You are on page 1of 897

,:carihant

ARIHANT PRAKASHAN (Series), MEERUT


All Rights Reserved

!:Ii ©Author
No part of this publication may be re-produced, stored in a retrieval system or by any means, electronic,
mechanical, photocopying, recording, scanning, web or otherwise without the written permission of the
publisher. Arihant has obtained all the information in this book from the sources believed to be reliable
and true. However, Arihant or its editors or authors or illustrators don't take any responsibility for the
absolute accuracy of any information published and the damage or loss suffered thereupon.

All disputes subject to Meerut (UP) jurisdiction only.

!:Ii Administrative & Production Offices


Regd. Office
'Ramchhaya' 4577/15, Agarwal Road, Darya Ganj, New Delhi -110002
Tele: 011- 47630600, 43518550

Head Office
Kalindi, TP Nagar, Meerut (UP) - 250002, Tel: 0121-7156203, 7156204

!:Ii Sales & Support Offices


Agra, Ahmedabad, Bengaluru, Bareilly, Chennai, Delhi, Guwahati,
Hyderabad, Jaipur, Jhansi, Kolkata, Lucknow, Nagpur & Pune.

!:Ii ISBN 978-93-26191-70-8

PO No: TXT-XX-XXXXXXX-X-XX
Published by Arihant Publications (India) Ltd.

PRODUCTION TEAM
Publishing Managers Mahendra Singh Rawat, Keshav Mohan Inner Designer Ankit Saini
Project Head Yojna Sharma Page Layouting Rajbhaskar Rana
Project Editor Mansi, Manish Proof Readers Ashish Kumar, Princi Mittal
Cover Designer Shanu Mansoori

For further information about the books published by Arihant, log on to


www.arihantbooks.com or e-mail at info@arihantbooks.com
Follow us on O @ a(§]
CHAPTER
0

Basic
Mathematics
TRIGONOMETRY
It is one of the important branch of mathematics which deal with relations of
sides and angles of triangle and also with the relevant functions of any angle.
Consider a ray OA. If this ray revolves about its end point O in anti-clockwise
direction and takes position OB, then we say that the angle ∠AOB has been
generated as shown in the following figure.
B Initial line
ne O A
l li
ina Te
rm rm
Te ina
l li
ne
O A
Initial line B
(i) Positive angle (ii) Negative angle
Fig. 0.1

Or simply say that angle is a measure of an amount of revolution of a given ray


about its initial point.
The angle is positive (or negative), if the initial line revolves in anti-clockwise
Inside
(or clockwise) direction to get the terminal line. 1 Trigonometry
2 Calculus (Differentiation)
System of measurement of angles 3 Integration
(i) Sexagesimal system In this system, a right angle is divided into 90 equal 4 Graphs
parts, called degree.
Thus, 1 right angle = 90 ° (degrees)
1° = 60 ′ (minutes)
1′ = 60 ′′ (seconds)
2 OBJECTIVE Physics Vol. 1

(ii) Circular system In this system, the unit of


measurement is radian. Trigonometrical ratios (or T-ratios)
l Consider the two fixed lines X ′OX andYOY ′ intersecting
θ = radian or rad each other at right angle at point O as shown in the
r θ
l
π following figure
O
1 right angle = rad r
Y
2 A
1 straight angle = π rad
Fig. 0.2 Circular system θ
and 1 complete angle = 2π rad X′
O B
X
c
One radian (denoted by 1 ) is the measure of an
angle subtended at the centre of a circle by an arc of
Y′
length equal to the radius of the circle.
Fig. 0.3 Trigonometrical ratios
180 °
1 rad (1c ) = ≈ 57°17′ 45 ′′ ≈ 57.3 ° Then,
π
22 π (i) Intersection point O is called origin.
Note (i) π = = 314
. (ii) 360° = 2 π radian, radian = 90°
7 2 (ii) X ′ OX andYOY ′ are called X-axis andY-axis,
Example 0.1 Find the radian measures corresponding to the respectively.
following degree measures. (iii) The portion XOY,YOX′, X ′ OY ′ andY ′ OX are
(i) 75° (ii) − 37°30′ (iii) 5°37′30′′ known as I, II, III and IV quadrants, respectively.
c Now, consider that the revolving line OA has traced out an
180°  π 
Sol. We have, 1c = ⇒ 1° =   angle θ in anti-clockwise direction (in I quadrant).
π 180
c c
From point A, draw AB ⊥ OX which results a right angled
 π   5π  ∆ABO, where AB = perpendicular, OA = hypotenuse and
(i) 75° =  75 ×  = 
 180  12  OB = base.
1 °  75 °
c c
  75 π   5π  The three sides of right angled triangle are related to each
(ii) − 37°30′ = −  37  = −   =− ×  =− 
 2  2  2 180   24  other through side having different ratios, called
trigonometrical ratios or T-ratios, which are given as
 5 °  45 °  45
c c
π   π
(iii) 5°37′30′′ =  5  =   =  ×  =  Perpendicular AB
 8  8  8 180  32 (i) sinθ = = (From Fig. 0.3)
Hypotenuse OA
Example 0.2 Find the degree measures corresponding to the Base OB
following radian measures. (ii) cos θ = =
c c Hypotenuse OA
 2π   π
(i)   (ii)   (iii) (− 2)c Perpendicular AB
 15   8 (iii) tan θ = =
Base OB
Sol. We have,
Base OB
 2π 180 ° (iv) cot θ = =
c
 2π 
(i)   =  ×  = 24° Perpendicular AB
 15   15 π 
Hypotenuse OA
 π  π 180 °  45 °  1 °
c
1 
′ (v) sec θ = =
(ii)   =  ×  =   = 22  = 22°  × 60 Base OB
 8 8 π   2   2   2 
Hypotenuse OA
= 22°30′ (vi) cosec θ = =
Perpendicular AB
 180 °  180 °  6°
(iii) (− 2)c =  − 2 ×  = − 2 × × 7 = − 114 
 π   22   11 Fundamental of T-ratios or
′ ′
6 
= − 114°  × 60 = − 114°
 8
32 
trigonometric functions
11   11 For any acute angle say θ ( < 90 ° ), the functions are given as
8 ″ (i) cosec θ =
1
(ii) sec θ =
1
= − 114°32 ′  × 60 • − [114° 32 ′ 44′′]
11  sin θ cos θ
Basic Mathematics 3

1 T-ratios of allied angles


(iii) cot θ =
tan θ
In trigonometry two angles are said to be allied angles
(iv) sin 2 θ + cos 2 θ = 1 when their sum or difference is a multiple of 90°.
The T-ratios of the following allied angles are as
(v) 1 + tan 2 θ = sec 2 θ
(i) When angle (say) θ is negative, then
(vi) 1 + cot 2 θ = cosec 2 θ (a) sin (− θ ) = − sin θ (b) cos (− θ ) = cos θ
Signs of trigonometric ratios or T-ratios in (c) tan (− θ ) = − tan θ
various quadrants (ii) When angle θ is less than 90°
(i.e., lies in I quadrant), then
II
only sin and cosec
I
All + ve
(a) sin(90° − θ ) = cos θ (b) cos(90° − θ ) = sin θ
are + ve (c) tan(90° − θ ) = cot θ
III IV
(iii) When angle θ lies between 90° and 180°
only tan and only cos and (i.e., lies in II quadrant), then
cot are + ve sec are + ve (a) ● sin(90° + θ ) = cos θ
Fig. 0.4 Sign of T-ratios ● cos(90 ° + θ ) = − sin θ
(i) In I quadrant, all T-ratios are positive. ● tan(90° + θ ) = − cot θ
(ii) In II quadrant, sin θ is + ve, cos θ and tan θ are − ve . (b) ● sin(180° − θ ) = sin θ
● cos(180 ° − θ) = − cos θ
(iii) In III quadrant, tan θ is + ve, sin θ and cos θ are − ve .
● tan(180° − θ ) = − tan θ
(iv) In IV quadrant, cos θ is + ve, sin θ and tan θ are − ve.
(iv) When angle θ lies between 180° and 270°
4 (i.e., lies in III quadrant), then
Example 0.3 If sin θ = , where θ lies in the first quadrant,
5 (a) ● sin(180° + θ ) = − sin θ
then find all the other T-ratios. ● cos(180° + θ ) = − cos θ

Sol. Let ∆PQR be right angled triangle, right angled at Q. ● tan(180° + θ ) = tan θ

P
(b) ● sin(270° − θ ) = − cos θ
● cos(270° − θ ) = − sin θ
5 4
● tan(270° − θ ) = cot θ

θ (v) When angle θ lies between 270° and 360°


R Q
(i.e., lies in IV quadrant), then
4
sin θ = (Given) (a) ● sin(270° + θ ) = − cos θ
5
● cos(270° + θ ) = sin θ
∴ PR = 5 and PQ = 4
● tan(270° + θ ) = − cot θ
On applying Pythagoras theorem in ∆PQR, we have
(PR )2 = (PQ )2 + (QR )2 (b) ● sin(360° − θ ) = − sin θ
● cos(360° − θ ) = cos θ
⇒ (5)2 = (4)2 + (QR )2
● tan(360° − θ) = − tan θ
⇒ QR = 25 − 16 = 9 = 3
Values of T-ratios of some standard angles
[Taking positive value of square root]
QR 3 0° 30° 45° 60° 90° 120° 135° 150° 180°
Now, cos θ = = , Angle (θ)  π   π   π   π   2π   = 3π   = 5π  (= π)
PR 5  =   =   =   =   =     
 6   4   3  2  3  4  6
PQ 4
tan θ = = ,
QR 3 0 1 1 3 1 3 1 1 0
sin θ
QR 3 2 2 2 2 2 2
cot θ = = ,
PQ 4 1 3 1 1 0 1 1 3 −1
cos θ − − −
PR 5 2 2 2 2 2 2
sec θ = =
QR 3
0 1 1 3 ∞ − 3 −1 1 0
tan θ −
PR 5
and cosec θ = = 3 3
PQ 4
4 OBJECTIVE Physics Vol. 1

Example 0.4 Find the value of 3 tan A − tan 3 A


● tan 3A =
(i) sin(− 45° ) (ii) tan 225° 1 − 3 tan 2 A
(iii) cos 300° (iv) sec 120°
● sin(A + B ) + sin(A − B ) = 2 sin A cos B
Sol. (i) sin(− 45° ) = − sin 45° [Q sin(− θ) = − sin θ] ● sin(A + B ) − sin (A − B ) = 2 cos A sin B
1 1
=− [Q sin 45° = ] ● cos(A + B ) + cos(A − B ) = 2 cos A cos B
2 2
● cos(A + B ) − cos(A − B ) = − 2 sin A sin B
(ii) tan 225° = tan(270° − 45° ) = cot 45°
[Q tan(270° − θ) = cot θ] C + D  C − D 
● sin C + sin D = 2 sin   ⋅ cos  
=1 [Q cot 45° = 1]  2   2 
(iii) cos 300° = cos(270° + 30° ) = sin 30°
C + D  C − D 
[Q cos(270° + θ) = sin θ] ● sin C − sin D = 2 cos   ⋅ sin  
 2   2 
1 1
= [Q sin 30° = ]
C + D  C − D 
2 2 ● cos C + cos D = 2 cos   ⋅ cos  
(iv) sec 120° = sec(180° − 60° ) = − sec 60°  2   2 
[Q sec(180° − θ) = − sec θ]
C + D  C − D 
= −2 ● cos C − cos D = − 2 sin   ⋅ sin  
 2   2 

Some important formulae of Example 0.5 Find the value of (i) sin 15° (ii) tan 75°

trigonometry Sol. (i) We have,


sin 15° = sin(45° − 30° )
● sin(A + B ) = sin A cos B + cos A sin B
= sin 45° cos 30° − cos 45° sin 30°
● sin(A − B ) = sin A cos B − cos A sin B [Q sin(A − B ) = sin A cos B − cos A sin B]
● cos(A + B ) = cos A cos B − sin A sin B
1 3 1 1
● cos(A − B ) = cos A cos B + sin A sin B = ⋅ − ⋅
2 2 2 2
tan A + tan B 1 3
● tan(A + B ) = (Q sin 45° = cos 45° = , cos 30° =
1 − tan A tan B 2 2
tan A − tan B 1
● tan(A − B ) = and sin 30° = )
1 + tan A tan B 2
3 −1
2 tan A =
● sin 2A = 2 sin A cos A = 2 2
1 + tan 2 A (ii) We have, tan 75° = tan(45° + 30° )
● cos 2A = cos 2 A − sin 2 A = 2 cos 2 A − 1 tan 45° + tan 30°  tan A + tan B 
= Q tan (A + B ) = 
1 − tan A
2
1 − tan 45°⋅ tan 30°  1 − tan A ⋅ tan B 
= 1 − 2 sin 2 A =
1 + tan 2 A 1
1+
2 tan A 3  1 
● tan 2A = = Q tan 45° = 1 and tan 30° = 
1 − tan 2 A 1  3
1 − 1⋅
3
● sin 3A = 3 sin A − 4 sin A 3

3 +1
● cos 3A = 4 cos 3A − 3 cos A =
3 −1
CHECK POINT 0.1 5 5
1. Find the radian measures corresponding to the following Ans. sin θ = − and tan θ =
degree measures. 13 12
(i) 25° (ii) − 47° 30′ 3
(iii) 39° 22′30′′ 4. Find the values of other five T -ratios, if tan θ = − and θ
4
5π 19π 7π lies in II quadrant.
Ans. (i) (ii) − (iii)
36 72 32 3 4 5 5 4
Ans. sin θ = , cos θ = − , cosec θ = , sec θ = − and cot θ = −
5 5 3 4 3
2. Find the degree measures corresponding to the following
radian measures.
5. Find the values of the following T -ratios
c (i) cosec 315° (ii) cos 210°
18 π 
(i)   (ii) (− 3)c (iii) sin(− 330°)
 5 
3 1
5π 
c
9π 
c Ans. (i) − 2 (ii) − (iii)
(iii)  −  (iv)   2 2
 6  5
6. Find the value of
Ans. (i) 648° (ii) − 171° 49 ′5′′ (iii) − 150°
(i) sec165° (ii) cot 105°
(iv) 324°
1− 3
12 Ans. (i) ( 2 − 6) (ii)
3. Find sin θ and tan θ, if cos θ = − and θ lies in the third 1+ 3
13
quadrant.

CALCULUS (DIFFERENTIATION)
Differentiation in calculus, is the process of finding dy
the derivative. The derivative is the instantaneous
Physical meaning of
rate of change of a function with respect to one of its
dx
variable. This is equivalent to finding the slope of the (i) The ratio of small change in the function y and the
tangent line to the function at a point. variable x is called the average rate of change of y w.r.t.
x.
Variable e.g. If a body covers a small distance ∆s in small time ∆t,
∆s
A quantity, which can take different values, is called a then average velocity of the body, v av =
variable quantity. A variable is usually represented by ∆t
x, y, z etc. ∆y
(ii) The limiting value of , when ∆ x → 0, i.e.,
∆x
Constant lim
∆y dy
= , is called the instantaneous rate of
A quantity, whose value remains unchanged during ∆x → 0 ∆ x dx
mathematical operations, is called a constant quantity. change of y w.r.t. x.
The integers, fractions such as π, e, etc are all Thus, the differentiation of a function w.r.t. a variable
constant. implies the instantaneous rate of change of the function
w.r.t. that variable.
Function e.g., Instantaneous velocity of the body,
A quantity y is called a function of a variable x, if ∆s ds
corresponding to any given value of x, there exists a lim =
∆t → 0 ∆ t dt
single definite value of y. The phrase ‘y is function of
x’ is represented as y = f (x ).
Theorems of differentiation
e.g., Consider that y is a function of the variable x d
which is given by y = 4x 2 + 3 x + 7 and (i) If c is constant, then (c ) = 0
dx
y = sin x + e x .
(ii) If y = cv , where c is a constant and v is a function of x,
Here, we will treat x as independent variable and y as dy d dv
dependent variable, i.e., the value y depends on x. If then = (c ⋅ v ) = c
dx dx dx
we change the value of x, then y will change.
6 OBJECTIVE Physics Vol. 1

(iii) If y = x n , where n is a real number, then x3 + 4


(iv) We have, y =
dy d x +1
= (x n ) = nx n − 1
dx dx On differentiating both sides w.r.t. x, we get
(iv) If y = u ± v ± w , where u, v and w are functions of d 3 d
(x + 1) (x + 4) − (x 3 + 4) (x + 1)
dy dx dx
dy d du dv dw =
x, then = (u ± v ± w ) = ± ± dx (x + 1)2
dx dx dx dx dx
(By division rule)
(v) Product rule If y = u ⋅ v , where u and v are
dy d dv du dy (x + 1) (3x + 0) − (x + 4) (1 + 0)
2 3

functions of x, then = (u ⋅ v ) = u +v =
dx dx dx dx dx (x + 1)2
u 3x 2 (x + 1) − x 3 − 4 3x 3 + 3x 2 − x 3 − 4
(vi) Division rule If y = , where u and v are functions = =
v (x + 1)2 (x + 1)2
du dv 2x 3 + 3x 2 − 4
v −u =
dy d u dx dx (x + 1)2
of x, then =   =
dx dx  v  v2
Formulae for differential coefficient of
Example 0.6 Differentiate the following functions trigonometric, logarithmic and exponential function
(i) y = x − 3 (ii) 6x 5 + 4x 3 − 3x 2 + 2x − 7 d d
● (sin x ) = cos x ● (cos x ) = − sin x
(x + 4)
3
dx dx
(iii) y = (x + 2) (x 2 + 1) (iv) y =
(x + 1) ●
d
(tan x ) = sec 2 x ●
d
(cot x ) = − cosec 2 x
Sol. (i) We have, y = x −3 dx dx
d
On differentiating both sides w.r.t. x, we get ● (sec x ) = sec x ⋅ tan x
dx
dy  d n 
= − 3x − 3 − 1 Q x = nx n − 1 d
dx  dx  ● (cosec x ) = − cosec x ⋅ cot x
dx
− 3
= − 3x − 4 = 4 d 1 d x
x ● (log x ) = ● (e ) = e x
dx x dx
(ii) Let y = 6x 5 + 4x 3 − 3x 2 + 2x − 7
On differentiating both sides w.r.t. x, we get Example 0.7 Differentiate the following functions
dy d 5 d 3 d d d (i) y = sin x + e x (ii) y = 3x 2 + log x + 4 e x + 5
=6 x +4 x − 3 x2 + 2 x − 7
dx dx dx dx dx dx (iii) y = e x ⋅ tan x
 d dv  Sol. (i) We have, y = sin x + e x
Q (c ⋅ v ) = c ⋅ 
 dx dx 
On differentiating both sides w.r.t. x, we get
dy dy d
⇒ = 6 ⋅ 5 x 5 − 1 + 4 ⋅ 3 x 3 − 1 − 3 ⋅ 2 x 2 − 1 + 2 ⋅ x1 − 1 − 0 = (sin x + e x ) = cos x + e x
dx dx dx
 d n n −1 d  (ii) We have, y = 3x 2 + log x + 4 e x + 5
Q x = nx and c = 0
 dx dx  On differentiating both sides w.r.t. x, we get
dy 1−1 dy d
⇒ = 30x 4 + 12x 2 − 6x + 2 × 1 (Q x = x = 1)
0
= (3x 2 + log x + 4 e x + 5)
dx dx dx
dy d d d d
⇒ = 30x 4 + 12x 2 − 6x + 2 = 3 x2 + (log x ) + 4 e x + 5
dx dx dx dx dx
(iii) We have, y = (x + 2) (x 2 + 1) 1 1
= 3 ⋅ 2x 2 − 1 + + 4e x + 0 = 6x + + 4e x
On differentiating both sides w.r.t. x, we get x x
dy d 2 d (iii) We have, y = e ⋅ tan x
x
= (x + 2) (x + 1) + (x 2 + 1) (x + 2)
dx dx dx On differentiating both sides w.r.t. x, we get
(By product rule) dy d d x
= ex ⋅ tan x + tan x e (By product rule)
= (x + 2) (2x + 0) + (x 2 + 1) (1 + 0) dx dx dx
= 2x (x + 2) + x 2 + 1 = 2x 2 + 4x + x 2 + 1 = 3x 2 + 4x + 1 = e x ⋅ sec 2 x + tan x ⋅ e x = e x (sec 2 x + tan x )
Basic Mathematics 7

Chain rule Applications of differentiation in physics


This rule is applied, when the given function is the (i) If the displacement is a function of time t, then to
function of function, i.e., a function is in the form of find the velocity, differentiate s w.r.t. t.
f [g (x )].
d ds
∴ f [g (x )] = f ′ [g (x )] ⋅ g ′ (x ) s = f (t ), v =
dx dt
Example 0.8 Differentiate sin(x 2 + 5) w.r.t. x. (ii) If the velocity is a function of time t, then to find
Sol. Let y = sin(x + 5)
2 acceleration, differentiate v w.r.t. t
dv
On differentiating both sides w.r.t. ‘x’, we get v = f (t ), a =
dy d d 2 dt
= sin(x 2 + 5) = cos(x 2 + 5) ⋅ (x + 5)
dx dx dx d  ds  d 2 s
= cos(x 2 + 5) ⋅ (2x + 0) = 2x cos(x 2 + 5) =   =
dt  dt  dt 2
Maxima and minima (iii) If the velocity is a function of displacement s, then
Let y = f (x ), where y is a function of x. to find the acceleration, differentiate v w.r.t. t and
dv
For y to be minimum or maximum, put
dy
= 0, then use the expression, a = v
dx ds
find x. (iv) Consider the motion along the X-axis
● If v > 0, s is increasing, then the particle is moving
d 2y d 2y
If < 0, y is maximum and if > 0, y is minimum. along the positive X-axis.
dx 2 dx 2
● If v < 0, s is decreasing, then the particle is moving
Note Most of time, it is known from physical situation whether the
quantity is a maximum or minimum; therefore, there is no need along the negative X-axis.
d 2y ● If a > 0, v > 0, then speed is increasing along the
to check maximum or minimum with the help of 2 .
dx positive X-axis.
Example 0.9 Divide a number 1000 in two parts such that ● If a > 0, v < 0, then speed is decreasing along the
there product is maximum. negative X-axis.
Sol. Let the two parts be x and (1000 − x ). ● If a < 0, v > 0, then the speed is decreasing along the

∴ Their product, P = x (1000 − x ) ⇒ P = 1000x − x 2 positive X-axis.


On differentiating both sides w.r.t. x, we get ● If a < 0, v < 0, then the speed is increasing along the

dP negative X-axis.
= 1000 − 2x ...(i)
dx Note
For P to be maximum or minimum, (i) If v and a have same sign, then the speed is increasing.
dP (ii) If v and a have opposite sign, then the speed is decreasing.
= 0 ⇒ 1000 − 2x = 0 ⇒ x = 500
dx
Example 0.10 The displacement of a particle as a function of
On differentiating both sides of Eq. (i) w.r.t. x, we get time t is given by s = α + βt + γt 2 + δt 4 , where α, β, γ and
d 2P δ are constants. Find the ratio of the initial velocity to the
= − 2< 0
dx 2 initial acceleration.
∴ P is maximum at x = 500 Sol. First find the velocity and acceleration in terms of time t,
On dividing equally, the two parts are (500, 500). then use t = 0 to find the initial values.
s = α + βt + γt 2 + δt 4 (Given)
Remembering points On differentiating both sides w.r.t. t, we get
All the problems of maxima/minima cannot be solved by the above ds
= β + 2γ t + 4δt 3

methods e.g., y = x 2, y is maximum when x is maximum. dt


● If y = sin x, by simple observation, y is maximum if sin x is  ds 
maximum, i.e., sin x = 1. The value of sine or cosine functions lies ⇒ v = β + 2γ t + 4δt 3 Q = v  ...(i)
 dt 
between − 1 and + 1.
● The product of the two parts is the maximum when the parts are On differentiating both sides of Eq. (i), w.r.t. t, we get
equal. dv
= 2 γ + 4 ⋅ 3 δt 2
dt
8 OBJECTIVE Physics Vol. 1

dv  dv  Sol. Given, x = 6 t − t 2 + 4
⇒ a= = 2γ + 12 δt 2 Q = a
dt  dt  dx d
v= = (6t − t 2 + 4) = 6 − 2t + 0 = 2(3 − t )
At t = 0, v = β and a = 2γ dt dt
Initial velocity β At t < 3, v > 0, then the particle is moving along the positive
∴ =
Initial acceleration 2γ x-direction.
At t > 3, v < 0, then particle is moving along the negative
Example 0.11 The position of a particle moving along the x-direction.
X-axis varies with time t as x = 6 t − t 2 + 4. Find the time At t = 3, v = 0
interval during which the particle is moving along the positive For time-interval t = 0 to t = 3, the particle is moving along
x-direction. the positive x-direction.

CHECK POINT 0.2


1. Differentiate the following function 2x
(v) (2x + 3) sec2(x 2 + 3x + 1) (vi) − 3 sin x + 7e x +
1 x2 + 1
(i) y = 3 x 4 + 2 2 + log x (ii) y = (x 2 + 1) (x + 2)
x 2. A particle is moving with velocity v = t 3 − 6 t 2 + 4, where v is
3x2 in m/s and t is in seconds. At what time will the velocity be
(iii) y = (iv) y =sin x
x +1 maximum/minimum and what is it equal to?
(v) y = tan(x 2 + 3 x + 1) Ans. vmax = 4 m/s at t = 0 s and vmin = − 28 m/s at t = 4 s
(vi) y = 3 cos x + 7e x + log(x 2 + 1) 3. If the time and displacement of particle along the positive
1
1 1 X-axis are related as t = (x 2 − 1) 2, then find the acceleration
Ans. (i)12x 3 − 4 + (ii) 3x 2 + 4 x + 1
x3 x in terms of x.
3x 2 + 6 x 1
(iii) (iv) cos x Ans.
(x + 1)2 x3

INTEGRATION
It means summation. It is the process of finding the function, Q d (sin x ) = cos x
whose derivative is given. In other word, integration is the

∫ cos x dx = sin x + C 
 dx


reverse process of differentiation. It’s symbol is ∫ . 1 Q d log x = 1 
Consider a function f (x ), whose derivative w.r.t. x is equal to

∫ x dx = log e x + C 
 dx e
x

f ′ (x ), then f (x ) + C is called integration of f ′ (x ), where C is Q d e x = e x 


∫e dx = e x + C
x
called constant of integration. Symbolically, it is written as ●
 
 dx 
∫ f ′(x )dx = f (x ) + C Example 0.12 Evaluate the following integrals.
Here, f ′ (x ) dx is called element of integration and ∫ is called 1
(i) ∫ (e x + + 2x 2 + 3) dx
indefinite integral. x
 3 4
(ii) ∫  cos x + 3x1/ 2 + + 2  dx
Some basic formulae of integration  x x 
x n +1  d  x n +1  n
 1 
Sol. (i) Let I = ∫ e + + 2x 2 + 3 dx
x
∫ x dx = + C; n ≠ − 1 Q  =x 
n
●  x 
n +1  dx  n + 1 
1
Q d x = 1 = ∫ e x dx + ∫ x dx + 2∫ x
2
dx + 3 ∫ 1 dx

∫ 1 dx = x + C 
 dx


x 2+1
Q d − (cos x ) = sin x = e x + log e x + 2 ⋅ + 3x + C

∫ sin x dx = − cos x + C 
 dx


2 +1
Basic Mathematics 9

2 3 b
= e x + log e x + x + 3x + C d
3 If f (x ) = f ′ (x ), then ∫ f ′ (x ) dx is called definite integral,
dx a
 3 4
(ii) Let I = ∫  cos x + 3x +
+  dx
1/ 2
where a and b are called lower and upper limit,
x x2 
respectively of variable x.
1 1
= ∫ cos x dx + 3∫ x1/ 2dx + 3∫ dx + 4∫ 2 dx After carrying out integration, the result is evaluated
x x between upper and lower limits as shown below
x (1/ 2) + 1 x −2 + 1 b
= sin x + 3 ⋅ + 3 log e x + 4 +C
∫ f ′(x ) dx = | f (x )|a = f (b ) − f (a )
b
(1/2) + 1 −2 + 1
a
4
= sin x + 2x 3/ 2 + 3 log e x − + C
x Example 0.14 Evaluate the following
2

∫ (4x + 2x 2 + 2x + 1) dx
3
Other important formula of integration (i)
0
f (ax + b )
I = ∫ f ′ (ax + b ) dx = π /4
 d  (ii) ∫ (sin x + cos x ) dx
 dx (ax + b ) 
  4
0

dx
Example 0.13 Evaluate the following
(iii) ∫ x
2
 1 
∫ (2x + 1) dx (ii) ∫ 
3 2
(i)  dx
a − x I= ∫ (4x + 2x 2 + 2x + 1) dx
3
Sol. (i) Let

∫ (x (iv) ∫ sin (2x 2 ) dx


0
(iii) 2
+ 3x + 4)4dx 2 2 2 2
3 +1
(2x + 1) = 4 ∫ x 3dx + 2∫ x 2dx + 2 ∫ x dx + ∫ 1 dx
I= ∫ (2x + 1) dx = +C
3
Sol. (i) Let 0 0 0 0
d 
(3 + 1)  (2x + 1) 2 2 2
dx  x4 x3 x2
=4 +2 +2 + | x |20
(2x + 1) (2x + 1)
4 4 4 0
3 0
2 0
= +C = +C
4⋅2 8  24 − 04   23 − 03   22 − 02 
 1  =4  + 2  +2 
log (a − x )  4   3   2 
(ii) Let I = ∫   dx = +C
a − x d
(a − x ) + (2 − 0)
dx 2
log (a − x ) = 16 + × 8 + 4 + 2
= + C = − log (a − x ) + C 3
−1
16 66 + 16 82
I = 22 + = =
(iii) Let I = ∫ (x + 3x + 4)4dx
2
3 3 3
π /4
(x 2 + 3x + 4)4 + 1
=
d 2
+C (ii) Let I = ∫ (sin x + cos x ) dx
(4 + 1) (x + 3x + 4) 0
dx π /4 π /4

=
(x 2 + 3x + 4)5
+C
= ∫ sin x dx + ∫ cos x dx
0 0
5 (2x + 3)
= |− cos x |π0 / 4 + |sin x |0π / 4
− cos (2x 2 )
(iv) Let I = ∫ sin (2x ) dx = +C
2
π π
d
(2x 2 ) = − cos + cos 0 + sin − sin 0
dx 4 4
− cos (2x 2 ) 1 1
= +C =− +1+ − 0 =1
4x 2 2
4
dx

4
Definite integral (iii) Let I = = log e x 2 = log e 4 − log e 2
2
x
When a function is integrated between a lower limit and
 4  a 
an upper limit, it is called a definite integral. = log e   = log e 2 Qlog e a − log e b = log e  b  
 2  
A definite integral has definite value.
10 OBJECTIVE Physics Vol. 1

Example 0.15 Evaluate the following = | x 3 + 2x 2 + 5x |40


2 π /4
= 43 + 2 × 42 + 5 × 4 − 0
∫e ∫ cos (2x
( x + 4)
(i) dx (ii) 2
+ x ) dx
1 0 = 64 + 32 + 20 = 64 + 52 = 116
2 4
dx
∫ (3x + 4) ∫ (3x + 4x + 5) dx
2
(iii) (iv) Application of integration in physics
1 0
ds dv dv
2 We know that, v = ,a = or v
dt dt ds
ex + 4
2
Sol. (i) Let I = ∫ e ( x + 4)dx = If the displacement is given and we have to find the
d
1 (x + 4) velocity and acceleration, then we use differentiation.
dx 1
2 But if the acceleration is given and we have to find the
ex + 4
= = e 2 + 4 − e1 + 4 = e 6 − e 5 = e 5 (e − 1) velocity and displacement, then we use integration.
1 1
Example 0.16 A particle is moving under constant
π /4
acceleration a = 3 t + 4 t 2 . If the position and velocity of the
(ii) Let I = ∫ cos (2x + x ) dx
2
particle at start, i.e., t = 0 are x 0 and v 0 , respectively, then
0
find the displacement and velocity as a function of time t.
π /4
π /4 Sol. O A
sin (2x + x )
2
sin (2x 2 + x )
= = t=0 t=t
d
(2x 2 + x ) 4x + 1 0 x = x0 x=x
v = v0 v=v
dx 0
 π2 π Given, a = 3t + 4t 2
sin2. +  dv  dv 
 16 4  sin(2 × 02 + 0) = 3t + 4t 2 Qa = 
= − dt  dt 
π 4 × 0 +1
4⋅ +1 v t
4 ⇒ ∫ dv = ∫ (3 t + 4 t
2
) dt (Integrating both sides)
 π 2 + 2π  v0 0
sin  t
 8  sin 0 t2 t3
= − ⇒ =3 +4
|v |vv 0
(π + 1) 1 2 30
 π + 2π 
2
3 2 4
sin  ⇒ (v − v 0 ) = (t − 0) + (t 3 − 0)
 8   1   π 2 + 2π  2 3
= − 0=   sin   3 4
π +1  π + 1  8  ⇒ v = v0 + t 2 + t 3 …(i)
2 3
2
dx 3 4  dx 
2 ⇒ = v0 + t 2 + t 3 Qv = 
dx log (3x + 4) dt 2 3  dt 
(iii) Let I = ∫ (3x + 4)
=
d x t
 3 2 4 3
(3x + 4)
1
dx 1
⇒ ∫ dx = ∫ v 0 + 2 t + 3 t  dt
x 0 0
2 t
log (3x + 4) 3 t3 4 t4 1 1
= ⇒ x − x 0 = v 0t + ⋅ + ⋅ = v 0t + ⋅ t 3 + t 4 − 0
3 1 2 3 3 40 2 3
log (3 × 2 + 4) − log (3 × 1 + 4) 1 1 1 1
= ⇒ x − x 0 = v 0t + ⋅ t 3 + t 4 ⇒ x = x 0 + v 0t + t 3 + t 4
3 2 3 2 3
1 1 10
= [log 10 − log 7] = log   Average value
3 3  7
4 If the velocity is variable and depends on time t, then find
(iv) Let I = ∫ (3x + 4x + 5) dx the average value of velocity (v ) for time interval t = t1 to
2

0 t = t 2.
4 t2
x 2 + 1 4x1 + 1
=3 +
2 +1 1+1
+ 5x
0
∫ v dt
t1
4 Let v = f (t ); v = t2
x3 x2
=3
3
+4
2
+ 5x
0
∫ dt
t1
Basic Mathematics 11

To find the average value of square of velocity Sol. We have, v = v 0 sin ω t


t2 t2 T /2 T /2
v 0 − cos (ωt ) 0
∫ v dt
2
∫ v dt
4
∫ v 0 sin (ωt ) dt
t1 t1 Average velocity, v = 0
= ω
v2 = ⇒v 4 = T /2
|t |T0 / 2
∫ dt
t2 t2

∫ dt ∫ dt 0
t1 t1 v0   T 
If velocity is a function of displacement, v = f (x ), for ω − cos ω 2  + cos 0
=  
average of v from x = x 1 to x = x 2 T 
 − 0
x2 2 
∫ v dt v0
x1
2π  2π T  
⇒ v = x2 − cos  T . 2  + 1 
T    2π 
∫ dt =
T
Qω =
 T

x1
2
The above procedure can be applied to find the average 2v 0T (1 − cos π ) v 0
= = [1 − (−1)]
value of any quantity; like acceleration, force, etc. 2π T π
(Q cos π = cos180° = − 1)
Example 0.17 The velocity of a particle is given by
v = v 0 sin ωt, where v 0 is constant and ω = 2π / T. Find the v0 2v 0
= (1 + 1) =
average velocity in time interval t = 0 to t = T / 2 . π π

CHECK POINT 0.3


1. Evaluate
 1
+ 2 2 + 3x 3  dx
1
(i) ∫  sin x +
x x 
4. Evaluate the following
2
dx
(ii) ∫ (3cos x + e x + 4 x 2 + x + 5) dx (i) ∫ (x 2 + 4 x + 1)
0
2 3 4
Ans. (i)− cos x + log x − + x + C 3
x 4 (ii) ∫ (4 x 3 + 3x 2 + 2x + 1) dx
4 3 x2 1
(ii) 3sin x + e x + x + + 5x + C
3 2 1
Ans. (i) log13 (ii) 116
8
2. Evaluate the following
1 5. A particle is moving in a straight line under acceleration
(i) ∫ (2x 2 + 4 x + 1)2 dx (ii) ∫ x 2 + 2 dx a = kt, where k is a constant. Find the velocity in term of t, if
(iii) ∫ cos (x + 2) dx the motion starts from rest.
kt 2
1 (2x 2 + 4 x + 1)3 1 Ans.
Ans. (i) + C (ii) log (x 2 + 2) + C 2
12 (x + 1) 2x
6. A particle is moving in a straight line such that its velocity
(iii) sin (x + 2) + C
varies as v = v 0 e − λt , where λ is a constant. Find the average
3. Evaluate
3 π/4 velocity during the time interval in which the velocity
 1 
(i) ∫  4 x + x + 1 dx (ii) ∫ (sin x − cos x) dx v
decrease from v 0 to 0 .
1 0 2
Ans. (i)18 + log 3 (ii)1 − 2 v0
Ans.
log e (2)
12 OBJECTIVE Physics Vol. 1

GRAPHS (b) If m is –ve, i.e., 90° < θ < 180°, then the lines will
It is defined as pictorial representation showing the be of the type,
relation between variable quantities, typically two
variables, each measured along one of a pair of axes at
θ
right angles.
90° < θ < 180°
(i) If a graph is concave up (curved upward), the slope
is increasing.
Y
Fig. 0.10

(c) If c is + ve, then the lines will cut theY-axis above


X the origin.
O Increasing slope
Y Y
Fig. 0.5
(ii) If a graph is concave downward or
c c
(curved downward), the slope is decreasing. X X
O O
Y
Fig. 0.11

(d) If c is –ve, then the lines will cut theY-axis below


X the origin.
O Decreasing slope
Y Y
Fig. 0.6

(iii) If the graph is a straight line, the slope is constant. O or O


X X
Y c c

Fig. 0.12
X
O Slope is constant (e) If c = 0, then the lines will pass through the origin.
Fig. 0.7 Y Y
(iv) The general equation of a straight line is of the form
y = mx + c where, m is the slope of line, m = tanθ O
X or O
X
and c is the intercept on theY-axis.
Y
y = mx + c

θ Fig. 0.13
X
c (v) Parabola Some standard forms of parabola are as
follows
Fig. 0.8 (a) y 2 = kx, a parabola passing through the origin and
(a) If m is + ve, i.e., 0° < θ < 90°, then lines will be of opens rightward.
the type,
Y

X
O
θ
0° < θ < 90°

Fig. 0.14

Fig. 0.9
Basic Mathematics 13

(b) y 2 = − kx, a parabola passing through the origin and (vii) Circle If equation of circle is x 2 + y 2 = a 2 , where
centre of circle ≡ (0, 0 ) and radius of circle = a
opens leftward.
Y
Y

X O (0,0)
O X

a a
Fig. 0.15

(c) x 2 = ky, a parabola passing through the origin and Fig. 0.20
opens upward. x2 y2
(viii) Ellipse Equation of ellipse is +
= 1 (a > b ),
Y a2 b 2
where, coefficient of x 2 ≠ coefficient of y 2 .
X Y
O

S′ S
Fig. 0.16 X 2b
(– ae,0) (ae,0)
(d) x = − ky, a parabola passing through the origin and
2
2a
opens downward.
Y
Fig. 0.21

O Here, 2a is major axis and 2b is minor axis.


X
b2
Eccentricity, e = 1 −
a2
Fig. 0.17 For ellipse, e < 1
(vi) Exponential graph The most popular graph based Focus, S ≡ (ae, 0 ), S ′ ≡ (−ae, 0 )
on exponential (e) are Area of ellipse = π ab
(a) y = e − x x2 y2
(ix) Hyperbola Equation of hyperbola is − = 1.
a2 b2
y Y

O x X
O (0,0)

Fig. 0.18 Fig. 0.22


−x
(b) y = 1 − e 1
(x) Rectangular Hyperbola If x ∝ or xy = constant,
then y
y Y

O x
X
Fig. 0.19 Fig. 0.23
14 OBJECTIVE Physics Vol. 1

Sketches of some standard curves


Equation of curve Diagram/Sketch Equation of curve Diagram/Sketch

1. (a) Straight lines Y 6. (a) Parabola y = 4 ax


2

x = a and x = − a, x=–a and y 2 = – 4 ax (– a, 2a)


(a, 2a)
y2=–4ax L¢ Y L y 2 = 4ax
where a > 0 x=a Vertex, O = (0, 0)
X′ X Focus, S = (a, 0)
(– a, 0) O (a, 0)
and ( – a, 0) X¢ X
(– a, 0) O S (a, 0)
Length of L1(a, –2a)
Y′ latusrectum = 4a L¢1 Y¢
(– a, – 2a)
(b) Straight lines Y
y = b and y = –b, (b) Parabola x 2 = 4 ay x2 = 4ay
y=b Y
where b > 0
(0, b) and x 2 = − 4 ay
(−2a, a) L1 L (2a, a)
X′ X (0, a) S
O Vertex, O = (0, 0)
(0, – b)
y = –b Focus, S = (0, a) O
X′ X
Y′ and (0, − a)
2. Straight lines Y Length of (0, −a)
(−2a, −a) L′1 L′(2a, −a)
latusrectum = 4 a
y = x and y = –x
x2= − 4ay
x
=

Y′
y

X′ X
x2 y 2
O 7. (a) Ellipse + = 1, Y
y = –x
a2 b 2
(0, b)
when a > b
Y′ (−a, 0)
Vertices = (± a, 0) (a, 0)
X′ X
3. Straight lines Y Centre, O = (0, 0) O (0, 0)
x y
+ =1, a ≠ b B (0, b)
a b x x (0, −b)
x + y = a, a = b b a + b =1 Y′
A(a, 0)
X′ X x2
y 2
O
a (b) Ellipse + = 1, Y
a2 b 2 (0, b)

when a < b
Y′
Vertices = (0, ± b ) (−a, 0) (a, 0)
X′ X
4. Modulus function Y O (0, 0)
Centre, O = (0, 0)
y =|x|
 x, for x ≥ 0
x
y

y =
=

y=

(0, −b)
-x

− x, for x < 0 X¢ X
O Y′
8. (a) Sine function Y
y = sin x

π 2π
5. Circle x 2 + y 2 = a 2 X′ X
Y O π/2
Centre = (0, 0) (0, a)
Radius = a Y′
(− a, 0) (a, 0)
X (b) Cosine function Y

O (0, 0) y = cos x
π
(0, −a) X′ X
O π/2 2π

Y′
Basic Mathematics 15

Example 0.18 Find the area of the region in the first Example 0.19 Find the area of region bounded by the
quadrant enclosed by the X-axis, the line y = x and the curve y 2 = 4x and the line x = 4.
circle x 2 + y 2 = 32. Sol. Given curve is a parabola, y 2 = 4x ...(i)
Sol. We have, circle Which is of the form y = 4aX having vertex (0, 0)
2

x 2 + y 2 = 32 ...(i) and line, x=4 ...(ii)


and line y=x ...(ii) Then, the region for which we have to find area is OACBO.
It is clear from the figure that region for finding area is Also, region OACO is symmetrical about X-axis.
OABO. Y A(4,4)

Y x=4
x2 + y2 = 32 O C
y=x X′ X
(0,0) (4, 0)
B
B
O Y′
X′ X
D A
On putting the value of x from Eq. (ii) in Eq. (i), we get
y 2 = 4(4) = 16 ⇒ y = ± 4
∴ Area of bounded region OACBO = 2(Area of region OACO )
Y′ (Q Parabola is symmetrical about X-axis)
4 4
On putting the value of y from Eq. (ii) in Eq. (i), we get = 2∫ y (parabola) dx = 2∫ 2 ⋅ x1/ 2 dx [From Eq. (i)]
x 2 + x 2 = 32 0 0
4
⇒ 2x 2 = 32 ⇒ x 2 = 16 ⇒ x = ± 4 4
 x 3/ 2  2
= 4∫ x1/ 2dx= 4  = 4 ⋅ [x 3/ 2]40
From Eq. (ii), we get y = ± 4 0  3 / 2  0 3
Thus, line and circle intersect each other at two points (4, 4) 8 8 8 64
and (− 4, − 4). So, coordinates of A(4 2, 0), and B (4, 4) in = [43/ 2 − 0] = × 4 4 = × 4 × 2 = sq units
3 3 3 3
I quadrant. 64
Now, area of OABO = Area of ODBO + Area of DABD Hence, the required area is sq units.
3
4 4 2
= ∫ y dx + ∫ circle
y dx Example 0.20 Find the area of the region bounded by the
0 line 4 line y = 3x + 2, the X-axis and the ordinates x = − 1 and
4 4 2 x = 1.
= ∫ xdx + ∫ 32 − x 2 dx Sol. Given lines are y = 3x + 2 ...(i)
0 4
y = 0 [on X-axis] ...(ii)
[From Eq. (i) and Eq. (ii)]
4 x = −1 ...(iii)
x 2 4 2
and x =1 ...(iv)
=  + ∫ 32 − x 2 dx
Now, table for y = 3x + 2
 2 0 4
4 2 2
4 −02
x 2
32  x  0 −
= + 32 − x 2 + sin− 1  
x
 4 2   4
3
2 2 2
y 2 0
4 2 4 32 
32 − 32 − 32 − 16 +
16  2 2 2
 The given region bounded by y = 3x + 2, X-axis and the
= + 
2  −1 4 2 32 − 1 4  ordinates x = − 1 and x = 1 is represented by shaded region.
 sin − sin
 4 2 2 4 2  Y B
x +2
 1  =3
= 8 + 0 − 2 × 4 + 16 sin− 1(1) − 16 sin− 1   y
 2 C (0,2)

 π  π X′
E x=1
X
= 8 − 8 + 16   − 16    D O A
 2  4 x = –1 −2 , 0

 3 
= 8π − 4π = 4π sq unit F
Hence, the required area of region is 4π sq units. Y′
16 OBJECTIVE Physics Vol. 1

∴ Required area Example 0.21 Find the area of the region bounded by the
= Area of region EFDE + Area of region ABDA x2 y2
ellipse + = 1.
− 2/ 3 1 a2 b2
= ∫ −1 y1 dx + ∫ − 2/ 3 y 2 dx
x2 y2
− 2/ 3 1
Sol. We have, =1 + ...(i)
a 2 b2
= ∫ −1 (3x + 2) dx + ∫ − 2/ 3 (3x + 2) dx
Since, power of x and y both are even in the equation of the
− 2/ 3 1 curve. So, it is symmetrical about the axes as shown in figure.
3 x 2  3 x 2 
=  + 2x + + 2x Y
 2 −1  2  − 2/ 3 B (0, b)
P (x,y)
  3  4 4  3   3  3  4 4 
=     −  −  − 2  +  + 2 −    −  X′ X
      2  9 3   A′ ( – a,0) O dx
 2 9 3 2   2 A(a,0)

 2 4 1  7 2 4 B′(0,–b)
=  − +  + − +  Y′
 3 3 2 2 3 3 
Area enclosed by the ellipse = 4 [Area enclosed by the ellipse
 4 − 8 + 3 21 − 4 + 8
=   + 
and coordinate axes in 1st quadrant]
 6   6  a a
b
⇒ A = 4 ∫ y dx = 4∫ a 2 − x 2 dx [From Eq. (i)]
 1 25 1 25 26 a
= −  + = + = 0 0
 6 6 6 6 6 a
4b 1 1 x
= x a 2 − x 2 + a 2 sin− 1 
a 2
13
= sq units 2 a 0
3
4b  1 2 − 1  4b 1 2  π 
13 = 0 + a sin (1) = × a   = πab sq units
Hence, the required area is sq units. a  2  a 2  2
3

CHECK POINT 0.4


1. Find the area of region bounded by the curve y = x 3, x2 y 2
3. For the curve + = 1, evaluate the area of the region
y = x + 6 and x = 0. 4 9
Ans. 10 sq units under the curve and above the X-axis.
2. Find the area of the region included between the parabola Ans. 3π sq units
3x 2 4. Find the area of the region bounded by y = x and y = x.
y= and the line 3 x − 2 y + 12 = 0. 1
4 Ans. sq units
6
Ans. 27 sq units
CHAPTER
01

Units, Dimensions
& Error Analysis
In this chapter, we will discuss about units and dimensions of different physical
quantities and errors that occur in measurement.

PHYSICAL QUANTITIES AND UNITS


All the quantities which are used to describe the laws of physics are called
physical quantities, e.g. length, mass, volume, etc.
To express the measurement of a physical quantity, we need to know two things
as given below
(i) The unit in which the quantity is measured.
(ii) The numerical value or the magnitude of the quantity.
i.e. The number of times that unit is contained in the given physical quantity
Inside
= nu 1 Physical quantities and units
1 System of units
∴ n∝ ⇒ nu = constant
u 2 Dimensions of physical
quantities
Here, n = numerical value of the physical quantity and u = size of unit.
Applications of dimensional
analysis
We may also write it as n 1u1 = n 2u 2 Defects or limitations
of dimensional analysis
where, n 1 and n 2 are values of the physical quantity in two different units u1
and u 2 . 3 Significant figures
Rules to determine significant
The standard amount of a physical quantity chosen to measure the physical figures
quantity of same kind is called a physical unit. Mathematical operations of
significant figures
The essential requirements of physical unit are given below Rounding off
(i) It should be of suitable size. Order of magnitude
(ii) It should be easily accessible. 4 Error in measurement
Expression of errors
(iii) It should not vary with time. Combination of errors
(iv) It should be easily reproducible.
(v) It should not depend on physical conditions like pressure, volume, etc.
18 OBJECTIVE Physics Vol. 1

System of units Fundamental quantities and their SI units


A complete set of units which is used to measure all kinds of SI units
fundamental and derived quantities is called a system of Base
quantity Name/
units. Definition
Symbol
Some of the commonly used systems of units are as follows
Length Metre It is defined by taking the fixed
(i) CGS system In this system, the units of length, mass numerical value of the speed of light in
(m)
and time are centimetre (cm), gram (g) and second (s), vacuum c to be 299792458 when
respectively. The unit of force in this system is dyne expressed in the unit ms −1, where the
and that of work or energy is erg. second is defined in terms of the
caesium frequency ∆νCs.
(ii) FPS system In this system, the units of length, mass and
time are foot (ft), pound (lb) and second (s), respectively. Mass Kilogram It is defined by taking the fixed
(kg) numerical value of the Planck constant h
The unit of force in this system is poundal. to be 6.62607015 × 10−34 when
(iii) MKS system In this system, the units of length, mass expressed in the unit Js, which is equal
and time are metre (m), kilogram (kg) and second (s), to kg-m 2s −1, where the metre and the
respectively. The unit of force in this system is newton second are defined in terms of c and
∆νCs.
(N) and that of work or energy is joule (J).
(iv) International system (SI) This system of units helps in Time Second It is defined by taking the fixed
(s) numerical value of the caesium
revolutionary changes over the MKS system and is frequency ∆νCs, the unperturbed
known as rationalised MKS system. It is helpful to ground state hyperfine transition
obtain all the physical quantities in physics. frequency of the caesium-133 atom, to
be 9192631770 when expressed in the
Note
unit Hz, which is equal to s −1.
(i) The FPS system is not a metric system. This system is not in much use
these days. Electric Ampere It is defined by taking the fixed
(ii) The drawback of CGS system is that many of the derived units on this current (A) numerical value of the elementary
system are inconveniently small. charge e to be 1.602176634 × 10−19
when expressed in the unit C.
Fundamental quantities and Thermo- Kelvin It is defined by taking the fixed
fundamental units dynamic (K) numerical value of the Boltzmann
temperature constant k to be 1.380649 × 10−23 when
Those physical quantities which are independent of each
expressed in the unit JK −1, which is
other and not defined in terms of other physical quantities, equal to kg-m 2s −2K −1, where the
are called fundamental quantities or base quantities. The kilogram, metre and second are defined
units of these quantities are called fundamental or base in terms of h, c and ∆νCs.
units.
Amount of Mole One mole contains exactly
substance (mol) 6 .02214076 × 1023 elementary entities.
Derived quantities and derived units This number is the fixed numerical
The quantities which can be expressed in terms of the value of the Avogadro constant N A ,
fundamental quantities are called derived quantities. The when expressed in the unit mol −1 and is
called the Avogadro number. The
units of these quantities are called derived units. amount of substance, symbol n, of a
e.g. Unit of speed = ms −1 can be derived from fundamental system is a measure of the number of
specified elementary entities. An
units, i.e. unit of length and time as elementary entity may be an atom, a
Distance m molecule, an ion, an electron, any other
Speed = = = ms −1 particle or specified group of particles.
Time s
Luminous Candela It is defined by taking the fixed
Supplementary quantities and intensity (cd) numerical value of the luminous
intensity of monochromatic radiation of
supplementary units frequency 540 × 1012 Hz, K cd, to be 683
Other than fundamental and derived quantities, there are when expressed in the unit lm W −1,
which is equal to cd sr W −1, or cd sr kg
two more quantities called as supplementary quantities. −1 −2 3
m s , where the kilogram, metre and
The units of these quantities are known as supplementary second are defined in terms of h, c and
units. ∆νCs.
Units, Dimensions & Error Analysis 19

Supplementary quantities and their SI units Time


SI units (i) 1 millisecond = 10 −3 s
Supplementary
quantity Name/Symbol Definition
(ii) 1 microsecond = 10 −6 s
(iii) 1 shake = 10 −8 s
One radian is the angle subtended
1 nanosecond = 10 −9 s
Plane angle Radian
(rad) at the centre by an arc equal in (iv)
length to the radius of the circle. (v) 1 picosecond = 10 −12 s
(vi) 1 hour = 60 min = 3600 s
r
ds (vii) 1 day = 24 hours = 86400 s
(viii) 1 year = 365 days = 3.15 × 10 7 s
O dθ
(ix) 1 century = 100 years
ds Common SI Prefixes and Symbols for
i.e. dθ =
r Multiples and Sub-multiples
Solid angle Steradian One steradian is the solid angle Multiple Sub-multiple
(sr) subtended at the centre of a
sphere, by that surface of the Factor Prefix Symbol Factor Prefix Symbol
sphere, which is equal in area, to 18 −18
10 Exa E 10 atto a
the square of radius of the sphere.
15 −15
10 Peta P 10 femto f
12 −12
r 10 Tera T 10 pico p
O dΩ
dA
109 Giga G 10−9 nano n
106 Mega M 10−6 micro µ
3 −3
dA 10 Kilo k 10 milli m
i.e. dΩ =
r2 10 2
Hecto h 10 −2
centi c
1 −1
10 Deca da 10 deci d

Some other units Example 1.1 The acceleration due to gravity is 9.8 ms −2 .
(Not contained in SI units) Give its value in ft s −2 .
Length Sol. As, 1 m = 3.28 ft
(i) 1 micron = 1µm = 10 m −6 ∴ 9.8 ms −2 = 9.8 × 3.28 ft s −2
= 32.14 ft s −2
(ii) 1 nanometre = 1 nm = 10 −9 m
≈ 32 ft s −2
(iii) 1 Angstrom = 1 Å = 10 −10 m = 10 −8 cm = 10 −4 µm
(iv) 1 fermi = 1fm = 10 −15 m Example 1.2 The value of gravitational constant G in MKS
system is 6.67 × 10 −11 N-m 2 kg −2 . What will be its value in
(v) 1 astronomical unit = 1 AU = 1.496 × 10 11 m CGS system?
(vi) 1 light year = 1ly = 9.467 × 10 15 m Sol. Given, G = 6.67 × 10−11 N-m 2 kg −2
(vii) 1 parsec = 3.08 × 10 16 m = 3.26 ly = 206267 AU = 6.67 × 10−11 (kg-ms −2) m 2 kg −2

Mass = 6.67 × 10−11 (m3 ) (s−2 ) (kg −1)

(i) 1 quintal = 100 kg = 6.67 × 10−11 (102 cm)3 (s)−2 (103 g )−1
(ii) 1 tonne or 1metric ton = 1000 kg = 10 quintal = 6.67 × 10−8 cm3 g −1 s −2
(iii) 1 megagram = 10 3 kg = 6.67 × 10−8 dyne-cm2 g −2

(iv) 1 gigagram = 10 6 kg Example 1.3 The wavelength of a light is of the order of


(v) 1 teragram = 10 9 kg 6400 Å. Express this in micron and metre.
(vi) 1 slug = 14.57 kg Sol. As, 1Å = 10−10 m
(vii) 1 pound = 1lb = 0.4536 kg ∴ Wavelength of light = 6400 Å
= 6400 × 10−10 m = 6.4 × 10−7m
20 OBJECTIVE Physics Vol. 1

Also, 1 micron = 10−6 metre Thus, the dimensions of density are 1 in mass and − 3 in
−7 length. The dimensions of all other fundamental quantities
6.4 × 10
∴ Wavelength of light (in micron) = micron are zero.
10−6
= 0.64 micron
Dimensional representation of
Example 1.4 How many microns are there in 1 light year?
physical quantities
Sol. 1 ly = 9.46 × 1015 m For convenience, the fundamental quantities are
As, 1m = 106 micron represented by one letter symbols. The dependence of all
∴ 1 ly = 9.46 × 1015 × 106 micron other physical quantities on these base quantities can be
= 9.46 × 1021 micron expressed in terms of their dimensions.
≈ 1022 micron (approx.) Thus, the seven dimensions of physical quantities are
represented as follows
Example 1.5 How many microseconds are there in
[M] for mass
10 minutes?
[L] for length
Sol. As, 1 second = 106 microseconds
[T] for time
10 minutes = 10 × 60 seconds
[A] for electric current
= 10 × 60 × 106
= 6 × 108 microseconds
[K] or [θ] for thermodynamic
temperature
Example 1.6 Calculate the angle of [cd] for luminous intensity
(i) 1° (degree) [mol] for amount of substance
(ii) 1′ (minute of arc or arc minute) and The physical quantity that is expressed in terms of the
(iii) 1′ ′ (second of arc or arc second) base quantities is enclosed in square brackets.
in radians. (Use 360° = 2π rad, 1° = 60′ and 1′ = 60′ ′)
Thus, from Eq. (i), dimensions of density can be
2π π
Sol. (i) 1° = rad = rad represented as [ML−3 ].
360 180
22 Dimensional formula and
= rad = 1.746 × 10−2 rad
7 × 180
1° 1 π
dimensional equation
(ii) 1 arc min = 1′ = = × rad = 2.91 × 10−4 rad The expression of a physical quantity in terms of its
60 60 180
1′ 1° 1 π dimensions is called its dimensional formula. e.g.
(iii) 1 arc second = 1′ ′ = = = × rad Dimensional formula for density is [ML−3 T 0 ], the
60 60 × 60 60 × 60 180
dimensional formula of force is [MLT −2 ] and that for
= 4.85 × 10−6 rad
acceleration is [M 0 LT −2 ].
DIMENSIONS OF PHYSICAL An equation which contains a physical quantity on one side
and its dimensional formula on the other side, is called the
QUANTITIES dimensional equation of that quantity.
The dimensions of a physical quantity are the powers (or Dimensional equations for a few physical quantities are
exponents) to which the fundamental quantities must be given below
raised to represent that quantity completely. Speed [v ] = [M0 LT −1]
Mass Mass
e.g. Density = = Area [A] = [M0 L2 T 0 ]
Volume (Length) 3
Force [F ] = [MLT −2 ], etc.
−3
or Density = (Mass) (Length) …(i)
The physical quantities having same derived units have
same dimensions.
Units, Dimensions & Error Analysis 21

Dimensional formulae of some physical quantities


The table given below gives the dimensional formulae and SI units of some physical quantities frequently used in physics
S. No. Physical quantity SI units Dimensional formula
1. Velocity = displacement/time m/s [M 0 LT −1]
2. Acceleration = velocity/time m/s 2 [M 0 LT −2]
3. Force = mass × acceleration kg-m/s 2 = newton or N [MLT −2]

4. Work = force × displacement kg-m 2/s 2 = N-m = joule or J [ML2T −2]

5. Energy Joule or J [ML2T −2]


6. Torque = force × perpendicular distance N-m [ML2T −2]
7. Power = work/time J/s or watt [ML2T −3]
8. Momentum = mass × velocity kg-m/s [MLT −1]
9. Impulse = force × time N-s [MLT −1]
10. Angle = arc/radius radian or rad [M 0 L0 T 0 ]
∆L ∆V
11. Strain = or No units [M 0 L0 T 0 ]
L V
12. Stress = force/area N/m 2 [ML−1T −2]`
13. Pressure = force/area N/m 2 [ML−1T −2]
14. Modulus of elasticity = stress/strain N/m 2 [ML−1T −2]
15. Frequency = 1/time period per second or hertz (Hz) [M 0 L0 T −1]
16. Angular velocity = angle/time rad/s [M 0 L0 T −1]
17. Moment of inertia = (mass) × (distance) 2 kg-m 2 [ML2T 0 ]

18. Surface tension = force/length N/m [ML0 T −2]

force × (distance)2
19. Gravitational constant = N-m 2/kg 2 [M −1L3T −2]
(mass)2
20. Angular momentum kg-m 2/s [ML2T −1]
21. Coefficient of viscosity N-s/m 2 [ML−1T −1]
22. Planck's constant J-s [ML2T −1]
23. Specific heat (s) J/kg-K [M 0 L2T −2 θ−1]
24. Coefficient of thermal conductivity (K) watt/m-K [M LT −3 θ−1]

25. Gas constant (R ) J/mol-K [M L2T −2 θ−1mol−1]

26. Boltzmann constant (k ) J/K [ M L2T −2 θ−1]

27. Wien's constant (b) m-K [ M 0 LT 0θ]


28. Stefan's constant (σ) watt/m 2-K 4 [ M L0 T −3 θ−4 ]
29. Electric charge C [ M 0 L0 TA]
30. Electric intensity N/C [ M LT −3A−1]
31. Electric potential volt (V) [ M L2T −3A−1]
32. Capacitance farad (F) [ M −1L−2 T 4A2]
33. Permittivity of free space C 2N −1m −2 [ M –1L–3T 4A2]
22 OBJECTIVE Physics Vol. 1

S. No. Physical quantity SI units Dimensional formula


34. Electric dipole moment C-m [M 0 LTA]
35. Resistance Ohm [ ML2T –3A–2]
36. Magnetic field tesla (T) or weber/m 2 (Wb/m 2) [ ML0 T –2A–1]
37. Coefficient of self-induction henry (H) [ ML2T –2A–2]
38. Magnetic flux Wb (weber) [ ML2T –2A–1]
39. Permeability of free space Hm −1 [ MLT –2A–2]
40. Magnetic moment Am 2 [ M 0 L2T 0A]

Quantities having same dimensions


S. No. Quantities Dimensions
1. Strain, refractive index, relative density, angle, solid angle, phase, distance gradient, relative [M 0 L0 T 0 ]
permeability, relative permittivity, angle of contact, Reynolds number, coefficient of friction,
mechanical equivalent of heat and electric susceptibility.
2. Mass and inertia [M1L0 T 0 ]
3. Momentum and impulse [M1L1T −1]
4. Thrust, force, weight, tension and energy gradient. [M1 L1 T −2]
5. Pressure, stress, Young’s modulus, bulk modulus, shear modulus, modulus of rigidity and energy [M1L−1T −2]
density.
6. Angular momentum and Planck’s constant (h). [M1L2T −1]
1 −2
7. Acceleration, acceleration due to gravity and gravitational field intensity. [M 0 LT ]
8. Surface tension, free surface energy (energy per unit area), force gradient and spring constant. [M1L0 T −2]
9. Latent heat and gravitational potential. [M 0 L2T −2]
10. Thermal capacity, Boltzmann constant and entropy. [ML2T −2θ−1]
11. Work, torque, internal energy, potential energy, kinetic energy, moment of force, (q 2 / C ), (LI 2 ), (qV ), [M1L2T −2]
V 2 
(V 2C ), (I 2Rt ),  t , (VIt ), (pV ), (RT ), (mL) and (mc ∆T ).
R 

12. Frequency, angular frequency, angular velocity, velocity gradient, radioactivity,


 R  ,  1  and  1  . [M 0 L0 T −1]
     
 L   RC   LC 
1/ 2 1/ 2 1/ 2
l R 
,   ,   , (RC ), ( LC ) and time.
m L
13.   ,  [M 0 L0 T1]
 g k   g R 

14. Power (VI ), (I 2R ) and (V 2R ). [ML2 T −3]

Example 1.7 Find the dimensional formulae of where, A is the area, v is the velocity, l is the length, I is the
(i) coefficient of viscosity, η (ii) charge, q electric current, t is the time and U is the energy.
(iii) potential,V (iv) capacitance, C and Sol. F ∆l
(i) η= −
(v) resistance, R A ∆v
Some of the equations containing above quantities are [F ][l ] [MLT−2] [L]
∴ [η] = = = [ML−1 T −1 ]
 ∆v  [A][v ] [L2][LT−1]
F = − ηA   , q = It , U = VIt ,
 ∆l  (ii) q = It
q = CV and V = IR ∴ [q ] = [I] [t ] = [AT]
Units, Dimensions & Error Analysis 23

(iii) U = VIt Example 1.10 In the formula x = 3 yz 2, x and y have


U dimensions of capacitance and magnetic induction
∴ V= respectively, then find the dimensions of y.
It
[U ] [ML2T−2] Sol. Given, x = 3 yz 2
or [ ]=
V = = [ML2T−3A−1] x Capacitance
[I] [t ] [A] [T] ⇒ y= 2 =
3z (Magnetic induction)2
(iv) q = CV
q [M−1 L−2 T4 A2]
∴ C = [ y] =
V [M T−2 A−1]2
[q ] [AT] = [M−3 L−2 T8 A4]
or [C ] = = = [M −1L−2T 4A2]
[ ] [ML2T−3A−1]
V
(v) V = IR Applications of dimensional
V
∴ R=
I
analysis
[V ] [ML2T−3A−1]
The method of studying a physical phenomenon on the
or [R ] = = = [ML2T−3A−2] basis of dimensions is called dimensional analysis.
[I] [A]
The three main uses of a dimensional analysis are
Example 1.8 If C and R denote capacitance and resistance, described in detail in the following section
then find the dimensions of CR.
Sol. The capacitance of a conductor is defined as the ratio of the 1. Checking the dimensional
charge given to the rise in the potential of the conductor, consistency of equation
q q2  W Every physical equation should be dimensionally balanced.
C = = QV = 
V W  q This is called the principle of homogeneity. This
ampere2 -s2 principle states that, the dimensions of each term on both
C = sides of an equation must be the same. On this basis, we
kg-metre2 /s2
can judge whether a given equation is correct or not. But a
Hence, dimensions of C are [M−1L−2T4A2] . dimensionally correct equation may or may not be
From Ohm’s law, V = iR, therefore dimensions of resistance, physically correct.
V Volt 1
R= = e.g. In the physical expression s = ut + at 2 , the
i Ampere 2
= kg-metre2s−3ampere−2 1 2
dimensions of s , ut and at all are same.
Dimensions of R = [ML2T−3A−2] 2
Note The physical quantities separated by the symbols + , − , = , > , <
∴ Dimensions of RC = [ML2T−3A−2][M−1L−2T4A2]
etc., should have the same dimension.
= [M0L0TA0]
Example 1.11 Show that the expression of the time period T
Example 1.9 Which amongst the following quantities is (are)
of a simple pendulum of length l given by T = 2π l / g is
dimensionless?
Work Momentum dimensionally correct.
(i) (ii) sin θ (iii) l
Energy Time Sol. Given, T = 2π
g
Sol. (i) Since, work and energy both have the same
dimensions [ ML2T−2 ], therefore their ratio is a Dimensionally, [T] =
[L]
= [T]
dimensionless quantity. [LT−2]
(ii) sin θ, here θ represents an angle. An angle is the As in the above equation, the dimensions of both sides are
ratio of two lengths, i.e. arc length and radius. same. Therefore, the given expression is dimensionally
Therefore, θ is dimensionless, hence sin θ is
correct.
dimensionless.
−1
 Momentum  MLT  −2 Example 1.12 Check the correctness of the relation
(iii)   =  = [MLT ]
 Time   T  s = ut +
1 2
at , where u is initial velocity, a is the
Hence, the given ratio is not dimensionless. 2
acceleration, t is the time and s is the displacement.
24 OBJECTIVE Physics Vol. 1

Sol. Writing the dimensions of either side of the given relation. Hence, [m 2] = [m1] = [M]
LHS = s = displacement = [M0LT0] Also, the quantity obtained by the addition of m1 and m 2
would have the same dimensions as that of mass.
RHS = ut = velocity × time = [M0LT−1] [T] = [M0LT0]
∴ [m1 + m 2] = [M]
1 2
and at = (acceleration) × (time) 2 Now, going back to Eq. (i),
2 [m + m 2][x]
= [M0LT−2] [T]2 = [M0LT0] [p] = 1
[A]
As LHS = RHS, so the relation is dimensionally correct. [M ][x ]
⇒ [ML−1T −2] = 2
Example 1.13 Write the dimensions of a and b in the relation, [L ]
b − x2 ⇒ [ML−1T −2] = [ML−2][x]
P=
at [ML−1T −2]
where, P is power, x the distance and t the time. ⇒ −2
= [x] ⇒ [x] = [LT −2]
[ML ]
Sol. The given equation can be written as Pat = b − x 2
Hence, the quantity x represents acceleration. In this example,
Now, [Pat ] = [b] = [x 2] or [b] = [x 2] = [M0L2T0] it is the acceleration due to gravity g . (m1 + m 2 ) g represents
[x 2] [L2] the weight exerted by two masses m1, m 2 on the area A.
and [a ] = = 2 −3
= [M−1L0T2]
[Pt ] [ML T ] [T]
2. To convert a physical quantity from one
Example 1.14 The velocity v of a particle depends upon the system of units to other system of units
c
time t according to the equation v = a + bt + ⋅ Write This is based on the fact that the product of the numerical
d +t value (n) and its corresponding unit (u) is a constant, i.e.
the dimensions of a, b, c and d.
Sol. From principle of homogeneity, n (u ) = constant or n 1[u1] = n 2 [u 2 ]
[a ] = [v ] or [a ] = [LT−1] Suppose the dimensions of a physical quantity are a in
[bt ] = [v ] mass, b in length and c in time. If the fundamental units in
one system are M1, L 1 and T1 and in the other system are
[v ] [LT−1]
or [b] = = or [b] = [LT−2] M2 , L 2 and T 2 , respectively. Then, we can write
[t ] [T]
n 1 [M1a Lb1 T1c ] = n 2 [M2a Lb2 T 2c ] ...(i)
Similarly, [d ] = [t ] = [T]
a b c
Further,
[c ]
= [v ] or [c ] = [v ] [d + t ] u1 M  L  T 
[d + t ] n 2 = n1 = n1  1   1   1 
u2  M2   L 2   T 2 
or [c ] = [LT−1] [T]
Here, n 1 and n 2 are the numerical values in two system of
or [c ] = [L] units, respectively. Using Eq. (i), we can convert the
∴ Dimensions of a = [LT−1] numerical value of a physical quantity from one system of
Dimensions of b = [LT−2] units into the other system.
Dimension of c = [L] Example 1.16 Find the value of 100 J on a system which has
Dimension of d = [T] 20 cm, 250 g and half minute as fundamental units of
length, mass and time.
Example 1.15 The following equation gives a relation between
Sol. The dimensional formula of work is = [ML2T−2]
the mass m1, kept on a surface of area A and the pressure p
exerted on this area To convert a physical quantity from one system of units to
(m1 + m 2 ) x other system of units, we use the following formula
p= a b c
A  M   L  T 
n 2 = n1 1   1   1 
What must be the dimensions of the quantities x and m 2?  M 2   L 2   T2 
Sol. Since, all the terms of a mathematical equation should have 1 2 −2
 1 kg   1m   1s 
the same dimensions. n 2 = 100   20 cm  0.5 min 
 (m + m 2 ) x  250 g 
Therefore, [p] =  1  ...(i) 1 2 −2
 A 1000 g  100 cm  1 s 
= 100      
Only the quantities having same dimensions and nature can  250 g   20 cm  30 s 
be added to each other.
= 100 × 4 × 25 × 30 × 30 = 9 × 106 new units
Here, m 2 is added to mass m1.
Units, Dimensions & Error Analysis 25

Example 1.17 The value of gravitational constant is Substituting these values in Eq. (i), we get
G = 6.67 × 10 N-m /kg in SI units. Convert it into
–11 2 2
k F
f = k (F )1/ 2 (l )−1(µ )− 1/ 2 or f =
CGS system of units. l µ
Sol. The dimensional formula of G is [M −1L3T −2]. 1
Experimentally, the value of k is found to be ⋅
To convert a physical quantity from one system of units to 2
other system of units, we use the following formula 1 F
Hence, f=
n1[M1−1L13 T1−2] = n 2[M 2−1 L32 T2−2] 2l µ
−1 3 −2
 M1   L1   T1 
n 2 = n1   L  T  Example 1.19 The centripetal force F acting on a particle
 M2   2  2 moving uniformly in a circle may depend upon mass (m),
−1 3 −2 velocity (v) and radius (r) of the circle. Derive the formula for
 1 kg   1 m  1 s
= 6.67 × 10−11  –3  10–2 m 1 s
F using the method of dimensions.
10 kg  Sol. Let F = k (m )x (v )y (r )z …(i)
−8
or n 2 = 6.67 × 10 Here, k is a dimensionless constant of proportionality.
Thus, value of G in CGS system of units is Writing the dimensions of RHS and LHS in Eq. (i), we have
6.67 × 10−8 dyne cm2 / g 2. [MLT−2] = [M]x [LT−1]y [L]z
= [Mx Ly +z
T −y ]
3. Deducing relation between the
Equating the powers of M, L and T on both sides, we have
physical quantities x = 1, y = 2 and y + z = 1
If we know the factors on which a given physical or z =1− y = −1
quantity depends, we can find a formula relating to Putting the values in Eq. (i), we get
those factors.
mv 2
F = kmv 2r −1 = k
Example 1.18 The frequency (f ) of a stretched string r
depends upon the tension F (dimensions of force), length l mv 2
of the string and the mass per unit length µ of string. F = (where, k = 1)
r
Derive the formula for frequency.
Sol. Suppose, the frequency f depends on the tension raised to
the power a, length raised to the power b and mass per unit
Defects or limitations of
length raised to the power c. dimensional analysis
Then, f ∝ (F )a (l )b ( µ )c The method of dimensional analysis has the following
limitations
or f = k (F ) (l ) (µ )
a b c
…(i)
(i) The value of dimensionless constant involved in a
Here, k is a dimensionless constant of proportionality.
formula cannot be deduced from this method.
Thus, [f ] = [F]a [l ]b [µ ]c
(ii) By this method, the equation containing
or [M0 L0T−1] = [MLT−2]a [L]b [ML−1]c trigonometrical, exponential and logarithmic terms
or [M0 L0T−1] = [Ma + cLa + b − c T−2a ] cannot be analysed.
For dimensional balance, the dimensions on both sides (iii) This method does not work when physical quantity
should be same. depends on more than three variables because we only
Thus, a +c = 0 …(ii) have three equations by equalising the power of M, L
a +b −c = 0 …(iii)
and T.
and − 2a = − 1 …(iv) (iv) If dimensions are given, physical quantity may not be
unique. e.g. Work, energy and torque all have the
Solving these three equations, we get
1 1
same dimensional formula [ML2 T −2 ].
a= , c=−
2 2 (v) It gives no information whether a physical quantity is
and b = −1 a scalar or a vector.
OBJECTIVE Physics Vol. 1

CHECK POINT 1.1


1. In the SI system, the unit of temperature is 13. The dimensional formula for Planck’s constant and angular
(a) degree centigrade momentum is
(b) kelvin (a) [ML2T −2] and [MLT −1] (b) [ML2T −1] and [ML2T −1]
(c) degree celsius (c) [ML3T −1] and [ML2T −2] (d) [MLT −1] and [MLT −2]
(d) degree Fahrenheit
1
2. Which amongst the following is not equal to watt? 14. The dimensions of ε 0 E 2(ε 0 is the permittivity of the space
2
(a) joule/second (b) ampere × volt and E is electric field), are
(c) (ampere)2 × ohm (d) ampere/volt (a) [ML2T −1] (b) [ML−1T −2] (c) [ML2T −2] (d) [MLT −1]
3. Joule × second is the unit of 15. The units of length, velocity and force are doubled. Which of
(a) energy (b) momentum the following is the correct change in the other units?
(c) angular momentum (d) power (a) Unit of time is doubled
(b) Unit of mass is doubled
4. Which amongst the following pairs has the same units? (c) Unit of momentum is doubled
(a) Wavelength and Rydberg constant (d) Unit of energy is doubled
(b) Relative velocity and relative density
(c) Thermal capacity and Boltzmann constant t 
16. Given that y = a cos  − qx , where t represents time and x
(d) Time period and acceleration gradient p 
5. Density of liquid in CGS system is 0.625 g cm−3. What is its represents distance; which amongst the following
magnitude in SI system? statements which is(are) true?
(a) 0.625 (b) 0.0625 (a) The unit of x is same as that of q
(c) 0.00625 (d) 625 (b) The unit of x is same as that of p
(c) The unit of t is same as that of q
6. Dimensions of surface tension are
(d) The unit of t is same as that of p
(a) [M 2L2T −2] (b) [M 2LT −2]
(c) [MT −2] (d) [MLT −2] a a − t2
17. The dimensions of in the equation p = , where p is
b bx
7. The dimensions of impulse are equal to that of pressure, x is distance and t is time, are
(a) force (b) linear momentum (a) [M 2LT −3] (b) [MT −2]
(c) pressure (d) angular momentum
(c) [LT −3] (d) [ML3T −1]
8. Which of the following does not possess the same
x 
dimensions as that of pressure? 18. The equation of a wave is given by y = a sin ω  − k
(a) Stress (b) Bulk modulus v 
(c) Thrust (d) Energy density where, ω is angular velocity and v is the linear velocity. The
dimensions of k will be
9. What is the dimensional formula of gravitational constant?
(a) [T −2] (b) [T −1]
(a) [ML2T −2] (b) [ML−1T −1]
(c) [T] (d) [LT]
(c) [M −1L3T −2] (d) None of these
19. If ‘muscle times speed equals power’, then what is the ratio
10. Which one of the following have same dimensions? of the SI unit and the CGS unit of muscle?
(a) Torque and force (a)105 (b)103
(b) Potential energy and force (c)107 (d)10−5
(c) Torque and potential energy
(d) Planck’s constant and linear momentum
20. If p represents radiation pressure, c represents speed of light
and Q represents radiation energy striking a unit area per
11. The force F on a sphere of radius a moving in a medium second, then for what values of non-zero integers x , y and z,
with velocity v is given by F = 6π ηa v. The dimensions of η p x Q y c z is dimensionless?
are
(a) x = 1, y = 1, z = − 1 (b) x = 1, y = − 1, z = 1
(a) [ML−3] (b) [MLT −2]
(c) x = − 1, y = 1, z = 1 (d) x = 1, y = 1, z = 1
(c) [MT −1] (d) [ML−1T −1]
21. Assuming that the mass m of the largest stone that can be
12. The dimensional representation of specific resistance in moved by a flowing river depends upon the velocity v of the
terms of charge Q is water, its density ρ and the acceleration due to gravity g.
(a) [ML3T −1Q−2] (b) [ML2T −2Q2] Then, m is directly proportional to
−2 −1
(c) [MLT Q ] (d) [ML2T −2Q−1] (a) v3 (b) v4
(c) v5 (d) v6
Units, Dimensions & Error Analysis 27

SIGNIFICANT FIGURES The significant figures are three in each measurement,


The significant figures are normally those digits in a because all zeros appearing in the base number in the
measured quantity which are known reliable or about scientific notation (in the power of 10) are not
which we have confidence in our measurement plus one significant.
additional digit that is uncertain. Rule 7 A choice of change of different units does not
e.g. If length of some object is 185.2 cm, then it has four change the number of significant digits or figures in a
significant figures. The digits 1,8 and 5 are reliable and digit measurement.
2 is uncertain. e.g. The length 7.03 cm has three significant figures. But
Note Significant figures indicate the precision of the measurement in different units, the same value can be written as
which depends on the least count of the measuring instrument.
0.0703 m or 70.3 mm. All these measurements have the
same number of significant figures (digits 7, 0 and 3)
Rules to determine significant namely three.
figures Rule 8 The exact numbers appearing in the
For determining number of significant figures, we use the mathematical formulae of various physical quantities
following rules have infinite number of significant figures. e.g.
Perimeter of a square is given by 4 × side. Here, 4 is an
Rule 1 All non-zero digits are significant, e.g. x = 2567 has
exact number and has infinite number of significant
four significant figures.
figures.
Rule 2 The zeros appearing between two non-zero digits ∴ It can be written as 4.0, 4.00, 4.0000 as per the
are significant, no matter where the decimal point is, if at requirement.
all, e.g. 6.028 has 4 significant figures.
Some significant figures of measured values given in the
Rule 3 If the number is less than 1, the zero(s) on the right table below
of decimal point but to the left of first non-zero digit are not
significant. Measured values
Number of significant
Rule
figures
e.g. 0.0042 has two significant digits.
Rule 4 The terminal or trailing zero(s) in a number without 12376 5 1
a decimal point are not significant. Thus, 426 m = 42600 cm 6024.7 5 2
= 426000 mm has three significant figures.
0.071 2 3
Rule 5 In a number with decimal, zeros to the right of last
non-zero digit are significant. 410 2 4

e.g. 4.600 or 0.002300 have four significant figures each. 2.40 3 5

1.6 × 10 10
2 6
Point of confusion and its remedy
Suppose we change the units of a physical quantity, then we ln 2 (l + b ), digit 2 Infinite 8
will write
Example 1.20 How many significant figures are there in the
2.30 m = 230 cm = 2300 mm = 0.00230 km
following measured values?
When we are considering 2300 mm, then from Rule-4, we (i) 227.2 g (ii) 3600 g
would conclude erroneously that the number has two
(iii) 0.00602 g (iv) 2.50 × 1010 g
significant figures, while in fact it has three significant
figures and a mere change of units cannot change the Sol. (i) 227.2 g has all the non-zero digits. Hence, it has four
number of significant figures. significant figures.
To remove such ambiguities in determining the number of (ii) According to rule number 4, trailing zeros are not
significant. Hence, 3600 g has 2 significant figures.
significant figures, apply following rules
(iii) According to the rule number 3, the zeros on the right
Rule 6 The power of 10 is irrelevant to the determination of decimal point but to the left of first non-zero digit
of significant figures. e.g. In the measurements, are not significant. Hence, 0.00602 g has 3 significant
2.30 m = 2.30 × 10 2 cm = 2.30 × 10 3 mm figures.
(iv) According to the rule number 6, it has 3 significant
= 2.30 × 10 −3 km figures.
28 OBJECTIVE Physics Vol. 1

As 1100 has minimum number of significant figures


Mathematical operations of (i.e. 2), therefore the result should also contain only two
significant figures significant digits. Hence, the result when rounded off to
The result of a mathematical operation involving measured two significant digits becomes 110.
values of quantities cannot be more accurate than the Example 1.23 The voltage across a lamp is 6.32V when the
measured value themselves. current passing through it is 3.4 A. Find the power
So, certain rules have to be followed while doing consumed upto appropriate significant figures.
mathematical operations with significant figures, so that Sol. Voltage across a lamp,V = 6.32 V (3 significant figures)
precision in final result is consistent with the precision of Current flowing through lamp, I = 3.4A (2 significant figures)
the original measured values. ∴ Power consumed, P = VI = (6.32)(3.4) = 21.488 W
Answer should have minimum number of significant figures.
Addition or subtraction Here, the minimum number of significant figures is 2.
Suppose in the measured values to be added or subtracted, ∴ Power consumed = 21W
the least number of significant digits after the decimal is n.
Then, in the sum or difference also, the number of Example 1.24 A thin wire has a length of 21.7 cm and radius
significant digits after the decimal should be n. 0.46 mm. Calculate the volume of the wire upto correct
significant figures.
e.g. 1.2 + 3.45 + 6.789 = 11.439 ≈ 11.4
Sol. Given, l = 21.7 cm, r = 0.46 mm = 0.046 cm
Here, the least number of significant digits after the
22
decimal is one. Hence, the result will be 11.4 (when Volume of wire,V = πr 2l = (0.046)2 (21.7)
rounded off to smallest number of decimal places). 7
Similarly, e.g. 12.63 − 10.2 = 2.43 ≈ 2.4 = 0.1443 cm3 −~ 0.14 cm3

Example 1.21 Add 6.75 × 10 3 cm to 4.52 × 10 2 cm with Example 1.25 The time taken by a pendulum to complete
regard to significant figures. 25 vibrations is 88.0 s. Find the time period of the pendulum
Sol. Let a = 6.75 × 10 cm, b = 4.52 × 10 cm
3 2 in seconds upto appropriate significant figures.
Total time taken
= 0.452 × 103 cm = 0.45 × 103 cm Sol. Time period of oscillation =
Number of oscillations
(upto 2 places of decimal)
88.0
∴ Addition of significant figures = s = 3.52 s
25
a + b = (6.75 × 103 + 0.45 × 103 ) cm = 7.20 × 103 cm Out of the two quantities given in the data, 25 is exact, hence
has infinite significant figures. Therefore, the answer should
Example 1.22 Two sticks of lengths 12.132 cm and 10.2 cm be reported to three significant figures, i.e. 3.52 s.
are placed end to end. Find their total length with due
regard to significant figures. Example 1.26 5.74 g of substance occupies 1.2 cm 3 . Express
Sol. Length of first stick = 12.132 cm (5 significant figures) its density by keeping the significant figures in view.
Length of second stick = 10.2 cm (3 significant figures) Sol. Here, mass, m = 5.74 g, volume,V = 1.2 cm3
∴ Total length of two sticks = 12132
. + 10.2 = 22.332 mass 5.74 g
As density, ρ= = = 4.783 g cm−3
The answer should be rounded off with least number of volume 1.2 cm3
significant digits after the decimal. As mass has 3 significant digits and volume has 2 significant
∴ Total length of two sticks will be 22.3 cm. digits, therefore as per rule, density will have only two
significant digits, rounding off, we get ρ = 4.8 gcm−3.
Multiplication or division
Suppose in the measured values to be multiplied or Rounding off
divided, the least number of significant digits be n, then in The process of omitting the non-significant digits and
the product or quotient, the number of significant digits retaining only the desired number of significant digits,
should also be n. incorporating the required modifications to the last
e.g. 1.2 × 36.72 = 44.064 ≈ 44 significant digit is called rounding off the number.
The least number of significant digits in the measured In physics, calculation is a vital part and during that we
values are two. Hence, the result when rounded off to two shall reduce the number to the required extent and that is
significant digits become 44. Therefore, the answer is 44. why there is a need to round off numbers.
1100 Like mathematical operations of significant figures,
Similarly, e.g. = 107.8431373 ≈ 110 rounding off numbers also follow certain rules.
10.2
Units, Dimensions & Error Analysis 29

(iii) 22.78 : Since, the digit to be dropped is 8 and is greater


Rules for rounding off a measurement than 5, therefore the preceding digit 7, is raised by 1.
Following are the rules for rounding off a measurement Hence, 22.8.
Rule 1 If the number lying to the right of cut-off digit is less (iv) 36.35 : Since, the digit to be dropped is 5 and the
than 5, then the cut-off digit is retained as such. However, if preceding digit 3 is odd, we can write the answer as 36.4.
it is more than 5, then the cut-off digit is increased by 1. Example 1.28 The length and the radius of a cylinder measured
e.g. x = 6.24 is rounded off to 6.2 to two significant digits with slide callipers are found to be 4.54 cm and 1.75 cm,
and x = 5.328 is rounded off to 5.33 to three significant respectively. Calculate the volume of the cylinder.
digits. Sol. Length of cylinder, h = 4.54 cm (3 significant figures)
Rule 2 If the insignificant digit to be dropped is 5, then the Radius of cylinder, r = 1.75 cm (3 significant figures)
rule is ∴ Volume of cylinder = πr 2h = 3.14 × (1.75)2 × 4.54 cm3
(i) if the preceding digit is even, the insignificant digit = 43.657775 cm3 = 43.6 cm3
is simply dropped.
(Rounded off upto 3 significant figures)
(ii) if the preceding digit is odd, the preceding digit is
raised by 1.
e.g. x = 6.265 is rounded off to x = 6.26 to three Order of magnitude
significant digits and x = 6.275 is rounded off to x = 6.28 Any physical quantity can be expressed in the form of
to three significant digits. a × 10 b (in terms of magnitude), where a is a number lying
Rule 3 The exact numbers like π, 2, 3 and 4, etc., that between 1 and 10; and b is any negative or positive
appear in formulae and are known to have infinite exponent of 10, then the exponent b is called the order of
significant figures, can be rounded off to a limited number magnitude of the physical quantity. And the expression of a
of significant figures as per the requirement. quantity as a × 10 b is called scientific notation.
Example 1.27 Round off the following numbers upto three e.g. The speed of light is given as 3.00 × 10 8 m/s. So, the
significant figures. order of magnitude of the speed of light is 8.
(i) 2.520 (ii) 4.645 (iii) 22.78 (iv) 36.35 The order of magnitude gives an estimate of the magnitude
Sol. (i) 2.520 : Since, 0 is less than 5, preceding digit is left of the quantity. The charge on an electron is 16 . × 10 −19 C.
unchanged. Hence, 2.52. Therefore, we can say that the charge possessed by an
(ii) 4.645 : Since, the digit to be dropped is 5 and the electron is of the order 10 −19 or its order of magnitude
preceding digit 4 is even. Hence, 4.64. is −19.

CHECK POINT 1.2


1. What is the number of significant figures in 0.0310 × 10 3? 6. Multiply 107.88 by 0.610 and express the result with correct
(a) 2 (b) 3 number of significant figures.
(c) 4 (d) 6 (a) 65.8068 (b) 64.807 (c) 65.81 (d) 65.8

2. The number of significant figures in 11.118 × 10 − 6 V is 7. The length, breadth and thickness of rectangular sheet of
(a) 3 (b) 4 metal are 4.234 m, 1.005 m and 2.01 cm, respectively. The
(c) 5 (d) 6 volume of the sheet upto correct significant figures is
(a) 0.0855 m3 (b) 0.086 m3 (c) 0.08556 m3 (d) 0.08 m3
3. In which of the following numerical values, all zeros are
significant? 8. The radius of a thin wire is 0.16 mm. The area of
(a) 0.2020 (b) 20.2 cross-section of the wire (in mm2) with correct number of
(c) 2020 (d) None of these
significant figures is
(a) 0.08 (b) 0.080
4. What is the number of significant figure in (c) 0.0804 (d) 0.080384
(3.20 + 4.80) × 10 5? 9. When 97.52 is divided by 2.54, the correct result
(a) 5 (b) 4 (considering significant figures) is
(c) 3 (d) 2 (a) 38.3937 (b) 38.394
(c) 65.81 (d) 38.4
5. Subtract 0.2 J from 7.26 J and express the result with
correct number of significant figures. 10. What is the order of magnitude of [(5.0 × 10 −6 ) (5.0 × 10 −8 )]
(a) 7.1 (b) 7.06 with due regards to significant digits?
(c) 7.0 (d) None of these (a) − 14 (b) − 15 (c) + 15 (d) + 14
30 OBJECTIVE Physics Vol. 1

ERROR IN MEASUREMENT
We use different kinds of instruments for measuring Causes of systematic errors
various quantities. However, these measurements Few causes of systematic errors are as follows
always has a degree of uncertainty related to it. This
(i) Instrumental errors may be due to erroneous
uncertainty is called as error in the measurement. Thus,
instruments. These errors can be reduced by using
the difference between the measured value and the true
more accurate instruments and applying zero
value of a quantity is known as the error of
correction, when required.
measurement.
(ii) Sometimes errors arise on account of ignoring certain
∴ Error = True value − Measured value facts. e.g. In measuring time period of simple
Errors may arise from different sources and are usually pendulum, error may creap because no consideration is
classified as follows taken of air resistance. These errors can be reduced by
applying proper corrections to the formula used.
1. Systematic errors (iii) Change in temperature, pressure, humidity, etc., may
These are the errors whose causes are known to us. also sometimes cause errors in the result. Relevant
They can be either positive or negative. corrections can be made to minimise their effects.
One of the common source of systematic errors is as 2. Random errors
follows
The errors which occur irregularly and at random, in
Instrumental errors magnitude and direction are called random errors. The causes
These errors are due to imperfect design or erroneous of random errors are not known. Hence, it is not possible to
manufacture or misuse of the measuring instrument. remove them completely. These errors may arise due to a
These are of following types variety of reasons.
(i) Zero error If the zero mark of vernier scale does e.g. The reading of a sensitive beam balance may change by
not coincide with the zero mark of the main scale, the vibrations caused in the building, due to persons moving
the instrument is said to have zero error. A metre in the laboratory or vehicles running nearby. The random
scale having worn off zero mark also has zero error can be minimised by repeating the observation a large
error. number of times and taking the arithmetic mean of all the
observations. The mean value would be very close to the
(ii) Least count or permissible error This error is most accurate reading.
due to the limitation imposed by the least count of
the measuring instrument. It is an uncertainty Example 1.29 In a vernier callipers, 1 main scale reading is
associated with the resolution of the measuring 1 mm and 9th main scale division coincide with 10th vernier
instrument. scale. Find the least count of vernier.
Note Least Count (LC ) of Sol. Given, 1 main scale reading or division (MSD)
(i) Vernier callipers =
Value of 1 MSD = 1 mm
Number of divisions 9 MSD = 10 VSD
or LC = 1 MSD − 1VSD on vernier scale 9 9 9
Pitch ⇒ 1 VSD = MSD = ×1= mm
(ii) Screw gauge = 10 10 10
Number of divisions on circular scale
9 1
∴ LC = 1 − = mm or 0.1 mm
(iii) Constant error The errors which affect each 10 10
observation by the same amount are called
constant errors. Such errors are due to faulty
calibration of the scale of the measuring
Expression of errors
instrument. Errors can be expressed in following way
(iv) Backlash error Backlash error occurs in screw (i) Absolute error The difference between the true value
gauge, when we try to rotate the screw very fast and the measured value of a quantity is called an
to measure a reading. Due to this, there is some absolute error. Usually the mean value am is taken as
slipping between the different screws instead of the true value. So, if
the rotation, which gives an incorrect reading. To a1 + a 2 + … + an 1n
avoid this we should rotate the screw slowly in am = = ∑ ai
n n i =1
only one direction.
Units, Dimensions & Error Analysis 31

Then by definition, absolute errors in the measured Thus, x = 2.48 ± 0.01 m


values of the quantity are ∆x mean
Percentage error, δx = × 100
∆a1 = a1 − am x
∆a 2 = a 2 − am =
0.01
× 100 = 0.40%
M M M 2.48
∆an = an − am Example 1.31 The diameter of a wire as measured by a
Absolute error may be positive or negative. screw gauge was found to be 2.620 cm, 2.625 cm,
2.630 cm, 2.628 cm and 2.626 cm. Calculate
Mean absolute error It is the arithmetic mean of (i) mean value of diameter,
the magnitudes of absolute errors. Thus, (ii) absolute error in each measurement,
|∆a1|+ | ∆a 2 |+ … + |∆an | 1 n (iii) mean absolute error,
∆a mean = = ∑ | ∆a i |
n n i =1 (iv) fractional error,
Thus final result of measurement can be written as (v) percentage error and
(vi) express the result in terms of percentage error.
a = am ± ∆a mean Sol. (i) Mean value of diameter,
This implies that value of a is likely to lie between 2.620 + 2.625 + 2.630 + 2.628 + 2.626
am =
am + ∆a mean and am − ∆a mean . 5
= 2.6258 cm = 2.626 cm
(ii) Relative or fractional error The ratio of mean (rounding off to three decimal places)
absolute error to the mean value of the quantity (ii) Taking a m as the true value, the absolute errors in
measured is called relative or fractional error. different observations are
∆a mean ∆a1 = 2.620 − 2.626 = − 0.006 cm
Thus, Relative error = ∆a 2 = 2.625 − 2.626 = − 0.001 cm
am
∆a 3 = 2.630 − 2.636 = + 0.004 cm
(iii) Percentage error When the relative error is ∆a 4 = 2.628 − 2.626 = + 0.002 cm
expressed in percent, it is called percentage error. It ∆a 5 = 2.626 − 2.626 = 0.000 cm
is denoted by δa.
(iii) Mean absolute error,
∆a mean | ∆a1| + | ∆a 2 | + | ∆a 3 | + | ∆a 4 | + | ∆a 5 |
Thus, δa = × 100% ∆a mean =
am 5
0.006 + 0.001 + 0.004 + 0.002 + 0.000
=
Example 1.30 The length of a rod as measured in an 5
experiment is found to be 2.48 m, 2.46 m, 2.49 m, 2.49 m = 0.0026 = 0.003
and 2.46 m. Find the average length, the absolute error in
(rounding off to three decimal places)
each observation and the percentage error.
∆a mean 0.003
Sol. Average length = Arithmetic mean of the measured (iv) Fractional error = ± =± = ± 0.001
am 2.626
values
2.48 + 2.46 + 2.49 + 2.49 + 2.46 12.38 (v) Percentage error = ± 0.001 × 100 = ± 0.1%
x mean = = = 2.476 m (vi) Diameter of wire can be written as
5 5
∴ True value, x mean = 2.48 m d = 2.626 cm ± 0.1%
Absolute errors in various measurements, Example 1.32 The refractive index (n) of glass is found to
| ∆x1| = | x1 − x mean | = |2.48 − 2.48| = 0.00 m have the values 1.49, 1.50, 1.52, 1.54 and 1.48. Calculate
| ∆x 2 | = |2.46 − 2.48| = 0.02 m (i) the mean value of refractive index,
| ∆x 3 | = |2.49 − 2.48| = 0.01 m (ii) absolute error in each measurement,
| ∆x 4 | = |2.49 − 2.48| = 0.01 m (iii) mean absolute error,
| ∆x 5 | = |2.46 − 2.48| = 0.02 m (iv) fractional error and
| ∆x1| + | ∆x 2| + | ∆x 3| + | ∆x 4| + | ∆x 5| (v) percentage error.
Mean absolute error =
5 Sol. (i) Mean value of refractive index,
(0.00 + 0.02 + 0.01 + 0.01 + 0.02) 0.06 1.49 + 1.50 + 1.52 + 1.54 + 1.48
= = = 0.012 nm =
5 5 5
∆x mean = 0.01 m = 1.506 = 1.51
(rounded off to two decimal places)
32 OBJECTIVE Physics Vol. 1

(ii) Taking n m as the true value, the asbolute errors in 2. Error in product
different observations are
Let x = ab
∆n1 = 1.49 − 1.51 = − 0.02
∆n 2 = 1.50 − 1.51 = − 0.01 Then, (x ± ∆x ) = (a ± ∆a ) (b ± ∆b )
∆n 3 = 1.52 − 1.51 = + 0.01  ∆x   ∆a   ∆b 
or x 1 ±  = ab 1 ±  1 ± 
∆n 4 = 1.54 − 1.51 = + 0.03  x   a   b 
∆n 5 = 1.48 − 1.51 = − 0.03 ∆x ∆b ∆a ∆a ∆b
(iii) Mean absolute error, or 1± = 1± ± ± ⋅ (Q x = ab )
x b a a b
| ∆n1 | + | ∆n 2 | + | ∆n 3 | + | ∆n 4 | + | ∆n 5 |
∆n mean = ∆x ∆a ∆b ∆a ∆b
5 or ± =± ± ± ⋅
0.02 + 0.01 + 0.01 + 0.03 + 0.03 x a b a b
= = 0.02 ∆a ∆b
5 Here, ⋅ is a very small quantity, so can be
± ∆n mean ± 0.02 a b
(iv) Fractional error = = = ± 0.0132
nm 1.51 neglected.
(v) Percentage error = (± 0.0132 × 100) = ± 1.32% ∆x ∆a ∆b
Hence, ± =± ±
x a b
Combination of errors ∆x  ∆a ∆b   ∆a ∆b 
Possible values of are  + ,  − ,
Most of our experiments involves the measurement of various x  a b   a b 
physical quantities. We then put these measurements in  ∆a ∆b   ∆a ∆b 
− +  and  − − .
appropriate formula, to calculate the required quantity.  a b   a b 
Therefore, we must know how the errors in all the
Hence, maximum possible value of
measurements combine and appear in the final quantity.
∆x  ∆a ∆b 
= ± + 
1. Error in sum or difference x  a b 
Let x = a ± b
Therefore, maximum fractional error in product of two
Further, let ∆a be the absolute error in the measurement (or more) quantities is equal to sum of fractional errors in
of a, ∆b be the absolute error in the measurement of b and the individual quantities.
∆x be the absolute error in the measurement of x.
Then, x + ∆x = (a ± ∆a ) ± (b ± ∆b ) 3. Error in division
= (a ± b ) ± (± ∆a ± ∆b ) a
Let x =
= x ± (± ∆a ± ∆b ) or ∆x = ± ∆a ± ∆b b
The four possible values of ∆x are (∆a − ∆b ), (∆a + ∆b ), a ± ∆a
Then, x ± ∆x =
(− ∆a − ∆b ) and (− ∆a + ∆b ). Therefore, the maximum b ± ∆b
absolute error in x is ∆a 

a 1 ± 
∆x = ± ( ∆a + ∆b )  ∆x   a 
or x 1 ±  =
i.e. The maximum absolute error in sum or difference of  x   ∆b 
b 1 ± 
two quantities is equal to sum of the absolute errors in the  b 
individual quantities. −1
 ∆x   ∆a   ∆b   a
Example 1.33 The volumes of two bodies are measured to be or 1 ±  = 1 ±  1 ±  Q x = 
V1 = (10.2 ± 0.02) cm 3 andV2 = (6.4 ± 0.01) cm 3 .  x   a   b   b
Calculate the sum and difference in volumes with error ∆b
As << 1, so expanding binomially, we get
limits. b
Sol. Given,V1 = (10.2 ± 0.02) cm3 and V2 = (6.4 ± 0.01) cm3  ∆x   ∆a   ∆b 
1 ±  = 1 ±  1 + 
∆V = ± (∆V1 + ∆V2) = ± (0.02 + 0.01) cm3 = ± 0.03 cm3  x   a  b 
V1 + V2 = (10.2 + 6.4) cm3 = 16.6 cm3 ∆x ∆a ∆b ∆a ∆b
or 1± = 1± + ± ⋅
and V1 − V2 = (10.2 − 6.4) cm = 3.8 cm
3 3
x a b a b
Hence, sum of volumes = (16.6 ± 0.03) cm3 ∆a ∆b
Here, ⋅ is a very small quantity, so can be neglected.
and difference of volumes = (3.8 ± 0.03) cm3 a b
Units, Dimensions & Error Analysis 33

∆x ∆a ∆b Example 1.35 The radius of sphere is measured to be


Hence, ±
=± +
x a b ( 2.1 ± 0.5) cm. Calculate its surface area with error limits.
∆x  ∆a ∆b   ∆a ∆b   22
are  − + Sol. Surface area, S = 4πr 2 = (4)   (2.1)2
Possible values of ,  ,  7
x  a b   a b 
 ∆a ∆b   ∆a ∆b  = 55.44 = 55.4 cm2
− −  and  − +  . Therefore, the maximum ∆S ∆r  ∆r 
 a b   a b  Further, =2 or ∆S = 2   (S )
value of S r  r 

∆x  ∆a ∆b  2 × 0.5 × 55.4
= ± +  = = 26.38 = 26.4 cm2
x  a b  2.1
∴ S = (55.4 ± 26.4) cm2
Therefore, the maximum value of fractional error in
division of two (or more) quantities is equal to the sum of Example 1.36 The mass and density of a solid sphere are
fractional errors in the individual quantities. measured to be (12.4 ± 0.1) kg and (4.6 ± 0.2) kg m –3 .
Calculate the volume of the sphere with error limits.
Example 1.34 Calculate focal length of a spherical mirror Sol. Here, m ± ∆m = (12.4 ± 0.1) kg
from the following observations. Object distance
u = (50.1 ± 0.5) cm and image distance v = (20.1 ± 0.2) cm. and ρ ± ∆ρ = (4.6 ± 0.2) kgm −3
Sol. Formula for focal length of a spherical mirror, m 12.4
Volume, V = = = 2.69 m3 = 2.7 m3
1 1 1 ρ 4.6
= + …(i)
f v u (rounding off to one decimal place)
uv (50.1) (20.1) ∆V  ∆ m ∆ ρ
or f= = = 14.3 cm Now, =± + 
u + v (50.1 + 20.1) V  m ρ
On differentiating Eq. (i), we get  ∆ m ∆ ρ
or ∆V = ±  +  ×V
∆f ∆u ∆v  m ρ
= + 2
f 2 u2 v
 0.1 0.2
=±  +  × 2.7 = ± 0.14
f2 f2 12.4 4.6
or ∆f = × ∆u + ∆v
u2 v2 ∴ V ± ∆V = (2.7 ± 0.14) m3
2 2
14.3 14.3
=  × 0.5 +   × 0.2 Example 1.37 A thin copper wire of length L increase in
.
 501 .
 201
length by 2% when heated from T1 to T 2 . If a copper cube
= 0.0407 + 01012
. having side 10 L is heated from T1 to T 2 , what will be the
= ± 01419
. cm −~ ± 01
. cm percentage change in
∴ f = (14.3 ± 01
. ) cm (i) area of one face of the cube and
(ii) volume of the cube ?
4. Error in quantity raised to some power Sol. (i) Area, A = 10 L × 10 L = 100 L2
n
a
Let x= Percentage change in area
bm ∆A ∆L
= × 100 = 2 × × 100
Then, ln (x ) = n ln (a ) − m ln (b ) A L
Differentiating both sides, we get = 2 × 2% = 4%
dx da db (ii) Volume, V = 10 L × 10 L × 10 L = 1000 L3
=n −m
x a b Percentage change in volume
In terms of fractional error, we may write ∆V ∆L
∆x ∆a ∆b = × 100 = 3 × 100 = 3 × 2% = 6%
± = ±n +m V L
x a b
Example 1.38 Calculate percentage error in determination of
Therefore, maximum value of time period of a pendulum
l
∆x  ∆a ∆b  T = 2π
= ± n +m  g
x  a b 
where, l and g are measured with ± 1% and ± 2% errors.
34 OBJECTIVE Physics Vol. 1

Sol. Percentage error in time period, ∆B


× 100 = 2%
∆T  1 ∆l 1 ∆g  B
× 100 = ±  × × 100 + × × 100
T 2 l 2 g  ∆C
× 100 = 3%
1 1  C
= ±  × 1% + × 2% = ±1.5 %
2 2  ∆D
and × 100 = 1%
D
A 4B1/ 3 ∆Z 1  3 
Example 1.39 Find the relative error in Z, if Z = and ∴ × 100 = (4 × 4%) +  × 2% + 3% +  × 1%
CD 3/ 2 Z 3  2 
the percentage error in the measurements of A, B, C and D
 2 3
are 4%, 2%, 3% and 1%, respectively. = 16 + + 3 + %
 3 2
∆Z  ∆ A 1  ∆ B  ∆ C 3  ∆ D  = 21.16%
Sol. Q = 4  +   + +  
Z  A  3 B  C 2 D  The percentage error in the measurement of Z is 21.16%.
∆A Therefore, the relative error in Z is 0.2116.
Given, × 100 = 4%
A

CHECK POINT 1.3


1. A spherometer has 100 equal divisions marked along the measurement of l is 1%, then the relative percentage error
periphery of its disc and one full rotation of the disc in measurement of V is
advances on the main scale by 0.01 cm. The least count of (a) 18% (b) 6%
this system is (c) 3% (d) 1%
(a) 10−2 cm (b) 10−4 cm 7. A force F is applied on a square plate of side L. If the
(c) 10−5 cm (d) 10−1 cm percentage error in the determination of L is 2% and that in
2. Three measurements are made as 18.425 cm, 7.21 cm and F is 4%. What is the permissible error in pressure?
5.0 cm. The mean of measurements should be written as (a) 8% (b) 6%
(a) 10.212 cm (b) 10.21 cm (c) 10.22 cm (d) 10.2 cm (c) 4% (d) 2%
3. If error in measuring diameter of a circle is 4%, the error in 8. The heat generated in a wire depends directly on the
measuring radius of the circle would be resistance, current and time. If the error in measuring the
(a) 2% (b) 8% above are 1%, 2% and 1%, respectively. The maximum error
(c) 4% (d) 1% in measuring the heat is
(a) 8% (b) 6% (c) 18% (d) 12%
4. The length of a rod is (11.05 ± 0.2) cm. What is the net
length of the system of rods, when these two rods are 9. If the error in the measurement of momentum of a particle
joined side by side? is (+ 100%), then the error in the measurement of kinetic
energy is
(a) (22.1 ±0.05) cm (b) (22.1 ± 0.1) cm
(a) 100% (b) 200% (c) 300% (d) 400%
(c) (22.10 ± 0.05) cm (d) (22.10 ± 0.4) cm
10. The radius of a ball is (5.2 ± 0.2) cm. The percentage error in
5. A body travels uniformly a distance of (13.8 ± 0.2) m in a
the volume of the ball is (approximately)
time (4.0 ± 0.3) s. The velocity of the body within error limit
is (a) 11% (b) 4% (c) 7% (d) 9%
(a) (3.45 ± 0.2) ms−1 (b) (3.45 ± 0.3) ms−1 11. The values of two resistors are (5.0 ± 0.2) kΩ and
(c) (3.45 ± 0.4) ms−1 (d) (3.45 ± 0.5) ms−1 (10.0 ± 0.1) kΩ. What is the percentage error in the
equivalent resistance when they are connected in parallel?
6. A cuboid has volume V = l × 2 l × 3 l , where l is the length of
(a) 2% (b) 5% (c) 7% (d) 3%
one side. If the relative percentage error in the
Chapter Exercises
(A) Taking it together
Assorted questions of the chapter for advanced level practice

1 If dimensions of A and B are different, then which of 11 The position of the particle moving along Y-axis is
the following operation is valid? given as y = At 2 − Bt 3, where y is measured in
A
(a) (b) e − A/B (c) A − B (d) A + B metre and t in second. Then, the dimensions of B are
B (a) [LT−2] (b) [LT−1] (c) [LT−3] (d) [MLT−2]
2 The diameter of a wire is measured to be
12 Out of the following four dimensional quantities,
0.0250 × 10 −4 m. The number of significant figures which one qualifies to be called a dimensional
in the measurement is constant?
(a) five (b) four (c) three (d) nine (a) Acceleration due to gravity
3 Dimensional formula for electromotive force is same (b) Surface tension of water
as that for (c) Weight of a standard kilogram mass
(a) potential (b) current (c) force (d) energy (d) The velocity of light in vacuum
4. The number of significant figures in 0.06900 is 13 If the random error in the arithmetic means of
[NCERT Exemplar] 100 observations is x, then the random error in the
(a) 5 (b) 4 arithmetic mean of 400 observations would be
(c) 2 (d) 3 1 1
(a) 4x (b) x (c) 2x (d) x
5 The sum of the numbers 436.32, 227.2 and 0.301 in 4 2
appropriate significant figures is [NCERT Exemplar] 14 The damping force on an oscillator is directly
(a) 663.821 (b) 664 proportional to the velocity. The unit of the constant
(c) 663.8 (d) 663.82 of proportionality is
6 The dimensional formula for magnetic flux is (a) kg ms−1 (b) kg ms−2 (c) kgs−1 (d) kgs
(a) [ML2T−2A−1] (b) [ML3T−2A−2] 15 The square root of the product of inductance and
(c) [M0L−2T−2A−2] (d) [ML2T−1A2] capacitance has the dimensions of
(a) length (b) time
7 If the dimensions of a physical quantity are given by
(c) mass (d) no dimension
[Ma Lb T c ], then the physical quantity will be
(a) force, if a = 0, b = − 1, c = − 2 16 The frequency of vibration of string is given by
1/ 2
(b) pressure, if a = 1, b = − 1, c = −2 p F 
f = . Here, p is number of segments in the
(c) velocity, if a = 1, b = 0, c = − 1 2l  m 
(d) acceleration, if a = 1, b = 1, c = − 2
string and l is the length. The dimensional formula
1 for m will be
8 What is the units of k = ?
4 πε 0 (a) [M0LT−1] (b) [ML0T−1]
(a) C2 N −1m−2 (b) N-m2C−2 (c) [ML−1T0] (d) [M0L0T0]
2 2
(c) N-m C (d) Unitless 17 The numbers 2.745 and 2.735 on rounding off to
9 The radius of a circle is 2.12 m. Its area according to 3 significant figures will give [NCERT Exemplar]
the rule of significant figures is (a) 2.75 and 2.74 (b) 2.74 and 2.73
(a) 14.1124 m2 (b) 14.112 m2 (c) 2.75 and 2.73 (d) 2.74 and 2.74
(c) 14.11 m2 (d) 14.1 m2 18 The mass and volume of a body are 4.237 g and
10 If the value of resistance is 10.845 Ω and the value 2.5 cm3 , respectively. The density of the material of
of current is 3.23 A, the value of potential with the body in correct significant figures is
significant numbers would be [NCERT Exemplar]
(a) 35.0 V (b) 3.50 V (a) 1.6048 g cm−3 (b) 1.69 g cm−3
(c) 35.029 V (d) 35.030 V (c) 1.7 g cm−3 (d) 1.695 g cm−3
36 OBJECTIVE Physics Vol. 1

19 Which of the following measurement is most 28 The length and breadth of a rectangular sheet are
precise? [NCERT Exemplar] 16.2 cm and 10.1 cm, respectively. The area of the
(a) 5.00 mm (b) 5.00 cm sheet in appropriate significant figures and error is
(c) 5.00 m (d) 5.00 km [NCERT Exemplar]
(a) 164 ± 3 cm2 (b) 163.62 ± 2.6 cm2
20 The mean length of an object is 5 cm. Which of the
following measurements is most accurate? (c) 163.6 ± 2.6 cm2 (d) 163.62 ± 3 cm2
[NCERT Exemplar]
29 Which of the following pairs of physical quantities
(a) 4.9 cm (b) 4.805 cm
does not have same dimensional formula?
(c) 5.25 cm (d) 5.4 cm [NCERT Exemplar]
21 If the energy (E ), velocity (v ) and force (F ) be taken (a) Work and torque
as fundamental quantities, then the dimensions of (b) Angular momentum and Planck’s constant
mass will be (c) Tension and surface tension
(d) Impulse and linear momentum
(a) [Fv −2] (b) [Fv −1]
(c) [Ev −2] (d) [Ev 2] 30 Measure of two quantities along with the precision
of respective measuring instrument is
22 The dimensional formula for molar thermal capacity
A = 2.5 ms −1 ± 0.5 ms −1, B = 0.10 s ± 0.01s. The
is same as that of
(a) gas constant (b) specific heat value of AB will be [NCERT Exemplar]
(c) Boltzmann’s constant (d) Stefan’s constant (a) (0.25 ± 0.08) m (b) (0.25 ± 0.5) m
(c) (0.25 ± 0.05) m (d) ( 0.25 ± 0.135) m
23 The displacement of an oscillating particle is given
31 Young’s modulus of steel is 1.9 × 10 11 Nm−2 . When
by y = A sin (Bx + Ct + D ). The dimensional formula
expressed in CGS units of dyne/cm2 , it will be equal
for (ABCD) is
to (1N = 10 5 dyne, 1m2 = 10 4 cm2 ) [NCERT Exemplar]
(a) [M0L−1T0] (b) [M0L0T−1] (c) [M0L–1T−1] (d) [M0 L0T0]
(a) 1.9 × 1010 (b) 1.9 × 1011 (c) 1.9 × 1012 (d) 1.9 × 1013
24 A force F is given by F = at + bt , where t is time.
2
32 If voltage V = (100 ± 5) V and current
The dimensions of a and b are
I = (10 ± 0.2) A, the percentage error in resistance R
(a) [MLT−3] and [MLT−4] (b) [MLT−4] and [MLT−3]
is
(c) [MLT−1] and [MLT−2] (d) [MLT−2] and [MLT0] (a) 5.2% (b) 25% (c) 7% (d) 10%
25 The length, breadth and thickness of a block are 33 A wire has a mass (0.3 ± 0.003) g, radius
given by l = 12 cm, b = 6 cm and t = 2.45 cm, (0.5 ± 0.005) cm and length (0.6 ± 0.006) cm. The
respectively. The volume of the block according to maximum percentage error in the measurement of its
the idea of significant figures should be density is
(a) 1 × 102 cm3 (b) 1.76 × 102 cm3 (a) 1 (b) 2 (c) 3 (d) 4
(c) 1.764 × 102 cm3 (d) None of these 34 If x = 10.0 ± 0.1 and y = 10.0 ± 0.1, then 2x − 2 y is
26 A physical quantity Q is calculated according to the equals to
expression (a) (0.0 ± 0.1) (b) zero (c) (0.0 ± 0.4) (d) (20 ± 0.2)
3 3
A B 35 Dimensions of Ohm are same as
Q=
C D h h2 h h2
(a) (b) (c) 2 (d) 2
If percentage errors in A, B, C , D are 2%, 1%, 3% and e e e e
4%, respectively. What is the percentage error in Q ? (where, h is Planck’s constant and e is charge)
(a) ± 8% (b) ± 10% (c) ± 14% (d) ± 12% 36 The equation of state of some gases can be expressed
27 With usual notation, the following equation said to as
give the distance covered in the nth second, i.e.  a 
(2n − 1)  p + 2  (V − b ) = RT
sn = u + a is  V 
2
where, p is the pressure,V is the volume, T is the
(a) only numerically correct
absolute temperature and a, b and R are constants.
(b) only dimensionally correct
The dimensions of a are
(c) Both dimensionally and numerically correct
(d) Neither numerically nor dimensionally correct (a) [ML5T−2] (b) [ML−1T−2] (c) [L3] (d) [L6]
Units, Dimensions & Error Analysis 37

37 Using mass (M ), length (L ), time (T ) and current (A) as Given that length is 5.0 cm and radius is 2.0 cm. The
fundamental quantities, the dimensions of percentage error in the calculated value of the volume
permeability are will be
(a) [M−1LT−2A] (b) [ML2T−2A−1] (a) 1.5% (b) 2.5% (c) 3.5% (d) 4%
(c) [MLT−2A−2] (d) [MLT−1A−1] 44 You measure two quantities as A = 1.0 m ± 0.2 m,
B = 2.0 m ± 0.2 m. We should report correct value
38 In a system of units, the units of mass, length and
time are 1 quintal, 1 km and 1 h, respectively. In for AB as [NCERT Exemplar]
this system, 1 N force will be equal to (a) 1.4 m ± 0.4 m (b) 1.41m ± 0.15 m
(a) 1 new unit (b) 129.6 new unit (c) 1.4 m ± 0.3 m (d) 1.4 m ± 0.2 m
(c) 427.6 new unit (d) 60 new unit 45 If E = energy, G = gravitational constant, I = impulse
dx x − a GIM 2
39 Given that ∫ = a n sin −1   and M = mass, then dimensions of are same as
2ax − x 2  a  E2
that of
where, a = constant. Using dimensional analysis, the
(a) time (b) mass (c) length (d) force
value of n is −α Z
(a) 1 (b) zero α kθ
(c) − 1 46 The relation p = e , where p is pressure, Z is
(d) None of these β
40 The magnetic force on a point charge is distance, k is Boltzmann constant and θ is
F = q ( v × B) temperature. The dimensional formula of β will be
Here, q = electric charge, (a) [M0L2T0] (b) [ML2T] (c) [ML0T−1] (d) [M0L2T−1]
v = velocity of point charge 47 If E, M, L and G denote energy, mass, angular
and B = magnetic field. momentum and gravitational constant respectively,
The dimensions of B are then the quantity (E 2L2/M 5G 2 ) has the dimensions
(a) [MLT−1A] (b) [M2LT−2A−1] of
(c) [MT−2A−1] (d) None of these (a) angle (b) length (c) mass (d) energy
ε 0 lV 48 If the energy E = G h c , where G is the universal
p q r

41 A quantity is given by X = , whereV is the gravitational constant, h is the Planck’s constant and
t
potential difference and l is the length. Then, X has c is the velocity of light, then the values of p, q and r
dimensional formula same as that of are respectively
(a) resistance (b) charge (a) − 1/2, 1/2 and 5/2 (b) 1/2, − 1/2 and − 5/2
(c) voltage (d) current (c) − 1/2, 1/2 and 3/2 (d) 1/2, 1/2 and − 3/2

42 The length of a strip measured with a metre rod is 10.0 cm. 49 A gas bubble formed from an explosion under water
Its width measured with a vernier callipers is 1.00 cm. The oscillates with a period T proportional to p ad b E c ,
least count of the metre rod is 0.1 cm and that of vernier where p is the pressure, d is the density of water
callipers is 0.01 cm. What will be error in its area? and E is the total energy of explosion. The values of
(a) ± 13% (b) ± 7% (c) ± 4% (d) ± 2% a, b and c are
43 The length of cylinder is measured with a metre rod (a) a = 1, b = 1, c = 2 (b) a = 1, b = 2, c = 1
having least count 0.1 cm. Its diameter is measured 5 1 1 5 1 1
(c) a = , b = , c = (d) a = − , b = , c =
with vernier callipers having least count 0.01 cm. 6 2 3 6 2 3

(B) Medical entrance special format questions


(a) If both Assertion and Reason are correct and Reason is the
Assertion and reason correct explanation of Assertion.
Directions (Q. Nos. 1-5) These questions consists of two (b) If both Assertion and Reason are correct, but Reason is not
statements each printed as Assertion and Reason. While the correct explanation of Assertion.
answering these questions you are required to choose any (c) If Assertion is true, but Reason is false.
one of the following four responses. (d) If Assertion is false, but Reason is true.
38 OBJECTIVE Physics Vol. 1

1 Assertion When we change the unit of 3 Which of the following statement(s) is/are incorrect?
measurement of a quantity, its numerical value (a) Dimensional formula of thermal conductivity (K) is
changes. [M1 L1 T−3K −1].
Reason Smaller the unit of measurement, smaller is ( ) is [M −1L2T − 3A−1].
(b) Dimensional formula of potential V
its numerical value. (c) Dimensional formula of permeability of free space (µ 0 )
is [M1 L1 T − 2A−2].
2 Assertion The error in the measurement of radius
of the sphere is 0.3%. The permissible error in its (d) Dimensional formula of RC is [M0 L0 T1].
surface area is 0.6%. 4 Which of the following statement(s) is/are correct?
Reason The permissible error is calculated by I. Out of two measurements l = 0.7 m and
the formula l = 0.70 m, the second one is more accurate.
∆A ∆r II. In every measurement, the last digit is not
=4
A r accurately known.
(a) Only I (b) Only II
an ∆x
3 Assertion If x = m
, then (c) Both I and II (d) None of these
b x
5 Which of the following statement(s) is/are correct?
 ± ∆a   ± ∆b 
=n   −m   I. A screw gauge having a smaller value of pitch has
 a   b  greater accuracy.
The change in a or b, i.e. ∆a or ∆b may be II. The least count of screw gauge is directly proport-
comparable to a and b. ional to the number of divisions on circular scale.
Reason The above relation is valid when (a) Only I (b) Only II
∆a << a and ∆b << b . (c) Both I and II (d) None of these
4 Assertion Dimensional formula of the given quantity
Magnetic dipole moment × magnetic induction
Match the columns
is 1 Match the following columns.
Moment of inertia
[M0 L0 T −1]. Column I Column II
Reason The given dimension is that of frequency. (A) R / L (p) Time
(B) CR (q) Frequency
Modulus of elasticity
5 Assertion has the (C) E / B (r) Speed
Density
(D) ε0 µ 0 (s) None
unit ms −1.
Reason Acceleration has the dimensions of Codes
1 A B C D A B C D
.
( ε0 µ0 ) t (a) p r q s (b) q p r s
(c) s q p r (d) p r s q
Statement based questions 2 Match the following columns.
1 Which of the following statement(s) is/are incorrect? Column I Column II
(a) Method of dimensions cannot be used for deriving (Physical quantity) (Dimensions)
formulae containing trigonometrical ratios. (A) GMeMs (p) [M 2L2T −3]
(b) The light year and wavelength consist of dimensions of
length. (B) 3 kT /M (q) [L2T −2]
(c) Both light year and wavelength represent time. (C) F 2 /q 2B 2 (r) [L2T −1θ−1]
(d) Pressure has the dimensions of energy density.
(D) GMe /R e (s) None
2 Which of the following statement(s) is/are incorrect?
(a) Systematic errors and random errors fall in the same Codes
group of errors. A B C D
(b) Both systematic and random errors are based on the (a) r r p q
cause of error. (b) p p s r
(c) Absolute error cannot be negative. (c) s q r s
(d) Absolute error is the difference between the real value
(d) s r q q
and the measured value of a physical quantity.
(C) Medical entrances’ gallery
Collection of questions asked in NEET & various medical entrance exams

1 A screw gauge has least count of 0.01 mm and there 10 If mass [M], distance [L] and time [T] are
are 50 divisions in its circular scale. fundamental quantities, then find the dimensions of
The pitch of the screw gauge is [NEET 2020] torque.
[JIPMER 2019]
(a) 0.25 mm (b) 0.5 mm (c) 1.0 mm (d) 0.01 mm
(a) [ML2T−2] (b) [MLT−2]
2 Taking into account of the significant figures, what is
(c) [MLT] (d) [ML2T]
the value of 9.99 m − 0.0099 m? [NEET 2020]
(a) 9.98 m (b) 9.980 m 11 What is the dimensions of energy in terms of linear
(c) 9.9 m (d) 9.9801 m momentum [p], area [A] and time [T]? [JIPMER 2019]
3 Dimensions of stress are [NEET 2020] (a) [p1A1T1] (b) [p 2A2T−1]
2
(a) [ML T ] −2 0
(b) [ML T ] −2 (c) [p1A1/2T−1] (d) [p1/2A1/2T−1]
(c) [ML−1T −2] (d) [MLT −2] 12 A student measured the diameter of a small steel ball
4 The angle of 1′ (minute of arc) in radian is nearly using a screw gauge of least count 0.001 cm. The
equal to [NEET 2020] main scale reading is 5 mm and zero of circular scale
(a) 2.91 × 10−4 rad (b) 4.85 × 10−4 rad
division coincides with 25 divisions above the
reference level. If screw gauge has a zero error of
(c) 4.80 × 10−6 rad (d) 1.75 × 10−2 rad − 0.004 cm, the correct diameter of the ball is
[NEET 2018]
5. Time intervals measured by a clock give the
(a) 0.053 cm (b) 0.525 cm (c) 0.521 cm (d) 0.529 cm
following readings 1.25 s, 1.24 s, 1.27 s, 1.21 s and
1.28 s. 13 In an experiment to measure the height of a bridge
What is the percentage relative error of the by dropping stone into water underneath. If the error
observations? [NEET 2020] in measurement of time is 0.2s at the end of 4s, then
(a) 2% (b) 4% (c) 16% (d) 1.6% the error in estimation of height of bridge will be
(neglect the water resistance, i.e. thrust) [AIIMS 2018]
6 The SI unit of thermal conductivity is [NEET 2019]
−1 −1 −1
(a) J m K (b) W m K
h
(c) W m −1K −1 (d) J m K −1
7 In an experiment, the percentage of error occurred (a) ± 19.68 m (b) ± 1722
. m
in the measurement of physical quantities A, B, C (c) ± 7.84 m (d) ± 12.22 m
and D are 1%, 2%, 3% and 4%, respectively. Then,
14 R = (65 ± 1) Ω, l = (5 ± 0.1) mm and
the maximum percentage of error in the
A 2B 1/ 2 d = (10 ± 0.5) mm. Find error in calculation of
measurement X, where X = 1/ 3 3 will be resistivity. [JIPMER 2018]
C D [NEET 2019] (a) 21% (b) 13% (c) 16% (d) 41%
3 15 Dimensions of force are
(a) 16% (b) −10% (c) 10% (d)   % [JIPMER 2018]
13 2 1 −1 1 1 −2
(a) [M LT ] (b) [M L T ]
8 The main scale of a vernier callipers has n (c) [M2L−1T−2] 1 −1
(d) [M1LT ]
divisions/cm. n divisions of the vernier scale
16 A physical quantity of the dimensions of length that
coincide with (n −1) divisions of main scale. The
least count of the vernier callipers is [NEET 2019]
e2
can be formed out of c, G and is [c is velocity
1 1 4πε 0
(a) cm (b) cm
(n + 1) (n − 1) n of light, G is universal constant of gravitation and e
1 1 is charge] [NEET 2017]
(c) 2 cm (d) cm 1/ 2 1/ 2
n n (n + 1) 1  e2   e2 
(a) 2 
G  (b) c 2 G 
9 Calculate the mean percentage error in five c  4πε 0   4πε 0 
observations, 80.0, 80.5, 81.0, 81.5 and 82. 1/ 2
[AIIMS 2019]
1  e2  1 e2
(c) 2   (d) G
(a) 0.74% (b) 1.74% (c) 0.38% (d) 1.38% c G 4πε 0  c 4πε 0
40 OBJECTIVE Physics Vol. 1

17 Planck’s constant (h ), speed of light in vacuum (c ) and 25 Match the column I with column II and mark the
Newton’s gravitational constant (G ) are three correct option from the codes given below.
[Guj. CET 2015]
fundamental constants. Which of the following
combinations of these has the dimensions of length? Column I Column II
[NEET 2016]
hG hG
(A) Electrical resistance 1. [ML3T −3A−2]
(a) (b)
c 3/ 2 c 5/ 2 (B) Electrical potential 2. [ML2T −3A−2]

(c)
hc
(d)
Gc (C) Specific resistance 3. [ML2T −3A−1]
3/ 2
G h (D) Specific conductance 4. None
18 If energy (E ), velocity (v ) and time (T ) are chosen as
Codes
the fundamental quantities, the dimensional formula
(a) A-2, B-3,C-1, D-4 (b) A-2, B-4, C-3, D-1
of surface tension will be [CBSE AIPMT 2015]
(c) A-1, B-2, C-4, D-3 (d) A-1, B-3, C-2, D-4
(a) [Ev −2T−1] (b) [Ev −1 T−2]
26 The three physical quantities x, y and z have units
−2 −2
(c) [Ev T ] (d) [E −2v −1T−3] g cm2 s −5, g s −1 and cms −2 , respectively. The relation
19 In terms of basic units of mass (M), length (L), time between x, y and z is [AFMC 2015]
(T ) and charge (Q ), the dimensions of magnetic (a) x = yz 2 (b) x = y 2z
permeability of vacuum ( µ 0 ) would be [AIIMS 2015] (c) y 2 = x z (d) z = x 2 y
−2 −1 −1
(a) [MLQ ] (b) [LT Q ]
27 If force (F), velocity (v) and time (T) are taken as
−1 −2
2
(c) [ML T Q ] (d) [LTQ −1] fundamental units, then the dimensions of mass are
[CBSE AIPMT 2014]
20 The dimensional formula for electric flux is
[AIIMS 2015] (a) [FvT−1] (b) [FvT–2]
(a) [ML3 I−1T−3] (b) [M 2L2I −1T−2] (c) [Fv –1T−1] (d) [Fv –1T]
(c) [ML3 I1T−3] (d) [ML−3I −1 T−3] 28 The dimensional formula for Reynold’s number is
[MHT CET 2014]
21 In terms of time t and distance x, the force F is given (a) [L0M0T0] (b) [LMT]
A
by F = A sin Ct + B cos Dx, then dimensions of and (c) [L–1MT] (d) [LMT–1]
B
C 29 The relation between force F and density d is
are given by x
D [UK PMT 2015] F = . The dimensions of x are
(a) [M0L0T 0], [M0LT −1] (b) [MLT−2], [M0L0T−1]
d [MHT CET 2014]
–1/2 3/2 –2 –1/2 1/2 –2
(c) [MLT −2], [M0L−1T 0] (d) [M0LT –1], [M0L0T 0] (a) [L M T ] (b) [L M T ]
(c) [L–1M3/2T–2] (d) [L–1M1/2T–2]
22 The wrong unit conversion among the following is
30 If the absolute errors in two physical quantities A
(a) 1 angstrom = 10−10 m [Kerala CEE 2015] and B are a and b respectively, then the absolute
(b) 1 fermi = 10−15 m error in the value of A − B are [EAMCET 2014]
(c) 1 light year = 9.46 × 1015 m (a) b − a (b) a =/ b
(d) 1 astronomical unit = 1.496 × 10−11 m (c) a + b (d) a − b
(e) 1 parsec = 3.08 × 1016 m 31 If the unit of force is kN, the length is 1 km and
time 100 s, then what will be the unit of mass?
23 The mass of the liquid flowing per second per unit [KCET 2014]
area of cross-section of the tube is proportional to (a) 1000 kg (b) 1 kg
(pressure difference across the ends) n and (average (c) 10000 kg (d) 100 kg
velocity) m of the liquid. Which one of the following 32 If n denotes a positive integer, h the Planck’s
relation is correct? [CG PMT 2015] constant, q the charge and B the magnetic field, then
(a) m = n (b) m = − n (c) m 2 = n (d) m = − n 2  nh 
the quantity   has the dimension of
24 The ratio of the dimensions of Planck’s constant and  2πqB  [WB JEE 2014]
that of moment of inertia has the dimensions of
(a) angular momentum (b) time [KCET 2015] (a) area (b) length
(c) velocity (d) frequency (c) speed (d) acceleration
Units, Dimensions & Error Analysis 41

33 In an experiment, four quantities a, b, c and d are 38 The quantities A and B are related by the relation
measured with percentage error 1%, 2%, 3% and m = A / B, where m is the linear density and A is the
4%, respectively. Quantity P is calculated as follows force. The dimensions of B are of [BCECE 2012]
a 3b 2 (a) pressure (b) latent heat
P = %, error in P is (c) work (d) None of these
cd [NEET 2013]
(a) 14% (b) 10% 39 A physical quantity is given by X = [Ma Lb T c ]. The
(c) 7% (d) 4% percentage error in measurement of M, L and T are
34 The density of glass is 2.8 g/cc in CGS system. The α, β and γ, respectively. Then, the maximum % error
value of density in SI unit is [Kerala CEE 2013] in the quantity X is [AFMC 2012]
(a) 2.8 × 10−3
(b) 2.8 × 10−2 (a) a α + b β + c γ (b) a α + b β − c γ
(c) 2.8 × 102 (d) 2.8 × 103 a b c
(c) + + (d) None of these
(e) 2.8 × 106 α β γ

 1 y 40 The dimensions of (µ 0 ε 0 ) −1/ 2 are [CBSE AIPMT 2011]


35. In the equation  = , where p is the pressure, (a) [L−1T] (b) [LT−1]
 pβ  kBT
(c) [L−1/2T1/2] (d) [L1/2T−1/2]
y is the distance, kB is Boltzmann constant and T is
the temperature. Dimensions of β are [EAMCET 2013] RT
41 If p = e −αV/RT , then dimensional formula of α
(a) [M−1 L1 T2] (b) [M0 L2 T0] (c) [M1 L−1 T−2] (d) [M0 L0 T0] V −b
is [UP CPMT 2011]
36 A physical quantity X is defined by the formula (a) p (b) R
IF v 2 (c) T (d)V
X=
WL3 42 Velocity v is given by v = at 2 + bt + c, where t is
where, I is moment of inertia, F is force, v is time. What are the dimensions of a, b and c,
velocity,W is work and L is length, the dimensions respectively?
of X are [MP PMT 2013] (a) [LT−3], [LT−2] and [LT−1] [UP CPMT 2011]
(a) [MLT−2] (b) [MT−2] (c) [ML2T−3] (d) [LT−1] (b) [LT−1], [LT−2] and [LT−3]
(c) [LT−2], [LT−3] and [LT−1]
37 A physical quantity X is given by
(d) [ LT−1], [LT−3] and [LT−2]
2k 3l 2
X=
m n 43 From the dimensional consideration, which of the
following equations is correct? [Haryana PMT 2011]
The percentage error in the measurements of k, l, m
and n are 1%, 2%, 3% and 4%, respectively. The R3 GM
value of X is uncertain by [AMU 2012] (a) T = 2π (b) T = 2π
GM R3
(a) 8% (b)10%
GM R2
(c) 12% (d) None of these (c) T = 2π 2
(d) T = 2π
R GM
ANSWERS
l CHECK POINT 1.1
1. (b) 2. (d) 3. (c) 4. (c) 5. (d) 6. (c) 7. (b) 8. (c) 9. (c) 10. (c)
11. (d) 12. (a) 13. (b) 14. (b) 15. (c) 16. (d) 17. (b) 18. (c) 19. (a) 20. (b)
21. (d)

l CHECK POINT 1.2


1. (b) 2. (c) 3. (b) 4. (c) 5. (c) 6. (d) 7. (a) 8. (b) 9. (d) 10. (a)

l CHECK POINT 1.3


1. (b) 2. (d) 3. (c) 4. (d) 5. (d) 6. (c) 7. (a) 8. (b) 9. (c) 10. (a)
11. (d)

(A) Taking it together


1. (a) 2. (c) 3. (a) 4. (b) 5. (c) 6. (a) 7. (b) 8. (b) 9. (d) 10. (a)
11. (c) 12. (d) 13. (b) 14. (c) 15. (b) 16. (c) 17. (d) 18. (c) 19. (a) 20. (a)
21. (c) 22. (a) 23. (b) 24. (a) 25. (b) 26. (c) 27. (c) 28. (a) 29. (c) 30. (a)
31. (c) 32. (c) 33. (d) 34. (c) 35. (c) 36. (a) 37. (c) 38. (b) 39. (b) 40. (c)
41. (d) 42. (d) 43. (b) 44. (d) 45. (a) 46. (a) 47. (d) 48. (a) 49. (d)

(B) Medical entrance special format questions


l Assertion and reason
1. (c) 2. (c) 3. (d) 4. (a) 5. (b)

l Statement based questions


1. (c) 2. (c) 3. (b) 4. (c) 5. (a)

l Match the columns


1. (b) 2. (d)

(C) Medical entrances’ gallery


1. (b) 2. (a) 3. (c) 4. (a) 5. (d) 6. (c) 7. (a) 8. (c) 9. (a) 10. (a)
11. (c) 12. (d) 13. (c) 14. (b) 15. (b) 16. (a) 17. (a) 18. (c) 19. (a) 20. (a)
21. (a) 22. (d) 23. (b) 24. (d) 25. (a) 26. (a) 27. (d) 28. (a) 29. (a) 30. (c)
31. (c) 32. (a) 33. (a) 34. (d) 35. (b) 36. (b) 37. (c) 38. (b) 39. (a) 40. (b)
41. (a) 42. (a) 43. (a)
Hints & Explanations
l CHECK POINT 1.1 21 (d) m ∝ v aρ b g c . Writing the dimensions on both sides,
h
3 (c) Since, (mvr ) = n ⋅ and E = hν [M] = [LT −1]a [ML−3]b [LT −2]c

[M] = [MbLa − 3b + c T − a − 2c ]
So, unit of h = joule second = angular momentum
mass ∴ b =1
5 (d) We know that, density = a − 3b + c = 0
volume
− a − 2c = 0
In CGS system, d = 0.625 g cm−3
Solving these, we get
0.625 × 10 −3 kg a=6
In SI system, d = = 625 kg m−3
10 −6 m3 Hence, m ∝v6
7 (b) Impulse = Change in linear momentum.
ρl
l CHECK POINT 1.2
12 (a) Since R = , where ρ is specific resistance.
A 3 (b) Only in 20.2, all zeros are significant because in this, zero
RA  V W lies between two non-zero digits.
∴ [ρ] = , R = ,V =
 l  i Q 6 (d) Out of 107.88 and 0.610, least number of significant digits
[ρ] = [ML3T −1 Q −2] is 3, so the product must contain 3 significant digits. So, the
right answer is 65.8.
15 (c) Since, units of length, velocity and force are doubled.
8 (b) Given, R = 0.16 mm
[force] [time] [length]
Hence, [m] = , [time] = 22
[velocity] [velocity] Hence, A = π R 2 = × (0.16)2 = 0.080457
7
Hence, unit of mass and time remains same.
Since, radius has two significant figures, so answer will also
Momentum is doubled.
have two significant figures.
a − t2
17 (b) Given, p = , where p is the pressure and t is the time Hence, option (b) is correct.
bx
9 (d) As 2.54 has least number of significant digits which is 3,
[pbx] = [a] = [t 2] so the result must have 3 digits as significant.
[t 2] Hence, the correct result is 38.4.
Hence, [b ] =
[px] 10 (a) The product must have two significant digit as both the
a figures being multiplied have two significant digits, i.e.
Dimensions of = [px] = [MT −2]
b 25 × 10 −14 . So, the order of magnitude of result is −14.
x
18 (c) Q have the same dimension as of k. l CHECK POINT 1.3
v
[x] [L] 1 (b) Given, number of divisions on circular scale = 100
∴Dimensional formula of [k] = = = [T]
[v ] [LT −1] Pitch = 0.01cm
Pitch
19 (a) Muscle × Speed = Power ∴Least count (LC) =
Number of divisions on circular scale
Power Work [ML2T −2]
or Muscle = = = = [MLT −2] =
0.01
= 10 −4 cm
Speed Time × Speed [T][LT −1] 100
= Mass × Acceleration = Force 3 (c) Since, D ∝ r
SI unit of force kg × m × s −2 where, D = diameter of a circle
Now, =
CGS unit of force g × cm × s −2 and r = radius of the circle.
= 10 3 × 10 2 = 10 5 Therefore, there will be no change in error, it will remain 4%
for radius also.
20 (b) Since, p xQ yc z is dimensionless. Therefore,
6 (c) Volume of cuboid,
[ML−1T −2]x [MT −3 ]y [LT −1]z = [M0 L0 T 0 ]
V = l × 2l × 3l = 6l 3
Only option (b) satisfies this expression.
∆V  ∆l   ∆l 
So, x =1 , y = −1 , z =1 ∴ × 100 = 3   = 3% Q × 100 = 1%
V  l   l 
44 OBJECTIVE Physics Vol. 1

7 (a) p =
F F
= = FL−2 p2  F  p 2F
f2 = 2   ⇒ m= 2 2
A L2 4l  m  4l f
% error in pressure = (% error in F) + 2 (% error in L)
[MLT −2]
= (4%) + 2 (2%) = 8% [m] = = [ML−1T 0 ]
[L2][T −1]2
8 (b) We know that, H = i 2 R t
17 (d) Rounding off 2.745 to 3 significant figures, it would be
∴ % error in H = 2 (% error in i) + (% error in R) + (% error in t) 2.74. Rounding off 2.735 to 3 significant figures, it would
= 2(2%) + 1% + 1% = 6% be 2.74.
18 (c) In this question, density should be reported to two
p2 significant figures.
9 (c) We know that, K =
2m 4.237g
Density = = 1.6948 gcm −3
The error in measurement of momentum is + 100%. 2.5 cm3
Therefore, the actual momentum with error, p′ = p + p = 2p On rounding off the number, we get
Q Kinetic energy with error, Density = 1.7 g cm −3
(p′ )2 (2p )2
K′ = = 19 (a) All given measurements are correct upto two decimal
2m 2m
places. As here 5.00 mm has the smallest unit and the error
4p 2 in 5.00 mm is least (commonly taken as 0.01 mm, if not
or K′ =
2m specified), hence 5.00 mm is most precise.
or K′ = 4K Note In solving these type of questions, we should be careful about units
So, percentage change in kinetic energy, although their magnitude is same.
 K′−K   4K − K  20 (a) Given, length, l = 5 cm
KE =   × 100 =   × 100
 K   K  Now, checking the errors with each options one by one, we
 4 − 1 get
=  × 100 = 300% ∆ l1 = 5 − 4.9 = 0.1cm
 1 
∆ l 2 = 5 − 4.805 = 0.195 cm
10 (a) Radius of ball = 5.2 cm
∆ l 3 = 5.25 − 5 = 0.25 cm
4
Volume, V = πR 3 ∆ l 4 = 5.4 − 5 = 0.4 cm
3
Error ∆ l1 is least.
∆V  ∆R 
=3  Hence, 4.9 cm is most accurate.
V  R 
23 (b) Displacement,
 ∆V   0.2
  × 100 = 3   × 100 −
~ 11% y = A sin (Bx + Ct + D )
V   5.2
A = y = [L]
As each term inside the brackets is dimensionless, so
(A) Taking it together 1
B = = [L−1]
6 (a) Magnetic flux, [φ] = [BS] = [MT −2A−1] [L2] = [ML2T −2A−1] x
1
11 (c) As, [ y ] = [B] [T 3 ] ⇒ [L] = [B] [T 3 ] ⇒ [B] = [LT −3 ] C = = [T −1]
t
13 (b) Since, error is measured for 400 observations instead of and D is dimensionless.
100 observations. So, error will reduce by 1/4 factor.
∴ [ABCD] = [L][L−1][T −1] = [M0 L0 T −1]
14 (c) Given, damping force ∝ velocity
F ∝ v ⇒ F = kv 25 (b) Using the relation for volume,
F V = Length × Breadth × Thickness
⇒ k=
v = 12 × 6 × 2 .45 = 176.4 cm3
Unit of F kgms −2 V = 1.764 × 10 2 cm3
Unit of k = = = kgs–1
Unit of v ms –1
The minimum number of significant figure is 3. Hence, the
1 1 T
15 (b) f = or LC = = (Q T = 1/ f ) volume will contain only 3 significant figures. Therefore,
2π LC 2π f 2π V−~ 1.76 × 10 2 cm 3.
Thus, LC has the dimensions of time.
A3B 3
1/ 2 26 (c) Given, Q =
p F  C D
16 (c) Given, f =
2l m  ∆Q  ∆A  ∆B   ∆C  1  ∆D 
= 3  + 3  +  +  
Squaring the equation on either side, we have Q  A   B   C  2 D 
Units, Dimensions & Error Analysis 45

∆A ∆B Maximum percentage error in resistance,


Here, × 100 = 2%, × 100 = 1%,
A B  ∆R   ∆V   ∆I 
 × 100 =  × 100 +  × 100
∆C ∆D  R  V   I 
× 100 = 3%, × 100 = 4%
C D  5   0.2 
∆Q 1 = × 100 +  × 100 = 5 + 2 = 7%
∴ × 100 = (3 × 2%) + (3 × 1%) + (3%) + × (4%) = ±14%  100   10 
Q 2
m
33 (d) Density, ρ=
28 (a) Given, length, l = (16.2 ± 0.1) cm πr 2L
Breadth, b = (10.1± 0.1) cm ∆ρ  ∆m 2∆r ∆L 
∴ × 100 = + + × 100
ρ  m r L 
Area, A = l × b = (16.2 cm) × (10.1 cm) = 163.62 cm2
Rounding off to three significant digits, area, A = 164 cm2 After substituting the values, we get the maximum
percentage error in density = 4%.
∆ A ∆l ∆b 0.1 0.1 1.01 + 1.62 2.63
Q = + = + = = 35 (c) Q ‘Ohm’ is the unit of resistance and having dimension
A l b 16.2 10.1 16.2 × 10.1 163.62
= [ML2T −3A−2]
2.63 2.63
⇒ ∆A = A × = 163.62 ×
163.62 163.62 From option (c),
= 2.63 cm2 [h] [ML2T −1]
⇒ 2= = [ML2T − 3A−2] = Dimensions of resistance (Ω )
∆A = 3 cm2 (By rounding off to one significant figure) [e ] [AT]2
B
∴ Area, A = A ± ∆A = (164 ± 3) cm2 37 (c) B = µNI ⇒ µ =
NI
29 (c) (a) Work = Force × Distance = [MLT −2][L] =[ML2T −2] As, Bqv = F
F
Torque = Force × Distance = [ML2T −2] ⇒ B=
qv
(b) Angular momentum = mvr = [M][LT −1][L] = [ML2T −1]
F
E [ML2T −2] So, µ = , where N is the number of turns per unit length
Planck’s constant = = = [ML2T −1] qvNI
ν [T −1]
(c) Tension = Force = [MLT −2] [MLT −2]
[µ] = = [MLT −2A−2]
Force [MLT −2] [AT] [A] [L−1]
Surface tension = = = [ML0 T −2]
Length [L] 38 (b) [Force] = [MLT −2]
−2 −1
(d) Impulse = Force × Time = [MLT ][T] = [MLT ]  1  1 
∴ 1N =    (3600) = 129.6 units
2
−1
Momentum = Mass × Velocity = [M][LT ] = [MLT ] −1  100   1000 
Note One should not be confused with the similar form tension in both 40 (c) Magnetic force, F = q (v × B) or F = q v B sinθ
the physical quantities-surface tension and tension. Dimensional
formula for both of them is not same. F  [MLT −2]
∴ [B] =   = −1
= [MT −2A−1]
30 (a) Given, A = 2.5 ms −1 ± 0.5 ms −1, B = 0.10 s ± 0.01s  
qv [AT][LT ]
x = AB = (2.5)(0.10) = 0. 25 m 42 (d) Area of strip = lb
∆x ∆A ∆B 0.5 0.01 0.05 + 0.025 0.075  ∆A  ∆l   ∆b 
= + = + = = ∴   × 100 =   × 100 +   × 100
x A B 2.5 0.10 0.25 0.25  A  l   b 
∆x = 0.075 = 0.08 m (rounding off to one significant 0.1 0.01
=× 100 + × 100 = ± 2%
figure) 10 1
AB = (0.25 ± 0.08) m
D
43 (b) Volume of cylinder,V = πr 2L, r =  
31 (c) Given, Young’s modulus,Y = 1.9 × 1011 Nm−2  2
1 N = 10 5 dyne  ∆V   ∆D   ∆L 
∴   × 100 = 2   × 100 +   × 100
Hence, Y = 19. × 1011 × 10 5 dyne/m2 V   D   L 
We know that, 1 m = 100 cm  0.01  0.1
=2  × 100 +   × 100 = 2.5%
∴ Y = 19
. × 1011 × 10 5 dyne/(100 )2 cm2  4.0   5
= 1.9 × 1016 − 4 dyne/cm2
44 (d) Given, A = 1.0 m ± 0.2 m, B = 2.0 m ± 0.2 m
Y = 1.9 × 1012 dyne/cm2
Let Y = AB = (1.0)(2.0) = 1.414 m
32 (c) Given, voltage,V = (100 ± 5) V
Rounding off to two significant digitY = 1.4 m
Current, I = (10 ± 0.2) A
∆Y 1  ∆A ∆B  1  0.2 0.2  0.6
V = + = + =
2  A B  2 1.0 2.0  2 × 2.0
Q
From Ohm’s law, V = IR ⇒ Resistance, R = Y
I
46 OBJECTIVE Physics Vol. 1

0.6Y 0.6 × 1.4 Match the columns


⇒ ∆Y = = = 0 . 212 l

2 × 2.0 2 × 2.0
1 (b) A → q; B → p; C→ r ; D → s
Rounding off to one significant digit, ∆Y = 0 . 2 m
[ML2T −3A−2]
Thus, correct value for AB = Y + ∆Y = 1.4 m ± 0.2 m (A) [R]/[L] = = [T] −1
[ML2T −2A−2]
αZ
46 (a) In the given equation, should be dimensionless. This is the dimensions of frequency.
kθ (A) → (q)
k θ  [ML2T −2K−1] [K]
∴ [α ] = ⇒ [α ] = = [MLT −2] (B) [CR] = [M −1L −2T 4 A 2] [ML 2T −3A −2] = [T]
 Z  [L]
This matches with the dimensions of time given in the
α α  [MLT −2] column.
and p= ⇒ [ β] =   = −1 −2
= [M0 L2T 0 ]
β  p  [ML T ] (B) → (p)
47 (d) The dimensions of E = [ML2T −2] [MLT −3A−1]
(C) [E]/ [B] = = [LT −1]
Dimension of M = [M] [MT −2A−1]
Dimensions of L = [ML2T −1] This is the dimensions of speed.
(C) → (r)
Dimensions of G = [M−1L3T −2]
(D) [ε 0µ 0 ]1/ 2 = {[M −1L −3T 4 A 2] [MLT −2A −2]}1/ 2 = [L −2T 2]1/ 2 = [L −1T]
 E 2L2  [ML2T −2] 2 [ML2T −1] 2
∴ Dimensions of  5 2  = This does not matches with the dimensions given in the
M G  [M] 5 [M−1 L3T −2] 2 column.
= [ML2T −2] = Energy (D) → (s)

49 (d) Given, T ∝ p ad bE c
We have, [M0 L0 T] = k [ML−1T −2] a [ML−3] b[ML2T −2] c
(C) Medical entrances’ gallery
1 (b) Given, least count = 0.01mm
where, k is a constant.
⇒ [M0 L0 T] = k [Ma + b + c L− a − 3b + 2c
T −2a − 2c] Number of divisions on circular scale = 50
Pitch of the screw gauge = Least count × Number of
On comparing powers of M, we have divisions on circular scale
0 =a + b + c …(i) = 0.01 × 50 = 0.5 mm
On comparing powers of L, we have 2 (a) The difference between 9.99 m and 0.0099 m is
0 = − a − 3b + 2c …(ii) = 9.99 − 0.0099 = 9.9801m
On comparing powers of T, we have
Taking significant figures into account, as both the values has
1 = − 2a − 2c …(iii) two significant figures after decimal.
On solving Eqs. (i), (ii) and (iii), we have So, their difference will also have two significant figures after
5 1 1
a =− ,b = ,c = decimal, i.e. 9.98 m.
6 2 3 Force
3 (c) Q Stress =
Area
(B) Medical entrance special format [MLT −2]
questions ∴ Dimensions of stress = = [ML−1T −2]
[L2]
Assertion and reason 1 1 π
4 (a) 1 minute = degree = × = 2 .91× 10 −4 rad
l

60 60 180
2 (c) A = 4πr 2
∆A ∆r 5 (d) Mean time interval,
× 100 = 2 × 100 = 2 (0.3) % = 0.6% 1.25 + 1.24 + 1.27 + 1.21 + 1.28 6 .25
A r T = = = 1.25 s
5 5
Modulus of elasticity [ML−1T −2]
5 (b) = = m/s Mean absolute error,
Density [ML−3T 0 ] | ∆T1 | + | ∆T2 | + | ∆T3 | + | ∆T4 | + | ∆T5 |
∆T =
5
l Statement based questions | 1.25 − 1.25 | + | 1.24 − 1.25 | + | 1.27 − 1⋅.25 |
1 (c) Both light year and wavelength has dimensions of length. + | 1.21 − 1.25 | + | 1.28 − 1.25 |
⇒ =
2 (c) Absolute error may be negative or positive. 5
[ ] = [ML2T −3A−1] 0 + 0.01 + 0.02 + 0.04 + 0.03 0.1
3 (b) Dimensional formula of potential V ⇒ = = = 0.02 s
5 5
5 (a) The least count of screw gauge is
∆T 0.02
Pitch ∴Percentage relative error = × 100 = × 100 = 1. 6%
LC = T 1. 25
Number of divisions on circular scale
Units, Dimensions & Error Analysis 47

6 (c) The SI unit of thermal conductivity is Wm−1K−1. 11 (c) Dimensions of energy in terms of linear momentum (p ),
2 1/ 2 area (A) and time (T), is related to
AB
7 (a) Given, X =
C 1/ 3D 3 E = p aAbT c … (i)
The percentage error in X is given by Writing dimensional formula of both sides, we get
∆X  ∆A 1  ∆B  1  ∆C  [ML2T −2] = [MLT −1]a [L2]b[T]c
× 100 = 2   × 100 +   × 100 +   × 100
X  A 2 B  3 C  [ML2T −2]] = [Ma La + 2bT − a + c ]
 ∆D  Comparing the exponents,
+ 3  × 100 …(i)
 D  a = 1, a + 2b = 2
∆A ∆B 2b = 1
Given, × 100 = 1%, × 100 = 2%,
A B 1
⇒ b=
∆C ∆D 2
× 100 = 3%, × 100 = 4%
C D −a + c = − 2
Substituting these values in Eq. (i), we get ⇒ − 1+ c = − 2
∆X 1 1 ⇒ c = −1
× 100 = 2 (1%) + (2%) + (3%) + 3(4%)
X 2 3 ∴From Eq. (i), we have
= 2% + 1% + 1% + 12% = 16% ⇒ E = [pA1/ 2T −1]
Thus, maximum % error in X is 16%.
12 (d) Given, least count of screw gauge,
8 (c) As it is given that, n divisions of vernier scale coincide LC = 0.001 cm
with (n − 1) divisions of main scale, i.e.
Main Scale Reading (MSR) = 5 mm = 0.5 cm
n (VSD) = (n − 1)MSD
Number of coinciding divisions on the circular scale, i.e.
(n − 1)
⇒ 1 VSD = MSD …(i) Vernier Scale Reading (VSR) = 25
n Here, zero error = −0.004 cm
The least count is the difference between one Main Scale Final reading obtained from the screw gauge
Division (MSD) and one Vernier Scale Division (VSD). = MSR + VSR × LC − zero error
∴Least Count (LC) = 1 MSD − 1 VSD = 0.5 + 25 × 0.001 − (−0.004)
(n − 1) = 0.5 + 0.025 + 0.004
= 1 MSD − MSD [From Eq. (i)]
n = 0.5 + 0.029 = 0.529 cm
 (n − 1) 1
= 1 −  MSD = MSD Thus, the diameter of the ball is 0.529 cm.
 n  n 1
1 13 (c) We know that, s = ut + at 2
Given, 1 MSD = cm 2
n 1  Qa = g 
1 1 1 or h = (0 ) t + × 9.8 × (4)2  
⇒ LC = × cm = 2 cm 2  and u = 0
n n n
= 78.4 m
9 (a) Mean of the five observations, Given, ∆t = 0.2 s, t = 4 s
80.0 + 80.5 + 81.0 + 81.5 + 82 405.0
µ= = = 81 ∆h  ∆t   0.2
5 5 Now, for error, = ± 2   = ± 2   = ± 0.1
h  t   4
| 80 − µ | + | 80.5 − µ | + | 810. −µ| 
 ∆h = ± 0.1× h = ± 0.1× 78.4 = ± 7.84 m
+ |82 − µ|
or
+ − µ
∴ Mean error = 
|815. |
5 14 (b) Given, R = 65 Ω, ∆R = 1Ω,
| 80 − 81| + | 80.5 − 81| + | 810 . − 81|  l = 5 × 10 −3 m, ∆l = 0.1 × 10 −3 m,
 + − + −  d = 10 × 10 −3 m and ∆d = 0.5 × 10 −3 m
= 
| 815
. 81 | |82 81|
5 RA Rπ (d / 2 ) 2 πRd 2
Q Resistivity, ρ = or ρ = =
1 + 0.5 + 0 + 0.5 + 1 3 l l 4l
= = = 0.6
5 5 ∆ρ ∆R ∆ d ∆l
∴ = +2 +
0.6 ρ R d l
∴ Mean % error = × 100% = 0.74%
81 ∆ρ 1  0.5 × 10 −3  0.1× 10 −3
⇒ = + 2  +
10 (a) Dimensions of torque, ρ 65  10 × 10 −3  5 × 10 −3
τ = [F × r] = [MLT −2] [L] ∆ρ ∆ρ
⇒ = 0.0153 + 0.1+ 0.02 ⇒ ≈ 0.1353
2 −2
= [ML T ] ρ ρ
So, error in calculation of resistivity is 13.5% ≈ 13%.
48 OBJECTIVE Physics Vol. 1

15 (b) We know that, F = ma 19 (a) The force per unit length experienced due to two wires in
which current is flowing in the same direction is given by
[M] [L]
∴ [F] = [M][a] = = [M1 L1 T −2] dF µ 0 2 I1I2 [M L T − 2] [A2]
T2 = ⇒ = [µ 0 ]
dl 4π d [L] [L]
e2 e2
16 (a) As, force, F = ⇒ = r 2 ⋅F [MLT − 2]  Q2 
4πε 0r 2 4πε 0 ⇒ = [µ 0 ]  2  ⇒ [ µ 0 ] = [MLQ −2]
[L ] T L 
Putting dimensions of r and F, we get
 e2  F
3 −2 20 (a) As electric flux is given by, φ E = EdS = dS
⇒  4πε  = [ML T ] …(i) q
 0
 MLT −2  2 3 −1 −3
Gm 2 ∴ Dimensions of φ E =   [L ] = [ML I T ]
Also, force, F = 2 ⇒ [G] = [M−1 L3 T −2] …(ii)  IT 
r
 1 1 21 (a) Given, F = A sin Ct + B cos Dx …(i)
and = = [L−2 T 2] …(iii)
c 2  [L2 T −2]
where, t = time and x = distance
Now, checking optionwise As, we know that trigonometric ratios are dimensionless. This
1/ 2 implies
1  Ge 2 
=   = [L−2 T 2] [L6 T − 4 ]1/ 2 = [L] sin Ct = dimensionless and cos Dx = dimensionless
c 2  4πε 0 
1  1
Also, [C ] = = [T −1] and [D] = = [L−1]
17 (a) In forms of h, c and G, length can be expressed as t   x 
L = (h )a (c )b (G )c As, Eq. (i) represents the force. So, A and B both have the
Writing dimensions on both sides, we get dimensions as that of force. So, A/B is dimensionless, i.e.
[M0 L0 T 0 ].
[M0 LT 0 ] = [ M L2T −1] a [ LT −1] b [M−1 L3T −2] c
−1
= M a − c L2a + b+ 3 c T − a − b− 2 c C   T 
While = = [M0 LT −1]
D   L−1 
On comparing powers of M, L and T on both sides, we get  
a − c = 0, 2a + b + 3 c = 1 22 (d) Option (d) is wrong because 1 astronomical unit
and − a − b − 2c = 0 = 1.5 × 10 11m
On solving, we get
23 (b) According to the question, we have
1 3
a = c = and b = − m
2 2 ∝ p n ⋅v m or m /t ⋅ A = kp nv m
t ⋅A
hG
∴ Dimensions of length, L = (h )1/ 2 (c )−3/ 2 (G )1/ 2 = 3/ 2 where, k is proportionality constant.
c Using principle of homogeneity, we get
Force [F]
18 (c) We know that, surface tension (S ) = [ML−2T −1] = k[ML−1T −2]n ⋅ [LT −1]m
Length [L]
or [ML−2T −1] = k[M] n [L] − n + m [T]− 2n − m
[MLT −2]
So, [S] = = [ML0 T −2] Equating both sides, we find n = − m or m = − n
[L]
24 (d) We know that, energy of an emitted particle,
Energy (E ) = Force × Displacement ⇒ [E] = [ML2T −2] E
Displacement E = hν ⇒ h =
Velocity (v ) = ⇒ [v] = [LT −1] ν
Time [ML2T −2]
S ∝ Eavb Tc Planck’s constant, [h] = = [ML2T −1] …(i)
As, [T −1]
where, a, b and c are constants. and moment of inertia, I = mr 2 ⇒ [I] = [ML2] …(ii)
From the principle of homogeneity,
On dividing Eq. (i) by Eq. (ii), we get
[LHS] = [RHS]
[h]  ML2 T −1  1
[ML0 T −2] = [ML2T −2]a [LT −1]b[T]c = = [T −1] =
⇒ [I]  ML2  T
⇒ [ML0 T −2] = [Ma L2a + bT −2a − b + c ] [h]
i.e. = [T −1] = Dimensions of frequency of a particle
Equating the power on both sides, we get [I]
a = 1, 2a + b = 0 ⇒ b = − 2 25 (a) The dimensions of electrical resistance,
and − 2a − b + c = − 2 W  W
 
⇒ c = (2a + b ) − 2 = 0 − 2 = − 2 V  q  It W
R= = = = = [ML2T −2 T −1 A−2] = [ML2 T −3 A−2]
So, [S] = [Ev−2T −2] I I (I ) I 2 t
Units, Dimensions & Error Analysis 49

Then, (A) → (2) Using Eqs. (i) and (ii), we get


The dimensions of electrical potential,  nh  mvr
 2πqB  = mv / r = [r ]
2
W W
V = = = [ML2T −2A−1T −1] = [ML2 T −3 A−1]  
q It
As, r has the dimension of length, thus the given quantity has
Then, (B) → (3) the dimension of area.
The dimensions of specific resistance,
a 3b 2
A 33 (a) Here, P =
ρ = R = [ML2T −3A−2] [L] = [ML3T −3A−2] cd
l
∆P  3∆a 2∆b ∆c ∆d 
Thus, (C) → (1) ∴ × 100 =  + + +  × 100
P  a b c d 
And the dimensions of specific conductance, ∆a ∆b ∆c ∆d
1 1 =3 × 100 + 2 × 100 + × 100 + × 100
σ= = = [M−1L−3T 3A2] a b c d
ρ [ML3T −3A−2]
= 3 × 1% + 2 × 2% + 3% + 4%
= not given in column = 3% + 4% + 3% + 4% = 14%
Thus, (D) → (4)
34 (d) Given, density of glass in CGS system= 2.8 g/cc
26 (a) Given, x = g cm2 s −5 = [ML2T −5]
= 2.8 g/cm3
−1 −1
y =gs = [ML T ]
0
2.8 × 10 −3 kg
−2 −2 Value of density in SI system =
and z = cms = [M LT ]0
10 −6 m3
Now, z 2 = [M0 L2T −4 ] = 2.8 × 10 3 kgm−3
−1 2 −4 2 −5
and yz = [ML T ][M L T ] = [ML T ] = x
2 0 0
1 y
35 (b) Given equation, =
i.e. x = yz 2 pβ kBT
mv Ft where, p = pressure, y = distance,
27 (d) We know that, F = ma ⇒ F = ⇒ m= kB = Boltzmann constant and T = temperature.
t v
[ Dimensions of kB ] [ Dimensions of T ]
∴ Dimensions of [m] =
[F][T]
= [Fv −1T] Dimensions of [β] =
[v] [ Dimensions of p] [ Dimensions of y]
[ ML2T −2K−1 ][K]
28 (a) Reynold’s number describes the ratio of inertial force per =
[ ML−1 T −2][ L]
unit area to viscous force per unit area for a flowing fluid.
Thus, Reynold’s number is the ratio of two physical quantity = [ M0 L2T 0 ]
of same dimension which cancel out each other. Hence, Dimensions of [IFv 2]
Reynold’s number is dimensionless [M0 L0 T 0 ] quantity. 36 (b) Dimensions of [X ] =
[ 3]
Dimensions of WL
x
29 (a) Substituting dimensions, [MLT −2] = [ML2] [MLT −2] [LT −1]2
[ML−3] =
[ML2T −2] [L3]
⇒ x = [M3/ 2L−1/ 2T −2]
[M2L5T −4 ]
= = [MT −2]
30 (c) The absolute error in the value A – B will be a + b. [ML5T −2]
31 (c) Let x kg be unit of mass, then 2k 3l 2
37 (c) Here, X =
f = (x kg )(1000 m)(100 s )−2 m n
1 Percentage error in X
⇒ 1000 = x × 1000 × 3 × ∆k ∆l ∆m 1 ∆n
10000 = × 100 + 2 × × 100 + × 100 + × × 100
∴ x = 10000 kg k l m 2 n
= 10 4 kg  1
=  3 × 1% + 2 × 2% + 3% × 1 + 4% ×  = 12%
 2
 nh 
32 (a) The quantity is given as  A
, where n and 2π are 38 (b) Given, m =
 2πqB  B
dimensionless quantities. [A]
nh ∴ Dimensions of B = [B] =
Q mvr = …(i) [m]

Here, A = force = [MLT − 2]
Also using, Bqr = mv
[M]
⇒ Bq = mv /r …(ii) and m = linear density = mass per unit length =
[L]
50 OBJECTIVE Physics Vol. 1

[MLT − 2] 42 (a) Dimensions of velocity are [v ] = [L][T −1]


∴ [B] = = [L2T − 2]
[ML− 1]
So, dimensions of [at 2] = [LT −1]
These are same dimensions as that of latent heat.
⇒ [a] [T 2] = [LT −1]
39 (a) Given, X = [Ma LbT c ]
Maximum % error in X = a α + b β + c γ [a] = [LT −3]
1 Dimensions of [bt] = [LT −1] ⇒ [b ] [T] = [LT −1]
40 (b) Q (µ 0 ε 0 )−1/ 2 = = speed of light
(µ 0 ε 0 )1/ 2 ⇒ [b ] = [LT −2]

⇒ [c] =
[L]
= [LT −1] Dimensions of [c] = [LT −1]
[T]
R3
RT −αV /RT 43 (a) Taking, T = 2π
41 (a) Given, p = e GM
V −b
αV Substituting the dimensions, LHS = T = [T]
So, is dimensionless.
RT R3 [L] 3
2 −2 −1 RHS = 2π = −1 3 −2
= [T] 2 = [T]
RT  [ML T θ ] [θ] GM [M L T ] [M]
Hence, [α ] = = = [ML−1T −2]
 V  [L3]
R3
This is the dimensional formula of pressure (p ). Thus, LHS = RHS for T = 2π .
GM
CHAPTER
02

Vectors
In physics, we study a large number of physical quantities. These physical
quantities can either have magnitude or magnitude and direction both. On this
basis, we have broadly categorised physical quantities into two categories : scalars
and vectors. In this chapter, we will study about the vector quantities and their
operations in detail.

SCALAR AND VECTOR QUANTITIES


Scalar quantities
A physical quantity which can be described completely by its magnitude only and
does not require a direction is called scalar quantity. Addition, subtraction,
multiplication or division of scalar quantities can be done according to the general
rules of algebra. Mass, volume, density, etc., are few examples of scalar
quantities.

Vector quantities
A physical quantity which has both magnitude and particular direction and obeys
the triangle law of vector addition or equivalently the parallelogram law of vector
addition is called a vector quantity. Displacement, velocity, acceleration, etc., are
few examples of vector quantities.
Note The physical quantity current has both magnitude and direction but it is still a scalar as it
disobeys the laws of vector algebra. Inside
General points regarding vectors 1 Scalar and vector quantities
General points regarding vectors
General points regarding vectors are as follows Types of vectors
Vector notation Usually a vector is represented by a bold capital letter with an Multiplication and division
→ → → of vectors by scalars
arrow (or without arrow) over it, as A, B, C or simply A, B, C. 2 Addition of two vectors
The magnitude of a vector A is represented by A or | A|. 3 Subtraction of two vectors
Graphical representation of a vector Graphically a vector is represented by an Resolution of vectors

arrow drawn to a chosen scale, parallel to the direction of the vector. The length 4 Product of two vectors
of the arrow represents the magnitude and the tip of the arrow (arrow-head) Scalar product of two vectors
Vector product of two vectors
represents the direction.
Suppose that a car A is running with a velocity of 10 m/s towards east; and
another car B is running with a velocity of 20 m/s towards north-east.
52 OBJECTIVE Physics Vol. 1

These velocities can be represented by vectors drawn in


Fig. 2.1. Types of vectors
Different types of vectors are given below
N
(i) Polar vectors These are the vectors which have a
initial starting point or a point of application.
W E e.g. Displacement, force, etc.
Velocity of (ii) Axial vectors These are the vectors which
car A
represent rotational effect and act along the axis of
S
Velocity of rotation in accordance with right hand screw rule or
car B right hand thumb rule. e.g. Angular velocity, angular
Fig. 2.1 acceleration, torque, etc.
To draw these vectors, it has been assumed that 1 cm ω (Angular velocity) τ
length represents a velocity of 5 m/s. The velocity vector A
F
of car A is an arrow of length 2 cm and its tip is directed
towards east, while that of B is an arrow of length 4 cm
with its tip directed towards north-east.
Angle between two vectors (θ ) Angle between two Axis of rotation
Fig. 2.5 Axial vectors
vectors is the smaller of two angles between the vectors
when they are placed tail to tail. (iii) Equal vectors These are the vectors which have
B B equal magnitude and same direction. In Fig. 2.6, A
A
and B are equal vectors. i.e., A = B
120°
⇒ A

θ = 60° B
A
(a) (b) Fig. 2.6 Equal vectors
Fig. 2.2
(iv) Parallel vectors These are the vectors which have
For example, in Fig. 2.2, angle between A and B is 60° same direction but their magnitude may be equal or
not 120°. Because in Fig. (a), their tails are not together different. The angle between two parallel vectors is
while in Fig. (b), they are drawn correctly. always 0°.
If a vector is displaced parallel to itself, it does not A
change.
B

Fig. 2.7 Parallel vectors


A B
(v) Anti-parallel vectors These are the vectors which
θ θ have opposite direction but their magnitude may be
equal or different. The angle between two
Fig. 2.3 anti-parallel vectors is always 180°.
∴ B=A A
If a vector is rotated by an angle –B
(θ ≠ 2nπ , n = 1, 2 , 3 , ... ), the vector is changed.
A B Fig. 2.8 Anti-parallel vectors

(vi) Collinear vectors These are the vectors which lie


along the same line. Angle between them can be 0°
or 180°. e.g.
θ
α
(a) (θ = 0°) (b) (θ = 0°)
Fig. 2.4
(c) (θ = 180°) (d) (θ = 180°)
∴ B≠A
Vectors 53

(vii) Zero or null vector A vector having zero i.e. A=B


magnitude is known as zero vector. Its direction is Here, A = i$ − 3$j + 5k$ and B = i$ − 3$j − ak$
not specified and hence is arbitrary. It is represented ⇒ i − 3j + 5k$ = i$ − 3$j − ak$
$ $ \
by 0. e.g. Displacement, velocity and acceleration of
a particle at rest or acceleration of a particle moving Comparing on both sides, we get
with uniform velocity. a=−5
 i$ j$ 
(viii) Unit vector A vector whose magnitude is one unit Example 2.2 Check whether the vector  +  is a unit
and points in a particular direction, is called unit vector or not.  2 2
vector. It is represented by A $ (A cap or A hat). The
Sol. A unit vector is a vector with magnitude equals to 1.
unit vector along the direction of A is The magnitude of given vector is
$ = Vector A
A =  i$ j$   1 
2
 1 
2
1 1
Magnitude of the vector |A |  +  =   +   = + = 1 =1
 2 2  2   2  2 2
(Here, A = A $i + A $j + A k$ )
x y z
As magnitude of given vector is 1.
∴ A=A
$ |A | ∴ It is a unit vector.
where, | A | = Ax2 + Ay2 + Az2 Example 2.3 Find the unit vector of 4i$ − 3j$ + k$ .
Y Sol. Let the given vector be A = A i$ + A j$ + A k$
x y z

= 4i$ − 3$j + k$
^j
∴ Ax = 4, Ay = − 3, Az = 1

| A | = Ax2 + Ay2 + Az2 = (4)2 + (−3)2 + (1)2 = 26


X
^i
$ = A = 4i − 3j + k
$ $ $
^k ∴ Unit vector, A
Z |A | 26
Fig. 2.9
Example 2.4 If A = 4i$ + 3$j and B = 24i$ + 7j$, find the vector
In cartesian coordinates $i, $j, k$ are the unit vectors having the same magnitude as B and parallel to A.
along X-axis, Y-axis and Z-axis, respectively. Sol. Magnitude of B , | B | = 242 + 72 = 25
where, | $i | = | $j | = | k$ | = 1 Magnitude of A, | A| = 42 + 32 = 5
Note $ = 4i + 3j
A unit vector is used to specify the direction of a vector. $ $
Unit vector A, A
(ix) Coplanar vectors These are the vectors which 5
always lie in the same plane. (4i$ + 3$j )
Required vector, r = 25 = 20 i$ + 15 $j
Note Two vectors are always coplanar vectors. 5
(x) Negative or opposite vector If the direction of a
vector is reversed, the sign of the vector get
Multiplication and division of
reversed. It is called negative vector of the original vectors by scalars
vector. The product of a vector A and a scalar m gives a vector
In Fig. 2.10, B is negative vector of A, i.e. A = − B. mA whose magnitude is m times the magnitude of A and
which is in the direction or opposite to A accordingly, if
A B the scalar m is positive or negative. Thus, m (A ) = mA
Fig. 2.10 Further, if m and n are two scalars, then
Example 2.1 If vectors i$ − 3j$ + 5k$ and i$ − 3j$ − ak$ are equal (m + n ) A = mA + nA and m (nA ) = n (mA ) = (mn ) A
vectors, then find the value of a. The division of vector A by a non-zero scalar m is defined
1
Sol. Two vectors are said to be equal, if their magnitude is equal as the product of A and ⋅
and direction is same. m
OBJECTIVE Physics Vol. 1

CHECK POINT 2.1


1. Which of the following is not the vector quantity? 5. Find the vector that must be added to the vector
(a) Torque (b) Displacement i$ − 3j$ + 2 k$ and 3i$ + 6j$ − 7k$ , so that the resultant vector
(c) Dipole moment (d) Electric flux
is a unit vector along theY-axis.
2. Surface area is (a) −4 $i − 2$j + 5 k
$ (b) −4 $i + 2$j + 5 k
$
(a) scalar (c) 4 i − 2j + 5k
$ $ $ (d) − 4 i − 2j − 5 k
$ $ $
(b) vector
(c) Both (a) and (b) 6. The magnitude of i$ + j$ is
(d) Neither scalar nor vector (a) 2 (b) 0
3. I. Pressure II. Temperature (c) 2 (d) 4

III. Momentum IV. Work 7. The direction of unit vector along i$ + j$ is


$i + $j $i + $j
Which of the following physical quantities are scalars ? $
(a) k (b) $i + $j (c) (d)
(a) I and II (b) I, II and III 2 2
(c) I, II and IV (d) II and III
8. What happens, when we multiply a vector by (–2)?
4. A vector multiplied by the number 0, results into (a) Direction reverses and unit changes
(a) 0 (b) A (b) Direction reverses and magnitude is doubled
(c) 0 $
(d) A (c) Direction remains unchanged and unit changes
(d) None of the above

ADDITION OF TWO VECTORS


Vectors cannot be added by simple laws of algebra, 2. Parallelogram law of vector addition
which are applicable to scalars. To add two vectors, we It states that, if two vectors acting on a particle at the same
must follow certain laws. These laws are described time can be represented with magnitude and direction by
below the two adjacent sides of a parallelogram drawn from a
point, then their resultant vector is represented in
1. Triangle law of addition magnitude and direction by the diagonal of the
of two vectors parallelogram drawn from the same point.
It states that, if two vectors acting on a particle at the (i) Magnitude of resultant vector Let R be the
resultant of two vectors A and B. According to
same time are represented with magnitude and direction
parallelogram law of vector addition, the resultant R is
by the two sides of a triangle taken in same order, then the diagonal of the parallelogram of which A and B
their sum or resultant is represented in magnitude and are the adjacent sides as shown in figure below.
direction by the third side of the triangle taken in R Q
opposite order.
As is evident from the figure that the resultant R is the B
A+ B sin θ
B R=
same irrespective of the order in which the vectors A B
β
and B are taken. θ
A
α θ
Thus, O S
A P
+B B cos θ
=A B
R Fig. 2.12
B R=B+A

Magnitude of R is given by
A
Fig. 2.11 R = A 2 + B 2 + 2AB cos θ …(i)
R = A + B =B + A Here, θ = angle between A and B.
This is the geometrical method of vector addition. Eq. (i) is also known as law of cosines.
Vectors 55

R A B Example 2.5 Two forces whose magnitude are in the ratio


Also, = = is known as law of sines.
sin θ sin β sin α 3 : 5 give a resultant of 28 N. If the angle of their inclination
is 60°, find the magnitude of each force.
Special cases
Sol. Let A and B be the two forces, then A = 3x , B = 5x
• Resultant of two vectors will be maximum when
they are parallel, i.e. angle between them is zero. Q R = 28 N and θ = 60°

or R max = A + B Now, R = A2 + B 2 + 2AB cos θ


• Resultant of two vectors will be minimum when ∴ 28 = (3x )2 + (5x )2 + 2(3x )(5x ) cos 60°
they are anti-parallel, i.e. angle between them is
180°. 28 = 9x 2 + 25x 2 + 15x 2 = 7x or x = 4
or R min = A − B ∴ A = 3 × 4 = 12 N
• Resultant of two vectors of unequal magnitude can and B = 5 × 4 = 20 N
never be zero.
Example 2.6 If A = B + C have scalar magnitudes of 5, 4, 3
(ii) Direction of resultant vector Let θ be the angle units respectively, then find the angle between A and C .
between A and B, then
Sol. Here, triangle OMN is given with vectors A, B and C are its
| A + B | = A 2 + B 2 + 2AB cos θ adjacent sides.
If R makes an angle α and β with A and B N
respectively, then θ
B sin θ A
tan α = C
A + B cos θ
A sin θ
tan β = O
B
M
B + A cos θ
MN  | C | − 1  3
As, cos θ = ⇒ θ = cos− 1   = cos  
Polygon law of vector addition for more ON  | A |  5
than two vectors Example 2.7 Find the sum of vectors A and B as shown in the
It states that, if n number of vectors acting on a particle at figure, also find the direction of sum vector. Given, A =
the same time are represented in magnitude and direction 4 unit and B = 3 unit.
by various sides of an open polygon taken in the same B
order, then their resultant is represented in magnitude and
direction by the closing side of the polygon taken in
opposite order.
D D C θ = 60°
A
E C
Sol. According to the question, we draw the following figure
E B
B R=A+B
R B

O A
A α
Fig. 2.13 θ A
Resultant of the vectors A and B,
Thus, in the figure,
OE = OA + AB + BC + CD + DE R = A2 + B 2 + 2AB cos θ
= 16 + 9 + 2 × 4 × 3 cos 60°
∴ R=A+B+C+D+E
= 37 unit
Note
(i) Resultant of two vectors is always located in their common plane. ∴ Direction of the sum vector,
(ii) Vector addition is commutative, i.e. A + B = B + A B sin θ
tan α =
(iii) Vector addition is associative A + B cos θ
i.e. A + (B + C ) = ( A + B) + C 3 sin 60°
(iv) If vectors are of unequal magnitude, then minimum three coplanar = = 0.472
4 + 3 cos 60°
vectors are required for zero resultant.
56 OBJECTIVE Physics Vol. 1

∴ α = tan−1 (0.472) = 25.3° θ


If θ = 60 °, then 2a sin   = a
Thus, resultant of | A | and | B | is 37 unit at angle 25.3°  2
from A in the direction shown in figure. i.e. | A − B | = | A | = | B | = a at θ = 60 °
• If two vectors are such that their sum and
SUBTRACTION OF TWO difference have equal magnitude, then angle
between the given vectors θ = 90 ° .
VECTORS i.e. | A + B | = | A − B |
Negative of a vector say −A is a vector of the same then cos θ = 0 or θ = 90 °
magnitude as vector A but pointing in a direction opposite • If A + B = A − B
to that of A.
then B = 0 (a null vector)
Thus, A − B can be written as A + (−B ) or A − B is the
Note
vector addition of A and − B.
(i) The vector subtraction does not follow commutative law,
i.e. A − B ≠ B − A.
A (ii) The vector subtraction does not follow associative law,
B ( = –A ) i.e. A − (B − C ) ≠ ( A − B) − C
Fig. 2.14 Example 2.8 Find the subtraction of vector A and B as shown
Suppose angle between two vectors A and B is θ. Then, in the figure, also find the direction of subtraction vector.
angle between A and − B will be 180° − θ as shown in Given, A = 4 unit and B = 3 unit.
Fig. 2.15 (b). B

B 180° – θ
A
α
θ = 60°
⇒ β
A
θ
A –B Sol. According to the question, we draw the following figure.
R=A–B
(a)
(b) θ
A
Fig. 2.15 α

Magnitude of resultant vector R = A − B will be thus


given by
−B
| R| = | A − B | = A 2 + B 2 + 2AB cos (180 ° − θ ) R=A−B

or | R| = A 2 + B 2 − 2AB cos θ …(i) Magnitude of resultant of the vectors A and B,


R = A2 + B 2 − 2AB cos θ
For direction of resultant vector R, we will either calculate
angle α or β, where = 16 + 9 − 2 × 4 × 3 cos 60°
B sin (180 ° − θ ) B sin θ
tan α = = …(ii) = 13 unit
A + B cos (180 ° − θ ) A − B cos θ and direction is
A sin (180 ° − θ ) A sin θ B sin θ
or tan β = = …(iii) tan α =
B + A cos (180 ° − θ ) B − A cos θ A − B cos θ
Special cases 3 sin 60°
= = 1.04
4 − 3 cos 60°
• If two vectors have equal magnitudes,
i.e. | A | = | B | = a and θ is the angle between ∴ α = tan−1 (1.04) = 461

them, then Thus, A – B is 13 unit at 46.1° from A in the direction
θ
| A − B | = a 2 + a 2 − 2a 2 cos θ = 2a sin   shown in figure.
 2
Vectors 57

Example 2.9 Obtain the magnitude of 2A − 3B, if Refer Fig. (a)


A = i$ + j$ − 2 k$ and B = 2i$ − j$ + k$ . We have resolved a two dimensional vector (in XY-plane)
Sol. The magnitude of 2A − 3B is R in mutually perpendicular directions x and y.
Y Y
2 A − 3B = 2 (i$ + $j − 2k$ ) − 3 (2i$ − $j + k$ )
= − 4i$ + 5 j$ − 7k$ Ry R Ry R
∴ Magnitude of 2 A − 3 B = (−4) + (5) + (−7) 2 2 2
β β β
α α
= 16 + 25 + 49 O X O X
Rx Rx
= 90 (a) (b)
Fig. 2.16
Example 2.10 Two vectors P and Q have equal magnitudes.
If the magnitude of (P + Q ) is ‘ k ’ times the magnitude of Component along X-axis = R x = R cos α or R sin β
(P − Q ), then calculate the angle between P and Q . Component alongY-axis = R y = R cos β or R sin α
Sol. Given, |P| = | Q| If $i and $j be the unit vectors along X andY-axes
or P =Q …(i)
respectively, we can write, R = R x $i + R y $j
Let magnitude of (P + Q ) is R and for (P − Q ) is R ′ Refer Fig. (b)
Now, R =P + Q Vector R has been resolved in two axes such that x and y
and R 2 = P 2 + Q 2 + 2PQ cos θ not perpendicular to each other. Applying sine law in the
R 2 = 2P 2 + 2P 2 cos θ …(ii) triangle shown , we have
R R Ry
Again, R′ = P − Q = x =
(R ′ )2 = P 2 + Q 2 − 2PQ cos θ sin [180° − (α + β)] sin β sin α
(R ′ )2 = 2P 2 − 2P 2 cos θ R sin β R sin α
…(iii) or Rx = and R y =
2 sin (α + β) sin (α + β)
R
Given, R = kR ′ or   = k 2
 R ′ If α + β = 90 °, R x = R sin β and R y = R sin α
Dividing Eq. (ii) by Eq. (iii), we get Ry Ry 
Also, tan α = or α = tan −1  
k 2 1 + cos θ
=
Rx  Rx 
1 1 − cos θ
Rectangular components of a vector in
k2 − 1 (1 + cos θ ) − (1 − cos θ )
or = three dimensional space
k +1
2
(1 + cos θ ) + (1 − cos θ )
Let R x , R y and R z are the components of resultant vector
2 cos θ R in X,Y and Z-axes respectively, and $i, $j and k$ are unit
= = cos θ
2
vectors along these directions. Then, a vector R and its
 k 2 − 1 −1  k − 1
2
magnitude can be written as
∴ cos θ =  2  or θ = cos  2 
 k + 1  k + 1 Y
R
Resolution of vectors Ry
β
β
The resolution of a vector is opposite to vector addition. If a α Rx
X
vector is resolved into two vectors whose combined effect is γ
Rz
the same as that of the given vector, then the resolved
Z
vectors are called the components of the given vector.
Fig. 2.17
Resolution of vectors into R = R x + R y + R z or R = R xi$ + R y $j + R z k$

rectangular components R = R x2 + R y2 + R z2
When a vector is splitted into components which are at This vector R makes an angle of
right angle to each other, then the components are called
R 
rectangular or orthogonal components of that vector. α = cos−1  x  with X-axis
R
58 OBJECTIVE Physics Vol. 1

Ry 
β = cos−1   withY -axis Sol. Consider the figure shown below.
R
R 
and γ = cos−1  z  with Z-axis C
E
R
45° B
Note
D O
(i) A vector can be resolved into maximum infinite number of
components.
For example,10i$ = i$ + i$ + i$ K 10 times A
i$ i$ i$ Resolve OC into two rectangular components,
or = + + K 20 times
2 2 2
OD = OC cos 45° and OE = OC sin 45°
(ii) A vector is independent of the orientation of axes but the
components of that vector depends upon the orientation of axes. To obtain zero resultant,
(iii) The component of a vector along its perpendicular direction is OE = OA or OC sin 45° = 10 N
always zero. 1
⇒ OC × = 10 N
Example 2.11 Find the angle that the vector A = 2$i + 3$j − k$ 2
makes withY-axis. |OC | = 10 2 N and OD = OB
Sol. According to the resolution of the vector, ⇒ OC cos 45° = OB
Ay 3 3 1
cos θ = = = ⇒ OB = 10 2 × = 10 N
A (2) + (3) + (− 1)
2 2 2 14 2

 3  Thus, the magnitude of OB and OC is 10 2 N and 10 N.


∴ θ = cos−1  
 14  Example 2.14 Find the resultant and direction of three
vectors as shown in the figure.
Example 2.12 A vector is given by A = 3 i$ + 4 j$ + 5 k$ . Find y
the magnitude of A, unit vector along A and angles made by 3m
A with coordinate axes.
Sol. We have, magnitude, | A| = A = Ax2 + Ay2 + Az2 1m
m
2
5√

= (3)2 + (4)2 + (5)2 = 5 2 45°


O x
$ = A = 3i$ + 4j$ + 5k$
Unit vector, A Sol. From the figure,
|A| 5 2
On X-axis, x = 5 2 cos 45° + 1 = 5 + 1 = 6 m
Angles made by A with coordinate axes,
OnY -axis, y = 5 2 sin 45° + 3 = 5 + 3 = 8 m
A 3  3 
cos α = x = ⇒ α = cos−1  
| A| 5 2  5 2 8

Ay 4  4 
cos β = = ⇒ β = cos−1  
| A| 5 2  5 2 R

Az 5  1  π θ
cos γ = = ⇒ γ = cos−1   = 6
| A| 5 2  2 4
∴ Magnitude of resultant of given vectors,
Example 2.13 Find the magnitude of vectors OB and OC . R = x2 + y 2
If sum of three vectors gives a value equals to 0 as shown
in figure below. = (6)2 + (8)2 = 10 m
∴ Direction of resultant vectors,
C
y 8 4
tan θ = = =
x 6 3
45°
 4
O B ⇒ θ = tan−1  
 3
 4
Thus, resultant vector makes an angle of tan −1   with
 3
A=10N
X-axis.
CHECK POINT 2.2
1. For the resultant of two vectors to be maximum, what must 8. Three vectors each of magnitude A are acting at a point
be the angle between them? such that angle between any two consecutive vectors in
(a) 0° (b) 60° same plane is 60°. The magnitude of their resultant is
(c) 90° (d) 180° (a) 2A (b) 2A
2. Minimum number of vectors of unequal magnitude whose (c) 3 A (d) 6A
vector sum can equal to zero is 9. If P + Q = P − Q, then
(a) two (b) three
(a) P = 0 (b) Q = 0
(c) four (d) Any
(c) P = 1 (d) Q Q | = 1
3. Two vectors having magnitudes 8 and 10 can have maximum
and minimum value of magnitude of their resultant as
10. Resultant of two vectors A and B is given by R = {A − B},
(a) 12, 6 (b) 10, 3 angle between A and B will be
(c) 18, 2 (d) None of these (a) 90° (b) 180°
(c) 0° (d) None of these
4. Given that, P + Q + R = 0. Which of the following statement
is true?
11. If|A| = 2 and|B| = 4 and angle between them is 60°, then
(a) |P| + |Q| = |R| (b) |P + Q| = |R|
| A − B|
(c) |P| − |Q| = |R| (d) |P − Q| = |R| (a) 13 (b) 3 3
(c) 3 (d) 2 3
5. (P + Q) is a unit vector along X-axis. If P = $i − $j + k$ , then
12. A vector inclined at an angle θ to the horizontal as shown in
what is the value of Q?
figure below. If its component along X-axis is 50 N, then its
(a) $i + $j − k
$ (b) $j − k
$ (c) $i + $j + k
$ (d) $j + k
$
magnitude in y-direction is
6. A = 2$i + $j , B = 3$j − k$ and C = 6$i − 2k$ .
Y
Value of A − 2 B + 3 C would be
(a) 20$i + 5$j + 4 k
$ (b) 20$i − 5$j − 4k
$

(c) 4 i + 5j + 20 k
$ $ $ (d) 5i + 4 j + 10 k
$ $ $

7. At what angle should the two vectors 2P and 2P act, so


that the resultant force is P 10? θ=60°
X
(a) 45° (b) 60°
(c) 90° (d) 120°
(a) 50 N (b) 72 N (c) 64 N (d) 87 N

PRODUCT OF TWO VECTORS


The multiplication of two vector quantities cannot be
done by simple algebraic method. The product of two
vectors may be a scalar as well as a vector. B

If the product of two vectors is a scalar quantity, then A.B = AB cos θ


it is called scalar product (or dot product); if the
product is a vector quantity, then it is called vector θ
product (or cross product). A
Fig. 2.18
Scalar product of two vectors A ⋅ B = AB cos θ (A scalar quantity)
The scalar product (or dot product) of two vectors is e.g. work done (W ) = F ⋅ s and power (P ) = F ⋅ v
defined as the product of their magnitude with cosine
Note (i) The scalar or dot product of two vectors A and B is denoted by A ⋅ B
of the angle between them. and is read as A dot B.
Thus, if there are two vectors A and B having angle θ (ii) Dot product is always a scalar, which is positive, if angle between
between them, then their scalar product is written as the vectors is acute (i.e. θ < 90°) and negative, if angle between
them is obtuse (i.e. 90° < θ < 180°).
60 OBJECTIVE Physics Vol. 1

Important points regarding dot product Example 2.16 Prove that the vectors A = 2i$ − 3$j + k$ and
The following points should be remembered regarding the B = i$ + $j + k$ are mutually perpendicular.
dot product Sol. A ⋅ B = (2i$ − 3$j + k$ ) ⋅ (i$ + $j + k$ )
(i) A ⋅ B = B ⋅ A (i.e. dot product is commutative) = (2)(1) + (−3)(1) + (1)(1)
(ii) A ⋅ (B + C) = A ⋅ B + A ⋅ C (i.e. dot product is distributive) = 0 = AB cos θ (Q A ⋅ B = AB cos θ)
∴ cos θ = 0 (As A ≠ 0, B ≠ 0)
(iii) A ⋅ A =A 2 (also called self-dot product)
or θ = 90° (Q cos 90° = 0)
(iv) A ⋅ B = A(B cos θ ) = A (component of B along A) or the vectors A and B are mutually perpendicular.
or A ⋅ B = B (A cos θ ) = B (component of A along B) Example 2.17 Find the angle between two vectors
(v) $i ⋅ $i = $j ⋅ $j = k$ ⋅ k$ = (1)(1) cos 0 ° = 1 A = 2i$ + $j − k$ and B = i$ − k$ .
(vi) $i ⋅ $j = $j ⋅ k$ = $i ⋅ k$ = (1)(1) cos 90 ° = 0
Sol. A = | A| = (2)2 + (1)2 + (−1)2 = 6
(vii) (a1$i + b 1$j + c 1k$ ) ⋅ (a 2 $i + b 2 $j + c 2 k$ )
B = | B | = (1)2 + (−1)2 = 2
= a1a 2 + b 1b 2 + c 1c 2
A ⋅ B = (2i$ + j$ − k$ ) ⋅ (i$ − k$ ) = (2)(1) + (−1)(−1) = 3
A⋅B
(viii) cos θ = (cosine of angle between A and B) A⋅B 3 3 3
AB Now, cos θ = = = =
AB 6⋅ 2 12 2
(ix) Two vectors are perpendicular (i.e. θ = 90 °), if their
dot product is zero. ∴ θ = 30°
(x) Dot product of two vectors will be maximum when Example 2.18 Find the component of vector A + B along
vectors are parallel (i.e. θ = 0) (A ⋅ B ) max = AB (i) X-axis (ii) and C.
Projection of A along B (Components of dot product) Given, A = i − 2j, B = 2i + 3k$ and C = i$ + $j .
$ $ $

Sol. A + B = ($i − 2$j ) + (2$i + 3k$ ) = 3$i − 2$j + 3k$


A (i) Component of A + B along X-axis is 3.
(ii) Component of A + B = R (say) along C is
R ⋅ C = RC cos θ
θ
B R ⋅ C (3i$ − 2$j + 3k$ ) ⋅ (i$ + $j ) 3 − 2 1
A cos θ ∴ R cos θ = = = =
Fig. 2.19 C (1) + (1)
2 2 2 2
(i) In scalar form : Projection or scalar component of Example 2.19 Find the (i) scalar component and (ii) vector
A along B component of A = 3i$ + 4$j + 5k$ on B = i$ + j$ + k$ .
A⋅B A⋅B
= A cos θ = A × = = A⋅B
$ Sol. (i) Scalar component of A along B is
AB B (i$ + $j + k$ )
(ii) In vector form : Projection or vector component of A cos θ = A ⋅ B$ = (3i$ + 4$j + 5k$ ) ⋅
3
A along B 3 + 4 + 5 12
= = =4 3
$ =  A × A ⋅ B  B
= (A cos θ ) B $ 3 3
 AB  (ii) Vector component of A along B is
A⋅B $ (i$ + j$ + k$ )
= ⋅ B = (A ⋅ B
$) B
$ (A cos θ ) B$ = (A ⋅ B$ ) B$ = (4 3 ) = 4i$ + 4j$ + 4k$
B 3
Example 2.15. Find the projection of A = 2i$ − $j + k$ on Vector product of two vectors
B = i$ − 2j$ + k$ .
The vector product or cross product of B
Sol. Projection of A on B = A cos θ (where, θ = angle between A
two vectors is defined as a vector having
and B) magnitude equal to the product of their
A⋅B (i$ − 2j$ + k$ ) magnitudes with the sine of angle
= = (2i$ − j$ + k$ ) ⋅
B (1)2 + (−2)2 + (1)2 between them, and its direction is θ
A
perpendicular to the plane containing
2 + 2 +1 5 Fig. 2.20
= = both the vectors according to right hand
6 6 screw rule.
Vectors 61

Thus, if A and B are two vectors, then their vector (iii) If two vectors are perpendicular to each other, we
product, i.e. A × B gives a vector C and is defined by have θ = 90 °, i.e. sin θ = 1. So that, A × B = AB n$ .
C = A × B = AB sin θ n$ . These vectors A, B and A × B thus form a right
where, n$ is a unit vector perpendicular to the plane of A handed system of mutually perpendicular vectors.
and B. It follows at once from the above that in case of the
The direction of C (or of n$ ) is determined by right hand orthogonal triad of unit vectors $i, $j and k$ (each
screw rule and right hand thumb rule. perpendicular to each other)
(i) Right Hand Screw Rule Rotate a right handed ∧ ∧
i i
screw from first vector (A ) towards second vector (B ).
The direction in which right handed screw moves
gives the direction of vector (C) as shown in Fig. 2.21. Plus Minus
∧ ∧ ∧ ∧
k j k j
C=A×B

Fig. 2.23

$i × $j = − $j × i$ = k$
$j × k$ = − k$ × $j = $i

θ and k$ × i$ = − i$ × k$ = $j

B (iv) A × (B + C) = A × B + A × C
A (v) A vector product can be expressed in terms of
Plane of A and B rectangular components of the two vectors and put
Fig. 2.21 in the determinant form as may be seen from the
following
The direction of C (or of n$ ) is perpendicular to the Let A = a1$i + b 1$j + c 1k$
plane containing A and B; and its sense is decided
by right hand screw rule. and B = a $i + b $j + c k$
2 2 2
(ii) Right Hand Thumb Rule If the fingers of the right Putting it in determinant form, we have
hand be curled in the direction in which vector A
must be turned through the smaller included angle θ
$i $j k$
to coincide with the direction of vector B, the thumb A × B = a1 b 1 c1
points in the direction of C as shown in Fig. 2.22. a2 b2 c2
AH B = C
It may be noted that the scalar components of the
first vector A occupy the middle row of the
determinant.
(vi) A unit vector ($n) perpendicular to A as well as B is
A×B
given by n$ =
|A × B |
A θ (vii) If A, B and C are coplanar, then [A ⋅ (B × C)] = 0
B (viii) Angle between (A + B ) and (A × B ) is 90°.
Fig. 2.22 (ix) Two vectors can be shown parallel to each other, if
Important points regarding vector product • The coefficient of $i, $j and k$ of both the vectors
(i) A × B = − B × A bear a constant ratio. For example, a vector
(ii) The magnitude of cross product of two parallel A = a1$i + b 1$j + c 1k$ is parallel to another vector
vectors is zero, as | A × B | = AB sin θ and θ = 0 ° for B = a 2 $i + b 2 $j + c 2 k$ , if
two parallel vectors. Thus,
a1 b 1 c1
$i × $i = $j × $j = k$ × k$ = 0 = =
a2 b2 c2
62 OBJECTIVE Physics Vol. 1

• The cross product of both the vectors is zero. For Example 2.22 Show that the vector A = i$ − j$ + 2k$ is parallel
instance A and B are parallel to each other, if to a vector B = 3i$ − 3$j + 6k$ .
$i $j k$ Sol. A vector A is parallel to an another vector B, if it can be
written as
A × B = a1 b1 c1 = 0 A = mB
1
a2 b2 c2 Here, A = (i$ − $j + 2k$ ) = (3i$ − 3$j + 6k$ )
3
(x) The area of triangle bounded by vectors A and B is (Q B = 3i$ − 3j$ + 6k$ )
1 1
| A × B |. ∴ A= B
2 B 3
This implies that A is parallel to B and magnitude of A is
1/3 times the magnitude of B.

Example 2.23 Find a unit vector perpendicular to


A = 2i$ + 3$j + k$ and B = i$ − j$ + k$ both.
O A
Fig. 2.24 Sol. Given, A = 2i$ + 3$j + k$ and B = i$ − j$ + k$
Now, C = A × B is a vector, perpendicular to both A and B.
Area of triangle ABC If position vector of A is a, Hence, a unit vector n$ is perpendicular to both A and B. It can
position vector of B is b and position vector of C is c, be written as
then C A ×B
1 n$ = =
Area of triangle ABC = | a × b + b × c + c × a | C | A × B|
2 $i $j k$
(xi) Area of parallelogram shown in figure is
Here, A×B= 2 3 1
B 1 –1 1

d1 d2 = $i (3 + 1) + $j (1 − 2) + k$ (−2 − 3)
= 4$i − $j − 5k$
O A
Fig. 2.25 Further, | A × B | = (4)2 + (−1)2 + (−5)2 = 42
1 A×B
= |A × B| = |d1 × d 2| ∴ The desired unit vector is n$ =
2 | A × B|
where, d 1 and d 2 are diagonals. 1
or n$ = (4i$ − $j − 5 k$ )
Example 2.20 If a × b = b × c ≠ 0 with a ≠ − c, then show 42
that a + c = k b, where k is scalar.
Example 2.24 Let A, B and C be unit vectors. Suppose that
Sol. Given, a× b= b× c
A ⋅ B = A ⋅ C = 0 and that the angle between B and C is π/6,
a × b= − c× b then prove that, A = ± 2 (B × C)
∴ a × b + c× b= 0 Sol. Since, A ⋅ B = 0, A ⋅C = 0
(a + c) × b = 0 Hence, (B + C) ⋅ A = 0
Given, a × b ≠ 0, b × c ≠ 0, a, b, c, d are non-zero vectors. So, A is perpendicular to (B + C) and A is a unit vector
(a + c) ≠ 0 perpendicular to the plane of vectors B and C.
Hence, a + c is parallel to b. B× C
A=
∴ a + c= k b (where, k is scalar) | B × C|
Example 2.21 Prove that, | a × b |2 = a 2b 2 − (a ⋅ b) 2 where, | B × C | = | B || C | sin θ
Sol. Let | a | = a, |b | = b π  π
= | B || C |sin Qθ = 
6  6
and θ be the angle between them.
1 1
∴ | a × b |2 = (ab sin θ )2 = a 2b 2 sin2 θ = 1× 1× =
2 2
= a 2b 2 (1 − cos2 θ ) = a 2b 2 − (ab cos θ )2 B× C
∴ A= = ± 2 (B × C)
= a 2b 2 − (a ⋅ b ) 2 = RHS |B × C|
Vectors 63

Example 2.25 If a = 3i$ + $j − 4k$, b = 6i$ + 5$j − 2k$, then find Example 2.27 The adjacent sides of a parallelogram is given
the area of a triangle whose adjacent sides are determined
by two vectors A and B, where A = 5i$ − 4$j + 3k$ and
by a and b.
B = 3i$ − 2$j − k$ . Calculate the area of parallelogram.
Sol. Cross product of vectors a and b,
Sol. Here, A and B represents the adjacent sides of a
 i$ $j k$  parallelogram.

a × b = 3 1 −4  B

 

6 5 −2
 
= i (−2 + 20) − j (−6 + 24) + k$ (15 − 6) = 18i$ − 18j$ + 9k$
$ $
Magnitude of a and b, O A
| a × b | = (18)2 + (−18)2 + (9)2 = 729 = 27
A = 5i$ − 4$j + 3k$
1 27
∴ Area of ∆ = | a × b | =
2 2 B = 3i$ − 2$j − k$
= 13.5 sq. units Area of parallelogram = | A × B |
 i$ $j k$ 
Example 2.26 If the diagonals of a parallelogram are 2 i$ and
∴ 
A × B = 5 −4 3
2 $j , then find its area.

 

Sol. Let A = 2i$ and B = 2$j 3 −2 −1
 
1 1 = i$ (4 + 6) − $j (−5 − 9) + k$ (−10 + 12)
Area of parallelogram = | A × B | = [2i$ × 2 j$ ]
2 2
1 $ $ 1 $ = 10i$ + 14j$ + 2k$
= [4(i × j )] = | 4k | (Q i$ × j$ = k$ )
2 2 ⇒ | A × B | = (10)2 + (14)2 + (2)2
= 2 sq. units
= 300 = 10 3 sq. units

CHECK POINT 2.3


1. The modulus of the vector product of two vectors is
1 5. The condition (a ⋅ b)2 = a 2b 2 is satisfied when
3 (a) a is parallel to b (b) a ≠ b
times their scalar product. The angle between vectors is (c) a ⋅ b = 1 (d) a ⊥ b
π π
(a) (b) 6. When A ⋅ B = −| A|| B |, then
6 2
π π (a) A and B are perpendicular to each other
(c) (d)
4 3
(b) A and B acts in the same direction
2. What is the dot product of two vectors having magnitude of (c) A and B acts in the opposite direction
3 and 5; and the angle between them is 60° ?
(d) A and B can act in any direction
(a) 5.2 (b) 7.5
(c) 8.4 (d) 8.6 7. Given|A| = 2 ,|B | = 5 and|A × B | = 8. If angle between
3. The vector projection of a vector 3$i + 4 k$ on Y-axis is A and B is acute, then A ⋅ B is
(a) 6 (b) 3 (c) 4 (d) 7
(a) 5 (b) 4
(c) 3 (d) zero 8. If|A × B| = 3 A ⋅ B, then the value of|A + B | is
4. Three vectors A, B and C satisfy the relation A ⋅ B = 0 and 1/ 2
(a)  A2 + B2 +
AB 

A ⋅ C = 0. Then, the vector A is parallel to  3
(a) B (b) C (b) A + B
(c) B ⋅ C (d) B × C (c) (A2 + B2 + 3 AB)1 / 2
(d) (A2 + B2 + AB)1 / 2
64 OBJECTIVE Physics Vol. 1

Chapter Exercises
(A) Taking it together
Assorted questions of the chapter for advanced level practice

1 Out of the following quantities, which is scalar? 10 Vector P = 6$i + 4 2$j + 4 2k


$ makes an angle with
(a) Displacement (b) Momentum Z-axis which is equal to
(c) Potential energy (d) Torque  2
(a) cos−1   (b) cos−1 (2 2 )
2 The vector quantity among the following is  5
(a) mass (b) time
2 2
(c) distance (d) displacement (c) cos−1   (d) None of these
 5 
3 A vector is added to an equal and opposite vector of
similar nature, forms a 11 Which of the following is the unit vector
(a) unit vector (b) position vector perpendicular to A and B?
(c) null vector (d) displacement vector $ × B$
A $ × B$
A A×B A×B
(a) (b) (c) (d)
4 The component of a vector along any other direction AB sin θ AB cos θ AB sin θ AB cos θ
is 12 Condition under which vectors (a + b ) and (a − b )
(a) always less than its magnitude are parallel is
(b) always greater than its magnitude (a) a ⊥ b (b) | a | = | b |
(c) always equal to its magnitude (c) a ≠ b (d) a is parallel to b
(d) None of the above
13 The forces, which meet at one point but their lines
5 Which of the following is a unit vector? of action do not lie on one plane, are called
(a) i$ + j$ (b) cos θ i$ − sin θ j$ (a) non-coplanar non-concurrent forces
1 $ $ (b) non-coplanar concurrent forces
(c) sin θ i$ + 2 cos θ j$ (d) (i + j )
3 (c) coplanar concurrent forces
(d) coplanar non-concurrent forces
6 A vector P = 3 $i − 2$j + ak
$ is perpendicular to the
14 Consider a vector F = 4$i − 3 $j . Another vector
vector Q = 2$i + $j − k$ . The value of a is
(a) 2 (b) 1
perpendicular to F is
(c) 4 (d) 3 (a) 4i$ + 3$j (b) 6i$ (c) 7k$ (d) 3i$ − 4j$
$ and B = − $i − $j − k$ , then what is
7 If A = $i + $j + k 15 If the angle between two non-zero vectors A and B
the angle made by (A − B ) with A? is 120°, its resultant C will be
(a) 0° (b) 180° (a) C = | A − B | (b) C < | A − B |
(c) 90° (d) 60° (c) C > | A − B | (d) C = | A + B |
8 The angle between the two vectors − 2$i + 3 $j + k
$ 16 Two vectors A and B inclined at angle θ have a
and $i + 2$j − 4k$ is resultant R which makes an angle φ with A. If the
directions of A and B are interchanged, then the
(a) 45° (b) 90°
resultant will have the same
(c) 30° (d) 60°
(a) magnitude
9 Component of the vector A = 2$i + 3 $j along the (b) direction
vector B = ($i + $j ) is (c) magnitude as well as direction
(d) None of the above
5
(a) (b) 4 2 17 Which of the following is correct?
2
(a) | a − b | = | a | − | b | (b) | a − b | ≤ | a | − | b |
2 (c) | a − b | ≥ | a | − | b | (d) | a − b | > | a | − | b |
(c) (d) None of these
3
Vectors 65

18 If A = B, then which of the following is not correct? 30 If a unit vector is represent by 0.5$i + 0.8$j + c k
$ , then
(a) A$ = B$ (b) | A | = | B | the value of c is
(c) AB$ = BA
$ (d) A + B = A $ + B$ (a) 1 (b) 0.11
19 The angle between vectors (A × B ) and (B × A ) is (c) 0.01 (d) 0.39
(a) zero (b) π 31 A and B are two vectors given by A = 2$i + 3 $j and
(c) π /4 (d) π / 2 B = 2$i + 4$j . The magnitude of the component of A
20 Unit vector parallel to the resultant of vector 8 $i and along B is
8$j will be (a)
5
(b)
3
(c)
8
(d)
5
(a) (24 i$ + 5j$ )/13 (b) (12 i$ + 5j$ )/13 2 2 5 13
(c) (6 i$ + 5j$ )/13 (d) None of these 32 If F1 and F2 are two vectors of equal magnitudes F
21 The area of the parallelogram represented by the such that | F1 ⋅ F2 | = | F1 × F2 |, then | F1 + F2 | equals to
vectors A = 2$i + 3 $j and B = $i + 4$j is (a) (2 + 2) F (b) 2F
(a) 14 units (b) 7.5 units (c) F 2 (d) None of these
(c) 10 units (d) 5 units
33 The angle between the vectors A and B is θ. The
22 A vector is represented by 3 $i + $j + 2k
$ . Its length in
value of the triple product A ⋅ (B × A ) is
XY-plane is (a) A2B (b) zero (c) A2B sin θ (d) A2B cos θ
(a) 2 (b) 14
$ is a unit vector in a given direction, then the
34 If A
(c) 10 (d) 5
$
dA
23 What is the angle between (P + Q ) and (P × Q )? $ ⋅
value of A is
dt
π π
(a) Zero (b) (c) (d) π (a) 0 (b) 1 (c)
1
(d) 2
2 4 2
24 If three vectors along coordinate axes represent the 35 Find the resultant of three vectors OA, OB and OC
adjacent sides of a cube of length b, then the unit
shown in the following figure. (Radius of the circle is R)
vector along its diagonal passing through the origin
will be C
i$ + j$ + k$ i$ + j$ + k$ i$ + j$ + k$ B
(a) (b) (c) i$ + j$ + k$ (d)
2 36 3 45°
45°
O A
25 The angle between A = $i + $j and B = $i − $j is
[NCERT Exemplar]
(a) 45° (b) 90° (c) − 45° (d) 180°
26 Resultant of which of the following may be equal to (a) 2R (b) R (1 + 2 )
zero?
(a) 10 N, 10 N, 10 N (b) 10 N, 10 N, 25 N (c) R 2 (d) R ( 2 − 1)
(c) 10 N, 10 N, 35 N (d) None of these 36 If A = 3 $i + 4$j and B = 7$i + 24$j , the vector having
27 A vector perpendicular to both the vectors the same magnitude as B and parallel to A is
2$i − $j + 5k$ and X-axis is (a) 5i$ + 20j$ (b) 15i$ + 10j$
(a) $j + 5j$ (b) $j − 5k$ (c) 5$j + k$ (d) i$ + j$ + k$ (c) 20i$ + 15j$ (d) 15i$ + 20j$
28 The resultant of A and B makes an angle α with 37 Let A = $i A cos θ + $jA sin θ be any vector. Another
A and β with B, then vector B which is perpendicular to A can be
(a) α < β (b) α > β, if A < B expressed as
(c) α < β, if A = B (d) α < β, if A < B (a) i$B cos θ − j$ B sin θ (b) i$ B sin θ − j$ B cos θ
29 The vector that must be added to the vectors
$i − 2$j + 3 k$ and 6$i + 3 $j − 7k$ , so that the resultant (c) i$B cos θ + $j B sin θ (d) i$ B sin θ + j$ B cos θ
vector is a unit vector along theY-axis is 38 If two vectors 2$i + 3 $j + k
$ and − 4$i − 6$j − λk$ are
(a) 4i$ + 2j$ + 5k$ (b) −7i$ + 4k$ parallel to each other, then value of λ is
(c) 3i$ + 4j$ + 5k$ (d) null vector (a) 0 (b) 2 (c) 3 (d) 4
66 OBJECTIVE Physics Vol. 1

39 If P + Q = R and | P | = | Q | = 3 and | R | = 3, then 48 If vectors A and B have an angle θ between them,


the angle between P and Q is $ −B
then value of | A $ | will be
(a) π /4 (b) π /6 θ θ
(a) 2 cos (b) 2 tan
(c) π / 3 (d) π /2 2 2
40 The angles which the vector A = 3 $i + 6$j + 2k
$ θ
(c) 2 sin (d) None of these
makes with the coordinate axes are 2
 3  6  2 49 Given A = 3 $i + 4$j and B = 6$i + 8$j , then which of
(a) cos−1   , cos−1   and cos−1  
 7  7  7 the following option is correct?
−1  4  −1  5  −1  3 
| A| 1
(b) cos   , cos   and cos   (a) A × B = 50 (b) =
 7  7  7 |B | 2
 3  4  1 (c) | A | = 15 (d) A ⋅ | B | = 48
(c) cos−1   , cos−1   and cos−1  
 7  7  7 50 The resultant of two vectors 3P and 2P is R. If the
(d) None of the above first vector is doubled, then the resultant is also
41 Resultant of two vectors of equal magnitude A is doubled. The angle between the two vectors is
(a) 60° (b) 120°
(a) 3A at 0° (b) 2A at 90°
(c) 90° (d) 180°
(c) 2A at 120° (d) A at 180°
51 The sum of two vectors A and B is at right angles to
42 The component of a vector r along X-axis will have
their difference. Then, the correct relation is
maximum value, if [NCERT Exemplar]
(a) A = B (b) A = 2B
(a) r is along positive Y-axis (c) B = 2A (d) None of these
(b) r is along positive X-axis
(c) r makes an angle of 45° with the X-axis 52 If the sum of two unit vectors is a unit vector, then
(d) r is along negative Y-axis magnitude of difference in two unit vectors is
43 If A ⋅ B = 0 and A × B = 1, then A and B are (a) 2 (b) 3 (c) 1/ 2 (d) 5
(a) perpendicular unit vectors 53 What is the angle between P and the cross product
(b) parallel unit vector of (P + Q ) and (P − Q )?
(c) parallel (a) 90° (b) tan−1 (P /Q )
(d) anti-parallel −1
(c) tan (Q /P ) (d) 0°
44 If a + b + c = 0, then a × b is equal to
(a) b × c (b) c × b
54 A vector having magnitude 30 unit makes equal
(c) a × c (d) None of these angles with each of X, Y and Z-axes. The components
of vector along each of X, Y and Z-axes are
45 If A = 4$i − 3 $j and B = 6$i + 8$j , then magnitude and
(a) 10 3 unit (b) 20 3 unit
direction of A + B with X-axis will be (c) 15 3 unit (d) 10 unit
(a) 5, tan−1 (3 / 4) (b) 5 5, tan−1 (1/2)
−1 55 The resultant of two forces, one double the other in
(c) 10, tan (5) (d) 25, tan−1 (3 / 4)
magnitude, is perpendicular to the smaller of the two
46 The direction cosines of vector (A − B ), if forces. The angle between the two forces is
A = 2$i + 3 $j + k$ , B = 2$i + 2$j + 3 k$ are (a) 120° (b) 135° (c) 90° (d) 150°

1 −2 2 1 56 What is the angle between P and the resultant of


(a) 0, , (b) 0, , (P + Q ) and (P − Q )?
5 5 5 5
(a) Zero (b) tan−1 (P /Q )
1
(c) 0, 0, (d) None of these (c) tan−1 (Q /P ) (d) tan−1 (P − Q )/(P + Q )
5
57 There are N coplanar vectors each of magnitudeV.
47 Two vectors A and B are such that A + B = C and
Each vector is inclined to the preceding vector at
A 2 + B 2 = C 2 . If θ is the angle between A and B, 2π
angle . What is the magnitude of their resultant?
then the value of θ is N
2π V
(a) π (b) (a) (b) V
3 N
π N
(c) 0 (d) (c) Zero (d)
2 V
Vectors 67

58 At what angle must the two forces (x + y ) and 65 Six vectors have magnitude and direction as indicated
in the figure. Which of the following expression
(x − y ) act, so that the resultant may be x 2 + y 2 ?
is true?
 x2 + y 2 
(a) cos−1 − 2  a
 2 (x − y )
2 b c

 −2 (x 2 − y 2 ) d f
(b) cos−1 − 
 x2 + y 2  e
 (x + y )
2 2
(a) b + e = f (b) b + c = f
(c) cos−1 − 
 (x 2 − y 2 ) (c) d + c = f (d) d + e = f
 (x 2 − y 2 ) 66 The sum of the magnitudes of two forces acting at a
(d) cos−1 − 
 (x 2 + y 2 ) point is 18 and the magnitude of their resultant is
12. If the resultant is at 90° with the force of smaller
59 If a and b are two vectors, then the value of magnitude, what are the magnitudes of forces?
(a + b ) × (a − b ) is (a) 12, 6 (b) 14, 4 (c) 5, 13 (d) 10, 8
(a) 2 (b × a ) (b) − 2 (b × a )
67 The resultant of two vectors P and Q is R. If Q is
(c) b × a (d) b × a
doubled, the new resultant is perpendicular to P.
60 Given that A + B = C and that C is perpendicular to
Then, R equal to
A. Further, if | A | = | C |, then what is the angle (a) P (b) (P + Q ) (c) Q (d) (P − Q )
between A and B?
π π 68 In the figure shown, ABCDEF is a regular hexagon.
(a) (b) What is the value of AB + AC + AD + AE + AF?
4 2
3π E D
(c) (d) π
4
61 The resultant of vectors A and B is R1. On reversing
F C
the direction of vector B, the resultant becomes R 2 . O
What is the value of R 12 + R 22 ?
(a) A2 + B 2 (b) A2 − B 2 A B
(c) 2(A + B )
2 2
(d) 2 (A − B )
2 2
(a) AO (b) 2AO
62 Unit vector perpendicular to vector (c) 4AO (d) 6AO
A = − 3 $i − 2$j − 3 k$ and B = 2$i + 4$j + 6k$ both is 69 Figure shows three vectors p, q and r, where C is the
3$j − 2k$ 3k$ − 2$j mid-point of AB. Then, which of the following
(a) (b) relation is correct?
13 13
A
− $j + 2k$ i$ + 3j$ − k$
(c) (d)
13 13
p C
63 The velocity of a particle is v = 6$i + 2$j − 2k
$ . The r
component of the velocity parallel to vector
a = $i + $j + k$ in vector form is O q B

(a) 6i$ + 2j$ + 2k$ (b) 2i$ + 2j$ + 2k$ (a) p + q = 2r (b) p + q = r (c) p − q = 2r (d) p − q = r
(c) i$ + j$ + k$ (d) 6i$ + 2j$ − 2k$ 70 A vector a is turned without a change in its length
64 If a$i + b$j is a unit vector and it is perpendicular to through a small angles dθ. The value of | ∆a| and ∆a
$i + $j , then value of a and b is are, respectively.
(a) 0, a dθ (b) a ⋅ d θ, 0
(a) 1, 0 (b) − 2, 0 (c) 0, 0 (d) None of these
(c) 0.5, − 0.5 (d) None of these
OBJECTIVE Physics Vol. 1

(B) Medical entrance special format questions


Assertion and reason
Directions (Q. Nos. 1-5) These questions consists of 2 Which one of the following statement is true?
two statements each printed as Assertion and Reason. [NCERT Exemplar]
While answering these question you are required to (a) A scalar quantity is the one that is conserved in a process.
choose any one of the following four responses (b) A scalar quantity is the one that can never take negative
(a) If both Assertion and Reason are correct and Reason values.
is the correct explanation of Assertion. (c) A scalar quantity is the one that does not vary from one
(b) If both Assertion and Reason are correct but Reason is point to another in space.
not the correct explanation of Assertion. (d) A scalar quantity has the same value for observers with
(c) If Assertion is true but Reason is false. different orientation of the axes.
(d) If Assertion is false but Reason is true. 3 Figure shows the orientation of two vectors u and v in
1 Assertion Angle between $i + $j and $i is 45°. the XY-plane.
If u = a$i + b$j and v = p$i + q$j , then which of the
Reason $i + $j is equally inclined to both $i and $j
following statement is correct? [NCERT Exemplar]
and the angle between $i and $j is 90°.
Y
2 Assertion (A + B ) ⋅ (A − B ) is always positive.
Reason This is positive if | A | > | B |.
v
3 Assertion A × B is perpendicular to both A + B u

as well as A − B.
Reason A + B as well as A − B lie in the plane O X
containing A and B while A × B lies
perpendicular to the plane containing A and B. (a) a and p are positive while b and q are negative.
(b) a, p and b are positive while q is negative.
4 Assertion (A × B ) ⋅ (B × A ) is − A 2B 2 sin 2 θ. (c) a, q and b are positive while p is negative.
Here θ is the angle between A and B. (d) a, b, p and q are all positive.
Reason (A × B ) and (B × A ) are two 4 Two unit vectors when added give a unit vector. Then,
anti-parallel vectors provided A and B are choose the correct statement.
neither parallel nor anti-parallel. (a) Magnitude of their difference is 3.
5 Assertion If | A | = | B |, then (A + B ), (A − B ) (b) Magnitude of their difference is 1.
and (A × B ) are three mutually perpendicular (c) Angle between the vectors is 90°.
vectors. (d) Angle between the sum and the difference of the two
vectors is 180°.
Reason Dot product of a null vector with any
other vector is always zero. 5 I. Displacing a vector parallel to itself leaves the vector
unchanged.
Statement based questions II. Three equal vectors cannot add upto zero.
Which of the following statement(s) is/are correct?
1 Which of the following statement is true? (a) Only I (b) Only II
(a) When the coordinate axes are translated, the (c) Both I and II (d) Neither I nor II
component of a vector in a plane changes.
(b) When the coordinate axes are rotated through 6 Unit vector
some angle, components of the vector change but I. has dimensions and a unit.
the vector’s magnitude remains constant. II. when multiplied by a scalar quantity, it results a scalar.
(c) Sum of a and b is R. If the magnitude of a alone is Which of the following statement(s) is/are incorrect?
increased angle between b and R decreases.
(a) Only I (b) Only II
(d) The cross product of 3i$ and 4j$ is 12. (c) Both I and II (d) Neither I nor II
Vectors 69

Match the columns Codes


A B C D
1 Vector A is pointing eastwards and vector B (a) p r q s
northwards. If | A | = | B |, then match the following two (b) r s p q
columns and mark the correct option from the codes (c) r q s p
given below. (d) q r p s
Column I Column II 3 Two vectors A and B have equal magnitude x .
(A) (A + B) (p) North-east Angle between them is 60°. Then, match the
(B) (A − B) (q) Vertically upwards following two columns and mark the correct
(C) (A × B) (r) Vertically downwards option from the codes given below.
(D) (A × B) × (A × B) (s) None
Column I Column II
Codes
A B C D A B C D (A) |A + B| (p) 3 2
x
(a) p r s q (b) p s q s 2
(c) q r p s (d) q p s r (B) |A − B| (q) x
2 A vector has a magnitude x. If it is rotated by an angle (C) A ⋅ B (r) 3x
θ, then magnitude of change in vector is nx. Match the
(D) |A × B| (s) None
following two columns and mark the correct option
from the codes given below.
Codes
Column I Column II A B C D
(A) θ = 60° (p) n= 3 (a) p s q r
(B) θ = 90° (q) n =1 (b) p r q s
(C) θ = 120° (r) n= 2 (c) r q s p
(d) r s p q
(D) θ = 180° (s) n=2

(C) Medical entrances’ gallery


Collection of questions asked in NEET and various medical entrance exams

1 If the magnitude of sum of two vectors is equal to (a) 1 : 1 (b) 2 : 1


the magnitude of difference of the two vectors, the (c) − 1 : 1 (d) 3 : 1
angle between these vectors is [NEET 2016] 5 Two equal vectors have a resultant equal to either of
(a) 90° (b) 45° (c) 180° (d) 0° the two. The angle between them is [UK PMT 2014]
2 If a vector 2$i + 3 $j + 8k
$ is perpendicular to the (a) 90° (b) 60°
(c) 120° (d) 0°
vector 4$j − 4$i + αk$ , then the value of α is
[Manipal 2015] 6 Which of the following not a vector quantity?
[KCET 2014]
(a) 1/2 (b) −1 (c) –1/2 (d) 1
(a) Weight
3 The angle θ between the vector p = $i + $j + k
$ and
(b) Nuclear spin
unit vector along X-axis is [MHT CET 2014] (c) Momentum
−1  1  −1  1  (d) Potential energy
(a) cos   (b) cos  
 3  2 7 The scalar product of two vectors A = 2$i + 2$j − k
$
 3 and B = − $j + k$ , is given by [J&K CET 2013]
 1
(c) cos− 1   (d) cos −1
  (a) A ⋅ B = 3 (b) A ⋅ B = 4
 2   2
(c) A ⋅ B = − 4 (d) A ⋅ B = − 3
4 Consider three vectors A = $i + $j − 2k
$ , B = $i − $j + k$
8 If A ⋅ B = A × B, then angle between A and B is
and C = 2$i − 3 $j + 4k$ . A vector X of the form [AMFC 2012]
αA + βB (α and β are numbers) is perpendicular to C. (a) 45° (b) 30°
The ratio of α and β is [EAMCET 2014] (c) 60° (d) 90°
70 OBJECTIVE Physics Vol. 1

9 If a vector A having a magnitude of 8 is added to a 10 The value of λ for which two vectors
vector B which lies along X-axis, then the resultant of a = 5$i + λ$j + k$ and b = $i − 2$j + k$ are perpendicular
two vectors lies alongY-axis and has magnitude twice to each other is
that of B. The magnitude of B is [JCECE 2012] [WB JEE 2011]
6 12 16 8 (a) 2 (b) − 2
(a) (b) (c) (d)
5 5 5 5 (c) 3 (d) − 3

ANSWERS
l CHECK POINT 2.1
1. (d) 2. (a) 3. (c) 4. (c) 5. (a) 6. (c) 7. (c) 8. (b)

l CHECK POINT 2.2


1. (a) 2. (b) 3. (c) 4. (b) 5. (b) 6. (b) 7. (a) 8. (a) 9. (b) 10. (b)
11. (d) 12. (d)

l CHECK POINT 2.3


1. (a) 2. (b) 3. (d) 4. (d) 5. (a) 6. (c) 7. (a) 8. (d)

(A) Taking it together


1. (c) 2. (d) 3. (c) 4. (a) 5. (b) 6. (c) 7. (a) 8. (b) 9. (a) 10. (c)
11. (c) 12. (d) 13. (b) 14. (c) 15. (c) 16. (a) 17. (c) 18. (d) 19. (b) 20. (d)
21. (d) 22. (c) 23. (b) 24. (d) 25. (b) 26. (a) 27. (c) 28. (b) 29. (b) 30. (b)
31. (c) 32. (a) 33. (b) 34. (a) 35. (b) 36. (d) 37. (b) 38. (b) 39. (c) 40. (a)
41. (b) 42. (b) 43. (a) 44. (a) 45. (b) 46. (a) 47. (d) 48. (c) 49. (b) 50. (b)
51. (a) 52. (b) 53. (a) 54. (a) 55. (a) 56. (a) 57. (c) 58. (a) 59. (a) 60. (c)
61. (c) 62. (a) 63. (b) 64. (d) 65. (d) 66. (c) 67. (c) 68. (d) 69. (a) 70. (b)

(B) Medical entrance special format questions


l Assertion and reason
1. (b) 2. (d) 3. (a) 4. (b) 5. (b)

l Statement based questions


1. (b) 2. (d) 3. (b) 4. (a) 5. (a) 6. (c)

l Match the columns


1. (b) 2. (d) 3. (c)

(C) Medical entrances’ gallery


1. (a) 2. (c) 3. (a) 4. (a) 5. (c) 6. (d) 7. (d) 8. (a) 9. (d) 10. (c)
Hints & Explanations
l CHECK POINT 2.1 8 (a) |A + C| = A2 + A2 + 2A2 cos 120 °
1 (d) Electric flux is a scalar quantity. =A
2 (a) Surface area has magnitude only. Therefore, it is a scalar Now, |A + B + C| = A + A = 2A
quantity.
9 (b) If P + Q = P − Q, then Q = 0 (a null vector).
3 (c) Pressure, temperature and work are scalar quantities while
momentum is a vector quantity. 10 (b) Resultant of two vectors A and B,
R = {| A | − | B | }
4 (c) When a vector is multiplied by zero, it results into
zero vector, i.e. a vector having zero magnitude. It is written then θ = 180 °
as 0. 11 (d) | A − B | = A2 + B 2 − 2AB cos θ
5 (a) ( $i − 3 $j + 2 k$ ) + (3 $i + 6$j − 7 k$ ) + r = $j
1
= 4 + 16 − 2 × 2 × 4 ×
⇒ r = − 4$i − 2 $j + 5 k$ 2
6 (c) Let the given vector be A, where = 12 = 2 3

A = $i + $j 12 (d) Let the vector be A as shown below.


Y
∴ | A | = (1) + (1) = 2
2 2

7 (c) Let A = $i + $j

| A | = (1)2 + (1)2 = 2 A
Ay
∴ Unit vector along A will be A
$ is given by
θ = 60°
$ = A = i+ j
$ $
A X
|A | 2 O Ax
Ay
8 (b) When a vector is multiplied by a negative scalar number tan θ = or Ay = Ax tanθ
(i.e. − 2), then its magnitude gets doubled and direction gets Ax
reversed.
Ay = 50 tan 60 ° = 50 3 = 86.6 N
l CHECK POINT 2.2 −
~ 87 N
1 (a) Resultant of two vectors will be maximum when they are
parallel, i.e. angle between them is zero.
l CHECK POINT 2.3
1
2 (b) Minimum three vectors of unequal magnitude are required 1 (a) AB sin θ = AB cos θ
3
to make vector sum equal to zero.
1 π
3 (c) Maximum resultant, Rmax = | A + B | = 18 ∴ tan θ = or θ = 30 ° =
3 6
Minimum resultant, Rmin = | A − B | = 2
2 (b) Let the two vectors be A and B.
4 (b) If P + Q + R = 0, then | P + Q | = | R |. We know, A ⋅ B = AB cos θ
5 (b) Given, P = $i − $j + k$ = (3)(5) cos 60 ° = 7.5
and P + Q = $i ⇒ Q = $i − $i + $j − k$ = $j − k$ 3 (d) As the multiple of $j in the given vector is zero, therefore
this vector lies in XZ-plane and projection of this vector on
6 (b) Given, A = 2$i + $j, B = 3$j − k$ and C = 6$i − 2 k$ Y-axis is zero.
∴ A − 2B + 3C = (2$i + $j ) − 2(3$j − k$ ) + 3(6$i − 2 k$ ) 4 (d) Given, A ⋅ B = 0
= 2$i + $j − 6$j + 2 k$ + 18$i − 6 k$ ∴ A⊥B
= 20 $i − 5$j − 4 k$ A⋅ C = 0
∴ A⊥C
7 (a) P 10 = 4P 2 + 2P 2 + 4 2P 2 cos θ
As, B × C is ⊥ to both B and C.
∴ θ = 45°
So, B × C is parallel to A.
72 OBJECTIVE Physics Vol. 1

5 (a) (a ⋅ b )2 = a 2b 2 cos 2 θ = a 2b 2 (Given) 10 (c) P = 6$i + 4 2 $j + 4 2 k$


∴ θ = 0° Let P makes angle γ with Z-axis.
⇒ a is parallel to b. P 
∴ γ = cos −1 z 
6 (c) We have, A ⋅ B = AB cos θ P

Here, A ⋅ B = − AB, i.e. cos θ = − 1 Here, Pz = 4 2


∴ θ = 180 ° and P = (6)2 + (4 2 )2 + (4 2 )2 = 10
So, A and B acts in the opposite direction.
 4 2  2 2
7 (a) We have, A × B = AB sin θ ∴ γ = cos −1  = cos −1 
 10   5 
8 = (2) (5) sinθ
∴ sin θ = 4/ 5 or cos θ = 3/ 5 11 (c) Unit vector perpendicular to A and B,
A×B A×B
∴ A ⋅ B = AB cos θ n$ = =
|A × B| AB sin θ
= (5) (2) (3/ 5) = 6
8 (d) AB sin θ = 3 AB cos θ or tanθ = 3 12 (d) Their cross product should be zero.
(a + b ) × (a − b ) = 0
∴ θ = 60 °
∴ 2 (a × b) = 0
Now, |A + B | = A2 + B 2 + 2AB cos 60 °
So, a is parallel to b.
= A2 + B 2 + AB $ perpendicular to XY-plane
14 (c) F lies in XY-plane. Hence, 7 k,
is also perpendicular to F.
(A) Taking it together 15 (c) Resultant of two vectors lies between |A + B | and |A − B |.
1 (c) Potential energy is a scalar quantity. i.e. |A − B| < C < |A + B|
2 (d) Displacement has both direction as well as magnitude. So, 16 (a) Only the magnitude will remain same.
it is a vector quantity. 17 (c) (a − b ) is nothing but addition of a and −b.
3 (c) When a vector is added to another vector of similar nature | a| = | a − b + b| ≤ | a − b| + |b|
having equal magnitude and opposite sign, it forms a null
vector. or | a| − |b| ≤ | a − b|
Hence, option (c) is correct.
4 (a) The component of a vector is always less than its
magnitude along any other direction. 19 (b) The angle between vectors (A × B ) and (B × A) is 180° or
5 (b) Unit vector has a magnitude equal to 1. π.
20 (d) Resultant of two given vectors will be
Here, cos 2 θ + sin2 θ = 1
(8)2 + (8)2 = 8 2
∴ Option (b) is the correct answer. $i + $j
6 (c) Since, vector P is perpendicular to the vector Q. ∴ Unit vector parallel to the given vectors will be,
2
∴ P ⋅ Q = 0 ⇒ (3$i − 2$j + ak$ ) ⋅ (2$i + $j − k$ ) = 0
21 (d) Area = |A × B|
⇒ 6− 2−a = 0 ⇒ a = 4
Here, A × B = (2$i + 3$j ) × ($i + 4$j )
7 (a) A − B = ( i + $j + k$ ) − (− $i − $j − k)
$ $
= 8 k$ − 3 k$ = 5 k$
= 2$i + 2$j + 2 k$ = 2A
∴ Area = |A × B| = 5 units
i.e. A − B and A are parallel.
22 (c) In XY-plane, vector is 3$i + $j
Hence, θ = 0 °
8 (b) Let A = − 2$i + 3$j + k$ ∴ Length in XY-plane = 9 + 1 = 10

B = $i + 2$j − 4 k$ 23 (b) P × Q is perpendicular to the plane formed by P and Q.

∴ A ⋅ B = (−2$i + 3$j + k$ ) ⋅ ($i + 2$j − 4 k$ ) P + Q lies in this plane. Hence, P + Q is perpendicular


to P × Q.
=− 2+ 6− 4 = 0
24 (d) Diagonal vector, A = b $i + b $j + b k$
Since, dot product of two vectors is zero, so vectors will be
perpendicular (i.e. θ = 90 °) to each other. or A = b2 + b2 + b2 = 3b
A⋅ B 2+ 3 5 $i + $j + k$
9 (a) Component of A along B = A cos θ = = = ∴ A=
A
=
B 1+ 1 2 |A| 3
Vectors 73

25 (b) Given, A = $i + $j and B = $i − $j 38 (b) The coefficients of $i, $j and k$ should be in constant ratio.
We know that, A ⋅ B = A B cos θ 2 3 1
or = = or λ = 2
⇒ ($i + $j ) ⋅ ($i − $j ) = ( 1 + 1) ( 1 + 1) cos θ −4 −6 −λ

where, θ is the angle between A and B. 39 (c) Resultant, R = P 2 + Q 2 + 2PQ cos θ

1− 0 + 0 − 1 0 3 = ( 3 )2 + ( 3 )2 + 2( 3 ) ( 3 ) cos θ
⇒ cos θ = = = 0 ⇒ θ = 90 °
2 2 2
Squaring both sides in the above equation, we get
26 (a) If three vectors form a triangle, then the resultant of these 9 = 3 + 3 + 6 cos θ
vectors will be zero when the sum of two smaller sides of 1 π
triangle is greater than its third side. This is only possible in ⇒ cos θ = ⇒ θ = 60 ° or
2 3
option (a).
40 (a) Magnitude of vector, |A| = (3)2 + (6)2 + (2)2 = 7
27 (c) Cross product of two perpendicular vector should be zero.
 3  6  2
$i $j k$ α = cos −1  , β = cos −1  , γ = cos −1 
 7  7  7
2 −1 5 = − (0 − 5)$j + (1) k$ = 5$j + k$
θ
1 0 0 41 (b) Resultant of two vectors AR = 2A cos = 2A cos 45° = 2A ,
2
The vector perpendicular to both vectors is 5$j + k$ . at θ = 90 °.
28 (b) Consider the figure, 42 (b) Let r makes an angle θ with positive X-axis, so component
of r along X-axis,
B
rX = r cos θ
R (rX ) maximum = r (cos θ ) maximum
β = r cos 0° (Q cos θ is maximum, if θ = 0°)
α A = r
Resultant is inclined towards a vector having greater As θ = 0 °
magnitude. i.e. r is along positive X-axis.
29 (b) $j = ($i − 2$j + 3k$ ) + (6$i + 3$j − 7k$ ) + C 43 (a) Given, A ⋅ B = 0
Hence, C = − 7$i + 4 k$ ⇒ A ⊥B ⇒ A × B =1
AB sin θ = 1 ⇒ AB sin 90 ° = 1
30 (b) For a unit vector, (0.5)2 + (0.8)2 + c 2 = 1
or AB = 1 ⇒ A = 1 and B = 1
On solving, we get c = 0.11 So, A and B are perpendicular unit vectors.
A⋅ B 4 + 12 16 8 44 (a) We have, a + b + c = 0
31 (c) Component of A along B = = = =
B (2)2 + (4)2 2 5 5
∴ a + c = −b
32 (a) F ⋅ F cos θ = F ⋅ F sin θ or tanθ = 1or θ = 45° or (a + c ) × b = − b × b = 0
| F1 + F2| = F 2 + F 2 + 2F ⋅ F cos 45° = ( 2 + 2 )F ⇒ (a × b ) + (c × b ) = 0
Hence, a ×b =b × c
33 (b) B × A is perpendicular to A. Hence, A ⋅ (B × A) will be
45 (b) A + B = 10 i + 5$j
$
zero.
$ is a unit vector in a given direction. It should be a ∴ |A + B| = 100 + 25 = 5 5
34 (a) Since, A
constant unit vector. Angle of A + B with X-axis,
 5  1
θ = tan−1  = tan−1 
$
or $ ⋅ dA = 0
A
dt  10   2

35 (b) Rnet = R + R 2 + R 2 = R + 2R = R ( 2 + 1) 46 (a) We have, A − B = $j − 2 k$ = C

36 (d) |A| = 32 + 42 = 5 C = 1+ 4 = 5
0 1 −2
$ = ( 72 + 242 ) × 3 i + 4 j = 15$i + 20 $j
$ $ cos α = = 0, cos β = and cos γ =
Desired vector, r = | B | A 5 5 5
5
37 (b) Dot product of two perpendicular vectors should be zero. 47 (d) Here, A + B = C and A2 + B 2 = C2
∴ B = $i B sin θ − $j B cos θ ∴ (A + B )2 = (C)2
[Q ($i A cos θ + $jA sin θ ) ⋅( $i B sin θ − $jB cos θ ) = 0] |A |2 + | B |2 + 2|A | | B | cos θ = | C |2
74 OBJECTIVE Physics Vol. 1

| C |2 + 2 |A | | B | cos θ = | C |2 57 (c) Since, each of N coplanar vector is inclined at



to the
N
∴ 2 |A|| B| cos θ = 0
preceding vector. Hence, they will form a closed polygon.
⇒ cos θ = 0 Therefore, their resultant must be equals to zero.
π
⇒ θ= 58 (a) Let the two forces be A and B.
2
Resultant, R 2 = A2 + B 2 + 2AB cos θ …(i)
$ − B$ | = 1 + 1 − 2 × 1 × 1 × cosθ = 2 sin θ
48 (c) |A
2 Substituting, A = (x + y ), B = (x − y )
$ and B$ is also θ, but their magnitudes are 1.
Angle between A and R = x2 + y2
49 (b) |A | = 9 + 16 = 5 and | B | = 36 + 64 = 10 On substituting these values in Eq. (i), we get
|A| 1
⇒ =  (x 2 + y 2 ) 
| B| 2 θ = cos −1 − 2 
 2 (x − y ) 
2

50 (b) Resultant, R = 9 P 2 + 4 P 2 + 12 P 2 cos θ ...(i)


59 (a) (a + b ) × (a − b ) = a × a − a × b + b × a − b × b
2R = 36 P 2 + 4 P 2 + 24 P 2 cos θ ...(ii) = 2(b × a)
Equating Eqs. (i) and (ii), we get 60 (c) From the figure, B cos θ = C …(i)
1
cos θ = − and B sinθ = A …(ii)
2
∴ θ = 120 ° B cos θ
51 (a) R ⋅ S = (A + B ) ⋅ (A − B )
B C
= A ⋅A − A ⋅ B + B ⋅A − B ⋅ B θ
or 0 = A2 − B 2 (Q A ⋅ B = B ⋅ A) B sin θ A
∴ A = B or A = B
2 2
Given, |A| = | C|
52 (b) Sum of two unit vectors is a unit vector, means angle ∴ | B cos θ| = | B sin θ| or θ = 45°
between those two unit vectors is 120°. 3π
Hence, angle between A and B is 135° or .
∴Difference, | S| = 1 + 1− 2 × 1× 1× cos 120 ° = 3 4
53 (a) Cross product of (P + Q) and (P − Q) is perpendicular to the 61. (c) We have, R12 = A2 + B 2 + 2AB cos θ
plane formed by (P + Q) and (P − Q) or P (or Q). and R22 = A2 + B 2 − 2AB cos θ
54 (a) We have, Ax = Ay = Az ∴ R12 + R22 = 2(A2 + B 2 )
Resultant, A = Ax2 + Ay2 + Az2 A×B
62 (a) Unit vector is given by n$ =
|A × B|
= 3 Ax
$i $j k$
A 30
∴ Ax = = = 10 3 unit A × B = −3 −2 −3
3 3
2 4 6
2A sin θ
55 (a) We have, tan 90 ° = =∞
A + 2A cos θ = $i(−12 + 12) − $j (−18 + 6) + k$ (−12 + 4) = 12$j − 8 k$
R
2A |A × B| = (12)2 + (− 8)2 = 208 = 4 13
12$j − 8 k$ 3$j − 2 k$
∴ n$ = =
4 13 13
θ 90°
A 63 (b) Magnitude of component of v along a
∴ A + 2A cos θ = 0 v⋅a 6+ 2− 2
= = =2 3
1 a 3
or cos θ = − or θ = 120 ° $i + $j + k$
2 Now, a$ =
56 (a) Resultant of (P + Q) and (P − Q) is P + Q + P − Q or 2P 3
which is parallel to P. ∴ Component of velocity in vector form
So, angle between P and 2P will be zero. = 2 3 a$ = (2$i + 2$j + 2 k$ )
Vectors 75

64 (d) (a$i + b$j ) ⋅ ($i + $j ) = 0 or a + b = 0 ...(i) = AB + (AB + BC) + (AB + BC + CD) + (AB + BC + CD + DE) + AF
= 4AB + 3BC + 2CD + DE + AF
Further, a 2 + b 2 = 1 or a 2 + b 2 = 1 ...(ii) (Q AB = −DE, BC = − EF, AF = CD)
Solving Eqs. (i) and (ii), we get = 3(AB + BC + CD) = 3AD = 6 AO

a=±
1
and b =±
1 69 (a) From the figure, OB + BC = r
2 2
or q + BC = r ...(i)
65 (d) On arranging vectors, OA + AC = r
e or p + AC = r ...(ii)
Adding these two equations, we get,
d p + q = 2r, as AC and BC are equal and opposite vectors.
f
 dθ 
70 (b) | ∆a | = a 2 + a 2 − 2 ⋅ a ⋅ a ⋅ cos (dθ ) = 2a sin  
 2
dθ dθ
For small angles sin ≈
f=d+e 2 2
66 (c) We have, P + Q = 18 dθ
...(i) ∴ | ∆a| = 2a ×
= a ⋅ dθ
2
and R = 12
∆a means change in magnitude of vector,
R
Q i.e. a − a = 0 ⇒ ∆a = 0

(B) Medical entrance special format


θ
P question
Q sinθ = P and Q cos θ = R
l Assertion and reason
Squaring and adding, we get 1 (b) $i and $j are mutually perpendicular to each other and angle
P 2 + R2 = Q 2 between ($i + $j ) and $i is 45°.

or Q 2 − P 2 = R 2 = 144 ...(ii) 2 (d) Value of (A + B ) ⋅ (A − B ) can be positive or negative.


Solving Eqs. (i) and (ii) , we get 3 (a) (A × B ) is perpendicular to (A + B ) and (A − B ) because
P = 5 and Q = 13. (A × B ) lies perpendicular to the plane containing A and B.
67 (c) We have, 2Q sinθ = P , as R ⊥ P 4 (b) Angle between A × B and B × A is 180°.
R ∴ (A × B ) ⋅ (B × A) = (AB sin θ ) (AB sin θ ) (cos 180 ° )
= − A2B 2 sin2 θ

2Q 5 (b) (A + B ) ⋅ (A − B ) = A2 − B 2
θ This is the dot product that possesses zero value, if A = B.
P Therefore, they are perpendicular. Further, (A × B ) is
perpendicular to both (A + B ) and (A − B ).
Now, R = P 2 + Q 2 + 2PQ cos(90 ° + θ )

= P 2 + Q 2 − 2PQ sinθ
l Statement based questions
1 (b) When the coordinate axes are rotated through some angle,
= (2 Q sin θ )2 + Q 2 − 2 (2 Q sin θ )Q sin θ only the components of the vector changes, whereas
magnitude remains constant.
= 4Q 2 sin2 θ + Q 2 − 4Q 2 sin2 θ = Q
2 (d) A scalar quantity is independent of direction, hence has
68 (d) AB + AC + AD + AE + AF the same value for observers with different orientation of the
E D axes.
3 (b) Clearly, from the diagram, u = a$i + b $j
As u is in the first quadrant and located upward, hence its
F C
O both components a and b will be positive.
For v = p $i + q $j, as it is in positive x-direction and located
downward, hence its x-component p will be positive and
A B y-component q will be negative.
76 OBJECTIVE Physics Vol. 1

4 (a) Sum, R = A2 + B 2 + 2AB cos θ ∴ Change in |A|, ∆A = x 2 + x 2 = 2x ⇒ n = 2


B R
(C) When θ = 120 °
A
∆A

θ = 120°
60° A
A
30°
∴ Change in |A |, ∆A = x + x × x = 3x
2 2 2

n= 3
–B S (D) When θ = 180 °
1 θ = 180°
We have, 1 = 1 + 1 + 2 cos θ or cos θ = −
2
∴ θ = 120 ° A′ A
Difference, S = A2 + B 2 − 2AB cos θ ∴ Change in |A | , ∆A = x + x + 2x 2 = 2x
2 2

 1 ⇒ n=2
= 1+ 1− 2 × 1× 1×  −  = 3
 2 Hence, A → q, B → r, C → p, D → s.
5 (a) If a vector is displaced parallel to itself, it does not change. 3 (c) (A) |A + B| = A2 + B 2 + 2AB cos θ
Thus, statement (I) is correct. Here, |A| =| B| = x and θ = 60 °
Three equal vectors can add upto zero when each of them is
∴ |A + B| = 3x
inclined at an angle of 120° with each other.
Thus, statement (II) is incorrect. (B) |A − B| = A2 + B 2 − 2AB cos θ = x

6 (c) A unit vector is a vector of unit magnitude and points in a x2


(C) A ⋅ B = AB cos θ =
particular direction. 2
If we multiply a unit vector, say n$ by a scalar λ, then the 3 2
(D) |A × B | = AB sin θ = x
result is a vector = λ$n. 2
So, both statements I and II are incorrect. Hence, A → r, B → q, C → s, D → p.
l Match the columns
(C) Medical entrances’ gallery
1 (b) Using right hand rule, direction of (A × B ) will be vertically
1 (a) Suppose two vectors are P and Q.
upwards and (A + B ) will point towards north-east.
The (A − B ) will point towards south-east and It is given that
|P + Q | = |P − Q |
(A × B ) × (A × B ) is a null vector having arbitrary direction.
Let angle between P and Q is φ.
Hence, A → p, B → s, C → q, D → s.
∴ P 2 + Q 2 + 2PQ cos φ = P 2 + Q 2 − 2PQ cos φ
2 (d) (A) When θ = 60 °
⇒ 4PQ cos φ = 0
⇒ cos φ = 0 (Q P, Q ≠ 0 )
A′
π
∆A ⇒ φ = = 90 °
2
θ = 60° 2 (c) Dot product of these two vectors should be equal to zero as
A they are perpendicular to each other.
⇒ A = A′ ∴ A ⋅B = − 8 + 12 + 8α = 0
∴ Change in |A|, ∆A = x 2 + x 2 − 2 ⋅ x ⋅ x ⋅ cos 60 ° = x −1
8α = − 4 ⇒ α =
2
⇒ n =1
(B) When θ = 90 ° 3 (a) The angle between, p = i + j + k
$ $ $

and X-axis, x = $i is given by


A′
p⋅ x ($i + $j + k$ ) ⋅ ($i) 1
∆A cos θ = = =
|p | | x | 1 +1 +1 ⋅ 1
2 2 2 2 3
θ = 90°  1
∴ θ = cos − 1  
A
 3
Vectors 77

4 (a) Vector X of the form αA + βB. ⇒ cos θ = − 1/ 2


⇒ θ = 120 °
X = αA + βB
= α ($i + $j − 2 k$ ) + β ($i − $j + k$ ) 6 (d) Potential energy is not a vector quantity.

X = i$ (α + β ) + $j (α − β ) + k$ (−2α + β ) 7 (d) Given, A = 2 $i + 2 $j − k$ and B = − $j + k$

A vector X is perpendicular to C, i.e. X ⋅ C = 0 Scalar product, A ⋅ B = (2 $i + 2 $j − k$ ) ⋅ (− $j + k$ )


[$i (α + β ) + $j (α − β ) + k$ (− 2α + β )] ⋅ [2$i − 3$j + 4k$ ] = 0 Using $i ⋅ $i = 1, $j ⋅ $j = 1, k$ ⋅ k$ = 1

⇒ 2 (α + β ) − 3 (α − β ) + 4 (− 2α + β ) = 0 We have, A ⋅ B = − 2 − 1= − 3
⇒ 2α + 2β − 3α + 3β − 8α + 4β = 0 8 (a) Given, A⋅ B = A × B
⇒ − 9α + 9β = 0 AB cos θ = A B sin θ
α 1 tan θ = 1
or α =β ⇒ =
β 1 θ = 45°
or α :β =1 :1 9 (d) According to given condition,
5 (c) Let two vectors are A and B, inclined at an angle θ. A + Bi$ = Rj$ (where, R is the resultant vector)
B Also, R = 2B
∴ A + Bi$ = 2 Bj$ or A = 2 Bj$ − Bi$
R ∴ A2 = 4B 2 + B 2 = 5B 2

θ Here, A= 8
O A ∴ 64 = 5B 2
Resultant of the two vectors A and B,
64 8
⇒ B= =
|R | = |A |2 + | B |2 + 2 |A | | B | cos θ …(i) 5 5
Let |A | = | B | = a 10 (c) For two vectors a and b to be perpendicular, a ⋅ b = 0
According to the question, |R | = a
Thus, (5$i + λ$j + k$ ) ⋅ ($i − 2$j + k$ ) = 0
From Eq. (i), we get
5($i ⋅ $i) − 2λ ($j ⋅ $j ) + 1 (k$ ⋅ k$ ) = 0
a = a 2 + a 2 + 2aa cos θ
⇒ 5 − 2λ + 1 = 0
⇒ a 2 = a 2 + a 2 + 2a 2 cos θ
⇒ 6 − 2λ = 0 ⇒ λ = 3
⇒ 2a 2 cos θ = − a 2
CHAPTER
03

Motion in
One Dimension
Motion is defined as the change in position of an object with time. When the
object moves along a single axis, the motion is known as one dimensional
motion or rectilinear motion and such a motion is along a straight line only,
which may be horizontal or vertical. In this chapter, we shall learn about motion
using the concepts of velocity and acceleration along with the basic physics of one
dimensional motion.

Frame of reference
A system of coordinate axes which defines the position of a particle or an event in
two or three dimensional space along with a clock constitutes a frame of
reference. The simplest frame of reference is the cartesian system of coordinates,
in which the position of the particle (P ) is specified by its three coordinates x, y
and z.
Y

P (x, y, z) Inside
1 Rest and motion
Some basic terms related to
X motion
O
(Origin) 2 Kinematic equations for
uniformly accelerated motion
Z
3 Motion under gravity
Fig. 3.1 Frame of reference for position of a particle P
Equations for motion
under gravity
Types of frame of reference 4 Non-uniformly accelerated
Frame of references are of two types motion
5 Graphical representation of
(i) Inertial frame of reference It is a frame of reference, where Newton’s law
motion
holds good. e.g. An object will remains at rest or in uniform motion unless 6 Relative velocity
acted by an external force. An inertial frame of reference is either at rest or
Different cases of relative
moving with a constant velocity. velocity
(ii) Non-inertial frame of reference An accelerating frame of reference is Examples of relative motion
called a non-inertial frame of reference. In this reference frame, Newton’s
law will not hold true.
Motion in One Dimension 79

Y
REST AND MOTION
If the position of an object does not change w.r.t. its M (x, y, z)
X
surrounding with the passage of time, it is said to be at O
rest. e.g. Book lying on the table, a person sitting on a Z
chair, etc. And if the position of an object is continuously
Fig. 3.4 Three dimensional motion
changing w.r.t. its surrounding, then it is said to be in the
state of motion. e.g. The walking man, crawling insects, In three dimensional motion, the object moves in a space.
water flowing down a dam, etc. e.g. Butterfly flying in garden, the motion of water
molecules, etc.
Rest and motion are relative terms
Rest and motion are always relative but never absolute. It
means an object, can be at rest for an observer but the
Some basic terms related to motion
same object can be in motion when observed by other 1. Point object
observer. e.g. A person sitting in his house is at rest w.r.t. An object is considered as point object, if the size of the
earth but is in motion w.r.t. moon. object is much smaller than the distance, it moves in a
reasonable duration of time. e.g. Earth can be considered as
Classification of motion a point object during its revolution around the sun because
On the basis of the number of coordinates required to it covers much larger distance than its own size.
specify the position of an object, the motion of the object
can be classified as 2. Distance and displacement
One dimensional motion Distance
The motion of an object is considered as one dimensional, The length of the path covered by the object in a given
if only one coordinate is needed to specify the position of time interval, is known as its distance. It is a scalar
the object. quantity. The unit of distance is metre in SI or MKS and
x centimetre in CGS. Its dimensional formula is [M0 LT 0 ].
−X O +X Example 3.1 A scooter is moving along a straight line AB
Fig. 3.2 One dimensional motion covers a distance of 360m in 24 s and returns back from B
to C and covers 240m in 18s. Find the total distance
In one dimensional motion, the object moves along a travelled by the scooter.
straight line. e.g. A boy running on a straight line, motion
Sol. From the above question, we draw the following figure
of freely falling body, etc.
240 m
Two dimensional motion
A B
The motion of an object is considered as two dimensional, C
360 m
if two coordinates are needed to specify the position of
the object. Hence, to find out the total path distance, it does not matter
Y how much time is taken by a scooter to reach at B and the
time taken to return at C.
M (x, y) ∴ Total distance = AB + BC = 360 + 240 = 600 m
Example 3.2 A wheel completes 2000 revolutions to cover
O X
the 9.5 km distance. Find the diameter of the wheel.
Fig. 3.3 Two dimensional motion
Sol. Given, number of revolutions, n = 2000
In two dimensional motion, the object moves in a plane. Distance, x = 9.5 km = 9.5 × 103 m = 9500 m
e.g. Motion of billiard ball. Q Distance covered in n revolutions is equal to the
circumference of the wheel,
Three dimensional motion x = n ⋅ 2πr ⇒ x = n ⋅ πD (Q Diameter = 2 × Radius)
The motion of an object is considered as three 9500 = 2000 × π × D
dimensional, if all the three coordinates are needed to 9500
specify the position of the object. ⇒ Diameter, D = = 1.5 m
2000 × 3.14
80 OBJECTIVE Physics Vol. 1

Displacement Example 3.3 A man starts from his home and walks 50m
towards north, then he turns towards east and walks 40m and
It is the shortest distance between the initial and final then reaches to his office after moving 20m towards south.
position of the moving object.
(i) What is the total distance covered by the man from his home
If x 1 and x 2 are the initial and final positions of an object, to office?
respectively. Then, displacement of the object is given by (ii) What is his displacement from his home to office?
∆x = x 2 − x 1 Sol. Let O represents the position of home, then according to the
question, the man moves from O to A (50 m) towards north,
● If x 2 > x1, then ∆x is positive. then from A to B (40 m) towards east and from B to C (20 m)
● If x1 > x 2 , then ∆x is negative. towards south as shown in figure.
● If x1 = x 2 , then ∆x is zero. A
40 m
B N
i.e. The displacement of an object in motion can be 20 m
positive, negative or zero while distance can never be 50 m C W E
negative or zero.
Displacement has both magnitude and direction. The unit θ S
O D
of displacement is metre in SI or MKS and centimetre in
(i) Total distance travelled by the man is
CGS. Its dimensional formula is [M0 LT 0 ]. OA + AB + BC = 50 + 40 + 20 = 110 m
To understand distance and displacement clearly, let us (ii) Displacement of the person is OC , which can be
consider the following example calculated by Pythagoras theorem, i.e.
Suppose a person (moving body) moves from A to B (4 km) OC 2 = OD 2 + CD 2 = (40)2 + (30)2
towards east and from B to C (3 km) due north as shown = 1600 + 900 = 2500
in figure, then the distance travelled by the person is ⇒ OC = 50 m
AB + BC = 4 km + 3 km = 7 km
Example 3.4 An object covers (1/4)th of the circular path.
C
N What will be the ratio of the distance and displacement of
the object?
m
5k 3 km W E Sol. Distance covered by object = 1/4th of the circular path
= AB through path (1)
A 2 πr πr
4 km B S = 1/4th of circumference of circular path = =
Fig. 3.5 4 2
B
But the displacement of the person is AC which can be 1
calculated by Pythagoras’ theorem, r
i.e. AC 2 = AB 2 + BC 2 A
O r
= (4) + (3 ) = 16 + 9 = 25
2 2

⇒ AC = 5 km
Displacement = Shortest distance between initial position (A)
Distance versus Displacement and final position (B)
(i) The magnitude of displacement may or may not be equal to AB = OA2 + OB 2 = r 2 + r 2 = r 2
the distance traversed by an object. Distance πr /2 π
(ii) The magnitude of the displacement for a course of motion ∴ = =
Displacement r 2 2 2
may be zero but the corresponding distance can never be zero.
(iii) If a particle moves in a straight line without change in Example 3.5 Displacement of a person moving from X to Y
direction, the magnitude of displacement is equal to the along a semicircular path of radius r is 200m. What is the
distance travelled otherwise displacement is always less than distance travelled by him?
distance. Thus,
|Displacement | ≤ Distance Sol. Given, displacement = 200 m
(iv) Distance depends on the path while displacement is Distance travelled by the person from X to Y is equal to the
independent of the path but depends only on initial and final circumference of the semicircular path,
positions. πD
x= ⇒ x = πr (Q Diameter = 2 × Radius) …(i)
2
Motion in One Dimension 81

According to the question, the shortest distance between the Time taken by athlete for completing one A
final position Y and initial position X is XY = 2r . round = 40 s
In 40 s, distance covered by athlete

200 m
4400
r = m
7
X Y ∴ Distance covered by athlete in 2 min
O
and 20 s (= 140 s) B
∴ The displacement traversed by the person is 2 r. 4400 140
200 = 2r ⇒ r = 100 m = × = 2200 m
7 40
Putting r = 100 m in Eq. (i), we get As the athlete returns to the initial point A in 40 s, so his
Distance, x = π × 100 = 3.14 × 100 ⇒ x = 314 m displacement = 0
In 40 s, the number of round, around the track = 1
Example 3.6 An athlete completes one round of a circular
track of diameter 200m in 40s. What will be the distance ∴ In 140 s, the number of rounds around the track
covered and the displacement at the end of 2 min 20s? 140 1
= =3
Sol. Diameter of circular track, D = 200 m 40 2
For each complete round, the displacement is 0.
Circumference of circular track
∴ For 3 complete rounds, the displacement will be 0.
= 2πr = π × (D )
Hence, the final displacement will be due to 1/2 round.
22 4400
= × 200 = m Thus, his displacement = diameter of circular track = 200 m
7 7 ∴ Displacement after 2 min 20 s = 200 m

CHECK POINT 3.1


1. Which of the following is a one-dimensional motion? 5. The three initial and final position of a man on the X-axis
(a) Landing of an aircraft are given as
(b) Earth revolving around the sun (i) (− 8 m, 7 m) (ii) (7 m, − 3 m)
(c) Motion of wheels of moving train (iii) (− 7 m, 3 m)
(d) Train running on a straight track
Which pair gives the negative displacement?
2. A person moves towards east for 3 m, then towards north (a) (i) (b) (ii)
for 4 m and then moves towards for 5 m. What is his (c) (iii) (d) (i) and (iii)
distance now from the starting point?
6. The numerical ratio of displacement to the distance covered
(a) 12 m (b) 5 m (c) 10 m (d) 20 m is always
3. A particle moves in a circle of radius R from A to B as shown (a) less than one
in figure. The distance covered by the object is (b) equal to one
(c) equal to or less than one
A B (d) equal to or greater than one

60° R 7. A particle moves along a circular path of radius R. The


distance and displacement of a particle after one complete
O revolution is
(a) 0, 2πr (b) 2πr, 0
(b) 0, πr (d) πr, 0
πR πR πR
(a) (b) (c) (d) πR 8. A particle starts from the origin, goes along X-axis to the
3 2 4 point (20 m, 0) and then returns along the same line to the
4. A wheel of radius 1 m rolls forward half a revolution on a point (− 20 m, 0). The distance and displacement of the
horizontal ground. The magnitude of displacement of the particle during the trip are
point of the wheel initially in contact with the ground is (a) 40 m, 0 (b) 40 m, 20 m
(a) 2π (b) 2 π (c) π2 + 4 (d) π (d) 40 m, − 20 m (d) 60 m, − 20 m
82 OBJECTIVE Physics Vol. 1

3. Speed Instantaneous speed


The time rate of distance travelled by an object in any The speed of a particle at any instant of time is known as
direction is called speed of the object. its instantaneous speed.

Distance travelled ∆s ds
Speed (v ) = Instantaneous speed = lim =
Time taken ∆t → 0 ∆t dt
where, s represents distance.
It is a scalar quantity.
The unit of speed in SI or MKS system is ms −1 and in CGS Example 3.7 Abdul while driving to school, computes the
system is cms −1. Its dimensional formula is [M 0 LT −1 ]. average speed for his trip to be 20 kmh −1. On his return trip
For a moving body, speed is always positive and can along the same route, there is less traffic and the average speed
is 40 kmh −1. What is the average speed for Abdul’s trip?
never be negative or zero.
Sol. Let t1 and t 2 be the time taken by Abdul to go to school
Average speed and come back from the school, respectively. Let s be its
The ratio of the total distance travelled by the object to distance covered in one way, then
the total time taken is called average speed of the object. s s
t1 = h and t 2 = h
Total distance travelled 20 40
i.e. Average speed = s s 3s
Total time taken Total time taken = t1 + t 2 = + = h
20 40 40
Average speed of particles in different cases Total distance covered = 2s
2s 80
Case I. If a particle travels distance s1, s 2, s 3, ..., etc., with ∴ Average speed = × 40 = = 26.67 kmh−1
speeds v1, v 2, v 3, K, etc., in same direction, then the 3s 3
distance travelled = s1 + s 2 + s 3 + ... Example 3.8 A car covers the first half of the distance
s s s between two places at a speed of 40 kmh −1 and second half
Total time taken = 1 + 2 + 3 + ... at 60 kmh −1. Calculate the average speed of the car.
v1 v 2 v 3
Sol. Given, speed in first half, v1 = 40 kmh−1
s + s 2 + ...
Average speed, v av = 1 Speed in second half, v 2 = 60 kmh−1
 s1 s 2 
 + +... Q Car covers equal distance with different speeds.
 v1 v 2 
∴ Average speed of car,
If s1 = s 2 = s , i.e. the body covers equal distances 2v1v 2
with different speeds, then v av =
v1 + v 2
2s
v av = 2(40) (60)
1 1 ⇒ v av = = 48 kmh−1
s +  40 + 60
 v1 v 2 
Example 3.9 A car moves from X toY with a uniform speed
2v 1v 2 v u and returns to X with a uniform speed v d . Find average
v av = speed for this round trip.
v1 + v 2
distance travelled
Sol. We know that, average speed =
Case II. If a particle travels with speeds v1, v 2, v 3, K, etc., time taken
during time intervals t1, t 2, t 3, ..., etc., then total Let t1 and t 2 be times taken by the car to go from X to Y and
distance travelled, s = v1 t1 + v 2 t 2 + v 3t 3 + ... then from Y to X, respectively.
Total time taken = t1 + t 2 + t 3 + ... XY XY v + vd 
Then, t1 + t 2 = + = XY  u 
v1t1 + v 2 t 2 + v 3 t 3 +... vu vd  vu v d 
So, average speed, v av =
t1 + t 2 + t 3 +... Total distance travelled = XY + XY = 2XY
Therefore, average speed of the car for this round trip is
Case III. If t1 = t 2 = t 3 = ... = t n , then we have
2XY
(v 1 + v 2 + ... + v n ) t v av =
v av = v + vd 
nt XY  u 
 vu v d 
v + v 2 + ... + v n
v av = 1 or v av =
2vu v d
n vu + v d
Motion in One Dimension 83

Example 3.10 A particle travelled half the distance with a Uniform and non-uniform velocity
speed v 0 . The remaining part of the distance was covered
An object is said to have uniform velocity, if the
with speed v1 for half the time and with speed v 2 for the
other half of the time. Find the average speed of the magnitude and direction of its velocity remains constant.
particle. This is only possible when the object moves along a
straight line without reversing its direction.
Sol. If s is the total distance travelled by the particle, then
s s However, an object is said to have non-uniform velocity, if
= v 0t1 ⇒ t1 = either magnitude or direction of velocity change w.r.t. time.
2 2v0
If t is the time taken by particle to travel remaining distance Velocity versus Speed
s v1t v 2 t t s
s/ 2, then = + = (v1 + v 2 ) or t = (i) Velocity of an object can be changed by changing the object’s
2 2 2 2 ( v1 + v 2 ) speed or direction of motion or both.
s s 2 v 0 ( v1 + v 2 ) (ii) For an object in a time interval (t ); |Velocity | ≤ Speed
Average speed = = =
t1 + t s
+
s v1 + v2 + 2v0 i.e. The magnitude of velocity of an object is always equal to or
2 v 0 ( v1 + v 2 ) less than its speed.
(iii) If a body is moving in a straight line, then the magnitude of its
4. Velocity speed and velocity will be equal.
(iv) Average velocity could be zero or positive or negative but
The rate of change of position or displacement of an object average speed is always positive for a moving body.
with time is called the velocity of that object.
Displacement Example 3.11 In one second, a particle goes from A
i.e. Velocity = point A to point B moving in a semicircular path
Time as shown in figure. Find the magnitude of average
velocity. 1m
It is a vector quantity. The unit of velocity in SI or MKS
Sol. Given, t = 1 s and radius, R = 1.0 m
system is ms −1 and in CGS system is cms −1. Its dimensional
formula is [M0 LT –1]. In 1-D motion, the velocity of an object The shortest distance between the final position
(B ) and initial position A is AB, which is the B
is taken to be positive, if the object is moving towards the displacement of the particle.
right of the origin and is taken to be negative, if the object is ∴ Total displacement (AB ) = 2R = 2 × 1.0 = 2 m
moving towards the left of the origin.
Total displacement
Thus, average velocity, v av =
Average velocity Total time taken
The ratio of the total displacement to the total time taken AB 2.0
= = ms −1 = 2 ms −1
is called average velocity. ∆t 1.0

Total displacement (∆x ) Example 3.12 A farmer has to go 500m due north, 400m
Average velocity = due east and 200m due south to reach his field. If he takes
Total time (∆t )
20 min to reach the field,
If velocity of the object changes at a uniform rate, then (i) what distance he has to walk to reach the field ?
Initial velocity + Final velocity (ii) what is the displacement from his house to the field ?
Average velocity = (iii) what is the average speed of farmer during the walk ?
2 (iv) what is the average velocity of farmer during the walk ?
Note For a given time interval, average velocity has single value Sol. (i) Distance = AB + BC + CD
while average speed can have many values depending on path
followed. = (500 + 400 + 200) = 1100 m
Instantaneous velocity (ii) Displacement = AD = (AB − CD )2 + BC 2
The velocity of a particle at any instant of time is known
= (500 − 200)2 + (400)2 = 500 m
as its instantaneous velocity.
400 m N
∆x dx B C
v = lim =
∆t → 0 ∆t dt 200 m
500 m D W E
Note The magnitude of average velocity may be less than or equal to the (Field)
average speed for a particular motion. But the magnitude of
instantaneous velocity is always equal to the instantaneous speed A
(Home) S
for a particular instant.
84 OBJECTIVE Physics Vol. 1

Total distance 1100 Example 3.14 The position of an object moving along X-axis
(iii) Average speed = = = 55 m/min
Total time 20 is given by x = 3t − 4t 2 + t 3, where x is in metres and t in
AD 500 seconds. Find the position of the object at the following
(iv) Average velocity = = = 25 m/min (along AD) values of t : (i) 2 s, (ii) 4s, (iii) What is the object’s
t 20 displacement between t = 0 s and t = 4 s ?; and (iv) What is
Example 3.13 Joseph jogs from one end A to the other end B its average velocity for the time interval from t = 2 s to
of a straight 300 m road in 2 min 50 s and then turns t = 4s ?
around and jogs 100 m back to point C in another 1 min. Sol. Using x = 3t − 4t 2 + t 3 with SI units …(i)
What are Joseph’s average speeds and velocities in jogging (i) (i) Substituting t = 2 s in Eq. (i), we get
from A to B and (ii) from A to C?
x 2 = 3(2) − 4(2)2 + (2)3 = − 2 m
Sol. Given, distance covered from A to B = 300 m
Thus, the position of the object at t = 2 s is x 2 = − 2 m.
Time = 2 min 50 s = (2 × 60) + 50 s = 170 s (ii) Substituting t = 4 s in Eq. (i), we get
300 m x 4 = 3(4) − 4(4)2 + (4)3 = 12 − 64 + 64 = 12 m
C
A 100 m B Thus, the position of the object at t = 4 s is x 4 = 12 m.
Distance covered (iii) The displacement of the object between t = 0 s and t = 4 s
(i) Average speed = can be calculated as follows :
Time
The position of the object at t = 0 s is x = 0.
300
= = 1.76 ms−1 ∆x = Final position – Initial position = x 4 − x 0
170
= 12 m − 0 m = 12 m
Displacement along AB Hence, the displacement between t = 0 s and t = 4 s is 12 m.
Average velocity =
Time (iv) The displacement of the object from t = 2 s to t = 4 s is
=
300
= 1.76 ms−1 along AB ∆x = x 4 − x 2 = 12 m − (−2 m) = 14 m
170 The time interval from t = 2 s to t = 4 s is
(ii) When Joseph turns around from B to C towards A, then ∆t = 4 s − 2 s = 2 s
The average velocity of the object from t = 2 s to t = 4 s is
Distance covered 400
Average speed = = = 1.74 ms−1 given by the relation
Time 230 ∆x 14 m
v av = =
Displacement AC 200 ∆t 2s
Average velocity = = along AC
Time 230
= 7 ms−1
= 0.87 ms along AC
–1

CHECK POINT 3.2


1. A car has to cover the distance 60 km. If half of the total 4. A particle is constrained to move on a straight line path. It
time, it travels with speed 80 kmh−1 and in rest half time, its returns to the starting point after 10 s. The total distance
speed becomes 40kmh−1, the average speed of car will be covered by the particle during this time is 30 m. Which of
the following statements about the motion of the particle is
(a) 60 kmh−1 (b) 80 kmh−1 (c) 120 kmh−1 (d) 180 kmh−1
true?
2. During the first 18 min of a 60 min trip, a car has an average (a) Displacement of the particle is zero
speed of 11 m min−1. What should be the average speed for (b) Average speed of the particle is 3 ms−1
remaining 42 min, so that car is having an average speed of (c) Displacement of the particle is 30 m
21 m min−1 for the entire trip? (d) Both (a) and (b)
(a) 25.3 m min−1 (b) 29.2 m min−1
−1
5. A 150 m long train is moving with a uniform velocity of
(c) 31 m min (d) 35.6 m min−1 45 kmh −1 . The time taken by the train to cross a bridge of
length 850 m is
3. A man walks on a straight road from his home to a market
(a) 56 s (b) 68 s (c) 80 s (d) 92 s
2.5 km away with a speed of 5 kmh −1 . Finding the market
closed, he instantly turns and walks back home with a 6. An insect crawls a distance of 4 m along north in 10 s and
speed of 7.5 kmh −1 . The average speed of the man over the then a distance of 3 m along east in 5 s. The average velocity
interval of time 0 to 40 min is equal to of the insect is
25 7 1
(a) 5 kmh −1 (b) kmh −1 (a) ms−1 (b) ms−1
4 15 5
30 45 1 4
(c) kmh −1 (d) kmh −1 (c) ms−1 (d) ms−1
4 8 3 5
Motion in One Dimension 85

7. A particle traversed (3/4) th of the circle of radius R in time 8. A boy is running over a circular track with uniform speed of
t. The magnitude of the average velocity of the particle in 10ms −1. What is the average velocity for movement of boy
this time interval is along semicircle (in ms −1)?
πR 3 πR 10 40
(a) (b) (a) (b)
t 2t π π
R 2 R 20
(c) (d) (c) 10 (d)
t 2t π

5. Acceleration
The time rate of change of velocity of a body is called is defined as the ratio of the total change in velocity of the
acceleration. object to the total time taken.

Change in velocity (∆v ) Total change in velocity


∴ Acceleration = Average acceleration =
Time interval (∆t ) Total time taken

If in a given time interval, the velocity of a body Note The average acceleration can be positive or negative depending
upon the sign of change of velocity. It is zero, if the change in
changes from u to v, then acceleration a is expressed as velocity of the object in the given interval of time is zero.

Final velocity − Initial velocity v − u Instantaneous acceleration


a= =
Time interval t The acceleration of an object at a given instant of time or at
It is a vector quantity. Its SI unit is ms −2
and CGS unit is a given point during the motion, is called its instantaneous
acceleration. i.e.
cms −2 .
Its dimensional formula is [M0 LT −2 ]. Its direction is ∆v dv d 2 s
a = lim = =
same as that of change in velocity (not of the velocity). ∆t → 0 ∆t dt dt 2
There are three possible ways by which change in
velocity may occur Key points regarding acceleration
(i) When only direction of velocity changes, then Following are the important points for motion of an object
acceleration is perpendicular to velocity. having some acceleration
e.g. Uniform circular motion. (i) A body falling down from a height or a body rolling
(ii) When only magnitude of velocity changes, down on a smooth inclined plane has uniform
then acceleration is parallel or anti-parallel to acceleration.
velocity. (ii) If a car travelling along a straight road, increases its
e.g. Motion under gravity. speed by unequal amounts in equal intervals of time,
(iii) When both magnitude and direction of then the car is said to be moving with non-uniform
velocity changes, then acceleration has two acceleration.
components : one is perpendicular to velocity and (iii) The acceleration is created by accelerator of the
another is parallel or anti-parallel to velocity. vehicles and the applications of breaks give the
e.g. Projectile motion. uniform deceleration to the vehicles. However, the
acceleration produced in spring block system is
Retardation non-uniform acceleration.
When the velocity of a body increases with time,
(iv) If a particle is accelerated for time t1 with acceleration
acceleration is positive and when the velocity of a body
a1 and for time t 2 with acceleration a 2 , then average
decreases with time (i.e. u > v ), then acceleration
acceleration is
becomes negative. This negative acceleration is also
a t + a 2t 2
called deceleration or retardation. In other words, a av = 1 1
retardation opposes the motion of body. t1 + t 2
Average acceleration (v) Acceleration can also be written as
When an object is moving with a variable acceleration, dv dv dx  dv 
a= = . = v 
then the average acceleration of the object for the given dt dx dt  dx 
motion
86 OBJECTIVE Physics Vol. 1

Example 3.15 A car starts from rest, attains a velocity of For motion of car from B to C,
18kmh –1 with an acceleration of 0.5 ms −2 , travels 4 km with s = 4 km = 4000 m
this uniform velocity and then comes to halt with a uniform
and v = 5 ms–1
deceleration of 0.4 ms −2 . Calculate the total time of travel of
the car. distance
t2 =
Sol. Let the car be accelerated from A to B, it moves with uniform velocity
velocity from B to C of 4 km distance and then moves with 4000
= = 800 s ...(iii)
uniform deceleration of 0.4 ms−2 from C to D as shown below. 5
A Acceleration B Uniform C Deceleration D
For motion of car from C to D, v = 0, u = 5 ms–1
velocity and a = − 0. 4 ms–2 (negative sign shows deceleration)
−2
For motion of car from A to B, a = 0. 5 ms v −u 0− 5
Time taken, t 3 = =
u = 0 and v = 18 km h−1 a − 0. 4
5 −5
= 18 × ms−1 = 5ms−1 = = 12 . 5 s ...(iv)
18 − 0. 4
v −u Total time taken, T = t1 + t 2 + t 3
Time, t1 = ...(i)
a Substituting values of t1, t 2 and t 3 from Eqs. (ii), (iii) and (iv)
Substituting given values of v, u and a for A to B motion, we respectively, we get
get T = (10 + 800 + 12.5 )s = 822 . 5
5−0
t1 = = 10 s ...(ii) Thus, total time of travel of the car is 822.5 s.
0.5

CHECK POINT 3.3


1. Acceleration of a particle changes when 4. A car travelling with a velocity of 80 km/h slowed down to
(a) direction of velocity changes 44 km/h in 15 s. The retardation is
(b) magnitude of velocity changes (a) 0.67 ms−2 (b) 1 ms−2 (c) 1.25 ms−2 (d) 1.5 ms−2
(c) Both (a) and (b) 5. An object is moving along the path OABO with constant
(d) speed changes speed, then
2. If a particle moves with an acceleration, then which of the
following can remain constant? B
(a) Both speed and velocity
(b) Neither speed nor velocity
(c) Only the velocity
(d) Only the speed
3. The average velocity of a body moving with uniform O A
acceleration travelling a distance of 3.06 m is 0.34 ms −1. If
(a) the acceleration of the object while moving along the path
the change in velocity of the body is 0.18 ms −1, then during OABO is zero
this time, its uniform acceleration is
(b) the acceleration of the object along the path OA and BO is zero
(a) 0.01 ms−2 (b) 0.02 ms−2
(c) there must be some acceleration along the path AB
(c) 0.03 ms−2 (d) 0.04 ms−2 (d) Both (b) and (c)

KINEMATIC EQUATIONS FOR UNIFORMLY


ACCELERATED MOTION
When a body is moving along a straight line with uniform The three equations of motion on a straight line are
acceleration, then its motion is called uniformly 1
accelerated motion. For this motion, we can establish the (i) v = u + at (ii) s = ut + at 2
2
relation between velocity, acceleration and the distance
travelled by the body in a particular time interval by a set (iii) v 2 − u 2 = 2as
of equations. These equations are known as kinematic where, u is the initial velocity of the body, a is the
equations or equations of motion. uniform acceleration of the body, v is the final velocity of
Motion in One Dimension 87

the body after t second and s is the distance travelled in Sol. Let both cars reach at same position in same time t, then
this time. 1
from s = ut + at 2
● Distance travelled by a body in n th second, 2
1 t2
1 For 1st car, s = 4(t ) + (1) t 2 = 4t + …(i)
sn = u + a (2n − 1) 2 2
2 1
For 2nd car, s = 2(t ) + (2)t 2 = 2t + t 2
2
Note
(i) If initial position of a particle is r0, then position at time t can be Equating above equations, we get
written as t2
1 4t + = 2t + t 2 ⇒ t = 4 s
r = r 0 + s = r 0 + u t + at 2 2
2
(ii) Stopping distance When brakes are applied to a moving vehicle,
Substituting the value of t in Eq. (i), we get
1
the distance it travels before stopping is called stopping distance. It s = 4(4) + (1)(4)2 = 16 + 8 = 24 m
is an important factor for road safety. It is given by 2

s=
u2 Example 3.17 A car was moving at a rate of 18 kmh −1.
2a When the brakes were applied, it comes to rest at a distance
where, u is initial velocity and a is the retardation produced by
of 100 m. Calculate the retardation produced by the brakes.
brakes. Sol. Given, v = 0, u = 18 kmh−1 = 5 ms−1, s = 100 m
Key points regarding kinematic equations Using the equation of motion,
Following are the important points in case of one v 2 − u 2 = 2as …(i)
dimensional motion with constant acceleration ⇒ −u = 2as
2
(Qv = 0)
(i) If the motion starts from rest, then initial velocity is u 2
⇒ a=−
taken as zero, i.e. u = 0. 2s
(ii) If the object comes to rest after the motion, then −5 × 5 1
⇒ a= = − = − 0.125 ms−2
final velocity is taken as zero, i.e. v = 0. 2 × 100 8
(iii) If velocity of moving object increases with time, So, the retardation produced by the brakes is 0.125 ms−2.
then acceleration is taken as positive and if velocity
decreases with time, acceleration is taken as Example 3.18 Two car travelling towards each other on a
negative. straight road at velocity 10 ms −1and 12 ms −1, respectively.
(iv) If velocity and acceleration both have same sign like When they are 150 m apart, both the drivers apply their
v > 0 ; a > 0 or v < 0 ; a < 0, then object is speeding brakes and each car decelerates at 2 ms −2 until it stops.
up. Similarly, if velocity and acceleration both have How far apart will they be when both of them come to rest?
opposite sign like v < 0 ; a > 0 or v > 0 ; a < 0. Then, Sol. Let x1 and x 2 be the distance travelled by the car before
the object is speeding down. they stop under deceleration.
(v) For motion of an object along a straight line, normally From third equation of motion,
we take vertically upward direction positive (and v 2 = u 2 + 2as
downward negative) and horizontally rightwards ⇒ 0 = (10)2 − 2 × 2x1 ⇒ x1 = 25 m
positive (or leftwards negative). Sign convention for
(a) motion in vertical direction (b) motion in horizontal and 0 = (12)2 − 2 × 2x 2 ⇒ x 2 = 36 m
direction is shown in figure. Total distance covered by the two cars
= x1 + x 2 = 25 + 36 = 61 m
+ve Distance between the two cars when they stop
−ve = 150 − 61 = 89 m
+ve −ve
(a) (b) Example 3.19 A train travelling at 20 kmh −1 is approaching
a platform. A bird is sitting on a pole on the platform. When
Fig. 3.6 Sign convention for vertical and horizontal direction
the train is at a distance of 2 km from pole, brakes are
applied which produce a uniform deceleration in it. At that
Example 3.16 Two cars start off a race with velocities 2 ms −1
instant, the bird flies towards the train at 60 kmh −1 and
and 4 ms −1 travel in straight line with uniform
after touching the nearest point on the train flies back to the
accelerations 2 ms −2 and 1 ms −2 , respectively. What is the
pole and then flies towards the train and continues repeating
length of the path, if they reach the final point at the same
itself. Calculate how much distance the bird covers before
time?
the train stops?
88 OBJECTIVE Physics Vol. 1

Sol. For retardation of train, v 2 = u 2 + 2as a2 t


or t1 =
⇒ 0 = (20) + 2(a )(2)
2 a1 − a 2
−2
⇒ a = − 100 kmh Substituting this value of t1 in Eq. (iii), we get
Time required to stop the train, v = u + at v = ( a1a 2 ) t
1
⇒ 0 = 20 − 100t ⇒ t = h Example 3.22 A body starting from rest has an acceleration
5 of 4 ms −2 . Calculate distance travelled by it in 5th second.
distance
For bird, speed = Sol. Given, u = 0, a = 4 ms−2
time
1 Distance travelled by the body in 5th second is
⇒ sB = vB × t = 60 × = 12 km 1
5 sn = u + a (2n − 1)
2
Example 3.20 A particle starts with an initial velocity and 1
passes successively over the two halves of a given distance with s5 = 0 + × 4(2 × 5 − 1)
2
accelerations a1 and a 2 , respectively. Show that the final 1 36
velocity is the same as if the whole distance is covered with a = × 4(9) = = 18 m
(a + a 2 ) 2 2
uniform acceleration 1 ⋅
2 Example 3.23 A particle starts from rest and moves under
Sol. In the first case, constant acceleration in a straight line. Find the ratio of
displacement (i) in successive second and (ii) in successive
u v1 v2 u a1 + a2 v
→ → → → 2s, → time interval t 0 .
s, a1 s, a2 2
First case Second case Sol. (i) Displacement in 1 s or 1st second,
1 1 1
v12 = u 2 + 2a1s …(i) s1 = ut + at 2 = 0 + a (1)2 = a
2 2 2
v 22 = v12 + 2a 2s K (ii) or
1 1 1
s1 = u + a (2t − 1) = 0 + a (2 × 1 − 1) = a
Adding Eqs. (i) and (ii), we get 2 2 2
a + a2 Displacement in the 2nd second,
v 22 = u 2 + 2  1  (2 s ) K (iii) 1 1 3
 2  s2 = u + a (2t − 1) = 0 + a (2 × 2 − 1) = a
2 2 2
In the second case,
Displacement in the 3rd second,
a + a2
v 2 = u2 + 2  1  (2 s ) K (iv) 1
s3 = 0 + a (2 × 3 − 1) = a
5
 2 
2 2
From Eqs. (iii) and (iv), we can see that 1 3 5
s1 : s2 : s3 : K = a : a : a :... = 1 : 3 : 5 : K
v2 = v 2 2 2
(ii) u = 0 a
Example 3.21 In a car race, car A takes a time t less than car
B at the finish point and passes the finishing point with speed v A s1 B s2 C s3 D
more than that of the car B. Assuming that both the cars starts
from rest and travel with constant acceleration a1 and a 2, A to B : Displacement in the first t 0 second,
respectively. Show that v = a1a 2 t. 1 1
s = ut + at 2 ⇒ s1 = 0 + at 02 = at 02
1
2 2 2
Sol. Let A takes t1 second, then according to the given problem B
A to C, t = t 0 + t 0 = 2t 0
will take (t1 + t ) seconds. Further, let v1 be the velocity of B at
1
finishing point, then velocity of A will be (v1 + v ). Writing s1 + s2 = 0 + a (t 0 + t 0 )2 = 2 at 02
equations of motion for A and B, 2
v1 + v = a1t1 K (i) Displacement in the next t 0 second,
v1 = a 2 (t1 + t ) 3
K (ii) s2 = at 02
From these two equations, we get 2
v = (a1 − a 2 ) t1 − a 2t K (iii) A to D, t = t 0 + t 0 + t 0 = 3t 0
1 9
Total distance travelled by both the cars is equal. s1 + s2 + s3 = 0 + a (t 0 + t 0 + t 0 )2 = at 02
or sA = sB 2 2
1 2 1 Displacement in the next t 0 second,
or a1t1 = a 2 (t1 + t )2 5
2 2 s3 = at 02 ⇒ s1 : s2 : s3 = 1 : 3 : 5
2
CHECK POINT 3.4
1. Velocity of a body moving along a straight line with (a) 2s (b) 4 s
3 (c) 6 s (d) 8 s
uniform acceleration (a) reduces by of its initial velocity
4 7. Two bodies A and B start from rest from the same point
in time t0 . The total time of motion of the body till its with a uniform acceleration of 2 ms −2. If B starts one
velocity becomes zero is
4 3
second later, then the two bodies are separated at the end
(a) t0 (b) t0 of the next second, by
3 2 (a) 1 m (b) 2 m
5 8
(c) t0 (d) t0 (c) 3 m (d) 4 m
3 3
8. A train accelerating uniformly from rest attains a maximum
2. The displacement of a body in 8 s starting from rest with an speed of 40 ms −1 in 20 s. It travels at this speed for 20 s and
−2
acceleration of 20 cms is
is brought to rest by uniform retardation in further 40 s.
(a) 64 m (b) 64 cm What is the average velocity during this period?
(c) 640 cm (d) 0.064 m (a) (80 / 3) ms−1 (b) 40 ms−1
3. The motion of a particle is described by the equation v = at. (c) 25 ms −1
(d) 30 ms−1
The distance travelled by the particle in the first 4 s is
(a) 4a (b) 12a 9. A particle starts from rest and traverses a distance l with
(c) 6a (d) 8a uniform acceleration, then moves uniformly over a further
−1 distance 2l and finally comes to rest after moving a further
4. A particle starts with a velocity of 2 ms and moves in a
distance 3l under uniform retardation. Assuming entire
. ms −2. The first time at
straight line with a retardation of 01 motion to be rectilinear motion, the ratio of average speed
which the particle is15 m from the starting point is over the journey to the maximum speed on its ways is
(a) 10 s (b) 20 s (a) 1 / 5 (b) 2/ 5 (c) 3/ 5 (d) 4 / 5
(c) 30 s (d) 40 s
10. A body travelling with uniform acceleration crosses two
5. A particle starts from rest, accelerates at 2 ms −2 for10 s and points A and B with velocities 20 ms −1 and 30 ms −1 ,
then moves with constant speed of 20 ms −1 for 30 s and respectively. The speed of the body at mid-point of A and B
then decelerates at 4 ms −2 till it stops after next 5 s. What is is
the distance travelled by it? (a) 25 ms−1 . ms−1
(b) 255
(a) 750 m (b) 800 m (c) 700 m (d) 850 m (c) 24 ms−1 (d) 10 6 ms−1
6. A body is moving with uniform velocity of 8 ms −1 . When 11. If a body starts from rest and travels 120 cm in the 6th
the body just crossed another body, the second one starts second, then what is the acceleration?
and moves with uniform acceleration of 4 ms −2. The time (a) 0.20 ms −2 (b) 0.027 ms −2
after which two bodies meet, will be (c) 0.218 ms −2 (d) 0.03 ms −2

MOTION UNDER GRAVITY Equations for motion under gravity


When the objects fall under the influence of gravity, its
The objects falling towards the earth under the influence motion is uniformly accelerated motion. Hence, equations
of gravitational force alone, are called freely falling objects of motion are applicable in this case also.
and such fall is called free fall.
Equation for motion under gravity are given below
Whenever an object falls towards the earth, an acceleration
(i) If particle is thrown vertically upwards
is involved, this acceleration is due to the earth’s gravitational
pull and is called acceleration due to gravity. The value of In this case, applicable kinematics relations
g
are
acceleration due to gravity near the earth surface is 9.8 ms −2 .
v = u − gt …(i)
It is independent of the mass of freely falling objects and is u
1
denoted by g. h = ut − gt 2 …(ii)
Though the value of g is independent of freely falling mass, a 2
Fig. 3.7
feather falls much slowly than a coin when released from a v 2 = u 2 − 2gh …(iii)
height. This is due to the resistance offered by air to the Here, h is the vertical height of the particle in
falling mass. If both the bodies were released at the same upward direction.
time in vacuum (no air resistance), they would reach the In this case, acceleration due to gravity is taken as
earth surface within the same duration of time. negative.
90 OBJECTIVE Physics Vol. 1

At maximum height (say h), v = 0


u2
(Q at maximum height, the particle stops moving h=
upwards that means its velocity becomes zero) 2g
∴ From the Eq. (i), u = gt (b) Velocity of particle at the time of striking the
u ground when released (u = 0 ) from a height h is
or t = , which is called time of ascent.
g v = 2gh
For motion under gravity, for the same distance, the (c) In above point (b), time of collision of particle
time taken to go up is same as time taken to fall with ground,
down.
∴ Time of ascent = Time of descent 2h
t=
Total flight time, T = 2 × Time of ascent or descent g

2u Example 3.24 A ball is thrown upwards from the top of a


Total flight time (T ) = tower 40 m high with a velocity of 10 m/s. Find the time
g
when it strikes the ground. (Take, g = 10 m/s 2 )
From Eqs. (ii) and (iii), we get Sol. According to the question, the condition is as shown below.
1
h = gt 2 u = +10 m/s
2 +ve
a = g = –10 m/s2
2
u
and u 2 = 2gh or h = s=0
2g
40 m
(ii) If particle is thrown vertically downward with
some velocity from some height.

u Given, u = + 10 m / s, a = − 10 m/s 2 and s = − 40 m


(at the point, where stone strikes the ground)
1
g Substituting in s = ut + at 2, we get
2
Fig. 3.8 − 40 = 10 t − 5t 2 or 5t 2 − 10 t − 40 = 0
In this case, v = u + gt …(i) or t2 − 2t − 8 = 0
1 2 Solving this, we have
h = ut + gt …(ii)
2 t = 4 s and −2 s. Taking the positive value t = 4 s.
v 2 = u 2 + 2gh …(iii) Note The significance of t = − 2 s can be understood by following figure

Here, h is the vertical height of particle in downward C C


direction. t=1s tAB = tDE = 2 s
tBC = tCD = 1 s
In this case, acceleration due to gravity is taken as
positive. t=0 B D
t=2s
(iii) If a particle is dropped from some height.
In this case, initial velocity is taken zero u=0
(u = 0 ), so equations of motion are g t=–2s A E t=4s
v = gt …(i)
1
h = gt 2 …(ii) Example 3.25 A rocket is fired vertically up from the ground
2 with a resultant vertical acceleration of 10 ms −2 . The fuel is
v = 2gh
2
…(iii) Fig. 3.9 finished in 1 min and it contiues to move up.
(i) What is the maximum height reached?
For fast calculation in objective problems,
remember the following results (ii) After finishing fuel, calculate the time for which it continues
its upwards motion. (Take, g = 10 ms −2)
(a) Maximum height attained by a particle, thrown
upwards from ground,
Motion in One Dimension 91

Sol. (i) The distance travelled by the rocket during burning of ⇒ u 2 − 2 g (− h ) = 4 (u 2 − 2 gh )


fuel (1 minute = 60 s) in which resultant acceleration is
10 gh
1
vertically upwards and is 10 ms −2 will be h1 = ut + gt 2 ∴ u2 =
2 3
= 0 × 60 + (1/2) × 10 × 602 = 18000 m = 18 km u 2 5h
Now, maximum height, h max = =
and velocity acquired by it will be 2g 3
v = u + at = 0 + 10 × 60 = 600 ms −1
Example 3.28 A ball is thrown vertically upwards with a
Now, after 1 min, the rocket moves vertically up with
velocity of 20 ms −1 from the top of a multistorey building.
initial velocity of 600 ms −1 and acceleration due to The height of the point from where the ball is thrown is
gravity opposes its motion. 25 m from the ground. How long it will take before the ball
So, it will go to a height h 2 from this point, till its hits the ground? (Take, g = 10 ms −2 )
velocity becomes zero such that
v 2 − u 2 = −2gh ⇒ 0 − (600)2 = − 2gh 2 (g = 10 ms −2 ) Sol. Let us take the positiveY -axis in the vertically upward
direction with zero at the ground.
or h 2 = 18000 m = 18 km
Now, v 0 = + 20 ms−1, a = − g = −10 ms−2, v = 0 ms−1
So, the maximum height reached by the rocket from the
ground, H = h1 + h 2 = 18 + 18 = 36 km The total time taken can also be calculated by noting the
(ii) As after burning of fuel, the initial velocity attained will coordinates of initial and final positions of the ball with
be 600 ms −1 and gravity opposes the motion of rocket, respect to the origin chosen and using equation
so from first equation of motion time taken by it till its 1
y = y 0 + v 0t + at 2
velocity v = 0 is given as, 2
0 = 600 − gt ⇒ t = 60 s Now, y 0 = 25 m, y = 0 m,
Example 3.26 A juggler throws balls into air. He throws one v 0 = 20 ms −1 , a = − 10 ms−2
ball whenever the previous one is at its highest point. How  1
∴ 0 = 25 + 20t +   (− 10)t 2
high does the balls rise, if he throws n balls each second?  2
Acceleration due to gravity is g.
Sol. Juggler throws n balls in one second, so time interval ⇒ 5t 2 − 20t − 25 = 0
1 Solving this quadratic equation for t, we get
between two consecutive throws is t = s
n t = 5s
Example 3.29 A ball is thrown upwards from the ground with
an initial speed u. The ball is at a height of 80 m at two
1
hmax t=
n
s times, for the time interval of 6s. Find the value of u.
Sol. Here, a = g = − 10 ms−2 and s = 80 m
1
Substituting the values in s = ut + at 2,
2
1 s = 80 m
Each ball takes s to reach maximum height.
n
2 + ve
1 1  1 g
So, h max = × gt 2 = × g   ⇒ h max =
2 2 n  2n 2
− ve u

Example 3.27 From an elevated point A, a stone is projected


vertically upwards. When the stone reaches a distance h below we get 80 = ut − 5t 2
A, its velocity is double of what it was at a height h above A. or 5t 2 − ut + 80 = 0
5
Show that the greatest height attained by the stone is h. u + u 2 − 1600
3 or t=
Sol. Let u be the velocity with which the stone is projected 10
vertically upwards. u − u 2 − 1600
and t=
Given, v − h = 2 v h or (v − h )2 = 4 v h2 10
Now, it is given that
According to the kinematic equation,
u + u 2 − 1600 u − u 2 − 1600
v h2 = u 2 − 2gh − =6
10 10
and v –2h = u 2 − 2g (− h )
92 OBJECTIVE Physics Vol. 1

u 2 − 1600 3 t 0, third drop for 2t 0, fourth drop for t 0 when fifth drop is
⇒ =6 about to fall. The location of drops are as shown in the figure.
5
⇒ u 2 − 1600 = 30
⇒ u 2 − 1600 = 900 5th drop
h4
∴ u 2 = 2500 4th drop h3
⇒ u = ± 50 ms−1 h2
3rd drop h1
Ignoring the negative sign, we get
u = 50 ms−1 2nd drop
h1 = 16 m
Example 3.30 A particle is thrown vertically upwards from 1st drop
the surface of the earth. Let TP be the time taken by the
particle to travel from a point P above the earth to its highest For 1st drop,
point and back to the point P. 1 1
h1 = g (4t 0 )2, 16 = g × 16t 02
Similarly, let TQ be the time taken by the particle to travel 2 2
from another point Q above the earth to its highest point and 1 2
back to the same point Q. If the distance between the points gt 0 = 1 m
2
P and Q is H, find the expression for acceleration due to 1
gravity in terms of TP , TQ and H. For 2nd drop, h 2 = g (3t 0 )2 = 9 m
2
Sol. Time taken by the particle to travel from point P back to 1
For 3rd drop, h 3 = g (2t 0 )2 = 4 m
point P, 2
TP = TPR + TRP 1 2
For 4th drop, h 4 = gt 0 = 1m
Here, TPR = TRP , then TP = 2TPR 2
Using second equation of motion, For 5th drop, h 5 = 0
1 2 Separation between drops
(H + h ) = gTPR
2 1st and 2nd : h1 − h 2 = 7 m
(H + h ) R 2nd and 3rd : h 2 − h 3 = 5 m
⇒ TPR = 2 Highest 3rd and 4th : h 3 − h 4 = 3 m
g point
h 4th and 5th : h 4 − h 5 = 1 m
2(H + h )
⇒ TP = 2 Q Note If the 1st drop is at the ground and the 5th drop is about to fall,
g H the time for which the first drop has fallen ( 5 − 1) t 0 = 4 t 0, where
P t 0 is the regular interval of time.
Then, similarly time taken by the particle
to travel from point Q back to point Q, Example 3.32 A ball is dropped from the top of a tower.
2h After 2s another ball is thrown vertically downwards with a
TQ = 2 speed of 40 ms −1. After how much time and at what
g
distance below the top of tower the balls meet?
8(h + H )
TP2 = Sol. Let the balls meet at distance h below the top of tower at t
g second after dropping of first ball. The second ball takes time
8h (t − 2) seconds.
and TQ2 =
g O O

8H 8H
⇒ TP2 = TQ2 + ⇒ g= 2 u=0 u = 40 ms−1
g TP − TQ2 h h

Example 3.31 From the top of a building, 16 m high water


drops are falling at equal intervals of time such that when
the first drop reaches the ground, the fifth drop just starts.
Find the distance between the successive drops at that First ball Second ball
instant. For first ball,
Sol. Let the interval of time be t 0. 1 2
h=
gt ...(i)
First drop is released at t = 0, second drop at t = t 0, third drop 2
at t = 2t 0, fourth drop at t = 3t 0, fifth drop at t = 4t 0. 1
For second ball, h = 40 (t − 2) + g (t − 2)2 ...(ii)
Therefore, first drop has fallen for time 4 t 0, second drop for 2
Motion in One Dimension 93

From Eqs. (i) and (ii), we get or ds = (10 + 2t + 3t 2 ) dt


1 1
40 (t − 2) + g (t − 2)2 = gt 2 s 1
2 2 or ∫ 20 ds = ∫ 0 (10 + 2t + 3t 2) dt
1 2
40 (t − 2) = g [t − (t − 2)2] or
1
s − 20 = [10t + t 2 + t 3] 0
2
1 or s = 20 + 12 = 32 m
40 (t − 2) = × 10 (2t − 2) × 2
2 (ii) Acceleration-time equation can be obtained by
4t − 8 = 2t − 2 ⇒ t = 3 s differentiating Eq. (i) w.r.t. time. Thus,
Distance below the top of tower, where the balls meet, dv d
a= = (10 + 2t + 3t 2)
1 1 dt dt
h = gt 2 = × 10 × 32 = 45 m
2 2 or a = 2 + 6t
Example 3.34 Displacement-time equation of a particle
NON-UNIFORMLY moving along X-axis is
x = 20 + t 3 − 12t (SI units)
ACCELERATED MOTION (i) Find position and velocity of particle at time t = 0.
When acceleration of particle is not constant, motion is (ii) State whether the motion is uniformly accelerated or not.
known as non-uniformly accelerated motion. Then, basic (iii) Find position of particle when velocity of particle is zero.
equations of velocity and acceleration can be written as Sol. (i) Given, displacement-time equation of a particle
ds dr
(i) v = or sometimes v = x = 20 + t 3 − 12t K(i)
dt dt
dv At t = 0, x = 20 + 0 − 0 = 20 m
(ii) a = Velocity of particle at time t can be obtained by
dt
differentiating Eq. (i) w.r.t. time, i.e.
(iii) ds = v dt
dx
(iv) dv = a dt v= = 3t 2 − 12 K(ii)
dt
For one dimensional motion, above relations can be At t = 0, v = 0 − 12 = − 12 m/s
written as under (ii) Differentiating Eq. (ii) w.r.t. time t, we get the acceleration,
ds dv dv
(i) v = (ii) a = =v dv
dt dt ds a= = 6t
dt
(iii) ds = v dt and (iv) dv = adt or v dv = a ds
As acceleration is a function of time, the motion is
Such problems can be solved either by differentiation or non-uniformly accelerated motion.
integration (with some boundary conditions). (iii) Substituting v = 0 in Eq. (ii), we get
s-t → v-t → a-t (Differentiation) 0 = 3t 2 − 12
a-t → v-t → s-t Positive value of t comes out to be 2 s from this
(Integration with boundary conditions) equation. Substituting t = 2 s in Eq. (i), we get
Note x = 20 + (2)3 − 12 (2) or x = 4 m
(i) By boundary condition, we mean that velocity or displacement at
some time (usually at t = 0) should be known to us. Otherwise we Example 3.35 The velocity of particle moving in the positive
cannot find constant of integration. direction of X-axis varies as v = α x , where α is a positive
(ii) Equation a = v dv / ds or v dv = a ds is useful when constant. Assuming that at moment t = 0, the particle was
acceleration-displacement equation is known and located at the point x = 0.
velocity-displacement equation is required.
Find
Example 3.33 Velocity-time equation of a particle moving in (i) the time dependence of the velocity of the particle.
a straight line is (ii) the mean velocity of the particle averaged over the time that
v = (10 + 2t + 3t 2) (SI units) the particle takes to cover first s metres of the path.
Find Sol. (i) Given, the velocity of the particle moving in the
positive direction of X-axis,
(i) displacement of particle from the mean position at time
t = 1 s, if it is given that displacement is 20 m at time t = 0. v =α x
(ii) and acceleration-time equation. dx
⇒ =α x
Sol. (i) The given equation can be written as dt
x dx t
v=
ds
= (10 + 2t + 3t 2) ...(i) By integrating ∫ = α ∫ dt
0 x 0
dt
94 OBJECTIVE Physics Vol. 1

x − 1/ 2 + 1 α 2t 2 α 2t 2
⇒ = αt (ii) Distance, x = ; for s distance s =
− (1/2) + 1 4 4
α 2t 2 4s
⇒ x= Time taken to cover first s distance, t =
4 α2
Time dependence of the velocity of the particle, The mean velocity of the particle,
dx 2α 2t 1 2 s s α s
v= = = αt v av = = =
dt 4 2 t 4s /α 2 2

CHECK POINT 3.5


1. If a stone is thrown up with a velocity of 9.8 ms −1 , then 9. A body is thrown vertically upwards from the top A of tower.
how much time will it take to come back? It reaches the ground in t1 second. If it is thrown vertically
(a) 1 s (b) 2s downwards from A with the same speed, it reaches the ground
(c) 3s (d) 4 s in t2 second. If it is allowed to fall freely from A, then the time
2. If a ball is thrown vertically upwards with speed u, the it takes to reach the ground is given by
t1 + t2 t1 − t2
distance covered during the last t second of its ascent is (a) t = (b) t =
(a) ut − (gt / 2)
2
(b) (u + gt) t 2 2
t1
(c) ut (d) gt 2 / 2 (c) t = t1 t2 (d) t =
t2
3. A person throws balls into air after every second. The
next ball is thrown when the velocity of the first ball is 10. A man in a balloon rising vertically with an acceleration of
zero. How high do the ball rise above his hand? 4.9 ms −2 releases a ball 2 s after the balloon is let go from the
ground. The greatest height above the ground reached by the
(a) 2 m (b) 5 m (c) 8 m (d) 10 m
ball is (Take, g = 9.8 ms −2)
4. A particle is thrown vertically upwards. Its velocity at (a) 14.7 m (b) 19.6 m
half of the height is10 ms −1 . Then, the maximum height (c) 9.8 m (d) 24.5 m
attained by it is (Take, g = 10 ms −2)
(a) 16 m (b) 10 m 11. A stone falls freely under gravity. The total distance covered by
(c) 20 m (d) 40 m it in the last second of its journey equals the distance covered
by it in first 3 s of its motion. The time for which stone remains
5. When a ball is thrown up vertically with velocity v 0 , it in air, is
reaches a maximum height of h. If one wishes to triple (a) 5s (b) 12s
the maximum height, then the ball should be thrown (c) 15s (d) 8 s
with velocity, 12. A body falls from a height h = 200 m. The ratio of distance
(a) 3 v0 (b) 3 v0 (c) 9 v0 (d) 3/ 2 v0
travelled in each 2 s, during t = 0 to t = 6 s of the journey is
6. A stone thrown upward with a speed u from the top of (a) 1 : 4 : 9 (b) 1 : 2 : 4
the tower reaches the ground with a speed 3u. The (c) 1 : 3 : 5 (d) 1 : 2 : 3
height of the tower is 13. A ball is released from height h and another from 2h. The ratio
(a) 3u / g2 2
(b) 4 u / g of time taken by the two balls to reach the ground is
(a) 1 : 2 (b) 2 :1
(c) 6 u2 / g (d) 9 u2 / g
(c) 2 : 1 (d) 1 : 2
7. A body thrown vertically up from the ground passes the 14. A particle is dropped under gravity from rest from a height
height of10. 2 m twice in an interval of10 s . What was h (g = 9.8 ms −2) and it travels a distance 9h / 25 in the last
its initial velocity? second, the height h is
(a) 52 ms−1 (b) 61 ms−1 (a) 100 m (b) 1225
. m
(c) 45 ms−1 (d) 26 ms−1 (c) 145 m (d) 167.5 m
8. A body is projected with a velocity u. It passes through a 15. A body dropped from the top of a tower covers a distance 7x in
the last second of its journey, where x is the distance covered
certain point above the ground after t1 second. The time in first second. How much time does it take to reach the
interval after which the body passes through the same ground?
point during the return journey is (a) 3s (b) 4 s (c) 5s (d) 6 s
u  u 
(a)  − t12  (b) 2  − t1  16. The displacement (in metre) of a particle moving along X-axis
g  g  is given by x = 18 t + 5 t 2. The average acceleration during the
u  u 2  interval t1 = 2s and t2 = 4 s is
(c)  − t1  (d)  2 − t1  (a)13 ms−2 (b)10 ms−2 (c) 27 ms−2 (d) 37 ms−2
g  g 
Motion in One Dimension 95

1 3 1 3
17. The displacement of a particle moving in a straight line is (a) v0 t + bt (b) v0 t + bt
described by the relation s = 6 + 12t − 2t 2. Here, s is in metre 6 3
1 1
and t is in second. The distance covered by particle in first 5 (c) v0 t + bt 2 (d) v0 t + bt 2
s is 3 2
(a) 20 m (b) 32 m 20. The acceleration a (in ms −2), of a particle is given by
(c) 24 m (d) 26 m a = 3 t 2 + 2 t + 2, where t is the time. If the particle starts out
18. The displacement of a particle moving in a straight line with a velocity v = 2 ms −1 at t = 0, then the velocity at the
depends on time as x = αt 3 + βt 2 + γt + δ. end of 2 s is
The ratio of initial acceleration to its initial velocity (a) 12 ms−1 (b) 14 ms−1
depends on (c) 16 ms−1 (d) 18 ms−1
(a) α and γ only (b) β and γ only
(c) α and β only (d) α only 21. A particle is moving such that s = t 3 − 6 t 2 + 18 t + 9, where s
19. The acceleration of a particle is increasing linearly with time is in metre and t is in second. The minimum velocity
t as bt. The particle starts from the origin with an initial attained by the particle is
velocity v 0 . The distance travelled by the particle in time t (a) 29 ms−1 (b) 5 ms−1
will be (c) 6 ms−1 (d) 12 ms−1

GRAPHICAL REPRESENTATION
OF MOTION
Motion of a point or body or a particle in all aspects can be
shown with the help of the graph, such as Different cases s-t graph Main features of graph
displacement-time graph and velocity-time graph, etc. Uniformly s Slope of s-t graph gradually
Displacement-time and velocity-time graphs for one accelerated motion 1
s = ut + at 2
goes on increasing.
dimensional motion are shown in tabular forms. with u ≠ 0 but 2
s = 0 at t = 0
t
1. Displacement-time graph
(i) Displacement-time graph gives instantaneous value of Uniformly retarded s θ is decreasing,
1 2
displacement at any instant. motion s = ut − at
2 so v is decreasing, a is
(ii) The slope of tangent drawn to the graph at any instant negative.
t
of time gives the instantaneous velocity at that instant. t0
(iii) The s-t graph cannot make sharp turns.
Different cases of displacement-time graph
2. Velocity-time graph
Different cases s-t graph Main features of graph (i) Velocity-time graph gives the instantaneous value of
At rest s Slope = v = 0 velocity at any instant.
(ii) The slope of tangent drawn on graph gives
instantaneous acceleration.
t
(iii) Area under v-t graph with time axis gives the value of
Uniform motion s Slope = constant, displacement covered in given time.
s =vt v = constant or a = 0 (iv) The v-t curve cannot take sharp turns.
t v
Uniformly s u = 0, i.e. slope of s-t graph
accelerated motion 1 at t = 0 should be zero. Area = Displacement
s = at 2 from t1 to t2
with u = 0, s = 0 at 2
t =0
t t1 t2
Fig. 3.10
96 OBJECTIVE Physics Vol. 1

Different cases in velocity-time graph 3. Acceleration-time graph


Different cases v-t graph Main features of graph The area of the a-t graph between time t1 to t 2 gives the
change in velocity.
Uniform motion v (i) θ = 0°
dv
(ii) v = constant As, a=
v = constant dt
(iii) Slope of v-t graph = a = 0
t ⇒ dv = a dt
v2 t2
Uniformly
accelerated motion
v Slope of v-t graph is constant,
so a = constant, u = 0,
⇒ ∫v 1
dv = ∫t 1
a dt
with u = 0 and v = at i.e. v = 0 at t = 0 t2
s = 0 at t = 0 v 2 − v1 = ∫t 1
a dt
t
v
Change in velocity = Ara of the a-t graph
Uniformly Positive constant acceleration
accelerated motion v = u + at because θ is constant and
u
with u ≠ 0 but < 90° but the initial velocity of a
s = 0 at t = 0 the particle is positive. Area = Change in velocity (v2 − v1)
t

Uniformly retarded v Slope of v-t graph = −a


motion u (retardation) t1 t2
v = u − at t
Fig. 3.13 Acceleration-time graph
t
t0
v
Example 3.36 A particle is moving along the X-axis and its
Non-uniformly Slope of v-t graph increases
position-time graph is shown. Determine the sign of
accelerated motion with time. θ is increasing, so
acceleration.
acceleration is increasing.
E
t
D
Non-uniformly v θ is decreasing, so acceleration
decelerating motion is decreasing. s B
A C

t
O t1 t2 t3 t4 t5
Note
(i) Slope of s-t or v-t graphs can never be infinite at any point because Sol. By observing the s-t graph, we can determine the sign of
infinite slope of s-t graphs means infinite velocity. Similarly, infinite acceleration. Recall, if the graph is concave upwards, the
slope of v-t graph means infinite acceleration. Hence, the following slope is increasing; if it is concave downward, the slope is
graphs are not possible. decreasing; and if the graph is straight line, the slope is
s v constant.
v is constant v is increasing v is decreasing

s s s
t t
Fig. 3.11
(ii) At one time, two values of velocity or displacement are not possible. O t O t O t
Hence, the following graphs are not acceptable.
OA : Slope is increasing, v is increasing and a is positive.
s v
AB : Slope is constant, v is constant and a = 0.
s1 v1
BC : Slope is decreasing, v is decreasing and a is negative.
s2 v2
CD : Slope is increasing, v is increasing and a is positive.
t
t0
t t0 DE : Slope is constant, v is constant and a = 0.
Fig. 3.12
Motion in One Dimension 97

Example 3.37 With the help of the given velocity-time graph, α + β αβt
find the or v max   = t or v max =
 αβ  α +β
(i) displacement in first three seconds and
(ii) acceleration (ii) Total distance = Displacement = Area under v-t graph
1
= × t × v max
+v ms−1 A 2
30
1 αβt
20 = ×t ×
10
2 α +β
B
1  αβt 2 
O
t
0 1 2 3 4 5 6 or Distance =  
10 2 α + β
− v ms−1 20
30 Example 3.39 Velocity-time graph of a particle moving in a
C
straight line is shown in figure.
Sol. (i) Displacement in first three seconds = Area of v (ms−1)
triangle OAB
1 1 C
= (OB ) × (OA) = (3) × (30) = + 45 m 20
2 2 A B
10
(ii) Acceleration = Slope of v-t graph
As, v-t graph is a straight line. So, consider the slope of line D
t (s)
O 2 4 6 8
AB.
y − y1 0 − 30
∴ Slope of line AB = 2 = = − 10 ms −2 Plot the corresponding displacement-time graph of the particle,
x 2 − x1 3 if at time t = 0, displacement s = 0.
So, the acceleration is negative. Sol. Displacement = Area under velocity-time graph
Example 3.38 A car accelerates from rest at a constant rate α Hence, sOA =
1
× 2 × 10 = 10 m
for some time, after which it decelerates at a constant rate β 2
to come to rest. If the total time elapsed is t second evaluate sAB = 2 × 10 = 20 m
(i) the maximum velocity reached and (ii) the total distance or sOAB = 10 + 20 = 30 m
travelled.
1
Sol. (i) Let the car accelerates for time t1 and decelerates for sBC = × 2(10 + 20) = 30 m
2
time t 2. Then, t = t1 + t 2 …(i)
or sOABC = 30 + 30 = 60 m
and corresponding velocity-time graph will be as shown
1
in figure.
v
and sCD = × 2 × 20 = 20 m
2
or sOABCD = 60 + 20 = 80 m
vmax A
Between 0 s to 2 s and 4 s to 6 s, motion is accelerated, hence
displacement-time graph is a parabola. Between 2 s to 4 s,
motion is uniform, so displacement-time graph will be a straight
B line. Between 6 s to 8 s, motion is decelerated, hence
t
O t1 t2
displacement-time graph is again a parabola but inverted in
From the graph, shape.
v max At the end of 8 s velocity is zero, therefore slope of
α = slope of line OA =
t1 displacement-time graph should be zero. The corresponding
graph is shown in figure.
v max
or t1 = …(ii)
α s (m)
v max
and β = − slope of line AB = 80
t2
v max 60
or t2 = …(iii)
β
From Eqs. (i), (ii) and (iii), we get 30
v max v max
+ =t 10
α β 2 4 6 8
t (s)
98 OBJECTIVE Physics Vol. 1

Example 3.40 A rocket is fired vertically upwards with a net Example 3.41 Acceleration-time graph of a particle moving in
acceleration of 4 ms −2 and initial velocity zero. After 5 s, its a straight line is shown in figure. Velocity of particle at time
fuel is finished and it decelerates with g. At the highest t = 0 is 2 ms −1. Find velocity at the end of fourth second.
point, its velocity becomes zero. Then, it accelerates a (ms−2)
downwards with acceleration g and return back to ground.
Plot velocity-time and displacement-time graphs for the 4
complete journey. (Take, g = 10 ms −2 )
Sol. In the graphs, v A = atOA = (4) (5) = 20 ms−1
t (s)
vB = 0 = v A − gt AB 0 2 4
v 20 Sol. According to acceleration time-graph, dv = a dt
⇒ t AB = A = = 2s
g 10 or change in velocity = area under a-t graph
∴ tOAB = (5 + 2) s = 7s Hence,
1
v f − vi = (4) (4) = 8 ms −1
2
v (ms−1) ∴ v f = vi + 8 = (2 + 8) ms −1 = 10 ms −1

Example 3.42 The acceleration versus time graph of a particle


moving along a straight line is shown in the figure. Draw the
A respective velocity-time graph.
20
B C
O 5 t (s) a
7 10.7
(m/s2)
2
0 2 4 6 t (s)
Now, sOAB = area under v-t graph between 0 to 7 s −2
1 −4
= (7) (20) = 70 m
2 Assume at t = 0, v = 0.

s (m) Sol. From t = 0 to t = 2 s, a = + 2 m/s 2 ⇒ v = at = 2t

B or v-t graph is a straight line passing through origin with


70 slope 2 m/s 2.
50 A At the end of 2 s, v = 2 × 2 = 4 m/s
From t = 2 to 4 s, a = 0.
Hence, v = 4 m/s will remain constant.
C
t (s) From t = 4 to 6 s, a = − 4 m/s 2.
O 5 7 10.7
Hence, v = u − at = 4 − 4t
1 2 v = 0 at t = 1 s or at 5 s from origin.
Now, | sOAB | = | sBC | = gtBC At the end of 6 s (or t = 2 s) v = − 4 m/s.
2
1 Corresponding v-t graph as shown below.
∴ 70 = 2
(10) tBC
2 v (m/s)
⇒ tBC = 14 = 3.7 s 4
∴ tOABC = 7 + 3.7 = 10.7 s
0 6 t (s)
Also, sOA = area under v-t graph between OA 2 4 5
1
= (5) (20) = 50 m −4
2
CHECK POINT 3.6
1. Which of the following graph represents the uniform 6. The variation of velocity of a particle with time moving
motion? along a straight line is illustrated in the adjoining figure.
The distance travelled by the particle in 4 s is

Velocity (ms–1)
(a) (b) 30

s s 20

t t 10

1 2 3 4
(c) (d) None of these Time (s)
s (a) 60 m (b) 55 m (c) 25 m (d) 30 m
t 7. A lift is going up. The

Velocity (ms−1)
2. A particle shows distance-time curve as given in this figure. variation in the speed of
The maximum instantaneous velocity of the particle is the lift is as given in the 3.6
around the point graph. What is the height
to which the lift takes the
passengers?
D (a) 36
. m
Distance (s)

(b) 28.8 m 2 Time (s) 10 12


C (c) 36.0 m
(d) Cannot be calculated from the above graph
A B 8. The velocity-time graph of a body moving in a straight line
is shown in the figure. The displacement and distance
Time (t) travelled by the body in 6 s are respectively
(a) A (b) B (c) C (d) D
v (ms–1)
3. From the displacement-time graph, find out the velocity of 5
a moving body. 4
3
2
1
3 t (s)
Time (s)

0 4 5 6
1 2
−1
−2
−3
30°
O (a) 8 m , 16 m (b) 16 m , 32 m
Displacement (m)
(c) 16 m , 16 m (d) 8 m , 18 m
1 1 9. The x- t equation is given as x = 2t + 1. The corresponding
(a) ms−1 (b) 3 ms−1 (c) 3 ms−1 (d) ms−1
3 3 v- t graph is
4. The distance-time graph of a particle at time t makes angle (a) a straight line passing through origin
45° with the time axis. After one second, it makes angle 60° (b) a straight line not passing through origin
with the time axis. What is the average acceleration of the (c) a parabola
particle? (d) None of the above
(a) 3 −1 (b) 3+1 (c) 3 (d) 1
10. Which of the following graphs correctly represents
5. The v- t graph of a moving object is shown in the figure. The velocity-time relationship for a particle released from rest to
maximum acceleration is fall freely under gravity?
v v
80
Velocity (cms−1)

(a) (b)
60

40 t t
v v
20

10 20 30 40 50 60 70 80 (c) (d)
Time (s)
t t
(a) 1 cms−2 (b) 2 cms−2 (c) 3 cms−2 (d) 6 cms−2
100 OBJECTIVE Physics Vol. 1

11. A particle projected vertically upwards returns to the 15. If the velocity v of a particle moving 20
ground in time T . Which graph represents the correct along a straight line decreases

v (in ms–1)
variation of velocity (v) against time ()
t?
linearly with its displacement s from
v v
20 ms −1 to a value approaching zero
at s = 30 m, then acceleration of the
(a) (b) particle at v = 10 ms −1 is
0 s (in m) 30
2 2
(a) ms−2 (b) − ms−2
O T/2 O T/2 T 3 3
T
20 20
(c) ms−2 (d) − ms−2
v v 3 3
16. v 2 versus s graph of a particle moving in a v2
(c) (d) straight line is shown in the figure below.
From the graph some conclusions are
O drawn. State which amongst the
O T/2 T T/2 T following statement(s) is wrong?
12. The velocity-time graph is shown in the figure, for a (a) The given graph shows a uniformly s
particle. The acceleration of particle is accelerated motion.
v (ms−1) (b) Initial velocity of particle is zero.
(c) Corresponding s-t graph will be a parabola.
15
(d) None of the above
10 17. A graph between the square of the velocity of a particle and
the distance s moved by the particle is shown in the figure
below. The acceleration of the particle is
5
t (s) v2 (ms–2)
1 2 3 4
25
−2 −2 −2 −2
(a) 225
. ms (b) 5 ms (c) − 5 ms (d) − 3 ms
13. The v- t plot of a moving object is shown in the figure. The 9
average velocity of the object during the first 10 s is s (m)
0 2
−2 −2
(a) − 8 ms (b) − 4 ms (c) − 16 ms−2 (d) None of these
5
18. A particle starts from rest at t = 0 and undergoes an
Velocity (ms–1)

Time (s) acceleration a in ms −2 with time t in second which is as


0 shown?
5 10 a
3

–5
0
t
(a) zero . ms−1
(b) 25 (c) 5 ms−1 (d) 2 ms−1 1 2 3 4

14. Which of the following graphs cannot possibly represent –3


one dimensional motion of a particle?
Which one of the following plot represents velocity v
Total distance

(in ms −1) versus time t (in s)?


Position
covered

6 6
v4 v4
Time Time (a)
I II
2 (b) 2

1 2 3 4 t 1 2 3 4 t
Velocity
Speed 6
6
v4
Time v4
Time
(c) 2 (d)
2
IV
1 2 3 4 t
III –1 1 2 3 4 t
(a) I and II (b) II and III (c) II and IV (d) All of these –2
Motion in One Dimension 101

Case II If both objects A and B move along parallel straight


RELATIVE VELOCITY lines in the opposite direction, then relative velocity
The term relative is frequently used for comparison of of B w.r.t. A is given as
displacement, velocity and acceleration of the two objects. v BA = v B − (− v A ) = v B + v A
The time rate of change of relative position of one object
and the relative velocity of A w.r.t. B is given by
with respect to another is called relative velocity.
v AB = v A − (− v B ) = v A + v B
Let two objects A and B are moving along the + ve
direction on X-axis. At time t, their displacement from the Case III If v A < v B , v A − v B is negative.
origin be x A and x B . Then, x − x 0 < 0
O
vA A vB
B ⇒ x < x0
xA
where, x 0 = initial displacement of object A w.r.t. B
xB
and x = displacement of object A w.r.t. B
Fig. 3.14
at time t.
dxA d xB
∴ Their velocities are v A = and v B = i.e. (x − x 0 ) is negative. It means the separation
dt dt between the two objects will go on decreasing and
The displacement of B relative to A, two objects will meet and object B will overtake
x BA = x B − x A object A at this time. In this case, relative velocity of
A w.r.t. B, i.e.
Rate of change of relative displacement w.r.t. time is
v AB = v A − v B
d (x BA ) d dx dx dx
= ( xB − x A ) ⇒ BA = B − A = − v BA
dt dt dt dt dt
∴ v BA = v B − v A B
A
Similarly, relative acceleration of A with respect to B is
a AB = a A − aB
x (m)
If it is a one dimensional motion, we can treat the vectors
as scalars just by assigning the positive sign to one
direction and negative to the other. So, in case of a one
dimensional motion, the above equations can be written as t (s)

v AB = v A − v B Fig. 3.16
and a AB = a A − aB Case IV If v A > v B , v A − v B is positive.
Further, we can see that v AB = − vBA or aBA = − a AB Then, x − x 0 > 0, i.e. (x − x 0 ) is positive.
It means the separation between the two objects will
Different cases of relative velocity go on increasing with time, i.e. the separation
Case I When the two objects move with equal velocities, (x − x 0 ) between them will increase by an amount
i.e. v A = v B or v B − v A = 0. It means, the two (v A − v B ) after each unit of time.
objects stay at constant distance apart during the Therefore, their position-time graphs will open out
whole journey. gradually as shown below.
In this case, the position-time graph of two objects
are parallel straight lines.
y B
t
jec
Ob XA (0)
x (m) tA
c x (m)
Obje XB (0)

x O
t (s) t (s)
Fig. 3.15 Fig. 3.17
102 OBJECTIVE Physics Vol. 1

Example 3.43 Seeta is moving due east with a velocity of | aBA| = (4)2 + (2)2
1ms −1 and Geeta is moving due west with a velocity of
2 ms −1. What is the velocity of Seeta with respect to Geeta? = 2 5 m / s2
Sol. It is a one dimensional motion. So, let us choose the east  4
direction as positive and the west as negative. and α = tan−1   = tan−1 (2)
 2
Now, given that
Thus, aBA is 2 5 m/s2 at an angle of α = tan−1 (2) from west
v S = velocity of Seeta = 1 ms −1
towards north.
and vG = velocity of Geeta = − 2 ms −1
Example 3.46 A police van moving on a highway with a
Thus, v SG = velocity of Seeta with respect to Geeta speed of 30 kmh −1 fires a bullet at a thief car which is
= v S − vG = 1 − (−2) = 3 ms −1 speeding away in the same direction with a speed of
Hence, velocity of Seeta with respect to Geeta is 3 m/s due 190 kmh −1. If the muzzle speed of the bullet is 150 ms −1,
east. find speed of the bullet with respect to the thief’s car.
Sol. Let v b is velocity of bullet, v p is velocity of police van and
Example 3.44 A man A moves due east with velocity 6 ms −1 and
another man B moves in N-30°E with 6 ms −1 . Find the vt is velocity of thief’s car.
velocity of B w.r.t. A. Then, speed of the bullet with respect to the thief’s car,
Sol. Given, v A = 6$i, v bp = v b − v p
18
v B = vB cos 60° $i + vB sin 60° $j v b = v bp + v p = 150 × kmh−1 + 30 kmh−1 = 570 kmh−1
5
 1  3 v bt = v b − vt = 570 kmh−1 − 190 kmh−1 = 380 kmh−1
= 6   i$ + 6   $j = 3 $i + 3 3 $j
 2  2  Example 3.47 Delhi is at a distance of 200 km from Ambala.
To find the velocity, Car A set out from Ambala at a speed of 30 kmh −1 and car B
v BA = v B − v A = (3 $i + 3 3 $j ) − 6 $i set out at the same time from Delhi at a speed of 20 kmh −1.
When will they meet each other? What is the distance of the
= − 3 i$ + 3 3 $j meeting point from Ambala?
Sol. Relative velocity, v AB = v A − vB
| v BA| = (− 3)2 + (3 3 )2
−1
= 30 − (−20) = 50 kmh−1
= 9 + 27 = 36 = 6 ms
Here, $i is − ve and $j is + ve. So, second quadrant is possible. 30 kmh−1 20 kmh−1

Direction,
coefficient of j$ 3 3 200 km
tan α = = =− 3 Ambala Delhi
coefficient of i$ −3
They will meet after time
⇒ α = − 60°
s 200
t= = =4h
Example 3.45 Car A has an acceleration of 2 m/s 2 due east v AB 50
and car B, 4 m/s 2 due north. What is the acceleration of car
Distance from Ambala, where they will meet,
B with respect to car A?
x = 30 × 4 = 120 km
Sol. It is a two dimensional motion, therefore
aBA = acceleration of car B with respect to car A
= aB − aA
Examples of relative motion
1. Relative velocity of rain
N
aBA aB = 4 m/s 2 Consider a man walking east with velocity v m represented
by OA. Let the rain be falling vertically downwards with
W E velocity v r , represented by OB. To find the relative
α
velocity of rain w.r.t. man (i.e. v rm ) being the man at rest
by imposing a velocity − v m on man and apply this
− aA = 2 m/s 2 S
velocity on rain also.
Here, aB = acceleration of car B = 4 m/s2 (due north) Now, the relative velocity of rain w.r.t. man will be the
resultant velocity of v r (= OB ) and − v m (= OA), which will
and a A = acceleration of car A = 2 m/s2 (due east) be represented by diagonal OC of rectangle OACB.
Motion in One Dimension 103

∴ v rm = v r2 + v m2 + 2v r v m cos 90 ° = v r2 + v m2 2. Crossing the river


To cross the river over shortest distance, i.e. to cross the
− vm O vm river straight, the man should swim upstream making an
A A angle θ with OB such that, OB gives the direction of
vrm q resultant velocity (v mR ) of velocity of swimmer and
vr velocity of river water as shown in figure.
C B Let us consider
A vR B
Fig. 3.18 Relative velocity of rain

If θ is the angle which v rm makes with the vertical vm vmR


direction, then θ
BC v v 
tan θ = = m or θ = tan −1  m  O
OB vr  vr 
Fig. 3.19 Crossing the river
Here, angle θ is from vertical towards west and is written
as θ, west of vertical. AB = v R (velocity of river water),
OA = v m (velocity of man in still river water)
Note In the above problem, if the man wants to protect himself from
the rain, he should hold his umbrella in the direction of relative and OB = v mR (relative velocity of man w.r.t. river).
velocity of rain w.r.t. man, i.e. the umbrella should be held
i.e. v mR = v m2 − v R2
making an angle θ from west of vertical.
vR
Example 3.48 To a man walking at the rate of 3 km/h, the In ∆OAB, sin θ =
vm
rain appears to fall vertically. When he increases his speed to
6 km/h, it appears to meet him at an angle of 45° with where, θ is the angle made by man with shortest distance OB,
vertical. Find the speed of rain. v vR
tan θ = R =
Sol. Let $i and $j be the unit vectors in horizontal and vertical v mR v2 − v2 m R
directions, respectively.
(i) Time taken to cross the river If d be the width of
^
Vertical ( j ) the river, then time taken cross to the river given by
d d
t1 = =
v mR v m2 − v R2
^
Horizontal ( i ) (ii) To cross the river in possible shortest time The
man should go along OA. Now, the swimmer will be
Let velocity of rain, going along OB, which is the direction of resultant
vr = a$i + b$j …(i) velocity of v m and v R .
A vR B
Then, speed of rain will be
x
| vr | = a + b 2 2
…(ii)
d vm −vmR
In the first case, v m = velocity of man = 3$i θ
∴ v = v − v = (a − 3)i$ + b$j
rm r Downstream
Upstream O
It seems to be in vertical direction. Hence,
Fig. 3.20 Crossing the river in possible shortest time
a − 3 = 0 or a = 3
AB v R
In the second case vrm = 6i$ In∆OAB, tan θ = = and v mR = v m2 + v R2
OA v m
∴ v = (a − 6)i$ + b$j = − 3i$ + b$j
rm
Time of crossing the river,
This seems to be at 45° with vertical. Hence, |b | = 3
Therefore, from Eq. (ii), speed of rain is d OB x2 + d2
t= = ⇒ t=
| vr | = (3)2 + (3)2 = 3 2 km/h vm v mR v m2 + v R2
104 OBJECTIVE Physics Vol. 1

The boat will be heading the point B instead of Drift in this case will be
v x dv x = vr t
point A. If AB = x, then tan θ = R = ⇒ x = R
vm d vm ∴ 120 = 10 vr K (ii)
In this case, the man will reach the opposite bank at B
a distance AB downstream.
v br w
Drift
A
It is defined as the displacement of man in the direction of For minimum time
river flow as shown below.
Shortest path is taken when v b is along AB. In this case,
x
v b = v br
2
− vr2
y
w w
vMR Now, 12. 5 = = K (iii)
d vM x vb v br − vr2
2

Solving these three equations, we get


vR v br = 20 m/min, vr = 12 m/min and w = 200 m
Fig. 3.21 Example 3.50 A man wants to reach point B on the opposite
bank of a river flowing at a speed u as shown in figure.
It is simply the displacement along X-axis. During the period, What minimum speed relative to water should the man have,
the man crosses the river. (v MR cos θ + v R ) is the components so that he can reach point B ? In which direction should he
of velocity of man in the direction of river flow and this swim ?
component of velocity is responsible for drift along the river
B
flow. If drift is x, then
d
x = (v MR cos θ + v R ) ×
v MR sin θ 45°
u

Note If vR > v MR, then it is not possible to have zero drift. In this case, A
the minimum drift corresponding to shortest possible path is
non-zero and the condition for minimum drift can be proved to be Sol. Let v be the speed of boatman in still water.
v v
cosθ = − MR or sin φ = MR for minimum but non-zero drift. B
vR vR
vb
v y
θ 45°
vR
vMR φ x
θ A u
Resultant of v and u should be along AB. Components of v b
Fig. 3.22 (absolute velocity of boatman) along x and y-directions are
v x = u − v sin θ and v y = v cos θ
Example 3.49 A man crosses a river in a boat. If he cross the vy
Further, tan 45° =
river in minimum time, he takes 10 min with a drift 120 m. vx
If he crosses the river taking shortest path, he takes 12.5 min,
v cos θ
find or 1=
(i) width of the river, u − v sin θ
(ii) velocity of the boat with respect to water u u
∴ v= =
(iii) and speed of the current. sin θ + cos θ 2 sin (θ + 45°)
Sol. Let vr = velocity of river, v is minimum, at
v br = velocity of boat in still water θ + 45° = 90°
and w = width of river. or θ = 45°
w u
Given, t min = 10 min or = 10 K (i) and v min =
v br 2
Motion in One Dimension 105

Example 3.51 A man can row a boat with 4 km/h in still Suppose boat starts at an angle θ from the normal direction
water. If he is crossing a river, where the current is 2 km/h. up stream as shown.
(i) In what direction will his boat be headed, if he wants to reach
B
a point on the other bank, directly opposite to starting point?
Drift = x C
(ii) If width of the river is 4 km, how long will the man take to
cross the river, with the condition in part (i)? y
(iii) In what direction should he head the boat, if he wants to d u
x
cross the river in shortest time and what is this minimum time? v v cos θ
θ
(iv) How long will it take him to row 2 km up the stream and
then back to his starting point? u − v sin θ
A
Sol. (i) Given, v br = 4 km/h and vr = 2 km/h
v  Component of velocity of boat along the river,
∴ θ = sin−1  r  v x = u − v sin θ
 v br 
and velocity perpendicular to the river,
 2 v y = v cos θ
= sin−1  
 4 Time taken to cross the river is
 1 d d
= sin−1   = 30° t= =
 2 v y v cos θ
Hence, to reach the point directly opposite to starting d
Drift x = (v x ) t = (u − v sin θ )
point he should head the boat at an angle of 30° with v cos θ
AB or 90° + 30° = 120° with the river flow. ud
(ii) Time taken by the boatman to cross the river, = sec θ − d tan θ
v
w = width of river = 4 km dx
The drift x is minimum when =0
v br = 4 km / h dθ
and θ = 30°  ud 
  (sec θ ⋅ tan θ ) − d sec θ = 0
2
or
4 2 v 
∴ t= = h
4 cos 30° 3 u v
sin θ = 1 ⇒ sin θ =
(iii) For shortest time θ = 0° v u
w 4 So, for minimum drift, the boat must move at an angle
and t min = = = 1h v  1
v br cos 0° 4 θ = sin−1   = sin−1  
 u n 
Hence, he should head his boat perpendicular to the from normal direction.
river current for crossing the river in shortest time and
this shortest time is 1 h.
(iv) t = tCD + tDC
3. Minimum distance between
two bodies in motion
When two bodies are in motion, the questions like, the
v br − vr v br + vr minimum distance between them or the time when one body
D C D C
overtakes the other can be solved easily by the principle of
relative motion. In these type of problems one body is
assumed to be at rest and the relative motion of the other
CD DC body is considered.
or t= +
v br − vr v br + vr By assuming so two body problem is converted into one
2 2 1 4 body problem and the solution becomes easy.
= + =1+ = h
4−2 4+2 3 3 Following example will illustrate the statement.
Example 3.52 A boat moves relative to water with a velocity v Example 3.53 Car A and car B start moving simultaneously in
is n times less than the river flow u. At what angle to the the same direction along the line joining them. Car A moves
stream direction must the boat move to minimise drifting? with a constant acceleration a = 4 ms −2 , while car B moves
Sol. In this problem, one thing should be carefully noted that the with a constant velocity v = 1 ms −1. At time t = 0, car A is
velocity of boat is less than the river flow velocity. Hence, 10 m behind car B. Find the time when car A overtakes
boat cannot reach the point directly opposite to its starting car B.
point, i.e. drift can never be zero.
106 OBJECTIVE Physics Vol. 1

Sol. Given, uA = 0, uB = 1 ms −1, a A = 4 ms −2 and a B = 0 2 ms−2 10 ms−1


20 ms−1
Assuming car B to be at rest, we get +ve
−1 B
uAB = uA − uB = 0 − 1 = − 1 ms
a AB = a A − a B = 4 − 0 = 4 ms −2 10 ms− 2
Here, L → Lift B → Ball
Now, the problem can be assumed in simplified form as
follows Solving this equation, we get
5
a = 4 ms −2 v = 1 ms −1 t=0 and t = s
3
5
10 m ∴ Ball will again meet the lift after s.
A B 3
2
+ve 5 1  5
(ii) At this instant, sL = sB = 10 × + ×2×  
3 2  3
Substituting the proper values in equation
175
1 = m = 19.4 m
s = ut + at 2, we get 9
2
(iii) For the ball u ↑ ↓a . Therefore, we will first find t 0, the
1
10 = − t + (4)(t 2) time when its velocity becomes zero.
2
u 20
or 2t 2 − t − 10 = 0 t0 = = =2s
a 10
uAB = − 1 ms−1 aAB = 4 ms− 2  5 
As t  = s  < t 0, distance and displacement are equal
 3 
10 m
A B or d = 19.4 m
At rest Example 3.55 Two ships A and B are 10 km apart on a
1 ± 1 + 80 1 ± 81 1 ± 9 line running south to north. Ship A farther north is
or t= = = streaming west at 20 kmh −1 and ship B is streaming north at
4 4 4 20 kmh −1. What is their distance of closest approach and
or t = 2.5 s and − 2 s how long do they take to reach it?
Ignoring the negative value, the desired time is 2.5 s. Sol. Ships A and B are moving with same speed 20 kmh −1 in the
Note The above problem can also be solved without using the concept directions as shown in figure. It is a two dimensional, two
of relative motion as follows body problem with zero acceleration. Let us find v BA.
At the time, when A overtakes B, s A = sB + 10 vA N
A
1
∴ × 4 × t 2 = 1 × t + 10 E
2 vB
or 2t 2 − t − 10 = 0 B
which on solving gives t = 2.5 s and –2 s, the same as we found AB = 10 km
above.
As per my opinion, this approach (by taking absolute values) is
more suitable in case of two body problem in one dimensional vBA = vB − v A
motion. Let us see one more example in support of it.
Here, | v BA| = (20)2 + (20)2 = 20 2 kmh −1
Example 3.54 An open lift is moving upwards with velocity
10ms −1. It has an upward acceleration of 2 ms −2 . A ball is i.e. vBA is 20 2 kmh −1 at an angle of 45° from east towards
projected upwards with velocity 20 ms −1 relative to ground. north. Thus, the given problem can be simplified as
Find vB = 20 kmh−1 vBA = 20√2 kmh−1
(i) time when ball again meets the lift,
(ii) displacement of lift and ball at that instant
(iii) and distance travelled by the ball upto that instant. 45°
(Take, g = 10 ms −2) − vA = 20 kmh−1
Sol. (i) At the time, when ball again meets the lift, sL = sB A is at rest and B is moving with vBA in the direction as
1 1 shown in figure. Therefore, the minimum distance between
∴ 10 t + × 2 × t 2 = 20 t − × 10 t 2 the two is
2 2
Motion in One Dimension 107

 1  (ii) Along line OA


smin = AC = AB sin 45° = 10   km = 5 2 km
 2 10

A
10 cos 37° A
C
vBA
45° 37° v cos θ v
B θ

Time taken by the plane to move from O to A,


and the desired time is d d
t= =
BC 5 2 10 cos 37° + v cos θ 8 + v cos θ
t= = (Q BC = AC = 5 2 km)
| v BA| 20 2 Example 3.57 An aircraft flies at 400 km/h in still air. A
1 wind of 200 2 km/h is blowing from the south. The pilot
= h = 15 min wishes to travel from A to a point B north-east of A. Find
4
the direction he must steer and time of his journey, if
4. Aircraft wind problems AB = 1000 km.
This is similar to river boat problem. The only difference Sol. Given, v w = 200 2 km/h, v aw = 400 km/h and v a should
is that v mR is replaced by v aw (velocity of aircraft with be along AB or in north-east direction. Thus, the direction of
respect to wind or velocity of aircraft in still air), v R is v aw should be such as the resultant of v w and v aw is along AB
replaced by v w (velocity of wind) and v m is replaced by or in north-east direction.
v a (absolute velocity of aircraft). Further, v a = v aw + v w . N
The following example will illustrate the theory. B

Example 3.56 An aeroplane has to go from a point O to 45° v = 200√2 km/h


va w
another point A, at distance d due 37° east of north. A wind
is blowing due north at a speed of 10 ms −1. Find the air C
speed of the plane is v, (i) the direction in which the pilot 45° α
A vaw = 400 km/h
should head the plane to reach the point A (ii) and the time E
taken by the plane to go from O to A.
N Let v aw makes an angle α with AB as shown in figure.
A Applying sine law in triangle ABC, we get
10 m/s
AC BC  BC   200 2  1 1
= or sin α =   sin 45° =   =
37° d sin 45° sin α  AC   400  2 2
O
E ∴ α = 30°
Therefore, the pilot should steer in a direction at an angle of
Sol. (i) If a particle moves in a straight line, the velocity of
(45° + α ) or 75° from north towards east.
particle perpendicular to straight line should be zero. If
an aeroplane moves along OA, the pilot should head the Further,
plane towards right of line OA. | v a| 400
=
10 m/s sin (180° − 45° − 30° ) sin 45°
A sin 105°
or | v a| = × (400) km /h
sin 45°
10 sin 37° 37° v
θ  cos15° 
=  (400) km/h
O  sin 45° 
v sin θ
 0.9659
=  (400) km/h
Perpendicular to line OA,  0.707 
v sin θ = 10 sin 37° = 6
= 546.47 km/h
6  6
sinθ = ⇒ θ = sin−1  ∴ The time of journey from A to B is
v v 
AB 1000
The pilot should head the plane at an angle t= = h
| v a | 546.47
 6
sin−1  east of line OA t = 1.83 h
v 
OBJECTIVE Physics Vol. 1

CHECK POINT 3.7


1. A100 m long train crosses a man travelling at 5 kmh −1 , in 8. The speed of boat is 5 kmh −1 in still water. It crosses a river
opposite direction in 7.2 s, then the velocity of train is of width1 km along the shortest possible path in15 min.
(a) 40 ms−1 (b) 25 ms−1 (c) 20 ms−1 (d) 45ms−1 Then, velocity of river will be
(a) 4.5 kmh−1 (b) 4 kmh−1
2. Two parallel rail tracks run north-south. Train A moves −1
north with a speed of 54 kmh −1 and train B moves south (c) 1.5 kmh (d) 1 kmh−1
with a speed of 90 kmh −1 . Find the relative velocity of B
9. A ship X moving due north with speed v observes that
w.r.t. A.
−1 −1 another ship Y is moving due west with same speed v. The
(a) 40 ms N to S (b) 10 ms N to S
actual velocity of Y is
(c) 10 ms−1 S to N (d) 40 ms−1 S to N
(a) 2v towards south-west (b) 2v towards north-west
3. Two bodies are held separated by 9.8 m vertically one above (c) 2v towards south-east (d) v towards north-east
the other. They are released simultaneously to fall freely
10. A river is flowing from west to east at a speed of 8 m per
under gravity. After 2 s, the relative distance between them is
min. A man on the south bank of the river, capable of
(a) 4.9 m (b) 19.6 m (c) 9.8 m (d) 39.2 m
swimming at 20 m/ min in still water, wants to swim across
4. A particle (A) moves due north at 3 kmh −1 and another the river in the shortest time. He should swim in a
particle (B) moves due west at 4 kmh −1 . The relative velocity direction,
of A with respect to B is (Take, tan37° = 3 / 4) (a) due north (b) 30° east of north
(c) 30° west of north (d) 60° east of north
(a) 5 kmh−1 , 37° north of east
11. The rowing speed of a man relative to water is 5 kmh −1 and
(b) 5 kmh−1 , 37° east of north the speed of water flow is 3 kmh −1 . At what angle to the
−1
(c) 5 2 kmh , 53° east of north river flow should he head, if he wants to reach a point on
the other bank, directly opposite to starting point?
(d) 5 2 kmh −1 , 53° north of east
(a) 127° (b) 143° (c) 120° (d) 150°
5. A man standing on a road has to hold his umbrella at 30° 12. A man wants to reach point B on the opposite bank of a
with the vertical to keep the rain away. He throws the
river flowing at a speed as shown in figure. With what
umbrella and starts running at10 kmh −1 . He finds that minimum speed and in which direction should the man
raindrops are hitting his head vertically. swim relative to water, so that he can reach point B ?
What is the speed of rain with respect to ground?
(a) 10 3 kmh−1 (b) 20 kmh−1 B
20 10
(c) kmh−1 (d) kmh−1 u
3 3 45°
6. A stationary man observes that the rain is falling vertically
downward. When he starts running with a velocity of A
12 kmh −1 , he observes that the rain is falling at an angle (a) u, 45° north-west (b) u, 45° north-east
60° with the vertical. The actual velocity of rain is u u
(c) , 45° north-west (d) , 45° north-east
(a) 12 3 kmh−1 (b) 6 3 kmh−1 2 2
(c) 4 3 kmh−1 (d) 2 3 kmh−1 13. Two trains, each 50 m long moving parallel towards each
7. A boy is running on the plane road with velocity v with a other at speeds 10 ms −1 and 15 ms −1 respectively, at what
long hollow tube in his hand. The water is falling vertically time will they pass each other?
downwards with velocity u. At what angle to the vertical, he (a) 8 s (b) 4 s
(c) 2 s (d) 6 s
must inclined the tube, so that the water drops enter it
without touching its sides? 14. A ball is dropped from the top of a building100 m high. At
the same instant, another ball is thrown upwards with a
(a) tan−1  v (b) sin−1  v
    velocity of 40 ms −1 from the bottom of the building. The
 u  u
two balls will meet after
(c) tan−1   (d) cos−1  
u v (a) 5s (b) 25
. s
 v  u (c) 2s (d) 3s
Chapter Exercises
(A) Taking it together
Assorted questions of the chapter for advanced level practice

1 A boy walks to his school at a distance of 6 km with 25


(a) 5 kmh −1 (b) kmh −1
−1 4
constant speed of 2.5 kmh and walks back with a
constant speed of 4 kmh −1. His average speed for (c)
30
kmh −1 (d)
45
kmh −1
round trip expressed (in kmh −1), is 5 8
(a) 24/13 (b) 40/13 (c) 3 (d) 1/2 7 A car moving with a velocity of 10 ms −1 can be
2 The distance travelled by a particle starting from rest stopped by the application of a constant force F in a
4 distance of 20 m. If the velocity of the car is
and moving with an acceleration ms −2 , in the third 30 ms −1, it can be stopped by this force in
3
second is 20
(a) m (b) 20 m (c) 60 m (d) 180 m
10 19 3
(a) m (b) m (c) 6 m (d) 4 m
3 3 8 A body A starts from rest with an acceleration a 1.
3 A vehicle travels half the distance l with speed v 1 After 2 seconds, another body B starts from rest with
and the other half with speed v 2 , then its average an acceleration a 2 . If they travel equal distance in
speed is [NCERT Exemplar] the 5th second, after the start of A, then the ratio
v1 + v 2 2v1 + v 2 2v1v 2 l (v1 + v 2 ) a1 : a 2 is equal to
(a) (b) (c) (d)
2 v1 + v 2 v1 + v 2 v1v 2 (a) 5 : 9 (b) 5 : 7 (c) 9 : 5 (d) 9 : 7

4 Which of the following speed-time (v -t ) graphs is 9 The displacement-time graph of a moving particle is
shown below. The instantaneous velocity of the
physically not possible?
particle is negative at the point
v v
Displacement

D
(a) (b)
C E F

t t
Time
v (a) E (b) F (c) C (d) D
10 Figure given shows the distance-time graph of the
(c) (d) All of these motion of a car. It follows from the graph that the car is
t
x
x =1.2 t2
5 A particle moves in a straight line with a constant
acceleration. It changes its velocity from 10 ms −1 to
t
20 ms −1 while passing through a distance 135 m in (a) at rest
t second. The value of t (in second) is (b) in uniform motion
(a) 12 (b) 9 (c) 10 (d) 1.8 (c) in non-uniform accelerated motion
6 A man walks on a straight road from his home to a (d) uniformly accelerated motion
market 2.5 km away with a speed of 5 kmh −1. 11 The velocity of a body depends on time according to
Finding the market closed, he instantly turns and t2
walks back home with a speed of 7.5 kmh −1. The the equation v = + 20. The body is undergoing
10
average speed of the man over the interval of time (a) uniform acceleration (b) uniform retardation
0 to 50 min is equal to (c) non-uniform acceleration (d) zero acceleration
110 OBJECTIVE Physics Vol. 1

12 The velocity v of a particle as a function of its 18 A body starts from rest with uniform acceleration a.
position (x ) is expressed as v = c 1 − c 2 x , where c 1 The acceleration of the body as function of time t is
and c 2 are positive constants. The acceleration of the given by the equation a = pt, where p is a constant,
particle is then the displacement of the particle in the time
c2 interval t = 0 to t = t1 will be
(a) c 2 (b) − 1 3 1 2
2 (a) pt1 (b) pt1
c1 + c 2 2 3
(c) c1 − c 2 (d) 1 1
2 (c) pt12 (d) pt13
2 6
13 A person walks up a stalled escalator in 90 s. When
19 A particle moves along a straight line OX. At a time
he is just standing on the same moving escalator,
t (in seconds), the distance x = 40 + 12t − t 3 . How
then he is carried for 60 s. The time it would take
him to walk up the moving escalator will be long would the particle travel before coming to rest?
(a) 27 s (b) 50 s (a) 24 m (b) 40 m (c) 56 m (d) 16 m
(c) 18 s (d) 36 s 20 A bullet emerges from a barrel of length 1.2 m with
14 Particle A is moving along X-axis. At time t = 0, it a speed of 640 ms −1. Assuming constant
has velocity of 10 ms −1 and acceleration − 4 ms −2 . acceleration, the approximate time that it spends in
the barrel after the gun is fired, is
Particle B has velocity of 20 ms −1 and acceleration (a) 4 ms (b) 40 ms (c) 400 µs (d) 1 s
− 2 ms −2 . Initially both the particles are at origin. At 21 The ratios of the distance traversed, in successive
time t = 2 s, distance between the two particles is intervals of time by a body, falling from rest, are
(a) 24 m (b) 36 m (c) 20 m (d) 42 m (a) 1 : 3 : 5 : 7 : 9 : K (b) 2 : 4 : 6 : 8 : 10 : K
15 The displacement of a body along X-axis depends on (c) 1 : 4 : 7 : 10 : 13 : K (d) None of these
time as x = t +1. Then, the velocity of body 22 A particle starts from rest. Its acceleration (a ) versus
(a) increases with time (b) decreases with time time (t ) graph as shown in the figure. The maximum
(c) independent of time (d) None of these speed of the particle will be
16 A car starts moving along a line, first with a
−2
acceleration a = 5 ms starting from rest, then 10 m/s2
uniformly and finally decelerating at the same rate a
and comes to rest. The total time of motion is 25 s.
The average speed during the time is 20 ms −1. How t (s)
11
long does particle move uniformly?
(a) 10 s (b) 12 s (a) 110 ms−1 (b) 55 ms−1
(c) 20 s (d) 15 s (c) 550 ms−1 (d) 660 ms−1
17 A cyclist starts from the centre O of a circular park 23 A ball is dropped onto the floor from a height of
of radius one kilometre, reaches the edge P of the 10 m . It rebounds to a height of 5 m . If the ball was
park, then cycles along the circumference and in contact with the floor for 0.01 s, what was its
returns to the centre along QO as shown in the average acceleration during contact?
figure. If the round trip takes ten minutes, the net (Take, g = 10 ms −2 )
displacement and average speed of the cyclist (in
(a) 2414 ms−2 (b) 1735 ms−2
metre and kilometre per hour) is −2
(c) 3120 ms (d) 4105 ms−2
Q
24 Two boys are standing at the ends A and B of a
ground, where AB = a. The boy at B starts running
O P in a direction perpendicular to AB with velocity v 1.
The boy at A starts running simultaneously with
velocity v and catches the other boy in a time t,
π+4 where t is
(a) 0, 1 (b) ,0
2 (a) a / v 2 + v12 (b) a 2 / (v 2 − v12 )
π+4 (c) a / (v − v1 ) (d) a / (v + v1 )
(c) 21.4, (d) 0, 21.4
2
Motion in One Dimension 111

25 A boggy of uniformly moving train is suddenly 33 Two particles P and Q simultaneously start moving
detached from train and stops after covering some from point A with velocities 15 ms −1 and 20 ms −1
distance. Then, which amongst the following option
respectively. The two particles move with
is correct about the relation between the distance
accelerations equal in magnitude but opposite in
covered by the boggy and distance covered by the
direction. When P overtakes Q at B, then its velocity
train in the same time?
(a) Both will be equal
is 30 ms −1. The velocity of Q at point B will be
(b) First will be half of second (a) 30 ms−1 (b) 5 ms−1
(c) First will be 1/4 of second (c) 20 ms−1 (d) 15 ms−1
(d) No definite ratio
34 A man is 45 m behind the bus when the bus start
26 A body moves for a total of nine second starting
accelerating from rest with acceleration 2.5 ms −2 .
from rest with uniform acceleration and then with
With what minimum velocity should the man start
uniform retardation, which is twice the value of
running to catch the bus?
acceleration and then stops. The duration of uniform
(a) 12 ms−1 (b) 14 ms−1
acceleration is
(c) 15 ms−1 (d) 16 ms−1
(a) 3 s (b) 4.5 s (c) 5 s (d) 6 s
35 A point moves in a straight line, so that its
27 The displacement x of a particle varies with time t as
−αt βt displacement x at time t is given by x 2 = t 2 + 1. Its
x = ae + be , where a, b, α and β are positive
acceleration is
constants. The velocity of the particle will
(a) go on decreasing with time (a) 1/ x (b) 1/x 3 (c) − 1/x 2 (d) − 1/ x 3
(b) be independent of α and β 36 A point moves with uniform acceleration and v 1, v 2
(c) drop to zero when α = β
and v 3 denote the average velocities in the three
(d) go on increasing with time
successive intervals of time t1, t 2 and t 3 . Which of
28 A stone is allowed to fall freely from rest. The ratio the following relations is correct?
of the time taken to fall through the first metre and (a) (v1 − v 2 ) : (v 2 − v 3 ) = (t1 − t 2 ) : (t 2 + t 3 )
the second metre distance is (b) (v1 − v 2 ) : (v 2 − v 3 ) = (t1 + t 2 ) : (t 2 + t 3 )
(a) 2 −1 (b) 2 +1 (c) (v1 − v 2 ) : (v 2 − v 3 ) = (t1 − t 2 ) : (t1 − t 3 )
(c) 2 (d) None of these (d) (v1 − v 2 ) : (v 2 − v 3 ) = (t1 − t 2 ) : (t 2 − t 3 )
37 The velocity-time graph for a particle moving along
29 Amongst the following equation of motion, which
X-axis is shown in the figure. The corresponding
represents uniformly accelerated motion? displacement-time graph is correctly shown by
t +a t +a x+a
(a) x = (b) x = (c) t = (d) x = t + a
b b b
v
30 A point initially at rest moves along X-axis. Its
acceleration varies with time as a = (6t + 5) ms −2 . If it
t
starts from origin, then the distance covered in 2 s is
(a) 20 m (b) 18 m (c) 16 m (d) 25 m
x x
31 A particle moves a distance x in time t according to
−1
equation x = (t + 5) . The acceleration of particle is
(a) (b)
proportional to
(a) (velocity) 3/ 2
(b) (distance) 2
t
t
−2 x
(c) (distance) (d) (velocity)2 / 3 x

32 A particle moves along a straight line. Its position at


(c) (d)
8t 3
any instant is given by x = 32t − , where x is in
4 t t
metre and t is in second. Find the acceleration of the
38 The vertical height of point P above the ground is
particle at the instant when particle is at rest.
twice that of Q. A particle is projected downward
(a) − 16 ms−2 (b) − 27.6 ms−2
with a speed of 5 ms −1 from P and at the same time,
(c) 32 ms−2 (d) 16 ms−2
another particle is projected upward with the same
112 OBJECTIVE Physics Vol. 1

speed from Q. Both particles reach the ground 42 The displacement x of a particle in a straight line
simultaneously, then motion is given by x = 1 − t − t 2 . The correct
(a) PQ = 30 m
representation of the motion is
(b) time of flight of stones = 3 s
(c) Both (a) and (b) are correct
(d) Both (a) and (b) are wrong x x
(a) (b)
39 A body falling from a high Minaret travels 40 m in
the last 2 seconds of its fall to ground. Height of t
t
Minaret in metre is (Take, g = 10 ms −2 )
(a) 60 (b) 45 (c) 80 (d) 50 (c) x (d) x
40 The acceleration-time (a -t ) graph for a particle
moving along a straight line starting from rest is t t
shown in figure. Which of the following graph is the 43 Among the four graph shown in the figure, there is
best representation of variation of its velocity (v ) only one graph for which average velocity over the
with time (t ) ? a time interval (0, T ) can vanish for a suitably chosen
T. Which one is it? [NCERT Exemplar]
x x

(a) t (b)
t
0
v v t
x x
(a) (b)
(c) (d)
t t
0 T 0 T
t t
v v
44 A lift is coming from 8th floor and is just about to
reach 4th floor. Taking ground floor as origin and
T positive direction upwards for all quantities, which
(c) 0 t (d) 0 t
T
one of the following is correct? [NCERT Exemplar]
(a) x < 0, v < 0, a > 0 (b) x > 0, v < 0, a < 0
(c) x > 0, v < 0, a > 0 (d) x > 0, v > 0, a < 0
41 The given graph shows the variation of velocity with
displacement. Which one of the graphs given below 45 The displacement of a particle is given by
correctly represents the variation of acceleration x = (t − 2) 2 , where x is in metre and t in second. The
with displacement? distance covered by the particle in first 4 seconds is
v [NCERT Exemplar]
v0 (a) 4 m (b) 8 m (c) 12 m (d) 16 m
46 A body falls freely from the top of a tower. It covers
36% of the total height in the last second before
x striking the ground level. The height of the tower is
x0
a a
(a) 50 m (b) 75 m
(c) 100 m (d) 125 m
(a) x (b) x 47 A particle moving along X-axis has acceleration f, at
 t
time t, given f = f 0 1 −  , where f 0 and T are
 T
a a
constants. The particle at t = 0 has zero velocity.
When f = 0, the particle’s velocity (v x ) is
(c) x (d) x
1 1
(a) f0T (b) f0T (c) f0T 2 (d) f0T −2
2 2
Motion in One Dimension 113

v v
48 An elevator car whose floor to ceiling distance is
2.7m starts ascending with a constant acceleration of
1.2 ms −2 . 2 s after the start, a bolt falls from the d
(a) h (b) h
ceiling of the car. The free fall time of the bolt is d
(Take, g = 9.8 ms −2 )
2.7 5.4
(a) s (b) s v v
9.8 9.8
5.4 5.4
(c) s (d) s
8.6 11 (c) d (d) d
h h
49 A parachutist after bailing out falls 50 m without
friction when parachute opens, it decelerates at
2 ms −2 . He reaches the ground with speed of 3 m/s.
At what height did he bail out? 54 The position x of a particle with respect to time t
(a) 293 m (b) 111m along X-axis is given by x = 9t 2 − t 3, where x is in
(c) 91 m (d) 182 m metres and t in second. What will be the position of
50 Two cars A and B are travelling in the same direction this particle when it achieves maximum speed along
with velocities v 1 and v 2 (v 1 > v 2 ). When the car A the positive x-direction?
is at a distance d ahead of the car B, the driver of the (a) 24 m (b) 32 m (c) 54 m (d) 81 m
car A applied the brake producing a uniform 55 A car A moves along north with velocity 30 km/h and
retardation a. There will be no collision when another car B moves along east with velocity 40 km/h.
(v − v 2 )2 The relative velocity of A with respect to B is
(a) d < 1
2a (a) 50 km/h north-east
v12 − v 22 (b) 50 km/h north-west
(b) d < (c) 50 km/h at angle tan−1 (3/4) north of west
2a
(v1 − v 2 )2 (d) 50 km/h at angle tan−1 (4/3) west of north
(c) d >
2a 56 Rain is falling vertically downward with velocity
v12 − v 22 4 m/s. A man is moving horizontally with velocity
(d) d > 3 m/s, the velocity of rain with respect to man is
2a
(a) 5 m/s at an angle tan−1 (4/3) with horizontal
51 Water drops fall at regular intervals from a tap
(b) 5 m/s at an angle tan−1 (3/4) with vertical
which is 5 m above the ground. The third drop is
(c) 5 m/s at an angle tan−1 (4/3) with vertical
leaving the tap at the instant, the first drop touches
(d) Both (a) and (b)
the ground. How far above the ground is the second
drop at that instant? 57 A ship is travelling due east at a speed of 15 km/h.
(a) 2.50 m (b) 3.75 m (c) 4.00 m (d) 1.25 m Find the speed of a boat heading 30° east of north, if
it always appears due north from the ship.
52 A body is thrown vertically up with a velocity u. It
15 3
passes three points A, B and C in its upward journey (a) 30 km/h (b) km/h (c) 10 3 km/h (d) 20 km/h
u u u 2
with velocities , and , respectively. The ratio of
2 3 4 58 A man takes 3 h to cover a certain distance along
the separations between points A and B and between the flow of river and takes 6 h to cover the same
AB distance opposite to the flow of river. In how much
B and C , i.e. is time, he will cross this distance in still water?
BC
(a) 1 (b) 2 (a) 3.5 h (b) 4 h (c) 4.5 h (d) 5 h
10 20 59 A river 500 m wide is flowing at a rate of 4 m/s. A
(c) (d)
7 7 boat is sailing at a velocity of 10 m/s with respect to
53 A ball is dropped vertically from a height d above the water in a direction perpendicular to the river.
the ground. It hits the ground and bounces up The time taken by the boat to reach the opposite
vertically to a height d / 2. Neglecting subsequent bank is
motion and air resistance, its velocity v varies with (a) 30 s (b) 40 s
(c) 50 s (d) 60 s
the height h above the ground can be plotted as
OBJECTIVE Physics Vol. 1

(B) Medical entrance special format questions


Assertion and reason
(a) Particle starts with zero velocity and variable
Directions (Q. Nos. 1-5) These questions consists of two acceleration.
statements each printed as Assertion and Reason. While (b) Particle starts with non-zero velocity and variable
answering these questions you are required to choose any one of acceleration.
the following four responses. (c) Particle starts with zero velocity and uniform
(a) If both Assertion and Reason are correct and Reason is the acceleration.
correct explanation of Assertion. (d) Particle starts with non-zero velocity and uniform
(b) If both Assertion and Reason are correct but Reason is not the acceleration.
correct explanation of Assertion.
2 A particle moves along X-axis as
(c) If Assertion is correct but Reason is incorrect.
(d) If Assertion is incorrect but Reason is correct. x = 4(t − 2) + a (t − 2) 2
1 Assertion A body is momentarily at rest at the instant, Which of the following statement is true ?
it reverses the direction. (a) The initial velocity of particle is 4.
Reason A body cannot have acceleration, if its velocity (b) The acceleration of particle is 2a.
(c) The particle is at origin at t = 0.
is zero at a given instant of time.
(d) None of the above
2 Assertion The v-t graph perpendicular to time axis is
3 In one dimensional motion, instantaneous speed v
not possible in practice.
satisfies 0 ≤ v < v 0 . Then, which of the following
Reason Infinite acceleration cannot be realised in practice. statement is true? [NCERT Exemplar]
3 Assertion In the s-t diagram as shown in figure, the (a) The displacement in time T must always take
body starts moving in positive direction but not from non-negative values.
s = 0. s (b) The displacement x in time T satisfies
− v 0 T < x < v 0T .
(c) The acceleration is always a non-negative number.
(d) The motion has no turning points.
t
t0 4 I. If a particle is thrown upwards, then distance
travelled in last second of upward journey is
independent of the velocity of projection.
Reason At t = t 0 , velocity of body changes its direction II. In last second, distance travelled is 4.9 m.
of motion. (Take, g = 9.8 ms −2 )
4 Assertion If acceleration of a particle moving in a Which amongst the statement(s) is/are correct?
straight line varies as a ∝ t n , then s ∝ t n + 2 . (a) Only I (b) Only II
(c) Both I and II (d) Neither I nor II
Reason If a-t graph is a straight line, then s-t graph
may be a parabola. 5 I. In the v-t diagram as shown in figure, average
velocity between the interval t = 0 and t = t 0
5 Assertion A lift is ascending with decreasing speed
is independent of t 0 .
means acceleration of lift is downwards. v
Reason A body always moves in the direction of its
acceleration. vm

Statement based questions


1 The displacement (x )-time (t ) graph of a particle is t0
t
shown in figure. Then, which of the following
1
statement is correct? II. Average velocity in the given interval is
vm .
x
2
Which amongst the statement(s) is/are correct?
(a) Only I
(b) Only II
(c) Both I and II
t (d) Neither I nor II
0
Motion in One Dimension 115

Column I Column II
Match the columns (A) Change in velocity (p) − 5/3 SI unit
1 Match the following columns and mark the correct (B) Average acceleration (q) − 20 SI unit
option from the codes given below.
(C) Total displacement (r) − 10 SI unit
Column I Column II
(D) Acceleration at t = 3 s (s) − 5 SI unit
(A) Constant positive acceleration (p) Speed may increase
(B) Constant negative acceleration (q) Speed may decrease Codes
(C) Constant displacement (r) Speed is zero A B C D
(D) Constant slope of a-t graph (s) Speed must increase (a) p r q s
(t) Speed must decrease (b) p r s q
(c) r p r s
Codes
(d) q p s r
A B C D A B C D
(a) q,r s p t (b) p,q p,q r p,q 4 Let us call a motion, A when velocity is positive and
(c) p s,r s q (d) q p t s,q increasing, A −1 when velocity is negative and
2 In the s-t equation (s = 10 + 20 t − 5t 2 ), match the increasing. R when velocity is positive and
decreasing and R −1 when velocity is negative and
following columns and mark the correct option from
decreasing. Now, match the following two columns
the codes given below.
for the given s-t graph and mark the correct option
Column I Column II from the codes given below.
(A) Distance travelled in 3 s (p) −20 units
s
(B) Initial acceleration (q) 15 units
(C) Velocity at 4 s (r) 25 units
(s) −10 units N P
Codes M Q
t
A B C A B C
(a) p q s (b) r q s
(c) r s p (d) q s r Column I Column II
3 For the velocity-time graph as shown in the figure, (A) M (p) A−1
in a time interval from t = 0 to t = 6 s, match the
(B) N (q) R −1
following columns and mark the correct option from
the codes given below. (C) P (r) A
v (ms−1) (D) Q (s) R
10
Codes
2 4 6 t (s) A B C D A B C D
(a) p r q s (b) r s p q
(c) s p r q (d) q p s r

(C) Medical entrances’ gallery


Collection of questions asked in NEET & Various Medical Entrance Exams

1 A ball is thrown vertically downward with a velocity window in 0.1 s. What is the velocity of the ball
of 20 m/s from the top of a tower. It hits the ground when it is at the topmost point of the window?
after some time with a velocity of 80 m/s. The (Take, g = 10 m/s 2 ) [NEET 2020]
height of the tower is (Take, g = 10 m/s 2 ) (a) 15.5 m / s (b) 14.5 m / s (c) 4.5 m / s (d) 20 m / s
[NEET 2020]
3 A person travelling in a straight line moves with a
(a) 340 m (b) 320 m (c) 300 m (d) 360 m
constant velocity v 1 for certain distance x and with a
2 A person sitting in the ground floor of a building constant velocity v 2 for next equal distance. The
notices through the window of height 1.5 m, a ball average velocity v is given by the relation
dropped from the roof of the building crosses the [NEET 2019]
116 OBJECTIVE Physics Vol. 1

1 1 1 2 1 1 10 A runner starts from O and goes to O following path


(a) = + (b) = +
v v1 v 2 v v1 v 2 OQRO in 1 h. What is net displacement and average
v v + v2 speed?
(c) = 1 (d) v = v1v 2 R
[JIPMER 2018]
2 2
−1
4 The speed of a swimmer in still water is 20 ms .
The speed of river water is 10 ms −1 and is flowing Q
O
due east. If he is standing on the south bank and 1km
wishes to cross the river along the shortest path the
angle at which he should make his strokes w.r.t.
(a) 0, 3.57 kmh −1 (b) 0, 0 kmh −1
north is given by [NEET 2019]
(c) 0, 2.57 kmh −1 (d) 0, 1 kmh −1
(a) 0° (b) 60°, west
(c) 45°, west (d) 30°, west 11 A ball is thrown upwards with a speed u from a
height h above the ground. The time taken by the
5 Find the average velocity when a particle complete
ball to hit the ground is [JIPMER 2018]
the circle of radius 1m in 10 s. [JIPMER 2019]
−1 −1 u 2 + 2gh u 2h
(a) 2 ms (b) 3.14 ms (a) 2h / g (b) 8h / g (c) (d) +
(c) 6.28 ms −1 (d) Zero g g g
6 Speed of a particle at 3rd and 8th second are 12 Preeti reached the metro station and found that the
20 ms −1 and zero respectively, then average escalator was not working. She walked up the
acceleration between 3rd and 8th second will be stationary escalator in time t1. On other days, if she
[JIPMER 2019] remains stationary on the moving escalator, then the
(a) 3 ms −2 (b) 4 ms −2 escalator takes her up in time t 2 . The time taken by
(c) 5 ms −2 (d) 6 ms −2 her to walk up on the moving escalator will be
[NEET 2017]
7 A toy car with charge q moves on a frictionless
t +t t t tt
horizontal plane surface under the influence of a (a) 1 2 (b) 1 2 (c) 1 2 (d) t1 − t 2
2 t 2 − t1 t 2 + t1
uniform electric field E. Due to the force q E, its
velocity increases from 0 to 6 ms −1 in one second 13 What will be the a versus x graph for the following
duration. At that instant, the direction of the field is graph?
reversed. The car continues to move for two more v(ms−1)
seconds under the influence of this field. The v0
average velocity and the average speed of the toy car
between 0 to 3 s are respectively [NEET 2018]
(a) 1 ms −1, 3.5 ms −1
(b) 1 ms −1, 3 ms −1 x0 x(m) [AIIMS 2017]
(c) 2 ms −1, 4 ms −1 a a
(d) 1.5 ms −1, 3 ms −1 (a) (b)
8 Assertion A body is momentarily at rest at the
instant, if it reverse the direction. x x

Reason A body cannot have acceleration, if its a a


velocity is zero at a given instant of time. [AIIMS 2018] (c) (d)
x x
(a) Assertion and Reason both are correct and Reason is
the correct explanation of Assertion.
(b) Assertion and Reason both are correct but Reason is not
the correct explanation of Assertion. 14 Which of the following statements is true for a car
(c) Assertion is correct but Reason is incorrect. moving on the road? [Manipal 2017]
(d) Assertion is incorrect but Reason is correct. (a) With respect to the frame of reference attached to the
9 Velocity is given by v = 4t (1 − 2t ), then find the ground, the car is at rest.
value of time at which velocity is maximum. (b) With respect to the frame of reference attached to the
[AIIMS 2018] person sitting in the car, the car is at rest.
(a) 0.25 s (b) 1 s (c) With respect to the frame of reference attached to the
(c) 0.45 s (d) 4 s person outside the car, the car is at rest.
(d) None of the above
Motion in One Dimension 117

15 If the velocity of a particle is v = At + Bt 2 , where A 21 A car starts from rest and accelerates uniformly to a
and B are constants, then the distance travelled by it speed of 180 kmh −1 in 10 s. The distance covered
between 1s and 2s is [NEET 2016] by the car in the time interval is [Manipal 2015]
3 7 (a) 200 m (b) 300 m
(a) 3A + 7B (b) A+ B (c) 500 m (d) 250 m
2 3
A B 3 22 The velocity-time graph for two bodies A and B are
(c) + (d) A + 4B
2 3 2 shown in figure. Then, the acceleration of A and B
16 A particle of unit mass undergoes one dimensional are in the ratio [KCET 2015]
motion such that its velocity varies according to B
v (x ) = β x −2n , where β and n are constants and x is

Velocity
the position of the particle. The acceleration of the A
particle as a function of x, is given by 40°
[CBSE AIPMT 2015]
(a) −2nβ 2 x −2 n −1 (b) −2nβ 2 x −4 n −1 25°
−2 n +1 −4 n +1 Time
(c) −2β x 2
(d) − 2n β x
2
(a) sin 25° to sin 50° (b) tan 25° to tan 40°
17 A ball is dropped from a bridge 122.5 m above a (c) cos 25° to cos 50° (d) tan 25° to tan 50°
river. After the ball has been falling for 2s, a second 23 A ball thrown vertically upwards after reaching a
ball is thrown straight down after it. What must be maximum height h returns to the starting point after
the initial velocity of the second ball, so that both a time of 10 s. Its displacement after 5 s is
ball hit the water at the same time? [AIIMS 2015] [Kerala CEE 2014]
(a) 40 ms −1 (b) 55.5 ms −1 (c) 26.1 ms −1 (d) 9.6 ms −1 (a) h (b) 2h (c) 10h (d) 20h
18 A ball is thrown vertically upwards from the ground (e) 5h
with a speed of 25.2 ms −1. How long does it take to 24 A police jeep is chasing with velocity of 45 kmh −1, a
reach its highest point and how high does it rise? thief in another jeep moving with velocity 153 kmh −1.
(Take, g = 9.8 ms −2 ) [UK PMT 2015] Police fires a bullet with muzzle velocity of
(a) 2.75 s, 3.24 m (b) 25.7 s, 34.2 m 180 ms −1. The velocity with which it will strike the
(c) 2.57 s, 32.4 m (d) 27.5 s, 3.42 m car of the thief is [EAMCET 2014]
19 A vehicle moving with a constant acceleration from (a) 150 ms −1 (b) 27 ms −1 (c) 450 ms −1 (d) 250 ms −1
A to B in a straight line AB, has velocities u and v at 25 A particle moves with constant acceleration along a
A and B, respectively. C is the mid-point of AB. If straight line starting from rest. The percentage
time taken to travel from A to C is twice the time increase in its displacement during the 4th second
taken to travel from C to B, then the velocity of the compared to that in the 3rd second is [WB JEE 2014]
vehicle v at B is [EAMCET 2015] (a) 33% (b) 40% (c) 66% (d) 77%
(a) 5 u (b) 6 u (c) 7 u (d) 8 u
26 A car covers the first half of the distance between the
20 The displacement of a particle as a function of time two places at 40 kmh −1 and another half at 60 kmh −1.
is shown in figure. It indicates that [Kerala CEE 2015]
The average speed of the car is [UK PMT 2014]
(a) 40 kmh−1 (b) 48 kmh−1 (c) 50 kmh−1 (d) 60 kmh−1
Displacement (in m)

30
27 A particle starts moving from rest with uniform
20 acceleration. It travels a distance x in first 2 s and
10 distance y in the next 2 s. Then, [EAMCET 2014]
(a) y = 3x (b) y = 4x (c) y = x (d) y = 2x
10 20 30 40 50 28 At time t = 0, two bodies A and B are at the same
Time (in second)
point. A moves with constant velocity v and B starts
(a) the velocity of the particle is constant throughout from rest and moves with constant acceleration.
(b) the acceleration of the particle is constant throughout
Relative velocity of B w.r.t. A when the bodies meet
(c) the particle starts with a constant velocity and is
each other is [EAMCET 2014]
accelerated
v v
(d) the motion is retarded and finally the particle stops (a) (b) (c) v (d) 2v
(e) None of the above 2 3
118 OBJECTIVE Physics Vol. 1

29 A car moves from A to B with a speed of 30 kmh −1 37 The velocity-time graph of robber’s car and a chasing
−1 police car are shown in the following graph. Police
and from B to A with a speed of 20 kmh . What is
the average speed of the car? [KCET 2014] car crosses the robber’s car in time [UP CPMT 2013]
(a) 25 kmh−1 (b) 24 kmh−1 ca
r
e
(c) 50 kmh−1 (d) 10 kmh−1 lic
Po

v (ms–1)
30 A body starts from rest and moves with constant 10 Robber’s car
acceleration for t second. It travels a distance x 1 in
first half of time and x 2 in next half of time, then
[KCET 2014]
5 10 15 20 25
(a) x 2 = x1 (b) x 2 = 2x1 (c) x 2 = 3x1 (d) x 2 = 4x1
t (s)
31 The acceleration of a moving body is found from the
(a) 10 s after it starts (b) 15 s after it starts
(a) area under velocity-time graph [Kerala CEE 2014]
(c) 20 s after it starts (d) Never crosses
(b) area under displacement-time graph
(c) slope of distance-time graph 38 Initial speed of an α-particle inside a tube of length
(d) slope of velocity-time graph 4m is 1 kms −1, if it is accelerated in the tube and
(e) None of the above comes out with a speed of 9 kms −1, then the time for
which the particle remains inside the tube is
32 A stone falls freely under gravity. It covers distances [BCECE 2013]
h1, h 2 and h 3 in the first 5 s, the next 5 s and the (a) 8 × 10 s −3
(b) 8 × 10 s −4
next 5 s, respectively.
(c) 80 × 10−3 s (d) 800 × 10−3 s
The relation between h1, h 2 and h 3 is [NEET 2013]
h2 h3 39 The motion of a particle along a straight line is
(a) h1 = 2h 2 = 3h 3 (b) h1 = = described by equation [CBSE AIPMT 2012]
3 5
(c) h 2 = 3h1 and h 3 = 3h 2 (d) h1 = h 2 = h 3 x = 8 + 12t − t 3
33 The motion of a particle in straight line is an where, x is in metre and t in second. The retardation
example of of the particle when its velocity becomes zero, is
(a) constant velocity motion [J&K CET 2013] (a) 24 ms−2 (b) zero (c) 6 ms−2 (d) 12 ms−2
(b) uniformly accelerated motion 40 A particle moves along with X-axis. The position x
(c) non-uniformly accelerated motion of particle with respect to time t from origin given
(d) zero velocity motion by x = b 0 + b 1t + b 2t 2 . The acceleration of particle is
34 The velocity-time graph of particle comes out to be a [AIIMS 2012]
non-linear curve. The motion is [J&K CET 2013] (a) b 0 (b) b1 (c) b 2 (d) 2b 2
(a) uniform velocity motion 41 A body X is projected upwards with a velocity of
(b) uniformly accelerated motion
98 ms −1, after 4 s, a second bodyY is also projected
(c) non-uniform accelerated motion
(d) Nothing can be said about the motion
upwards with the same initial velocity. Two bodies
will meet after [BCECE 2012]
35 A body is thrown vertically upward from a point A (a) 8 s (b) 10 s (c) 12 s (d) 14 s
125 m above the ground. It goes up to a maximum
42 A scooter starts from rest have an acceleration of
height of 250 m above the ground and passes
through A on its downward journey. The velocity of 1 ms −2 while a car 150 m behind it starts from rest
the body when it is at a height of 70 m above the with an acceleration of 2 ms −2 . After how much
ground is (Take, g = 10 ms −2 ) time, the car catches up with the scooter? [BHU 2012]
[EAMCET 2013]
(a) 700 s (b) 300 s
(a) 50 ms −1 (b) 60 ms −1 (c) 80 ms −1 (d) 20 ms −1
(c) 150 s (d) None of these
36 A person reaches a point directly opposite on the
other bank of a river. The velocity of the water in 43 Let r1 (t ) = 3t$i + 4t 2 $j
the river is 4 ms −1 and the velocity of the person in and r2 (t ) = 4t 2 $i + 3 t$j
still water is 5 ms −1. If the width of the river is 84.6 represent the positions of particles 1 and 2,
m, time taken to cross the river (in seconds) is respectively, as function of time t, r1 (t ) and r2 (t ) are in
[EAMCET 2013] metre and t in second. The relative speed of the two
(a) 28.2 (b) 9.4 particles at the instant t = 1s, will be
[AMU 2012]
(c) 2 (d) 84.6
(a) 1 m/s (b) 3 2 m/s (c) 5 2 m/s (d) 7 2 m/s
ANSWERS
l CHECK POINT 3.1
1. (d) 2. (a) 3. (a) 4. (c) 5 . (b) 6. (c) 7. (b) 8. (d)

l CHECK POINT 3.2


1. (a) 2. (a) 3. (d) 4. (d) 5. (c) 6. (c) 7. (c) 8. (d)

l CHECK POINT 3.3


1. (c) 2. (d) 3. (b) 4. (a) 5. (d)

l CHECK POINT 3.4


1. (a) 2. (c) 3. (d) 4. (a) 5. (a) 6. (b) 7. (c) 8. (c) 9. (c) 10. (b)
11. (c)

l CHECK POINT 3.5


1. (b) 2. (d) 3. (b) 4. (b) 5. (a) 6. (b) 7. (a) 8. (b) 9. (c) 10. (a)
11. (a) 12. (c) 13. (a) 14. (b) 15. (b) 16. (b) 17. (d) 18. (b) 19. (a) 20. (d)
21. (c)

l CHECK POINT 3.6


1. (d) 2. (c) 3. (c) 4. (a) 5. (d) 6. (b) 7. (c) 8. (a) 9. (b) 10. (a)
11. (a) 12. (c) 13. (a) 14. (d) 15. (d) 16. (b) 17. (b) 18. (a)

l CHECK POINT 3.7


1. (d) 2. (a) 3. (c) 4. (b) 5. (b) 6. (c) 7. (a) 8. (d) 9. (b) 10. (a)
11. (a) 12. (c) 13. (b) 14. (b)

(A) Taking it together


1. (b) 2. (a) 3. (c) 4. (d) 5. (b) 6. (c) 7. (d) 8. (a) 9. (a) 10. (d)
11. (c) 12. (b) 13. (d) 14. (a) 15. (a) 16. (d) 17. (d) 18. (d) 19. (c) 20. (b)
21. (a) 22. (b) 23. (a) 24. (b) 25. (b) 26. (d) 27. (d) 28. (b) 29. (c) 30. (b)
31. (a) 32. (b) 33. (b) 34. (c) 35. (b) 36. (b) 37. (d) 38. (c) 39. (b) 40. (a)
41. (a) 42. (b) 43. (b) 44. (a) 45. (b) 46. (d) 47. (a) 48. (d) 49. (a) 50. (c)
51. (b) 52. (d) 53. (a) 54. (c) 55. (c) 56. (d) 57. (a) 58. (b) 59. (c)

(B) Medical entrance special format questions


l Assertion and reason
1. (c) 2. (a) 3. (c) 4. (b) 5. (c)

l Statement based questions


1. (a) 2. (b) 3. (b) 4. (c) 5. (c)

l Match the columns


1. (b) 2. (c) 3. (c) 4. (b)

(C) Medical entrances’ gallery


1. (c) 2. (b) 3. (b) 4. (d) 5. (d) 6. (b) 7. (b) 8. (c) 9. (a) 10. (a)
11. (c) 12. (c) 13. (c) 14. (b) 15. (b) 16. (b) 17. (c) 18. (c) 19. (c) 20. (d)
21. (d) 22. (d) 23. (a) 24. (a) 25. (b) 26. (b) 27. (a) 28. (c) 29. (b) 30. (c)
31. (d) 32. (b) 33. (b) 34. (c) 35. (b) 36. (a) 37. (c) 38. (b) 39. (d) 40. (d)
41. (c) 42. (b) 43. (c)
Hints & Explanations
l CHECK POINT 3.1 4 (d) Displacement of the particle will be zero because it comes
2 (a) Distance from starting point = 3 + 4 + 5 = 12 m back to its starting point.
Total distance 30 m
60 πR Average speed = = = 3 ms −1
3 (a) Distance = Length AB = 2 πR × = Total time 10 s
360 3
d 150 + 850
4 (c) Horizontal distance covered by the wheel in half 5 (c) t = = = 80 s
v 5
revolution = πR 45 ×
A′ Final 18
Total displacement (4)2 + (3)2 1 −1
2R 6 (c) v av = = = ms
Total time 10 + 5 3
Displacement 2R
A πR 7 (c) v av = =
Initial Time t
So, the displacement of the point which was initially in Displacement 2R 2v 20
8 (d) v av = = = = ms −1
contact with ground = AA′ = (πR )2 + (2R )2 Time (πR /v ) π π

=R π2 + 4 = π2 + 4 (As, R = 1m ) l CHECK POINT 3.3


5 (b) A B C O D E 1 (c) Acceleration is a vector quantity. So, it changes when
−x +x either magnitude or direction of velocity changes or both
−9 −8 −7 −6 −5 −4 −3 −2 −1 0 1 2 3 4 5 6 7 8
(m) (m) changes.
Distance 3.06
(i) The displacement of the man from A to E is 3 (b) Time = = = 9s
∆x = x 2 − x1 = 7m − (−8 m) = +15 m directed in the positive Average velocity 0.34
x-direction. Change in velocity 0.18
(ii) The displacement of the man from E to C is Acceleration = = = 0.02 m/s 2
Time 9
∆x = − 3 m − (7 m) = −10 m directed in the negative
Change in velocity v f − vi
x-direction. 4 (a) a = =
(iii) The displacement of the man from B to D is Time taken t
∆x = 3 m − (−7 m) = +10 m directed in the positive  5  5
 44 ×  −  80 × 
x-direction.  18  18
= = − 0.67 m/s 2
6 (c) Since, displacement is always less than or equal to distance 15
but never greater than distance. Hence, numerical ratio of Negative sign represents the retardation.
displacement to the distance covered is always equal to or less 5 (d) For path OA and BO, the magnitude of velocity (speed) and
than one. direction is constant, hence acceleration is zero.
l CHECK POINT 3.2 For path AB, since this path is a curve, so the direction of the
Total distance 80t + 40 t velocity changes every moment but the magnitude of velocity
1 (a) v av = = = 60 kmh−1 (speed) remains constant.
Total time 2t
Since, the direction of velocity is changing, i.e. there must be
11 × 18 + 42 × v some acceleration along the path AB.
2 (a) 21 =
60
∴ v = 25.3 m min−1 l CHECK POINT 3.4
3u
3 (d) Man walks from his home to market with a speed of 1 (a) u− = u − at0
2.5 1 4
5 kmh −1. Distance = 2.5 km and time = = h = 30 min 3u u 4
5 2 a= or = t0
and he returns back with speed of 7.5 km/h in rest time, i.e. 4t0 a 3
10 min. u 4
10 Now, 0 = u − at or t = = t0
Distance = 7.5 × = 1.25 km a 3
60 1 2 1
total distance 2 (c) s = at = × (0.2) (64) = 6.4 m = 640 cm
So, average speed = 2 2
total time 1 2 1
(2.5 + 1.25) km 45 3 (d) s = at ⇒ s = × a × (4)2 ⇒ s = 8 a
= = kmh−1 2 2
(40 / 60 ) h 8
Motion in One Dimension 121

1 CHECK POINT 3.5


4 (a) 15 = 2t − × (0.1) t 2 or t = 10 s l

2 2u 2 × 9.8
1 (b) t = ⇒ t= = 1.96 ≈ 2 s
5 (a) s = s1 + s2 + s3 g 10
1 1 2 (d) Let t second of upward journey = first t second of
= × 2 × (10 )2 + (20 ) (30 ) + × 4 × (5)2
2 2 downward journey (with zero initial velocity).
= 100 + 600 + 50 = 750 m 1
∴ Desired distance = gt 2
6 (b) Displacements of both should be equal. 2
1 3 (b) Time taken to reach maximum height is 1 s.
or 8t = × 4 × t 2 or t = 4 s
2 1
Height = free fall distance in1s = gt 2 = 5 m
1 2
7 (c) sB = × 2 × (1)2 = 1 m
2 (10 )2
1 4 (b) h = =5m
sA = × 2 × (2)2 = 4 m 2g
2 h
∴ sA − sB = 3 m
8 (c) 40 = (20 ) a1 ⇒ a1 = 2 ms −2 10 ms–1
Further 40 = (40 ) a 2
∴ a 2 = 1 ms −2
Therefore, acceleration is 2 ms −2 and retardation is 1 ms −2. ∴ Total height = 2h = 10 m
1 2 1 5 (a) Hmax ∝ v 2
Now, s1 = a1 t1 = × 2 × (20 )2 = 400 m
2 2 ⇒ v ∝ Hmax
s2 = vmax t2 = 40 × 20 = 800 m
i.e. To triple the maximum height, ball should be thrown with
v2 (40 )2
s3 = max = = 800 m velocity 3 v 0 .
2a 2 2 × 1
total displacement 6 (b) v 2 = u 2 + 2gh
Now, average velocity =
total time ⇒ (3u )2 = (− u )2 + 2gh
400 + 800 + 800
= = 25 ms −1 4u 2
20 + 20 + 40 ⇒ h=
g
9 (c) Let v m be the maximum speed,
7 (a) tABC = 10 s ⇒ tAB = 5 s
v m = a11
t = 2a1l (Q u = 0)
B B
2l
t2 = and v m = a 2t3 = 2a 2 (3l )
vm 5s 5s
l + 2l + 3l
Now, average speed, v av =
t1 + t2 + t3 v A C
6l
v av = 10.2 m
(v m /a1) + (2l /v m ) + (v m /a 2 )
u
6l 6l 3v
= = = m
 v m   2l   v m  (10 l / v m ) 5
 2  +  +  At B, velocity becomes zero. Hence at A, velocity should be
v m / 2l  v m  v m2 / 6 l  50 ms −1.
v av 3
⇒ = Now, (50 )2 = (u 2 ) − 2 × 10 × 10.2
vm 5
∴ u = 52 ms −1
10 (b) (30 )2 = (20 )2 + 2a (2s ) or 2as = 250
8 (b) Velocity of particle of this instant will be v = (u − gt1)
Now, v 2 = u 2 + 2as ⇒ v 2 = (20 )2 + 250 ⇒v 2 = 650
∴ v = 25.5 ms −1 v t0 t0
v
t0 = g
a a
11 (c) sn = u + (2n − 1) ⇒ 1.2 = 0 + (2 × 6 − 1)
2 2
1.2 × 2 −2 t1
⇒ a= = 0.218 ms
11 u
122 OBJECTIVE Physics Vol. 1

2v 1
Now, the desired time interval will be . 15 (b) In first second distance travelled, x = × g ×t2 = 5 m
g 2
2 (u − gt1) u  and similarly, in last second distance travelled, 7x = 35 m
or = 2  − t1  1 
g g  Now, st = u + at − a
 2 
9 (c) Taking downward direction as the positive direction. 1
u ⇒ 35 = 0 + 10 × t − × 10
2
∴ t =4s
16 (b) x = 18t + 5t 2
h
u dx d
v= = (18t + 5t 2 ) = 18 + 10t
1 2 dt dt
+ h = − ut1 + gt1 …(i)
2 ∴ v = 10t + 18
1 2 At t1 = 2 s, v1 = 10 (2) + 18 = 38 m/s
+ h = ut2 + gt2 …(ii)
2 At t2 = 4 s, v 2 = 10 (4) + 18 = 58 m/s
v −v 20
Multiplying Eq. (i) by t2 and Eq. (ii) by t1 and adding, we get ∴ a= 2 1= = 10 m/s 2
1 1 t 2
h (t1 + t2 ) = gtt12 (t1 + t2 ) or h = gt1t2
2 2 ds
17 (d) v = = 12 − 4t
1 dt
For free fall from rest, h = gt 2
2 Comparing with v = u + at, u = 12 ms −1 and a = − 4 ms −2
∴ t 2 = t1 t2 ⇒ t = t1t2 Velocity will become zero at time t0 ⇒ 0 = 12 − 4t0 or
t0 = 3 s.
10 (a) After 2 s, velocity, v = 4.9 × 2 = 9.8
Since, the given time t = 5 s is greater than t0 = 3 s
1
and h = × 4.9 × (2)2 = 9.8 m distance > displacement
2
u2 1
v2 Distance, d = s0 − t 0 + st − t 0 = + a (t − t0 )2
Greatest height, hmax = h + = 14.7 m 2 a 2
2g
(12)2 1
= + × 4 × (2)2 = 26 m
11 (a) Here, u = 0 8 2
1 2 1 dx d
S3 = 0 + gt = × 10 × 9 = 45 18 (b) v = = (αt 3 + βt 2 + γt + δ )
2 2 dt dt
St th = u + (2t − 1) g / 2 v = 3αt 2 + 2 βt + γ ; v t = 0 = vi = γ
⇒ St th = 0 + 5 (2t − 1)
45 = 5 (2t − 1) a = 6 αt + 2 β : a t = 0 = ai = 2β
vi γ
⇒ 2t − 1 = 9 ⇒ t = 5 s ∴ =
ai 2 β
1
12 (c) h1 = × g × (2)2 = 2g
2 19 (a) a = bt
1 v t bt 2
h2 = × g × (4)2 − 2g = 6g
2

0
∫v dv = ∫ adt = ∫ bt ⋅ dt ⇒ v = v 0 +
0 2
1 Further integrating the above equation w.r.t. time, we get
h3 = × g × (6)2 − 8g = 10 g
2 bt 3
s = v 0t +
∴ h1 : h2 : h3 = 1: 3 : 5 6

2h t h 20 (d) dv = a d t
13 (a) t = or t ∝ h ⇒ 1 = = 1: 2 v 2
g t2 2h ∴ ∫ 2 dv = ∫ 0 (3t + 2t + 2) dt = [t 3 + t 2 + 2t]20
2

1 2
14 (b) Let h be the distance covered in t second, h = gt ...(i) or v = 18 ms −1
2
1 ds
Distance covered in t th second = g (2t − 1) 21 (c) s = t 3 − 6t 2 + 18 t + 9 ⇒ v =
= 3t 2 − 12t + 18
2 dt
9h g v is minimum or maximum at time t, which can be calculated as
⇒ = (2t − 1) ...(ii)
25 2 dv
a= = 6t − 12 = 0
From Eqs. (i) and (ii), we get dt
h = 122.5 m ⇒ t = 2s
Motion in One Dimension 123

At t = 2 s, 13 (a) Since, total displacement is zero, hence average velocity is


d 2v also zero.
= 6 > 0, i.e. v is minimum at t = 2 s
dt 2 14 (d) I is not possible because total distance covered by a particle
vmin = 3 (2)2 − 12 (2) + 18 = 6 m/s cannot decrease with time. II is not possible because at a
particular time, position cannot have two values. III is not
l CHECK POINT 3.6 possible because at a particular time, velocity cannot have two
values. IV is not possible because speed can never be negative.
1 (d) Uniform motion means uniform velocity or constant slope
 dv   20  20
of s- t graph. 15 (d) a = v   = (10 )  −  = − ms −2
 ds   30  3
3 (c) Slope of the s-t graph is velocity, v = tanθ
But it is valid only when angle is measured with time axis. 16 (b) For uniformly accelerated motion, v 2 = u 2 + 2 as, i.e.
So, for the given graph, angle from time axis v 2 versus s graph is a straight line with intercept u 2 and slope
= 90 ° − 30 ° = 60 ° 2 a. Since, intercept is non-zero, initial velocity is non-zero.
Now, v = tan 60 ° = 3 ms −1 16
17 (b) v 2 = u 2 + 2 as , slope = 2a = − = − 8 ms −2
2
4 (a) vi = slope of s-t graph = tan 45° = 1 ms −1
or a = − 4 ms −2
v f = slope of s-t graph = tan 60 ° = 3 ms −1
18 (a) Taking the motion from 0 to 2 s,
v f − vi 3 −1
Now, a av = = = ( 3 − 1) units u = 0, a = 3 ms −2, t = 2s , v = ?
∆t 1
v = u + at = 0 + 3 × 2 = 6 ms −1
5 (d) Maximum acceleration means maximum change in
velocity in minimum time interval. Taking the motion from 2s to 4s,
In time interval, t = 30 s to t = 40 s v = 6 + (− 3)(2) = 0 ms −1
∆v 80 − 20 60
∴ a= = = = 6 cms −2 l CHECK POINT 3.7
∆t 40 − 30 10
100
6 (b) Distance = Area under v - t graph = A1 + A 2 + A 3 + A 4 1 (d) 7.2 = or v = 45 ms −1
(v + 5) (5/18)
2 (a) Let positive direction of motion be from south to north.
30
v A = + 54 kmh −1 = 15 ms −1 ,
Velocity (ms−1)

Given,

20 v B = − 90 kmh −1 = − 25 ms −1
∴ The relative velocity of B w.r.t. A,
10 v BA = v B − v A = − 25 − 15 = − 40 ms −1
A1 A2 A3 A4
i.e. The train B appears to A to move with a speed of 40 ms −1
1 2 3 4 from north to south.
Time (s)
3 (c) Both the particles will fall same distance in same time
1 1 interval.
= × 1 × 20 + (20 × 1) + (20 + 10 ) × 1 + (10 × 1)
2 2 So, the relative separation will remain unchanged.
= 10 + 20 + 15 + 10 = 55 m 4 (b) v AB = v A − v B
1
7 (c) Distance = Area of trapezium = × 3.6 × (12 + 8) = 36 m N
2
dx
9 (b) v = = 2 ms −1 = constant
dt E

vA vAB
v (ms−1) 2

t (s)
α
–vB
10 (a) Velocity will continuously increase (starting from rest).
v AB = v A2 + v B2 = 5 kmh−1
11 (a) Velocity first decreases in upwards direction, then
increases in downward direction. v   3
α = tan−1  A  = tan−1  
15 v B   4
12 (c) a = Slope of v-t graph = − = − 5 ms −2
3 = 37° east of north
124 OBJECTIVE Physics Vol. 1

B
5 (b) Velocity of rain is at 30° in vertical direction. So, its
v
horizontal component is vR sin 30 ° = R . When man starts vb
2
walking with 10 kmh−1 rain appears vertical. So, horizontal v y
θ 45°
v
component R is balanced by his speed of 10 kmh−1. Thus,
2 x
A u
vR
= 10 or vR = 20 kmh−1 v x = u − v sin θ and v y = v cos θ
2
v 12 vy v cos θ
6 (c) tan 60° = H or 3 = Further, tan 45° = or 1=
vV vV vx u − v sin θ
∴ vV = 4 3 kmh−1 ∴ v=
u
=
u
sin θ + cos θ 2 sin (θ + 45°)
vH v
7 (a) tanθ = = v is minimum at,
vV u
u
v 
−1 θ + 45° = 90 ° or θ = 45° and vmin =
or θ = tan   2
 u
1 13 (b) v r = 10 + 15 = 25 ms −1
8 (d) v b = = 4 kmh−1
1/ 4 where, v r is relative velocity.
vr 50 + 50
∴ t= =4s
25
vbr 14 (b) Relative acceleration = 0, relative velocity is 40 ms −1 and
vb relative separation is 100 m.
100
∴ t= = 2.5 s
v br = 5 kmh−1 40
∴ v r = v br − v b = 1kmh−1
(A) Taking it together
9 (b) v x = − v $j 2v1v 2 2 × 2.5 × 4
1 (b) Average speed of a boy = =
N(j)
^
v1 + v 2 2.5 + 4
20 40
= = km/h
^
E( i ) 6.5 13
W
a
2 (a) sn = u + (2n − 1)
2
4/ 3 10
S ⇒ s3 = 0 + (2 × 3 − 1) ⇒ s3 = m
2 3
∴ vY = vYX + v X = − v$i + v$j l/2 l
3 (c) Time taken to travel first half distance, t1 = =
v1 2v1
| vY | = 2 v
l
Direction of vY is north-west. Time taken to travel second half distance, t2 =
2v 2
10 (a) For shortest time, one should swim at right angles to river
current. l l l  1 1
Total time = t1 + t2 = + =  + 
v 3 2v1 2v 2 2 v1 v 2 
11 (a) sinθ = r =
v br 5 We know that, v av = Average speed
vr total distance l 2v v
= = = 1 2
total time l  1 1  v1 + v 2
 + 
vbr vb 2  v1 v 2 
θ
4 (d) In all the given graphs, we have more than one value of
∴ θ = 37° speed corresponding to a single time. However, the graph
must have a unique value of speed corresponding to a single
The required angle is therefore
time. Thus, all the three graphs are not possible.
90 ° + θ = 90 ° + 37° = 127°
5 (b) From third equation of motion, v 2 = u 2 + 2as
12 (c) Let v be the speed of boatman in still water. Resultant of v
and u should be along AB. Components of v b (absolute velocity v 2 − u 2 (20 )2 − (10 )2 300 10
⇒ a= = = = ms −2
of boatman) along x and y-directions are, 2s 2 × 135 270 9
Motion in One Dimension 125

From first equation of motion, 1


14 (a) At t = 2 s; x A = 10 × 2 − × 4 × (2)2 = 12 m
v − u 20 − 10 10 2
v = u + at ⇒ t = = = = 9s
a 10 / 9 10 / 9 1
and x B = 20 × 2 − × 2 × (2)2 = 36 m
2
6 (c) Man walks from his home to market with a speed of
∴ Distance between A and B at that instant is 24 m.
5 kmh −1. Distance = 2.5 km and time
d 2.5 1 15 (a) Given, x =t+1
= = = = 30 min Squaring both sides, we get
v 5 2
and he returns back with speed of 7.5 kmh −1 in rest a time of x = (t + 1)2 = t 2 + 2t + 1
20 min. Differentiating it w.r.t. time t, we get
20 dx
Distance = 7.5 × = 2.5 km = 2t + 2
60 dt
total distance (2.5 + 2.5) 30 dx
So, average speed = = = km/h Velocity, v = = 2t + 2
total time (50 / 60) h 5 dt
Total displacement
7 (d) s ∝ u 2. If u becomes 3 times, then s will become 9 times, 16 (d) Average velocity =
Total time
i.e. 9 × 20 = 180 m 25 − 2t
t t
8 (a) Distance travelled by body A in 5th second and distance
travelled by body B in 3rd second of its motion are equal.
v = at = 5t
a a
0 + 1 (2 × 5 − 1) = 0 + 2 (2 × 3 − 1) 1 1
× a × t 2 + (at ) (25 − 2t ) + × a × t 2
2 2
or 20 = 2 2
a 5 25
9a1 = 5a 2 ⇒ 1 =
a2 9 Solving this equation with a = 5 ms −2, we get
9 (a) Q Slope of displacement-time graph at the point E is t = 5s
negative. Thus, at the point E, the instantaneous velocity of Thus, the particle moved uniformly for (25 − 2 t ) or 15 s.
the particle is negative.
17 (d) Net displacement = 0 and total distance = OP + PQ + QO
10 (d) Since x = 1.2 t 2, comparing it with equation of motion
2π × 1 14.28
1 2 = 1+ + 1= km
x= at . 4 4
2
14.28 6 × 14.28
Thus, the motion is uniformly accelerated. Average speed = = = 21.42 km/h
4 × 10 / 60 4
t2
11 (c) It is given that, v = + 20 18 (d) Integrate twice to convert a-t equation into s-t equation.
10
v
Differentiating both sides w.r.t. time, we get Q a= ⇒ v = at
t
dv 2t t
= +0= pt 2
dt 10 5
dv t
⇒ ∫ dv = ∫ a ⋅ dt ⇒ v = ∫ pt ⋅ dt =
2
∴ Acceleration, a = = ≠ constant
dt 5 Q s = v × t ⇒ ds = v dt
t1 1 3
Hence, it is a case of non-uniform acceleration. ⇒ ∫ ds = ∫0 v dt ⇒ s = 6 pt1
12 (b) Given, v = c1 − c 2x
19 (c) Distance travelled by the particle is x = 40 + 12 t − t 3
dv
We know that, a = v We know that, speed is defined as the rate of change of
dx
dx
d distance, i.e. v=
⇒ a = c1 − c 2x ⋅ 9 ( c1 − c 2x ) dt
dx
d
1 c ∴ v = (40 + 12t − t 3 ) = 0 + 12 − 3t 2
= (− c 2 ) = − 2 dt
2 2
But final velocity, v = 0
s s
13 (d) v1 = , v2 = 12
90 60 ∴ 12 − 3t 2 = 0 ⇒ t 2 = =4
s s 3
Now, t = = ⇒ t = 2s
v1 + v 2 s
+
s
90 60 Hence, distance travelled by the particle before coming to rest
90 × 60 is given by
= = 36 s
90 + 60 x = 40 + 12(2) − (2)3 = 40 + 24 − 8 = 64 − 8 = 56 m
126 OBJECTIVE Physics Vol. 1

20 (b) Given, s = 1. 2 m, v = 640 ms −1, 26 (d) Let acceleration is a and retardation is − 2a.
a = ?; u = 0; t = ? Then, for accelerated motion, v
v vmax
2as = v 2 − u 2 t1 = …(i)
a a, t 1
⇒ 2a × 1. 2 = 640 × 640 −2a, t2
For retarding motion,
8 × 64 × 100
⇒ a= v
3 t2 = …(ii) O t
2a
v = u + at
v v 3v v
v 640 × 3 Given, t1 + t2 = 9 ⇒ + =9 ⇒ = 9⇒ = 6
⇒ t= = = 37.5 × 10 −3 s ≈ 40 ms a 2a 2a a
a 8 × 64 × 100
v
21 (a) Here, u = 0, a = g Hence, duration of acceleration, t1 = = 6 s.
a
Distance travelled in nth second is given by
27 (d) Given, x = ae − αt + be βt
a
Dn = u + (2n − 1) dx
2 So, velocity, v = = − aαe − αt + bβe βt
dt
∴ Dn ∝ (2n − 1)
dv
∴ D1 : D2 : D3 : D4 : D5K = 1: 3 : 5 : 7 : 9 : K a= = aα 2e − αt + bβ 2e βt = + ve all time.
dt
22 (b) The area under acceleration-time graph gives change in ∴ v will go on increasing.
velocity.
1 2
As acceleration is zero at the end of 11 s, 28 (b) 1 = g t12 or t1 =
2 g
1
i.e. vmax = Area of ∆OAB = × 11× 10 = 55 ms −1
2 1 2 4
2= g t or t =
v f − vi 2 g
23 (a) a av = (As they are in opposite direction)
∆t 4 2
2gh f + 2gh i But t2 = t − t1 = −
2 × 10 × 5 + 2 × 10 × 10 g g
= =
∆t 0.01
−2 t1 2/ g 2
= 2414 . 21≈ 2414 ms ∴ = =
t2 4/ g − 2/ g 2− 2
24 (b) Let two boys meet at point C after time ‘t ’ from the
2 (2 + 2)
starting. Then, AC = vt, BC = v1t (see figure) = = ( 2 + 1)
v1t
2
B C x+a
29 (c) t = or (x + a ) = bt 2 or x = − a + bt 2
a b
vt
Comparing this equation with general equation of uniformly
A
1
accelerated motion, s = si + ut + at 2
(AC )2 = (AB )2 + (BC )2 ⇒ v 2t 2 = a 2 + v12t 2 2
By solving, we get we see that si = − a, u = 0 and acceleration = 2b.
a2 30 (b) Given, acceleration, a = 6 t + 5
t=
v − v12
2
dv
∴ a= = 6 t + 5, dv = (6 t + 5)dt
dt
25 (b) Let a be the retardation of boggy, then distance covered by
it be sb. If u is the initial velocity of boggy after detaching Integrating it, we have
v t
from train (i.e. uniform speed of train)
∫0dv = ∫0 (6 t + 5)dt
v = u + 2as ⇒ 0 = u − 2as
2 2 2
v = 3 t 2 + 5 t + C , where C is a constant of integration.
2
u
⇒ sb = When t = 0, v = 0, so C = 0
2a ds
Time taken by boggy to stop ∴ v= = 3 t 2 + 5 t or ds = (3t 2 + 5) dt
dt
u
v = u + at ⇒ 0 = u − at ⇒ t = Integrating it within the conditions of motion, i.e. as t changes
a from 0 to 2s, s changes from 0 to s, we have
u2 s 2
∫ 0 ds = ∫ 0 (3 t + 5 t )dt‘
2
In this time t, distance travelled by train, st = ut =
a
2
sb 1  5 
Hence, ratio = . ∴ s = t 3 + t 2 = 8 + 10 = 18 m
st 2  2  0
Motion in One Dimension 127

1 1
31 (a) Given, x = and v 3 = u + a (t1 + t2 ) + at3
t+5 2
Differentiating both sides w.r.t. t, we get ∴ (v1 − v 2 ) : (v 2 − v 3 ) = (t1 + t2 ) : (t2 + t3 )
dx 1 37 (d) Motion is first uniformly accelerated in positive direction,
=− ×1
dt (t + 5)2 then it is uniform in negative direction.
dx 1 38 (c) h = − 5t + 5t 2 and 2h = 5t + 5t 2
∴ Velocity, v = =− …(i)
dt (t + 5)2 5 ms−1
P +ve
Again, differentiating both sides w.r.t. t, we get
dv 1× 2 2 5 ms−1
= (1) = …(ii)
dt (t + 5)3 (t + 5)3
2h Q
Now, from Eqs. (i) and (ii), we get
dv 1 h
= − 2v × = (−2v ) ( v ) = − 2(v )3/ 2
dt (t + 5)
dv From these two equations, we get t = 3 s and h = distance
∴ Acceleration, a = = − 2v 3/ 2 ⇒ a ∝ (velocity)3/ 2
dt between P and Q = 30 m .
dx 39 (b) Let height of Minaret be H and body take time T to fall
32 (b) v = = 32 − 6t 2, v = 0 at t = 2.30 s
dt from top to bottom.
dv
a= = − 12 t
dt (H − 40) m (T − 2) s
At 2.30 s, a = − 27.6 ms −2
T H
33 (b) For P, 30 = 15 + at or at = 15 ms −1
2s
For Q, v = 20 − at or v = 20 − 15 = 5 ms −1 40 m

34 (c) Let the man will catch the bus after t second. So, he will 1 2
cover distance ut. H=gT …(i)
2
1
Similarly, distance travelled by the bus will be at 2. In last 2 s, body travels distance of 40 m, so in (T − 2) second,
2 distance travelled = (H − 40 ) m
For the given condition, 1
1 ⇒ (H − 40 ) = g (T − 2)2 …(ii)
ut = 45 + at 2 = 45 + 1.25 t 2 (As, a = 2 . 5 ms −2) 2
2 By solving Eqs. (i) and (ii), we get
45
⇒ u= + 1.25 t T = 3 s and H = 45 m
t
40 (a) Given acceleration-time graph is shown in figure.
To find the minimum value of u
du du − 45 a
=0 ⇒ = 2 + 1.25 = 0
dt dt t a0
45
So, we get t = 6 s, then u = + (1.25 × 6) = 15 ms −1
6
dx dx
35 (b) x 2 = t 2 + 1 or 2x ⋅ = 2t or x ⋅ =t t
dt dt 0 T
dx t t
∴ v= = = Let a (t = 0 ) = a 0 = constant
dt x t +1
2
From the graph, we have
t2 t a
t2 + 1 − + =1
dv d 2x t2 + 1 1 1 T a0
a= = 2 = = =
dt dt (t + 1)
2
(t 2 + 1)3/ 2 x 3 ⇒ a 0t + aT = a 0T …(i)
Also, we know that,
36 (b) Average velocity in uniformly accelerated motion is given by
dv
1 a= …(ii)
ut + at 2 dt
s 2 1
v av = = = u + at From Eqs. (i) and (ii), we get
t t 2
1 1 a 0T − a 0t dv
Now, v1 = u + at1 , v 2 = (u + at1) + at2 =
2 2 T dt
128 OBJECTIVE Physics Vol. 1

45 (b) Given, x = (t − 2)2


Integrating both sides, we get dx d
t v Velocity, v= = (t − 2)2 = 2(t − 2) ms −1
∫0 (a 0T − a 0t ) dt = T ∫ dv
0
dt dt
dv d
a 0t 2
a Acceleration, a = = [2(t − 2)] = 2 (1 − 0 ) = 2 ms −2
⇒ a 0Tt − = Tv ⇒ v = a 0t − 0 t 2 dt dt
2 2T
When t = 0 ; v = − 4 ms −1, t = 2 s ; v = 0 ms −1
Therefore, v-t graph must be parabolic.
Now, at t = 0, v = 0 t = 4 s ; v = 4 ms −1
aT v-t graph is shown in figure below.
t = T, v = 0 = constant v
2
4 ms−1 B
These conditions are satisfied by graph (a).
41 (a) Given line has positive intercept but negative slope. So, its
equation can be written as O A D
 v  t
v = − mx + v 0  where, m = tanθ = 0  …(i) 2s 4s
 x0 
By differentiating with respect to time, we get
-4 ms−1 C
dv dx
= −m = − mv
dt dt Distance travelled = Area under the v-t graph
Now, substituting the value of v from Eq. (i), we get = |Area OAC | + Area ABD
dv 4×2 1
= − m (− mx + v 0 ) = m 2x − mv 0 = + × 2× 4= 8m
dt 2 2
∴ a = m 2x − mv 0
46 (d) Let height of tower is h and body takes t time to reach to
i.e. The graph between a and x should have positive slope but ground when it fall freely.
negative intercept on a-axis. So, graph (a) is correct. 1
∴ h = gt 2 …(i)
dx 2
42 (b) = v = − 1 − 2t
dt In last second, i.e. tth second body travels = 0.36 h
Comparing with v = u + at, we get It means in rest of the time, i.e. in (t − 1) second, it travels
u = − 1 ms −1 and a = − 2 ms −2. = h − 0.36 h = 0.64h
Now, applying equation of motion for (t − 1) second
At t = 0, x = 1m. Then, u and a both are negative. Hence,
1
x-coordinate of particle will go on decreasing. 0.64h = g (t − 1)2 ...(ii)
2
43 (b) In graph (b) for one value of x
From Eqs. (i) and (ii), we get
displacement, there are two
different points of time. Hence, for t = 5 s and h = 125 m
A B
one time, the average velocity is  t
47 (a) Acceleration, f = f0 1 − 
positive and for other time is  T
equivalent negative.
dv  t  dv 
As there are opposite velocities in = f0 ⋅ 1 −  Q f = 
O T
t dt  T  dt 
the interval 0 to T, hence average
 t  t
velocity can vanish in (b). This can be seen in the figure alongside. ⇒ dv = f0 ⋅ 1 −  dt ⇒
 T ∫ dv = ∫ f0 1 − T  dt
Here, OA = BT (same displacement) for two different points of
time. t2
v = f0t − f0 +C …(i)
44 (a) As the lift is coming in downward x 2T
directions, displacement will be 8th floor where, C is constant.
negative. We have to see whether the When t = 0, v = 0 thus from Eq. (i), C = 0
motion is accelerating or retarding. x<0 f t2
We know that due to downward v = f0t − 0 ⋅ …(ii)
T 2
motion, displacement will be negative.
4th floor  t
When the lift reaches 4th floor and is x<0 As, f = f0 1 − 
0  T
about to stop, hence motion is Ground floor
retarding in nature, hence x < 0; a > 0. When f =0
 t
As displacement is in negative direction, velocity will also be ∴ f0 1 −  = 0 ; f0 ≠ 0 ⇒ t = T
 T
negative, i.e. v < 0.
Motion in One Dimension 129

Putting t = T in Eq. (ii), we get u 2  8 3 u 2 5


∴ AB =  −  = ⋅
f T 2 f0T 2g  9 4 2g 36
v = f0T − 0 ⋅ =
T 2 2 u 2  15 8 u 2 7
BC =  −  = ⋅
48 (d) Relative to lift, 2g  16 9 2g 144
u r = 0, a r = (9.8 + 1.2) = 11 ms −2 AB 5 144 20
∴ = × =
1 1 5.4 BC 36 7 7
sr = a rt 2 ⇒ 2.7 = × 11 × t 2 ⇒ t= s
2 2 11 53 (a) For the given condition, initial height h = d and velocity of
49 (a) After bailing out from point A, parachutist falls freely the ball is zero. When the ball moves downward, its velocity
under gravity. The velocity acquired by it will be v. increases and it will be maximum, when the ball hits the
ground and just after the collision, it becomes half and in
From v 2 = u 2 + 2as = 0 + 2 × 9.8 × 50 = 980 opposite direction. As the ball moves upwards, its velocity
(As, u = 0, a = 9.8 ms −2, s = 50 m) again decreases and becomes zero at height d /2 . This
At point B, parachute opens and it moves with retardation of explanation match with graph (a).
2 ms −2 and reach at ground (point C with velocity of 3 ms −1 ). 54 (c) The position x of a particle w.r.t. time t along X-axis,
For the part BC by applying the equation v 2 = u 2 + 2as x = 9t 2 − t 3 …(i)
v = 3 ms −1, u = 980 ms −1, a = − 2 ms −2, s = h Differentiating Eq. (i) w.r.t. time, we get speed, i.e.
dx d
⇒ (3)2 = ( 980 )2 + 2 × (−2) × h v= = (9t 2 − t 3 )
dt dt
⇒ 9 = 980 − 4h
⇒ v = 18 t − 3t 2 …(ii)
980 − 9 971
⇒ h= = = 242.7 ≅ 243 m Again, differentiating Eq. (ii) w.r.t. time, we get acceleration,
4 4
i.e.
So, the total height by which parachutist bails out
dv d
= 50 + 243 = 293 m a= = (18 t − 3 t 2 )
dt dt
50 (c) Initial relative velocity = v1 − v 2, final relative velocity = 0 ⇒ a = 18 − 6 t …(iii)
From v 2 = u 2 − 2as ⇒ 0 = (v1 − v 2 )2 − 2 × a × s Now, when speed of particle is maximum, its acceleration is
zero.
(v1 − v 2 )2
⇒ s= a=0
2a ⇒ 18 − 6 t = 0
If the distance between two cars is s, then collision will take ⇒ t =3s
place. To avoid collision d > s
Putting in Eq. (i), we obtain position of particle at that time
(v1 − v 2 )2 x = 9(3)2 − (3)3 = 9(9) − 27
∴ d >
2a
= 81 − 27 = 54 m
where, d = actual initial distance between two cars.
55 (c) Relative velocity of car A w.r.t. ground
2h vAg = 30 km/h
51 (b) Time taken by first drop to reach the ground, t =
g
2× 5 Relative velocity of car B w.r.t. ground
⇒ t= = 1s
10 vBg = 40 km/h
As the water drops fall at regular intervals from a tap,
1 ∴ Relative velocity of A w.r.t. B, N
therefore time difference between any two drops = s. vAB
2 v AB = v Ag − v Bg = v Ag + (−v Bg )
In this given time, distance of second drop from the tap |v AB | = (30 )2 + (40 )2 = 50 km/h
2 θ
1  1 10 W E
= g  = = 1.25 m 30 3
∴ tanθ = =
2  2 8 40 4
Its distance from the ground = 5 − 1.25 = 3.75 m ⇒ θ = tan−1(3/ 4) N of west
S
u2 C
52 (d) A ⇒ − u 2 = − 2gh1 56 (d)
4 Relative velocity of man w.r.t. ground
B vmg = 3 m/s
u2
B⇒ − u 2 = − 2gh2
9 A Relative velocity of rain w.r.t. ground,
h3 h2 h1
u2 vrg = 4 m/s
C⇒ − u 2 = − 2gh3 u
16 Relative velocity of rain w.r.t. man, v rm = ?
130 OBJECTIVE Physics Vol. 1

v rm = v rg − v mg = v rg + (−v mg ) (B) Medical entrance special format


3
β questions
α
l Assertion and reason
1 (c) When a particle is released from rest, v = 0 but a ≠ 0.
vrm
dv
4 2 (a) a = = slope of v-t graph.
dt
v rm = 5 m/s Perpendicular to t-axis, slope = ∞, therefore a = ∞
3 4
tanα = , tanβ = 3 (c) Slope of s-t graph = Velocity = Positive
4 3
At t = 0, s ≠ 0. Further at t = t0 : s = 0, v ≠ 0.
 3  4
v rm = 5 m/s at tan−1  with vertical or at tan−1  with Thus, v is constant.
 4  3
horizontal. 4 (b) By differentiating a-t equation two times, we will get s-t
equation.
57 (a) N
vbg = v Further
a s
30°

W E
vsg = 15 km/h
t t
S Straight line Parabola

where, v bg is the velocity of boat w.r.t. ground and v sg is the 5 (c) Ascending means velocity is in upward direction. Speed is
velocity of ship w.r.t. ground. decreasing. It means acceleration is downwards.
Relative velocity of boat w.r.t. ship v bs is along north. Further, the body moves in the direction of velocity.
v bs is resultant of v bg and v sg in opposite direction. l Statement based questions
v cos 30° = v bs 1 (a) From the given x-t graph, it is clear that
v sin 30 ° = 15 ⇒ v = 30 km/h at t = 0,
dx
=0
dt
58 (b) A B dx
∴ Velocity, v = = 0, at t = 0
dt
d dx
As time passes increases and then decreases.
dt
u : v rg (velocity of river w.r.t. ground) ∴ Velocity and hence, acceleration changes.
v : v mr (velocity of man w.r.t. river)
2 (b) x = 4(t − 2) + a (t − 2)2
d
Along the flow, v mg = v + u, 3 = ...(i) At t = 0, x = − 8 + 4a = 4a − 8
v +u
dx
where, v mg is velocity of man w.r.t. ground. v= = 4 + 2a (t − 2)
dt
Opposite to the flow,
d At t = 0, v = 4 − 4a = 4(1 − a )
v mg = v − u , 6 = ...(ii) d 2x
v −u But acceleration, a = = 2a
d dt 2
⇒ t0 = = 4h
v 3 (b) For maximum and minimum displacement, we have to keep
B in mind the magnitude and direction of maximum velocity.
59 (c)
As maximum velocity in positive direction is v 0 maximum
y velocity in opposite direction is also v 0 .
d = 500m 10 m/s
x Maximum displacement in one direction = v 0T
Maximum displacement in opposite directions = − v 0T
A 4 m/s
Hence, − v 0T < x < v 0T .
y : displacement = velocity × time 4 (c) Distance travelled in last second of upward journey is
500 = 10t independent of velocity of projection.
⇒ t = 50s 1 1
Distance travelled = g t 2 = × 9.8 × (1)2 = 4.9 m
2 2
Motion in One Dimension 131

Displacement 1 2 u m/s
5 (c) Average velocity = h = ut + gt
Time 2
Area of v - t graph 1 Window
= ⇒ 1. 5 = u × 0.1+ × 10 × (0.1)2 1.5 m
Time 2
(1/ 2) v m t0 1 ⇒ 1. 5 = 0.1u + 0.05
= = vm 1. 5 − 0.05 1. 45
t0 2 ⇒ u= = = 14. 5 m/s
0. 1 0. 1
l Match the columns 3 (b) For distance x, the person moves with constant velocity v1
1 (b) With constant positive acceleration, speed will increase and for another x distance, he moves with constant velocity of
when velocity is positive, speed will decrease, if velocity is v 2, then
negative. Total distance travelled by the person,
Similarly, with constant negative acceleration, speed will
increase, if velocity is negative and speed will decrease, if D = x + x = 2x
velocity is positive. Total time taken to cover that distance,
Hence, A → p,q, B → p,q, C → r, D → p,q. x x  Distance
T = t1 + t2 = + Q t = 
2 (c) At t = 3 s, s = 10 + 20 (3) − 5 (3) = 25 unit
2 v1 v 2  Velocity 
ds Average velocity,
v= = 20 − 10t
dt Total distance D
v av = =
At t = 4 s, v = 20 − 10 (4) = − 20 unit Total time T
dv 2x 2
a= = − 10 units v= = (Qv av = v )
dt x x 1 1
+ +
Hence, A → r, B → s, C → p. v1 v 2 v1 v 2
3 (c) vi = + 10 ms −1 and v f = 0 2 1 1
−1
⇒ = +
∴ ∆v = v f − vi = − 10 ms v v1 v 2
∆v − 10 − 5 4 (d) Given, speed of river, vR = 10 ms −1
a av = = = ms −2
∆t 6 3
Speed of swimmer in still water, v SN = 20 ms −1
Total displacement = Area under v-t graph (with sign)
vR
1 1 1 River flow N
= × 10 × 2 − × 2 × 10 − × 2 × 10 = − 10 m
2 2 2 vSN vS W E
and acceleration = slope of v-t graph q
10 S
=− = − 5 ms −2
2 For the shortest path to cross the river, he should swim at an
Hence, A → r, B → p, C → r, D → s. angle (90° + θ ) with the stream flow. From the above figure,
4 (b) For M, slope of s-t graph is positive and increasing. v SN = vR + v S
Therefore, velocity of the particle is positive and increasing. So, angle θ is given by
Hence, it is A type motion. Similarly, N, P and Q can be
vR 10 1
observed from the slope. sinθ = = = ⇒ θ = 30 °
v SN 20 2
Hence, A → r, B → s, C → p, D → q.
As the river is flowing in east direction, so he should swim
towards west.
(C) Medical entrances’ gallery 5 (d) When a particle completes one revolution in circular
1 (c) Given, u = 20 m/s, v = 80 m/s and h = ? motion, then average displacement travelled by particle is
zero.
From kinematic equation of motion, v 2 = u 2 + 2gh Hence, average velocity
v 2 − u 2 (80 )2 − (20 )2 average displacement 0
⇒ h= = (Q Given, g = 10 m/s 2 ) = = =0
2g 2 × 10 ∆t ∆t
= 300 m 6 (b) Time interval between 8th and 3rd second,
2 (b) According to question, time taken by the ball to cross the ∆t = 8 − 3 = 5 s, i.e. ∆t = 5s
window, Change in velocity,
t = 0.1s , h = 1.5 m ∆v = 20 − 0 = 20 ms −1
If u be the velocity at the top most point of the window, then ∆v 20
from equation of motion, ∴Average acceleration = = = 4 ms −2
∆t 5
132 OBJECTIVE Physics Vol. 1

1 1 1
7 (b) According to the question, =× 1× 6 + × 1× 6 − × 6 × 1 = 3 m
For the time duration 0 < t < 1s, 2 2 2
3 −1
the velocity increase from 0 to 6 ms −1. ∴ Average velocity = = 1ms .
3
As the direction of field has been reversed, for 1 < t < 2 s, the
velocity firstly decreases from 6 ms −1 to 0. Total distance travelled, d = 9 m
9
Then, for 2 < t < 3 s; as the field strength is same; the ∴ Average speed = = 3 ms −1
magnitude of acceleration would be same, but velocity 3
increases from 0 to − 6 ms −1. 8 (c) When a particle is released from rest position under
0<t<1s
gravity, then v = 0, but a ≠ 0.
B 1< t < 2 s
A C Thus, Assertion is correct but Reason in incorrect.
v=0 a v = 6 ms -1 −a
v=0
D 9 (a) To find value of time at which velocity is maximum,
v = − 6 ms-1 − a taking differentiation of v with respect to time
2 < t < 3s dv
=0
Acceleration of the car, dt
v −u 6−0 Given, v = 4t (1− 2t )
|a | = = = 6 ms −2
t 1 d
v = 4t − 8t 2 ⇒ (4t − 8t 2 ) = 0
The displacement of the particle is given as dt
1 1
s = ut + at 2 ⇒ 4 − 16t = 0 ⇒ t = s = 0.25 s
2 4
For t = 0 to t = 1 s, Again, taking differentiation, we get
1
u = 0, a = + 6 ms −2 ⇒ s1 = 0 + × 6 × (1)2 = 3 m ⇒
d 2v
= −16 < 0
2
dt 2
For t = 1 s to t = 2 s, So, at t = 0.25 s, velocity is maximum.
u = 6 ms −1, a = − 6 ms −2 10 (a) Q Runner starts from O and goes to O following the path
1 OQRO, so net displacement is zero.
⇒ s2 = 6 × 1 − × 6 × (1)2 = 6 − 3 = 3 m
2 Total distance OQ + QR + RO
Q Average speed = =
For t = 2 s to t = 3 s, Total time Total time
1 km + (2πr ) (1/ 4) + 1 km
1 =
u = 0, a = − 6 ms −2 ⇒ s3 = 0 − × 6 × (1)2 = − 3 m 1h
2
π
∴ Net displacement, = 2 + = 3.57 kmh −1
2
s = s1 + s2 + s3 = 3 m + 3 m − 3 m = 3 m
Hence, average velocity 11 (c) Time taken to reach the highest point from the height h is
obtained from equation, v = u − gt
net displacement 3
= = = 1 ms −1 ∴ 0 = u − gt or t =
u
total time 3 g
Total distance travelled, d = 9 m
(Q At highest point, final velocity of ball = 0)
Hence, average speed
Height attained above h is obtained from v 2 − u 2 = 2 (− g )h1
total distance 9
= = = 3 ms −1
total time 3 u2
or 0 − u 2 = 2 (− g )h1 or h1 =
Alternate Method 2g
Given condition can be represented through graph also as  u2 
shown below. Total height, h2 = h1 + h = + h
 2g 
v
(ms−1) + 6 A Time taken to hit the ground is obtained from
1 u2 1
O′ t =2 t =3 h2 = ut + at 2 or + h = 0 + gt 2
2 2g 2
O t=1 B D t (s)
(u 2 + 2g h )
−6 ∴ Total time taken, t =
C g
∴ Displacement in three seconds h
12 (c) Speed of walking, v1 =
= Area under the graph t1
h
= Area of ∆OAO′ + Area of ∆AO ′B − Area of ∆BCD Speed of escalator, v 2 =
t2
Motion in One Dimension 133

Time taken when she walks over moving escalator, 16 (b) Given, v = βx −2n
h dv dx dv
t= a= = ⋅
v1 + v 2 dt dt dx
1 v1 v 2 1 1 dv
⇒ = + = + ⇒ a =v = (βx −2n )(−2nβx −2n −1)
t h h t1 t2 dx
tt12 ⇒ a = − 2nβ 2x −4 n −1
⇒ t=
t1 + t2 17 (c) Let the ball hit water in t second.
13 (c) For the given v-x graph, 1
For first ball, s = ut + at 2
Slope = −v 0 / x 0 2
1
and intercept = v 0 122.5 = 0 + × 9.8 × t 2 = 4.9 t 2
2
From the general equation of straight line, i.e.
122.5
y = mx + c ⇒ t= = 25 = 5 s
4.9
where, m is slope and c is the intercept.
1
The equation of motion of the v-x graph can be given as For second ball, 122.5 = u (5 − 2) + × 9.8 × (5 − 2)2
2
v x
v = − 0 + v0 … (i) = 3u + 44.1
x0
⇒ 3u = 122.5 − 44.1
(−ve sign signifies that the slope is decreasing) 3u = 78.4 ⇒ u = 26.1ms −1
On differentiating Eq. (i) with respect to t, we get
18 (c) Given, upward velocity of ball = 25.2 ms −1
dv v dx v
=− 0 + 0 or a = − 0 v … (ii) Height attained by the ball,
dt x 0 dt x0
u 2 25.2 × 25.2
Putting the value of v from Eq. (i) in Eq. (ii), we get H= = = 32.4 m
v  v  v2 v2 2g 2 × 9.8
a = − 0 − 0 x + v 0  = 02 x − 0
x0  x0  x0 x0 Now, time taken by the ball to attain 32.4 m is
u 25.2
This equation shows that at x = 0, t= = = 2.57s
g 9.8
v2
a = − 0 = constant and at a = 0, x = x 0 .
x0 19 (c) According to question, the given condition can be depicted as
So, option (c) is correct.
u Mid-point v
14 (b) For a car in motion, if we describe this event w.r.t. a
frame of reference attached to the person sitting inside the A C B
car, the car will appear to be at rest as the person inside the s/2 s/2
car (i.e. observer) is also moving with same velocity and in the Time t
same direction as car. Time 2t

15 (b) Velocity of the particle is given as v = At + Bt 2


For motion from A to C,
where, A and B are constants. s 1
= 2 ut + a × (2t )2 = 2ut + 2at 2 …(i)
dx  dx  2 2
⇒ = At + Bt 2 Qv = 
dt  dt  Also, for motion from C to B,
⇒ dx = (At + Bt 2 ) dt s 1 1
= v ′ t + at 2 = (u + 2at )t + at 2
Integrating both sides, we get 2 2 2
x2 2 (Let initial velocity be v ′ = u + 2at )
∫x 1
dx = ∫ (At + Bt 2 )dt
1
= ut + 2at + at
2 1 2
2 2 2
⇒ ∆x = x 2 − x1 = A ∫ t dt + B ∫ t 2dt
1 1 Now, from Eq. (i), we get
2 2 1 at
t 2  t 3  2ut + 2at 2 = ut + 2at 2 + at 2 ⇒ u =
=A  + B   2 2
 2 1  3 1 For overall motion,
A B
= (22 − 12 ) + (23 − 13 ) at 7at  at 
v = u + 3at = + 3at = = 7u Q u = 
2 3 2 2  2
∴ Distance travelled by the particle between 1s and 2s is
20 (d) At first, the slope is decreasing, therefore the motion is
A B 3A 7B
∆x = × (3) + (7) = + retarded. Finally, the displacement becomes constant, thus the
2 3 2 3 particle stops.
134 OBJECTIVE Physics Vol. 1

21 (d) Given, u = 0, v = 180 kmh −1 = 50 ms −1 Total distance travelled by the body (d )


= ...(i)
v − u 50 Total time taken (t1 + t2)
Time taken, t = 10s, a = = = 5 ms −2
t 10 So, now time taken in 1st half of the distance,
Distance covered by the car, Distance d
t1 = Time = =
1 1 Velocity 2 × 40
s = ut + at 2 = 0 + × 5 × (10 )2
2 2 d d
⇒ t1 = (Q Distance = , velocity = 40 kmh −1)
500 80 2
= = 250 m
2 d
Time taken for 2nd half, t2 =
22 (d) The slope of velocity-time graph gives acceleration, 2 × Velocity
a A tan θ1 d d
i.e. = So, t2 = =
a B tan θ 2 2 × 60 120
Here, θ1 is the angle made by line A with time axis and θ 2 is d
(Q distance = , velocity = 60 kmh −1)
the angle made by line B with time axis 2
a A tan 25° d
⇒ = Now, average velocity, v av = [From Eq. (i)]
a B tan 50 ° d d
+
80 120
23 (a) The position of an object is always expressed w.r.t. s2
d × 80 × 120
some reference point. If the initial position of an object = = 48 kmh −1
w.r.t. a reference points is s1 and after sometime, it 200d
changes to s2, then the magnitude of the displacement 1
27 (a) The distance covered in 2 s, x = a (2)2 (Q u = 0)
of the object is s2 − s1. Since, the given ball returns to h 2
its starting point in 10 s. So, in 5s, it reaches the = 2a
highest point which is at s2.
The distance covered in next 2 s,
Given, s2 is at a height h w.r.t. s1. Thus, the 1 1
displacement after 5s will be s2 − s1 = h. s1 y = a (4)2 − a (2)2 = 6a
2 2
2a 1
Now, x /y = = ⇒ y = 3x
24 (a) Let v PG = velocity of police w.r.t. ground, 6a 3
vTG = velocity of thief w.r.t. ground, 28 (c) A → v A (constant velocity)
vTP = velocity of thief w.r.t. police = vTG − v PG
B → a (constant acceleration)
 153 − 45 −1
=  × 5 = 30 ms C be the fixed point at which both A and B meet in time t.
 18 
Now, s = v At and v B = at
v BC = velocity of bullet w.r.t. car A → vA =v
= 180 − 30 = 150 ms −1 B→a C
1
25 (b) We know that, snth = u + a (2n − 1) O
s
2
1 5 1
s3rd = 0 + a (2 × 3 − 1) = a (For n = 3s ) s = at 2 = v At
2 2 2
1 7 2v
s4rd = 0 + a (2 × 4 − 1) = a (For n = 4s ) ⇒ a= A …(i)
2 2 t
s4 th − s3rd vB = u + at (Qv = 0)
So, the percentage increase = × 100
s3rd ⇒ vB = at = 2vA [From Eq. (i)]
7 5 2a v BA = velocity of B w.r.t. A
a− a ⇒ vBA = vB − vA = 2vA − vA = vA
= 2 2 × 100 = 2 × 100 = 2 × 20 = 40%
5 5 ⇒ vBA = v (Q vA = v )
a a
2 2 29 (b) vAB = Speed of car from A to B
26 (b) As from the question, A B
d D
A B vAB = 30 kmh −1
C
d/2 d/2 v BA = Speed of car from B to A
t1 t2 vBA = 20 kmh −1
v = 40 kmh–1 v = 60 kmh–1 2v v
vav = AB BA = 24 kmh −1
Average speed of the car, v av vAB + vBA
Motion in One Dimension 135

36 (a) Given, velocity of water, v w = 4 ms −1


A x1 C x2 B
30 (c) (0, 0)
u=0 t/2 t/2 Velocity of person, v p = 5 ms −1, width of the river = 84.6 m
a = constant s
Let A be the original point. We know that, time taken, t =
v p2 − vW
2
From equation of motion,
1 1 84.6
x = ut + at 2 = at 2 (Q u = 0 ) ⇒ t=
2 2 25 − 16
1 1 84.6 ~
For point C, x1 = a (t /2)2 ⇒ x1 = at 2 ⇒ t= = 28.2 s
2 8 3
Distance travelled by the body in t second is
37 (c) From graph, velocity of robber’s car = 10 ms −1
1
x1 + x 2 = a (t )2 = 4x1 Let police car crosses it after t second.
2
x 2 = 4x1 − x1 = 3x1 Distance travelled by robber’s car = 10 t m
Police car is moving with a constant acceleration of 1 ms −2 as
31 (d) The acceleration of a moving body is found from the slope
of velocity-time graph. it attains a velocity of 10 ms −1 in 10 s after starting from rest.
1 1
Y Distance travelled in t second = ⋅ a ⋅ t 2 = t 2
2 2
When the police car crosses the robber’s car, distance
travelled by the both cars should be same from the starting
Velocity

point of chase.
1 2
∴ t = 10 t ⇒ t = 20 s
2
38 (b) Given, initial speed, u = 1kms −1 = 1000 ms −1
X
0 Time

a=
dv
= Slope of v-t curve Final speed, v = 9 kms −1 = 9000 ms −1
dt
By using the relation, v 2 = u 2 + 2as
32 (b) The distance h1 covers by stone,
(9000 )2 = (1000 )2 + 2 × a × 4 ⇒ a = 10 7 ms −2
1 1
h1 = gt 2 × 10 × (5)2 = 125 ∴ The time for which the particle remains in the tube
2 2
v = u + at
The distance h2 covers by stone,
v − u 9000 − 1000
1 1
h2 = × 10 × (10 )2 − × 10 × (5)2 = 375 ⇒ t= = = 8 × 10 −4 s
2 2 a 10 7
The distance h3 covers by stone, 39 (d) Given, x = 8 + 12 t − t 3
1 1
h3 = × 10 × (15)2 − × 10 × (10 )2 = 625 We know that , v =
dx
and a =
dv
2 2 dt dt
h2 h3
The relation between h1, h2 and h3 is h1 = = So, v = 12 − 3t 2
3 5
and t = 2 s,
33 (b) When velocity of a body changes by equal amount in
equal intervals of time, then the body is said to have uniform v =0
acceleration, this holds true for straight line motion. and a = − 12 ms −2
34 (c) Velocity-time graph gives the instantaneous value of So, retardation of the particle = 12 ms −2.
acceleration at any instant. For non-uniformly accelerated
40 (d) Distance, x = b 0 + b1t + b 2t 2
motion, v-t graph is non-linear.
 dx 
35 (b) The initial velocity of the body, u 2 = 2 gh Velocity, v =   = b1 + 2 b 2t
 dt 
⇒ u = 2gh = 2 × 10 × 125 = 2500 = 50 ms −1
d 2x
The final velocity from equation of motion, Acceleration, a = = 2 b2
dt 2
v 2 = u 2 + 2 gh
41 (c) Let t second be the time of flight of the first body after
v = (50 )2 + 2 × 10 × 55 meeting, then (t − 4) second will be the time of flight of the
second body.
v = 2500 + 1100
As the initial velocity at which the bodies A and B projected
= 3600 = 60 ms −1 are same and also the position of meeting will be also same.
136 OBJECTIVE Physics Vol. 1

So, hx = hy At t = 1s,
1 2 1 dr
∴ 98t − gt = 98 (t − 4) − g (t − 4)2 v1 = 1 = 3$i + 8$j
2 2 dt
On solving, t = 12 s Again, r2 (t ) = 4t 2$i + 3t$j
42 (b) Let after time t, car catches the scooter and the distance dr2
⇒ = 8t$i + 3$j
travelled by scooter in time t, dt
1 t2 At t = 1s,
x = × (1) × t 2 = …(i)
2 2 dr
v 2 = 2 = 8$i + 3$j
The distance travelled by car in time t dt
1
x + 150 = × 2 × t 2 = t 2 …(ii) Relative speed, vrel = v 2 − v1
2 = (8$i + 3$j ) − (3$i + 8$j )
Solving Eqs. (i) and (ii), we get
= 5$i − 5$j
t = 300 s
∴ |vrel| = (5)2 + (− 5)2
43 (c) Given, r1(t ) = 3 t $i + 4 t 2$j
= 5 2 m/s
dr1
∴ = 3$i + 8t$j
dt
CHAPTER
04

Motion in a Plane
and Projectile
Motion
In this chapter, we will study motion of those particles which do not move in a
straight line rather, they move in a plane. In our daily life, examples of motion in
a plane are motion of a football, circular motion, motion of a stone fired from a
slingshot (gulel), etc.

MOTION IN A PLANE
(TWO DIMENSIONAL MOTION)
Motion of an object is called two dimensional motion when two of the coordinates
(x-y, y-z or z-x) from the three coordinates (x, y, z ) are required to specify the
change in position of the object in space, with respect to time. In two dimensional
motion, the object moves in XY-plane, YZ-plane or ZX-plane, therefore it is
called motion in a plane. e.g. Projectile motion, circular motion, etc.
When an object moves in a plane or in two dimensional motion, different physical Inside
quantities related with it changes with time. e.g. Position vector, displacement
1 Motion in a plane
vector, velocity vector and acceleration vector. In XY-plane, these physical (Two dimensional motion)
quantities are studied in terms of their x, y-components. Position vector
Displacement vector
Position vector Velocity vector
Acceleration vector
A vector that extends from a reference point to the point at which particle is
located is called position vector. 2 Projectile

Let r be the position vector of a particle P located in a plane with reference to the Projectile motion
Projectile projected obliquely on
origin O in XY-plane as shown in the figure.
the surface of the earth
OP = OA + OB Y
Projectile fired at an angle
with the vertical
Position vector, r = x$i + y $j
3 Projectile from a point
B P
Direction of this position vector r is given above the ground
y ^j r Projectile projected from
by the angle θ with X-axis, where a tower
θ
 y X Shooting a freely falling target
tanθ =   O x ^i A
x Fig. 4.1 Representation of position vector
138 OBJECTIVE Physics Vol. 1

 y ⇒ Magnitude of displacement, | ∆r| = (∆x ) 2 + (∆y ) 2


⇒ θ = tan −1  
x
= (x 2 − x1 ) 2 + ( y 2 − y1 ) 2
In three dimensions, the position vector is represented as
Y
r = x $i + y$j + z k$
∆r ∆yj^
Example 4.1 A particle moves in a plane such that its
θ
coordinates changes with time as x = at and y = bt, where a
∆x^i
and b are constants. Find the position vector of the particle
O X
and its direction at any time t.
Sol. Coordinates of the particle are x = at and y = bt Fig. 4.3 Component of displacement

∴ Position vector of the particle at any time t is Direction of the displacement vector ∆r is given by
r = x $i + y $j = (at )$i + (bt )$j
∆y  ∆y 
y
−1   bt  b tanθ = ⇒ θ = tan −1  
Direction of r, θ = tan   = tan−1   = tan−1  ∆x  ∆x 
 x  at  a 
where, θ = angle made by ∆r with X-axis.
Displacement vector Similarly, in three dimensions, the displacement vector
can be represented as
Consider a particle moving in XY-plane with a uniform
velocity v and point O as an origin for measuring time and ∆r = (x 2 − x 1 ) $i + ( y 2 − y 1 ) $j + (z 2 − z 1 ) k$
position of the particle. Let the particle be at positions A
Note Magnitude of displacement (∆r ) between two points is always
and B at timings t1 and t 2 , respectively. The position
less than or equal to distance ( s) between corresponding points.
vectors are OA = r1 and OB = r2 .
i. e. ∆r≤s
Y
Example 4.2 An object moves from position (3,4) to (6,5) in
y2 the XY-plane. Find the magnitude and direction of
∆y A ∆r B displacement vector of the particle.
y1 Sol. Position vectors of the particle are
r1 r2
r1 = x1i$ + y1$j = 3i$ + 4$j and r 2 = x 2i$ + y 2$j = 6i$ + 5$j

X
∴ Displacement vector, ∆r = (x 2 − x1)i$ + ( y 2 − y1)j$
O x1 x2
∆x = (6 − 3) i$ + (5 − 4)j$ = 3i$ + j$
Fig. 4.2 Representation of displacement vector ∴ Magnitude of displacement vector,
Then, the displacement of the particle in time interval | ∆r| = (3)2 + (1)2 = 10
(t 2 − t1 ) is AB. From triangle law of vector addition, we get Direction of ∆r with X-axis,
OA + AB = OB  ∆y   1
θ = tan−1  = tan−1  ≈ 18.43°
⇒ AB = OB − OA  ∆x   3
AB = r2 − r1 …(i)
If the coordinates of the particle at points A and B are Velocity vector
(x 1, y 1 ) and (x 2, y 2 ) , then Velocity of an object in motion is defined as the ratio of
r = x i$ + y j$ displacement and the corresponding time interval taken by
1 1 1
the object, i.e.
and r2 = x 2 i$ + y 2 j$
Displacement
Substituting the values of r1 and r2 in Eq. (i), we get Velocity =
Time interval
AB = (x i$ + y j$ ) − (x i$ + y $j )
2 2 1 1
Velocity is a vector quantity as it has both the magnitude
Displacement, AB = (x 2 − x1 ) i$ + ( y 2 − y1 ) $j (speed) and direction.
It is of two types
or Displacement, ∆r = ∆x$i + ∆y$j (i) Average velocity (ii) Instantaneous velocity
Motion in a Plane and Projectile Motion 139

Average velocity Instantaneous velocity


It is defined as the ratio of the displacement and the The velocity of the object at an instant of time (t ) is known
corresponding time interval. as instantaneous velocity. The average velocity will
displacement become instantaneous, if ∆t approaches to zero.
Thus, average velocity =
time taken ∴ Instantaneous velocity,
∆r r2 − r1 ∆r dr
Average velocity, v av = = v = lim =
∆t t 2 − t1 ∆t → 0 ∆ t dt
Y Now, we can write
d r = dx$i + dy$j
∆v ∆vy ^j
dx$i + dy$j dx $ dy $
θ ∴ v= = i+ j
∆vx ^i dt dt dt
X
O
⇒ v = v x $i + v y $j
Fig. 4.4 Components of velocity
Similarly, in three dimensions, we can write
Velocity can be expressed in the component form as
v = v $i + v $j + v k$
∆x $ ∆y $ x y z
v av = i+ j = ∆v x $i + ∆v y $j dx
∆t ∆t where, v x = is magnitude of instantaneous velocity in
where, ∆v x and ∆v y are the components of average dt x-direction,
velocity along x-direction and y-direction, respectively. dy
vy = is magnitude of instantaneous velocity in
The magnitude of v av is given by dt y-direction
dz
and vz = is magnitude of instantaneous velocity in
v av = ∆v x2 + ∆v y2 dt z-direction.

and the direction of v av is given by angle θ Magnitude of instantaneous velocity, | v | = v x2 + v y2

∆v y Direction of instantaneous velocity v with X-axis,


tan θ = (From X-axis)
∆v x Y

Example 4.3 A particle moves in XY-plane from position


(1m, 2m) to (3m, 4m) in 2 s. Find the magnitude and
direction of average velocity. vy v
Sol. Given, position vectors of the particle are θ X
vx
r = x i$ + y $j = i$ + 2$j
1 1 1
Fig. 4.5 Direction of instantaneous velocity
and r 2 = x 2i$ + y 2j$ = 3i$ + 4j$
vy vy 
Displacement, ∆r = r 2 − r1 = 2i$ + 2j$ tanθ = ⇒ θ = tan −1  
vx vx 
∆r
∴ Average velocity, v av =
∆t Example 4.4 Position vector of a particle is given as
2i$ + 2j$ r = 2t i$ + 3t 2 $j
⇒ v av =
2 where, t is in seconds and the coefficients have the proper units,
for r to be in metres.
⇒ v av = ∆v xi$ + ∆v y j$ = (i$ + j$ ) ms −1 (i) Find instantaneous velocity v (t ) of the particle.
(ii) Find magnitude and direction of v (t ) at t = 2s.
⇒ | v av | = (1)2 + (1)2 = 2 ms −1
Sol. Given, r = 2t$i + 3t 2$j
Direction of average velocity with X-axis,
dr d $
(i) Instantaneous velocity, v (t ) = = (2t i + 3t 2$j )
−1  ∆ v y
 −1 1 dt dt
θ = tan   = tan   = 45°
 ∆v x  1 ⇒ v = v xi$ + v y $j = 2i$ + 6t $j
140 OBJECTIVE Physics Vol. 1

(ii) Magnitude of v (t ), Angle θ made by average acceleration with X-axis is


| v (t )| = v x2 + v y2 = (2) + (6 t ) = 4 + 36 t
2 2 2
a (av) y  a (av) y 
At t = 2s, | v (t )| = 4 + 36 × 4 = 148 ms −1 tanθ = ⇒ θ = tan −1  
a (av) x  a (av) x 
vy   6t
Direction of v (t ), θ = tan−1  = tan−1   = tan−1(3 t )
 vx  2 Example 4.5 Velocity of a particle changes from (3i$ + 4j$ ) m/s
At t = 2 s, θ = tan−1 (3 × 2 ) ≈ tan−1(6) rad to (6 i$ + 5 j$ ) m/s in 2 s. Find magnitude and direction of
average acceleration.
Sol. Given, velocity vectors of the particle,
Acceleration vector v = 3i$ + 4j$
1
It is defined as the rate of change of velocity. It can be
expressed as and v 2 = 6i$ + 5j$
Change in velocity Change in velocity,
Acceleration =
Time taken ∆v x = (v 2 )x − (v1)x i$ = (6 − 3)i$ = 3i$

It is of two types as follows ∆v y = (v 2 )y − (v1)y )j$ = (5 − 4)j$ = j$


(i) Average acceleration ∴ ∆v = ∆v x + ∆v y = 3i$ + j$
(ii) Instantaneous acceleration
Average acceleration,
Average acceleration ∆v 3i$ + j$
a av = = (Q t = 2 s)
It is defined as the change in velocity (∆v ) divided by ∆t 2
the corresponding time interval (∆t ). It can be expressed as = 1.5 i + 0.5j$
$

Y Direction of average acceleration,


a (av) y 0.5 1
tan θ = = =
a (av) x 1.5 3
aav  1
a(av)y ^
j ⇒ θ = tan−1 
θ  3
a(av)x^
i
X ≈ 18.43° with X-axis
O
Fig. 4.6 Components of acceleration
Instantaneous acceleration
∆v ∆v x $i + ∆v y $j It is defined as the limiting value of the average
Average acceleration , a av = = acceleration as the time interval approaches to zero.
∆t ∆t
∆ It can be expressed as
∆v x $ v y $
= i+ j
∆t ∆t ∆v dv
a = lim =
Average acceleration = a (av) x $i + a (av) y $j ∆t → 0 ∆t dt

which is expressed in component form Instantaneous acceleration,


∆v x a = a x $i + a y $j
where, a (av)x = = average acceleration in x-direction where,
∆t
a x = magnitude of instantaneous acceleration in x-direction
∆v y
and a (av)y = = average acceleration in y-direction. dv
= x
∆t dt
In three dimensions, we can write a y = magnitude of instantaneous acceleration in y-direction
a =a $i + a $j + a k$ dv y
av (av) x (av) y (av) z
=
Magnitude of average acceleration is given by dt
The magnitude of instantaneous acceleration is given by
| a av | = (a (av) x ) 2 + (a (av) y ) 2
a = a x2 + a y2
Motion in a Plane and Projectile Motion 141

If acceleration a makes an angle θ with X-axis, then In terms of rectangular components, we can express it as
ay ay  v x = v 0x + a x t and v y = v 0y + a y t
tanθ = ⇒ θ = tan −1   It can be concluded that, each rectangular component of
ax  ax  velocity of an object moving with uniform acceleration in
Y
a plane depends upon time as if it were the velocity vector
of one dimensional uniformly accelerated motion.

ay
a Path of particle under
O
θ
X
constant acceleration
ax
Now, we can also find the position vector (r ). Let r0 and r
Fig. 4.7 Direction of instantaneous acceleration be the position vectors of the particle at time t = 0 and
In three dimensions, we can write t = t and their velocities at these instants be v 0 and v,
a = a x $i + a y $j + a z k
$ respectively. Then, the average velocity is given by
v +v
Example 4.6 The position of a particle is given by v av = 0
2
r = 3 t $i + 2 t 2 $j + 8 k$
where, t is in seconds and the coefficients have the proper
Displacement is the product of average velocity and time
units for r to be in metres. interval.
(i) Find v (t ) and a (t ) of the particle. It is expressed as
(ii) Find the magnitude and direction of v (t ) and a (t ) at t = 1s.  v + v0   (v0 + at ) + v0 
r − r0 =  t = t
Sol. Position of particle, r = 3 t i$ + 2 t 2j$ + 8 k$  2   2 
dr dv 1
(i) As, v (t ) = and a (t ) = ⇒ r − r0 = v0 t + at 2
dt dt 2
d
∴ v (t ) = (3 t i + 2 t j + 8 k$ ) = 3 i$ + 4 t j$
$ 2$
⇒ r = r0 + v 0 t +
1 2
at
dt
2
dv
a (t ) = = 4 $j
dt In terms of rectangular components, we get
vy  1
x i$ + y $j = x 0 i$ + y 0 $j + (v 0x i$ + v 0y $j ) t + (a x i$ + a y j$ ) t 2
(ii) v (t ) = v x2 + v y2 and θ = tan−1 
 vx  2
Now, equating the coefficients of i$ and $j ,
Velocity, v (t ) = 3 i + 4 t j
$ $
At t = 1 s, 1
−1 x = x 0 + v 0x t + a x t 2 ....... along X-axis
v = (3) + (4) = 5 ms
2 2
2
 4
⇒ Direction of v (t ) = θ = tan−1   = 53° with X-axis 1
 3 and y = y 0 + v 0y t + a y t 2 ....... along Y-axis
2
Direction of a (t ), at t = 1 s,
ay   4 Note Motion in a plane (two dimensional motion) can be treated as
θ′ = tan−1   = tan−1  = tan−1(∞ ) = 90° with X-axis two separate simultaneous one dimensional motions with
ax   0
constant acceleration along two perpendicular directions.
Motion in plane with uniform dv d |v |
Example 4.7 (i) What does and represent?
acceleration (ii) Can these be equal?
dt dt
An object is said to be moving with uniform acceleration, d |v | dv
if its velocity vector undergoes the same change in the (iii) Can = 0 while ≠ 0?
dt dt
same interval of time (however small).
d |v | dv
Let an object is moving in XY-plane and its acceleration a (iv) Can ≠ 0 while = 0?
dt dt
is constant. At time t = 0, the velocity of an object be v 0
(say) and v be the velocity at time t. dv
Sol. (i) is the magnitude of total acceleration. While
According to definition of average acceleration, we get dt
v − v0 v − v0 d | v|
represents the time rate of change of speed (called
a= = ⇒ v = v0 + at dt
t −0 t
142 OBJECTIVE Physics Vol. 1

the tangential acceleration, a component of total Example 4.9 An object has a velocity v = (2i$ + 4j$ ) ms −1 at
acceleration) as | v| = v. time t = 0 s. It undergoes a constant acceleration
(ii) These two are equal in case of one dimensional motion a = ($i − 3$j) ms −2 for 4s. Then,
(without change in direction). (i) find the coordinates of the object, if it is at origin at t = 0.
(iii) In case of uniform circular motion, speed remains (ii) find the magnitude of its velocity at the end of 4s.
constant while velocity changes.
Sol. (i) Here, initial position of the object,
d | v| dv
Hence, = 0 while ≠ 0. r 0 = x 0i$ + y 0$j = 0i$ + 0$j
dt dt
d | v| Initial velocity, v 0 = v 0x i$ + v 0y $j = 2i$ + 4 $j
(iv) ≠ 0 implies that speed of particle is not constant.
dt Acceleration, a = a xi$ + a y $j = i$ − 3 $j
Velocity cannot remain constant, if speed is changing.
dv and t = 4 s
Hence, cannot be zero in this case. So, it is not Let the final coordinates of the object be (x, y ). Then,
dt
according to the equation for the path of particle under
dv d | v| constant acceleration,
possible to have = 0 while ≠ 0.
dt dt 1 1
x = x 0 + v 0xt + a xt 2 = 0 + 2 × 4 + (1 ) × 42
2 2
Example 4.8 A particle starts from origin at t = 0 with a 1
velocity of 15 $i ms −1 and moves in XY-plane under the action ⇒ x = 16 m and y = y 0 + v 0y t + a y t 2
2
of a force which produces a constant acceleration of 1
(15 i$ + 20j$ ) ms −2 . Find the y-coordinate of the particle at the = 0 + 4 × 4 + (−3) × 4 ⇒ y = − 8 m
2
2
instant when its x-coordinate is 180 m.
Therefore, the object lies at (16 i$ − 8j$ ) at t = 4 s.
Sol. The position of the particle is given by
1 1 (ii) Using equation
r (t ) = v 0t + at 2 = 15i$ t + (15i$ + 20j$ ) t 2 v = v0 + at
2 2
⇒ v = (2i$ + 4j$ ) + (i$ − 3$j ) × 4
= (15t + 7.5t )i + 10t j
2 $ 2$

= (2i$ + 4j$ ) + (4i$ − 12j$ ) = (2 + 4) i$ + (4 − 12) j$


∴ x (t ) = 15 t + 7.5 t 2 and y (t ) = 10t 2
⇒ v = 6i$ − 8j$
If x (t ) = 180 m, t = ?
∴ Magnitude of velocity, | v | = 62 + 82 = 10 ms −1
180 = 15 t + 7.5 t 2 ⇒ t = 4 s
 − 8
Its direction with X-axis, θ = tan−1  ≈ − 53°
∴ y -coordinate, y (t ) = 10 × 16 = 160 m  6 

CHECK POINT 4.1


1. The x and y-components of a position vector P have 4. The displacement of an object along the three axes are
numerical values 5 and 6, respectively. Direction and given by, x = 2 t 2 , y = t 2 − 4 t and z = 3 t − 5. The initial
magnitude of vector P are velocity of the particle is
(a) tan−1   and 61
6 (a) 10 units (b) 12 units
 5 (c) 5 units (d) 2 units

(b) tan−1   and 61


5 5. A particle moves along the positive branch of the curve
 6 x2 t2
y= , where x = , x and y are measured in metres and t
(c) 60° and 8 2 2
(d) 30° and 9 in second. At t = 2 s, the velocity of the particle is
(a) (2$i − 4$j) ms −1 (b) (4$i + 2$j) ms −1
2. An object moves from positions (6, 8) to (12, 10) in the
XY-plane. Magnitude and direction of displacement is (c) (2$i + 4$j) ms −1 (d) (4$i − 2$j) ms −1
(a) 40 and18.43° (b) 40 and 71.56° 6. The position vector of a particle is
(c) 10 and 53° (d) 244 and 53° r = a sin ωt $i + a cos ωt $j
3. A particle moves in XY-plane from positions (2 m, 4 m) to The velocity of the particle is
(6 m, 8 m) is 2 s. Magnitude and direction of average (a) parallel to position vector
velocity is (b) perpendicular to position vector
(c) directed towards the origin
(a) 2 ms−1 and 45° (b) 2 2 ms−1 and 45°
(d) directed away from the origin
(c) 4 2 ms−1 and 30° (d) 3 2 ms−1 and 60°
Motion in a Plane and Projectile Motion 143

7. The position vector of an object at any time t is given by 10. A particle’s velocity changes from (2$i + 3$j) ms −1 to
3 t 2$i + 6 t$j + k$ . Its velocity (in m/s) along Y-axis has the (3$i − 2$j) ms −1 in 2 s. Its average acceleration (in ms −2) is
magnitude (a) − ($i + 5$j) (b) ($i + 5$j)/ 2 (c) zero (d) ($i − 5$j)/ 2
(a) 6t (b) 6
(c) 0 (d) 9 11. A particle has an initial velocity of 4 $i + 3$j and an
8. The coordinates of a moving particle at any time t are given acceleration of 0.4$i + 0.3$j. Its speed after 10 s is
by, x = 2 t 3 and y = 3 t 3. Acceleration of the particle is given (a) 10 units (b) 7 units (c) 7 2 units (d) 8.5 units
by
(a) 468 t (b) t 468
12. A body lying initially at point (3 , 7) starts moving with a
(c) 234 t 2 (d) t 234 constant acceleration of 4 $i. Its position after 3 s is given by
the coordinates
9. The position of a particle moving in the XY-plane at any (a) (7, 3) (b) (7, 18) (c) (21, 7) (d) (3, 7)
time t is given by x = (3 t 2 − 6 t) m, y = (t 2 − 2 t) m. Select the
13. The initial position of an object at rest is given by 3$i − 8$j. It
correct statement about the moving particle from the
following. moves with constant acceleration and reaches to the
(a) The acceleration of the particle is zero at t = 0 s position 2$i + 4 $j after 4 s. What is its acceleration?
1 $ 3$ 1
(b) The velocity of the particle is zero at t = 0 s (a) − i+ j (b) 2$i − $j
(c) The velocity of the particle is zero at t = 1 s 8 2 8
1 3
(d) The velocity and acceleration of the particle are zero (c) − $i + 8$j (d) 8$i − $j
2 2

PROJECTILE Projectile projected obliquely


When an object or body released into the space with some on the surface of the earth
initial velocity, moves freely under the effect of gravity is Let OX be a horizontal line on the ground, OY be a
known as projectile. vertical line perpendicular to the ground and O be the
origin for XY-axes on a plane.
Projectile motion Suppose an object is projected from point O with velocity
If a constant force (and hence, constant acceleration) acts (u ), making an angle (θ) with the horizontal direction OX,
on a particle at an angle θ (≠ 0 ° or 180°) with the direction such that x 0 = 0 and y 0 = 0 when t = 0.
of its initial velocity (≠ zero), the path followed by the Y
particle is parabolic and the motion of the particle is called
projectile motion. It is a two dimensional motion, i.e. A u cos θ
x
motion of the particle is constrained in a plane. u sin θ P (x, y)
u cos θ
In other words, if a particle moves in horizontal as well as u
g
y H
vertical motion simultaneously, the motion of the particle θ g
is known as projectile motion. O
u cos θ B
X
Fig. 4.9 Oblique projectile motion
+ ≡ While resolving velocity (u ) into two components, we get
Horizontal
motion of Vertical Projectile (i) u cosθ along OX and (ii) u sin θ along OY
particle motion of motion
particle
As there is no force acting in horizontal direction, so the
horizontal component of velocity (u cos θ ) remains constant
Fig. 4.8 Motion of particle
throughout the entire motion, so there is no acceleration in
Note the horizontal direction (if air resistance is assumed to be
(i) If the angle between acceleration and velocity is θ and where zero).
0° < θ < 180° , then particle is executing projectile motion.
However, the vertical components of velocity (u sin θ)
(ii) In projectile motion, change in velocity of particle in magnitude
and direction both act simultaneously. decreases continuously with height from O to A, due to
downward force of gravity and becomes zero at highest
When a particle is thrown obliquely near the earth’s point A. At this point, the object attains maximum height,
surface, it moves in a parabolic path, provided that the now it has only horizontal component of velocity. From
particle remains close to the surface of earth and the air point A, the object starts to fall down and reaches at point
resistance is negligible. This is an example of projectile B on the ground.
motion.
144 OBJECTIVE Physics Vol. 1

Let us make ourselves familiar with certain terms used in Similarly, vertical distance,
projectile motion 1
y = (u sin θ ) t − gt 2
2
1. Equation of path of projectile 1
= (20 sin 60° ) × 0.5 − × 9.8 × (0.5)2 ⇒ y = 7.43 m
2
Suppose at any time t, the object reaches at point P (x, y ). (ii) Velocity along horizontal direction,
So, x is the horizontal distance travelled by object in time t v x = u cos θ = 20 cos 60° = 10 ms −1
and y is the vertical distance travelled by object in time t.
Velocity along vertical direction,
v y = u sin θ − gt = 20 sin 60°− 9.8 × 0.5 = 12.42 ms −1
Motion along horizontal direction
The velocity of the object in horizontal direction, i.e. along Example 4.11 A stone is thrown with a speed of 10 ms −1 at an
OX is constant, so the acceleration a x in horizontal direction angle of projection 60°. Find its height above the point of
is zero. projection when it is at a horizontal distance of 3 m from the
thrower? (Take, g = 10 ms −2 )
∴ Position of the object at time t along horizontal direction
1 Sol. Considering the equation of trajectory,
is given by, x = x 0 + u x t + a x t 2 y = (tan θ 0 ) x −
g
x2
2 2(v 02 cos2 θ 0 )
But x 0 = 0, u x = u cos θ, a x = 0 and t = t Here, θ 0 = 60°, v 0 = 10 ms −1, x = 3 m
∴ x = u cos θ t 10
∴ y = (tan 60° ) × 3 − (3)2
x 2 (100 cos2 60° )
⇒ Time, t = …(i)
u cosθ 9 15 3 − 9
=3 3 − = m = 3.396 m
5 5
Motion along vertical direction
The vertical component of velocity of the object is 2. Time of flight of projectile
decreasing from O to P due to gravity, so acceleration a y It is defined as the total time for which projectile is in flight,
is − g. i.e. time during the motion of projectile from O to B. It is
∴ Position of the object at any time t along the vertical denoted by T.
1 Time of flight consists of two parts such as
direction, i.e. along OY is given by, y = y 0 + u y t + a y t 2 (i) Time taken by an object to go from point O to A. It
2
is also known as time of ascent (t a ).
But y 0 = 0, u y = u sin θ, a y = − g and t = t
(ii) Time taken by an object to go from point A to B. It
1 1
So, y = u sin θ t + (−g ) t 2 = u sin θ t − gt 2 …(ii) is also known as time of descent (t d ).
2 2 Total time can be expressed as
Substituting the value of t from Eq. (i) in Eq. (ii), we get T = t a + t d = 2t ⇒ t = T /2 (Q t a = t d = t )
2 The vertical component of velocity of the projectile
 x  1  x 
y = u sin θ   − g  becomes zero at the highest point H.
 u cos θ 2  u cos θ Let us consider vertical upward motion of the
g  x 
2 object from O to A, we get
= x tan θ −   u y = u sin θ, a y = − g, t = T /2 and v y = 0
2  u cos θ
T
Since, v y = u y + a y t ⇒ 0 = u sin θ − g
1 g  2 2
Vertical displacement, y = x tan θ −   x 2 u sin θ
 2 u cos θ 
2 2
∴ Time of flight, T =
g
This equation is in the form of y = ax − bx 2 , which
represents a parabola and it is known as equation of Example 4.12 A cricket ball is thrown at a speed of 28 ms −1
trajectory of a projectile. in a direction 30° above the horizontal. Calculate the time
Example 4.10 A body is projected with a velocity of 20ms −1 in taken by the ball to return to the same level.
a direction making an angle of 60° with the horizontal. Sol. Given, speed, u = 28 ms−1 and θ = 30°
Determine its (i) position after 0.5 s and (ii) the velocity after 0.5 s. ∴ The time taken by the ball to return the same level is
Sol. Given, u = 20 ms−1, θ = 60°, t = 0.5 s 2u sin θ 2 × 28 × sin 30° 28
T = = = = 2.85 −~2.9 s
(i) Since, horizontal distance, g 9.8 9.8
x = (u cos θ ) t = (20 cos 60° ) × 0.5 = 5 m
Motion in a Plane and Projectile Motion 145

sin θ
3. Maximum height of a projectile i.e. OB = R = u cos θ × T = u cos θ × 2 u
g
It is defined as the maximum vertical height attained by
the projectile above the point of projection during its u 2 sin 2θ
Horizontal range, R = (Q sin 2 θ = 2 sin θ cos θ)
flight. It is denoted by H. g
Y
The horizontal range will be maximum, if angle of
projection is 45°.
u A u2
∴ Maximum horizontal range, R max =
g
H
θ B For same value of initial velocity, horizontal range of
O X
projectile is same for complementary angles.
Fig. 4.10 Maximum height of projectile
So, R 30 ° = R 60 ° or R 20 ° = R 70 °
Let us consider the vertical upward motion of the
Some important points related to projectile motion
projectile from O to A.
(i) Velocity of the projectile at any instant t,
We have,
u y = u sin θ, a y = − g, y 0 = 0, y = H, v = v $i + v $j = u cos θ$i + (u sin θ − gt ) $j
x y

T u sin θ | v| = u 2 + g 2t 2 − 2ugt sin θ


t= =
2 g
Y
1 v
Using this relation, y = y 0 + uy t + ay t 2 A
2 u
2 β
 u sin θ  1  u sin θ 
We have, H = 0 + u sin θ   + (−g )  
 g  2  g  θ
O X
u 2
1 u sin θ 2 2 B
= sin 2 θ − Fig. 4.11
g 2 g
● This velocity makes an angle β with the horizontal
u 2 sin 2 θ given by
Maximum height, H=
2g vy u sin θ − gt
tan β = = …(i)
Example 4.13 Assume that a ball is kicked at an angle of 60° vx u cos θ
with the horizontal, so if the horizontal component of its ● When the projectile reaches at point B, substitute
velocity is 19.6 ms −1, determine its maximum height. t = T in Eq. (i).
Sol. Given, θ = 60° (ii) When a projectile is thrown upward, its kinetic
Horizontal component of velocity = u cos 60° = 19.6 ms−1 energy decreases, potential energy increases but the
19.6 19.6 total energy always remains constant.
∴ u= = = 39.2 ms−1
cos 60° 0.5 (iii) Total energy of projectile = Kinetic energy + Potential
Therefore, maximum height, energy
2 1 2 1 1
u 2 sin2 60° (39.2)2  3  = mv cos 2 θ + mv 2 sin 2 θ = mv 2
H= = ×   = 58.8 m 2 2 2
2g 2 × 9.8  2 

4. Horizontal range of a projectile (iv) In projectile motion, speed (and hence, kinetic
energy) is minimum at highest point of its trajectory
The horizontal range of a projectile is defined as the
and given as
horizontal distance covered by the projectile during its
time of flight. It is denoted by R. Speed = (cos θ ) times the speed of projection
If the object having uniform velocity u cosθ (i.e. horizontal and kinetic energy = (cos 2 θ ) times the initial kinetic
component) and the time of flight is T, then the horizontal energy.
range covered by the projectile. Here, θ = angle of projection.
146 OBJECTIVE Physics Vol. 1

(v) In projectile motion, it is sometimes better to write the Example 4.16 Find the angle of projection of a projectile for
equations of H, R and T in terms of u x and u y as which the horizontal range and maximum height are equal.
2u y u y2 Sol. Given, R =H
T = , H=
g 2g u sin 2 θ u 2 sin2 θ
2
∴ =
2u x u y g 2g
and R = sin2 θ
g or 2 sin θ cos θ =
2
(vi) In projectile motion H = R , when u y = 4 u x or sin θ
tan θ = 4. or = 4 or tan θ = 4
cos θ
(vii) Equation of trajectory can also be written as ∴ θ = tan−1(4)
 x
y = x 1 −  tanθ Example 4.17 There are two angles of projection for which
 R
the horizontal range is the same. Show that the sum of the
where, R is horizontal range. maximum heights for these two angles is independent of the
(viii) All the above expressions for T, H and R are derived angle of projection.
by neglecting air resistance. If air resistance is Sol. Let the angles of projection be α and 90°− α for which the
considered, then values may differ slightly. horizontal range R is same.
(ix) Due to air resistance when a net speed of projectile u 2 sin2 θ
decreases, then R decreases, T increases and H Now, H1 =
2g
decreases. Reverse is the case when net speed
increases. u 2 sin2 (90° − θ ) u 2 cos2 θ
and H2 = =
2g 2g
Example 4.14 An object is projected with a velocity of
30 ms −1 at an angle of 60° with the horizontal. Determine u2 u2
Therefore, H1 + H 2 = (sin2 θ + cos2 θ ) =
the horizontal range covered by the object. 2g 2g
Sol. Given, initial velocity, u = 30 ms −1 Clearly, the sum of the heights for the two angles of
Angle of projection, θ = 60° projection is independent of the values of projection angles.
Therefore, the horizontal range (or distance) covered by the Example 4.18 Prove that the maximum horizontal range is four
object will be given as
times the maximum height attained by the projectile; when
u 2 sin 2θ (30)2 sin 2 (60° ) fired at an inclination so as to have maximum horizontal
R = =
g g range.
(30) 2 sin 60° cos 60°
2
Sol. For θ = 45°, the horizontal range is maximum and is given by
=
9.8 u2
= 79.53 m R max =
g
⇒ R = 79.53 m
Maximum height attained,
Example 4.15 A projectile has a range of 40 m and reaches a
u 2 sin2 45° u 2 R max
maximum height of 10 m. Find the angle at which the H max = = =
projectile is fired. 2g 4g 4
u02 sin 2 θ 0 or R max = 4 H max
Sol. Range of a projectile, R = = 40 m …(i)
g
u02 sin2 θ 0 Projectile fired at an angle
H= = 10 m …(ii)
2g
On dividing Eq. (i) by Eq. (ii), we get
with the vertical
2 (sin 2 θ 0 ) Let a particle be projected vertically with an angle θ with
=4 vertical and it’s speed of projection is u.
sin2 θ 0
4 sin θ 0 cos θ 0
Clearly, the angle made by the velocity of projectile at
=4 point of projection with horizontal is (90°−θ ).
sin2 θ 0
In this case
⇒ tan θ 0 = 1
2u sin (90 ° − θ ) 2u
⇒ θ 0 = 45° (i) Time of flight = = cos θ
g g
Motion in a Plane and Projectile Motion 147

u 2 sin 2 (90 ° − θ ) u 2 cos 2 θ (v) Velocity at any time t,


(ii) Maximum height = =
2g 2g v = u 2 + g 2t 2 − 2ugt sin (90 °− θ )
Y
= u 2 + g 2 t 2 − 2ugt cos θ
A u sin θ
This velocity makes an angle β with the horizontal
u
direction, then
u cos θ
θ
H u sin (90 °− θ ) − gt u cos θ − gt
tan β = =
θ

u cos (90 °− θ ) u sin θ



90

O X
u sin θ B
Fig. 4.12 Projectile fired at an angle with the vertical Example 4.19 A football is kicked at an angle of 30° with
the vertical, so if the horizontal component of its velocity is
(iii) Horizontal range 20 ms −1, determine its maximum height.
u2 u2 u2 Sol. Given, θ = 30°
= sin 2 (90 ° − θ ) = sin (180 ° − 2θ ) = sin 2θ
g g g Horizontal component of velocity = u sin 30° = 20 ms−1
(iv) Equation of path of projectile, 20 20
⇒ u= = = 40 ms−1
1 gx 2 sin 30° 1/2
y = x tan (90 ° − θ ) −
2 u 2 cos 2 (90 ° − θ ) Therefore, maximum height,
2 2
gx u 2 cos2 30° (40)2  3
= x cot θ − 2 H= = ×   = 61.22 m
2u sin 2 θ 2g 2 × 9.8  2 

CHECK POINT 4.2


1. At the top of the trajectory of a projectile, the directions of 6. A football player throws a ball with a velocity of 50 ms −1 at
its velocity and acceleration are an angle 30° from the horizontal. The ball remains in the air
(a) parallel to each other for (Take, g = 10 ms −2)
(b) anti-parallel to each other (a) 2.5 s (b) 1.25 s
(c) inclined to each other at an angle of 45° (c) 5 s (d) 0.625 s
(d) perpendicular to each other
7. A particle is projected with a velocity of 20 ms −1 at an angle
2. The value of acceleration at the top of the trajectory of a of 60° to the horizontal. The particle hits the horizontal
particle when thrown obliquely is plane again during its journey. What will be the time of
(a) maximum (b) minimum impact?
(c) zero (d) g (a) 3.53 s (b) 2.4 s
3. In the motion of a projectile falling freely under gravity, its (c) 1.7 s (d) 1s
(neglect air friction) 8. If two balls are projected at angles 45° and 60° and the
(a) total mechanical energy is conserved maximum heights reached are same, what is the ratio of
(b) momentum is conserved their initial velocities?
(c) mechanical energy and momentum both are conserved
(a) 2: 3 (b) 3: 2
(d) None is conserved
(c) 3 : 2 (d) 2 : 3
4. During the flight of a projectile thrown obliquely, which
amongst the quantities remains constant? 9. If the initial velocity of a projection is doubled, keeping the
(a) Angular momentum angle of projection same, the maximum height reached by it
(b) Linear momentum will
(c) Vertical component of velocity (a) remain the same (b) be doubled
(d) Horizontal component of velocity (c) become four times (d) be halved
5. A stone is projected with speed of 50 ms −1 at an angle of 60° 10. For a projectile, the ratio of maximum height reached to the
with the horizontal. The speed of the stone at highest point square of flight time is (Take, g = 10 ms −2 )
of trajectory is (a) 5 : 4 (b) 5 : 2
(a) 75 ms−1 (b) 25 ms−1 (c) 5 : 1 (d) 10 : 1
(c) 50 ms−1 (d) Cannot find
148 OBJECTIVE Physics Vol. 1

11. A particle is projected from ground with speed u and at an (a) 2R (b) R/ 2
angle θ with horizontal. If at maximum height from ground, (c) R (d) 4R
the speed of particle is 1/2 times of its initial velocity of 16. An object is thrown along a direction inclined at an angle of
projection, then find its maximum height attained. 45° with the horizontal direction. The horizontal range of
2 2
u 2u the particle is equal to
(a) (b)
g g (a) vertical height
u2 3u2 (b) twice the vertical height
(c) (d)
2g 8g (c) thrice the vertical height
12. A projectile thrown with velocity v 0 at an angle α to the (d) four times the vertical height
horizontal, has a range R. It will strike a vertical wall at a 17. An object is projected at an angle of 45° with the horizontal.
distance R/ 2 from the point of projection with a speed of The horizontal range and the maximum height reached will
(a) v0 (b) v0 sinα be in the ratio
gR (a) 1 : 2 (b) 2 : 1
(c) v0 cosα (d)
2 (c) 1 : 4 (d) 4 : 1
13. A particle is projected at an angle of 45° with a velocity of 18. The horizontal range of a projectile is 4 3 times of its
9.8 ms −1. The horizontal range will be (Take, g = 9.8 ms −2 ) maximum height. The angle of projection will be
(a) 9.8 m (b) 4.9 m (a) 60° (b) 37°
9.8 (c) 16.1° (d) 45°
(c) (d) 9.8 2
2 19. A ball is projected with a velocity 20 3 ms −1 at an angle 60°
14. Two projectiles A and B are projected with same speed at to the horizontal. The time interval after which the velocity
angles 30° and 60° to the horizontal, then which one is vector will make an angle 30° to the horizontal is (Take,
wrong? g = 10 ms −2)
(a) R A = RB (b) HB = 3H A (a) 5s (b) 2s (c) 1 s (d) 3s
(c) TB = 3 TA (d) None of these
20. A projectile is thrown with a velocity of10 ms −1 at an angle
15. A projectile fired with initial velocity u at some angle θ has a of 60° with horizontal. The interval between the moments
range R. If the initial velocity be doubled at the same angle when speed is 5 g m/ s is (Take, g = 10 ms −2)
of projection, then the range will be
(a) 1 s (b) 3s
(c) 2s (d) 4 s

PROJECTILE FROM A POINT


ABOVE THE GROUND
Projectile projected from a tower
When an object is at some height above the ground, then Horizontal components Vertical components
its projectile motion depends on the angle of projection. ux = u uy = 0
Projectile projected horizontally from a tower ax = 0 (QFx = 0) ay = − g
Assume that an object is thrown horizontally with some
velocity u from point O, on a tower of height h above Different terms related to this type of projectile motion are
the ground level and after time t, it reaches ground at point 1 g 
E. (i) Equation of trajectory, y =  2  x 2
O u
2 u 
X
y 2h
(ii) Time of flight, T =
x D (x, y) vx g
h β
v
2h
vy (iii) Horizontal range, R = u
C g
Ground level
Y E (iv) Velocity of projectile at any instant,
Fig. 4.13 Horizontal projectile
v = u 2 + g 2t 2
Motion in a Plane and Projectile Motion 149

Example 4.20 A bomb is released from an aeroplane flying at (ii) the distance of the point, where the particle hits the ground
a speed of 720 kmh −1 in the horizontal direction 8000 m from foot of the hill and
above the ground. At what horizontal distance from the (iii) the velocity with which the projectile hits the ground.
initial position of aeroplane, it strikes the ground? (Take, g = 9.8 ms −2)
Sol. Sol. In this problem, we cannot apply the formulae of R, H and
u = 720 kmh−1 T directly. Here, it will be more convenient to choose x and y
O
directions as shown in figure.
O x
Here, ux = 98 ms −,1 a x = 0, uy = 0 and a y = g.
h = 8000 m
y (i) At A, sy = 490 m . So, applying
1
A sy = u y t + a y t 2
2
B 1
x ⇒ 490 = 0 + ( 9.8 ) t 2 ∴ t = 10 s
2
According to the figure, during motion of the bomb from O
1
to B, (ii) BA = sx = uxt + a xt 2 or BA = (98)(10) + 0 (Q a x = 0)
2
u = 720 ×
5 1
= 200 ms −1 ⇒ y = h = gt 2 or BA = 980 m
18 2
(iii) Horizontal velocity, v x = ux = 98 ms −1
1
⇒ 8000 = × 10t ⇒ t = 40 s
2
Vertical velocity, v y = uy + a y t = 0 + ( 9.8 ) (10) = 98 ms −1
2
∴ x = ut = 200 × 40 = 8000 m ∴ Resultant velocity, v = v x2 + v y2 = (98)2 + (98)2
Example 4.21 A body is thrown horizontally from the top of a
= 98 2 ms −1
tower and strikes the ground after three seconds at an angle
of 45° with the horizontal. Then, find 98 vy
and = = 1 ∴ β = 45°
tan β =
(i) the height of the tower. v x 98
(ii) the speed of projection of the body. Thus, the projectile hits the ground with a velocity
Sol. (i) Let H be the height of the tower. 98 2 ms −1 at an angle of β = 45° with horizontal as
2H shown in the given figure.
The time of flight, Tf = =3s
g
g × (3)2 9.8 × 9
Projectile projected upward from a tower
⇒ H= = = 44.1 m Consider a projectile projected upward at an angle (θ ) from
2 2
point O which is situated on a tower at height h above the
(ii) Let the speed of projection be v 0.
ground. Now, from the diagram, we have
Then, for horizontal projection,
u x = u cos θ, a x = 0
v x = v 0 ⇒ v y = − gt
u y = u sin θ, a y = − g
At t = Tf = 3 s, v y = − 9.8 × 3 = −29.4 ms−1
(i) Equation of horizontal motion, x = u cosθt …(i)
The angle which the final velocity makes with the
horizontal = θ = 45° (Given) (ii) Equation of vertical motion,
− vy 1
⇒ tan 45° = ⇒ − vy = vx − h = u sinθt − gt 2 …(ii)
vx 2
So, v x = 29.4 ms−1 u vx = ux = u cos θ
vy = 0
Example 4.22 A projectile is fired horizontally with a A
O θ B u cos θ a y = -g
velocity of 98 ms −1 from the top of a hill 490 m high.
uy = u sin θ

u cos θ θ
Find u sin θ
(i) the time taken by the projectile to reach the ground, h
u = 98 ms−1
O
x
P D C
y Fig. 4.14 Projectile projected upward from a tower

u sin θ u 2 sin 2 θ 2h
B A vx (iii) Time of flight, T = ± +
β g g2 g
vy
150 OBJECTIVE Physics Vol. 1

(iv) Horizontal distance covered (in time of flight T), Example 4.24 A boy standing on the top of a tower 36 m
high has to throw a packet to his friend standing on the
PC = (u cos θ )T
ground 48 m horizontally away. If he throws a packet
(v) Horizontal distance covered from the top of tower, directly aiming the friend with a speed of 10 ms −1, how
u 2 sin 2θ short will the packet fall?
OB = Sol. The packet will strike at point C instead of reaching to
g
point B because the packet will move in a parabolic path
In such case for range PC to become maximum, θ should instead of straight line.
be 45°. O u cos θ
θ O x
Example 4.23 A boy playing on the roof of a 10 m high

u sin θ
−1 u
building throws a ball with a speed of 10 ms at an angle

36 m
of 30° with the horizontal. How far from the throwing point y
will the ball be at the height of 10 m from the ground?
Sol. The ball will be at point P 10 ms−1 θ
when it is at a height of 10 m A x C B
from the ground. So, we have to O 30° P
X 48 m
find distance OP, which can be
36 3
calculated by direct considering 10 m From geometry, tan θ = =
it, as a projectile on a level OX at 48 4
a height h from the horizontal 3 4
level. Ground
⇒ sin θ = , cos θ =
5 5
u 2 sin 2θ 1 2
OP = R = For O to C , sy = u sin θt + gt
g 2
102 × sin (2 × 30° ) 3 1
⇒ R= = 8.66 m ⇒ 36 = 10 × t + × 10t 2 = 6t + 5t 2
10 5 2
⇒ 5t 2 + 6t − 36 = 0
Projectile projected downward from a tower ∴ t = 215
. s,
Consider a projectile projected downward at an angle θ 4
sx = u cos θt = 10 × × 2.15
from point O which is on a tower of height h above the 5
ground. = 17.2 m = AC
Now, from the diagram, we have ∴ BC = AB − AC = 48 − 17.2 = 30.8 m
u x = u cos θ, a x = 0 The packet will fall at distance 30.8 m in front of his friend.
⇒ u y = − u sin θ, a y = − g
y Shooting a freely falling target
uy = u sin θ

ux cos θ Aiming a target situated at some height is also an example


O
θ
of projectile motion. For example, consider a stone of a
a y = −g slingshot (gulel) which aimed at a mango A as shown in
u
figure.
h

x
P A A
Fig. 4.15 Projectile projected downward from a tower
G A′
1
(i) Equation of motion, − h = (− u sin θ ) t + (− g )t 2
2
or gt 2 + (2u sin θ )t − 2h = 0 Fig. 4.16 Shooting a freely falling target

Suddenly the mango starts falling and at the same time stone
− 2u sin θ 4u 2 sin 2 θ + 8gh
(ii) Time of flight, T = ± is fired from the gulel. The stone hits the mango at points
2g 2g A′. Reason behind this is stone and mango falls through
(iii) Horizontal distance covered from the base of tower, same height under same value of gravitational acceleration
PA = (u cos θ ) T (g) in same time, hence stone hits the mango.
CHECK POINT 4.3
1. A bomb is dropped from an aeroplane moving horizontally 5. An aeroplane is flying at a constant height of 1960 m with
at constant speed. If air resistance is neglected, then the speed 600 kmh −1 above the ground towards point directly
bomb over a person struggling in flood water. At what angle of
(a) falls on the earth exactly below the aeroplane sight with the vertical should the pilot release a survival kit,
(b) falls on the earth behind the aeroplane if it is to reach the person in water? (Take, g = 9.8 ms −2 )
(c) falls on the earth ahead of the aeroplane (a) 45° (b) 30°
(d) flies with the aeroplane (c) 60° (d) 90°
2. A body is projected horizontally with a velocity of 4 ms −1 6. A ball is projected horizontally from the top of a tower with
from the top of a high tower. The velocity of the body after a velocity v 0 . It will be moving at an angle of 60° with the
horizontal after time,
0.7 s is nearly (Take, g = 10 ms −2)
v0 3 v0 v0 v0
(a) 10 ms−1 (b) 8 ms−1 (a) (b) (c) (d)
3g g g 2g
(c) 19.2 ms−1 (d) 11 ms−1
7. A man standing on a hill top projects a stone horizontally
3. A particle is projected horizontally with speed 20 ms −1 from with speed v 0 as shown in figure. Taking the coordinate
the top of a tower. After what time, velocity of particle will system as given in the figure. The coordinates of the point,
be at 45° angle from initial direction of projection? where the stone will hit the hill surface are
(a) 1 s (b) 2 s (c) 3 s (d) 4 s
y
4. An aeroplane is travelling horizontally at a height of 2000 m v0 x
from the ground. The aeroplane when at a point P, drops a (0, 0)
bomb to hit a stationary target Q on the ground. In order
that the bomb hits the target, what is the angle θ, the line
PQ makes with the vertical?
(Take, g = 10 ms −2 ) θ
−1
100 ms
P  2v2 tanθ 2v2 tan2 θ   2v 2 2v2 tan2 θ 
(a)  0 ,− 0  (b)  0 , − 0 
 g g   g g 
 2v2 tanθ 2v 2 
2000 m (c)  0 ,− 0
 g g 
θ
 2v2 tan2 θ 2v2 tanθ 
Q (d)  0 ,− 0 
 g g 
(a) 45° (b) 30° (c) 60° (d) 90°
Chapter Exercises
(A) Taking it together
Assorted questions of the chapter for advanced level practice

1 In a two dimensional motion, instantaneous speed v 0 h h


is a positive constant. Then which of the following
are necessarily true? [NCERT Exemplar]
(a) (b)
(a) The average velocity is not zero at any time.
(b) Average acceleration must always vanish. t t
O O
(c) Displacements in equal time intervals are equal.
h h
(d) Equal path lengths are traversed in equal intervals.
2 In a two dimensional motion, instantaneous speed v 0
(c) (d)
is a positive constant. Then, which of the following
are necessarily true? [NCERT Exemplar]
t
O O t
(a) The acceleration of the particle is zero.
(b) The acceleration of the particle is bounded. 7 Which of the following is the altitude-time graph for
(c) The acceleration of the particle is necessarily in the a projectile thrown horizontally from the top of the
plane of motion. tower?
(d) The particle must be undergoing a uniform circular
motion. h h

3 A particle velocity changes from (2$i − 3 $j) ms −1 to


(a) (b)
(3 $i − 2$j) ms −1 in 2 s. If its mass is 1 kg, the
acceleration (ms −2 ) is O t O t
(a) − ($i + $j) (b) ($i + $j)/2
h h
(c) zero (d) ($i − $j)/2
4 A projectile is thrown at an angle θ with the (c) (d)
horizontal and its range is R 1. It is then thrown at an
angle θ with vertical and the range is R 2 , then O t O t
(a) R1 = 4R 2 (b) R1 = 2R 2
(c) R1 = R 2 (d) None of these 8 The horizontal range of a projectile fired at an angle
5. Figure shows four paths for a kicked football. of 15° is 50 m. If it is fired with the same speed at
Ignoring the effects of air on the flight, rank the an angle of 45°, its range will be [NCERT Exemplar]
paths according to initial horizontal velocity (a) 60 m (b) 71 m (c) 100 m (d) 141 m
component highest first. 9 A body is thrown horizontally from the top of a
y tower of height 5 m. It touches the ground at a
distance of 10 m from the foot of the tower. The
initial velocity of the body is (Take, g = 10 ms −2 )
(a) 2.5 ms−1 (b) 5 ms−1 (c) 10 ms−1 (d) 20 ms−1

0 1 2 3 4 x 10 Velocity and acceleration of a particle at some


instant of time are v = (3 $i + 4$j) ms −1 and
(a) 1, 2, 3, 4 (b) 2, 3, 4, 1
a = − (6$i + 8$j) ms −2 , respectively. At the same
(c) 3, 4, 1, 2 (d) 4, 3, 2, 1
instant particle is at origin, maximum x-coordinate
6. Which of the following is the graph between the of particle will be
height (h ) of a projectile and time (t ), when it is (a) 1.5 m (b) 0.75 m (c) 2.25 m (d) 4 m
projected from the ground?
Motion in a Plane and Projectile Motion 153

11 Two paper screens A and B are separated by a The percentage increase in the horizontal range will
distance of 100 m . A bullet pierces A and then B. be
The hole in B is 10 cm below the hole in A. If the (a) 20% (b) 15% (c) 10% (d) 5%
bullet is travelling horizontally at the time of hitting 19 A body is projected at an angle of 30° with the
A, then the velocity of the bullet at A is horizontal with momentum p. At its highest point,
(a) 100 ms−1 (b) 200 ms−1 the magnitude of the momentum is
(c) 600 ms−1 (d) 700 ms−1 3 2 p
(a) p (b) p (c) p (d)
12 Two stones having different masses m1 and m 2 are 2 3 2
projected at an angle α and (90° − α ) with same 20 A projectile is fired from ground level at an angle θ
speed from same point. The ratio of their maximum above the horizontal. The elevation angle φ of the
heights is highest point as seen from the launch point is related
(a) 1 : 1 (b) 1 : tan α (c) tan α : 1 (d) tan2 α : 1 to θ by the relation
1
13 Two projectiles A and B are thrown from the same (a) tan φ = tan θ (b) tan φ = tan θ
point with velocities v and v /2, respectively. If B is 4
thrown at an angle 45° with horizontal, what is the 1
(c) tan φ = tan θ (d) tan φ = 2 tan θ
inclination of A when their ranges are the same? 2
 1 1  1 21 A ball is thrown up with a certain velocity at an
(a) sin−1   (b) sin−1  
 4 2  4 angle θ to the horizontal. The kinetic energy KE of
 1 1  1 the ball varies with horizontal displacement x as
(c) 2 sin−1   (d) sin−1  
 4 2  8
14 A particle moves in the XY-plane according to the (a) KE (b) KE
law x = kt, y = kt (1 − αt ), where k and α are positive
constants and t is time. The trajectory of the particle
O x O x
is
αx 2
(a) y = kx (b) y = x −
k
ax 2
(c) y = − (d) y = αx (c) KE (d) KE
k
15 The maximum range of a gun on horizontal terrain is O x O x
1 km. If g = 10 ms −2 , what must be the muzzle
velocity of the shell? 22 A body projected with velocity u at projection angle
(a) 400 ms−1 (b) 200 ms−1
θ has horizontal range R. For the same velocity and
projection angle, its range on the moon surface will
(c) 100 ms−1 (d) 50 ms−1 be (g moon = g earth /6)
16 The equation of trajectory of a projectile is R R
(a) 36R (b) (c) (d) 6R
g 36 16
y = 3 x − x 2 , the angle of its projection is
2 23 Three balls of same masses are projected with equal
(a) 90° (b) zero (c) 60° (d) 30° speeds at angle 15°, 45°, 75°, and their ranges are
17 A projectile is thrown upward with a velocity v 0 at respectively R 1, R 2 and R 3 , then
(a) R1 > R 2 > R 3 (b) R1 < R 2 < R 3
an angle α to the horizontal. The change in velocity
(c) R1 = R 2 = R 3 (d) R1 = R 3 < R 2
of the projectile when it strikes the same horizontal
plane is 24 A projectile is thrown with an initial velocity of
(a) v 0 sin α vertically downward (a$i + b$j) ms −1. If the range of the projectile is twice
(b) 2v 0 sin α vertically downward the maximum height reached by it, then
(c) 2v 0 sin α vertically upward (a) a = 2b (b) b = a (c) b = 2a (d) b = 4a
(d) zero
25 The ratio of the speed of a projectile at the point of
18 The maximum height attained by a projectile is projection to the speed at the top of its trajectory is
increased by 10% by increasing its speed of x. The angle of projection with the horizontal is
projection, without changing the angle of projection. (a) sin−1 x (b) cos−1 x (c) sin−1 (1/ x ) (d) cos−1 (1/ x )
154 OBJECTIVE Physics Vol. 1

26 A man can throw a stone such that it acquires 36 A stone is thrown at an angle θ with the horizontal,
maximum horizontal range 80 m. The maximum reaches a maximum height H. Then, the time of
height to which it will rise for the same projectile flight of stone will be
(in metre) is 2H 2H 2 2H sin θ 2H sin θ
(a) (b) 2 (c) (d)
(a) 10 (b) 20 (c) 40 (d) 50 g g g g
27 The velocity at the maximum height of a projectile is 37 A ball is thrown up with a certain velocity at an
half of its initial velocity of projection (u ). Its range angle θ to the horizontal. The kinetic energy KE of
on horizontal plane is the ball varies with height h as
3u 2 3 u2 u2 3 u2
(a) (b) ⋅ (c) (d) ⋅
g 2 g 3g 2 g
28 A projectile is thrown from a point in a horizontal (a) KE (b) KE
plane such that the horizontal and vertical velocities
are 9.8 ms −1 and 19.6 ms −1. It will strike the plane O h O h
after covering distance of
(a) 39.2 m (b) 19.6 m (c) 9.8 m (d) 4.9 m
29 The greatest height to which a man can throw a (c) KE (d) KE
stone is h. The greatest distance to which he can
throw it will be
O h O
h h
(a) (b) h (c) 2h (d) 3h
2 38 For a given velocity, a projectile has the same range
30 Four bodies P, Q, R and S are projected with equal R for two angles of projection. If t1 and t 2 are the
velocities having angles of projection 15°, 30 °, 45° time of flight in the two cases, then t1t 2 is equal to
and 60° with the horizontal respectively. The body 2R R 4R R
(a) (b) (c) (d)
having shortest range is g g g 2g
(a) P (b) Q (c) R (d) S
39 A cricket ball is hit for a six by the bat at an angle of
31 A stone is projected in air. Its time of flight is 3 s 45° to the horizontal with kinetic energy K. At the
and range is 150 m. Maximum height reached by the highest point, the kinetic energy of the ball is
stone is (Take, g = 10 ms −2 ) (a) zero (b) K (c) K /2 (d) K / 2
(a) 37.5 m (b) 22.5 m (c) 90 m (d) 11.25 m 40 The equation of motion of a projectile is
32 A boy throws a ball with a velocity u at an angle θ with 3 2
y = 12x − x
the horizontal. At the same instant, he starts running 4
with uniform velocity to catch the ball before it hits What is the range of the projectile?
the ground. To achieve this, he should run with a (a) 12 m (b) 16 m (c) 20 m (d) 24 m
velocity of
41 A ball of mass m is projected from the ground with
(a) u cos θ (b) u sin θ (c) u tan θ (d) u sec θ
an initial velocity u making an angle of θ with the
33 Galileo writes that for angle of projection of a vertical. What is the change in velocity between the
projectile at angle (45° − θ ) and (45° + θ ), the point of projection and the highest point?
horizontal ranges described by the projectile are in (a) u cos θ downward (b) u cos θ upward
the ratio of (if θ ≤ 45° ) (c) u sin θ upward (d) u sin θ downward
(a) 2 : 1 (b) 1 : 2 (c) 1 : 1 (d) 2 : 3 1
42 The equation of projectile isY = 3X − gX 2 . The
34 If time of flight of a projectile is 10 s. Range is 2
500 m. The maximum height attained by it will be velocity of projection is
(a) 125 m (b) 50 m (c) 100 m (d) 150 m (a) 1 ms−1 (b) 2 ms−1 (c) 3 ms−1 (d) 1.2 ms −1
35 A body of mass m is thrown upwards at an angle θ 43 The range of a projectile when launched at an angle
with the horizontal with velocity v. While rising up θ is same as when launched at an angle 2θ. What is
the velocity of the mass after t seconds will be the value of θ ?
(a) (v cos θ )2 + (v sin θ )2 (b) (v cos θ − v sin θ )2 − gt (a) 15° (b) 30°
(c) 45° (d) 60°
(c) v + g t − (2v sin θ ) gt (d)
2 2 2
v + g t − (2v cos θ ) gt
2 2 2
Motion in a Plane and Projectile Motion 155

44 A particle is thrown with a speed u at an angle θ If the initial velocity of the ball is 20 ms −1 and the
with the horizontal. When the particle makes an horizontal distance between O and C is 10 m. Find
angle φ with the horizontal, its speed changes to v, the value of h.
where B
(a) v = u cos θ
(b) v = u cos θ cos φ h
(c) v = u cos θ sec φ
A
(d) v = u sec θ cos φ
45 An aeroplane moving horizontally with a speed of 20 ms–1
−1 30°
720 kmh drops a food pocket, while flying at a O C
10 m
height of 396.9 m. The time taken by a food pocket
to reach the ground and its horizontal range is g g g g
(a) m (b) m (c) m (d) m
(Take g = 9.8 ms −2 ) 6 10 3 12
(a) 3 s and 2000 m (b) 5 s and 500 m 50 A ball is thrown from a point with a speed v 0 at an
(c) 8 s and 1500 m (d) 9 s and 1800 m
angle of projection θ. From the same point and at the
46 A bullet is to be fired with a speed of 2000 ms −1 to same instant, a person starts running with a constant
hit a target 200 m away on a level ground. If speed v 0 /2 to catch the ball. Will the person be able
g = 10 ms −2 , the gun should be aimed to catch the ball? If yes, what should be the angle of
(a) directly at the target projection?
(b) 5 cm below the target (a) Yes, 60° (b) Yes, 30°
(c) 5 cm above the target (c) No (d) Yes, 45°
(d) 2 cm above the target 51 The initial velocity of a particle of mass 2 kg is
47 A projectile has the maximum range 500 m. If the (4$i + 4$j) ms −1. A constant force of − 20 $j N is
projectile is thrown up a smooth inclined plane of 30° applied on the particle. Initially, the particle was at
with the same (magnitude) velocity, the distance (0, 0 ). Find the x-coordinate of the point, where its
covered by it along the inclined plane till it stops will y-coordinate is again zero.
be (a) 3.2 m (b) 6 m (c) 4.8 m (d) 1.2 m
(a) 250 m (b) 500 m
(c) 750 m (d) 1000 m
52 An object of mass m is projected with a momentum p
at such an angle that its maximum height is 1/4th of
48 A projectile A is thrown at an angle 30° to the
its horizontal range. Its minimum kinetic energy in
horizontal from point P. At the same time, another
its path will be
projectile B is thrown with velocity v 2 upwards from
p2 p2 3p 2 p2
the point Q vertically below the highest point A (a) (b) (c) (d)
v 8m 4m 4m m
would reach. For B to collide with A, the ratio 2
v1 53 The equation of motion of a projectile are given by
should be x = 36 t m and 2 y = 96 t − 9.8 t 2 m. The angle of
v1
projection is
 4  3
v2 (a) sin−1   (b) sin−1  
 5  5
30°
P  4  3
(c) sin−1   (d) sin−1  
Q
3  3  4
(a) (b) 2
2 54 Two stones are projected so as to reach the same
1 2 distance from the point of projection on a horizontal
(c) (d)
2 3 surface. The maximum height reached by one exceeds
49 A ball is thrown from a point O aiming a target at the other by an amount equal to half the sum of the
angle 30° with the horizontal, so that the ball hits height attained by them. Then, angle of projection of
the target at B but the ball hits at point A, a vertical the stone which attains smaller height is
distance h below B. (a) 45° (b) 60° (c) 30° (d) tan−1 (3 / 4)
156 OBJECTIVE Physics Vol. 1

55 An arrow is shot into air. Its range is 200 m and its 61 A particle moves along a parabolic path y = − 9x 2 in
time of flight is 5 s. If g = 10 ms −2 , then horizontal such a way that the x-component of velocity remains
component of velocity and the maximum height will constant and has a value 1/3 ms −1. The acceleration
be respectively of the particle is
1 −2 2 −2
(a) 20 ms−1, 62.50 m (b) 40 ms−1, 31.25 m (a) ms (b) 3 ms−2 (c) ms (d) 2 ms−2
3 3
(c) 80 ms−1, 62.5 m (d) None of these
62 A projectile can have same range from two angles of
56 A body is projected from the ground with a velocity projection with same initial speed. If h1 and h 2 be
v = (3 $i + 10 $j ) ms −1. The maximum height attained the maximum heights, then
and the range of the body respectively are (a) R = h1 h 2 (b) R = 2 h1 h 2
(Take, g = 10 ms −2 ) (c) R = 2 h1 h 2 (d) R = 4 h1 h 2
(a) 5 m and 6 m (b) 3 m and 10 m
(c) 6 m and 5 m (d) 3 m and 5 m 63 A ball is projected from ground with a speed of
57 A cricket fielder can throw the cricket ball with a 20 ms −1 at an angle of 45° with horizontal. There is
speed v 0 . If he throws the ball while running with a wall of 25 m height at a distance of 10 m from the
speed u at an angle θ to the horizontal, what is the projection point. The ball will hit the wall at a height of
effective angle to the horizontal at which the ball is (a) 5 m (b) 7.5 m (c) 10 m (d) 12.5 m
projected in air as seen by a spectator? 64 A boy can throw a stone up to a maximum height of
[NCERT Exemplar] 10 m. The maximum horizontal distance that the boy
−1 v cos θ  can throw the same stone up to will be
(a) tan  0 
 u + v 0 sin θ  (a) 20 2 m (b) 10 m (c) 10 2 m (d) 20 m
 v sin θ  65 At the height 80 m, an aeroplane is moving with
(b) tan−1 0 
 u + v 0 cos θ  150 ms −1. A bomb is dropped from it so as to hit a
 
target. At what distance from the target should the
u
(c) tan−1  bomb be dropped? (Take, g = 10 ms −2 )
 v 0 cos θ + v 0 sin θ 
(a) 605.3 m (b) 600 m (c) 80 m (d) 230 m
 v sin θ + v 0 cos θ 
(d) tan−1 0  66 A ball is projected upwards from the top of a tower
 u 
with velocity 25 ms −1 making an angle of 30° with
−1
58 A particle is projected with a velocity of 30 ms , at the horizontal. If the height of the tower is 70 m,
3 after what time from the instant of throwing, will
an angle of θ 0 = tan −1   . After 1 s, the particle is the ball reach the ground? (Take, g = 10 ms −2 )
 4
moving at an angle θ to the horizontal, where tanθ (a) 2 s (b) 5 s (c) 7 s (d) 9 s
will be equal to (Take, g = 10 ms −2 ) 67 From the top of a tower of height 40 m, a ball is
1 1 projected upwards with a speed of 20 ms −1 at an
(a) 1 (b) 2 (c) (d)
2 3 angle of elevation of 30°. The ratio of the total time
taken by the ball to hit the ground to its time of
59 A bomber plane moves horizontally with a speed of
flight (time taken to come back to the same
500 ms −1 and a bomb released from it, strikes the elevation) is (Take, g = 10 ms −2 )
ground in 10 s. Angle at which it strikes the ground
(a) 2 : 1 (b) 3 : 1 (c) 3 : 2 (d) 1.5 :1
will be (Take, g = 10 ms −2 )
68 The coordinates of a moving particle at any time t are
 1  1
(a) tan−1  (b) tan−1  given by x = ct and y = bt 2 . The speed of the particle
 5  2
(c) tan−1(1) (d) tan−1(5)
is given by
(a) 2t b 2 − c 2 (b) 4b 2t 2 + c 2
60 Two balls are thrown simultaneously from ground
(c) 2t (b + c ) (d) 2t (b − c )
with same velocity of 10 ms −1 but different angles of
69 A ball rolls off the edge of a horizontal table top 4 m
projection with horizontal. Both balls fall at same
high. If it strikes the floor at a point 5 m horizontally
distance 5 3 m from point of projection. What is the away from the edge of the table, what was its speed
time interval between balls striking the ground? at the instant it left the table?
(a) ( 3 − 1) s (b) ( 3 + 1) s (c) 3s (d) 1 s (a) 2.5 ms−1 (b) 3.5 ms−1 (c) 5.55 ms−1 (d) 6.5 ms−1
Motion in a Plane and Projectile Motion 157

70 A ball is thrown from the ground to a wall 3 m high it is at height half of the maximum height. Find the
at a distance of 6 m and falls 18 m away from the angle of projection α with the horizontal.
wall, the angle of projection of ball is (a) 30° (b) 45°
(c) 60° (d) 37°
 3  2  1  3
(a) tan−1   (b) tan−1   (c) tan−1   (d) tan−1  
 2  3  2  4 77 A very broad elevator is going up vertically with a
constant acceleration of 2 ms −2 . At the instant when
71 The horizontal range and maximum height attained
by a projectile are R and H, respectively. If a its velocity is 4 ms −1, a ball is projected from the
constant horizontal acceleration a = g /4 is imparted floor of the lift with a speed of 4 ms −1 relative to the
to the projectile due to wind, then its horizontal floor at an elevation of 30°. The time taken by the
range and maximum height will be ball to return the floor is (Take, g = 10 ms −2 )
H  H
(a) (R + H ), (b) R +  , 2H 1 1 1
2  2 (a) s (b) s (c) s (d) 1 s
2 3 4
(c) (R + 2H ), H (d) (R + H ), H 78 A grasshopper can jump a maximum distance of
72 An object is projected with a velocity of 20 ms −1 1.6 m. How far can it go in 10 s?
making an angle of 45° with horizontal. The (a) 5 2 m (b) 10 2 m (c) 20 2 m (d) 40 2 m
equation for trajectory is h = Ax − Bx 2 , where h is
79 A body of mass 1kg is projected with velocity
height, x is horizontal distance, A and B are
50 ms −1 at an angle of 30° with the horizontal. At
constants. The ratio A : B is (Take, g = 10 ms −2 )
the highest point of its path, a force 10 N starts
(a) 1 : 5 (b) 5 : 1 (c) 1 : 40 (d) 40 : 1
acting on body for 5 s vertically upward besides
73 If the instantaneous velocity of a particle projected gravitational force. What is the horizontal range of
as shown in figure is given by v = a$i + (b − ct ) $j. the body? (Take, g = 10 ms −2 )
where a, b and c are positive constants, the range on (a) 125 3 m (b) 200 3 m
the horizontal plane will be (c) 500 m (d) 250 3 m
y
80 A projectile is thrown at an angle θ such that it is
v just able to cross a vertical wall at its highest point
as shown in the figure.
The angle θ at which the projectile is thrown is given
x by
(a) 2ab /c (b) ab /c (c) ac /b (d) a /2bc
74 Two particles are simultaneously projected in v0
opposite directions horizontally from a given point in H
space, where gravity g is uniform. If u1 and u 2 be
their initial speeds, then the time t after which their θ
velocities are mutually perpendicular is given by
√3 H
u1 u2 u12 + u22
(a) (b)  1 
g g (a) tan−1   (b) tan−1 3
 3
u1 (u1 + u2 ) u2 (u1 + u2 )
(c) (d)  3
 2 
g g (c) tan−1   (d) tan−1  
 3  2 
75 A projectile is fired at an angle of 30° to the
horizontal such that the vertical component of its 81 A jet aeroplane is flying at a constant height of 2 km
initial velocity is 80 ms −1 . Its time of flight is T. Its with a speed 360 kmh −1 above the ground towards a
velocity at t = T /4 has a magnitude of nearly target and releases a bomb. After how much time, it
(a) 200 ms−1 (b) 300 ms−1 will hit the target and what will be the horizontal
distance of the aeroplane from the target, so that the
(c) 100 ms−1 (d) None of these
bomb should hit the target? (Take, g = 10 ms −2 )
76 A projectile is thrown with some initial velocity at (a) 10 s, 1 km (b) 20 s, 2 km
an angle α to the horizontal. Its velocity when it is at (c) 30 s, 3 km (d) 40 s, 4 km
the highest point is (2/5) 1/ 2 times the velocity when
158 OBJECTIVE Physics Vol. 1

82 Balls A and B are thrown from two points lying on 87 For a ground-to-ground projectile, an object is at
the same horizontal plane separated by a distance T 5T
point A at t = , at point B at t = and reaches the
120 m. Which of the following is true? 3 6
ground at t = T . The difference in heights between
50 ms−1 points A and B is
30 ms−1 gT 2 gT 2 gT 2 gT 2
(a) (b) (c) (d)
A 37° B
6 12 18 24
88 A cart is moving horizontally along a straight line
120 m
with a constant speed of 30 ms −1. A projectile is to
(a) The two balls can never meet
be fired from the moving cart in such a way that it
(b) The balls can meet, if the ball B is thrown 1 s later
(c) The two balls meet at a height of 45 m will return to the cart (at the same point on cart)
(d) None of the above after the cart has moved 80 m. At what velocity
(relative to the cart) must projectile be fired? (Take,
83 Two particles are projected from the same point with
g = 10 ms −2 )
same speed u at angles of projection α and β from
20 −1
horizontal. The maximum heights attained by them (a) 10 ms−1 (b) ms
are h1 and h 2 respectively, R is the range for both. If 3
t1 and t 2 are their times of flight respectively, then 40 −1 80 −1
(c) ms (d) ms
which amongst the option(s) is/are incorrect? 3 3
π 89 Two second after projection, a projectile is travelling
(a) α + β =
2 in a direction inclined at 30° with the horizontal.
(b) R = 4 h1h 2 After 1 more second, it is travelling horizontally.
Then, (Take, g = 10 ms −2 )
t1
(c) tan α = = h1h 2 (a) the velocity of projection is 20 3 ms−1
t2
(b) the angle of projection is 30° with horizontal
h1 (c) Both (a) and (b) are correct
(d) tan α =
h2 (d) Both (a) and (b) are incorrect

84 A particle is projected from the ground at an angle θ 90 A ball rolls off top of a stair way with a horizontal
with the horizontal with an initial speed u. Time velocity u ms −1. If the steps are h metres high and
after which velocity vector of the projectile is b metres wide, the ball will just hit the edge of nth
perpendicular to the initial velocity. step, if n equals to
(a) u / g sin θ (b) u / g cos θ hu 2 u 2g
(a) (b)
(c) 2u / g sin θ (d) 2u tan θ gb 2 gb 2
2
85 A particle is projected from horizontal making an 2hu 2u 2g
−1 (c) (d)
angle of 53° with initial velocity of 100 ms . The gb 2 hb 2
time taken by the particle to make angle 45° from 91 A hill is 500 m high. Supplies are to be sent across
horizontal is the hill using a canon that can hurl packets at a
(a) 14 s (b) 2 s
(c) Both (a ) and (b ) (d) None of these speed of 125 ms −1 over the hill. The canon is
located at a distance of 800 m from the foot of
86 A large number of bullets are fired in all directions hill and can be moved on the ground at a speed of
with same speed v. What is the maximum area on 2 ms −1, so that its distance from the hill can be
the ground on which these bullets will spread? adjusted. What is the shortest time in which a
v2 v4 packet can reach on the ground across the hill?
(a) π (b) π
g g2 (Take, g = 10 ms −2 ) [NCERT Exemplar]
4
v v2 (a) 10 s (b) 25 s
(c) π 2 (d) π 2
g2 g2 (c) 35 s (d) 45 s
(B) Medical entrance special format questions
Assertion and reason vector can be anything between 0 to π (excluding
the limiting case.)
Directions (Q. Nos. 1-3) These questions consists of two
statements each printed as Assertion and Reason. While II. In projectile motion, acceleration vector is always
answering these questions you are required to choose any pointing vertically downwards. (Neglect air
one of the following four responses friction).
(a) If both Assertion and Reason are correct and Reason is the Which of the following statement(s) is/are correct?
correct explanation of Assertion. (a) Only I (b) Only II
(b) If both Assertion and Reason are correct but Reason is not (c) Both I and II (d) Neither I nor II
the correct explanation of Assertion.
(c) If Assertion is correct but Reason is incorrect. 4 I. Particle-1 is dropped from a tower and particle-2
(d) If Assertion is incorrect but Reason is correct. is projected horizontal from the same tower, then
1 Assertion In case of projectile motion, the both the particles reach the ground
magnitude of rate of change of velocity is variable. simultaneously.
Reason In projectile motion, magnitude of velocity first II. Both the particles strike the ground with different
decreases and then increases during the motion. speeds.
2 Assertion At highest point of a projectile, dot Which of the following statement(s) is/are correct?
product of velocity and acceleration is zero. (a) Only I (b) Only II
(c) Both I and II (d) Neither I nor II
Reason At highest point, velocity and acceleration are
mutually perpendicular. Match the columns
3 Assertion If a particle is projected vertically 1 A particle is projected from ground with velocity u at
upwards with velocity u, the maximum height angle θ from horizontal. Match the following two
attained by the particle is h1. The same particle is columns and mark the correct option from the codes
projected at angle 30° from horizontal with the same given below.
speed u. Now the maximum height is h 2 . Thus, Column I Column II
h1 = 4h 2 .
(A) Average velocity between initial and (p) u sinθ
Reason In first case, v = 0 at highest point and in final points
second case, v ≠ 0 at highest point. (B) Change in velocity between initial (q) u cosθ
and final points
Statement based questions (C) Change in velocity between initial (r) zero
and peak points
1 A particle (A) is dropped from a height and another (D) Average velocity between initial and (s) None
particle (B) is thrown in horizontal direction with highest points
speed of 5 m/s from the same height. The correct
Codes
statement is A B C D A B C D
(a) both particles will reach at ground simultaneously (a) p s r q (b) p r q s
(b) both particles will reach at ground with same speed (c) q s p s (d) r p q s
(c) particle (A) will reach at ground first with respect to 2 A particle is projected horizontally from a tower
particle (B ) with velocity 10 ms −1. Taking, g = 10 ms −2 . Match
(d) particle (B ) will reach at ground first with respect to
the following two columns at time t = 1s and mark
particle (A)
the correct option from the codes given below.
2 A ball is rolled off the edge of a horizontal table at a
Column I Column II
speed of 4 ms −1. It hits the ground after 0.4 s.
(A) Horizontal component of velocity (p) 5 SI units
Which statement given below is true?
(a) It hits the ground at a horizontal distance 1.6 m from (B) Vertical component of velocity (q) 10 SI unit
the edge of the table. (C) Horizontal displacement (r) 15 SI unit
(b) The speed with which it hits the ground is 4.0 ms−1. (D) Vertical displacement (s) 20 SI unit
(c) Height of the table 1m.
(d) It hits the ground at an angle of 60° to the horizontal. Codes
A B C D A B C D
3 I. In projectile motion, the angle between (a) p q s r (b) q q s p
instantaneous velocity vector and acceleration (c) q p r s (d) q q q p
OBJECTIVE Physics Vol. 1

(c) Medical entrances’ gallery


Collection of questions asked in NEET & various medical entrance exams

1 When an object is shot from the bottom of a long 6 Assertion When θ = 45° or 135°, the value of R
smooth inclined plane kept at an angle 60º with remains the same, only the sign changes.
horizontal, it can travel a distance x 1 along the u 2 sin 2θ
plane. But when the inclination is decreased to 30º Reason R = [AIIMS 2017]
g
and the same object is shot with the same velocity, it
can travel x 2 distance. Then, x 1 : x 2 will be (a) Both Assertion and Reason are correct and Reason is the
[NEET 2019] correct explanation of Assertion.
(a) 2:1 (b) 1: 3 (c) 1:2 3 (d) 1: 2 (b) Both Assertion and Reason are correct and Reason is not
the correct explanation of Assertion.
2 Two bullets are fired horizontally and (c) Assertion is correct but Reason is incorrect.
simultaneously towards each other from roof tops of (d) Assertion is incorrect but Reason is correct.
two buildings 100 m apart and of same height of
7 The x and y-coordinates of a particle moving in a
200 m with the same velocity of 25 ms −1. When and
where will the two bullets collides? plane are given by x (t ) = a cos(pt ) and y (t ) = b sin(pt ),
where a , b (< a ) and p are positive constants of
(Take, g = 10 ms −2 ) [NEET (Odisha) 2019]
appropriate dimensions and t is time. Then, which of
(a) After 2s at a height of 180 m the following is not true? [JIPMER 2017]
(b) After 2s at a height of 20 m
(a) The path of the particle is an ellipse.
(c) After 4s at a height of 120 m
(b) Velocity and acceleration of the particle are
(d) They will not collide π
perpendicular to each other at t = .
3 Assertion The maximum height of projectile is 2p
always 25% of the maximum range. (c) Acceleration of the particle is always directed towards
Reason For maximum range, projectile should be a fixed point.
projected at 90°. [AIIMS 2018] (d) Distance travelled by the particle in time interval
π
(a) Both Assertion and Reason are correct and Reason is the between t = 0 and t = is a.
correct explanation of Assertion. 2p
(b) Both Assertion and Reason are correct and Reason is 8 A particle moves, so that its position vector is given
not the correct explanation of Assertion.
by r = cos ωtx$ + sin ωt y$ , where ω is a constant.
(c) Assertion is correct but Reason is incorrect.
(d) Assertion is incorrect but Reason is correct.
Which of the following is true? [NEET 2016]
(a) Velocity and acceleration both are parallel to r.
4 What is the range of a projectile thrown with (b) Velocity is perpendicular to r and acceleration is
velocity 98 ms −1 with angle 30° from horizontal ? directed towards to origin.
(a) 490 3 m (b) 245 3 m [JIPMER 2018] (c) Velocity is perpendicular to r and acceleration is
(c) 980 3 m (d) 100 m directed away from the origin.
(d) Velocity and acceleration both are perpendicular to r.
5 A block is dragged on a smooth plane with the help
of a rope which moves with a velocity v as shown in 9 A particle is projected with an angle of projection θ
the figure. The horizontal velocity of the block is to the horizontal line passing through the points
(P, Q ) and (Q, P ) referred to horizontal and vertical
v
axes (can be treated as X-axis andY-axis,
θ
respectively).
The angle of projection can be given by [AIIMS 2015]
P 2 + PQ + Q 2  P 2 + Q 2 − PQ 
[AIIMS 2017] (a) tan− 1   (b) tan− 1  
 PQ   PQ 
v
(a) (b) v sin θ
sin θ P 2 + Q 2  P 2 + Q 2 + PQ 
v (c) tan− 1   (d) sin− 1  
(c) (d) v cos θ  2PQ   2PQ 
cos θ
Motion in a Plane and Projectile Motion 161

10 An object is thrown towards the tower which is at a 17 A body is projected horizontally from the top of a
horizontal distance of 50 m with an initial velocity of tower with a velocity of 10 ms −1. If it hits the
10 ms −1 and making an angle 30° with the horizontal. ground at an angle of 45°, then the vertical
The object hits the tower at certain height. The height
from the bottom of the tower, where the object hits component of velocity when it hits ground (in ms −1) is
[EAMCET 2014]
the tower is (Take, g = 10 ms −2 ) [EAMCET 2015]
(a) 10 2 (b) 5 2 (c) 5 (d) 10
50  10  50  10 
(a) 1− m (b) 1 − 3  m 18 A body is projected with an angle θ. The maximum
3  3  3   height reached is h. If the time of flight is 4 s and
100 
1−
10  100  10  g = 10 ms −2 , then value of h is
1 − 3  m
(c) m (d) [EAMCET 2014]

3  3  3   (a) 40 m (b) 20 m (c) 5 m (d) 10 m
11 The range of a projectile is R when the angle of 19 The velocity of a projectile at the initial point A is
projection is 40°. For the same velocity of projection (2 $i + 3 $j ) ms −1. Its velocity (in ms −1) at point B is
and range, the other possible angle of projection is [NEET 2013]
[Kerala CEE 2015] Y
(a) 45° (b) 50° (c) 60° (d) 40°
(e) 90°
12 A particle is projected with a velocity v, so that its
horizontal range twice the greatest height attained. B
The horizontal range is [KCET 2015] A X
2 2 2 2
4v v v 2v
(a) (b) (c) (d)
5g g 2g 3g (a) −2 i$ − 3$j (b) −2 i$ + 3$j (c) 2 i$ − 3$j (d) 2 i$ + 3j$

13 If the angle of projection of a projector with same 20 A projectile is thrown with initial velocity u 0 and
initial velocity exceed or fall short of 45° by equal angle 30° with the horizontal. If it remains in the air
amount α, then the ratio of horizontal ranges is for 1s, what was its initial velocity? [J & K CET 2013]
[Kerala CEE 2014] (a) 19.6 ms −1 (b) 9.8 ms −1 (c) 4.9 ms −1 (d) 1 ms −1
(a) 1 : 2 (b) 1 : 3 (c) 1 : 4 (d) 1 : 1 21 A projectile is projected at 10 ms −1 by making at an
(e) 1 : 2 angle 60° to the horizontal. After some time, its
14 A particle is moving such that its position velocity makes an angle of 30° to the horizontal. Its
coordinates (x, y ) are (2m, 3m ) at time t = 0, (6m, 7m ) speed at this instant is [KCET 2013]
at time t = 2 s and (13m, 14m ) at time t = 5 s. Average (a)
10
(b) 10 3 (c)
5
(d) 5 3
velocity vector (v av ) from t = 0 to 5 s is 3 3
[CBSE AIPMT 2014]
1 7 22 Two particles are projected upwards with the same
(a) (13$i + 14$j) (b) ($i + $j) initial velocity v 0 in two different angles of projection
5 3
11 $ $ such that their horizontal ranges are the same. The
(c) 2 ($i − $j) (d) ( i + j) ratio of the heights of their highest point will be
5 [UP CPMT 2013]
15 A cricket ball thrown across a field is at heights h1 (a) tan 2 θ1 (b) v 02 sin 2 θ1 (c) v 0 sin θ1 (d) v 0 / cos θ1
and h 2 from the point of projection at times t1 and t 2 23 The velocity vector of the motion described by the
respectively after the throw. The ball is caught by a position vector of a particle r = 2t$i + t 2 $j is given by
fielder at the same height as that of projection. The [J & K CET 2013]
time of flight of the ball in this journey is [WB JEE 2014]
(a) v = 2i$ + 2t $j (b) v = 2t i$ + 2t $j
 h t 2 − h 2t12   h t 2 − h 2t 22 
(a)  1 2  (b)  1 1  (c) v = t i$ + t 2j$ (d) v = 2i$ + t 2j$
 h1t1 − h 2t 2   h1t 2 − h 2t1 
24 The horizontal range and the maximum height of a
 h t 2 − h 2t12 
(c)  1 2  (d) None of these projectile are equal. The angle of projection of the
 h1t 2 − h 2t1  projectile is [CBSE AIPMT 2012]

16 For an object thrown at 45° to the horizontal, the  1


(a) θ = tan−1  (b) θ = tan−1(4)
maximum height H and horizontal range R are  4
related as [UK PMT 2014]
(c) θ = tan−1(2) (d) θ = 45°
(a) R = 16 H (b) R = 8 H (c) R = 4 H (d) R = 2 H
162 OBJECTIVE Physics Vol. 1

25 Trajectories of two projectiles are shown in figure, 26 If for the same range, the two heights attained are
let T1 and T 2 be the periods and u1 and u 2 are their 20 m and 80 m, then the range will be [BHU 2012]
speeds of projections, then [UP CPMT 2012] (a) 20 m (b) 40 m
Y (c) 120 m (d) 160 m
27 A ball thrown by one player reaches the other in 2 s.
The maximum height attained by the ball above the
2
point of projection will be (Take, g = 10 ms −2 )
1 [BHU 2012]
X (a) 2.5 m (b) 5 m
(a) T2 > T1 (b) T1 = T2 (c) 7.5 m (d) 10 m
(c) u1 > u2 (d) u1 < u2

ANSWERS
CHECK POINT 4.1
1. (a) 2. (a) 3. (b) 4. (c) 5. (c) 6. (b) 7. (b) 8. (b) 9. (c) 10. (d)
11. (a) 12. (c) 13. (a)

CHECK POINT 4.2


1. (d) 2. (d) 3. (a) 4. (d) 5. (b) 6. (c) 7. (a) 8. (b) 9. (c) 10. (a)
11. (d) 12. (c) 13. (a) 14. (d) 15. (d) 16. (d) 17. (d) 18. (c) 19. (b) 20. (a)

CHECK POINT 4.3


1. (a) 2. (b) 3. (b) 4. (a) 5. (c) 6. (b) 7. (a)

(A) Taking it together


1. (d) 2. (c) 3. (b) 4. (c) 5. (d) 6. (c) 7. (d) 8. (c) 9. (c) 10. (b)
11. (d) 12. (d) 13. (d) 14. (b) 15. (c) 16. (c) 17. (b) 18. (c) 19. (a) 20. (c)
21. (d) 22. (d) 23. (d) 24. (c) 25. (d) 26. (b) 27. (d) 28. (a) 29. (c) 30. (a)
31. (d) 32. (a) 33. (c) 34. (a) 35. (c) 36. (b) 37. (a) 38. (a) 39. (c) 40. (b)
41. (a) 42. (b) 43. (b) 44. (c) 45. (d) 46. (c) 47. (b) 48. (c) 49. (a) 50. (a)
51. (a) 52. (b) 53. (a) 54. (c) 55. (b) 56. (a) 57. (b) 58. (d) 59. (a) 60. (a)
61. (d) 62. (d) 63. (b) 64. (d) 65. (a) 66. (c) 67. (a) 68. (b) 69. (c) 70. (b)
71. (d) 72. (d) 73. (a) 74. (a) 75. (d) 76. (c) 77. (b) 78. (c) 79. (d) 80. (c)
81. (b) 82. (c) 83. (d) 84. (a) 85. (c) 86. (b) 87. (d) 88. (c) 89. (a) 90. (c)
91. (d)

(B) Medical entrance special format questions


l Assertion and reason
1. (d) 2. (a) 3. (b)
l Statement based questions
1. (a) 2. (a) 3. (c) 4. (c)
l Match the columns
1. (c) 2. (d)
(C) Medical entrances’ gallery
1. (b) 2. (a) 3. (c) 4. (a) 5. (a) 6. (a) 7. (d) 8. (b) 9. (a) 10. (a)
11. (b) 12. (a) 13. (d) 14. (d) 15. (c) 16. (c) 17. (d) 18. (b) 19. (c) 20. (b)
21. (a) 22. (a) 23. (a) 24. (b) 25. (d) 26. (d) 27. (b)
Hints & Explanations
l CHECK POINT 4.1 ∴Magnitude of the velocity of the object along Y-axis = 6 m/s
1 (a) Vector P is shown in the figure, Y 8 (b) Given, x = 2t 3
then according to the given information, dx dv
Px = 5, Py = 6 ∴ vx = = 6t 2 ⇒ a x = x = 12t
Py P dt dt
∴ | P | = Px2 + Py2 = 25 + 36 θ X dy dv y
Px Also, y = 3t ⇒ v y =
3
= 9t 2 ⇒ a y = = 18t
dt dt
⇒ | P | = 61
∴ Acceleration, a = a x2 + a y2 = t 468
Py6  6
and tan θ = = ⇒ θ = tan−1  
Px 5  5 dx dv
9 (c) v x = = 6t − 6, a x = x = 6 ms −2
dt dt
Thus, P have magnitude of 61and lies in XY-plane at an
dv y
 6
angle tan−1   to the X-axis. v y = 2t − 2, a y = = 2 ms −2
 5 dt
At t = 1s, v x and v y both are zero. Hence, net velocity is zero.
2 (a) Positions of the object are
∆v 3$i − 2$j − 2$i − 3$j $i − 5$j
r = 6$i + 8$j and r = 12$i + 10 $j 10 (d) aav = = =
1 2 ∆t 2 2
∴ ∆r = r2 − r1 = 6$i`+ 2$j ⇒ | ∆r| = 40 11 (a) v = u + a t = (4 $i + 3$j ) + (0.4$i + 0.3$j ) (10 ) = (8$i + 6$j )
 ∆y   1
θ = tan−1  = tan−1  = 18.43° ∴ v = (8)2 + (6)2 = 10 units
 ∆x   3
12 (c) Position vector of the body, r = 3$i + 7$j
3 (b) Displacement, ∆r = r2 − r1 = 4$i + 4$j
Q Acceleration of the body,
∆r 4$i + 4$j
∴ v av = = = 2($i + $j ) ms −1 a = 4$i and t = 3 s
∆t 2
1 4$i
⇒ Magnitude of velocity, Using s = u0t + at 2 = 0 × 3 + × 3 × 3 ⇒ s = 18$i
2 2
|v av| = 2 12 + 12 = 2 2 ms −1 ∴ New position vector of the body,
 ∆v y  r1 = 3$i + 7$j + 18$i = 21 $i + 7$j
−1 2
Direction, θ = tan−1 −1
 = tan   = tan 1 = 45°
 ∆v x   2 Coordinates becomes (21, 7)
4 (c) v x = 4,t v y = 2t − 4, v z = 3 13 (a) Initial position vector, r1 = 3$i − 8$j
At t = 0, v x = 0, v y = − 4 and v z = 3 Final position vector, r2 = 2$i + 4$j
∴ v = v y2 + v z2 = 5 units Change in position,
∆r = r2 − r1 = 2$i + 4$j − 3$i + 8$j = − $i + 12$j
dx 2t
5 (c) As, vx = = =t 1 2
dt 2 Using s = u0t + at
At t = 2 s, v x = 2 ms −1 2
1 − $i + 12$j − $i 3 $
x 2 (t 2 / 2)2 t 4 dy t 3 ⇒ − $i + 12$j = 0 + a (4)2 ⇒ a = = + j
Further, y= = = ⇒ vy = = 2 8 8 2
2 2 8 dt 2
At t = 2 s, v y = 4 ms −1 l CHECK POINT 4.2
So, velocity of particle is (2$i + 4$j ) ms −1. 1 (d) Velocity is horizontal and acceleration is vertically
downward. Therefore, at the top of the trajectory of a
6 (b) Given, r = (a sin ωt ) $i + (a cos ωt ) $j projectile, the direction of its velocity and acceleration are
dr perpendicular to each other.
∴ v= = (−aω sin ωt ) $j + (aω cos ωt ) $i
dt 2 (d) Acceleration throughout the projectile motion remains
r⋅ v = 0 ⇒ r ⊥ v constant and equal to g.
7 (b) Position vector, r = 3t 2$i + 6t$j + k$ 3 (a) An external force of gravity is present throughout the
dr motion. So, momentum will not be conserved. Its total
Velocity vector, v = = 6t$i + 6$j + 0 = 6t$i + 6$j mechanical energy is conserved.
dt
164 OBJECTIVE Physics Vol. 1

4 (d) a x = 0 ∴ u x = constant
17 (d) R = 4H cot θ
5 (b) At highest point, vertical component of velocity is zero. R 4
Only horizontal component of velocity is present. If θ = 45°, then =
H 1
v x = u cos θ = 50 cos(60 ° ) = 25 ms −1
18 (c) Given, range, R = 4 3H
2u sin θ 2 × 50 × sin 30 °
6 (c) Time of flight, T = = =5s u 2 sin θ cos θ  u 2 sin2 θ  1
g 10 ⇒ =4 3  ⇒ tanθ =
g  2g  2 3
7 (a) The particle hits the horizontal plane again at time,
2u sin θ 40 × sin 60 ° ∴ θ = 16.1°
T= = = 3.53 s
g 9.8 vy u y − gt (20 3 sin 60 ° ) − 10 t
19 (b) tan 30° = = =
u12 sin2 45° u 22 sin2 60 ° vx ux (20 3 cos 60 ° )
8 (b) =
2g 2g or 10 = 30 − 10t
u1 sin 60 ° 3 /2 ∴ t =2s
∴ = = = 3: 2
u 2 sin 45° (1/ 2 ) 20. (a) v 2 = v y2 + v x2 or 5g = (u y − gt )2 + u x2
u 2 sin2 θ Since, v y = u sin 30 ° = 10 sin 30 ° = 5 3 ms −1
9 (c) H= ⇒ H ∝ u2
2g
v x = u cos 30 ° = 10 cos 30 ° = 5 ms −1
If initial velocity be doubled, then maximum height reached
by the projectile will become four times. or 50 = (5 3 − 10t )2 + (5)2
u 2 sin2 θ ∴ (5 3 − 10t ) = ± 5
10 (a) Maximum height, H =
2g 5 3+5 5 3−5
⇒ t1 = and t2 =
2u sinθ 10 10
and time of flight, T =
g ∴ t1 − t2 = 1 s
H u 2 sin2 θ/ 2g g 5
So, = = = l CHECK POINT 4.3
T 2 4u 2 sin2 θ/ g 2 8 4
1 (a) Horizontal component of velocity of the bomb is same as
11 (d) Given at maximum height, velocity of the aeroplane. Therefore, the bomb falls exactly
1 1 below the aeroplane.
u cos θ = u ⇒ cos θ = ⇒ θ = 60 °
2 2 2 (b) v y = gt = 7 ms −1, v x = 4 ms −1
u 2 sin2 θ u 2 sin2 60° 3 u 2
∴ H= = = ∴ v = v x2 + v y2 ≈ 8 ms −1
2g 2g 8g
12 (c) Projectile will strike at highest point of its path with 3 (b) Let horizontal direction is x and y is downward direction.
velocity v 0 cos α. After time t, v x = u x = 20 ms −1 vx
u sin 2 θ (9.8) sin 90 °
2 2 a = 45°
Velocity in y-direction,
13 (a) Horizontal range, R = = = 9.8 m
g 9.8 v y = u y + a yt = 0 + gt = gt
vy
T sin 30 ° 1 vy gt v
14 (d) T ∝ sin θ , A = = or TB = 3 TA tan α = =
TB sin 60 ° 3 v x 20
H A sin2 30 ° 1 10 × t
H ∝ sin2 θ , = = or H B = 3 H A ∴ 1= ⇒ t =2s
H B sin2 60 ° 3 20
As, Rθ = R90 ° − θ 4 (a) Let t be the time taken by bomb to hit the target.
∴ RA = RB 1 2
h = 2000 = gt ⇒ t = 20 s
u 2 sin 2θ 2
15 (d) R= 100 ms−1
g
∴ R ∝ u2
θ
If initial velocity be doubled at some angle of projection, then h = 2000
range will become four times. θ
u 2 sin 2θ R
R g 2 sin θ cos θ 2g
16 (d) = 2 2 = × 2 = 4 cot θ R = ut = (100 )(20 ) = 2000 m
H u sin θ g sin θ
R 2000
2g Q tan θ = = = 1 ⇒ θ = 45°
h 2000
⇒ R = 4H cot θ; If θ = 45°, then R = 4 H cot(45° ) = 4 H
Motion in a Plane and Projectile Motion 165

5 500 4 (c) We know that, Rθ = R90 ° − θ


5 (c) Plane is flying at a speed = 600 × = ms −1
18 3
horizontally (at a height 1960 m) ∴ R1 = R2
u 2 sin 2θ 2u xu y
5 (d) R = =
A g g
∴ Range ∝ Horizontal initial velocity component (u x )
θ
In path 4, range is maximum, so football has maximum
h horizontal velocity component in this path and in path 1,
range is minimum. So, horizontal velocity component is
minimum.
x P 8 (c) Given, θ = 15 ° and R = 50 m

Time taken by the kit to reach the ground, u 2 sin 2 θ


Range, R =
2h 2 × 1960 g
t= = = 20 s
g 9.8 Putting all the given values in the formula, we get
u 2 sin (2 × 15° )
In this time, the kit will move horizontally by R = 50 m =
500 10000 g
x = ut = × 20 = m
3 3 1
⇒ 50 × g = u 2 sin 30 ° = u 2 × ⇒ 50 × g × 2 = u 2
So, the angle of sight, 2
x 10000 10 ⇒ u 2 = 50 × 9.8 × 2 = 980
tan θ = = =
h 3 × 1960 5.88 ⇒ u = 980
~ 3 or θ = 60 °
= 1.7 − = 31.304 ms −1
v gt u sin 2 × 45° u 2
2
6 (b) tan 60 ° = V = For θ = 45°, R = = (Q sin 90° = 1)
vH v0 g g
3 v0 (14 5 )2 14 × 14 × 5
∴ t= ⇒ R= = = 100 m
g g 9.8
1 2
gt 2h 5
7 (a)
AB 2
= = tanθ 9 (c) s = u × ⇒ 10 = u 2 ×
BC v 0t g 10
2v 0 tanθ
A v0 ⇒ u = 10 ms −1
∴ t=
g 10 (b) Given, u x = 3 ms −1, a x = − 6 ms −1
2v 02 tanθ θ
Now, x-coordinate = v 0t = u x2 9
g
B C ∴ Xmax = = = 0.75 m
2 |a x | 12
and y-coordinate
1 2
1 2v 2 tan2 θ θ 11 (d) h= gt (In vertical direction)
= − gt 2 = − 0 2
2 g
2h 2 × 0.1
⇒ t= = = 0.141 s
g 10
(A) Taking it together Now, in horizontal direction, v x =
sx
=
100
≈ 700 ms −1
1 (d) We know that, t 0.141
ds u 2 sin2 α
Instantaneous speed, v = 12 (d) Maximum height, H = ⇒ H ∝ sin2 α
dt 2g
Since, v is a positive constant, hence in this situation, equal H1 sin2 α
∴ = = tan2 α
path lengths are traversed in equal intervals of time by the H 2 sin (90 ° − α )
2

moving object.
13 (d) Given, RA = RB
2 (c) As given motion is two dimensional motion and it is given
that instantaneous speed v 0 is positive constant. Acceleration v 2 sin 2θ (v / 2)2 sin 90 °
⇒ =
is rate of change of velocity (instantaneous speed), hence it g g
will also be in the plane of motion. 1 1  1
⇒ sin 2θ = or θ = sin−1  
v f − vi (3$i − 2$i ) − (2$j − 3$j ) $i + $j 4 2  8
3 (b) a = = =
t 2 2
166 OBJECTIVE Physics Vol. 1

x u2
14 (b) Q x = kt ⇒ t = 26 (b) At θ = 45°, Rmax = = 80
k g
 x  x
Now, y = k   1 − α ⋅  ∴ u 2 = 800 ms −2
k  k
α x2 u 2 sin2 45° (800 ) (1/ 2)
or y =x− Now, H= = = 20 m
k 2g 20
u2 u
15 (c) Rmax = at θ = 45° 27 (d) Given, u cos θ = ⇒ θ = 60 °
g 2
u 2 sin 2θ u 2 sin 120 ° 3 u2
∴ u = g Rmax = 100 ms −1 Now, R = = =
g g 2g
16 (c) Compare the given equation with 2u xu y 2 × 9.8 × 19.6
gx 2 28 (a) R = = = 39.2 m
y = x tan θ − 2 g 9.8
2u cos 2 θ
u2
⇒ tanθ = 3 ⇒ θ = 60 ° 29 (c) Maximum height, H = =h (Given)
2g
17 (b) ∆ v = a ∆t, (As, a = constant) u 2 sin 90 ° u 2
Now, Rmax = = = 2h
 2v sin α  g g
= (− g $j )  0  = (− 2v 0 sin α )$j
 g  30 (a) When the angle of projection is very far from 45°, then
i.e. Change in velocity is 2v 0 sin α, vertically downwards. range will be minimum. Therefore, the body P with angle of
projection 15° will have a shortest range.
2u y
18 (c) H and R both are proportional to u 2. Hence, percentage 31 (d) Time of flight, T = =3
increases in horizontal range will also be 10%. g
3v ∴ u y = 15 ms −1
19 (a) At highest point, velocity will remain v cos 30° or .
2 u y2 (15)2
3p Now, H = = = 11.25 m
Therefore, momentum will also remain . 2g 20
2
32 (a) Velocity of boy should be equal to the horizontal
H 2H (u 2 sin2 θ / g )
20 (c) tan φ = = = 2 component of velocity of ball, i.e. u cos θ.
R /2 R u sin 2θ / g u 2 sin(90 ° − 2θ ) u 2 cos 2θ
33 (c) For angle (45° − θ ), R = =
sin2 θ tan θ g g
⇒ tan φ = =
2 sin θ cos θ 2 u 2 sin(90 ° + 2θ ) u 2 cos 2θ
For angle (45° + θ ), R = =
21. (d) The kinetic energy of projectile first decreases from ground g g
to highest point and after that increases upto ground. At
highest point, kinetic energy will be minimum but not zero. Thus, ratio becomes 1: 1.
1 2u sin θ
22 (d) R ∝ 34 (a) Time of flight, T = = 10 s
g g
∴ Rmoon = 6 Rearth
1
(Q gmoon = gearth) ⇒ u sinθ = 50 ms −1
6
u 2 sin2 θ (u sin θ )2 50 × 50
23 (d) At 45°, range is maximum. At 15° and 75°, ranges are ∴ H= = = = 125 m
2g 2g 2 × 10
equal.
As, Rθ = R90 ° − θ 35 (c) Instantaneous velocity of rising mass after t second will be
Q R15° = R75° < R45° v t = v x2 + v y2
⇒ R1 = R3 < R2 where, v x = v cos θ = horizontal component of velocity
2u xu y 2u y2 v y = v sinθ − gt = vertical component of velocity.
24 (c) R = 2H or =
g 2g ⇒ v t = (v cos θ )2 + (v sin θ − gt )2
or 2u x = u y or 2a = b
⇒ v t = v 2 + g 2 t 2 − (2v sin θ ) gt
25 (d) Speed of projectile at the point of projection = u
u 2 sin2 θ 2u sin θ 4u 2 sin2 θ
Speed of projectile at the top of its trajectory = u cos θ 36 (b) H= and T = ⇒T2 =
2g g g2
u 1  1
∴ = x or cos θ = ⇒ θ = cos −1  
u cos θ x  x T2 8 8H 2H
∴ = ⇒ T= =2
H g g g
Motion in a Plane and Projectile Motion 167

2
37 (a) K = K0 − mgh; Here K = kinetic energy at height h, 1  1 1
h= × 10 ×   = m = 5 cm
K0 = initial kinetic energy. Variation of K with h is linear. At 2  10  20
highest point, kinetic energy is not zero. ∴ Gun should be aimed 5 cm above the target.
u 2 sin 2θ u2
38 (a) R = at angles θ and 90° − θ 47 (b) Rmax = = 500
g g
2u sinθ 2u sin(90 ° − θ ) 2u cos θ u2 u2 u2
Now, t1 = and t2 = = Now, s = = = = 500 m
g g g 2a 2g sin 30 ° g
2  u 2 sin 2θ  2R
∴ tt12 =   = 48 (c) Vertical component of velocity of projectile A should be
g g  g equal to vertical velocity of projectile B.
1 v
39 (c) At highest point, speed will remain times (= u cos 45° ). or v1 sin 30 ° = v 2 or 1 = v 2
2 2
Therefore, kinetic energy will become 1/2 times. v2 1
∴ =
3 v1 2
40 (b) Substitute y = 0, 0 = 12x − x 2 ⇒ x = 16 m
4 49 (a) In moving horizontal distance 10 m, the ball will fall by
T  u cos θ 1
41 (a) ∆v = a∆t = a ⋅   = (− g $j )   = (− u cos θ )$j distance gt 2.
 2  g  2
As, R = u cos θt
Therefore, change in velocity is u cos θ in downward
direction. 3 1
⇒ 10 = 20 × cos 30 ° t = 20 × t ⇒ t= s
gx 2 2 3
42 (b) Comparing with y = x tan θ − , we have 2
2u cos 2 θ
2 1 2 1  1 g
AB = h = gt = × g ×   = m
tanθ = 3 or θ = 60 ° and u 2 cos 2 θ = 1 2 2  3 6
or u = sec θ = sec 60 ° = 2 ms −1 50 (a) Person will catch the ball, if his velocity will be equal to
horizontal component of velocity of the ball.
43 (b) Range of projectile launched at an angle θ is same as the v 1
range of projectile launched at angle 2θ. ⇒ 0 = v 0 cos θ ⇒ cos θ = ⇒ θ = 60 °
2 2
u 2 sin 2(2θ ) u 2 sin 2θ
⇒ = 51 (a) We can see that it is like a projectile motion, with
g g
u x = 4 ms −1, u y = 4 ms −1 and a y = 10 ms −2.
⇒ sin 2(2θ ) = sin 2θ 2u xu y 2 × 4 × 4
⇒ 2 sin 2θ cos 2θ = 2 sin θ cos θ x-coordinate = Range = = = 3.2 m
g 10
4 sin θ cos θ cos 2θ = 2 sin θ cos θ
R
1 52 (b) It is given that, H =
⇒ 4 cos 2θ = 2 ⇒ cos 2θ = 4
2
u sin θ 2u sin θ cos θ
2 2 2
⇒ cos 2θ = cos 60 ° ∴ = or tanθ = 1 ⇒ θ = 45°
2g 4g
⇒ 2θ = 60 °
p
∴ θ = 30 ° At highest point, momentum will remain .
2
44 (c) Horizontal component of velocity remains unchanged.
(p / 2 ) 2 p2
Hence, v cos φ = u cos θ ∴ K= =
∴ v = u cos θ sec φ 2m 4m

2h 2 × 396.9 53 (a) It is given that, x = 36 t


45 (d) T = = = 9s
g 9.8 dx
∴ vx = = 36 ms −1 and y = 48t − 4.9 t 2
and u = 720 km/h = 200 m/s dt
∴ R = u × T = 200 × 9 = 1800 m ∴ v y = 48 − 9.8 t
At t = 0 v x = 36 ms −1 and v y = 48 ms −1
46 (c) Let t be time taken by the bullet to hit the target.
∴ 200 m = 2000 ms −1t v y 
So, angle of projection, θ = tan−1  
200 m 1 v x 
⇒ t= = s
2000 ms −1 10  48  4
= tan−1  = tan−1  
For vertical motion, here u = 0  36  3
1  4
∴ h = gt 2 or θ = sin−1  
2  5
168 OBJECTIVE Physics Vol. 1

H1 + H 2 v y = 0 + 10 × 10 = 100 ms −1
54 (c) It is given that, H1 − H 2 = or H1 = 3H 2
2 vx = 500 ms–1
u 2 sin2 θ  u 2 sin2 (90 ° − θ )
∴ =3 
2g  2g 
⇒ tan2 θ = 3
500 ms–1
∴ tanθ = 3 or θ = 60 ° θ
Therefore, the other angle is 90° − θ or 30°.
100 ms–1
2u y
55 (b) Time of flight, T = ∴ Angle with which it strikes the ground,
g
v y   100  −1  1
gT θ = tan−1   = tan−1   = tan  
⇒ uy = = 25 ms −1 v x   500   5
2
u y2 (25)2 (10 )2 sin 2θ 3
60 (a) Given, 5 3 = or sin 2θ =
Now, H = = = 31.25 m g 2
2g 20
∴ 2θ = 60 ° or θ = 30 °
Further, R = ux T
R Two different angles of projection are therefore θ and 90° − θ
⇒ u x = = 40 ms −1 or 30° and 60°.
T
2u sin 30 °
56 (a) Given, v = (3$i + 10 $j ) ms −1, v x = 3 ms −1, v y = 10 ms −1 ∴ T1 = = 1s
g
v y2 100 2u sin 60 °
H= = = 5m ∴ T2 = = 3s
2g 2 × 10 g
2 vy Hence, ∆t = T2 − T1 = ( 3 − 1) s
R =vx × T =vx × = 6m
g 61 (d) Comparing with the trajectory of projectile in which particle
57 (b) Consider the adjacent diagram. is projected from certain height horizontally (θ = 0 ° ).
gx 2
Y y = x tan θ − 2
2u cos 2 θ
Putting θ = 0 ° and g = a = 18u 2 = 2 ms −2
 1 −1 
v0 sin θ

v0 A
Q y = − 9x , u = ms 
2
 3 
θ u 2 sinθ
X
O u + v0cos θ B 62 (d) R = at angles θ and 90° − θ
g
Initial velocity in x-direction, u x = u + v 0 cos θ
u 2 sin2 θ u 2 sin2 (90 ° − θ ) u 2 cos 2 θ
Initial velocity in y-direction, u y = v 0 sinθ Now, h1 = and h2 = =
2g 2g 2g
where, angle of projection is θ.
2
Now, we can write  u 2 sin 2θ  1 R2
1 2 = 
hh  ⋅ =
uy v sin θ  g  16 16
tan θ = = 0
u x u + v 0 cos θ ∴ R = 4 hh
1 2
 v sin θ 
⇒ θ = tan− 1 0  63 (b) Since, u sinθ × t = 10 m
 u + v 0 cos θ 
⇒ 20 sin 45° t = 10
4 10 1
58 (d) u x = u cos θ 0 = 30 × = 24 ms −1 ⇒ t= = s
5 20 sin 45° 2
3
and u y = u sinθ 0 = 30 × = 18 ms −1 1
Now, y = (20 sin 45° ) t − gt 2
5 2
After 1 s, u x will remain same and u y will decrease by 10 ms −1 1 1 1 1
= 20 × × − × 10 × = 7.5 m
or it will become 8 ms −1. 2 2 2 2
vy 8 1 u2
∴ tanθ = = = 64 (d) Hmax = = 10 (Q θ = 90 ° )
vx 24 3 2g
u2
59 (a) Horizontal component of velocity, v x = 500 ms −1 and ⇒ u 2 = 200 ⇒ Rmax = = 20 m
g
vertical component of velocity while striking the ground,
Motion in a Plane and Projectile Motion 169

65 (a) The horizontal distance covered by bomb, u 2 sin 2θ


70 (b) Here, R = = 24 …(i)
2h 2 × 80 g
BC = v H × = 150 = 600 m
g 10
A vH

3m

6m 18 m
gx 2
B C Q y = x tan θ −
2u cos 2 θ 2

∴The distance of target from dropping point of bomb, 36g


3 = 6 tan θ − 2 …(ii)
2u cos 2 θ
AC = (AB )2 + (BC )2 = (80 )2 + (600 )2 = 605 . 3 m
From Eq. (i), we get
1 2 sin 2θ sin θ cos θ
66 (c) sy = u yt + a yt or − 70 = 25t − 5t 2 g
= =
2
u2 24 12
⇒ 5t 2 − 25t − 70 = 0 Substituting in Eq. (ii), we get
⇒ t 2 − 5t − 14 = 0 3 9  2
3 = 6 tan θ − tan θ = tanθ ⇒ θ = tan−1  
2 2  3
⇒ t = 7, t = − 2
∴ t =7s 71 (d) H ′ = H (as vertical component of acceleration has not
1 2 changed)
67 (a) From sy = u yt + a yt , we have
2 1 1 g 4u 2 sin2 θ
R′ = u x T + a x T 2 = R + × ×
− 40 = (20 sin 30 ° ) t − 5t 2 or t 2 − 2t − 8 = 0 2 2 4 g2
Solving this, we get t = 4 s u 2 sin2 θ
=R+ = (R + H )
2u sin θ 2 × 20 × sin 30 ° 2g
T= = =2s
g 10 72 (d) Standard equation of projectile motion,
t gx 2
⇒ = 2: 1 y = x tan θ − 2
T 2u cos 2 θ
dx
68 (b) Given, x = ct ⇒ vx = =c Comparing with given equation,
dt g
dy A = tanθ and B = 2
Also, y = bt 2 ⇒ vy = = 2bt 2u cos 2 θ
dt
A tan θ × 2u 2 cos 2 θ
∴ Speed = | v | = v x2 + v y2 = (c )2 + (2bt )2 So, = = 40 : 1
B g
= c 2 + 4b 2t 2 (As, θ = 45°, u = 20 ms −1, g = 10 ms −2 )
1 2 73 (a) u x = a, u y = b, g = c
69 (c) Using, h = gt , we get
2 2u xu y 2ab
∴ Horizontal range, R = =
v g c
A
74 (a) Given, v1 ⊥ v 2
4m
∴ v1 ⋅ v 2 = 0
B C or (u1 $i − gt $j ) ⋅ (− u 2 $i − gt $j ) = 0
5m
u1u 2
∴ g 2t 2 = u1u 2 or t =
1 2 g
hAB = gtAC
2 uy
2hAB 2×4 75 (d) tan 30° =
or tAC = = = 0.9 s ux
g 9.8 1 80
∴ = or u x = 80 3 ms −1
Further, BC = vtAC 3 ux
BC 5
or v= = = 5.55 ms −1 2u y 2 × 80
tAC 0.9 T= = = 16 s
g 10
170 OBJECTIVE Physics Vol. 1

T
At t = = 4 s ⇒ v x = u x = 80 3 ms −1 and v y = u y + a yt 5
u = 360 × = 100 ms −1
4 18
or v y = 80 + (− 10 ) (4) = 40 ms −1 The time taken by the bomb to hit the target is
2
∴ Velocity = (80 3) + (40 )2 = 144. 2 ms −1 2H 2 × 2000
T= = = 20 s
g 10
2
76 (c) (u cos α ) = (u cos α )2 + {(u sin α )2 − 2gh } …(i) The horizontal distance of the aeroplane from the target,
5
2H
H u 2 sin2 α R =u = 100 × 20 = 2000 m = 2 km
Here, h= = …(ii) g
2 4g
82 (c) Two balls will meet, if (50 cos 37° ) tA = 120 or tA = 3 s
Solving Eqs. (i) and (ii), we get
α = 60º Vertical component of A is 50 sin 37° or 30 ms −1, so they will
77 (b) The motion relative to elevator, meet, if thrown simultaneously.
a r = a b − a e = (− 10 ) − ( + 2) = − 12 ms −2 1
hA = hB = 30 × 3 − × 10 × (3)2 = 45 m
2u y 2 × u sin θ 2 × 4 × sin 30 ° 1 2
Now, T = = = = s
|a r | |a r | 12 3 83 (d) (a) Range becomes equal at complimentary angle. Hence,
u2 β = 90 ° − α or α + β = 90 ° = π / 2
78 (c) Rmax = = 1.6 m (At θ = 45° )
g u 2 sin2 α
(b) h1 =
or u = 16 = 4 ms −1 2g

2u y 2(4/ 2 ) 4 u 2 cos 2 α
Now, T= = = s ⇒ h2 = (As, β = 90 ° − α )
g 10 5 2 2g
10 2u 2 sin α cos α u 2 sin 2α
∴Total distance travelled in 10 s = 1.6 × = 20 2 m ∴ 4 hh
1 2 = = =R
4/ 5 2 g g
t1 (2u sin α / g )
79 (d) For 5 s, weight of the body is balanced by the given force. (c) = = tan α
Hence, it will move in a straight line as shown in the figure. t2 (2u cos α / g )
h1
(d) = tanα
h2

84 (a) Since, v ⊥ u or v ⋅ u = 0
5s or (u + gt ) ⋅ u = 0
u sin 2 θ
2 or u ⋅ u + (g ⋅ u ) t = 0
R= + (u cos θ ) (5)
g or u 2 + gut cos (90 ° + θ ) = 0
(50 )2 ⋅ sin 60 ° (Angle between u and g is 90° + θ)
= + (50 × cos 30 ° ) (5) = 250 3 m or u − g t sinθ = 0
10
u
R /2 3H ∴ t=
80 (c) From the given diagram, = = 3 g sin θ
H H
(v 02 sin θ cos θ )/ g 85 (c) Component (100 cos 53°) 60 ms −1 will remain unchanged.
or = 3 ⇒ 2 cot θ = 3
(v 02 sin2 θ )/ 2g Velocity will make 45° with horizontal when vertical
component also becomes ± 60 ms −1.
2  2
or tanθ = or θ = tan−1  80 ms−1
3  3
100 ms−1
81 (b) Given, H = 2000 m, u = 360 kmh −1

u
53°

60 ms−1
H
Using v = u + at (In vertical direction)
+ 60 = 80 + (− 10 ) t1
∴ t1 = 2 s ⇒ − 60 = 80 + (− 10 ) t2
Target
∴ t2 = 14 s
R
Motion in a Plane and Projectile Motion 171

86 (b) Area in which bullet will spread = πr 2 91 (d) Given, speed of packets = 125 ms −1
v2 Height of the hill = 500 m.
For maximum area, r = Rmax = (Whenθ = 45° )
g To cross the hill, the vertical component of the velocity
2 should be sufficient to cross such height.
v 2  πv 4
Maximum area = π Rmax
2
=π  = 2 u y ≥ 2gh
 g g
≥ 2 × 10 × 500 ≥ 100 ms −1
2u y gT
87 (d) As, T = ⇒ uy =
g 2 But u 2 = u x2 + u y2
2
1 2  2u y  1  2u y  ∴ Horizontal component of initial velocity,
⇒ hA = u ytA − gtA = u y   − g  
2  3g  2  3g 
u x = u 2 − u y2
2
4 u y  4   gT 
2
gT 2  gT 
= =    = Q u y = 
9 g  9g   2  9  2 = (125)2 − (100 )2

= 75 ms −1
2
 5 2u y  1  5 2u y 
⇒ hB = u y  ×  − ×g× × 
6 g  2 6 g  Time taken to reach the top of the hill,
2 2
5 uy 5  gT  5 2h 2 × 500
= =   = gT 2 t= = = 10 s
18 g 18g  2 72 g 10
gT 2 Time taken to reach the ground from the top of the hill,
∴ hA − hB =
24 t ' = t = 10 s
88 (c) The time taken by cart to cover 80 m, Horizontal distance travelled in 10 s,
s 80 8 x = u x × t = 75 × 10
= = s
v 30 3 = 750 m
The projectile must be fired (relative to cart) in vertically ∴ Distance through which canon has to be moved
upward direction. = 800 − 750 = 50 m
8/ 3 4 Speed with which canon can move = 2 ms −1
i.e. a = − g = − 10 ms −2, v′ = 0 and t = = s
2 3
50
4 40 ∴ Time taken by canon, t′ ′ =
∴ v ′ = u + at or 0 = u − 10 × or u = ms −1 2
3 3
⇒ = 25 s
89 (a) T /2 = 2 + 1 = 3 s or T = 6 s ∴ Total time taken by a packet to reach on the ground
2u y
⇒ =6 = t' ' + t + t'
g = 25 + 10 + 10 = 45 s
∴ u y = 30 ms −1

Further, tan 30 ° =
vy
=
u y − gt
=
30 − 20 (B) Medical entrance special format
vx ux ux questions
or u x = 10 3 ms −1
l Assertion and reason
or u = u x2 + u y2 = 20 3 ms −1
dv
uy 30 1 (d) = |a| = 9.8 ms −2 = constant and it is true that in case of
tanθ = = = 3 or θ = 60 ° dt
ux 10 3
projectile motion, the magnitude of velocity first decreases
90 (c) If the ball hits the nth step, the horizontal and vertical and then increases during the motion.
distances traversed are nb and nh, respectively. Let t be the 2 (a) At highest point, velocity is horizontal and acceleration is
time taken by the ball for these horizontal and vertical vertical, i.e. both are perpendicular to each other and hence,
displacement. Then, velocity along horizontal direction their dot product is zero.
remains constant = u, initial vertical velocity is zero.
u2 u 2 sin2 30 ° u 2
∴ nb = ut …(i) 3 (b) h1 = , h2 = =
2g 2g 8g
nh = 0 + (1/2) gt 2 …(ii)
⇒ h1 = 4h2
From Eqs. (i) and (ii), we get by eliminating t,
Also, at highest point, v = 0 in first case
2hu 2
nh = (1/ 2) g (nb /u )2 ⇒ n = and v ≠ 0 in second case.
gb 2
172 OBJECTIVE Physics Vol. 1

l Statment based questions (C) Medical entrances’ gallery


2h
1 (a) For both cases, t = = constant. 1 (b) The motion of the object shot in two cases can be depicted
g as below
Because, initial vertical downward component of velocity will
be zero for both the particles and both move under the effect
of g.
2 (a) Vertical component of velocity of ball at point P. u
uV = 0 + gt 60°
x1
= 10 × 0.4 = 4 ms −1 60° g cos 60°
g sin 60°
Horizontal component of velocity = initial velocity g
Case I
⇒ v H = 4 m/s

4 m/s u
30°
x2
P vH 30°
q g sin 30° g cos 30°
g
u
vV Case II
Using third equation of motion, v 2 = u 2 − 2gh … (i)
So, the speed with which it hits the ground,
As the object stops finally, so v = 0
v = v H2 + vV2 = 4 2 m/s
For inclined motion, g = g sinθ and h = x
vV 4
and tanθ = = = 1 ⇒ θ = 45° Substituting these values in Eq. (i), we get
vH 4 ⇒ u 2 = 2g sinθ x
It means the ball hits the ground at an angle of 45° to the
u2
horizontal. ⇒ x=
1 1 2g sinθ
Height of the table h = gt 2 = × 10 × (0.4)2 = 0.8 m
2 2 u2
For case I x1 =
Horizontal distance travelled by the ball from the edge of 2g sin 60 °
table h = ut = 4 × 0.4 = 1.6 m u2
For case II x2 =
3 (c) The angle between instantaneous velocity vector and 2g sin 30 °
acceleration vector before attaining the maximum height is
acute (0 to π/2) and after is obtuse (π/2 to π). At the highest x1 u2 2g sin 30 °
Q = ×
point, it is perpendicular (π/2). x 2 2g sin 60 ° u2
2h 1 2 1
4 (c) t1 = t2 = (where, h = height of tower) = × = or 1 : 3
g 2 3 3
v1 = 2gh 2 (a) Given, distance between the two buildings, d = 100 m
Height of each tower, h = 200 m
While v 2 = v12 + v 02 (where, v 0 = initial horizontal velocity)
Speed of each bullet, v = 25 ms −1
Therefore, both statements are correct.
The situation can be shown as below.
l Match the columns
x
2 (d) In horizontal projectile motion, 25 ms-1
200 m 200 m
Horizontal component of velocity,
u x = u = 10 ms −1
Vertical component of velocity, 100 m
−1 where, x be the vertical distance travelled from the top of the
u y = gt = 10 × 1 = 10 ms
building and t be the time at which they collide.
Horizontal displacement As two bullets are fired toward each other, so their relative
= u × t = 10 × (1) = 10 m velocity will be
1 1 vrel = 25 − (−25) = 50 ms −1
Vertical displacement = gt 2 = × 10 × (1)2 = 5 m
2 2
Motion in a Plane and Projectile Motion 173

d 100 When θ = 45°, maximum horizontal range,


Then, time, t= = = 2s
vrel 50 u2 u2
Rmax = sin 90 ° =
The distance or height at which they collide is calculated g g
from equation of motion, When θ = 135°, maximum horizontal range,
1
x = ut + at 2 u2 − u2
2 R= sin 270 ° =
g g
The bullet is initially at rest, i.e. u = 0 and as it is moving
under the effect of gravity a = − g, so Negative sign implies opposite direction.
1
x = − gt 2 7 (d) Given, x = a cos(pt ), y = b sin(pt )
2 x2 y2
1 ∴ + = 1, i.e. equation of ellipse
x = − × 10 (2 )2 = − 20 m a2 b 2
2
Now, r = x$i + y$j = a cos(pt ) $i + b sin(pt )$j
The negative sign shows that the bullets will collide 20 m
dr
below the top of tower, i.e. at a height of (200 − 20 ) = 180 m v= = − pa sin(pt ) $i + pb cos(pt )$j
from the ground after 2 s. dt
dv
3 (c) To obtain maximum range, angle of projection must be 45°, a= = − p 2a cos(pt ) $i − p 2b sin(pt )$j
i.e. θ = 45°. dt
π
u 2 sin(2 × 45° ) u 2 At t= ,
Rmax = = 2p
g g
v = − pa$i and a = − p 2b$j
u 2 sin2 45° u 2 Rmax
Q H max = = = Thus, velocity is perpendicular to acceleration.
2g 4g 4
Also, a = − p 2b , i.e. directed towards a fixed point as p and b
So, Hmax is 25 % of Rmax.
are positive constants.
4 (a) Given, u = 98 ms −1 and θ = 30 ° dr
π

Q Range of a projectile, As, v= ⇒ ∆r = ∫ 2p vdt


dt 0
π
u 2 sin (2θ ) 98 × 98 × sin 60 °
R= = ⇒ ∆r = [a cos (pt )$i + b sin (pt )$j ] 20p = − a$i + b$j
g 9.8
R = 490 3 m So, | r| = a 2 + b 2

5 (a) Let at any instant of time, the length AB be l, here angle θ 8 (b) Position vector of the particle is given by
and length l vary with time, then using Pythagoras theorem in r = cos ωt x$ + sin ωt y$
∆ABC, where, ω is a constant.
C x B Velocity of the particle is
v
dr d
y v= = (cos ωt x$ + sin ωt y$ )
l
dt dt
θ
m = (− sin ωt ) ωx$ + (cos ωt ) ωy$
A
= − ω (sin ωt x$ − cos ωt y$ )
Acceleration of the particle,
dv d
a= = (− ω sin ωt x$ + ω cos ωt y$ )
x 2 + y 2 = l2 dt dt
On differentiating both sides w.r.t t, we get = − ω 2 cos ωt x$ − ω 2 sin ωt y$
dx dy dl ⇒ a = − ω 2r = ω 2 (− r)
or 2x + 2y = 2l
dt dt dt
Assuming the particle is at P, then its position vector is
As, there is no vertical motion of the block, so directed as shown in the diagram.
dy dx dl
= 0, = v x and =v Y
dt dt dt
P
∴ 2xv x = 2lv
r
l v v
or v x = v or v x = =
x  x  sin θ
 
l
X
O
u 2 sin 2θ
6 (a) Horizontal range, R = Therefore, acceleration is directed towards − r, i.e. towards O
g
(origin).
174 OBJECTIVE Physics Vol. 1

v ⋅ r = − ω( sin ωt x$ − cos ωt y$ ) ⋅ ( cos ωt x$ + sin ωt y$ ) 12 (a) Given, a particle having horizontal


= − ω [sin ωt ⋅ cos ωt + 0 + 0 − sin ωt ⋅ cos ωt] range is twice the greatest height
= − ω (0 ) = 0 attained by it, i.e. R = 2H √5
⇒v ⊥r v 2 sin 2 θ 2v 2 sin2 θ 2
Thus, velocity is perpendicular to r. =
g 2g
θ
9 (a) The equation of trajectory, ⇒ 2 sin θ cos θ = sin2 θ ⇒ tan θ = 2 1
 x
y = x tan α 1 − (Gives)
⇒ sin θ =
2
⇒ cos θ =
1
 R 
5 5
 P v 2 sin 2θ v 2 × 2 sin θ cos θ
Q = P tan θ 1 − …(i) ∴ R= =
 R  g g
 Q 2 2 1
and P = Q tan θ 1 − … (ii) ⇒ R =v2 × × ×
 R  g 5 5
On dividing Eq. (i) by Eq. (ii), we get 4v 2
i.e. R=
Q 2 [1 − P / R] 5g
=
P 2 [1 − Q / R] 13 (d) Given, θ1 = θ 2 = 45°
1 3
[P − Q 3] = P 2 − Q 2 θ1 + θ 2 = (45° + α ) + (45° − α ) = 90 °
R
u1 = u 2 = u ⇒ R1 = R2 = ?
P 3 − Q 3 P 2 + PQ + Q 2
⇒ R= 2 =
P − Q2 P +Q u12 sin (θ1 + θ 2 )
R1 =
g
Q  P (P + Q ) 
Now, = tan θ 1 − 2 2 u 2 sin 90 ° u 2
P  P + PQ + Q  ⇒ R1 = = …(i)
g g
P 2 + PQ + Q 2 − P 2 − PQ 
= tan θ   u 22 sin (θ1 + θ 2 )
 P 2 + PQ + Q 2  Similarly, R2 =
g
P 2 + Q 2 + PQ
⇒ tan θ = u 2 sin 90 ° u 2
PQ ⇒ R2 = = …(ii)
g g
P 2 + PQ + Q 2 
⇒ θ = tan− 1   The ratio of horizontal ranges
 PQ 
R1 u2/g
= = 1: 1 ⇒ R1 : R2 = 1: 1
10 (a) Consider the diagram R2 u2/g
1 Net displacement
m s– 14 (d) Average velocity, v av =
10 sin 30° 10 Time taken
(Let, h) Tower
(13 − 2)$i + (14 − 3)$j
=
θ=30° 5
10 cos 30° 11i + 11j 11 $ $
$ $
50 m = = (i + j)
5 5
For horizontal motion, 50 = 10 cos 30 ° × t 1 2
5 5 10 15 (c) For vertical motion, h1 = u sin θt1 − gt1 (For h1)
⇒ t= = = 1 2
cos 30 ° 3 3 h1 + gt12
2 ⇒ t1 = 2 …(i)
For vertical motion, u sin θ
2
10 1  10 
h = 10 sin 30 ° × − × 10 ×   ⇒ h2 = u sin θt2 −
1 2
3 2  3 gt2 (For h2)
2
1 10 100 5 100 1 5 1 2
= 10 × × − × =  −  h2 + gt2
2 3 3 3 3 2 3 2
⇒ t2 = …(ii)
50  10  u sin θ
= 1 − 3  m
3 On dividing Eq. (i) by Eq. (ii), we get
 1 2
11 (b) Range of projectile is same for θ and 90° − θ. h + gt  /u sin θ
t1  1 2 1 
Given, θ = 40 ° =
t2  1 2
Another angle = 90 ° − θ = 90 ° − 40 ° = 50 ° h2+ gt2 /u sin θ
 2 
Motion in a Plane and Projectile Motion 175

g 2 2 2 u sinθ 2u sinθ
⇒ ht12 − h21
t = (tt12 − t1t2 ) 18 (b) As, t= ⇒ 4= …(i)
2 g g
The time of flight of the ball,
u sinθ
2 u sin θ 2
T= = (u sin θ ) u
g g
2  h1 + 1/ 2 gt12 
=   [From Eq. (i)]
g t1  θ
u cos θ
2  h t 2 ⇒ u sin θ = 2g
=  1 + 1
t1  g 2 u 2 sin2 θ 4g 2
and h= = [From Eq.(i)]
h 2 h  tt 2 − t 2t  2g 2g
= 1 × + t1 = 1 ×  12 1 2  + t1
t1 g t1  ht12 − h21
t ⇒ h = 20 m
htt 2
− ht112t2 + ht112t2 − h21
t3  ht122 t 2
− h21 19 (c) From the question’s figure, the x-component remain
= 112
=  unchanged, while the y-component is reverse. Then, the
t1 (ht12 − h21
t)  ht12 − h21
t
velocity at point B is (2$i − 3$j) ms −1.
16 (c) As we know that, maximum height of a projectile is 2u sinθ
given by 20 (b) Time of flight, T =
u 2 sin2 θ g
Hmax = Given, u = u 0 , T = 1s, θ = 30 °, g = 9.8 ms −2
2g
where, u = initial velocity of projectile u 0 = g = 9.8 ms −1
g = acceleration due to gravity 21 (a) As the velocity makes an angle of 30° with horizontal, so
and θ = angle of projection. the horizontal component of velocity at the instant will be
As per question, v cos 30°.
v 2 sin2 45° ⇒ v cos 30 ° = 5
Hmax = …(i) (As, u = v, θ = 45°) 5 5 10
2g ⇒ v= = = ms −1
cos 30 ° 3/2 3
Now, range of a projectile is given by
u 2 sin 2 θ 22 (a) As the horizontal ranges are the same.
R=
g v 02 sin 2θ1 v 02 sin 2θ 2
∴ =
v 2 sin (2 × 45° ) g g
⇒ R=
g So, sin 2θ1 = sin 2θ 2
v 2 sin 90 ° or 2θ1 = π − 2θ 2
⇒ R= …(ii)
g ⇒ θ1 + θ 2 = π / 2
On dividing Eq. (i) by Eq. (ii), we get v 02 sin2 θ1
∴ (h1)max =
Hmax v 2 sin2 45° × g 1 2g
= =
R 2g × v 2 sin 90 ° 4 × 1 v 02 sin2 θ 2
and (h2 )max =
⇒ R = 4Hmax = 4H 2g
17 (d) The horizontal component of velocity, (h1)max sin2 θ1 sin2 θ1
v x = u = 10 ms −1 ∴ = = = tan2 θ1
(h2 )max sin2 θ 2 cos 2 θ1
u =10 ms−1  2 π  
Q sin θ 2 = sin  2 − θ 2 = cos θ / 2
2 2

 
23 (a) Given, r = 2t $i + t 2$j
dr d n
vx Velocity vector, v = and using x = nx n −1
45° dt dx
dr
vy We have, = 2$i + 2t $j
dt
vy vy
Now, tan θ = tan 45° = = 24 (b) Given, R = H
vx 10
u 2 (2 sin θ cos θ )
⇒ v y = 10 ms −1 Range, R =
g
176 OBJECTIVE Physics Vol. 1

u 2 sin2 θ u 2 sin2 θ
Height, H = So, h=
2g 2g
u 2 (2 sin θ cos θ ) u 2 sin2 θ u2 u2
Hence, = ⇒ 20 = or = 200
g 2g 10 g g
sin θ u 2 × 2 sin θ cos θ 200 × 2 × 2
2 cos θ = ∴ The range, R = =
2 g 5× 5
⇒ tanθ = 4 ⇒ θ = tan−1(4) 200 × 4
= = 160 m
25 (d) Maximum height and time of flight depends upon the 5
vertical component of initial velocity. 27 (b) Since, the ball reaches from one player to another in 2 s, so
H1 > H 2 the time period of the flight, T = 2 s
2 u sin θ
⇒ u y1 > u y 2 ⇒ =2s
g
Range R2 > R1
Here, u is the initial velocity and θ is the angle of projection.
So, u 2 > u1 ⇒ u sin θ = g …(i)
26 (d) According to given condition, Now, we know that the maximum height of the projection,
h1 sin2 θ 20 u 2 sin2 θ (u sin θ )2
= = H= or H =
h2 sin (90 ° − θ ) 80
2 2g 2g
1 1 On putting the value of u sin θ from Eq. (i), we get
⇒ tan2 θ = ⇒ tanθ =
4 2 g2 g g 10
H= = or H = = m or H = 5 m
1 2 2g 2 2 2
sinθ = and cos θ =
5 5
CHAPTER
05

Laws of Motion
We normally observe around us, a number of objects or bodies at rest or in motion
and find that, the objects at rest do not move by themselves or the objects in
motion do not come to rest by themselves but they require some external force to
do so. e.g. To move a book kept on a table, we need to push or pull it, or to stop a
vehicle in motion, breaks are required.
The factor which is necessary for causing motion or change in motion is termed as
force. This cause of motion (force) and effects of motion are governed by Newton’s
laws of motion.
Inside
In this chapter, we will discuss the motion of a body by taking into consideration 1 Force
the cause of motion, i.e. the external force which produces the motion or change Inertia
the motion. Momentum
2 Newton’s laws of motion
Newton’s Ist law of motion
FORCE Newton’s IInd law of motion
Resultant force
Force is an effort in the form of push or pull causing or tending to cause motion, Impulse
change in motion or deformations in a body. Newton’s IIIrd law of motion
There are basically two types of forces which are commonly observed 3 Law of conservation of
linear momentum
(i) Distant forces (Field forces) The forces acting between two or more
4 Forces in equilibrium
objects, which do not require the physical contact between the objects are
Newton’s first law for forces
called distant forces or field forces. Gravitational force between two bodies, in equilibrium
electrostatic force between two charges, weak forces and nuclear forces are 5 Common forces in mechanics
examples of distant forces. Free body diagram
Weight (w = mg ) of a body also comes in this category. 6 Apparent weight of a man in a lift
7 Applications of Newton’s laws
(ii) Contact forces The forces acting between two or more objects, which of motion
require the physical contact between the objects are called contact forces. Motion of bodies
Friction force, normal reaction, tension, spring force, etc., are some connected through strings
or springs
examples of contact forces.
Bodies attached through pulley
(using strings or springs)
Inertia 8 Force of friction
The term inertia means resistance or opposition to the change of state. It is Types of friction
9 Equation of motion on a rough
defined as the inherent property of a body by virtue of which it remains in its inclined plane
state of rest or of uniform motion in a straight line. This term was first used by
Galileo.
178 OBJECTIVE Physics Vol. 1

There are three types of inertia


(i) Inertia of rest It is defined as the tendency of a body
Newton’s IInd law of motion
to remain in its position of rest. i.e. A body at rest This law states that, “the rate of change of momentum of a
remains at rest and cannot start moving on its own. body is directly proportional to the external force applied
on the body and the change takes place in the direction of
(ii) Inertia of motion It is defined as the tendency of a the applied force.”
body to remain in its state of uniform motion along a
straight line. i.e. A body in uniform motion can We may also state Newton’s second law of motion as
neither gets accelerated nor get retarded on its own, “If the unbalanced external force (net force) acts on a
also it cannot stop on its own. body, the body accelerates. The direction of acceleration is
(iii) Inertia of direction It is defined as inability of a the same as the direction of the net force.”
body to change by itself its direction of motion.
Calculating force with the help
Relation between mass and inertia of Newton’s IInd law
Mass of a body is the measurement of its inertia. Let F be external force applied on the body in the
A body with greater mass shows greater inertia, i.e. it direction of motion of the body for time interval ∆t, then
is more difficult to change its state of rest or uniform the velocity of a body of mass m changes from v to v + ∆ v,
motion as compared to that of a body having smaller mass. i.e. change in momentum, ∆ p = m ∆v.
Quantitatively, the inertia of an object is measured by its According to Newton’s second law,
mass. Thus, the SI unit for mass as well as inertia is ∆p ∆p
kilogram (kg), whereas the units in the CGS system and in F∝ or F = k
∆t ∆t
the British Imperial system for mass or inertia are gram (g)
and slug (sl), respectively. where, k is a constant of proportionality.
∆p
If limit ∆t → 0, then the term becomes the
Momentum dp dp ∆t
derivative . Thus, F = k
Momentum of a body is the quantity of motion possessed dt dt
by a moving body. It is measured as the product of mass For a body of fixed mass m, we have
and velocity of a body. It is represented by p. d (m v ) dv
F=k = km ⇒ F = km a
i.e. Momentum (p ) = Mass (m ) × Velocity (v ) dt dt
Now, a unit force may be defined as the force which
SI unit of momentum is kg -ms −1 and CGS unit of produces unit acceleration in a body of unit mass.
momentum is g -cms −1 . So, F = 1, m = 1, a = 1 ⇒ k = 1
The dimensional formula of momentum is [MLT −1].
So, Force, F = m a
It is a vector quantity.
In scalar form, this equation can be written as, F = ma.
NEWTON’S LAWS OF If v is fixed and m is variable, then F = kd
(mv ) kvdm
=
dt dt
MOTION Q k = 1, then F = vdm /dt
Newton’s laws of motion are the three physical laws that, The force is a vector quantity.
together laid the foundation for classical mechanics. These
three laws of motion were first proposed by Sir Isaac Newton. Note The slope of momentum-time graph is equal to force on the
particle, e.g.
p
Newton’s Ist law of motion
This law states that, “every body continues in its state of A
rest or of uniform motion in a straight line unless it is
θ
compelled by some external force to change its state.” O t
Thus, it can be concluded that if the net external force on Fig. 5.1 Momentum-time graph
a body is zero, its acceleration is zero. Acceleration can be At point A, F =
dp
= slope = tanθ
non-zero only if there is a net external force on the body. dt
Newton’s first law defines force qualitatively. This graph depicts the motion of a body on which increasing force
is acting.
Laws of Motion 179

Units of force ⇒ a = − 6750 ms −2


● In SI system, absolute unit of force is newton. From second law of motion,
● One newton is defined as that much force which Retarding force, F = ma = 0.06 × 6750
produces an acceleration of 1 ms −2 in a body of mass 1 kg. ⇒ F = 405 N

1 N = 1 kg × 1 ms −2 Example 5.3 A stone of mass 1 kg is thrown with a velocity


−2 of 20 ms −1 across the frozen surface of a lake and it comes
1 N = 1 kg ms to rest after travelling a distance of 50 m. What is the
● In CGS system, absolute unit of force is dyne. magnitude of the force opposing the motion of the stone?
One dyne is that much of force which produces an Sol. Given, u = 20 ms −1, v = 0, s = 50 m and m = 1 kg
acceleration of 1 cms −2 in a body of mass 1 g. To calculate force, we have the formula F = ma , but we have to
1 dyne = 1 g × 1 cms −2 first calculate acceleration a.
1 dyne = 1 g cms −2 Using the third equation of motion, i. e .
● In MKS system, gravitational unit of force is kilogram v 2 = u 2 + 2as
weight (kg-wt). One kg-wt is that much of force which (0)2 = (20)2 + 2 × a × 50
produces an acceleration of 9.80 ms −2 in a body of mass 1 ⇒ 100a = − 400
kg. It is also known as kilogram-force (kgf). ⇒ a = − 4 ms −2
1 kg-wt = 1kgf = 9.8 N
Acceleration a = − 4 ms −2 (− ve sign shows that speed of the
● In CGS system, gravitational unit of force is gram weight
stone decreases, i.e. retardation)
(g-wt) or gram force (gf). It is defined as that force which
Now, F = ma = (1 kg ) × (− 4 ms −2 )
produces an acceleration of 980 cms −2 in a body of mass 1 g.
= − 4 kg - ms −2 = − 4 N
1 g -wt = 1 gf = 980 g cm s −2 or 1 gf = 980 dyne
Thus, force of opposition between the stone and the ice is − 4 N.
Relation between newton and dyne The negative value of force shows that the opposing force acts
1 N = 1 kg × 1 ms −2 = 1000 g × 100 cms −2 in a direction opposite to the direction of motion.
= 10 5 gcms −2 (1 dyne = 1 g cms −2 ) Example 5.4 A block of 5 kg is resting on a frictionless
1 N = 10 5 dyne plane. It is struck by a jet releasing water at a rate of
3 kgs −1 at a speed of 4 ms −1. Calculate the acceleration of
Example 5.1 If an electron is subjected to a force of 10 −25 N the block.
in an X-ray machine, then find out the time taken by the dm
electron to cover a distance of 0.2 m. Sol. Force exerting on block, F = v = 4 × 3 = 12 N
dt
(Take, mass of an electron = 10 −30 kg)
F 10−25 5 kg
Sol. The acceleration of the electron, a = = = 105 ms −2
m 10−30
F 12
The time taken by the electron (t ) to cover the distance (s ) of So, acceleration of the block, a = = = 2.4 ms−2
0.2m can be given by m 5
1
s = ut + at 2
2 Resolution of force into different

1
0.2 = 0 + × 105 × t 2 components
2
If the force applied, imparts acceleration a to a body, then
⇒ t 2 = 0.4 × 10−5 = 4 × 10−6 ⇒ t = 2 × 10−3 s its components in X, Y and Z-axis are a = a x $i + a y $j + a z k.
$
Example 5.2 A bullet of mass 0.06 kg moving with a speed Components of force will be Fx , Fy and Fz
of 90 ms −1 enters a heavy wooden block and is stopped after
a distance of 60 cm. What is the average resistive force As, F = ma
exerted by the block on the bullet? ⇒ Fx $i + Fy $j + Fz k
$ = m (a $i + a $j + a k
x y
$
z )
Sol. The retardation a of the bullet is given by dv x d 2x dp x
u 2 −90 × 90 Thus, Fx = ma x = m =m =
a=− = (From, v 2 = u 2 + 2as, v = 0) dt dt 2 dt
2s 2 × 0.6
dv y d 2y dp y
(Q Given, s = 60 cm = 0.6 m, u = 90 ms −1 ) Fy = ma y =m =m =
dt dt 2 dt
180 OBJECTIVE Physics Vol. 1

dv z d 2 z dp Resultant force
Fz = ma z = m =m 2 = z
dt dt dt When two or more forces act on a body simultaneously,
The component form of Newton’s second law tells that if then the single force which produces the same effect as
the applied force makes some angle with the velocity of the produced by all the forces acting together is known as the
body, it changes the component of velocity along the resultant force.
direction of force. F1

The component of velocity normal to the force remains


unchanged. F4 F3

Example 5.5 A force of 50 N acts in the direction as shown


in figure. The block of mass 5 kg, resting on a smooth F2
horizontal surface. Find out the acceleration of the block. Fig. 5.2
50 N
Resultant force, F = F1 + F2 + F3 + F4
60°
a
Balanced force
5 kg
When a number of forces acting simultaneously on a body
do not bring about any change in its state of rest or of
Sol. Free body diagram
uniform motion along a straight line, then the forces acting
50 N on the body are said to be balanced forces.
a In other words, when different forces acting on a body
60° give a zero resultant, then the forces are said to be
balanced. Balanced forces do not produce any acceleration.
Fy = 50 cos 60°
Fx = 50 sin 60° e.g. When two opposite forces having the same magnitude
F act on a block placed on a smooth horizontal table, they
where, F x = horizontal component of the force fail to move the block.
and F y = vertical component of the force.
50 3
Horizontal component of the force = 50 sin 60° = N F F
2
Acceleration of the block, a
Component of force in the direction of acceleration Fig. 5.3
=
Mass This is because the net force is equal to zero. Similarly,
50 3 1 −2
= × = 5 3 ms two opposite forces having the same magnitude cannot
2 5 change the speed of a moving body.
Example 5.6 A force F = (6$i − 8$j + 10k
$ ) N produces
acceleration of 2 ms −2 in a body. Calculate the mass of Unbalanced force
the body. When a number of forces acting simultaneously on a body
Sol. Q Acceleration, a =
|F| bring about a change in its state of rest or of uniform
m motion along a straight line, then these forces acting on
|F| 62 + 82 + 102 the body are said to be unbalanced forces.
⇒ Mass, m = = = 10 kg
a 2 In this case, different forces acting on a body do not give
zero resultant. If an unbalanced force is applied on the
Example 5.7 A force F = (2t $i + 3t 2 $j) N acts on an object object, there will be a change either in its speed/velocity
moving in XY-plane. Find magnitude of change in or in the direction of its motion.
momentum of the object in time interval t = 0 to t = 2s.
dp Thus, to accelerate an object, an unbalanced force is required.
Sol. Given, F = (2t $i + 3t 2$j) N ⇒ = 2t$i + 3t 2$j When net force on the body is not equal to zero, then the
dt
body at rest starts moving in the direction of resultant force.
⇒ dp = 2tdt$i + 3t 2dt$j
2 2
⇒ ∫ dp = 2 ∫0 tdt$i + 3 ∫ t 2dt$j
0
F1 F2
2 2
⇒ ∆p = +
[t 2]0 $i [t 3]0 $j
Fig. 5.4
⇒ ∆p = 4$i + 8$j If F2 > F1, then F2 − F1 > 0 and as a result, the car
−1 accelerates in the direction of F2 .
⇒ | ∆p | = 16 + 64 = 80 ≈ 9 kg ms
Laws of Motion 181

Example 5.8 Let us consider two forces F1 and F2 acting or I = Fav ⋅ t = p 2 − p1 = ∆p


on a body of mass 2 kg as shown in the figure. F1 = 10 N, Thus, impulse is also equal to total change in momentum.
F2 = 2 N, what will be the acceleration? This is known as impulse-momentum theorem.

F1 F2
Calculation of impulse: graphical method
(i) When applied force is constant, then the graph
between this force and the time of application of this
Sol. Unbalanced external force, F = F1 − F 2 = 10 − 2 = 8 N
force is a straight line parallel to time axis.
So, F = ma
Y
F 8
⇒ Acceleration, a= = = 4 ms −2
m 2

F (newton)
A B
a
F
F

The body moves in direction of F1. C


O X
∆t
Impulse Time (second)

When a large force acts on a body for very small time, Fig. 5.5 Impulse of a constant force
then product of the average of total force for that small
Here, impulse is given by the area covered by the
time period and the time period itself is called impulse.
graph.
Impulse = Average force × Time i.e. I = F × ∆t
t2 where, F = OA, ∆t = OC
I = F∆t or I = ∫t
1
F dt
∴ I = OA × OC = Area of rectangle OABC .
It is a vector quantity and its direction is same as that of (ii) When applied force is variable for the time of
force. Dimensional formula of impulse is same as that of application (∆t ), then graph between force and time
momentum, i.e. [MLT −1]. will be a curve as given in the figure below
SI unit of impulse is N-s or kg ms −1 and CGS unit of Y
it is g-cms −1. dt
A B
Relation between momentum and impulse
Force (newton)

Suppose F is the value of force during impact at any time


and p is the momentum of the body at that time, then
according to Newton’s IInd law of motion,
t
dp X
t1 t2
F= or F ⋅ dt = dp …(i) D C
Time (second)
dt
Fig. 5.6 Impulse of a variable force
Suppose that the impact lasts for a small time t and during
this time, the momentum of the body changes from p 1 to Here, impulse = force × time
p 2 . Then, integrating the above equation, we get
= F × dt
t p2 t
= Area of shaded region
∫0 F dt = ∫p ∫0 F dt = p 2 − p1
p
dp = | p |p 2 ⇒
1 1
123 Total impulse for the force applied during period
Impulse from t1 to t 2
t2
From this equation, we found that impulse is equal to
change in momentum of the body.
= ∫t
1
F ⋅ dt

Also, if Fav is the average force (constant) during the = Area under F-t curve from t1 to t 2
impact, then
t t Total impulse for the force applied
Impulse, I = ∫0 Fav dt = Fav ∫0 dt = p 2 − p1 = Area covered between the curve and time-axis
182 OBJECTIVE Physics Vol. 1

Example 5.9 A baseball player hits back the ball straight in


the direction of the bowler without changing its initial speed
Newton’s IIIrd law of motion
of 12 ms −1. If the mass of the ball is 0.15 kg, then find the This law states that, “to every action, there is always an
impulse imparted to the ball. (Consider the ball in linear motion) equal and opposite reaction or the mutual actions of two
bodies upon each other are always directed to contrary parts.”
. kg, v = 12 ms −1 and u = − 12 ms −1
Sol. Given, m = 015
Change in momentum,
p 2 − p1 = m (v − u ) = 015
. [12 − (−12)] = 015
. × 24
Analysis of forces
p 2 − p1 = 3.60 kg - ms −1
From Newton’s IIIrd law, it can be analysed that we cannot
produce a single isolated force in nature. Thus, forces occur
Impulse, I = p 2 − p1 ⇒ I = 3.6 N-s in equal and opposite pairs. Whenever object A exerts a force
Example 5.10 A hammer of mass 1 kg moving with a speed on object B, object B must also exert a force on object A. The
of 6 ms −1 strikes a wall and comes to rest in 0.1 s. Calculate two forces are equal in magnitude and opposite in direction.
(i) impulse of the force,
(ii) average retarding force that stops the hammer A B
(iii) and average retardation of the hammer. FAB FBA

Sol. (i) Impulse = F × t = m (v − u) = 1(0 − 6) = − 6 N-s Fig. 5.7 Forces acting on bodies A and B

(ii) Average retarding force that stops the hammer, As shown in figure, if FBA is the force exerted by body A
Impulse 6 on B and FAB is the force exerted by B on A, then
F = = = 60 N
Time 0.1 according to Newton’s third law,
(iii) Average retardation, a =
F
=
60
= 60 ms−2 FAB = − FBA
m 1
Force on A by B = − Force on B by A
Example 5.11 A cricket ball of mass 150 g is moving with a
velocity of 12 ms −1 and is hit by a bat, so that the ball is Important features of Newton’s IIIrd law of motion
turned back with a velocity of 20 ms −1. If the duration of (i) Newton’s third law of motion is applicable
contact between the ball and bat is 0.01 s, find the impulse irrespective of the nature of the forces The
and the average force exerted on the ball by the bat.
forces of action and reaction may be mechanical,
Sol. According to given question, change in momentum of the ball, gravitational, electric or of any other nature.
∆p = p f − pi = m (v − u) = 150 × 10−3 [20 − (−12)] = 4.8 N-s (ii) Action and reaction always act on two different
So, by impulse-momentum theorem, impulse, I = ∆p = 4.8 N-s bodies If they act on the same body, the resultant
and by time averaged definition of force in case of impulse force would be zero and there could never be
I ∆p 4.80 accelerated motion.
⇒ F av = = = = 480 N
∆t ∆t 0.01 (iii) The force of action and reaction cannot cancel
Example 5.12 Figure shows an estimated force-time graph each other This is because action and reaction,
for a baseball struck by a bat. though equal and opposite forces always act on
P B different bodies and so cannot cancel each other.
18000
(iv) No action can occur in the absence of a reaction
In a tug of war, one team can pull the rope only if
Force (in N)

12000
the other team is pulling the other end of the ropel;
6000 no force can be exerted, if the other end is free. One
team exerts the force of action and the other team
A C
0
O 1 1.5 2 2.5 3
provides the force of reaction.
Time (in s)
Example 5.13 A block of mass 25 kg is raised by a 50 kg man
From this curve, determine in two different ways as shown in figure. What is the action on
(i) impulse delivered to the ball the floor by the man in the two cases? If the floor yields to a
(ii) and average force exerted on the ball. normal force of 700 N, which mode should the man adopt to
Sol. (i) Impulse = Area under F-t curve lift the block without the floor yielding? (Take, g = 9.8 ms −2 )
1 1
= Area of ∆ABC = × OP × AC = × 18000 × (2.5 − 1) 50 kg
2 2
= 1.35 × 104 kg - ms −1 or N-s
50 kg

Impulse 1.35 × 104


(ii) Average force = = = 9000 N 25 kg 25 kg
Time (2.5 − 1)
(a) (b)
Laws of Motion 183

Sol. In mode (a), the man applies a force equal to 25 kg weight Conservation of linear momentum for
in upward direction. According to Newton’s third law of
motion, there will be a downward force of reaction on the floor. the collision of two bodies
∴ Total action on the floor by the man (i) Head-on collision (collision in a straight line) Two
= 50 kg-wt + 25 kg -wt bodies of masses m1 and m 2 collide on frictionless
surface moving in the same direction with respective
= 75 kg -wt
velocities u1 and u 2 . After collision, both the bodies
= 75 × 9.8 N = 735 N separate with a variation in their velocities, i.e. v 1
In mode (b), the man applies a downward force on rope equal and v 2, respectively.
to 25 kg-wt. According to Newton’s third law, the reaction
will be in the upward direction by the rope on the man, so he Initial momentum (before collision),
becomes light by 25 kg-wt. p 1(initial) = m1u 1, p 2 (initial) = m 2 u 2
∴ Total action on the floor by the man Final momentum (after collision),
= 50 kg -wt – 25 kg-wt p 1(final) = m1v 1, p 2 (final) = m 2 v 2
= 25 kg-wt
u2
= 25 × 9.8 N = 245 N u1
As the floor yields to a downward force of 700 N, so the man
should adopt mode (b). m1 m2
(a) Before collision

LAW OF CONSERVATION F12 F21 v1


v2

OF LINEAR MOMENTUM m1 m2
(b) At the time of collision
The total momentum of an isolated system (a system (c) After collision
having no external force acting on it) of constant mass Fig. 5.8
remains constant or conserved and does not change with time.
During collision, particle 1 exerts a force F21 on
If the momentum of two particles system of masses m1 and
particle 2 and simultaneously particle 2 exerts a force
m 2 are p 1 and p 2 respectively, then the net momentum of
F12 on particle 1.
whole system is given by
F12 = rate of change of momentum of particle 1
p = p 1 + p 2 = constant m v − m1u1 m1 (v 1 − u1 )
= 1 1 =
This principle is a consequence of Newton’s second and t t
third law of motion. Similarly, F21 = rate of change of momentum of
particle 2
Conservation of linear momentum for a m v − m 2u 2 m 2 (v 2 − u 2 )
system of two or more particles = 2 2 =
t t
Force applied on particle 1 by particle 2 is F12 and force
According to Newton’s IIIrd law of motion,
applied on particle 2 by particle 1 is F21 and their
respective momentum are p 1 and p 2 . F12 = − F21
From Newton’s IInd law, m1 (v 1 − u1 ) m (v − u 2 )
⇒ =− 2 2
dp dp t t
F12 = 1 and F21 = 2
dt dt or m1u1 + m 2u 2 = m1v 1 + m 2v 2
From Newton’s IIIrd law,
dp1 dp i.e. Total momentum before collision remains
F12 = − F21 ⇒ =− 2 same as total momentum after collision.
dt dt
dp1 dp 2 d (ii) Oblique collision (collision of ball with wall) A ball
or + =0 ⇒ (p1 + p 2 ) = 0 of mass m strikes a wall with velocity u at an angle θ
dt dt dt
from the normal of wall and rebounds with the same
Thus, p1 + p 2 = constant speed in time t.
For n number of particles, Here, initial momentum of the particle,
p1 + p 2 + p 3 + K + pn = constant pi = mu cos θ$i − mu sin θ$j
184 OBJECTIVE Physics Vol. 1

Final momentum of the particle, Example 5.14 A bullet of mass 10 g is fired from a gun of
p f = − mu cos θ$i − mu sin θ$j mass 1 kg. If the recoil velocity is 5 ms −1. Find the velocity
of the muzzle.
pi = mui Wall
Sol. From the law of conservation of momentum,
– mu sin θ ^j m G vG = m BvB
mu cos θ ^i where, m G , vG = mass and velocity of gun
Normal θ
θ m B, vB = mass and velocity of bullet
– mu cos θ ^i
– mu sin θ ^j m G vG 1× 5
⇒ vB = = = 500 ms −1
mB 10 × 10−3
pf = muf
Fig. 5.9 Collision of ball with wall Example 5.15 On a mine site a rock is exploded. On
Now, change in momentum, ∆p = p f − pi explosion, rock breaks into three parts. Two parts go off at
right angles to each other. Of these two, 1 kg first part is
= − mu cos θ$i − mu sin θ$j − mu cos θ$i + mu sin θ$j moving with a velocity of 12 ms −1 and 2 kg second part is
moving with a velocity of 8 ms −1. If the third part flies off
= − 2mu cos θ $i with a velocity 4 ms −1, what will be its mass?
It means that momentum changes only along the Sol. v1 = 12 ms–1
normal to the wall but not along the wall. This is so v3 = 4 ms–1
because force is acting on the ball only normal to the Explosion
⇒ m1 = 1 kg
wall (this force is reaction force of wall) and no force
acts parallel to the wall. m3 = ?
v2 = 8 ms–1
| ∆p | = 2 mu cos θ m2 = 2 kg

∆p −2 mu cos θ $ From the law of conservation of momentum,


Force, F== i p 3 = p12 + p 22
∆t t
2 mu cos θ ⇒ m 3 × 4 = (1 × 12)2 + (2 × 8)2 = 20
|F |=
t ⇒ m 3 = 5 kg
There are two possible cases Example 5.16 Two objects each of mass 5 kg are moving in
Case I If θ = 0 ° , means ball is thrown perpendicular to the the same straight line but in the opposite directions towards
wall, then ∆p = − 2mu cos 0 ° $i each other with same speed of 3 m/s. They stick to each other
after collision. What will be the velocity of the combined
⇒ ∆p = − 2 mu$i object after collision?
∆p −2mu $ Sol. Given, m1 = m 2 = 5 kg,
∴ F= = i
∆t t u1 = 3 m/s, u2 = −3 m/s
Before collision,
Case II If θ is the angle measured from the wall, then m1 m2
∆p = − 2mu sin θ$i ⇒ | ∆p | = 2mu sin θ u1 u2

−2mu sin θ $ 2mu sin θ Let the velocity of the combined object be v.
F= i ⇒ |F|=
t t Then, after collision,
m1 m2 v
Newton’s IIIrd law can be derived from principle
of conservation of linear momentum Total momentum of the system before collision is
If two particles of masses m1 and m 2 are moving under action of their m1u1 + m 2u2 = 5 × 3 + 5 × (−3) = 0
mutually interacting forces with each other, such that no external force Total momentum of the system after collision is
acts on the system. Then, momentum of system remains constant. m1v + m 2v = (m1 + m 2 )v = (5 + 5) v = 10 v
∆ p1 ∆p 2 According to the law of conservation of momentum,
i.e. ∆ p1 + ∆ p 2 = 0 ⇒ ∆ p1 = − ∆ p 2 ⇒ =−
∆t ∆t Momentum before collision = Momentum after collision
⇒ F12 = − F21
∴ 0 = 10 v ⇒ v = 0
Force on 1st due to 2nd = − Force on 2nd due to 1st Hence, the velocity of the combined object after collision is zero.
Laws of Motion 185

Example 5.17 A ball of mass m strikes a rigid Sol. According to the question,
wall with speed v and gets reflected without (i) pi = mv sin 30° $i − mv cos 30° $j, p f = − mv sin 30° $i − mv cos 30° $j
any loss of speed, as shown in the figure.
∴ Impulse = ∆p = p f − pi = − 2mv sin 30° $i = − mv$i
Magnitude of impulse = | ∆p | = mv
30° mv sin 30° (–^i )
pi

30° 30°
30°
mv cos 30° (–^j ) pf
mv sin 30° ( ^i ) mv cos 30° (–^j )
(i) What is the magnitude of the impulse
imparted to the ball by the wall? (ii) Negative sign of the impulse shows that it is along negative x-direction.
(ii) What is the direction of the force on the wall Since, impulse and force are in the same direction, the force on the ball is
due to the ball? along the negative direction of X-axis. Hence, the force on the wall will
be along positive X-axis.

CHECK POINT 5.1


1. Inertia of an object is directly dependent on 7. A constant force acting on a body of mass 3 kg change its
(a) impulse (b) momentum (c) mass speed from 2 ms −1 to 3.5 ms −1 in 25 s, in the direction of
(d) density the motion of the body. What is the magnitude and
2. A body of mass 6 kg is acted upon by a force, so that its direction of the force?
velocity changes from 3 ms −1 to 5 ms −1 , then change in (a) 0.18 N in the direction of motion
momentum is (b) 0.32 N in the direction of motion
(a) 48 N-s (b) 24 N-s (c) 0.64 N in the direction of motion
(c) 30 N-s (d) 12 N-s (d) 0.16 N in the direction of motion

3. The momentum p (in kg-ms −1 ) of a particle is varying with 8. A body of mass 5 kg is acted upon by two perpendicular force
time t (in second) as p = 2 + 3 t . The force acting on the
2 8 N and 6 N, find the magnitude and direction of the
acceleration.
particle at t = 3 s will be
(a) 3 ms −2, θ = cos−1 (0.8) from 8 N
(a) 18 N (b) 54 N
(c) 9 N (d) 15 N (b) 2 ms −2, θ = cos−1 (0.6) from 6 N
4. A body is acted upon by balanced forces, (c) 3 ms −2, θ = cos−1 (0.9) from 6 N
(a) if it is in rest only (d) 5 ms −2, θ = cos−1 (0.81) from 8 N
(b) if it is moving with constant speed
(c) if even number of forces are acting on it 9. A ball of mass m is moving towards a player with velocity v. If
(d) if it is not accelerating player stopped it, then impulse applied by the player is
(a) − mv (b) + mv (c) − 2mv (d) + 2mv
5. A force of 72 dyne is inclined to the horizontal at an angle of
60°, find the acceleration of a mass of 9 g which moves in the 10. A constant retarding force of 50 N is applied to a body of
effect of this force in a horizontal direction. mass 20 kg moving initially with a speed of 15 ms −1 . How
−1 −1 much time does the body take to stop?
(a) 5 cms (b) 4 cms
(c) 2 ms −1 (d) 3 ms −1 (a) 6 s (b) 8 s (c) 9 s (d) 10 s

6. A man of mass 60 kg is standing on a horizontal conveyor 11. A ball of mass 1 kg is dropped from height 9.8 m, strikes
belt. When the belt is given an acceleration of 1 ms −2, the with ground and rebounds at height of 4.9 m, if the time of
man remains stationary with respect to the moving belt. If contact between ball and ground is 0.1 s, then find impulse
g = 10 ms −2, the net force acting on the man is and average force acting on ball.
(a) 23.52 N-s, 235.2 N (b) 235.2 N-s, 23.53 N
a = 1 ms –2 (c) 42.5 N-s, 525 N (d) 52.5 N-s, 525 N
12. A body of mass 5 kg is moving with velocity of
v = (2$i + 6$j) ms −1 at t = 0 s. After time t = 2 s, velocity of body
is (10$i + 6$j), then change in momentum of body is
(a) 40$i kg -ms−1 (b) 20$i kg -ms−1
(a) zero (b) 120 N
(c) 60 N (d) 600 N (c) 30$i kg -ms−1 (d) (50$i + 30$j) kg -ms−1
186 OBJECTIVE Physics Vol. 1

13. If impulse I varies with time t as F(kg - ms −1) = 20 t 2− 40 t. The 16. A bullet of mass 0.1 kg is fired with a speed of 100 ms −1.
change in momentum is minimum at The mass of gun being 50 kg, then the velocity of recoil
(a) t = 2 s (b) t = 1 s becomes
1
(c) t = s (d) t = s
3 (a) 0.05 ms−1 (b) 0.5 ms−1
2 2 (c) 0.1 ms−1 (d) 0.2 ms−1
14. A ball of mass 0.5 kg moving with a velocity of 2 ms −1 17. A ball is moving with speed 20 ms −1 collides with a smooth
strikes a wall normally and bounces back with the same surface as shown in figure. The magnitude of change in
velocity. If the time of contact between the ball and the wall is velocity of the ball will be
one millisecond, the average force exerted by the wall on the
ball is
v = 20 ms–1 v = 20 ms–1
(a) 2000 N (b) 1000 N
(c) 5000 N (d) 125 N
30° 30°
15. An initially stationary device lying on a frictionless floor
explodes into two pieces and slides across the floor. One
piece is moving in positive x-direction, then other piece is (Smooth horizontal surface)
moving in
(a) positive y-direction (b) negative y-direction (a) 10 3 ms −1 (b) 20 3 ms −1
(c) negative x-direction (d) at angle from x-direction (c) 40 3 ms −1 (d) 40 ms −1

FORCES IN EQUILIBRIUM Sol. Horizontal component of F is


Forces which have zero linear resultant will not cause FV
any change in the motion of the object to which they are
applied. Such forces (and the object) are said to be in
F
equilibrium. For understanding the equilibrium of an
object under two or more concurrent or coplanar forces, let 45°
FH
us first discuss the resolution of force.
FH = F cos 45°
Resolution of a force  1 
= (8)   = 4 2 N
When a force is replaced by an equivalent set of components,  2
it is said to be resolved. One of the most useful ways to and vertical component of F is
resolve a force is to choose only two components (although FV = F sin 45°
a force may be resolved in three or more components also)
 1 
which are at right angles also. The magnitude of these = (8)   = 4 2 N
 2
components can be very easily found using trigonometry.
B Example 5.19 Resolve a weight of 10 N in two directions
F
F2 F sin θ which are parallel and perpendicular to a slope inclined at
30° to the horizontal.
Sol. Component perpendicular to the plane,
θ F cos θ
F1 A C
w ||
Fig. 5.10 Component of force
30° w⊥
In the above figure,
F1 = F cos θ = component of F along AC 30° w = 10 N
F2 = F sin θ = component of F perpendicular to
AC or along AB 3
w ⊥ = w cos 30° = (10) =5 3N
The component of a force in a direction perpendicular to 2
itself is zero. and component parallel to the plane,
 1
Example 5.18 Resolve horizontally and vertically a force w|| = w sin 30° = (10)   = 5 N
 2
F = 8 N which makes an angle of 45° with the horizontal.
Laws of Motion 187

Example 5.21 Determine the tensions T1 and T 2 in the


Newton’s first law for forces strings as shown in figure.
in equilibrium 60°
When a particle is at rest or is moving with constant
velocity in an inertial frame of reference, the net force T1
acting on it, i.e. the vector sum of all the forces acting on it,
must be zero, i.e. ∑ F = 0 (Particle in equilibrium). T2
4 kg-wt
We most often use this equation in component form,
∑ Fx = 0, ∑ Fy = 0 and ∑ Fz = 0 T1 sin 60°
Sol. Resolving the tension T1 along
Equilibrium under concurrent forces (i.e. those forces which horizontal and vertical directions. As
act on same particle at same time) may be seen as the body is in equilibrium, 60°
F1
F3 T1 sin 60° = 4 × 9.8 N …(i) T1
F1 and T1 cos 60° = T2 …(ii) 60°
T1 cos 60°
F1 F2 T2
F2 F2
From Eq. (i), we get
F3 (i) (ii) 4 × 9.8 4 × 9.8 × 2
(a) (b) T1 = = = 45.26 N
sin 60° 3 w = 4 × 9.8 N
Fig. 5.11
Putting this value in Eq. (ii), we get
In Fig. (a) T2 = T1 cos 60° = 45.26 × 0.5 = 22.63 N
F1 + F2 = 0 Note Tension is a type of force produced in strings.
In Fig. (b)-(i) and (b)-(ii),
F1 + F2 + F3 = 0 Lami’s theorem
It states that, “if three forces acting on a particle are in
Example 5.20 An object is in equilibrium under four
equilibrium, then each force is proportional to the sine of
concurrent forces in the directions shown in figure. Find the
magnitude of F1 and F2 . the angle between the other two forces.”
F1
If an object O is in equilibrium under three concurrent
4N coplanar forces F1, F2 and F3 as shown in figure.

Y
30° Then, F1 F F
= 2 = 3
60° sin α sin β sin γ
8N
30° O X
F2
F2

Sol. The object is in equilibrium.


α γ
ΣF x = 0
F1
∴ 8 + 4 cos 60° − F 2 cos 30° = 0 F3
O
β
3
or 8 + 2 − F2 =0 Fig. 5.12
2
20 If more than three forces are given in the problem, then
or F2 = N solve the problem by using component approach. If three
3
forces are in equilibrium, then the resultant of two forces
Similarly, ΣF y = 0
is equal and opposite to the third.
∴ F1 + 4 sin 60° − F 2 sin 30° = 0 F2
4 3 F2
or F1 + − =0 F3
2 2 90°
F
or F1 = 2 − 2 3
2 F1
10
= −2 3 Fig. 5.13
3
4
or F1 = N F2 = F12 + F32
3
188 OBJECTIVE Physics Vol. 1

Here, the object is in equilibrium under three concurrent


Working with Newton’s laws forces. So, we can apply Lami’s theorem,
Normally any problem relating to Newton’s laws is solved in following F 8 T
= =
four steps sin (180° − θ ) sin (90° + θ ) sin 90°
(i) First of all we decide the system on which the laws of motion are
F 8
to be applied. The system may be a single particle, a block or a or = =T
combination of two or more blocks, two blocks connected by a sin θ cos θ
string, etc. The only restriction is that all parts of the system
8
should have the same acceleration. ∴ T =
(ii) Once the system is decided, we make the list of all the forces cos θ
acting on the system. Any force applied by the system on other 8
bodies is not included in the list of the forces. = = 10 N
(iii) Then, we make a free body diagram of the system and indicate
4 /5
the magnitude and directions of all the forces listed in step (ii) in 8 sin θ
and F =
this diagram. cos θ
(iv) In the last step, we choose any two mutually perpendicular axes
(8) (3 / 5)
say X and Y in the plane of the forces in case of coplanar forces. = =6N
Choose the X-axis along the direction in which the system is (4 / 5)
known to have or is likely to have the acceleration. A direction
perpendicular to it may be chosen as the Y-axis. If the system is in Example 5.23 A block of mass m is at rest on a rough wedge
equilibrium, any mutually perpendicular directions may be as shown in figure. What is the force exerted by the wedge on
chosen. Write the components of all the forces along the X-axis the block?
and equate their sum to the product of the mass of the system
and its acceleration, i.e.
ΣFx = ma …(i) m
This gives us one equation. Now, we write the components of the
forces along the Y-axis and equate the sum to zero. This gives us θ
another equation, i.e.
ΣFy = 0 …(ii) Sol. n)
ictio
(a) If the system is in equilibrium, we will write the two equations R
f (Fr
as
∑ Fx = 0 and ∑ Fy = 0
m sθ
θ co
(b) If the forces are collinear, the second equation, i.e. ∑ Fy = 0 sin θ mg
is not needed. mg θ mg

Example 5.22 One end of a string 0.5m long is fixed to a Since, the block is permanently at rest, it is in equilibrium.
point A and the other end is fastened to a small object of Net force on it should be zero. In this case, only two forces
weight 8N. The object is pulled aside by a horizontal force are acting on the block.
F, until it is 0.3 m from the vertical through A. Find the (i) Weight = mg (downwards)
magnitudes of the tension T in the string and the force F. (ii) Contact force (resultant of normal reaction and friction
force) applied by the wedge on the block.
A For the block to be in equilibrium, these two forces should be
T
equal and opposite. Therefore, force exerted by the wedge on
B F the block is mg (upwards).
C
Alternate method From Newton’s third law of motion, force
exerted by the block on the wedge is also mg but downwards.
8N
The result can also be obtained in a different manner.
Sol. AC = 0.5 m, BC = 0.3 m A The normal force on the block is R or N = mg cos θ and the
θ friction force on the block is f = mg sin θ (not µ mg cos θ)
∴ AB = 0.4 m T
θ because it is not the case of limiting friction.
and if ∠BAC = θ B F These two forces are mutually perpendicular.
C
AB 0.4 4
Then, cos θ = = = ∴ Net contact force would be N 2 + f 2
AC 0.5 5
8N
BC 0.3 3 or (mg cos θ )2 + (mg sin θ )2 which is equal to mg.
and sin θ = = =
AC 0.5 5
CHECK POINT 5.2
1. Four forces act on a point object. The object will be in 7. The below figure is the part of a horizontally stretched net.
equilibrium, if Section AB is stretched with a force of 10 N. The tensions in
(a) they are opposite to each other in pairs the sections BC and BF are
(b) sum of x , y and z-components of forces is zero separately E
(c) they can be represented by a closed figure of 4 sides by 150° 150°
direction and magnitude. D
(d) All of the above
2. A block of mass10 kg is suspended by three strings as
120°
shown in the figure. The tension T 2 is C 90° F
G H
B
60° 30° 120° 120°
A
T2 T3 (a) 10 N, 11 N (b) 10 N, 6 N
(c) 10 N, 10 N
T1
(d) Cannot be calculated due to insufficient data
10 kg

(a) 100 N (b)


100
N (c) 3 × 100 N (d) 50 3 N
8. A body of mass 60 kg suspended by means of three strings,
3 P , Q and R as shown in the figure is in equilibrium. The
3. An object is resting at the bottom of two strings which are tension in the string P is
inclined at an angle of120° with each other. Each string can Rod
withstand a tension of 20 N. The maximum weight of the 30°
object that can be sustained without breaking the strings is
(a) 10 N (b) 20 N (c) 20 2 N (d) 40 N R
90°
4. The pulleys and strings shown in the figure are smooth and P
of negligible mass. For the system to remain in equilibrium,
Q
the angle θ should be
M = 60 kg
Wall
θ (a) 130.9 kgf (b) 60 kgf
(c) 50 kgf (d) 103.9 kgf
2m
9. A man of mass 50 g stands on a frame of
m m mass 30 g. He pulls on a light rope which
(a) 0° (b) 30° (c) 45° (d) 60° passes over a pulley. The other end of the
rope is attached to the frame. For the system
5. A non-uniform rod AB of weight w is supported horizontally to be in equilibrium, what force man must
in a vertical plane by two light strings PA and QB as shown
exert on the rope?
in the figure. G is the centre of gravity of the rod. If PA and
QB make angles 30° and 60° respectively with the vertical, (a) 40 g (b) 80 g
AG (c) 30 g (d) 50 g
the ratio is
BG 10. Two particles of equal mass are connected to a rope AB of
P negligible mass, such that one is at end A and the other
Q dividing the length of the rope in the ratio1 : 2 from A. The
30º rope is rotated about end B in a horizontal plane. Ratio of
G 60º the tensions in the smaller part to the other is (ignore effect
A B of gravity)
w (a) 4 : 3 (b)1 : 4 (c)1 : 2 (d)1 : 3
1 1 1 11. A body is under the action of two mutually perpendicular
(a) (b) 3 (c) (d)
2 3 3 forces of 3 N and 4 N. The resultant force acting on the body is
(a) 7 N (b) 1 N
6. A weight w is suspended from the mid-point of a rope, (c) 5 N (d) zero
whose ends are at the same level. In order to make the rope
perfectly horizontal, the force applied to each of its ends 12. Two equals forces are acting at a point with an angle of 60°
between them. If the resultant force is equal to 40 3 N, the
must be
(a) less than w (b) equal to w magnitude of each force is
(c) equal to 2w (d) infinitely large (a) 40 N (b) 20 N
(c) 80 N (d) 30 N
190 OBJECTIVE Physics Vol. 1

13. When a force F acts on a body of mass m, the acceleration (a) 0.5 N (b) 1.5 N
produced in the body is a. If three equal forces 3
(c) N (d) 3 N
F1 = F2 = F3 = F act on the same body as shown in figure. 4
The acceleration produced is
F2 15. A ball of mass1 kg hangs in equilibrium from two strings
OA and OB as shown in figure. What are the tensions in
strings OA and OB ? (Take, g = 10 ms −2)
135º 90º
F1 B
m A
30º 60º
F3
(a) ( 2 − 1) a (b) ( 2 + 1) a (c) 2a (d) a 90º T
T1 2
14. Three forces acting on a body are shown in the figure. To 120º O 150º
have the resultant force only along the y-direction, the
magnitude of the minimum additional force needed is w = 10 N
4N y 1N (a) 5 N, 5 N
(b) 5 3 N, 5 3 N
30°

60° (c) 5 N , 5 3 N
x
(d) 5 3 N , 5N
2N

COMMON FORCES IN MECHANICS


Some of the common forces that we come across in mechanics are 3. Tension (T )
described below When a body is connected through a string or
rope, a force may act on the body by the string
1. Weight (w ) or rope due to the tendency of extension. This
The weight of an object is equal to the gravitational force with force is called tension.
which the earth pulls it downwards.
T=F
Weight of an object, w = mg
where, m = mass of the object and g = acceleration due to gravity. Fig. 5.16 Tension force

2. Normal reaction (R or N) Regarding tension and string, the following


It is a contact force between two surfaces in contact, which is points are important to remember
always perpendicular to the surfaces in contact. The following (i) Force of tension acts on a body in the
diagrams show normal reaction between two surfaces direction away from the point of contact or
M
tied ends of the string.
(a) ⇒ R (ii) If a string is inextensible, the magnitude of
M R acceleration of any number of masses
Fig. 5.14 connected through string is always same.
Block pushes ground downward with force R and ground pushes
the block back with force R, where R = normal reaction force. a
a
a
M R M m F
R
(b) M
a
m
⇒ (a) M

(b)
R R Fig. 5.17 Masses connected through string having
(c) m2 m1 F m2 m1 F same acceleration
Fig. 5.15 Normal reaction between two surfaces (iii) If a string is massless, the tension in it is
same everywhere. However, if a string has
Here, m1 pushes m 2 towards left by force R and m 2 pushes m1 a mass, tension at different points will be
towards right by force R. different.
Laws of Motion 191

(iv) If there is friction between string and pulley, A free body diagram of the book alone would consist of its
tension is different on two sides of the pulley, weight (w = mg ), acting through the centre of gravity and the
but if there is no friction between pulley and reaction (N ) exerted on the book by the surface.
string, tension will be same on both sides of the
Example 5.24 A cylinder of weight w is resting on a V-groove as
pulley. shown in figure. Draw its free body diagram.
These points can be understood in diagram as
follows
T T T1 T1
m m
T T2
Sol. The free body diagram of the cylinder is as shown in figure.
T T2
M M

(a) String is massless and (b) String is massless and N1 N2


there is no friction there is friction between w
between pulley and string string and pulley
T1 T2 Here, w = weight of cylinder and N1 and N 2 are the normal
m reactions between the cylinder and the two inclined walls.
T3
Example 5.25 Three blocks A, B and C are placed one over the
T4 other as shown in figure. Draw free body diagrams of all the
M three blocks.
A
(c) String is not massless and there is
friction between pulley and string
B
Fig. 5.18
C
(v) If a force is directly applied on the string, the
tension will be equal to the applied force Sol. Free body diagrams of A, B and C are shown below.
irrespective of the motion of the pulling agent.
N1 N2
4. Spring force
wA wB wC
The resistive force developed in a spring, when its
length is changed is called spring force. N1 N2 N3
∴ Spring force, F = − kx FBD of A FBD of B FBD of C
where, x = change in length of the spring Here, N1 = normal reaction between A and B,
and k = spring constant. N 2 = normal reaction between B and C
and N 3 = normal reaction between C and ground.
Free body diagram Example 5.26 A block of mass m is attached with two strings as
A free body diagram (FBD) consists of a diagrammatic shown in figure. Draw the free body diagram of the block.
representation of a single body or a sub-system of
bodies isolated from its surroundings showing all the
forces acting on it. θ
N

Sol. The free body diagram of the block is as shown in figure.


T1sin θ
Mass of book = m T1
w = mg
Fig. 5.19 Free body diagram T2 θ T1cos θ
Consider, for example, a book lying on a horizontal
surface. mg
192 OBJECTIVE Physics Vol. 1

Example 5.27 All surfaces are smooth in following figure. Example 5.28 A bob of mass m is suspended from the ceiling
Find F such that block remains stationary with respect to of a train moving with an acceleration a as shown in figure.
wedge. Find the angle θ in equilibrium position.

m θ a
F
M
θ

F
Sol. Acceleration of (block + wedge), a = Sol. This problem can also be solved by both the methods
(M + m ) (i) Inertial frame of reference (ground) FBD of bob w.r.t.
Let us solve the problem by both the methods ground (only real forces), which is also moving with an
(i) From inertial frame of reference (ground) FBD of acceleration a is shown below.
block (only real forces) with respect to ground, which is T cos θ
moving with an acceleration a is shown below.
N cos θ T
y θ a
T sin θ
N sin θ x
y
x
mg mg mg
a
∴ ΣF x = ma
∴ ΣF y = 0
⇒ T sin θ = ma …(i)
⇒ N cos θ = mg …(i)
and ΣF y = 0
and ΣF x = ma ⇒ T cos θ = mg …(ii)
⇒ N sin θ = ma …(ii) From Eqs. (i) and (ii), we get
From Eqs. (i) and (ii), we get a
a = g tan θ tan θ =
g
∴ F = (M + m )a = (M + m ) g tan θ
a 
(ii) From non-inertial frame of reference (wedge) FBD of or θ = tan−1  
block w.r.t. wedge (real forces + pseudo force) is shown  g
below. (ii) Non-inertial frame of reference (train) FBD of bob
N cos θ w.r.t. train (real forces + pseudo force) is shown below.
T cos θ
Fp = ma N sin θ T
θ

mg ma
Fp = ma T sin θ
As, w.r.t. wedge, block is stationary.
∴ ΣF y = 0 ⇒ N cos θ = mg …(iii) mg mg
ΣF x = 0 ⇒ N sin θ = ma …(iv)
In this way, the block’s net acceleration becomes zero. As, with respect to train, bob is in equilibrium.
Because all the forces acting on it balance each other for ∴ ΣF x = 0
an observer on the wedge and not for an observer on the ⇒ T sin θ = ma …(iii)
ground. ∴ ΣF y = 0
From Eqs. (iii) and (iv), we will get the same result ⇒ T cos θ = mg …(iv)
i.e. F = (M + m ) g tan θ From Eqs. (iii) and (iv), we get the same result, i.e.
Note In non-inertial frame, a force acting in a direction opposite to the a 
direction of acceleration of frame, is called pseudo force. θ = tan−1  
 g
FP = − ma
Laws of Motion 193

(iv) When the lift falls freely under gravity, if the


APPARENT WEIGHT OF A supporting cable of the lift breaks. Then, a = g.
MAN IN A LIFT The net downward force on the man is
R = m (g − g ) ⇒ R = 0
Let us consider a man of mass m is standing on a weighing
machine placed in an elevator/lift. The actual weight mg Thus, the apparent weight of the man becomes
of the man acts on the weighing machine and offers a zero. This is because both the lift and man are
reaction R given by the reading of the weighing machine. moving downwards with the same acceleration g and
so there is no force of action and reaction existing
This reaction R exerted by the surface of contact on the between the man and lift. Hence, a person develops
man is apparent weight of the person. a feeling of weightlessness when the lift falls freely
Now, we consider how R is related to mg in the different under gravity.
conditions.
Example 5.29 A spring balance is attached to the ceiling of
(i) When the lift moves upwards with R an elevator. A boy hangs his bag on the spring and the
acceleration a as shown in figure, the spring reads 49 N, when the elevator is stationary. If the
net upward force on the man is G a elevator moves downward with an acceleration of 5 ms −2 ,
R − mg = ma ⇒ R = ma + mg what will be the reading of the spring balance?
Apparent weight, R = m (g + a ) mg Sol. When the elevator is stationary, then
So, when a lift accelerates upwards, w = mg
Fig. 5.20
the apparent weight of the man ⇒ 49 = m × 9.8
inside it increases. 49
⇒ m= = 5 kg
(ii) When the lift moves downwards with 9.8
R
acceleration a as shown in figure, the When the elevator is moving downward with an acceleration,
net downward force on the man is G a R = m (9.8 − a ) = 5 (9.8 − 5) = 24 N
mg − R = ma
mg Example 5.30 If in a stationary elevator, a man is standing
Apparent weight, R = m (g − a ) with a bucket full of water. The bucket has a hole at its
So, when a lift accelerates Fig. 5.21 bottom. The rate of flow of water through this hole is R 0 .
downwards, the apparent weight of If the elevator starts to move up and then down with same
the man inside it decreases. acceleration, then the rate of flow of water are R u and R d .
Find the relation between R 0, R u and R d .
(iii) When the lift is at rest or moving with uniform
Sol. Rate of flow will be more when elevator will move in
velocity v downward or upward as shown in figure.
upward direction with some acceleration because the net
downward pull will be more and vice-versa.
R
F upward = Ru = m (g + a )
F downward = R d = m (g − a )
a=0
⇒ F at rest = R 0 = mg
Thus, relation between Ru , R 0 and R d is Ru > R 0 > R d .
mg
Example 5.31 In the adjoining figure, a wedge is fixed to an
Fig. 5.22 elevator moving upwards with an acceleration a. A block of
mass m is placed over the wedge. Find the acceleration of
Then, acceleration a = 0. So, net force on the man is the block with respect to wedge. Neglect friction.
R − mg = m × 0
⇒ R = mg a
or Apparent weight = Actual weight
m
So, when the lift is at rest, the apparent weight
of the man is his actual weight. θ
194 OBJECTIVE Physics Vol. 1

Sol. Since, acceleration of block w.r.t. wedge (an accelerating or Example 5.32 Two blocks of masses 20 N
4 kg and 2 kg are placed side-by-side 4 kg 2 kg
non-inertial frame of reference) is to be find out.
FBD of block w.r.t. wedge is shown in figure. on a smooth horizontal surface as
shown in the figure. A horizontal force of 20 N is applied
N on 4 kg block.
Find
(i) the acceleration of each block

) si (ii) and the normal reaction between two blocks.
+a
= (g Sol. (i) Since, both the blocks will move with same
θ a net
θ acceleration (say a) in horizontal direction.
Let us take both the blocks as a system. Net external force
mg + Fp = mg + ma Pseudo-acceleration (–a) on the system is 20 N in horizontal direction.
for non-inertial frame
a y
The acceleration would had been g sin θ (down the plane), if 20 N
the lift were stationary or when only weight (i.e. mg) acts 4 kg 2 kg
downwards. x

Here, downward force is m (g + a ) Using ΣF x = ma x


∴ Acceleration of the block (of course w.r.t. wedge) will be 10
ms −2
20 = (4 + 2)a = 6a or a =
(g + a ) sin θ down the plane. 3
(ii) The free body diagram of both the blocks are shown in
the figure.
APPLICATIONS OF y
20 N N
NEWTON’S LAWS OF MOTION 4 kg
N
2 kg

Consider two bodies of masses m1 and m 2 placed in direct a a x


contact with each other on a smooth platform. Using ΣF x = ma x
10
F For 4 kg block, 20 − N = 4a = 4 ×
m1 m2 3
a 40 20
⇒ N = 20 − = N
Fig. 5.23 Motion of two connected bodies
3 3
10 20
For 2 kg block, N = 2 a = 2 × = N
Suppose a horizontal force is applied and both the bodies 3 3
moves with acceleration a, then Here, N is the normal reaction between the two blocks.
F = (m1 + m 2 ) a Note In free body diagram of the blocks, we have not shown the
forces acting on the blocks in vertical direction, because normal
Now, we can calculate normal reaction between the reaction between the blocks and acceleration of the system can
bodies, using free body diagram (FBD). be obtained without using ΣFy = 0.
FBD of Ist body Example 5.33 Three blocks of masses 3 kg, 2 kg and 1 kg
a are placed side-by-side on a smooth surface as shown in
F m1 N figure. A horizontal force of 12 N is applied on 3 kg block.
Find the net force on 2 kg block.
Fig. 5.24
12 N
∴ F − N = m1a (From Newton’s IInd law) 3 kg 2 kg 1 kg

FBD of IInd body


a
Sol. All the blocks will move with same acceleration (say a) in
N horizontal direction. Let us take all the blocks as a system,
m2
net external force on the system is 12 N in horizontal direction.
Fig. 5.25 y

∴ N = m 2a 12 N
3 kg 2 kg 1 kg
x
In the same way, we can calculate the acceleration and
normal reaction for three bodies in contact.
a
Laws of Motion 195

Using ΣF x = ma x , we get m   m + 2M   F 
(iii) T1 =  + M  a =    
12 = (3 + 2 + 1)a = 6a or a =
12
= 2 ms−2 2   2  m + M
6
(m + 2M )F
Now, let F be the net force on 2 kg block in x-direction, then Tension in rope at mid-point, T1 =
using ΣF x = ma x for 2 kg block, we get 2(m + M )
F = (2)(2) = 4 N L/2 T1 T1 L/2 F
M
Note Here, net force F on 2 kg block is the resultant of N1 and m/2 m/2
N2 (N1 > N2 ), where N1 = normal reaction between 3 kg and 2 kg
block and N2 = normal reaction between 2 kg and 1 kg block.
Example 5.35 In the arrangement shown in the figure, the
Thus, F = N1 − N2 .
strings are light and inextensible. The surface over which
blocks are placed is smooth. Find
Motion of bodies connected 4 kg 2 kg 1 kg
F = 14 N

through strings or springs (i) the acceleration of each block


Horizontal motion (ii) the tension in each string.
Sol. (i) Let a be the acceleration of each block and T1 and T2 be
When two bodies are connected through an inextensible
the tensions, in the two strings as shown in the figure.
weightless string and a force is applied to impart an
y
acceleration a in both the bodies, as given in the figure
T2 T1 F = 14 N
below, then equation of motion from the FBD can be 4 kg 2 kg 1 kg
given by x

For A, T = m1a …(i) Taking the three blocks and the two strings as the system.
a
For B, F − T = m 2a …(ii)
a F = 14 N
a B 4 kg 2 kg 1 kg
A
T m2 F
m1
Using ΣF x = ma x
Fig. 5.26 Motion of two bodies connected through string 14
or = 2 ms−2
14 = ( 4 + 2 + 1)a or a =
7
From Eqs. (i) and (ii), F = m1a + m 2a …(iii)
(ii) Free body diagram (showing the forces in x-direction
m1F
From Eqs. (i) and (iii), T = only) of 4 kg block and 1 kg block are shown in figure.
(m1 + m 2 ) y
a = 2 ms–2 a = 2 ms–2
In the same way, we can calculate the acceleration and
T2 T1 F = 14 N
tension in three bodies. 4 kg 1 kg
x
Example 5.34 A block of mass M is pulled along a
horizontal frictionless surface by a rope of mass m as shown Using ΣF x = ma x
in figure. A horizontal force F is applied to one end of the For 1 kg block, F − T1 = (1)(a ) or 14 − T1 = (1) (2) = 2
rope. Find (i) the acceleration of the rope and block, (ii) the ∴ T1 = 14 − 2 = 12 N
force that the rope exerts on the block (iii) and tension in the
For 4 kg block, T2 = (4)(a )
rope at its mid-point.
m
∴ T2 = (4)(2) = 8 N
M F
Vertical motion
F In the same manner, as discussed above, we can calculate
Sol. (i) Acceleration, a =
(m + M ) acceleration, tension in the string and common force in
(ii) Force exerted by rope on the block is two or more blocks in vertical direction also.
M ⋅F
T = Ma = Example 5.36 The blocks of masses 2 kg, 3 kg and 200 N
(m + M ) 5 kg are connected by light, inextensible strings as
2 kg
a shown in figure.
T T a The system of blocks is raised vertically upwards by 3 kg
M F applying a force F 0 = 200 N . Find the common
L
FBD of block acceleration and tension in the strings.
FBD of rope 5 kg
196 OBJECTIVE Physics Vol. 1

200 N T1
Sol.
3 kg
a 2 kg a
T1 T1 − 80 = (3 + 5)a
T1 T1 − 80 = 8 × 10
3 kg T1 = 160 N
a 5 kg
T2
T2
(3 + 5)g = 80 N
a 5 kg
200 N For spring-block system
In the same manner, as discussed above, we can calculate
a 2 kg the tension in a spring connected to a block of mass m,
which produces an extension x in it, i.e. T = kx.
T1
where, k = spring constant.
2g = 20 N ↑ : 200 − T1 − 20 = 2a …(i)
T1 Example 5.37 Two masses are connected to two identical
springs of spring constant 100 N/m as shown in figure. Find
a 3 kg the extension in both the springs.
T2
3g = 30 N ↑ : T1 − T2 − 30 = 3a …(ii)
T2
1kg
a 5 kg

5g = 50 N
↑ : T2 − 50 = 5a …(iii)
2kg
Adding Eqs. (i), (ii) and (iii), we get
100 = 10a ⇒ a = 10 ms−2 Sol. The FBD for mass 2 kg,
T2
T1 = 160 N and T2 = 100 N
Alternate Method Taking three blocks together as a system 2kg
200 N
2g = 20N
2 kg
a ⇒ T2 − 20 = 0 or T2 = 20 N …(i)
3 kg
The FBD for mass 1 kg,
T1

5 kg 1kg
T2
1g = 10N
(2 + 3 + 5)g = 100 N
⇒ T1 = T2 + 10 ⇒ T1 = 30 N [Using Eq. (i)]
↑ : 200 − 100 = (2 + 3 + 5)a As, these tensions in the springs are the spring forces,
⇒ a = 10 ms−2 ∴ T1 = kx1 ⇒ 30 = 100 × x1 or x1 = 0.3 m
and T2 = kx 2 ⇒ 20 = 100 × x 2 or x 2 = 0.2 m
Two blocks at a time
200 N
Bodies attached through pulley
a
2 kg
200 − 50 − T2 = (2 + 3)a (using strings or springs )
150 − T2 = 5 × 10 Case I Consider two bodies of masses m1 and
⇒ T2 = 100 N m 2 (m 2 > m1 ) connected to a massless and
inextensible string which passes through a
3 kg
smooth pulley. If they are allowed to move freely,
T2 they move with common acceleration a. The
(2 + 3)g = 50 N tension in the string due to mass m1 and m 2 is T.
Laws of Motion 197

From FBD, Example 5.39 The pulley is light and smooth; the strings are
For A, T − m1g = m1a …(i) inextensible and light. The system is released from rest.
Find the acceleration of each block, tensions in the strings
For B, m 2 g − T = m 2a …(ii) and reaction on pulley.

Pulley

3 kg
T T

a m1
5 kg
A m2 2 kg
a
B
Fig. 5.27 Motion of connected bodies Sol.
On solving Eqs. (i) and (ii), we get
(m − m1 ) T2 T2
⇒ a= 2 g T2
(m 2 + m1 ) a
3 kg
 2m1m 2  T1
⇒ T =  g
 m1 + m 2  T2
T1
5 kg
Reaction at suspension of pulley R = 2T . a 2 kg
Example 5.38 Two blocks of masses 2.6 kg and 4.1 kg are
tied together by a light string looped over a frictionless pulley. T1

(a) What will the acceleration of each mass be? 5 kg a


(b) Find the value of tension in the string.
Sol. Here, m1 = 2.6 kg, m 2 = 4.1 kg
5g = 50 N ↓ : 50 − T1 = 5a …(i)
T2
Pulley 3 kg a

T T T1

a m1
3g = 30 N ↓ : 30 + T1 − T2 = 3a …(ii)
m2 a
m1g T2
m2g
2 kg a

FBD for m1, T − m1g = m1a (upward)


T − 2.6g = 2.6a …(i) 2g = 20 N ↑ : T2 − 20 = 2a …(iii)
FBD for m 2, m 2g − T = m 2a (downward) Solving Eqs. (i), (ii) and (iii), we get
4.1 g − T = 4.1a …(ii) a = 6 ms−2, T1 = 20 N, T2 = 32 N
(a) Adding Eqs. (i) and (ii), we get Forces on pulley, R − 2T2 = 0 (Pulley is massless)
4.1g − 2.6g = (2.6 + 4.1)a R = 2T2 = 64 N
1.5g 1.5 × 10
⇒ a= =
6.7 6.7 R
−2
= 2.2 ms
(b) Putting the value of a in Eq. (i), we get
T − 2.6g = 2.6 × 2.2
T2 T2
⇒ T = 2.6 × 2.2 + 2.6 × 10
where, R is the reaction on the pulley.
= 31.72 N
198 OBJECTIVE Physics Vol. 1

Case II When two bodies are attached through a pulley as Example 5.41 The strings are inextensible and light; the
given in the figure below. pulleys are smooth and light. Find the acceleration of each
block and tensions in the strings.
T
m1
a 2 kg
A Smooth

T
5 kg 3 kg

B m2 a a
Sol. T2
T1
Fig. 5.28 2 kg
Smooth
Here, m 2 > m1, then from FBD,
T1 T2
For A, T = m1a …(i)
a a
For B, m 2a = m 2 g − T …(ii) 5 kg 3 kg

Solving Eqs. (i) and (ii), we get


5g = 50 N 3g = 30 N
m2
a= g ↓ : 50 − T1 = 5a
(m1 + m 2 ) 5 kg : …(i)
2 kg : ← : T1 − T2 = 2a …(ii)
m1m 2
⇒ T = g 3 kg : ↑ : T2 − 30 = 3a …(iii)
(m1 + m 2 )
Adding Eqs. (i), (ii) and (iii), we get
Example 5.40 A 2 kg mass placed on a level table is 20 = 10a ⇒ a = 2 ms−2
attached to a 5 kg mass by a string passing over the edge of
a table as illustrated in the diagram. T1 = 40 N and T2 = 36 N

F1 Example 5.42 Consider the situation shown in figure.


Initially the spring is unstretched when the block of mass is
T
m1 2 kg
released from rest. Assume the pulley frictionless and light,
the spring and string massless. Find the maximum extension
T
of the spring.
F2
5 kg m2

F3
m
(a) Calculate the magnitude of acceleration of the system.
(b) Calculate the tension in the string.
Sol. From FBD in m1, Sol. The FBD of given mass m
k kx
T − m1a = 0 ⇒ mg − kx = ma or a = g − x
m
⇒ T − 2a = 0 …(i)  k  m a
vdv =  g − x dx
FBD in m 2,  m 
mg
m 2g − T = m 2a Integrating both sides, we get
5g − T = 5a
v x
 k  v2 kx 2
Putting T = 2a from Eq. (i), we get ∫ vdv = ∫ m 
 g − x dx ⇒
2
= gx −
2m
0 0
5g − 2a = 5a 1/ 2
 kx 2 
7a = 5g or v = 2gx − 
5 50 −2  m 
(a) a = × 10 = ms
7 7 When the block will stop, v = 0 (at maximum extension),
1/ 2
(b) Again, tension in the string,  kx 2  kx 2 2mg
50 100 2gx −  = 0 ⇒ 2gx = ⇒ x = xm =
T = 2a = 2 × = N  m  m k
7 7
Laws of Motion 199

Case III When pulley is attached to the edge of an Example 5.43 Consider the situation shown in the figure.
inclined plane To understand this case, first of The surface is smooth and the string and the pulley are light.
all we need to understand the motion of an Find the acceleration of each block and tension in the string.
object placed on a frictionless inclined plane.

R
g
3k
m1 2 kg
30°
θ
θ m1g cos θ
in
gs m1g Sol.
m1 θ
a
Fig. 5.29 g
3k
Consider an object of mass m1 is placed on a 2 kg a
smooth inclined plane with angle of inclination θ. 30°
From the FBD of object, different forces acting on Let 3 kg block be moving downward.
it, are
(i) Normal reaction R, acting perpendicular to N T
the plane. a
(ii) Component of weight m1g cos θ, acting
perpendicular to plane. 30° 3 0°
30
sin cos
30
(iii) Component of weight m1g sin θ downward 3g = 30 N
along the inclined plane. 30°

Here, R and m1g cos θ will cancel each other (as Along the plane: 30 sin 30° − T = 3a …(i)
here is no motion in vertically upward and T
vertical downward direction), so these forces can
be ignored. 2 kg a
R
a
T
m1 2g = 20 N
a
T
θ T − 20 = 2a …(ii)
in
gs m1g cos θ m2 Adding Eqs. (i) and (ii), we get
m1
m1g a = − 1 ms−2 ⇒ T = 18 N
m2 g
θ
Since, the acceleration is negative, i.e. the block of mass 3 kg
Fig. 5.30 is moving upward.

Now, a pulley is connected at the edge of inclined Example 5.44 In the arrangement shown, inclined plane is
plane and a block of mass m 2 (m1 > m 2 ) is smooth, strings and pulleys are massless. Find
T1
.
connected through a string passing over the pulley T2
to mass m1 as shown in Fig. 5.30.
T1 g
3k
Now, equation of motion for m 2 , T2
g
5k
T − m 2 g = m 2a …(i) 2 kg
Equation of motion for m1, 30°

m1g sinθ − T = m1a …(ii) 5 kg 3 kg 2 kg


Sol.
From Eqs. (i) and (ii), we get a
N1 T1 N2 T2 T2
(m g sin θ − m 2 g ) a T1
a= 1
(m1 + m 2 ) 2 kg a
0° 0° 0° 0°
0s in 3 os 3 0s in 3 os 3
5 50c 3 30c
m1m 2
T = (1 + sin θ ) g 30° 50 N 30° 30 N
m1 + m 2 20 N
200 OBJECTIVE Physics Vol. 1

5 kg : along the plane : 50 sin 30° − T1 = 5a …(i) Total mass being pulled = 1 + 3 + 2 = 6 kg
3 kg : along the plane : 30 sin 30° + T1 − T2 = 3a …(ii) 24.64
∴ Acceleration of the system, a = = 4.10 ms−2
2 kg : ↑ : T2 − 20 = 2a …(iii) 6
Adding Eqs. (i), (ii) and (iii), we get (ii) For the tension in the string between A and B
FBD of A
20 = 10a ⇒ a = 2 ms−2
a T1
T1 = 15 N and T2 = 24 N
T1 15 5 A
∴ = =
T2 24 8 6 0°
in
gs
m A
Example 5.45 In the arrangement shown, all the surfaces are
smooth, strings and pulleys are light. Find the tension in the
string. m A g sin 60° − T1 = (m A )(a )
∴ T1 = m A g sin 60° − m A a
= m A (g sin 60° − a )
B A  3 
2k
g ∴ T1 = (1) 10 × − 4.10 = 4.56 N
2k

 2 
g

37° 53° For the tension in the string between B and C


FBD of C T2
Sol. a
C
a T T a
B A
kg mC g sin 30°
2k

2
g

37
°
20 N
T2 − m C g sin 30° = (m C ) (a )
20

in
0s 37° 20 N 53° ∴ T2 = m C (a + g sin 30° )
sin

2
53

  1 
°

∴ T2 = 2 4.10 + 10    = 18.2 N
Block A : along the plane, 20 sin 53° − T = 2a …(i)   2 
Block B : along the plane, T − 20 sin 37° = 2a …(ii)
Adding Eqs. (i) and (ii), we get Example 5.47 Calculate the net acceleration produced in the
20 (sin 53° − sin 37° ) = 4a arrangements shown below.
(i)
 4 3
20  −  = 4 = 4a ⇒ a = 1 ms−2
 5 5
3
T = 20 sin 37° + 2a = 20 × + 2 × 1 = 14 N
5
Example 5.46 In the adjacent figure, masses of A, B and C
a m 3m a
are 1 kg, 3 kg and 2 kg, respectively. Find

mg 3mg
B

C (ii) a
m
A

60° 30°

(i) the acceleration of the system 2m a


(ii) and tension in the string.
Neglect friction. (g = 10 ms −2 ) 2mg
a
Sol. (i) In this case, net pulling force (iii)
= m Ag sin 60° + m B g sin 60° − m C g sin 30° m

 3  3  1
= (1)(10)   + (3)(10)   − (2)(10)  
 2   2   2 a 2m 3m a

= 24.64 N
2mg 3mg
Laws of Motion 201

(iv) (ii) Pulling force = 2mg


Total mass = 3m
2mg 2g
a= =
3m 3
m a
mg
(iii) Net pulling force = 3mg − 2mg = mg
Total mass = 3m + m + 2m = 6m
a 2m 3m a
mg g
a= =
2mg 3mg 6m 6
(iv) Net pulling force = 3mg + mg − 2mg
(v)
= 2mg
a Total mass = 3m + m + 2m = 6m
4m 2mg g
° m a a= =
30 6m 3
in
gs (v) Net pulling force = 4mg sin 30° − mg
4m 30° mg
= mg
Sol. (i) Net pulling force = 3mg − mg = 2mg Total mass = 4m + m = 5m
Total mass = 3m + m = 4m mg g
2mg g ⇒ a= =
a= = 5m 5
4m 2
CHECK POINT 5.3
1. Find the force exerted by 5 kg block on floor of lift, as T1 T2 T3
shown in figure. (Take, g = 10 ms −2) m1 m2 m3

(a) 20 N (b) 40 N (c) 10 N (d) 32 N


5. The surface is frictionless, the ratio between T1 and T 2 is
5 ms–2
F
2 kg T2 T1 30°
3 kg 12 kg 15 kg
5 kg

(a) 100 N (b) 115 N (a) 3 :1 (b) 1 : 3 (c) 1 : 5 (d) 5 : 1


(c) 105 N (d) 135 N
6. Two blocks of masses 2 kg and 1 kg are in contact with each
2. A 50 kg boy stands on a platform having spring scale in a other on a frictionless table. When a horizontal force of 3 N
lift that is going down with a constant speed of 3 ms −1 . If is applied to the block of mass 2 kg, the value of the force of
the lift is brought to rest by a constant deceleration in a contact between the two blocks is
distance of 9 m, what does the scale read during this (a) 4 N (b) 3 N (c) 5 N (d) 1 N
period? (Take, g = 9.8 ms −2 ) 7. Three blocks are placed at rest on a smooth inclined plane
(a) 500 N (b) 465 N with force acting on m1 parallel to the inclined plane. Find
(c) 515 N (d) Zero the contact force between m2 and m3.
3. An elevator and its load have a total mass of 800 kg. If the
elevator, originally moving downward at10 ms −1 , is brought m3
to rest with constant deceleration in a distance of 25 m, the m2
tension in the supporting cable will be (Take, g = 10 ms −2) m1
(a) 8000 N (b) 6400 N θ
(c) 11200 N (d) 9600 N F

4. Three blocks of masses m1 , m2 and m3 are connected by (m1 + m2 + m3) F m3 F


(a) (b)
massless strings as shown on a frictionless table. They are m3 m1 + m2 + m3
pulled with a force T 3 = 40 N. If m1 = 10 kg, m2 = 6 kg and
m3 = 4 kg, the tension T 2 will be (c) F − (m1 + m2) g (d) None of these
202 OBJECTIVE Physics Vol. 1

8. In the arrangement shown, the mass m will ascend with an 12. The acceleration of the 2 kg block, if the free end of string is
acceleration (pulley and rope are massless) pulled with a force of 20 N as shown, is

F = 20 N

m
3 2 kg
m
2
(a) zero (b) 10 ms−2 upward
g
(a) zero (b) (c) 5 ms−2 upward (d) 5 ms−2 downward
5
(c) g (d) 2g 13. In the arrangement shown in the figure, the pulley has a
9. Two masses are connected by a string which passes over mass 3m. Neglecting friction on the contact surface, the
a pulley accelerating upwards at a rate A as shown in force exerted by the supporting rope AB on the ceiling is
figure. If a1 and a2 be the accelerations of bodies 1 and 2
respectively, then A

A
B

a1
a2 m
2m
1

2
(a) 6 mg (b) 3 mg
(a) A = a1 − a 2 (b) A = a1 + a 2 (c) 4 mg (d) None of these
a − a2 a + a2
(c) A = 1 (d) A = 1
2 2 14. In the figure given below, with what acceleration does the
block of mass m will move? (Pulley and strings are massless
10. Three equal weights A , B and C of mass 2 kg each are
hanging on a string passing over a fixed pulley which is and frictionless)
frictionless as shown in figure. The tension in the string
connecting weight B and C is

m 2m
A
B
3m
C g 2g 2g g
(a) (b) (c) (d)
(a) zero (b) 13 N (c) 3.3 N (d) 19.6 N 3 5 3 2
11. A light string going over a clamped pulley of mass m 15. Three masses of 1 kg, 6 kg and 3 kg are connected to each
supports a block of mass M as shown in the figure. The other with threads and are placed on a table as shown in
force on the pulley by the clamp is given by
figure. What is the acceleration with which the system is
moving? (Take, g = 10 ms −2 )
m a
T1 T2
6 kg

M T1
1 kg 3 kg
(a) 2 Mg (b) 2 mg
(a) Zero (b) 2 ms−2
(c) g (M + m)2 + m2 (d) g (M + m)2 + M 2
(c) 4 ms−2 (d) 3 ms−2
Laws of Motion 203

16. Two bodies of masses m1 and m2 are connected by a light a = 1 ms–2


F
string which passes over a frictionless massless pulley. If
g
the pulley is moving upward with uniform acceleration ,
2
then tension in the string will be (a) 50 N (b) 100 N
3m1 m2 (c) 200 N (d) 10 N
(a) g
m1 + m2 19. Two masses M1 and M2 are attached to the ends of a string
m + m2
(b) 1 g which passes over a pulley attached to the top of an
4m1 m2 inclined plane. The angle of inclination of the plane is 30°
2m1 m2 and M1 = 10 kg, M2 = 5 kg. What is the acceleration of mass
(c) g
m1 + m2 M2?
m1 m2
(d) g
m1 + m2
17. In the system shown in figure, assume that all the surfaces M1
are smooth, string and spring are massless. When masses
connected are released, the acceleration of the system is M2
θ
m (a) 10 ms−2 (b) 5 ms−2
(c) Zero (d) None of these
2m 20. Two blocks, each having a mass M, rest on frictionless
surfaces as shown in the figure. If the pulleys are light and
frictionless and M on the incline is allowed to move down,
then the tension in the string will be
3m

M
17 50
(a) ms −2 (b) ms −2
5 6
60 60
(c) ms −2 (d) ms −2
5 7 θ M

18. A block of mass 200 kg is set into motion on a frictionless 2 3


horizontal surface with the help of frictionless pulley and a (a) Mgsinθ (b) Mgsinθ
3 2
rope system as shown in figure. What horizontal force F Mg sin θ
should be applied to produce in the block an acceleration of (c) (d) 2Mg sinθ
1 ms −2? 2

FORCE OF FRICTION
f
Whenever a body moves or tends to move over the If a body is at rest and no pulling force is acting on it, then force
surface of another body, a force comes into play of friction on it is zero.
which acts parallel to the surface of contact and Direction of
opposes the relative motion. This opposing force is motion
Contact of
called friction. surface
Consider a wooden block placed on a horizontal
f (Force of friction)
surface and is given a gentle push. The block slides
through a small distance and comes to rest.
According to Newton’s second law, a retarding force Direction of
motion
must be acting on the block. This retarding force or f (Force of
opposing force is called friction or friction force. friction)

As shown in the figure, the force of friction always Contact of


acts tangential to the surface in contact and in a surface
direction opposite to the direction of (relative) motion Fig. 5.31 Friction in different directions
of the body.
204 OBJECTIVE Physics Vol. 1

Friction between the two bodies depends upon (iii) It is independent of the speed of sliding, provided that the
(i) Nature of medium of contact between two resulting heat does not alter the condition of the surface.
bodies, the roughness of medium increases the Note If more than two blocks are placed one over the other on a horizontal
friction. ground, then normal reaction between two blocks will be equal to the
weight of the blocks over the common surface.
(ii) Normal reaction, as normal reaction increases,
the interlocking between two surfaces in contact A
increases because they press harder against each B
other, so friction increases. C

D
Types of friction
Fig. 5.32 Normal reaction between blocks
Friction is mainly of two types
e.g. N1 = normal reaction between A and B = mA g
N2 = normal reaction between B and C
1. Static friction = ( mA + mB) g and so on.
It is the friction force which comes into existence
when one object tends to move over the surface of Example 5.48 Suppose a block of mass 1 kg is
placed over a rough surface and a horizontal F
other object but the actual force has not yet started.
force F is applied on the block as shown in
If we keep on increasing the applied force to create figure. What are the values of force of friction
movement, then a stage comes when the object is on f, if the force F is gradually increased.
the verge of moving over the other object. Given that, µ s = 0.5, µ k = 0.4 and g = 10 ms −2 .
Friction at this time is called limiting friction. Sol. Free body diagram of block is
Limiting friction is maximum value of static friction. N
a
y
Laws of static friction
F
(i) The force of static friction is proportional to the
normal force exerted by other surface normally on f x
the former. mg
(ii) The static frictional force is given by f s ≤ µ s N, ΣF y = 0
where µ s is the coefficient of static friction. ∴ N − mg = 0 or N = mg = (1)(10) = 10 N
(iii) The less than or equal to sign in the above fL = µ s N = (0.5)(10) = 5 N
equation represents the adjusting nature of the and fk = µ kN = (0.4 )(10) = 4 N
force of static friction. Example 5.49 Blocks A and B of masses 5 kg and 10 kg are
(iv) The equality sign in the equation holds only placed as shown in figure. If block A is pulled with 50 N, find
when the static frictional force has its maximum out acceleration of the A and B. If coefficient of friction between
value. A and B is 0.5 and between B and ground is 0.4.
(v) It is independent of the area of contact between A 5 kg 50 N
the two surfaces.
B 10 kg
2. Kinetic friction
The friction force that comes into existence when one Sol. Limiting friction between A and B,
object is actually moving over the surface of other f1 = µ1R1 = 0.5 × 5 g = 24.5 N
object, is called kinetic or dynamic friction. Limiting friction between B and ground,
f2 = µ 2R 2 = 0.4 × 15g = 58.8 N
Laws of kinetic friction
R2 R1
(i) The force of kinetic friction is proportional to
the normal force, which presses the two surfaces A 5 kg 50 N
together. Mathematically, fk ≤ µ k N, where µ k is f1
5g
the coefficient of kinetic friction. B 10 kg
f2
(ii) It is independent of the surface area of contact.
15g
Laws of Motion 205

For block A, 50 − f1 = 5 × a Contact force is the resultant of force of friction and


⇒ 50 − 24.5 = 5 × a normal reaction, so
⇒ a = 51. ms−2
C = f 2 + N 2 = (40)2 + (100)2 = 107.7 N
For block B, accelerating force f1 = 24.5 N is less than limiting
C
friction f2 = 58.8 N, so block B will remain at rest. N

Example 5.50 12 N of force required to be applied on A of


mass 4 kg to slip on B of mass 5 kg. Find the maximum
horizontal force F to be applied on B, so that A and B move
f
together.
A (ii) F = 60 N is greater than the limiting friction on the
FB body, so body will start moving. Force of friction acting
B on the body
fk = kinetic friction = µ kN = 0.45 × 100 N = 45 N
Sol. Let µ be friction coefficient between A and B. FBD of the body
a
As 12 N force on A is required for slipping, so
F = 60 N
µm Ag = 12
f = fk = 45 N
12
⇒ µ= = 0.3 ∴ Acceleration of the body is
4 × 10
F − fk 60 − 45
Maximum force (FB ) applied on B, so that A and B move a= = = 1.5 ms−2
together m 10
FB = (m A + m B ) a Contact force, C = f 2 + N 2 = fk2 + N 2
where, a = µg
⇒ FB = (m A + m B ) µg = (45)2 + (100)2 ≈ 109.7 N
= (4 + 5)(0.3)(10) = 27 N
Example 5.52 In the adjoining figure, the coefficient of
Example 5.51 A body of mass 10 kg is kept on a horizontal friction between wedge (of mass M) and block (of mass m)
floor of coefficient of static friction µ s = 0.5 and coefficient is µ. Find the minimum horizontal force F required to keep
of kinetic friction µ k = 0.45 as shown in figure. the block stationary with respect to wedge.

10 kg F M m
F
µs = 0.5
µk = 0.45
Sol. Such problems can be solved with or without using the concept
of pseudo force. Let us solve the problem by both the methods.
Find the acceleration, force of friction and contact force on
a = Acceleration of (wedge + block) in horizontal direction
the body by the plane when the driving force is (g = 10 ms −2 )
F
(i) 40 N (ii) 60 N =
M +m
Sol. (i) FBD of the body
(i) Inertial frame of reference (ground) FBD of block
N with respect to ground (only real forces have to be
F
applied); with respect to ground block is moving with an
acceleration a.
f
mg F = µN
y
Normal reaction, N = mg = 100 N
N x
Limiting friction on the body,
fL = µ s N = 0.5 × 100 N = 50 N
F = 40 N is less than the limiting friction, so the body is mg
static, then a = 0. a

Force of friction acting on the body is static friction, Therefore, ΣF y = 0


f = driving force = 40 N and ΣF x = ma
206 OBJECTIVE Physics Vol. 1

⇒ mg = µN and N = ma N F
g
∴ a=
µ
g λ
∴ F = (M + m )a = (M + m )
µ µN (f)
(ii) Non-inertial frame of reference (wedge) FBD of m mg
with respect to wedge (real + one pseudo force); with Fig. 5.33 Angle of friction
respect to wedge, block is stationary.
F = µN The resultant of these two forces is F and it makes an
angle λ with the normal, where
Fp = ma N µ N f
tan λ = s = µ s =
N N
λ = tan −1 (µ s )
mg
or …(i)
∴ ΣF x = 0 = ΣF y
∴ mg = µN and N = ma or µ s = tan λ


g
a = and F = (M + m )a = (M + m )
g This angle λ is called the angle of friction.
µ µ Thus, the coefficient of static friction is equal to the
From the above discussion, we can see that from both tangent of the angle of friction.
the methods results are same.
Angle of repose (α )
Example 5.53 If the coefficient of friction between an insect
and bowl is µ and the radius of the bowl is r, find the The minimum angle of inclination of plane with the
maximum height to which the insect can crawl up in the bowl. horizontal at which the body placed on the plane just
begins to slide down, is known as angle of repose.
Sol. The insect will crawl up the bowl till the component of its
weight along the bowl is balanced by limiting frictional force. So, N
resolving weight perpendicular to the bowl and along the bowl. f

R = mg cos θ …(i) m
θ
FL = mg sin θ …(ii) n θ
mg si cos θ
mg mg
Dividing Eq. (ii) by Eq. (i), we get
θ
F
tan θ = L or tan θ = µ (Q FL = µR )
R Fig. 5.34 Angle of repose

O Suppose a block of mass m is placed on an inclined plane


r
θ
whose inclination θ can be increased or decreased. Let µ
FL R y be the coefficient of friction between the block and the
A plane. At any angle of inclination,
h
mg cosθ mg sinθ Normal reaction, N = mg cos θ
Limiting friction, fL = µ s N = µ s mg cos θ
(r 2 − y 2 ) and the driving force (or pulling force),
From ∆OAB , =µ
y F = mg sin θ (Down the plane)
r
or y= From these three equations; we see that when θ is
1 + µ2 increased from 0° to 90°, normal reaction N and hence,
 the limiting friction fL is decreased while the driving force
1 
So, h = r − y = r 1 −  F is increased. There is a critical angle called angle of
 1 + µ 2  repose (α ) at which these two forces are equal.
Now, if θ is further increased, then the driving force F
Angle of friction (λ) becomes more than the limiting friction fL and the block
The angle which the resultant of the force of limiting starts sliding.
friction and normal reaction makes with the direction of Thus, fL = F
normal reaction is known as angle of friction. At θ =α
Laws of Motion 207

or µ mg cos α = mg sin α From Eqs. (ii) and (iii), we get


or tan α = µ ⇒ F = mg (sin θ + µ cos θ )
or α = tan −1 (µ ) …(ii) ⇒ a = g (sin θ + µ cos θ )
From Eqs. (i) and (ii), we see that angle of friction (λ ) is Note To calculate the work done over a rough inclined surface,
multiply force with distance travelled by the body, i.e.
numerically equal to the angle of repose. W = F × s = ma × s = mgs [ sinθ + µ cosθ] (For body moving
or λ =α upwards) and W = mgs (sinθ − µ cosθ) (For body moving
downwards).
From the above discussion, we can conclude that
(i) if θ < α, F < fL , the block is stationary. Example 5.54 A block of mass 5 kg rests on an inclined
plane at an angle of 30° with the horizontal. If the block just
(ii) if θ = α, F = fL , the block is on the verge of sliding,
begins to slide, then what is the coefficient of static friction
(iii) if θ > α, F > fL , the block slides down with acceleration. between the block and the surface?
Sol. In equilibrium, the resultant of these forces must be zero.
EQUATION OF MOTION ON A fs
N

ROUGH INCLINED PLANE


(i) When body is moving downward due to a mg sin θ
acceleration (a ) and the force of friction (f ) between mg cos θ
mg
inclined plane and the body. 30°

R ∴ mg sin θ = fs …(i)
f = force of friction
and mg cos θ = N …(ii)
a θ fs
in
θ mg cos θ ∴ tan θ = [Dividing Eq. (i) by Eq. (ii)]
m gs mg N
As, fs = µ sN
Fig. 5.35 Downward motion of a body (FBD)
⇒ tan θ = µ s
From FBD, R = mg cos θ ∴ µ s = tan 30°
As the body is moving downward with acceleration ⇒ µ s = 0.577
a, then net force in downward direction = ma Example 5.55 A block of wood of 1 kg resting on an inclined
mg sin θ − f = ma plane of angle 30°, just starts moving down. If the coefficient
of friction is 0.2, then find its velocity (in ms −1) after 5 s.
We know that, f = µR = µmg cos θ
(Take, g = 10 ms −2 )
Thus, a = (sin θ − µ cos θ ) g Sol. Acceleration of block down the plane is given by
R
(ii) When body is moving upward due to a force F fs
applied on it as shown in the figure.
θ
θ c os
a sin mg
mg θ mg
R F

For the motion of body along the incline,


θ mg sin θ − µmg cos θ
θ mg cos θ a= (As, R = mg cos θ and fs = µR )
sin f mg m
mg θ
= 10 × sin 30° − 0.2 × 10 cos 30°
Fig. 5.36 Upward motion of a body (FBD)
1 3
= 10 ×−2× = 3.268 ms−2
From FBD, R = mg cos θ …(i) 2 2
From first equation of motion, v = u − at
F = mg sin θ + f …(ii)
Velocity after 5 s, v = 0 + 3 . 268 × 5 = 16.34 ms−1
f = µR = µmg cos θ …(iii)
208 OBJECTIVE Physics Vol. 1

Example 5.56 A rod AB rests with Y ∠ABC = 60°


the end A on rough horizontal B ∴ ∠BAC = 30°
ground and the end B against a Let N1 be the reaction of the wall and N 2 be the reaction
smooth vertical wall. The rod is of the ground.
uniform and of weight w. If the rod
is in equilibrium in the position Force of friction f between the ladder and the ground
acts along BC.
shown in figure. 30° A
O X For horizontal equilibrium, f = N1 …(i)
Find
(a) frictional force at A, For vertical equilibrium, N 2 = w …(ii)
(b) normal reaction at A Taking moments about B, we get for equilibrium,
(c) and normal reaction at B. N1(4 cos 30° ) − w (2 cos 60° ) = 0 …(iii)
Sol. Let length of the rod be 2l. Using the three conditions of Here, w = 250 N
equilibrium, anti-clockwise moment is taken as positive. Solving these three equations, we get
Y
f = 72.17 N
B NB and N 2 = 250 N
f 72.17
∴ µ= = = 0.288
N2 250
NA
w Example 5.58 A block of mass 1 kg is pushed against a
30° A rough vertical wall with a force of 20 N (coefficient of static
X
O fA
friction being 1/4). Another horizontal force of 10 N is
Q ΣF x = 0 applied on the block in a direction parallel to the wall. Will
∴ N B − fA = 0 the block move? If yes, in which direction? If no, find the
or N B = fA …(i) frictional force exerted by the wall on the block.
(Take, g = 10 ms −2)
Q ΣF y = 0
∴ NA − w = 0 Sol. Normal reaction on the block from the wall will be
N = F = 20 N
or NA = w …(ii)
Q Στ 0 = 0
F = 20 N
∴ N A (2l cos 30° ) − N B (2l sin 30° ) − w (l cos 30° ) = 0 N
A

3
or 3N A − N B − w=0 …(iii)
2 Therefore, limiting friction,
Solving these three equations, we get  1
fL = µN =   (20) = 5 N
3 3  4
(a) fA = w (b) N A = w (c) N B = w
2 2 Weight of the block is
Example 5.57 A 4 m long ladder weighing 25 kg rests with w = mg = (1)(10) = 10 N
its upper end against a smooth wall and lower end on rough A horizontal force of 10 N is applied to the block.The
ground. What should be the minimum coefficient of friction resultant of these two forces will be10 2 N in the direction
between the ground and the ladder for it to be inclined at shown in figure. Since, this resultant is greater than the
60° with the horizontal without slipping? (Take, limiting friction, so the block will move in the direction of
g = 10 ms −2 ) Fnet with acceleration,
Sol. In figure, AB is a ladder of weight w which acts at its
10 N
centre of gravity G.
N1 A
45°

30°

G Fnet = 10√2 N 10 N
N2 w
F net − fL 10 2 − 5
60° C a= = = 9.14 ms−2
B f m 1
CHECK POINT 5.4
1. The limiting value of static friction between two surfaces 9. A block is gently placed on a conveyor belt moving
in contact is horizontally with constant speed. After t = 4 s, the velocity
(a) proportional to normal force between the surfaces in of the block becomes equal to the velocity of the belt.
contact
If the coefficient of friction between the block and the belt
(b) independent of area of contact
(c) depends on the microscopic area of constant magnitude is µ = 0.2, then the velocity of the conveyor belt is
(d) All of the above (a) 8 ms−1 (b) 4 ms−1
(c) 6 ms−1 (d) 18 ms−1
2. A mass placed on an inclined plane is just in equilibrium. If
µ is coefficient of friction of the surface, then maximum 10. The breaking strength of the cable used to pull a body is
inclination of the plane with the horizontal is 40 N. A body of mass 8 kg is resting on a table of coefficient
(a) tan−1 µ (b) tan−1 (µ / 2) of friction µ = 0. 2. The maximum acceleration which can be
produced by the cable connected to the body is (Take,
(c) sin−1 µ (d) cos−1 µ
g = 10 ms −2)
3. A 30 kg block rests on a rough horizontal surface. A force of (a) 6 ms−2 (b) 3 ms−2
200 N is applied on the block. The block acquires a speed of (c) 8 ms−2 (d) 8 ms−2
4 ms −1, starting from rest in 2 s. What is the value of
coefficient of friction? 11. A block of mass m is placed on the top of another block of
10 3 mass M as shown in the figure . The coefficient of friction
(a) (b) between them is µ.
3 10
(c) 0.47 (d) 0.184
m
4. A car having a mass of1000 kg is moving at a speed of
−1
30 ms . Brakes are applied to bring the car to rest. If the a
M
frictional force between the tyres and the road surface is
5000 N, the car will come to rest in
(a) 5 s (b) 10 s
(c) 12 s (d) 6 s The maximum acceleration with which the block M may
move, so that m also moves along with it, is
5. A 100 N force acts horizontally on a block of mass10 kg M
placed on a horizontal rough table of coefficient of friction (a) µg (b) µ g
m
µ = 0.5. If g at the place is10 ms −2, the acceleration of the m g
block is (c) µ g (d)
M µ
(a) zero (b) 10 ms−2
−2
(c) 5 ms (d) 5.2 ms−2 12. In the shown arrangement, mass of A = 1 kg and mass of
6. A block of mass 2 kg is placed on the floor. The coefficient B = 2 kg. Coefficient of friction between A and B = 0.2.
of static friction is 0.4. If a force of 2.8 N is applied on the
block parallel to the floor, the force of friction between the A
block and floor is B F = 10 N
(Take, g = 10 ms −2)
(a) 2.8 N (b) 8 N (c) 2 N (d) zero
7. A body is projected along a rough horizontal surface with a There is no friction between B and ground. The frictional
velocity 6 ms −1. If the body comes to rest after travelling force exerted by A on B equals
9 m, then coefficient of sliding friction is (Take, g = 10 ms −2 ) (a) 2 N (b) 3 N
(c) 4 N (d) 5 N
(a) 0.5 (b) 0.4 (c) 0.6 (d) 0.2
13. A block of mass 4 kg is placed on a rough horizontal plane.
8. The coefficient of friction between the tyres and road is 0.4. A time dependent force F = kt 2 acts on the block, where
The minimum distance covered before attaining a speed of
k = 2 Ns −2 and coefficient of friction µ = 0.8.
8 ms −1 starting from rest is nearly (Take, g = 10 ms −2 )
Force of friction between block and the plane at t = 2 s is
(a) 8 m (b) 4 m (a) 8 N (b) 4 N
(c) 10 m (d) 16 m (c) 2 N (d) 32 N
210 OBJECTIVE Physics Vol. 1

14. A block of weight 5 N is pushed against a vertical wall by a of static friction between the block and the plane is 0.7. The
force 12 N. The coefficient of friction between the wall and frictional force on the block is
block is 0.6. The magnitude of the force exerted by the wall (a) 9.8 N (b) 0.7 × 9.8 × 3N
on the block is
(c) 9.8 × 3 N (d) 0.7 × 9.8 N

12 N 17. A minimum force F is applied to a block of mass 102 kg to


prevent it from sliding on a plane with an inclination angle
30° with the horizontal. If the coefficients of static and
kinetic friction between the block and the plane are 0.4 and
0.3 respectively, then the force F is
(a) 12 N (b) 5 N (a) 157 N (b) 224 N
(c) 7.2 N (d) 13 N (c) 315 N (d) zero
15. A body of mass 10 kg is placed on rough surface, pulled by a 18. A box of mass 8 kg is placed on a rough inclined plane of
force F making an angle of 30° above the horizontal. If the inclination θ. Its downward motion can be prevented by
angle of friction is also 30°, then the minimum applying an upward pull F and it can be made to slide
magnitude of force F required to move the body is equal to upwards by applying a force 2F. The coefficient of friction
(Take, g = 10 ms −2) between the box and the inclined plane is
(a) 100 N (b) 50 2 N 1
(a) tanθ (b) 3tanθ
(c) 100 2 N (d) 50 N 3
16. A block of mass 2 kg rests on a rough inclined plane 1
(c) tanθ (d) 2tanθ
making an angle of 30° with the horizontal. The coefficient 2
Chapter Exercises
(A) Taking it together
Assorted questions of the chapter for advanced level practice

1 A metre scale is moving with uniform velocity. This 7 A block of mass m is placed on a smooth plane
implies inclined at an angle θ with the horizontal. The force
(a) the force acting on the scale is zero, but a torque about exerted by the plane on the block has a magnitude
the centre of mass can act on the scale (a) mg (b) mg sec θ (c) mg cos θ (d) mg sin θ
(b) the force acting on the scale is zero and the torque
acting about centre of mass of the scale is also zero 8. A block has been placed on an inclined plane. The
(c) the total force acting on it need not be zero but the slope angle θ of the plane is such that the block
torque on it is zero slides down the plane at a constant speed. The
(d) Neither the force nor the torque need to be zero coefficient of kinetic friction is equal to
2 Conservation of momentum in a collision between (a) sin θ (b) cos θ (c) g (d) tan θ
particles can be understood from 9 A body is projected up a 45° rough incline. If the
(a) conservation of energy coefficient of friction is 0.5, then the retardation of
(b) Newton’s first law only the block is
(c) Newton’s second law only g g 3g g
(d) Both Newton’s second and third law (a) (b) (c) (d)
2 2 2 2 2 2
3 If the elevator in the shown 10 A rope of length L and mass M is hanging from a
figure is moving upwards with 1 ms –2
rigid support. The tension in the rope at a distance x
constant acceleration 1 ms −2 , A from the rigid support is
the tension in the string L − x  L  x
connected to block A of mass (a) Mg (b)   Mg (c)   Mg (d) Mg
 L  L − x L
6 kg would be
(Take, g = 10 ms −2 ) 11 Consider the shown arrangement. Assume all
(a) 60 N (b) 66 N surfaces to be smooth. If N represents magnitudes of
(c) 54 N (d) 42 N normal reaction between block and wedge, then
acceleration of M along horizontal is equal to
4 A block is placed on the top of a smooth inclined
plane of inclination θ kept on the floor of a lift. Y
When the lift is descending with a retardation a, the
block is released. The acceleration of the block
relative to the incline is m X
(a) g sin θ (b) a sin θ M
θ
(c) (g − a ) sin θ (d) (g + a ) sin θ
5 A cricket ball of mass 150 g has an initial velocity N sin θ
u = (3 $i + 4$j) ms −1 and a final velocity (a) along + ve X -axis
M
v = −(3 $i + 4$j) ms −1, after being hit. The change in N cos θ
(b) along − ve X -axis
momentum (final momentum − initial momentum) is M
(in kg-ms −1) (c)
N sin θ
along − ve X -axis
(a) zero (b) − (0.45$i + 0.6$j) M
N sin θ
(c) − (0.9$i + 1.2$j) (d) −5 ($i + $j)$i (d) along − ve X -axis
m +M
6 In the previous question (5), the magnitude of the
momentum transferred during the hit is 12 In the above problem, normal reaction between
−1 ground and wedge will have magnitude equal to
(a) zero (b) 0.75 kg-ms
(a) N cos θ + Mg (b) N cos θ + Mg + mg
(c) 1.5 kg-ms−1 (d) 14 kg-ms−1 (c) N cos θ − Mg (d) N sin θ + Mg + mg
212 OBJECTIVE Physics Vol. 1

13 A block of mass m is at rest on an inclined plane 18 In the figure shown, a person wants to raise a block
which is making angle θ with the horizontal. The lying on the ground to a height h. In both the cases,
coefficient of friction between the block and plane is if time required is the same, then in which case, he
µ. Then, frictional force acting between the surfaces has to exert more force. (Assume pulleys and strings
is are light)

θ
(i) (ii)
(a) µ mg (b) µ mg sin θ (a) (i) (b) (ii)
(c) µ (mg sin θ − mg cos θ) (d) mg sin θ (c) Same in both (d) Cannot be determined
14 If a body loses half of its velocity on penetrating 19 A body of mass 2kg travels according to the relation
3 cm in a wooden block, then how much will it x (t ) = pt + qt 2 + rt 3 , where q = 4 ms −2, p = 3 ms −1
penetrate more before coming to rest?
(a) 1cm (b) 2 cm and r = 5 ms −3 . The force acting on the body at
(c) 3 cm (d) 4 cm t = 2 s is
15 An insect crawls up a hemispherical surface very (a) 136 N (b) 134 N (c) 158 N (d) 68 N
slowly (see the figure). The coefficient of friction 20 A body with mass 5 kg is acted upon by a force
between the insect and the surface is 1/3. If the line F = (−3 $i + 4$j) N. If its initial velocity at t = 0 is
joining the centre of the hemispherical surface to the u = (6$i − 12$j) ms −1, the time at which it will just
insect makes an angle α with the vertical, the
have a velocity along theY-axis is
maximum possible value of α is given by (a) zero (b) 10 s (c) 2 s (d) 15 s
21 A 5000 kg rocket is set for vertical firing. The
α exhaust speed is 800 ms −1. To give an initial upward
acceleration of 20 ms −2 , the amount of gas ejected
per second to supply the needed thrust will be (Take,
g = 10 ms −2 )
(a) cotα = 3 (b) tan α = 3
(c) sec α = 3 (d) cosec α = 3 (a) 127.5 kgs −1 (b) 187.5 kgs −1
(c) 185.5 kgs −1 (d) 137.5 kgs −1
16 A hockey player is moving northward and suddenly
turns westward with the same speed to avoid an 22 Two blocks are in contact on a frictionless table. One
opponent. The force that acts on the player is has mass m and the other 2m. A force F is applied on
(a) frictional force along westward 2m as shown in the figure. Now, the same force F is
(b) muscle force along southward applied from the right on m. In the two cases, the
(c) frictional force along south-west ratio of force of contact between the two blocks will
(d) muscle force along south-west be
2m
17 A car of mass m starts from rest and acquires a m
F
velocity along east v = v$i (v > 0 ) in two seconds.
Assuming the car moves with uniform acceleration,
the force exerted on the car is (a) same (b) 1: 2 (c) 2 : 1 (d) 1 : 3
mv
(a) eastward and is exerted by the car engine 23 A 4 kg block A is placed on the top of 8 kg block B
2 which rests on a smooth table.
mv
(b) eastward and is due to the friction on the tyres
2 A
exerted by the road B F
mv
(c) more than eastward exerted due to the engine and
2 A just slips on B when a force of 12 N is applied on
overcomes the friction of the road A. Then, the maximum horizontal force F applied on
mv B to make both A and B move together, is
(d) exerted by the engine
2 (a) 12 N (b) 24 N (c) 36 N (d) 48 N
Laws of Motion 213

24 Find the value of friction forces between the blocks A 29 A block of mass 1 kg is at rest relative to a smooth
−2
and B; and between B and ground. (Take, g =10 ms ) wedge moving leftwards with constant acceleration
µ = 0.1 a = 5 ms −2 .
A 5 kg
F = 80 N
B 15 kg µ = 0.6 1 kg
Ground
(a) 90 N, 5 N (b) 5 N, 90 N (c) 5 N, 75 N (d) 0 N, 80 N
25 A block of mass 5 kg is kept on a horizontal floor a
having coefficient of friction 0.09. Two mutually θ
perpendicular horizontal forces of 3 N and 4 N act
on this block. The acceleration of the block is Let N be the normal reaction between the block and
(Take, g = 10 ms −2 ) the wedge. Then, (Take, g = 10 ms −2 )
(a) zero (b) 0.1 ms −2 (c) 0.2 ms −2 (d) 0.3 ms −2
(a) N = 5 5 N (b) N = 15 N
26 Two masses A and B of 10 kg and 5 kg respectively 1
are connected with a string passing over a (c) tan θ = (d) tan θ = 2
3
frictionless pulley fixed at the corner of a table as
shown in figure. The coefficient of friction of A with 30 A block of mass m is kept on an inclined plane of a
the table is 0.2. The minimum mass of C that may lift moving down with acceleration of 2 ms −2 . What
be placed on A to prevent it from moving is equal to should be the coefficient of friction to let the block
move down with constant velocity relative to lift?
C

A
m
2 ms–2
B

30º
(a) 15 kg (b) 10 kg
(c) 5 kg (d) 20 kg 1 3
(a) µ = (b) µ = 0.4 (c) µ = 0.8 (d) µ =
27 In the figure, pulleys are smooth and 3 2
strings are massless, m1 = 1 kg and
1 31 Two blocks of mass 5 kg and 3 kg are attached to
m 2 = kg. To keep m 3 at rest, mass the ends of a string passing over a smooth pulley
3 fixed to the ceiling of an elevator. The elevator is
m 3 should be accelerated upwards. If the acceleration of the blocks
2 m3
(a) 1 kg (b) kg 9
3 is g, the acceleration of the elevator is
1 m1 32
(c) kg (d) 2 kg m2 g g g g
4 (a) (b) (c) (d)
3 4 8 6
28 If the coefficient of friction between A and B is µ,
32 A balloon of weight w is falling vertically downward
the maximum acceleration of the wedge A for which with a constant acceleration a (< g ). The magnitude
B will remain at rest with respect to the wedge is of the air resistance is
 a
B (a) w (b) w 1 + 
 g
A  a a
(c) w 1 −  (d) w
 g g
a
45º 33 A smooth inclined plane of length L, having an
inclination θ with horizontal is inside a lift which is
1 + µ  1 − µ  g moving down with retardation a. The time taken by
(a) µg (b) g   (c) g   (d) a block to slide down the inclined plane from rest
1 − µ  1 + µ  µ
will be
214 OBJECTIVE Physics Vol. 1

2L 2L weight, his upward acceleration would be


(a) (b)
a sin θ g sin θ (a)
g
(b)
g
2 4
2L 2L
(c) (d) (c) g (d) zero
(g − a ) sin θ (g + a ) sin θ
40 A 40 N block supported by two ropes. One rope is
34 A body of mass M at rest explodes into three pieces, horizontal and the other makes an angle of 30°
two of which of mass M /4 each are thrown off in with the ceiling. The tension in the rope attached
perpendicular directions with velocities of 3 ms −1 to the ceiling is approximately
and 4 ms −1, respectively. The third piece will be (a) 80 N (b) 40 N
thrown off with a velocity of 40
(c) 40 3 N (d) N
(a) 1.5 ms −1 (b) 2 ms −1 (c) 2.5 ms −1 (d) 3 ms −1 3
35 A wooden box of mass 8 kg slides down an inclined 41 Starting from rest, a body slides down a 45° inclined
plane of inclination 30° to the horizontal with a plane in twice the time, it takes to slide down the
constant acceleration of 0.4 ms −2 . What is the force of same distance in the absence of friction. The
friction between the box and inclined plane? coefficient of friction between the body and the
(Take, g = 10 ms −2 ) inclined plane is
(a) 36.8 N (b) 76.8 N (a) 0.2 (b) 0.25 (c) 0.75 (d) 0.5
(c) 65.6 N (d) None of these
42 A block of mass 0.1 kg is held against a wall
36 A mass of 3 kg descending vertically downwards applying a horizontal force of 5 N on the block. If
supports a mass of 2 kg by means of a light string the coefficient of friction between the block and the
passing over a pulley. At the end of 5 s, the string wall is 0.5, the magnitude of the frictional force
breaks . How much high from now the 2 kg mass
acting the block is
will go? (Take, g = 9.8 ms −2 )
(a) 2.5 N (b) 0.98 N (c) 4.9 N (d) 0.49 N
(a) 4.9 m (b) 9.8 m (c) 19.6 m (d) 2.45 m
43 A block of mass m is given an initial downward
37 The rear side of a truck is open and a box of mass velocity v 0 and left on an inclined plane (coefficient
20 kg is placed on the truck 4 m away from the open of friction = 0.6). The block will
. and g = 10 ms −2 ). The truck starts from
end (µ = 015
rest with an acceleration of 2 ms −2 on a straight
road. The box will fall off the truck when it is at a v0
distance from the starting point equal to
(a) 4 m (b) 8 m
(c) 16 m (d) 32 m 30º

38 Two blocks of masses m = 5 kg and F (a) continue of move down the plane with constant
M = 10 kg are connected by a string velocity v 0
passing over a pulley B as shown. A (b) accelerate downward
Another string connects the centre of (c) decelerate and come to rest
pulley B to the floor and passes over (d) first accelerate downward then decelerate
another pulley A as shown. An B 44 Pushing force making an angle θ to the horizontal is
upward force F is applied at the applied on a block of weight w placed on a horizontal
centre of pulley A. Both the pulleys table. If the angle of friction is φ, the magnitude of
are massless. The accelerations of m M force required to move the body is equal to
blocks m and M, if F is 300 N are
w cos φ w sin φ
(Take, g = 10 ms −2 ) (a) (b)
cos(θ − φ ) cos(θ + φ )
(a) 5 ms −2 , zero (b) zero, 5 ms −2
(c) zero, zero (d) 5 ms −2 , 5 ms −2 w tan φ w sin φ
(c) (d)
sin(θ − φ ) tan(θ − φ )
39 A man of mass m stands on a
platform of equal mass m and 45 In the arrangement shown in figure, there is a friction
pulls himself by two ropes force between the blocks of masses m and 2m kept on
passing over pulleys as shown a smooth horizontal surface. The mass of the
in figure. If he pulls each rope suspended block is m. The block of mass m is
with a force equal to half his stationary with respect to block of mass 2 m.
Laws of Motion 215

The minimum value of coefficient of friction 50 Two blocks are connected


between m and 2 m is over a massless pulley as
shown in figure. The mass A
of block A is 10 kg and the
m coefficient of kinetic
B
m friction is 0.2. Block A 30°
2m
slides down the incline at
constant speed. The mass of block B (in kg) is
Smooth (a) 5.4 (b) 3.3 (c) 4.2 (d) 6.8
1 1 1 1 51 A block of mass 5 kg resting on a horizontal surface
(a) (b) (c) (d)
2 2 4 3 is connected by a cord, passing over a light
46 Two masses m and M are attached with strings as frictionless pulley to a hanging block of mass 5 kg.
shown. For the system to be in equilibrium, we have The coefficient of kinetic friction between the block
and the surface is 0.5. Tension in the cord is
θ (Take, g = 9.8 ms −2 )
M
A

45º 45º 5 kg

m 5 kg B
2M 2m
(a) tan θ = 1 + (b) tan θ = 1 +
m M (a) 49 N (b) zero (c) 36.75 N (d) 12.75 N
M m
(c) tan θ = 1 + (d) tan θ = 1 + 52 A block of mass 3 kg is at rest on a rough inclined
2m 2M plane as shown in the figure. The magnitude of net
47 A man of mass m has fallen force exerted by the surface on the block will be
into a ditch of width d. 3 kg
Two of his friends are
slowly pulling him out
using a light rope and two 30º
fixed pulleys as shown in d
figure. Both the friends (a) 15 3 N (b) 15 N (c) 10 N (d) 30 N
exert force of equal magnitudes F.
53 Two blocks of masses M and m are connected to
When the man is at a depth h, the value of F is each other by a massless string and spring of force
mg
(a) d 2 + 4h 2 (b) hmg constant k as shown in the figure. The spring passes
4h over a frictionless pulley connected rigidly to the
mg
(c) dmg (d) h2 + d2 edge of a stationary block A. The coefficient of
2h
friction between block M and the plane horizontal
48 Two weights w 1 and w 2 are suspended from the ends
surface of A is µ. The block M slides over the
of a light string passing over a smooth fixed pulley.
If the pulley is pulled up with an acceleration g. The horizontal top surface of A and block m slides
tension in the string will be vertically downwards with the same speed. The
4w1 w 2 2w1 w 2 w1 − w 2 w1 w 2 mass M is equal to
(a) (b) (c) (d)
w1 + w 2 w1 + w 2 w1 + w 2 2 (w1 + w 2 )
M
49 A dynamometer D is attached to two bodies of masses
M = 6 kg and m = 4 kg. Forces F = 20 N and f = 10 N
are applied to the masses as shown. The dynamometer A
reads D
m
F M m f

(a) 10 N (b) 20 N (c) 6 N (d) 14 N m 2m


(a) (b) (c) mµ (d) µmg
µ µ
216 OBJECTIVE Physics Vol. 1

54 A block of mass m, lying on a rough horizontal Then, the normal reaction acting between the two
plane, is acted upon by a horizontal force P and blocks is
another force Q, inclined at an angle θ to the vertical F F
(a) F (b) (c) (d) 3 F
upwards. The block will remain in equilibrium, if 2 3
minimum coefficient of friction between it and the 58 A uniform rope of length l lies on a table. If the
surface is coefficient of friction is µ, then the maximum length
(P + Q sin θ ) (P cos θ + Q ) l1 of the hanging part of the rope which can
(a) (b)
(mg − Q cos θ ) (mg − Q sin θ ) overhang from the edge of the table without sliding
(P − Q cos θ ) (P sin θ − Q ) down is
(c) (d)
(mg + Q sin θ ) (mg − Q cos θ ) (a) l /µ (b) l /(µ + 1)
(c) µl / (µ + 1) (d) µl / (µ − 1)
55 In the figure shown, if coefficient of friction is µ,
59 Block A of mass m rests on the plank B of mass 3m
then m 2 will start moving upwards, if
which is free to slide on a frictionless horizontal
surface. The coefficient of friction between the
block and plank is 0.2. If a horizontal force of
magnitude 2mg is applied to the plank B, the
acceleration of A relative to the plank and relative to
m2 the ground respectively, are
m1 A
θ B 2 mg

m1 m1 g 2g 3g g 2g g
(a) > sin θ − µ cos θ (b) > sin θ + µ cos θ (a) 0, (b) 0, (c) , (d) ,
m2 m2 2 3 5 5 5 5
m m
(c) 1 > µ sin θ − cos θ (d) 1 > µ sin θ + cos θ
m2 m2 60 Two blocks of masses m and 2m are placed one over
the other as shown in figure.The coefficient of
56 A block of mass M rests on a rough horizontal
friction between m and 2m is µ and between 2m and
surface as shown. Coefficient of friction between the
µ
block and the surface is µ. A force F = Mg acting at ground is . If a horizontal force F is applied on
angle θ with the vertical side of the block pulls it. In 3
which of the following cases, the block can be pulled upper block and T is tension developed in string,
along the surface? then choose the incorrect alternative.
F
θ m
M
µ 2m

θ  µ
(a) tan θ ≥ µ (b) tan  ≥ µ  (a) If F = mg, T = 0 (b) If F = µmg, T = 0
2  3
µmg
 θ (c) If F = 2 µmg, T = (d) If F = 3 µmg, T = 0
(c) cos θ ≥ µ (d) cot   ≥ µ 3
 2
61 A block of mass m is placed on a
57 Two blocks A and B each of mass m are placed on a wedge of mass 2 m which rests
smooth horizontal surface. Two horizontal force F on a rough horizontal surface. m
2m
and 2 F are applied on the blocks A and B There is no friction between the
respectively as shown in figure. The block A does block and the wedge. The
not slide on block B. minimum coefficient of friction 45º

A B between the wedge and the


ground, so that the wedge does not move, is
F m m 2F
(a) 0.1 (b) 0.2
30º (c) 0.3 (d) 0.4
Laws of Motion 217

62 If the coefficient of friction between all surfaces is 16 mg 25 mg


(a) (b)
0.4, then find the minimum force F to have 25 39
39 mg
equilibrium of the system. (Take, g = 10 ms −2 ) (c) (d) None of these
21
64 A pendulum of mass m hangs from a support fixed
to a trolley. The direction of the string (i.e. angle
θ) when the trolley rolls up a plane of inclination
α with acceleration a is
25 kg 15 kg F
Wall a

(a) 62.5 N (b) 150 N (c) 135 N (d) 50 N θ

63 A sphere of mass m is held between two smooth


3
inclined walls. For sin 37° = , the normal reaction
5
of the wall (2) is equal to α

(2) (a) 0 (b) tan−1 α


 a + g sin α  a
(1) (c) tan−1   (d) tan−1
37º
 g cos α  g
37º

(B) Medical entrance special format questions


Assertion and reason 4 Assertion In the system of µ2 m F
Directions (Q. Nos. 1-6) These questions consists of two two blocks of equal masses
statements each printed as Assertion and Reason. While as shown, the coefficient of m µ1
answering these questions you are required to choose any friction between the blocks
one of the following four responses (µ 2 ) is less than coefficient
(a) If both Assertion and Reason are correct and Reason is the of friction (µ 1 ) between lower block and ground.
correct explanation of Assertion. For all values of force F applied on upper block,
(b) If both Assertion and Reason are correct but Reason is not lower block remains at rest.
the correct explanation of Assertion.
(c) If Assertion is true but Reason is false.
Reason Frictional force on lower block due to upper
(d) If Assertion is false but Reason is true. block is not sufficient to overcome the frictional force
on lower block due to ground.
1 Assertion When a person walks on a rough surface,
the net force exerted by surface on the person is in 5 Assertion In the figure
the direction of his motion. shown, block of mass m is
stationary with respect to lift.
Reason It is the force exerted by the road on the Force of friction acting on the m
person that causes the motion. a
block is greater than mg sinθ.
2 Assertion A body of mass 10 kg is placed on a Reason If lift moves with
rough inclined surface ( µ = 0.7). The surface is constant velocity, then force of θ
inclined to horizontal at angle 30°. Acceleration of friction is equal to mg sinθ.
the body down the plane will be zero.
6 Assertion Block A is resting on one corner of a box
Reason Force of friction is zero.
as shown in figure. Acceleration of box is
3 Assertion Static friction acting on a body is always (2$i + 2$j ) ms −2 . Let N1 is the normal reaction on
greater than the kinetic friction acting on this body.
block from vertical wall and N 2 from ground to box.
Reason Coefficient of static friction is more than the N 1
coefficient of kinetic friction. Then, 1 = . (Neglect friction)
N2 6
218 OBJECTIVE Physics Vol. 1

Reason N1 = 0, if lift is stationary.


Match the columns
1 In the diagram shown in figure, match the following
columns (Take, g = 10 ms −2 )
y
A 20√2 N
x

45º
Statement based questions
4 kg
1 Which of the following statement (s) is/are correct?
(a) The weighing machine measures the weight of a body.
µs = 0.8, µk = 0.6
(b) During free fall of a person one feels weightlessness
because his weight becomes zero.
(c) During free fall, the person falls with an acceleration of g. Column I Column II
(d) All of the above
(A) Normal reaction (p) 12 SI unit
2 Which of the following statement (s) is/are incorrect?
(a) Static friction is self-adjusting while kinetic friction is (B) Force of friction (q) 20 SI unit
constant. (C) Acceleration of block (r) zero
(b) Friction always opposes the motion of two bodies.
(s) 2 SI unit
(c) Without friction, one can move on a smooth surface.
(d) Some mechanical energy is lost in the form of heat due Codes
to air friction. A B C A B C
3 Which of the following statement (s) is/are correct? (a) p q r (b) q p r
I. A string has a mass m. If it is accelerated, (c) q p s (d) p s r
tension is non-uniform and if it is not accelerated, 2 In the diagram shown in figure, match the following
tension is uniform. columns. (Take, g = 10 ms −2 )
II. Tension force is an electromagnetic force.
(a) Only I (b) Only II F2 = 18 N
(c) Both I and II (d) None of these 1 kg
2 kg
4 In the diagram shown in figure,
string is massless and pulley is 3 kg Smooth
smooth, then which of the
following statement (s) is/are
correct? 2 kg F1 = 60 N θ = 30º
10
I. Net force on 1 kg block is N. 1 kg
3 Column I Column II
II. Net force on both the blocks will be same.
(A) Acceleration of 2 kg (p) 8 SI unit
(a) Only I (b) Only II
block
(c) Both I and II (d) None of these
(B) Net force on 3 kg block (q) 25 SI unit
5 Two forces are acting on a rope lying on a smooth
table as shown in figure. (C) Normal reaction between (r) 2 SI unit
B A 2 kg and 1 kg
F 2F (D) Normal reaction between (s) 45 N
Which of the following statement (s) is/are correct? 3 kg and 2 kg
I. In moving from A to B, tension on string decreases (t) None
from 2F to F.
Codes
II. Situation will becomes indeterminant, if we take A B C D A B C D
it a massless string. (a) q r t t (b) p s r t
(a) Only I (b) Only II (c) s s p t (d) r t q t
(c) Both I and II (d) None of these
Laws of Motion 219

3 In the diagram shown in figure. Match the following 4 In the diagram shown in figure, all pulleys are smooth
columns. and massless and strings are light. Match the following
µ = 0.4 columns. F = 80 N
10 ms–1
1 kg
20 ms–1
2 kg

µ = 0.6

Column I Column II 1 kg 3kg

(A) Absolute acceleration of (p) 11 ms−2 2 kg 4 kg


1 kg block
Column I Column II
(B) Absolute acceleration of (q) 6 ms−2
2 kg block (A) 1 kg block (p) will remain stationary
(B) 2 kg block (q) will move down
(C) Relative acceleration (r) 17 ms−2
between the two (C) 3 kg block (r) will move up
(s) None (D) 4 kg block (s) 5 ms−2
(t) 10 ms−2
Codes
A B C A B C Codes
(a) p q r (b) s p s A B C D A B C D
(c) r q p (d) p p r (a) r,t p q q,s (b) p r,s q,t s
(c) p,s r q,s t (d) p t r s

(C) Medical entrances’ gallery


Collection of questions asked in NEET & various medical entrance exams
1 Two bodies of masses 4 kg and 6 kg are tied to the (a) 1.25 m/s 2 (b) 1.50 m/s 2
ends of a massless string. The string passes over a (c) 1.66 m/s 2 (d) 1.00 m/s 2
pulley which is frictionless (see figure). The 3 A truck is stationary and has a bob suspended by a
acceleration of the system in terms of acceleration light string, in a frame attached to the truck. The
due to gravity g is [NEET 2020] truck, suddenly moves to the right with an
acceleration of a. The pendulum will tilt
[NEET (Odisha) 2019]
(a) to the left and the angle of inclination of the pendulum
4 kg  g
with the vertical is sin−1  
6 kg a 
(b) to the left and angle of inclination of the pendulum
(a) g/2 (b) g/5 (c) g/10 (d) g
a 
2 Calculate the acceleration of the block and trolley with the vertical is tan−1  
 g
system shown in the figure. The coefficient of
kinetic friction between the trolley and the surface (c) to the left and angle of inclination of the pendulum
is 0.05 (g = 10 m/s 2 , mass of the string is negligible a 
with the vertical is sin−1  
and no other friction exists). [NEET 2020]  g
Trolley (d) to the left and angle of inclination of the pendulum
 g
10 kg with the vertical is tan−1  
a 
4 A body of mass m is kept on a rough horizontal
surface (coefficient of friction = µ). Horizontal force is
2 kg
applied on the body, but it does not move.
Block
220 OBJECTIVE Physics Vol. 1

The resultant of normal reaction and the frictional A


m
force acting on the object is given F, where F is
[NEET (Odisha) 2019]
(a) | F | = mg + µ mg (b) | F | = µmg a
q
(c) | F | ≤ mg 1 + µ 2 (d) | F | = mg C B

5 A particle moving with velocity v is acted by three g


(a) a = g cos θ (b) a =
forces shown by the vector triangle PQR. sin θ ]
The velocity of the particle will g
[NEET 2019] (c) a = (d) a = g tan θ
P cosec θ
10 Which one of the following statements is incorrect?
(a) Frictional force opposes the relative motion. [NEET 2018]
(b) Limiting value of static friction is directly proportional
to normal reaction.
R Q
(c) Rolling friction is smaller than sliding friction.
(a) decrease (d) Coefficient of sliding friction has dimensions of length.
(b) remain constant
11 In the figure, block A and B of
(c) change according to the smallest force QR
masses 2m and m are connected with
(d) increase
a string and system is hanged
6 Assertion A glass ball is dropped on concrete floor vertically with the help of a spring.
can easily get broken compared, if it is dropped on Spring has negligible mass. Find out
wooden floor. magnitude of acceleration of masses B m 2m A
Reason On concrete floor, glass ball will take less 2m and m just after the instant when
time to come to rest. [AIIMS 2019] the string of mass m is cut [AIIMS 2018]
(a) Both Assertion and Reason are true and Reason is the g g g g
correct explanation of Assertion. (a) g, g (b) g, (c) , g (d) ,
2 2 2 2
(b) Both Assertion and Reason are true, but Reason is not
the correct explanation of Assertion.
9
12 In the figure, mass of a ball is times mass of the
(c) Assertion is true, but Reason is false. 5
(d) Both Assertion and Reason are false. rod. Length of rod is 1 m. The level of ball is same
7 A gun applies a force F on a bullet which is given as rod level. Find out time taken by the ball to reach
at upper end of rod. [AIIMS 2018]
by F = (100 − 0.5 × 10 5 t ) N. The bullet emerges out
with speed 400 m/s. Then, find out the impulse
exerted till force on bullet becomes zero. [AIIMS 2019]
(a) 0.2 N-s (b) 0.3 N-s
(c) 0.1 N-s (d) 0.4 N-s
8 Assertion Even though net external force on a body
is zero, momentum need not to conserved. Rod
Reason The internal interaction between particles of Ball
a body cancels out momentum of each other.
[AIIMS 2019] (a) 1.4 s (b) 2.45 s
(a) Both Assertion and Reason are true and Reason is the (c) 3.25 s (d) 5 s
correct explanation of Assertion. 13 Assertion Angle of repose is equal to angle of
(b) Both Assertion and Reason are true, but Reason is not limiting friction.
the correct explanation of Assertion. Reason When a body is just at the point of motion,
(c) Assertion is true, but Reason is false. the force of friction of this stage is called as limiting
(d) Both Assertion and Reason are false. friction. [AIIMS 2018]
9 A block of mass m is placed on a smooth inclined (a) Both Assertion and Reason are correct and Reason is
wedge ABC of inclination θ as shown in the figure. the correct explanation of Assertion.
The wedge is given an acceleration a towards the (b) Both Assertion and Reason are correct, but Reason is not
the correct explanation of Assertion.
right. The relation between a and θ for the block to (c) Assertion is correct, but Reason is incorrect.
remain stationary on the wedge is [NEET 2018] (d) Assertion is incorrect, but Reason is correct.
Laws of Motion 221

14 A wooden wedge of mass M and The tension T1, T 2 and T 3 will be


[AIIMS 2017]
inclination angle α rest on a F
m T1 T2 T3
F m m m m
smooth floor. A block of mass m is
kept on wedge. A force F is α
1 3 1
(a) T1 = F , T2 = F , T3 = F
applied on the wedge as shown in M 4 2 4
the figure, such that block remains 1 1 1
(b) T1 = F , T2 = F , T3 = F
stationary with respect to wedge. The magnitude of 4 2 2
force F is [AIIMS 2018] 3 1 1
(c) T1 = F , T2 = F , T3 = F
(a) (M + m ) g tan α (b) g tan α 4 2 4
(c) mg cos α (d) (M + m )g cosec α 3 1 1
(d) T1 = F , T2 = F , T3 = F
15 A piece of ice slides down a rough inclined plane at 4 2 2
45° inclination in twice the time that it takes to slide 21 A body of mass 5 × 10 − 3 kg is launched upon a rough
down an identical but frictionless inclined plane.
inclined plane making an angle of 30° with the
What is the coefficient of friction between ice and
horizontal. Obtain the coefficient of friction between
incline? [AIIMS 2018]
the body and the plane, if the time of ascent is half
3 4 3 7
(a) (b) (c) (d) of the time of descent. [AIIMS 2017]
7 cotθ 7 cotθ 4 cotθ 9 cotθ
(a) 0.346 (b) 0.921 (c) 1.926 (d) 2.912
16 A body of mass 5 kg is suspended by a spring 22 A long block A of mass M is at rest on a smooth
balance on an inclined plane as shown in figure. horizontal surface. A small block B of mass M /2 is
placed on A at one end and projected along A with
some velocity v. The coefficient of friction between
m the block is µ. Then, the accelerations of blocks A
30°
and B before reaching a common velocity will be
So, force applied on spring balance is [AIIMS 2018] respectively [JIPMER 2017]
(a) 50 N (b) 25 N (c) 500 N (d) 10 N B
17 A force of 10N acts on a body of mass 0.5 kg for A
0.25s starting from rest. What is its impulse? µg µg
[JIPMER 2018] (a) (towards right), (towards left)
(a) 0.25 N-s (b) 2.5 N-s (c) 0.5 N-s (d) 0.75 N-s 2 2
(b) µg (towards right), µg (towards left)
18 A mass M is hung with a light inextensible string as
µg
shown in the figure. Find the tension of the (c) (towards right), µg (towards left)
2
horizontal string. [JIPMER 2018] µg
B (d) µg (towards right), (towards left)
2
30°
A P
T1 23 A box of mass 8 kg is placed on a rough inclined
plane of inclination 30°. Its downward motion can
be prevented by applying a horizontal force F, then
M value of F for which friction between the block and
Mg
the incline surface is minimum, is [JIPMER 2017]
80 40
(a) 2 Mg (b) 3 Mg (c) 2 Mg (d) 3 Mg (a) N (b) 40 3 N (c) N (d) 80 3 N
3 3
19 Two blocks A and B of masses 3m and m
24 Two masses 10 kg and 20 kg respectively are
respectively are connected by a massless
and inextensible string. The whole system connected by a massless spring as shown in figure. A
is suspended by a massless spring as shown A 3m force of 200 N acts on the 20 kg mass. At the instant
in figure. The magnitudes of acceleration of shown in figure, the 10 kg mass has acceleration of
B m
A and B immediately after the string is cut, 12 m/s 2 . The value of acceleration of 20 kg mass is
are respectively [NEET 2017] [JIPMER 2017]
g g g g 10 kg F F 20 kg
(a) g, (b) , g (c) g, g (d) , 200 N
3 3 3 3
20 Four blocks of same mass connected by strings are (a) 4 m / s2 (b) 10 m / s2
pulled by a force F on a smooth horizontal surface as
(c) 20 m / s2 (d) 30 m / s2
shown in figure.
222 OBJECTIVE Physics Vol. 1

25 The upper half of an inclined plane of inclination θ


A B
is perfectly smooth while the lower half rough. A C
block starting from rest at the top of the plane will
again come to rest at the bottom, if the coefficient of (a) 2 N (b) 6 N (c) 8 N (d) 18 N
friction between the block and the lower half of the 31 A block A of mass m1 rests on a horizontal table. A
plane is given by [JIPMER 2017, NEET 2013] light string connected to it passes over a frictionless
(a) µ = 2 tan θ (b) µ = tan θ pulley at the edge of table and from its other end,
(c) µ = 2 / (tan θ ) (d) µ = 1 / tan θ another block B of mass m 2 is suspended. The
26 A rigid ball of mass m strikes a rigid
m coefficient of kinetic friction between the block and
wall at 60° and gets reflected without
v the table is µ k . When the block A is sliding on the
loss of speed as shown in the figure. 60°
table, the tension in the string is [CBSE AIPMT 2015]
(m 2 + µ km1 )g (m 2 − µ km 1 ) g
The value of impulse imparted by the 60° (a) (b)
wall on the ball will be [NEET 2016] (m1 + m 2 ) (m 1 + m 2 )
v
(a) mv (b) 2 mv m1m 2 (1 + µ k ) g m1m 2 (1 − µ k ) g
(c) mv /2 (d) mv /3 (c) (d)
(m 1 + m 2 ) (m 1 + m 2 )
27 A body of mass 40 kg resting on rough horizontal
surface is subjected to a force P which is just enough 32 A balloon with mass m is descending down with an
to start the motion of the body. If µ s = 0.5, acceleration a (where, a < g). How much mass should
µ k = 0.4, g = 10 ms −2 and the force P is continuously be removed from it, so that it starts moving up with
applied on the body, then the acceleration of the an acceleration a? [CBSE AIPMT 2014]
body is [AIIMS 2015]
(a) zero (b) 1 ms −2 (c) 2 ms −2 (d) 24 ms −2
28 A balloon of mass 10 kg is raising up with an B a
−2
acceleration of 20 ms . If a mass of 4 kg is removed mg
from the balloon, its acceleration becomes (Take, 2 ma 2 ma ma ma
g = 10 ms −2 ) [EAMCET 2015]
(a)
g +a
(b)
g −a
(c)
g +a
(d)
g −a
(a) 40 ms−2 (b) 20 ms−2
33 A bullet moving with a velocity of 30 2 ms −1 is
(c) 30 ms−2 (d) 12 ms−2
fired into a fixed target. It penetrated into the target
29 Block B lying on a table weighs w. The coefficient of to the extent of s metre. If the same bullet is fired
static friction between the block and the table is µ. s
Assume that the cord between B and the knot is into a target of thickness metres and of the same
2
horizontal. The maximum weight of the block A for material with the same velocity, then the bullet
which the system will be stationary is [WB JEE 2015] comes out of the target with velocity, is
[EAMCET 2014]
(a) 20 ms −1 (b) 30 ms −1
Knot
θ
(c) 20 2 ms −1 (d) 10 2 ms −1
B m2
34 A system consists of three m3
P
masses m1, m 2 and m 3
connected by a string
A passing over a pulley P.
The mass m1 hangs freely
w tan θ m1
(a) (b) µw tan θ and m 2 and m 3 are on a
µ rough horizontal table (the coefficient of friction
(c) µw 1 + tan2 θ (d) µw sin θ = µ). The pulley is frictionless and of negligible
mass. The downward acceleration of mass m1 is
30 Three blocks A, B and C of masses 4 kg, 2 kg and (Assume, m1 = m 2 = m 3 = m) [CBSE AIPMT 2014]
1 kg respectively are in contact on a frictionless g (1 − gµ ) 2gµ
surface as shown. If a force of 14 N is applied on the (a) (b)
9 3
4 kg block, then the contact force between A and B g (1 − 2 µ ) g (1 − 2 µ )
is [CBSE AIPMT 2015]
(c) (d)
3 2
Laws of Motion 223

35 Three identical blocks of masses m = 2 kg are drawn accelerations of the two blocks (in ms −2 ) are
by a force 10.2 N on a frictionless surface. What is (Take, g = 10 ms −2 ) [EAMCET 2013]
the tension (in newton) in the string between the
µ = 0.2 3 kg 20 N
blocks B and C ?
10 kg
C B A F
[UK PMT 2014] 13 14
(a) , 0.6 (b) ,3
(a) 9.2 (b) 8 (c) 3.4 (d) 9.8 4 4
13 14
36 A wooden block of mass 8 kg slides down an (c) ,3 (d) , 0.6
4 3
inclined plane of inclination 30° to the horizontal
42 A 60 kg person is weighed by a balance as 54 kg in
with constant acceleration 0.4 ms −2 . The force of
friction between the block and inclined plane is a lift which is accelerated downwards. The
acceleration of the lift is [Kerala CEE 2013]
(Take, g = 10 ms −2 ) [MHT CET 2014]
(a) 1.26 ms−2 (b) 1.76 ms−2
(a) 12.2 N (b) 24.4 N
(c) 1.98 ms−2 (d) 0.98 ms−2
(c) 36.8 N (d) 48.8 N
(e) None of these
37 To determine the coefficient of friction between a
43 A lift starting from rest with a constant upward
rough surface and a block, the surface is kept acceleration moves 15 . m in 0.4 s. If a person standing
inclined at 45° and the block is released from rest. in the lift holds a packet of 2 kg by a string, then the
The block takes a time t in moving a distance d. The tension in the string due to motion is
rough surface is then replaced by a smooth surface [UP CPMT 2013]
and the same experiment is repeated. The block now (a) 5.89 N (b) 57.1 N
takes a time t /2 in moving down the same distance (c) 67.1 N (d) None of these
d. The coefficient of friction is [WB JEE 2014] 44 A body of mass 0.25 kg is projected with muzzle
(a) 3/4 (b) 5/4 velocity 100 ms −1 from a tank of mass 100 kg. What
(c) 1/2 (d) 1/ 2 is the recoil velocity of the tank? [AIIMS 2012]

38 A body of mass m is placed on a rough surface with (a) 5 ms −1 (b) 25 ms −1 (c) 0.5 ms −1 (d) 0.25 ms −1
coefficient of friction µ inclined at θ. If the mass is 45 A rocket with a lift-off mass 3.5 × 10 4 kg is blast
in equilibrium, then [KCET 2014] upward with an initial acceleration of 10 ms −2 .
 1 Then, the initial thrust of the blast is [AIIMS 2012]
(a) θ = tan−1 µ (b) θ = tan−1  
µ  (a) 1.75 × 105 N (b) 3.5 × 105 N
m µ (c) 7 × 105 N (d) 14 × 105 N
(c) θ = tan−1 (d) θ = tan−1
µ m
46 A marble block of mass 2 kg lying on ice when given
a velocity of 6 ms −1 is stopped by friction in 10 s.
F v
39 Three blocks with masses m, 2m and 3m
m
are connected by strings as shown in the Then, the coefficient of friction is [AIIMS 2012]
figure. After an upward force F is applied 2m
(a) 0.01 (b) 0.02 (c) 0.03 (d) 0.06
on block m, the masses move upward at 47 A 40 kg slab rests on a frictionless floor. A 10 kg
constant speed v. What is the net force on 3m block rests on the top of the slab as shown in figure.
the block of mass 2m ? (g is the acceleration Block
due to gravity) [NEET 2013] 100 N
10 kg
(a) Zero (b) 2 mg (c) 3 mg (d) 6 mg
40 A balloon starting from rest ascends vertically with 40 kg Slab
uniform acceleration to a height of 100 m in 10 s.
The force on the bottom of the balloon by a mass of The static coefficient of friction between the block
50 kg is (Take, g = 10 ms −2 ) [EAMCET 2013] and the slab is 0.60 while the kinetic coefficient is
(a) 100 N (b) 300 N (c) 600 N (d) 400 N 0.40. The 10 kg block is acted upon by a horizontal
41 A 3 kg block is placed over a 10 kg block and both force of 100 N. If g = 9.8 ms −2 , the resulting
are placed on a smooth horizontal surface. The acceleration of the slab will be [BHU 2012]
coefficient of friction between the blocks is 0.2. If a (a) 61 ms −2 (b) 152 ms −2
horizontal force of 20 N is applied to 3 kg block, (c) 147 ms −2 (d) 0.98 ms −2
224 OBJECTIVE Physics Vol. 1

48 An inclined plane of height h and length l have the 54 A monkey of mass 25 kg is holding a vertical rope.
angle of inclination θ. The time taken by a body to The rope does not break, if a body of mass 30 kg is
come from the top to the bottom of this inclined suspended from it, but the rope breaks, if the mass of
plane will be [BCECE 2012] the body suspended with the rope exceeds 30 kg.
2h 1 2h What will be the maximum acceleration with which
(a) sin θ (b)
g sin θ g the monkey can climb up along the rope? (Take,
2h 2l g = 10 ms −2 ) [JCECE 2012]
(c) (d)
g g (a) 2 ms−2 (b) 25 ms−2 (c) 3 ms−2 (d) 4 ms−2

49 If a coin is dropped in a lift it takes t 1time to reach 55 A body of weight 50 N placed on a horizontal
the floor and takes t 2 time when lift is moving up surface is just moved by a force of 282 N. The
with constant acceleration, then which one of the frictional force and normal reaction are [JCECE 2012]
following relation is correct? [BCECE (Mains) 2012] 28.2 N
(a) t1 = t 2 (b) t1 > t 2 45°
(c) t 2 > t1 (d) t1 >> t 2
50 A 60 kg mass is pushed with a enough force to start
it moving and the same force is continued to act 50 N
afterwards. If the coefficient of static friction and
(a) 2 N, 3N (b) 5 N, 6 N
sliding friction are 0.5 and 0.4 respectively, then the
(c) 10 N, 15 N (d) 20 N, 30 N
acceleration of the body will be [BCECE (Mains) 2012]
(a) 1ms−2 (b) 3.9 ms−2 56 Diwali rocket is ejecting 50 g of gases/s at a velocity
(c) 4.9 ms −2
(d) 6 ms −2 of 400 ms −1. The accelerating force on the rocket
will be [RPMT 2011]
51 A body is hanging from a rigid support by an (a) 22 dyne (b) 20 N (c) 20 dyne (d) 100 N
inextensible string of length l . It is struck 57 A rocket of mass 1000 kg is to be projected
inelastically by an identical body of mass m with vertically upwards. The gases are exhausted
horizontal velocity v = 2 gl , the tension in the vertically downwards with velocity 100 ms −1 with
string increases just after striking by [AFMC 2012] respect to the rocket. What is the minimum rate
(a) mg (b) 3 mg of burning of fuel, so as to just lift the rocket
(c) 2 mg (d) None of these upwards against the gravitational attraction?
52 A block A of mass 100 kg rests on another block B of (Take,g = 10 ms −2 ) [AMU 2011]
mass 200 kg and is tied to a wall as shown in the (a) 50 kg s−1 (b) 100 kg s−1
figure. The coefficient of friction between A and B is
(c) 200 kg s−1 (d) 400 kg s−1
0.2 and that between B and the ground is 0.3. The
minimum force F required to move the block B is 58 In a non-inertial frame, the second law of motion is
(Take, g = 10 ms −2 ) [AFMC 2012] written as [DUMET 2011]
(a) F = ma (b) F = ma + Fp (c) F = ma − Fp (d) F = 2 ma
A where, Fp is pseudo force while a is the acceleration
F of the body relative to the non-inertial frame.
B
59 A man of mass 60 kg is riding in a lift. The weight
(a) 900 N (b) 200 N of the man when the lift is accelerating upwards and
(c) 1100 N (d) 700 N downwards at 2 ms −2 are respectively (Take,
53 A machine gun fires a bullet of mass 40 g with a g = 10 ms −2 ) [AMU 2011]
velocity 1200 ms −1. The man holding it can exert a (a) 720 N and 480 N (b) 480 N and 720 N
maximum force of 144 N on the gun. How many (c) 600 N and 600 N (d) None of these
bullets can be fired per second at the most? 60 An object is moving on a plane surface with uniform
[AFMC 2012]
velocity 10 ms −1 in presence of a force 10 N. The
(a) Only one
frictional force between the object and the surface is
(b) Three [DUMET 2011]
(c) Can fire any number of bullets
(a) 1 N (b) − 10 N (c) 10 N (d) 100 N
(d) 144 × 48
ANSWERS
l CHECK POINT 5.1
1. (c) 2. (d) 3. (a) 4. (a) 5. (b) 6. (c) 7. (a) 8. (b) 9. (a) 10. (a)
11. (a) 12. (a) 13. (b) 14. (a) 15. (c) 16. (d) 17. (b)

l CHECK POINT 5.2


1. (d) 2. (d) 3. (b) 4. (c) 5. (c) 6. (d) 7. (c) 8. (d) 9. (a) 10. (d)
11. (c) 12. (a) 13. (a) 14. (a) 15. (c)

l CHECK POINT 5.3


1. (c) 2. (c) 3. (d) 4. (d) 5 (d) 6. (d) 7. (b) 8. (b) 9. (c) 10. (b)
11. (d) 12. (b) 13. (d) 14. (c) 15 (b) 16. (a) 17. (b) 18. (b) 19. (c) 20. (c)

l CHECK POINT 5.4


1. (d) 2. (a) 3. (c) 4. (d) 5. (c) 6. (a) 7. (d) 8. (a) 9. (a) 10. (b)
11. (a) 12. (a) 13. (a) 14. (d) 15. (d) 16. (a) 17. (a) 18. (a)

(A) Taking it together


1. (b) 2. (d) 3. (b) 4. (c) 5. (c) 6. (c) 7. (c) 8. (d) 9. (c) 10. (b)
11. (c) 12. (a) 13. (d) 14. (a) 15. (a) 16. (d) 17. (b) 18. (a) 19. (a) 20. (b)
21. (b) 22. (b) 23. (b) 24. (d) 25. (b) 26. (a) 27. (a) 28. (b) 29. (a) 30. (a)
31. (c) 32. (c) 33. (d) 34. (c) 35. (a) 36. (a) 37. (a) 38. (a) 39. (d) 40. (a)
41. (c) 42. (b) 43. (c) 44. (b) 45. (c) 46. (a) 47. (a) 48. (a) 49. (d) 50. (b)
51. (c) 52. (d) 53. (a) 54. (a) 55. (b) 56. (d) 57. (d) 58. (c) 59. (d) 60. (c)
61. (b) 62. (a) 63. (d) 64. (c)

(B) Medical entrance special format questions


l Assertion and reason
1. (a) 2. (c) 3. (a) 4. (a) 5. (b) 6. (b)

l Statement based questions


1. (d) 2. (b) 3. (b) 4. (a) 5. (c)
l Match the columns
1. (c) 2. (d) 3. (b) 4. (a)

(C) Medical entrances’ gallery


1. (b) 2. (a) 3. (b) 4. (c) 5. (b) 6. (a) 7. (c) 8. (d) 9. (d) 10. (d)
11. (c) 12. (a) 13. (a) 14. (a) 15. (c) 16. (b) 17. (b) 18. (b) 19. (b) 20. (c)
21. (a) 22. (c) 23. (a) 24. (a) 25. (a) 26. (a) 27. (b) 28. (a) 29. (b) 30. (b)
31. (c) 32. (a) 33. (b) 34. (c) 35. (c) 36. (c) 37. (a) 38. (a) 39. (a) 40. (c)
41. (d) 42. (d) 43. (b) 44. (d) 45. (c) 46. (d) 47. (d) 48. (b) 49. (b) 50. (a)
51. (c) 52. (c) 53. (b) 54. (a) 55. (d) 56. (b) 57. (b) 58. (c) 59. (a) 60. (b)
Hints & Explanations
l CHECK POINT 5.1 v 2 = 2 × 9.8 × 9.8
2 (d) Given, mass, m = 6 kg ⇒ v = 9.8 2 m/s
Velocity, ∆v = v 2 − v1 = 5 − 3 = 2 ms −1 Again, v = u 2 − 2gh
2

∴ Momentum, ∆p = m∆v = 6 × 2 = 12 N-s


0 = u 2 − 2 × 9.8 × 4.9
3 (a) Given, p = 2 + 3t 2
⇒ u 2 = (9.8)2 ⇒ u = 9.8 m/s
Differentiate w.r.t. t, we get
dp Impulse = mv
[ − (−u )]
= 0 + 3 × 2t = 6t = 1[9.8 2 + 9.8] = 9.8 ( 2 + 1)
dt
dp = 9.8 × 2.4 = 23.52 N-s
If t = 3 s, then = 6 × 3 = 18 N
dt Impulse 23.52
Average force = = = 235.2 N
∴ Force acting on the particle = 18 N Time 0.1
5 (b) Given, F = 72 dyne ⇒ F = 72 × 10 −5 N, 12 (a) Here, mass, m = 5 kg
−3 Change in velocity, ∆v = v f − vi = [(10 − 2)$i + (6 − 6)$j] = 8$i
θ = 60 °, m = 9g = 9 × 10 kg
F ′ = ma Change in momentum = m∆v = 5 [8$i] = 40 $i kg-ms −1
In this case, F ′ = F cos 60 °
13 (b) Impulse is defined as rate of change of momentum. For
F ′ F cos 60 °
⇒ a= = change in momentum to be minimum,
m 9 × 10 −3 d
(20t 2 − 40t ) = 0
72 × 10 −5 × cos 60 ° 1 dt
= = 8 × 10 −2 × 40t − 40 = 0 ⇒ t = 1s
9 × 10 −3 2
∆v 0.5 [2 − (− 2)]
= 4 × 10 −2 ms −1 = 4 cms −1 14 (a) Fav = ma av = m = = 2000 N
∆t 10 −3
6 (c) Given, m = 60 kg and a = 1ms −2 15 (c) From law of conservation of momentum, pi = p f
∴ Net force, Fnet = Mass × Acceleration = 60 × 1 = 60 N
and initial momentum, pi = mu = m (0 ) = 0
. ms −1 and ∆t = 25 s
7 (a) Given, m = 3 kg, ∆v = 3.5 − 2 = 15 ∴ p f should also be zero.
∆v 1.5 Hence, other piece will move in negative x-direction.
Force, F = ma = m = 3× = 0.18 N, in the direction of
∆t 25
16 (d) From the law of conservation of momentum,
motion
total initial momentum = total final momentum
8 (b) Given, m = 5 kg, F1 = 8 N and F2 = 6 N
⇒ m1u1 + m 2u 2 = mv
1 1 + m 2v 2
Resultant force, F = 82 + 62 = 10 N ∴ 0.1 × 0 + 50 × 0 = 0.1 × 100 + 50 (−v 2 )
F 10 ⇒ 0 = 10 − 50v 2
∴a = = = 2 ms −2
m 5 10
∴ v2 = = 0.2 ms −1
 6 50
and θ = cos −1  = cos −1 (0.6), from 6N
 10  17 (b) From the given figure,
10 (a) Given, F = 50 N, m = 20 kg and v = 15 ms −1 Initial velocity, vi = (− 20 sin 30 ° $i − 20 cos 30 ° $j ) m/s
Impulse = F∆t = mv Final velocity, v = (− 20 sin 30 ° $i + 20 cos 30 ° $j ) m/s
f
mv 20 × 15
∴Time, ∆t = = =6s Change in velocity, ∆v = v f − vi
F 50
∆v = (− 20 sin 30 ° $i + 20 cos 30 ° $j )
11 (a)
− (− 20 sin 30 ° $i − 20 cos 30 ° $j )
9.8 m
4.9 m ∆v = 2 × 20 cos 30 ° j
$
3$
∆v = 2 × 20 × j ⇒ ∆v = 20 3$j
v 2 = u 2 + 2gh 2
v 2 = 0 + 2 × 9.8 × 9.8 Magnitude, | ∆v| = 20 3 m/s
Laws of Motion 227

CHECK POINT 5.2 mg


l
⇒ T=
2 cos θ
2 (d) T3 sin 60 ° = T2 sin 30 °
T2
∴ T3 =
3 T θ T
θ
60º T3 cos 60° 30º
T2 T2 cos 30°
30º T3
60º
mg
T2 sin 30° T3 sin 60°
T1 For the string to be horizontal,
θ = 90 °
w = 100 N mg
T=
Now, T2 cos 30 ° + T3 cos 60 ° = T1 = 100 2 cos 90 °
3T2 T2 1 ⇒ T =∞
or + × = 100
2 3 2 7 (c) As shown in figure,
8T2 T1 sin 30°
or = 100 C F
4 3 T1 T2 sin 30°
or T2 = 50 3 N T2
30° 30°
3 (b) 2T cos 60 ° = w T1 cos 30° 90°
B
90° T2 cos 30°

T T = 20 N 10 N
60º A
60º
T1 cos 30 ° = T2 cos 30 °
∴ T1 = T2 = T (Let)
Again, T1 sin 30 ° + T2 sin 30 ° = 10 ⇒ 2T sin 30 ° = 10
w
1
⇒ 2T ⋅ = 10 ⇒ T = 10 N
or w =T 2
or wmax = Tmax = 20 N Thus, the tension in section BC and BF are 10 N and 10 N,
4 (c) Equilibrium of m : T = mg … (i) respectively.

Equilibrium of 2 m : 2T cos θ = 2 mg … (ii) 8 (d) The given figure can be drawn as

Solving these two equations, we get θ = 45°


5 (c) T1 cos 60 ° = T2 cos 30 ° R cos θ
°

R
60

T1 3T2
⇒ = T R sin θ
2 2
T1 3 T1 T2 T2
2 2
60° G 30° M 60 kg
T1 cos 60° T2 cos 30°
A B
w
Taking component of forces,
or T1 = 3T2 …(i) R cos θ = Mg ⇒ R cos 60° = Mg ...(i)
3T1 T and R sin 60° = T ...(ii)
∴ × AG = 2 × BG
2 2 By Eqs. (i) and (ii), we get
AG T2 T
⇒ = ⇒ tan 60° = ⇒ T = Mg tan 60 °
BG 3T1 Mg

But
T2
=
1
[from Eq. (i)] or T = 60 × g × 3 = 103.9 kgf
T1 3
9 (a) Force exerted by man on rope transfers to it in the form of
AG 1
Hence, = tension.
BG 3 Net upward force on the system is 2T or 2F.
6 (d) For equilibrium of body, Net downward force is (50 + 30 ) g = 80 g.
mg = 2T cos θ For equilibrium of system, 2F = 80 g or F = 40 g
228 OBJECTIVE Physics Vol. 1

10 (d) Let x be distance from A to O and L be the total length of l CHECK POINT 5.3
string. Then, ratio of tensions is
1 (c) R
T1 x 1
= =
T2 L 3 2 kg
–2
5 kg 5 ms

A 1 O 2 B
T1
m m T2 70 N
x
L
R − 70 = 7 × 5)
11 (c) The resultant force will be ∴ R = 105 N
FR = | FR | = 32 + 42 + 2 × 3 × 4 cos 90 ° = 5 N 2 (c) Given, v = 0, u = 3 ms −1
12 (a) Let equal forces F1 = F2 = F newton So, using v 2 = u 2 − 2as
Angle between the forces, θ = 60 ° 0 = (3)2 − 2(a )(9)
Resultant force, R = 40 3 N
1
⇒ a= = 0.5 ms −2 (upwards)
Now, R = F12 + F22 + 2FF
1 2 cos θ 2
∴ 40 3 = F 2 + F 2 + 2FF cos 60 ° Now, N = m (g + a ) (for deceleration)

1 = 50 (9.8 + 0.5) = 515 N


⇒ 40 3 = 2F 2 + 2F 2 ×
2 3 (d) Given, u = 10 ms −1, v = 0
⇒ F = 40 N So, using v 2 = u 2 − 2as
13 (a) As, resultant of F1 and F2,
0 = (10 )2 − 2(a )(25)
FR = F12 + F22 + 2FF
1 2 cos 90 °
∴ a = 2 ms −2 (upwards)
⇒ =F 2
Now, T − 800 g = 800a
∴ Resultant of three forces F1, F2 and F3 will be ( 2 − 1)F.
∴ T = 800 (10 + 2) = 9600 N
As, F = ma
4 (d) Acceleration of the system,
Therefore, acceleration of body is also ( 2 − 1)a. T3 40
a= = = 2 ms −2
14 (a) Minimum additional force needed m1 + m 2 + m 3 10 + 6 + 4
F = − (Fresultant )x Equation of motion of m 3 is
Fresultant = [(4 − 2)(cos 30 $j − sin 30 i$ ) + (cos 60 i$ + sin 60 $j)] T3 − T2 = m 3a
∴ T2 = T3 − m 3a
  3 $ 1 $  1 $ 3 $ 
= 2 j − i +  i + j  = 40 − 4 × 2 = 32 N
  2 2   2 2  m1F cos 30 ° 3F cos 30 °
5 (d) T2 = =
 (m1 + m 2 + m 3 ) (3 + 12 + 15)
3  $  $ $i  
=  3 +  j + − i +   3F cos 30 ° F cos 30 °
 2  2  = =
30 10
 1 3 3 $ $i 3 3$ (m1 + m 2 ) F cos 30 °
= − $i + j = − + j and T1 =
 2 2  2 2 (m1 + m 2 + m 3 )
 i$  (3 + 12) F cos 30 °
⇒ Fx =  −  =
 2 3 + 12 + 15
15 F cos 30 ° F cos 30 °
Hence, F =| Fx | = 0.5 N = =
30 2
15 (c) Apply Lami’s theorem at O,
The ratio between T1 and T2 is
T1 T2 10 10
= = = = 10 F cos 30 ° / 2
sin 150 ° sin 120 ° sin 90 ° 1 T1 : T2 = = 5:1
F cos 30 °/10
1
∴ T1 = 10 sin 150 ° = 10 × = 5 N F=3N
2 6 (d) 2 kg 1 kg
T2 = 10 sin 120 °
3
= 10 × =5 3N
2
Laws of Motion 229

As the blocks are rigid under the action of a force F, so both Since, pulley is in equilibrium, clamp will exert the same
will move with same acceleration. amount of force in opposite direction or pulley will also exert
F 3 3 this much force on clamp.
a= = = = 1 ms −2
m +M 2+1 3 12 (b) Upward force on 2 kg block in upward direction will be
40 N ( = 2F ) in the form of tension.
The force is applied to 2 kg, then its action on 1 kg will be
40 N
FN = ma = 1 × 1 = 1N
Net pushing force
7 (b) Acceleration of system, a =
Total mass 2 kg a
F − (m1 + m 2 + m 3 )g sin θ
or a=
(m1 + m 2 + m 3 )
Equation of motion for m 3, 20 N
N − m 3g sinθ = m 3a 40 − 20
∴ a= = 10 ms −2 (upward)
F − (m1 + m 2 + m 3 )g sin θ  2
or N = m 3g sin θ + m 3  
 (m1 + m 2 + m 3 )  Net pulling force
13 (d) Acceleration of the system, a =
m 3F Total mass
= 2mg − mg g
m1 + m 2 + m 3 = =
3  3m 3
m −m g
 m 2 − m1   2  g Now, from equation of motion of m, TAB
8 (b) Acceleration, a =   g= = mg
m1 + m 2  
m+ m
3  5 T − mg = ma =
 2  3
4mg
9 (c) (x P − x1) + (x P − x 2 ) = length of string = constant ∴ T=
3
Differentiating twice w.r.t. time, we get For equilibrium of pulley, T T

A TAB = 2T + Weight of pulley


8mg 17mg w
= + 3mg =
3 3
a1
 m − m1 
14 (c) Acceleration, a =  2 g
1
a2 xP m 2 + m1 
x1  5m − m 
= g
x2
2  5m + m 
4m 2
= g= g
a1 + a 2 6m 3
aP =
2 15 (b) Acceleration of the system,
Here a P = A, a1 is positive and a 2 is negative. (m 2 − m1) g
a=
a − a2 (m1 + m 2 + M )
Hence, A = 1
2 (3 − 1) g
=
10 (b) Acceleration of the system, (1 + 3 + 6)
Net pulling force 4g − 2g g 2g g 10
a= = = = = = = 2 ms −2
Total mass 6 3 10 5 5
Equation of motion of block C is
m C g − TBC = m Ca 16 (a) Pulley is moving upward with acceleration a 0 ,
∴ TBC = m C (g − a )  mm 
then tension, T = 2  1 2  [g + a 0 ]

= 2  9.8 −
9.8
 = 13 N m1 + m 2 
 3 g
T Here, a 0 =
11 (d) T = Mg 2
Force on the pulley (other than from clamp),  mm   g
∴ T =2 1 2  g +
 2 
Fnet = (T + mg )2 + T 2 mg
T
m1 + m 2  

= g (M + m )2 + M 2  mm 
Mg = 3g  1 2 
m1 + m 2 
230 OBJECTIVE Physics Vol. 1

a F 5000
17 (b)
T 4 (d) Retardation, a = = = 5 ms −2
m 1000
m
⇒ v = u + at ⇒ 0 = 30 − 5t
T ∴ t = 6s
2m F − µmg 100 − 0.5 × 10 × 10
a 5 (c) a = = = 5 ms −2
m 10
6 (a) fmax = µmg = 0.4 × 2 × 10 = 8 N
3m Since, the applied force is less than fmax, so force of friction
will be equal to the applied force or 2.8 N.
For mass m, T = ma …(i)
µmg
For mass 2m, 2mg + kx − T = 2ma …(ii) 7 (d) Retardation, a = = µg = 10 µ
m
For mass 3m, 3mg − kx = 3ma …(iii)
Now, v = u − 2as or 0 = (62 ) − 2 (10 µ )(9)
2 2
Adding Eqs. (i), (ii) and (iii), we get
∴ µ = 0.2
5mg = 6ma
µmg
5g 50 8 (a) Retardation, a = = µg = 4 ms −2
⇒ a= = m/s 2 m
6 6
Using v 2 = u 2 + 2as
18 (b) As shown in figure, when force F is applied at the end of
the string, the tension in the lower part of the string is also F. (8)2 = (0 )2 + 2(4)(s )
If T is the tension in string connecting the pulley and the ∴ s=8 m
block, then
a = 1 ms–2 9 (a) Due to friction (a = µg ), velocity of block will become
equal to velocity of belt. Relative motion between two will
F T T stop.
F ∴ v = at = µgt = 0.2 × 10 × 4 = 8 ms −1
T = 2F Tmax − µmg 40 − 0.2 × 8 × 10
10 (b) amax = = = 3 ms −2
But T = ma = (200 )(1) = 200 N m 8
∴ 2 F = 200 N 11 (a) m will move by friction,
or F = 100 N µmg
fmax = µmg ⇒ (a m )max = = µg
19 (c) Since, M1g sin 30 ° − M2g = (M1 + M2 ) a m
1
⇒ 10 × g × − 5 g = 15a 12 (a) Block A moves due to friction . Maximum acceleration of
2 f µmg
A can be max or or µg = 0.2 × 10 = 2 ms −2. If both the
⇒a = 0 m m
blocks move together, then combined acceleration of A and B
10
can be = 3.33 ms −2. Since, this is more than the maximum
M1 3
acceleration of A. Slipping between them will take place and
M1g sin θ a
force of friction between A and B is µm Ag = 2 N.
M2
θ = 30° 13 (a) fmax = µmg = 0.8 × 4 × 10 = 32 N
Net pulling force Mg sinθ At t = 2 s , F = k t 2 = (2)(2)2 = 8 N
20 (c) Acceleration of the system, a = =
Total mass 2M
Since, applied force F < fmax, force of friction will be 8 N.
1
a = g sinθ 14 (d) N = applied force = 12 N
2
Now, the block on ground is moving due to tension. ∴ fmax = µN = 7.2N
Mg sinθ Since weight, w < fmax
Hence, T = Ma =
2 Force of friction, f = 5 N
∴Net contact force = N 2 + f 2 = (12)2 + (5)2 = 13 N
l CHECK POINT 5.4
1
2 (a) Maximum inclination of the plane with horizontal = angle 15 (d) µ = tan 30 ° = (Q angle of friction = 30°)
of repose = tan−1(µ ). 3
F sin 30° F
v 4 N
3 (c) a = = = 2 ms −2 ⇒ F − f = ma 30º
t 2 F cos 30°
f 10 kg
or 200 − µ × 30 × 10 = 30 × 2
∴ µ = 0.47 mg
Laws of Motion 231

 F ∴ ∆p = Change in momentum
N = mg − F sin 30 ° = 100 − 
 2 = Final momentum − Initial momentum = mv − mu
F cos 30° = µN = m (v − u) = (0.15)[− (3$i + 4$j ) − (3$i + 4$j )]
3F 1  F = (0.15)[−6$i − 8$j )] = − [0.15 × 6$i + 0.15 × 8$j )]
⇒ = 100 − 
2 3 2 = − (0.9$i + 1.20 $j )
3F F
⇒ = 100 − ⇒ F = 50 N Hence, ∆p = − (0.9$i + 1.2$j )
2 2
16 (a) Angle of repose, θ r = tan−1(µ s ) = tan−1(0.7) or tan θ r = 0.7 6 (c) By previous solution, ∆p = − (0.9$i + 1.2$j )

Angle of plane is θ = 30 °, tan θ = tan 30 ° = 0.577 ∴Magnitude = ∆p = (0.9)2 + (1.2)2


Since tan θ < tan θ r , θ < θ r = 0.81+ 1.44 = 1.5 kg-ms −1
Block will not slide or f = mg sin θ ≠ µmg cos θ 7 (c) Force of friction is zero. Only contact force is the normal
or f = (2)(9.8) sin 30 ° = 9.8N reaction which is mg cos θ.
17 (a) mg sin θ = (102)(10 ) sin 30 ° = 510 N 8 (d) Angle of plane is just equal to the angle of repose.
µ smg cos θ = (0.4)(102)(10 ) cos 30 ° = 353 N i.e. µ = tan θ
F = 510 − 353 = 157 N Hence, the coefficient of kinetic friction, µ k = tan θ
18 (a) F = mg sin θ − µmg cos θ … (i) 9 (c) Acceleration, a = (g sin θ + µg cos θ )

2F = mg sin θ + µmg cos θ … (ii) = (g sin 45° + 0.5 × g × cos 45° )


g 1 3g
⇒ 2mg sin θ − 2 µmg cos θ = mg sin θ + µmg cos θ = + 0.5 × g × =
1 2 2 2 2
∴ µ = tan θ
3 M
10 (b) Mass per unit length of rope =
L
(A) Taking it together M
Tx = mass of rope of length (L − x ) × g = (L − x )g
L
1 (b) To solve this question, we have to apply Newton’s second
law of motion, in terms of force and change in momentum.
Tx x
dp L
We know that, F = P
dt (L − x)
Given that, metre scale is moving with uniform velocity. Mg
Hence, dp = 0. So, force, F = 0.
As all parts of the scale is moving with uniform velocity and total 11 (c) Wedge moves due to horizontal component of normal
force is zero, hence torque on centre of mass will also be zero. reaction.
dp N N sinθ
2 (d) We know that for a system, Fext = Thus, a= H = (along − ve X-axis)
dt M M
(from Newton’s second law) 12 (a) Net force on M in vertical direction should be zero.
If Fext = 0, dp = 0 ⇒ p = constant N′
Hence, momentum of a system will remain conserved, if
external force on the system is zero.
In case of collision between particles equal and opposite
N cos θ + Mg
forces will act on individual particles by Newton’s third law.
Hence, total force on the system will be zero. In vertically downward direction, two forces N cos θ and Mg
Note We should not confuse with system and individual are acting. Therefore, N′ the normal reaction from ground
particles. As total force on the system of both particles is zero, but should be equal to N cos θ + Mg.
force acts on individual particles.
13 (d) The block of mass m is at rest on an inclined plane.
3 (b) The tension T in the string is T = 6 (g + a ) = 6 (10 + 1) = 66N
Hence, frictional force acting between the surfaces is
4 (c) If lift is accelerating, reading will be more, if it is f = mg sinθ
decelerating, reading will be less and if it is moving with
constant velocity, reading will be same.
Hence, the acceleration of the block relative to the incline f
is (g − a ) sin θ.
θ θ
5 (c) Given, u = (3$i + 4$j ) ms −1 and v = − (3$i + 4$j ) ms −1 sin
mg mg mg cosθ
θ
Mass of the ball = 150 g = 0.15 kg
232 OBJECTIVE Physics Vol. 1

14 (a) Assuming the resistance force or retardation to be In second case,


T2 = 2F2
constant. 2
v 
  = v − 2as1
2
… (i)
 2
2
v 
0 =   − 2as2 … (ii)
 2
∴ In first case, person will have to apply more force.
Solving these two equations, we get
s 3 19 (a) Given, mass, m = 2 kg
s2 = 1 = = 1cm (Given, s1 = 3 cm)
3 3 x (t ) = pt + qt 2 + rt 3
15 (a) α = angle of repose dx
Velocity, v= = p + 2qt + 3rt 2
dt
α
dv
Acceleration, a= = 0 + 2q + 6 rt
α dt
At t = 2s, a = 2q + 6 × r × 2 = 2q + 12r
1
⇒ tanα = µ = = 2 × 4 + 12 × 5 = 8 + 60 = 68 ms −1
3
Force, F = ma = 2 × 68 = 136 N
∴ cot α = 3
20 (b) Given, mass, m = 5 kg
16 (d) Consider the adjacent diagram
N Acting force, F = (−3$i + 4$j ) N
B A Initial velocity at t = 0, u = (6$i − 12$j ) ms −1
F  3$i 4$j 
W E Retardation, a = =  − +  ms −2
O m  5 5
As final velocity is alongY-axis only, its x-component must be zero.
S From v = u + at, for x-component only,
Let OA = p1 = Initial momentum of player northward 3 5× 6
0 = 6− t ⇒ t = = 10 s
5 3
AB = p 2 = Final momentum of player towards west.
dm
Clearly, OB = OA + AB 21 (b) F = u = m (g + a )
dt
B A dm m (g + a ) 5000 × (10 + 20 )
⇒ = = = 187.5 kgs −1
dt u 800
22 (b) Acceleration in both the cases will be same.
R O N1 = ma, but N 2 = (2m )a
Muscle force (AR) = Change in momentum = p 2 − p1 ∴
N1 1
=
= AB − OA = AB + (− OA) N2 2
23 (b) When force is applied on A, acceleration produced will be
Clearly, resultant AR will be along south-west.
FA µm Ag
17 (b) Given, mass of the car = m = … (i)
mA + mB mB
As car starts from rest, u = 0
When force is applied on B, acceleration produced will be
Velocity acquired along east, v = v$i
FB µm Ag
Duration, t = 2 s = … (ii)
mA + mB mA
From first equation of motion,
Dividing these two equations, we get
v
v = u + at ⇒ v$i = 0 + a × 2 ⇒ a = $i FB m B
2 =
mv $ FA m A
Force, F = ma = i
2 m 8
∴ FB = B ⋅ FA = × 12 = 24 N
mv mA 4
Hence, force acting on the car is towards east due to
2
24 (d) Maximum force of friction between ground and B is
friction on the tyres exerted by the road.
fmax = (M + m ) µg = ( 15 + 5)(0.6)(10 ) = 120 N
18 (a) In first case,
T1 = F 1 A f=0
f=0
B F = 80 N
F = 80 N
Laws of Motion 233

As fmax > F 31 (c) Let acceleration of lift is a upwards. Then, with respect to lift,
∴ Frictional force between B and ground will be 80 N and
that between A and B is zero. a

25 (b) Net external force, F = (4) + (3) = 5 N


2 2

ar
Maximum friction, fmax = µmg = (0.09)(5)(10 ) = 4.5 N
Since F > fmax , block will move with an acceleration,
3 kg
F − fmax 5 − 4.5 ar
a= = = 0.1ms −2
m 5
(3g + 3a)
26 (a) µ(m A + m C )g = m B g 5 kg
m 5
∴ mC = B − mA = − 10 = 15 kg
µ 0.2 (5g + 5a)

27 (a) m 3 is at rest. Therefore, Net pulling force (5g + 5a ) − (3g + 3a )


ar = =
Total mass 8
9 (g + a )
⇒ g= (Given)
32 4
g
2T 2T ∴ a=
8
m3 P 32 (c) Suppose air resistance is F (upwards), then from equation
T a T of motion of balloon, we have
w
m1 a F′ = Mass × Acceleration = ⋅ a
m2 g
 a
∴ F = w − F ′ = w 1− 
2T = m 3 g …(i)  g
Further, if m 3 is at rest, then pulley P is also at rest.
33 (d) Moving down with retardation a means, lift is accelerated
Writing equations of motion, m1g − T = m1a …(ii) upwards.With respect to lift, pseudo force on the block will
T − m 2g = m 2a …(iii) be ma in downward direction, where m is the mass of block.
Solving Eqs. (i), (ii) and (iii), we get So, downward force mg on the block will be replaced by m (g + a ).
Therefore, acceleration of block relative to plane will be
m 3 = 1kg
N a r = (g + a ) sinθ (Down the plane) … (i)
28 (b) N sin θ + µN cos θ = ma N cosθ
1 2  1 2
N cos θ − µN sin θ = mg From L = a rt Q s = at 
a 2  2 
Putting θ = 45° and solving these two N sinθ
2L 2L
equations, we get ⇒ t= = [From Eq. (i)]
ar (g + a ) sinθ
f = µN
 1+ µ  mg
a=g  34 (c) Momentum of one piece =
M
×3
 1− µ  4
29 (a) N sinθ = ma = 1 × 5 = 5 M
N cosθ Momentum of second piece = ×4
4
N cos θ = mg = 1 × 10 = 10 N N a
9M 2 5M
From these two equations, we get ∴ Resultant momentum = + M2 =
16 4
N =5 5N N sinθ
The third piece should also have the same momentum. Let its
(Adding and squaring them) 5M M 5
1 velocity be v, then = × v ⇒ v = = 2.5 ms −1
and tanθ = mg 4 2 2
2 35 (a) mg sinθ − f = ma (where, f = force of friction)
30 (a) m (g − a ) sin θ = µm (g − a ) cos θ  1 
or f = m (g sin 30 ° − a ) = 8 10 × − 0.4 = 36.8 N
N  2 
36 (a) Acceleration of system before breaking the string,
Net pulling force 3g − 2g g
a= = =
Total mass 5 5
θ
g
mg – ma
After 5 s velocity of system, v = at = × 5 = g ms −1
5
v 2 g2 g
1 Now, h= = = = 4.9 m
⇒ µ = tan θ = tan 30 ° = 2g 2g 2
3
234 OBJECTIVE Physics Vol. 1

37 (a) Maximum acceleration of the box can be µg or 1.5 ms −2, 3


43 (c) fmax = µmg cos θ = 0.6 × mg × = 0.52mg
while acceleration of truck is 2 ms −2 . Therefore, relative 2
acceleration of the box will be a r = 0.5 ms −2 (backward). It
will fall off the truck in a time, f
2l 2×4  1 2 m
t= = =4 s Q s = at 
ar 0.5  2  v0
θ mg cos θ
Displacement of truck upto this instant, in mg
m gs θ = 30°
1 1
sr = a rt 2 = × 0.5 × (4)2 = 4 m mg
2 2 mg sin θ = = 0.5mg
F F 2
38 (a) = 75 N or < 100 N Since, fmax > mg sin θ
4 4
Block will decelerate and come to rest.
F = 300 N
44 (b) N = w + F sinθ
∴ fmax = µN = (tan φ )(w + F sin θ )
F/2
F F
N
2 θ

F F
4 4

w
To move the body, F cos θ = fmax = (tan φ ) (w + F sin θ )
50 N 100 N
Solving this equation, we get
75 − 50
Therefore, a M = 0, a m = = 5 m/s 2 w sin φ
5 F =
cos(θ + φ )
 mg 
39 (d) Total upward force = 2  = mg.
 2 45 (c) Maximum acceleration due to friction of mass m over mass
(Weight of man is balance by total tension acting upwards) 2m can be µg. Now, for the whole system,
Total downward force is also mg. Net pulling force
a=
∴ Fnet = 0 = anet Total mass
40 (a) T sin 30 ° = w = 40 N mg
∴ µg =
T sin 30º
4m
30º 1
T or µ=
30º 4
T cos 30º
46 (a) 2T1 cos 45° = mg
mg
w = 40N ∴ T1 = … (i)
2
T T2 sin θ
∴ = 40 or T = 80 N T2
2 θ
41 (c) For smooth surface, a1 = g sin 45°, t1 = t T1
T1 M T1 T2 cos θ
For rough surface, a 2 = g sin 45° − µg cos 45°, t2 = 2t 45º 45º
√2
M
1 2 1 2  1 2
∴ s = a11
t = a 2t2 Q s = at  T1
2 2  2  Mg +
m √2
⇒ (g sin 45° )t 2 = (g sin 45° − µg cos 45° ) (2t )2
T1 mg
T2 cos θ = =
3 2 2
⇒ 1 = (1 − µ ) 4 ⇒ µ = = 0.75
4 mg
T2 sin θ = Mg +
42 (b) Here, N = F = 5 N, µ = 0.5, m = 0.1kg 2
∴ fmax = µN = 2.5 N F=5N
mg
Mg +
Weight, w = mg = 0.1 × 9.8 = 0.98 N tanθ = 2 = 1 + 2M
mg m
Since w < fmax, force of friction will be 0.98 N.
2
Laws of Motion 235

47 (a) From the free body diagram, 51 (c) a


A
F cos θ + F cos θ − mg = 0 5 kg T
2F cos θ = mg f
d/2
F F ⇒ T − f = 5a, T − µmg = 5a
h T − 0.5 × 5 × 9.8 = 5a …(i)
F θ θ F T

5 kg a
mg

So, as the man moves up, θ increases, cos θ decreases. 5g


h
Q cos θ = ⇒ 5g − T = 5a …(ii)
2
d  From Eqs. (i) and (ii), we get
  +h
2
 2 5 × 9.8 − 0.5 × 5 × 9.8 = 10a
Now, when the man is at depth h, then the force is ⇒ 0.5 × 5 × 9.8 = 10a
d2 ⇒ a = 2.45 m/s 2
mg h 2 +
mg 4 = mg d 2 + 4h 2 From Eq (ii), we get
F = =
2 cos θ 2h 4h 5 × 9.8 − 5 × 2.45 = T
48 (a) Writing equation of motion for two weights ⇒ T = 36.75 N
a=g
52 (d) Since, the block is at rest under two forces
(i) weight of block.
(ii) contact force from the plane (resultant of force of friction
and normal reaction).
Contact force should be equal to weight (or 30 N) in upward
ar ar + a ar – a
w2 w2 direction. Because under the action of two forces, a body
w1 ar w1 remains in equilibrium when both the forces are equal in
magnitude, but opposite in direction, i.e. 30 N.
Acceleration relative to pulley Acceleration relative to ground
w 53 (a) When the system of two blocks of masses M and m is
w1 − T = 1 (a r − a ) …(i) either at rest or moving with the same speed, no net force
g acts on them.
w2 R
T −w2 = (a r + a ) …(ii) T
g M
µR
Solving Eqs. (i) and (ii), we get T
4w1 w 2
T= with a = g Mg
w1 + w 2 A
49 (d) Acceleration of system, m
F −f 20 − 10
a= = = 1ms −2 (towards left) mg
M+m 6+ 4
In this equilibrium state,
Let F0 be the reading of dynamometer, then equation of
T = µR = µMg and T = mg
motion of mass m would be
m
F0 − f = ma ∴ µMg = mg ⇒ M =
µ
⇒ F0 = f + ma = 10 + (4)(1) = 14 N
54 (a) N = mg − Q cos θ
50 (b) Net pulling force on the system should be zero, as velocity
Q
is constant. Hence, N θ
m Ag sin 30 ° = µm Ag cos 30 ° + m B g µN m
P

 m   µm 3 
∴ mB =  A  −  A  mg
 2  2 
P + Q sin θ = µN = µ (mg − Q cos θ )
1 3
= 10  − 0.2 × = 3.3 kg (P + Q sin θ )
2 2  ∴ µ=
(mg − Q cos θ )
236 OBJECTIVE Physics Vol. 1

55 (b) m1g > m 2g sin θ + µm 2g cos θ Now, let us draw free body diagram of m and 2m in all four cases.
µ µ
m1 (i) 3 mg m F= mg
⇒ > sin θ + µ cos θ 3
m2 µ
mg
θ T=0 3
56 (d) N = Mg − F cos θ = Mg − Mg cos θ = 2Mg sin 2
2m
2
µ
Further, block can be pulled, if F sinθ ≥ µN 3
mg
θ θ θ µmg F = µmg
⇒ 2Mg sin ⋅ cos ≥ 2 µMg sin2 m
2 2 2 (ii)
 θ T=0
⇒ cot  ≥ µ 2m µmg
 2
µmg
2F − F F
57 (d) a = = (towards left)
2m 2m µmg m F = 2µmg
Writing equation of right hand side block, (iii)
F T=0
2F − N sin 30 ° = m a = 2m µmg
2
µmg
N F 3F
= 2F − = ⇒ N = 3F
2 2 2 µmg m F = 3µmg
(iv)
58 (c) In critical case, weight of hanging part = force of friction
of the part of rope lying on table. T=0
2m µmg
m m
∴ ⋅ l1g = µ (l − l1)g µmg
l l
Solving above equation, we get As tension is zero in all four cases. So, option (c) is incorrect.
 µ  g
l1 =  l 61 (b) For block, a = g sinθ =
 1+ µ  2

59 (d) Block A moves due to friction. Maximum value of friction N /√2


can be µm Ag. Therefore, maximum acceleration of A can be
µm Ag g aH
or µg = 0.2g = . When force 2 mg is applied on lower N /√2 m
mA 5 45º
block, common acceleration (if both move together) will be
aV
m 0.2 mg mg sinθ mg
0.2 mg
2 mg
3m a H = aV = a cos 45°
g 1 g
= × =
Net force 2mg g 2 2 2
a= = =
Total mass 4m 2 N mg
= … (i)
Since, a = g /2 is greater than maximum acceleration of A 2 2
which can be given to it by friction.Therefore, slipping will take N mg
place. mg − = … (ii)
2 2
g
a A = 0.2g = For wedge, N′
5
2mg − 0.2mg 3g
aB = = 0.6g = µ N′ N
3m 5 2m 2
 g 3g 
a AB = a A − a B =  −  N + 2mg
5 5 2
−2g N
= (in backward direction) N′ = + 2mg … (iii)
5 2
60 (c) f1 = maximum value of friction between m and 2m = µmg µN ′ =
N
… (iv)
f2 = maximum value of friction between 2m and ground 2
µ Solving above four equations, we get
= (3m )g = µ mg
3 µ = 0.2
Laws of Motion 237

62 (a) 2T = 250 5 (b) f − mg sin θ = ma sin θ


0.4F 0.4F a sin θ
2T T
f

F F mg sin θ
F F

0.4F ∴ f = m (g + a ) sinθ > mg sinθ


250 N 150 N
6 (b) N1 = ma x = 2m
⇒ T = 125 N
Also, T + 0.4F = 150 N 2 = m (g + a y ) = 12m
N1 1
⇒ F = 62.5 N ∴ =
N2 6
63 (d) N1 sin 37° = N 2 sin 74°
⇒ N1 = 2N 2 cos 37° l Statement based questions
N1 2 (b) The statement given in option (b) is incorrect, which can
be corrected as,
37º Friction opposes the relative motion of the bodies in contact
not the motion.
3 (b) Only Statement II is correct while Statement I is incorrect,
74º
N2 which can be corrected as,
mg Tension is non-uniform even, if string is not accelerated.
Now, N1 cos 37° − N 2 cos 74° = mg 4 (a) Fnet = ma
⇒ 2N 2 cos 37° − N 2 (2 cos 37° − 1) = mg
2 2

For 1 kg mass
 16  32 
⇒ 2N 2   − N 2  − 1 = mg ⇒ N 2 = mg
 25  25 

64 (c) T sin θ − mg sin α = ma For 2 kg mass


T cos θ T
θ
θ in a
Ts 10
Solving, we get a = m/s 2
3
mg sin α 10 20
mg cos α Force on 1 kg, F1 = N and on 2 kg, F2 = N
α 3 3
and T cos θ = mg cos α Hence, Statement I is correct while Statement II is incorrect
From these two equations, we get 5 (c) As forces at two ends of string are different, so tension on
 a + g sin α  it decreases from 2F to F in moving from A to B.
a + g sin α
tan θ = ⇒ θ = tan−1  2F − F
g cos α  g cos α  In case of massless string, a = =∞
0
Hence, both statements are correct.
(B) Medical entrance special format
questions l Match the columns
1 (c) N = mg − 20 2 sin 45° = 20 N, f = µ kN = 12 N
l Assertion and reason
Since, 20 2 cos 45° > µN block will move and its acceleration
1
2 (c) mg sinθ = 10 × 10 × = 50 N will be
2
20 2 cos 45° − µ kN 20 − 12
ffriction = µmg cos θ = 0.7 × 10 × 10 ×
3
= 60.62 N a= = = 2 ms −2
2 m 4
Since, µmg cos θ is more, block will remain stationary. Hence, A → q, B → p, C → s.

4 (a) (f2 )max = µ 2mg 2. (d) Acceleration of system,


60 − 18 − (m1 + m 2 + m 3 )g sin 30 °
(f1)max = µ1(2m )g a= = 2 ms −2
(m1 + m 2 + m 3 )
Since, (f1)max > (f2 )max
Net force on 3 kg block = m 3a = 6 N
Lower block will not move at all.
238 OBJECTIVE Physics Vol. 1

From free body diagram of 1 kg block, we have


N12 − m1g sin 30 ° − 18 = m1 a (C) Medical entrances’ gallery
⇒ N12 = 25 N 1 (b) Given, m1 = 4 kg, m 2 = 6 kg and a = ?
From free body diagram of 3 kg block, we have The free body diagram of given system is shown below
60 − m 3g sin 30 ° − N 32 = m 3a
a
∴ N 32 = 39 N T T
Hence, A → r, B → t, C → q, D → t. 4 kg
3 (b) Force diagram of both the blocks are as shown. m1g
6 kg a
1 kg
4N m2g
4 The balancing equations for given system are
⇒ a1 = = 4 m/ s 2
1 T – m1 g = m1a …(i)
2 kg
4N and m 2g – T = m 2a …(ii)
18 N Adding Eqs. (i) and (ii), we get
22 m 2g – m1g = m1a + m 2a
⇒ a2 = = 11m/ s 2
2
 m − m1  6 − 4 g
⇒ a 21 = a 2 – a1 = 7m/ s 2 ⇒ a= 2  ⋅g =   ×g=
 m1 + m 2   4 + 6 5
Hence, A → s, B → p, C → s.
2 (a) The given situation is shown in the following diagram.
4 (a) Since, the pulleys are smooth, net force on each pulley
a
should be zero. With this concept, tensions on all strings are
shown below. 10 kg
T
F = 80 N

T a

40 N 40 N 2 kg

If a be the acceleration of the system,


20 N then equation of motion of 10 kg trolley,
20 N 20 N
T − µR = 10a
1 kg 3 kg 20 N ⇒ T − 0.05 × 10 g = 10a (Given, µ = 0.05, R = 10 g)
2 kg 4 kg ⇒ T − 5 = 10a … (i)
Equation of motion of 2kg block,
Now, we can draw free body diagrams of all the four blocks.
20 N 20 N 2g − T = 2a
⇒ 20 − T = 2a … (ii)
Adding Eqs. (i) and (ii), we get
1 kg a2 = 0 2 kg 15 5
15 = 12a ⇒ a = = = 1.25 ms − 2
12 4
a1 = 10 ms–2
3 (b) As the truck move to the right, so the bob will move to the
10 N 20 N
left due to inertia of rest with acceleration a.
20 N 20 N Thus, the given situation can be drawn as
ma cos θ
10 ms –2 3 kg 4 kg a4 = 20 = 5 ms–2
a3 = 4 θ θ
3 a a ⇒ ma
θ
mg sin θ
30 N 40 N mg
(a)
Hence, A → (r, t), B → p, C → q, D → (q, s). (b)
Laws of Motion 239
2 × 10 −3
 t2 
Now, from equilibrium of forces in above diagram (b), we get = 100t − 0.5 × 10 5 ⋅ 
 20
ma cos θ = mg sin θ
sin θ ma 0.5 × 10 5
⇒ = = 100 × 2 × 10 −3 − × (4 × 10 −6 − 0 )
cos θ mg 2
= 0.2 − 0.1 = 0.1N-s
a  a
⇒ tanθ = ⇒ θ = tan−1  8 (d) By Newton’s second law of motion, net external force
g  g
applied on a system is equal to rate of change of momentum.
4 (c) The situation can be drawn as dp
i.e. Fext =
F N dt
dp
If F ext = 0, then = 0 ⇒ p = constant
dt
f FH
i.e. When F ext = 0, then momentum of the system remains
conserved.
The internal interaction between particles of a body do not
mg
contribute to change in total momentum of body.
The frictional force, f = µN = µ mg (Q N = mg) Hence, both Assertion and Reason are false.
From free body diagram, the resultant force is 9 (d) According to the question, the free body diagram of the
given condition will be
|F | = N + f 2 2
A
R cos q
= (mg )2 + (µmg )2 = mg 1 + µ 2 R
q
This is the minimum force required to move the object. But as ma R sin q
the body is not moving. (Pseudo q
force)
∴ | F | ≤ mg 1 + µ 2 mg q a
B C
5 (b) As the three forces are represented by three sides of a
Since, the wedge is accelerating towards right with a, thus a
triangle taken in order, then they will be in equilibrium.
pseudo force acts in the left direction in order to keep the
P block stationary. As, the system is in equilibrium.
∴ ΣFx = 0 or ΣFy = 0
⇒ R sin θ = ma …(i)
Q Similarly, R cos θ = mg …(ii)
R
Dividing Eq. (i) by Eq (ii), we get
⇒ Fnet = FPQ + FQR + FRP = 0 R sin θ ma
dv =
Fnet = m × a = m =0 R cos θ mg
dt a
dv ⇒ tan θ = or a = g tan θ
⇒ = 0 or v = constant g
dt
∴ The relation between a and g for the block to remain
So, the velocity of particle remain constant. stationary on the wedge is a = g tan θ.
6 (a) Force exerted by concrete floor is more as compared to 10 (d) The opposing force that comes into play when one body is
wooden floor due to greater change in momentum. actually sliding over the surface of the other body is called
Since on concrete floor, glass ball will take less time to come sliding friction.
to rest, so a glass ball dropped on concrete floor can easily get The coefficient of sliding is given as µ s = N /Fsliding
broken compared, if it is dropped on wooden floor.
where, N is the normal reaction and Fsliding is the sliding force.
Hence, both Assertion and Reason are true and Reason is the
correct explanation of Assertion. As, the dimensions of N and Fsliding are same. Thus, µ s is a
dimensionless quantity. When body is rolling, then it reduces
7 (c) Force applied by gun, F = (100 − 0.5 × 10 5 t ) N the area of contact of surfaces, hence rolling friction is smaller
Speed of bullet, v = 400 m/s than sliding friction.
When F = 0, then 100 − 0.5 × 10 5 t = 0 Hence, statement (d) is incorrect.

⇒ t = 2 × 10 −3 s 11 (c) When the system is in equilibrium, then the spring force is


3 mg. When the string is cut, then net force on block A
2 × 10 −3
= 3 mg − 2 mg = mg
Impulse, I = ∫ Fdt = ∫ (100 − 0.5 × 10 5 t ) dt
0
Hence, acceleration of block A at that instant,
240 OBJECTIVE Physics Vol. 1

Force on block A mg g 1 1
a= = = So, s1 = 0 + g (sin θ − µ cos θ )t12 and s2 = 0 + g sinθt22
Mass of block A 2m 2 2 2
1 1
When string is cut, then block B falls freely with an As, s1 = s2 , g (sin θ − µ cos θ )t12 = g sin θt22
acceleration equal to g. 2 2
2T sin θ − µ cos θ t22
12 (a) Let a1 and a 2 be accelerations of a ball T ⇒ = 2
sin θ t1
(upward) and rod (downward), respectively.
t22
Clearly, from the diagram, a2 ⇒ 1− µ cot θ = (Qt1 = 2t2)
a1 9 (2 t2 )2
2 a1 = a 2 …(i) 5
mg
1 3
⇒ 1 − µ cot θ = 1/ 4 ⇒ µ cot θ = 1 − =
Now, for the ball, 4 4
9 9 mg 3
2 T − mg = ma1 …(ii) ∴ µ=
5 5 4 cot θ
and for the rod, mg − T = ma 2 …(iii) 16 (b) Acceleration of the body down the inclined plane = g sinθ.
On solving Eqs. (i), (ii) and (iii), we get

N
g
a1 = m/ s 2 ↑ (upward)
29 m
2g
a2 = m/ s 2 ↓ (downward) θ
29
mg mg cos θ

θ
So, acceleration of ball w.r.t. rod = a1 + a 2 = (3g / 29) m / s 2

sin
θ=30°

mg
Now, displacement of ball w.r.t. rod when it reaches the
upper end of rod is 1m. ∴ Force applied on spring balance
1 1
Using second equation of motion, s = ut + at 2 = mg sinθ = 5 × 10 × sin 30 ° = 5 × 10 × = 25 N
2 2
1 3 × 10 2
1= 0 + × t 17. (b) Given, F = 10 N, vi = 0, m = 0.5 kg and ∆t = 0.25 s
2 29
As impulse, I = p f − pi
⇒ t = 58/ 30 = 1.4 s (approx)
Also, I = F ⋅ ∆t = 10 × 0.25 = 2.5 N-s
13 (a) Angle of repose is equal to angle of limiting friction as
maximum value of static friction is called the limiting friction. 18 (b) As there is a load at P, so tension in AP and PB will be
different. Let these be T2 and T1, respectively. For vertical
Hence, both Assertion and Reason are correct and Reason is
equilibrium at P,
the correct explanation of Assertion.
B
14 (a) Since, F = (M + m )a …(i)
30°
ma T1 cos 60°
cos 60°
αα
N T1
A P
T1 sin 60°
ma T2
ma sin α
α mg
mg cos α mg sin
α α M

Normal, N = (mg cos α + ma sin α ) Mg


So, we apply pseudo force on the block by observing it from T1 cos 60 ° = Mg, i.e. T1 = 2 Mg …(i)
the wedge.
and for horizontal equilibrium at P,
Now, from free body diagram of block, we get
T2 = T1 sin 60 ° = T1 ( 3 / 2 )
ma cos α = mg sin α
sin α Substituting the value of T1 from Eq. (i), we get
⇒ a=g ⇒ a = g tanα …(ii)
cos α T2 = (2Mg ) × ( 3 / 2 ) = 3 Mg
Now, from Eqs. (i) and (ii), we get 19 (b) Initially system is in equilibrium with a total weight of 4mg
F = (M + m )g tanα over spring.
15 (c) Given, θ = 45°, s1 = s2, u = 0, t1 = 2 t2 kx
On the rough incline, a1 = g (sin θ − µ cos θ ) (3m+m)
On the frictionless incline, a 2 = g sinθ A 3m
1 Cutting
As, s = ut + at 2 4 mg plane
2 B m
Laws of Motion 241

∴ kx = 4mg Case 2 For body coming down the plane,


When string is cut at the location as shown above. 1
s = ut + a 2t22
Force on mass m, FB = mg 2
As, u=0
∴ Acceleration of mass m, a B = g
⇒ t2 = 2s / a 2 … (iii)
For mass 3m
Downward acceleration,
If a A = acceleration of block of mass 3m, then Fnet = 4mg − 3mg
g mg sin 30 ° − µR
⇒ 3m ⋅ a A = mg or a A = a2 =
3 m
So, accelerations of blocks A and B are a A =
g
and a B = g mg sin 30 ° − µmg cos 30 °
=
3 m
20 (c) Given situation can be represented as g µg 3
F ← m1 → T1 ← m 2 → T2 ← m 3 → T3 ← m 4 = − …(iv)
2 2
(m 2 + m 3 + m 4 ) t1 = t2 / 2
⇒ T1 = F Given,
m1 + m 2 + m 3 + m 4
Substituting values of t1 and t2 from Eqs. (i) and (iii), we get
Given, m1 = m 2 = m 3 = m 4 = m 2s 1 2s
3 =
∴ T1 = F a1 2 a 2
4
Now, squaring both sides, we get
(m 3 + m 4 )F
Similarly, T2 = a1 = 4a 2 … (v)
m1 + m 2 + m 3 + m 4
Substituting values of a1 and a 2 from Eqs. (ii) and (iv) in
1
∴ T2 = F Eq. (v), we get
2
3 g  g µg 3 
m 4F 1 µg + = 4 − 
Also, T3 = ⇒ T3 = F 2 2 2 2 
m1 + m 2 + m 3 + m 4 4
Solving for µ, we get
21 (a) According to question, the situation can be shown as
3 1. 732
R tio
n µ= =
Mo 5 5
⇒ µ = 0 . 346
θ
sin
mg 22 (c) The force causing the motion of A is frictional force
θ between A and B,
µR mg cosθ
θ=30°
mg µMB g = MAa A
So, acceleration of A ,
Case 1 For body projected up the plane, v = 0 M 
aA = µ  B  g
Using third equation of motion, v 2 = u 2 − 2a1s  MA 
⇒ 0 = u 2 − 2a1s µg
= (towards right)
∴ u = 2a1s 2
Block B experiences friction force toward left,
Also, v = u − a11 t
MBa B = µMB g
⇒ 0 = u − a11 t
⇒ a B = µg (towards left)
⇒ t1 = u / a1
23 (a) For friction to be minimum,
2a1s 2s
∴ t1 = = … (i)
a1 a1 os θ
Fc
As, ma1 = µR + mg sin θ θ
µR + mg sin 30 ° F
∴Retardation, a1 = nθ
g si
m m θ
30° N
Also, R = mg cos 30 ° mg
µmg cos 30 ° + mg sin 30 °
⇒ a1 =
m F cos θ = mg sin θ
80
3 g ⇒ F = mg tanθ = 8 × 10 × tan 30 ° = N
= µg + … (ii) 3
2 2
242 OBJECTIVE Physics Vol. 1

24 (a) Given, m1 = 10 kg, m 2 = 20 kg, F2 = 200 N, a1 = 12 m/s 2, ⇒ 300 − 60 = 6 a ⇒ 240 = 6 a


240
a2 = ? ∴ a= = 40 ms −2
6
For mass m1, F1 = m1a1 = 10 × 12 = 120 N
For mass m 2, 29 (b) Let weight of A is w′. From the free body diagram, for
equilibrium of the system,
F2 − F1 = m 2a 2 T sinθ
200 − 120 = 20 × a 2 N T
80 θ
⇒ a2 = = 4 ms −2 B
20 f = µN T cosθ
w A
25 (a) Suppose length of plane is L. When block descends the
w'
plane, rise in kinetic energy = fall in potential energy
= mg Lsin θ T cos θ = µN = µw …(i)
Work done against
T sinθ = w ′ …(ii)
friction is zero where, T = tension in the thread lying between knot and the
for smooth surface support.
On dividing Eq. (ii) by Eq. (i), we get
T sin θ w ′ w′
Loss in PE = ⇒ tan θ =
= Gain in KE T cos θ µw µw
θ mg sin θ ⇒ w ′ = µw tan θ
mg cos θ
(Reaction) mg 30 (b) Given, m A = 4 kg, m B = 2 kg
θ
Work done against and m C = 1kg
friction on rough surface a
= µ (mg cos θ) × (L/2)
F
Work done by friction A B C
= µ × (Reaction) × Distance
= 0 + µ (mg cos θ ) × (L / 2) So, total mass, M = 4 + 2 + 1 = 7 kg
= µ (mg cos θ ) × (L / 2) Now, F = Ma ⇒ 14 = 7a ⇒ a = 2 ms −2
Now, work done = change in KE FBD of block A ,
mgL sin θ = µ (mg cos θ ) × (L / 2) ⇒ tan θ = µ / 2 a

∴ µ = 2 tan θ F F′
4 kg
26 (a) Impulse is imparted due to change in perpendicular
components of momentum of ball, F − F ′ = 4a ⇒ F′ = 14 − 4 × 2 ⇒ F′ = 6 N
J = ∆p = mv f − mvi Hence, the contact force between A and B is 6N.
= mv cos 60 ° − (− mv cos 60 ° ) 31 (c) FBD of block A ,
1 a
= 2mv cos 60 ° = 2mv × = mv
2 T m1
27 (b) Force, P = fm = µ s mg (when body is at rest)
fk
When the body starts moving with acceleration a, then
T − m1a = fk …(i)
P − Fk = ma T
FBD of block B,
µ s mg − µ k mg = ma
⇒ a = (µ s − µ k )g = (0.5 − 0.4)10
= 0.1 × 10 ms −2 = 1 ms −2 m2 a

28 (a) For the motion of balloon,


Let upward force is F, then F − mg = ma
m2g
⇒ F − 10 × 10 = 10 × 20
⇒ F − 100 = 200 ⇒ F = 300 N m 2g − T = m 2 a …(ii)
When 4 kg mass is removed, then weight of balloon Adding Eqs. (i) and (ii), we get
= 6 × 10 = 60 N m 2g − m1a = m 2a + fk
This is smaller than the upward force. ⇒ m 2g − m1a = m 2a + µ km1g
(m − µ km1)g
So, F − 60 = 6 × a ⇒ a= 2
where, a = new upward acceleration m1 + m 2
Laws of Motion 243

From Eq. (ii), we get Adding Eqs. (i) and (ii), we get
 (m − µ km1)  g
T = m 2 (g − a ) = m 2 1 − 2 g mg (1 − 2 µ ) = 3m × a ⇒ a = (1 − 2 µ )
 m1 + m 2  3
m m (1 + µ k )
T= 1 2 g 35 (c) Given, m1 = m 2 = m 3 = 2 kg, F = 10.2 N
(m1 + m 2 )
The FBD of given system is as shown below
32 (a) When the balloon is descending down with acceleration a,
T T T1 T1
mg − B = m × a …(i) C B A F
where, B = Buoyant force.
For body A , F − T1 = ma ...(i)
Here, we should assume that while removing some mass, the For body B, T1 − T = ma ...(ii)
volume of balloon and hence, buoyant force will not change.
For body C, T = ma ...(iii)
Let the new mass of the balloon is m′.
Adding Eqs. (i), (ii) and (iii), we get
So, mass removed = m − m′ 10.2
When the balloon is moving up with acceleration a, then F = 3 ma ⇒ a = = 1.7 ms −2
3× 2
B − m′g = m′ × a …(ii)
Alternately Acceleration can be found as net acceleration of
Adding Eqs. (i) and (ii), we get a system, i.e.
⇒ mg − m′g = ma + m′a Total net force 10.2 10.2
a= = = = 1.7 ms −2
⇒ (mg − ma ) = m′ (g + a ) Total mass (2 + 2 + 2) 6
⇒ m (g − a ) = m ′ (g + a ) So, net tension in string between the blocks B and C is
m (g − a ) T = m × a = 2 × 1.7 = 3.4 N
∴ m′ =
(g + a )
So, mass removed, 36 (c) From FBD shown below, mg sin θ − f = ma
 (g − a )   (g + a ) − (g − a ) 
∆m = m − m′ = m 1 −  =m   f
 (g + a )   (g + a ) 
 g + a − g + a  2ma ma
=m  = g+a θ mg sinθ
 g+a  mg cosθ
mg
−1 θ
33 (b) Given, u = 30 2 ms
Distance = s metre mg sin θ − ma = f
Let a be the acceleration of the bullet. ⇒ 8 × 10 sin (30 ° ) − 8 × 0.4 = f
According to the first condition, (v = 0 ) ⇒ 40 − 3.2 = f ⇒ f = 36.8 N
From third equation of motion, v 2 = u 2 − 2as 37 (a) If the same wedge is made rough, then time taken by it to
900 −2 come down becomes n times more.
0 = 900 × 2 − 2as ⇒ a = ms
k

s
k
oc

oc
Bl

Bl
n

For second condition,


io
ot
M

900 s
v 2 = u 2 − 2as = (30 2 )2 − 2 × ×
s 2 d d
⇒ v 2 = 1800 − 900
⇒ v = 900 = 30 ms −1 Rough surface Smooth surface
45° 45°
34 (c) First of all consider the forces on the blocks
2s
a For rough surface, t= …(i)
g sin θ − µg cos θ
2 T T 3
T1 2 3 t 2s
m m For smooth surface, = …(ii)
n g sin θ
µmg µmg
a T1 Squaring Eqs. (i) and (ii) and dividing them, we get
 1
1 µ = 1 − 2  tan θ
 n 
mg  1  1
∴ µ = 1 − 2 tan θ = 1 − 2 tan 45°
For the Ist block, mg − T1 = m × a (Q m1 = m 2 = m 3 ) …(i)  n   2 
⇒ Let us consider 2nd and 3rd blocks as a system. 4−1 3
⇒ µ= =
So, T1 − 2 µmg = 2m × a …(ii) 4 4
244 OBJECTIVE Physics Vol. 1

1.5 × 2
38 (a) Consider a body of mass m placed on a rough inclined ⇒ a= = 18.75 ms −2
surface and it is just on the point of sliding down, with (0.4)2
coefficient of friction µ inclined at angle θ, as shown in figure. As the string is moving upwards with this acceleration.
R F ∴T = m (g + a ) = 2(9.8 + 18.75) = 57.1N
44 (d) Using law of conservation of momentum, we get
θ
in
mg
s θ 100 × v = 0.25 × 100 ⇒ v = 0.25 ms −1
mg cos θ
θ mg
45 (c) Initial thrust must be
At the equilibrium point O, m ( g + a ) = 3.5 × 10 4 (10 + 10 ) = 7 × 10 5 N
In case of limiting condition, F = mg sin θ …(i) 46 (d) From first equation of motion, v = u − at
Normal force, R = mg cos θ …(ii) 0 = u − µgt
On dividing Eq. (i) by Eq. (ii), we get u 6
F mg sin θ ⇒ µ= = = 0.06
= gt 10 × 10
R mg cos θ
47 (d) Force of limiting friction for block = µ smg
µ = tan θ (Q Force of friction, F = µR)
= 0.60 × 10 × 9.8 = 58.8 N
⇒ θ = tan−1 (µ )
If the applied force is greater than 58.8 N, then the block will
This is maximum value of θ for mass m to be at rest. move over the slab.
For smaller θ, body will be at rest, i.e. in equilibrium.
Kinetic friction acting on the block towards right
So, angle of repose, i.e. θ = tan−1 µ. = µ k mg = 0.40 × 10 × 9.8 = 39.2 N
39 (a) Since, all the blocks are moving with constant velocity, This is also equal to the force of friction acting on slab
then the net force on all the blocks will be zero. towards left. This is the only force acting on slab.
40 (c) Given, h = 100 m and t = 10 s F 39.2
So, acceleration of the slab, a = = = 0.98 ms −2
1 2 m 40
From second equation of motion, h = at
2 48 (b) Force down the plane = mg sinθ
2h 2 × 100 ∴ Acceleration down the plane = g sinθ
⇒ a= 2 = = 2 ms −2
t (10 )2
Now, F = m (g + a ) = 50 (10 + 2) = 600 N l
41 (d) We know that, F − f = ma h
20 − 0.2 × 3 × 10
Here, for small body, a1 =
3 θ
14 −2 1
⇒ a1 = ms Using the relation, h = ut + gt 2
3 2
Similarly, 0.2 × 3 × 10 = 10 × a 2 1
6 l = 0 + (g sin θ ) t 2
⇒ a2 = = 0.6 ms −2 2
10 2l 2h  h 
⇒ t =
2
= Q l = 
42 (d) Here, Meff g = M (g − a ) g sin θ g sin2 θ  sinθ 
g M g 60 1 2h
or = ⇒ = or t=
g − a Meff g − a 54 sinθ g
g 10 2h
⇒ = ⇒ 9g = 10 g − 10a 49 (b) Time t1 for stationary lift =
g− a 9 g
⇒ − g = − 10a ⇒ g = 10a When lift is moving up with constant acceleration, then
g 9.8
⇒ a= ⇒ a= = 0.98 ms −2 t2 =
2h
[Q Relative acceleration = (g + a )]
10 10 g+a
43 (b) Given, u = 0, s = 1.5 m, t = 0.4 s ∴ t1 > t2
1
From second equation of motion, s = ut + at 2 50 (a) Kinetic friction force = µ k R = µ k mg = 0.4 × 60 × 10 = 240 N
2 and the limiting friction force
1
⇒ 1.5 = 0 + a (0.4)2 = µ s R = µ s mg = 0.5 × 60 × 10 = 300 N
2
Laws of Motion 245

So, the force applied on the body is 300 N and if the body is 55 (d) Frictional force, f = 28.2 cos 45°
moving afterwards with the same force, then
1
= 28. 2 × = 20 N
2
Net accelerating force = Applied force − Kinetic force
R
ma = 300 − 240 28.2 sin 45°
60 60
⇒ a= = = 1 ms −2
m 60 28.2 cos 45°
51 (c) Tension in the string,
f
2 m (v ′ )2 m (2gl )
T= + 2 mg = + 2 mg
l 2l 50 N
 2m (v ′ )2 v gl  Normal reaction, R = 50 − 28.2 sin 45°
Q = Centrifugal force and v ′ = = 
 l 2 2 = 50 − 20 = 30 N
T = 3 mg
56 (b) The accelerating force on the rocket = upward thrust
∴ Increase in tension = 3 mg − mg = 2 mg
∆m
= ⋅v
52 (c) Friction force between block A and block B ; and between ∆t
block B and surface will oppose the force F. ∆m
Given, = 50 × 10 −3 kgs −1
∴ F = FAB + FBS ∆t
and v = 400 ms −1
So, accelerating force = 50 × 10 −3 × 400 = 20 N
57 (b) The velocity of exhaust gases with respect to the rocket
A
= 100 ms −1
FAB
The minimum force on the rocket to lift it,
FBS B F Fmin = mg = 1000 × 10 = 10000 N
Ground
Hence, minimum rate of burning of fuel is given by
dm Fmin 10000
= = = 100 kgs −1
= µ AB m Ag + µ BS (m A + m B ) g dt v 100
= 0.2 × 100 × 10 + 0.3(100 + 200 ) 10 58 (c) In a non-inertial frame, the second law of motion is written
= 200 + 900 = 1100 N as F = ma − Fp
This is the required minimum force to move the block B. where, Fp is the pseudo force while a is the acceleration of
53 (b) Given, m b = 40 g, v b = 1200 ms −1, the body relative to non-inertial frame.

F = 144 N 59 (a) When the lift is accelerating upwards with a constant


acceleration a, then the apparent weight,
m bv b
∴ F = n ma = ×n w = m (g + a ) = 60 (10 + 2)
t
40 × 1200 = 60 × 12 = 720 N
⇒ 144 = ×n ⇒ n = 3 When the lift is accelerating downwards with acceleration a,
1000
then apparent weight,
54 (a) The maximum weight which can be suspended with the
w ′ = m (g − a ) = 60 (10 − 2)
rope without breaking it = 30 kg-wt.
or Fmax = 30 × 10 = 300 N = 60 × 8 = 480 N
∴ 300 = mg + ma 60 (b) When an object moves in a plane surface with uniform
⇒ ma = 300 − mg = 300 − 25 × 10 = 50 N velocity in the presence of a force, then the frictional force
50 50 between the object and the surface has opposite value of the
⇒ a= = = 2 ms −2 present force. So, the frictional force between the object and
m 25
the surface is − 10 N.
CHAPTER
06

Work, Energy
and Power
The terms work and energy are quite familiar to us and we use these terms in
many ways. In physics, work is said to be done on a body, when a force acts on it
and the point of application of force actually moves.
Energy is the capacity to do work and the term power is usually associated with the
time in which the work is done.
In this chapter, we are going to develop better understanding of these three
physical quantities in detail.

WORK
Work is said to be done when a force acts on a body in such a way that the body
is displaced through some distance in the direction of force.
We define the work done by a constant force on a body as the product of the
force F and the displacement s through which the body is displaced in the direction
of force.

F F

Initial position s Final position Inside


Fig. 6.1 Work done, when force and displacement are in the same direction
1 Work
2 Conservative forces
Then, the work done W is given by W = F ⋅s
3 Non-conservative forces
On the other hand, in a situation, when the constant force F acting on the body 4 Energy
makes angle θ with the horizontal and body is displaced through a distance s. Kinetic energy
Work-energy theorem
Then, F can be resolved into two components Potential energy
(i) F cosθ in the direction of displacement of body. 5 Equilibrium
(ii) F sinθ in the perpendicular direction of the displacement of body. 6 Law of conservation
of energy
F
7 Conversion of mass
θ and energy
8 Power
s Instantaneous power

Fig. 6.2 Work done, when force and displacement


are inclined to each other
Work, Energy and Power 247

Thus, in this case, work done by a constant force F is Sol. Given, displacement, s = 30 m
given by Force, F = 30 kg-wt = 300 N
W = (component of force along the displacement) θ = 60°
× (displacement) The work done by the gardener,
or W = (F cos θ ) (s ) W = F ⋅ s = Fs cos θ = 300 × 30 × cos 60°
1
or W = F⋅s (From the definition of dot product) = 300 × 30 × ⇒W = 4500 J
2
So, work done is a scalar or dot product of F and s. Example 6.2 A body moves a distance of 10 m along a straight
Nature of work in different situations : line under an action of 5 N force. If work done is 25 J, then
(i) If θ is acute, thenW is positive and force tries to find the angle between the force and direction of motion of
increase the speed of the body. the body.
F
Sol. Work is measured by the product of the applied force and
F
θ θ the displacement of the body in the direction of the force.
∴ Work = Applied force × Displacement
s W = (F cos θ ) × s = Fs cos θ
Fig. 6.3 Given, W = 25 J, F = 5 N, s = 10 m
(ii) If θ = 90 °, thenW is zero and there is no change in W 25 1
∴ cos θ = = =
speed of the body due the force. F ⋅ s 5 × 10 2
F F  1
⇒ θ = cos−1  = 60°
θ θ  2
Hence, angle between the force and direction of motion of the
s body is 60°.
Fig. 6.4 Example 6.3 A block of mass F
m = 2 kg is pulled by a force
(iii) If θ is obtuse, thenW is negative and force tries to F = 40 N upwards through a
decrease the speed of the body. height h = 2 m. Find the work
m

F F done on the block by the applied


θ θ mg
force F and its weight mg.
(Take, g = 10 ms −2 )
s Sol. Weight of the block, w = mg = (2 ) (10) = 20 N
Fig. 6.5 Work done by the applied force,
Work is a scalar quantity, its dimensional formula is WF = Fs cos θ = F h cos 0°
[ML2 T –2 ]. SI unit of work is joule and CGS unit is erg. (Here, s = h = 2 m and the angle between force and
displacement is 0°)
1 joule = 10 7 erg or WF = (40) (2) (1) = 80 J
Similarly, work done by its weight,
Gravitational unit of work done is kg-m. Wmg = (mg ) (h ) cos 180°
1 kg-m = 9.8 J or Wmg = (20) (2) (−1) = − 40 J

Some other convenient units of work are Example 6.4 A 10 g block placed on a rough horizontal floor
eV (electron volt), MeV (mega electron volt) and kWh is being pulled by a constant force 50 N. Coefficient of
(kilowatt hour) kinetic friction between the block and the floor is 0.4. Find
−19 work done by each individual force acting on the block over
● 1 eV = 1.6 × 10 J, 1 J = 6.25 × 1018 eV displacement of 5 m.
● 1 MeV = 1.6 × 10 −13 J, 1 J = 6.25 × 1012 MeV F

● 1 kilowatt hour (kWh) = 3.6 × 10 J 6

Sol. Given, mass, m = 10 g


Example 6.1 A lawn roller has been pushed by a gardener
Constant force, F = 50 N
through a distance of 30 m. What will be the work done by
him, if he applies a force of 30 kg-wt in the direction Coefficient of kinetic friction, µ k = 0.4
inclined at 60° to the ground? Displacement, s = 5 m
248 OBJECTIVE Physics Vol. 1

Forces acting on the block are; its weight (mg = 100 N ), For the motion of 2 kg block in downward direction, applying
normal reaction (N = 100 N) from the ground, force due to Newton’s law, we get
kinetic friction (f = µ k mg = 40 N) and the applied force mg − T = ma
(F = 50 N ) which are shown in the given figure. 20 − T = 2a = 2 × 2
⇒ T = 16 N
mg = 100 N
1
F = 50 N
∴ Work done by the string,W = − Ts = − 16 × = − 4 J
s=5m 4
Negative sign indicates the opposite direction of tension and
f = 40 N
N = 100 N displacement of the block.

Work done by the gravity, i.e. weight of the block, Example 6.6 Two unequal masses of 1 kg and 2 kg are
attached at the two ends of a light inextensible string passing
Wg = 0 J (Qmg ⊥ s) over a smooth pulley as shown in figure. If the system is
Work done by the normal reaction, released from rest, find the work done by string on both the
WN = 0 J (Q N ⊥ s) blocks in 1 s. (Take, g = 10 ms −2 )
Work done by the applied force,
WF = 50 × 5 × cos 0° (Q F || s)
= 250 J
Work done by the force of kinetic friction,
Wf = 40 × 5 × cos 180°
Wf = − 200 J (Q F and s are anti-parallel) 1 kg

Example 6.5 A block of mass 2 kg is being brought down by 2 kg


a string. If the block acquires a speed 1 ms −1 in dropping
down 25 cm, find the work done by the string in the process. Sol. Net pulling force acting on the system,
1 F net = 2g − 1g = 20 − 10 = 10 N
Sol. Given, distance moved by block, s = 25 cm = m
4 Total mass being pulled,
Initial velocity of block, u = 0 m = (1 + 2) = 3 kg
Final velocity of block, v = 1 ms−1

1 kg T

2 kg 2 kg a 2 kg a
u=0 1g
From the third equation of motion for block,
2g 20 N
1
v 2 = u 2 + 2as ⇒ (1)2 = 0 + 2a × ⇒ a = 2 ms−2 (i) (ii)
4
∴ Acceleration of the block, a = 2 ms−2 Therefore, acceleration of the system will be
F 10 −2
The free body diagram of the block is as shown below a = net = ms
m 3
Displacement of both the blocks in 1 s is
1 1 10 5
s = at 2 =   (1)2 = m
2 2 3 3
Free body diagram of 2 kg block is shown in Fig. (ii).
T Using Σ F = ma , we get
25 cm
10
2 kg 20 − T = 2a = 2  
a 3
v = 1 ms–1
20 40
mg = 20 N or T = 20 − = N
3 3
Work, Energy and Power 249

∴ Work done by string (tension) on 1 kg block in 1 s is Final position, s2 = (i$ + $j + 2 k$ )m


 40  5 200
Displacement of the particle, s = s2 − s1 = i$ + j$ + 2 k$ − k$
W1 = (T ) (s ) cos 0° =     (1) = J
 3   3 9
⇒ s = (i$ + j$ + k$ )m
Similarly, work done by string on 2 kg block in 1 s will be
and force, F = F1 + F2 = 2i$ + 3$j − k$ + i$ − 2$j + 2 k$
 40  5 200
W2 = (T )(s ) (cos 180° ) =     (−1) = − J
 3   3 9 = (3i$ + j$ + k)N
$
Substituting given values in Eq. (i), we get
Work done on a particle moving in W = (3i$ + j$ + k$ ) ⋅ (i$ + j$ + k$ ) = 3 + 1 + 1 = 5 J
three dimensional space
Let a particle moves in space under the action of a constant
Work done by a variable force
force given by The force is said to be variable force, if it changes its
direction or magnitude or both. The work done by a
F = F $i + F $j + F k
x
$
y z variable force can be calculated as
Let initial position of the particle be r2

s = x $i + y $j + z k
$
W= ∫r 1
F ⋅ dr
1 1 1 1
and final position of the particle be where, integration is performed along the path of particle
s = x $i + y $j + z k $ and dr is the position vector of the particle.
2 2 2 2
If the particle moves from r1 (x 1, y 1, z 1 ) to r2 (x 2, y 2, z 2 ), i.e.
Then, work done by the force F given by when the magnitude and direction of the force vary in three
W = F ⋅ s = F ⋅ (s 2 − s 1 ) dimensions, then the work done by force F, is given by
= [Fx $i + Fy $j + Fz k$ ] ⋅ [(x 2 − x 1 ) $i x2 y2 z2
W = ∫ dW = ∫ Fx dx + ∫ Fy dy + ∫ Fz dz
+ ( y 2 − y 1 ) $j + (z 2 − z 1 ) k
$]
x1 y1 z1

⇒ W = Fx (x 2 − x 1 ) + Fy ( y 2 − y 1 ) + Fz (z 2 − z 1 ) where, Fx , Fy and Fz are the rectangular components of


force in x, y and z-directions, respectively.
Example 6.7 A constant force F = ($i + 3$j + 4k
$ ) N acts on a xf
particle and displaces it from (−1m, 2m, 1m ) to (2m, − 3m, 1m ).
● If the motion is one dimensional,W = ∫xi F x dx
Find the work done by the force.
Here, Fx is the component of force along motion.
Sol. Given, F = ($i + 3$j + 4k$ )N
Example 6.9 A position dependent force F = (7 − 2x + 3x 2 )N
Initial position of the particle is given by acts on a small body of mass 2 kg and displaces it from
s = (− $i + 2$j + k$ )m
1 x = 0 to x = 5 m. Calculate the work done (in joule).
x2 5
Sol. Work done,W = ∫x F dx = ∫0 (7 − 2x + 3x
2
Final position, s 2 = (2$i − 3$j + k$ )m ) dx
1
Displacement of the particle, Here, the body changes its position from x = 0 to x = 5m
s = s2 − s1 = (2$i − 3$j + k) − (–$i + 2$j + k)
$

5
2x 2 3x 3 
⇒ W = 7x − +  = [7(5) − (5) + (5) − 0 ] = 135 J
2 3
= 3$i − 5$j + 0 k$ = (3$i − 5$j)m  2 3 0
Work done by the force F,
Example 6.10 A force F = ( 2 + x ) acts on a particle in
W = F ⋅ s = ($i + 3$j + 4k$ ) ⋅ (3$i − 5$j) = (3 − 15) = − 12 J x-direction, where F is in newton and x in metre. Find the
work done by this force during a displacement from x = 1 m
Example 6.8 A particle is shifted from point (0, 0, 1m ) to to x = 2 m.
(1m, 1m, 2 m ), under simultaneous action of several forces. Sol. As the force is variable, we shall find the work done in a
Two of the forces are F1 = (2i$ + 3j$ − k$ ) N and small displacement from x to x + dx and then integrate it to find
F2 = (i$ − 2j$ + 2k$ ) N . Find work done by resultant of these the total work. The work done in this small displacement is
two forces. dW = F dx = (2 + x ) dx
Sol. Work done by a constant force is equal to dot product of the 2 2
force and displacement vectors.
Thus, W= ∫1 dW = ∫1 (2 + x ) dx

i.e. W = F⋅s …(i)  x2 


2
 4  1 
Initial position of the particle, s = k$ metre = 2x +  =  4 + 2  − 2 + 2  = 3.5 J
1
 2 1  
250 OBJECTIVE Physics Vol. 1

k
Example 6.11 A force F = − (x ≠ 0) acts on a particle in F (N)
x2 B C
x-direction. Find the work done by this force in displacing 2
the particle from x = + a to x = + 2 a. Here, k is a positive
constant.
k A D E
Sol. Given, F = − , where x is the position of particle. 0 d (m)
x2 3 7 12
+2 a  − k = 2 × (7 − 3) +
1
× (12 − 7) × 2
Work done by this force,W = ∫ F dx = ∫+a  2  dx
x  2
+2 a 1
k  k k k =8+ × 10 = 8 + 5 = 13 J
=  = − = − 2
 +a
x 2a a 2 a
Example 6.13 For the force-displacement graph shown below,
Note It is important to note that work comes out to be negative which
calculate the work done by the force in displacing the body
is quite obvious as the force acting on the particle is in negative
 k
from x = 1 cm to x = 5 cm.
x-direction  F = − 2  while displacement is along positive
 x  y
x-direction (from x = a to x = 2 a). 20
10
Calculation of work done by F 0
1 2 3 4 5 6 7 8
(in dyne) x (cm)
force-displacement graph –10
–20
The area under force-displacement curve gives work done.
F Sol. Work done = Area under the curve and displacement axis
= 10 × 1 + 20 × 1 − 20 × 1 + 10 × 1 = 20 erg
Force

W = Area Example 6.14 A force F acting on a particle varies with the


position x as shown in figure. Find the work done by this
x force in displacing the particle from
xi dx xf
Displacement F(N)
Fig. 6.6 Force-displacement graph 10
xf xf
W= ∫x i
dW = ∫x i
F ⋅ dx –2
2
x(m)

where, x i and x f are the initial and final position, respectively.


–10
xf
∴ W= ∫x i
(Area under curve)
(i) x = − 2 m to x = 0 (ii) x = 0 to x = 2 m .
⇒ W = Area under curve between x i and x f . Sol. (i) From x = − 2 m to x = 0, displacement of the particle is
along positive x-direction while force acting on the particle
Example 6.12 Force F acting on a particle moving in a
is along negative x-direction. Therefore, work done is
straight line varies with distance d as shown in the figure. negative and is given by the area under F-x graph.
Find the work done on the particle during its displacement of 1
12 m. ∴ W = − (2) (10) = − 10 J
2
F (N)
(ii) From x = 0 to x = 2 m, displacement of particle and
force acting on the particle both are along positive
2 x-direction. Therefore, work done is positive and is
given by the area under F -x graph.
1
∴ W = (2) (10) = 10 J
0 d (m) 2
3 7 12

Sol. Work done = Area under force-displacement graph Spring block system
= Area of rectangle ABCD + Area of ∆DCE Consider an elastic spring of negligibly small mass having
1
= Length × Breadth + × Base × Height spring constant k with its one end attached to a rigid
2 support and its other end is attached to a block of mass m
1
= (AB × AD ) + × DE × CD that can slide over a smooth horizontal surface.
2
Work, Energy and Power 251

1 2
Suppose a force F is applied on the spring to stretch it Sol. (i) Work done by the spring force, WS = − kx
from natural length to produce an elongation x in it. 2
Here, k = 100 Nm−1 and x = 10 cm = 0.1 m
1
k ∴ WS = − × 100 (0.1)2 = − 0.5 J
F 2
Negative sign indicates that the work done by the spring
Fig. 6.7 Spring block system
force is negative.
(ii) When the block attains equilibrium, its speed is maximum.
The work done in stretching the spring by external applied ∴ Work done on the block by external force F ,
x x  10 
force,W = ∫0 F dx = ∫0 kx dx W = F ⋅ x = (10) 
100
 = 1J

1 2 [QF = kx = 100 × 01
. = 10 N]
W= kx
2 Thus, net work done on the block,
WN = WS +W = − 0.5 + 1 = 0.5 J
The work done by stretching or compressing force is
positive. But the work done by the spring is negative Dependence of work done on
because the force exerted by the spring is always opposite
to elongation or contraction. frame of reference
Work depends on frame of reference. With change of
1
∴ Work done by the spring,W = − kx 2 frame of reference, inertial force does not change while
2 displacement may change.
When length of spring changes from x = x i to x = x f A
s
xf xf 1
W = −∫ Fdx = −∫ kxdx = k (x i2 − x f2 )
xi xi 2
Example 6.15 The work done in extending a spring by x 0 is
Fig. 6.8 Dependence of work done on frame of reference
W0 . Find the work done in further extension x 0 .
So, the work done by a force will be different in different
Fs = kx
F frames. e.g. If a person A is pushing a box inside a moving
bus, then work done as seen by him from the frame of
reference of bus is F ⋅ s while as seen by a person on the
Sol. Force, F = F s = kx ground it is F⋅ (s + s 0 ). Here, s 0 is the displacement of bus
kx 02
x0 relative to ground.
W0 = ∫ F dx =
∫0 kx dx =
2 Example 6.17 A train is moving with a speed of 90 kmh −1. A
LetW be the work done in extending a spring by 2 x 0. passenger X inside the train displaces his 40 kg luggage
2x 0 slowly on the floor through 1 m in 10 s. Coefficient of
W = ∫ F dx = ∫x kx dx friction of the floor of the train is 0.2. Find the work done by
0
this passenger X and the luggage as seen by
k 3
= [(2x 0 )2 − x 02] = kx 02 (i) a fellow passengerY
2 2
(ii) a person on the ground [Take, g = 10 ms −2].
⇒ W = 3W0
Sol. Given, speed of the train,
Example 6.16 Consider a block connected to a light spring of
−1 90 ×1000
spring constant 100 Nm . Now, the block is displaced by v = 90 kmh −1 = = 25 ms−1
applying a constant force F which gives zero resultant force 60 × 60
when spring is stretched through 10 cm. (i) Displacement of the luggage with respect to the train, s = 1m.
As luggage is displaced slowly, the force applied on it
–1
must be same as frictional force on it by the floor,
k = 100 Nm
f = µ mg = (0.2) × 40 × 10 = 80 N
F
Work done by the passenger X as seen by fellow
passengerY ,
Evaluate W = fs = (80) (1) = 80 J
(i) work done by the spring force when the block attains equilibrium. (ii) The luggage is displaced for 10 s. Therefore, distance
(ii) net work done on the block when it attains maximum speed. moved by train with respect to ground during this interval
is
252 OBJECTIVE Physics Vol. 1

s0 = 25 × 10 = 250 m If the force acting on the particle is conservative, then


Therefore, work done by passenger X on the luggage as
seen by a person on the ground, WABCDA = ∫ F ⋅ dL = ∫ F ⋅ dL + ∫ F ⋅ d L = 0
WG = f (s + s0 ) = (80) (1 + 250) ABCDA ABC CDA

= (251) (80) = 20.08 kJ ⇒ WABC + WCDA = 0


⇒ WABC = − WCDA
CONSERVATIVE FORCES ⇒ WI = −WII
A force is said to be conservative, if the work done by or
against the force on a body is independent of path Example 6.18 An object is displaced from point A (2m, 3m, 4m )
to a point B (1m, 2m, 3m ) under a constant force
followed by the body and depends only on initial and final
F = (2$i + 3$j + 4k$ ) N . Find the work done by this force in this
positions. process.
e.g. Gravitational force, spring force, coulomb force, etc. Sol. Work done by force F,
rf (1m, 2 m, 3 m)
Work done by conservative forces W= ∫ri
F ⋅ ds = ∫( 2m, 3m, 4m) (2$i + 3j$ + 4k$ ) ⋅ (dxi$ + dyj$ + dzk$ )
(1m, 2 m, 3 m)
One of the following two equivalent conditions on work = [2x + 3 y + 4z] ( 2m, 3m, 4m) = − 9 J
done must be satisfied by conservative forces
Alternate Solution
(i) Work done by or against a conservative force in
Since, F = constant, we can also use
moving a body from one position to another
W = F⋅ s
depends only on the initial and final positions of
the body. It does not depend upon the nature of Here, s = s f − si = ($i + 2$j + 3k$ ) − (2$i + 3$j + 4k$ )
the path followed by the body in going from initial
= (− $i − $j − k$ ) m
position to the final position.
∴ W = (2$i + 3$j + 4k$ ) ⋅ (− $i − $j − k$ )
II
= −2−3− 4 = − 9J
I Example 6.19 An object is displaced from position vector
A B r1 = (2$i + 3$j) m to r2 = (4$i + 6$j)m under a force
III F = (3x 2 $i + 2 y$j) N. Find the work done by this force.
r2 r2
∫r ∫r
Fig. 6.9 Work done by conservative forces
Sol. Work done,W = F ⋅ dr = (3x 2$i + 2 y$j) ⋅ (dx$i + dy$j + dzk$ )
1 1

LetW1,W2 andW3 denote the net work done in r2

moving a body from A to B along three different


= ∫r 1
(3x 2 dx + 2 y dy ) = [x 3 + y 2](( 42,, 63)) = 83 J

paths I, II and III respectively, as shown in figure. Note In the above two examples, we saw that while calculating the work
If the force is conservative, then done we did not mention the path through which the object was

WI =WII =WIII = ∫ F ⋅ dL
displaced. Only initial and final coordinates were required.
It shows that in both the examples, the work done is path
B B B independent or work done will be equal whichever path we
⇒ ∫ F ⋅ dL = ∫ F ⋅ dL = ∫ F ⋅ dL follow.
Therefore, above two forces in which work done is path
A A A
Path I Path II Path III independent are examples of conservative forces.
(ii) Work done by or against a conservative force in
moving a particle along a closed path NON-CONSERVATIVE FORCES
(round trip) is zero.
A force is said to be non-conservative, if work done by or
I against the force in moving a body depends upon the path
A B between initial and final positions.
e.g. Frictional force, viscous force, air resistance, etc.
D C
II Work done by non-conservative forces
Fig. 6.10
LetW1, W2 andW3 denote the net work done in moving a
Assume that a particle is moving along a closed body from A to B along three different paths 1, 2 and 3
path ABCDA as shown in figure. respectively, as shown in Fig. (a).
Work, Energy and Power 253

If the force is non-conservative, thenW1 ≠ W2 ≠ W3 W1 = 0.3 × 10 sin 30° × 10(− 1) [Q s = l = 10 m]


W1 = − 15 J
2 Work done by force of friction,
1 1 W2 = (µ mg cos θ )s cos 180°
A B A B W2 = 0.15 × 0.3 × 10 cos 30° × 10(− 1)
3 2
W2 = − 3.897 J
(b)
(a) Work done by external force,W3 = F ext × s × cos 0°
Fig. 6.11 W3 = (mg sin θ + µ mg cos θ ) × 10 × 1
`W3 = 18.897 J
Work done by a non-conservative force in a round trip as
shown in Fig. (b) is not zero. Downward journey

i.e. W1 +W2 ≠ 0 µ mg cos 30°

Example 6.20 A body of mass 0.3 kg is taken up an inclined °


30
g sin
plane of length 10 m and height 5 m, and then allowed to m
slide down the bottom again. The coefficient of friction
30°
between the body and the plane is 0.15. What is the
(i) work done by the applied force over the upward journey? mg sin 30° > µ mg cos 30°
(ii) work done by the gravitational force over the round trip? Work done by the gravitational force,
(iii) work done by the frictional force over the round trip? W4 = mg sin 30° × s cos 0°
Which of the above forces (except applied force) is/are 1
conservative forces? W4 = 0.3 × 10 × × 10 × 1 = + 15 J
2
Work done by the frictional force,
m
10 W5 = µmg cos 30° × s cos 180°
l=
h=5m
3
m = 0.15 × 0.3 × 10 × × 10 × (− 1) = − 3.897 J
µ=0.15 2
30° (i) Work done by applied force over upward journey,
Sol. Upward journey Let us calculate work done by different W3 = 18.897 J
forces over upward journey. (ii) Work done by gravitational force over the round trip,
t
F ex W1 + W4 = − 15 + 15 = 0 J
θ (iii) Work done by frictional force over the round trip,
sin
mg W2 + W5 = − 3.897 + (− 3.897) = − 7.794 J
f µ mg cos θ
Work done by gravitational force over a closed path is
30° zero but due to frictional force, it is non-zero.
Work done by gravitational force, Therefore, gravitational force is conservative and
W1 = (mg sin θ )s cos 180° frictional force is non-conservative.

CHECK POINT 6.1


1. The net work done by kinetic friction 4. How much work must be done by a force on 50 kg body in
(a) is always negative (b) is always zero order to accelerate, it in the direction of force from rest to
(c) may be negative or positive (d) is always positive 20 ms −1 in 10 s?
2. A man pushes a wall but fails to move it. He does (a)10−3 J (b)104 J (c) 2 ×103 J (d) 4 ×104 J
(a) negative work
(b) positive but not maximum work 5. The coefficient of friction between the block and plank is µ
(c) maximum positive work and its value is such that block becomes stationary with
(d) no work at all respect to plank before it reaches the other end. Then,
3. A gardener pushes a lawn roller through a distance 20 m. If which of the following is not correct ?
he applies a force of 20 kg-wt in a direction inclined at 60°
m v0
to the ground, the work done by him is
(a) 1960 J (b) 196 J M
(c) 1.96 J (d) 196 kJ
254 OBJECTIVE Physics Vol. 1

(a) The work done by friction on the block is negative. 14. The force F is acting on a particle moving in a straight line
(b) The work done by friction on the plank is positive. as shown in figure. What is the work done by the force on
(c) The net work done by friction is negative. the particle in the 4 m of the trajec tory?
(d) Net work done by the friction is zero. F (N)
6. A horizontal force F pulls a 20 kg box at a constant speed
along a rough horizontal floor. The coefficient of friction 5
between the box and the floor is 0.25. The work done by
force F on the block in displacing it by 2 m is
(a) 49 J (b) 98 J
(c) 147 J (d) 196 J O x (m)
1 3 4
7. A force (3$i + 4 $j) N acts on a body and displaces it by (a) 5 J (b) 10 J (c) 15 J (d) 2.5 J
(3$i + 4 $j) m. The work done by the force is
15. A position dependent force F is acting on a particle and its
(a) 10 J (b) 12 J force-position curve is shown in the figure. Work done on
(c) 16 J (d) 25 J the particle, when its displacement is from 0 to 5 m is
8. A particle moves from point P (1 m , 2 m , 3 m) to F (N)
Q(2 m , 1 m , 4 m) under the action of a constant force + 10
F = (2$i + $j + k$) N. Work done by the force is
x (m)
(a) 2 J (b) 4 J 1 2 3 4 5
(c) 16 J (d) 8 J – 10
9. Work done by a force F = ($i + 2$j + 3 k$) N acting on a particle
in displacing it from the point r1 = ($i + $j + k$ )m to the point (a) 35 J (b) 25 J (c) 15 J (d) 5 J
r2 = ($i − $j + 2 k$ )m is 16. A spring of force constant 800 Nm −1
has an extension of 5 cm.
(a) – 3 J (b) – 1 J The work done in extending it from 5 cm to 15 cm is
(c) zero (d) 2 J (a) 16 J (b) 8 J (c) 32 J (d) 24 J
10. A body constrained to move in the y-direction, is subjected 17. A spring 40 mm long is stretched by the application of a
to a force F = (− 2$i + 15$j + 6 k$) N. What is the work done by force. If 10 N force is required to stretch the spring through
this force in moving the body through a distance of 10 m 1 mm, then the work done in stretching the spring through
along the Y-axis? 40 mm is
(a) 20 J (b) 150 J (a) 84 J (b) 68 J (c) 23 J (d) 8 J
(c) 160 J (d) 190 J 18. A porter with a suitcase of mass 20 kg on his head moves
11. A particle moves along the X-axis from x = 0 to x = 5 m up a staircase upto a height of 4m. The amounts of work
done by the upward lifting force relative to him and relative
under the influence of a force given by F = 10 − 2 x + 3 x .2
to a person on the ground respectively, are
Work done in the process is (a) 0 ; 800J (b) 400J ; 400J
(a) 70 units (b) 270 units (c) 0 ; 400J (d) 800J ; 0
(c) 35 units (d) 150 units
19. For the path PQR in a conservative force field, the amounts
12. A force F = Ay 2 + By + C acts on a body in the y-direction. of work done in carrying a body from P to Q and from Q to R
The work done by this force during a displacement from are 5J and 2J, respectively. The work done in carrying the
y = − a to y = a is body from P to R will be
2Aa 3 P
(a)
3
2Aa 3
(b) + 2Ca
3
2Aa 3 Ba 2
(c) + + Ca
3 2
(d) None of the aboe Q R
13. Force acting on a particle is (2$i + 3$j) N. Work done by this (a) 7J (b) 3J
(c) 21 J (d) zero
force is zero, when a particle is moved on the line
3 y + kx = 5. Here, value of k is 20. Which of the following is a non-conservative force?
(a) 2 (b) 4 (a) Spring force (b) Frictional force
(c) 6 (d) 8 (c) Gravitational force (d) All of these
Work, Energy and Power 255

ENERGY Q
K1 v12 K
= 2 ⇒ 1 =
v2
The energy of a body is defined as its capacity or ability K2 v2 2K1 (v + 2)2
for doing work. Like work, energy is a scalar quantity ⇒ v2 − 4v − 4 = 0
having magnitude only and no direction. The dimensions
of energy are the same as the dimensions of work, i.e. 4 + 16 + 16 4 + 32
This gives, v1 = =
[ML2 T −2 ]. It is measured in the same unit as work, i.e. 2 2
joule in SI and erg in CGS system. ⇒ v1 = 2( 2 + 1) ms−1
Energy can exist in various forms such as mechanical Example 6.22 In a ballistics demonstration, a police officer
energy (potential energy and kinetic energy), sound fires a bullet of mass 50 g with speed 200 ms −1 on soft
energy, heat energy, light energy, etc. plywood of thickness 2 cm. The bullet emerges with only
10% of its initial kinetic energy. What is the emergent speed
of the bullet?
Kinetic energy
1 50
The energy possessed by a body by virtue of its motion is Sol. Initial kinetic energy, Ki = × × 200 × 200 J = 1000 J
called kinetic energy. Kinetic energy of a body can be 2 1000
calculated by the amount of work done in stopping the 10
Final kinetic energy, K f = × 1000 J = 100 J
moving body or from the amount of work done in giving it 100
same velocity from state of rest. If v f is emergent speed of the bullet, then
If an object of mass m has velocity v, then its kinetic 1 50
1 1 × × v f2 = 100
energy is given by KE = mv 2 = mv ⋅ v 2 1000
2 2 ⇒ v f2 = 4000
The kinetic energy of a system having n particles is equal
to the sum of the kinetic energies of all its constituent ⇒ v f = 63.2 ms−1
particles.
Example 6.23 A body of mass 0.8 kg has initial velocity
1 1 1 1
i.e. K = m1v 12 + m 2v 22 + m 3v 32 + ...... + mn v n2 (3i$ − 4j$ ) ms −1 and final velocity (− 6$j + 2k$ ) ms −1. Find
2 2 2 2 change in kinetic energy of the body.
n
1 Sol. Change in kinetic energy,
K = ∑ 2 mi v i2 1 1
∆KE = mv f2 − mvi2
i =1 2 2
Regarding the kinetic energy, the following two points are where, v f = 62 + 22 = 40 ms−1
important to note
and vi = 32 + 42 = 25 ms−1
(i) Since, both m and v 2 are always positive. Hence,
1
kinetic energy is always positive and does not ∴ ∆KE = × 0.8 [ 40 )2 − ( 25 )2]
2
depend on the direction of motion of the body.
= 0.4 [40 − 25] = 0.4(15) = 6 J
(ii) Kinetic energy depends on the frame of reference.
e.g. The kinetic energy of a person of mass m sitting Relation between kinetic energy
in a train moving with speed v is zero in the frame
of train but (1/2) mv 2 in the frame of earth. and linear momentum
The linear momentum of a body is given by p = mv ,
Example 6.21 When a man increases his speed by 2 ms −1, he
finds that his kinetic energy is doubled. Find the original where m is the mass and v is the velocity of a body.
speed of the man. Then, kinetic energy of the body,
Sol. Man possesses kinetic energy because of his velocity (v ). If m 1 1
1 KE = mv 2 = (m 2v 2 )
is mass of man, then K = mv 2 2 2m
2 p2
Given, v1 = v , m1 = m 2 = m KE = or p 2 = 2m KE
2m
When v 2 = ( v + 2) ms− 1, then
K 2 = 2K1 ⇒ Linear momentum, p = 2m KE
256 OBJECTIVE Physics Vol. 1

Example 6.24 Two bodies A and B having masses in the ratio W = ∫ F ⋅ dr = ∫ (F1 + F2 + ...) ⋅ dr
of 3 : 1 possess the same kinetic energy. Obtain the ratio of
linear momentum of B to that of A. = ∫ F1 ⋅ dr + ∫ F2 ⋅ dr + ...
Sol. Kinetic energy of the body is given by
1 where, ∫ F1 ⋅ dr is the work done on the particle by F1 and
EK = mv 2 ...(i)
2 so on. Thus, work-energy theorem can also be stated as
and linear momentum, p = mv ...(ii) work done by the resultant force is equal to the sum
From Eqs. (i) and (ii), we get of the work done by the individual forces.
m 2v 2 p 2 Example 6.26 The position (x) of a particle of mass 1 kg
EK = =  1 
2m 2m moving along X-axis at time t is given by  x = t 2  metre.
 2 
Now, EK1 = EK 2
Find the work done by force acting on it in time interval
p12 p2 p m1 t = 0 to t = 3 s.
⇒ = 2 or 1 =
2m1 2m 2 p2 m2 1 2
Sol. Given, x = t
p1 3 p 1 2
or = or 2 = dx 1
p2 1 p1 3 ⇒ v= = (2 t ) = t
dt 2
Example 6.25 Kinetic energy of a particle is increased by ∴ At t = 0, vi = 0
300%. Find the percentage increase in momentum.
⇒ At t = 3 s, v f = 3 ms−1
1
Sol. Kinetic energy, E = mv 2 According to work-energy theorem,
2
and momentum, p = mv 1 1
W = ∆K = K f − Ki = mv f2 − mvi2
When E is increased by 300%, 2 2
E ′ = E + 3E = 4E  1  1 
=  × 1 × 32 −  × 1 × 02 = 4.5 J
1   2  2 
= 4  mv 2 = 2mv 2
2  Example 6.27 A bullet weighing 10 g is fired with a velocity
If v ′ is velocity of body, then 800 ms −1. After passing through a mud wall 1 m thick, its
1
m (v ′ )2 = 2mv 2 velocity decreases to 100 ms −1. Find the average resistance
2 offered by the mud wall.
⇒ v′ = 2v Sol. According to work-energy theorem, work done by the average
So, p ′ = mv ′ = 2mv resistance offered by the wall = change in kinetic energy of
Hence, percentage change in momentum the bullet.
2mv − mv ∴
1 1
W = F ⋅ s = mv 2 − mu 2
= × 100 = 100%
mv 2 2
m ( v 2 − u 2 ) 0.01 (1002 − 8002 )
⇒ F = = = − 3150 N
Work-energy theorem 2s 2 ×1
This theorem states that work done by all the forces ⇒ Resistance offered = 3150 N
(conservative or non-conservative, external or internal)
Example 6.28 An object of mass 5 kg falls from rest through
acting on a particle or an object is equal to the change in a vertical distance of 20 m and attains a velocity of 10 ms −1.
kinetic energy of it. How much work is done by the resistance of the air on the
∴ W net = ∆KE = K f − K i object? (Take, g = 10 ms −2 )
1 Sol. Applying work-energy theorem, work done by all the
Wnet = m (v f2 − v i2 ) forces = change in kinetic energy
2 1
or Wmg + Wair = mv 2
⇒ Wconservative +Wnon-conservative +Wext.force = ∆KE 2
IfWnet is positive, then kinetic energy will increase and 1 1
∴ Wair = mv 2 − Wmg = mv 2 − mgh
vice-versa. 2 2
Let F1, F2, ... be the individual forces acting on a particle. 1
= × 5 × (10) − (5) × (10) × (20)
2
The resultant force is F = F1 + F2 + ... and the work done by 2
the resultant force is = − 750 J
Work, Energy and Power 257

Example 6.29 A particle of mass m moves with velocity v = 2R { g (1 − cos θ ) + a 0 sin θ )}


v = a x , where a is a constant. Find the total work done by
all the forces during a displacement from x = 0 to x = d. The work done by normal reaction N is zero because it is
always perpendicular to displacement.
Sol. Work done by all forces,
1 1
W = ∆KE = mv 22 − mv12
2 2
Potential energy
The energy possessed by a body or system by virtue of its
Here, v1 = a 0 = 0, v 2 = a d
position or configuration is known as the potential energy.
1 1 e.g. A block attached to a compressed or elongated spring
So, W = ma 2d − 0 = ma 2d
2 2 possesses some energy called elastic potential energy.
This block has a capacity to do work.
Example 6.30 A vehicle of mass m is moving in x-direction
Similarly, a stone when released from a certain height also
with a constant speed. It is subjected to a retarding force
has energy in the form of gravitational potential energy.
F = − 0.1 x Jm −1 during its travel from x = 20 m to
Two charged particles kept at certain distance has electric
x = 30 m. Evaluate the change in its kinetic energy.
potential energy.
Sol. According to work-energy theorem,
The units and dimensions of potential energy are same as
work done = change in kinetic energy of the vehicle that of kinetic energy.
∴ W = K f − Ki or F ⋅ dx = K f − Ki
x = 30
There are mainly two types of potential energies as
or ∫x = 20 (− 0.1)x dx = K f − Ki discussed below
x = 30
x2  1. Gravitational potential energy
or − 0.1   = K f − Ki
 2  x = 20 Gravitational potential energy of a body is the energy
associated with it due to the virtue of its position above
 (30)2 (20)2 
or − 0.1  −  = K f − Ki the surface of earth. If an object of mass m is placed at
 2 2  height h above earth’s surface, then
or K f − Ki = − 0.1 (450 − 200) Gravitational potential energy, U = mgh
∴ K f − Ki = − 25 J
Example 6.31 A block is placed at the top of a smooth Example 6.32 A stone of mass 0.4 kg is thrown vertically up
with a speed of 9.8 ms −1. Find the potential energy after
hemisphere of radius R. Now, the hemisphere is given a
half second.
horizontal acceleration a 0 . Find the velocity of the block
relative to the hemisphere as a function of θ as it slides Sol. Here, mass, m = 0.4 kg, speed, u = 9.8 ms−1,
down. 1
acceleration, a = − g = − 9.8 m/s2 and time, t = s
Sol. In the reference frame of hemisphere, apply a pseudo force 2
on m. From second equation of motion,
1
m h = s = ut + at 2
a0 2
 1  1 1 1
R =  9.8 ×  +  × (–9.8) × ×  = 3.67m
 2   2 2 2
∴ Potential energy = mgh = (0.4) (9.8) (3.67) = 14.386 J
From the work-energy theorem,

u=0 N 2. Elastic potential energy of a spring


ma0 m Potential energy of a spring is the energy associated with
R (1 − cos θ) the state of compression or expansion of an elastic spring.
mg
When the spring is stretched or compressed by an amount x
m

v R
R θ from its unstretched position, then elastic potential energy
of spring,
R sin θ
U = 1/2 kx 2 (where, k = spring constant)
Wtotal = ∆K
Note that elastic potential energy is always positive.
⇒ Wgr + Wpseudo = ∆K
1 2 Natural length of spring indicates reference point, where
mgR (1 − cos θ ) + ma 0R sin θ = mv − 0 potential energy of spring is taken zero.
2
258 OBJECTIVE Physics Vol. 1

Example 6.33 Two springs of spring constants 1500 Nm −1 Change in potential energy
and 3000 Nm −1 respectively are stretched with the same
force, slowly. Compute the ratio of their potential energies. Potential energy is defined for a conservative force field
only. For non-conservative forces, it has no meaning. The
Sol. The work done in pulling the string is stored as potential
energy in the spring which is given by change in potential energy (dU ) of a system corresponding
1 to a conservative internal force is given by
U = kx 2 …(i)  dU 
2 dU = − F ⋅ dr = − dW Q F = − 
where, k is spring constant and x is distance through which it  dr 
is pulled. Uf rf rf
or ∫Ui
dU = − ∫ ri
F ⋅ dr or U f − Ui = − ∫r
i
F ⋅ dr

We generally choose the reference point at infinity and


assume potential energy to be zero there, i.e. if we take
ri = ∞ (infinite) and Ui = 0, then we can write
pulled r
U=− ∫ ∞ F ⋅dr = − W
In case of spring, applied force is given by
F = kx or Potential energy of a body or system is the negative
of work done by the conservative forces in bringing it
where, k is spring constant.
from infinity to the present position.
F
Putting x = in Eq. (i), we get Regarding the potential energy it is worth noting that
k
2
(i) Potential energy should be considered to be a
1 F  F2 1 property of the entire system rather than assigning it
U = k   = ⇒U ∝ …(ii)
2 k 2k k to any specific particle.
U1 k 2 3000 2 (ii) Potential energy depends on frame of reference.
∴ = = = or U1 :U 2 = 2 : 1
U 2 k1 1500 1
For three dimensional motion of the particle
Example 6.34 A block of mass 8 kg is released from the top  ∂U $ ∂U $ ∂U $ 
F=− i+ j+ k
of an inclined smooth surface as shown in figure. If spring  ∂x ∂y ∂z 
constant of spring is 200 Nm −1 and block comes to rest after
compressing spring by 1 m, then find the distance travelled ∂U
where, = partial derivative of U w.r.t. x,
by block before it comes to rest. ∂x
8k ∂U
g = partial derivative of U w.r.t. y
∂y
∂U
and = partial derivative of U w.r.t. z
∂z
30°
Example 6.35 Calculate the work done in lifting a 300 N
Sol. Let d be the distance travelled by block along the plane and weight to a height of 10 m with an acceleration 0.5ms −2 .
h be the height upto which it come in downward direction, (Take, g = 10 ms −2 )
h = d sin 30° Sol. Given, weight, w = 300 N, height, h = 10 m and acceleration,
Now, when block will comes to rest, the decrease in its a = 0.5 ms−2
potential energy will be stored in spring in the form of its
w 300
potential energy. Q Mass, m = = = 30 kg, a net = g + a
1 g 10
∴ mgh = ky 2 (y is compression in spring)
2 ∴ Work done,W = − U = − m (a + g )h
1 = − 30(0.5 + 10)10 = − 3150 J
⇒ mg d sin 30° = ky 2
2 Example 6.36 Two cylindrical vessels of equal cross-sectional
d 1 area A contain water up to heights h1 and h 2 . The vessels
⇒ 8 × 10 × = × 200 × 12 are inter-connected, so that the levels in them are equal.
2 2
Calculate the work done by the force of gravity during the
⇒ 40 d = 100 process. The density of water is ρ.
⇒ d = 2.5 m
Work, Energy and Power 259

Sol. According to the question, we draw the following diagram. M L M


Mass of hanging part of chain = × =
L x x
Now, assume hanging portion of the chain as a point mass
h1 centred at its centre of mass.
h2 h
Therefore, work done to raise the centre of mass of the
Case-I (Before) Case-II (After) chain on the table,
M L MgL
Since, the mass of water remains same,
W= ×g× or W =
x 2x 2x 2
h1 + h 2
ρAh1 + ρAh 2 = ρAh + ρAh ⇒ h = Example 6.38 A uniform chain of length 4m is kept on a table
2
such that a length of 120 cm hangs freely from the edge of
Potential energy of first case, the table. The total mass of the chain is 4 kg. What is the
h h ρAg 2 work done in pulling the entire chain on the table?
U1 = (ρAh1) g 1 + (ρAh 2 ) g 2 = (h1 + h 22 )
2 2 2 Sol. Fraction of length of the chain hanging from the table,
Potential energy of second case, L 10
= 120 cm ⇒ x =
h ρAg (h1 + h 2 )2 x 3
U 2 = (ρAh ) g ×2=
2 4 Work done in pulling the chain on the table,
Since, the gravitational force is a conservative force, then
ρAg
Wgr = U1 − U 2 = (h1 − h 2 )2
4
120 cm
Example 6.37 A particle is moving on frictionless XY-plane.
It is acted upon a conservative force described by the
potential energy function:
1 mgL 4 × 10 × 4
U (x, y, z ) =k (x 2 + y 2 + z 2 ) W= = = 7. 2 J
2 2x 2 2 × (10 / 3)2
where, k = constant Alternative Solution
Derive an expression for the force acting on the particle. Centre of mass of hanging part of the chain,
Sol. Force acting on the particle in x-direction is given by 120
h= = 60 cm = 0.6 m
∂U 1 2
Fx = − = − k (2x + 0 + 0) = − kx
∂x 2 M 4
Mass per unit length of chain, µ = = = 1 kg/m
Similarly, F y = − ky and F z = − kz L 4
∴ Force acting on the particle is Mass of hanging part of chain of length, l = 120 cm = 12
. m
F = F $i + F $j + F k$ = − k (x$i + y$j + zk$ ) = − kr
x y z M ′ = µl = 1 × 12
. = 12
. kg
∴ Work done in pulling the entire chain on the table,
where, r = position vector of the particle = x$i + y$j + zk$
W = Gravitational potential energy
= M ′ gh = 12
. × 10 × 0.6 = 72
. J
Work done in pulling a chain against gravity
Consider a chain of length L and mass M is held on a
frictionless table with L /x of its length hanging over the EQUILIBRIUM
edge. We have to find the work done in pulling the
If a large number of forces act on a system or on an object
hanging portion of the chain on the table.
simultaneously in such a way that the resultant force on it
is zero, then it is said to be in translational equilibrium. If
the forces acting on the object are conservative and it is in
equilibrium, then
dU dU
L/x Fnet = 0 ⇒ − = 0 or =0
dr dr
So, when force is conservative and object is in equilibrium,
slope of U-r graph is zero or its potential energy is either
Fig. 6.12
minimum or maximum or constant.
For this, we can assume centre of mass of hanging portion On this basis, equilibrium of object or a system can be
of the chain at the middle. divided into three types
260 OBJECTIVE Physics Vol. 1

(i) Stable equilibrium An object is said to be in stable At x = a, b , U is minimum ⇒ stable equilibrium


equilibrium, if on slight displacement from x = c, U is maximum ⇒ unstable equilibrium
equilibrium position, it has tendency to come back. x = d , U is constant ⇒ neutral equilibrium
Here, potential energy in equilibrium position is
minimum as compared to its neighbouring points or Example 6.39 The potential energy of a conservative system
d 2U is given by U = ax 2 − bx (where, a and b are positive
= positive. constants). Find the equilibrium position and discuss whether
dr 2
the equilibrium is stable, unstable or neutral.
(ii) Unstable equilibrium An object is said to be in dU
unstable equilibrium, if on slight displacement from Sol. In a conservative field, F = −
dx
equilibrium position, it moves in the direction of
d
displacement. ∴ F =− (ax 2 − bx ) = b − 2ax
dx
In unstable equilibrium, potential energy is
For equilibrium, F = 0 ⇒ b − 2ax = 0
d 2U
maximum or 2 = negative. ∴ x=
b
dr 2a
(iii) Neutral equilibrium An object is said to be in d 2U
From the given equation, we can see that 2 = 2a (positive),
neutral equilibrium, if on displacement from its dx
i.e. U is minimum.
equilibrium position, it has neither the tendency to
b
move in direction of displacement nor to come back Therefore, x = is the stable equilibrium position.
to equilibrium position. In neutral equilibrium, 2a
d 2U Example 6.40 The potential energy for a conservative force
potential energy of the object is constant or 2 = 0. system is given by U = ax 2 − bx, where a and b are
dr
constants. Find out (i) an expression of force (ii) potential
Potential energy curve for equilibrium energy at equilibrium.
E mech = constant is represented by horizontal dotted line in dU
Sol. (i) For conservative force, F = − = − (2ax − b ) = − 2ax + b
the following graph. dx
b
At x, kinetic energy, K = E mech − U (x ) (ii) At equilibrium, F = 0 ⇒ −2ax + b = 0 ⇒ x =
2a
U 2 2
b b b b2 b2
∴ U =a   −b   = − =−
Emech  2a   2a  4a 2a 4a

v
LAW OF CONSERVATION
xmin x0 x xmax OF ENERGY
Fig. 6.13 Energy can neither be created nor be destroyed, it can
The points x = x max and x = x min are called turning points. only be transformed from one form to another form.
At these points, velocity of the particle decreases to zero Conservation of mechanical energy The total
and reverses. mechanical energy (sum of kinetic energy and potential
dU
From x min to x 0 , slope of U (x ) is negative, F = − is energy) of a system is conserved, if the forces acting on it
dx are conservative.
positive and acts towards x 0 .
At x 0 , F = 0. So, x 0 is known as stable equilibrium point. Example 6.41 A body of mass 5 kg is thrown vertically up
with a kinetic energy of 490 J. What will be height at which
Beyond x 0 , slope is positive, indicating a negative force,
the kinetic energy of the body becomes half of the original
towards x 0 . value? (Take, acceleration due to gravity = 9.8ms −2 )
From the PE and position graph, Sol. Given, m = 5 kg and Ki = 490 J, g = 9.8 ms–2
U
From the law of conservation of energy, Ki + U i = Kf + U f
K  Ki 
⇒ Ki + 0 = i + mgh Q K f = 
2  2
490 = 245 + 5 × 9.8 × h
x 490 − 245 245
a c b d ⇒ h= = =5m
Fig. 6.14 5 × 9.8 49
Work, Energy and Power 261

Example 6.42 A bullet of mass m moving with velocity v strikes Example 6.46 Auto manufactures study the collision of cars
a suspended wooden block of mass M and remains embedded with mounted springs of different spring constants. Consider
in it. If the block rises to a height h, find the initial velocity of a car of mass 1500 kg moving with a speed of 36 kmh −1 on
the bullet. a smooth road and colliding with a horizontally mounted
Sol. Initial kinetic energy of the block when the bullet strikes . × 10 3 Nm −1. Find the
spring of spring constant 75
1
maximum compression of the spring.
= (m + M ) v 2 Sol. At maximum compression, KE of car gets converted
2
completely into PE of the spring.
Due to this kinetic energy, the block will rise to a height h. Its
1 1
potential energy = (m + M ) gh KE of car, K = mv 2 = × 1500 × 10 × 10
So, from the law of conservation of energy, 2 2
 5 
1 v2 = 7.5 × 104 J Qv = 36 × = 10 ms−1
(M + m ) v 2 = (M + m ) gh ⇒ = gh  18 
2 2
1 2
Initial velocity of the bullet, v = 2gh U = kx m = K = 7.5 × 104 J
2
Example 6.43 A particle of mass m makes SHM in a smooth 2 × 7.5 × 104
⇒ xm = = 4.47 m
hemispherical bowl ABC and it moves from A to C and back 7.5 × 103
to A via ABC, so that PB = h. Find the speed of the ball
when it just crosses the point B. Example 6.47 A spherical ball of mass 20 kg is stationary at
A P C
the top of a hill of height 100 m. It rolls down a smooth
surface to the ground, then climbs up another hill of height
30 m and finally rolls down to a horizontal base at a height
m h of 20 m above the ground. Find the velocity attained by the
B
ball, when moving at horizontal base.
Sol. According to conservation of energy,
Sol. From law of conservation of energy, PE at A = KE at B
1 1
1 2 mgH = mv 2 + mgh 2 ⇒ mg (H − h 2 ) = mv 2
or mgh = mv or v = 2gh 2 2
2 where, H = height of the first hill,
Example 6.44 A child is swinging on a swing. Minimum and h1 = height of the second hill,
maximum heights of swing from the earth’s surface are 0.75 m h 2 = height of the horizontal base
and 2 m, respectively. What will be the maximum velocity of this and v = velocity attained by the ball.
swing?
Sol. From energy conservation, ⇒ v = 2g (100 − 20) = 2 × 10 × 80 = 40 ms −1
gain in kinetic energy = loss in potential energy
Example 6.48 A smooth narrow tube in the form of an arc
⇒ 2
(1/2) mv max = mg (H 2 − H1) AB of a circle of centre O and radius r is fixed, so that A is
Here,H1 = minimum height of swing from earth’s surface = 0.75 m vertically above O and OB is horizontal. Particles P of mass
and H 2 = maximum height of swing from earth’s surface = 2 m m and Q of mass 2 m with a light inextensible string of length
(π r /2) connecting them are placed inside the tube with P at A
∴ 2
(1/2) mv max = mg (2 − 0.75)
and Q at B and released from rest. Assuming the string
or v max = 2 × 10 × 1.25 = 25 = 5 ms−1 remains taut during motion, find the speed of particles when
P reaches B.
Example 6.45 A machine which is 75% efficient, uses 12 J of
A
energy in lifting 1 kg mass through a certain height. The P
mass is then allowed to fall through the same height. Find
the velocity at the end of its fall. r

Sol. Potential energy of the mass at a height above the earth’s


75 O B
surface = × 12 = 9 J …(i) Q
100
1 Sol. All surfaces are smooth. Therefore, mechanical energy of
Now, kinetic energy of the mass at the end of fall = mv 2 …(ii) the system will remain conserved.
2
Applying law of conservation of energy, ∴ Decrease in PE of both the blocks
= Increase in KE of both the blocks
1 2 2×9 18
mv = 9 ⇒ v = = = 18 ms−1  πr  1 2
2 m 1 ∴ (mgr ) + (2mg )   = (m + 2m )v 2or v = (1 + π )gr
2 2 3
262 OBJECTIVE Physics Vol. 1

Example 6.49 In the arrangement shown in figure, string is between the block and the table. Neglect friction elsewhere.
light and inextensible and friction is absent everywhere. (Take, g = 10 ms −2 )

A
B B

Sol. From constraint relations, we can see that v A = 2 vB


Find the speed of both the blocks after the block A has
ascended a height of 1 m. Given that, m A = 1 kg and Therefore, v A = 2(0.3) (Given, vB = 0.3 ms−1)
m B = 2 kg (Take, g = 10 ms −2 ).
= 0.6 ms−1
Sol. Friction is absent. Therefore, mechanical energy
Applying Wnc = ∆U + ∆K , we get
of the system will remain conserved. From constraint
1 1
relations, we see that speed of both the blocks will be same. − µ m AgsA = − m B gsB + m Av A2 + m BvB2
Suppose it is v. Here, gravitational potential energy of 2 kg 2 2
block is decreasing while gravitational potential energy of 1 kg Here, sA = 2sB (Given, sB = 1 m)
block is increasing. = 2m
Similarly, kinetic energy of both the blocks is also increasing. 1 1
1 1 ∴ − µ (4.0)(10) (2) = − (1)(10)(1) + (4)(0.6)2 + (1) (0.3)2
∴ m B gh = m A gh + m Av 2 + m Bv 2 2 2
2 2
1 1 or − 80 µ = −10 + 0.72 + 0.045
or (2) (10) (1) = (1) (10) (1) + (1) v + (2) v 2
2
or 80 µ = 9.235 or µ = 0.115
2 2
or 20 = 10 + 0.5 v + v
2 2
or 1.5 v 2 = 10 Example 6.52 A 20 kg body is released from rest, so as to
2 −2 −1 slide in between vertical rails and compresses a vertical
∴ v = 6.67 m s
2
or v = 2.58 ms
spring (k = 1920 Nm −1 ) placed at a distance h = 1 m from
Example 6.50 In the arrangement shown in figure, m A = 1kg,
the starting position of the body. The rails offer a frictional
m B = 4 kg. String is light and inextensible while pulley is force of 40 N opposing the motion of body.
smooth. Coefficient of friction between block A and the table Find
is µ = 0.2. Find the speed of both the blocks when block B (i) the velocity v of the body just before striking with the spring,
has descended a height h = 1 m. (Take, g = 10 ms −2 ) (ii) the maximum compression of the spring and
Sol. According to the question, we draw the following diagram.
A
1
2 v
1m 3
B x

Sol. From constraint relation, we see that


v A = vB = v (say)
Force of friction between block A and table will be
f = µm Ag = (0.2) (1) (10) = 2 N (1) (2) (3)
∴ Wnc = ∆U + ∆K
∴ − fs = − m B gh + (1/2) (m A + m B ) v 2 (i) Between diagrams (1) and (2),
1 Loss in PE = Gain in KE + Work done against friction
or (−2) (1) = − (4) (10) (1) + (1 + 4) v 2 1
2
mgh = mv 2 + fh (Q h = s )
−2 = − 40 + 2.5 v 2 2
2.5 v 2 = 38 1
or 20 × 10 × 1 = × 20 v 2 + 40 × 1 ⇒ v = 4 m/s
2
or v 2 = 15.2 m2s−2 or v = 3.9 ms−1
(ii) Between diagrams (2) and (3),
Example 6.51 In the arrangement shown in figure, m A = 4 kg Loss in PE + Loss in KE = Gain in spring energy + Work
and m B = 1 kg. The system is released from rest and block B done against friction
1 1 2
is found to have a speed 0.3 m/s after it has descended mgx + × mv = kx + fx
2

through a distance of 1 m. Find the coefficient of friction 2 2


Work, Energy and Power 263

1 1 So, the law of conservation of mass and law of


20 × 10 × x + × 20 × 42 = × 1920x 2 + 40x
2 2 conservation of energy have been unified by this relation
into a single law of conservation of mass energy.
960x 2 − 160x − 160 = 0
⇒ 6x − x − 1 = 0
2
Nuclear energy
1± 1+ 4 × 6 ×1 1± 5 1 When U 235 nucleus breaks up into lighter nuclei on being
⇒ x= = = m
12 12 2 bombarded by a slow neutron, a tremendous amount of
energy is released. Thus, the energy so released is called
nuclear energy and this phenomenon is known as nuclear
CONVERSION OF MASS fission. Nuclear reactors and nuclear bombs are the
AND ENERGY sources of nuclear energy.
In 1905, Einstein discovered that mass can be converted Example 6.53 Calculate the energy in MeV equivalent to the
rest mass of an electron. Given that the rest mass of an electron,
into energy and vice-versa. He showed that mass and . × 10−31 kg, 1 MeV = 1.6 × 10−13 J and speed of light,
m 0 = 91
energy are equivalent and related by the relation c = 3 × 108ms −1.
E = mc 2 Sol. According to the conversion of mass and energy,
E = m 0c 2 = 9.1 × 10−31 × (3 × 108 )2
where, m is mass that disappears, E is energy that appears
and c is velocity of light in vacuum. Conversely, when = 81.9 × 10−15 J
energy E disappears, a mass m (= E /c 2 ) appears. 81.9 × 10−15
=
Thus, according to modern physics, mass and energy are not 1.6 × 10−13
conserved separately, but are conserved as a single entity = 0.512 MeV
called mass energy.

CHECK POINT 6.2


1. If the force acting on a body is inversely proportional to its 5. If the linear momentum is increased by 50%, then kinetic
speed, the kinetic energy of the body is energy will be increased by
(a) constant (a) 50 % (b) 100 %
(b) directly proportional to time (c) 125 % (d) 25 %
(c) inversely proportional to time 6. The graph between EK and 1/p is
(d) directly proportional to square of time (EK = kinetic energy and p = momentum)
−1
2. If the speed of a vehicle is increased by 1 ms , its kinetic
energy is doubled, then original speed of the vehicle is
(a) √EK (b) √EK
(a) ( 2 + 1) ms−1 (b) 2( 2 − 1) ms−1
(c) 2( 2 + 1) ms−1 (d) 2( 2 + 1) ms−1
1/p 1/p
3. A running man has half the kinetic energy that of a boy
whose mass is half the mass of the man. The man speeds
up by 1 ms −1 and then has the same kinetic energy as that
of boy. The original speeds of man and boy (in ms −1) (c) √EK (d) √EK
respectively, are
(a) ( 2 + 1), ( 2 − 1) (b) ( 2 + 1), 2( 2 + 1)
1/p 1/p
(c) 2, 2 (d) ( 2 + 1), 2( 2 − 1)
7. The kinetic energy acquired by a body of mass m in travelling
4. Two bodies of masses m1 and m2 have same momentum. a certain distance starting from rest under a constant force is
The ratio of their kinetic energy is (a) directly proportional to m
m2 m1
(a) (b) (b) directly proportional to m
m1 m2
m1 m2 (c) inversely proportional to m
(c) (d)
m2 m1 (d) independent of m
264 OBJECTIVE Physics Vol. 1

8. Under the action of a force, a 2 kg body moves such that its 14. A body of mass 5 kg is raised vertically to a height of 10 m by
t3 a force 170 N. The velocity of the body at this height will be
position x as a function of time t is given by x = , where x (a) 9.8 ms−1 (b) 15 ms−1
3
(c) 22 ms−1 (d) 37 ms−1
is in metre and t in second. The work done by the force in
the first two seconds is 15. A body of mass 0.1 kg moving with a velocity of 10 ms −1 hits
(a) 1600 J (b) 160 J a spring (fixed at the other end) of force constant
(c) 16 J (d) 1.6 J
1000 N m−1 and comes to rest after compressing the spring.
9. An object of mass 5 kg is acted upon by a force that varies The compression in the spring is
with position of the object as shown in the figure. If the (a) 0.01 m (b) 0.1 m
object starts out from rest at a point x = 0, what is its speed (c) 0.2 m (d) 0.5 m
at x = 50m? 16. A mass of 2 kg falls from a height of 40 cm on a spring of a
F(N)
force constant 1960 Nm −1 . The spring is compressed by
(Take, g = 9.8 ms −2)
(a) 10 cm (b) 1 cm
10 (c) 20 cm (d) 5 cm
17. In which of the following cases, the potential energy is
x(m) defined?
25 50 (a) Both conservative and non-conservative forces
(a) 12.2 ms−1 (b) 18.2 ms−1 (b) Conservative force only
(c) 16.4 ms−1 (d) 20.4 ms−1 (c) Non-conservative force only
(d) Neither conservative non-conservative forces
10. A block of mass 20 kg is moving in x-direction with a
18. The potential energy of a system increases, if work is done
constant speed of10 ms −1 . It is subjected to a retarding force
(a) by the system against a conservative force
F = (− 01
. x) N during its travel from x = 20 m to (b) by the system against a non-conservative force
x = 30 m. Its final kinetic energy will be (c) upon the system by a conservative force
(a) 975 J (b) 450 J (d) upon the system by a non-conservative force
(c) 275 J (d) 250 J
19. The potential energy for a conservative force system is
11. Velocity-time graph of a particle of mass 2 kg moving in a given by
straight line as shown in figure. Work done by all the forces 7 2
on the particle is U= x − 3x
2
v (ms–1) The potential energy at equilibrium is
9 9
20 (a) + units (b) − units
14 14
13 13
(c) + units (d) − units
2 2
20. A pendulum of length 2 m is left at A. When it reaches B, it
t(s) loses 10% of its total energy due to air resistance. The
2
velocity at B is
(a) 400 J (b) − 400 J (c) − 200 J (d) 200 J
A
12. A particle of mass 0.01 kg travels with velocity given by
(4 $i + 16k$) ms −1 . After sometime, its velocity becomes
(8$i + 20k$) ms −1 . The work done on particle during this B
interval of time is
(a) 0.32 J (b) 6.9 J (a) 6 ms −1 (b) 1 ms −1
(c) 9.6 J (d) 0.96 J (c) 2 ms −1 (d) 8 ms −1
13. A mass of 1 kg is acted upon by a single force 21. A body of mass m thrown vertically upwards attains a
F = (4 $i + 4 $j)N. Under this force, it is displaced from (0, 0) maximum height h. At what height will its kinetic energy
be 75% of its initial value?
to (1m, 1m). If initially, the speed of the particle was 2 ms −1, h h
its final speed should be (a) (b)
6 5
(a) 6 ms−1 (b) 4.5 ms−1 h h
(c) 8 ms−1 (d) 4 ms−1 (c) (d)
4 3
Work, Energy and Power 265

POWER Example 6.54 A train has a constant speed of 40 ms −1 on a


level road against resistive force of magnitude 3 × 10 4 N.
Power is defined as the rate at which work is done or Find the power of the engine.
energy is transferred. If a force does workW in time t,
then its average power is given by Sol. At constant speed, there is no acceleration, so the
forces acting on the train are in equilibrium.
Average power (Pav ) = Rate of doing work
Therefore, F = R = 3 × 104 N, v = 40 ms −1
Work done W Now, power, P = Fv
Average power (Pav ) = = P = 3 × 104 × 40
Time taken t
= 1.2 × 106 W
Power is a scalar quantity because it is the ratio of two
scalar quantities, work (W ) and time (t ). The dimensional Example 6.55 A machine gun fires 240 bullets per minute. If
formula of power is [ML 2 T −3 ]. the mass of each bullet is 10 g and the velocity of the bullets
is 600 ms − 1, then find power (in kW) of the gun.
The SI unit of power is watt (W). The power of an agent is
one watt, if it does work at the rate of 1 joule per second. Sol. Work done by the gun
1 joule = Total kinetic energy of the bullets
1 watt = = 1 Js −1  1  1
1 second = n mv 2 = 240 × × 10 × 10− 3 × (600)2
 2  2
Another popular units of power are kilowatt and horsepower.
= 120 × 10 × 10− 3 × 600 × 600
1 kilowatt = 1000 watt or 1 kW = 10 3 W Work done
∴ Power of gun =
1 horsepower = 746 watt or 1 hp = 746 W Time taken
Horsepower is used to describe the output of automobiles, 120 × 10 × 10− 3 × 600 × 600
=
motorbikes, engines, etc. 60
Note = 72
. kW
(i) Kilowatt hour (kWh) or Board Of Trade (BOT) is the commercial
unit of electrical energy. Example 6.56 In unloading grain from the hold of a ship, an
(ii) Relation between kWh and joule elevator lifts the grain through a distance of 12 m. Grain is
1 kWh = 1 kW × 1 h = 1000 W × 1 h
discharged at the top of the elevator at a rate of 2 kg each
= 1000 Js −1 × 3600 s or 1kWh = 36
. × 106 J
second and the discharge speed of each grain particle is
3 ms −1. Find the minimum horsepower of the motor that can
(iii) Efficiency of an engine, η =
Output power
elevate grain in this way. (Take, g = 10 ms −2 )
Input power
Sol. The work done by the motor each second, i.e.
1
Power = mgh + mv 2, as t = 1 s.
Instantaneous power 2
The instantaneous power of an agent is defined as the Given, m = 2 kg, v = 3 ms−1 and h = 12 m
limiting value of the average power of the agent in a small 1 249
time interval, i.e. when the time interval approaches to zero. ∴ Power = 2 × (10) × (12) + (2) (3)2 = 249 W =
2 746
When work done by a force F for a small displacement dr = 0.33 hp
is dW = F ⋅ dr, then instantaneous power can be given as The motor must have an output of at least 0.33 hp.
∆W dW
P = lim = Example 6.57 A pump can take out 7200 kg of water per
∆t → 0 ∆ t dt
hour from a well 100 m deep. Calculate the power of the
Now, dW = F ⋅ d r pump, assuming that its efficiency is 50%.
(Take, g = 10 ms −2 )
dr
∴ P = F⋅ mgh 7200 × 10 × 100
dt Sol. Output power = = = 2000 W
t 3600
Again d r/dt = v, instantaneous velocity of the agent. Therefore, Efficiency, η =
Output power
P = F ⋅ v = Fv cosθ Input power
where, θ is the angle between F and v. Input power =
Output power
Power is zero, if force is perpendicular to velocity. e.g. η
Power of a centripetal force is zero in a circular motion. 2000 × 100
= = 4 kW
50
266 OBJECTIVE Physics Vol. 1

Example 6.58 A block of mass m is pulled by a constant Sol. Power, P = constant


power P placed on a rough horizontal plane. The coefficient Work done upto time t,W = Pt
of friction between the block and surface is µ. Find the 1
maximum velocity of the block. From work-energy theorem,W = ∆KE or Pt = mv 2
2
Sol. Power, P = F ⋅v = constant 1/ 2
 2Pt 
P 1 ∴ v= 
F = or F ∝ m 
v v
As v increases, F decreases. Example 6.62 A train of mass 2 × 10 5 kg has a constant
When F = µmg, net force on block becomes zero, i.e. it has 1
maximum or terminal velocity. speed of 20 ms −1 up a hill inclined at θ = sin −1   to the
 50
P
∴ P = ( µmg ) v max or v max = horizontal when the engine is working at 8 × 10 5 W. Find
µmg
the resistance to motion of the train. (Take, g = 9.8 ms −2 )
Example 6.59 The force required to row a boat at constant
Sol. Power, P = Fv
velocity is proportional to the speed. If a speed of 4 kmh −1
requires 7.5 kW, how much power does a speed of P 8 × 105
⇒ F = = = 4 × 104 N
12 kmh −1 require? v 20
Sol. Let the force be F = αv, where v is speed and α is a At constant speed, the forces acting on the train are in
constant of proportionality. The power required is equilibrium. Resolving the forces parallel to the hill,
F
P = Fv = αv 2
Let P1 be the power required for speed v1 and P2 be the power ain
Tr
required for speed v 2. R θ mg cos θ
sin mg
P1 = 7.5 kW and v 2 = 3 v1 mg

v 
2 F = R + mg sin θ
⇒ P2 =  2  P1 1
 v1  ⇒ F = R + (2 × 105 ) g ×
50
⇒ P2 = (3)2 × 7.5 kW = 67.5 kW 4 × 104 = R + 39200 or R = 800 N
Example 6.60 An engine pumps 400 kg of water through Therefore, the resistance is 800 N.
height of 10 m in 40 s. Find the power of the engine, if its
Example 6.63 A small body of mass m moving with velocity
efficiency is 80%. (Take, g = 10 ms −2 )
v 0 on rough horizontal surface, finally stops due to friction.
Sol. Work done by engine against gravity, Find the mean power developed by the friction force during
W = mgh = 400 × 10 × 10 = 40 kJ the motion of the body, if the frictional coefficient, µ = 0.27,
Power used by engine (output power) m = 1 kg and v 0 = 1.5 ms −1.
W 40 × 103 Sol. The retardation due to friction,
= = W = 1 kW
∆t 40 Force of friction µmg
a= = = µg
If power of the engine is P (input power), then its efficiency, Mass m
Output power Now, v 0 = at
η= v v
Input power Therefore, t= 0 = 0 K (i)
a µg
1 kW
Q P= From work-energy theorem,
80 %
work done by force of friction = change in kinetic energy
1000 × 100
⇒ P= W 1
80 or W = mv 02 K (ii)
2
100
= kW W
80 Mean power =
t
= 1.25 kW From Eqs. (i) and (ii), we get
1
Example 6.61 An automobile of mass m accelerates, starting Pmean = µmgv 0
from rest. The engine supplies constant power P, show that 2
1/ 2 Substituting the values in above equation, we get
 2Pt 
the velocity is given as a function of time by v =   . 1
Pmean = × 0.27 × 1.0 × 9.8 × 1.5 ≈ 2 W
m  2
CHECK POINT 6.3
1. A particle of mass M starting from rest undergoes uniform of energy. How much power is generated by the turbine?
acceleration. If the speed acquired in time t is v, the power (Take , g = 10 ms −2 )
delivered to the particle is (a) 12.3 kW (b) 7 kW
Mv2 1 Mv2 (c) 8.1 kW (d) 10.2 kW
(a) (b)
t 2 t2 5. If the heart pushes 1 cc of blood in one second under
Mv2
(c) 2 (d)
1 Mv2 pressure 20000 Nm −2, the power of heart is
t 2 t (a) 0.02 W (b) 400 W
(c) 5 × 10−10 W (d) 0.2 W
2. An engine develops 10 kW of power. How much time will it
take to lift a mass of 200 kg to a height of 40 m? 6. A body of mass 10 kg moves with a constant speed v of
(Take, g = 10 ms −2 ) 2 ms −1 along a circular path of radius 8 m. The power
(a) 4 s (b) 5 s
produced by the body will be
(c) 8 s (d) 10 s (a)10 Js−1 (b) 98 Js−1
(c) 49 Js−1 (d) zero
3. An engineer claims to make an engine delivering 10 kW
power with fuel consumption of 1 gs −1. The calorific value of 7. A force of (2$i + 3$j + 4 k$) N acting on a body for 4 s, produces
fuel is 2 kcal g −1. This claim is a displacement of (3$i + 4 $j + 5k$) m. The power used is
(a) valid (a) 9.5 W (b) 7.5 W
(b) invalid (c) 6.5 W (d) 4.5 W
(c) dependent on engine design 8. A body of mass 2 kg is projected at 20 ms −1 at an angle 60°
(d) dependent on load above the horizontal. Power due to the gravitational force at
4. Water falls from a height of 60 m at the rate of 15 kg/s to its highest point is
operate a turbine. The losses due to frictional forces are 10% (a) 200 W (b)100 3 W (c) 50 W (d) zero
Chapter Exercises
(A) Taking it together
Assorted questions of the chapter for advanced level practice

1 A body is falling freely under the action of gravity 7 A body of mass m was slowly pulled up the hill by a
alone in vacuum. Which of the following quantities force F which at each point was directed along the
remain constant during the fall? [NCERT Exemplar] tangent of the trajectory. All surfaces are smooth.
(a) Kinetic energy (b) Potential energy Find the work performed by this force.
(c) Total mechanical energy (d) Total linear momentum
2 An electron and a proton are moving under the
influence of mutual forces. In calculating the change F
h
in the kinetic energy of the system during motion, m
one ignores the magnetic force of one on another. l
This is because [NCERT Exemplar] (a) mgl (b) – mgl
(a) the two magnetic forces are equal and opposite, so they (c) mgh (d) zero
produce no net effect
(b) the magnetic forces do not work on each particle 8 Two masses of 1 g and 4 g are moving with equal
(c) the magnetic forces do equal and opposite (but kinetic energies. The ratio of the magnitudes of their
non-zero) work on each particle momenta is
(d) the magnetic forces are necessarily negligible (a) 4 : 1 (b) 2:1 (c) 1 : 2 (d) 1 : 16
3 An engine exerts a force F = (20 $i − 3 $j + 5k
$ ) N and
9 If v, p and E denote velocity, linear momentum and
$ ) ms −1. The
moves with velocity v = (6$i + 20 $j − 3 k kinetic energy of the particle respectively, then
power of the engine (in watt) is dE dE
(a) p = (b) p =
(a) 45 (b) 75 (c) 20 (d) 10 dv dt
dv dE dE
4 A rod of mass m and length l is lying on a horizontal (c) p = (d) p = ×
table. Work done in making it stand on one end will dt dv dt
be 10 The energy required to accelerate a car from rest to
(a) mgl (b)
mgl
(c)
mgl
(d) 2mgl 10 ms −1 isW. The energy required to accelerate the
2 4 car from 10 ms −1 to 20 ms −1 is
5 A bicyclist comes to a skidding stop in 10 m. During (a) W (b) 2W
this process, the force on the bicycle due to the road is (c) 3W (d) 4W
200 N and is directly opposed to the motion. The work 11 Which of the following diagrams most closely shows
done by the cycle on the road is [NCERT Exemplar] the variation in kinetic energy of the earth as it
(a) + 2000 J (b) − 200 J (c) zero (d) − 20000 J moves once around the sun in its elliptical orbit?
6 The pointer reading versus load graph for a spring [NCERT Exemplar]
balance is as shown in the figure. KE KE
Extension (cm)

(a) (b)

10 t

KE KE
Load (kgf)
1.0
The spring constant is (c) (d)
(a) 15 kgf/cm (b) 5 kgf/cm
(c) 0.1 kgf/cm (d) 10 kgf/cm
Work, Energy and Power 269

12 A particle is moved from (0, 0) to (a, a ) under a force If there are no frictional forces acting on the particle,
F = (3 $i + 4$j) from two paths. Path 1 is OP and path the graph will look like
2 is OQP. LetW1 andW2 be the work done by this
force in these two paths. Then,
y W
(a) W (b)
P (a, a)

v v

45°
x
O Q
(c) W (d) W
(a) W1 = W2 (b) W1 = 2W2 (c) W2 = 2W1 (d) W2 = 4W1
13 A body moves from rest with a constant v v
acceleration. Which one of the following graphs
represents the variation of its kinetic energy K with 17 A car moving with a speed of 40 kmh −1 can be
the distance travelled x? stopped by applying brakes in 2 m. If the car is
K K
moving with a speed of 80 kmh −1, the minimum
stopping distance under similar brakes will be
(a) (b) (a) 8 m (b) 2 m (c) 4 m (d) 6 m
18 A long spring is stretched by 2 cm. Its potential
O x
O
x energy is U. If the spring is stretched by 10 cm, its
potential energy would be
K K U U
(a) (b) (c) 5U (d) 25U
25 5
(c) (d)
19 A body is falling under gravity. When it loses a
x x gravitational potential energy by U, its speed is v.
O O The mass of the body will be
14 Which of the following diagrams represents variation 2U U 2U U
(a) (b) (c) (d)
of total mechanical energy of a pendulum oscillating v 2v v2 2v 2
in air as function of time? [NCERT Exemplar] 20 How much mass is converted into energy per day in
E E Tarapur nuclear power plant operated at 10 7 kW?
(a) (b)
(a) 9.6 g (b) 9.63 kg (c) 8.6 g (d) 7 g

t 21 A proton is kept at rest. A positively charged particle


t
is released from rest at a distance d in its field.
Consider two experiments: one in which the charged
E E particle is also a proton and in another a positron. In
the same time t, the work done on the two moving
charged particles is [NCERT Exemplar]
(c) (d)
(a) same as the same force law is involved in the two
t
t experiments
(b) less for the case of a positron, as the positron moves
away more rapidly and the force on it weakens
15 An object of mass m, initially at rest under the action (c) more for the case of a positron, as the positron moves
of a constant force F attains a velocity v in time t. away a larger distance
Then, the average power supplied to mass is (d) same as the work done by charged particle on the
mv 2 Fv stationary proton
(a) (b)
2t 2 22 The potential energy function for a particle
(c) Both (a) and (b) (d) None of these 1 2
executing linear SHM is given byV (x ) = kx
16 A particle at rest on a frictionless table is acted upon 2
by a horizontal force which is constant in magnitude where, k is the force constant of the oscillator (see
and direction. A graph is plotted for the work done on figure). For k = 0.5 Nm −1, the graph ofV (x ) versus x
the particleW against the speed of the particle v. is shown in the figure.
270 OBJECTIVE Physics Vol. 1

A particle of total energy E turns back when it 28 The system shown in the figure is released from rest.
reaches x = ± xm . IfV and K indicate the PE and KE At the instant when mass M has fallen through a
respectively of the particle at x = + xm , then which distance h, the velocity of m will be
of the following is correct? [NCERT Exemplar]
V(x)
M>m

x
–xm xm m M
(a) V = 0, K = E (b)V = E , K = 0 2ghM
(c) V < E , K = 0 (d) V = 0, K < E (a) 2gh (b)
m
23 The potential energy between the atoms in a 2gh (M − m ) 2gh (M + m )
a b (c) (d)
molecule is given by U (x ) = − m +M m −M
x 12 x 6
29 The curved portions are smooth and horizontal
where, a and b are positive constants and x is the
surface is rough. The block is released from P. At
distance between the atoms . The atom is in
what distance from A, it will stop (if µ = 0.2)?
equilibrium when
1/ 6 P
a
(a) x = 0 (b) x =   h =1m
 2b 
1/ 6 1/ 6 A B
 2a  11a 
(c) x =   (d) x =   2m
b  5b 
(a) 1m (b) 2 m (c) 3 m (d) 4 m
24 A ball is thrown vertically upwards with a velocity
30 A particle moves on a rough horizontal ground with
of 10 ms −1. It returns to the ground with a velocity
3
of 9 ms −1. If g = 9.8 ms −2 , then the maximum height some initial velocity, say v 0 . If   th of its kinetic
 4
attained by the ball is nearly (assume air resistance
to be uniform) energy is lost due to friction in time t 0 , then
(a) 5.1 m (b) 4.1 m (c) 4.61 m (d) 5 m
coefficient of friction between the particle and the
ground is
25 A body is moving down an inclined plane of slope v0 v0 3v 0 v0
37°. The coefficient of friction between the body (a) (b) (c) (d)
2gt 0 4gt 0 4gt 0 gt 0
and plane varies as µ = 0.3 x, where x is the distance
travelled down the plane by the body. The body will 31 If a body of mass 200 g falls from a height 200 m
3 and its total potential energy is converted into
have maximum speed (Take, g = 10 ms −2, sin 37° = ) kinetic energy at the point of contact of the body
5
(a) at x = 1.16 m (b) at x = 2 m with the surface, then decrease in potential energy
(c) at bottom of plane (d) at x = 2.5 m of the body at the contact is (Take, g = 10 ms −2 )
26 A mass-spring system oscillates such that the mass (a) 900 J (b) 600 J
moves on a rough surface having coefficient of (c) 400 J (d) 200 J
friction µ. It is compressed by a distance a from its 32 A stone of mass 2 kg is projected upwards with
normal length and on being released, it moves to a kinetic energy of 98 J. The height at which the
distance b from its equilibrium position. The kinetic energy of the body becomes half of its
decrease in amplitude for one half-cycle (−a to b ) is original value will be (Take, g = 9.8ms −2 )
µmg 2µmg µg k
(a) (b) (c) (d) (a) 5 m (b) 2.5 m (c) 1.5 m (d) 0.5 m
k k k µmg
33 A toy gun uses a spring of very large value of force
27 A block of mass 5 kg slides down a rough inclined constant k. When charged before being triggered in
surface. The angle of inclination is 45°. The the upward direction, the spring is compressed by a
coefficient of sliding friction is 0.20 . When the small distance x. If mass of shot is m, on being
block slides 10 cm, the work done on the block by triggered, it will go upto a height of
force of friction is kx 2 x2 kx 2 (kx )2
1 (a) (b) (c) (d)
(a) − J (b) 1J (c) − 2 J (d) −1 J mg kmg 2mg mg
2
Work, Energy and Power 271

34 Water falling from a 50 m high fall is to be used for 41 Power supplied to a particle of mass 2 kg varies with
generating electrical energy. If 1.8 × 10 kg of water
5
3t 2
time as P = W. Here, t is in second. If particle is
falls per hour and half the gravitational potential 2
energy can be converted into electrical energy, how rest at t = 0, then velocity of particle at time t = 2 s
many 100 W bulb can be lit? will be
(a) 50 (b) 125 (c) 150 (d) 200 (a) 1 ms −1 (b) 4 ms −1 (c) 2 ms −1 (d) 2 2 ms −1
35 Given that the displacement of the body (in metre) is 42 An open knife edge of mass m is dropped from a
a function of time as follows height h on a wooden floor. If the blade penetrates
x = 2t 4 + 5 upto the depth d into the wood, the average
The mass of the body is 2 kg. What is the increase resistance offered by the wood to the knife edge is
2
in its kinetic energy one second after the start of  h  h
motion? (a) mg 1 +  (b) mg 1 + 
 d  d
(a) 8 J (b) 16 J (c) 32 J (d) 64 J
 h  d
(c) mg 1 −  (d) mg 1 + 
36 An object of mass m is  d  h
tied to a string of length L
θ L 43 If v be the instantaneous velocity of the body
and a variable horizontal
force is applied on it dropped from the top of a tower, when it is located
which starts at zero and at height h, then which of the following remains
m F
gradually increases until constant?
the string makes an angle θ with the vertical. v2 v2
(a) gh + v 2 (b) gh + (c) gh − (d) gh − v 2
Work done by the force F is 2 2
(a) mgL (1 − sin θ ) (b) mgL 44 A block of mass 1 kg slides down a rough inclined
(c) mgL (1 − cos θ ) (d) mgL (1 + cos θ ) plane of inclination 60° starting from its top. If
37 A body is attached to the lower end of a vertical coefficient of kinetic friction is 0.5 and length of the
spiral spring and it is gradually lowered to its plane d = 2 m, then work done against friction is
equilibrium position. This stretches the spring by a (a) 2.45 J (b) 4.9 J (c) 9.8 J (d) 19.6 J
length d. If the same body attached to the same 45 A raindrop falling from a height h above ground,
spring is allowed to fall suddenly, what would be the attains a near terminal velocity when it has fallen
maximum stretching in this case? through a height (3/4)h. Which of the diagrams
(a) d (b) 2d (c) 3d (d) (1/2) d shown in figure correctly shows the change in
38 Kinetic energy of a particle moving in a straight line kinetic and potential energy of the drop during its
varies with time t as K = 4t 2 . The force acting on fall up to the ground? [NCERT Exemplar]

the particle h PE PE
(a) is constant (b) is increasing
(a) (b)
(c) is decreasing h/4
(d) first increases and then decreases KE
KE
t t
39 Three particles A, B and C are projected from the
top of a tower with the same speed. A is thrown h
straight upwards, B straight down and C PE h PE
horizontally. They hit the ground with speeds v A , v B (c) (d)
and v C , then which of the following is correct? KE
KE
(a) v A = vB > vC (b) v A = vB = vC O t O t
(c) v A > vB = vC (d) vB > vC > v A
46 In a shotput event, an athlete throws the shotput of
40 A particle is moving in a conservative force field mass 10 kg with an initial speed of 1 ms −1 at 45°
from point A to point B.U A and UB are the potential from a height 1.5 m above ground. Assuming air
energies of the particle at points A and B; andWC is resistance to be negligible and acceleration due to
the work done by conservative forces in the process gravity to be 10 ms −2 , the kinetic energy of the
of taking the particle from A and B. Which of the shotput when it just reaches the ground will be
following is true? [NCERT Exemplar]
(a) WC = U B − U A (b) WC = U A − U B (a) 2.5 J (b) 5 J
(c) U A > U B (d) U B > U A (c) 52.5 J (d) 155 J
272 OBJECTIVE Physics Vol. 1

47 A block of mass m is attached to two unstretched 51 The force required to stretch a F


springs of spring constants k, each as shown. The spring varies with the distance as
A
block is displaced towards right through a distance x shown in the figure . If the
and is released. experiment is performed with
The speed of the block as it passes through the mean the above spring of half the
position will be length, the line OA will O x
(a) shift towards F -axis
(b) shift towards X-axis
m
(c) remain as it is
k k (d) become double in length
52 A motor drives a body along a straight line with a
m 2k m 2k constant force. The power P developed by the motor
(a) x (b) x (c) x (d) x
2k m k m must vary with time t as
48 A particle of mass 1 g executes an oscillatory motion
on the concave surface of a spherical dish of radius
2m placed on a horizontal plane. If the motion of the (a) P (b) P
particle begins from a point on the dish at a height of
1 cm from the horizontal plane, the total distance
covered by the particle before it comes to rest, is t t
(Curved surface is smooth and µ = 0.01 for
horizontal surface)
(a) 2 cm (b) 10 cm (c) 1 cm (d) 20 cm (c) P (d) P
49 The net work done by the tension in the figure when
the bigger block of mass M touches the ground is t t

53 A force F acting on a body depends on its


displacement s as F ∝ s −1/ 3 . The power delivered by
F will depend on displacement as
T (a) s 2 / 3 (b) s −5 / 3 (c) s1/ 2 (d) s 0
m 54 The force acting on a body moving along X-axis
M varies with the position of the particle as shown in
d the figure. The body is in stable equilibrium at
F

(a) + Mgd (b) − (M + m )gd (c) − mgd (d) zero


50 A block A of mass M rests on a wedge B of mass 2M
and inclination θ. There is sufficient friction x1 x2 x
between A and B, so that A does not slip on B. If
there is no friction between B and ground, the (a) x = x1 (b) x = x 2
compression in spring is (c) Both x1 and x 2 (d) Neither x1 nor x 2
B 55 A body of mass 0.5 kg travels in a straight line with
M velocity v = a x 3/ 2 , where a = 5 m− 1/ 2 s −1. The work
k
A done by the net force during its displacement from
2M
x = 0 to x = 2 m is [NCERT Exemplar]
θ (a) 1.5 J (b) 50 J
(c) 10 J (d) 100 J
Mg cos θ Mg cos θ sin θ
(a) (b) 56 The figure shows a particle sliding on a frictionless
k k
Mg sin θ track, which terminates in a straight horizontal
(c) (d) zero section.
k
Work, Energy and Power 273

If the particle starts slipping from the point A, how 61 A force of F = 0.5 N is applied on lower block as
far away from the track will the particle hit the shown in figure . The work done by lower block on
ground? upper block for a displacement of 3 m of the upper
A block with respect to ground is (Take, g = 10 ms −2 )
1m
0.5 m µ = 0.1

1 kg
(a) 1 m (b) 2 m (c) 3 m (d) 4 m
57 A uniform chain has a mass M and length L. It is 2 kg F
placed on a frictionless table with length l 0 hanging Smooth
over the edge. The chain begins to slide down . Then,
the speed v with which the end slides down from the (a) − 0.5 J (b) 0.5 J
edge is given by (c) 2 J (d) − 2 J

g g 62 A body is moving unidirectionally under the


(a) v = (L + l 0 ) (b) v = (L − l 0 ) influence of a source of constant power supplying
L L
energy. Which of the following diagrams shown in
g 2 2
(c) v = (L − l 0 ) (d) v = 2g (L − l 0 ) figure correctly represents the displacement-time
L curve for its motion? [NCERT Exemplar]
58 A bead of mass 1/2 kg starts from rest from A to B d d
move in a vertical plane along a smooth fixed
quarter ring of radius 5 m, under the action of a (a) (b)
constant horizontal force F = 5 N as shown in figure.
The speed of bead as it reaches the point B is t t
(Take, g = 10 ms −2 ) d
d
A

(c) (d)

t t
R=5m
63 A small block of mass m is kept on a rough inclined
B surface of inclination θ fixed in an elevator. The
(a) 14.14 ms −1 (b) 7.07 ms −1 (c) 5 ms −1 (d) 25 ms −1 elevator goes up with a uniform velocity v and the
block does not slide on the wedge.
59 A car of mass m is accelerating on a level smooth
road under the action of a single force F. The power The work done by the force of friction on the block
delivered to the car is constant and equal to P. If the in a time t will be
(a) zero (b) mgvt cos2 θ
velocity of the car at an instant is v, then after
1
travelling how much distance will it become double? (c) mgvt sin2 θ (d) mgvt sin 2θ
2
m
F 64 A pendulum of mass 1 kg and length l = 1m is
released from rest at angle θ = 60 °. The power
delivered by all the forces acting on the bob at angle
θ = 30 ° will be (Take, g = 10 ms −2 )
7mv 3 4 mv 3 mv 3 18 mv 3
(a) (b) (c) (d) (a) 13.5 W (b) 20.4 W
3P 3P P 7P (c) 24.6 W (d) zero
60 A particle is released from a height H. At certain 65 A uniform flexible chain of mass m and length l
height, its kinetic energy is two times its potential hangs in equilibrium over a smooth horizontal pin of
energy. Height and speed of particle at that instant negligible diameter. One end of the chain is given a
are small vertical displacement, so that the chain slips
H 2gH H gH over the pin. The speed of chain when it leaves pin is
(a) , (b) ,2
3 3 3 3 gl
(a) (b) gl (c) 2gl (d) 3gl
2H 2gH H 2
(c) , (d) , 2gH
3 3 3
274 OBJECTIVE Physics Vol. 1

66 The potential energy of a particle of mass 1 kg is of friction is µ, then the work done by the applied
U = 10 + (x − 2) . Here, U is in joule and x in metre
2 force is
on the positive X-axis. Particle travels upto x = + 6 m. F
Choose the correct option. θ
(a) On negative X-axis, particle travels upto x = − 2 m
(b) The maximum kinetic energy of the particle is 16 J
(c) Both (a) and (b) are correct µmgd µmgd cos θ
(d) Both (a) and (b) are incorrect (a) (b)
cos θ + µ sin θ cos θ + µ sin θ
67 A plank of mass 10 kg and a block of mass 2 kg are µmgd sin θ µmgd cos θ
(c) (d)
placed on a horizontal plane as shown in the figure. cos θ + µ sin θ cos θ − µ sin θ
There is no friction between plane and plank. The
coefficient of friction between block and plank is 69 An ideal massless spring S can be compressed 1 m
0.5. A force of 60 N is applied on plank horizontally. by a force of 100 N in equilibrium. The same spring
is placed at the bottom of a frictionless inclined
In first 2 s, the work done by friction on the block is
plane inclined at 30° to the horizontal. A 10 kg
2 kg block M is released from rest at the top of the incline
and is brought to rest momentarily after compressing
10 kg 60 N the spring by 2 m. If g = 10 ms −2 , what is the speed
of mass just before it touches the spring?
(a) − 100 J (b) 100 J M
(c) zero (d) 200 J
S
68 A block of mass m is pulled along a horizontal h
surface by applying a force at an angle θ with the 30°
horizontal. If the block travels with a uniform
velocity and has a displacement d and the coefficient (a) 20 ms −1 (b) 30 ms −1 (c) 10 ms −1 (d) 40 ms −1

(B) Medical entrance special format questions


Assertion and reason 3 Assertion If the surface between the blocks A and B
is rough, then work done by friction on block B is
Directions (Q. Nos. 1-6) These questions consists of two always negative.
statements each printed as Assertion and Reason. While
answering these questions you are required to choose any A
one of the following four responses
(a) If both Assertion and Reason are correct and Reason is the B F
correct explanation of Assertion. Smooth
(b) If both Assertion and Reason are correct but Reason is not
the correct explanation of Assertion. Reason Total work done by friction in both the
(c) If Assertion is correct but Reason is incorrect. blocks is always zero.
(d) If Assertion is incorrect but Reason is correct. 4 Assertion Force applied on a block moving in one
1 Assertion At stable equilibrium position of a body, dimension is producing a constant power, then the
kinetic energy cannot be zero. Because it is maximum. motion should be uniformly accelerated.
Reason During oscillations of a body, potential Reason This constant power multiplied with time is
energy is minimum at stable equilibrium position. equal to the change in kinetic energy.
2 Assertion If work done by conservative force is 5 Assertion Total work done by spring may be
negative, then potential energy associated with that positive, negative or zero.
force should increase. Reason Direction of spring force is always towards
Reason This is according to the relation, ∆U = − W . mean position.
Here, ∆U is change in potential energy andW is 6 Assertion At any instant, the magnitude of rate of
work done by conservative force. change of potential energy of the projectile of mass
Work, Energy and Power 275

1 kg is numerically equal to magnitude of a ⋅ v 5 Which of the following statement(s) is/are correct?


(where, a is acceleration due to gravity and v is I. If momentum of a body increases by 50%, its
velocity at that instant) kinetic energy will increase by 125%.
Reason The graph representing power delivered II. Kinetic energy is proportional to square of
by the gravitational force acting on the velocity.
projectile with time will be straight line with (a) Only I (b) Only II
negative slope. (c) Both I and II (d) Neither I nor II

Statement based questions Match the columns


1 A body is moved along a straight line by a machine
1 Mark out the correct statement(s).
(a) Total work done by internal forces on a system is delivering a power proportional to time (P ∝ t ).
always zero. Then, match the following columns and mark the
(b) Total work done by internal forces on a system may correct option from the codes given below.
sometimes be zero.
(c) Total work done by friction can never be zero. Column I Column II
(d) Total work done by friction is always zero. (A) Velocity is proportional to (p) t
2 Two inclined frictionless tracks, one gradual and the (B) Displacement is proportional to (q) t2
other steep meet at A from where two stones are (C) Work done is proportional to (r) t3
allowed to slide down from rest, one on each track
as shown in figure. [NCERT Exemplar] Codes
A B C A B C
Which of the following statement(s) is/are correct?
(a) p q r (b) p p r
A (c) p q q (d) r p q

I II 2 F-x and corresponding U-x graphs are as shown in


h figures. Three points A, B and C in F-x graph may be
corresponding to P, Q and R in the U-x graph. Match
θ1 θ2
B the following columns and mark the correct option
from the codes given below.
(a) Both the stones reach the bottom at the same time but
not with the same speed.
F U
(b) Both the stones reach the bottom with the same speed
and stone I reaches the bottom earlier than stone II. B x Q R x
(c) Both the stones reach the bottom with the same speed
A C P
and stone II reaches the bottom earlier than stone I.
(d) Both the stones reach the bottom at different times and
with different speeds.
3 In position A, kinetic energy of a particle is 60 J and Column I Column II
potential energy is − 20 J. In position B, kinetic (A) A (p) P
energy is 100 J and potential energy is 40 J. Choose
(B) B (q) Q
the correct statement, if the particle moves from A
to B. (C) C (r) R
(a) Work done by conservative forces is 50 J. (s) None
(b) Work done by external forces is 40 J.
(c) Net work done by all the forces is 40 J. Codes
(d) Net work done by all the forces is 100 J. A B C
(a) r s p
4 Which of the following statement(s) is/are correct?
(b) p s r
I. Spring force is a conservative force. (c) p r q
II. Potential energy is defined only for conservative (d) q r s
forces.
3 Acceleration versus x and potential energy versus x
(a) Only I
(b) Only II graph of a particle moving along X-axis are as shown
(c) Both I and II below . Mass of the particle is 1 kg and velocity at
(d) Neither I nor II x = 0 is 4 ms −1.
276 OBJECTIVE Physics Vol. 1

Match the following columns for x = 8 m and mark Match the following columns for work done on the
the correct option from the codes given below. block and mark the correct option from the codes
a (ms–2 ) U (J) given below.
2 120
x (m) 8 x (m)
4 8 4
−120 a
m
Column I Column II
θ
(A) Final kinetic energy (p) 120 J
(B) Work done by conservative forces (q) 240 J Column I Column II
(C) Total work done (r) −120 J (A) By gravity (p) 144 J
(D) Work done by external forces (s) 128 J
(B) By normal reaction (q) 32 J
Codes (C) By friction (r) –160 J
A B C D A B C D
(D) By all the forces (s) 48 J
(a) p s r q (b) s q p r
(c) s p r q (d) p q r s Codes
A B C D
4. A block of mass m is stationary with respect to a (a) r q s p
rough wedge is shown in figure. Starting from rest in (b) q r s p
time t (m = 1 kg, θ = 30 °, a = 2 ms −2, t = 4 s). (c) p q r s
(d) r p s q

(C) Medical entrances’ gallery


Collection of questions asked in NEET & Various Medical Entrance Exams
1 A force F = 20 + 10 y acts on a particle in (a) 18 ms −1 and 24.4 ms −1 (b) 23 ms −1 and 24.4 ms −1
y-direction, where F is in newton and y is in metre. (c) 23 ms −1 and 20.6 ms −1 (d) 18 ms −1 and 20.6 ms −1
Work done by this force to move the particle from 4 Initially spring in its natural length. Now, a block of
y = 0 to y = 1m is [NEET 2019]
mass 0.25 kg is released, then find out the value of
(a) 5 J (b) 25 J (c) 20 J (d) 30 J maximum force by system on the floor. [AIIMS 2019]
2 When a block of mass M is suspended by a long wire 0.25 kg
of length L, the length of the wire becomes (L + l).
The elastic potential energy stored in the extended
wire is [NEET 2019]
1 1
(a) MgL (b) Mgl (c) MgL (d) Mgl
2 2 2 kg
3 An object of mass 500 g, initially at rest acted upon (a) 15 N (b) 20 N (c) 25 N (d) 30 N
by a variable force whose x component varies with x in 5 The rate of decrease of kinetic energy is 9.6 Js −1.
the manner shown in figure. The velocities of the Find the magnitude of force acting on particle when
object at points x = 8 m and x = 12 m, would be the its speed is 3 ms −1. [JIPMER 2019]
respective values of (nearly) [NEET (Odisha) 2019] (a) 3.2 N (b) 4.8 N (c) 2.4 N (d) 5.6 N
F (N)
6 If a machine perform 4000 J output work and 1000 J
20 is inside loss due to friction, then find the efficiency.
(a) 80% (b) 30% [JIPMER 2018]
10
(c) 25% (d) 60%
x (m)
4 5 8 10 12 7 Kinetic energy of a particle is increased by 4 times.
–10 What will be the relation between initial and final
–20 momentum? [JIPMER 2018]
–25 (a) p 2 = 2p1 (b) p 2 = p1 /2 (c) p 2 = p1 (d) p 2 = 4p1
Work, Energy and Power 277

8 1000 N force is required to lift a hook and 10000 N (a) 18 × 103 times (b) 24 × 103 times
force is requires to lift a load slowly. Find power (c) 30 × 103 times (d) 21 × 103 times
required to lift hook with load with speed v = 0.5 ms −1.
[JIPMER 2018] 14 A skier starts from rest at point A and slides down
(a) 5 kW (b) 1.5 kW (c) 5.5 kW (d) 4.5 kW the hill without turning or breaking. The friction
9 A spring of force constant k is cut into lengths of coefficient is µ. When he stops at point B, his
ratio 1 : 2 : 3. They are connected in series and the horizontal displacement is s. What is the height
new force constant is k ′. If they are connected in difference between points A and B?
parallel and force constant is k ′ ′, then k ′ : k ′ ′ is (The velocity of the skier is small, so that the
[NEET 2017] additional pressure on the snow due to the curvature
(a) 1 : 6 (b) 1 : 9 (c) 1 : 11 (d) 1 : 14
can be neglected. Neglect also the friction of air and
the dependence of µ on the velocity of the skier.)
10 A force F = − k ( y$i + x$j ), where k is a positive [JIPMER 2017]
µ
constant, acts on a particle moving in the XY-plane. (a) h = µs (b) h =
s
Starting from the origin, the particle is taken along (c) h = 2µs (d) h = µs 2
the positive X-axis to the point (a, 0 ) and then
15 A body of mass 1 kg begins to move under the action
parallel to the Y-axis to the point (a, a ). The total
of a time dependent force F = (2t $i + 3t 2 $j) N, where
work done by the force on the particle is [AIIMS 2017] $i and $j are unit vectors along X andY-axis. What
(a) − 2ka 2 (b) 2ka 2 (c) − ka 2 (d) ka 2
power will be developed by the force at the time (t )?
11 Assertion A spring of force constant k is cut into (a) (2 t 2 + 4 t 4 ) W (b) (2 t 3 + 3 t 4 ) W [NEET 2016]
two pieces having lengths in the ratio 1 : 2. The (c) (2 t 3 + 3 t 5 ) W (d) (2 t + 3 t 3 ) W
force constant of series combination of the two parts
16 A particle of mass 10 g moves along a circle of
is 3k /2.
radius 6.4 cm with a constant tangential
Reason The spring connected in series are acceleration. What is the magnitude of this
represented by k = k1 + k 2 . [AIIMS 2017] acceleration, if the kinetic energy of the particle
(a) Both Assertion and Reason are correct and Reason is becomes equal to 8 × 10 −4 J by the end of the second
the correct explanation of Assertion. revolution after the beginning of the motion?
(b) Both Assertion and Reason are correct but Reason is not [NEET 2016]
the correct explanation of Assertion. (a) 0.15 ms −2 . ms −2
(b) 018
(c) Assertion is correct but Reason is incorrect. (c) 0.2 ms −2 (d) 0.1 ms −2
(d) Both Assertion and Reason are incorrect.
17 A block of mass 10 kg moving in x-direction with a
12 The figure shows a mass m on a frictionless surface. constant speed of 10 ms −1, is subjected to a retarding
It is connected to rigid wall by the mean of a force F = − 01 . x Jm −1 during its travel from x = 20 m
massless spring of its constant k. Initially, the spring to 30 m. Its final KE will be [CBSE AIPMT 2015]
is at its natural position. If a force of constant (a) 475 J (b) 450 J
magnitude starts acting on the block towards right, (c) 275 J (d) 250 J
then the speed of the block when the deformation in
spring is x, will be [AIIMS 2017] 18 A particle of mass m is driven by a machine that
k m delivers a constant power k watts. If the particle
F starts from rest, the force on the particle at time t is
[CBSE AIPMT 2015]
2F ⋅ x − kx 2 F ⋅ x − kx 2 mk −1/ 2
(a) (b) (a) t (b) mk t −1/ 2
m m 2
1
x (F − k ) F ⋅ x − kx 2 (c) 2mkt −1/ 2 (d) mkt −1/ 2
(c) (d) 2
m 2m
19 A force F = (10 + 0.5x ) acts on a particle in the
13 A person of weight 70 kg wants to loose 7 kg by
going up and down 12m high stairs. Assume he x-direction. What would be the work done by this
burns twice as much fat while going up than going force during a displacement from x = 0 to x = 2m (F is
down. If 1 kg of fat is burnt on expending in newton and x in metre) [AIIMS 2015, UK PMT 2015]
9000 k-cal. How many times must he go up and (a) 31.5 J (b) 63 J
down to reduce his 7 kg weight?(Take, g = 10 ms − 2 ) (c) 21 J (d) 42 J
[AIIMS 2017]
278 OBJECTIVE Physics Vol. 1

20 A block of mass m =11.7 kg is to be pushed a distance 27 A body of mass m = 3.90 kg slides on a horizontal
of s = 4.65 m along an incline and raised to a frictionless table with a speed of v = 1.20 ms −1. It is
distance of h = 2.86 m. Assuming frictionless brought to rest in compressing a spring in its path.
surface, calculate the work done in applying a force How much does spring is compressed, if its force
parallel to the incline to push the block up at a constant k is 135 Nm −1? [UK PMT 2015]
constant speed. (Take, g = 9.8 ms −2 ) [UK PMT 2015]
(a) 0.204 m (b) 0.408 m
(c) 0.804 m (d) 4.04 m
28 A string of length L and force constant k is stretched
s = 4.65 m
to obtain extension l. It is further stretched to obtain
h = 2.86 m
extension l1. The work done in second stretching is
[MHT CET 2014]
θ
1 1
(a) kl1 (2l + l1 ) (b) kl12
(a) 328 J (b) 656 J 2 2
(c) 164 J (d) 530 J 1 2 1
(c) k (l + l12 ) (d) k (l12 − l 2 )
2 2
21 An elevator weighing 500 kg is to be lifted up at a
29 A body is initially at rest. It undergoes one
constant velocity of 0.20 ms −1. What would be the
dimensional motion with constant acceleration. The
minimum horse power of the motor to be used?
power delivered to it at time t is proportional to
[CGPMT 2015, UK PMT 2015]
[Uttarakhand PMT 2014]
(a) 10.30 hp (b) 5.15 hp (a) t1/ 2 (b) t
(c) 2.62 hp (d) 1.31 hp
(c) t 3 / 2 (d) t 2
22 Two bodies of different masses are moving with
30 If two persons A and B take 2 s and 4 s respectively
same kinetic energy. Then, the ratio of their momenta to lift an object to the same height h, then the ratio
is equal to the ratio of their [Kerala CEE 2015]
of their powers is [Kerala CEE 2014]
(a) masses (b) square of masses
(a) 1 : 2 (b) 1 : 1
(c) square root of masses (d) cube root of masses (c) 2 : 1 (d) 1 : 3
23 Two bodies of masses 1 kg and 2 kg moving with (e) 3 : 1
same velocities are stopped by the same force. Then, 31 If a machine gun fires n bullets per second each with
the ratio of their stopping distances is [Kerala CEE 2015]
kinetic energy K, then the power of the machine gun
(a) 1 : 2 (b) 2 : 1
is [Kerala CEE 2014]
(c) 2 : 1 (d) 1 : 2 K
(a) nK 2 (b)
24 A bob of mass m accelerates uniformly from rest to v 1 n
in time t1. As a function of t, the instantaneous power (c) n 2K (d) nK
delivered to the body is [Manipal 2015]
n
(e)
mvit mv1 t K
(a) (b)
t1 t1 32 A uniform force of (3 $i + $j ) N acts on a particle of
mv1 t 2 mv12 t mass 2 kg. Hence, the particle is displaced from
(c) (d)
t1 t1
2 position (2i$ + k$ ) m to position (4 $i + 3 $j − k$ ) m. The
work done by the force on the particle is [NEET 2013]
25 A uniform chain of length l is placed on a smooth (a) 9 J (b) 6 J
horizontal table, such that half of its length hangs (c) 13 J (d) 15 J
over one edge. It is released from rest, the velocity
with which it leaves the table is [EAMCET 2015] 33 The power (P) of an engine lifting a mass of 100 kg
3gl 3gl upto a height of 10 m in 1 min is [J&K CET 2013]
(a) (b) (a) 163.3 W (b) 9800 W
4 2
(c) 10000 W (d) 5000 W
2gl gl
(c) (d) 34 A body of mass 300 kg is moved through 10 m along a
3 3
smooth inclined plane of angle 30°. The work done in
26 The kinetic energy of a body of mass 4 kg and
moving (in joules) is (Take, g = 9.8 ms −2 ) [EAMCET 2013]
momentum 6 N-s will be [KCET 2015]
(a) 4.5 J (b) 2.5 J (a) 4900 (b) 9800
(c) 5.5 J (d) 3.5 J (c) 14700 (d) 2450
Work, Energy and Power 279

35 Force constants of two wires A and B of the same and r is the distance of particle from the centre of
material are k and 2k, respectively. If the two wires the field. For stable equilibrium, the distance of the
are stretched equally, then the ratio of work done in particle is [CBSE AIPMT 2012]
W  (a) B/2A (b) 2A/B
stretching  A  is [EAMCET 2013] (c) A/B (d) B/A
WB 
39 The particle of mass 50 kg is at rest. The work done
1 1
(a) (b) to accelerate it by 20 ms −1 in 10 s is [AIIMS 2012]
3 2 3 4
(a) 10 J (b) 10 J
3 1
(c) (d) (c) 2 × 103 J (d) 4 × 104 J
2 4
36 A truck accelerates from speed v to 2v. Work done 40 The slope of kinetic energy and displacement curve
during this is [KCET 2013] for a particle in motion will be [BCECE Mains 2012]
(a) three times as the work done in accelerating it from (a) equal to the acceleration of the particle
rest to v (b) directly proportional to the acceleration of the particle
(b) same as the work done in accelerating it from rest to v (c) inversely proportional to the acceleration of the particle
(c) four times as the work done in accelerating it from rest (d) None of the above
to v 41 Two masses m and 2m are attached to two ends of an
(d) less than the work done in accelerating it from rest to v
ideal spring as shown in figure. When the spring is
37 A block of 200 g mass is dropped from a height of in the compressed state, the energy of the spring is
2 m on to a spring and compresses the spring to a 60 J, if the spring is released, then at its natural
distance of 50 cm. The force constant of the spring is length, [BHU 2012]
[Kerala CET 2013] m 2m
(a) 20 Nm −1 (b) 40 Nm −1 (c) 30 Nm −1 (d) 60 Nm −1
(e) 10 Nm −1 (a) energy of smaller body will be 20 J
(b) energy of smaller body will be 40 J
38 The potential energy of a particle in a force field is (c) energy of smaller body will be 10 J
A B
U= 2
− , where A and B are positive constants (d) energy of both the bodies will be same
r r
OBJECTIVE Physics Vol. 1

ANSWERS
l CHECK POINT 6.1
1. (c) 2. (d) 3. (a) 4. (b) 5. (d) 6. (b) 7. (d) 8. (a) 9. (b) 10. (b)
11. (d) 12. (b) 13. (a) 14. (c) 15. (d) 16. (b) 17. (d) 18. (a) 19. (a) 20. (b)

l CHECK POINT 6.2


1. (b) 2. (a) 3. (b) 4. (d) 5. (c) 6. (c) 7. (d) 8. (c) 9. (a) 10. (a)
11. (b) 12. (d) 13. (b) 14. (c) 15. (b) 16. (a) 17. (b) 18. (a) 19. (b) 20. (c)
21. (c)

l CHECK POINT 6.3


1. (d) 2. (c) 3. (b) 4. (c) 5. (a) 6. (d) 7. (a) 8. (d)

(A) Taking it together


1. (c) 2. (b) 3. (a) 4. (b) 5. (c) 6. (c) 7. (c) 8. (c) 9. (a) 10. (c)
11. (d) 12. (a) 13. (c) 14. (c) 15. (c) 16. (d) 17. (a) 18. (d) 19. (c) 20. (a)
21. (c) 22. (b) 23. (c) 24. (c) 25. (d) 26. (b) 27. (a) 28. (c) 29. (a) 30. (a)
31. (c) 32. (b) 33. (c) 34. (b) 35. (d) 36. (c) 37. (b) 38. (a) 39. (b) 40. (b)
41. (c) 42. (a) 43. (b) 44. (b) 45. (b) 46. (d) 47. (b) 48. (b) 49. (d) 50. (d)
51. (a) 52. (a) 53. (d) 54. (b) 55. (b) 56. (a) 57. (c) 58. (a) 59. (a) 60. (b)
61. (b) 62. (b) 63. (c) 64. (a) 65. (a) 66. (c) 67. (b) 68. (b) 69. (a)

(B) Medical entrance special format questions


l Assertion and reason
1. (d) 2. (a) 3. (c) 4. (d) 5. (b) 6. (c)

l Statement based questions


1. (b) 2. (c) 3. (c) 4. (c) 5. (c)
l Match the columns
1. (c) 2. (a) 3. (b) 4. (d)

(C) Medical entrances’ gallery


1. (b) 2. (b) 3. (c) 4. (c) 5. (a) 6. (a) 7. (a) 8. (c) 9. (c) 10. (c)
11. (d) 12. (a) 13. (d) 14. (a) 15. (c) 16. (d) 17. (a) 18. (a) 19. (c) 20. (a)
21. (d) 22. (c) 23. (a) 24. (d) 25. (a) 26. (a) 27. (a) 28. (a) 29. (b) 30. (c)
31. (d) 32. (a) 33. (a) 34. (c) 35. (b) 36. (a) 37. (b) 38. (b) 39. (b) 40. (b)
41. (b)
Hints & Explanations
l CHECK POINT 6.1 10 (b) Work done,W = ( y -component of force )
1 (c) Work done by kinetic friction may be positive or negative. × (displacement along Y - axis )
2 (d) As, work done,W = F ⋅ s = Fs cos θ = 15 × 10 = 150 J
x2 5 5
When a man pushes a wall but fails to move it, then 11 (d) W = ∫ Fdx = ∫ F dx = ∫ (10 − 2x + 3x 2 ) dx
x1 0 0
displacement of wall, s = 0
∴ Work done,W = F × 0 × cos θ = 0 = [10 x − x 2 + x 3] 50 = 150 J
Therefore, man does no work at all. +a
+a Ay 3 By 2  2Aa 3
12 (b) W = ∫ Fdy =  + + Cy  = + 2Ca
3 (a) Given, F = 20 kg-wt = 20 × 9.8 N, s = 20 m and θ = 60 ° −a
 3 2 −a 3
∴ Work done = F s cos θ = 20 × 9.8 × 20 × cos 60 ° = 1960 J
13 (a) Given, force, F = (2$i + 3$j)N
4 (b) From first equation of motion,
v = u + at ⇒ 20 = 0 + a × 10 Displacement, ds = dx$i + dy$j + dzk$
⇒ a = 2 m/s 2
Work done,W = ∫ F ⋅ ds = ∫ (2dx + 3dy )
1 2 1
Now, distance, s = ut + at ⇒ s = 0 + × 2 × 10 × 10 3dy
2 2 Also, 3 y + kx = 5 ⇒ +k=0
dx
or s = 100 m
∴ Work done,W = F ⋅ s = mas = 50 × 2 × 100 = 10 4 J ⇒ 3dy = − kdx ⇒ W = ∫ ( 2dx − kdx ) = 0

5 (d) The free body diagram is as shown below. ⇒ 2x = kx ⇒ k = 2


f Alternative Solution
3  k
m
sm
M Q m1 m 2 = −1⇒ −  = −1 ⇒ k=2
f
sM 2  3
s m ≠ sM , sm > sM 14 (c) W = Area under F -x graph
Therefore, net work done by the friction cannot be zero. 1
∴W = Area of trapezium = × ( 4 + 2) × 5 = 15 J
6 (b) Frictional force acting on the box, 2
F = µmg = 0.25 × 20 × 9.8 = 49 N 15 (d) Work done = Area under F-x graph
This force must be same as applied force F, so that the box = Area of ∆ABG + Area of rectangle of BGHC
moves with constant speed. + Area of ∆CDH + Area of ∆DEI + Area of rectangle EFJI
Work done by the applied force,W = Fs = 49 × 2 = 98 J
F(N)
7 (d) Given, force, F = (3$i + 4$j ) N
B C
and displacement, s = (3$i + 4$j ) m +10
∴ Work done,W = F ⋅ s = (3$i + 4$j ) ⋅ (3$i + 4$j )
G H D E F
W = 9 + 16 = 25 J A
1 x(m)
2 3 4 5
8 (a) Particle moves from P (1m, 2 m, 3 m) to Q (2 m, 1m, 4 m) –10
I J
∴ Distance, d = PQ = (2i$ + $j + 4k$ ) − (i$ + 2$j + 3k$ )
1  1  1 
=  × 10 × 1 + (10 × 1) +  × 10 × 1 +  (−10 ) × 1
= ($i − $j + k$ ) m 2  2  2 
Force, F = (2i$ + $j + k$ ) N + (−10 × 1)
⇒ W = 5J
Work done by the force, 1 1
W = F ⋅ d = (2i$ + $j + k$ ) ⋅ ($i − $j + k$ ) = 2 − 1 + 1 = 2 J 16 (b) W1 = k (5)2 × 10 −4 and W2 = k (15)2 × 10 −4
2 2
9 (b) W = F ⋅ r = F ⋅ (r2 − r1) where,W1 andW2 are the work done in extending spring to
5 cm and 15 cm, respectively.
= ($i + 2$j + 3 k$ ) ⋅ [($i − $j + 2 k$ ) − ($i + $j + k$ )] W1→ 2 = W2 − W1
= ($i + 2$j + 3 k$ ) ⋅ (− 2$j + k$ ) = − 4 + 3 = − 1J 1 1
= k [(15)2 − (5)2] × 10 −4 = × 800 (200 ) × 10 −4 = 8 J
2 2
282 OBJECTIVE Physics Vol. 1

10 N So, the graph between p and E K will be straight line but


17 (d) Spring constant, k = = 10 × 10 3 Nm–1
10 –3 m graph between 1/p and E K will be a hyperbola as shown
Now, work done in stretching the spring through 40 mm below
1
(= 40 × 10 −3 m), W = × 10 × 10 3 × (40 × 10 −3 )2 = 8J
2
18 (a) Work done by the force relative to porter will be zero
because displacement relative to porter is zero. √EK
Now, work done by the force relative to a person on the
ground, WG = mgh = 20 × 10 × 4 = 800 J
19 (a) In case of a conservative force field, the work done is 1/p
independent of the path followed.
∴Required work done = 5J + 2J = 7J 7 (d) From work-energy theorem, K2 − K1 = W = Fs
20 (b) Amongst the given forces, frictional force is a K2 − 0 = Fs
non-conservative force, whereas spring and gravitational
forces are conservative forces. K is independent of m. (Q Force is constant)
dx d  t 3
l CHECK POINT 6.2 8 (c) Speed of the body, v = =   = t2
dt dt  3 
1
1 (b) According to question, F ∝ At t = 0, v = 0
v
λ At t = 2 s, v = 4 ms −1
⇒ F = where, λ = constant
v From work-energy theorem,W = Change in kinetic energy
dv λ v t 1 1
ma = m = ⇒ m ∫ v dv = λ ∫0dt = Kf − Ki = m (v f2 − vi2 ) = × 2 × (16 − 0 ) = 16 J
dt v 0 2 2
v
v2 1 2 9. (a) Change in kinetic energy = Work done
m = λ | t |0t ⇒ mv = K = λt = Area under F - x graph
2 0
2
1 1
K ∝t Q × 5 × v 2 = 10 × 25 + × 25 × 10 = 375
2 2
2 (a) It is given that, Kf = 2Ki ∴ v = 12.2 ms −1

1 1  10 (a) Applying work-energy theorem, Kf − Ki = W


or m (v + 1)2 = 2  mv 2 or v + 1 = 2 v
2 2  1 30
⇒ Kf = Ki + W = × 20 × (10 )2 + ∫ (− 0.1x )dx
1 2 20
or v= = ( 2 + 1) ms −1
2 −1 0.1 2 30
= 1000 − (x )20 = 1000 − 25 = 975 J
1 2 11 m  2
3 (b) According to the question, mv1 = × × v 22
2 2  2 2  11 (b) Work done by all forces = Change in kinetic energy
where, v1 = speed of man and v 2 = speed of boy. 1 1
= m (v f2 − vi2 ) = × 2(0 − 400 ) = − 400 J
1 1 m 2 2
Now, m (v1 + 1)2 = × × v 22
2 2 2 1
12 (d) Work done on the particle,W = ∆KE = m (v f2 − vi2 )
Solving these two equations, we get 2
v1 = ( 2 + 1) ms −1 and v 2 = 2( 2 + 1) ms −1 1
= × 0.01[(64 + 400 ) − (16 + 256)] = 0.96 J
2
p2 p2 p2
4 (d) Kinetic energy, K = ⇒ K1 = and K2 = 13 (b) Given, force, F = (4$i + 4$j ) N
2m 2m1 2m 2
K1 m 2 r1 = 0 $i + 0 $j
Q =
K2 m1 r2 = $i + $j
2 2 2
5 (c) K =
p
, K′ =
(1.5p )
= 2.25
p
= 2.25 K ∆r = r 2 − r1 = i$ + $j
2m 2m 2m
Initial speed, v1 = 2 ms −1
2.25 K − K
∴ % increase in kinetic energy = × 100 = 125%.
K From work-energy theorem, we have
6 (c) p = 2mE K ∆W = ∆K
1
It is clear that p ∝ E K ⇒ F ⋅ ∆r = m (v 22 − v12 )
2
Work, Energy and Power 283

1 21 (c) From conservation of mechanical energy, we have


⇒ (4$i + 4$j ) ⋅ ($i + $j ) = × 1 (v 22 − 4)
2 1 2
1 ⇒ mvi = mgh
⇒ 4 + 4 = (v 22 − 4) 2
2 3 h
∴ mgh + mgh′ = mgh ⇒ h′ =
⇒ v 22 = 20 ⇒ v 2 = 4.5 ms −1 4 4
14 (c) Forces acting on the body are l CHECK POINT 6.3
weight, w = 5 × 10 = 50 N 1
1 (d) From work-energy theorem, work done,W = Mv 2
External force, F = 170 N 2
Net upward force acting on the body, Fnet = 170 − 50 = 120 N W 1  Mv 2 
1 ∴Power delivered, P = =  
Now, applying work-energy theorem, (Fnet ) h = mv 2 t 2 t 
2
Work done by the engine W
1 2 (c) Power, P = =
⇒ 120 × 10 = × 5 × v 2 Time taken t
2
2 × 12 × 100 W mgh 200 × 10 × 40
⇒ v2 = ⇒ v = 22 ms −1 ⇒ t= = = =8s
5 P P 10 × 10 3

15 (b) Decrease in kinetic energy = Increase in elastic potential 3 (b) Given, Poutput = 10 kW
energy Now, Pinput = 2 × 10 3 calg −1 × 1 g s −1
1 2 1 2
∴ mv = kx = 2 × 10 3 cal s −1 = 2 × 10 3 × 4.2 Js −1
2 2
m 0.1 = 8.4 kW
or x= ⋅v = × 10 = 0.1m
k 1000 As Poutput > Pinput, hence the claim given in question is never
possible.
16 (a) Decrease in gravitational potential energy
mgh
= Increase in elastic potential energy 4 (c) Power given to turbine, Pin =
t
1 2 1
or mg (h + x ) = kx or 2 × 9.8(0.4 + x ) = × 1960 × x 2 m
2 2 Pin =   × g × h
t
Solving this equation, we get
x = 0.1m or 10 cm ⇒ Pin = 15 × 10 × 60 ⇒ Pin = 9000 W
⇒ Pin = 9 kW
17 (b) Potential energy is only associated with conservative force.
As efficiency of turbine is 90%, therefore power generated,
 ∂U $ ∂U $ ∂U $  90
Force, F = −  i+ j+ k Pout = 90 % of 9 kW = 9 ×
 ∂x ∂y ∂z  100
where, F = conservative force. ⇒ Pout = 8.1 kW
 ∂U $ ∂U $ ∂U $  Work done Pressure × Change in volume
 i+ j+ k = Partial derivative of potential energy 5 (a) Power = =
 ∂x ∂y ∂z  Time Time
w.r.t. x, y and z, respectively. 20000 × 1× 10 −6
=
1
18 (a) The potential energy of a system increases, if work is done
by the system against a conservative force. = 2 × 10 −2 = 0.02 W
− ∆U = Wconservative force 6 (d) Power is defined as the rate of change of energy in a
dU system or the time rate of doing work.
19 (b) For conservative force, F = − = − (7x − 3) = 3 − 7x
dx dE dW
⇒ P = =
At equilibrium, F = 0 dt dt
3
⇒ 3 − 7x = 0 ⇒ x = Also, work,W = force × displacement = F × d
7
Since, the displacement is zero.
2
7  3  3 9 9 9 d d
∴ U=   −3  = − =− units ∴ P = (F × d ) = × 0 = 0
2  7  7 14 7 14 dt dt
20 (c) Gain in KE = Loss in PE F ⋅ s (2 $i + 3 $j + 4 k$ ) ⋅ (3$i + 4 $j + 5 k$ )
7 (a) P = F ⋅ v = =
1 2 t 4
⇒ mv = (0.1)(mgh )
2 38
= = 9.5 W
∴ v = 0.2gh = 0.2 × 10 × 2 = 2 ms −1 4
284 OBJECTIVE Physics Vol. 1

8 (d) P = F ⋅ v = mg (u cos θ ) cos 90 ° = 0 13 (c) From work-energy theorem,


u work done = change in kinetic energy
ucosθ ⇒K ∝ x (as F = constant because a = constant)
θ Therefore, K -x graph is a straight line passing through origin.
mg 14 (c) When a pendulum oscillates in air, it will lose energy
continuously in overcoming resistance due to air. Therefore,
total mechanical energy of the pendulum decreases
(A) Taking it together continuously with time.
1 (c) As the body is falling freely under gravity, the potential The variation is correctly represented by curve (c).
energy decreases and kinetic energy increases but total W KE mv 2
mechanical energy (PE + KE) of the body and earth system 15 (c) ∴Average power = = =
t t 2t
will remain constant, as external force on the system is zero.
 1 2
2 (b) When electron and proton are moving under the influence F  at 
2  1
of their mutual forces, the magnetic forces will be ⇒ P = = Fv
t 2
perpendicular to their motion. Hence, no work is done by
these forces. 16 (d) From work-energy theorem,
1 2
3 (a) Power of the engine, W = Change in kinetic energy or W = mv
P = F ⋅ v = (20 $i − 3$j + 5k$ ) ⋅ (6$i + 20 $j − 3k$ ) 2
∴ W- v graph is a parabola.
= 120 − 60 − 15 = 45 W
l 17 (a) Speed is doubled. Therefore, kinetic energy will become
4 (b) Centre of mass of the rod moves a height, h = . four times. Hence, minimum stopping distance will also
2
become four times, i.e. 4 × 2 m = 8 m.
Work done,W = mgh = mgl / 2
1
5 (c) Here, work is done by the frictional force on the cycle and 18 (d) Potential energy of the spring, U = kx 2 or U ∝ x 2
2
is equal to −200 × 10 = − 2000 J. As the road is not moving,
hence work done by the cycle on the road = zero. Stretch is increased by 5 times. Therefore, stored potential
energy will be increased by 25 times.
6 (c) F = kx or k = slope of F-x graph [when F (load) is along
19 (c) Applying conservation of mechanical energy, we have
Y-axis and extension (x ) is along X-axis.]
1 2 2U
Here, F is along X-axis. mv = U or m = 2
2 v
1
So, k= = 0.1 kgf/cm
10 20 (a) Here, P = 10 7 kW = 1010 W = 1010 Js −1

7 (c) ∴WF + Wg = ∆K = 0 ⇒ WF = − Wg = mgh Time, t = 1day = 24 × 60 × 60 s


Energy produced per day,
8 (c) P = 2Km or P ∝ m
E = Pt = 1010 × 24 × 60 × 60 = 864 × 1012 J
P m1 1 1
∴ 1= = = As, E = mc 2
P2 m2 4 2
E 864 × 1012 J
1 ⇒ Mass, m = =
9 (a) Kinetic energy, E = mv 2 c 2
(3 × 10 8 )2 ms −1
2
dE dE = 9.6 × 10 −3 kg = 9.6 g
∴ = mv or p = (Q mv = p)
dv dv 21 (c) Force between two protons is same as that of between
1 proton and a positron.
10 (c) W = × m × (10 )2 = 50m …(i)
2 As, positron is much lighter than proton, it moves away
1 1 through much larger distance as compared to proton.
andW ′ = m (v 22 − v12 ) = m × (400 − 100 ) = 150 m = 3W We know that, work done = force × distance.
2 2
[from Eq. (i)]
As forces are same in case of proton and positron but distance
11 (d) When the earth is closest to the sun, speed of the earth is moved by positron is larger, hence work done will be more.
maximum, hence kinetic energy is maximum. When the earth
22 (b) Total energy, E = PE + KE ...(i)
is farthest from the sun, speed is minimum, hence kinetic
energy is minimum but it will never be zero and negative. When particle is at x = x m, i.e. at extreme position, it returns
This variation is correctly represented by option (d). back. Hence, at x = x m; v = 0; KE = 0
From Eq. (i), we get
12 (a) Given force is a constant force and work done by a
constant force is always path independent. 1
E = PE + 0 = PE ⇒ E =V (x m ) = kx m2
∴ W1 = W2 2
Work, Energy and Power 285

dU v 02
23 (c) At equilibrium, F = − = 0 ⇒ (− 12)ax −13 + (6 bx −7 ) = 0 ∴ v2 =
v
⇒v = 0 = v 0 − at0 = v 0 − µgt0
dx 4 2
1/ 6
 2a  v0
∴ x=  or µ=
b 2gt0
1
24 (c) For upward motion, m (10 )2 = mgh + Rah …(i) 31 (c) Mass of the body, m = 200 g = 200 × 10 −3 kg
2
where, Ra = air resistance. Height, h = 200 m
1 Potential energy, U = mgh = 200 × 10 −3 × 10 × 200 = 400 J
For downward motion, mgh = m (9)2 + Rah …(ii)
2 Therefore, decrease in potential energy at the surface = 400 J.
Solving Eqs. (i) and (ii) , we get 32 (b) At this height, half energy will be potential energy.
1 98
2mgh = m (100 + 81) ⇒ h = 4.61m ∴ mgh = or 2 × 9.8 × h = 49 or h = 2.5 m
2 2
25 (d) Body will have maximum speed, where 33 (c) Increase in gravitational potential energy
mg sin θ = µ mg cos θ = Decrease in elastic potential energy
or sin 37° = (0.3x ) ⋅ cos 37° or x = 2.5 m 1 2 kx 2
∴ mgh = kx ⇒ h=
26 (b) From − a to b, decrease in elastic potential energy 2 2mg
= work done against friction. 34 (b) Loss in potential energy = mgh
1 2 1 2 2µmg
∴ k a − kb = µmg (a + b ) or (a − b ) = mgh
2 2 k Power generated =
2×t
2µmg
∴ Decrease in amplitude = mgh 1.8 × 10 5 × 10 × 50
k Number of bulbs, n = = = 125
2 × t × 100 2 × 3600 × 100
27 (a) Work done by frictional force,Wf = fs cos 180 ° dx
35 (d) Speed, v = = 8 t 3, v 0 s = 0, v1s = 8 ms −1
= (µmg cos θ )(− 1)(s ) dt
1 1 1
= − 0.20 × 5 × 10 × × 0.1 (Q θ = 45°) ∴ ∆ KE = m (v12s − v 02 s ) = × 2 × (64 − 0 ) J = 64 J
2 2 2
1
=− J 36 (c) W = Change in potential energy = mgh = mgL (1 − cos θ )
2
mg
28 (c) From conservation of mechanical energy, 37 (b) In equilibrium, kd = mg or d = …(i)
k
1
(M − m )gh = (M + m )v 2 If allowed to fall suddenly, the body does not stop in its
2 equilibrium position. In that case,
2gh (M − m ) decrease in gravitational PE = increase in elastic PE
∴ v=
m+M 1 2mg
or mgd ′ = k d ′ 2 ⇒ d ′ = = 2d [From Eq. (i)]
29 (a) The distance travelled by the block on the rough surface can 2 k
be calculated from energy conservation, 38 (a) K = 4t 2 or v 2 ∝ t 2
Ki + Ui = Kf + U f + WF ∴ v ∝t
where,WF = work done by friction. v varies linearly with time when acceleration or force is
h 1 constant.
⇒ 0 + mgh = 0 + 0 + µmgs ⇒ s = = = 5m
µ 0.2 39 (b) Change in potential energy for all three particles is same.
A B Hence, change in kinetic energy will also be same.
A B or vA =vB =vC

A C 40 (b) Work done by conservative force, WC = − ∆U


1m
⇒ WC = − (U B − U A )
2m
⇒ WC = U A − U B
The block stops at distance 1 m from A.
Initial mechanical energy is same for all.
3
30 (a) th of kinetic energy is lost. Hence, left kinetic energy is 1 2 2  3t 2 
4 41 (c) Work done,W = ∫ Pdt or mv = ∫ Pdt = ∫   dt = 4 J
2 0 2 
1
th.
4 8
⇒ v= = 2 ms −1
2
286 OBJECTIVE Physics Vol. 1

42 (a) Work done by resistive force = Change is PE 49 (d) Work done by tension on M is negative (force and
 h displacement are in opposite directions). But work done by
⇒ F ⋅ d = mg (h + d ) ⇒ F = mg 1 + 
 d tension on m is positive. Net work done will be zero.
1 50 (d) On M, horizontal components of N and f are balanced (as
43 (b) PE + KE = mgh + mv 2 = constant
2 Mg is vertical). Hence, on 2M also, they will be balanced.
= mgH (Here, H = initial height) ∴Horizontal force kx on 2M should be zero.
v2
or gh + = constant F
51 (a) Spring constant, k = = slope of F-x graph.
2 x
44 (b) Work done against friction = (µmg cos θ ) d k∝
1
(Q x −
~ l)
= 0.5 × 1 × 9.8 × cos 60 ° × 2 = 4.9 J l
45 (b) When drop falls, velocity first increases, hence kinetic Length is reduced to half. Therefore, k will become two
energy also increases. After sometime, speed (velocity) is times. Slope will increase. Hence, the line OA will shift
constant which is known as terminal velocity, hence kinetic towards F-axis.
energy also becomes constant. Potential energy decreases
continuously as the drop is falling continuously. 52 (a) F = constant
The variation in potential energy and kinetic energy is best ∴ a = constant or v = at
represented by (b). Now, P = F ⋅v = F ⋅ at or P ∝ t (As F and a both are constants)
46 (d) Given, h = 1.5 m, v = 1ms −1, m = 10 kg, g = 10 ms − 2 Hence, P - t graph is a straight line passing through origin.
From conservation of mechanical energy, 53 (d) F ∝ s −1/ 3 or a ∝ s −1/ 3
(PE)i + (KE) i = (PE)f + (KE)f  dv 
or vdv ∝ s −1/ 3ds Q a =v ⋅ 
1  ds 
⇒ mgh + mv 2 = 0 + (KE)f
2 On integrating, we have
1
⇒ (KE)f = mgh + mv 2 v 2 ∝ s 2/ 3 or v ∝ s1/ 3
2
1 Now, power, P ∝ F ⋅v ∝ s −1/ 3
⋅ s1/ 3 or P ∝ s 0
⇒ (KE)f = 10 × 10 × 1.5 + × 10 × (1)2
2 54 (b) When displaced from x 2 in negative direction, force is
= 150 + 5 = 155 J
positive. So, this force is restoring in nature and will bring the
47 (b) k k body back. Hence, at x 2 , body is in stable equilibrium
m
position.
1
55 (b) Given, v = ax 3/ 2 ,
m
x
m = 0.5 kg, a = 5 m−1/ 2 s −1, work done, W = ?
v dv dv d
m Acceleration, a 0 = =v = ax 3/ 2 (ax 3/ 2 )
dt dx dx
2 3 3
= ax 3/ 2 × a × × x1/ 2 = a 2x 2
Total PE of spring = Total KE of block 2 2
1 2 1 2 1 2 1 3 2 2
kx + kx = mv ⇒ kx 2 = mv 2 Now, Force = ma 0 = m a x
2 2 2 2 2
2k x=2 23
⇒ Speed of the block, v = x Work done,W = ∫ Fdx = ∫ ma 2x 2dx
m x=0 0 2

48 (b) Loss in mechanical energy = Work done against friction. 3 x3


2

= ma 2 ×  
(2 − h) θ 2m 2  3 0
1 2 1
= ma × 8 = × (0.5) × (25) × 8 = 50 J
h 2 2
56 (a) The given figure is
h
mgh = µmg cos θ × s ⇒ s = 1
µg cos θ
u=0
1 × 10 −2  (2 − h )  2
= Q cos θ = v
 2 – h  2  1m
0.01 × 10 ×   0.5m
 2 
x
= 0 . 1005 m
1 2
s−
~ 10 cm From law of conservation of energy, mg (1) = mv + mg (0.5)
2
Work, Energy and Power 287

Fds
⇒ v 2 = 2g (1 − 0.5) = g P = = constant (Q P = constant)
dt
x
Also, t = Now, writing dimensions, [F] [v] = constant
v
⇒ [MLT − 2] [LT − 1] = constant
1 gx 2 10 x 2
Q y = gt 2 ⇒ 0.5 = = ⇒ L2 T − 3 = constant (Q mass is constant)
2 2 v 2 2(10 )
⇒ L ∝T 3/ 2
⇒ x 2 = 1 ⇒ x = 1m
⇒ Displacement (d ) ∝ t 3/ 2

57 (c) Decrease in gravitational potential energy = increase in


63 (c) As block does not slide. Hence, force of friction,
kinetic energy
v
C1 s
L
Ml0 l0 C2 U2= – mg θ
U1= g 2
L 2 °–
90
f

MgL M l 1 g 2 2 θ
− ⋅ l0 g 0 = Mv 2 or v = (L − l0 )
2 L 2 2 L f = mg sinθ
1 In time t, displacement, s = vt
58 (a) From work-energy theorem, WF + Wmg = mv 2
2 ∴ Wf = f ⋅ s ⋅ cos (90 ° − θ )
1 = (mg sin θ )(vt )(sin θ ) = mgvt sin2 θ
⇒ F ⋅ R + mgR = mv 2
2 64 (a) Power due to tension = 0
1 1 1
⇒ 5 × 5 + × 10 × 5 = × × v 2 60º
2 2 2 30º
⇒ . ms −1
v = 200 = 1414
 dv  T
59 (a) Power , P = F ⋅v = m v ⋅  ⋅v
 ds 
2v 2 P s h
∴ ∫v v ⋅ dv =
m ∫0
⋅ ds
2v v 60°
v 3  Ps 7mv 3
 3  = m or s = 3 P mg
 v
mgv
60 (b) Applying conservation of mechanical energy, we get Power due to mg = (mg )(v ) cos 60 ° =
2
(mgH − mgh ) = 2mgh  3 1
Here, v = 2gh and h = l (cos 30 ° − cos 60 ° ) = 1 − 
H  2H  gH  2 2
∴ h= ⇒ v = 2g (H − h ) = 2g   = 2
3  3 3  3 − 1
∴ v = 2 × 10 ×   = 10 ( 3 − 1) = 2 .7 ms −1
 2 
61 (b) Maximum acceleration of 1 kg block, amax = µg = 1ms −2
mgv 1× 10
Common acceleration without relative motion between two ∴ Power, P = = × 2.7 = 13.5 W
0.5 −2 2 2
blocks, a= ms
3 65 (a) Decrease in gravitational potential energy
Since, a < a max = Increase in kinetic energy.
There will be no relative motion and blocks will move with l
0.5 −2 Initially, centre of mass of chain was at distance below the
acceleration ms . 4
3 l
pin and in final position, it is at distance below the pin.
Force of friction by lower block on upper block, 2
l
 0.5 1 Hence, centre of mass has descended .
f = ma = (1)  = N (towards right) 4
 3 6
∴ Work done,W = f × s = 0.5 J

62 (b) Given, power = constant
dW F ⋅ ds F ds cos 0° F ds
As, power, P = = = =
dt dt dt dt
l 1 2 gl
(Q Body is moving unidirectionally) ∴ mg = mv or v =
4 2 2
288 OBJECTIVE Physics Vol. 1

66 (c) At x = 6 m, U = 26 J (extreme position) 3 (c) Work done by friction on block A is positive and on B is
negative. If there is no slipping between the two blocks, then
On the other side, U = 26 = 10 + (x − 2) 2
or x − 2 = ± 4
sA = sB . Therefore, net work done will be zero, otherwise not.
∴ x = 6 m and x = − 2 m sB sA
Umin = 10 J, at x = 2 m
KEmax = E − Umin = 16 J, at x = 2 B A
f f
67 (b) Maximum acceleration of 2 kg block due to friction can be 1 2
µg or 5 ms −2. 4 (d) W = Pt = mv
2
Combined acceleration, if both move together with same v ∝ t1/ 2
60
acceleration, would be a = = 5 ms −2 On differentiating, we get
12 a ∝ t −1/ 2
Since, both accelerations are equal, upper block will move
5 (b) WAC = + ve
with acceleration 5 ms −2 due to friction. x x
1 1
In first two seconds, s = at 2 = × 5 × 4 = 10 m
2 2
and force of friction, f = ma = 10 N C
∴ Wf = fs cos 0 ° = 100 J x=0
A B
68 (b) Normal force on the block, N = mg − F sinθ
WCB = − ve andWACB = 0
Block moves with uniform velocity. Hence, net force = 0
Also, F ∝ (− x )
or F cos θ = µN = µ (mg − F sin θ )
dU d d  1 
µmg 6. (c) = (mgh ) = mg u y t − gt 2
∴ F = dt dt dt  2 
cos θ + µ sin θ
= mg (u y − gt ) = mgv y
F sin θ
dU
∴ = mg |v y |
dt
θ F cos θ dU
For m = 1kg, = g|v y | = g (u y − gt ) …(i)
dt
µ mgd cos θ a ⋅v = (− g$j ) ⋅ [u $i + (u − gt )$j]
x y
W = Fs cos θ =
cos θ + µ sin θ = − g (u y − gt ) = g|v y | …(ii)
69 (a) ∴ F = kx From Eqs. (i) and (ii), we can see that two magnitudes are
F 100 equal.
⇒ k= = Nm −1 = 100 Nm −1 P = F ⋅ v = (− mg$j ) ⋅ [v x $i + (u y − gt )$j] = − mgu y + mg 2t
x 1
Now, from energy conservation between natural length of So, P versus t graph has the positive slope.
spring and its maximum compression state.
1 2 1 2
l Statement based questions
i.e. mv + mgh = kxmax
2 2 1 (b) Work done by friction may be positive, negative and zero.
2 2 Work done by conservative, internal forces may be zero in a
kxmax (100 )(2)
⇒ v= − 2gh = − (2)(10 )(1) round trip but it is non-zero for non-conservative internal
m 10 forces.
= 20 ms −1 2 (c) As the given tracks are frictionless, hence mechanical
energy will be conserved, as both the tracks having common
(B) Medical entrance special format height h.
questions From conservation of mechanical energy,
1 2
mv = mgh (For both tracks I and II)
l Assertion and reason 2
1 (d) If oscillations are not taking place, then kinetic energy ⇒ v = 2gh
may be zero at stable equilibrium position. Hence, speed is same for both stones.
2 (a) For conservative forces, ∆U = − ∆W For stone I, a1 = acceleration along inclined plane = g sin θ1
where, ∆U = change in potential energy Similarly, for stone II, a 2 = g sin θ 2 as θ 2 > θ1, hence a 2 > a1.
and ∆W = work done by conservative force. And length for track II is also less, hence stone II reaches
earlier than stone I.
Work, Energy and Power 289

3 (c) Work done by conservative forces = Ui − U f From work-energy theorem,


Work done by all the forces = ∆KE = 32 J
= − 20 − 40 = − 60 J
Work done by gravity, Wg = − mgh = − (1) (10) (16) = − 160 J
Work done by external forces = E f − Ei = 140 − 40 = 100J
Writing equation of motion, we have
and net work done by all the forces = Kf − Ki
= 100 − 60 = 40 J Σ Fy = ma
4 (c) Spring force is conservative in nature. Potential energy is f
Y
N
only associated with conservative forces. θ a
X
5 (c) As momentum of a body increases by 50% of its initial
3 θ
momentum, p 2 = p1 + 50% of p1 = p1
2 θ
3 mg = 10 N
∴ v 2 = v1
2
N cos 30 ° + f sin 30 ° − 10 = ma = 2
9
As K ∝ v , so K2 = K1
2
or 3 N + f = 24 …(i)
4
9 Σ Fx = 0
K1 − K1
K2 − K1
Increase in KE = × 100 = 4 × 100 = 125% ∴ N sin 30 ° = f cos 30 ° or N = 3f …(ii)
K1 K1
Solving Eqs. (i) and (ii), we have
l Match the columns f = 6N and N = 6 3 N
1 (c) Power, P ∝ t Now, work done by normal reaction, WN = (N cos θ )(s )
 3
Work done, W = ∫ Pdt = ∫ αt dt or W ∝ t 2 = (6 3 )  (16) = 144 J
 2
Since, work done is equal to change in kinetic energy. 1
Work done by friction, Wf = (f sin θ )(s ) = (6) (16) = 48 J
Hence, v 2 ∝ t 2 or v ∝ t 2
ds Work done by all the forces,
Further, v=
dt W = Wg + WN + WF = − 160 + 144 + 48 = 32 J
ds Hence, A → r, B → p, C → s, D → q.
∴ ∝ t or ds ∝ t dt or s ∝ t 2 (By integration)
dt
Hence, A → p, B → q, C → q. (C) Medical entrances’ gallery
2 (a) A is the point of stable equilibrium, so potential energy is 1 (b) Work done by a force F, which is variable in nature in
minimum, i.e. at R. Similarly, point C is the unstable y2

equilibrium position, where potential energy should be moving a particle from y1 to y 2 is given byW = ∫ F ⋅ dy … (i)
maximum, i.e. at P. y1

Hence, A → r, B → s, C → p. Given, force, F = 20 + 10 y , y1 = 0 and y 2 = 1m


Substituting the given values in Eq. (i), we get
3 (b) v f − vi = Area of a-x graph = 12 ms −1 1
1
 10 y 2 
W = ∫ (20 + 10 y )dy = 20 y +
∴ v f = 12 + 4 = 16 ms −1 0  2  0
1 = 20 (1 − 0 ) + 5(1 − 0 )2 = 25 J
∆ KE = m (v f2 − vi2 ) = 120 J
2 ∴Work done by the force will be 25 J.
Total work done by all the forces = ∆ KE = 120 J 2 (b) In stretching a wire, the work
1 done against internal restoring force
Final KE, Kf = mv f2 = 128 J is stored as elastic potential energy in L L
2
the wire and is given by
Work done by conservative forces = Ui − U f = 240 J 1
U = W = × Force (F ) ×
Work done by external forces 2 l
Elongation (l )
= Total work done − Work done by conservative forces
1 1 1 M
= − 120 J = Fl = × Mg × l = Mgl
2 2 2
Hence, A → s, B → q, C → p, D → r. Mg
3 (c) The area under the
1
4 (d) In t = 4 s, v = at = 8 ms −1 and s = at 2 = 16 m force-displacement curve gives the amount of work done.
2 From work-energy theorem,W = ∆KE …(i)
1 2
KE = mv = 32 J ∴ At x = 8 m, W = Area ABDO + Area CEFD
2
290 OBJECTIVE Physics Vol. 1

= 20 × 5 + 10 × 3 = 130 J Differentiating above equation w.r.t. t, we get


dK 1 dv dv dp  dv dp 
= m 2v = vm =v Q m = 
20
A B
dt 2 dt dt dt  dt dt 
dK  dp 
= vF Q Force, F = 
10
C E K L
dt  dt 
M 1 dK 1
0
O D F J x(m) ∴ F = ⋅ = × 9.6 = 3.2 N
4 5 8 10 12 v dt 3
–10 W
6 (a) Q Efficiency = o × 100
G Wi
–20 I
–25 Given, Wo = 4000 J and Wi = 4000 + 1000 = 5000 J
H
4000
Given, m = 500 g = 500 × 10 −3 kg Q Efficiency (η) = × 100 = 80 %
5000
Using Eq. (i), we get 7 (a) Relation between kinetic energy and momentum is
1 1
⇒ 130 = mv 2 = × 500 × 10 −3 × v 2 p1 = 2mK1
2 2
Q Kinetic energy is increased by 4 times, then K2 = 4K1
⇒ v = 2 130 = 22 .8 ms −1 ≈ 23 ms −1
Hence, p 2 = 2mK2 = 2m (4K1)
At x = 12 m,
p 2 = 2 2mK1 or p 2 = 2p1
W = Area ABDO + Area CEFD
+ Area FGHIJ + Area KLMJ 8 (c) Net force required to lift a hook and load,
 1  Fnet = 1000 + 10000 = 11000 N
W = 20 × 5 + 10 × 3 +  −20 × 2 + × (−5) × 2
 2  Power required to lift the hook, P =
W
+ 10 × 2 t
(Q Area FGHIJ = Area FGIJ + Area GHI ) As, W = Fnet d
= 100 + 30 − 40 − 5 + 20 = 105 J F d d   d
∴ P = net = Fnet   = Fnetv Qv = 
t t  t
Using Eq. (i), we get
1 or P = 11000 × 0.5 = 5500 W = 5.5 kW
∴ 105 = × 500 × 10 −3 × v 2
2 9 (c) When the spring is cut into pieces, they will have the new
⇒ ~ 20.6 ms −1
v = 2 105 − force constant. The spring is divided into 1 : 2 : 3 ratio.
When the pieces are connected in series, the resultant force
4 (c) If x be the compression in the spring, when a block of constant will be given by
0.25 kg is released. 1 1 1 1
= + +
From law of conservation of energy, k′ k1 k2 k3
Potential energy of spring = Potential energy of block 1 1 1 1 6x
= + + ⇒ k′ =
1 2
kx = mgx k′ x 2x 3x 11
2 In parallel, the net force constant,
where, k is force constant of spring. k′′ = x + 2x + 3x = 6x
kx = 2 mg = 2 × 0.25 g k′ 6x /11
The required ratio = = 1: 11
kx = 0.5 g k′′ 6x
∴Force applied by the spring on the block, F = kx = 0.5 g 10 (c) Given, F = − k ( y$i + x$j)
From free body diagram, N
As, work done, W = ∫ F ⋅ dr
∴ Maximum force by system on the floor,
N = F + 2g = 0.5 g + 2 g = 2 . 5g So, W = −k ∫ ( y$i + xj
$$ ) ⋅ (dxi$ + dy$j)
2kg
(Q g = 10 ms −2)
⇒ W = −k ∫ ( ydx + xdy )
= 25 N
F 2g Y
5 (a) Given, rate of decrease in kinetic energy,
dK
= 9.6 Js −1
dt (a, a)
When speed, v = 3 ms −1, then force F = ? r
1
Kinetic energy, K = mv 2 (0, 0) (a, 0)
X
2
Work, Energy and Power 291

⇒ W = − k∫
(a , a )
d (xy ) [Q ydx + xdy = ∫ d (xy )] For a sufficiently safe-horizontal displacement, ∆s can be
(0 , 0 ) considered straight. If the corresponding length of path
Hence, W = −k (xy )((0a ,, a0 )) = −ka 2 element is ∆L, the frictional force is given by
∆F = µmg (∆L cos θ )
F 1 ∆s
11 (d) As we know, k = ⇒ k∝ From ∆OPQ, cos θ =
l l ∆L
k2 l1 1
⇒ = =  ∆s 
k1 l2 2 So, ∆F = µmg∆L   = µmg∆s
 ∆L 
k1 = 2k, k2 = k
On adding up, we find that along the whole path, the total
1 1 1 1 1 3 work done by the friction force is µmgs. By law of conservation
In series, = + = + =
k′ k1 k2 2k k 2k of energy, this must equals to the decrease in potential energy
2k of skier.
Q k′ =
3 ∴ µmgs = mgh
So, both Assertion and Reason are incorrect. Hence, h = µs

12 (a) Free body diagram of block is as shown below. 15 (c) According to question, a body of mass 1 kg begins to move
under the action of time dependent force,
N
F = (2t $i + 3t 2 $j ) N
dv
kx F m = 2t$i + 3t 2$j
dt

mg ∫ ∫
dv = (2t$i + 3t 2$j )dt (Q m = 1kg)

v = t 2$i + t 3$j
Now, from law of conservation of energy,W = ∆K
1 ∴ Power developed by the force at the time t will be given by
⇒ WF + Wsp = mv 2 P = F ⋅ v = (2t$i + 3t 2$j ) ⋅ (t 2$i + t 3$j )
2
1 2 1 2 = (2t ⋅ t 2 + 3t 2 ⋅ t 3 )
⇒ F ⋅ x − kx = mv
2 2 P = (2t 3 + 3t 5 ) W
2F ⋅ x − kx 2
∴ v= 16 (d) From work-energy theorem,
m Wtangential = ∆KE = Kf − Ki = Kf − 0
13 (d) Given, m = 70 kg, g = 10 ms − 2 and h = 12 m (maT ) × S = 8 × 10 – 4
In going up and down once, Here, S = 2 × 2πr
 mgh  8 × 10 − 4 2 × 10 − 2
Number of k-cal burnt = mgh +  aT = − 2 =
 2  10 (2πr × 2) π × 6 . 4 × 10 − 2
3 3 70 × 10 × 12
= mgh = × ~ 0.1 ms −2

2 2 4.2 × 1000
17 (a) From work-energy theorem,
= 3 k-cal
Work done = Change in kinetic energy ⇒ W = Kf − Ki
Total number of k-cal to be burnt to loose 7 kg of weight x2 1
= 7 × 9000 = 63000 k-cal ⇒ Kf =W + Ki = ∫ Fdx + mv 2
x1 2
∴Number of times the person has to go up and down the 30 1
stairs = ∫ − 0.1x dx + × 10 × 10 2
20 2
63000
= = 21000 = 21 × 10 3 times  x2 
30
3 = −0.1  + 500 = − 0.05 [30 2 − 20 2] + 500
14 (a) According to question, the given situation is shown in the  2  20
figure below. = − 0.05 [900 − 400] + 500
Q ⇒ Kf = − 25 + 500 = 475 J
∆L
A θ 18 (a) As, the machine delivers a constant power.
O P
∆s So, F ⋅v = constant = k (watts)
B
∆L dv k
⇒ m ⋅v = k ⇒ ∫ v dv = ∫ dt
dt m
s v2 k 2k
∆s ⇒ = t ⇒ v= t
2 m m
292 OBJECTIVE Physics Vol. 1

Now, force on the particle is given by 24 (d) From v = u + at, v1 = 0 + at1


1
mv  v
dv d  2kt  2 F = ma = 1 Q a = 1
F =m =m    t1 
dt dt  m  t1
v
Velocity acquired in t second = at ⇒ v = 1 t
 1 − 1 mk −1/ 2 t1
= 2km  t 2 = ⋅t
2  2  mv  v   mv  2
Power = F v =  1  1 t =  21  t
 t1   t1   t1 
1
19 (c) Given, force, F = 10 + 0.5x = 10 + x
2 25 (a) Consider the diagram. Let the mass of chain be m.
Let during small displacement, the work done by the force is m
∴ Mass per unit length =
dW = Fdx. l
So, work done during displacement from x = 0 to x = 2 is Consider an elementary length of the chain at a depth x below
x 2 2 1  the table. The potential energy of their parts is given by
W = ∫ dW = ∫ Fdx = ∫ 10 + x dx
0 0 0 2  l/2
x 2 2
= 10 [x]20 +   = 21J Surface of table x
 4 0 l/2

20 (a) Given, m = 11.7 kg, s = 4.65 m, h = 2.86 m


h 2.86
From the figure, sin θ = = = 0.615 m
s 4.65 dU = − gx dx
Also, component of mg along the incline is mg sin θ. l
∴ Work done by this force = mg sin θ ⋅ s Now, potential energy of hanging part,
l
= 11.7 × 9.8 × 0.615 × 4.65 l
m m x2 2 mgl
= 11.7 × 9.8 × 2.86 = 327.9 −
~ 328 J U1 = ∫ dU = − ∫ 2 gx dx = − g   = −
0 l l  2 0 8
21 (d) Given, mass of elevator = 500 kg
When chain is completely leaves the table, then potential
Velocity = 0.20 ms −1 energy,
Weight of elevator = 500 × 9.8 = F lm mg 2 l − mgl
U2 = − ∫ gx dx = [x ]0 =
Now, power, P = Fv = 500 × 9.8 × 0.20 = 980 W 0 l 2l 2
1 − mgl mgl
Therefore, hp-rating of motor = × 980 = 1.31hp Now, loss in potential energy = +
746 8 2
22 (c) Let the kinetic energy is K and masses of bodies are m1 and From law of conservation of energy,
m 2. Loss in potential energy = Gain in kinetic energy
For first body, p1 = 2m1 K ⇒
1 2 3mgl
mv = ⇒ v=
3gl
2 8 4
∴ p12 = 2m1 K …(i)
26 (a) Given, m = 4 kg, p = 6 N-s
Similarly, for second body, p 22 = 2m 2 K …(ii)
We know that, kinetic energy of a body,
Dividing Eq. (i) by Eq. (ii), we get
p2 (6)2 36
p12 2m1K p1 m1 K= ,K = ⇒ K= = 4.5 J
= , = 2m 2×4 2×4
p 22 2m 2K p 2 m2
p1 27 (a) Given, m = 3.90 kg, v = 1.20 ms −1
⇒ = square root of masses.
p2 and k for spring = 135Nm−1
From law of conservation of energy,
23 (a) Given, m1 = 1 kg and m 2 = 2 kg
1 2 1 2 mv 2
Let kinetic energies of bodies be K1 and K2. mv = kx ⇒ x =
For first body, K1 = Fs1 …(i) 2 2 k
For second body, K2 = Fs2 …(ii) 3 .90 × 1.20 × 1.20
=
Dividing Eq. (i) by Eq. (ii), we get 135
s1 K1 s m v2 = 0.204 m
= ⇒ 1 = 1 12
s 2 K2 s2 m 2 v 2 28 (a) Work done in stretching a string to obtain an extension l,
s m s 1 1
Given, v1 = v 2, so 1 = 1 , 1 = W1 = kl 2
s2 m 2 s2 2 2
Work, Energy and Power 293

Similarly, work done is stretching a string to obtain extension 1


1 and WB = (2k )x 2 = kx 2
(l + l1), W2′ = k (l1 + l )2 2
2 Hence, the ratio of work done in stretching the wires,
Now, work done in second stretching,
WA (1/ 2) kx 2 W 1
1 1 1 = ⇒ A =
W2 = W2′ −W1 = k (l1 + l )2 − kl 2 = kl1(2l + l1) WB kx 2 WB 2
2 2 2
1 2
29 (b) For an instantaneous displacement dx, we can write 36 (a) Work done to accelerate from 0 to v,W1 = mv
2
dW  dx 
P = = F ⋅   = F ⋅v 1
dt  dt  Work done to accelerate from v to 2v,W2 = m (2v )2
2
P = (F ) (at ) = (ma ) (at ) (Q F = ma ) 1 1
= (ma 2 ) t ⇒ P ∝ t (Q ma 2 = constant) ⇒ ∆W = W2 − W1 = m (2v )2 − mv 2
2 2
30 (c) Given, t1 = 2s, t2 = 4 s and h1 = h2 = h = 4W1 − W1 = 3W1
mgh1 37 (b) Given, m = 200 g = 0.2 kg, g = 10 ms −2 and
As, PA = …(i)
t1 H eff = 2 + 0.5 = 2.5 m
mgh2 1 2 2 mg H eff
and PB = …(ii) Here, kx = mg H eff ⇒ k =
t2 2 x2
2 × 0.2 × 10 × 2.5 10
On dividing Eq. (i) by Eq. (ii), we get ⇒ k= ⇒ k= = 40 Nm−1
(0.5)2 2.5
mgh1/t1  h1   t2  t2 4 2
PA : PB = =    = = = (Q h1 = h2 )
mgh2 /t2  h2   t1  t1 2 1 38 (b) Given, the potential energy of a particle in a force field,
A B
⇒ PA : PB = 2 : 1 U= 2−
r r
31 (d) Power of the machine gun dU
1 For stable equilibrium, F = − =0
n ⋅ mv 2 dr
Total work done 2
= = 2A B 2A
Time t 0 = − 3 + 2 or =B
r r r
1 mv 2  1 2 
=n⋅ Q K = mv , t = 1 s The distance of particle from the centre of the field, r =
2A
2 t  2 
B
∴ The power of the machine gun = nK 1 2 1
39 (b)W = KE = mv = × 50 × (20 )2 = 10 4 J
32 (a) Given, force, F = (3 $i + $j) N 2 2
r1 = (2$i + k$ ) m and r2 = (4$i + 3$j − k$ ) m 1
40 (b) Kinetic energy of a particle in motion, E = mv 2
2
∴ s = r − r = (4$i + 3$j − k$ ) − (2$i + k$ )
2 1
Differentiating above equation w.r.t. x, we get
= (2$i + 3$j − 2k$ ) m
dE 1 dv dv dt a
∴ W = F ⋅ s = (3$i + $j ) ⋅ (2$i + 3$j − 2k$ ) = m × 2v = mv × × = mv × = ma
dx 2 dx dt dx v
= 3 × 2 + 1× 3 + 0 = 6 + 3 = 9 J So, the slope of kinetic energy-displacement curve is directly
Work done proportional to the acceleration of the particle.
33 (a) Power =
Time 41 (b) According to law of conservation of momentum,
Work done = mgh = 100 × 9.8 × 10 v
mv1 + 2 mv 2 = 0 or v 2 = − 1 …(i)
Time = 1 min = 60 s 2
100 × 9.8 × 10 According to law of conservation of energy,
∴ P = = 163.3 W
60 1 2 1
mv1 + 2mv 22 = 60
34 (c) Work done, W = mg sin θ × s 2 2
2
= 300 × 9 . 8 × sin 30 ° × 10 1 2  −v 
⇒ mv1 + m  1 = 60
1 2  2 
= 300 × 9 . 8 × × 10 = 14700 J
2 1 2 1 2
⇒ mv1 + mv1 = 60
1 2 4
35 (b) Work done in a stretched wire,W = kx 2
2 3mv12
Given, kA = k and kB = 2 k ⇒ = 60
4
1 1 2
⇒ WA = kx 2 ⇒ mv1 = 40 J
2 2
CHAPTER
07

Circular Motion
Everyday in different activities of our daily life, we use circular motion. For
example, to grind wheat from a flour mill, its wheel is given circular motion; to
wash clothes, parts of a washing machine perform circular motion. Similarly, some
natural phenomena like motion of the earth around the sun, motion of moon
around the earth, etc., all are examples of circular motion. In this chapter, we will
study circular motion in detail.

CIRCULAR MOTION AND ITS TYPES


In circular motion, an object moves along the perimeter or circumference of a circle.

Fig. 7.1 Circular motion

Circular motion can be categorised into two types as follows


Uniform circular motion If a particle moves along a circular path with a
constant speed (i.e. it covers equal distances along the circumference of the circle
in equal intervals of time), then its motion is said to be uniform circular motion.
e.g. (i) Motion of a point on the rim of a wheel rotating uniformly. Insisde
ide
(ii) Motion of the tip of the second hand of a clock.
In
1 Circular motion and its types
Non-uniform circular motion If the speed of the particle in circular motion 2 Kinematics of circular motion
1 Circular motion and its types
changes with respect to time, then its motion is said to be non-uniform circular Centripetal acceleration
2 Kinematics of circular motion
motion. Acceleration of a particle in
Centripetal acceleration
non-uniform circular motion
e.g. Motion of a stone tied to a string moving in vertical circle. Acceleration of a particle in
3 Dynamics of circular
non-uniform circularmotion
motion
Centripetal force
3 Dynamics of circular motion
KINEMATICS OF CIRCULAR MOTION Centrifugal
Centripetal force
force
Examples
Centrifugal forforce
obtaining
For a particle in circular motion, following variables are needed to describe its centripetal
Examples for force in daily life
obtaining
motion centripetal
Motion force in daily
of a particle life
tied to a
string
Motion inof
vertical circle
a particle tied to a
string in verticle circle
Radius vector (r)
When a particle moves on a circular path, its distance from the centre is fixed and
it is equal to radius of the circle. If the centre of the circle is taken as origin, then
Circular Motion 295

the vector joining centre to the particle is called radius It is directed along a line passing through centre (O) and
vector. It is directed from centre to the particle and its perpendicular to the plane of circular motion, containing
magnitude is same as radius. r and ∆s.
B t=t
Note If a particle makes N revolutions, its angular displacement is
r θ = 2 πN rad
O A t=0
Angular velocity (ω )
r

Rate of change of angular displacement of a particle


Fig. 7.2 Radius vector in circular motion performing circular motion is called angular velocity.
∆θ
Angular position Angular velocity, ω =
∆t
Suppose a particle P is moving on a circular path of radius ∆θ dθ
r and centre O, as shown in figure. Instantaneous angular velocity, ω = lim =
Y ∆t → 0 ∆ t dt
P ′∆s
P
It is also a vector quantity. Its unit is rads–1, rpm, rps, etc.,
∆θ and its dimensional formula is [M 0 L0 T −1].
θ Direction of angular velocity Direction of angular
O r X
velocity is given in the same way as that of angular
displacement.
Observer Observer
Fig. 7.3 Moving particle on a circle
ω
The position of the particle P at a given instant may be
described by the angle θ between OP and OX. This angle θ
is called the angular position of the particle. As the particle
moves on the circle, its angular position θ changes. Here, P ω
and P ′ are given as P (r, θ ) and P ′ (r, θ + ∆θ ), respectively. Screw Screw
advancement advancement
Angular displacement ( ∆θ ) (a) Anti-clockwise motion (b) Clockwise motion
The angle traced out by the radius vector at the centre of
Fig. 7.5 Directions of angular velocities
the circular path in the given time is called angular
displacement. It is denoted by ∆θ and expressed in Let the particle rotates in anti-clockwise direction as seen
radians. In the Fig. 7.3, OP is the initial and OP′ is the by observer, angular velocity is along a line passing
final position vectors of the particle. through the centre and perpendicular to the plane of the
Angular displacement circle and towards the observer. This is along a direction,
Linear displacement between two positions where screw advances as shown in figure.
= Similarly, when the particle rotates in clockwise direction as
Radius vector
∆s seen by the observer, angular velocity is away from the observer.
∆θ = Important points regarding the angular velocity are given below
r
Small angular displacement is taken as a vector quantity (i) If the particle performing circular motion completes
and its SI unit is radian. one rotation around the circular path inT seconds, then
Direction of angular displacement Direction of ∆θ is 2π
Angular velocity, ω = rad s −1
given by right handed (i.e. the direction, where screw T
advances) screw rule as shown in figure. (ii) If the particle performing circular motion makes
∆θ n rotations per second around the circular path, then
Angular velocity, ω = 2πn rad s −1
r
O ∆s (iii) If two particles are moving on the same circle in the
r same direction with different constant angular
speeds ω1 and ω 2, then the angular speed of particle
2 with respect to 1 for an observer at centre will be
Fig. 7.4 Direction of advancement of screw ω = ω 2 − ω1
296 OBJECTIVE Physics Vol. 1

(iv) Angular velocity depends on axis of rotation. Relation between linear velocity and angular velocity
α B A particle performing circular motion also has linear
ωO =
t velocity (as it cover linear displacement along circular
θ α
θ α /2 ω o
P A path) along with angular velocity. If linear velocity of
O
ωP = = = particle performing circular motion is v and angular
t t 2 velocity is ω, then both of these velocities are related as
Fig. 7.6
v = rω
Example 7.1 Calculate the average angular velocity of the
hour hand of a clock. where, r is radius of circular path. In vector form,
v = ω ×r
Sol. The hour hand completes one round in 12 h. One round
makes an angular displacement 2π. Linear velocity is always along the tangent to the circular path.
∆θ 2 π rad
∴ Average angular velocity, ω av = = Example 7.4 A particle moves in a circle of radius 4 m with a
∆t 12 h linear velocity of 20 ms −1. Find the angular velocity.
2π Sol. Given, linear velocity, v = 20 ms−1
= rad s−1
12 × 3600 Radius, r = 4 m
π
= rad s−1 As, linear velocity, v = r ω
21600
v 20
⇒ Angular velocity, ω = = = 5 rad s −1
Example 7.2 An object revolves uniformly in a circle of r 4
diameter 0.80 m and completes 100 rev min −1. Find its time
period and angular velocity. Example 7.5 If the length of the second’s hand in a stop clock
Sol. Here, diameter = 0.80 m is 3 cm, find the angular velocity and linear velocity of the
Diameter 0.80 tip.
∴ Radius, r = = = 0.4 m
2 2 Sol. Given, radius, r = 3 cm = 3 × 10−2 m
100 Time period of stop clock, T = 60 s
Frequency, n = 100 rev/min = rev/s
60 2π 2π
1 60 Angular velocity, ω = = = 0.1047 rad s −1
Therefore, time period, T = = s = 0.6 s T 60
n 100 and linear velocity, v = ω r = 0.1047 × 3 × 10−2
2π 2 × 3.14
∴ Angular velocity, ω = = rad/s = 0.00314 ms −1
T 0.6
= 10.467 rad/s
Angular acceleration (α )
Example 7.3 A threaded rod with 12 turns per cm and
diameter 1.18 cm is mounted horizontally. A bar with a The rate of change of angular velocity of a particle performing
threaded hole to match the rod is screwed onto the rod. The circular motion is called angular acceleration.
bar spins at the rate of 216 rpm. Determine the velocity of ∆ω
Angular acceleration, α =
the bar with which it will move the length of 1.50 cm along ∆t
the rod. Also, find the time taken by it.
Instantaneous angular acceleration,
216
Sol. Given, frequency, n = 216 rpm = rps
60 ∆ω dω d 2θ
1 α ins = lim = = 2
Length of one turn = cm ∆t → 0 ∆t dt dt
12
∴ Number of rotations required to move a distance of 1.5 cm,
Distance 1.5 The SI unit of angular acceleration is rad s −2 and its
N = = = 18 dimensional formula is [M 0 L0T –2 ] . If α = 0, circular
Length of one turn 1 12
motion is said to uniform.
Therefore, angular displacement, θ = 2πN = 2π × 18 = 36 π rad
216 It has same characteristics as that of angular velocity.
∴ Angular velocity of the bar, ω = 2πn = 2π ×
60 Note Angular acceleration is an axial vector, when axis of rotation is
. π rad s−1
= 72 fixed, angular acceleration and angular velocity vectors both lie
along that axis.
Angular displacement (θ)
∴ Required time (t ) = Example 7.6 A point on the rim of a disc starts circular
Angular velocity (ω )
motion from rest and after time t, it gains an angular
36π acceleration which is given by α = 3 t − t 2 . Calculate the
= = 5s
. π
72 angular velocity after 2 s.
Circular Motion 297

dω because of change in direction of motion. Therefore,


Sol. Angular acceleration, α = = 3t − t2
ω t
dt motion of the particle is accelerated. This acceleration is
3t 2 t 3
⇒ ∫ d ω = ∫ (3t − t ) dt ⇒ ω = 2 − 3
2
called centripetal acceleration and it is directed along
0 0 radial direction towards centre of the circle. It is given by
10
At t = 2 s, ω= rad s −1 v2
3 ar = a c =
r
Relation between angular variables
where, v = magnitude of linear velocity of the particle
(Kinematic equations of circular motion)
and r = radius of circular path.
If angular acceleration is constant, then we have kinematic It is also called radial acceleration.
equations of circular motion as follows
If we put, v = r ω
1
(i) θ = ω 0 t + αt 2 (ii) ω = ω 0 + αt where, ω is angular velocity, then centripetal acceleration,
2
1 ar = a c = r ω 2
(iii) ω 2 = ω 20 + 2αθ (iv) θ t = ω 0 + α (2t − 1)
2
Example 7.9 Determine the magnitude of centripetal
ω + ω0  acceleration of a particle on the tip of a fan blade, 0.30 m in
(v) θ =  t
 2  diameter, rotating at 1200 rev min −1.
Here, ω 0 and ω are the angular velocities at time t = 0 and Sol. Given, diameter = 0.30 m
t; θ and θ t are the angular displacements in time t and 0.30
∴ Radius, r = = 0.15 m
t thsecond, respectively. 2
Example 7.7 The wheel of a motor rotates with a constant 1200
and frequency, n = 1200 rev min −1 = = 20 rps
acceleration of 4 rad s −2 . If the wheel starts from rest, how 60
many revolutions will it make in the first 20 s?
∴ Angular velocity, ω = 2πn = 2π × 20 = 40 π rad s −1
Sol. The angular displacement in the first 20 s is given by
Therefore, centripetal acceleration, a c = r ω 2
1 1
θ = ω 0t + αt 2 = (4 rads −2 )(20 s )2 ⇒ a c = 0.15 × (40π )2 ⇒ a c = 2368.7 ms−2
2 2
(Q Angular velocity, ω 0 = 0)
= 800 rad Example 7.10 Find the acceleration of a particle placed on
As, the wheel turns by 2π radian in each revolution, the
the surface of the earth at the equator, due to the earth’s
θ 800 rotation. The radius of earth is 6400 km and time period of
number of revolutions in 20s is N = = revolution of the earth about its axis is 24 h.
2π 2π
= 127.38 −~ 127 Sol. Given, radius of earth, R e = 6400 km = 6400 × 103 m
Time period, T = 24 h = 24 × 60 × 60 s
Example 7.8 The wheel of a car accelerated uniformly from
2π 2π
rest, rotates through 1.5 rad during the first second. Find the Angular speed of the earth, ω = = rad s −1
angle rotated during the next second. T 24 × 60 × 60
Sol. As the angular acceleration is constant, we have Acceleration of the particle,
2
1 1
θ = ω 0t + αt 2 = αt 2 (Q Angular velocity, ω 0 = 0)  2π  −2
a c = ω 2R e =   × (6400 × 10 ) = 0.034 ms
3
2 2  24 × 60 × 60
1
∴ 1.5 rad = α (1)2
2 Example 7.11 Two particles A and B start at the origin O
Angular acceleration, α = 3 rads −2 and travel in opposite directions along the circular path at
constant speeds v A = 0.7 ms −1 and v B = 1.5 ms −1,
The angular displacement in first two second is given by respectively. Determine the time when they collide and the
1 magnitude of the acceleration of B just before this happens.
θ1 = × (3) (2)2 = 6 rad
2 Y
Thus, the angle rotated during the 2nd second
= θ1 − θ = 6 rad − 1.5 rad = 4.5 rad m
5.0

Centripetal acceleration B
When a particle is in uniform circular motion, its speed A
remains constant but velocity changes continuously O
X
vB = 1.5 ms–1 vA = 0.7 ms–1
298 OBJECTIVE Physics Vol. 1

Sol. From the condition given in the question, it is clear that


total distance (= velocity × time) will be equal to the Net acceleration
circumference of circular path, Consider the particle moving on circular path in anti-clockwise
i.e. vBt + v At = circumference of circular path direction with increasing speed as shown in figure.
1.5 t + 0.7t = 2πR = 10π at
10π
∴ Time, t = = 14.3 s
2.2 a
2
v (1.5)2 φ
Hence, acceleration, a = B = = 0.45 ms−2
R 5 O ar

Acceleration of a particle in
non-uniform circular motion Fig. 7.7 Non-uniform circular motion
If a particle is in non-uniform circular motion, i.e. its We know that, linear velocity, v = ω × r
speed is not constant, then the particle has both radial and Differentiating on both sides w.r.t. time t, we get
tangential components of acceleration. dv dω dr
Net acceleration, = × r +ω ×
dt dt dt
Radial component (ar ) ⇒ Net acceleration, a = α × r + ω × v ⇒ a = at + ar
This component of acceleration is towards the centre. This
is responsible for change in direction of velocity. This is (Q at = α × r and ar = ω × v)
v2
equal to or rω 2 . Magnitude of net acceleration, a = ar2 + at2
r
v2 This resultant acceleration makes an angle φ with the
Thus, ar = = rω 2
r at
radius, where tan φ =
Note The radial acceleration ( ar ) is also sometimes called normal
ar
acceleration ( an).
Note
(i) In accelerated circular motion, dv / dt is positive and hence,
Tangential component (at ) tangential acceleration of the particle is parallel to velocity v.
This is the component of acceleration in the direction of (ii) In decelerated circular motion, dv /dt is negative and hence,
tangential acceleration is anti-parallel to velocity v.
velocity, which is responsible for change in speed of
particle. It is also equal to rate of change of speed. Example 7.13 A car is travelling along a circular curve that
Hence, at = component of a along v has a radius of 50 m. If its speed is 16 ms −1 and is
increasing uniformly at 8 ms −2 , determine the magnitude of
dv d | v |
= = its acceleration at this instant.
dt dt Sol. Given, tangential acceleration a t = 8 ms−2
dω Radius, R = 50 m, speed, v = 16 ms−1
⇒ at = ×r (Q v = ω × r )
dt v 2 (16)2 256
∴ Radial acceleration, a r = = = ms −2
⇒ at = α × r R 50 50
Magnitude of net acceleration of the car,
⇒ at = rα
2
This component is tangential. 2 2  256 −2
a = a t + a r = (8)2 +   = 9.5 ms
 50 
Example 7.12 A particle moves in a circle of radius 0.5 m at
a speed that uniformly increases. Find the angular Example 7.14 The speed of a particle moving in a circle of
acceleration of particle, if its speed changes from 2 ms −1 to radius r = 2 m varies with time t as v = t 2 , where t is in second
4 ms −1 in 4 s. and v in ms −1. Find the radial, tangential and net acceleration
Sol. The tangential acceleration of the particle is at t = 2 s.
dv 4 − 2 Sol. Given in the question, v = t 2
at = = = 0.5 ms−2
dt 4
Linear speed of particle at t = 2s is v = (2)2 = 4 ms−1
a 0.5
The angular acceleration, α = t = = 1 rad s−2
v 2 (4)2
r 0.5 ∴ Radial acceleration, a r = = = 8 ms−2
r 2
Circular Motion 299

The tangential acceleration is a t =


dv d 2
= (t ) = 2t
constant rate of 0.5 ms −1. Determine the magnitude and
dt dt direction of the net acceleration of the cyclist on the circular
∴ Tangential acceleration at t = 2 s is a t = (2) (2) = 4 ms −2 turn.
5
∴ Net acceleration of particle at t = 2 s is Sol. Linear speed of the cyclist, v = 18 km h−1 = 18 × = 5 ms−1
18
a = (a r )2 + (a t )2 = (8)2 + (4)2 = 64 + 16 and centripetal acceleration of the cyclist,
or a = 80 ms −2 v2 25 1
ac = = = ms−2
R 25 2 2
Example 7.15 A particle moves in a circle of radius 2 cm at a
speed given by v = 4 t, where v is in cms −1 and t in second. v

(i) Find the tangential acceleration at t = 1 s.


(ii) Find total acceleration at t = 1 s. ac
Sol. Given, v = 4t θ
v 2 (4t )2 16t 2
Radial acceleration, a r = = or a r = = 8t2
R R 2 anet at
At t = 1 s, a r = 8 cms −2 dv 1 −2
Tangential acceleration of the cyclist, a t = = ms
dv d dt 2
(i) Tangential acceleration, a t = or a t = (4 t ) = 4 cms −2
dt dt ∴ Net acceleration of the cyclist,
i.e. a t is constant or tangential acceleration at t = 1 s 2 2
 1   1 3
is 4 cms−2. a net =   +   = = 0.86 ms−2,
 2  2 2
(ii) Total acceleration, a = a t2 + a r2
a c 1/ 2 2
tan θ = = = = 2
or a = (4)2 + (8)2 = 80 = 4 5 cms −2 at 1/2 2
Example 7.16 A cyclist is riding with a speed of 18 kmh −1. Angle made by resultant acceleration with tangential
As he approaches a circular turn on the road of radius acceleration,
25 2 m, he applies brakes and reduces his speed at the θ = tan−1 ( 2 )

CHECK POINT 7.1


1. The angular speed of a flywheel making 120 rev min −1 is 6. A body is moving in a circular path with acceleration a. If its
(a) 2 π rad s−1 (b) 4 π 2 rad s−1 speed gets doubled, find the ratio of centripetal acceleration
(c) π rad s−1 (d) 4 π rad s−1 after and before the speed is changed.
(a) 1 : 4 (b) 1 : 2 (c) 2 : 1 (d) 4 : 1
2. The ratio of angular speeds of minute hand and hour hand 7. The circular orbit of two satellites have radii r1 and r2
of a watch is respectively (r1 < r2). If angular velocities of satellites are
(a) 1 : 12 (b) 6 : 1 (c) 12 : 1 (d) 1 : 6 same, then their centripetal accelerations are related as
3. The wheel of a toy car rotates about a fixed axis. It slows (a) a1 > a 2 (b) a1 = a 2
down from 400 rps to 200 rps in 2 s. Then, its angular (c) a1 < a 2 (d) Data insufficient
retardation (in rad s −2) is (rps = revolutions per second) 8. A particle is moving on a circular track of radius 30 cm with
(a) 200 π (b) 100 π a constant speed of 6 ms −1. Its acceleration is
(c) 400π (d) None of these
(a) zero (b) 120 ms−2 (c) 1.2 ms−2 (d) 36 ms−2
4. A wheel is rotating at 900 rpm about its axis. When the
power is cut off, it comes to rest in 1 min. The angular
9. A particle starts moving along a circle of radius (20/ π) m with
retardation (in rad s −2) is constant tangential acceleration. If velocity of the particle is
π π π π 50 m/s at the end of the second revolution after motion has
(a) (b) (c) (d) began, the tangential acceleration
2 4 6 8
(in ms −2) is
5. The motor of an engine is rotating about its axis with an (a) 1.6 (b) 4 (c) 15.6 (d) 31.25
angular velocity of 100 rev min −1 . It comes to rest in 15 s
after being switched off, assuming constant angular 10. Let ar and at represent radial and tangential accelerations.
deceleration. What is the number of revolutions made by it The motion of a particle may be circular, if
before coming to rest? (a) a r = 0, a t = 0 (b) a r = 0, a t ≠ 0
(a) 12.5 (b) 40 (c) 32.6 (d) 15.6 (c) a r ≠ 0, a t = 0 (d) None of these
300 OBJECTIVE Physics Vol. 1

11. A point starts from rest and moves along a circular path 14. A particle moves in a circular path of radius R with an
with a constant tangential acceleration. After one rotation, angular velocity ω = a − bt, where a and b are positive
the ratio of its radial acceleration to its tangential constants and t is time. The magnitude of the acceleration
acceleration will be equal to 2a
1 of the particle after time is
(a) 1 (b) 2π (c) π (d) 4π b
2 a
(a) (b) a 2R (c) R (a 2 + b) (d) R a 4 + b 2
12. A particle is moving on a circular path of 10 m radius. At R
any instant of time, its speed is 5 ms −1 and the speed is 15. The distance of a particle moving on a circle of radius 12 m
increasing at a rate of 2 ms −2. At this instant, the measured from a fixed point on the circle is given by s = 2 t 3
magnitude of the net acceleration will be
(in metre). The ratio of its tangential to centripetal
(a) 3.2 ms−2 (b) 2 ms−2 (c) 2.5 ms−2 (d) 4.3 ms−2
acceleration at t = 2s is
13. A point on the rim of a flywheel has a peripheral speed of (a) 1 : 1 (b) 1 : 2 (c) 2 : 1 (d) 3 : 1
10 ms −1 at an instant when it is decreasing at the rate of
16. A body is moving on a circle of radius 80 m with a speed
60 ms −2. If the magnitude of the total acceleration of the 20 m/s which is decreasing at the rate 5 ms −2 at an instant.
point at this instant is 100 ms −2, the radius of the flywheel is The angle made by its acceleration with its velocity is
(a) 1.25 m (b) 12.5 m (a) 45° (b) 90°
(c) 25 m (d) 2.5 m (c) 135° (d) 0°

DYNAMICS OF CIRCULAR F = mr ω 2 or F = mv ω
MOTION Putting ω=

In this section, we will start with the forces in circular T
motion and further we will discuss their practical or ω = 2π n
utilisation in the applications of circular motion. F = 2πmvn
2πmv
Centripetal force ⇒ F =
T
When a body moves along a circular path with uniform
speed, its direction changes continuously, i.e. velocity In vector form, centripetal force is given by
keeps on changing on account of a change in direction.
mv 2
According to Newton’s first law of motion, a change in the F=− r$ = − mω 2r
direction of motion of the body can take place only if some r
external force acts on the body. Centripetal Force in Different Situations
Thus, a particle performing circular motion is acted upon Situation The centripetal force
by a force directed along the radius towards the centre of
A particle tied to a string and Tension in the string
the circle, this force is called the centripetal force. whirled in a horizontal circle
v
Vehicle taking a turn on a level Frictional force exerted by the
road road on the tyres
F A vehicle on a speed breaker Weight of the body or a
O m ω
component of weight
Revolution of earth around the sun Gravitational force exerted by
the sun
Fig. 7.8 Centripetal force on the particle
Electron revolving around the Coulomb attraction exerted by
If m is mass of the particle performing circular motion as nucleus in an atom the protons on electrons
shown in Fig. 7.8, then magnitude of centripetal force is A charged particle describing a Magnetic force exerted by the
given by circular path in a magnetic field magnetic field
Centripetal force = Mass × Centripetal acceleration Few important points related to circular motion
v 2  (i) In non-uniform circular motion, the particle
∴ F =m   (In magnitude)
 r  simultaneously possesses two forces
Putting v = r ω, we get mv 2
(a) Centripetal force, Fc = ma c = = mrω 2
r
Circular Motion 301

(b) Tangential force, Ft = mat F = mrω 2 = 0.12 × 0.5 × (24.2)2 = 35.1 N


This is the breaking tension of the string.
∴ Net force, Fnet = ma = m a c2 + at2 Example 7.19 A spaceman in training is rotated in a seat at
the end of a horizontal arm of length 5 m. If he can
(ii) If a moving particle comes to stand still, then the withstand accelerations upto 9 g, then what is the maximum
particle will move along the radius towards the number of revolutions per second permissible? (Take,
centre and if radial acceleration ar is zero, the body g = 10 ms − 2 )
will fly off along the tangent. So, a tangential Sol. In circular motion, necessary centripetal force to the
velocity and a radial acceleration (hence, force) is a spaceman is provided by effective weight of spaceman.
must for uniform circular motion.
∴ m × 9 g = mrω 2 = mr × 4π 2n 2
(iii) The work done by the centripetal force is always
zero as it is perpendicular to velocity and 9g 9 × 10
⇒ n= ⇒n=
displacement. 4π 2r 4 × (3.14)2 × 5
Further, by work-energy theorem, = 0.675 rev s− 1 or hertz (Hz)
Work done = Change in kinetic energy Example 7.20 A string breaks under a load of 4.8 kg. A
∴ ∆K = 0 (Q ∆W = 0 ) mass of 0.5 kg is attached to one end of the string 2 m long
i.e. K (kinetic energy) remains constant. and is rotated in a horizontal circle. Determine the number of
Work 0 revolution per minute that the mass can make without
Power = = =0 breaking the string.
Time t
Sol. Given, m = 0.5 kg, r = 2 m, g = 9.8 ms−2
or Power = Fc ⋅ v = Fc v cos 90 ° = 0
The maximum tension that the string can withstand,
(iv) Total work done,W = Ft ⋅ s = Ft s cos 0 ° = Ft s
F = 4.8 kg-wt = (4.8 × 9.8)N
(where, s is distance travelled by the particle)
Let the maximum number of revolutions per second be n.
Example 7.17 A ball of mass 0.25 kg attached to the ends of Q F = mr ω 2 = mr (2πn )2
a string of length 1.96 m is rotating in a horizontal circle.
The string will break, if tension is more than 25 N. What is F 9.8 × 4.8
⇒ n2 = = = 1.193
the maximum velocity with which the ball can be rotated? 4π 2mr 4 × (3.14)2 × 0.5 × 2
mv 2 ⇒ n = 1.193 = 1.092 rps
Sol. The centripetal force, F =
r
⇒ n = 1.092 × 60 = 65.52 rpm
In the given situation, centripetal force (F ) is provided by
tension (T ) in the string.
The string will break, if F = T ≥ 25 N Centrifugal force
Hence, string will not break, if F = T ≤ 25 Centrifugal force can be defined as the radially directed
mv 2
25 × r 25 × 1.96
outward force acting on a body in circular motion.
⇒ ≤ 25 ⇒ v 2 ≤ ⇒ v2 ≤ ⇒ v ≤ 14 v
r m 0.25
−1
∴ Maximum velocity of the ball, v max = 196 = 14 ms
m
O
Example 7.18 A ball of mass 0.12 kg is being whirled in a T Centrifugal
horizontal circle at the end of a string 0.5 m long. It is force on body
capable of making 231 revolutions in one minute. Find the mg
breaking tension of the string .
Fig. 7.9 Outward force on a body
Sol. Given, m = 0.12 kg, r = 0.5 m,
Centrifugal force = Mass × Centrifugal acceleration
2π × 231
ω = 231 rpm = rads−1 = 24.2 rad s −1 mv 2
60 or F = = mr ω 2
r

O
T or F = mv ω
r
This can be written in vector form as

When body performs circular motion, it is acted upon by a mv 2


F= r$
centripetal force, magnitude of which is given by r
302 OBJECTIVE Physics Vol. 1

mv 2
where, $r is the unit vector acting along r. Required centripetal force, f =
(i) In an inertial frame, the centrifugal force does not r
act on the object. Further, limiting value of f is µN
(ii) In non-inertial frame, pseudo force arises in the form or fL = µN = µ mg (Q N = mg )
of centrifugal force. Here, N = normal reaction force on the car by the
road
Example 7.21 A gramophone disc rotates at 60 rpm. If coin
of mass 18 g is placed at a distance of 8 cm from the centre.
and µ = coefficient of friction between road and
Determine the centrifugal force on the coin. tyre of the car.
2π Therefore, for a safe turn without sliding,
Sol. Since, angular velocity, ω = 60 × = 2π rad s−1
60 mv 2 mv 2 v2
≤ fL or ≤ µ mg or µ ≥
∴ Centrifugal force, F = mω 2r = 18 × (2π )2 × 8 r r rg
= 18 × 4 × (3.14)2 × 8 or v ≤ µ rg
⇒ F = 5679.13 dyne ≈ 0.06 N
Here, two situations may arise. If µ and r are known
to us, the speed of the vehicle should not exceed
Examples for obtaining µ rg and if v and r are known to us, the coefficient
centripetal force in daily life of friction should be greater than v 2 /rg.
Some examples for obtaining centripetal force in daily life Maximum velocity for no skidding or slipping,
are given below
v max = µrg
1. Circular turning of roads
It is most popular example of circular motion. When Note You might have seen that if the speed of the car is too high, car
starts skidding outwards. With this radius of the circle increases
vehicles go through turns, they travel along a nearly
or the necessary centripetal force is reduced
circular arc. There must be some force which will produce Q centripetal force ∝ 1 .
the required centripetal acceleration. If the vehicles travel  
 r
on a horizontal circular path, this resultant force is also
Example 7.22 Determine the maximum speed at which a car
horizontal. can turn round a curve of 30 m radius on a level road, if the
The necessary centripetal force is being provided to the coefficient of friction between the tyres and the road is 0.4.
vehicles by following three ways (Take, g = 10 ms −2 )
(i) By friction (ii) By banking of roads Sol. Given, µ = 0.4, r = 30 m, g = 10 ms −2
(iii) By friction and banking of roads Maximum speed, v max = µ gr
In practical, the necessary centripetal force is provided by ⇒ v max = 0.4 × 10 × 30 = 10.95 ≈ 11 ms −1
friction and banking of roads. Now, let us write equations
of motion in each of the three cases separately and see Example 7.23 A cyclist speeding at 4.5 km h −1 on a level
road takes a sharp circular turn of radius 3 m without
what are the constraints in each case. reducing the speed. The coefficient of static friction between
(i) By friction : motion of a car on level road the road and the tyres is 0.1. Will the cyclist slip while
Suppose, a car of mass m is moving with a speed v in a taking the turn (i) with a speed of 4.5 km h −1 and (ii) with a
horizontal circular arc of radius r. In this case, the speed of 9 km h −1?
necessary centripetal force to the car will be Sol. Frictional force provides the necessary centripetal force. He
provided by force of friction f acting towards centre. will slip, if the turn is too sharp (i.e. too small a radius) or if
his speed is too large.
v Maximum speed for no slipping is
v max = µ srg = 0.1 × 3 × 9.8 = 1.72 ms −1
O 5 5
(i) If v = 4.5 km h −1 = 4.5 × = ms −1
r
18 4
= 1.25 ms −1 < 1.72 ms−1, hence he will not slip.
5 5
(ii) If v = 9 km h −1 = 9 × = ms −1 = 2.5 ms −1 > 1.72 ms −1,
Fig. 7.10 Motion of a car on a level road
18 2
hence he will slip.
Circular Motion 303

(ii) By banking of roads : motion of a car on banked h v2 h


Also, tan θ = ⇒ =
road Friction is not always reliable at circular l rg l
turns, if high speed and sharp turns are involved.
v 2l
To avoid dependence on friction, the roads are ⇒ h=
banked by an angle (θ ) at the turn, so that the rg
outer part of the road is somewhat raised (20)2 × 1 1
∴ h= = m
compared to the inner part. 2000 × 10 50
Applying Newton’s second law along the radius and 100
the first law in the vertical direction for the motion = = 2 cm
50
of a car on the banked road having mass m.
(iii) By friction and banking of roads If on a banked
N N cos θ circular turning, there is a frictional force between
mv 2 G car and road, then the vector sum of normal reaction
r N sin θ force (N ) and frictional force (f ) provides the
necessary centripetal force.
Centre of bank r

N
mg N cos θ
N θ
Fig. 7.11 Motion of a car on banked road θ N sin θ
mv 2 f cos θ θ
We get, N sinθ = and N cosθ = mg
r f sin θ
h
Here, r = radius of circular turn, θ = banking angle. f f
From these two equations, we get θ mg mg
θ
b
v2
tanθ = or v = rg tanθ (a) (b)
rg Fig. 7.12 Banked road with friction
Example 7.24 A turn of radius 600 m is banked for a vehicle mv 2
of mass 200 kg going with a speed of 180 kmh −1. Determine ∴ N sin θ + f cos θ = …(i)
r
the banking angle of its path.
and N cos θ = mg + f sin θ …(ii)
Sol. The turn is banked for speed,
5 (Q Vertical force is balanced)
v = 180 kmh −1 = 180 × ms −1 = 50 ms −1
18 Taking limiting condition of friction, we can write
and radius, r = 600 m f = µ sN …(iii)
v2 50 × 50 Here, µ s = coefficient of static friction.
Q tan θ = =
rg 600 × 10 To obtain the value of N, solving above three
25 equations properly, we get
⇒ tan θ = = 0.4167
60 mg
N=
⇒ Banking angle, θ = tan−1 (0.4167) ⇒ θ = 22.62° cos θ − µ s sin θ
Example 7.25 A train has to describe a curve of radius 2000m . After putting the value of N in Eq. (i), we get
By how much should the outer rail be raised with respect to  rg (sin θ + µ s cos θ ) 
1/ 2
inner rail for a speed of 72 km h −1. The distance between v max = 
the rails is 1 m. (Take, g = 10 ms −2 )  cos θ − µ s sin θ 
Sol. Given, v = 72 km h −1 1/ 2
 rg (µ s + tan θ ) 
= 72 ×
5
= 20 ms −1, v max = 
18 h  1 − µ s tan θ 
l = 1 m, r = 2000 m, g = 10 ms−2 θ
2 l
If the vehicle is moving upward on inclined road, then
v we can find maximum speed for no skidding from the
We have, tan θ =
rg above formula.
304 OBJECTIVE Physics Vol. 1

If the vehicle is moving downward on inclined road, From the above figure, we get
then minimum velocity for no skidding is Resultant of normal reaction force (N ) and frictional
1/ 2 force (f ), F = N2 + f 2
 (tan θ − µ s ) 
v min = rg  This resultant of N and f , i.e.F should pass through G, the
 1 + µ s tan θ  centre of gravity of cyclist (for complete equilibrium,
rotational as well as translational). Hence,
Note
(i) For no slipping or skidding, we have v min < v ≤ v max f
This speed is greater than the maximum possible speed of a car on tan θ =
level road (v = µgr ). N
(ii) If µ s = 0, v o = ( gr tan θ)1/ 2 mv 2 v2
This speed is known as optimum speed. where, f = and N = mg ⇒ tan θ =
r rg
Example 7.26 A circular race track of radius 300 m is
banked at an angle of 15°. If the coefficient of friction Note The angle through which cyclist should bend will be greater, if
(i) the radius of the curve is small.
between the wheels of a race car and the road is 0.2, then
(ii) the velocity of the cyclist is large.
what will be the maximum permissible speed to avoid
slipping? (Take, tan 15° = 0.26) Example 7.27 A cyclist speeding at 6 ms −1 in a circle of
Sol. We know that, maximum permissible speed on a banked radius 18 m makes an angle θ with the vertical. Determine
road to avoid slipping is the value of θ. Also, determine the minimum possible value
1/ 2 of coefficient of friction between the tyres and the ground.
rg (µ s + tan θ )
v max =   Sol. Given, v = 6 ms −1, r = 18 m, g = 9.8 ms −2
 (1 − µ s tan θ ) 
v2 6× 6
Now, putting the values given in the question, Since, tan θ = ⇒ tan θ = = 0.2041
rg 18 × 9.8
r = 300 m, θ = 15°
g = 9.8 (≈ 10) ms−2 and µ s = 0.2 ⇒ θ = tan−1 (0.2041) ⇒ θ = 11° 53′
1/2 Also, minimum possible value of coefficient of friction,
300 × 9.8 (0.2 + tan 15° )
We obtain, v max =   v2
 1 − 0.2 tan 15° µ = tan θ = = 0.2041 ⇒ µ = 0.2041
1/ 2
rg
300 × 9.8 (0.2 + 0.26)
= 
 1 − 0.2 × 0.26 3. Conical pendulum
After solving this, we get If a small particle of mass m tied to a string is whirled in a
v max = 37.8 ms−1 horizontal circle as shown in figure. The arrangement is
called the conical pendulum.
2. Bending of a cyclist In case of conical pendulum, the vertical component of
To take a safe turn on a circular turning, cyclist bend tension balances the weight while its horizontal
himself inward. When the cyclist is inclined to the centre component provides the necessary centripetal force.
of the rounding off its path, the resultant of N, f and mg is mv 2
directed horizontally to the centre of the circular path of Thus, T sin θ = …(i)
r
the cycle. This resultant force imparts a centripetal
acceleration to the cyclist. θ
F L

N G O N T T cos θ
G mv 2 θ
r
θ r r m
C
mg T sin θ
θ
f r = L sin θ mg
mg Fig. 7.14 Conical pendulum
Fig. 7.13 Bending of a cyclist from vertical
and T cos θ = mg …(ii)
Circular Motion 305

From these two equations, we can find v = rg tan θ Example 7.29 A ball of mass (m ) 0.5 kg is attached to the end
of a string having length (L) 0.5 m. The ball is rotated on a
v g tan θ horizontal circular path about vertical axis. The maximum
∴ Angular speed, ω = = tension that the string can bear is 324 N. Find the maximum
r r
possible value of angular velocity of ball (in rads −1).
So, the time period of pendulum is
2π r L cos θ
T = = 2π = 2π
ω g tan θ g
L
[since, r = L sinθ]
L cos θ m
or T = 2π
g Sol. Consider the forces acting on the ball as shown in the figure.

Example 7.28 A particle of mass 200 g tied to one end of


string is revolved in a horizontal circle of radius 50 cm about θ
l
a vertical axis passing through the point of suspension, with
angular speed 60 rev per minute (rpm). Find (i) linear speed, θ T
(ii) the acceleration and (iii) horizontal component of tension
C
in the string. What will happen, if string is broken?
mg
(Take, π 2 = 10) r
2πn 2π × 60 r = l sin θ
Sol. Angular speed, ω = =
60 60 The component T cos θ will cancel mg.
= 2π rad s −1 = 6.28 rad s −1 The component T sin θ will provide necessary centripetal
Horizontal component of the tension, TH = T sin θ force to the ball towards centre C.
Vertical component of tension, TV = T cos θ = mg ∴ T sin θ = mrω 2 = m (l sin θ ) ω 2 or T = mlω 2
T
Angular velocity, ω =
θ ml
Tmax 324
= 36 rads−1
T T
or ω max = =
TV
ml 0.5 × 0.5
θ
TH
Example 7.30 A boy whirls a stone of mass 2 kg in a
C r horizontal circle of radius 1.5 m, which is attached to a
string having length 1 m. What is the time period of the
r = L sin θ mg given system of stone and string? (Take, cos 15° = 0.96)
(i) Linear speed, v = rω = 0.5 × 2π = π ms −1 = 3.14 ms −1
m= 2 kg
v 2 (π )2 10 O
(ii) Acceleration, a c = = ≈ = 20 ms−2
r 0.5 0.5
L
(iii) Horizontal component of tension,
15°
mv 2
TH = = 0.2 × 20 = 4 N
r
When the string is broken, tension TH (i.e. centripetal force)
vanishes and body moves along the tangent in a straight line Sol. Given, m = 2 kg, r = 1.5 m,
with speed 3.14 ms −1. L = 1 m, θ = 15° , g = 10 ms −2
v Time period of the given system of stone and string is given
as
v
L cos θ 1 × cos15° 0.96
T = 2π = 2π = 2π
g 10 10
O = 2 × 314
. × 0.31 = 1.95 −~ 2 s
When the string
is broken Thus, the time period is 2 s (approx).
306 OBJECTIVE Physics Vol. 1

4. ‘Death well’ or rotor µ s mv 2  mv 2 


or = mg QN = 
r  r 
In case of death well, a person drives a bicycle on a
vertical surface of a large wooden well, while in case of a rg 2 × 10
or v= = = 10 ms−1
rotor, at its certain angular speed, a person hangs resting µs 0.2
against the wall without any support from the bottom. In
death well, walls are at rest and person revolves while in
case of rotor, person is at rest and the walls rotate. Motion of a particle tied to a
r
f
string in vertical circle
N Suppose a particle of mass m is attached to an inextensible
f
r
light string of length R. This particle is moving in a
N mg
mg vertical circle of radius R about a fixed point O.
It is imparted a velocity u in horizontal direction at lowest
point A. Let v be its velocity at point B of the circle as
(a) (b) shown in Fig. 7.16.
Death well Rotor
Fig. 7.15
In both cases, friction balances the weight of person while
O
reaction provides the centripetal force for circular motion, T v
θ
mv 2 R B
θ mg cos θ
i.e. f = mg and N = = mrω 2 (Q v = rω ) h
r mg s mg
A u in θ
gr
f ≤ µN ⇒ mg ≤ µ mv /r ⇒ v ≥ 2 2
Fig. 7.16 Motion of a particle in vertical circle
µ
gr Here, we have
∴ Safe speed, v = h = R (1 − cos θ)
µ
Now, from conservation of mechanical energy, we have
Example 7.31 In a rotor, a hollow vertical cylinder rotates 1
about its axis and a person rest against the inner wall. At a m (u 2 − v 2 ) = mgh …(i)
particular speed of the rotor, the floor below the person is 2
removed and the person hangs resting against the wall The necessary centripetal force is provided by the resultant
without any floor. If the radius of the rotor is 2 m and the of tension T and mg cos θ.
coefficient of static friction between the wall and the person
mv 2
is 0.2. Find the minimum speed at which the floor may be ∴ T − mg cos θ = …(ii)
removed. R
Sol. The situation is shown in figure below. As speed of the particle decreases with height, tension in the
string is maximum at the bottom. The particle will complete the
circle, if the string does not slack even at the highest point.
fs Now, following conclusions can be made using above
N Eqs. (i) and (ii)
mg (i) Minimum velocity at highest point, so that
particle complete the circle v min = gR , at this
velocity, tension in the string is zero.
When the floor is removed, the forces on the person are
(i) weight mg downward. (ii) Minimum velocity at lowest point, so that particle
(ii) normal force N due to the wall towards the centre. complete the circle v min = 5gR , at this velocity,
(iii) frictional force fs parallel to the wall, upwards.
tension in the string is 6 mg.
The person is moving in a circle with a uniform speed, so its
acceleration is v 2 /r towards the centre. (iii) When string is horizontal, then minimum velocity
is 3Rg and tension in this condition is 3 mg.
For the minimum speed when the floor may be removed, the
friction is limiting one and so equals µ s N. (iv) If velocity at lowest point is less than 5 gR , then
This gives,
tension in the string becomes zero before reaching
µ s N = mg
Circular Motion 307

the highest point, now the particle will leave the Example 7.34 A simple pendulum is constructed by attaching
circle and will move on parabolic path. a bob of mass m to a string of length L fixed at its upper end.
The bob oscillates in a vertical circle. It is found that the
In this condition, if 2gR < v < 5 gR , then tension speed of the bob is v when the string makes an angle α with
in the string becomes zero but velocity is not zero, the the vertical. Find the tension in the string and the magnitude
particle will leave circle at 90 ° < θ < 180 ° or h > R . of net force on the bob at that instant.
T=0 Sol. (i) The force acting on the bob are
v≠0 (a) the tensionT (b) the weight mg
P O

O
θ α
h>R
R
T
u
A
Fig. 7.17
α
(v) If velocity at lowest point is 0 < v ≤ 2 gR, the mg sin α mg cos α
particle will oscillate. In this condition, velocity mg
becomes zero but tension is not zero. The particle As the bob moves in a circle of radius L with centre at
will oscillate in lower half of circle, i.e. 0 ° < θ < 90 °. mv 2
O. A centripetal force of magnitude is required
L
towards O. This force will be provided by the resultant
of T and mg cos α. Thus,
T≠0 mv 2
v=0 T − mg cos α =
θ L
h≤R  v2
u or T = m  g cos α + 
Fig. 7.18  L
2
Note The above points have been derived for a particle moving in a  mv 2 
vertical circle attached to a string. The same conditions apply, if (ii) | Fnet| = (mg sin α ) + 
2

a particle moves inside a smooth spherical shell of radius R. The  L 
only difference is that the tension is replaced by the normal
v4
reaction N. =m g 2 sin2 α +
L2
Example 7.32 One end of a string of length 1 m is tied to a
body of mass 0.5 kg. It is whirled in a vertical circle with Example 7.35 A heavy particle hanging from a fixed point by
angular velocity 4 rad s −1. Find the tension in the string a light inextensible string of length l is projected horizontally
when the body is at the lower most point of its motion. with speed gl . Find the speed of the particle and the
(Take, g = 10 ms −2 ) inclination of the string to the vertical at the instant of the
Sol. At lower most point, the tension in the string, motion when the tension in the string is equal to the weight
T = m ω 2r + mg = 0.5 × (4)2 × 1 + 0.5 × 10 of the particle.
Sol. Let T = mg at angle θ as shown in figure.
= 13 N
h = l (1 − cos θ ) …(i)
Example 7.33 A ball of mass 0.6 kg attached to a light
inextensible string rotates in a vertical circle of radius 0.75 m Applying conservation of mechanical energy between points
such that it has speed of 5 ms − 1 when the string is horizontal. A and B, we get
Tension in string when it is horizontal on other side is 1
(Take, g = 10ms −2 ) m (u 2 − v 2 ) = mgh
2
Sol. Tension in the string when it makes angle θ with the vertical,
mv 2
T = + mg cos θ
r
When the string is horizontal, θ = 90° T
θ B
mv 2 mv 2
∴ T = + mg × 0 = θ mg cos θ
r r h mg mg
sin
0.6 × (5)2 θ
= = 20 N A u = √gl
0.75
308 OBJECTIVE Physics Vol. 1

Here, u 2 = gl …(ii) In this situation, we can write


and v = speed of particle in position B v 2 = u 2 − 2gh = u 2 − 2gL (1 − cos 60° )
∴ v 2 = u 2 − 2 gh …(iii)  1
= 10gL − 2gL 1 −  = 9 gL
 2
mv 2
Further, T − mg cos θ = mv 2 m
l Also, T − mg cos 60° = = × 9 gL = 9 mg
mv 2 L L
or mg − mg cos θ = (QT = mg ) mg
l ⇒ T − = 9 mg
2
or v 2 = gl (1 − cos θ ) …(iv) 19
2 2
⇒ T = mg
Substituting the values of v , u and h from Eqs. (iv), (ii) and 2
(i) in Eq. (iii), we get (b)
°
gl (1 − cos θ ) = gl − 2gl (1 − cos θ ) 60
g cos v
2  2 m
or cos θ = or θ = cos−1  60° 60
°
L cos 60°
3  3 T
O mg
2 h
Substituting cos θ = in Eq. (iv), we get L
3 L
gl u
v= m
3
In this situation, we have
Example 7.36 A particle of mass m is attached to a string of
v 2 = u 2 − 2 gh = 10 gL − 2 gL (1 + cos 60° ) = 7 gL
length L and given velocity 10gL in the horizontal direction
at the lowest point. Find tension in the string when the mv 2
Also, T + mg cos 60° =
particle is at (i) (a) lowest position (b) highest position; (ii) when L
the string makes an angle 60° with (a) lower vertical and (b) mg m
upper vertical. ⇒ T + = × 7 gL = 7mg
2 L
Sol. (i) v 13 mg
⇒ T =
T2 2
mg
Example 7.37 A hemispherical bowl of radius R is rotating
O about its axis of symmetry which is kept vertical. A small
L ball kept in the bowl rotates with the bowl without slipping
T1 on its surface. If the surface of the bowl is smooth and the
u angle made by the radius through the ball with the vertical is
mg
α. Find the angular speed at which the bowl is rotating.
(a) At the lowest position, Sol. Let ω be the angular speed of rotation of the
mu 2 m bowl.Two forces are acting on the ball
T1 − mg = = × 10gL ⇒ T1 = 11 mg
L L (i) normal reaction N
(b) At the highest position, (ii) weight mg ω
v 2 = u 2 − 2gh = u 2 − 2g × 2 L
R
= 10gL − 4gL = 6gL α
N
mv 2 m
Also, T2 + mg = = × 6 gL = 6 mg r A
L L
⇒ T2 = 5 mg mg
The ball is rotating in a circle of radius r (= R sin α ) with
(ii) (a) O centre at A at an angular speed ω. Thus,
L
L cos 60°
60° T N sin α = mrω 2 = mRω 2 sin α
v ⇒ N = mRω 2 …(i)
and N cos α = mg …(ii)
60°
L (1 – cos 60°) mg cos 60° On dividing Eq. (i) by Eq. (ii), we get
mg 1 ω 2R g
u = ⇒ ω=
cos α g R cos α
CHECK POINT 7.2
1. A particle of mass 2 kg is moving along a circular path of 9. Radius of the curved road on national highway is R. Width
radius 1 m. If its angular speed is 2π rad s −1, the centripetal of the road is b. The outer edge of the road is raised by h
force on it is with respect to inner edge, so that a car with velocity v can
(a) 4π N (b) 8π N pass safely over it. The value of h is
(c) 4 π 4 N (d) 8 π 2 N v2b v
(a) (b)
Rg Rgb
2. Two particles of equal masses are revolving in circular paths v2R v2b
of radii r1 and r2 respectively with the same speed. The ratio (c) (d)
of their centripetal forces is bg R

(a)
r2
(b)
r2 10. A motor cyclist moving with a velocity of 72 km h −1 on a
r1 r1 flat road takes a turn on the road at a point, where the
2 2 radius of curvature of the road is 20 m. The acceleration due
r  r 
(c)  1  (d)  2  to gravity is 10 ms −2. In order to avoid skidding, he must not
 r2   r1  bend with respect to the vertical plane by an angle greater
than
3. A particle of mass m is executing uniform circular motion on
a path of radius r. If p is the magnitude of its linear (a) θ = tan−1 (6) (b) θ = tan−1 (2)
−1
momentum. The radial force acting on the particle is (c) θ = tan (2592
. ) (d) θ = tan−1 (4)
rm
(a) pmr (b) 11. A car of mass 1000 kg negotiates a banked curve of radius
p 90 m on a frictionless road. If the banking angle is 45°, the
mp 2 p2 speed of the car is
(c) (d)
r rm (a) 20 ms −1 (b) 30 ms −1
4. A stone of mass of 16 kg is attached to a string 144 m long and (c) 5 ms −1 (d) 10 ms −1
is whirled in a horizontal circle on a smooth surface. The 12. Keeping the angle of banking unchanged, if the radius of
maximum tension in the string that it can withstand is 16 N. curvature is made four times, the percentage increase in the
The maximum velocity of revolution that can be given to the maximum speed with which a vehicle can travel on a
stone without breaking it, will be circular road is
(a) 20 ms−1 (b) 16 ms−1 (a) 25 % (b) 50%
(c) 14 ms−1 (d) 12 ms−1 (c) 75% (d) 100%

5. If mass, speed and radius of the circle, of a particle moving 13. A person wants to drive on the vertical surface of a large
uniformly in a circular path are all increased by 50%, the cylindrical wooden ‘well’ commonly known as ‘death well’
necessary force required to maintain the body moving in in a circus. The radius of the well is R and the coefficient of
the circular path will have to be increased by friction between the tyres of the motorcycle and the wall of
the well is µ s . The minimum speed, the motorcycle must
(a) 225% (b) 125% (c) 150% (d) 100%
have in order to prevent slipping, should be
6. A string of length 0.1 m cannot bear a tension more than Rg µs
100 N. It is tied to a body of mass 100 g and rotated in a (a) (b)
µs Rg
horizontal circle. The maximum angular velocity can be
µs g R
(a) 100 rad s −1 (b) 1000 rad s −1 (c) (d)
(c) 10000 s −1 (d) 0.1 rad s −1 R µs g

7. A mass of 2 kg is whirled in a horizontal circle by means of 14. A motorcyclist wants to drive on the vertical surface of
a string at an initial speed of 5 rev min −1 . Keeping the wooden ‘well’ of radius 5 m, with a minimum speed of
radius constant the tension in the string is doubled. The 5 5 ms −1. The minimum value of coefficient of friction
new speed is nearly between the tyres and the wall of the well must be
(a)
5
rpm (b) 10 rpm (Take, g = 10 ms −2)
2 (a) 0.10 (b) 0.20
(c) 10 2 rpm (d) 5 2 rpm (c) 0.30 (d) 0.40

8. A mass of 100 g is tied to one end of a string 2 m long. The 15. A block of mass m at the end of a string is whirled round in
body is revolving in a horizontal circle making a maximum a vertical circle of radius R. The critical speed of the block at
of 200 rev min −1 . The other end of the string is fixed at the top of its swing below which the string would slacken
centre of the circle of revolution. The maximum tension before the block reaches the bottom is
that the string can bear is (approximately) (a) 5 Rg (b) 3 Rg
(a) 8.76 N (b) 8.94 N (c) 2 Rg (d) Rg
(c) 89.42 N (d) 87.64 N
310 OBJECTIVE Physics Vol. 1

16. A stone is attached to one end of a string and rotated in a 19. A stone of mass 1 kg is tied to the end of a string 1 m long.
vertical circle. If string breaks at the position of maximum It is whirled in a vertical circle. The velocity of the stone at
tension, it will break at the bottom of the circle is just sufficient to take it to the top
C of circle without slackening of the string. What is the tension
in the string at the top of the circle? (Take, g = 10 ms −2 )
(a) Zero (b) 1 N
D B (c) 10 N (d) 10 N
20. A small sphere of mass m is suspended by a thread of
length l. It is raised upto the height of suspension with
A thread fully stretched and released. Then, the maximum
tension in thread will be
(a) A (b) B
(a) mg (b) 2 mg
(c) C (d) D
(c) 3 mg (d) 6 mg
17. A particle of mass m attached at the end of a string is being
circulated on a vertical circle of radius r. If the speed of 21. A child is swinging a swing. Minimum and the maximum
particle at the highest point be v, such that the string would heights of swing from earth’s surface are 0.75 m and 2 m,
not slacken before the string reached the bottom, then respectively. The maximum velocity of this swing is
mv2 mv2
(Take, g = 10 ms −2 )
(a) mg = (b) mg > (a) 5 ms−1 (b) 10 ms−1
r r
(c) 15 ms−1 (d) 20 ms−1
mv2 mv2
(c) mg ≤ (d) mg ≥
r r 22. A national roadway bridge over a canal is in the form of an
arc of a circle of radius 49 m. What is the minimum speed
18. A particle is moving in a vertical circle. The tensions in the with which a car can move without leaving the ground at
string when passing through two positions at angles 30° the highest point? (Take, g = 9.8 ms −2)
and 60° from vertical (lowest position) are T1 and T 2
respectively, then (a) 19.6 ms−1
(a) T1 = T2 (b) T2 > T1 (b) 40 ms−1
(c) T1 > T2 (d) Data insufficient (c) 22 ms−1
(d) None of the above
Chapter Exercises
(A) Taking it together
Assorted questions of the chapter for advanced level practice
1 A particle moving along a circular path due to a 8 A car when passes through a convex bridge with
centripetal force having constant magnitude is an velocity v exerts a force on it at the topmost point is
example of motion with equal to
(a) constant speed and velocity Mv 2 Mv 2
(b) variable speed and variable velocity (a) Mg + (b)
r r
(c) variable speed and constant velocity
Mv 2
(d) constant speed and variable velocity (c) Mg − (d) None of these
r
2 A particle moving on a circular path makes 600 rpm.
In how much time, it will complete one revolution? 9 A stone tied to one end of rope and rotated in a
(a) 0.2 s (b) 0.1 s circular motion. If the string suddenly breaks, then
(c) 0.4 s (d) 0.3 s the stone travels
(a) in perpendicular direction
3 A wheel rotates at 50 rpm about its axis. The (b) in direction of centrifugal force
angular retardation that can stop the wheel in one (c) towards centripetal force
minute is (d) in tangential direction
π π
(a) rad s−2 (b) rad s−2 10. A car is moving on a circular path and takes a turn.
36 18
π π If R 1 and R 2 be the reactions on the inner and outer
(c) rad s−2 (d) rad s−2 wheels respectively, then
72 9
(a) R1 = R 2 (b) R1 < R 2
4 The speed of a particle moving in a circle is (c) R1 > R 2 (d) R1 ≥ R 2
increasing. The dot product of its acceleration and
11 An unbanked curve has a radius of 60 m. The
velocity is
maximum speed at which a car can make a turn, if
(a) negative
(b) zero
the coefficient of static friction is 0.75, is
(c) positive (a) 2.1 ms−1 (b) 14 ms−1 (c) 21 ms−1 (d) 7 ms−1
(d) may be positive or negative 12 A motorcyclist riding at 36 kmh −1 has to turn a
5. A particle moves with constant angular velocity in a corner. Find the least radius of the curve, he should
circle. During the motion, its follow for safe travelling, if the coefficient of friction
(a) energy is conserved between the tyres and the road is 0.2.
(b) momentum is conserved (a) 10 m (b) 25 m (c) 50 m (d) 100 m
(c) energy and momentum both are conserved 13 A circular curve of a highway is designed for traffic
(d) None of the above
moving at 72 kmh −1. If the radius of the curved path
6 An object is moving in a circle of radius 100 m with is 100 m, the correct angle of banking of the road
a constant speed of 31.4 ms −1. What is its average should be
speed for one complete revolution?  2  3
(a) Zero (b) 31.4 ms−1 (a) tan−1   (b) tan−1  
 5  5
(c) 3.14 ms−1 (d) 2 × 31.4 ms−1
 1  1
(c) tan−1   (d) tan−1  
7 A bucket full of water is rotated in a vertical circle  5  4
of radius R. If the water does not split out, the speed
14 A particle of mass m is circulating on a circle of
of the bucket at topmost point will be
radius r having angular momentum L about centre.
(a) Rg (b) 5gR
Then, the centripetal force will be
R  L2 L2 L2 L
(c) 2Rg (d)   (a) (b) (c) (d)
 g mr mr 2 mr 3 mr 2
312 OBJECTIVE Physics Vol. 1

15 A particle is moving along a circular path of radius 23 A pendulum bob on a 2m string is displaced 60°
5 m with a uniform speed 5 ms −1 . What will be the from the vertical and then released. What is the
average acceleration when the particle completes speed of the bob as it passes through the lowest
half revolution? point in its path?
(a) Zero (b) 10 ms−2 (a) 2 ms −1 (b) 9.8 ms −1
−2 10 −2 1
(c) 10π ms (d) ms (c) 4.43 ms −1 (d) ms −1
π 2
16 A fan makes 2400 rpm. If after it is switched off, it 24 A pendulum bob having length of string 0.2 m
comes to rest in 10 s, then find the number of times describes an arc of circle in a vertical plane. If the
it will rotate before it comes to rest after it is tension in the cord is 3 times the weight of the bob
switched off.
when the cord makes an angle 30° with the vertical,
(a) 400 (b) 100 (c) 200 (d) 50
the acceleration of the bob in that position is
17 The distance between the rails of the track is g
(a) g (b)
1.67 m. How much the outer rail be elevated for 2
curve of 0.5 km radius, so that a train moving with 3g g
speed 54 kmh −1 can take safe turn on track. (c)
2
(d)
4
(a) 80 mm (b) 75 mm (c) 60 mm (d) 75 cm
25 A jeep runs around a curve of radius 0.3 km at a
18 A body of mass 1 kg is moving in a vertical circular constant speed of 60 ms −1. The jeep covers a curve of
path of radius 1 m. The difference between the 60° arc, then
kinetic energies at its highest and lowest positions is (a) resultant change in velocity of jeep is 60 ms−1
(a) 20 J (b) 10 J
(b) instantaneous acceleration of jeep is12 ms−2
(c) 4 5 J (d) 10( 5 − 1) J (c) average acceleration of jeep is approximately 11.5 ms−2
19 If the banking angle of curved road is given by (d) All are correct
3 26 A coin placed on a rotating turn-table slips, when it
tan −1   and the radius of curvature of the road is
 5 is placed at a distance of 9 cm from the centre. If the
6 m, then the safe driving speed should not exceed angular velocity of the turn-table is trippled, it will
just slip, if its distance from the centre is
(Take, g = 10 ms −2 )
(a) 27 cm (b) 9 cm (c) 3 cm (d) 1 cm
(a) 86.4 km h −1 (b) 43.2 km h −1 27 A circular disc of radius R is rotating about its axis O
(c) 21.6 km h −1 (d) 30.4 km h −1 with a uniform angular velocity ω rad s −1 as shown
−1
20 A motorcyclist moving with a velocity of 144 kmh in the figure. The magnitude of the relative velocity
on a flat road takes a turn on the road at a point, of point A relative to point B on the disc is
where the radius of curvature of the road is 40 m.
B
The acceleration due to gravity is 10 ms −2 . In order ω
to avoid sliding, he must bend with respect to the
vertical plane by an angle
O θ
(a) θ = tan−1 (4) (b) θ = 45°
(c) θ = tan−1 (2) (d) θ = tan−1 (6)
21 A train has to negotiate a curve of radius 800 m. By A
how much height should the outer rail be raised  θ
(a) zero (b) Rω sin  
with respect to inner rail for a speed of 96 kmh −1?  2
The distance between the rails is 1 m.  θ  θ
(c) 2Rω sin   (d) 3 Rω sin  
(a) 4.4 cm (b) 9 cm (c) 8.9 cm (d) 3.3 cm  2  2
22 A car wheel is rotated to uniform angular acceleration 28 When the angular velocity of a uniformly rotating
about its axis. Initially, its angular velocity is zero. It body has increased thrice, the resultant of forces
rotates through an angle θ 1 in the first 2 s. In the next applied to it increases by 60 N. Find the
2 s, it rotates through an additional angle θ 2 , the ratio accelerations of the body in the two cases. The mass
θ of the body m = 3 kg.
of 2 is
θ1 (a) 2.5 ms−2, 7.5 ms−2 (b) 7.5 ms−2, 22.5 ms−2
(a) 1 (b) 2 (c) 3 (d) 4 (c) 5 ms−2, 45 ms−2 (d) 2.5 ms−2, 22.5 ms−2
Circular Motion 313

29 An automobile enters a turn of radius R. If the road 37 Three particles A, B and C move in a circle of radius
is banked at an angle of 45° and the coefficient of 1
friction is 1, the minimum speed with which the r = m, in anti-clockwise direction with speed
π
automobile can negotiate the turn without skidding is
1ms −1 , 2.5 ms −1 and 2 ms −1, respectively. The
rg rg
(a) (b) (c) rg (d) zero initial positions of A, B and C are as shown in figure.
2 2
30 A mass is attached to the end of a string of length l B
which is tied to a fixed point O. The mass is released
from the initial horizontal position of the string.
Below the point O at what minimum distance, a peg A C
P should be fixed, so that the mass turns about P and O
can describe a complete circle in the vertical plane?
 3  2 l 2l
(a)   l (b)   l (c) (d)
 5  5 3 3
The ratio of distance travelled by B and C by the
31 A stone is rotated in a vertical circle. Speed at instant A, B and C meet for the first time is
bottommost point is 8gR , where R is the radius of (a) 3 : 2 (b) 5 : 4
circle. The ratio of tension at the top and the bottom
(c) 3 : 5 (d) 3 : 7
is
(a) 1 : 2 (b) 1 : 3 (c) 2 : 3 (d) 1 : 4 38 A stone tied to one end of spring 80 cm long is
whirled in a horizontal circle with a constant speed.
32 A body is moving in a vertical circle of radius r such If stone makes 14 revolutions in 25 s, the magnitude
that the string is just taut at its highest point. The of acceleration of stone is
speed of the particle when the string is horizontal, is (a) 850 cm s−2 (b) 992 cm s−2
(a) gr (b) 2gR (c) 3gr (d) 4gR (c) 720 cm s−2 (d) 650 cm s−2
33 A small ball is pushed from a height h along a 39 A student whirles a stone in a horizontal circle of
smooth hemispherical bowl of radius R. With what radius 3 m and at height 8 m above level ground.
speed should the ball be pushed, so that it just The string breaks, at lowest point and the stone flies
reaches the top of the opposite end of the bowl? off horizontally and strikes the ground after
(a) 2gh (b) 2g (R + h ) travelling a horizontal distance of 20 m. What is the
magnitude of the centripetal acceleration of the
(c) 2g (R − h ) (d) None of these
stone while breaking off?
34 A 50 kg girl is swinging on a swing from rest. Then, (a) 150 ms−2 (b) 140 ms−2
the power delivered when moving with a velocity of (c) 81.4 ms−2 (d) 163 ms−2
2 ms −1 upwards in a direction making an angle 60° 40 A stone is tied to a string of length l and is whirled
with the vertical is
in a vertical circle with the other end of the string as
(a) 980 W (b) 490 W (c) 490 3 W (d) 245 W the centre. At a certain instant of time, the stone is
35 A simple pendulum of length l has a maximum at its lowest position and has a speed u. The
angular displacement θ. The maximum kinetic magnitude of the change in velocity as it reaches a
energy of the bob of mass m will be position, where the string is horizontal (g being
(a) mgl (1 − cos θ ) (b) mgl cos θ acceleration due to gravity) is
(c) mgl sin θ (d) None of these
(a) 2(u 2 − gl ) (b) u 2 − gl
36 Toy cart tied to the end of an unstretched string of
length a, when revolved moves in a horizontal circle (c) u − u 2 − 2gl (d) 2gl
of radius 2a with a time period T. Now, the toy cart
is speeded up until it moves in a horizontal circle of 41 A ball suspended by a thread swings in a vertical
radius 3a with a period T ′. If Hooke’s law (F = kx ) plane, so that its acceleration at the extreme position
holds, then and lowest position are equal. The angle θ of thread
3  3 deflection in the extreme position will be
(a) T ′ = T (b) T ′ =   T (a) tan−1 (2) (b) tan−1 ( 2 )
2  2 
 1  1
 3 (c) tan−1   (d) 2 tan−1  
(c) T ′ =   T (d) T ′ = T  2  2
 2
314 OBJECTIVE Physics Vol. 1

42 A body of mass m hangs at one end of a string of 48 A heavy particle is tied to the end A of a string of
length l, the other end of which is fixed. It is given a length 1.6 m. Its other end O is fixed. It revolves as
horizontal velocity, so that the string would just a conical pendulum with the string making 60° with
reach, where it makes an angle of 60° with the the horizontal. Then,
vertical. The tension in the string at bottommost 4π
(a) its period of revolution is s
point position is 7
(a) 2 mg (b) mg (b) the tension in the string is1/ 3 times the weight of the
(c) 3 mg (d) 3 mg particle
(c) the speed of the particle is 2.8 3 ms −1
43 A simple pendulum oscillates in a vertical plane.
9.8
When it passes through the bottommost point, the (d) the centripetal acceleration of the particle is ms−2
tension in the string is 3 times the weight of the 3
pendulum bob. What is the maximum displacement 49 A pendulum bob has a speed of 3 ms −1 at its lowest
of the pendulum of the string with respect to the position. The pendulum is 0.5 m long. The speed of
vertical? the bob, when string makes an angle of 60° to the
(a) 30° (b) 45° (c) 60° (d) 90° vertical is (Take, g = 10 ms −2 )
44 A stone of mass 1 kg tied to a light inextensible 1
(a) 2 ms −1 (b) ms −1
10 2
string of length L = m, whirling in a circular path
3 (c) 1 ms −1 (d) 2.5 ms −1
in a vertical plane. The ratio of maximum tension to 50 A block is released from rest at the top of an inclined
the minimum tension in the string is 4. If g is taken plane which later curves into a circular track of radius r
to be 10 ms −2 , the speed of the stone at the highest as shown in figure. The minimum height h from where
point of the circle is it should be released, so that it is able to complete the
(a) 10 ms −1 (b) 5 2 ms −1 (c) 10 3 ms −1 (d) 20 ms −1 circle, is
45 A string of length l fixed at one end carries a mass m A
2
at the other end. The strings makes rev s −1 around
π B
the axis through the fixed end as shown in the
figure, the tension in the string is h
2r

θ
l
T (a) r (b) 2.5 r
(c) 1.5 r (d) 0.5 r
m 51 A small body of mass m slides without friction from
r
the top of a hemisphere of radius r. At what height
(a) 16 ml (b) 4 ml (c) 8 ml (d) 2 ml
will the body be detached from the centre of the
hemisphere?
46 A particle starts travelling on a circle with constant
tangential acceleration. The angle between velocity
vector and acceleration vector, at the moment when h
particle complete half the circular track, is
(a) tan−1 (2π ) (b) tan−1 (π ) (c) tan−1 (3π ) (d) zero
r r 2r r
47 A wet open umbrella is held vertical and it whirled (a) h = (b) h = (c) h = (d) h =
2 3 3 4
about the handle at a uniform rate of 21 rev in 44 s.
If the rim of the umbrella is a circle of diameter 1 m 52 The maximum tension that an inextensible ring of
and the height of the rim above the floor is 4.9 m, radius 1m and mass density 0.1 kg m−1 can bear is
then the locus of the drop on floor is a circle of 40 N. The maximum angular velocity with which it
radius can be rotated in a circular path is
(a) 2.5 m (b) 1 m (a) 20 rad s−1 (b) 18 rad s−1
(c) 3 m (d) 1.5 m (c) 16 rad s−1 (d) 15 rad s−1
Circular Motion 315

53 Two bodies of masses m and 4m are attached to a 57 A ball is placed on a smooth inclined plane of
string as shown in the figure. The body of mass m inclination θ = 30 ° to the horizontal, which is
hanging from a string of length l is executing rotating at frequency 0.5 Hz about a vertical axis
periodic motion with amplitude θ = 60 ° while other passing through its lower end. At what distance from
body is at rest on the surface. the lower end does the ball remain at rest?
4m (a) 0.87 m (b) 0.33 m
(c) 0.5 m (d) 0.67 m
58 A particle suspended by a light inextensible thread
θ
of length l is projected horizontally from its lowest
position with velocity 7gl / 2. The string will slack
m after swinging through an angle equal to
(a) 30° (b) 90°
The minimum coefficient of friction between the
(c) 120° (d) 150°
mass 4m and the horizontal surface must be
1 1 1 2 59 The kinetic energy K of a particle moving along a
(a) (b) (c) (d)
4 3 2 3 circle of radius R depends on the distance covered s
as K = as 2 . The force acting on the particle is
54 A bullet of mass m moving with a horizontal velocity
1/ 2
u strikes a stationary wooden block of mass M 2as 2  s2 
(a) (b) 2as 1 + 2 
suspended by a string of length L = 50 cm. The R  R 
u 1/ 2
bullet emerges out of the block with speed . If  s2 
4 (c) as 1 + 2  (d) None of these
M = 6 m, the minimum value of u, so that the block  R 
can complete the vertical circle, is
60 A simple pendulum is vibrating with an angular
(Take, g = 10 ms −2 ) amplitude of 90° as shown in the figure. For what
(a) 10 ms −1 (b) 20 ms −1 (c) 30 ms −1 (d) 40 ms −1 value of α, is the acceleration directed?
55 Three identical particles are joined together by a
thread as shown in figure. All the three particles are α
moving in a circle in horizontal plane. If the velocity
of the outermost particle is v 0 , then the ratio of
tensions in the three sections of the string is
(TBC : T AB : TOA )
(i) Vertically upwards
O A B C (ii) Horizontally
l l l
(iii) Vertically downwards
 1   1 
(a) 0°, cos−1   , 90° (b) 90°, cos−1   , 0°
(a) 3 : 5 : 7 (b) 3 : 4 : 5 (c) 7 : 11 : 6 (d) 3 : 5 : 6  3  3
56 A particle moves from rest at A on the surface of a  1 
(c) 0°, cos−1 3 , 90° (d) cos−1   , 90° , 0°
smooth circular cylinder of radius r as shown in the  3
figure. At B, it leaves the cylinder. The equation
61. A small block of mass m is released from the top of a
relating α and β is
smooth hemisphere of radius R with the horizontal
A speed u. What is the angle with vertical, where it
B
loses contact with the hemisphere?
r α  u2 2  u2 2
β (a) sin−1  +  (b) cos−1  + 
 3gR 3  3gR 3
(a) 3 sin α = 2 cos β (b) 2 sin α = 3 cos β  u2 4  u2 2
(c) 3 sin β = 2 cos α (d) 2 sin β = 3 cos α (c) cos−1  +  (d) sin−1  + 
 6 gR 3   6 gR 3
OBJECTIVE Physics Vol. 1

(B) Medical entrance special format questions


Assertion and reason the shown path without loosing contact with road
anywhere else.
Directions (Q. Nos. 1–5) These questions consist of two Reason For car to loose contact with road, the
statements each printed as Assertion and Reason. While normal reaction between car and road should be
answering these questions you are required to choose any zero.
one of the following four responses
(a) If both Assertion and Reason are correct and Reason is the Car
correct explanation of Assertion.
(b) If both Assertion and Reason are correct but Reason is not
the correct explanation of Assertion.
(c) If Assertion is correct but Reason is incorrect.
(d) If Assertion is incorrect but Reason is correct. Statement based questions
1 Assertion A particle is rotating in a circle of radius 1 A particle moves in a uniform circular motion.
1m. At some given instant, its speed is 2 ms −1. Choose the incorrect statement.
Then, acceleration of particle at the given instant (a) The particle moves with constant speed.
is 4 ms −2 . (b) The acceleration is always normal to the velocity.
Reason Centripetal acceleration at this instant is (c) The particle moves with uniform acceleration.
4 ms −2 towards centre of circle. (d) The particle moves with variable velocity.

2 Assertion When a car takes a circular turn on a 2 Which of the following statement is incorrect?
(a) Uniform circular motion is uniformly accelerated
horizontal road, then normal reaction on inner motion.
wheels is always greater than the normal reaction on (b) Acceleration in uniform circular motion is always
outer wheels. towards centre.
Reason This is for rotational equilibrium of car. (c) In circular motion, dot product of v and ω is always
zero.
3 Assertion A ball tied by thread is undergoing (d) In any curvilinear path, average speed and average
circular motion (of radius R) in a vertical plane. velocity are never equal.
(Thread always remains in vertical plane). The
3 Which of the following statement(s) is/are correct?
difference of maximum and minimum tension in
(a) Centripetal force mv 2 / R acts on a particle rotating in a
thread is independent of speed (u ) of ball at the circle.
lowest position (u > 5gR ). (b) If a particle is rotating in a circle, then centrifugal
force is acting on the particle in radially outward
Reason For a ball of mass m tied by thread direction.
undergoing vertical circular motion (of radius R), (c) Centrifugal force is equal and opposite to the
difference in maximum and minimum magnitude of centripetal force.
centripetal acceleration of the ball is independent of (d) All of the above
speed (u ) of ball at the lowest position (u > 5gR ). 4 Which of the following statement(s) is/are correct?
I. When water in a bucket is whirled fast overhead,
4 Assertion One end of a massless rod of length l is
the water does not fall out at the top of the
hinged, so that it is free to rotate in a vertical plane circular path.
about a horizontal axis. If a particle is attached to the
other end of the rod, then the minimum speed at II. The centripetal force in this position on water is
more than the weight of water.
lower most position of the particle is 5gl to (a) Only I (b) Only II
complete the circular motion. (c) Both I and II (d) None of these
Reason Work done by centripetal force on the 5 A small block of mass m is rotating
particle is always zero. in a circle inside a smooth cone as
5 Assertion A car moves along a road with uniform shown in figure. Which of the
speed. The path of car lies in vertical plane and is following statement(s) is/are correct θ
shown in figure. The radius of curvature (R ) of the regarding this?
path is same everywhere. If the car does not loose I. In this case, the normal reaction, N ≠ mg cosθ.
contact with road at the highest point, it can travel
Circular Motion 317

II. In this case, acceleration of the block is not along 2 A particle is suspended from a string of length R. It
the surface of cone. It is horizontal. is given a velocity u = 3 gR at the bottom. Match
(a) Only I (b) Only II
(c) Both I and II (d) None of these
the following columns and mark the correct option
from the codes given below.
Match the columns C

1 Three balls each of mass 1 kg are attached with three


strings each of length 1 m as shown in figure. They D B

are rotated in a horizontal circle with angular velocity


ω = 4 rads −1 about point O. Match the following A
u

columns and mark the correct option from the codes


given below. Column I Column II
ω
O (A) Velocity at B (p) 7mg
T1 T2 T3
(B) Velocity at C (q) 5gR
Column I Column II
(A) T1 (p) Maximum (C) Tension in string at B (r) 7gR
(B) T2 (q) 80 N
(D) Tension in string at C (s) 5 mg
(C) T3 (r) 48 N
(t) None
(s) 90 N
Codes Codes
A B C A B C A B C D A B C D
(a) p q r (b) s q p (a) p q r s (b) r q p t
(c) p s q (d) s r p (c) p s q t (d) t q p s

(C) Medical entrances’ gallery


Collection of questions asked in NEET & various medical entrance exams
1 A point mass m is moved in a vertical circle of radius 4 Find the maximum radius of circle, so that the block
r with the help of a string. The velocity of the mass can complete the circular motion. [JIPMER 2019]
is 7 gr at the lowest point. The tension in the string
at the lowest point is [NEET 2020]
(a) 6 mg (b) 7 mg (c) 8 mg (d) 1 mg H = 5cm R

2 A block of mass 10 kg is in contact against the inner


wall of a hollow cylindrical drum of radius 1 m. The (a) 5 cm (b) 3 cm
coefficient of friction between the block and the (c) 2 cm (d) 4 cm
inner wall of the cylinder is 0.1. The minimum
5 A body initially at rest and sliding along a frictionless
angular velocity needed for the cylinder to keep the
track from a height h (as shown in the figure) just
block stationary when the cylinder is vertical and
completes a vertical circle of diameter AB = D . The
rotating about its axis, will be (Take, g = 10 m/s 2 )
height h is equal to [NEET 2018]
[NEET 2019]
10
(a) rad/s (b) 10 rad/s (c) 10π rad/s (d) 10 rad/s
2π h B
3 A mass m is attached to a thin wire and whirled in a
vertical circle. The wire is most likely to break when A
(a) the wire is horizontal [NEET 2019] 7
(b) the mass is at the lowest point (a) D (b) D
5
(c) inclined at an angle of 60° from vertical 3 5
(d) the mass is at the highest point (c) D (d) D
2 4
318 OBJECTIVE Physics Vol. 1

6 One end of the string of length l is connected to a 11 What is the minimum velocity with which a body of
particle of mass m and the other end is connected to mass m must enter a vertical loop of radius R, so that
a small peg on a smooth horizontal table. If the it can complete the loop? [NEET 2016]
particle moves in circle with speed v, the net force (a) 2gR (b) 3gR
on the particle (directed towards centre) will be (c) 5gR (d) gR
(T represents the tension in the string) [NEET 2017]
12 A particle of mass 10 g moves along a circle of
mv 2
(a) T (b) T + radius 6.4 cm with a constant tangential
l acceleration. What is the magnitude of this
mv 2 acceleration, if the kinetic energy of the particle
(c) T − (d) zero
l becomes equal to 8 × 10 −4 J by the end of the second
7 Assertion For looping a vertical loop of radius r, the revolution after the beginning of the motion?
minimum velocity at lowest point should be 5gr . [NEET 2016]
(a) 0.15 ms −2 . ms −2
(b) 018
Reason In this event, the velocity at the highest (c) 0.2 ms −2 (d) 0.1 ms −2
point will be zero. [AIIMS 2017]
13 A particle is moving in a curved path. Which of the
(a) Both Assertion and Reason are correct and Reason is
the correct explanation of Assertion.
following quantities may remain constant during its
(b) Both Assertion and Reason are correct but Reason is not motion? [CG PMT 2015]
the correct explanation of Assertion. (a) Acceleration (b) Velocity
(c) Assertion is correct but Reason is incorrect. (c) Magnitude of acceleration (d) None of these
(d) Both Assertion and Reason are incorrect.
14 The ratio of angular speed of a second hand to the
8 In the given figure, a = 15 m/s 2 represents the total hour-hand of a watch is [KCET 2015]
acceleration of a particle moving in the clockwise (a) 3600 :1 (b) 720 : 1
direction in a circle of radius R = 2.5 m at a given (c) 72 : 1 (d) 60 : 1
instant of time. The speed of the particle is
[NEET 2016] 15 If the length of second’s hand of a clock is 10 cm,
the speed of its dip (in cm s −1 ) is nearly
[Kerala CEE 2014]
R 30°
a (a) 2 (b) 0.5 (c) 1.5 (d) 1
O (e) 3
16 A particle is moving uniformly in a circular path of
radius r. When it moves through an angular
(a) 4.5 m/s (b) 5.0 m/s
displacement θ, then the magnitude of the
(c) 5.7 m/s (d) 6.2 m/s
corresponding linear displacement will be
9 A car is negotiating a curved road of radius R. The [WB JEE 2014]
road is banked at angle θ. The coefficient of friction  θ
(a) 2 r cos  
 θ
(b) 2 r cot  
between the tyres of the car and the road is µ s . The  2  2
maximum safe velocity on this road is [NEET 2016]  θ  θ
(c) 2 r tan   (d) 2 r sin  
 2  2
 µ + tan θ  g  µ s + tan θ 
(a) gR  s  (b)  
1 − µ s tan θ  R 1 − µ s tan θ  17 A rotating wheel changes angular speed from
1800 rpm to 3000 rpm in 20 s. What is the angular
g  µ s + tan θ  µ + tan θ 
2 acceleration assuming to be uniform?
(c)   (d) gR  s  [KCET 2014]
R 2 1 − µ s tan θ  1 − µ s tan θ  (a) 60π rad s−2 (b) 90π rad s− 2
−2
10 A uniform circular disc of radius 50 cm at rest is (c) 2π rad s (d) 40π rad s−2
free to turn about an axis which is perpendicular to
its plane and passes through its centre. It is subjected 18 A stone tied to a rope is rotated in a vertical circle
to a torque which produces a constant angular with uniform speed. If the difference between the
acceleration of 2 rad s −2 . Its net acceleration maximum and minimum tensions in the rope is
(in ms −2 ) at the end of 2 s is approximately 20 N, mass of the stone (in kg) is (Take, g = 10 ms −2 )
[NEET 2016] [EAMCET 2013]
(a) 7 (b) 6 (a) 0.75 (b) 1.0
(c) 3 (d) 8 (c) 1.5 (d) 0.5
Circular Motion 319

19 A car of mass 1000 kg negotiates a banked curve of 20 A car is moving in a circular horizontal track of
radius 90 m on a frictionless road. If the banking angle radius 10.0 m with a constant speed of 10.0 ms − 1. A
is 45°, the speed of the car is [CBSE AIPMT 2012] plumb bob is suspended from the roof of the car by a
(a) 20 ms −1 light rigid rod of length 10.0 m. The angle made by
(b) 30 ms −1 the rod with the track is (Take, g = 10 ms − 2 )
(c) 5 ms −1 [AFMC 2011]
(d) 10 ms −1 (a) zero (b) 30° (c) 45° (d) 60°

ANSWERS
l CHECK POINT 7.1
1. (d) 2. (c) 3. (a) 4. (a) 5. (a) 6. (d) 7. (c) 8. (b) 9. (d) 10. (c)
11. (d) 12. (a) 13. (a) 14. (d) 15. (b) 16. (c)

l CHECK POINT 7.2


1. (d) 2. (a) 3. (d) 4. (d) 5. (b) 6. (a) 7. (d) 8. (d) 9. (a) 10. (b)
11. (b) 12. (d) 13. (a) 14. (d) 15. (d) 16. (a) 17. (c) 18. (c) 19. (a) 20. (c)
21. (a) 22. (c)

(A) Taking it together


1. (d) 2. (b) 3. (a) 4. (c) 5. (a) 6. (b) 7. (a) 8. (c) 9. (d) 10. (b)
11. (c) 12. (c) 13. (a) 14. (c) 15. (d) 16. (c) 17. (b) 18. (a) 19. (c) 20. (a)
21. (c) 22. (c) 23. (c) 24. (a) 25. (d) 26. (d) 27. (c) 28. (d) 29. (d) 30. (a)
31. (b) 32. (c) 33. (c) 34. (c) 35. (a) 36. (b) 37. (b) 38. (b) 39. (c) 40. (a)
41. (d) 42. (a) 43. (d) 44. (a) 45. (a) 46. (a) 47. (a) 48. (d) 49. (a) 50. (b)
51. (c) 52. (a) 53. (c) 54. (d) 55. (d) 56. (c) 57. (d) 58. (c) 59. (b) 60. (a)
61. (b)

(B) Medical entrance special format questions


l Assertion and reason
1. (b) 2. (d) 3. (a) 4. (b) 5. (d)

l Statement based questions


1. (c) 2. (a) 3. (d) 4. (c) 5. (c)

l Match the columns


1. (a) 2. (b)

(C) Medical entrances’ gallery


1. (c) 2. (b) 3. (b) 4. (c) 5. (d) 6. (a) 7. (c) 8. (c) 9. (a) 10. (d)
11. (c) 12. (d) 13. (c) 14. (b) 15. (d) 16. (d) 17. (c) 18. (b) 19. (b) 20. (c)
Hints & Explanations
l CHECK POINT 7.1 Given, t = 2T =
8
s
2π 5
1 (d) ω = 120 rev/min = 120 × rad s −1 = 4π rad s −1 ∴ ω = αt
60
2π 2π 5π 8
2 (c) ωmin = rad/min and ω hr = rad/min ⇒ =α ×
60 12 × 60 2 5
25π
ωmin 2π / 60 12 ⇒ α= rads −2
∴ = = 16
ω hr 2π / 12 × 60 1
 20   25π 
∆ω ∴ a t = r (α ) =    
3 (a) α = but ω = 2πf  π   16 
∆t
2π∆f 2π × 200 = 31.25 ms −2
∴ Angular retardation, α = =
∆t 2 10 (c) In circular motion, a r can never be zero. So, option (c) is
= (200π ) rad s −2 correct.
ar v 2 / R
4 (a) As, ω = ω 0 + αt 11 (d) = (Let a t = a )
at a
(2π × 900 ) 4π
Here, ω 0 = 900 rpm = rad s −1, ω = 0 Here, v 2 = 2al = 2a (2πR ) = 4πaR. Therefore, the ratio is .
60 1
and t = 60 s
12 (a) a = a t2 + a n2
2π × 900
Q 0= + α × 60 a t = rate of change of speed = 2 ms −2
60
2π × 900 π v 2 (5)2
⇒ α=− =− rad s −2 an = = = 2.5 ms −2
60 × 60 2 R 10
π ∴ a = a t2 + a n2 = (2)2 + (2.5)2 = 3.2 ms −2
∴ |α | = rad s −2
2
13 (a) a n = a 2 − a t2 = (100 )2 − (60 )2 = 80 ms −2
5 (a) From equation of motion (angular),
ω (100 × 2π )/ 60 v2 v 2 (10 )2
0 = ω 0 − αt ⇒ α = 0 = = 0.69 rad s −2 Now, a n = or R = = = 1.25 m
t 15 R an 80
≅ 0.7 rads −2 14 (d) Given, ω = a − bt
Now, angle rotated before coming to rest is given by dω
α= = −b
 100 × 2π 
2 dt
  a t = Rα = − Rb
 60 
ω 20 …(i)
θ= = = 78.25 rad 2a
2α 2 × 0.7 At t = , ω = − a
θ b
∴ Number of revolutions = = 12 .5 a n = Rω 2 = Ra 2 …(ii)

v2
a (2v ) 4 2
Now, a = a t2 + a n2 =R a + b
4 2
6 (d) a = or a ∝ v 2 ⇒ after = 2 =
R a before v 1
15 (b) The ratio of tangential to centripetal acceleration is
7 (c) a = r ω 2 or a ∝ r ⇒ so if r1 < r2, then a1 < a 2 dv d  ds 
 
 at  dt  dt 
v 2 (6)2   = dt2 = r
8 (b) a t = 0, a = a n = = = 120 ms −2  ac  t = t v  ds 
2
R 0.3  
r  dt 
9 (d) As, v = ωr
d 2
 20  50 π 5π (6 t )
⇒ 50 = ω   ⇒ ω = = (12 t ) × 12 4
 π = dt 2 2 r = = 3
20 2 (6 t ) 36 t 4 t
2π 5π  at  4 1
⇒ = ∴   = =
T 2  a c  t = 2 s (2)3 2
4
⇒ T= s
5
Circular Motion 321

v2 20 × 20 v2 v2
ac 20 × 20 11 (b) tanθ = ⇒ tan 45° = ⇒ v = 30 m/s
16 (c) tan φ = = r = 80 = =1 rg (90 )(10 )
at dv 5 400
v 12 v 22
dt 12 (d) tanθ = = ⇒ v 2 = 2v1
a at Rg 4Rg
v 2 
φ  − 1 = 1
 v1 

ac θ % increase = 100%
mv 2
µ mv 2
13 (a) N = , mg = µ s N = s
R R
v Rg
∴ v=
⇒ φ = 45° µs
Angle between a (net) and v, θ = π − φ = 135° mv 2 µmv 2
14 (d) N = , mg = µN =
l CHECK POINT 7.2 R R
∴ Coefficient of friction between the tyres and wall of the well
1 (d) Centripetal force = mRω 2 = (2) (1) (2π )2 = 8π 2 N Rg 5 × 10
will be µ = 2 = = 0.40
mv 2 1 v (5 5 ) 2
2 (a) F = . If m and v are constants, then F ∝
r r 2
mvmin
15 (d) mg = or vmin = Rg (at topmost point)
F1  r2  R
∴ = 
F2  r1  16 (a) Maximum tension will be at bottommost point. Therefore,
mv 2
m  p p
2 2 the string will break at A.
3 (d) Radial force = =   = mv 2 mv 2
r r m mr 17 (c) At the highest point, T + mg = ⇒ mg ≤
r r
mv 2 16 × v 2
4 (d) Maximum tension = = 16 N ⇒ = 16 mv 2
r 144 18 (c) Tension, T = + mg cos θ
r
∴ v = 12 ms −1
mv 2
mv 2 (15 . v )2
. m ) (15 mv 2 For θ = 30 °, T1 = 1 + mg cos 30 °
5 (b) F = , F′ = = 2.25 = 2.25 F r
r (15
. r) r
mv 22
2.25F − F For θ = 60 °, T2 = + mg cos 60 °
Percentage increase = × 100 = 125% r
F
Q cos 30 ° > cos 60 °
Therefore, F has to be increased by 125%.
∴ T1 > T2 as v1 > v 2
6 (a) T = 100 N = mω 2mr
19 (a) In critical case, tension at topmost point is zero.
⇒ 100 = 100 × 10 −3 × ω 2m × 0.1 mv 2
−1 20 (c) T − mg = or T = 3 mg
⇒ ω m = 100 rads l
7 (d) Tension in the string, T = mω 2r = 4π 2n 2mr
h=l
n T v2 = 2gh = 2gl
∴ T ∝n 2
⇒ 2= 2 h
n1 T1 T
2T v
∴ n2 = 5 = 5 2 rpm
T
mg
8 (d) Maximum tension = m ω 2 r = m × 4π 2 × n 2 × r
By substituting the values, we get Tmax = 87.64 N 21 (a) vmax = 2gh = 2g (hmax − hmin )

9 (a) For safe turning, = 2 × 10 × 1.25 = 5 ms −1


v2 h v vb 2 2
tanθ = ⇒ = ⇒ h= ~ 22 ms −1
22 (c) vmin = gR = 9.8 × 49 = 21.9 −
gr b gR Rg
v
v 2 20 × 20
10 (b) To avoid skidding, tanθ = = =2
gr 10 × 20
⇒ θ = tan−1(2)
322 OBJECTIVE Physics Vol. 1

 80 π + 0 
⇒ θ= × 10 = 400 π rad
(A) Taking it together  2 
∴ Fan will turn angle 400 π after it is switched off.
2 (b) Here, rpm mean rotation per minute, i.e. it is frequency of
600 θ 400 π
rotation, f = 600 rpm ⇒ f = rps = 10 Hz ⇒ The number of rotation = = = 200
60 2π 2π
1 1 17 (b) h is elevation of outer rail over inner rail.
⇒ Time period, T = = = 0.1s
f 10 dv 2
h=
3 (a) The initial angular velocity, gr h
5π 1.67 × (15)2
ω 0 = 50 rpm = rad s −1 ⇒ h=
3 10 × 0.5 × 1000 d
Using ω = ω 0 + αt = 0.075 m = 75 mm
5π 18 (a) Difference between the kinetic energies at its highest and
0−
ω − ω0 3 rad s −2 = − π rad s −2 lowest positions is
Q α= =
t 60 36 = 2mgr = 2 × 1 × 10 ×1= 20 J
4 (c) Speed of particle is increasing due to tangential component v2 3 v2
of acceleration. Hence, dot product of a and v is positive. 19 (c) tan θ = ⇒ =
gr 5 10 × 6
6 (b) As the speed is constant throughout the circular motion, ⇒ v = 6 ms −1 = 21.6 kmh−1
therefore its average speed is equal to instantaneous speed.
5
7 (a) Minimum velocity at topmost point is Rg. 20 (a) Given, v = 144 kmh −1 = 144 × ms −1 = 40 ms −1
18
Mv 2 Mv 2 v2 (40 )2 1600
8 (c) Mg − N = or N = Mg − ∴ tan θ = = = =4
r r Rg 40 × 10 400
N ⇒ θ = tan−1(4)
v
5
21 (c) Given, v = 96 kmh−1 = 96 × ms −1 = 26.67 ms −1
18
Mg

9 (d) If the string suddenly breaks, the centripetal force will be


h
zero as the tension of string become zero but the tangential
force will be present due to tangential velocity, so the stone
travels in tangential direction. θ
1m
12 (c) For safe circular turn, v = µ rg
v2 (26.67)2
v2 (10 )2 ∴ tan θ = = = 0.089
⇒ r= ⇒ r= = 50 m Rg 800 × 10
µg 0.2 × 10
h
5 ⇒ 0.089 = ∴ h = 0.089 m = 8.9 cm
13 (a) Given, v = 72 kmh −1 = 72 × ms −1 = 20 ms −1 1
18
22 (c) α = constant
R = 100 m ∴ θ=
1 2
αt
v 2  400  −1  2
2
∴ θ = tan−1   = tan−1   = tan  
 Rg   100 × 10   5 Q θ ∝ t2
2
L θ 2 + θ1  2 + 2 θ
14 (c) L = mvr ⇒v = or =  = 4 or 2 = 3
mr θ1  2  θ1
mv 2 L2 l
∴ F = = 3 23 (c) From figure, h = l (1 − cos 60 ° ) = = 1m
r mr 2
∆v 2 × 5 10 l
15 (d) a av = = = ms −2
∆t π π
60
°

2400
16 (c) Given, f = 2400 rpm = = 40 Hz
60
Q ω 0 = 2πf = 80 π rad s −2
ω 0 + ω 
Now, θ= t h
 2  v
Circular Motion 323

Applying conservation of mechanical energy, we get 30 (a) From conservation of mechanical energy, v 2 = 2gl = 5gR
v = 2gh = 2 × 9.8 × 1 = 4.43 ms −1 O
2 x
 T − mg cos 30 °
24 (a) a = a n2 + a t2 =   + (g sin 30 ° )
2
 m 
R

30° T v
∴ R = 0.4 l
3
or x = l − R = 0.6 l or l
mg sin 30° 5
mu 2
31 (b) T = + mg cos θ; v 2 = u 2 − 2gh
30° R
mg
mg cos30° Ttop = 3mg and Tbottom = 9 mg
Ttop 3 mg 1
 3
2 Therefore, the ratio = =
 1 Tbottom 9 mg 3
=g  3−  +  =g
 2  4
32 (c) v = gr
Distance travelled
25 (d) ∆t = v = gr
Speed
(2πR / 6) 3.14 × 300
= = = 5.23 s r
v 60 × 3 v=?
(i) | ∆v | = | v f | − | vi | = v 2 + v 2 − 2 v ⋅v cos 60 °
= 2 v sin 30 ° = 60 ms −1
(KE )i + (PE )i = (KE )f + (PE )f
θ θ θ 1 2 1
⇒ mv + 0 = m ( gr )2 + mgr
v sinθ v sinθ 2 2
θ = 30°
⇒ v 2 = gr + 2gr ⇒ v = 3gr
2
v
(ii) ai = = 12 ms −2 33 (c) h′ = R − h
R
| ∆v | 60 ∴ vmin = 2gh′ ⇒ vmin = 2g (R − h )
(iii) | a av | = = = 11.5 ms −2
∆t 5.23
26 (d) Necessary centripetal force to the coin is provided by v h′
µg
friction. Thus, mr ωmax
2
= µmg or r = 2 h
ωmax
ωmax is made three times. Therefore, distance from centre r 34 (c) Two forces are acting on girl, tension and weight. Power
1 of tension will be zero and that of weight is,
will become times, i.e. 1 cm.
9
27 (c) | v AB | = | v A − v B | = (Rω )2 + (Rω )2 − 2(Rω )(Rω ) cos θ
θ
 θ T
= 2R ω sin   v = 2ms–1
 2
θ = 60°
29 (d) N
θ
45°
mg
amin
P = mgv cos (90 ° + θ )
45° 3
= − mgv sin 60 ° = − 50 × 9.8 × 2 ×
mg 2
= − 490 3 W
rg (tan θ − µ )
vmin = ⇒ vmin = 0, as µ = 1 ∴ Power delivered is 490 3 W.
1 + µ tan θ
324 OBJECTIVE Physics Vol. 1

35 (a) Height, h = l (1 − cos θ ) v 2 2gh 2gR (1 − cos θ )


41 (d) g sin θ = = =
v m2 = 2gh = 2gl (1 − cos θ ) R R R
1 2 or sin θ = 2 (1 − cos θ )
∴ Maximum kinetic energy, Km = mv m = mgl (1 − cos θ )
2 θ θ  θ
2 sin cos = 2  2 sin2 
2 2  2
36 (b) In the given problem, centripetal force will be equal to
F = kx θ 1 θ  1  1
⇒ tan = ⇒ = tan−1   or θ = 2 tan−1  
mv 2 kxr 2 2 2  2  2
⇒ kx = ⇒ v=
r m Note In extreme position of pendulum, only tangential component of
acceleration ( at = g sin θ) is present. In lowest position, only
2πr mr normal acceleration ( an = v 2 / R ) is present.
⇒ T= = 2π
v kx
42 (a) When body is released from the position P (inclined at
r
⇒ T∝ angle θ from vertical), then velocity at mean position,
x
v = 2gl (1 − cos θ )
r′ x
∴ T′ = T
rx′
3a × a  3 θ
l
=T = T
2a × 2a  2  T
d v t v d 2.5 5 l
37 (b) Q B = B = B ⇒ B = =
d C v Ct v C dC 2 4 P
v
38 (b) Given, radius of the horizontal circle, r = 80 cm = 0.80 m, mg sinθ θ
n = 14 and t = 25 s mg mg cosθ
Angular speed of revolution of the stone,
2πn n mv 2
ω= = 2π   ∴Tension at the lowest point = mg +
t  t l
m
22  14 88 = mg + [2gl (1 − cos 60 ° )] = mg + mg = 2 mg
ω = 2× ×  = rads −1 l
7  25 25
mv 2
Q Magnitude of centripetal acceleration = rω 2 43 (d) At mean position, mg + = 3 mg ⇒ v = 2gl …(i)
2 r
 88 ~ 992 cms −2
= 80 ×   = 9912
. − and if the body displace by angle θ with the vertical, then
 25
v = 2gl (1 − cos θ ) …(ii)
2h 2×8
39 (c) t = = = 128
. s On comparing Eqs. (i) and (ii), we get
g 9.8
cos θ = 0 ⇒ θ = 90°
20
Q v= = 15.63 ms −1 44 (a) Minimum tension is at topmost point (speed = v ) and
t
maximum tension at bottommost point (speed = u).
v2
∴ a= = 81. 4 ms −2 mu 2
R mg +
Tmax L =4
40 (a) From figure, we have h = l Q =
Tmin mv 2
v − mg +
L
⇒ u 2 = 4v 2 − 5gL …(i)
and u = v + 2g (2L )
2 2
…(ii)
l
On solving Eqs. (i) and (ii), we get
v = 10 ms −1
u
45 (a) Balancing horizontal forces, T sin θ = mrω 2
∴ v = u 2 − 2gh = u 2 − 2gl or T sin θ = m (l sin θ ) ω 2
∴ T = mlω 2 = ml (2πf )2
⇒ | ∆v | = | v f − vi | = v 2 + u 2 − 2vu ⋅ cos 90 ° 2
 2
= ml  2π ×  = 16 ml
= v + u = (u − 2gl ) + u = 2 (u − gl )
2 2 2 2 2  π
Circular Motion 325

46 (a) v = 2a t s = 2 a t (πR ) Q In right angled ∆ AOB,


OA 1 0.5 − h
an
a net cos 60° = ⇒ = ⇒ R = 1 − 2h
AB 2 R
⇒ 0.5 = 1 − 2h (Q R = 0.5 m)
1
⇒ h=
4
1
θ at
Q v = u 2 − 2gh = 9 − 2 × 10 × = 9 − 5 = 2 ms −1
4

v2 a a ∴ v = 2 ms −1
∴ an = = 2πa t or n = 2π ⇒ tan θ = n = 2π
R at at 50 (b) Complete the circle, the body should not loose contact with
∴ −1
θ = tan (2π ) the track anywhere, so
v ≥ gr
d 21 × 2π × 0.5
47 (a) v = = = 1.5 ms −1
t1 44 A

2h 2 × 4.9
t2 = = =1s B
g 9.8
h
2r

0.5 m r
Applying law of conservation of mechanical energy between
points A and B, i.e. magnitude of change in kinetic energy
1.5 m
equals the magnitude of change in potential energy.
Horizontal distance travelled by drop = vt2 = 1.5 m ⇒
1 2
mv − 0 = mg (h − 2r )
2
∴ r = (1.5)2 + (0.5)2 = 2.5 m
1 5
m ( gr )2 = mg (h − 2r ) ⇒ h = r
48 (d) R = 1.6 cos 60 ° = 0.8 m 2 2
O Hence, h must be atleast equal to 2.5 r.
51 (c) When released from top with zero velocity block leaves
contact at point B.
1.6 m
T A

60°
A r–r cos θ N
R B

θ θ
2mg mv 2 r cos θ cos
T sin 60° = mg or T = and T cos 60 ° = mg
R FC
3 mg mg sin θ
v2 T g 9.8
∴ = = = ms −2 O s
R 2m 3 3 According to diagram,
9.8 mv 2
⇒ v= × 0.8 ms −1 mg cos θ − N =
3 r
R v 2 9.8 When the body is detached, then N = 0.
Time period = 2π and a c = = ms −2
v R 3 mv 2
∴ mg cos θ =
R r
49 (a) h = R (1 − cos 60 ° ) =
2 v2
A ⇒ cos θ = …(i)
rg
60° R v Applying the conservation of mechanical energy at position A
0.5 – h
and B,
0.5 O B KA + U A = KB + U B
1
h h 0 + mgr (1 − cos θ ) = mv 2 + 0
2
u
C
326 OBJECTIVE Physics Vol. 1

v2 Particle will leave contact at B, if component of weight is just


2 (1 − cos θ ) = …(ii)
rg equal to centripetal force (towards centre).
From Eqs. (i) and (ii), we get mv 2
or mg sin β = or sin β = 2 cos α − 2 sin β
2 − 2 cos θ = cos θ r
2 ∴ 3 sin β = 2 cos α
⇒ cos θ =
3 57 (d) ω = 2πf = 2π (0.5) = π rad s −1
2 2r
∴ h = r cos θ = r × = ω
3 3 N
θ
52 (a) To find tension in the ring, let us take an arc which
subtends angle 2(d θ ) at centre. Tangential components of T
cancel out each other, while inward components provide the mg
d
necessary centripetal force. Thus, θ
R
dθ dθ
From the figure,
T T
Rω 2
⇒ N sin θ = mR ω 2 and N cos θ = mg or tan θ =
ω dθ dθ g
g tan θ
∴ R=
ω2
10 × tan 30 ° 1
2T sin (dθ ) = (dm ) R ω 2 = (λ 2R ⋅ d θ ) (Rω 2 ) ⇒ R= ≈ m
(π 2 ) 3
Here, λ = linear mass density.
R 1/ 3 1/ 3 2
For small angles, sin dθ ≈ dθ Now, d = = = = = 0.67 m
cos θ cos 30 ° 3/ 2 3
∴ 2Tdθ = 2λR 2ω 2dθ or T = λR 2ω 2
Tmax 1 40 1 58 (c) h = l + l sin θ = l (1 + sin θ )
⇒ ωmax = ⋅ = × = 20 rad s −1
λ R 0.1 1 v 2 = u 2 − 2gh = u 2 − 2gl (1 + sin θ )
l v
53 (c) h = l (1 − cos 60 ° ) = , v 2 = 2gh = gl
2
mv 2 θ
Now, Tmax − mg = (at bottommost point)
l h
l
∴ Tmax = 2mg = µ s (4 mg )
1
∴ µ s = 0.5 or u
2
String will slack, where component of weight towards centre
54 (d) From momentum conservation, is just equal to centripetal force.
u
mu = m + (6 m ) 5 × 10 × 0.5 mv 2 m 2
4 or mg sin θ = = [u − 2gl (1 + sin θ )]
l l
Solving, we get
7gl
u = 40 ms −1 Substituting u 2 = , we get
2
55 (d) Let ω is the angular speed of revolution. 1
sin θ = or θ = 30 °
T1 T2 T3 2
O
A B C ∴The desired angle is 90 ° + 30 ° or 120°.
l l l 1
59 (b) Given, mv 2 = as 2
T3 = m ω 2 (3l ) 2
mv 2 2as 2
or Fn = = …(i)
T2 − T3 = m ω 2 2l R R
⇒ T2 = mω 2 (5l ) 2a dv 2a ds 2a
Further, v= ⋅s or a t = = ⋅ = ⋅v
T1 − T2 = mω l 2
⇒ T1 = mω (6l ) 2 m dt m dt m
T3 : T2 : T1 = 3 : 5 : 6 2a 2a 2as
= ⋅ ⋅s =
m m m
56 (c) hAB = (r cos α − r sin β )
∴ Ft = ma t = 2as …(ii)
Velocity of particle at B, 2
s
v = 2ghAB = 2g (r cos α − r sin β ) ∴ Fnet = Fn2 + Ft2 = 2as 1 +
R2
Circular Motion 327

60 (a) When a is horizontal 3 (a) Let the minimum and maximum tensions be Tmin and Tmax
and the minimum and maximum speed be u and v.
α
at = g v
v2
an =
R
Tmin
At α = 90°, acceleration
is downwards At α = 0°, acceleration
Tmax
is upwards

α
u
an
mu 2
∴ Tmax = + mg
R
mv 2
α Tmin = − mg
R
Horizontal
at  u 2 v 2
∴ ∆T = m  −  + 2mg
an v2 /l 2gh / l R R
tan α = = =
a t g sin α g sin α u2 v 2
From conservation of energy, − = 4g
2g l cos α / l R R
= = 2 cot α
g sin α ⇒ ∆T is independent of u and ∆T = 6 mg.
1 ∴ Reason is the correct explanation of Assertion.
⇒ tan α = 2 or cos α =
3 4 (b) In case of massless rod, minimum speed at lower most
−1  1  position of the particle is 5gl to complete the circular
∴ α = cos  
 3 motion.
Work done by centripetal force on the particle is always zero.
61 (b) h = R − R cos θ
u N 5 (d) The normal reaction is not least at topmost point, hence
h Assertion is false.
v
θ

Statement based questions


R cos θ

l
cos
mg

mg 2 (a) Direction of acceleration continuously changes. Also, v is


θ
always perpendicular to ω.
` 5 (c) In vertical direction, N cos θ = mg
v 2 = u 2 + 2gh = u 2 + 2gR (1 − cos θ ) ...(i)
mv 2
2 In horizontal direction, N sinθ =
mv R
mg cos θ − N = ...(ii)
R
l Match the columns
When the particle loses contact,
N =0 ...(iii) 1 (a) T3 = (1) (3) (4)2 = 48 N (Q F = mRω 2 )
From Eqs. (i), (ii) and (iii), we get T2 − T3 = (1) (2) (4)2 = 32 N
⇒ v 2 = gR cos θ = u 2 + 2gR (1 − cos θ ) ∴ T2 = 80 N
⇒ 3gR cos θ = u 2 + 2gR ⇒ T1 − T2 = (1) (1) (4)2 = 16 N
u2 2  u2 2 ∴ T1 = 96 N
⇒ cos θ = + ⇒ θ = cos −1 + 
3gR 3  3gR 3 Hence, A → p, B → q, C → r.
2 (b) v B2 = u A2 − 2ghAB = (9gR ) − (2gR ) = 7gR
(B) Medical entrance special format
∴ v B = 7gR
questions mv B2
l Assertion and reason Further, TB = = 7 mg
R
v 2 (2)2 Again, v C2 = v A2 − 2ghAC = (9gR ) − 2g (2R ) = 5gR
1 (b) a c = = = 4 ms −2
R 1
∴ v C = 5gR
But no information is given for tangential acceleration a t .
328 OBJECTIVE Physics Vol. 1

mv C2 Thus, the minimum angular velocity,


Further, TC + mg =
R g 10
ωmin = = = 10 rad/s
∴ TC = 4 mg rµ 1 × 0.1
Hence, A → r, B → q, C → p, D → t.
3 (b) Let the mass m which is attached to a thin wire and is
whirled in a vertical circle as shown in the figure below.
(C) Medical entrances’ gallery C

1 (c) Velocity of point mass in vertical circle at lowest point,


vl = 7gr
∴ vl = 7gr > 5gr D B
T
Hence, point mass will have completed the vertical circular path. l m
P
We know that,
θ mg cos θ
A
mg
The tension in the string at any point P be T.
According to Newton’s law of motion in equilibrium, net force
Tbottom towards the centre = centripetal force
mv 2
⇒ T − mg cos θ =
l
√7gr where, l = length of wire and v = linear velocity of the mass
mg whirling in a circle.
mv 2
mv 2 m ⇒ T = mg cos θ +
Tbottom − mg = = ( 7gr )2 l
r r
mv 2
⇒ Tbottom − mg = 7mg At point A, θ = 0 ° ⇒ TA = mg +
l
⇒ Tbottom = 8mg
mv 2
2 (b) Given, mass of cylinder, m = 10 kg At point B, θ = 90 ° ⇒ TB =
l
radius of cylinder, r = 1m mv 2
and coefficient of friction, µ = 0.1 At point C, θ = 180 ° ⇒ TC = − mg +
l
The given situation can be as shown in the figure given mv 2
below. At point D, θ = 90 ° ⇒ TD = TB =
l
So, from the above analysis, it can be concluded that the
fl
tension is maximum at point A, i.e. the lowest point of circle.
fc = mrω2 So, chance of breaking is maximum.
N
4 (c) From energy conservation,
r 1
mgH = mv 2
2
mg ⇒ v 2 = 2 gH or v = 2gH …(i)
From the above figure, it can be concluded that the block will To complete circular loop, minimum speed at bottom point
be stationary when the limiting friction (fl ) is equal to or should be vmin = 5gR …(ii)
greater than the downward force or weight of block,
i.e. fl ≥ mg …(i) From Eqs. (i) and (ii), we get
Also, the magnitude of limiting friction between two bodies is 2H
2gH ≥ 5gR ⇒ R ≤
directly proportional to the normal reaction (N) between them, 5
i.e. fl ∝ N or fl = µ N …(ii) 2H 2(5)
⇒ Rmax = = = 2 cm
From Eqs. (i) and (ii), we get 5 5
µN ≥ mg 5 (d) If a body is moving on a frictionless surface, then its total
or µ(mrω 2 ) ≥ mg (Q N = mrω 2 ) mechanical energy remains conserved.
g According to the conservation of mechanical energy,
⇒ ω≥
rµ (TE)initial = (TE)final
Circular Motion 329

⇒ (KE )i + (PE )i = (KE )f + (PE )f 3


⇒ v 2 = R × 15 ×
1 2
0 + mgh = mv A2 + 0
2 3
= 2.5 × 15 ×
v2 2
⇒ gh = A
2 ∴ v = 5.7 ms −1
v2
or h= A …(i) 9 (a) According to question, a car is negotiating a curved road of
2g radius R. The road is banked at angle θ and the coefficient of
In order to complete the vertical circle, the velocity of the friction between the tyres of car and the road is µ s . So, the
body at point A should be given situation can be drawn as shown in figure below.
v A = vmin = 5gR N cos θ
where, R is the radius of the body. N
AB D θ
Here, R= = N sin θ
2 2 fl cos θ
θ
5
⇒ vmin = v A = gD fl
fl sin θ
2
θ
Substituting the value of v A in Eq. (i), we get mg
2
 5 
  gD  Considering the case of vertical equilibrium,
 2 
h= N cos θ = mg + fl sin θ
2g ⇒ mg = N cos θ − fl sin θ …(i)
5 gD 5 Considering the case of horizontal equilibrium,
= = D
2 × 2g 4 mv 2
N sin θ + fl cos θ = …(ii)
6 (a) Consider the string of length l connected to a particle as R
shown in the figure. On dividing Eq. (ii) by Eq. (i), we get
v 2 sin θ + µ s cos θ
= (Q fl ∝ µ s )
v Rg cos θ − µ s sin θ

O T Fc  sin θ + µ s cos θ 
l ⇒ v = Rg  
 cos θ − µ s sin θ 

 tan θ + µ s 
⇒ v = Rg  
 1 − µ s tan θ 
Speed of the particle is v. As, the particle is in uniform
circular motion, net force on the particle must be equal to 10 (d) According to given question, a uniform circular disc of
centripetal force which is provided by the tension (T ). radius 50 cm at rest is free to turn about an axis
perpendicular to its plane and passes through its centre. This
∴ Net force = Centripetal force = Tension in string
situation can be shown by the figure.
mv 2
⇒ =T
l
7 (c) At the lowest point of a vertical circle, the minimum 0.5 m
velocity , vmin = 5gr
Velocity at highest point, v = gr
So, Assertion is correct but Reason is incorrect.
Angular acceleration, α = 2 rad s −2 (Given)
8 (c) Centripetal acceleration of a particle moving on a circular −1
path is given by Angular speed, ω = αt = 4 rad s
v 2 ∴ Centripetal acceleration, a c = ω 2r = (4)2 × 0.5 = 16 × 0.5
aC =
R a c = 8 ms −2
−2
In the given figure, a C = a cos 30 ° = 15 cos 30 ° ms Hence, linear acceleration at the end of 2s,
v 2 a t = αr = 2 × 0.5
⇒ = 15 cos 30 °
R ⇒ a t = 1ms −2
330 OBJECTIVE Physics Vol. 1

Therefore, the net acceleration at the end of 2 s is given by ⇒ v 2 = 2a t s or v 2 = 2a t (4πr )


a= a c2 + a t2 (Q particle covers 2 revolutions)
a = (8) + (1) = 65
2 2 v2 16 × 10 −2
⇒ at = =
−2
8πr 8 × 314. × 6.4 × 10 −2
⇒ a ≈ 8 ms
[Using Eq. (i)]
11 (c) According to question, we have ∴ a t ≈ 0.1ms −2
C
13 (c) A particle moving in a curved path is experiences an
R TC acceleration. Even if it moving around the perimeter of a
B circle with a constant speed, there is still a change in velocity
TB and subsequently an acceleration. This acceleration is directed
TA
towards the centre of the circle. The quantity which may
v0 remain constant, here is magnitude of acceleration.
A
14 (b) We know that, angular speed of a watch,
mg
θ 1
Let the tension at point Abe TA. So, from Newton’s second law, ω= ⇒ ω∝
t t
mv C2 ω second thour 12 × 3600
TA − mg = Hence, = = = 720
R ω hour tsecond 60
1 2 ω second 720
Energy at point A = mv 0 …(i) i.e. =
2 ω hour 1
1
Energy at point C = mv C2 + mg × 2R …(ii) π
2 15 (d) We know that, ω = rads −1
30
Applying Newton’s second law at point C,
π
mv C2 ∴ v = r ω = 10 × (Q r = 10 cm)
TC + mg = 30
R π 3.14
= = = 1.046 ≅ 1 cm s −1
To complete the loop TC ≥ 0 3 3
mv C2 16 (d) The figure is shown as below
So, mg =
R 2 r sin θ/2
⇒ v C = gR …(iii)
A B C
From Eqs. (i) and (ii) by conservation of energy, θ
2 θ
1 2 1 2 r
mv 0 = mv C + 2mgR
2 2 O
1 2 1
⇒ mv 0 = mgR + 2mgR [Using Eq. (iii)]
2 2
⇒ v 02 = gR + 4gR
θ AB
⇒ v 0 = 5gR In ∆AOB, sin =
2 AO
θ θ
12 (d) Given, mass of particle, m = 0.01 kg ⇒ AB = AO sin ⇒ AB = r sin (Q AO = r )
2 2
Radius of circle along which particle is moving, r = 6.4 cm
Q AC = AB + BC
Q Kinetic energy of particle, KE = 8 × 10 −4 J θ θ
AC = r sin + r sin (Q AB = BC )
1 2 2 2
⇒ mv = 8 × 10 −4 J
2 θ
AC = 2 r sin
16 × 10 −4 2
⇒ v2 = = 16 × 10 −2 …(i)
0.01 So, the magnitude of the corresponding linear displacement
θ
As it is given that kinetic energy of particle is equal to will be 2 r sin .
8 × 10 −4 J by the end of second revolution after the beginning 2
of motion of particle. It means, its initial velocity (u ) is 0 ms −1 17 (c) We know that, ω = 2πn
at this moment. Given, n1 = 1800 rpm, n 2 = 3000 rpm, ∆t = 20 s
By Newton’s third equation of motion, 1800
ω1 = 2π n1 = 2π × = 2π × 30 = 60 π rad/s
v 2 = u 2 + 2a t s 60
Circular Motion 331

3000
Similarly, ω 2 = 2πn 2 = 2π × = 2π × 50 = 100 π rad/s 19 (b) The angle of banking,
60 v2
If the angular velocity of a rotating wheel about an axis is tanθ =
rg
changed by change in angular velocity in a time interval ∆t,
then the angular acceleration of rotating wheel about that axis Given, θ = 45°, r = 90 m and g = 10 ms −2
is given by v2
Change in angular velocity tan 45° =
α= 90 × 10
Time interval
⇒ v = 90 × 10 × tan 45°
ω 2 − ω1 100 π − 60 π
= =
∆t 20 Speed of car, v = 30 ms −1
40 π −2 20 (c) If angle of banking is θ, then
= = 2π rads
20
mv 2 /r v2
18 (b) We know that, the difference between the maximum and tan θ = ⇒ tan θ =
mg rg
minimum tensions in the rope,
Given, v = 10 ms −1, r = 10 m and g = 10 ms − 2
Tmax − Tmin = 2mg
Tmax − Tmin = 20 (10 )2
Here, So, tan θ = =1
10 × 10
20 20
Now, mass of the stone, m = = = 1kg ∴ θ = 45°
2g 2 × 10
CHAPTER
08

COM, Conservation
of Momentum
and Collision
CENTRE OF MASS (COM)
For a system of particles or a body, the centre of mass is defined as a point at
which the total mass of the system or of the body is supposed to be concentrated.
On application of external forces, centre of mass of the system of particles moves
in the same way as a point having mass equal to that of the whole system moves.

Position of centre of mass for a system of two particles


Consider two particles of masses m1 and m 2 located at position vectors r1 and r2 .
y
m1

r1 rCM
m2

r2
x

z
Inside
Fig. 8.1 System of two particles 1 Centre of mass (COM)
2 Motion of centre of mass
Then, position of centre of mass rCM is given as Linear momentum of a
m r + m 2r2 system of particles
rCM = 11 3 Collision
m1 + m 2 Types of collisions
Newton’s law of restitution
m1r1 + m 2r2 Σmi ri
rCM = =
m m
where, m = m1 + m 2 = total mass of system.
COM, Conservation of Momentum and Collision 333

The x and y-coordinates of centre of mass can be written m1y1 + m 2 y 2 (1) (2) + (2) (3) 2 + 6 8
Similarly, y= = = =
as m1 + m 2 1+ 2 3 3
m x + m 2x 2  −1 8 
x CM = 1 1 Therefore, the coordinates of centre of mass will be  ,  .
m1 + m 2  3 3
m1 y 1 + m 2 y 2 Example 8.2 Two particles of masses 1 kg and 2 kg are
and y CM = located at x = 0 and x = 3 m. Find the position of their
m1 + m 2
centre of mass.
Hence, the centre of mass of two particles system lies Sol. Since, both the particles lie on X-axis, so the CM will also
between the two particles on the line joining them and the lie on X-axis. Let the CM be located at x from 1 kg mass, then
distance of the centre of mass from masses is in inverse r1 = distance of CM from the particle of mass 1 kg = x
ratio of masses of the particles. and r2 = distance of CM from the particle of mass 2 kg
1 r1 m 2 = (3 − x )
i.e. r ∝ ⇒ = m1 = 1 kg m2 = 2 kg
m r 2 m1 CM

x=0 x = xm x = 3m
Consider two particles of masses m1 and m 2 at distance r
r1 = x m r2 = (3 – x) m
from each other. Their centre of mass (CM) must lie in
between them on the line joining them. Using
r1 m 2
= , we get
Let distances of these particles from the centre of mass be r2 m1
r1 and r 2 . x 2
= or x = 2 m
r 3−x 1
r1 r2
Thus, the CM of the two particles is located at x = 2 m.
m1 CM m2
Example 8.3 Two point objects of masses 1.5 g and 2.5 g
Fig. 8.2
respectively are 16 cm apart, the centre of mass is at a
1 distance x from the object of mass 1.5 g. Find the value
Q r ∝ ⇒ m1r1 = m 2r 2 of x.
m
Sol. As, centre of mass of two particles system lies between the
m 2r 2 m1r1
r1 = and r2 = two particles on the line joining them.
m1 m2 16 cm

If the two particles have the same mass, i.e. m1 = m 2 = m, 1.5 g CM 2.5 g
then x (16 – x) cm
mr + mr 2 r1 + r 2 ∴ From
r1 m 2
= ⇒
x
=
2.5
⇒ x = 10 cm
r CM = 1 = r2 m1 16 − x 1.5
2m 2
Thus, for a system of two particles of equal mass, the
centre of mass lies exactly midway between them.
Position of centre of mass for a
If m1 ≠ m 2, centre of mass is nearer to the particle of larger
system of large number of particles
mass. If we have a system consisting of n particles of masses
m1, m 2, K, mn with r1, r2, ..., rn as their position vectors at a
Example 8.1 Two bodies of masses 1 kg and 2 kg are located
at (1, 2) and (−1, 3), respectively. Calculate the coordinates given instant of time. The position vector rCM of the
of centre of mass. centre of mass of the system at that instant is given by
n
Sol. Let the coordinates of centre of mass be (x, y ). Σ mi ri
m1 r1 + m 2 r2 + K + mn rn i =1
Given, mass, m1 = 1 kg, m 2 = 2 kg rCM = =
Coordinates, x1 = 1, x 2 = − 1, y1 = 2 and y 2 = 3 m1 + m 2 + K + mn n
Σ mi
m x + m 2 x2 i =1
Q x= 1 1 n
m1 + m 2
Σ mi ri
1 × 1 + (2) (−1) i =1
⇒ x= or rCM =
1+ 2 M
1 − 2 −1 Here, M = m1 + m 2 + K + mn and Σ mi ri is called the
= =
3 3 first moment of the mass.
334 OBJECTIVE Physics Vol. 1

Further, ri = x i $i + y i $j + z i k
$ m1y1 + m 2 y 2 + m 3 y 3 + m 4 y 4
Similarly, yCM =
m1 + m 2 + m 3 + m 4
and rCM = x CM $i + y CM $j + z CM k
$
1(1) + 2(1) + 3(0) + 4(0) 3
= = m = 0.3 m
So, the cartesian coordinates of the CM will be 1+ 2+ 3+ 4 10
n
∴ Centre of mass is at (x CM , y CM) = (0.5, 0.3 )
Σ mi x i
m1x 1 + m 2 x 2 + K + mn x n i =1
x CM = = Example 8.6 Three point masses m1 = 2 kg, m 2 = 4 kg and
m1 + m 2 + K + mn Σ mi m 3 = 6 kg are kept at the three corners of an equilateral
n triangle of side 1 m. Find the location of their centre of
Σ mi x i mass.
i =1
or x CM = Sol. Assume m1 to be at the origin and X-axis along the line
M
joining m1 and m 2 as shown in figure.
n n
Σ mi y i Σ mi z i Y
m3
i =1 i =1
Similarly, y CM = and z CM =
M M 1m 1m
Example 8.4 The position vectors of three particles of masses
X
m1 = 1 kg, m 2 = 2 kg and m 3 = 3 kg are r1 = ($i + 4$j + k$ ) m, m1 1m m2

r2 = ($i + $j + k$ )m and r3 = (2$i − $j − 2 k$ ) m, respectively. Find From the figure, it is clear that the coordinates of m1 are
the position vector of their centre of mass. (x1, y1) = (0, 0) that of m 2 are (x 2, y 2 ) = (1, 0) and that of m 3 are
1 3
Sol. The position vector of CM of the three particles will be given by (x 3, y 3 ) =  , 
m1r1 + m 2r2 + m 3r3 2 2 
rCM =
m1 + m 2 + m 3 Coordinates of centre of mass are
2 × 0 + 4 × 1 + 6 × 1/2 7
Substituting the given values in above equation, we get x CM = = m
2+4+6 12
1 ($i + 4$j + k$ ) + 2 ($i + $j + k$ ) + 3 (2$i − $j − 2k$ )
rCM = 2 × 0 + 4 × 0 + 6 × 3 /2 3 3 3
1+ 2 + 3 and y CM = = = m
2+4+6 12 4
9 i + 3 j − 3k
$ $ $ 1
= ⇒ rCM = (3$i + $j − k$ ) m  7 3
6 2 ∴ Centre of mass is at (x CM , y CM ) =  , .
12 4 
Example 8.5 Four particles of masses 1 kg, 2 kg, 3 kg and
4 kg are placed at the four vertices A, B, C and D of a
square of side 1 m. Find the position of centre of mass of the
Position of centre of mass of
particles. continuous bodies
Sol. Assuming D as the origin, DC as X-axis and DA as Y-axis, For a real body, which has a continuous distribution of
we have matter, point masses are differential mass elements dm and
Y
m1 m2 their centre of mass is defined as
∫ x dm = ∫ x dm
(x1 , y1 ) A B (x2 , y2 )
x CM =
∫ dm M

∫ y dm = ∫ y dm
m4 m3
X
(x4 , y4 ) D C (x3 , y3 ) y CM =
m1 = 1 kg; (x1, y1) = (0, 1 m)
∫ dm M

m 2 = 2 kg; (x 2, y 2) = (1 m, 1 m)
and z CM =
∫ z dm = ∫ z dm
m 3 = 3 kg; (x 3, y 3) = (1 m, 0)
and m 4 = 4 kg; (x 4, y 4) = (0, 0) ∫ dm M

Coordinates of their CM, where, M is total mass of that real body.


m x + m 2x 2 + m 3x 3 + m 4x 4 If we choose the origin of coordinate axes at centre of
xCM = 1 1 mass, then
m1 + m 2 + m 3 + m 4

=
1(0) + 2(1) + 3(1) + 4(0) 5 1
= = m = 0.5 m ∫ xdm = ∫ ydm = ∫ z dm = 0
1+ 2+ 3+ 4 10 2
COM, Conservation of Momentum and Collision 335

Example 8.7 The linear density of a thin rod of length 1 m


varies as λ = (1 + 2x ), where x is the distance from its one
Position of centre of mass of
end. Find the distance of its centre of mass from this end. symmetrical bodies
Sol. Let the X-axis be along the length of the rod and origin at one Given below are three points which are very important
of its end as shown in figure. regarding the centre of mass of symmetrical bodies
(i) For the bodies symmetrical about both the axes (X or
x Y) or all the three axes (X, Y, Z ), the centre of mass
dx
As, rod is along X-axis, for all points on it Y and Z will be will lie at point of intersection of symmetrical axes.
zero, so y CM = 0 and z CM = 0, i.e. centre of mass will be on There is no need to determine centre of mass.
the rod. Now, consider an element of rod of length dx at a (ii) The bodies which are symmetrical about one axis,
distance x from the origin. centre of mass will lie on that axis. Determine only
Mass of this element, dm = λdx = (1 + 2x )dx that coordinate about which there is symmetry.
1

∫ x dm ∫ x (1 + 2x )dx (iii) If an arrangement or body is not symmetrical about


any axis, then determine all the required coordinates.
x CM = = 0

∫ dm
1
Centre of mass of some symmetric bodies are given in a
∫ (1 + 2x )dx table below
0
1 Body Figure Position of CM
x 2
2x 3
 +  Uniform sphere Centre of the sphere
 2 3 0
= (hollow or solid)
[x + x 2]10 C

 (1)2 2(1)3  1 2
 + −0− 0 +
7
= 2 3 =2 3 = m Uniform circular Centre of the ring
 1 + (1) − 0 − 0
2 ring
 2 12
C
 

Example 8.8 A straight rod of length L has one of its ends at the
origin and the other at x = L. If the mass per unit length of the Uniform circular Centre of the disc
rod is given by Ax, where A is constant, where is its centre of disc
C
mass?
Sol. Let the mass of an element of length dx of the rod located at
a distance x away from left end be Axdx. Uniform rod C Centre of the rod

Y
A plane square Point of intersection
x
lamina of diagonals
O X C
(0,0) x=0 dx x=L

L
Triangular lamina Point of intersection
The x-coordinate of the centre of mass is given by of the medians
1 1 L
x CM =
M ∫ x dm =
M ∫0
x (Axdx )
C
L

∫ x ⋅ Axdx [Ax 3 /3]L0


2
Rectangular cubical
block
Point of intersection
of the diagonals
= 0
= = L C
L
[Ax 2 /2]L0 3
∫ Axdx
0
1 Cylinder Middle point of the
The y-coordinate is yCM =
M ∫ y dm = 0 (hollow or solid) axis of the cylinder
and similarly, z CM = 0 C
2  2
Hence, the centre of mass is at  L, 0, 0 or at L from one
3  3
end.
336 OBJECTIVE Physics Vol. 1

Body Figure Position of CM Example 8.10 A small disc of radius 2 cm is cut from a disc
Cone or pyramid On the axis of the of radius 6 cm. If the distance between their centres is
cone at a distance 3.2 cm, what is the shift in the centre of mass of the disc?
h 3h / 4 from the vertex, Sol. Let radius of complete disc be a and that of small disc be b.
C
h/4 where h is the height Also, let centre of mass now shifts to O 2 at a distance l from
of the cone. original centre.
Uniform Coordinates of
CM =  0,
semicircular wire 2R 
C  or a
2R  π
π (0, 0.64R )
O b
O2 O1 X-axis
O R
(0, 0)
l x1
Uniform Coordinates of
semicircular plate or
CM =  0,
4R 
C 
disc 0, 4R  3π  The position of new centre of mass is given by

(0, 0) O R or (0, 0.42R ) A2x 2 − A1x1 − σπb 2x1
x CM = = (Q x 2 = 0)
A2 − A1 σπa 2 − σπb 2
Example 8.9 Find the position of centre of mass of the where, σ = mass per unit area.
uniform lamina as shown in figure, if small disc of radius a/2 Here, a = 6 cm, b = 2 cm, x1 = 3.2 cm
is cut from disc of radius a.
− σ × π (2)2 × 3.2
Y Hence, x CM =
σ × π × (6)2 − σ × π × (2)2
12.8 π
=− = − 0.4 cm
32π
a
O X Negative sign indicates the left side shift from the centre.

Example 8.11 Two identical rods each of mass m and length


L are connected as shown in the figure. Locate the centre of
mass.
Sol. Here, A1 = area of complete circle = πa 2
A2 = area of small circle
2
a  πa 2
=π  =
 2 4
(x1, y1) = coordinates of centre of mass of large circle
= (0, 0) Sol. This system is symmetrical about the X-axis. Hence, we
a  need to find x CM. Here, we will take coordinates of centre
(x 2, y 2 ) = coordinates of centre of mass of small circle =  , 0
2  of mass of rods.
A1x 1 − A 2x 2 Y
Using x CM = , we get
A1 − A 2
π a 2 a  1 2
πa 2 × 0 −   LO
4  2 X
x CM =
πa2
πa2 − L
4
 1 mass, m1 = m
−  For rod 1,
 8 a
= a=− x-coordinate, x1 = 0
 3 6
  For rod 2, mass, m 2 = m
 4
x-coordinate, x 2 = L /2
and yCM = 0 as y1 and y 2 both are zero.Therefore, coordinates
 a  m1x1 + m 2x 2
of CM of the lamina as shown in figure are  − , 0 . ∴ x CM =
 6  m1 + m 2
COM, Conservation of Momentum and Collision 337

L L Sol. Distance OC of the centre of mass from the centre,


m × 0+m × m
x CM = 2 = 2 =L Y
m +m 2m 4
C
yCM
Similarly, y CM = 0
L  π/4 X
∴ Centre of mass =  , 0 O xCM
4 
r sin π / 4 2 2r
OC = =
Example 8.12 Find the coordinates of centre of mass of a π/4 π
quarter ring of radius r placed in the first quadrant of a  2r 2r 
cartesian coordinate system with centre at origin. ∴ Coordinates of centre of mass (x CM, y CM ) are  ,  .
 π π

CHECK POINT 8.1


1. Two bodies of masses 1 kg and 2 kg are lying in xy-plane at (a) (0.8, 0.6) m (b) (0.6, 0.8) m
(−1, 2) and (2, 4), respectively. What are the coordinates of (c) (0.4, 0.4) m (d) (0.5, 0.6) m
the centre of mass? 7. Centre of mass of three particles of masses 1 kg, 2 kg and
(a) 1, 
10
(b) (1, 0) 3 kg lies at the point (1, 2, 3) and centre of mass of another
 3 system of particles of total mass 3 kg lies at the point
(c) (0, 1) (d) None of these (−1 , 3 , − 2). Where should we put a particle of mass 5 kg, so
2. The centre of mass of a system of two particles divides the that the centre of mass of the entire system lies at the
distance between them centre of mass of first system?
(a) in inverse ratio of square of masses of particles (a) (0, 0, 0) (b) (1, 3, 2)
(b) in direct ratio of square of masses of particles (c) (−1 , 2, 3) (d) None of these
(c) in inverse ratio of masses of particles 8. Three identical spheres, each of mass 1 kg are placed
(d) in direct ratio of masses of particles touching each other with their centres on a straight line.
3. In carbon monoxide molecules, the carbon and the oxygen Their centres are marked P, Q and R, respectively. The
distance of centre of mass of the system from P is
atoms are separated by distance 1.2 × 10 −10 m. The distance
PQ + PR + QR PQ + PR
of the centre of mass from the carbon atom is (a) (b)
3 3
(a) 0.48 × 10−10 m (b) 0.51 × 10−10 m
PQ + QR
(c) 0.56 × 10−10 m (d) 0.69 × 10−10 m (c) (d) None of the above
3
4. The centre of mass of a system of particles does not depend
on 9. Four rods AB, BC, CD and DA have masses m, 2 m, 3 m and
(a) masses of the particles 4m, respectively. The centre of mass of all the four rods lies
(b) internal forces on the particles
D C
(c) position of the particles
(d) relative distance between the particles 1 2

5. All the particles of a body are situated at a distance R from O


the origin. The distance of centre of mass of the body from 3 4
the origin is A B
(a) = R (b) ≤ R (c) > R (d) ≥ R
(a) in region 1 (b) in region 2
6. Three point masses m1 , m2 and m3 are placed at the corners
(c) in region 3 (d) at O
of a thin massless rectangular sheet (1.2 m × 1 m) as shown.
Centre of mass will be located at the point 10. The linear density of a rod of length L varies as λ = A + Bx,
where x is the distance from one of its ends.
The position of centre of mass will be
m3 = 2.4 kg L  3A + BL  L  3A + 2BL 
C (a)   (b)  
2  2A + 2BL  3  2A + BL 
L 2A + 3BL  L 2A + 3BL 
1m (c)   (d)  
m1 = 1.6 kg
5  A + 3BL  3  3A + 2BL 
A 1.2 m B m2 = 2 kg
338 OBJECTIVE Physics Vol. 1

11. Three rods of the same mass are placed as shown in the 14. Four particles of masses m1 = 2m, m2 = 4 m, m3 = m and m4
figure. What will be the coordinate of centre of mass of the are placed at four corners of a square. What should be the
system? value of m4 , so that the centre of mass of all the four
Y
particles are exactly at the centre of the square?
m4 m3
(0, a)

O (a, 0) X m1 m2

(a) 2m (b) 8 m
(b) 
a a 
(a)  ,
a
(d)  ,
a 2a 2a a a
 ,  (c) ,  (c) 6m (d) None of these
2 2  2 2 3 3 3 3
12. Figure shows a composite system of two uniform rods of 15. A square plate of side 20 cm has uniform thickness and
lengths as indicated. Then, the coordinates of the centre of density. A circular part of diameter 8 cm is cut out
mass of the system of rods are symmetrically as shown in figure. The position of centre of
y mass of the remaining portion is

2L

O O1
x
O L

(a)  ,  (b)  ,
L 2L L 2L 

2 3 4 3 (a) at O1

(c)  ,  (d)  ,
L 2L L L (b) at O

6 3 6 3 (c) 0.54 cm from O on the left hand side
(d) None of the above
13. A uniform metal rod of length 1 m is bent at 90°, so as to
form two arms of equal length. The centre of mass of this 16. A uniform metal disc of radius R is taken and out of it a disc
bent rod is of diameter R is cut-off from the end. The centre of mass of
the remaining part will be
 1 
(a) on the bisector of the angle,   m from vertex R
 2 (a) from the centre
4
 1 
(b) on the bisector of angle,   m from vertex R
 2 2 (b) from the centre
3
on the bisector of the angle,   m from vertex
1
(c) R
 2 (c) from the centre
5
 1 
(d) on the bisector of the angle,   m from vertex R
 4 2 (d) from the centre
6

MOTION OF CENTRE OF MASS


Let us consider the motion of a system of n particles of Acceleration of centre of mass,
individual masses m1, m 2, ..., mn and total mass M. It is
assumed that no mass enters or leaves the system during m1a1 + m 2 a 2 + K + mn an
a CM = …(ii)
its motion, so that M remains constant. Then, M
velocity of centre of mass, n
m v + m 2 v 2 + .. + mn v n Σ m i ai
v CM = 1 1 …(i) i =1
M or aCM =
M
n
Σ mi v i Further, in accordance with Newton’s second law of
i =1
or v CM = motion, F = m a. Hence, Eq. (ii) can be written as
M
FCM = F1 + F2 + ... + Fn
Differentiating Eq. (i) w.r.t. t, we get
COM, Conservation of Momentum and Collision 339

Force on centre of mass, Example 8.14 Find the velocity of centre of mass of the
system shown in the figure.
n
FCM = Σ Fi …(iii) y
i =1 1 kg 2 ms−1

From expression (iii), it is clear that the centre of mass of x 2 kg


30°
a system of particles moves as though it is a particle
of mass equal to that of the whole system with all the 2 ms−1
external forces acting directly on it.
There are some important points related to motion of Sol. Here, m1 = 1 kg, v1 = 2i$,
centre of mass m 2 = 2 kg, v 2 = 2 cos 30° i$ − 2 sin 30° j$
(i) If a system consists of more than one particle (or m v + m 2v 2
v CM = 1 1
bodies) and net external force on the system in a m1 + m 2
particular direction is zero with centre of mass at 1 × 2i$ + 2(2 cos 30° i$ − 2 sin 30° j$ )
rest. Then, the centre of mass will not move along =
1+ 2
that particular direction even though some particles
(or bodies) of the system may move along that direction. 2i + 2 3i − 2j  2 + 2 3  $ 2 $
$ $ $
v CM = = i− j
(ii) Motion of centre of mass of a system or object is not 3  3  3
affected by any of internal forces as they always Example 8.15 Two particles of masses 2 kg and 4 kg are
make action-reaction pairs, so their net contribution approaching each other with acceleration 1 ms −2 and
to acceleration of centre of mass is zero. 2 ms −2 respectively, on a smooth horizontal surface. Find
(iii) If two particles of masses m1 and m 2 are placed on a the acceleration of centre of mass of the system.
smooth surface separated by distance r and they Sol. The acceleration of centre of mass of the system,
move towards each other due to the mutual m a + m 2a 2 2 × 1 − 4 × 2
attractive force, then a CM = 1 1 = = −1ms−2
m1 + m 2 (2 + 4)
(a) In the absence of any external force, the (Negative sign indicates that direction of 4 kg is opposite
acceleration of CM is zero, irrespective of the to that of 2 kg)
individual acceleration of particles. Since, |m 2a 2| > |m1a1|, so the direction of acceleration of centre
(b) In the absence of any external force, the of mass will be directed towards m1.
velocity of CM is also constant. Example 8.16 Two particles of masses m1 and m 2 are
(c) If initially the centre of mass is at rest, projected from the top of a tower. The particle m1 is
i.e. v CM = 0 and the external force is absent, projected vertically downward with speed u and m 2 is
i.e. Fext = 0, the location of CM is fixed. projected horizontally with same speed. Find acceleration of
(d) Under the action of external forces, the CM CM of system of particles by neglecting the effect of air
resistance.
moves just as all the mass were concentrated at
that point, Sol. As effect of air is neglected, therefore the only force acting
on the particles is the gravitational force in downward
i.e. ΣFext = Ma ext direction.
Example 8.13 Two blocks of masses 5 kg and 2 kg are Let the point of projection is taken as origin and downward
placed on a frictionless surface and connected by a spring. direction as negative Y-axis, then
An external kick gives a velocity of 14 ms −1 to the heavier acceleration of 1st point mass, a1 = − g $j
block in the direction of lighter one. Calculate the velocity
gained by the centre of mass. acceleration of 2nd point mass, a2 = − g $j

Sol. Given, m1 = 5 kg, m 2 = 2 kg, v1 = 14 ms−1 and v 2 = 0 ∴ aCM =


m1a1 + m 2a2 m1(− g $j) + m 2 (− g $j)
= = − g $j
m1v1 + m 2v 2 m1 + m 2 m1 + m 2
∴ v CM =
m1 + m 2 i.e. Acceleration of CM is equal to acceleration due to gravity
and is in downward direction.
5 × 14 + 2 × 0
= Note If large number of particles are projected under the effect of
5+2
gravity only in different directions, then acceleration of CM is
70
= = 10 ms −1 in the direction of lighter one. equal to the acceleration due to gravity irrespective of
7 directions of projection of particles.
340 OBJECTIVE Physics Vol. 1

Example 8.17 Two particles of masses 2m and 3m separated m AaA + m B aB


Now, aCM =
by distance d are placed on a smooth surface. They move m A + mB
towards each other due to mutual attractive force. Find (i)
2 (a ) − 1 (a ) a g
acceleration of CM, (ii) velocity of CM when separation = = = (downwards)
between particles becomes d /3 and (iii) at what distance 1+ 2 3 9
from the initial position of mass 2m will the particles collide?
Example 8.19 Two particles A and B of masses 1 kg and
Sol. The given situation is shown below. 2 kg respectively are projected in the directions as shown in
3m
figure with speeds u A = 200 ms −1 and uB = 50 ms −1.
2m F F
Initially, they were 90 m apart. Find the maximum height
d Smooth attained by the centre of mass of the particles. Assume
A B acceleration due to gravity to be constant.
(i) In the absence of any external force, the acceleration of (Take, g = 10 ms −2 )
CM is zero, B
i.e. Fext = 0 ⇒ a CM = 0
uB
(ii) Initially, the particles are at rest, i.e. v1 = v 2 = 0, therefore
90 m
v CM = 0. Since Fext = 0, the velocity of CM is constant and uA
hence, v CM is always zero whatever be the separation
between the particles.
A
(iii) The position of CM will be
2m × 0 + 3m × d 3d Sol. Using m ArA = m BrB or 1 (rA) = 2 (rB )
x CM = =
2m + 3m 5 or rA = 2rB …(i)
CM and rA + rB = 90 m …(ii)
A B
Solving these two equations, we get
3d/5 rA = 60 m and rB = 30 m
i.e. CM is at height 60 m from the ground at time t = 0.
Since, the CM is at rest and its position is fixed, hence
m a + m B aB
particles will meet at CM, i.e. at distance 3d / 5 from A. Further, aCM = A A = g = 10 ms −2 (downwards)
m A + mB
Example 8.18 In the arrangement shown in figure,
As, aA = aB = g (downwards)
m A = 2 kg and m B = 1 kg. String is light and inextensible.
m u + m B uB
Find the acceleration of centre of mass of both the blocks. u CM = A A
Neglect friction everywhere. m A + mB
1 (200) − 2 (50) 100
= = ms −1 (upwards)
1+ 2 3
Let h be the height attained by CM beyond 60 m.
Using v 2CM = uCM
2
+ 2aCMh
A 2
100
or 0=  − (2) (10) h
B  3 
(100)2
Sol. Net pulling force on the system = (m A − m B ) g or h= = 55.56 m
180
= (2 − 1) g = g
Therefore, maximum height attained by the centre of mass,
H = 60 + 55.56
= 115.56 m

a A
Linear momentum of a
B a
system of particles
For a system of n-particles, total linear momentum is
Total mass being pulled = m A + m B = 3 kg vector sum of linear momenta of individual particles,
Net pulling force g where linear momentum of an individual particle is
∴ a= = product of its mass and velocity (p = mv ).
Total mass 3
COM, Conservation of Momentum and Collision 341

So, linear momentum of system is given by Here, rate of change of momentum is zero, i.e. momentum
of system remains constant.
p = p1 + p 2 + p 3 +… + pn
So, p initial = p final
or p = m1v 1 + m 2 v 2 + m 3 v 3 + ⋅ ⋅ ⋅ +mn v n …(i) Above expression represents the law of conservation of
(Q p = mv ) linear momentum for system of particles.
From the concept of centre of mass, we know that, Example 8.20 A man of mass m1 is standing on a platform of
m1v 1 + m 2 v 2 + m 3 v 3 + ⋅ ⋅ ⋅ + mn v n = M v CM …(ii) mass m 2 kept on a smooth horizontal surface. The man
starts moving on the platform with a velocity v r relative to
From Eqs. (i) and (ii), we get the platform. Find the recoil velocity of platform.
Total linear momentum, p = M v CM …(iii) Sol. Absolute velocity of man = vr − v , where v = recoil velocity
of platform. Taking the platform and the man as a system, net
Thus, the total momentum of a system of particles is external force on the system in horizontal direction is zero.
equal to the product of the total mass and velocity of Initially, both the man and the platform were at rest, hence
its centre of mass. the linear momentum of the system remains constant.
Note Relation between momentum and kinetic energy,
p = 2 mK vr − v

v
Conservation of linear momentum for
system of particles Hence, 0 = m1(vr − v ) − m 2v ⇒ v =
m1vr
According to law of conservation of linear momentum, m1 + m 2
total linear momentum of a system of particles remains
Example 8.21 A wooden plank of mass 20 kg is resting on a
constant or conserved in the absence of any external force. smooth horizontal floor as shown in figure. A man of mass
Total linear momentum of system of particles, 60 kg starts moving from one end of the plank to the other
p = Mv CM end. The length of the plank is 10 m. Find the displacement
of the plank over the floor when the man reaches the other
Differentiating both sides w.r.t. t, we get end of the plank.
dp d
= (Mv CM )
dt dt
dv
= M CM
dt
dv CM
Here, = a CM, acceleration of centre of mass. 10 m
dt
Sol. Here, the system is man + plank. Net force on this system
dp
∴ = Ma CM in horizontal direction is zero and initially the centre of mass
dt of the system is at rest. Therefore, the centre of mass does not
Here, from Newton’s second law of motion, Ma CM will be move in horizontal direction.
equal to the external force. Let x be the displacement of the plank. Assuming the origin,
i.e. x = 0 at the position as shown in figure.
dp
∴ = Fext
dt
∴ If Fext = 0, then
dp
= 0 or p = constant 10
CM
dt 2 x
∴ M v CM = constant x=0 10 m Initial position

v CM = constant
So, we can conclude that, if net external force on the
system is zero, the linear momentum of the system is
constant, hence centre of mass will move with
x 10 − x Final position
constant velocity.
342 OBJECTIVE Physics Vol. 1

As, the centre of mass will not move in horizontal direction On solving Eqs. (i) and (ii), we get
(X-axis). Therefore, for centre of mass to remain stationary,
2Mg (R − r ) m 2Mg (R − r )
xi = x f v1 = and v 2 =
M +m M M +m
10 10 
60 (0) + 20   60 (10 − x ) + 20  − x
2 2 
= Example 8.23 A disc of mass 100 g is kept floating
60 + 20 60 + 20 horizontally in air by firing bullets, each of mass 5 g with the
10  same velocity at the same rate of 10 bullets per second. The
6(10 − x ) + 2  − x bullets rebound with the same speed in opposite direction.
5  2 
or = Find the velocity of each bullet at the time of impact.
4 8
Sol. From the law of conservation of momentum,
60 − 6x + 10 − 2x
= rate of change in momentum of bullets = weight of disc
8
2m ′vn = mg
or 5 = 30 − 3x + 5 − x
mg 100 × 980
⇒ 4x = 30 ∴ v= =
2m ′n 2 × 5 × 10
30
⇒ x= m = 980 cm/s
4
⇒ x = 7.5 m Example 8.24 A plank of mass 5 kg is placed on a frictionless
Note The centre of mass of the plank lies at its centre. horizontal plane as shown in figure. Further, a block of mass
1 kg is placed over the plank. A massless spring of natural
Example 8.22 A block of mass M with a semi-circular track length 2 m is fixed to the plank by its one end. The other
of radius R rests on a smooth floor. A sphere of mass m and end of spring is compressed by the block by half of spring’s
radius r is released from rest at point A. Find the velocity of natural length. The system is now released from the rest.
sphere and track when the sphere reaches at B. What is the velocity of the plank when block leaves the
plank ? (The stiffness constant of spring is 100 Nm −1)
m O
A r 1 kg
R
5 kg
M B 4m
Smooth
Sol. Let the velocities of the block and the plank, when the
block leaves the spring, be u and v, respectively.
Sol. According to the question,
1 1 1
By conservation of energy, kx 2 = mu 2 + Mv 2
A O 2 2 2
(where, M = mass of the plank, m = mass of the block)
R–r
v2 ⇒ 100 = u 2 + 5v 2 K (i)
v1
B By conservation of momentum,
mu + Mv = 0 ⇒ u = − 5v K (ii)
When the sphere reaches at point B Solving Eqs. (i) and (ii), we get
Let v1 = velocity of m 30 v 2 = 100
v 2 = velocity of M 10
Using conservation of momentum in the horizontal direction, ⇒ v= ms −1
3
mv1 = Mv 2 …(i)
From this moment until block falls, both plank and block keep
Applying the conservation of energy between A and B, their velocity constant.
1
mg (R − r ) = (mv12 + Mv 22 ) …(ii)
2 Thus, when block falls, velocity of plank = 10 /3 ms −1.
CHECK POINT 8.2
1. A body falling vertically downwards under gravity breaks in 2 ms−1 2 ms−1
two parts of unequal masses. The centre of mass of the two
4 kg 4 kg
parts taken together shifts horizontally towards
(a) heavier piece Origin (4.5 m, 0)
(b) lighter piece
At t = 0 , the position of blocks are shown, then the
(c) does not shift horizontally coordinates of centre of mass t = 3 s will be
(d) depends on the vertical velocity at the time of breaking (a) (1, 0) (b) (3, 0) (c) (5, 0) (d) (2.25, 0)
2. Two balls are thrown simultaneously in air. The 10. Two particles of equal mass have coordinates (4 m, 4m, 6m)
acceleration of the centre of mass of the two balls in air and (6m, 2m, 8m). Of these, one particle has a velocity
(a) depends on the direction of the motion of the balls
v1 = (2 $i) ms −1 and another particle has velocity v 2 = (2$j) ms −1
(b) depends on the masses of the two balls
at time t = 0. The coordinates of their centre of mass at time
(c) depends on the speeds of two balls
t = 1 s will be
(d) is equal to g
(a) (4m, 4m, 7m) (b) (5m, 4m, 7m)
3. Consider a system of two identical particles. One of the (c) (2m, 4m, 6m) (d) (4m, 5m, 4m)
particles is at rest and the other has an acceleration a. The
centre of mass has an acceleration 11. An isolated particle of mass m is moving in horizontal
1 xy-plane, along the X-axis, at a certain height above the
(a) zero (b) a (c) a (d) 2a ground. It suddenly explodes into two fragments of masses
2 m 3m
and . An instant later, the smaller fragment is at
4. Two balls of equal mass are projected from a tower 4 4
simultaneously with equal speeds, one at angle θ above the y = + 15 cm. The larger fragment at this instant is at
horizontal and the other at the same angle θ below the (a) y = − 5cm (b) y = + 20 cm
horizontal. The path of the centre of mass of the two balls is (c) y = + 5cm (d) y = − 20 cm
(a) a vertical straight line
(b) a horizontal straight line 12. Two particles A and B initially at rest, move towards each
(c) a straight line at angle α (< θ) with horizontal
other under a mutual force of attraction. At the instant,
when the speed of A is v and the speed of B is 2v, the speed
(d) a parabola
of centre of mass of the system is
5. A ball kept in a closed box moves in the box making (a) zero (b) v (c) 1.5 v (d) 3v
collisions with the walls. The box is kept on a smooth
surface. The velocity of the centre of mass 13. A man of mass m is standing on a plank of equal mass m
resting on a smooth horizontal surface. The man starts
(a) of the box remains constant
moving on the plank with speed u relative to the plank.
(b) of the box plus the ball system remains constant
The speed of the man relative to the ground is
(c) of the ball remains constant
m
(d) of the ball relative to the box remains constant
6. Two blocks of masses 10 kg and 4 kg are connected by a
spring of negligible mass and placed on a frictionless
horizontal surface. An impulse gives a velocity of 14 ms −1 to
the heavier block in the direction of the
u u
lighter block. The velocity of the centre of mass is (a) 2u (b) (c) zero (d)
−1 −1 2 4
(a) 30 ms (b) 20 ms
(c) 10 ms −1 (d) 5 ms −1 14. A shell of mass m is moving horizontally with velocity v 0
7. A metre stick is placed vertically at the origin on a and collides with the wedge of mass M just above point A as
frictionless surface. A gentle push in + x-direction is given to shown in the figure. As a consequence, wedge starts to
the top most point of the rod. When it has fallen move towards left and the shell returns with a velocity in
completely, the x-coordinate of centre of rod is at xy-plane. The principle of conservation of momentum can
(a) origin (b) − 0.5 m (c) −1 m (d) + 0.5 m be applied for C
8. Two bodies having masses m1 and m2 and velocities v1 and
v 2 collide and form a composite system. If
m1 v1 + m2v 2 = 0 (m1 ≠ m2), the velocity of composite system
M
will be
(a) v1 − v2 (b) v1 + v2 v0 m
v + v2 θ
(c) 1 (d) zero
2 B A
9. Blocks A and B are resting on a smooth horizontal surface (a) system (m + M) along any direction
given equal speeds of 2 ms −1 in opposite sense as shown in (b) system (m + M) vertically
the figure. (c) system (m + M) horizontally
(d) None of the above
344 OBJECTIVE Physics Vol. 1

15. A stationary bomb explodes into two parts of masses 3 kg 17. A shell is fired from cannon with velocity v at an angle θ
and 1 kg. The total kinetic energy of the two parts after with the horizontal direction. At the highest point in its
explosion is 2400 J. The kinetic energy of the smaller part is
path, it explodes into two pieces of equal mass. One of the
(a) 600 J (b) 1800 J (c) 1200 J (d) 2160 J
pieces retraces its path to the cannon and the speed (in
16. An object of mass 3m splits into three equal fragments. Two ms −1) of the other piece immediately after the explosion is
fragments have velocities v $j and v $i. The velocity of the (a) 3v cos θ (b) 2v cos θ
third fragment is 3 3
(c) v cos θ (d) v cos θ
v($i + $j)
(a) v($j − $i) (b) v($i − $j) (c) − v($i + $j) (d) 2 2
2

COLLISION
A collision is an isolated event in which two or more Types of collisions
colliding bodies exert strong forces on each other for a Collision between two bodies may be classified in two ways
relatively short time. For a collision to take place, the
Elastic and inelastic collision A collision is said to be
actual physical contact is not necessary.
elastic, if along with linear momentum, kinetic energy
Total linear momentum is conserved in all collisions, also remains conserved before and after collision.
i.e. the initial momentum of the system is equal to final
A collision is said to be inelastic, if only linear momentum
momentum of the system.
remains conserved but not the kinetic energy.
∴ Total momentum before collision = Total momentum after
The collision is said to be perfectly inelastic, if
collision
approaching particles permanently stick to each other and
m1v 1 + m 2v 2 = m1v 1′ + m 2v 2′
move with common velocity.
In the absence of any dissipative forces, the mechanical Head on and oblique collision If velocity vectors of the
energy of the system will also remain conserved, i.e. colliding bodies are directed along the line of impact, the
impact is called as direct or head on collision.
1 1 1 1
m1v 12 + m 2v 22 = m1v 1′2 + m 2v 2′ 2 And if velocity vectors of both or any of the bodies are not
2 2 2 2 along the line of impact, the impact is called oblique
collision.
Example 8.25 Two blocks A and B of equal mass m =1 kg
are lying on a smooth horizontal surface as shown in figure.
A spring of force constant k = 200 Nm −1 is fixed at one end
Head on elastic collision
of block A. Block B collides with block A with velocity Let the two balls of masses m1 and m 2 collide with each
v 0 = 2 ms −1. Find the maximum compression of the spring. other elastically with velocities v 1 and v 2 in the directions
2 ms−1 shown in figure below. Their velocities become v 1′ and v 2′
after the collision along the same line.
B A
m2 m1
v2 v1
Sol. At maximum compression (x m ), velocity of both the blocks
is same, say v. Applying conservation of linear momentum,
we have (a) Before collision
(m A + m B ) v = m Bv 0 m2 m1
v'2 v1'
or (1 + 1) v = (1) v 0
v0 2
or = = 1 ms −1
v= (b) After collision
2 2
Fig. 8.3 Head on elastic collision
Using conservation of mechanical energy, we have
1 1 1 Applying law of conservation of linear momentum, we get
m Bv 02 = (m A + m B ) v 2 + kx m2
2 2 2 m1v 1 + m 2v 2 = m1v 1′ + m 2v 2′ …(i)
Substituting the given values in above equation, we get In an elastic collision, kinetic energy before and after
1 1 1 collision is also conserved. Hence,
× (1) × (2)2 = × (1 + 1) × (1)2 + × (200) × x m2
2 2 2
1 1 1 1
or x m = 0.1 m = 10 cm m v 2 + m v 2 = m v ′2 + m 2v 2′ 2 …(ii)
2 1 1 2 2 2 2 1 1 2
COM, Conservation of Momentum and Collision 345

Solving Eqs. (i) and (ii) for v 1′ and v 2′ , we get i.e. The second particle (of mass 2m) comes to rest while the
first (of mass m) moves with velocity 3v in the direction
 m − m2   2m 2  shown in figure given above.
v 1′ =  1  v1 +   v2 …(iii)
 m1 + m 2   m1 + m 2  Head on inelastic collision
 m − m1   2m1  In an inelastic collision, due to permanent deformation
and v 2′ =  2  v2 +   v1 …(iv)
 m1 + m 2   m1 + m 2  the kinetic energy of the particles no longer remains
conserved. However, in the absence of external forces, law
Special cases of head on elastic collision of conservation of linear momentum still holds good.
(i) If m1 = m 2, then from Eqs. (iii) and (iv), we can see that
v2 v1
v 1′ = v 2 and v 2′ = v 1 m2 m1

i.e. When two particles of equal mass collide Before collision


elastically in head on condition, they exchange their v'2 v'1
m2 m1
velocities.
+ ve
m
(ii) If m1 > > m 2 and v 1 = 0, then 2 ≈ 0 After collision
m1 Fig. 8.4 Head on inelastic collision
 m 
With these two substitutions  v 1 = 0 and 2 = 0  , Suppose the velocities of two particles of masses m1 and
 m 1 
m 2 before collision be v 1 and v 2 in the directions as shown
we get the following two results, in figure. Let v 1′ and v 2′ be their velocities after
v 1′ ≈ 0 and v 2′ ≈ − v 2 collision. The law of conservation of linear momentum
i.e. If a lighter particle collides with a heavier gives
particle at rest, then heavier particle remains at rest m1v 1 + m 2v 2 = m1v 1′ + m 2v 2′ …(i)
but lighter particle bounces back with same velocity.
v'
(iii) If m 2 > > m1 and v 1 = 0
With the substitution m1 /m 2 ≈ 0 and v 1 = 0, we get
the results, v 1′ ≈ 2v 2 and v 2′ ≈ v 2
i.e. If a heavier particle collides with a lighter Fig. 8.5 After inelastic collision
particle at rest, then lighter particle moves with
twice the velocity of heavier particle while velocity Collision is said to be perfectly inelastic, if both the
of heavier particle remains same. particles stick together after collision and move with same
Example 8.26 Two particles of masses m and 2m moving in velocity, say v ′ as shown in figure. In this case, Eq. (i) can be
opposite directions collide elastically with velocities v and 2v. written as
Find their velocities after collision. m1v 1 + m 2v 2 = (m1 + m 2 )v ′
Sol. Here, v1 = − v, v 2 = 2v, m1 = m and m 2 = 2m . m v + m 2v 2
or v′ = 1 1 …(ii)
2v m1 + m 2
2m v
m +ve
Loss in kinetic energy,
1 1  1
Substituting these values in Eqs. (iii) and (iv), we get ∆K =  m1v 12 + m 2v 22  − (m1 + m 2 ) v ′ 2
 m − 2m   4m  2 2  2
v1′ =   (− v ) +   (2v )
 m + 2m   m + 2m   m v + m 2v 2 
2
1 1  1
=  m1v 12 + m 2v 22  − (m1 + m 2 )  1 1 
or v1′ =
v 8v
+ = 3v 2 2  2  m1 + m 2 
3 3
Solving above equation, we get
 2m − m   2m 
and v 2′ =   (2v ) +   (− v )
 2m + m   m + 2m  1  m1m 2 
∆K =   (v − v 2 )
2

or v 2′ =
2 2
v− v=0
2  m1 + m 2  1
3 3
which is positive. Therefore, some kinetic energy is
2m v2′ = 0 m
3v
always lost in an inelastic collision.
346 OBJECTIVE Physics Vol. 1

Example 8.27 A simple pendulum of length 1 m has a 10−2 × 1 + 10−3 × 0 = (10−2 + 10−3 ) v
wooden bob of mass 1 kg. It is struck by a bullet of mass 10−2 10
10 −2 kg moving with a speed of 2 × 10 2 ms −1. The bullet or v= = ms −1
1.1 × 10−2 11
gets embedded into the bob. Obtain the height to which the
bob rises before swinging back. v 2 (10/11)2
Now, h= = = 4.1 × 10−2 m
Sol. Applying principle of conservation of linear momentum, 2g 2 ×10
mu = (M + m ) v ⇒ 10−2 × (2 × 102 ) = (1 + 0.01) v Example 8.30 An object of mass 40 kg having velocity
⇒ v=
2
ms −1
4 ms −1 collides with another object of mass 60 kg having
1.01 velocity 2 ms −1. What is the loss of energy during this
process, if it is a perfectly inelastic collision?
Sol. Given, m1 = 40 kg, m 2 = 60 kg, v1 = 4 ms −1, v 2 = 2 ms −1
1 m m  1  40 × 60
∴ ∆K =  1 2  (v1 − v 2 )2 =   (4 − 2) = 48 J
2
2  m1 + m 2  2  40 + 60
( M + m)

v
h Example 8.31 A particle of mass m moving with speed u
m collides perfectly inelastically with another particle of mass
M
u 2m at rest. Find loss of kinetic energy of system in the
Kinetic energy of the block with bullet in it is converted into collision.
potential energy as it rises through a height h.
Sol. Let velocity of system (m + 2m ) of particles after collision be v.
1
∴ (M + m ) v 2 = (M + m ) gh u
2 From law of conservation of momentum, mu = 3mv ⇒ v =
2 3
v2  2  1 Now, loss of kinetic energy = Ki − K f
⇒ h= =  × = 0.2 m
2g 1.01 2 × 9.8 2
1 1  u
⇒ ∆K = mu 2 − (3m )  
Example 8.28 A body falling on the ground from a height of 2 2  3
10 m, rebounds to a height 2.5 m, calculate 1 1
⇒ ∆K = mu 2 − mu 2
(i) the percentage loss in kinetic energy 2 6
(ii) ratio of the velocities of the body just before and just after 1
⇒ ∆K = mu 2
the collision. 3
Sol. Let v1 and v 2 be the velocities of the body just before and
Example 8.32 A railway carriage of mass 8000 kg moving
just after the collision. with on speed of 54 km h −1 collides with an another
1
KE1 = mv12 = mgh1 …(i) stationary carriage of same mass. Determine the loss in
2 kinetic energy in this process.
1
and KE 2 = mv 22 = mgh 2 …(ii) Sol. Given, m1 = 8000 kg, v1 = 54 km h−1
2
5
mg (h1 − h 2 ) = 54 × ms−1 = 15 ms−1
(i) Percentage loss in kinetic energy = × 100 18
mgh1
10 − 2.5 m 2 = 8000 kg, v 2 = 0
= × 100 = 75% From law of conservation of momentum,
10
m1v1 + m 2v 2 = (m1 + m 2 )v
v12 h1 10 v
(ii) = = = 4 ⇒ 1 =2 m1v1
v 22 h 2 2.5 v2 v= (Q v 2 = 0)
m1 + m 2
Example 8.29 A pendulum bob of mass 10 −2 kg is raised to a 8000 × 15
height 5 × 10 −2 m and then released. At the bottom of its = = 7.5 ms −1
8000 + 8000
swing, it picks up a mass 10 −3 kg. To what height will the
Loss of kinetic energy = KE before collision − KE after collision
combined mass rise? (Take, g = 10 ms −2 )
1 1
Sol. Velocity of pendulum bob in mean position, = m1v12 − (m1 + m 2 ) v 2
2 2
v1 = 2gh = 2 × 10 × 5 × 10−2 = 1 ms −1 1 1
= × 8000 × (15)2 − (8000 + 8000)(7.5)2
When the bob picks up a mass 10−3 kg at the bottom, then by 2 2
conservation of linear momentum, the velocity of coalesced = 400 × 225 − 8000 × 56.25
mass is given by
= 450 kJ
m1v1 + m 2v 2 = (m1 + m 2) v
COM, Conservation of Momentum and Collision 347

Newton’s law of restitution Example 8.33 A ball is moving with velocity 2 ms −1 towards
a heavy wall moving towards the ball with speed 1 ms −1 as
When two objects are in direct (head on) impact, the speed shown in figure. Assuming collision to be elastic, find the
with which they separate after impact is usually less than velocity of ball immediately after the collision.
or equal to their speed of approach before impact.
According to Newton’s law of restitution, the ratio of
2 ms−1 1 ms−1
relative velocity of separation after collision to relative
velocity of approach before collision remains constant.
Relative velocity of separation (after collision)
e=
Relative velocity of approach (before collision) Sol. The speed of wall will not change after the collision. So, let
v be the velocity of the ball after collision in the direction as
The ratio e is called the coefficient of restitution and is shown in figure. Since, collision is elastic (e = 1),
constant for two particular objects.
v − v1
e= 2
u1 − u 2 2 ms−1 1 ms−1 v 1 ms−1

where, u1 and u 2 are velocities of two bodies before


collision and v 1 and v 2 are their velocities after collision.
For elastic collision, e = 1and for inelastic collision, Before collision After collision
0 < e < 1 while for perfectly inelastic collision, e = 0. separation speed = approach speed
Solving the following equations, or v − 1 = 2 + 1 or v = 4 ms−1
m1u1 + m 2u 2 = m1v 1 + m 2v 2 and v 2 − v 1 = e (u1 − u 2 ),
we get
m − em 2 (1 + e )m 2 Example 8.34 A ball of mass 2 kg moving with speed 5 ms −1
v1 = 1 u1 + u2 collides directly with another ball of mass 3 kg moving in the
m1 + m 2 m1 + m 2 same direction with speed 4 ms −1. The coefficient of
m 2 − em1 (1 + e )m1 restitution is 2/3. Find their velocities after collision.
and v2 = u2 + u1 Sol. Denoting the first ball by A and the second ball by B,
m1 + m 2 m1 + m 2
velocities immediately before and after the impact are shown
Putting e = 1, we will get formulae of v 1 and v 2 for an in the figure.
uA = 5 ms−1 uB = 4 ms−1 vA vB
elastic collision.
Putting e = 0, we will get formulae of v 1 and v 2 for A B A B
perfectly inelastic collision. Immediately before Immediately after
impact starts impact ends
The loss in kinetic energy during an inelastic collision is
Applying principle of conservation of momentum, we have
1 m1m 2 m BvB + m Av A = m AuA + m BuB
∆E = (1 − e 2 ) (u1 − u 2 ) 2
2 m1 + m 2 ⇒ 3 vB + 2 v A = 2 × 5 + 3 × 4
However, if the target is massive (i.e. m 2 >> m1) and 3 vB + 2 v A = 22 …(i)
u 2 = 0, then the lighter body loses all its kinetic energy. Applying equation of coefficient of restitution, we have
Note 2
vB − v A = e (uA − uB ) ⇒ vB − v A = (5 − 4)
In the situation shown in figure, if e is the u=0 3
coefficient of restitution between the ball 3vB − 3v A = 2 …(ii)
and the ground, then after nth collision with h
the floor, the speed of ball will remain e nv 0 Solving Eqs. (i) and (ii), we get
and it will go upto a height e 2nh, i.e. v A = 4 ms−1 and vB = 4.67 ms−1
vn = e v0 = e
n n
2 gh
and hn = e 2nh v 0 = √2gh Example 8.35 A block of mass 5 kg moves from left to right
Total distance travelled by the ball before it with a velocity of 2 ms −1 and collides with another block of
Fig. 8.6
stops bouncing is mass 3 kg moving along the same line in the opposite
H = h
1 + e 2 
and total time taken by the
direction with velocity 4 ms −1.
2
1 − e  (i) If the collision is perfectly elastic, determine velocities of both
ball to stop bouncing will be the blocks after their collision.
1 + e  2h
T =   (ii) If coefficient of restitution is 0.6, determine velocities of both
1− e  g the blocks after their collision.
348 OBJECTIVE Physics Vol. 1

Sol. Denoting the first block by A and the second block by B, Sol. Let v1 and v 2 be the final velocities of 1st and 2nd object,
velocities immediately before and after the impact are shown respectively.
in the figure.
u v1 v2
A A m m m m
B vA B vB
uA = 2 ms–1 uB = 4 ms–1
Rest
Before collision After collision
Immediately before Immediately after
impact starts impact ends From law of conservation of momentum,
Applying principle of conservation of momentum, we have mu + m × 0 = mv1 + mv 2
m BvB + m Av A = m AuA + m BuB ⇒ v1 + v 2 = u …(i)
v − v1
⇒ 3 vB + 5 v A = 5 × 2 + 3 × (−4) Now, e= 2
3 vB + 5 v A = − 2 …(i) u1 − u2
Applying equation of coefficient of restitution, we have v 2 − v1
⇒ =e
v A − vB = e (uA − uB ) u
⇒ vB − v A = e {2 − (−4)} ⇒ v 2 − v1 = eu …(ii)
vB − v A = 6 e …(ii) Adding Eqs. (i) and (ii), we get
(i) For perfectly elastic impact, e = 1.Using this value in Eq. (ii), 2v 2 = u + eu
we get 1 + e 
⇒ v2 =  u
vB − v A = 6 …(iii)  2 
Now, solving Eqs. (i) and (iii), we obtain Subtracting Eq. (ii) from Eq. (i), we get
v A = − 2.5 ms−1 and vB = 3.5 ms−1 2v1 = (1 − e ) u
(ii) For e = 0.6, Eq. (ii) is modified as 1 − e 
⇒ v1 =  u
vB − v A = 3.6 …(iv)  2 
Example 8.38 Three identical balls, ball I, ball II and ball III
Now, solving Eqs. (i) and (iv), we obtain are placed on a smooth floor in a straight line at the
v A = − 1.6 ms−1 and vB = 2 ms−1 separation of 10 m between balls as shown in figure.
Block A reverses back with speed 1.6 ms −1 and B also moves Initially balls are stationary.
in opposite direction to its original direction with speed 2 ms Ball I is given velocity of 10 ms −1 towards ball II, collision
−1 between balls I and II is inelastic with coefficient of
.
restitution 0.5 but collision between balls II and III is
Example 8.36 A particle of mass 2 kg moving with a perfectly elastic.
velocity 5 $i ms −1 collides head on with another particle of What is the time interval between two consecutive collisions
mass 3 kg moving with a velocity − 2 $i ms −1. After the between balls I and II ?
collision, the first particle has speed of 1.6 ms −1 in negative
x-direction. Find I II III
(i) velocity of the centre of mass after the collision,
(ii) velocity of the second particle after the collision,
10 m 10 m
(iii) coefficient of restitution.
m u + m 2u 2 Sol. Let velocity of 1st ball and 2nd ball after collision be v1 and
Sol. (i) vc = 1 1 = 0.8 $i ms −1
m1 + m 2 v 2.
v 2 − v1 = 0.5 × 10 = 5 K (i)
(Velocity of CM before and after collision will be same)
mv 2 + mv1 = m × 10 K (ii)
(ii) v1 = − 1.6 $i ms −1 ⇒ v 2 + v1 = 10
Using law of conservation of momentum, Solving Eqs. (i) and (ii), we get
m1u1 + m 2u2 = m1v1 + m 2v 2 ⇒ v 2 = 2.4 $i ms −1 v1 = 2.5 ms −1 and v 2 = 7.5 ms −1
v 2 − v1 4 Ball II after moving 10 m collides with ball III elastically and
(iii) e = =
u1 − u 2 7 stops. But ball I moves towards ball II. Time taken between
two consecutive collisions,
Example 8.37 An object of mass m moving with speed u 2.5
collides one dimensionally with another identical object at 10 − 10 ×
10 7.5 = 4 s
rest. Find their velocities after collision, if coefficient of t= +
7.5 2.5
restitution of collision is e.
CHECK POINT 8.3
1. In an elastic collision, (a) 0.12 m (b) 1.5 m
(a) both momentum and KE are conserved (c) 0.5 m (d) 0.15 m
(b) only momentum is conserved 9. A smooth sphere of mass M moving with velocity u directly
(c) only KE is conserved collides elastically with another sphere of mass m at rest.
(d) Neither KE nor momentum is conserved After collision, their final velocities are v′ and v, respectively.
2. A ball hits the floor and rebounds after an inelastic collision. The value of v is
In this case, 2u M 2um 2u 2u
(a) (b) (c) (d)
m M m M
(a) the momentum of the ball just after the collision is the 1+ 1+
M m
same as that just before the collision
10. A body of mass m1 moving with velocity 3 ms −1 collides with
(b) the mechanical energy of the ball remains the same in the
another body at rest of mass m2. After collision, the velocities
collision
of the two bodies are 2 ms −1 and 5 ms −1 respectively along the
(c) the total momentum of the ball and the earth is conserved m
direction of motion of m2. The ratio 1 is
(d) the total energy of the ball and the earth is conserved m2
3. If a body of mass m collides head on, elastically with 5 1 12
(a) (b) 5 (c) (d)
velocity u with another identical body at rest. After 12 5 5
collision, velocity of the second body will be
11. A body of mass m moving with velocity v collides head on
(a) zero (b) u with another body of mass 2m which is initially at rest. The
(c) 2u (d) data insufficient ratio of KE of colliding body before and after collision will
4. A body of mass M1 collides elastically with another body of be
(a) 1 : 1 (b) 2 : 1 (c) 4 : 1 (d) 9 : 1
mass M2 at rest. There is maximum transfer of energy when
(a) M1 > M 2 (b) M1 < M 2 12. The two diagrams show the situations before and after a
collision between two spheres A and B of equal radii moving
(c) M1 = M 2 along the same straight line on a smooth horizontal surface.
(d) same for all values of M1 and M 2 The coefficient of restitution e is
5. Two particles of masses m A and mB and velocities v A and v B Before collision After collision
respectively collides. After collision, they interchanges their A B A B
m
velocities, then ratio of A is
mB
vA vB v A + vB
(a) (b) (c) (d) 1
vB vA vB − v A 8 ms−1 2 ms−1 2 ms−1 5 ms−1
1 1 2 3
6. Two perfectly elastic particles A and B of equal mass are (a) (b) (c) (d)
−1 3 2 3 4
travelling along the line joining them with velocities 15 ms
and 10 ms −1 . After collision, the respective velocities of A 13. Two balls of equal masses have a head on collision with
and B will be speed 6 ms −1 each. If the coefficient of restitution is 1/3, the
−1 −1 −1 −1 relative speed of separation of balls after impact will be
(a) 10 ms , 10 ms (b) 15 ms , 15 ms
−1
(c) 10 ms , 15 ms −1
(d) 15 ms −1 , 10 ms −1 (a) 18 ms −1 (b) 4 ms −1
(c) 6 ms −1 (d) data insufficient
7. The collision of two balls of equal mass takes place at the
origin of coordinates. Before collision, the components of 14. A block of mass m moving at a velocity v collides with
velocities are (v x = 50 cms −1 , v y = 0) and (v x = − 40 cms −1 another block of mass 2m at rest. The lighter block comes to
−1
rest after collision. Find the coefficient of restitution.
and v y = 30 cms ). The first ball comes to rest after 1 1 1
(a) (b) 1 (c) (d)
collision. The velocity components v x and v y respectively of 2 3 4
the second ball are
15. A sphere of mass m moving with a constant velocity u hits
(a) 10 and 30 cms −1 (b) 30 and 10 cms −1 another stationary sphere of same mass. If e is the
(c) 5 and 15 cms −1 (d) 15 and 5 cms −1 coefficient of restitution, then ratio of velocities of the two
v
8. A mass of 0.5 kg moving with a speed of 1.5 ms −1 on a spheres 1 after collision will be
horizontal smooth surface, collides with a nearly weightless v2
spring of force constant k = 50 Nm −1 . The maximum 2
u
1
compression of the spring would be
1−e 1+ e e +1 e −1
(a) (b) (c) (d)
1+ e 1−e e −1 e +1
Chapter Exercises
(A) Taking it together
Assorted questions of the chapter for advanced level practice

1 If the net external forces acting on the system of 7 A machine gun fires a steady stream of bullets at the
particles is zero, then which of the following may rate of n per minute into a stationary target in which
vary? the bullets get embedded. If each bullet has a mass m
(a) Momentum of the system and arrives at the target with a velocity v, the
(b) Velocity of centre of mass average force on the target is
(c) Position of centre of mass 60 v mnv mv
(a) 60 mnv (b) (c) (d)
(d) None of the above mn 60 60n
2 For which of the following does the centre of mass 8 A machine gun fires a bullet of mass 40 g with a
lie outside the body? [NCERT Exemplar] velocity 1200 ms −1 . The man holding it, can exert a
(a) A pencil (b) A shotput (c) A dice (d) A bangle
maximum force of 144 N on the gun. How many
3 Conservation of momentum in a collision between bullets can be fired per second at the most?
particles can be understood from [NCERT Exemplar] (a) One (b) Four (c) Two (d) Three
(a) Conservation of energy
9 A particle of mass m moving with speed v hits
(b) Newton’s first law
elastically another stationary particle of mass 2m
(c) Newton’s second law
inside a smooth horizontal circular tube of radius r.
(d) Both Newton’s second and third laws
The time after which the second collision will take
4 A body of mass a moving with velocity b strikes a place is
body of mass c and gets embedded into it. The 2πr 4πr 3πr πr
velocity of the system after collision is (a) (b) (c) (d)
v v 2v v
a +c ab
(a) (b)
ab a +c 10 A bullet of mass 20 g moving with 600 ms −1 collides
a a with a block of mass 4 kg hanging with the string of
(c) (d) length 0.4 m. What is velocity of bullet when it
b +c a +b
comes out of block, if block rises to height 0.2 m
5 A cannon ball is fired with a velocity 200 ms −1 at an after collision? (Take, g = 10 ms −2 )
angle of 60° with the horizontal. At the highest
(a) 200 ms −1 (b) 150 ms −1 (c) 400 ms −1 (d) 300 ms −1
point of its flight, it explodes into 3 equal fragments,
one going vertically upwards with a velocity 11 A mass of 10 g moving horizontally with a velocity
100 ms −1, the second one falling vertically of 100 cms −1 strikes a pendulum bob of mass 10 g.
downwards with a velocity 100 ms −1. The third Length of string is 50 cm. The two masses stick
fragment will be moving with a velocity together. The maximum height reached by the system
(a) 100 ms −1 in the horizontal direction now is (Take, g = 10 ms −2 )
(b) 300 ms −1 in the horizontal direction (a) 7.5 cm (b) 5 cm
(c) 300 ms −1 in a direction making an angle of 60° with (c) 2.5 cm (d) 1.25 cm
the horizontal
12 In a gravity free space, a man of mass M standing at
(d) 200 ms −1 in a direction making an angle of 60° with
the horizontal a height h above the floor, throws a ball of mass m
straight down with a speed u. When the ball reaches
6 Two balls of equal mass have a head on collision with the floor, the distance of the man above the floor
speed 4 ms −1 each travelling in opposite directions. If will be
the coefficient of restitution is 1/2, the speed of each  m  M
ball after impact will be (a) h 1 +  (b) 1 +  h
 M  m
(a) 1 ms −1 (b) 2 ms −1 m
(c) 3 ms −1 (d) data insufficient (c) h (d) h
M
COM, Conservation of Momentum and Collision 351

13 A cracker is thrown into air with a velocity of 10 ms −1 19 A particle of mass 1 kg is thrown vertically upward
at an angle of 45° with the vertical. When it is at a with speed 100 ms −1 . After 5 s, it explodes into two
height of (1/2) m from the ground, it explodes into a parts. One part of mass 400 g emerges with speed
number of pieces which follow different parabolic 25 ms −1 in downward direction , what is the
paths. What is the velocity of centre of mass, when velocity of other part just after explosion?
it is at a height of 1 m from the ground? (Take, g = 10 ms −2 )
(Take, g = 10 ms −2 )
(a) 100 ms −1 upward (b) 600 ms −1 upward
(a) 4 5 ms −1 (b) 2 5 ms −1 (c) 100 ms −1 downward (d) 300 ms −1 upward
(c) 5 4 ms −1 (d) 5 ms −1
20 A circular plate of diameter a
14 Two blocks of masses 10 kg and 30 kg are placed is kept in contact with a
along a vertical line. The first block is raised through square plate of edge a as
a height of 7 cm. By what distance should the shown in figure. The density
a a
second mass be moved to raise the centre of mass by of the material and the
1 cm? thickness are same everywhere. The centre of mass
(a) 2 cm upward (b) 1 cm upward of the composite system will be
(c) 2 cm downward (d) 1 cm downward (a) inside the circular plate
15 A cricket ball of mass 150 g moving with a speed of (b) inside the square plate
(c) at the point of contact
126 kmh −1 hits at the middle of the bat, held firmly
(d) outside the system
at its position by the batsman. The ball moves
straight back to the bowler after hitting the bat. 21 A ladder is leaned against a smooth wall and is
Assuming that collision between ball and bat is allowed to slip on a frictionless floor. Which figure
completely elastic and the two remain in contact for represents trace of its centre of mass?
0.001 s, the force that the batsman had to apply to
hold the bat firmly at its place would be
(a) 10.5 N (b) 21 N [NCERT Exemplar] (a) (b)
(c) 1.05 ×104 N (d) 2.1 × 104 N
16 Which of the following points is the likely position
Time Time
of the centre of mass of the system as shown in
figure? [NCERT Exemplar]
Hollow sphere
Air (c) (d)

R/2 A
B
Time Time
C
R/2 22 Both the blocks as shown in the given arrangement
D are given together a horizontal velocity towards
Sand right. If a CM be the subsequent acceleration of the
(a) A (b) B centre of mass of the system of blocks, then a CM
(c) C (d) D will be
17 A metal ball falls from a height of 32 m on a steel 1kg µ = 0.1
plate. If the coefficient of restitution is 0.5, to what
2kg µ = 0.2
height will the ball rise after second bounce?
(a) 2 m (b) 4 m
5
(c) 8 m (d) 16 m (a) zero (b) ms −2
3
18 10000 small balls, each weighing 1 g, strikes 1 cm 2 7 −2
of area per second with a velocity 100 ms −1 in a (c) ms (d) 2 ms −2
3
normal direction and rebound with the same
velocity. The value of pressure on the surface will 23 In a free space, a rifle of mass M shoots a bullet of
be mass m at a stationary block of mass M at a distance
(a) 2 × 10 Nm
3 −2
(b) 2 × 10 Nm5 −2 D away from it. When the bullet has moved through
−2 a distance d towards the block, the centre of mass of
(c) 10 Nm
7
(d) 2 × 107 Nm−2 the bullet-block system is at a distance of
352 OBJECTIVE Physics Vol. 1

(D − d ) m 28 A bullet of mass m is fired into a block of wood of


(a) from the bullet
M +m mass M which hangs on the end of pendulum and
md + MD gets embedded into it. When the bullet strikes the
(b) from the block wooden block, the pendulum starts to swing with
M +m
maximum rise R. Then, the velocity of the bullet is
2md + MD given by
(c) from the block
M +m M M +m
(a) 2gR (b) 2gR
(D − d ) M m +M m
(d) from the bullet
M +m M
(c) 2gR (d) None of these
24 A man of mass M stands at one end of a plank of m
length L which lies at rest on a frictionless surface. 29 A ball falling freely from a height of 4.9 m , hits a
The man walks to the other end of the plank. If the horizontal surface. If e = 3 /4, then the ball will hit
M
mass of the plank is , the distance that the man the surface second time after
3 (a) 1 s (b) 1.5 s (c) 2 s (d) 3 s
moves relative to the ground is
3L L 4L L 30 In a one dimensional collision between two identical
(a) (b) (c) (d) particles A and B, B is stationary and A has
4 4 5 3
momentum p before impact. During impact, B gives
25 A 2 kg block of wood rests on a long table top. A 5 g an impulse J to A. Then, coefficient of restitution
bullet moving horizontally with a speed of 150 ms −1 between the two is
is shot into the block and sticks to it. The block then 2J 2J J J
slides 2.7 m along the table top and comes to a stop. (a) −1 (b) +1 (c) +1 (d) −1
p p p p
The force of friction between the block and the
table is 31 A particle of mass m kg moving with a velocity
(a) 0.052 N (b) 3.63 N (c) 2.50 N (d) 1.04 N (3 $i + 2$j ) ms −1 collides with a stationary body of
26 A particle falls from a height h upon a fixed mass M kg and finally moves with a velocity
horizontal plane and rebounds. If e is the coefficient m 1
(− 2$i + $j ) ms −1. If = , then
of restitution, the total distance travelled before M 13
rebounding has stopped is (a) the impulse is ±m (5i$ + $j) kg-ms −1
1 + e 2 1 − e 2
1 $ $
(a) h   (b) h   (b) the velocity of the M is (5 i + j) ms −1
1 − e 2  1 + e 2  13
h 1 − e 2  h 1 + e 2  (c) Both (a) and (b) are wrong
(c)   (d)   (d) Both (a) and (b) are correct
2 1 + e 2  2 1 − e 2 
32 A ball falls freely from a height of 45 m. When the
27 Two identical balls bearing in 1 2 3 ball is at a height of 25 m, it explodes into two equal
contact with each other and pieces. One of them moves horizontally with a speed
resting on a frictionless table of 10 ms −1. The distance between the two pieces on
are hit head on by another ball v the ground is
bearing the same mass moving (a) 20 m (b) 30 m (c) 40 m (d) 60 m
initially with a speed v as shown in figure.
33 A ball of mass m is released from the top of an
If the collision is elastic, which of the following inclined plane of inclination θ as shown in figure. It
(figure) is a possible result after collision? 3h
[NCERT Exemplar] strikes a rigid surface at a distances from top
4
1 1 2 3 elastically. Impulse imparted to ball by the rigid
(a) (b)
surface is

v=0 v/2 v=0 v m

1 2 3 1 2 3 l
h
th

(c) (d)
oo
Sm

v/3 θ
v/1 v/2 v/3
COM, Conservation of Momentum and Collision 353

3 Which of the following cannot be the coordinates of


(a) m gh (b) m 3gh
2 centre of mass of the object?
(c) 2m 3gh (d) m 6gh Y

34 A block A of mass M moving with speed u collides B


elastically with block B of mass m which is
connected to block C of mass m with a spring. X
A u A
B C
M m m
R R  R R 
(a)  ,  (b)  , 
When the compression in spring is maximum, the  3 3  2 2
velocity of block C with respect to block A is R R 
(Neglect the friction everywhere) (c)  ,  (d) None of these
 4 4
 M   m  m 
(a) zero (b)   u (c)  u (d)   u 39 A circular ring of mass 6 kg and radius a is placed
M + m  M + m  M
such that its centre lies at the origin. Two particles
35 A particle of mass m moving with velocity u makes an of masses 2 kg each are placed at the intersecting
elastic one dimensional collision with a stationary points of the circle with positive X-axis and positive
particle of mass m. They are in contact for a brief time Y-axis. Then, the angle made by the position vector
T. Their force of interaction increases from zero to F0 of centre of mass of entire system with X-axis is
T  4
linearly in time and decreases linearly to zero in (a) 45° (b) 60° (c) tan−1   (d) 30°
2  5
T
further time . The magnitude of F0 is 40 A particle A of mass m initially at rest slides down a
2 height of 1.25 m on a frictionless ramp, collides with
mu 2mu mu
(a) (b) (c) (d) None of these and sticks to an identical particle B of mass m at rest
T T 2T as shown in the figure.
36 A T-shaped object with dimensions as shown in the m
figure, is lying on a smooth floor. A force F is A
applied at the point P parallel to AB, such that the 1.25 m
object has only the translational motion without B C
rotation. Find the location of P with respect to C .
m 2m
l
A B Then, particles A and B together collide elastically
with particle C of mass 2m at rest. The speed of
P particle C after the collision with combined body
F 2l
(A + B ) would be (Take, g = 10 ms −2 )
C (a) 2 ms −1 (b) 1.25 ms −1 (c) 2.5 ms −1 (d) 5 ms −1

4 41 A man of mass m moves with a constant speed on a


(a) l (b) l plank of mass M and length l kept initially at rest on
3
2 3 a frictionless horizontal surface from one end to the
(c) l (d) l other in time t. The speed of the plank relative to
3 2
ground while man is moving, is
37 A ball is projected vertically down with an initial l M l  m 
velocity from a height of 20 m onto a horizontal (a)   (b)  
t m  t m + M
floor. During the impact, it loses 50% of its energy
and rebounds to the same height. The initial velocity l  M 
(c)   (d) None of these
of its projection is (Take, g = 10 ms −2 ) t M + m 
(a) 20 ms−1 (b) 15 ms−1 (c) 10 ms−1 (d) 5 ms−1 42 You are supplied with three identical rods of same
38 An object comprises a uniform ring of radius R and length and mass. If the length of each rod is 2π.
its uniform chord AB (not necessarily made of the Two of them are converted into rings and then
same material) as shown in figure. placed over the third rod as shown in figure. If
point A is considered as origin of the coordinate
354 OBJECTIVE Physics Vol. 1

system, the coordinates of the centre of mass will be 46 A girl throws a ball with initial velocity v at an
(you may assume AB as X-axis of the coordinate inclination of 45°. The ball strikes the smooth
system) vertical wall at a horizontal distance d from the girl
and after rebounding returns to her hand. What is
the coefficient of restitution between wall and the
ball?
A B
gd
(a) v 2 − gd (b)
 π 1  π 2  1  2 v − gd
2
(a)  ,  (b)  ,  (c)  π,  (d)  π, 
 2 3  2 3  3  3
gd v2
(c) (d)
43 A pendulum consists of a wooden bob of mass m and v2 gd
length l. A bullet of mass m1 is fired towards the
47 A disc of mass 10 g is kept floating horizontally by
pendulum with a speed v 1 and it emerges from the
v throwing 10 marbles per second against it from
bob with speed 1 . The bob just completes motion below. If the mass of each marble is 5 g. What will
3 be velocity with which the marbles are striking the
along a vertical circle. Then, v 1 is disc? Assume that, the marble strikes the disc
normally and rebound downwards with the same
speed.
O
(a) 2.98 ms −1 (b) 0.98 ms −1
(c) 0.49 ms −1 (d) 1.96 ms −1
m1 v1 m m1 v1 /3 48 Two blocks of masses m and 2m are kept on a
m 3m smooth horizontal surface. They are connected by an
(a) 5gl (b) 5gl ideal spring of force constant k. Initially, the spring
m1 2m1
is unstretched. A constant force is applied to the
2 m m  heavier block in the direction as shown in figure.
(c)   5gl (d)  1 gl
3  m1 m Suppose at time t, displacement of smaller block is x,
44 From a circular disc of radius R, a square is cut out then displacement of the heavier block at this
with a radius as its diagonal. The centre of mass of moment would be
F
remaining portion is at a distance (from the centre) m 2m
R R
(a) (b)
(4π − 2) 2π
R R
(c) (d) x Ft 2 x x Ft 2 x
(π − 2) (2π − 2) (a) (b) + (c) (d) −
2 6m 3 3 4m 2
45 A uniform circular disc of radius a is taken. A 49 Three identical blocks A, B and C are placed on
circular portion of radius b has been removed from it horizontal frictionless surface. The blocks B and C
as shown in the figure. If the centre of hole is at a are at rest but A is approaching towards B with a
distance c from the centre of the disc, the distance
speed 10 ms −1 .
x 2 of the centre of mass of the remaining part from A B C
the initial centre of mass O is given by

The coefficient of restitution for all collisions is 0.5.


a The speed of the block C just after collision is
b
approximately
O2 O O1 X-axis (a) 5.6 ms −1 (b) 6.4 ms −1
x2 c (c) 3.2 ms −1 (d) 4.6 ms −1
50 A train of mass M is moving on a circular track of
radius R with constant speed v. The length of the
πb 2 cb 2 train is half of the perimeter of the track. The linear
(a) (b)
(a 2 − c 2 ) (a 2 − b 2 ) momentum of the train will be
πc 2 ca 2 2Mv πMv
(c) (d) (a) πMv (b) (c) (d) Mv
(a − b )
2 2
(c − b )
2 2 π 2
COM, Conservation of Momentum and Collision 355

51 n elastic balls are placed at rest on a smooth 52 A small ball rolls off the top landing of the staircase.
horizontal plane which is circular at the ends with It strikes the mid-point of the first step and then the
radius r as shown in the figure. The masses of the mid-point of the second step. The steps are smooth,
m m m and identical in height and width. The coefficient of
balls are m, , 2 , K, n − 1 , respectively. What is
2 2 2 restitution between the ball and the first step is
the minimum velocity which should be imparted to 3
(a) 1 (b)
the first ball of mass m such that this nth ball will 4
complete the vertical circle? 1 1
(c) (d)
2 4
53 Two identical blocks A and B, each of mass m
r resting on smooth floor are connected by a light
m spring of natural length L and spring constant k with
the spring at its natural length. A third identical
n −1 n −1
 3  4 block C (mass m) moving with a speed v along the
(a)   5gr (b)   5gr line joining A and B collides with A, the maximum
 4  3
n −1 n −1 compression in the spring is
 3  2
(c)   5gr (d)   5gr m v mv mv
 2  3 (a) v (b) m (c) (d)
2k 2k k 2k

(B) Medical entrance special format questions


Assertion and reason
Linear momentum of the system will not remain
Directions (Q. Nos. 1-6) These questions consist of two
statements each printed as Assertion and Reason. While constant till the spring reaches its initial natural
answering these questions, you are required to choose any length.
one of the following four responses
(a) If both Assertion and Reason are correct and Reason is the A B
correct explanation of Assertion.
(b) If both Assertion and Reason are correct but Reason is not Reason An external force will act from the wall on
the correct explanation of Assertion. block A.
(c) If Assertion is correct but Reason is incorrect.
(d) If Assertion is incorrect but Reason is correct. 5. Assertion Two blocks of masses m A and
1 Assertion The relative velocity of the two particles mB (mB > m A ) are thrown towards each other with
in head on elastic collision is unchanged both in same speed over a rough ground. The coefficient of
magnitude and direction. friction of both the blocks with ground is same.
Reason The relative velocity is unchanged in Initial velocity of CM is towards left.
magnitude but gets reversed in direction. A v v B
2. Assertion If net force on a system is zero, then Rough
momentum of every individual body remains
constant. Reason Initial acceleration of centre of mass is
towards right.
Reason If momentum of a system is constant, then
kinetic energy of the system may change. 6. Assertion Two identical spheres are half filled with
two liquids of densities ρ1 and ρ 2 ( > ρ1 ). The centre of
3. Assertion Two bodies moving in opposite
mass of both the spheres lie at same level.
directions with same magnitude of linear momentum
collide with each other. Then, after collision both
the bodies will come to rest.
Reason Linear momentum of the system of bodies is zero.
(a) (b)
4. Assertion Two blocks A and B are connected at
the two ends of an ideal spring as shown in figure. Reason The centre of mass will lie at centre of the
Initially spring was relaxed. Now, block B is pressed. sphere.
356 OBJECTIVE Physics Vol. 1

Statements based questions II. In elastic collision, kinetic energy during the
collision time ∆t is constant.
1 Two trains A and B are running in the same
Which of the following statement(s) is/are correct?
direction on parallel rails such that A is faster than B.
(a) Only I (b) Only II
Packets of equal weight are transferred between (c) Both I and II (d) Neither I nor II
them. Which of the following statement is correct?
(a) A will be accelerated, but B will be retarded. Match the columns
(b) B will be accelerated, but A will be retarded.
(c) There will be no change in A, but B will be accelerated. 1 In the diagram shown in figure, mass of both the
(d) There will be no change in B, but A will be accelerated. balls is same. Match the following columns and mark
2 In a two block system, an initial velocity v 0 with the correct option from the codes given below.
v v'
respect to ground is given to block A. Which of the
1 2 ⇒ 2
following statement(s) is/are correct?
Before After
A v0 Rough collision collision
B Column I Column II
Smooth (A) For v ′ = v (p) e=0
(a) The momentum of block A is not conserved. (B) For v ′ = v / 2 (q) e =1
(b) The momentum of system of blocks A and B is (C) For v ′ = (3 / 4 ) v (r) e = 1/ 2
conserved.
(s) Data is insufficient
(c) The increase in momentum of B is equal to the
decrease in momentum of block A. Codes
(d) All of the above A B C A B C
O (a) p q r (b) q p r
3 The bob A of a simple pendulum
(c) s r q (d) s p r
is released when the string
makes an angle of 45° with the 45° 2 A particle of mass 1 kg has velocity v 1 = (2t ) $i and
vertical. It hits another bob B of another particle of mass 2 kg has velocity
the same material and same A v 2 = (t 2 ) $j . Match the following columns and mark
mass kept at rest on a table. If B the correct option from the codes given below.
the collision is elastic, which of
Column I Column II
the following statement is correct?
(a) Both A and B rise to the same height. (A) Net force on centre of mass at 2 s (p) 20 unit
(b) Both A and B come to rest at B. 9
(c) Both A and B move with the same velocity of A. (B) Velocity of centre of mass at 2 s (q) 68 unit
(d) A comes to rest and B moves with the velocity of A. (C) Displacement of centre of mass in 2 s (r) 80 / 3 unit
4 I. Linear momentum of the system remains (s) None
constant. Codes
II. Centre of mass of the system remains at rest. A B C A B C
Which of the following statement(s) is/are correct? (a) q r p (b) q p r
(a) I implies II and II implies I (c) p r s (d) s q r
(b) I does not imply II and II does not imply I 3 A particle of mass m, kinetic energy K and
(c) I implies II but II does not imply I momentum p collides head on elastically with
(d) II implies I but I does not imply II another particle of mass 2 m at rest. Match the
5 I. Linear momentum of a system of particles is following columns (after collision) and mark the
zero. correct option from the codes given below.
II. Kinetic energy of a system of particles is zero. Column I Column II
Which of the following statement(s) is/are correct? (A) Momentum of first particle (p) 4 p/3
(a) I implies II and II implies I (B) Momentum of second particle (q) K/9
(b) I does not imply II and II does not imply I (C) Kinetic energy of first particle (r) − p/3
(c) I implies II but II does not imply I (D) Kinetic energy of second particle (s) 8K/9
(d) II implies I but I does not imply II
A B C D A B C D
6 I. In elastic collision, initial kinetic energy is equal (a) q r s p (b) p s r q
to the final kinetic energy. (c) r p q s (d) s q r p
(C) Medical entrances’ gallery
Collection of questions asked in NEET & various medical entrance exams

1 Two particles of masses 5 kg and 10 kg respectively inelastically and sticks to it. Then, loss in kinetic
are attached to the two ends of a rigid rod of length energy of the system will be [AIIMS 2019]
1 m with negligible mass. The centre of mass of the (a) 7.5 kJ (b) 15 kJ (c) 10 kJ (d) 5 kJ
system from the 5 kg particle is nearly at a distance 7 Assertion There is no loss in energy in elastic
of [NEET 2020] collision.
(a) 50 cm (b) 67 cm Reason Linear momentum is conserved in elastic
(c) 80 cm (d) 33 cm collision. [AIIMS 2019]
2 Three identical spheres, each of mass M, are placed (a) Both Assertion and Reason are correct and Reason is the
at the corners of a right angle triangle with the correct explanation of Assertion.
mutually perpendicular sides equal to 2 m (see (b) Both Assertion and Reason are correct, but Reason is not
figure). Taking the point of intersection of the two the correct explanation of Assertion.
mutually perpendicular sides as the origin, find the (c) Assertion is correct, but Reason is incorrect.
(d) Both Assertion and Reason are incorrect.
position vector of centre of mass. [NEET 2020]
8 One object of mass 20 kg is moving with speed
10 ms −1 in west direction and another object of mass
j

M 10 kg is moving with 15 ms −1 in north direction.


Both collide and stick together. Choose the correct
2m alternative. [JIPMER 2019]
M (a) Their kinetic energy is conserved as it is inelastic
M 2m i
collision.
(b) Their kinetic energy is conserved as it is elastic
2 $ $ 4 $ $ collision.
(a) 2(i$ + $j ) (b) (i$ + j$ ) (c) (i + j ) (d) (i + j ) (c) Their momentum is conserved as it is inelastic collision.
3 3
(d) Their momentum is conserved as it is elastic collision.
3 Body A of mass 4m moving with speed u collides
with another body B of mass 2m at rest. The collision 9 Two objects of mass m each moving with speed
is head on and elastic in nature. After the collision, u ms −1 collide at 90°, then final momentum is
the fraction of energy lost by the colliding body A is (assume collision is inelastic) [JIPMER 2019]
[NEET 2019] (a) mu (b) 2 mu (c) 2 mu (d) 2 2 mu
8 4 5 1
(a) (b) (c) (d) 10 A moving block having mass m, collides with another
9 9 9 9
stationary block having mass 4m. The lighter block
4 An object flying in air with velocity(20 $i + 25$j − 12k
$)
comes to rest after collision. When the initial
suddenly breaks in two pieces whose masses are in velocity of the lighter block is v, then the value of
the ratio 1 : 5. The smaller mass flies off with a coefficient of restitution (e ) will be [NEET 2018]
velocity (100 $i + 35$j + 8k
$ ). The velocity of the larger
(a) 0.8 (b) 0.25 (c) 0.5 (d) 0.4
piece will be [NEET (Odisha) 2019] 11 Three bodies having masses 5 kg, 4 kg and 2 kg are
(a) 4i$ + 23j$ − 16 k$ (b) − 100i$ − 35j$ − 8k$ moving at the speeds of 5 ms −1, 4 ms −1 and 2 ms −1,
(c) 20i$ + 15j$ − 80k$ (d) − 20i$ − 15j$ − 80k$ respectively along X-axis. The magnitude of velocity
of centre of mass is [AIIMS 2018]
5 A particle of mass 5m at rest suddenly breaks on its
own into three fragments. Two fragments of mass m (a) 1.0 ms −1 (b) 4 ms −1
each move along mutually perpendicular directions (c) 0.9 ms −1 (d) 1.3 ms −1
each with speed v. The energy released during the 12 Body of mass M is much heavier than the other body
process is [NEET (Odisha) 2019] of mass m. The heavier body with speed v collides
3 5 3 4 with the lighter body which was at rest initially
(a) mv 2 (b) mv 2 (c) mv 2 (d) mv 2
5 3 2 3 elastically. The speed of lighter body after collision is
[AIIMS 2018]
6 A body of mass 5 × 10 3 kg moving with speed v
(a) 2 v (b) 3 v (c) v (d)
2 ms −1 collides with a body of mass 15 × 10 3 kg 2
358 OBJECTIVE Physics Vol. 1

13 Assertion Two particles are moving in the same to a wedge fixed rigidly with the horizontal part. A
direction do not lose all their energy in completely 40 g mass is released from rest while situated at a
inelastic collision. height 5 m of the curved track. The minimum
Reason Principle of conservation of momentum deformation in the spring is nearly equal to (Take,
holds true for all kinds of collisions. [AIIMS 2018] g = 10 ms −2 )
[AIIMS 2015]
(a) Both Assertion and Reason are correct and Reason is
the correct explanation of Assertion.
(b) Both Assertion and Reason are correct but Reason is not
5m
the correct explanation of Assertion.
(c) Assertion is correct but Reason is incorrect.
(d) Assertion is incorrect but Reason is correct.
14 A ball of 0.5 kg collided with wall at 30° and (a) 9.8 m (b) 9.8 cm (c) 0.98 m (d) 0.009 km
bounced back elastically. The speed of ball was 20 A block having mass m collides with an another
12ms −1. The contact remained for 1s. What is the stationary block having mass 2 m. The lighter block
force applied by wall on ball? [JIPMER 2018] comes to rest after collision. If the velocity of first
(a) 12 3 N (b) 3 N (c) 6 3 N (d) 3 3 N block is v, then the value of coefficient of restitution
will must be [AIIMS 2015]
15 A body of mass 4 kg moving with velocity 12 ms −1 (a) 0.5 (b) 0.4 (c) 0.6 (d) 0.8
collides with another body of mass 6 kg at rest. If
two bodies stick together after collision, then the loss 21 A smooth curved surface of height 10 m is ended
of kinetic energy of system is [AIIMS 2017] horizontally. A spring of force constant 200 Nm –1 is
(a) zero (b) 288 J (c) 172.8 J (d) 144 J fixed at the horizontal end as shown in figure. When
16 Two masses of 6 and 2 unit, are at positions (6$i −7$j) an object of mass 10 g is released from the top, it
travels along the curved path and collides with the
and (2$i + 5$j − 8k$ ), respectively. The coordinates of spring. Then, the maximum compression in the
the centre of mass are [JIPMER 2017] spring is (Take, g = 10 ms −2 ) [EAMCET 2015]
(a) (2,−5,3) (b) (5,−5, −3) (c) (5,−4,−2) (d) (5,−4,−4)
17 A block C of mass m is moving with velocity v 0 and
collides elastically with block A of mass m and
connected to another block B of mass 2m through
spring of spring constant k. What is the value of k, if
x 0 is compression of spring, when velocity of A and (a) 10 m (b) 0.1 m (c) 1 m (d) 0.01 m
B is same? [JIPMER 2017] 22 A frog sits on the end of a long board of length
C v0 A B
L = 10 cm. The board rests on a frictionless
horizontal table. The frog wants to jump to the
mv 02 mv 02 3 mv 02 2 mv 02 opposite end of the board. What is minimum take off
(a)
x 02
(b)
2 x 02
(c)
2 x 02
(d)
3 x 02 speed v in ms −1 relative to the ground that the frog
follows to do the trick? [Assume that, the board and
18. Two particles of masses m1 and m 2 move with initial frog have equal masses.] [UP CPMT 2015]
velocities u1 and u 2 . On collision, one of the particles (a) 2 5 ms −1 (b) 5 ms −1 (c) 5 2 ms −1 (d) 10 2 ms −1
get excited to higher level, after absorbing energy ε.
23 A particle of mass m collides with another stationary
If final velocities of particles be v 1 and v 2 , then we
particle of mass M. If the particle m stops just after
must have [CBSE AIPMT 2015]
collision, then the coefficient of restitution for
(a) m12 u1 + m 22 u2 − ε = m12 v1 + m 22 v 2 collision is equal to [Manipal 2015]
1 1 1 1 m M −m m
(b) m1 u12 + m 2 u22 = m1 v12 + m 2v 22 − ε (a) 1 (b) (c) (d)
2 2 2 2 M M +m M +m
1 1 1 1
(c) m1 u12 + m 2 u22 − ε = m1 v12 + m 2v 22 24 A body from height h is dropped, if the coefficient of
2 2 2 2
1 2 2 1 2 2 1 2 2 1 2 2 restitution is e, then calculate the height achieved
(d) m1 u1 + m 2 u2 + ε = m1 v1 + m 2v 2 after one bounce. [Manipal 2015]
2 2 2 2
(a) h1 = e 4h (b) h = e h1
19 Consider the situation as shown in figure. A spring
of spring constant 400 Nm −1 is attached at one end (c) h1 = e h
2
(d) h = h1 /e
COM, Conservation of Momentum and Collision 359

25 Three particles of masses 0.5 kg, 1 kg, 1.5 kg are 31 A gun fires a small bullet with kinetic energy K.
placed at the three corners of a right angled triangle Then, kinetic energy of the gun while recoiling is
of sides 3 cm, 4 cm, 5 cm as shown in adjoining [KCET 2013]
figure. What would be coordinates (x, y) of the (a) K (b) more than K
centre of mass of system? (c) less than K (d) K
[UK PMT 2015, UP CPMT 2015] 32 The linear momentum is conserved in [J&K CET 2013]
1.5 kg (a) elastic collisions (b) inelastic collisions
(c) Both (a) and (b) (d) Neither (a) nor (b)
3 cm 5 cm
33 Three particles, each of mass m are placed at the
vertices of a right angled triangle as shown in figure.
0.5 kg 4 cm 1 kg
The position vector of the centre of mass of the
$ are unit vectors)
system is (O is the origin and $i , $j, k
Y [EAMCET 2013]
(a) (1.3, 1.5) (b) (2.3, 1.5)
(c) (1.3, 2.5) (d) (2.3, 2.5)
B m
26 A large number of particles are placed around the
origin, each at a distance R from the origin. The b
distance of the centre of mass of the system from the
origin is [WB JEE 2015] m
O m A X
(a) equal to R (b) less than or equal to R a
(c) greater than R (d) greater than or equal to R 1 $ $ 2 $ $
(a) (a i − bj) (b) (a i − bj)
27 A body of mass 4m is lying in xy-plane at rest. It 3 3
suddenly explodes into three pieces. Two pieces 2 1
(c) (a$i + b$j) (d) (a$i + b$j)
each of mass m move perpendicular to each other 3 3
with equal speeds v. The total kinetic energy 34 A ball of mass m moving with a horizontal velocity v
generated due to explosion is [CBSE AIPMT 2014] strikes the bob of a pendulum at rest. Mass of the
(a) mv 2 (b) (3 / 2) mv 2 bob of the pendulum is also m. During this collision,
(c) 2mv 2 (d) 4 mv 2 the ball sticks with the bob of the pendulum. The
height to which the combined mass rises will be
28 The linear momentum of a particle varies with time t (g = acceleration due to gravity) [EAMCET 2013]
as p = a + bt + ct 2 . Then, which of the following is v2 v2 v2 v2
correct? [EAMCET 2014]
(a) (b) (c) (d)
4g 8g g 2g
(a) Velocity of particle is inversely proportional to time
(b) Displacement of the particle is independent of time 35 In an inelastic collision, [Kerala CEE 2013]
(c) Force varies with time in a quadratic manner (a) momentum is not conserved
(d) Force is dependent linearly on time (b) momentum is conserved but kinetic energy is not
conserved
29 The position of centre of mass of a system of (c) both momentum and kinetic energy are conserved
particles does not depend upon the [Kerala CEE 2014] (d) neither momentum nor kinetic energy is conserved
(a) mass of particles (e) kinetic energy is conserved but not momentum
(b) symmetry of the body
(c) position of the particles
36 Two spheres A and B of masses m1 and m 2
(d) nature of particles respectively collide. A is at rest initially and B is
(e) relative distance between the particles moving with velocity v along X-axis. After collision,
30 An explosion breaks a rock into three parts in a
B has a velocity v /2 in a direction perpendicular to
horizontal plane. Two of them go off at right angles the original direction. The mass A moves after
to each other. The first part of mass 1 kg moves with collision in the direction [CBSE AIPMT 2012]
(a) same as that of B
a speed of 12 ms −1 and the second part of mass 2 kg
(b) opposite to that of B
moves with speed of 8 ms −1. If the third part flies  1
off with speed of 4 ms −1 , then its mass is [NEET 2013] (c) θ = tan−1   to the X-axis
 2
(a) 3 kg (b) 5 kg
 −1
(c) 7 kg (d) 17 kg (d) θ = tan−1  to the X-axis
2
360 OBJECTIVE Physics Vol. 1

37 Two persons of masses 55 kg and 65 kg respectively 42 A mass of 10 g moving horizontally with a velocity
are at the opposite ends of a boat. The length of the of 100 cm s −1 strikes a pendulum bob of same mass.
boat is 3 m and weighs 100 kg. The 55 kg man The two masses after collision stick together.
walks up to the 65 kg man and sits with him. If the What will be the maximum height reached by the
boat is in still water, the centre of mass of the system now? (Take, g = 10 ms −2 ) [JCECE 2012]
system shifts by [CBSE AIPMT 2012]
(a) 3.0 m (b) 2.3 m
(c) zero (d) 0.75 m
38 A body of mass 0.25 kg is projected with muzzle
velocity 100 ms −1 from a tank of mass 100 kg. What
is the recoil velocity of the tank? [AIIMS 2012]
(a) Zero (b) 1.25 cm (c) 2.5 cm (d) 5 cm
−1 −1 43 In the diagram shown below, m1 and m 2 are the
(a) 5 ms b) 25 ms
(c) 0.5 ms −1 (d) 0.25 ms −1 masses of two particles and x 1 and x 2 are their
39 When a body of mass m1 moving with uniform respective distances from the origin O. The centre of
mass of the system is [J&K CET 2011]
velocity 40 ms −1 collides with another body of mass
O m1 m2
m 2 at rest, then the two together begin to move with
x1
uniform velocity of 30 ms −1. The ratio of the masses x2
(i.e. m1 /m 2 ) of the two bodies will be
[BCECE (Mains) 2012] m1x 2 + m 2x 2 m1 + m 2
(a) (b)
(a) 1 : 3 (b) 3 : 1 m1 + m 2 2
(c) 1 : 1.33 (d) 1 : 0.75 m x + m 2x 2 m m + x1x 2
40 A ball moving with velocity 9 ms −1 collides with (c) 1 1 (d) 1 2
m1 + m 2 m1 + m 2
another similar stationary ball. If after the collision,
both the balls move in directions making an angle of 44 A bullet of mass m moving with velocity v strikes a
30° with the initial direction, then their speeds after suspended wooden block of mass M. If the block rises
collision will be [BHU 2012] to a height h, then initial velocity of the block will be
(a) 5.2 ms−1 (b) 0.52 ms−1 [Haryana PMT 2011]
(c) 52 ms−1 (d) 26 ms−1 M +m
(a) 2gh (b) gh
m
41 A body of mass m1 = 4 kg moves at 5 $i ms −1 and m M +m
(c) 2gh (d) 2gh
another body of mass m = 2 kg moves at 10 $i ms −1.
2 M +m M
The kinetic energy of centre of mass is [Manipal 2012] 45 A particle of mass m1 moves with velocity v 1 and
200 500 collides with another particle at rest of equal mass.
(a) J (b) J
3 3 The velocity of the second particle after the elastic
400 800 collision is [DUMET 2011]
(c) J (d) J
3 3 (a) 2v1 (b) v1 (c) −v1 (d) 0
ANSWERS
CHECK POINT 8.1
1. (a) 2. (c) 3. (d) 4. (b) 5. (b) 6. (c) 7. (d) 8. (b) 9. (a) 10. (b)
11. (d) 12. (c) 13. (d) 14. (d) 15. (d) 16. (d)

CHECK POINT 8.2


1. (c) 2. (d) 3. (b) 4. (d) 5. (b) 6. (c) 7. (a) 8. (d) 9. (d) 10. (b)
11. (a) 12. (a) 13. (b) 14. (c) 15. (b) 16. (c) 17. (a)

CHECK POINT 8.3


1. (a) 2. (c) 3. (b) 4. (c) 5. (d) 6. (c) 7. (a) 8. (d) 9. (c) 10. (b)
11. (d) 12. (b) 13. (b) 14. (a) 15. (b)

(A) Taking it together


1. (c) 2. (d) 3. (d) 4. (b) 5. (b) 6. (b) 7. (c) 8. (d) 9. (a) 10. (a)
11. (d) 12. (a) 13. (a) 14. (d) 15. (c) 16. (c) 17. (a) 18. (d) 19. (a) 20. (b)
21. (a) 22. (d) 23. (d) 24. (b) 25. (a) 26. (a) 27. (b) 28. (b) 29. (b) 30. (a)
31. (d) 32. (a) 33. (d) 34. (c) 35. (b) 36. (a) 37. (a) 38. (b) 39. (a) 40. (c)
41. (b) 42. (d) 43. (b) 44. (a) 45. (b) 46. (b) 47. (b) 48. (d) 49. (a) 50. (b)
51. (a) 52. (b) 53. (a)

(B) Medical entrance special format questions


l Assertion and reason
1. (d) 2. (d) 3. (d) 4. (a) 5. (b) 6. (c)

l Statement based questions


1. (b) 2. (d) 3. (d) 4. (d) 5. (d) 6. (a)

l Match the columns


1. (b) 2. (a) 3. (c)

(C) Medical entrances’ gallery


1. (b) 2. (c) 3. (a) 4. (a) 5. (d) 6. (a) 7. (b) 8. (c) 9. (c) 10. (b)
11. (b) 12. (a) 13. (a) 14. (c) 15. (c) 16. (c) 17. (d) 18. (c) 19. (b) 20. (a)
21. (b) 22. (c) 23. (b) 24. (c) 25. (a) 26. (b) 27. (b) 28. (d) 29. (d) 30. (b)
31. (c) 32. (c) 33. (d) 34. (b) 35. (b) 36. (c) 37. (c) 38. (d) 39. (b) 40. (a)
41. (c) 42. (b) 43. (c) 44. (a) 45. (b)
Hints & Explanations
l CHECK POINT 8.1 m × 0 + m × PQ + m × PR
8 (b) X CM (from P) = (Q m = 1kg )
1 (a) Let the coordinates of the centre of mass be (x, y ). m+m+m
Y
m1x1 + m 2x 2 1 × (−1) + 2 × 2 −1 + 4
x= = = =1
m1 + m 2 3 3 P Q R
X
m y + m 2 y 2 1 × 2 + 2 × 4 2 + 8 10
and y = 1 1 = = =
m1 + m 2 3 3 3
 10  PQ + PR
Therefore, the coordinates of centre of mass be 1,  . or X CM =
 3 3
1 9. (a) C1 → Position of centre of mass of rods AB and CD (nearer
2 (c) r ∝
m to CD, as it is heavy)
3 (d) Distance distributes in inverse ratio of masses. 3m
D C
r m r 16 1 2
Hence, C = O ⇒ C = C1
d − rC m C d − rC 12
C
4 4
⇒ rC = × d = × 1.2 × 10 −10 4m 2m
7 7 C2 O
= 0.69 × 10 −10 m 3 4
4 (b) CM does not depend on the internal forces acting on the A
m B
particles.
C 2 → Position of centre of mass of rods BC and DA.
5 (b) For a single particle, distance of centre of mass from origin
is R. For more than one particle, distance ≤ R. C → Overall centre of mass of all four rods and it lies in
region 1.
m R R m
10 (b) Let the rod be along X-axis with origin at one of its ends.
x=0
As, the rod is along X-axis, so y CM = z CM = 0, i.e. centre of
For example, for two particles of equal masses, kept as shown mass will be on the rod.
in figure, distance = 0.
6 (c) Given, m1 = 1.6 kg; (x1, y1) = (0, 0) dx

m 2 = 2 kg; (x 2 , y 2 ) = (1.2, 0) x
m 3 = 2.4 kg; (x 3 , y 3 ) = (0, 1)
∴ Coordinates of centre of mass will be
m x + m 2x 2 + m 3x 3 Now, consider an element of rod of length dx at a distance x
x CM = 1 1 from the origin.
m1 + m 2 + m 3
Mass of element, dm = λdx = (A + Bx )dx
(1.6)(0) + (2)(1.2) + (2.4)(0)
= L L
AL2 BL3
1.6 + 2 + 2.4 ∫ xdm ∫ x (A + Bx ) dx +
⇒ x CM = 0.4 m ∴ x CM = 0L = 0L = 2 3
m y + m2y 2 + m3y 3 BL2
and y CM = 1 1
m1 + m 2 + m 3 ∫ dm ∫ (A + Bx )dx AL + 2
0 0
(1.6)(0) + (2)(0) + (2.4)(1) L (3A + 2BL )
= =
1.6 + 2 + 2.4 3(2A + BL )
⇒ y CM = 0.4 m
11 (d) Here, the coordinate of CM of inclined rod is (a/2, a/2).
∴ Coordinates of centre of mass = (0.4, 0.4) m m x + m 2x 2 + m 3x 3
y
Q x CM = 1 1 (0, a)
7 (d) Centre of mass of 1st system already lies at (1, 2, 3). m1 + m 2 + m 3
Therefore, centre of mass of 3 kg and 5 kg should lie at (1, 2, 3).
 a
m × 0 + (m )   + m
 a
 
( —2a , —2a )
3(− $i + 3$j − 2 k$ ) + 5 r 5  2  2 a 1 3
∴ = ($i + 2$j + 3 k$ ) = =
(3 + 5) m+m+m 3
11 7 x
On solving, we get r 5 = $i + $j + 6 k$ a O (a, 0)
5 5 Similarly, y CM = 2
3
i.e. 5 kg mass should be kept at (11/5, 7/5, 6)
COM, Conservation of Momentum and Collision 363

12 (c) As rods are uniform, therefore centre of mass of both rods 16 (d) Centre of mass of complete disc should lie at point O. C1 is
will be at their geometrical centres. The coordinates of CM of the position of centre of mass of remaining portion and C 2 is
L  the position of centre of mass of the removed disc.
first rod, C1 are  , 0 and second rod, C 2 are (0, L).
2  R
y ∴ x (Area of remaining portion) = (Area of removed disc)
2
C2 2M
(0, L)

CM C1
O C2

M x x R/2
O
C1 L , 0
2

 L
M   + 2M (0 )  πR 2  R  πR 2 
 2 L ∴ x  πR 2 − =
∴ x CM =
M + 2M
=
6  4  2  4 
M (0 ) + 2M (L ) 2 L R
y CM = = ∴ x=
M + 2M 3 6
 L 2L  CHECK POINT 8.2
Hence, coordinates of CM are  ,  . l

6 3
1 (c) Centre of mass does not change its path during explosion.
1
13 (d) As, here, OC 1 = m Therefore, it will keep on falling vertically and will not shift
4 horizontally as, Fx = 0.
2 (d) Both the balls in air have acceleration g in downward
C2 direction. Hence, the acceleration of their centre of mass will
C also be g in downward direction.
(m ) (0 ) + (m ) (a ) 1
3 (b) Acceleration of centre of mass, a CM = = a
45° m+m 2
O C1
4 (d) Vertical component of velocity of CM is zero. Horizontal
1
∴ OC = OC1 cos 45° = m component of velocity of CM is non-zero. Acceleration of CM
4 2 is g downwards. Hence, path of CM is a parabola as shown in
y figure.
14 (d) Let centre of square is at origin. m4 m v sinq
x CM = 0 CM v
a
m1x1 + m 2x 2 + m 3x 3 + m 4 x 4
or =0 x q
m1 + m 2 + m 3 + m 4 q
v cosq + v cosq
a
or (2m ) (− a ) + 4m (a ) + m (a ) + m 4 (−a ) = 0
or m 4 = 3m 2m a a 4m v
v sinq
Similarly, y CM = 0
5 (b) Net external force is zero. Hence, velocity of CM of the
or (2m ) (− a ) + 4m (− a ) + m (a ) + m 4 (a ) = 0 or m 4 = 5m box and ball system will remain constant.
Since, value of m 4 is different to be satisfied by both x CM = 0 10 (14) + 4(0 )
and y CM = 0. 6 (c) Velocity of centre of mass, v CM = = 10 ms −1
10 + 4
Hence, it is not possible.
 π 2 7 (a) Since, there is no external force on the rod in horizontal
  (8)
A2  4 direction, the centre of mass of the rod will not move in
15. (d) A1 x1 = A2 x 2 ⇒ x1 = ⋅ x2 = ×6 horizontal direction.
A1 (20 )2
In vertical direction, we have gravitational force as an
= 0.75 cm from O external force, so CM of rod will come 0.5 m down, but will
not move in x-direction. So, CM will be the origin.
Thus, option (a) is correct.
O O1 m v + m 2v 2
8 (d) v CM = 1 1 =0
x2 m1 + m 2
x1
As m1v1 + m 2v 2 = 0 is given. Hence, velocity of composite
system will be zero.
364 OBJECTIVE Physics Vol. 1

9 (d) At t = 0, centre of mass is at mid-point or at (2.25m, 0). 16. (c) Initial momentum of 3m mass = 0 ...(i)
Velocity of centre of mass is zero. Hence, centre of mass will Due to explosion, this mass splits into three fragments of
remain at this position all the time. equal masses.
10 (b) After 1 s, coordinates of first particle will become v
(4 m, 4m, 6m) and coordinates of second particle will become At rest m
(6m, 4m, 8m). 3m
4+ 6
∴ X CM = = 5m Before splitting
m
v
2
4+ 4
YCM = = 4m m
2 v After splitting
6+ 8
and ZCM = = 7m Final momentum of system
2
= mv + mv $i + mv $j ...(ii)
11 (a) Centre of mass will not move along Y-axis.
From law of conservation of linear momentum,
or YCM = 0 (always)
m y + m2y 2 mv + mv $i + mv $j = 0
Q YCM = 1 1
m1 + m 2 ⇒ v = − v ( $i + $j )
(m / 4) 15 + (3m / 4) ( y 2 )
0= 17 (a) From conservation of linear momentum,
(m / 4 + 3m / 4)
m m/2 m/2
15m 3m v'
⇒ − = ( y 2) v cos q Þ
4 4 v cos q
⇒ y 2 = − 5 cm
m m
12 (a) External force on system is zero, i.e. pi = p f = 0 m (v cos θ ) = v ′ − v cos θ
2 2
∴ Centre of mass should be at rest at all instants. ∴ v ′ = 3v cos θ
13 (b) m (u − v ) = mv (using figure) l CHECK POINT 8.3
u
∴ v= 2 (c) Net force on ball and earth system is zero. Hence, total
2 momentum of the ball and the earth is conserved.
3 (b) In elastic collision of two identical masses, velocities are
interchanged after collision. Therefore, after collision,
u–v velocity of second body will be u.
v
5 (d) From law of conservation of momentum,
m (u – v) = mv
m Av A + m B v B = m Av B + m Bv A
u m A (v A − v B ) = m B (v A − v B )
∴ Speed of man relative to ground = u − v =
2 mA
⇒ =1
14. (c) In horizontal direction, net force on the system is zero. mB
Therefore, principle of conservation of momentum can be
applied for system (m + M ) horizontally. 6 (c) In perfectly elastic collision between two bodies of equal
masses, velocities are exchanged. So, after collision, particle A
15. (b) K1 + K2 = 2400 ...(i) will move with 10 ms −1 and particle B with 15 ms −1.
p1 = p 2
7 (a) Along x-direction, m × 50 − m × 40 = m × 0 + mv x
∴ 2K1m1 = 2K2m 2
⇒ v x = 10 cm/s
or
K1 m 2 3
= = ...(ii) Along y-direction, m × 0 + m × 30 = m × 0 + mv y
K2 m1 1 ⇒ v y = 30 cm/s
From Eqs. (i) and (ii), we get 8 (d) From law of conservation of energy,
K1 3 1 2 1
= mv = × kx 2
2400 − K1 1 2 2
⇒ 7200 − 3 K1 = K1 ⇒ (0.5)(1.5)2 = 50 x 2
⇒ 4K1 = 7200 1.125
⇒ x2 =
⇒ K1 = 1800 50
K1 = Kinetic energy of smaller part = 1800 J ⇒ x = 0.15 m
COM, Conservation of Momentum and Collision 365

9 (c) m2 m1 m2 15 (b) From conservation of linear momentum,


m1 M m M m
u ⇒ v2 1 v1
u2 = 0 v1 = v' v2 = v 2 1 2
u1 = u
Before collision After collision mu = mv1 + mv 2 or u = v1 + v 2 ...(i)
From definition of e, v1 − v 2 = eu ...(ii)
 m − m1 2m1u1
Q v2 =  2  u2 + Solving these two equations, we get
 m1 + m 2  m1 + m 2
 1+ e   1− e 
v1 =   u and v 2 =  u
2Mu  2   2 
∴ v= (Q u 2 = 0)
M+m v1  1 + e 
∴ = 
⇒ v=
2u v 2  1− e 
m
1+
M
(A) Taking it together
10 (b) If target is at rest, then final velocity of bodies are
1 (c) The centre of mass under the given condition may be at
u1 v1 v2
m1 m2 m1 m2 rest or may be moving with constant velocity, i.e. position of
u2 = 0 CM may be at rest or moving with constant velocity.
Before collision After collision 2 (d) A bangle is in the form of a ring as shown
in the adjacent diagram. The centre of mass
 m − m2  lies at the centre which is outside the body
v1 =  1  u1 ...(i) C
 m1 + m 2  (boundary). Centre
2m1u1 dp
and v2 = ...(ii) 3. (d) We know that, for a system, Fext =
m1 + m 2 dt
From Eqs. (i) and (ii), we get (i.e. from Newton’s second law)
v1 m1 − m 2 2 m If Fext = 0, dp = 0 ⇒ p = constant
= = ⇒ 1 =5
v2 2m1 5 m2 Hence, momentum of a system will remain conserved, if
external force on the system is zero.
 m − m1  2m1 
11. (d) Qv ′ 2 =  2  v2 +   v1 In case of collision between particles, equal and opposite forces
 m 2 + m1  m1 + m 2 
will act on individual particles as per Newton’s third law.
and v1 = 0 Note We should not confuse with system and individual particles. As
v 2  m 2 + m1  m + 2m  total force on the system of two particles is zero, but force acts on
∴ =  =  = −3
v ′ 2  m 2 − m1  m − 2m  individual particles.
2 4 (b) As, m1v1 + m 2v 2 = (m1 + m 2 ) v
K2  v 2 
∴ =   = 9:1 ab
K′ 2  v ′ 2  ∴ a ⋅ b + c ⋅ 0 = (a + c ) v ⇒ v =
a+c
Relative velocity of separation 5 − 2 1
12 (b) e = = =
Relative velocity of approach 8 − 2 2 5 (b) At highest point, pi = p f and perpendicular velocity = 0
13 (b) Relative speed of approach is 12 ms −1, i.e. u1 − u 2 = 12 ms −1 So, (3m ) (100 $i ) = m (100 $j ) − m (100 $j ) + m (v )
1 v −v ∴ v = 300 $i (of third part)
e= = 2 1
3 u1 − u 2 1
–1 6 (b) Relative speed of approach is 8 ms −1, e = . Therefore,
6 ms–1 6 ms v1 v2 2
Þ
relative speed of separation will be 4 ms −1.
12 4 ms–1 2 ms–1 2 ms–1
⇒ v 2 − v1 = = 4 ms −1 4 ms–1

3
Therefore, relative speed of separation will be 4 ms −1. 7 (c) F = rate of change of linear momentum
14 (a) From conservation of linear momentum, we can see that n
v In 1 s, bullets are embedded. Momentum of each bullet is mv.
velocity of 2m will become after collision (as mass is 60
2  n
∴ F =   mv
double).  60 
m 2m m 2m ∆p
v ⇒ v/2 8 (d) F = = n (mv )
∆t
Rest
Here, n = number of bullets fired per second.
relative velocity of separation v / 2 1
Now, e = = = ∴ n=
F
=
144
=3
relative velocity of approach v 2 mv 0.04 × 1200
366 OBJECTIVE Physics Vol. 1

F (∆ p / ∆ t ) N
9 (a) In elastic collision, ∴ Pressure = =N = n × 2m × u [Q n = = 10 8 ]
relative speed of separation = relative speed of approach = v A A A∆t
2πr = 10 8 × 2 × 10 −3 × 100
∴ Time of next collision =
v = 2 × 10 7 Nm −2
10 (a) Velocity of block just after collision = 2gh = 2 × 10 × 0.2 19 (a) Velocity of particle after 5 s,
= 2 ms −1 v = u − gt = 100 − 10 × 5
Now, applying conservation of linear momentum just before = 100 − 50 = 50 ms −1 (upwards)
and just after collision,
Conservation of linear momentum gives
0.02 × 600 = 4 × 2 + 0.02 ×v
Mv = m1v1 + m 2v 2 ...(i)
∴ v = 200 ms −1 Taking upward direction positive,
11 (d) From conservation of linear momentum, velocity of v1 = − 25 ms −1, v = 50 ms −1
combined mass just after collision will be 50 cms −1, as mass M = 1kg, m1 = 400 g = 0.4 kg
has doubled. m 2 = M − m1 = 1 − 0.4 = 0.6 kg
u 2 (0.5)2 From Eq. (i), we get
Now, H = = m = 1.25 cm
2g 20 1 × 50 = 0.4 × (−25) + 0.6 v 2 or v 2 = 100 ms −1 (upwards)
12 (a) Centre of mass will remain at height h. π 2
M 20 (b) Area of circle, A1= a and area of square A2 = a 2. Since,
m × 0 + MH 4
∴ hCM = =h A 2 > A1, so centre of mass will lie inside the square plate.
m+M H CM
 m 21 (a) Due to net force in downward direction and towards left,
∴ H = h 1 +  h
 M m centre of mass will move downward and will shift left
horizontally.
13 (a) During explosion of a cracker, path of centre of mass does
R1
not change. At height 1 m,
v = u 2 − 2gh (from conservation of mechanical energy)
−1 R2
= 4 5 ms
mg
m1y1 + m 2 y 2 (10 ) (7) + (30 ) y 2
14 (d) y CM = or +1 =
m1 + m 2 10 + 30 Hence, the correct trace of centre of mass is shown in
∴ y 2 = − 1 cm option (a).
5 External force Force of friction from ground
15 (c) Here, u = − v = 126 kmh−1 = 126 × = 35 m/s 22 (d) a CM = =
18 Total mass Total mass
Change in momentum of the ball, 0.2 × (2 +1) (10)
= = 2 ms −2
∆p = m (v − u ) =
150
(−35 − 35) 1+ 2
1000
23 (d) Distance between bullet and block at this instant is
3 21
= ( − 70 ) = − kg -ms −1 (D − d ).
20 2 Distance of CM from bullet
∆p −21/ 2 M (D − d ) + m × 0 M
Now, force, F = = N = −1.05 × 10 4 N = = (D − d )
∆t 0.001 M+m M+m
Here, negative sign shows that direction of force will be m
Similarly, distance of CM from block = (D − d )
opposite to the direction of movement of the ball before M+m
hitting.
Hence, option (d) is correct.
16 (c) The position of centre of the system shown in the given
24 (b) Let plank moves x distance in opposite direction. Then,
figure is likely to be at C. displacement of man relative to ground will be (L − x ).
This is because lower part of the sphere containing sand is Applying law of conservation of momentum,
heavier than upper part of the sphere containing air. x x M
4
mR R = m L L or M (L − x ) = x
 1 32 t t 3
17 (a) hn = he 2n = 32   = = 2 m (here, n = 2, e = 1/ 2)
 2 16 Solving this equation, we get x =
3L
4
18 (d) In 1 cm2 area, 10 4 balls are striking per second. Therefore, 3L L
in 1m2 area, 10 8 balls will strike per second. ∴ Displacement of man relative to ground = L − =
4 4
Change in momentum of each ball per second will be 2 mu.
COM, Conservation of Momentum and Collision 367

25 (a) Velocity of block just after collision, 32 (a) Remaining time for the pieces to reach the ground,
5 × 10 −3 × 150
v=
(2 + 5 × 10 −3 ) 10 ms–1 10 ms–1 20 ms
(from conservation of linear momentum)
= 0.374 ms −1 25 ms
Let F be the force of friction, then work done against friction x x
= initial kinetic energy
1
F × 2.7 = × 2.005 × (0.374)2 ⇒ F = 0.052 N 2 × 45 2 × 20
or t= − = 1s
2 10 10
27 (b) When two bodies of equal masses collide elastically, their x = 10 × 1 = 10 m
velocities are interchanged. ∴ Distance between two pieces will be 20 m.
When ball 1 collides with ball 2, then velocity of ball 1, v1 33 (d) Loss in PE = Gain in KE
becomes zero and velocity of ball 2, v 2 becomes v, i.e.
1
v1 = 0 and v 2 = v mgh1 = mv 2
2
Similarly, when ball 2 collides with ball 3 , v 2 = 0 and v 3 = v .
3 1 3gh
Hence, figure (b) is correct. mg × h = mv 2 ⇒v =
4 2 2
28 (b) We know that, v ′ = 2gR
3gh
From conservation of linear momentum, Now, impulse imparted, J = 2mv = 2m = m 6gh
2
mv = (M + m ) v ′
M − m 2 Mu
M+m M + m 34 (c) Here, v A =   u and v B =
∴ v= ⋅v ′ =   2gR M + m M+m
m  m 
Velocity of C at maximum compression, v C = v B /2
29 (b) Velocity on hitting the surface
= 2gh = 2 × 9.8 × 4.9 1  2M   M 
∴ vC =   u= u
2 M + m  M + m
= 9.8 ms −1
3  m 
Velocity after first bounce, v = × 9.8 ms −1 ∴ v CA = v C − v A =  u
4 M + m
2v
Time taken from first bounce to the second bounce = 35 (b) In one dimensional elastic collision between two equal
g masses, their velocities are interchanged. Therefore, change
3 1 in linear momentum of any of the particle will be mu.
=2× × 9.8 × = 1.5 s
4 9.8 Now, impulse or area under F-t graph gives the change in
linear momentum.
30 (a) Let u A and u B be the velocities of A and B respectively
F
before impact and v A and v B be the velocities of A and B after
impact.
F0
p−J
A uA B ⇒ A B J
t
Before impact, u B = 0 T/2 T
p = mu A 1 2mu
∴ F0T = mu or F0 =
After impact, p − J = mv A and J = mv B 2 T
v −vA J − (p − J )
Coefficient of restitution, e = B = 36 (a) Here, the force F must be acting on CM of system.
uA − uB p
Let, m1 = m, m 2 = 2m
2J − p 2J
= = −1 D m
p p A x
y1 B
31 (d) m (3$i + 2$j ) = m (−2$i + $j ) + M v l F P

(5$i + $j ) 2m
∴ v= ms −1 (put, M = 13m)
13 l
Impulse, Jm = p f − pi = ± m [(−2$i + $j ) − (3$i + 2$j )] y2
C
∴ Jm = ± m (5i$ + $j ) kg-ms −1 y
368 OBJECTIVE Physics Vol. 1

m × 0 + 2ml 2l l
Taking D as origin, y CM = = 41 (b) We know that, v r =
m + 2m 3 t
2l 4l Now, m (v r − v ) = Mv (v = speed of plank)
From C, CP = 2l − =
3 3 mv r l  m 
∴ v= =  
37 (a) Let the ball be projected vertically downward with M + m t M + m
velocity v from height h.
A 42 (d) Here, 2πR = 2π ⇒ R = 1
v v=0
m × 0 + m × 1+ m × 1 2
∴ y CM = =
m+m+m 3
h m (π ) + m (0 ) + m (2π )
and x CM = =π
m+m+m
43 (b) From conservation of linear momentum,
v 3m
Total energy at point A =
1 2
mv + mgh m1v1 = m 5gl + m1 1 or v1 = 5gl
2 3 2 m1
During collision, loss of energy is 50% and the ball rises up to 44 (a) Here, A1(CC1) = A2 (CC 2 )
same height. This means it possesses only potential energy at
same level. where, A = area of square and A2 = area of remaining portion.
1 
50%  mv 2 + mgh = mgh
2 
11 2  C2 C C1
 mv + mgh = mgh ⇒v = 2gh = 2 × 10 × 20
2 2 
∴ v = 20 ms −1
38 (b) The centre of mass of the object must lie on the line
segment joining (0, 0) and (R / 2, R / 2 ). Here, (0, 0) is the R
Side of square will be .
centre of mass of the ring and (R / 2, R / 2 ) is the centre of 2
mass of the chord. A1 (R / 2 )2  R  R 
∴ CC 2 = (CC1) =   = 
Hence, here option (b) cannot be the coordinate of CM of the A2 πR 2 − (R / 2 )2  2  4π − 2
system.
39 (a) P is the position of centre of mass of particles at 2 and 3. 45 (b) Centre of mass of remaining portion was at point O 2.
Q is position of centre of mass of all three particles. Hence, x 2 (area of remaining portion) = c (area of removed
disc)
y
3 cb 2
∴ x 2 (π a 2 − π b 2 ) = c (π b 2 ) ⇒ x 2 = 2
a − b2
P

Q
θ d d  1 2d
x 46 (b) T = + = 1 + 
1 2 v / 2 ev / 2  e  v
2v / 2  1 2d gd
or = 1 +  or e = 2
g  e v v − gd
y CM m1y1 + m 2 y 2 + m 3 y 3 ∆p ∆ (mn v )
tan θ = = 47 (b) F = = v
x CM m1x1 + m 2x 2 + m 3x 3 ∆t ∆t
6× 0 + 2× 0 + 2×a Here, m = mass of one marble = 5 g = 5 × 10 −3 kg
= = 1 or θ = 45°
6× 0 + 2×a + 2× 0 n
= number of molecules striking per second
40 (c) Velocity of A just before collision ∆t v
= 10
−1
= 2gh = 2 × 10 × 1.25 = 5 ms n
∴ Mg = m   | ∆v | (Q | ∆v | = 2v )
5  ∆t 
Velocity of (A + B ) just after collision = = 2.5 ms −1
2 Here, M = mass of disc
In elastic collision between two bodies of equal masses, (10 × 10 −3 ) (9.8) = (5 × 10 −3 ) (10 ) 2 v
velocities are interchanged.
or v = 0.98 ms −1
Hence, velocity of C will become 2.5 ms −1.
COM, Conservation of Momentum and Collision 369

ev2
mx + 2mx 2m v1
48 (d) X CM = C
m + 2m x
D v1 ⇒ v1
1 x + 2x 2m
∴ a CMt 2 = tAB = x/v1 A
2 3 tCD = x/2v1 v2
3 F  2 B
∴   t = x + 2x 2m [Q F = Ma CM]
2  3m 
v2
Ft 2 x From C to D, v 2 = 2gx = gt ⇒ t =
∴ x 2m = − g
4m 2
From A to B, time will become two times.
49 (a) For collision between A and B,
1
10 ms−1 Applying s = ut + at 2 in vertical direction, we have
A B ⇒ A vA B vB 2
1
− x = (ev 2 ) (2t ) − × g × (2t )2
m × 10 = mv A + mv B ⇒v B + v A = 10 ...(i) 2
1 v −vA 2ev 22 2v 22
e= = B or v B − v A = 5 ...(ii) −x = − ⇒ − x = 2e (2x ) − 2(2x )
2 10 g g
Solving Eqs. (i) and (ii), we get 3
∴ e=
v B = 7.5 ms −1 4
Hence, A has given 75% of its speed to B and B will also 53 (a) After striking with A, the block C comes to rest and block
transfer its 75% speed to C. A moves with velocity v. When compression in spring is
75 maximum, both A and B will be moving with common
∴ vC = × 7.5 = 5.625 ms −1 −
~ 5.6 ms −1
velocity v.
100
π M C A B
 2Mv
50 (b) p net = ∫ dp sin θ = ∫  ⋅ d θ v ⋅ sin θ =
0 π  π m m m

dp
From law of conservation of linear momentum,
90° – v
θ mv = (m + m )V ⇒ V = …(i)
2
dp From law of conservation of energy,
dθ KE of block C = KE of system + PE of system
θ
1 2 1 1
mv = (2m )V 2 + kx 2
2 2 2
2
51 (a) In head on elastic collision, 1 2 1 v  1
⇒ mv = (2m )   + kx 2 [from Eq. (i)]
2 2  2 2
m / 2 − m  2(m )  4
v1 =   (0 ) +   (v ) = 3 v 1 m
m / 2 + m m / 2 + m  ⇒ kx 2 = mv 2 ⇒ x = v
2 2k
m m/2 m/2 m/4
2
v
1 ⇒
v1
(B) Medical entrance special
n −1
format questions
 4
Finally, vn =   ⋅v = 5gr Assertion and reason
 3 l

n −1 | RVOS, i. e . relative velocity of separation |


 3 1 (d) e =
∴ v =  5gr | RVOA, i. e . relative velocity of approach |
 4
In elastic collision, e = 1
52 (b) The given situation is shown below ∴ | RVOS | = | RVOA |
2 (d) If two bodies are released from rest F F
x in space, net force on the system is 1 2
zero. Momentum of system is constant
x/2 x/2 but momentum of individual body is not constant. Further,
x
kinetic energy of system is also increasing.
x/2 x/2
3 (d) Only in case of perfectly inelastic collision, they will come
to rest.
370 OBJECTIVE Physics Vol. 1

4 (a) Linear momentum will not remain constant till spring will 1 80
∴ | v CM | = 16 + 64 = unit
remain compressed. Therefore, a force will act on block A 3 3
from the wall. 2
m (+ v ) + m B (− v ) s1 = ∫ v1 dt = (4$i )
5 (b) v CM = A = − ve 0
mA + mB 8
s2 = ∫
2
v 2 dt =  $j
Both A and B will have same acceleration (µ g) on A towards 0 3 
left and on B towards right. Since, B have more mass, so
8 
acceleration of CM will be towards right. (1) (4$i ) + 2  $j
m1s1 + m 2s 2  3   4 $ 16 $
6 (c) In this case, centre of mass of half filled sphere will Now, s CM = = = i+ j
m1 + m 2 3 3 9 
depend only on radius and not on density of liquid inside.
Since, both spheres are of same radius, so both will have CM 16 256 20
∴ | s CM | = + = unit
at the same level. 9 81 9
l Statement based questions Hence, A → q, B → r, C → p.

1 (b) Packet from train A falls with greater momentum on train 3 (c) p1 + p 2 = p ...(i)
B. Therefore, train B is slightly accelerated while A will be p,K p1 p2
retarded. m 2m ⇒ m 2m
2 (d) Force of friction on A is backward and force of friction on
B is forward. Net external force on the system is zero. Hence, Further, K1 + K2 = K
2
momentum of system will remain conserved. As the p1 p2 p2
momentum of system is conserved, so increase in momentum or + 2 = or 2p12 + p 22 = 2p 2 ...(ii)
2m 4m 2m
of B is equal to decrease in momentum of A.
Solving Eqs. (i) and (ii), we get
3 (d) Due to the same mass of A and B as well as due to elastic 4 p K 8K
collision, velocities of spheres get interchanged after the p 2 = p and p1 = − , K1 = and K2 =
3 3 9 9
collision. So, A comes to rest and B moves with the velocity of A.
Hence, A → r, B → p, C → q, D → s.
4 (d) If centre of mass is at rest, it definitely means momentum
of the system is constant. But if momentum of the system is
constant, it does not mean centre of mass is at rest.
(C) Medical entrances’ gallery
1 (b) Given, m1 = 5 kg, m 2 = 10 kg
5. (d) If kinetic energy of the system is zero, it definitely means
momentum is zero. But if momentum of the system is zero, it and r = 1m = 100 cm
does not mean kinetic energy is zero. Let the centre of mass lies at origin O.
v v
m1 O m2
m m
A r1 r2 B
6 (a) Statement I is correct and Statement II is incorrect and it
r
can be corrected as,
r − m 2r2
m11 r
During collision time, some kinetic energy is stored as ∴ = 0 ⇒ 5r1 − 10r2 = 0 ⇒ r2 = 1
potential energy in the form of deformation. m1 + m 2 2
r1
l Match the columns Also, r1 + r2 = 100 ⇒ r1 + = 100
2
1 (b) When e = 1, collision is elastic and equal masses exchange 200
⇒ 3r1 = 200 ⇒ r1 = −~ 67 cm
their velocities. 3
For e = 0, collision is perfectly inelastic. Hence, velocity of
each will remain half. 2 (c) The given situation as shown in the figure.
3
In the last case, when v 2 = v ′ = v . j
v 4
Then, v1 = (from conservation of momentum) M B
4 3v v

∴ e= 4 4= 1 2m
v 2
Hence, A → q, B → p, C → r. OM
A
i
M
2 (a) F = F + F = m a + m a = (2$i + 8$j )
CM 1 2 11 2 2 2m
∴ | FCM | = 4 + 64 ⇒ | FCM | = 68 unit OA = 2$i
m1v1 + m 2v 2 (1) (4$i ) + (2) (4$j ) 4$i + 8$j OB = 2$j
v CM = = =
m1 + m 2 3 3
COM, Conservation of Momentum and Collision 371

Position vector of centre of mass, ∴Net decrease in kinetic energy of A ,


M r + M2r2 + M3r3 M (OA ) + M (OB) ∆KE = (KE )A − (KE′ )A = 2mu 2 − 2mv12
R CM = 1 1 =
M1 + M2 + M3 M+M+M = 2m (u 2 − v12 )
M × 2i + M × 2 j 2 $ $
$ $ Substituting the value of v1, we get
= = (i + j )
3M 3  u 2  16 mu 2
∆KE = 2m u 2 −  =
3 (a) The given situation of collision can be drawn as  9 9
4m 2m ∴The fractional decrease in kinetic energy,
∆KE 16 mu 2 1 8
u u'= 0 = × 2
=
(KE )A 9 2 mu 9
A B
144442444443 4 (a) Let m be the mass of an object flying with velocity v in air.
Before collision When it gets split into two pieces of masses in ratio 1 : 5, the
4m 2m 5m
mass of smaller piece is m/6 and of bigger piece is .
6
v1 v2
This situation can be interpreted diagrammatically as below.
A B v1
144442444443
After collision m/6

Applying law of conservation of linear momentum, m v


Initial momentum of system = Final momentum of system
5m/6
⇒ (4m )u + (2m )u′ = (4m )v1 + (2m )v 2
v2
4mu + (2m ) × 0 = 4mv1 + 2mv 2
or 2u = 2v1 + v 2 … (i) As, the object breaks in two pieces, so the momentum of the
The kinetic energy of A before collision, system will remains conserved, i.e. the total momentum
(before breaking) = total momentum (after breaking)
1
(KE)A = (4m )u 2 = 2 mu 2 m 5m v 5v 2
2 mv = v 1 + v2 ⇒ v = 1 + …(i)
6 6 6 6
Kinetic energy of B before collision,
(KE)B = 0 Given, v = 20 i$ + 25$j − 12k$
The kinetic energy of A after collision, and v 1 = 100 i$ + 35$j + 8k$
1 Putting these values in Eq. (i), we get
(KE′ )A = (4m )v12 = 2mv12
2 (100 i$ + 35$j + 8k$ ) 5v 2
Kinetic energy of B after collision, (20 $i + 25$j − 12k$ ) = +
6 6
1
(KE′ )B = (2m )v 22 = mv 22 ⇒ (120 $i + 150 $j − 72k$ ) = (100 i$ + 35$j + 8k$ ) + 5v 2
2
1
As, initial kinetic energy of the system = final kinetic energy ⇒ v2 = (20 i$ + 115$j − 80 k$ )
of the system 5
⇒ (KE )A + (KE )B = (KE′ )A + (KE′ )B = 4$i + 23$j − 16k$
2 mu 2 + 0 = 2mv12 + mv 22 5 (d) The particle of mass 5m breaks into three fragments of
2mu 2 = 2mv12 + mv 22 masses m, m and 3m, respectively. Two fragments of mass m
each move in perpendicular directions with velocity v and the
or 2u 2 = 2v12 + v 22 … (ii) left fragment will move in a direction with velocity v′ such
Solving Eqs. (i) and (ii), we get that the total momentum of the system must remain
1 4 conserved.
v1 = u and v 2 = u v
3 3
or the final velocity of A can be directly calculated by using v
5m m m
the formula,
 m − m2  2m 2u 2 v =0
v1 =  1  u1 + 3m
 m1 + m 2  m1 + m 2 v
 4m − 2m  2(2m ) × 0 By law of conservation of momentum,
= u + (Q u 2 = u′ = 0 )
 4m + 2m  (4m + 2m ) 5 m × 0 = mv $i + mv $j + 3mv ′
2m 1 v $ v $
= u= u ⇒ v′ = − i− j
6m 3 3 3
372 OBJECTIVE Physics Vol. 1

2 2 relative velocity of separation


 v  v v 2 e=
∴ | v′ | =  −  +  −  = relative velocity of approach
 3  3 3
v
∴ Energy released, −0
v 2 − v1
1 2 1 2 1 v 2 
2
=− =−4 [from Eq. (i)]
E = mv + mv + × 3m   u 2 − u1 0 −v
2 2 2  3 
= 1/ 4
mv 2 4 2
= mv 2 + = mv ∴ e = 0.25
3 3
11 (b) Velocity of centre of mass,
6 (a) Given, mass of body, m1 = 5 × 10 3 kg mv + m 2v 2 + m 3v 3
v CM = 1 1
Velocity, v1 = 2 ms −1 m1 + m 2 + m 3
and mass of another body, m 2 = 15 × 10 3 kg Given, m1 = 5 kg, v1 = 5 ms −1,
For perfectly inelastic collision, e = 0. m 2 = 4 kg, v 2 = 4 ms −1, m 3 = 2 kg and v 3 = 2 ms −1
∴Loss in kinetic energy of system,
Substituting all these values in above equation, we get
1 m1m 2
∆E K = × v12 5 × 5 + 4 × 4 + 2 × 2 25 + 16 + 4
2 m1 + m 2 v CM = =
5+ 4+ 2 11
1 5 × 10 3 × 15 × 10 3 45 −1
= × × (2)2 ∴ v CM = = 4.09 ≈ 4 ms
2 5 × 10 3 + 15 × 10 3 11
= 7.5 × 10 3 J 12. (a) From law of conservation of momentum,
= 7.5 kJ Mv + m × 0 = Mv1 + mv 2
⇒ M (v − v1) = mv 2 …(i)
7 (b) In elastic collision, total energy, kinetic energy and
momentum remain conserved, therefore no loss in energy Again, from the conservation of kinetic energy (as collision is
of elastic nature),
occurs in elastic collision.
1 1 1 1
Hence, both Assertion and Reason are correct but Reason is Mv 2 + m × 0 = Mv12 + mv 22
2 2 2 2
not the correct explanation of Assertion.
⇒ M (v 2 − v12 ) = mv 22 …(ii)
8 (c) When object of mass 20 kg moving with speed 10 ms −1 in
west direction collides with object of mass 10 kg and both of On solving Eqs. (i) and (ii), we get
them stick together, hence it is perfectly inelastic collision. In M (v − v1) mv 2
=
inelastic collision, only momentum is conserved. M (v + v1)(v − v1) mv 22
9 (c) Speed of objects = u ms −1 v 2 = v + v1 …(iii)
Since, both objects collide at 90°. Now, solving Eqs. (i) and (iii), we get
Hence, by the law of conservation of momentum, (M − m )v
v1 =
Total momentum before collision (M + m )
= Total momentum after collision 2Mv
and v2 =
|mu $i + mu $j| = p f (M + m )
m 2u 2 + m 2u 2 = p f As, M >> m
So, v1 = v and v 2 = 2v
⇒ p f = 2 mu
Hence, velocity of lighter body (m) is 2v.
10 (b) Since, the collision mentioned is an elastic head on 13. (a) If two particles are initially moving in the same direction,
collision. Thus, according to the law of conservation of linear then their resultant momentum will not be zero. Therefore,
momentum, we have their resultant momentum cannot be zero after a completely
m1u1 + m 2u 2 = mv
1 1 + m 2v 2
inelastic collision.
where, m1 and m 2 are the masses of the two blocks As, kinetic energy is directly proportional to the square of the
respectively, u1 and u 2 are their initial velocities and v1 and v 2 momentum, hence kinetic energy cannot be zero. This
are their final velocities, respectively. implies, not all the energy in inelastic collision is lost.
Given, m1 = m, m 2 = 4m Hence, both Assertion and Reason are correct and Reason is
u1 = v, u 2 = 0 and v1 = 0 the correct explanation of Assertion.
∴ mv + 4m × 0 = 0 + 4mv 2 14 (c) Given, m = 0.5 kg, v = 12 ms −1, ∆t = 1s
⇒ mv = 4mv 2 or v 2 =
v
…(i) and θ = 30 °
4 Force applied by wall on ball,
Now, the coefficient of restitution,
COM, Conservation of Momentum and Collision 373

∆p (p f )H − (pi )H v 02
F = or F = ∴ mv 02 = kx 02 + (3m )
∆t ∆t 9
mv 02 2mv 02
m ⇒ kx 02 = mv 02 − ⇒ kx 02 =
3 3
2mv 02
∴ k=
θ 3x 02
θ 1 1
18 (c) Total initial energy = m1u12 + m 2 u 22
2 2
Since, after collision one particle absorbs energy ε.
m 1 1
∴ Total final energy = m1v12 + m 2v 22 + ε
Q In this elastic collision, final and initial velocities will be 2 2
same but direction changes From conservation of energy,
(p f )H = mv cos θ 1 1 1 1
m1u12 + m 2 u 22 = m1v12 + m 2v 22 + ε
and (pi )H = −mv cos θ 2 2 2 2
mv cos θ + mv cos θ 2mv cos θ 1 1 1 1
∴ F = = ⇒ m1u1 + m 2 u 2 − ε = m1v1 + m 2v 22
2 2 2

∆t ∆t 2 2 2 2
2 × 0.5 × 12 × cos 30 ° 19 (b) Applying the law of conservation of momentum to the
F = = 6 3N
1 1
system, mgh = kx 2
1 m12u12 m1m 2u12 2
15 (c) Loss of kinetic energy = m1u12 − = Given, m = 0.04 kg, h = 5 m, k = 400 Nm −1
2 2 (m1 + m 2 ) 2 (m1 + m 2 )
and x = deformation (compression) in the spring
Given, m1 = 4 kg,u1 = 12 ms −1,
2mgh 2 × 0.04 × 10 × 5
m 2 = 6 kg and u 2 = 0 ⇒ x= =
k 400
1 4×6 1 24
∴ ∆ KE = (12)2 = × × (12)2 1
2 (4 + 6) 2 10 = m = 10 cm ≈ 9.8 cm
10
12
= × 144 = 172.8 J 20 (a) Let the velocity of block of mass 2 m after the collision be
10
v′, then from law of conservation of momentum,
16 (c) Given, masses, m1 = 6 unit and m 2 = 2 unit
v
Positions = 6$i − 7$j and 2$i + 5$j − 8k$ mv = 2mv ′ ⇒ v ′ =
2
Coordinates of centre of mass are calculated below Now, the coefficient of restitution,
m x + m 2x 2 6 × 6 + 2 × 2 36 + 4 velocity of separation v ′ v / 2 1
x CM = 1 1 = = = 5$i e= = = = = 0.5
m1 + m 2 6+ 2 8 velocity of approach v v 2
m y + m 2 y 2 6 × (− 7) + 2 × (5)
y CM = 1 1 = 21 (b) Let the maximum compression in the spring be x. From
m1 + m 2 6+ 2
1
− 42 + 10 law of conservation of energy, mgh = kx 2
= = – 4$j 2
8
1
m z + m 2z 2 6 × (0 ) + 2 × (− 8) − 16 10 × 10 −3 × 10 × 10 = × 200 × x 2
and z CM = 1 1 = = = − 2k$ 2
m1 + m 2 6+ 2 8
⇒ x 2 = 10 × 10 −3 = 10 −2
∴ Centre of mass lies at 5$i – 4$j − 2k$ .
1 1
⇒ Coordinates of centre of mass are (5, −4, −2). ⇒ x= = m = 0.1 m
100 10
17 (d) Using the law of conservation of linear momentum, we Thus, compression in the spring is 0.1m.
have
22 (c) Let the speed of the board be u and frog jumps with angle
mv 0 = mv + 2mv ⇒ v = v 0 / 3
of inclination to the board θ, then from law of conservation of
momentum in horizontal direction,
m m 2m mv cos θ − mu = 0, u = v cos θ …(i)
C v0 A B Let distance moved by board be x.
So, L − x = ut …(ii)
Using law of conservation of energy, we have and x = v cos θ t …(iii)
1 2 1 2 1 Solving above equations, we get
mv 0 = kx 0 + (3m )v 2
2 2 2 L
x=
where, x 0 is compression in the spring. 2
374 OBJECTIVE Physics Vol. 1

v 2 sin 2θ 27 (b) mv
Also, x= +Y
g
L v 2 sin 2 θ
⇒ = 45°
2 g −X mv
45° +X
gL
⇒ v=
2 sin 2θ v′
−Y
Hence, v should be minimum for sin 2 θ = 1(i.e. maximum)
gL 10 2(mv ) = Resultant momentum of two small masses
⇒ vmin = = 10 × = 50
2 2 v
2 (mv ) = (2m ) × v ′ ⇒ v ′ =
= 5 2 ms −1 2
So, total kinetic energy generated by the explosion
23 (b) As, net horizontal force acting on the system is zero, hence
1 1 1
momentum must remain conserved. = mv 2 + mv 2 + (2m ) v ′ 2
Hence, mu + 0 = 0 + Mv 2 2 2 2
2
mu  v  mv 2 3
⇒ v2 = = mv 2 + m ×   = mv 2 + = mv 2
M  2 2 2
| (v 2 − v1)| |v 2 − 0|
Coefficient of restitution, e = = 28 (d) Given, p = a + bt + ct 2
| (u 2 − u1)| |0 − u|
Differentiating with respect to t, we get
mu
dp
v2 M m = 0 + b + 2ct
= = = dt
u u M
dp
24 (c) When a body falls from height h, it strikes the ground with From Newton’s second law of motion, F ∝
dt
a velocity u = 2gh . Let it rebounces with a velocity v and
⇒ F ∝ t or force is dependent linearly on time.
rise to a height h1.
v h 29 (d) The position of centre of mass of a system of particles does
v = 2gh1 ⇒ e = = 1 not depend upon the nature of particles.
u h Σmi ri
rCM =
Clearly, h1 = e 2h Σmi
25 (a) The figure given in question is 30 (b) We have, p1 + p 2 + p 3 = 0 (Q p = mv )
Y ∴ 1 × 12 $i + 2 × 8$j + p 3 = 0
C 1.5 kg
⇒ 12$i + 16$j + p 3 = 0 ⇒ p 3 = − (12$i + 16$j )
5c
(0,

3 cm m ∴ p 3 = (12)2 + (16)2 = 144 + 256


3)

= 20 kg-ms −1
(0, 0) (4, 0)
A 4 cm B X p 3 20
Now, p 3 = m 3v 3 ⇒ m 3 = = = 5 kg
0.5 kg 1 kg v3 4
Let X andY-axes be along AB and AC, respectively. The
31 (c) After firing, the momentum of gun and bullet is same.
coordinates of centre of mass would be
p2
1.5 × 0 + 1 × 4 + 1.5 × 0 4 Therefore, by the relation, K =
XCM = = = 1.3 2m
1.5 + 0.5 + 1 3 1
We have, K∝ (As p is same)
0.5 × 0 + 4 × 0 + 1.5 × 3 1.5 × 3 m
and YCM = = = 1.5
1.5 +1 +0.5 3 As, the mass of gun is greater, hence its kinetic energy will be
Hence, (X CM, YCM ) = (1.3 , 1.5) less.
26 (b) As large number of particles are situated at a distance R 32 (c) In all types of collisions, total linear momentum of
from the origin. If particles are uniformly distributed and colliding particles remains conserved.
make a circular boundary around the origin, then centre of 33 (d) Given, mass = m
mass will be at the origin.
ma $i + mb $j + m (0 ) k$
While, if the particles are not uniformly distributed, then Position of centre of mass, rCM =
centre of mass will lie between particle and origin. This 3m
implies that the distance between centre of mass and origin is 1 $
always less than or equal to R. ⇒ rCM = (a i + b$j )
3
COM, Conservation of Momentum and Collision 375

34 (b) From the law of conservation of momentum, 40 (a) The given condition as shown in the figure below.
mv + m × 0 = (m + m )v ′ mv1 sin θ
m
mv v
⇒ v′ = ⇒ v′ =
(m + m ) 2 m m
30° mv1 cos θ
2
u 30° mv2 cos θ
and we know that, h = (Q v − u = 2gh )
2 2 Before
2g collision
m
mv2 sin θ
Here, u = v′
After collision
(v ′ )2  v
So, h= Q v ′ =  After collision, mv1 sin θ = mv 2 sin θ ⇒ v1 = v 2 = v
2g  2
∴ 2 mv cos θ = m × 9
v2
⇒ h= ⇒ 2v cos 30 ° = 9
8g 9
⇒ v= = 3 3 = 5.2 ms −1
35 (b) In an inelastic collision, kinetic energy before collision is 3
not equal to kinetic energy after collision. But the linear
41 (c) Velocity of centre of mass,
momentum is conserved in all types of collisions.
dr dr
36 (c) Initial momentum, p = m v $i + m × 0 m1 1 + m 2 2
dt = 4 × 5 i + 2 × 10 i = 40 i = 20 $i
$ $ $
i 2 1
v CM = dt
y m1 + m 2 4+ 2 6 3
v/2 The kinetic energy of centre of mass,
B 1 2 1 20 × 20
v K = mv CM = × (4 + 2) ×
θ x 2 2 3×3
B A
(m2) Rest A v1 1 20 × 20 400
(m1) = ×6× = J
2 3×3 3
Final momentum, p f = m 2 (v / 2) $j + m1 × v1 42 (b) From law of conservation of momentum,
u 1
From law of conservation of momentum, mu = (m + m ) v ⇒ v = = m/s
pi = p f 2 2
v Now, maximum height reached by the system,
m 2v $i = m 2 $j + m1 × v1
2 v2 1 1 5
h= = = × 100 = = 1.25 cm
m2 $ m2 v $ 2g 8g 80 4
v1 = vi− j
m1 m1 2
43 (c) The centre of mass of the system,
1
From this equation, we can find, tan θ = m x + m 2x 2
2 rCM = 1 1
m1 + m 2
 1
⇒ θ = tan−1   to the X-axis. m1 m2
 2 O
x1
37 (c) Here, net external force on the entire system is zero, x2
hence position of centre of mass remains unchanged.
44 (a) Final kinetic energy of the block along with the bullet
38 (d) Using law of conservation of momentum, we get 1
100 × v = 0.25 × 100 when the bullet strikes = (m + M )v 2
2
⇒ v = 0.25 ms −1 Due to this kinetic energy, the block will rise to a height h.
39 (b) Initial momentum of the system Its potential energy = (m + M ) gh
= m1 × 40 + m 2 × 0 = 40 m1 So, from the law of conservation of energy,
1
Final momentum of the system (M + m )v 2 = (M + m )gh ⇒ v = 2gh
= (m1 + m 2 ) × 30 2
By the law of conservation of momentum, 45 (b) Given, mass, m1 = m 2
Initial momentum = Final momentum Velocity, u1 = v1 and u 2 = 0
40m1 = (m1 + m 2 )30  m − m1 2m1u1
40 m1 − 30 m1 = 30 m 2 For elastic collision, v 2 =  2  u2 +
 m1 + m 2  m1 + m 2
⇒ 10 m1 = 30 m 2
After putting the given values, we get
m1 3
⇒ = 2m v
m2 1 v 2 = 1 1 ⇒ v 2 = v1
2m1
CHAPTER
09

Rotation
In this chapter, we will discuss rotational motion, i.e. the motion of an object
around its own axis and the variables which describe rotational motion. Motion of
wheels, gears, motors, planets, the hands of a clock, the rotor of jet engines,
blades of helicopters, etc. are examples of rotational motion.

Rigid body
Ideally, a body is said to be a rigid body when it has a definite shape and size. For
a rigid body, it is considered that its constituent particles do not change their
mutual distances even when an external force is applied on it.
e.g. A wheel can be considered as rigid body by ignoring a little change in its
shape.

Motion of rigid body


A rigid body can possess pure translational motion, pure rotational motion or a
combination of both of these motions.
Inside
In previous chapters, we have discussed translational motion, i.e. the motion of 1 Rotational motion
an object along a line (both straight and curved). So, here we will study about the (Fixed axis of rotation)
rotational motion of a body. Moment of inertia
Radius of gyration
Theorems on moment of inertia
ROTATIONAL MOTION 2 Torque or moment of force
3 Angular momentum
(FIXED AXIS OF ROTATION)
Angular momentum of a
In any kind of motion of a body, there is either change in position or change in particle about some point
orientation or change in both. If a body changes its orientation during its motion, Angular momentum of a
it is said to have rotational motion. rigid body rotating about a
fixed axis
In pure rotational motion, every particle of the rigid body moves in circles of Law of conservation of angular
different radii about a fixed line, which is known as axis of rotation. momentum
4 Rotational kinetic energy of
e.g. In case of motion of a ceiling fan, the vertical rod supporting the fan (here,
the particles
axis of rotation) remains stationary and all the particles on the fan move in
Work-energy theorem in
circular paths. The centres of circular paths are on the axis of rotation. rotational motion
Also, it is not necessary that axis of rotation passes through the body. 5 Rolling motion
Classification of rolling motion
Physical quantities required to describe rotational motion Kinetic energy of rolling motion
and equations of rotational motion Rolling down on a rough
All the physical quantities required to describe rotational motion are same as in inclined plane
circular motion.
Similarly, equations of rotational motion are same as that of circular motion.
Rotation 377

Example 9.1 A ring of diameter 0.4 m and of mass 10 kg is


Moment of inertia rotating about its geometrical axis at the rate of
Moment of inertia plays the same role in rotational motion 35 rotations/second. Find the moment of inertia.
as mass plays in translational motion. It is the property of Sol. Moment of inertia of the ring about its axis,
a body due to which it opposes any change in its state of I = MR 2
rest or uniform rotation.
I = 10 × (0.2)2 = 10 × 0.04 = 0.4 kg-m2
The moment of inertia is calculated about some axis
(usually the rotational axis) and it depends on the mass as Example 9.2 Three point masses m1, m 2 and m 3 are located
well as its distribution about that axis. at the vertices of an equilateral triangle of side a. Determine
the moment of inertia of the system about an axis along the
Moment of inertia of a single particle altitude of the triangle passing through m1.
The moment of inertia of a single particle about an axis is Sol. As shown in the figure given below, the axis of rotation
passes through mass m1. The distances of m1, m 2 and m 3 from
given by I = mr 2 …(i)
the axis of rotation are 0, a /2, a /2, respectively.
Axis of rotation
m1
r
m
a a

m2 m3
Fig. 9.1 a/2 a/2
Here, m is the mass of the particle and r is distance from
Therefore, moment of inertia of the system about the axis
the axis under consideration.
passing through m1 is
The SI unit of moment of inertia is kg-m 2 . Its dimensions I = m11
r 2 + m 2r22 + m 3r32
are [ML 2 ]. 2 2
a  a 
= m1(0)2 + m 2   + m 3  
Moment of inertia of a system of particles  2  2
The moment of inertia of a system of particles about an a2
⇒ I= (m 2 + m 3 )
axis is given by I = Σ mi ri 2 …(ii) 4
i
r1
Example 9.3 Calculate the moment of inertia w.r.t. rotational
m1 axis XX ′ in following figures.
r2 Y
m2 4 kg (0, 3)
r3 Massless rod
m3 X′ X
X
1 kg 4 kg
2 kg (0, −2)
Fig. 9.2
20 cm 70 cm X′
where, ri is the perpendicular distance from the axis of the 100 cm 3 kg (0, −4)
ith particle of mass mi . Fig. (i) Fig. (ii)

Moment of inertia of rigid bodies Sol. (i) Moment of inertia about axis XX ′,
For a continuous mass distribution found in a rigid body, I XX′ = m11
r 2 + m 2r22 = 4 × (1 − 0.7)2 + 1 × (1 − 0.2)2
we replace the summation of Eq. (ii) I XX′ = 4 × (0.3)2 + 1 × (0.8)2 = 1 kg-m2
by an integral. If the system is divided (ii) Moment of inertia about axis XX ′,
into infinitesimal elements of mass dm r
I XX′ = 4 × (3)2 + 2 × (2)2 + 3 × (4)2 = 92 kg-m2
and if r is the distance from a mass dm
element to the axis of rotation, then Example 9.4 Find the moment of inertia of a uniform cylinder
moment of inertia is about an axis through its centre of mass and perpendicular
I = ∫ r 2dm to its base. Mass of the cylinder is M and radius is R.
Fig. 9.3 Sol. The cylinder can be divided into annular shells of width dr
where, the integral is taken over the and length l as shown in figure.
system.
378 OBJECTIVE Physics Vol. 1

The moment of inertia of one of these shells, Example 9.5 A wheel of mass 8 kg has moment of inertia equals
I = ∫ r 2 dm = r 2 (ρ dV ) to 0.5 kg-m 2 . Determine its radius of gyration.
Sol. Given, mass, M = 8kg
Moment of inertia, I = 0.5 kg-m2

dr Q I = MK 2
R O
l
r I
⇒ K2 =
M
I
⇒ K=
Here, ρ = density, dV = volume of shell = 2πrl dr M
∴ I = 2πρlr 3 dr 0.5
⇒ Radius of gyration, K =
The cylinder’s moment of inertia is found by integrating this 8
expression between 0 and R ⇒ K = 0.25 m
R πρl 4 K = 25 cm
So, I = 2πρl ∫ r 3 dr = R …(i) or
0 2
Example 9.6 The moment of inertia of a solid cylinder about
The density ρ of the cylinder is the mass divided by the
MR 2
volume, its axis of rotation is . What is the value of the radius
M 2
i.e. ρ= …(ii) of gyration of the cylinder about this axis?
πR 2l
Sol. Moment of inertia of solid cylinder,
From Eqs. (i) and (ii), we have
1 MR 2
I = MR 2 I= ...(i)
2 2
Also, moment of inertia,
Radius of gyration I = MK 2 ...(ii)
The radius of gyration of a body about a given axis is the Comparing Eqs. (i) and (ii), we get
perpendicular distance of a point from the axis at which R2
K2 =
the whole mass of the body could be concentrated without 2
any change in the moment of inertia of the body about R
⇒ K=
that axis. 2

Axis of rotation
Theorems on moment of inertia
M

K M There are two important theorems on moment of inertia,
which enable calculation of the moment of inertia of a
body about an axis, if its moment of inertia about some
other axis is known. Let us now discuss both of them.
Fig. 9.4
Theorem of parallel axes
If a body has mass M and radius of gyration K, then It states that moment of inertia of a rigid body about any
moment of inertia, axis is equal to the sum of its moment of inertia about a
I = MK 2 ⇒ K = I /M parallel axis through its centre of mass, the product of the
mass of the body and the square of the perpendicular
Radius of gyration is also defined as the root-mean- distance between the two axes.
square-value of distances of all the particles about the axis
of rotation.
r12 + r 22 + r 32 + K + rn2
i.e. K = CM
n
It depends on the shape and size of the body, position and
configuration of the axis of rotation, distribution of mass of
R
the body with respect to the axis of rotation.
Radius of gyration does not depend on the mass of body. Fig. 9.5
Rotation 379

Two such axes are shown in figure for a body of mass Let X andY-axes be chosen in the plane of the body and
M. If R is the distance between the axes and I CM and I Z-axis perpendicular to this plane, three axes being mutually
are the respective moments of inertia about these axes, perpendicular, then according to the theorem,
then I Z = I X + IY
I = I CM + MR 2 Z Y
Note From the above theorem, we can conclude that among many
parallel axes, moment of inertia is least about an axis which
passes through centre of mass, e.g. I 2 is least among I1 , I 2 and I 3. xi
P
Similarly, I5 is least among I 4 , I5 and I 6 . ri
yi
1 X
2 O
3 4
5
CM CM Fig. 9.7
6

where, I X , IY and I Z are the moments of inertia about the


Fig. 9.6 X,Y and Z-axes respectively.
Note
Theorem of perpendicular axes (i) Theorem of parallel axes is applicable for any type of rigid body
whether it is a two dimensional or three dimensional, while the
It states that the moment of inertia of a plane lamina theorem of perpendicular axes is applicable for laminar type or two
about an axis perpendicular to its plane is equal to the dimensional bodies only.
sum of the moments of inertia of the lamina about any (ii) In theorem of perpendicular axes, the point of intersection of the three
two mutually perpendicular axes in its plane and axes (X, Y and Z) may be any point on the plane of body (it may even
intersecting each other at the point, where the lie outside the body). This point may or may not be the centre of mass
of the body.
perpendicular axis passes through it.

Moment of Inertia of Simple Geometrical Objects


Moment of
S.No. Body Axis of Rotation Figure
Inertia
K K 2 /R 2

(i) Thin circular ring, (a) About an axis passing MR 2 R 1


radius R through CG and
perpendicular to its plane
R

(b) About its diameter 1 R 1


MR 2
2 2 2

(ii) Circular disc, (a) Perpendicular to disc at 1 R 1


MR 2
radius R centre 2 2 2

(b) About its diameter 1 R 1


MR 2
4 2 4
380 OBJECTIVE Physics Vol. 1

Moment of
S.No. Body Axis of Rotation Figure
Inertia
K K 2 /R 2

(iii) Long uniform thin rod (a) About an axis passing ML2 L
through its centre of mass 12 12
and perpendicular to the
rod
L

(b) About an axis passing


through its edge and ML2 L
perpendicular to the rod 3 3
L

(iv) Rectangular lamina Passing through the centre M 2


[l + b 2] l2 + b2
of mass and perpendicular b
12 12
to the plane
l

(v) Hollow cylinder, About its own axis


radius R
R

MR 2 R 1
L

Axis

(a) Tangential line passing


(vi) Cylindrical shell through its surface parallel 2MR 2 2R 2
to its axis

(b) About an axis passing


through its CG and
perpendicular to its own  L2 R 2  L2 R 2
M +  +
axis 12 2 12 2

 L2 R 2  L2 R 2
(c) About the diameter of M +  +
one of faces of the cylinder 3 2 3 2
Rotation 381

Moment of
S.No. Body Axis of Rotation Figure
Inertia
K K 2 /R 2

1 R 1
(vii) Solid cylinder (a) About its own axis MR 2
L 2 2 2

Axis

(b) Tangential line passing 3 3


MR 2 3
through its surface and R
2 2 2
parallel to its axis

L
(c) About an axis passing
through its CG and  L2 R 2 
M + L2 R 2
perpendicular to its own R  +
axis 12 4 12 4

(d) About the diameter of  L2 R 2  L2 R 2


M +  +
one of the faces of the R
3 4 3 4
cylinder

2 2 2
(viii) Spherical shell (a) About its diametric axis MR 2 R
3 3 3

5 5 5
(b) About a tangential axis MR 2 R
3 3 3
382 OBJECTIVE Physics Vol. 1

Moment of
S.No. Body Axis of Rotation Figure
Inertia
K K 2 /R 2

2 2 2
(ix) Solid sphere, radius R (a) About its diametric axis MR 2 R
5 5 5

7 7 7
(b) About a tangential axis MR 2 R
5 5 5

(x) Hollow sphere of inner (a) About its diametric axis 2  R 25 − R15 
radius R1 and outer M  3 3
radius R 2
5  R 2 − R1 

2  R 25 − R15 
 + MR 2
2
M
(b) Tangential 5  R 23 − R13 

Example 9.7 Find the moment of inertia of a sphere about a Applying the theorem of parallel axes, the moment of inertia
tangent to the sphere, while the mass of the sphere is M and of the cylinder about tangent (XX ′ ),
the radius of the sphere is R. 1
Y X It = I + MR 2 = MR 2 + MR 2
Sol. The moment of inertia of a sphere about its 2
2MR 2 3
( ′ ) is
own axis YY . ⇒ It = MR 2
5 2
Applying the theorem of parallel axes, the Here, M = 25 kg and R = 5 m
moment of inertia of the sphere about tangent 3
(XX ′ ), ∴ It = (25) (5)2
Y′ X′ 2
It = I + MR 2
= 937.5 kg-m2
2 7
It = MR 2 + MR 2 = MR 2
5 5 Example 9.9 Find the moment of inertia of a solid sphere of
mass 5kg and radius 2m about an axis XX′ as shown in figure.
Example 9.8 The mass of the cylinder is 25 kg and radius of
cylinder is 5 m. Find the moment of inertia of a solid X
cylinder about a tangent to the cylinder. Y X
Sol. The moment of inertia of a cylinder about its
( ′)
own axis YY
1
= MR 2
2
X′
Y′ X′
Rotation 383

Sol. From theorem of parallel axes, the moment of inertia of Ml 2 + 3Ml 2 4Ml 2 Ml 2
solid sphere is = = =
12 12 3
X
Ml 2 2(5)2 50
ICD = = = kg-m2
3 3 3
Example 9.12 Two masses m1 and m 2 are placed at a
distance r from each other. Find out the moment of inertia of
X′ system about an axis passing through their centre of mass.
r=R
Sol. We have, m1r1 = m 2r 2 and r1 + r2 = r
IXX′ = I CM + MR 2
m 2r m1r
2 7 7 ⇒ r1 = , r2 =
= MR 2 + MR 2 = MR 2 = (5)(2)2 = 28 kg-m2 m1 + m 2 m1 + m 2
5 5 5
r
Example 9.10 What is the moment of inertia of a m1 m2
(i) uniform circular ring of mass 2 kg about its 4 m diameter? CM
r1 r2
(ii) a thin disc of mass M and radius R about an axis coinciding
with a diameter? Moment of inertia of the system about an axis passing through
Sol. (i) The moment of inertia of a uniform ring about an axis their centre of mass,
passing through its centre and perpendicular to its plane is 2 2
 m 2r   m1r 
given by I = MR 2. I = m1r12 + m 2r 22 = m1   + m2 
 m1 + m 2   m1 + m 2 
According to the theorem of perpendicular axis, I Z = I X + IY
Z  mm 
Y =  1 2  r2
 m1 + m 2 
X Note Here, I = µr 2, where µ is called reduced mass which is equal to
m1 m2
.
m1 + m2

Now, X andY -axes are along the diameter of the disc. By Example 9.13 Two identical rods each of
symmetry, I X = IY ⇒ I Z = 2I X ⇒ I Z = MR 2 mass M and length L are kept according to
figure. Find the moment of inertia of rods
The moment of inertia of ring about of its diameter, about an axis passing through O and
MR 2 (2)(2)2 perpendicular to the plane of rods. O
IX = = = 4 kg-m2
2 2 Sol. Moment of inertia of each rod about an axis passing through
(ii) Similarly, moment of inertia about the diameter of disc, ML2
an end =
1 1  1 3
I =  MR 2 = MR 2 According to perpendicular axes theorem,

2 2  4
ML2 ML2 2 ML2
∴ I given system = + =
Example 9.11 Calculate the moment of inertia of a rod of 3 3 3
mass 2 kg and length 5 m about an axis perpendicular to it Example 9.14 Consider a uniform rod of mass m and length
and passing through one of its ends. 2l with two particles of mass m each at its ends. Let AB be a
Sol. For the rod of mass M and length l, the moment of inertia of line perpendicular to the length of the rod and passing
the rod about an axis AB passing through its centre of mass, through its centre. Find the moment of inertia of the system
MI 2 about AB.
I AB =
12 A
A C
l l
CG
m m
l
l 2 B
B D
Sol. Moment of inertia of the system about AB,
According to the parallel axes theorem, I AB = I rod + I both particles
2
l Ml 2 Ml 2 m (2l )2 7
ICD = I AB + M   = + = + 2 (ml 2) = ml 2
 2 12 4 12 3
384 OBJECTIVE Physics Vol. 1

Example 9.15 Three rods each of mass m and length l are Sol. Taking origin at O, the centre of mass will be located at
joined together to form an equilateral triangle as shown in L  3L 
M × + 3M × L + 
figure. Find the moment of inertia of the system about an axis 2  2
passing through its centre of mass and perpendicular to the x CM = = 2L
M + 3M
plane of the triangle.
A is CM of system of rods.
A
O
A
L/2 2L – L/2
(5L/2 – 2L)
2L
CM 5L/2
O
So, using theroem of parallel axes,
B C ML2  L 
2
I A = I1 + I 2 =  + M 2L −  
Sol. Moment of inertia of rod BC about an axis perpendicular to
 12  2 
plane of triangle ABC and passing through the mid-point of 
rod BC (i. e . D ) is 3M (3L )2  5L  
2
+ + 3M  − 2L 
A  12  2  

 1 9  27 3
= ML2  +  +  + 
12 4 12 4 
CM
O 16ML2
r =
3
30°
B C
D Example 9.17 Two thin uniform rings made of same material
and of radii R and 4R are joined as shown in figure. The
ml 2 l2 3 mass of smaller ring is m. Find the moment of inertia about
I1 = , in ∆BDO, AD = l 2 − = l an axis passing through the centre of mass of system of rings
12 4 2
and perpendicular to the plane.
AD 3l l
r = OD = = =
3 3×2 2 3
From theorem of parallel axes, moment of inertia of this rod about O1 A O2
O
the axis passing through CM and perpendicular to planeABC is R 4R
2
ml 2  l  ml 2
I 2 = I1 + mr 2 = +m   =
12 2 3 6
∴ Moment of inertia of all the three rods, Sol. Here, m1 = m and m 2 = 4 m (Q m ∝ radius)
 ml 2  ml 2 Location of the centre of mass from O,
I = 3I 2 = 3   = m × R + 4 m (2R + 4R )
 6  2 x CM = = 5R
m + 4m
Example 9.16 Two thin uniform rods A (M, L) and B (3M, 3L)
A is CM of system of rings, so using theorem of parallel axes,
are joined as shown in figure. Find the moment of inertia
about an axis passing through the centre of mass of the I A = I1 + I 2 = {mR 2 + m (5R − R )2 }
system of rods and perpendicular to the length. + {4m (4R )2 + 4m (6R − 5R )2 }
M 3M = mR 2 [(1 + 16) + (64 + 4)]
L 3L I A = 85mR 2
CHECK POINT 9.1
1. A wheel rotates with a constant angular velocity of 11. A particle of mass 1 kg is kept at (1m, 1m, 1m). The
300 rpm. The angle through which the wheel rotates in 1 moment of inertia of this particle about Z-axis would be
s is (a) 1 kg-m2 (b) 2 kg-m2
(a) π rad (b) 5π rad (c) 3 kg-m2 (d) None of these
(c)10π rad (d) 20π rad
12. Three thin rods each of length L and mass M are placed
2. A motor is rotating at a constant angular velocity of along X, Y and Z -axes such that one end of each rod is at
500 rpm. The angular displacement per second is origin. The moment of inertia of this system about Z-axis is
3 3π 2 4 ML 2
(a) rad (b) rad (a) ML 2 (b)
50π 50 3 3
25π 50 π 5ML 2 ML 2
(c) rad (d) rad (c) (d)
3 3 3 3
3. A body rotating with uniform angular acceleration covers 13. The radius of gyration of a solid sphere of radius R about its
100 π rad in the first 5 s after the start. Its angular speed at tangential axis is
the end of 5 s (in rad/s) is 7 2
(a) 40 π (b) 30 π (a) R (b) R
(c) 20 π (d)10 π 5 5
5
4. A rotating wheel changes angular speed from 1800 rpm to (c) R (d) R
7
3000 rpm in 20 s. What is the angular acceleration,
assuming it to be uniform? 14. Moment of inertia of a rod of mass m and length l about its
(a) 60π rad s–2 (b) 90π rad s–2 one end is I. If one-fourth of its length is cut away, then
moment of inertia of the remaining rod about its one end
(c) 2π rad s–2 (d) 40π rad s–2
will be
5. The angular velocity of a wheel increases from 100 to 300 3 9
(a) I (b) I
rad/s in 10 s. The number of revolutions made during that 4 16
time is 27 I
(c) I (d)
(a) 600 (b) 1500 64 16
(c) 1000 (d) 318 15. One circular ring and one circular disc both having the
6. A wheel has angular acceleration of 3 rad s −2 and an initial same mass and radius. The ratio of their moments of inertia
angular speed of 2 rad s −1. In a time of 2 s, it has rotated about the axes passing through their centres and
through an angle (in rad) of perpendicular to planes will be
(a) 6 (b) 10 (c) 12 (d) 4 (a) 1 : 1 (b) 2 : 1
(c) 1 : 2 (d) 4 : 1
7. A body rotates about a fixed axis with an angular
16. The moment of inertia of a solid cylinder of mass M, length
acceleration of 3 rad s −2. The angle rotated by it
2R and radius R about an axis passing through the centre of
during the time when its angular velocity increases from 10 mass and perpendicular to the axis of the cylinder is I1 . For
rad s −1 to 20 rad s −1 (in rad) is the same cylinder, its value about an axis passing through
(a) 50 (b) 100 one end of the cylinder and perpendicular to the axis of
(c) 150 (d) 200 cylinder is I 2. The relation between I1 and I 2 is
8. A wheel which is initially at rest is subjected to a constant (a) I 2 − I1 = MR 2 (b) I 2 = I1
angular acceleration about its axis. It rotates through an I 19
angle of 15° in time t second. The increase in angle through (c) 2 = (d) I1 − I 2 = MR 2
I1 12
which it rotates in the next 2 t second is
(a) 90° (b) 120° 17. I1 is the moment of inertia of a thin rod about an axis
(c) 30° (d) 45° perpendicular to its length and passing through its centre of
mass. If I 2 is the moment of inertia of the ring formed by
9. Analogue of mass in rotational motion is
bending the rod about an axis perpendicular to the plane of
(a) moment of inertia (b) angular momentum
ring and passing through its centre, then
(c) radius of gyration (d) None of these I1 3 I1 2
(a) = (b) =
10. Moment of inertia of a body depends upon I2 π 2 I2 π 2
(a) axis of rotation (b) torque I π2 I π2
(c) angular momentum (d) angular velocity (c) 1 = (d) 1 =
I2 2 I2 3
386 OBJECTIVE Physics Vol. 1

TORQUE OR MOMENT OF FORCE


Torque is a quantity which measures the capability of a force Example 9.20 The body in figure shown is pivoted at O and
to rotate a body. Torque due to a force is also known as the two forces act on it as shown.
moment of a force. Torque is the rotational analogue of
force of linear motion. θ1 θ2
r1 r2
It is defined as the product of the
force and the perpendicular distance F F1 O F2
r
between the line of action of the O
θ A
r⊥
force and the axis of rotation. This (i) Find an expression of the net torque on the body about the
perpendicular distance is known as pivot.
the force arm. Fig. 9.8 (ii) If r1 = 1.30 m, r2 = 2.15 m, F1 = 4.20 N, F 2 = 4.90 N, θ1 = 75°
Torque, τ = Force arm × Force and θ 2 = 60°, what is the net torque about the pivot?
τ = r×F Sol. (i) Take a torque that tends to cause a counter clockwise
rotation from rest to be positive and a torque that tends
τ = rF sin θ to cause a clockwise rotation from rest to be negative.
Thus, a positive torque of magnitude r1F1 sin θ1 is
where, θ is the angle between r and F.
associated with F1 and a negative torque of magnitude
The SI unit of torque is N-m and its dimensions are r 2F 2 sin θ 2 is associated with F 2. Both of these are about
[ML2 T −2 ]. O. The net torque about O is τ = r1F1 sin θ1 − r 2F 2 sin θ 2.
Torque is an axial vector, i.e. its direction is always (ii) Substituting the given values in above equation, we get
perpendicular to the plane containing vectors r and F in τ = (1.30 m)(4.20 N) sin75°− ( 2.15 m) (4.90 N) sin60°
accordance with right-hand screw rule. = − 3.85 N-m
There is a mathematical relation between torque (τ ), Example 9.21 A small ball of mass 1 kg is attached to one
angular acceleration (α ) and moment of inertia (I ) and that end of a 1 m long massless string and the other end of the
can be given as τ = Iα string is hung from a point. When the resulting pendulum is
The above expression is similar to the expression of Newton’s 30° from the vertical, what is the magnitude of torque about
the point of suspension? [Take, g = 10 ms −2 ]
second law for translational motion with constant mass.
Note Sol. Two forces are acting on the ball
(i) If θ = 0° or 180°, the torque of force is zero. (i) tension ( T ) (ii) weight (mg )
(ii) τ = r × F is a vector product, so on reversing the direction of F, the Torque of tension about point O is zero, as it passes through O.
direction of torque is reversed. If directions of both r and F are r⊥
O
reversed, the direction of τ remains the same. P
(iii) If the total torque acting on the rigid body vanishes, i.e. the vector 30°
sum of the torques on the rigid body is zero. 1m
n
τ1 + τ 2 + τ 3 K τ n = Σ τ = 0
i =1

Then, the body is said to be in rotational equilibrium. T 30°

Example 9.18 Find the torque of a force F = ($i + 2$j − 3k


$ )N
about a point O. The position vector of point of application of mg
force about O is r = (2$i + 3$j − k$ ) m.
τ mg = F × r⊥
$i $j k$
Here, r⊥ = OP = 1 sin 30° = 0.5 m
Sol. Torque, τ = r × F = 2 3 −1 ∴ τ mg = (mg)(0.5) = (1)(10)(0.5) = 5N-m
1 2 −3
Example 9.22 A solid flywheel of mass 20 kg and radius
τ = $i (−9 + 2) + $j (−1 + 6) + k$ (4 − 3) 120 mm revolves at 600 rev min −1. With what force must a
brake lining be pressed against it for the flywheel to stop in
or τ = (−7$i + 5$j + k)N
$ -m
3s, if the coefficient of friction is 0.1?
Example 9.19 A force 10 (− k) $ N acts on the origin of the Sol. Given, revolutions per minute, n 0 = 600 rev min−1
coordinate system. Find the torque about the point
600
(1 m, −1 m, 0 m). = rev s−1
60
Sol. Torque, τ = r × F = [(1 − 0) $i + (−1 − 0) $j + (0 − 0 )k$ )] × (−10 ) k$
Revolutions per second, n 0 = 10 rev s −1
= 10 ($i − $j) × (− k$ ) = 10 ($i + $j) N - m
Rotation 387

So, initial angular velocity, ω 0 = 2πn 0 = (2π )(10) = 20π rad s −1 Work done dθ
P = =τ (dt = time taken for turning by angle dθ)
Time taken dt
Let α be the constant angular retardation, then applying
ω = ω 0 − αt or P = τω
or 0 = (20 π ) − 3(α ) or α =
20
π rad s −2 So, power required = 200 × 100 = 20000 W
3 = 20 kW [Q1 kW = 1000 W]
τ
Further, α=
I Example 9.24 The power output of an automobile engine is
Here, torque τ = µNR (R = radius) advertised to be 200 HP at 6000 rpm. What is the
corresponding torque?
or τ = µFR (F = applied force)
1 Sol. P = 200 HP = 200 × 746 [Q1 HP = 746 W]
as N = F and I = mR 2
2 = 1.49 × 10 W
5

From the above equations, we get and ω = 6000 rev/min


20 µFR 2 µF 2π
π= = = 6000 × = 628 rad/s
3 1 mR 60
mR 2
2 P 1.49 × 105
τ= = = 237.3 N - m
10 πmR ω 628
or Force, F =

Example 9.25 A flywheel of moment of inertia 10 kg-m 2 is
Substituting the values, we have rotating at 50 rad/s. It must be brought to rest in 10 s.
10 × 22 × 20 × 0.12 (i) How much work must be done to stop it?
F = = 251.43 N (ii) What is the required average power?
3 × 7 × 0.1
Sol. (i) Given, ω 0 = 50 rad/s and t = 10 s
Work done by torque From the equation, ω = ω 0 − αt
Consider a rigid body acted upon by a force F at a 0 = 50 − α × 10
perpendicular distance r from the axis of rotation. Let α = 5 rad/s2
under the action of this force, the body rotates through an 1 1
Again, θ = ω 0t − αt 2 = 50 × 10 − × 5 × (10)2
angle dθ. 2 2
F
= 500 − 250 = 250 rad
F Retarding torque, τ = Iα = 10 × 5 = 50 N-m
P
Work done by retarding torque,
P0 W = τθ = 50 × 250 = 12500 J
dθ r (ii) Required average power,
O W 12500
P= = = 1250 W
t 10

Fig. 9.9 ANGULAR MOMENTUM


Work done = Torque × Displacement = τdθ A mass moving in a straight line has linear momentum (p).
The power associated with the work done by a torque When a mass rotates about some point/axis, there is
acting on a rotating body is given by momentum associated with rotational motion called the
dW dθ angular momentum (L ).
Power = =τ = τω Just as net external force is required to change the linear
dt dt
momentum of an object, a net external torque is required
where, τω is also known as instantaneous rotational to change the angular momentum of an object.
power.
Example 9.23 To maintain a rotor at a uniform angular speed Angular momentum of a particle
−1
of 100 rad s , an engine needs to transmit a torque of
200 N-m. What is the power required by engine? Assume
about some point
efficiency of engine is 100%. Angular momentum of a particle (of mass m) about a point
is defined as the moment of its linear momentum (p) about
Sol. Work done by torque in turning rotor by an angle dθ is τdθ,
the point.
so power delivered by engine,
388 OBJECTIVE Physics Vol. 1

L = r × p = r × (mv ) = m(r × v ) Example 9.27 A particle of mass m is moving along the line
y = b, z = 0 with constant speed v. State whether the angular
momentum of particle about origin is increasing, decreasing
A θ
p = mv
or constant.
Sol. Angular momentum of particle,
r
r⊥ = r sin θ | L | = mvr sin θ
= mvr⊥ = mvb (Given y = r⊥ = b)
θ
O
Y
Fig. 9.10 P
v
Here, r is the radius vector of particle A about O at that
instant of time. The magnitude of L is r r⊥ = b

L = mvr sin θ = mvr ⊥ θ


X
Here, r ⊥ = r sin θ is the perpendicular distance of line of O
action of velocity v from point O. The direction of L is
same as that of r × v. The SI unit of angular momentum is ∴ |L | = constant, as m, v and b all are constants.
kg-m 2 s −1 and its dimensions are [ML2 T −1]. Direction of r × v also remains the same. Therefore, angular
momentum of particle about origin remains constant with
time.
Important concept related to angular momentum Note In this problem, | r | is increasing, θ is decreasing but r sin θ,
Angular momentum is given by L = r × p i.e. b remains constant. Hence, the angular momentum remains
We can obtain the three rectangular components of angular constant.
momentum, i.e. L x ,L y and L z as we did in the case of torque.
L = L i$ + L $j + L k$
x y z Example 9.28 A particle of mass m is projected with velocity
r = x i$ + y$j + zk$ v at an angle θ with the horizontal. Find its angular momentum
about the point of projection when it is at the highest point of its
p = p x $i + p y $j + p zk$ trajectory.
Using determinant method, we get
Sol. At the highest point, it has only horizontal velocity
 i$ $j k$  v x = v cos θ …(i)
$ $ $ 
L = (L x i + L y j + L zk) = x y z 

p 
p pz  y
 x y

= ( yp z − zp y ) i$ + ( zp x − xp z) $j + ( xp y − yp x ) k$
Comparing the components of both sides, we get
L x = ( yp z − zp y ) …(i)
L y = ( zp x − xp z) …(ii) H
L z = ( xp y − yp x ) …(iii) x
O
If the particle is moving in yz-plane, it has the angular momentum
component only in x-direction,
Length of the perpendicular to the horizontal velocity from O
i.e. L y = L z = 0 and L x ≠ 0.
is the maximum height, where
Similarly, if the particle is moving in xz-plane, L y ≠ 0,L x = L z = 0 and
if the particle is moving in xy-plane only, L z ≠ 0,L x = L y = 0. v 2 sin2 θ
H max =
2g

Example 9.26 A car of mass 300 kg is travelling on a Angular momentum = mvr = mv x H max
circular track of radius 100 m with a constant speed of  v 2 sin2 θ 
60 m/s. Calculate the angular momentum. = mv x  
 2g 
Sol. Momentum, p = mv = 300 × 60 = 18000 kg-m/s
mv 2 sin2 θ
(directed along the tangent to the circle) = v cos θ [from Eq. (i)]
2g
Angular momentum, L = r × p
L = rp sin θ (Here, θ = 90°) mv 3 sin2 θ cos θ
=
∴ L = rp = 100 × 18000 = 18 × 105 kg-m2s −1 2g
Rotation 389

Example 9.29 Find the components along the X, Y, Z-axes of L = Σ mi ri v i


the angular momentum J of a particle, whose position vector i
is r with components x, y and z and linear momentum is p Here, v i = ri ω
with components p x , p y and p z . Show that if the particle ∴ L = Σ mi ri 2ω or L = ω Σ mi ri 2
moves only in the xy-plane, then the angular momentum has i i
only a z component. or L = Iω (as Σ mi ri 2 = I )
i
Sol. The position vector r and the linear momentum p of the
particle are written in terms of x, y, z components as Here, I is the moment of inertia of the rigid body about AB.
● The relation between torque and angular momentum can
r = xi$ + yj$ + zk$
be given as follows
and p = p xi$ + p y $j + p z k$ As angular momentum, L = r × p
By definition, the angular momentum of the particle is dL dr dp
∴ = ×p+r×
J = r × p = (xi$ + yj$ + zk$ ) × (p i$ + p j$ + p k$ )
x y z
dt dt dt
= v × mv + r × F = 0 + r × F = τ ext
 i$ j$ k$  dL
⇒ = τ ext
= x y z dt
 
p
 x
py pz
 Thus, rate of change of angular momentum is equal to
the torque due to an external force.
= i$ ( yp z − zp y ) + $j (zp x − xp z ) + k$ (xp y − yp x ) …(i) ● Angular impulse =
∫ τdt = ∫ d L = L f − L i
The angular momentum may be written in terms of x, y,
Hence, angular impulse of torque is equal to total
z components as
change in angular momentum of the body in given time.
J = J xi$ + J y $j + J z k$ …(ii)
Example 9.30 A 40 kg flywheel in the form of a uniform
Comparing Eqs. (i) and (ii), we have circular disc of 1m radius is making 120 rpm. Determine the
J x = yp z − zp y , angular momentum.
J y = zp x − xp z , Sol. Here, M = 40 kg, R = 1m, ν = 120 rpm =
120
rps = 2 rps
and J z = xp y − yp x . 60
If the particle moves only in the xy-plane, then z = 0 and 1 1
Now, moment of inertia, I = MR 2 = × 40 × (1)2 = 20 kg - m2
p z = 0. Now, Eq. (i) becomes 2 2
J = (xp − yp ) k$ and ω = 2πν = 2π × 2 = 4π rad s − 1
y x

i.e. J has only z-component. So, angular momentum, L = Iω = 20 × 4π = 20 × 4 × 3.14


L = 251.2 kg-m2s− 1
Angular momentum of a rigid
Example 9.31 A torque of 10 N-m is applied on a wheel
body rotating about a fixed axis having angular momentum of 2 kg-m 2 s −1, calculate the
The angular momentum of rigid A
ω angular momentum of the wheel after 4 s.
body, i.e. a system of particles is the dL
sum of angular momentum of all the r Sol. Torque, τ =
O P dt
particles within the system. L − L1
⇒ τ= 2
Suppose a particle P of mass m is t
going in a circle of radius r and at Angular momentum of the wheel,
some instant the speed of the particle L2 = τt + L1 = 10 × 4 + 2
is v. For finding the angular
= 40 + 2 = 42 kg-m2s−1
momentum of the particle about the B
axis of rotation, the origin may be Fig. 9.11 Example 9.32 The diameter of a solid disc is 0.5 m and its
chosen anywhere on the axis. We mass is 16 kg. What torque will increase its angular velocity
choose it at the centre of the circle. In this case, r and p from 0 to 120 rotations/min in 8 s?
are perpendicular to each other and r × p is along the axis. Sol. Moment of inertia of solid disc,
Thus, component of r × p along the axis is mvr itself. The 2
1 1  0.5 1
angular momentum of the whole rigid body about AB is I= MR 2 = × 16 ×   = kg-m2
2 2  2  2
the sum of components of all particles, i.e.
390 OBJECTIVE Physics Vol. 1

120 × 2 π As L = Iω, the law of conservation of momentum leads us


Angular velocity, ω = = 4 π rad/s and change in
60 to the following conclusion.
angular momentum, ∆L = Iω − 0 = Iω For an isolated system, Iω = constant or I 1ω1 = I 2ω 2
Angular impulse, τ × t = ∆L This principle is often used by gymnast, swimmers, circus
∆L Iω 1 1 π acrobats and ballet dancers cat while catching birds, etc.
⇒ τ= = = × × 4 π = N-m
t t 8 2 4 Constant angular momentum is extremely useful when
dealing with the orbits of planets and satellites and also
Example 9.33 A solid ball of radius 0.2 m and mass 1 kg is when analysing the Bohr’s model of the atom.
given an instantaneous impulse of 50 N-s at point P as
shown in figure. Find the number of rotations made by the Example 9.34 The maximum and minimum distances of a
ball about its diameter before hitting the ground. Initially, comet from sun are 1.4 × 1012 m and 7 × 1010 m,
the ball is kept on smooth surface. respectively. If velocity nearest to sun is 6 × 10 4 ms −1, what
50 N-s is the velocity of comet when it is farthest from the sun?
Comet’s tail
P θ
Sun
30°
Comet
0.2 m
Sol. Consider comet as a point mass and its path as circular.
Applying conservation of angular momentum, I1ω1 = I 2ω 2
Sol. Impulse gives translational velocity, v v
mr12 × 1 = mr22 × 2 ⇒ v11 r = v 2r2
Impulse r1 r2
u= (along impulse )
Mass v r 6 × 104 × 7 × 1010
⇒ v 2 = 11 = = 3000 ms −1
50 r2 1.4 × 1012
= = 50 m/s
1
Example 9.35 A wheel of moment of inertia I and radius R is
T = time of flight of projectile rotating about its axis at an angular speed ω 0 . It picks up a
2u sin θ 2 × 50 × sin 60° stationary particle of mass m at its edge. Find the new
= = = 5 3s angular speed of the wheel.
g 10
Impulse also gives angular velocity, Sol. Net external torque on the system is zero. Therefore,
angular momentum will remain conserved, thus
Impulse × R Impulse × R
ω= or ω = Iω
I 2 I1ω1 = I 2 ω 2 or ω 2 = 1 1
mR 2 I2
5
5 × Impulse 5 × 50 Here, I1 = I, ω1 = ω 0, I 2 = I + mR 2
= = = 625 rad/s
2 × mR 2 × 1 × 0.2 Iω 0
∴ ω2 =
ωT 3125 3 I + mR 2
Number of rotations, n = =
2π 2π
Example 9.36 If the radius of the earth contracts to half of its
present value without change in its mass, what will be the
Law of conservation of angular new duration of the day?
momentum Sol. Present angular momentum of earth, L1 = Iω =
2
MR 2ω
5
It states that when no external torque acts on an object or
isolated system of objects, then no change of angular New angular momentum due to change in radius,
2
momentum occurs. Hence, the angular momentum before 2 R 
L2 = M   ω′
an event involving only internal torques or no torques is 5  2
equal to the angular momentum after the event. If external torque is zero, then angular momentum must be
dL conserved, i.e. L1 = L2
We know that, τ ext = 2 1 2
dt MR ω = × MR 2ω ′
2

dL 5 4 5
If τ ext = 0, then = 0 or L must be a constant. i.e. ω′ = 4 ω
dt 1  2π 
Therefore, in the absence of any external torque, the ∴ New duration of the day, T ′ = T Qω = 
4  T
total angular momentum of a system must remain
conserved. 1
= × 24 = 6 h
4
Rotation 391

Example 9.37 A wheel is rotating at a rate of 1000 rpm and


ROTATIONAL KINETIC its kinetic energy is 10 6 J. Determine the moment of inertia
ENERGY OF THE PARTICLES of the wheel about its axis of rotation.
1000 50
The energy due to rotational motion of a body is known as Sol. Given, ν = 1000 rpm = rps = rps
60 3
rotational kinetic energy.
100π
Let us consider a body rotating about a fixed axis. For a ∴ ω = 2πν = rads−1
rotating rigid body, kinetic energy is the sum of kinetic 3
energies of individual particles. 1 2
Q Rotational kinetic energy = Iω
2
Y 2
1 100π 
v2 v3 v4 ∴ 106 = ×I ×  
v1 2  3 

2 × 106 × 9 200 × 9
⇒ I= = = 182.4 kg-m2
X (100) π
2 2
9.87

Example 9.38 A flywheel of mass 0.2 kg and radius 10 cm


is rotating with (5/π) rev/s about an axis perpendicular to its
plane passing through its centre. Calculate angular
Fig. 9.12 Rotation of a rigid body momentum and kinetic energy of flywheel.
5
So, kinetic energy of a rotating body, Sol. Angular velocity, ω = 2πν = 2π × = 10 rad/s
π
1 1 1
K = m1v 12 + m 2v 22 + K + mn v n2 and moment of inertia, I = mr 2 = (0.2)(0.1)2 = 2 × 10−3 kg-m2
2 2 2
n Angular momentum = Iω = 2 × 10−3 × 10 = 2 × 10−2 J-s
1
or K = ∑ mi v i2 = 2 × 10−2 kg-m2s−1
i =1 2
1 2 1
n Kinetic energy = Iω = × 2 × 10−3 × (10)2 = 0.1 J
1
Substituting v = rω, K = ∑ mi ri 2ω 2
2 2
i =1 2
where, mi is the mass of ith particle and v i is its linear Work-energy theorem in rotational
velocity. motion
As, the angular velocity ω is same for all the particles of According to work-energy theorem, the change in the
rigid body. rotational kinetic energy of a rigid body is equal to the
1  n 
So, K = ω 2  ∑ mi ri 2  …(i) work done by external torques acting on the body.
2  i =1  Work done by torque = Change in rotational kinetic
The quantity ∑ (mi ri 2 ) depends on distribution of mass
energy

around axis of rotation. This quantity is called moment of 1 2 1 2


inertia (I ). W= Iω − Iω
2 2 2 1
1
Kinetic energy, K = Iω 2 …(ii)
2 Example 9.39 An energy of 484 J is spent in increasing the
1 speed of a flywheel from 60 rpm to 360 rpm. Calculate
When ω = 1, rotational kinetic energy = I
2 moment of inertia of flywheel.
or Moment of inertia, I = 2 × Rotational kinetic energy Sol. Given, energy spent,W = 484 J
60
Hence, the moment of inertia of a rigid body about an Initial speed, ω 1 = 60 rpm = × 2π = 2π rad s −1
axis of rotation is numerically equal to twice the 60
rotational kinetic energy of the body when it is 360
Final speed, ω 2 = 360 rpm = × 2π = 12π rad s −1
rotating with unit angular velocity about that axis. 60
Rotational kinetic energy depends upon axis of rotation. Moment of inertia, I = ?
392 OBJECTIVE Physics Vol. 1

1 2 1 2 E 2 − E1
Energy spent,W = E 2 − E1 = Iω 2 − Iω1 Increase in momentum = × 100
2 2 E1
1
= I[(12π )2 − (2π )2] = 70Iπ 2 4E − E
2 = × 100 = 100 %
E
484
⇒ I= = 0.7 kg-m 2 Example 9.41 A thin metre scale is kept vertical by placing its one
70 × π 2
end on floor. Keeping the end in contact stationary, it is allowed
Example 9.40 If the rotational kinetic energy of a body is to fall. Calculate the velocity of its upper end when it hits the
increased by 300%, then determine percentage increase in its floor.
angular momentum. Sol. Loss in potential energy = Gain in rotational kinetic energy
Sol. Percentage increase in angular momentum,
L − L1 l
= 2 × 100 2
L1 l CM
As, L ∝ E and E1 = E
300
⇒ E2 = E + E mgl 1 2 1 ml 2 v 2
100 ⇒ = Iω = × 2 ⇒ v = 3gl
⇒ E 2 = 4E 2 2 2 3 l

CHECK POINT 9.2


1. The torque of a force F = − 6$i acting at a point r = 4 $j about 7. A disc is rotating with angular velocity ω. A force F acts at a
origin will be point whose position vector with respect to the axis of
(a) − 24 k
$ $
(b) 24 k rotation is r. The power associated with torque due to the
$
(c) 24 j $
(d) 24 i
force is given by
(a) (r × F) ⋅ ω (b) (r × F) × ω (c) r × (F ⋅ ω) (d) r ⋅ (F × ω)
2. Moment of a force of magnitude 20 N acting along positive 8. Angular momentum is
x-direction at point (3m, 0, 0) about the point (a) moment of momentum
(0, 2m, 0) has magnitude of (in N-m)
(b) product of mass and angular velocity
(a) 20 (b) 60
(c) product of moment of inertia and velocity
(c) 40 (d) 30
(d) moment in angular motion
3. The torque of force F = − 3 $i + $j + 5k$ acting on a point
9. The unit mass has r = 8 $i − 4 $j and v = 8 $i + 4 $j. Its angular
r = 7$i + 3$j + k$ about origin will be
momentum is
(a) 14i$ − 38j$ + 16 k$ (b) 4i$ + 4j$ + 6 k$ (a) 64 units in − k
$ direction
(c) −14$i + 38$j − 16 k
$ (d) −21 $i + 3$j + 5k
$
(b) 64 units in + k
$ direction

4. A door 1.6 m wide requires a force of 1 N to be applied at (c) 64 units in + $j direction


the free end to open or close it. The force that is required at (d) 64 units in + $i direction
a point 0.4 m distant from the hinges for opening or closing
the door is 10. If the earth is a point mass of 6 × 10 24 kg revolving around
(a) 1.2 N (b) 3.6 N the sun at a distance of 1.5 × 10 8 km and in time
(c) 2.4 N (d) 4 N
T = 3.14 × 10 7 s, then the angular momentum of the earth
5. A flywheel of moment of inertia 2 kg-m 2 is rotated at a around the sun is
speed of 30 rad/s. A tangential force at the rim stops the (a) 1.2 × 1018 kg -m2 s−1 (b) 1.8 × 1029 kg -m2 s−1
wheel in 15 s. Average torque of the force is 2 −1
(a) 4 N-m (b) 2 N-m (c) 1.5 × 10 37
kg -m s (d) 2.7 × 1040 kg -m2 s−1
(c) 8 N-m (d) 1 N-m 11. A particle with the position vector r has linear momentum
6. A mass of 10 kg connected at the end of a rod of negligible p. Which of the following statement is true in respect of its
mass is rotating in a circle of radius 30 cm with an angular angular momentum L about the origin?
velocity of 10 rad/s. If this mass is brought to rest in 10 s by (a) L acts along p.
a brake, what is the magnitude of the torque applied? (b) L acts along r.
(a) 0.9 N-m (b) 1.2 N-m (c) L is maximum when p and r are parallel.
(c) 2.3 N-m (d) 0.5 N-m (d) L is maximum when p is perpendicular to r.
Rotation 393

12. By keeping moment of inertia of a body constant, if we 16. If the radius of earth contracts1/ n of its present day value,
double the time period, then angular momentum of body the length of the day will be approximately
(a) remains constant (b) becomes half 24 24
(a) h (b) h
(c) doubles (d) quadruples n n2
13. If torque is zero, then (c) 24nh (d) 24 n2h
(a) angular momentum is conserved
17. A thin circular ring of mass M and radius R is rotating about
(b) linear momentum is conserved its axis with a constant angular velocity ω. Two objects each
(c) energy is conserved of mass m are attached gently to the ring. The wheel now
(d) angular momentum is not conserved rotates with an angular velocity is
14. The angular momentum of a rotating body changes from A0 ωM ω (M − 2m)
(a) (b)
2 −1
kg m min to 4 A0 kg m min 2 −1
in 4 min. The torque acting (m + M) (M + 2m)
ωM ω (M + 2m)
on the body is (c) (d)
3 3 (M + 2m) M
(a) A0 (b) 4 A0 (c) 3A0 (d) A0
4 2 18. A circular disc of mass 2 kg and radius 1 m is rotating about
15. A disc of mass 2 kg and radius 0.2 m is rotating with an axis perpendicular to its plane and passing through its
angular velocity 30 rad s −1 . What is angular velocity, if a centre of mass with a rotational kinetic energy of 8 J. The
mass of 0.25 kg is put on periphery of the disc? angular momentum (in J-s) is
(a) 24 rad s−1 (b) 36 rad s−1 (a) 8 (b) 4
(c) 2 (d) 1
(c) 15 rad s−1 (d) 26 rad s−1

ROLLING MOTION
When a body performs translatory motion as well as Linear velocity of the disc, v = 10 ms − 1
rotational motion, then this type of motion is known as Q v = Rω
rolling motion. e.g. Motion of football rolling on a surface. 1
In rolling motion, the velocity of centre of mass represents ⇒ 10 = ω
2
linear motion while angular velocity represents rotational
⇒ ω = 10 × 2 = 20 rad s − 1
motion.
Also, moment of inertia of disc about an axis through its centre,
1
I = MR 2
2
2
CM 1  1 5
vCM CM vCM ⇒ I= ×5×   = kg-m
2
2  2 8
ω
ω 1 1
Translational kinetic energy = Mv 2 = × 5 × (10)2 = 250J
O 2 2
(a) (b) 1 2
Fig. 9.13 Rotational kinetic energy = Iω
2
1 5
The kinetic energy of a rolling body is equal to the = × × (20)2 = 125 J
sum of kinetic energies of translational and rotational 2 8
motion. ∴ Total kinetic energy = (250 + 125) J = 375 J
∴ Total kinetic energy of a rolling body = Rotational
kinetic energy + Translational kinetic energy Classification of rolling motion
1 1 Depending on the fact that relative velocity of point of
= Iω 2 + mv CM 2
contact of the body undergoing rolling motion, with the
2 2
platform is zero or non-zero. Rolling motion is classified
Example 9.42 A disc of mass 5 kg and radius 50 cm rolls on into two categories, which are as follows
the ground at the rate of 10 ms −1. Determine the kinetic (i) Pure rolling or rolling without slipping/sliding or
energy of the disc. perfect rolling motion.
Sol. Given, mass of the disc, M = 5 kg (ii) Impure rolling or rolling with slipping/sliding or
1 imperfect rolling motion.
Radius of the disc, R = 50 cm = m
2
394 OBJECTIVE Physics Vol. 1

Applying conservation of linear momentum, we get


Pure rolling
mv = (M + m ) v CM
If the velocity of point of contact with respect to the
mv
surface is zero, then it is known as pure rolling. ⇒ v CM =
(M + m )
Now, applying conservation of angular momentum about O,
CM
v
⇒ we get
2 
ω P mvh = I 0ω =  MR 2 + mR 2 ω
Rω v 3 
Q
mvh
(a) (b) ω=
 2  2
Fig. 9.14  M + m R
3 
For pure rotational motion, velocity of centre of mass
For pure rolling,
is zero given by the relation, v = Rω
v CM = ωR
In the above figure, when we observe the motion of contact
mv mvh
point P on the disc, we find that it has two simultaneous ⇒ =
motions M +m 2 
 M + m R
3 
(i) Linear forward velocity v because whole body is
moving forward. 2 
 M + m R
(ii) Linear backward velocity ωR due to rotation of the 3 
⇒ h=
disc as a whole in clockwise direction. In case of pure (M + m )
rolling, these two velocities become equal. 2R
= (Since, m << M )
This is the condition of pure rolling on a stationary 3
ground. Sometimes, it is simply known as rolling.
Suppose the base over which the disc in rolling is also Kinetic energy of rolling motion
moving with some velocity (say v 0 ), then in that case The kinetic energy of a rolling body is the sum of kinetic
condition of pure rolling is different. energy of translational motion and kinetic energy of
e.g. In the above figure, v P = v Q or v − Rω = v 0 rotational motion.
Thus, in this case v − Rω ≠ 0, but v − Rω = v 0 . In uniform ∴ Kinetic energy of rolling motion,
pure rolling, v and ω are constants, i.e. they are neither 1 1
increasing nor decreasing. E = Iω 2 + Mv CM 2
2 2
Impure rolling Substituting the value of I = MK 2, where K is radius of
If the velocity of point of contact with respect to the gyration and v CM = Rω, we get
surface is not zero, then is known as impure rolling. In 1 v  1
2
impure rolling, the body slides on the surface in contact E = MK 2  CM  + Mv CM
2
2  R  2
and it depends on the nature of surface.
In above case shown in Fig. 9.13, if v > ωR , disc will 1  K2
⇒ E = 2
Mv CM 1 + 2 
starts slipping on the ground. 2  R 
Example 9.43 A hollow sphere of mass M lies on a rough The ratio of translational energy, rotational energy and
horizontal plane when a particle of mass m travelling with total energy can be calculated as
speed v collides and sticks with it. If line of motion of the
particle is at height h above the centre of sphere, find h if E translational : E rotational : E total
the body rolls without slipping after collision.
K2 K2
Sol. Mass m moving with velocity v creating rotation in sphere. 1 : : 1+
R2 R2
m v m
Example 9.44 A solid cylinder of mass m and radius r starts
h
rolling down an inclined plane of inclination θ. Friction is
O ≡ O v CM
enough to prevent slipping. Find the speed of its centre of
R ω mass when its centre of mass has fallen a height h.
Rotation 395

Sol. Considering the two shown positions of the cylinder. As it Example 9.46 A horizontal force F acts on the hollow sphere
does not slip, hence total mechanical energy will be at its centre as shown in figure. Coefficient of friction
conserved. between ground and sphere is µ. What is the maximum
value of F for which there is no slipping?
1
h
2 F

θ
f (Force of friction)
Energy at position 1, E1 = mgh
Sol. As, F − f = Ma …(i)
1 1
Energy at position 2, E 2 = mv 2CM + I CM ω 2 Q τ = fR = Iα
2 2
2 2 a
v CM mr ⇒ f ⋅ R = MR 2 (Q a = Rα)
Q = ω and I CM = 5 R
r 2
2
3 ⇒ f = Ma
⇒ E 2 = mv CM
2
5
4
5
From law of conservation of energy, ⇒ Ma = f
2
E1 = E 2
2
3 2 ⇒ f= F [using Eq. (i)]
⇒ mgh = mv CM 7
4
2 7
4 ∴ For no slipping, F ≤ µmg ⇒ F ≤ µmg (Q f = µmg)
⇒ v CM = gh 7 2
3
Example 9.47 A tangential force F acts at the top of a thin
Example 9.45 A disc of radius R starts at time t = 0 moving spherical shell of mass m and radius R. Find the acceleration
along the positive X-axis with linear speed v and angular of the shell, if it rolls without slipping.
speed ω. Find the x and y-coordinates of the bottom most
point at any time t. F

Y
R

C v
ω
X
Sol. Let f be the force of friction between the shell and the
O horizontal surface.
Sol. At time t, the bottom most point will rotate at an angle F
θ = ωt with respect to the centre of the disc C. The centre C
will travel a distance, s = vt .

Y f (Force of friction)

C For translational motion, F + f = ma K(i)


C R For rotational motion,
θ a
P Q FR − fR = Iα = I (Q a = Rα for pure rolling)
R
O s=vt M
a
⇒ F − f =I 2 K (ii)
In the figure, PQ = R sin θ = R sin ωt R
and CQ = R cos θ = R cos ωt Adding Eqs. (i) and (ii), we get
Coordinates of point P at time t are  I   2   2 2
2 F = m + 2  a = m + m  a QI shell = mR 
x = OM − PQ = vt − R sin ωt  R   3   3 
and y = CM − CQ = R − R cos ωt 5 5 6F
= ma or F = ma ⇒ a =
∴ (x, y ) ≡ (vt − R sin ωt, R − R cos ωt ) 3 6 5m
396 OBJECTIVE Physics Vol. 1

object will reach the bottom with same speed because


Rolling down on a rough inclined the ratio K 2 /R 2 will be same for similar bodies.
plane g sin θ g sin θ
A body of mass m and radius R rolling down a plane Acceleration, a rolling = =
inclined at an angle θ with the horizontal. If the body rolls 1 + (K /R )
2 2
1+ n 2
without slipping, then the centre of mass of the body
It is also independent of mass and radius.
moves in a straight line.
K2
Different forces acting on the body are If body slides, then =0 (For frictionless surface)
(i) Frictional force (f ) acting upwards at the point of R2
contact with inclined plane and parallel to the ∴ a sliding = g sinθ
inclined plane.
Time taken by rolling body to reach the bottom,
(ii) Normal reaction of inclined plane (N).
(iii) Weight of the body (mg) acting vertically downwards 1 2h  K 2 
through the centre of mass of body. t= 1 + 
sinθ g  R2
R
A body with smaller value of K 2 /R 2 will take less
N
f time to reach the bottom.
θ θ Change in kinetic energy due to rolling (v 2 > v 1 )
in h
ω s mg cos θ
mg s
mg 1  K2 2
vCM = m 1 +  (v 2 − v 12 )
θ 2  R2
Fig. 9.15 Note
(i) For pure rolling motion, there should be friction on surface because
For rolling without slipping, the friction force is static and it provides torque to the body. On smooth surface, rolling is not
no work is done by frictional force. possible.
(ii) Velocity of falling and sliding bodies from inclined plane are equal
So, from law of conservation of mechanical energy, and is more than that of rolling body.
1 2 1 (iii) For rolling along inclined plane, acceleration is maximum in case of
mgh = mv CM + Iω 2 falling and minimum in case of rolling. Falling body reaches the
2 2
bottom first and rolling body in the last.
Putting I = mK 2 , where K = radius of gyration and (iv) When ring (R), disc (D), hollow sphere (H) and solid sphere (S) rolls
v CM = Rω, we get on same inclined plane, then
v S > vD > vH > vR ⇒ aS > aD > aH > aR ⇒ t S < t D < t H < t R
2gh
v rolling = Example 9.48 A sphere of mass m attached to a spring on
1 + K 2 /R 2 inclined plane as shown in figure is held in unstretched
position of spring. Suddenly sphere is left free, what is the
From Fig. 9.15, h = s sinθ maximum extension of spring, if friction allows only rolling
of sphere about horizontal diameter?
2g s sin θ
∴ v rolling =
1 + K 2 /R 2

2g s sin θ
= (where, K = nR )
1+ n 2 θ

K2 Sol. At the point of maximum extension, sphere is at rest. No


If body slides, then 2
=0 work is done by frictional force in rolling.
R
⇒ Loss in gravitational potential energy = Gain in potential
⇒ v sliding = 2gh = 2g s sinθ energy of spring
1 2
From the above formula, we see that velocity of rolling ⇒ kx = mgx sin θ
2
motion is independent of its mass (M ) and radius (R ). 2 mg sin θ
It means all uniform solid spheres or any other similar ⇒ x=
k
Rotation 397

Example 9.49 Calculate the kinetic energy of rolling ring of 1 2 1 2 1 2 1  2 2 2


mgh = mv + I ω = mv +  mr  ω
mass 0.2 kg about an axis passing through its centre of mass 2 2 2 2 5 
and perpendicular to it, if its centre of mass is moving with a
velocity of 3 m/s. 1 2 1 2 7 7v 2
= mv + mv = mv 2 or h =
1 2  K2 2 5 10 10g
Sol. Kinetic energy, KE = mv CM 1 + 2 
2  R  ∴ The distance rolled up on the ramp,
K 2
h 7v 2 7 (2 ms −1)2 4
Here, for ring = 1, s= = = = m ~ 57 cm
R 2 sin θ 10g sin 30° 10(9.8ms −2 ) × 1/2 7
m = 0.2 kg and v CM = 3 m/s
1 Example 9.52 When a body is under pure rolling, the fraction
∴ KE = × 0.2 × 9 (1 + 1) = 0.2 × 9 = 1.8 kg-m2s−2 of its total kinetic energy which is the purely rotational is
2 2/5. Identify the body.
Example 9.50 A solid sphere is rolling down an inclined Sol. In these type of questions, calculate the expression for
plane without slipping of height 20 m. Calculate the moment of inertia I and that helps in identifying the body.
maximum velocity with which it will reach the bottom of the 1
plane. (Take, g = 10 m /s 2 ) Rotational kinetic energy = Iω 2
2
Sol. When the sphere rolls down, its potential energy changes to 1
Translational kinetic energy = Mv 2
kinetic energy of rotation. Therefore, kinetic energy = 2
potential energy 1 1
∴ Total kinetic energy = I ω 2 + Mv 2
2 2
2 Rotational kinetic energy
Given, =
5 Total kinetic energy
20 m 2 (1/2) Iω 2
or =
5 (1/2) Iω 2 + (1/2) Mv 2
θ 2 Iω 2
For pure rolling, put v = ωR ⇒ = 2
5 Iω + Mω 2R 2
1 2 K2 2gh
mv 1 + 2  = mgh ⇒ v =
2  R  (1 + K 2 /R 2 ) 2MR 2
∴ I=
3
2
Moment of inertia of solid sphere = MR 2 So, the rolling body is hollow sphere.
5
2 2 Example 9.53 A uniform sphere of mass m and radius R rolls
∴ K2 = R without slipping down an inclined plane set at an angle θ to
5
the horizontal.
2gh 2gh × 5 2 × 10 × 20 × 5
Hence, v = = = Find
 2  7 7 (i) the friction coefficient when slipping is absent and
1 + 
 5 (ii) the kinetic energy of the sphere t seconds after the beginning
= 285.714 = 16.9 m/s
of motion.
Sol. (i) If a body rolls without slipping,
Example 9.51 A solid sphere is rolling without slipping on a tan θ tan θ 2  K 2 2
level surface at a constant speed of 2 ms −1. How far can it µ min = = = tan θ  For sphere, = 
R2 5 7  R 2 5
roll up on a ramp inclined at 30° before it stops? 1+ 2 1+
K 2
Sol. The moment of inertia of the sphere about its axis of g sin θ g sin θ 5
2 (ii) For pure rolling, a = = = g sin θ
rotation is I = mr 2. Suppose it rises to height h before K2 +
2 7
5 1+ 2 1
R 5
stopping. Using the conservation of energy, we get 5g sin θ
Linear velocity after time t, v = u + at = 0 + t
7
=0 In pure rolling, kinetic energy is given by
0 ,v
ω=
1  K2 1  2
K = mv 2 1 + 2  = mv 2 1 + 
2  R  2  5
ω 2
s h 7 7  5g sin θt  5
v = mv 2 = m  = mg 2 sin2 θ t 2
θ 10 10  7  14
OBJECTIVE Physics Vol. 1

CHECK POINT 9.3


1. At any instant, a rolling body may be considered to be in 9. The ratio of the time taken by a solid sphere and that taken
pure rotation about an axis through the point of contact. by a disc of the same mass and radius to roll down a rough
This axis is translating forward with speed inclined plane from rest, from the same height is
(a) equal to speed of centre of mass (a) 15 : 14 (b) 15 : 14 (c) 14 : 15 (d) 14 : 15
(b) zero
(c) twice of speed of centre of mass 10. A solid sphere, a hollow sphere and a disc, all having same
(d) None of the above mass and radius, are placed at the top of an inclined plane
2. A wheel of radius R rolls on the ground with a uniform and released. The coefficients of friction between the
velocity v. The velocity of top most point relative to the objects and the incline are same and not sufficient to allow
bottom most point is pure rolling. Least time will be taken in reaching the
(a) v bottom by
(b) 2v (c) v/ 2
(a) the solid sphere
(d) zero
(b) the hollow sphere
3. The centre of a wheel rolling on a plane surface moves with (c) the disc
a speed v 0 . A particle on the rim of the wheel at the same (d) All will take same time
level as the centre will be moving at speed
(a) zero (b) v0 11. The speed of a uniform spherical shell after rolling down an
(c) 2v0 (d) 2v0 inclined plane of vertical height h from rest is
10 gh 6 gh 4 gh
(a) (b) (c) (d) 2gh
4. A sphere is rolling down a plane of inclination θ to the 7 5 5
horizontal. The acceleration of its centre down the plane is
(a) g sin θ (b) less than g sin θ 12. A solid sphere, a hollow sphere and a disc, all having same
(c) greater than g sin θ (d) zero mass and radius, are placed at the top of a smooth inclined
5. Two uniform solid spheres having unequal masses and and released. Least time will be taken in reaching the
unequal radii are released from rest from the same height bottom by
on a rough incline. If the spheres roll without slipping, (a) the solid sphere (b) the hollow sphere
(a) the heavier sphere reaches the bottom first (c) the disc (d) all will take same time
(b) the bigger sphere reaches the bottom first
(c) Both the spheres reach the bottom together
13. A solid cylinder rolls down an inclined plane of height 3 m
(d) the information given is not sufficient to tell which sphere and reaches the bottom of plane with angular velocity
will reach the bottom first 2 2 rad s −1. The radius of cylinder must be (Take, g = 10 ms −2 )
6. A solid sphere of mass m rolls without slipping on an (a) 5 cm (b) 0.5 cm (c) 10 cm (d) 5 m
inclined plane of inclination θ. The linear acceleration of the 14. A uniform solid spherical ball is rolling down a smooth
sphere is
inclined plane from a height h. The velocity attained by the
7 2
(a) g sin θ (b) g sin θ ball when it reaches the bottom of the inclined plane is v. If
5 7
3 5 the ball is now thrown vertically upwards with the same
(c) g sin θ (d) g sin θ velocity v, the maximum height to which the ball will
7 7
rise is
7. An inclined plane makes an angle of 30° with the 5h 3h 5h 7h
(a) (b) (c) (d)
horizontal. A solid sphere rolling down this inclined plane 8 5 7 9
from rest without slipping, has a linear acceleration equal to 15. A solid sphere of mass 10 kg is placed on a rough surface
g 2g
(a) (b) having coefficient of friction, µ = 01
. . A constant force
3 3
5g 5g F = 7N is applied along a line passing through the centre of
(c) (d) the sphere as shown in figure. The frictional force acting on
7 14
the sphere will be
8. A thin uniform circular ring is rolling down an inclined
F =7 N
plane of inclination 30° without slipping. Its linear
acceleration along the inclined plane is
g g g µ = 0.1
(a) g (b) (c) (d)
2 3 4 (a) 1N (b) 2N (c) 3N (d) 7N
Chapter Exercises
(A) Taking it together
Assorted questions of the chapter for advanced level practice

1 The angular momentum of a system of particles is 9 A wheel is at rest. Its angular velocity increases
conserved uniformly and becomes 80 rad s −1 after 5 s. The
(a) when no external force acts upon the system total angular displacement is
(b) when no external torque acts upon the system (a) 800 rad (b) 400 rad
(c) when no external impulse acts upon the system (c) 200 rad (d) 100 rad
(d) when axis of rotation remains same
10 A ring of diameter 0.4 m and mass 10 kg is rotating
2 If a person standing on a rotating disc stretches out about its axis at the rate of 1200 rpm. The angular
his hands, the angular speed will momentum of the ring is
(a) increase (b) decrease (a) 60.28 kg-m2s−1 (b) 55.26 kg-m2s−1
(c) remain same (d) None of these 2 −1
(c) 40.28 kg-m s (d) 50.28 kg-m2s−1
3 A diver in a swimming pool bends his head before
diving, because it 11 A body is in pure rotation. The linear speed v of a
(a) decreases his moment of inertia particle, the distance r of the particle from the axis
(b) decreases his angular velocity and the angular velocity ω of the body are related as
(c) increases his moment of inertia v
ω = . Thus,
(d) decreases his linear velocity r
1
4 A body is under the action of two equal and (a) ω ∝ (b) ω ∝ r
oppositely directed forces and the body is rotating r
with constant non-zero angular acceleration. Which (c) ω = 0 (d) ω is independent of r
of the following cannot be the separation between 12 A particle performs uniform circular motion with an
the lines of action of the forces? angular momentum L. If the frequency of the particle
(a) 1 m (b) 0.4 m (c) 0.25 m (d) zero motion is doubled, the angular momentum becomes
5 Five particles of masses 2 kg each are attached to L L
(a) 2L (b) 4L (c) (d)
the rim of a circular disc of radius 0.1 m and 2 4
negligible mass. Moment of inertia of the system 13 A particle of mass 2 kg located at the position
about the axis passing through the centre of the disc $ ) ms −1. Its
($i + $j) m has a velocity 2 (+$i − $j + k
and perpendicular to its plane is
angular momentum about Z-axis (in kg-m2 s −1) is
(a) 1 kg-m2 (b) 0.1 kg-m2 (c) 2 kg-m2 (d) 0.2 kg-m2
(a) + 4 (b) + 8 (c) − 4 (d) − 8
6 The rotational kinetic energy of a body is E and its
14 A constant torque of 1000 N-m turns a wheel of
moment of inertia is I. The angular momentum is
E
moment of inertia 200 kg-m2 about an axis through
(a) EI (b) 2 EI (c) 2EI (d) its centre. Its angular velocity after 3 s is
I
(a) 1 rad s −1 (b) 5 rad s −1 (c) 10 rad s −1 (d) 15 rad s −1
7 A wheel is rotating at 900 rpm about its axis. When
15 A flywheel having a radius of gyration of 2 m and
power is cut-off, it comes to rest in 1 min. The
angular retardation (in rad s −2 ) is mass 10 kg rotates at an angular speed of 5 rad s −1
π π π π about an axis perpendicular to it through its centre.
(a) (b) (c) (d) The kinetic energy of rotation is
2 4 6 8
(a) 500 J (b) 2000 J (c) 1000 J (d) 250 J
8 A wheel is subjected to uniform angular acceleration
16 A rod is placed along the line y = 2x with its centre
about its axis. Initially, its angular velocity is zero. In
the first 2 s, it rotates through an angle θ 1, in the next at origin. The moment of inertia of the rod is
θ maximum about
2 s, it rotates through an angle θ 2 . The ratio of 2 is (a) X-axis (b) Y -axis
θ1
(c) Z-axis (d) Data insufficient
(a) 1 :1 (b) 2 : 1 (c) 3 : 1 (d) 5 : 1
400 OBJECTIVE Physics Vol. 1

17 Let I A and I B be moments of inertia of a body about ω ω ω ω


(a) (b) (c) (d)
6 3 2 5
two axes A and B, respectively. The axis A passes
through the centre of mass of the body but B does 26 A uniform square plate has a small piece Q of an
not. Choose the correct option. irregular shape removed and glued to the centre of
(a) I A < IB the plate leaving a hole behind as shown in figure.
(b) I A < IB , whether the axes are parallel or not parallel The moment of inertia about the Z-axis is
(c) If the axes are parallel, I A < IB [NCERT Exemplar]
(d) If the axes are not parallel, I A ≥ IB y y

18 A rigid body rotates with an angular momentum L. If


Hole
its rotational kinetic energy is made 4 times, its Q
angular momentum will become x x
(a) 4L (b) 16L (c) 2L (d) 2L
19 Work done by friction in case of pure rolling (a) increased
(a) is always zero (b) decreased
(b) is always positive (c) the same
(c) is always negative (d) changed in unpredicted manner
(d) may be positive, negative or zero
27 Moment of a force of magnitude 10 N acting along
20 A disc of mass m and radius R is rolling on horizontal positive y-direction at point (2m, 0, 0) about the
ground with linear velocity v. What is the angular point (0, 1m, 0) (in N-m) is
momentum of the disc about an axis passing through (a) 10 (b) 20 (c) 10 2 (d) 30
bottom most point and perpendicular to the plane of
motion? 28 A rigid body rotates about a fixed axis with variable
3 1 4 angular velocity equal to α − βt, at the time t, where
(a) mvR (b) mvR (c) mvR (d) mvR
2 2 3 α, β are constants. The angle through which it
rotates before it stops is
21 A sphere cannot roll without applying external force on
α2 α2 − β2 α2 − β2 (α − β ) α
(a) a smooth inclined surface (a) (b) (c) (d)
(b) a smooth horizontal surface 2β 2α 2β 2
(c) a rough inclined surface 29 The radius of gyration of a uniform rod of length L
(d) a rough horizontal surface about an axis passing through its centre of mass is
22 A solid sphere of mass 2 kg rolls up a 30° incline L L2 L L
−1 (a) (b) (c) (d)
with an initial speed of 10 ms . The maximum 2 3 12 3 2
height reached by the sphere is (Take, g = 10 ms −2 ) 30 The motor of an engine is rotating about its axis with
(a) 3.5 m (b) 7 m (c) 10.5 m (d) 14 m an angular velocity of 100 rpm. It comes to rest in
23 A solid sphere and a solid cylinder of same mass are 15 s, after being switched OFF. Assuming constant
rolled down on two inclined planes of heights h1 and angular deceleration, what are the numbers of
h 2 , respectively. If at the bottom of the plane, the revolutions made by it before coming to rest?
two objects have same linear velocities, then the (a) 12.5 (b) 40 (c) 32.6 (d) 15.6
ratio of h1 : h 2 is 31 A particle of mass 5 g is moving with a uniform
(a) 2 : 3 (b) 7 : 5 (c) 14 : 15 (d) 15 : 14 speed of 3 2 cm s −1 in the XY-plane along the
24 If F be a force acting on a particle having the line y = 2 5 cm. The magnitude of its angular
position vector r and τ be the torque of this force momentum about the origin (in g-cm2 s −1) is
about the origin, then (a) zero (b) 30
(a) r ⋅ τ = 0 and F ⋅ τ = 0 (b) r × τ = 0 and F × τ ≠ 0 (c) 30 2 (d) 30 10
(c) r ⋅ τ ≠ 0 and F ⋅ τ ≠ 0 (d) r × τ ≠ 0 and F × τ = 0
32 A flywheel is in the form of a uniform circular disc
25 A uniform disc of radius a and mass m is rotating of radius 1 m and mass 2 kg. The work which must
freely with angular speed ω in a horizontal plane be done on it to increase its frequency of rotation
about a smooth fixed vertical axis through its centre.
from 5 rps to 10 rps is approximately
A particle of mass m is suddenly attached to the rim
of the disc and rotates with it. The new angular (a) 1.5 × 102 J (b) 3.0 × 102 J
speed is (c) 1.5 × 10 J 3
(d) 3.0 × 103 J
Rotation 401

33 A thin uniform circular disc of mass M and radius R 38 The figure shows the angular velocity versus time
is rotating in a horizontal plane about an axis passing graph of a flywheel. The angle (in radians) through
through its centre and perpendicular to its plane which the flywheel turns during 25 s is
with an angular velocity ω. Another disc of same
1
dimensions but of mass M is placed gently on the 30
4
first disc co-axially. The angular velocity of the
system is
ω (rad s–1)
2 4 3 1
(a) ω (b) ω (c) ω (d) ω 0 5 t(s) 20 25
3 5 4 3
34 The ratio of the radii of gyration of a hollow sphere (a) 120 (b) 480 (c) 600 (d) 750
and a solid sphere of the same radii about a 39 If a disc of mass m and radius r is reshaped into a
tangential axis is ring of radius 2r, the mass remaining the same, the
7 5 21 25 radius of gyration about centroidal axis
(a) (b) (c) (d)
3 21 5 9 perpendicular to plane goes up by a factor of
(a) 2 (b) 2 (c) 2 2 (d) 4
35 A square lamina is as shown in figure. The moment
of inertia of the frame about the three axes as shown 40 A ball rolls without slipping. The radius of gyration
in figure are I 1, I 2 and I 3 , respectively. Select the of the ball about an axis passing through its centre of
correct alternative. mass is K. If radius of the ball be R, then the fraction
I3 I2 of total energy associated with its rotational energy
will be
K2 R2 K2 + R 2 K2
(a) (b) (c) (d)
K2 + R 2 K2 + R 2 R2 R2

I1 41 Two discs have same mass and thickness. Their


materials are made of densities d 1 and d 2 . The ratio
of their moments of inertia about an axis passing
(a) I 2 = I 3 > I1 (b) I1 > I 2 > I 3
through the centre and perpendicular to the plane is
2 2
(c) I 2 = I 3 < I1 (d) I1 < I 2 < I 3 d  d 
(a) d1 : d 2 (b) d 2 : d1 (c)  1  (d)  2 
36 A merry-go-round made of a ring-like platform of d2  d1 
radius R and mass M, is revolving with angular speed 1
ω. A person of mass M is standing on it. At one 42 If earth suddenly shrinks to th of its original
8
instant, the person jumps off the round, radially volume while mass remaining the same, then
away from the centre of the round (as seen from the duration of day (in hour) will be
round). The speed of the round afterwards is (a) 6 (b) 8
[NCERT Exemplar] (c) 12 (d) 3
ω
(a) 2 ω (b) ω (c) (d) 0 43 The speed of a homogeneous solid sphere after
2
rolling down an inclined plane of vertical height h,
37 A disc is rolling without slipping on a horizontal from rest without sliding is
surface with C as its centre and Q and P the two  g
points equidistant from C. Let v P , v Q and v C be the (a) gh (b)   gh
 5
magnitudes of velocities of points P, Q and C
respectively, then  4 10
(c)   gh (d)   gh
 3  7
Q
44 The moment of inertia of a cube of mass m and side
C a about one of its edges is equal to
P 2 4
(a) ma 2 (b) ma 2
3 3
(a) vQ > vC > vP (b) vQ < vC < vP 8
1 (c) 3ma 2 (d) ma 2
(c) vQ = vP , vC = vP (d) vQ < vC > vP 3
2
402 OBJECTIVE Physics Vol. 1

45 Figure represents the moment of inertia of the solid sphere is M and its radius is R, then what is the
sphere about an axis parallel to the diameter of the angular momentum of the sphere about the point of
solid sphere and at a distance x from it. Which one of contact?
the following represents the variations of I with x? 5 7 3 1
(a) Mv 0R (b) Mv 0R (c) Mv 0R (d) Mv 0R
2 5 5 2
51 The ratio of the radii of gyration of a circular disc
C x and a circular ring of the same radii about a
tangential axis perpendicular to plane of disc or ring
is
I I (a) 1 : 2 (b) 5: 6 (c) 2 : 3 (d) 3 :2
52 The ratio of the radii of gyration of a circular disc
(a) (b) and a circular ring of the same radius about a
x x tangential axis in the plane is
I I (a) 3: 4 (b) 5: 6
(c) 6: 5 (d) 4: 3
(c) (d)
53 A particle of mass m is projected with a velocity v
x x
O O making an angle of 45° with the horizontal. The
magnitude of angular momentum of projectile about
46 Two uniform, thin identical rods each of mass M and
the point of projection when the particle is at its
length l are joined together to form a cross. What maximum height h is
will be the moment of inertia of the cross about an mvh
axis passing through the point at which the two rods (a) zero (b) (c) mvh (d) 2 mvh
2
are joined and perpendicular to the plane of the
cross? 54 A particle of mass m is moving in Z
Ml 2
Ml 2
Ml 2
Ml 2 YZ-plane with a uniform velocity
(a) (b) (c) (d) v with its trajectory running a
12 6 4 3 v
parallel to + veY-axis and
47 A disc of radius R rolls on a rough horizontal surface. intersecting Z-axis at z = a as
The distance covered by the point A in one shown in figure. The change in its Y
revolution is angular momentum about the origin as it bounces
R elastically from a wall at y = constant is
[NCERT Exemplar]
C (a) mva e$ x (b) 2 mva e$ x (c) ymv e$ x (d) 2 ymv e$ x
A 55 A ring is kept on a rough inclined surface. But the
(a) 2πR (b) 2R (c) 8R (d) πR
coefficient of friction is less than the minimum
value required for pure rolling. At any instant of
48 When a body is projected at an angle with the time, let K T and K R be the translational and
horizontal in a uniform gravitational field of the rotational kinetic energies of the ring respectively,
earth, the angular momentum of the body about the then
point of projection, as it proceeds along its path
(a) KR = KT (b) KR > KT (c) KT > KR (d) KR = 0
(a) remains constant
(b) increases 56 A ring and a disc of different masses are rotating
(c) decreases with the same kinetic energy. If we apply a
(d) initially decreases and after its highest point increases retarding torque τ on the ring, it stops after making n
49 A particle of mass m = 5 units is moving with a revolutions. After how many revolutions will the
disc stop, if the retarding torque on it is also τ?
uniform speed v = 3 2 units in the XY-plane along
n
the line y = x + 4 . The magnitude of the angular (a)
2
(b) n
momentum about origin is (c) 2n (d) Data insufficient
(a) zero (b) 60 units (c) 7.5 units (d) 40 2 units
57 O is the centre of an equilateral triangle ABC. F1, F2
50 A sphere rolls without slipping on a rough horizontal and F3 are three forces acting along the sides AB, BC
surface with centre of mass speed v 0 . If mass of the and AC respectively as shown in figure.
Rotation 403

What should be the magnitude of F3 , so that the total 61 A uniform rod of mass 2 kg and length 1 m lies on a
torque about O is zero? smooth horizontal plane. A particle of mass 1 kg
A
moving at a speed of 2 ms −1 perpendicular to the
1
length of the rod strikes it at a distance m from the
O
4
F3 centre and stops. What is the angular velocity of the
B
F1 C
F2 rod about its centre just after the collision?
F1 + F 2 (a) 3 rad s −1 (b) 4 rad s −1 (c) 1 rad s −1 (d) 2 rad s −1
(a) (b) F1 − F 2 (c) F1 + F 2 (d) 2(F1 + F 2 )
2 62 For the uniform T shaped structure with mass 3M,
58 Four point masses each of mass m are placed at the moment of inertia about an axis normal to the plane
corners of a square ABCD of side l. The moment of and passing through O would be
inertia of the system about an axis passing through A 2l
and parallel to BD is O
Y

A
B l

l
X
O
2
D l C (a) Ml 2 (b) Ml 2
3
(a) 3 ml 2 (b) 3 ml 2 (c) ml 2 (d) 2 ml 2 Ml 2
(c) (d) None of these
3
59 A table fan rotating at a speed of 2400 rpm, is
switched OFF and the resulting variation of the rpm 63 A disc is free to rotate about a smooth horizontal axis
with time as shown in the figure. The total number passing through its centre of mass. A particle is fixed
of revolutions of the fan before it comes to rest is at the top of the disc. A slight push is given to the
disc and it starts rotating. During the process,
2400

rpm

600

t (s)
8 24
(a) 420 (b) 190 (c) 280 (d) 380 (a) only mechanical energy is conserved
60 Forces are applied on a wheel of radius 20 cm as (b) only angular momentum (about the axis of rotation) is
conserved
shown in the figure. The torque produced by the
(c) Both mechanical energy and angular momentum are
forces 4 N at A, 8 N at B, 6 N at C and 9 N at D at conserved
angles indicated is (d) Neither the mechanical energy nor the angular
4N momentum are conserved
A
90° 64 A solid sphere rolls without slipping and presses a
spring of spring constant k as shown in figure. Then,
30° 20 cm D the compression in the spring will be
8N
B 90°
M
v
k
C 9N

6N
(a) 5.4 N-m anti-clockwise (b) 1.80 N-m clockwise 2M 2M 5k 7M
(c) 2.0 N-m clockwise (d) 3.6 N-m clockwise (a) v (b) v (c) v (d) v
3k 5k 7M 5k
404 OBJECTIVE Physics Vol. 1

65 The figure shows a uniform rod lying along the 69 A square is made by joining four rods P
X-axis. The locus of all the points lying on the each of mass M and length L. Its
XY-plane, about which the moment of inertia of the moment of inertia about an axis PQ, 45°
rod is same as that about O is in its plane and passing through one
Y of its corner is
4 L
(a) 6ML 2 (b) ML 2
3
8 10 Q
(c) ML 2 (d) ML 2
X 3 3
O

(a) an ellipse 70 A rod of length L whose lower end is fixed along the
(b) a circle horizontal plane starts to topple from the vertical
(c) a parabola position. The velocity of the upper end of the rod
(d) a straight line when it hits the ground is
66 A portion of a ring of radius Y (a) 3gL (b) 2gL (c) gL (d) 5gL
R has been removed as 71 A cord is wound around the circumference of wheel
shown in figure. Mass of the of radius r. The axis of the wheel is horizontal and
A
remaining portion is m. O
X
moment of inertia is I. A weight mg is attached to
Centre of the ring is at origin the end of the cord and falls from rest. After falling
O. Let I A and I O be the through a distance h, the angular velocity of the
moments of inertia passing wheel will be
through points A and O are
2gh 2mgh
perpendicular to the plane of the ring. Then, (a) (b)
I + mr 2
I + mr 2
(a) IO = mR 2 (b) IO = I A
2mgh
(c) IO > I A (d) I A > IO (c) (d) 2gh
I + 2mr 2
67 A wheel comprises a ring of radius R and mass M
and three spokes each of mass m. The moment of 72 A particle P is moving in a circle of radius a with
inertia of the wheel about its axis is uniform speed u. C is the centre of the circle and AB
is its diameter. The angular velocities of P about A
and C are in the ratio
(a) 1 : 1 (b) 1 : 2 (c) 2 : 1 (d) 4 : 1
R 73 The moment of inertia of a semicircular ring of mass
M and radius R about an axis which is passing
through its centre and at an angle θ with the line
joining its ends as shown in figure, is
 m
(a) M +  R 2 (b) (M + m ) R 2
 4 R
M + m  2 θ
(c) (M + 3m ) R 2
(d)  R
 2  MR 2 MR 2
(a) , if θ = 0° (b) , if θ = 0°
68 A thin bar of mass m and length l is free to rotate 4 2
about a fixed horizontal axis through a point at its MR 2 MR 2
(c) , if θ = any angle (d) , if θ = 90°
end. The bar is brought to a horizontal position 2 2
(θ = 90 °) and then released. The angular velocity 74 Two uniform rods of equal length
when it reaches the lowest point is but different masses are rigidly
(a) directly proportional to its length and inversely joined to form an L-shaped body, m M
proportional to its mass
(b) independent of mass and inversely proportional to the
which is then pivoted as shown in 90°
square root of its length figure. If in equilibrium, the body 60°
(c) dependent only upon the acceleration due to gravity is in the shown configuration, ratio
and the mass of the bar M /m will be
(d) directly proportional to its length and inversely
proportional to the acceleration due to gravity (a) 2 (b) 3 (c) 2 (d) 3
(B) Medical entrance special format questions
Reason For a body performing pure rolling motion,
Assertion and reason the angular momentum is conserved about any point
Directions (Q. Nos. 1-5) These questions consists of two in space.
statements each printed as Assertion and Reason. While
answering these questions, you are required to choose any Statement based questions
one of the following four responses.
1 When a disc rotates with uniform angular velocity,
(a) If both Assertion and Reason are correct and Reason is the choose the incorrect statement. [NCERT Exemplar]
correct explanation of Assertion.
(b) If both Assertion and Reason are correct but Reason is not (a) The sense of rotation remains same.
the correct explanation of Assertion. (b) The orientation of the axis of rotation remains same.
(c) If Assertion is correct but Reason is incorrect. (c) The speed of rotation is non-zero and remains same.
(d) If Assertion is incorrect but Reason is correct. (d) The angular acceleration is non-zero and remains
same.
1 Assertion A body is moving along a circle with a
constant speed. Its angular momentum about the 2 A particle is moving in a circular orbit with
centre of the circle remains constant. constant speed. Choose the incorrect statement.
(a) Its linear momentum is conserved.
Reason In this situation, a constant non-zero torque (b) Its angular momentum is conserved.
acts on the body. (c) It is moving with variable velocity.
2 Assertion If a particle moves with a constant (d) It is moving with variable acceleration.
velocity, then angular momentum of this particle 3 A sphere can roll on a surface inclined at an angle θ,
about any point remains constant. if the friction coefficient µ > (2 / 7) g sin θ. Now,
Reason Angular momentum has the units of Planck’s suppose the friction coefficient is (1 / 7) g sin θ and
constant. the sphere is released from rest on the incline, then
3 Assertion Two identical solid spheres are rotated choose the correct statement.
from rest to same angular velocity ω about two (a) The sphere will stay at rest.
different axes as shown in figure. More work has to (b) The sphere will make pure translational motion.
be done to rotate the sphere in case 2. (c) The sphere will translate and rotate about the centre.
(d) The angular momentum of the sphere about its centre
Reason Moment of inertia in case 2 is more. will remain constant.
1 2
4 Consider three solid spheres, sphere (i) has radius r
and mass m, sphere (ii) has radius r and mass 3m,
sphere (iii) has radius 3r and mass m. All can be
placed at the same point on the same inclined
plane, where they will roll without slipping to the
bottom. If allowed to roll down the incline, then at
the bottom of the incline, choose the correct
4 Assertion A solid sphere cannot roll without statement.
slipping on smooth horizontal surface. (a) Sphere (i) will have the largest speed.
(b) Sphere (ii) will have the largest speed.
Reason If the sphere is left free on smooth inclined
(c) Sphere (iii) will have the largest kinetic energy.
surface, it cannot roll without slipping.
(d) All the spheres will have equal speeds.
5 Assertion A uniform disc of radius R is performing
5 A solid homogeneous sphere is moving on a rough
impure rolling motion on a rough horizontal plane as
horizontal surface, partly rolling and partly sliding.
shown in figure. After some time, the disc comes to
Then, choose the correct statement.
ω R
rest. It is possible only when v 0 = 0 . (a) Total kinetic energy is conserved.
2 (b) Angular momentum of the sphere about the point of
contact with the plane is conserved.
(c) Only the rotational kinetic energy about centre of
ω0 v0 mass is conserved.
(d) Angular momentum about centre of mass is
conserved.
406 OBJECTIVE Physics Vol. 1

6 I. Angular momentum of a particle moving in a Codes


straight line is always constant with respect to A B C D A B C D
fixed point. (a) r s p q (b) q p s r
(c) s r q p (d) p r q s
II. Moment of inertia of a body remains same
irrespective of position of axis of rotation. 3 A solid sphere is rotating about an
Which of the statement(s) is/are correct? axis as shown in figure. An insect Insect
(a) Only I (b) Only II follows the dotted path on the
(c) Both I and II (d) Neither I nor II circumference of sphere as shown in
7 I. Speed of any point on rigid body executing rolling the figure.
motion can be calculated by the expression v = rω, Then, match the following columns
where r is the distance of point from instantaneous and mark the correct option from the codes given
centre of rotation. below.
II. Rolling motion of rigid body can be considered as a Column I Column II
pure rotation about instantaneous centre of rotation.
(A) Moment of inertia (p) will remain constant
Which of the above statement(s) is/are correct?
(B) Angular velocity (q) will first increase, then
(a) Only I (b) Only II
decrease
(c) Both I and II (d) Neither I nor II
(C) Angular momentum (r) will first decrease, then
increase
Match the columns (D) Rotational kinetic energy (s) will continuously decrease
1 If radius of earth is reduced to half without changing (t) will continuously increase
its mass, then match the following columns and mark
the correct option from the codes given below. Codes
A B C D A B C D
Column I Column II (a) p r q t (b) s q r p
(A) Angular velocity of earth (p) will become two times (c) s t r p (d) q r p r

(B) Time period of rotation of (q) will become four times 4 Four rods of equal length l and each of mass m form
earth a square as shown in figure.
(C) Rotational kinetic energy of (r) will remain constant Moments of inertia about three axes 1, 2 and 3 are
earth say I 1, I 2 and I 3 . Then, match the following columns
and mark the correct option from the codes given
(s) None
below.
Codes 3
A B C A B C
(a) r s p (b) q r p
1
(c) q s q (d) q p s
2 A disc rolls on ground 2
without slipping. Velocity
C v
of centre of mass is v. Column I Column II
There is a point P on the θ 4 2
circumference of disc at P (A) I1 (p) ml
3
angle θ. Suppose v P is the
2 2
speed of this point. Then, match the following (B) I2 (q) ml
columns and mark the correct option from the codes 3
given below. 1 2
ml
(C) I3 (r) 2
Column I Column II
(s) None
(A) If θ = 60° (p) v P = 2v

(B) If θ = 90° (q) v P = v


Codes
A B C A B C
(C) If θ = 120° (r) v P = 2v (a) q s p (b) p s q
(D) If θ = 180° (s) vP = 3v (c) r p q (d) q s q
(C) Medical entrances’ gallery
Collection of questions asked in NEET & various medical entrance exams

1 Find the torque about the origin when a force of 9 The moment of the force F = 4$i + 5$j − 6k$ at
3 $j N acts on the particle whose position vector is (2, 0, − 3 ), about the point (2, − 2, − 2) is given by
[NEET 2018]
2 k$ m. [NEET 2020]
(a) −7i$ − 8$j − 4k$ (b) −4i$ − $j − 8k$
(a) 6 $j N-m (b) − 6 i$ N-m (c) 6 k$ N-m (d) 6 i$ N-m
(c) −8$i − 4$j − 7k$ (d) −7$i − 4$j − 8k$
2 The angular speed of the wheel of a vehicle is
increased from 360 rpm to 1200 rpm in 14 s. Its 10 A solid sphere is rotating freely about its symmetry
angular acceleration is [NEET 2020] axis in free space. The radius of the sphere is
2 2 increased keeping its mass same. Which of the
(a) 2π rad/s (b) 28π rad/s
following physical quantities would remain constant
(c) 120π rad/s 2 (d) 1 rad/s 2
for the sphere? [NEET 2018]
3 A solid cylinder of mass 2 kg and radius 4 cm is (a) Rotational kinetic energy
rotating about its axis at the rate of 3 rpm. The (b) Moment of inertia
torque required to stop after 2π revolutions is (c) Angular velocity
[NEET 2019] (d) Angular momentum
(a) 2 × 10−3 N-m (b) 12 × 10−4 N-m
11 A solid sphere is in rolling motion. In rolling motion,
(c) 2 × 106 N-m (d) 2 × 10−6 N-m a body possesses translational kinetic energy (K t ) as
4 A disc of radius 2 m and mass 100 kg rolls on a well as rotational kinetic energy (K r )
horizontal floor. Its centre of mass has speed of simultaneously. The ratio K t :(K t + K r ) for the sphere
20 cm s −1. How much work is needed to stop it? is [NEET 2018]
[NEET 2019] (a) 10 : 7 (b) 5 : 7
(a) 30 kJ (b) 2 J (c) 1 J (d) 3 J (c) 7 : 10 (d) 2 : 5
5 A solid cylinder of mass 2 kg and radius 50 cm rolls 12 A thin horizontal circular disc is rotating about a
up an inclined plane of angle inclination 30°. The vertical axis passing through its centre. An insect is
centre of mass of cylinder has speed of 4 ms −1. The at rest at a point near the rim of disc. The insect
distance travelled by the cylinder on the inclined now moves along a diameter of the disc to reach its
surface will be other end. During the journey of the insect, the
angular speed of the disc
(Take, g = 10 ms −2 )
[AIIMS 2018]
[NEET (Odisha) 2019]
(a) continuously decreases
(a) 2.2 m (b) 1.6 m (c) 1.2 m (d) 2.4 m (b) continuously increases
6 A sphere pure rolls on a rough inclined plane with (c) first increases and then decreases
initial velocity 2.8 ms −1. Find the maximum (d) remains unchanged
distance on the inclined plane. [AIIMS 2019] 13 Find the ratio of radius of gyration of a disc and ring
of same radii at their tangential axis in plane.
[JIPMER 2018]
5 5 2
(a) (b) (c) 1 (d)
6 3 3

30º 14 Which of the following statements are correct?


[NEET 2017]
(a) 2.74 m (b) 5.48 m (c) 1.38 m (d) 3.2 m I. Centre of mass of a body usually coincides
7 Two discs have mass ratio 1 : 2 and diameter ratio with the centre of gravity of the body.
2 : 1, then find the ratio of their moments of inertia. II. Centre of mass of a body is the point at which the
[JIPMER 2019] total gravitational torque on the body is zero.
(a) 2 : 1 (b) 1 : 1 (c) 1 : 2 (d) 2 : 3 III. A couple on a body produces both translational and
8 If an object starts from rest and covers angle of rotational motion in a body.
60 rad in 10 s in circular motion, then magnitude of IV. Mechanical advantage greater than one means that
its angular acceleration will be [JIPMER 2019] small effort can be used to lift a large load.
(a) 1.2 rad s −2 (b) 1.5 rad s −2 (a) II and IV (b) I and II
(c) II and III (d) III and IV
(c) 2 rad s −2 (d) 2.5 rad s −2
408 OBJECTIVE Physics Vol. 1

15 Two discs of same moment of inertia rotating about When it has turned through an angle θ, its angular
their regular axis passing through centre and velocity ω is given by [JIPMER 2017]
perpendicular to the plane of disc with angular  6g  θ  6g  θ
(a)   sin (b)   cos
velocities ω1 and ω 2 . They are brought into contact  l  2  l  2
face to face coinciding the axis of rotation. The
 6g   6g 
expression for loss of energy during this process is (c)   sin θ (d)   cos θ
[NEET 2017]  l   l 
1 1
(a) I(ω1 + ω 2 )2 (b) I(ω1 − ω 2 )2 21 A bicycle wheel rolls without slipping on a
2 4
I
horizontal floor. Which one of the following is true
(c) I(ω1 − ω 2 )2 (d) (ω1 − ω 2 )2 about the motion of points on the rim of the wheel,
8
relative to the axis at the wheel’s centre?
16 A rope is wound around a hollow cylinder of mass [JIPMER 2017]
3 kg and radius 40 cm. What is the angular
acceleration of the cylinder, if the rope is pulled
with a force of 30 N? [NEET 2017]
(a) 25 ms −2 (b) 0.25 rad s −2 (c) 25 rad s −2 (d) 5 ms −2
17 Assertion The total kinetic energy of a rolling solid (a) Points near the top move faster than points near the
sphere is the sum of translational and rotational bottom
kinetic energies. (b) Points near the bottom move faster than points near the
top
Reason For all solid bodies, total kinetic energy is (c) All points on the rim move with the same speed
always twice of translational kinetic energy. (d) All points have the velocity vectors that are
[AIIMS 2017]
pointing in the radial direction towards the centre of
(a) Both Assertion and Reason are correct and Reason is the wheel
the correct explanation of Assertion.
(b) Both Assertion and Reason are correct but Reason is not 22 A cylinder rolls up an inclined plane, reaches some
the correct explanation of Assertion. height and then rolls down (without slipping
(c) Assertion is correct but Reason is incorrect. throughout these motions). The directions of the
(d) Both Assertion and Reason are incorrect. frictional force acting on the cylinder are
[JIPMER 2017]
18 A thin hollow sphere of mass m is completely filled
(a) up the incline while ascending and down the incline
with a liquid of mass m. When the sphere rolls with while descending
a velocity v, kinetic energy of the system is (neglect (b) up the incline while ascending as well as descending
friction) [AIIMS 2017] (c) down the incline while ascending and up the incline
(a) (1/2) mv 2 (b) mv 2 (c) (4 /3) mv 2 (d) (4 / 5) mv 2 while descending
(d) down the incline while ascending as well as descending
19 ABC is right angled triangular plane of uniform
23 A wheel of radius 10 cm can
thickness. The sides are such that AB > BC as shown
rotate freely about its centre as
in figure. I 1, I 2, I 3 are moments of inertia about AB,
shown in figure. A string
BC and AC, respectively. Then, which of the
wrapped over its rim is pulled by
following relations is correct? [JIPMER 2017]
A
a force of 5N. It is observed that
the torque produces angular 5N
I3 acceleration of 2 rad s −2 in the
I1
wheel. What is the moment of inertia of the wheel?
[NEET 2016]
2 2
B
I2
C (a) 0.25 kg-m (b) 0.45 kg-m
(a) I1 = I 2 = I 3 (b) I 2 > I1 > I 3 (c) 0.15 kg-m2 (d) 0.16 kg-m2
(c) I 3 < I 2 < I1 (d) I 3 > I1 > I 2 24 From a disc of radius R and mass M, a circular hole
A
20 A uniform rod of length l is free to A′
of diameter R, whose rim passes through the centre
rotate in a vertical plane about a is cut. What is the moment of inertia of the
fixed horizontal axis through B. l
remaining part of the disc about a perpendicular
The rod begins rotating from rest axis, passing through the centre ? [NEET 2016]
θ
from its unstable equilibrium position. (a) 13 MR 2/32 (b) 11 MR 2/32
(c) 9 MR 2/32 (d) 15 MR 2/32
Rotation 409

25 A disc and a sphere of same radius but different 30 A massless rod S having length 2l has ω
masses roll off on two inclined planes of the same equal point masses attached to its two l
altitude and length. Which one of the two objects ends as shown in figure. The rod is m
gets to the bottom of the plane first? [NEET 2016] rotating about an axis passing through α
(a) Sphere its centre and making an angle α Rod

(b) Both reach at the same time with the axis. The magnitude of l
(c) Depends on their masses dL
(d) Disc change of momentum of rod, i.e. m
dt
26 A uniform circular disc of radius 50 cm at rest is equals [AIIMS 2015] axis
free to turn about an axis which is perpendicular to (a) 2 m l ω sin θ ⋅ cos θ
3 2
(b) ml ω sin 2θ
2 2
its plane and passes through its centre. It is subjected
(c) ml sin 2θ
2
(d) m1/ 2 l1/ 2 ω sin θ ⋅ cos θ
to a torque which produces a constant angular
acceleration of 2 rad s −2 . Its net acceleration (in ms −2 ) 31 A uniform sphere of mass 500 g rolls without
at the end of 2 s is approximately [NEET 2016] slipping on a plane surface, so that its centre moves
(a) 7 (b) 6 at a speed of 0.02 ms −1. The total kinetic energy of
(c) 3 (d) 8 rolling sphere would be (in joule) [AIIMS 2015]

27 A mass m moves in a circle on a smooth horizontal (a) 1.4 × 10− 4 (b) 0.75 × 10− 3
plane with velocity v 0 at a radius R 0 . The mass is (c) 5.75 × 10− 3 (d) 4.9 × 10− 5
attached to a string which passes through a smooth 32 A particle travels in a circle of radius 20 cm at a
hole in the plane as shown in the figure. uniformly increasing speed. If the speed changes
The tension in the string is increased gradually and from 8ms −1 to 9ms −1 in 2s, what would be the
finally m moves in a circle of radius R 0 /2. The final
angular acceleration (in rad s −2 )? [UK PMT 2015]
value of the kinetic energy is [CBSE AIPMT 2015]
(a) 1.5 rad s−2 (b) 2.5 rad s−2
v0 (c) 3.5 rad s−2 (d) 4.5 rad s−2
m
R0 33 A solid sphere of radius r is rolling on a horizontal
surface. The ratio between the rotational kinetic
energy and total energy
[EAMCET 2015, AIIMS 2012, JCECE 2010]
1 1 5 2 1 1
(a) mv 02 (b) mv 02 (c) 2 mv 02 (d) mv 02 (a) (b) (c) (d)
4 2 7 7 2 7
28 Three identical spherical shells, each of X 34 If two circular discs A and B are of same mass but of
mass m and radius r are placed as radii r and 2r respectively, then the moment of
shown in figure. Consider an axis XX′, inertia of A is [Kerala CEE 2015]
which is touching the two shells and (a) one-fourth that of B (b) twice that of B
passing through diameter of third shell. (c) four times that of B (d) half that of B
Moment of inertia of the system (e) same as that of B
consisting of these three spherical shells X′ 35 Choose the wrong statement. [Kerala CEE 2015]
about XX′ axis is [CBSE AIPMT 2015] (a) The centre of mass of a uniform circular ring is at its
11 2 geometric centre.
(a) mr (b) 3 mr 2
5 (b) Moment of inertia is a tensor quantity.
16 2 (c) Radius of gyration is a vector quantity.
(c) mr (d) 4 mr 2
5 (d) Force in translational motion is analogous to torque in
rotational motion.
29 A uniform metallic rod rotates about its perpendicular (e) For same mass and radius, the moment of inertia of a
bisector with constant angular speed. If it is heated ring is twice that of a uniform disc.
uniformly to raise its temperature slightly, then
[AIIMS 2015] 36 Two particles A and B are moving as shown in the
(a) its speed of rotation increases figure.
(b) its speed of rotation decreases 6.5 kg 2.2 ms−1
A 3.6 ms−1
(c) its speed of rotation remains same
1.5 m
(d) its speed increases because its moment of inertia
increases B
O 2.8 m 3.1 kg
410 OBJECTIVE Physics Vol. 1

Their total angular momentum about the point O is to point Q as shown in figure. When string is cut,
[WB JEE 2015] the initial angular acceleration of the rod is
(a) 9.8 kg-m2s−1 (b) zero
(c) 52.7 kg-m2s−1 (d) 37.9 kg-m2s−1
37 A rod of mass 5 kg is connected to the string at point
B. The span of rod is along horizontal. The other end P Q
L
of the rod is hinged at point A. If the string is massless, [NEET 2013]
then the reaction of hinge at the instant when string 3g g 2g 2g
(a) (b) (c) (d)
is cut, is (Take, g = 10 ms −2 ) [UP CPMT 2015] 2L L L 3L
(a) 10.1 N (b) 12.5 N 45 A small object of uniform density rolls up a curved
(c) 5 N (d) 15 N surface with an initial velocity v ′. It reaches upto a
38 A particle moves with a constant velocity parallel to maximum height of 3v 2 /4 g with respect to the
the X-axis. Its angular momentum with respect to initial position. The object is [NEET 2013]
the origin [UP CPMT 2015] (a) ring (b) solid sphere
(a) is zero (b) remains constant (c) hollow sphere (d) disc
(c) goes on increasing (d) goes on decreasing
46 The conservation of angular momentum demands
39 A ring of radius 0.5 m and mass 10 kg is rotating that [J&K CET 2013]
about its diameter with angular velocity of 20 rad s −1. (a) the external force on the system must be zero
Its kinetic energy is [Manipal 2015] (b) the external torque on the system must be zero
(a) 10 J (b) 100 J (c) 500 J (d) 250 J (c) Both the external force as well as the external torque
must be zero
40 Moment of inertia of ring about its diameter is I. The (d) Neither of them must be zero
moment of inertia of the same ring about that axis
perpendicular to its plane and passing through centre 47 The moment of inertia (I) and the angular
is [KCET 2014] momentum (L) are related by the expression
[J&K CET 2013]
I I
(a) (b) 2I (c) (d) 4I (a) I = Lω (b) L = Iω (c) L = I 2ω (d) ω = LI
2 4
41 Two bodies have their moments of inertia I and 2I 48 The moment of inertia (I) of a sphere of radius R and
respectively, about their axis of rotation. If their mass M is given by [J&K CET 2013]
kinetic energies of rotation are equal, their angular (a) I = MR 2 (b) I = (1 / 2) MR 2
velocities will be in the ratio [UK PMT 2014] (c) I = (4 / 3) MR 2 (d) I = (2 / 5) MR 2
(a) 2 : 1 (b) 1 : 2 (c) 2 : 1 (d) 1 : 2
49 A particle of mass m is attached to a thin uniform
42 A body having a moment of inertia about its axis of a
2 rod of length a at a distance of from the mid-point
rotation equal to 3 kg-m is rotating with angular 4
velocity of 3 rad s –1 . Kinetic energy of this rotating C as shown in the figure. The mass of the rod is 4 m.
body is same as that of a body of mass 27 kg moving The moment of inertia of the combined system about
with a velocity v. The value of v is [KCET 2014] an axis passing through O and perpendicular to the
(a) 1 ms–1 (b) 0.5 ms–1 rod is [EAMCET 2013]
Mass (m)
(c) 2 ms–1 (d) 1.5 ms–1 C
43 A uniform solid spherical ball is rolling down a O
a a
smooth inclined plane from a height h. The velocity 2
x
4
attained by the ball when it reaches the bottom of 91 27 51 64
the inclined plane is v. If the ball is now thrown (a) ma 2 (b) ma 2 (c) ma 2 (d) ma 2
48 48 48 48
vertically upwards with the same velocity v, the
maximum height to which the ball will rise is 50 A particle moving in a circular path has an angular
[WB JEE 2014] momentum of L. If the frequency of rotation is
5h 3h 5h 7h halved, then its angular momentum becomes
(a) (b) (c) (d) [Kerala CEE 2013]
8 5 7 9
L L L
44 A rod PQ of mass M and length L is hinged at end P. (a) (b) L (c) (d)
2 3 4
The rod is kept horizontal by a massless string tied (e) 2L
Rotation 411

51 The torque of a force F = 2$i − 3 $j + 5 k


$ acting at a 60 A rod of length L is composed of a uniform length
point whose position vector r = 3 $i − 3 $j + 5k
$ about the 1/2 L of wood whose mass is m w and a uniform
length 1/2 L of brass whose mass is mb . The moment
origin is [Kerala CEE 2013]
of inertia I of the rod about an axis perpendicular to
(a) −3 $i + 5k$ (b) −5$j + 3k$ the rod and through its centre is equal to [AMU 2012]
(c) −5$j − 3k$ (d) 3$i − 5$j + 3k$ (a) (m w + m b )L2 / 12 (b) (m w + m b )L2 / 6
(e) − $j + 2k$ (c) (m w + m b )L2 / 3 (d) (m w + m b )L2 / 2
52 Moment of inertia of a disc of radius R about a 61 The instantaneous angular position of a point on a
−2 rotating wheel is given by the equation
diametric axis is 25 kg m . The moment of inertia
of the disc about a parallel axes at a distance R /2 θ(t ) = 2t 3 − 6t 2 . The torque on the wheel becomes
from the centre is [MPPMT 2013] zero at [CBSE AIPMT 2011]
(a) 31.25 kg-m2 (b) 37.5 kg-m2 (a) t = 0 . 5 s (b) t = 0 . 25 s
(c) 50 kg-m2 (d) 62.5 kg-m2 (c) t = 2 s (d) t = 1 s
53 A thin hollow sphere of mass m is completely filled 62 The moment of inertia of a thin uniform rod of mass
with a liquid of mass m. When the sphere rolls with M and length L about an axis passing through its
a velocity v, kinetic energy of the system is (neglect mid-point and perpendicular to its length is I 0 . Its
friction) [AIIMS 2012] moment of inertia about an axis passing through one
1 4 4 of its ends and perpendicular to its length is
(a) mv 2 (b) mv 2 (c) mv 2 (d) mv 2 [CBSE AIPMT 2011]
2 3 5
(a) I 0 + ML2/4 (b) I 0 + 2ML2
54 The moment of inertia of a circular loop of radius
(c) I 0 + ML2 (d) I 0 + ML2/2
R, at a distance of R /2 around a rotating axis
parallel to horizontal diameter of loop is [AIIMS 2012] 63 A solid sphere of mass M rolls without slipping on an
(a) MR 2 1
(b) MR 2 (c) 2MR 2 3
(d) MR 2 inclined plane of inclination θ. What should be the
2 4 minimum coefficient of friction, so that the sphere
55 A wheel having moment of inertia 2 kg-m2 about its rolls down without slipping ? [NEET 2011]
2 2
vertical axis, rotates at the rate of 60 rpm about this (a) tan θ (b) tan θ
axis. The torque which can stop the wheel’s rotation 5 7
5
in one minute would be [UP CPMT 2012] (c) tan θ (d) tan θ
π 2π π π 7
(a) N-m (b) N-m (c) N-m (d) N-m
15 15 18 12 64 A force F = 2.0 N acts on a particle P in the
56 What is the moment of inertia of solid sphere of XZ-plane. The force F is parallel to X-axis. The
density ρ and radius R about its diameter? particle P (as shown in the figure) is at a distance
[UP CPMT 2012] 3 m and the line joining P with the origin makes
105 5 105 2 176 5 186 2 angle 30° with the X-axis. The magnitude of torque
(a) R ρ (b) R ρ (c) R ρ (d) R ρ
176 176 105 105 on P w.r.t. origin O (in N-m) is [AMU 2011]

57 The radius of gyration of a body depends upon F


z
(a) shape and size of body [UP CPMT 2012]
(b) nature of mass distribution of body P 3m
(c) choice of axis of rotation y
30°
(d) All of the above O
x
58 If two discs have same mass and thickness but their
densities are ρ1 and ρ 2 , then the ratio of their (a) 2 (b) 3
moments of inertia about central axis will be (c) 4 (d) 5
[BCECE (Mains) 2012]
65 One solid sphere A and another hollow sphere B are
(a) 1 : ρ1 ρ2 (b) ρ1 ρ2 : 1 (c) ρ1 : ρ2 (d) ρ2 : ρ1
of same mass and same outer radius. Their moments
59 If a disc starting from rest acquires an angular of inertia about their diameters are respectively, I A
velocity of 240 rev min −1 in 10 s, then its angular and I B such that
acceleration will be [BCECE (Mains) 2012] (a) I A = IB (b) I A > IB [J&K CET 2011]
(a) 1.52 rad s −1
(b) 3.11 rad s −1 (c) I A < IB (d) None of these
(c) 2.51 rad s −1 (d) 1.13 rad s −1
ANSWERS
CHECK POINT 9.1
1. (c) 2. (d) 3. (c) 4. (c) 5. (d) 6. (b) 7. (a) 8. (b) 9. (a) 10. (a)
11. (b) 12. (a) 13. (a) 14. (c) 15. (b) 16. (a) 17. (d)

CHECK POINT 9.2


1. (b) 2. (c) 3. (a) 4. (d) 5. (a) 6. (a) 7. (a) 8. (a) 9. (b) 10. (d)
11. (d) 12. (b) 13. (a) 14. (a) 15. (a) 16. (b) 17. (c) 18. (b)

CHECK POINT 9.3


1. (a) 2. (b) 3. (c) 4. (b) 5. (c) 6. (d) 7. (d) 8. (d) 9. (d) 10. (d)
11. (b) 12. (a) 13. (d) 14. (c) 15. (b)

(A) Taking it together


1. (b) 2. (b) 3. (a) 4. (d) 5. (b) 6. (c) 7. (a) 8. (c) 9. (c) 10. (d)
11. (d) 12. (a) 13. (d) 14. (d) 15. (a) 16. (c) 17. (c) 18. (d) 19. (a) 20. (a)
21. (b) 22. (b) 23. (c) 24. (a) 25. (b) 26. (b) 27. (b) 28. (a) 29. (a) 30. (a)
31. (d) 32. (c) 33. (b) 34. (b) 35. (c) 36. (a) 37. (a) 38. (c) 39. (c) 40. (a)
41. (b) 42. (a) 43. (d) 44. (a) 45. (d) 46. (b) 47. (c) 48. (b) 49. (b) 50. (b)
51. (d) 52. (b) 53. (b) 54. (b) 55. (c) 56. (b) 57. (c) 58. (b) 59. (c) 60. (b)
61. (a) 62. (b) 63. (a) 64. (d) 65. (b) 66. (d) 67. (b) 68. (b) 69. (c) 70. (a)
71. (b) 72. (b) 73. (c) 74. (d)

(B) Medical entrance special format questions


l Assertion and reason
1. (c) 2. (b) 3. (a) 4. (d) 5. (c)
l Statement based questions
1. (d) 2. (a) 3. (c) 4. (d) 5. (b) 6. (a) 7. (a)

l Match the columns


1. (c) 2. (b) 3. (d) 4. (d)

(C) Medical entrance’s gallery


1. (b) 2. (a) 3. (d) 4. (d) 5. (d) 6. (c) 7. (a) 8. (a) 9. (d) 10. (d)
11. (b) 12. (c) 13. (a) 14. (b) 15. (b) 16. (c) 17. (c) 18. (c) 19. (b) 20. (a)
21. (a) 22. (b) 23 (a) 24. (a) 25. (a) 26. (d) 27. (c) 28. (d) 29. (b) 30. (b)
31. (a) 32. (b) 33. (b) 34. (a) 35. (c) 36. (a) 37. (b) 38. (b) 39. (d) 40. (b)
41. (c) 42. (a) 43. (c) 44. (a) 45. (d) 46. (b) 47. (b) 48. (d) 49. (a) 50. (a)
51. (c) 52. (b) 53. (c) 54. (d) 55. (a) 56. (c) 57. (d) 58. (d) 59. (c) 60. (a)
61. (d) 62. (a) 63. (b) 64. (b) 65. (c)
Hints & Explanations
l CHECK POINT 9.1 8 (b) If angular acceleration is constant, we have
300 1
1 (c) Given, frequency of wheel, ν = 300 rpm = rps = 5 rps θ = ω 0t + αt 2
60 2
Angle described by wheel in one rotation = 2π rad Given, θ = 15°, ω 0 = 0
Therefore, angle described by wheel in 1s = 2π × 5 For the 1st condition (time = t second),
= 10π rad 1
15° = αt 2 …(i)
2 (d) Angular displacement per second, 2
500 × 2π 50 π For the 2nd condition (time = 3t second),
θ = ωt = = rad
60 3 1 1
θ1 = α (3t )2 = (α ) 9t 2 …(ii)
3 (c) Given, θ = 100 π rad, t = 5 s, ω = ? 2 2
θ 100 π So, the increase in angle through which it rotates in the next 2 s,
Angular speed, ω = = = 20 π 1 1 1 1 
t 5 ∆θ = θ1 − αt 2 = 9 × αt 2 − αt 2 = 8  αt 2
2 2 2 2 
4 (c) We know that, ω = 2πn ⇒ ω1 = 2πn1
= 8 × 15° = 120 ° [from Eq. (i)]
where, n1 = 1800 rpm, n 2 = 3000 rpm
1800 9 (a) The role of moment of inertia in the study of rotational
ω1 = 2π × = 2π × 30 = 60 π rad s –1 motion is analogous to that of mass in the study of
60 translational motion.
3000
Similarly, ω 2 = 2πn 2 = 2π × 10 (a) Moment of inertia of a body depends on position and
60
orientation of the axis of rotation with respect to the body.
= 2π × 50 = 100 π rad s –1
11 (b) Perpendicular distance from Z-axis
If the angular velocity of a rotating wheel about an axis changes
due to change in angular velocity in a time interval ∆t, then the = (1)2 + (1)2 = 2 m
angular acceleration of rotating wheel about that axis is ∴ I = Mr 2 = (1) ( 2 )2 = 2 kg- m2
Change in angular velocity
α= 12 (a) Moment of inertia of the rod lying along Z-axis will be
Time interval
zero. Moment of inertia of the rods along X andY-axes will be
ω 2 − ω1 100 π − 60 π
α= ⇒ α= ML 2
∆t 20 each about Z-axis. Hence, total moment of inertia about
3
40 π
α= = 2π rad s −2 2
Z-axis is ML 2.
20 3
5 (d) Angular displacement during given time, θ = (ω 2 − ω1)t 13 (a) Radius of gyration of solid sphere,
Here, ω1 = 100 rad/s, 7
MR 2
ω 2 = 300 rad/s and t = 10 s I 7
K= = 5 = R
⇒ θ = (300 − 100 ) × 10 = 2000 M M 5
Therefore, number of revolutions made during this time ml 2
14 (c) Moment of inertia, I = …(i)
2000 3
= = 318.31 −~ 318
2π New moment of inertia,
6 (b) The kinematical equation for rotational motion is (3m / 4) (3l / 4)2
I′ =
1 3
θ = ω 0t + αt 2
2 27 ml 2  27

=   = I [from Eq. (i)]
Here, α = 3 rad s −2, ω 0 = 2 rad s −1 and t = 2 s 64  3  64
1
Hence, θ = 2 × 2 + × 3 × (2)2 or θ = 4 + 6 = 10 rad 15 (b) Moments of inertia of circular ring and circular disc about
2 the axes passing through their centres and perpendicular to
7 (a) Given, α = 3 rad s −2, ω1 = 10 rad s −1, ω 2 = 20 rad s −1 1
their planes are MR 2 and MR 2, respectively. So, the desired
2
ω 22 − ω12 (20 )2 − (10 )2
Angle, θ = = = 50 rad MR 2
2α 2× 3 ratio is = 2:1
(1/ 2) MR 2
414 OBJECTIVE Physics Vol. 1

16 (a) From theorem of parallel axes, I2 = I1 + MR 2 ∴ | L| = [(8$i − 4$j) × (8$i + 4$j)]


I2 − I1 = MR 2 $i $j k$
⇒ L= 8 −4 0
2R 8 4 0
R R
⇒ L = $i (0 − 0 ) − $j(0 − 0 ) + k$ (32 + 32) ⇒ L = 64 k$ units

10 (d) Angular momentum, L = mvr = mωr 2 = m × ×r2
T
I1 I2
6 × 10 24 × 2 × 3.14 × (1.5 × 1011)2
= = 2.7 × 10 40 kg-m2 s −1
17 (d) According to the question, 3.14 × 10 7
l 11 (d) As, L = r × p = rp sinθ
l = 2πR ⇒ R =
2π Angular momentum L is maximum when p is perpendicular
Ml 2
∴ I1 = to r.
12
12 (b) The angular momentum of the body is given by
Ml 2
and I2 = MR 2 = 2π 1 L T
4π 2 L = Iω or L = I × or L ∝ ⇒ 1= 2
T T L2 T1
I1 π 2
Q = ⇒
L 2T
= (Q T2 = 2T and L1 = L)
I2 3 L2 T
CHECK POINT 9.2 L
⇒ L2 =
l

2
1 (b) Torque, τ = r × F
Thus, on doubling the time period, angular momentum of
⇒ τ = 4$j × (− 6$i ) = − (−24 k$ ) = 24 k$ body becomes half.
2 (c) Torque, τ = r × F = [(0 − 3) $i + (2 − 0 ) $j + (0 − 0 ) k$ ] × [20 $i] dL
13 (a) Torque, τ =
dt
= [−3 $i + 2 $j ] × [20 $i ] = − 40 k$
where, L = angular momentum.
3 (a) Torque of the force, τ = r × F dL
If τ = 0, then = 0.
$i $j k$ dt
i.e. L = constant or conserved.
τ= 7 3 1 = (14$i − 38$j + 16k$ )
dL
−3 1 5 14 (a) Torque = Rate of change of angular momentum =
dt
4 (d) Torque required to open/close the door will remain Given, dL = 4A0 − A0 = 3A0, dt = 4 min
constant. 3
⇒ Torque acting on the body, τ = A0
∴ r1 × F1 = r2 × F2 4
1.6 × 1 = 0.4 × F2 ⇒ F2 = 4 N 15 (a) If no external torque acts on a system of particles, then
5 (a) Given, I = 2 kg-m2, ω = 30 rad s −1 and t = 15 s angular momentum of the system remains constant, i.e. τ = 0.
dL
ω 30 ⇒ = 0 ⇒ L = I ω = constant
Average torque of the force, τ = Iα = I = 2 × = 4 N-m dt
t 15
1 1
6 (a) ω1 = 10 rad s −1, ω 2 = 0, t = 10 s ⇒ I1ω1 = I2ω 2 ⇒ Mr 2ω1 = (M + 2m ) r 2ω 2 …(i)
2 2
ω 2 − ω1 0 − 10 Here, M = 2 kg , m = 0.25 kg , r = 0.2 m, ω1 = 30 rad s −1
∴ α= = = − 1 rad s −2
t 10
Substituting the given values in Eq. (i), we get
Negative sign indicates retardation.
1 1
Now, I = MR 2 = 10 × (0.3)2 = 0.9 kg-m2 × 2 × (0.2)2 × 30 = × (2 + 2 × 0.25) (0.2)2 × ω 2
2 2
∴ Magnitude of torque, τ = Iα = 0.9 × (1) = 0.9 N-m or 60 = 2.5ω 2 or ω 2 = 24 rad s −1
8 (a) As L = r × p, so angular momentum is the moment of
16 (b) According to law of conservation of angular momentum,
momentum.
Iω = constant
9 (b) For a body of mass m rotating with velocity v in a circle of R2  1 1
radius r, angular momentum is given by L = m (r × v) or = constant Q I ∝ R and ω ∝
2
and I ∝ 
T  T ω
For unit mass, m = 1
Rotation 415

∴ T ∝ R2 5 (c) In case of pure rolling,


1 g sin θ I 2
As, R′ = R a= ; for solid sphere, 2
=
n I mR 5
1+
T R
2 mR 2
⇒ =  5
T ′  R′  ∴ a = g sin θ
7
2
T R × n
⇒ =  Since, a is independent of m and R, both the spheres reach
T′  R  the bottom together.
T 24
∴ T′ = 2
= 2h 6 (d) The linear acceleration of the sphere,
n n g sin θ g sin θ 5
a= = = g sin θ
17 (c) From conservation of angular momentum, I 2 7
1+ 1 +
ω1 I1 = ω 2 I2 mR 2 5
ωI 7 (d) The linear acceleration,
⇒ ω 2 = 11
I2 g sin θ I 2
a= and 2
= (For solid sphere)
   M  I mR 5
MR 2 1+
⇒ ω2 = ω   =ω   mR 2
 MR + 2mR 
2 2
 M + 2m 
g sin 30 ° 5g
1 2 ∴ a= =
18 (b) Rotational kinetic energy = Iω = 8 J 1+
2 14
2 5
1 1 2 2 1
⇒ × mr ω = 8 or × 2 × (1)2ω 2 = 8 8 (d) In case of pure rolling,
2 2 4
g sinθ
or ω 2 = 16 or ω = 4 rad s −1 Acceleration, a =
I
1+
Angular momentum, L = Iω =
1 2
mr ω MR 2
2 I
For ring, =1
1 MR 2
= × 2 × (1)2 × 4 = 4 J-s
2 g
∴ a= (Q θ = 30 ° )
4
l CHECK POINT 9.3 g sin θ
9 (d) In case of pure rolling, a =
1 (a) In this case, instantaneous axis of rotation is always below I
1+
the centre of mass. This is possible only when point of contact mR 2
on this axis moves with a speed equal to that of centre of I 2 I 1
mass. = (for solid sphere) and = (for disc).
mR 2 5 mR 2 2
2 (b) Required velocity of the particle at top most point, 5
∴ For solid sphere, a1 = g sin θ
v P = r ω = (2R ) ω = 2v 7
P 2
vP Similarly, for disc, a 2 = g sin θ
3
r R
v 1 2 2s
From s = at , t =
ω 2 a
v = Rω t1 a 2/ 3 14
⇒ = 2 = =
3 (c) Speed of particle, v P = r ω = ( 2 R ) ω = 2 v 0 t2 a1 5/ 7 15

10 (d) For all bodies, a = g sin θ − µg cos θ

v0 ⇒ P v0 As, µ is same for all bodies.


ω r v0 2 v0 Hence, all will take same time.

v0 = Rω 11 (b) In pure rolling, mechanical energy remains conserved.


ω Therefore, at bottom most point, total kinetic energy will be mgh.
4 (b) The acceleration of its centre down the plane, Ktotal = Krot + Ktrans
g sin θ
a= or a < g sin θ 1 2 1 2 1 2  v  1
2

1+
I = Iω + mv =  mR 2  2  + mv 2
mR 2 2 2 2 3  R  2
416 OBJECTIVE Physics Vol. 1

5 2 τ fR 5f
Ktotal = mv Angular acceleration, α = = =
2
6 I mR 2 2mR
5 2 5
Now, mv = mgh
6  5f  5f
Also, a = Rα ⇒ a = R   =
∴The speed of a uniform spherical shell,  2mR  2m
5f
6gh a= …(ii)
v= 20
5
Equating Eqs. (i) and (ii), we get
12 (a) Time taken by rolling body to reach the bottom, 7 − f 5f
= ⇒ f = 2N
1 2h  K 2  10 20
t= 1+ 
sinθ g  R2
(A) Taking it together
K2
A body with smaller value of will take less time to reach dL dL
R2 1 (b) Torque, τ = . When τ = 0, then =0
dt dt
the bottom. Hence, solid sphere will take least time to reach
the bottom. ⇒ L = constant, i.e. L is conserved.

13 (d) The relation between linear velocity (v ) and angular 2 (b) We have, Iω = L = constant ⇒ ω ∝ 1/I
velocity (ω) is On stretching hands, I will increase, therefore ω will decrease.
v
v =rω ⇒ ω = 3 (a) On bending the head before diving in swimming pool, the
r moment of inertia of the diver decreases.
1 2 1 2
Total kinetic energy = mv + Iω 4 (d) It cannot be zero, otherwise net torque or net angular
2 2
acceleration will become zero.
1 2 1 1  v 
2
= mv + ×  mr 2  2  5 (b) Moment of inertia of the system,
2 2 2  r 
I = 5mR 2 = 5 × 2 × (0.1)2 = 0.1 kg-m2
1 1 3
= mv 2 + mv 2 = mv 2 1 2 2E
2 4 4 6 (c) Rotational kinetic energy, E = Iω ⇒ ω =
3 2 I
mv 2 = mg (3) ⇒ v = 2 10 ms −1
4 Now, angular momentum, L = Iω = 2EI
v 2 10 900 × 2π
Now, r= = = 5m 7 (a) Angular velocity, ω 0 = rad s −1 = 30 π rad s −1
ω 2 2 60
Now, 0 = ω 0 − αt
14 (c) Total kinetic energy of a body which is rolling without
ω 0 30 π π
slipping is given by Ktotal = Krot + Ktrans ∴ Angular retardation, α = = rad s −2 = rad s −2
t 60 2
2
For solid spherical ball, I = mR 2 and v = Rω 1
5 8 (c) We have, θ1 = (α ) (2)2 = 2α
(Along the diameter) 2
where, R is radius of spherical ball. 1 1
and θ 2 = α (4)2 − α (2)2 = 6α
12  1 2 2
So, Ktotal =  mR 2 ω 2 + mR 2ω 2 θ2 3
25  2 Q =
θ1 1
7 7
= mR 2ω 2 ⇒ Ktotal = mv 2
10 10 9 (c) Angular velocity, ω = αt
Potential energy = Kinetic energy ω 80
∴ α= = = 16 rad s −2
7 10 t 5
mgh = mv 2 ⇒ v 2 = gh …(i)
10 7 1 1
Now, θ = αt 2 = (16) (5)2 = 200 rad
For vertical projection, 2 2
v ′ 2 = v 2 − 2gh′ ⇒ 0 = v 2 − 2gh′ ⇒ v 2 = 2gh′ 10 (d) Here, r = 0.2 m, M = 10 kg, ν = 1200 rpm = 20 rps
10 5 ∴Angular momentum, L = Iω = (Mr 2 )(2πν )
⇒ gh = 2gh′ ⇒ h′ = h
7 7 22
= 10 × (0. 2)2 × 2 × × 20 = 50.28 kg-m2s −1
15 (b) Let f be the frictional force towards left for pure rolling. 7
F −f 7− f 11 (d) If the distance of point from axis (r) will increase, then
Now, linear acceleration, a = = …(i)
m 10 corresponding v of the point will also increase accordingly.
Hence, ω is constant or independent of r.
Rotation 417

12 (a) Angular momentum, L = Iω = I (2πf ) 1


20 (a) Angular momentum, L = mvR + ICω = mvR + mvR
2
Frequency f is doubled. Hence, angular momentum will 3
become 2L. = mvR
2
13 (d) About origin, angular momentum will be
21 (b) On inclined surface, gravitational force provides torque.
 $i $j k$  On rough surface, friction provides torque for rolling but on

L = m (r × v ) = 2 1 1 0 = 4$i − 4$j − 8k$ horizontal smooth surface, external force is needed to create
  torque.
 2 −2 2  K 2
22 (b) In case of pure rolling of sphere, R =
Therefore, component of angular momentum about Z-axis KT 5
would be − 8 kg-m2s −1. ∴ Total kinetic energy = 7/ 5 times the translational kinetic
τ 1000 energy. At highest point, whole of the kinetic energy will be
14 (d) Angular acceleration, α = = = 5 rad s −2 converted into potential energy.
I 200
7  1 2 7v 2 7(10 )2
Now, angular velocity, ω = αt = (5) (3) = 15 rad s −1 ∴ mgh =  mv  or h = = = 7m
5 2  10 g 10 × 10
15 (a) ∴ I = mK 2 = 10 (2)2 = 40 kg-m2
23 (c) According to the question,
Kinetic energy of rotation,
(KT )sphere = (KT )cylinder (m and v are same)
1 1
KR = Iω 2 = (40 ) (5)2 = 500 J 5 2
2 2 ∴ (mgh1) = (mgh2 )
7 3
16 (c) The given situation of rod is shown below. h1 14
⇒ =
Y h2 15
24 (a) Torque, τ = r × F, i.e. τ is perpendicular to both r and F.
2x
y=

P ∴ r ⋅ τ = 0 and F ⋅ τ = 0
25 (b) From the conservation of angular momentum, I1ω1 = I2ω 2
y I1 (1/ 2 ma 2 ) ω
⇒ ω2 = ω1 = ω=
I2 (1/ 2 ma ) + ma
2 2
3
a X 26 (b) In the given diagrams, when the small piece Q removed
O x
and glued to the centre of the plate, the mass comes closer to
the Z-axis, hence moment of inertia decreases.
Z 27 (b) Moment of force = Force × r⊥ = 10 × 2 = 20 N-m
Y
Suppose a mass particle situated at point P (x, y ).
Hence, distance of mass particle from X-axis = x, distance of r⊥ 10 N
mass particle fromY-axis = y and distance of mass particle 1m

from Z-axis, a = x 2 + y 2 X
2m
Clearly, a > x
a>y 28 (a) ω = α − βt
Since, moment of inertia, I ∝ (distance) .
2
Comparing with ω = ω 0 − αt, we get
Hence, moment of inertia of the rod is maximum about Z-axis. Initial angular velocity = α
Angular retardation = β
17 (c) ICM is less than I about any other axis not passing through
∴ Angle rotated before it stops is
centre of mass only when two axes are parallel.
α2
18 (d) Angular momentum, L = 2KI (just like p = 2 Km ) 0 = α 2 − 2 βθ ⇒ θ =

⇒ L∝ K
29 (a) For uniform rod, the moment of inertia about an axis
If K is made 4 times, L will become 2 times.
passing through its centre is mL2 / 12 .
19 (a) Work done by friction in pure rolling is always zero. It is
because there is no displacement by frictional force. Friction I (mL 2 / 12) L
∴Radius of gyration, K = = =
works only in case of rolling with sliding. m m 2 3
418 OBJECTIVE Physics Vol. 1

30 (a) From the equation, 0 = ω 0 − αt From Eq. (i), I1ω 1 = I 2 ω 2


ω 0 (100 × 2π ) / 60 (where, ω1andω 2 are angular velocities before and after jumping)
∴ α= = = 0.7 rad s −2
t 15 I
⇒ Iω = × ω 2
ω 20 2
Now, angle rotated before coming to rest, θ =
2α (As mass reduced to half, hence moment of
inertia also reduced to half)
(100 × 2π / 60 )2
or θ= = 78.33 rad ⇒ ω 2 = 2ω
2 × 0.7
θ 37 (a) Velocity, v = r ω or v ∝ r
Now, number of rotations, n = = 12.47 −~ 12.5

31 (d) The angular momentum, L = mvr⊥ = (5) (3 2 ) (2 5 )
= 30 10 g -cm2s −1
Y
m v ω

rQ > rC > rP ⇒ v Q > v C > v P


r⊥ = y = 2√5 cm
38 (c) Angle rotated, θ = Area under ω-t graph.
X
Area under ω-t graph = Area of trapezium OABCO.
1 1
32 (c) W = Change in rotational kinetic energy = I (ω 2f − ωi2 ) Q Area of ( OABCO ) = (Sum of parallel sides) × Distance
2 2
11  between them
=  mR 2 [(2πff )2 − (2πfi )2]
2 2 
1  A B
=  mR 2 [4π 2 (ff2 − fi 2 )] 30
4 
1
= × 2 × (1)2 × 4 × π 2 (100 − 25) ω (rad s–1)
4
D E C
= 1.48 × 10 3 J ≈ 1.5 × 10 3 J O 5 20 25
t (s)
MR 2 MR 2 1  M  2 5
33 (b) ∴ I1 = and I2 = +   R = MR 2
2 2 2  4 8 1 1
= (AB + OC ) × AD = (15 + 25) × 30 = 600 rad
By conservation of angular momentum, 2 2
I 1 2
I1ω1 = I2ω 2 ⇒ ω 2 = 1 ω1 mr
I2 I 2 r
39 (c) Radius of gyration of disc, KD = D = =
(1/ 2) MR 2 4 m m 2
= 2
ω ⇒ ω2 = ω
(5/ 8)MR 5 IR m (2r )2
Radius of gyration of ring, KR = = = 2r
2 m m
mR 2 + mR 2 KR
I 3 5 ∴ =2 2
34 (b) For hollow sphere, KH = = = R
m m 3 KD
2
mR 2 + mR 2 40 (a) Moment of inertia, I = mK 2
I 5 7
For solid sphere, KS = = = R Q Ball is rolling without slipping, v = Rω
m m 5
1
KH 25 5 Translational kinetic energy, KT = mv 2
∴ = = 2
KS 21 21 1 mK 2v 2
Rotational kinetic energy, KR = Iω 2 =
35 (c) For square lamina, I2 = I3. These values will be less than I1 2 2R 2
because distribution of masses are nearer to the axes 1 2 mK 2v 2
corresponding to I2 and I3. ∴Total kinetic energy = mv +
2 2R 2
36 (a) As no external torque acts on the system, angular mK 2 v 2
momentum should be conserved. K2
Required fraction = 2R 2 = 2
Hence, I ω = constant ...(i) 1 2 mK v 2 2
R + K2
mv +
where, I is moment of inertia of the system and ω is angular 2 2R 2

velocity of the system.


Rotation 419

41 (b) We have, (πR 2td ) = m (Q Mass = Density × Volume) 47 (c) Path of A is cycloid and distance travelled in one rotation
m is 8R.
∴ R2=
πtd 48 (b) Torque, τ 0 = mg × r⊥
2
1 m Y
Now, I= mR 2 =
2 2πtd
1
or I∝
d X
O r⊥ mg
I d
∴The ratio of moments of inertia, 1 = 2
I2 d1 As, r⊥ is continuously increasing or torque is continuously
increasing on the particle. Hence, angular momentum is
42 (a) Original volume of the earth,
continuously increasing.
4
V = πR 3
3 49 (b) From the figure, r⊥ = 4 sin 45° = 2 2 units
Volume of earth after shrinking Y v = 3√2 units
V
V′ = 45°
4 r⊥ 4 sin45°
4 1 4 3 45° X
⇒ πR′ = × πR
3
O
3 8 3
R
R′ =
2
By conservation of angular momentum, The magnitude of the angular momentum,
I′ ω′ = Iω L = mvr⊥ = 5(3 2 ) (2 2 ) = 60 units
2 2π 2 2π
⇒ MR′ ×
2
= MR 2 × 50 (b) About bottom most point,
5 T′ 5 T
2
T R
⇒ = 
T ′  R′  v0
2
T  R 
⇒ = 
T ′  R /2
The angular momentum of sphere,
T 2 v  7
⇒ =4 L = MRv 0 + MR 2  0  = MRv 0
T′ 5 R 5
T 24
⇒ T′ = = [Q T = 24 h] I
4 4 51 (d) Radius of gyration, K =
m
= 6h
1
43 (d) At bottom most point, total kinetic will be mgh. mR 2 + mR 2
2 3
1 1 ∴ Kdisc = = R
Total kinetic energy = mv 2 + Iω 2 m 2
2 2
1 2
2 mR 2 + mR 2
1  v  7 and Kring = = 2R
= mv 2 +  mR 2   = mv 2
2 2 5   R 10 m

7 10 Kdisc 3/ 2 3
Now, mv 2 = mgh ⇒v = gh ∴ = =
10 7 Kring 2 2

44 (a) From theorem of perpendicular axes, we have 52 (b) Radius of gyration of a circular disc about a tangential axis,
 a 
2
 ma 2 ma 2  ma 2 2 I1 (5 / 4) m1R 2 5
I = IC + m   =  +  + = ma 2 K1 = = = R
 2  12 12  2 3 m1 m1 4
45 (d) Moment of inertia, I = ICM + mx 2 Radius of gyration of a circular ring about a tangential axis,

i.e. I-x graph is a parabola not passing through origin. I2 (3 / 2) m 2R 2 3


K2 = = = R
M (l / 2) 2 2 m2 m2 2
Ml
46 (b) Moment of inertia, I = 4  = 6
 2 3  K1 5
∴ =
K2 6
420 OBJECTIVE Physics Vol. 1

53 (b) The magnitude of angular momentum, L = mvr⊥ 60 (b) Force 8 N can also be resolved in tangential and radial
directions.
mvh
⇒ L= τ net = (9 + 8 sin 30 ° − 4) (0.2) N-m clockwise
2
= 1.80 N-m clockwise

v cos θ = v/√2 61 (a) From law of conservation of angular momentum, (about


centre of mass of the rod) Li = Lf
r⊥
ML2
h mvr = ⋅ω
12
12mvr
54 (b) The initial velocity is vi = v e$ y and after reflection from ⇒ ω=
ML2
the wall, the final velocity is v f = − v e$ y . The trajectory is (12) (1) (2) (1/4)
described as position vector r = y $ey + a$ez. = = 3 rad s −1
(2) (1)2
Hence, the change in angular momentum
= r × m (v f − v i ) 62 (b) Moment of inertia,
= ( y e$ y + a e$ z ) × (− mv e$ y − mv e$ y ) Ml 2 
I =3  = Ml
2
= ( ye$ y + ae$ z ) × (− 2mv e$ y ) [Q e$ y × e$ y = 0 and$ez × e$ y = − e$ x ]  3 
= 2mva $ex 63 (a) Angular momentum will not remain conserved due to the
torque produced by weight of particle acting about axis of
55 (c) Friction in this case is backward. If it is insufficient, it is
rotation. Hence, only mechanical energy is conserved.
the case of forward slipping or KT > KR .
64 (d) In case of pure rolling, ratio of rotational to translational
56 (b) Work done by retarding torque = Change in rotational
kinetic energy is 2/5. Therefore, total kinetic energy is 7/5
kinetic energy
times the translational kinetic energy. At maximum
Since, rotational kinetic energy of both ring and disc are compression, whole of energy is elastic potential. Hence,
equal, both will rotate with same number of revolutions.
71 2 1 2
 Mv  = kxmax
57 (c) For total torque about O to be zero, 5 2  2
A ∴The compression of the spring,
7M
r r xmax = v
5k
O
r F3 F2 65 (b) From theorem of parallel axes, moment of inertia about an
B
F1 C axis passing through a distance l /2 from centre in all
ml 2
F1 r + F2 r − F3 r = 0 directions will have moment of inertia . All such points at
3
⇒ F3 = F1 + F2
same distance from centre will lie in a circle.
58 (b) From the figure, AC = BD = l 2 + l 2 = 2 l 66 (d) Whole mass has equal distance from the centre O. Hence,
Moment of inertia of four point masses about BD, I0 = mR 2. Further, centre of mass of the remaining portion
 l 2
2
 l 2
2 will be to the left of point O. More the distance of axis from
IBD = m   + m × 0 + m  +m ×0 centre of mass, more is the moment of inertia. Hence, IA > IO.
 2   2 
67 (b) The moment of inertia,
IBD = ml 2 I = Iring + 3 Ispoke
Applying the theorem of parallel axes,  mR 2 
2 = MR 2 + 3   = (M + m )R 2
 l   3 
IXY = IBD + 4m (AO )2 = ml 2 + 4m   = 3 ml 2
 2
68 (b) Decrease in gravitational potential energy = Increase in
59 (c) Area under n-t graph will give total number of revolution rotational kinetic energy about an axis passing through end
before coming to stop. point.
Area = Total number of revolutions l 1  ml 2  2
∴ mg =  ω
1  1800  600 1  600  2 2 3 
= ×8×  + 8× + × 16 ×  
2  60  60 2  60 
3g g
= 120 + 80 + 80 = 280 ⇒ Αngular velocity, ω = or ω ∝
l l
Rotation 421

69 (c) By theorem of parallel axes, I1 = I2 + (4M ) r 2 74 (d) Net torque about O should be zero.
O
1 2
60° 30°
30° 60°
r

mg Mg

2
ML2   2L  8 ⇒ Mg
l l
sin 30 ° = mg sin 60 °
=4 sin2 45° + 4M   = 3 ML
2
2 2
 3   2 
M sin 60 °
= = 3
70 (a) Decrease in gravitational potential energy = Increase in m sin 30 °
rotational kinetic energy about point O.

(B) Medical entrance special format


questions
O O
ω l Assertion and reason
L 1  mL 2
or mg =   ω2 1 (c) L about centre = mvR = constant
2 2 3 
dL
As, τ = and derivative of any constant is zero.
3g dt
or ω=
L ∴ τ centre = 0
Now, v =r ω (In pure rotation) 2. (b) L = mvr sin θ or mvr⊥
3g In case of constant velocity, m, v and r⊥ all are constants.
∴ v =L (Q r = L) Therefore, angular momentum is constant.
L
h
= 3gL Further, L = n (in Bohr’s theory).

71 (b) Velocity, v = r ω Therefore, L and h have same units.
Decrease in gravitational potential energy = 2
3 (a) Moment of inertia of sphere in case 1 is MR 2.
Increase in kinetic energy ω 5
1 1 7
∴ mgh = Iω 2 + mv 2 Moment of inertia of sphere in case 2 is MR 2.
2 2 5
1 2 1 So, more work has to be done to rotate the sphere in case 2.
= Iω + m (r ω )2 h
2 2 4 (d) Sphere can roll without slipping on ω
2mgh surface, if v = r ω on an inclined plane.
∴ Angular velocity, ω = v
I + mr 2 It is friction which creates rotation of v
sphere. So, smooth surface cannot
72 (b) From the property of a circle, if an arc subtends an angle θ create rotation.
at any point A on circumference, then it will subtend an angle
5 (c) Angular momentum about bottom
2θ at centre C. So, in the same time interval, a particle 1
rotating in a circle turns double the angle with respect to most point, L = mv 0R − mR 2ω 0
centre point C compared to point A. 2
∴ ω C = 2ω A ω 0R
If L = 0 or v 0 = , disc will come to rest after some time.
ωA 1 2
or =
ωC 2 In case of pure rotation, since frictional force is acting,
therefore angular momentum is not conserved.
73 (c) If we complete the ring, its mass will become 2M.
1 l Statement based questions
∴ Iwhole ring = (2M ) (R 2 ) = MR 2
2 1 (d) As, angular acceleration,
(Iwhole ring = moment of inertia about any diameter) dω
1 α =
∴ Ihalf ring = MR 2 (About any radius) dt
2
where, ω is angular velocity of the disc.
This value is independent of angle θ.
422 OBJECTIVE Physics Vol. 1

Given, ω = constant  90 ° 1
(B) If θ = 90 °, then v P = 2v sin   = 2v × = 2v
dω 0  2  2
⇒ α = = = 0
dt dt  120 ° 3
(C) If θ = 120 °, then v P = 2v sin   = 2v × = 3v
Hence, angular acceleration is zero.  2  2
2 (a) Direction of linear velocity always keeps on changing in  180 °
(D) If θ = 180 °, then v P = 2v sin   = 2v
circular orbit. Hence, linear momentum is varying.  2 
3 (c) Here, µ < µmin but µ ≠ 0. Therefore, sphere will roll with Hence, A → q, B → p, C → s, D → r.
forward slipping or translation. 3 (d) Torque, τ = 0
g sin θ L2
4 (d) Acceleration, a = ∴ L = constant, K = and Iω = constant. The insect first
I
1+ 2I
mr 2 moves away from the axis, then towards it. Hence, I will first
I 2 increase and then decrease.
For a sphere, 2
=
mr 5 Hence, A → q, B → r, C → p, D → r.
5  ml 2  2
Hence, a = g sin θ = constant. l 2
7 4 (d) (A) I1 = 2   + 2(m )   = ml 2
 12   2 3
Hence, speeds of all spheres will be same at the bottom.
Sphere (ii) has the largest mass. Hence, it will have the  ml 2   5
(B) I2 = 0 + 2   + ml 2 =   ml 2
maximum kinetic energy.  3   3
5 (b) Since, the sphere is sliding, maximum force of friction will ml 2  2
act on passing through the bottom most point. Torque of this (C) I3 = 4  sin2 45° = ml 2 = I1
force about point of contact is zero. Hence, angular  3  3
momentum will remain conserved about the point of contact Hence, A → q, B → s, C → q.
but not about the centre of mass because torque of friction
will not be zero about centre of mass.
6 (a) Statement I is correct while Statement II is incorrect.
(C) Medical entrances’ gallery
The corrected statement is; 1 (b) Given, position vector, r = 2 k$ m
Moment of inertia changes with axis of rotation. Force, F = 3$j N
7 (a) Statement I is correct while Statement II is incorrect. As, torque, τ = r × F = 2k$ × 3$j = 6(− $i ) = − 6$i N-m
The corrected statement is;
2 (a) Initial angular speed of wheel,
Every rolling motion cannot be considered as pure rotation. 360
For pure rotation, the condition is ω 0 = 2πf0 = 2π × rad/s = 12π rad/s
60
linear velocity, v = ωr
Final angular speed of wheel,
Match the columns 1200
ω = 2πf = 2π ×
l
rad/s
2 60
1 (c) L1 = Iω = MR 2ω
5 = 40π rad/s
2
2  R t = 14 s
L2 = M   ω′
5  2 From the equation of rotational motion, ω = ω 0 + αt
As, L1 = L2 ⇒ ω′ = 4ω ω − ω 0 40 π − 12π 28π
⇒ α= = = = 2π rad/s 2
2π  2π  T t 14 14
∴ =4  ⇒ T′ =
T′ T 4 3 (d) Given, mass of cylinder, m = 2 kg
2
L
Further, K= Radius of cylinder, r = 4 cm = 4 × 10 −2 m
2I
2π π
Since, angular momentum is constant and I has become (1/4)th. Rotational velocity, ω = 3 rpm = 3 × = rad s −1 and
60 10
Therefore, kinetic energy will become 4 times.
θ = 2π × revolution = 2π × 2π = 4π 2 rad
Hence, A → q, B → s, C → q.
 θ The work done in rotating an object by an angle θ from rest is
2 (b) In general, v P = 2v sin   given by W = τθ
 2
As the cylinder is brought to rest, so the work done will be
 60 ° 1
(A) If θ = 60 °, then v P = 2v sin   = 2v × = v negative.
 2  2
Rotation 423

According to work-energy theorem, where, m = mass of body, v = velocity and K = radius of


Work done = Change in rotational kinetic energy gyration.
1 1 1
− τ θ = Iω 2f − Iωi2 = I (ω 2f − ωi2 )
2 2 2
I (ωi2 )
⇒ τ= (Q ω f = 0 ) x h

–1
s
1  1 2 ωi2  1  4m
=  mr  Q I = mr 2 (for cylinder) v= 30°
2 2  θ  2 

1 2 ω2 Given, m = 2 kg, θ = 30 ° and v = 4 ms −1


= mr (Q ωi = ω)
4 θ Let h be the height of the inclined plane, then from law of
2
1 π 1 conservation of energy,
= × 2 × (4 × 10 −2 )2 ×   ×
4  10  4π 2 KE = PE
1 π2 1 1 2 K 2
= × 2 × 16 × 10 −4 × × mv 1 + 2  = mgh
4 100 4π 2 2  R 
2
= × 10 −4 = 2 × 10 −6 N-m Substituting the given values in the above equation, we get
100  K 2 1
1  1
× 2 × 161 +  = 2 × 10 × h For cylinder, 2 = 
4 (d) Given, radius, R = 2 m, mass, m = 100 kg 2  2  R 2
and v CM = v = 20 cm s −1 = 20 × 10 −2 m s −1 3
⇒ 8× = 10 h ⇒ h = 1.2 m
Then, according to work-energy theorem, the work done in 2
stopping the disc is equal to the change in its kinetic energy, h
From the above diagram, sinθ =
i.e. x
W = (KE )f − (KE )i ...(i) h 1.2
⇒ x= =
As, the disc stops at the end, so final velocity is zero. sin θ sin 30 °
Thus, (KE )f = 0 = 1.2 × 2 = 2.4 m
Since, the disc is rolling, so its initial kinetic energy would 6 (c) Given, initial velocity of sphere, u = 2 .8 ms −1
have both rotational and translational kinetic energy
components. Acceleration on the inclined plane,
1 1 g sinθ
(KE )i = (KE )r + (KE )t = Iω 2 + mv 2 a= …(i)
2 2 K2
1+ 2
1 1  1  1  R
=  mR 2 ω 2 + mv 2 Q For disc, I = mR 2

2 2  2  2  where, K and R are the radius of gyration and radius of
1 2 2 1 2 sphere, respectively.
= mR ω + mv 2
4 2 Moment of inertia of sphere, I = mK 2 = mR 2
5
1 2 v2 1
= mR 2 + mv 2 (Qv = rω ) K2 2
4 R 2 For sphere, =
3 R2 5
= mv 2 Putting this value in Eq. (i), we get
4
3 g sin 30 °
∴ W = |KEi | = mv 2 [From Eq. (i)] a= [Q θ = 30 ° (given)]
2
4 1+
5
Substituting the given values in above equation, we get
5g 25 −2
3 = = ms [Q g = 10 ms −2]
W = × 100 × (20 × 10 −2 )2 14 7
4
3 If s be the maximum distance travelled by the sphere, then at
= × 400 × 100 × 10 −4 = 3 J maximum distance, v = 0.
4
∴ From third equation of motion, v 2 = u 2 − 2 as
5 (d) When a body rolls, i.e. have rotational motion, the total
kinetic energy of the system will be 0 = u 2 − 2 as
1 2 K 2 u 2 (2 . 8)2 × 7
KE = mv 1 + 2  ⇒ s= = = 1.09 m −
~ 1.38 m
2  R  2a 2 × 25
424 OBJECTIVE Physics Vol. 1

7 (a) Given, mass ratio of two discs, For a solid sphere, moment of inertia about its diametric axis,
m 1 2
m1 : m 2 = 1: 2, i.e. 1 = I = mR 2
m2 2 5
d1 2 r 2 Substituting the value of I in Eq. (ii), we get
and diameter ratio, = ⇒ 1= 12  1 2
d 2 1 r2 1 Kr + Kt =  mR 2 ω 2 + mv CM
25  2
∴Ratio of their moments of inertia,
2
r2 12 2  v  1 2
m11 =  mR   CM  + mv CM [Qv CM = Rω]
25   R 
2
m r 
2
I1 1  2 2 2
= 2 2 = 1 ⋅  1 =   =
I2 m 2r2 m 2  r2  2  1 1 1 2 1 2
= mv CM + mv CM
2 5 2
∴ I1 : I2 = 2 : 1  1 1 7
=  +  mv CM 2
= 2
mv CM …(iii)
 5 2 10
8 (a) Given, initial angular velocity of object, ω 0 = 0 1 2
mv CM
Angular displacement, θ = 60 rad ∴ Ratio,
Kt
= 2 = ×
1 10 5
=
and ∆t = 10 s Kt + Kr 7 2
mv CM 2 7 7
1 2 10
From equation of rotational motion, θ = ω 0t + αt
2 ∴ Kt : ( Kt + Kr ) = 5 : 7
1 12 (c) Moment of inertia of the insect-disc system,
60 = 0 × t + × α × 10 2
2 1
MI = MR 2 + mx 2
60 2
⇒ α= ⇒ α = 1.2 rad s −2 where, m = mass of insect
50
and x = distance of insect from centre.
9 (d) Moment of force is defined as the cross product of the Clearly, as the insect moves along the diameter of the disc.
force and the force arm. Moment of inertia first decreases and then increases.
Given, F = 4i$ + 5$j − 6$j , r1 = 2i$ − 2$j − 2k$ By conservation of angular momentum, angular speed first
and r = 2$i + 0 $j − 3k$ increases and then decreases.
2

Moment of force = r × F = (r2 − r1) × F 13 (a) For disc, moment of inertia about the diameter, Id = mr 2 / 4
= [( 2i$ + 0 $j − 3k$ ) − (2i$ − 2$j − 2k$ )] × (4$i + 5$j − 6k$ ) and moment of inertia about the tangential axis,
$i $j k$ Idisc = Id + mr 2
mr 2 5mr 2
= (0 i + 2 j − 1k ) × (4i + 5 j − 6k ) = 0 2 −1
$ $ $ $ $ $ Idisc = + mr 2 =
4 4
4 5 −6
Let the radius of gyration of disc be Kdisc .
= $i [(−6 × 2) − (−1× 5)] − $j [(−6 × 0 ) − (−1× 4)] + k$ [(0 × 5) − 2 × 4] Idisc = mKdisc
2

= −7$i − 4$j − 8k$ 5mr 2 5


⇒ = mKdisc
2
⇒ Kdisc
2
= r
dL 4 4
10 (d) As we know that, external torque, τ ext =
dt For ring, moment of inertia about the diameter,
where, L is the angular momentum. mr 2
Ia =
dL 2
Since, in the given condition, τ ext = 0 ⇒ =0 and moment of inertia about the tangential axis,
dt
⇒ L = constant mr 2 3
Iring = Ia + mr 2 = + mr 2 = mr 2
Hence, when the radius of the sphere is increased keeping its 2 2
mass same, only the angular momentum remains constant. But Let the radius of gyration of ring be Kring .
other quantities like moment of inertia, rotational kinetic Iring = mKring
2

energy and angular velocity changes.


3 2 3
11 (b) Translational kinetic energy of a rolling body, ⇒ mr = mKring
2
⇒ Kring = r
1 2 2 2
Kt = mv CM …(i)
2 5
r
Total kinetic energy of a rolling body Kdisc 4 = 5
Q =
= Rotational KE + Translational KE = Kr + Kt Kring 3
r
6
1 1 2 2
= Iω 2 + mv CM …(ii)
2 2
Rotation 425

14 (b) Usually, the centre of gravity coincides with the centre of For different solid bodies, total kinetic energy is not always
mass for bodies, when the gravitational field is uniform for twice of translation kinetic energy. Hence, Assertion is
bodies of small height. correct but Reason is incorrect.
Since, the gravitational force can be taken to be centred or 18 (c) Total kinetic energy (KE) of the system
confined at the centre of gravity of body, so the torque of
gravitational force about the centre of gravity must be zero. = KE (translational) + KE (rotational)
2
A couple always produces the rotation without translation. 1 1 1 12 v
= (2m )v 2 + I ω 2 = (2m )v 2 +  mR 2 2
Mechanical advantage is the ratio of the force produced by a 2 2 2 23 R
machine to the force applied on it. It is used in assessing the 1 1 4
performance of a machine. So, option (b) is correct. = (2m )v 2 + mv 2 = mv 2
2 3 3
15 (b) When no external torque acts on system, then angular
19 (b) The moment of inertia of a body about an axis depends not
momentum of system remains constant.
only on the mass of the body but also on the distribution of
Angular momentum before contact mass about the axis. For a given body, mass is same, so it will
= I1ω1 + I2ω 2 = Iω1 + Iω 2 = I (ω1 + ω 2 ) depend only on the distribution of mass about the axis. The
Angular momentum after the discs brought into contact mass is farthest from axis BC, so I2 is maximum, mass is
nearest to axis AC, so I3 is minimum.
= Inet ω = (I1 + I2 ) ω = 2Iω
Hence, the correct sequence will be I2 > I1 > I3.
From law of conservation of angular momentum,
ω + ω2 20 (a) When the rod rotates through an angle θ, the centre of
I (ω1 + ω 2 ) = 2Iω ⇒ ω = 1 gravity falls through a distance h.
2
Now, to calculate loss of energy, we subtract initial and final A ω
energies of system. A′
l/2
1 1 1 1
Loss of energy = Iω12 + Iω 22 − (2I ) ω 2 = I (ω1 − ω 2 )2
2 2 2 4 G
h
16 (c) Torque (τ ) acting on a body and angular acceleration (α ) C G′
θ
produced in it are related as
τ = Iα
B
Consider a hollow cylinder around which a rope is wounded.
(l / 2) − h l
Torque acting on the cylinder due to the force F is τ = Fr. In ∆BCG′, cos θ = or h = (1 − cos θ )
l/2 2
Now, we have τ = Iα l
where, I = moment of inertia of the cylinder about the axis Decrease in potential energy = mg (1 − cos θ ) …(i)
2
through the centre. The decrease in potential energy is equal to the kinetic
⇒ I = mr 2 energy of rotation.
Given, F = 30 N, m = 3 kg and r = 40 cm = 40 × 10 −2 m 1 2 1  ml 2  2
∴ (KE)rotational = Iω =  ω …(ii)
Angular acceleration, 2 2 3 
τ Fr  ml 2 
α= = 2 Q I = 
I mr  3 
F 30 From Eqs. (i) and (ii), we get
= =
mr 3 × 40 × 10 −2 1  ml 2  2 l 6g θ
  ω = mg (1− cos θ ) ⇒ ω = sin
100 2 3  2 l 2
= = 25 rad s −2
4
21 (a) Velocity of the particle at Q, v Q = r ω = Rω
17 (c) The kinetic energy of a rolling solid sphere
1 1 Velocity of the particle at P, v P = r ω = (2R ) ω = 2v Q
= Ktrans + Krot = mv 2 + Iω 2 P
2 2 2vQ
1 1 2
= mv 2 + × mR 2ω 2 r
2 2 5 v ⇒
1 1  v ω
= mv 2 + mv 2 Q ω =  Q
2 5  R vQ = Rω Q ω
= 7 / 10 mv 2
Hence, total kinetic energy is sum of translation and Hence, points near the top move faster than points near the
rotational kinetic energies. bottom.
426 OBJECTIVE Physics Vol. 1

22 (b) The direction of the frictional I 2/ 5 mr 2 2


N For solid sphere, = =
force will be up the inclined plane α f mr 2
mr 2 5
in both the motions. The friction g sin θ 2
acts opposite to the relative motion ∴ a disc = = g sin θ = 0.66 g sin θ
θ 1 3
of the point of contact on ground. mg sin θ 1+
In both cases, the force acting on mg mg cos θ 2
θ g sin θ 5
the point of cylinder in contact a solid sphere = = g sin θ = 0.71 g sin θ
with the plane is mg sin θ. As there 2 7
1+
is pure rolling, the velocity and acceleration of the point of 5
contact must be zero, so friction acts upwards along the Clearly, a solid sphere > a disc
inclined plane in both the motions (ascending or descending).
Hence, sphere gets to the bottom of the plane first.
τ
23 (a) Torque, τ = Iα or moment of inertia of wheel, I = 26 (d) According to given question, a uniform
α
5 × 10 × 10 −2 circular disc of radius 50 cm at rest is free 0.5m
Moment of inertia of wheel, I = kg-m2 to turn about an axis having perpendicular
2 to its plane and passes through its centre.
= 0.25 kg-m2 This situation can be shown by the given
24 (a) Considering the information given in the question, let us figure.
draw the figure. ∴ Angular acceleration, α = 2 rad s −2 (Given)
I −1
Angular speed, ω = αt = 4 rad s
Q Centripetal acceleration, a c = ω 2r = (4)2 × 0.5 = 16 × 0.5
R R/2
a c = 8 ms −2
Q Linear acceleration at the end of 2 s,
a t = αr = 2 × 0.5 ⇒ a t = 1ms −2
If the above figure is considered, then moment of inertia of
disc will be given as Therefore, the net acceleration at the end of 2 s is given by
I = Iremaining + I(R / 2) a = a c2 + a 2t
⇒ Iremaining = I − I(R / 2)
a = (8)2 + (1)2 = 65 ⇒ a ≈ 8 ms −2
Putting the values, we get
M 2  27 (c) According to law of conservation of angular momentum,
 R
   2 R 
MR 2
4  2 M  R  Li = Lf ⇒ mv 0R0 = mv ′  0  ⇒ v ′ = 2v 0
= − +    2
2  2 4  2 
  So, final kinetic energy of the particle,
 
1 1 1
MR 2 MR 2 MR 2  MR 2 MR 2 + 2MR 2  Kf = mv ′ 2 = m (2v 0 )2 = 4 × mv 02 = 2mv 02
= − + = − 2 2 2
2  32 16  2  32 
 28 (d) The total moment of inertia of the system, X
MR 2 3MR 2 16MR 2 − 3MR 2 I = I1 + I2 + I3 …(i)
= − =
2 32 32 2 1
Here, I1 = mr 2 m r
13MR 2 3
Iremaining =
32 2
I2 = I3 = mr 2 + mr 2 m r r m
3
25 (a) Acceleration of an object rolling down an inclined plane is
given by (From parallel axes theorem) 2 3
g sin θ
a= 5 2
= mr
1 + I /mr 2 3
X′

where, θ = angle of inclination of the inclined plane, From Eq. (i), we get
m = mass of the object 2 5  2 10 
I = mr 2 + 2 × mr 2 = mr 2  +  ⇒ I = 4 mr 2
and I = moment of inertia about the axis through centre of 3 3 3 3
mass. 29 (b) When a metallic rod is heated, it expands. Its moment of
Type of sphere is not mentioned in the question. Therefore, inertia (I) about a perpendicular bisector increases. According
we will assume the given sphere as solid sphere. to law of conservation of angular momentum, its angular
1
I 1/ 2 mr 2 1 speed (ω) decreases, since ω ∝ .
For disc, 2
= = I
mr mr 2 2
Rotation 427

30 (b) The radius of the circle followed by the masses, For disc B , IB = m (2r )2 = 4mr 2 …(ii)
r = l sin α
As, angular momentum, L = r × p = r × mv On dividing Eq. (i) by Eq. (ii), we get
⇒ | L | = l sin θ (m ω l sin θ ) IA mr 2 I 1 1
∴ = ⇒ A = ⇒ IA = IB
On differentiating both sides, we get IB 4mr 2 IB 4 4
d |L | dθ
= mωl 2 2 sin θ ⋅ cos θ So, moment of inertia of A is one-fourth that of B.
dt dt
35 (c) Radius of gyration is a scalar quantity.
dL
⇒ = 2ml ω sin θ ⋅ cos θ = ml 2ω 2 sin 2 θ
2 2
Hence, statement of option (c) is incorrect.
dt
36 (a) Total angular momentum about O is given as
31 (a) Let the velocity of centre of sphere be v.
L = L1 + L2 = m1v1r1 + m 2v 2r2
The angular speed of the sphere about its centre is given by
= − 6.5 × 2.2 × 1.5 + 3.1 × 3.6 × 2.8
v
ω=
r = − 21.45 + 31.248 = 9.8 kg-m 2 s −1
where, r = radius of sphere.
37 (b) When string is cut, the weight of the rod constitutes a
The total kinetic energy = Translational kinetic energy l
+ Rotational kinetic energy torque about the hinge point A . So, τ A = mg …(i)
2
1 2 1 1 2 1 Also, from Newton’s law, τ A = I α …(ii)
= Iω + Mv 2 = ⋅ Mr 2ω 2 + Mv 2
2 2 2 5 2 where, α = angular acceleration of the rod
1 1 7 7 and I = moment of inertia of rod.
= Mv 2 + Mv 2 = Mv 2 = × 0.5 × (0.02)2 = 1.4 × 10 − 4 J From Eqs. (i) and (ii), we get
5 2 10 10
l l
32 (b) Given, radius of the circle = 20 cm = 20 × 10 −2 m Iα = mg ⇒ α = mg / I
2 2
When speeds are 8 ms −1 and 9 ms −1, then angular speeds are
l2
respectively, ω1 =
8
= 40 rad s −1 As, I =m
20 × 10 −2 3
mg (l / 2) 3 g
9 So, α= =
and ω2 = = 45 rad s −1 ml 2 2l
20 × 10 −2
3
ω 2 − ω1 45 − 40
∴ Angular acceleration = = = 2.5 rad s −2 Now, acceleration of centre of mass of rod, a CM = α ⋅ r
t 2
Here, r = distance between hinge point A and centre of the rod.
1
33 (b) The rotational kinetic energy, Kr = Iω 2 3 g l 3g
2 ⇒ a CM = =
2 l 2 4
where, I = moment of inertia of solid sphere.
⇒ mg − RA = ma CM
The total kinetic energy is the sum of rotational kinetic
energy and translational kinetic energy. where, RA = reaction at A.
3g
1
KT = mv 2 + Iω 2
1 ⇒ mg − RA = m ×
2 2 4
mg 5 × 10
1
= m ω2r 2 + Iω 2
1
(For pure rolling, v = ωr ) ⇒ RA = = = 12.5 N
2 2 4 4
1 2 38 (b) Consider the diagram.

Kr 2 I
Now, = = 2 Particle
KT 1 2
ω (mr + I )
2 mr +I v = constant
r
2 r sin θ r sin θ = perpendicular
2
As, I = mr 2 (For solid sphere) X-axis
5 O
2 2 2 2 As, the particle goes with constant linear velocity, the
mr mr
Kr 5 2 5 2 perpendicular distance of velocity vector from the axis is
⇒ = = 5 = × =
KT mr 2 + 2 mr 2 7 mr 2 5 7 7 constant. So, angular momentum mvr sin θ is constant.
1
5 5 39 (d) Moment of inertia of a ring about its diameter, I = mr 2
2
34 (a) Given, radius of circular disc A , r1 = r
and kinetic energy is given by
Radius of circular disc B, r2 = 2r 1 1 1
K = Iω 2 ⇒ K = mr 2 ω 2 = × 10 × (0.5)2 × (20 )2 = 250 J
For disc A , IA = mr 2 …(i) 2 4 4
428 OBJECTIVE Physics Vol. 1

40 (b) For a ring, Iz = MR 2 Torque on the rod = Moment of weight of the rod about P,
From perpendicular axes theorem, Ix + Iy = Iz ...(i) L
τ = Mg …(i)
Given, Ix = Iy = I 2
From Eq. (i), we get ML2
Q Moment of inertia of rod about P, I = …(ii)
2I = Iz 3
41 (c) Consider two bodies A and B having moment of inertia I As, τ = Iα
and 2I. From Eqs. (i) and (ii), we get
Kinetic energy of rotation for the body A, KA = (1/ 2) Iω 2A L ML2
Mg = α
where, ω A is angular speed of the body A . 2 3
1 3g
Kinetic energy of rotation for the body B, KB = (2 I ) ω B2 ∴ The initial angular acceleration of the rod, α =
2 2L
According to the question, 2gh
45 (d) Velocity, v =
KA = KB 1 + K 2 /R 2
1 2 1
⇒ Iω A = (2 I ) ω B2 3v 2
2 2 Given, h=
2 4g
ω 
⇒ ω 2A = 2 ω 2B ⇒  A  = 2 2gh 2g 3v 2
 ωB  ⇒ v2 = =
K 2
 K 2
ωA 1 + 2 4 g 1 + 2 
⇒ = 2 R  R 
ωB
3 K2 3
⇒ ωA : ωB = 2 : 1 1= or 1 + =
 K 2
R2 2
1 2 1 2 2 1 + 2 
42 (a) We know that, the kinetic energy, K = mv = Iω  R 
2 2
where, m = 27 kg (mass of the body), K2 3 1
or = − 1=
ω = 3 rads −1 (angular velocity) R2 2 2
and I = 3 kg-m2 (moment of inertia). ⇒
1
K2 = R2 (Equation of disc)
2
v = ? or mv 2 = Iω 2
Hence, the object is disc.
Iω 2 3 × 32 27
⇒ v2 = ⇒ v2 = ⇒ v2 = =1 46 (b) From law of conservation of angular momentum, if no
m 27 27
–1 external torque acts on a system, then total vector sum of
⇒ Velocity of body, v = 1 = 1ms angular momentum of different particles of the system
43 (c) Total kinetic energy of a body which is rolling without remains conserved.
slipping is given by dL
i.e. if τ ext = 0, then =0
Ktotal = Krot + Ktrans dt
2 ⇒ L = constant.
For solid spherical ball, I = mR 2 and v = Rω
5 47 (b) Angular momentum, L = Iω
(Along the diameter)
2
where, R is radius of spherical ball. 48 (d) Moment of inertia about its diameter, I = MR 2
5
12  1 7 7
So, Ktotal =  mR 2 ω 2 + mR 2ω 2 = mR 2ω 2 = mv 2 49 (a) In given condition, the moment of inertia of the combine
25  2 10 10 system about an axis passing through O is
Now, potential energy = kinetic energy 4 2  3a 
2
I = ma + m  
⇒ mgh =
7
mv 2 ⇒ v 2 =
10
gh …(i) 3  4
10 7 91
For vertical projection, Q I= ma 2
48
10 5
v 2 = u 2 + 2gh′ ⇒ gh = 0 + 2gh′ ⇒ h′ = h 50 (a) We know that, L = Iω = 2 π mr 2f
7 7
Now, ω′ = ω / 2
44 (a) When the string is cut, the rod can be shown as
ω
L/2 Hence, L′ = Iω′ = mr 2 = πmr 2 f
P 2
L 2 m πr 2f L
Mg ∴ = ⇒ L′ =
L′ π mr 2f 2
Rotation 429

51 (c) We know that, torque , τ = r × F 1 1  M


58 (d) Moment of inertia of disc, I = MR 2 = M  
2 2  πρt 
Given, F = 2$i − 3 $j + 5k$
 M M M
⇒ r = 3$i − 3$j + 5 k$ Q ρ = = , ∴ R2 = 
 V πR 2t πρt 
$i $j k$
1 M2
Now, τ= 3 −3 5 I=
2 πρt
2 −3 5 1
∴ I∝ (As, M and t are constants)
τ = (−15 + 15) $i − (15 − 10 )$j (−9 + 6)k$ ρ
I1 ρ 2
τ = 0 − 5 $j − 3 k$ Q =
I2 ρ1
⇒ τ = − 5 $j − 3 k$ 240
59 (c) Here, ω 2 = 2πn 2 = 2 × and ω1 = 2πn1 = 0
52 (b) Moment of inertia of a disc about a parallel axes, 60
ma 2 ω − ω1
I = Iv + md 2 = + md 2 (Q a = R and d = R) The angular acceleration, α = 2
2 t
⇒ Moment of inertia,  240 
2π  − 0
 60 
3 3
I = ma 2 = (25) ∴ α= = 2 .51rad s −1
2 2 10
75 60 (a) The given situation as shown in the figure.
∴ I = = 37.5 kg-m2
2 −L/2 L/2
53 (c) Total energy = kinetic energy + Rotational kinetic energy mw mb
1 1 4
= (2 m ) v 2 + mv 2 = mv 2 Moment of inertia for the wooden part,
2 3 3
(L / 2)2 L2
54 (d) According to theorem of parallel axes, M1 = m w × = mw
2
3 12
 R
I = ICM + M   Moment of inertia for the brass part,
 2
(L / 2)2 L2
1 MR 2 M2 = m b × = mb ×
⇒ I = MR 2 + 3 12
2 4 Total moment of inertia,
3
Moment of inertia, I = MR 2 L2 L2 L2
4 M = M1 + M2 = m w + mb × = (m w + m b )
12 12 12
55 (a) We have, ω 2 = ω1 + αt
61 (d) According to question,
⇒ ω1 = ω 2 − αt torque, τ = Iα = 0
Here, ω1 = 0 d 2θ
⇒ α = 0 or α= =0
and ω 2 = 60 rpm dt 2
60 × 2π θ(t ) = 2t 3 − 6t 2
= rad s −1 Given,
60 dθ
60 So, = 6t 2 − 12t
⇒ 0 = 2π × − α ⋅ 60 dt
60
d 2θ  d 2θ 
2π ⇒ = 12t − 12 Q α = 2 = 0
or α= dt  dt 
60
2π π 12t − 12 = 0 ⇒ t = 1s
∴ Torque, τ = Iα = 2 × = N-m
60 15 62 (a) According to parallel axes theorem, we have
2 24  8 IO = Ixy
56 (c) I = MR 2 =  πR 3ρ R 2 = πR 5ρ Ix' y'
5 53  15
176 5  22
I= R ρ Q π = 
105  7 L
57 (d) The radius of gyration does not depend on the total mass of a
body but depends upon the shape and size of body, distribution
of mass within the body and choice of rotation of axis. Ix ′ y ′ = Ixy + Md 2
430 OBJECTIVE Physics Vol. 1

2 Further, the force of friction calculated in Eq. (iv) for pure


 L
= I0 + M   rolling to take place should be less than or equal to the
 2
maximum friction,
ML2 Mg sin θ 
⇒ Ix ' y ' = I0 + 2 2
i.e. ≤ µ Mg cos θ Q I = MR 
4 MR 2  5 
1+
63 (b) This is the case of rolling on rough inclined plane. Force of I
friction in this case will be backward. tan θ tan θ 2
Mg sinθ − f ⇒ µmin ≤ = = tan θ
Hence, a= …(i) MR 2 +
5 7
M 1+ 1
I 2
fR
and α= …(ii) 64 (b) As, torque, τ = F ⋅ r sin θ
I
M, R, I = 2 × 3 × sin 30 ° (Given)
a 1
⇒ τ = 6 × = 3 N-m
2
θ 65 (c) Let same mass and same outer radii of solid sphere and
s in
Mg hollow sphere be M and R, respectively. The moment of
inertia of solid sphere A about its diameter,
2
I A = MR 2 …(i)
5
For pure rolling to take place, a = Rα … (iii)
The moment of inertia of hollow sphere (spherical shell) B
Solving Eqs. (i), (ii) and (iii), we get 2
Mg sinθ about its diameter, I B = MR 2 …(ii)
f = …(iv) 3
MR 2 From Eqs. (i) and (ii), we get
1+
I IA < IB
CHAPTER
10

Gravitation
Each body in this universe attracts other bodies towards itself with a force known
as gravitational force, thus gravitation is the phenomena of mutual attraction
between any two bodies in the universe. It is the force of gravitation due to
which earth attracts everything towards itself.
In this chapter, we will study the basic laws which govern gravitational
interactions.

Planets
The celestial objects which moves in fixed semi-circular orbits around the sun are
known as planets. Including the earth mainly we have 8 planets in our solar
system. These planets move around the sun in their fixed semi-circular orbits
with increasing distance from the sun, name of these planets are
(i) Mercury (ii) Venus (iii) Earth
(iv) Mars (v) Jupiter (vi) Saturn
Inside
(vii) Uranus (viii) Neptune 1 Kepler’s laws of planetary
motion
Satellites 2 Newton’s law of gravitation
Vector form of Newton’s law of
Celestial objects which are moving around the planets are known as satellites. gravitation
There are two types of satellites namely; natural and artificial satellites. Principle of superposition of
gravitational forces
For example, the moon is a natural satellite while INSAT-1B is an artificial
3 Gravity
satellite of the earth.
Acceleration due to gravity
on earth
Variation in the value of
acceleration due to gravity
Earth 4 Gravitational field
Gravitational field intensity for
Moon different bodies
5 Gravitational potential
Fig. 10.1 Earth and moon Gravitational potential for
different bodies
Relation between gravitational
KEPLER’S LAWS OF PLANETARY MOTION field and potential
6 Gravitational potential energy
Kepler’s laws of planetary motion consist of three scientific laws describing 7 Binding energy
motion of planets around the sun, which are as given below Escape velocity
Escape energy
Kepler’s first law of orbits 8 Motion of satellites
According to this law, “all planets move in elliptical orbits around the sun, 9 Artificial satellite
situated at one of the foci of the ellipse.”
432 OBJECTIVE Physics Vol. 1

This law identifies that the distance between the sun (S ) In simple terms, this law states that the planet will
and planet (P ) is constantly changing as the earth revolve move slowly (v min ) only when it is farthest from the sun
around its orbit as shown in figure. and move rapidly (v max ) when it is nearest to the sun.
B
This law is similar to law of conservation of angular
P′ momentum.
Example 10.1 A planet moving around sun sweeps area A1 in
Perihelion Aphelion
2b 2 days, A2 in 3 days and A3 in 6 days, then find the relation
or perigee P A or apogee
S
S′ between A1, A2 and A3 .
A3
C
2a
Sun A2
Fig. 10.2 Law of orbit

Above figure shows an ellipse traced out by a planet around


the sun S. The closest point P is called perihelion (perigee) A1
and the farthest point A is called aphelion (apogee).
Sol. According to Kepler’s second law, when a planet revolves
It is the characteristics of an ellipse that the sum of the around the sun, its areal velocity is constant.
distances of any point (P ′) on it from its two foci (S and S′) dA
is constant, i.e. SP ′+S ′ P ′ = constant. ∴ = constant
dt
Note Shorter the orbit of the planet around the sun, shorter is the time
A1 A2 A3
to cover one complete revolution. Hence, = =
t1 t2 t3
Kepler’s second law of areas Given, t1 = 2 days, t 2 = 3 days and t 3 = 6 days
According to this law, “the line joining the planet sweeps A1 A2 A3
∴ = = ⇒ 3A1 = 2A2 = A3
out equal areas in equal interval of time.” It means that 2 3 6
areal velocity is constant. This is the relation between A1, A2 and A3.
Example 10.2 The figure shows an elliptical orbit of a planet
Sun P about the sun S. The shaded area CSD is twice the
C B
vmax dA vmin shaded area ASB. If t1 is the time taken by the planet to
D r A move from C to D and t 2 is the time to move from A to B,
S determine the ratio t1 /t 2 .
v
P
Fig. 10.3 Law of area C
B
In the given figure, if the line joining the sun and the A S
planet swept out area dA in a small time interval dt, then D
dA d  1  1
Areal velocity = =  r (vdt )  = rv
dt dt  2  2 Sol. According to Kepler’s second law, the areal velocity of the
planet remains constant.
mvr L
= = A1 A2
2m 2m ∴ =
t1 t2
where, L = angular momentum of the earth-planet system
t1 A1 Area CSD
= constant (for gravitational force between the Hence, = = =2
t 2 A2 Area ASB
sun and the planet)

∴ Areal velocity =
dA
= constant
Kepler’s third law of period
dt According to this law, “the square of the time period (T ) of
revolution of a planet around the sun is proportional to the
Also, if the planet moves from A to B in some time interval cube of the semi-major axis (a ) of its elliptical orbit.”
t and from C to D in the same time interval t, then
area (ASB ) = area (CSD ) i.e. T 2 ∝ a3 ...(i)
Gravitation 433

3
As shown in figure, we have  3.5 R 
⇒ T22 = T12  
C  7R 

Apogee (Q R 2 = R + 2.5R and R1 = R + 6R )


Perigee F r2
Sun T12 T1
A B b or T22 = ⇒ T2 =
8 2 2
r1
a 24
⇒ T2 = (Q T1 = 24 h)
D 2 2
Fig. 10.4 Law of period
∴ T2 = 6 2 h
AB = AF + FB ⇒ 2a = r1 + r 2
r + r2 NEWTON'S LAW OF
∴ a= 1
2
So, substituting the value of a in Eq. (i), we get
GRAVITATION
3
According to this law, ‘‘every particle in the universe
 r + r2  attracts every other particle with a force whose magnitude
T 2
∝ 1 
 2  is directly proportional to the product of their masses and
inversely proportional to the square of distances between
where, r1 = shortest distance of planet from sun (perigee) their centres.’’ This tendency of particles (or bodies) to
and r 2 = longest distance of planet from sun (apogee). attract each other is called gravitation.
It can be shown that, if e is the eccentricity of an elliptical
F F
r 1+ e m2
orbit of the planet, then 2 = . m1
r1 1 − e r
Note Kepler’s laws are applicable not only to the solar system but Fig. 10.5 Force between two point masses
also to artificial satellites as well as to the moon revolving
around the planets. Thus, the magnitude of gravitational force F between the
two particles of masses m1 and m 2 placed at distance r can be
Example 10.3 The planet neptune travels around the sun
given as
with a period of 165 yr. What is the radius of the orbit
approximately, if the orbit is considered as circular? m1m 2 Gm1m 2
F∝ or F = …(i)
Sol. As, T1 = Tearth = 1 yr and T2 = Tneptune = 165 yr r 2
r2
Let R1 and R 2 be the radii of the circular orbits of the earth where, G is the universal gravitational constant.
and neptune, respectively.
According to Kepler’s third law,
The magnitude of G in MKS or SI is 6.67 ×10 −11N-m2 kg −2
T12 R13 and in CGS, it is 6.67 × 10 −8 dyne-cm2 g −2 .
=
T22 R 23 The dimensional formula of G is [M−1L3 T −2 ].
R13 T22 Important points about force of gravitation
⇒ R 23 =
T12 (i) The gravitational constant G is called a universal
R13 × (165) 2 constant because it has same value for any two bodies
⇒ R 23 = anywhere in the space.
12
⇒ R 23 = 165 R13 ⇒ R 2 ≈ 30 R1
2 (ii) Gravitational force is always attractive in nature
and is independent of the nature of medium between
Example 10.4 A satellite of time period 24 h is orbiting the two masses and presence or absence of other bodies.
earth at a height 6R above the surface of earth, where R is (iii) Gravitational forces are central forces as they act
radius of earth. What will be the time period of another along the line joining the centre of gravity of two
satellite at a height 2.5 R from the surface of earth? bodies.
Sol. By Kepler’s third law, T 2 ∝ R 3 (iv) Gravitational forces are conservative in nature, so
T12 R13 T  R 
2 3 work done by gravitational forces does not depend
∴ = or  2  =  2  upon the path.
T22 R 23  T1   R1 
434 OBJECTIVE Physics Vol. 1

(v) For the gravitational force on a body outside a Example 10.7 Two particles A and B having masses M and
spherically symmetric body (solid spheres or 4M respectively are kept at a distance 2.73 m apart.
spherical shells), the entire mass of the body is Another small particle of mass m is to be placed, so that the
net gravitational force on it is zero. What will be its distance
assumed to be concentrated at its centre.
from body A?
(vi) The force of attraction due to a hollow spherical shell
Sol. Two particles A and B are placed as shown below and a
of uniform density on a point mass situated inside it
third particle is placed at C.
is zero.
A x m (y − x) B
(vii) In calculating the gravitational force between two
non-spherical bodies, we cannot assume their masses M C 4M
to be concentrated at their centres of mass but if the y
separation between them is very large in comparison
Now, according to the given conditions,
to their sizes, then the bodies may be considered as
FCA =FCB
particles. That means if we have to find gravitational
force between a sphere and a circular ring at infinite GMm G 4Mm
∴ =
separation, then the mass of sphere can be assumed x 2
( y − x )2
at its centre. 1 4
⇒ =
Generally, the gravitational force between x2 ( y − x )2
non-spherical bodies are found by integrating the
⇒ y − x = 2x
individual forces on individual particles within the
⇒ y = 3x
bodies.
y 2.73
(viii) Force developed between any two masses is called Q x= = ⇒ x = 0.91 m
3 3
gravitational force and the force between earth and
Hence, a particle of mass m should be placed at 0.91 m apart
any other body is called force of gravity. from body at A.
Example 10.5 Two heavy particles of masses 40 kg and
Example 10.8 Spheres of the same material and of same
60 kg attracts each other with a force of 4 × 10 −5 N. If G is
radius r are touching each other. Show that gravitational
6 × 10 −11 N -m 2 kg −2 , calculate the distance between them.
force between them is directly proportional to r 4 .
Sol. Given, m1 = 40 kg, m 2 = 60 kg, F = 4 × 10−5 N,
Sol. Two spheres are touching as shown below.
−11 −2 −2
G = 6 × 10 N -m kg , g = 10 ms
2

Gm1m 2
According to Newton’s law of gravitation, F =
r2
Gm1m 2 6 × 10−11 × 40 × 60
∴ Distance, r = =
F 4 × 10−5 2r
= 0.06 m = 6 cm According to the question,
4 
Example 10.6 If the distance between the two spherical m1 = m 2 = (volume)(density) =  πr 3 ρ
bodies is increased to four times, then by how many times, 3 
the mass of one of the bodies is to be changed to maintain 4  4 
G  π r 3   π r 3  ρ2
the same gravitational force? G m1m 2 3  3 
∴ Gravitational force, F = =
Gm1m 2 GM1M 2 r2 r2
Sol. Let F1 = 2
and F 2 = 2
r R ⇒ F ∝r4
Now, R = 4 r
Also, let m1 remains unchanged, so m1 = M1 Example 10.9 Force between two objects of equal masses is F.
If 25% mass of one object is transferred to the other object,
Gm1m 2 GM1M 2 then find the new force.
∴ =
r2 R2 Sol. Suppose mass of one object, m = 100 kg
m2 M2 M Q Masses of both objects are equal.
⇒ = ⇒ m2 = 2
r2 (4r )2 16 ∴ m1 = m 2 = 100 kg
∴ M 2 = 16 m 2 According to Newton’s law of gravitation,
mm
Hence, the mass of one body should be increased to 16 times F = G 122
to maintain the same gravitational force. R
Gravitation 435

G × 100 × 100 G × 10000 Gmm Gm 2


∴ F = 2
= 2 F = = ...(i)
R R ( 2r )2 4r 2
G F
⇒ 2
= …(i) If the speed of each particle is v, then the centripetal force,
R 10000
mv 2
If 25% mass of one object is transferred to the other object, then F = ...(ii)
r
m1 = 100 − 25 = 75 kg Equating Eqs. (i) and (ii), we get
m 2 = 100 + 25 = 125 kg Gm
v=
G × 75 × 125 G × 9375 4r
∴ New force, F ′ = =
R 2
R2 2 πr 2 πr 4π 3/ 2
∴ Time period, T = = = r
G F′ v Gm Gm
⇒ = …(ii)
R 2 9375 4r
From Eqs. (i) and (ii), we get

F′ =
15
F
Vector form of Newton’s
16 law of gravitation
Example 10.10 A particle of mass m is placed at a distance The vector form of Newton’s law of gravitation signifies
d from one end of a uniform rod with length L and mass M that the gravitational forces acting between the two
as shown in the figure. Find the magnitude of the particles form action and reaction pair.
gravitational force on the particle due to the rod.
Y
m M A m1
F12
r21=– r12
d L
r1 F21
Sol. Let us consider an elementary mass dm of length dr at a m2
B
distance r from the particle of mass m. Here, dm = (M /L )dr . r2
The gravitational force dF on m due to this elementary mass X
dm is O
m
Fig. 10.6 Vector form of gravitational force (F12 ) on m1 due to m2
dr
r
In figure, it can be seen that the two particles of masses m1
Gm GmM and m 2 are placed at a distance r, therefore according to
dF = dm = dr
r2 Lr 2 Newton’s law of gravitation, force on m1 due to m 2 ,
L+ d Gm1m 2
GmM dr F12 = − ⋅ $r12 …(i)
∴ Force on the particle, F =
L ∫ r2 | r12 |2
d
L+ d r1 − r2  r1 − r2 
GmM  1 F12 = − Gm1m 2 Q r$12 = 
= − r  r1 − r2 |
L | r1 − r2 | 3
 |
d

GmM  1 1 GmM where, $r12 is a unit vector pointing from m 2 to m1.


=− L + d − d  = d (L + d )
L The negative sign in Eq.(i) indicates that the direction of
force F12 is opposite to that of $r12 .
Example 10.11 Two particles of equal mass m are moving
round a circle of radius r due to their mutual gravitational Similarly, force on m 2 due to m1,
interaction. Find the time period of each particle. Gm1m 2
F21 = − ⋅ $r21 …(ii)
Sol. Two particles will always remain on diametrically opposite | r21|2
points, so that the gravitational force is centripetal. Here,
mutual gravitational force is where, $r21 is a unit vector pointing from m1 to m 2 .
v From Eqs. (i) and (ii), we get
m
r F12 = − F21
F
O ⇒ | F12 | = | F21| …(iii)
r
F As F12 and F21 are directed towards the centres of the two
m
v
particles, so gravitational force is conservative in nature.
436 OBJECTIVE Physics Vol. 1

GM
Principle of superposition Resultant of F1 and F3 is Fr = 2
a2
and its direction is along

of gravitational forces the diagonal, i.e. towards 2.


GM GM
Suppose F1, F2, K , Fn be the individual forces due to the Force on m due to mass M at corner 2, F 2 = =
masses m1, m 2, m 3, K, mn which are given by the universal ( 2a )2 2a 2
law of gravitation, then from the principle of superposition, and Fr and F 2 act in the same direction.
each of these forces acts independently and uninfluenced Resultant of these two gives net force,
by the other bodies as shown in figure.
2GM GMGM  1
Fr = + =
2+ 
a 2 
m4
m3 a2 2a 2 2
r41 r31
It is directed along the diagonal as shown in figure.
F14 F13
r21
m1 m2 Example 10.13 Three equal masses of 1 kg each are placed
F12 at the vertices of an equilateral ∆PQR and a mass of 2 kg is
1n
F

placed at the centroid O of the triangle which is at a


rn1
distance of 2 m from each of the vertices of the triangle.
Find the force (in newton) acting on the mass of 2 kg.
mn
Sol. Given, OP = OQ = OR = 2 m
Fig. 10.7 Gravitational forces on m1 due to m2 , m3, ..., mn using
principle of superposition of gravitational forces P
1 kg

So, the resultant force F on mass m1 can be expressed in


FOP
vector addition of various forces
O 2 kg
F = F12 + F13 + F14 + ... + F1n FOQ cos30° FOR cos30°
FOQ sin 30°
It states that, the resultant gravitational force F acting on a
particle due to number of point masses is equal to the Q 30° 30° R
1 kg S 1 kg
vector sum of forces exerted by the individual masses on
FOR sin 30°
the given particle.
mm mm mm The gravitational force on mass 2 kg due to mass 1 kg at P,
Clearly, F = − G 1 2 2 r$ 21 − G 1 2 3 r$ 31 − ..... − G 1 2 n $rn1
r12 r13 r1n 2 ×1
FOP = G = G along OP
m  ( 2 )2
m m
Resultant force, F = − Gm 1  22 r$ 21 + 23 r$ 31 +... + 2n r$n 1  2 ×1
 r12 r13 r1n  Similarly, FOQ = G = G along OQ
( 2 )2
Example 10.12 Three masses each equal to M are placed at 2 ×1
the three corners of a square of side a. Calculate the force of and FOR = G = G along OR
attraction on unit mass at the fourth corner. ( 2 )2
FOQ cos 30° and FOR cos 30° are equal and acting in opposite
Sol. Force on m due to masses at 1 and 3 are F1 and F3,
directions, hence cancel out each other.
GM Force on mass 2 kg along OS
F1 = F 3 = (Q m = 1)
a2
= FOQ sin 30° + FOR sin 30°
M F1 m = 1 kg
1 = 2FOQ sin 30° (Q FOQ = FOR )
F2
F3 = FOQ
a Fr
The resultant force on the mass 2 kg at O,
F = FOP − FOQ
M a M
= G − G = 0 (zero)
2 3
CHECK POINT 10.1
1. Kepler’s second law is based on 10. Two point masses each equal to 1 kg attract one another
(a) Newton’s first law with a force of10−9 kg-wt. The distance between the two
(b) Newton’s second law point masses is approximately (G = 6.67 × 10 −11MKS units )
(c) special theory of relativity (a) 8 cm (b) 0.8 cm
(d) conservation of angular momentum (c) 80 cm (d) 0.08 cm
2. For a satellite in elliptical orbit, which of the following 11. Gravitational force between a point mass m and M
quantities does not remain constant? separated by a distance r is F. Now, if a point mass 2m is
(a) Angular momentum (b) Momentum placed next to m in contact with it, the force on M due to m
(c) Areal velocity (d) All of these and the total force on M are
3. A planet moves around the sun. It is closest to sun at a (a) 2F, F (b) F, 2F
distance d1 and have velocity v1 .At farthest distance d 2 , its (c) F, 3F (d) F, F
speed will be 12. The distance of the centres of moon and earth is D. The
d 2v d v d v d2 v mass of earth is 81 times the mass of the moon. At what
(a) 1 21 (b) 2 1 (c) 1 1 (d) 2 21
d2 d1 d2 d1 distance from the centre of the earth, the gravitational force
on a particle will be zero?
4. Kepler’s law states that, square of the time period of any (a)
D
(b)
2D
planet moving around the sun in an elliptical orbit of 2 3
semi-major axis (R) is directly proportional to 4D 9D
1 1 (c) (d)
(a) R (b) (c) R3 (d) 3 10
R R3
13. A spherical planet far out in space has mass 2M and radius
5. The distance of two planets from the sun are1013m and a. A particle of mass m is falling freely near its surface.
1012 m, respectively. The ratio of the periods of the planet is What will be the acceleration of that particle?
1 GM 3GM
(a) 100 (b) (a) (b)
10 a2 a2
(c) 10 (d) 10 10 2GM 4 GM
(c) 2 (d) 2
a a
6. The period of revolution of planet A round the sun is
8 times that of B. The distance of A from the sun is how 14. Two balls each of radius R having equal mass and density
many times greater than that of B from the sun? are placed in contact, then the force of gravitation between
(a) 5 (b) 4 (c) 3 (d) 2 them is proportional to
1
7. The ratio of mean distances of three planets from the sun (a) F ∝ (b) F ∝ R
are 0.5 : 1 : 1.5, then the square of time periods are in the R2
1
ratio of (c) F ∝ R4 (d) F ∝
(a) 1 : 4 : 9 (b) 1 : 9 : 4 R
(c) 1 : 8 : 27 (d) 2 : 1 : 3 15. If the distance between the sun and the earth is increased by
8. A body is orbiting around earth at a mean radius which is three times, then attraction between two will
two times greater than the parking orbit of a satellite, the (a) remain constant
period of body is (b) decrease by 63%
(a) 4 days (b) 16 days (c) increase by 63%
(c) 2 2 days (d) 64 days (d) decrease by 89%
9. Which of the following statements about the gravitational 16. Three equal masses of 2 kg each are placed at the vertices of
constant is true? an equilateral triangle and a mass of 4 kg is placed at the
(a) It is a force. centroid of the triangle which is at a distance of 2 m from
(b) It has no unit. each of the vertices of the triangle. The force (in newton)
(c) It has same value in all systems of unit. acting on the mass of 4 kg is
(d) It does not depend on the nature of the medium in (a) 2 (b) 2
which the bodies are kept. (c) 1 (d) zero
438 OBJECTIVE Physics Vol. 1

Example 10.14 The acceleration due to gravity at the


GRAVITY moon’s surface is 1.67 ms −2 . If the radius of the moon is
In Newton’s law of gravitation, the force of attraction 1.74 × 10 6 m, calculate the mass of the moon.
between the two bodies is known as gravitation. If one of GM gR 2
these bodies is earth, then gravitation is called gravity. Sol. The acceleration due to gravity, g = 2
or M =
R G
Hence, gravity is another force by which earth attracts
This relation is true not only for the earth but also for any
another body towards its centre.
heavenly body which is assumed to be spherical.
Now, g = 1.67 ms−2, R = 1.74 × 106 m
Acceleration due to
and G = 6.67 × 10−11 N-m2 kg −2
gravity (g) on earth 1.67 × (1.74 × 106 )2
The force of gravity acting on a body having unit mass ∴ Mass of the moon, M = kg
6.67 × 10−11
placed on or near the surface of the earth is known as
acceleration due to gravity. = 7.58 × 1022 kg
Or
When a body is dropped from a certain height above the Example 10.15 Assume that, if the earth were made of lead
ground, it begins to fall towards the centre of the earth of relative density 11.3, then what would be the value of
acceleration due to gravity on the earth’s surface?
under the effect of gravity. The acceleration produced in
the body due to gravity, is known as acceleration due to Sol. Since, density of earth can be given as
gravity. ρ = Relative density × Density of water = 11.3 × 103 kgm−3
It is a vector quantity and denoted by g. Its average value ∴ Acceleration due to gravity on the earth’s surface,
on the earth’s surface at mean sea level is 9.8 ms −2 . GM G 4 4
g = 2 = 2 ⋅ πR 3 ⋅ ρ = πGR ⋅ ρ
R R 3 3
Relation between acceleration due to gravity (g) 4 22
and gravitational constant (G) = × × 6.67 × 10−11 × 6.4 ×106 × 11.3 × 103
3 7
Suppose that the mass of the earth is M, its radius is R, = 20.21 ms−2
then the force of attraction acting on a body of mass m
close to the surface of the earth is Example 10.16 What will be the relation between the
GMm acceleration due to gravity on the surface of the earth and on
F = a planet respectively, whose mass and radius are four times that
R2 of the earth?
According to Newton’s second law, the acceleration due to Sol. Let the acceleration due to gravity of the planet be g p .
gravity,
GM p
F GM As, gp =
g= = 2 …(i) R p2
m R
According to the question,
This expression is free from m. It means acceleration due Mass of planet, M p = 4M e
to gravity does not depend on the mass of the object, thus and radius of planet, R p = 4R e
it is same for all. If in Eq. (i), we put the value of G as G 4M e
∴ gp =
6.67 × 10 −11N-m2kg −2 , M as 6 × 10 24 kg and R as 16R e2
6.4 × 10 6 m, then on solving it, we will get the value of 1  GM 
∴ gp = ge Q g e = 2 e 
g as 9.8 ms −2 . 4  Re 
If two bodies of different masses are allowed to fall freely,
then they will have the same acceleration, i.e. if they are Variation in the value of
allowed to fall from the same height, then they will reach
the earth simultaneously. The dimensions of acceleration
acceleration due to gravity (g)
The value of acceleration due to gravity varies due to
due to gravity are [LT −2 ].
following factors
Note
(i) The value of g is independent of mass, shape, size, etc., of the body
(i) Height above the surface of earth
and depends upon the mass and radius of the earth. (ii) Depth below the surface of earth
(ii) The value of g on the surface of the moon is equal to 1/6 times the (iii) Shape of the earth
value of g on the surface of the earth.
(iv) Axial rotation of the earth
Gravitation 439

Example 10.17 At what altitude, the acceleration due to


1. Variation in g due to height gravity reduces to one-fourth of its value as that on the
above the surface of earth surface of the earth? (Take, radius of earth as 6.4 × 10 6 m
Consider a body of mass m lying on the surface of the and g on the surface of the earth as 9.8 ms −2 )
earth of mass M and radius R. Acceleration due to gravity g
GM Sol. Acceleration due to gravity at height h, g ′ = 2
at the surface of the earth, g = 2  h
1 + 
R  R
m
If acceleration due to gravity reduced to one-fourth of its
h g′
value, then
2
B g g  h h
⇒ = ⇒ 1 +  = 4 ⇒ 1 + = 2
4  h
2  R  R
R 1 + 
 R
R
M h
⇒ = 1 ⇒ h = R = 6.4 × 106 m
r R
(a) (b) ∴ Acceleration due to gravity is reduced to one-fourth of its
Fig. 10.8 Variation of g with altitude value on the earth’s surface at an altitude of 6.4 × 106 m.
The force of gravity on an object of mass m at a height h Example 10.18 At what height, the acceleration due to gravity
above the surface of the earth is decreases by 51% of its value on the surface of the earth?
GMm 2
F =  R 
Sol. As, g h = g 
51
(R + h ) 2  and g h = g − 51% of g = g − g
R + h  100
∴ Acceleration due to gravity at this height will be 49
⇒ gh = g
F GM 100
g′ = = 2
m (R + h ) 2 49  R 
g=g 
100 R + h 
This can also be written as
7 R
=
GM g  GM  10 R + h
g′ = ⇒ g′ = Q 2 = g 
2 2  R  ⇒ 7R + 7h = 10R
2h  h
R 1 +  1 +  3R
 R  R ∴ 7h = 3R ⇒ h =
7
Thus, g′ < g
i.e. The value of acceleration due to gravity g goes on 2. Variation in g due to depth
decreasing as we go above the surface of the earth. below the surface of earth
Also, if (R + h ) = r Let an object of mass m is situated at a depth d below the
GM gR 2 1 earth’s surface. Its distance from the centre of the earth is
g′ = = ; g′ ∝ (R − d ). This mass is situated at the outer surface of the
(R + h ) 2 r2 r2
inner solid sphere. The gravitational force of attraction on a
Further, by using Binomial theorem, we get mass inside a spherical shell is always zero. Therefore, the
object experiences gravitational attraction only due to
−2
 h  2h  inner solid sphere.
g ′ = g 1 +  ⇒ g ′ ≈ g 1 −  (Q h < < R )
 R  R
d g′
m
Note With height h, the decrease in the value of g is g − g ′ = 2 gh
R R–d
g − g ′ 2h O
∴ Fractional decreases in the value of g is = Straight line
g R
R
 g − g ′
∴ Percentage decreases in the value of g =   ×100 d
 g  (a) (b) R
2h
= ×100% Fig. 10.9 Variation of g with depth
R
440 OBJECTIVE Physics Vol. 1

 M  4 Note The acceleration due to gravity is maximum (having standard


The mass of this sphere is M ′ =  3 3
π (R − d ) 3 value of 9.8 ms −2) at the earth’s surface. It decreases either
( 4 /3 )πR  when we go at higher altitudes or we move below the surface.

Here, M being the mass of whole sphere of radius R. Example 10.19 At what depth from the surface of the earth,
(R − d ) 3 the acceleration due to gravity will be half the value of g on
Therefore, M′ = 3
M the surface of the earth?
R
 d
The force of gravity on an object of mass M at a depth d Sol. As, g ′ = g 1 − 
 Re
below the surface of earth,
GM ′ m GMm (R − d ) F GM According to the question,
F = = and g ′ = = 3 (R − d ) g
(R − d ) 2
R 3 m R g′ =
2
GM ....(i) g  d
Since, g= 2 ⇒ = g 1 − 
R 2  Re
Substituting the values in Eq. (i), we get 1 d
∴ =1−
2 Re
 d
g ′ = g 1 −  ⇒
d
=1−
1
 R Re 2
Since, g is a constant at a given place of the earth and R is ⇒ d = 0.5 R e
also a constant. Example 10.20 At what depth from earth’s surface,
∴ (g − g ′ ) ∝ d acceleration due to gravity is decreased by 1% ?
Hence, the acceleration due to gravity decreases as we Sol. Fractional decrease in the value of g with depth,
move down from the surface of the earth. ∆g d d
=
g g Re
Note Decrease in the value of g with depth d is g − g ′ = d
R
1 d
g − g′ d ⇒ =
∴ Fractional decrease in the value of g with depth = = 100 6400
g R
g − g′ ⇒ d = 64 km
∴Percentage decrease in the value of g = ×100
g
d Example 10.21 Assuming earth to be a sphere of uniform
= ×100 mass density, how much would a body weigh half-way
R
down the centre of the earth, if it weighed 100 N on the
We can see from this equation that g ′ = 0 at d = R, i.e.
surface?
acceleration due to gravity is zero (minimum value) at the centre
of the earth. Sol. Given, w = mg = 100 N
 d
If r is distance from the centre of earth. As, g ′ = g 1 − 
 R
 d  gr
For r ≤ R , g ′ = g 1 −  = (Q R − d = r ) d 1
It is also given that, =
 R R
R 2
or g′ ∝ r
gR 2 1  1 g
g ∴ g ′ = g 1 −  =
For r > R , g′ = 2
= 2
or g ′ ∝ 2  2 2
 h r r
1 +  Weight of body half-way down the centre of the earth,
 R mg 100
w ′ = mg ′ = = = 50 N
The graphical representation of change in the value of g′ 2 2
with height and depth is as follows
Example 10.22 Determine the decrease in the weight of a
g′
body when it is taken 32 km below the earth’s surface.
(Take, radius of the earth as 6400 km)
GM
1 Sol. Given, R = 6400 km, d = 32 km
R2 g ′∝ r g ′∝ r 2
 d  32   199 
Q g ′ = g 1 −  = g 1 − = g
 R   6400  200
r
O r=R 199 g
∴ g − g′ = g − g=
Fig. 10.10 Graphical representation for variation of g′ 200 200
Gravitation 441

The percentage decrease in the weight of the body Sol. The acceleration due to gravity on the new planet can be
mg − mg ′ written using the relation,
= × 100
mg GM
g= 2 …(i)
g − g′ R
= × 100
g But M = (4 /3) πR 3ρ, where ρ be the density.
g Thus, Eq. (i) becomes
= 200 × 100 G × (4 /3) πR 3ρ 4
g g= =G × π Rρ
R2 3
1 ⇒ g ∝R (Q ρ = constant)
= × 100 = 0.5%
200 g′ R′ g ′ 3R
∴ = ⇒ =
g R g R
3. Variation in g due to the g′
⇒ = 3 ⇒ g ′ = 3g
shape of the earth g
The earth is not a perfect sphere. It is somewhat flat at the
two poles. The equitorial radius (R eq ) is approximately 4. Variation in g due to axial
21 km more than the polar radius (R p ).
rotation of the earth
N
Suppose the earth is rotating on its axis with angular
velocity ω. Consider a particle P at rest on the surface of
Rp
the earth, at latitude φ, then the pseudo force acting on the
W
Req
E particle is mrω 2 in outward direction. The acceleration due
to gravity g is acting towards the centre O of the earth.
Thus, the effective acceleration due to gravity g′ is the
21 km S 21 km resultant of g and rω 2 .
Fig. 10.11 Variation in g due to shape of the earth After derivation, we can find the following relation
Now, acceleration due to gravity at poles, g ′ = g − Rω 2 cos 2 φ
GM
gp = 2 Y
ω
Rp
r P rω2
Acceleration due to gravity at equator, g
g′
GM φ
g eq = 2 O X
R eq R

Q R p < R eq
∴ g p > g eq Fig. 10.12 Axial rotation of the earth
Difference in g at poles and equator due to shape, Following conclusions can be drawn from the above
(∆g ) = g p − g eq = 0.02 ms −2 discussion
The value of g is minimum at the equator and (i) The effective value of g is not truely vertical.
maximum at the poles. So, this is the reason why the (ii) The effect of centrifugal force due to rotation of the
weight of the body increases when it is taken from equator earth is to reduce the effective value of g.
to the pole. (iii) At equators φ = 0 °.
Note If weight = constant, i.e. mg = constant, then m ∝
1
. Therefore, g ′ = g − Rω 2 and at poles φ = 90 °.
g
Therefore, g ′ = g
It means that, it will be profitable to buy sugar at the equator in
comparison to poles because more mass (amount) will be Thus, at equator g′ is minimum while at poles g′ is
obtained there. maximum.
Note
Example 10.23 Imagine a new planet having the same (i) If the angular velocity of earth increases, the value of g will
density as that of earth but it is 3 times bigger than the earth decrease at all places except at poles.
in size. If the acceleration due to gravity on the surface of (ii) Since, earth rotates west to east, we also project our rockets from
earth is g and that on the surface of the new planet is g′, west to east, so that effective acceleration due to gravity
then find the relation between g and g′ . ( g ′ = g − Rω 2) on the rocket is less.
442 OBJECTIVE Physics Vol. 1

Example 10.24 Find the imaginary angular velocity of the Sol. Let acceleration due to gravity at pole is g and at equator, it
earth for which the effective acceleration due to gravity at is g ′. If mass of the person is m and angular velocity of
the equator shall be zero. (Take, g = 10 m/s 2 for the rotation of the earth is ω, then
acceleration due to gravity, if the earth were at rest and
radius of earth equal to 6400 km and φ = 60° ) g ′ = g − ω 2R ⇒ mg ′ = mg − mω 2R
3
Sol. Acceleration due to gravity, g ′ = g − ω 2R cos2 φ ⇒ mg = mg − mω 2R
4
0 = g − ω 2R cos2 60° (Q g ′ = 0) 1
⇒ ω 2R = g
ω R
2
4
0=g −
4 g 9.8
⇒ ω= =
ω =2
g
=2
10 4R 4 × 6400 × 103
R 6400 × 1000 1 1 1
∴ ω= × × × 98 = 618. × 10−4 rads −1
1 2 80 102
⇒ ω= = 2.5 × 10−3 rad/s
400
Example 10.27 Suppose the earth increases its speed of
Example 10.25 Calculate the angular speed of rotation of the rotation. At what new time period, will the weight of a body
earth, so that the apparent g at the equator becomes half of on the equator becomes zero? (Take, g = 10 ms −2 and radius
its value at the surface. Also, calculate the length of the day of earth R = 6400 km )
in this situation. Sol. The weight will become zero, when
Sol. The apparent acceleration due to gravity, g ′ = 0 or g − Rω 2 = 0
g = g 0 − ω 2R = g 0 /2 (On the equator, g ′ = g − Rω2)
g
g0 9.8 or ω=
⇒ ω= = = 8.75 × 10−4 rads−1 R
2R 6.4 × 10 ×26
2π g R
The length of the day = Time period of rotation of the earth ∴ = or T = 2π
T R g
2π 2R 2 × 6.4 × 106
= = 2π = 2π Substituting the given values in above expression, we get
ω g0 9.8
= 7180 s ≈ 2 h 6400 × 103

T = 10 h or T ≈1.4 h
Example 10.26 Find the value of angular velocity of axial 3600
rotation of the earth, such that weight of a person at equator
Thus, the new time period should be 1.4 h instead of 24 h for
becomes (3/4)th of its weight at pole. Radius of the earth at the weight of a body to be zero at the equator.
equator is 6400 km.

CHECK POINT 10.2


1. The mass of a planet is twice the mass of earth and diameter 4. The acceleration due to gravity g and mean density of earth
of the planet is thrice the diameter of the earth, then the ρ are related by which of the following relations?
acceleration due to gravity on the planet’s surface is (G = gravitational constant and R = radius of earth)
(a) g / 2 (b) 2g
4 πgR2 4 πgR3 3g 3g
(c) 2g / 9 (d) 3g/ 2 (a) ρ = (b) ρ = (c) ρ = (d) ρ =
3G 3G 4 πGR 4 πGR3
2. If the earth suddenly shrinks (without changing mass) to 5. The diameters of two planets are in the ratio 4:1 and their
half of its present radius, then acceleration due to gravity mean densities in the ratio of 1:2. The acceleration due to
will be gravity on the planets will be in ratio
(a) g / 2 (b) 4g (a) 1 : 2 (b) 2 : 3
(c) g /4 (d) 2g (c) 2 : 1 (d) 4 : 1
3. If G is universal gravitational constant and g is acceleration 6. If density of earth increased 4 times and its radius reduced
G to half of what it is, our weight will
due to gravity, then the unit of the quantity is (a) be four times its present value
g
2 −1
(b) be doubled
(a) kg-m (b) kgm (c) remain same
(c) kgm−2 (d) m2 kg −1 (d) be halved
Gravitation 443

7. A body has a weight 72 N. When it is taken to a 12. The weight of a body at the centre of the earth is
height h = R = radius of earth, it would weight (a) zero
(a) 72 N (b) 36 N (c) 18 N (d) zero (b) infinite
8. The height above the surface of the earth, where (c) same as on the surface of earth
acceleration due to gravity is (1/64)th of its value at surface (d) None of the above
of the earth is approximately
13. Weight of a body is maximum at
(a) 45 × 106 m (b) 54 × 106 m
(a) poles (b) equator
(c) 102 × 106 m (d) 72 × 106 m (c) centre of earth (d) at latitude 45°
9. The depth d, at which the value of acceleration due to 14. When a body is taken from the equator to the poles, its
gravity becomes1 / n times the value at the surface is weight
(Here, R = radius of the earth) (a) remains constant
n − 1  n 
(b) R 
R R (b) increases
(a)  (c) 2 (d) R  
n  n  n  n + 1 (c) decreases
(d) increase at N-pole and decrease at S-pole
10. If the change in the value of g at a height h above the
15. The angular speed of earth in rad s −1 , so that the object on
surface of earth is the same as at a depth d below it (both h
and d are much smaller than the radius of the earth), then equator may appear weightless, is (radius of earth = 6400 km)
(a) d = h (b) d = 2h (a) 1.23 × 10−3 (b) 6.20 × 10−3(c) 1.56 (d) 1.23 × 10−5
(c) d = h / 2 (d) d = h2
16. Earth is supposed to be sphere of radius R. If g 30 ° is value of
11. At what depth below the surface of the earth, acceleration
due to gravity will be half of its value at 1600 km above the acceleration due to gravity at latitude of 30° and g at the
surface of the earth? equator, then value of g − g 30 ° is
(a) 4.3 × 106 m (b) 2.4 × 106 m 1 2 3 2 1 2
(a) ωR (b) ωR (c) ω2R (d) ωR
(c) 3.2 × 106 m (d) 1.6 × 106 m 4 4 2

GRAVITATIONAL FIELD Its SI unit is ms −2 or Nkg −1 and dimensions are [M0 LT −2 ].


An invisible field in space around a body due to its mass, It is a vector quantity and is always directed towards the
in which any other body experiences a force of attraction centre of gravity of body whose gravitational field
is called the gravitational field of the first body. intensity is to be considered.
In case of our planet earth, since it is surrounded by a Note
gravitational field, so any body brought in this field (i) Inertial mass of a body can be measured as the ratio of the
experiences a force of attraction towards the centre of earth. magnitude of the external force applied, to the magnitude of
acceleration produced in its motion.
(ii) Gravitational mass of a body is defined as the gravitational pull
Gravitational field intensity experienced by the body in a gravitational field of unit intensity.
The intensity of gravitational field at any point in the
gravitational field due to a given mass is defined as the
force experienced by a unit mass placed at that point
Gravitational field intensity
provided the presence of that unit mass does not alter the for different bodies
original gravitational field.
Usually, it is denoted by E or I.
1. Field due to a point mass
Suppose a point mass M is placed at point O and we want
M
to find the intensity of gravitational field E at point P, at a
F distance r from O.
m
So, the magnitude of force F acting on the particle of mass
r m placed at P is
M F m
Fig. 10.13 Gravitational field intensity O P
r
Thus, if F be the gravitational force experienced by the
Fig. 10.14 Gravitational field due to a point mass
test mass m, then the gravitational field intensity at any
point will be GMm F GM
F F = 2
⇒ E = = 2
E= r m r
m
444 OBJECTIVE Physics Vol. 1

GM GM $ GM $ GM $
or E = = i + 2 i + 2 i + … ∞ terms
r 2 (1)2 (2) (4)
 1 1   1
GM GM = GM $i 1 + + + … ∞  Here, a = 1 and r = 
In vector form, E = (− $r) = − r  4 16   4
r2 r3
The direction of force F and E is from P to O as shown in  a $
So, Enet = GM  i
the figure above. 1 − r 
Here, the negative sign shows the attractive force on body.  1   1 
= GM $i   = GM $i  
Note 1 − 1/ 4  3 / 4
(i) If r = ∞ ,E = 0. It means the intensity of gravitational field is zero
4
only at infinite distance from the body. ⇒ Enet = GM $i
(ii) If a unit mass ( m) is placed on the surface of earth, then the 3
gravitational force acting on the rest mass m will be equal to the
weight w of the test mass. 2. Field due to a uniform solid sphere
w mg
Now, E= = = g or E = g Field at an external point A uniform solid sphere may be
m m
(iii) We can also apply the principle of superposition to gravitational treated as a single particle of same mass placed at its centre
field intensity in the same way as that of gravitational force. (O) for calculating the gravitational field at an external
i.e. E = E1 + E 2 + … + E n point (P).
R
Example 10.28 Two point masses of mass 10 kg and
1000 kg are at a distance 1m apart. At which points on the P
line joining them, will the gravitational field intensity be O
zero?
Sol. Let the resultant gravitational intensity be zero at a r
distance x from the mass of 10 kg on the line joining the Fig. 10.15 Gravitational field at an external point P
centre of two bodies. At this point, the gravitational
intensities due to the two bodies must be equal and opposite. GM
Thus, E (r ) =
G × 10 G × 1000 r2
∴ =
x 2
(1 − x )
2 1
For r ≥ R or E (r ) ∝
⇒ 100x = (1 − x )
2 2
r2
⇒ 10x = 1 − x Here, r is the distance of the point from the centre of the
⇒ 11x = 1 sphere and R is the radius of sphere.
1 Field at an internal point The gravitational field due to
⇒ x=  m
11 a uniform sphere at an internal point (P) is proportional to
the distance (r) of the point from the centre of the sphere.
Example 10.29 Figure shows a system of point masses placed
on X-axis. Find the net gravitational field intensity at the
origin.
O P
Y R
M M M M r
∞ X
O
1m
2m
Fig. 10.16 Gravitational field at an internal point P
4m
8m The gravitational field at the centre is zero and at surface
GM
a is 2 , where R is the radius of the sphere.
(Take, sum of an infinite GP as S = , R
1−r
GM
where a = first term and r = least common ratio.) Thus, E (r ) = 3 . r
Sol. Net gravitational field intensity at the origin, R
Enet = E1 + E2 + E3 + E4 + … ∞ terms For r ≤ R or E (r ) ∝ r
Gravitation 445

Thus, E versus r graph is as shown in figure below Sol. Let mass of the solid sphere is M and radius is R.
E It is given that,
GM
E surface = 2 = 1.5 × 10−4 N kg –1
R
GM
GM
R2 1 ∴ E (r = R /2) = 3 (R / 2)
E∝r E∝ R
r2
GM 1.5 × 10−4
= 2
= = 7.5 × 10−5 N kg −1
r 2R 2
O r=R
Fig. 10.17 E versus r graph for solid sphere
3. Field due to a uniform spherical shell
Example 10.30 Two solid spheres of radius 10 cm and At an external point For an external point (P), the shell
masses 800 kg and 600 kg, are at a distance 0.25 m apart. may be treated as a single particle of same mass placed at
Calculate the magnitude of the gravitational field intensity at its centre (O).
a point distance 0.20 m from the 800 kg sphere and 0.15 m
from the 600 kg sphere and does not lie on the line joining
their centres. (Take, G = 6.6 × 10 −11 N - m 2kg −2 ) R

Sol. Let E A be the gravitational field intensity at P due to P


O r
800 kg sphere at A.
P

EA EB Fig. 10.18 Gravitational field at an external point P


0.20 m 0.15 m
Thus, at an external point the gravitational field is given
by
GM
A
0.25 m B E (r ) = (for r ≥ R )
r2
800 kg 600 kg
At r = R (the surface of shell),
800
Then, E A = G = 2 × 104 G, along PA GM
(0.2)2 E = 2
Let EB be the gravitational field intensity at P due to 600 kg R
sphere at B. 1
and otherwise, E ∝ 2
600 80000 G r
Then, EB = G = , along PB
(0.15)2 3 At an internal point The field inside a uniform spherical
The angle between E A and EB is 90°. (Q AB 2 = AP 2 + BP 2 ) shell is zero.
If E be the magnitude of the resultant intensity, then Thus, E versus r graph is as shown in figure below
2
 80000 G  E
E = E A2 + EB2 = (2 × 104 G )2 +  
 3 
64 GM
=G 4 × 108 + × 108
9 R2 1
E=0 E∝
r2
16
= 6.67 × 10−11 × 2 × 104 1+
9
5 r
= 6.67 × 2 × × 10−7 N kg −1 O r =R
3
Fig. 10.19 E versus r graph for spherical shell
⇒ E = 2.22 × 10−6 N kg −1
Example 10.32 Two concentric spherical shells have masses
Example 10.31 Gravitational field at the surface of a solid
−4 −1 m1 and m 2 and radii r1 and r2 (r2 > r1 ). What is the force
. × 10 N kg . Find the gravitational field at a
sphere is 15
exerted by this system on a particle of mass m 3 , if it is
point situated inside the sphere at a distance equal to half of
placed at a distance r (r1 < r < r2 ) from the centre?
the radius of the solid sphere.
446 OBJECTIVE Physics Vol. 1

Sol. The outer shell will have no contribution in the gravitational Example 10.33 Mass of 2 kg is distributed uniformly over a
field at point P. ring of radius 2m. Find the gravitational field at a point lying
Gm on the axis of the ring at a distance of 2 3 m from the centre.
∴ EP = 2 1
r Sol. Given, mass of the ring, M = 2 kg
m2
Radius of the ring, R = 2 m
m1 Distance of the point, r = 2 3 m
m3 ∴ Required gravitational field,
O P
r1 r GMr G ×2×2 3
E (r = 2 3 ) = =
(R + r )
2 2 3/ 2
( 4 + 12 )3
r2
4 3G 3G 6.67 × 10−11 × 3
= = =
64 16 16
Thus, force on mass m 3 placed at P is,
= 0.72 × 10−11= 7.2 × 10−12 Nkg −1
Gm1m 3
F = m 3 EP or F =
r2
The field EP and the force F both are towards centre O. GRAVITATIONAL
POTENTIAL
4. Field due to a uniform circular
The work done in bringing a unit mass slowly from reference
ring at a point on the axis point (infinity) to a given point in the gravitational field, is
Gravitational field intensity at a point P on the axis of a called the gravitational potential at that point.
circular ring of radius R and mass M is given by This work is done by the external agent in bringing the
GMr mass. The gravitational potential is denoted by V.
E (r ) = LetW joule of work is to be done by external agent in
(R + r 2 ) 3/ 2
2
bringing a test mass m from infinity to some point, then
gravitational potential at the point will be

W
r V =
P m
E
The SI unit of gravitational potential is J kg −1 or m2 s −2
R

and it is a scalar quantity because it is defined as work


Fig. 10.20 Gravitational field at a point P on the axis of ring done per unit mass. Its dimensions are [M0 L2 T −2 ].
This is directed towards the centre of the ring. It is zero at Note Negative work means force is opposite to displacement.
the centre of the ring and maximum at r = R / 2
(can be obtained by putting dE /dr = 0). Gravitational potential
2GM
for different bodies
The maximum value is E max =
3 3R 2 1. Potential due to a point mass
Thus, E versus r graph is as shown in figure below Suppose a point mass M is situated at a point O. We want
to find the gravitational potential due to this mass at a
E
point P situated at a distance r from O.
M r P
Emax = 2GM2 O
3 3R
Fig. 10.22 Gravitational potential due to a point mass
For this let us find work done in taking the unit mass from
r P to infinity. This will be
O
r= R ∞ ∞ GM GM
2 W = ∫ F ⋅ dr = ∫ 2
dr =
Fig. 10.21 E versus r graph for a circular ring r r r r
Gravitation 447

Hence, the work done in bringing unit mass from infinity to P − 6.67 × 10−11 3
∴ V= [10 + 105]
GM 100
will be − . Thus, the gravitational potential at P will be
r Hence, V = − 6.73 × 10−8 J kg −1
GM
V =− Example 10.36 Infinite number of bodies, each of mass 2 kg
r
are situated on X-axis at distance 1m, 2m, 4m and 8m
Gravitational potential at a point is always negative. respectively from the origin. What is the resulting
Note gravitational potential due to this system at the origin?
(i) When r = ∞ from above formula, then V = 0, hence gravitational Sol. The resulting gravitational potential,
potential is maximum (zero) at infinity.
1 1 1 1 
(ii) At surface of the earth r = R, then V = −
GM V = − 2G  + + + …
R
at the surface of the 1 2 4 8 
earth.  1 1 1 
= − 2G 1 + + 2 + 3 …
Example 10.34 A point of mass 15 kg is placed at the origin  2 2 2 
of coordinate axis. Find the gravitational potential at a point − 2G − 2G
located at x = 5 m on X-axis. ⇒ V= = = − 4G
 1 1
1 − 
Sol. Given, M = 15 kg  2 2
Gravitational potential at x = 5 m,
− GM − 6.67 × 10−11 × 15 3. Potential due to a uniform solid sphere
V= =
x 5 Potential at an external point The gravitational
−11 −1
= − 20.01 × 10 J kg potential due to a uniform solid sphere at an external point
P is same as that due to a single particle of same mass
2. Gravitational potential due to a placed at its centre.
system of more than one point mass
Consider a system of point masses M1, M 2, M 3, …, Mn . R
A point P is situated at a distances r 1, r 2, r 3, …, rn
respectively from these points. O r P
M1
r1
M2 Fig. 10.24 Potential at an external point P
r2
GM
P
r3
M3 Thus, V (r ) = − , r ≥R
rn
r
Mn GM
At the surface, r = R and V = −
Fig. 10.23 Potential due to system of point masses R
Potential at an internal point At some internal point,
Gravitational potential at this point P due to these point potential at a distance r from the centre is given by
masses,
GM
V (r ) = − 3 (1.5R 2 − 0.5r 2 ) , r ≤ R
 GM1 GM 2 GM 3 GMn  R
VP = −  + + +K+
 r1 r2 r3 rn 

Example 10.35 Two heavy point masses of mass 10 3 kg and O


r
10 5 kg are separated by a distance of 200 m. What will be P
R
the potential at the mid-point of the line joining them?
Sol. The total potential at A,
P Q Fig. 10.25 Potential at an internal point P
A
GM
100 m 100 m At r = R, V = −
200 m R
GMP GMQ 1.5 GM
V = VP + VQ = − − while at r = 0, V = −
r r R
448 OBJECTIVE Physics Vol. 1

i.e. At the centre of the sphere, the potential is 1.5 treated as a point mass of same magnitude at its centre.
times the potential at surface. Thus, potential at a distance r is given by
TheV versus r graph is as shown in figure below.
R
V
O r P

r<R R r>R
r
O
GM Hyperbolic Fig. 10.27 Potential at an external point P

R − GM
GM
–1.5 R Parabolic V (r ) = , where r ≥ R
r
Fig. 10.26 V versus r graph for solid sphere GM
At r = R,V = −
R
Example 10.37 The radius of the earth is 6.37 × 10 6 m Potential at an internal point The potential due to a
and its mean density is 5.5 × 10 3 kg m −3 and uniform spherical shell is constant throughout at any point
G = 6.67 × 10 −11 N-m 2 kg −2 . Find the gravitational GM
potential on the surface of the earth. inside the shell and this is equal to − .
R
Sol. Given, radius of the earth, R = 6.37 × 106m
Density, ρ = 5.5 × 103 kg m−3 , G = 6.67 × 10−11 N-m2 kg −2
R
4 r
Q Mass of the earth, M = Volume × Density = πR 3ρ O P
3
∴ Gravitational potential on the earth’s surface,
− GM − G 4 −4
V= = × πR 3ρ = πGR 2ρ Fig. 10.28 Potential at an internal point P
R R 3 3
−4 Thus,V versus r graph for a spherical shell is as shown in
= × 3.14 × 6.67 × 10−11 × (6.37 × 106 )2 × 5.5 × 103
3 figure below
⇒ V = − 6.23 × 107 J kg −1 V

Example 10.38 At a point above the surface of the earth, the r<R R r>R
r
gravitational potential is − 5.12 × 10 7 J/kg and the O
acceleration due to gravity is 6.4 m / s 2 . Assuming the mean
radius of the earth to be 6400 km, calculate the height of the
point above the earth’s surface. −
GM
R
Sol. If r is the distance of the given point from the centre of the
earth, then gravitational potential at the point,
Fig. 10.29 V versus r graph for spherical shell
GM
V=− = − 5.12 × 107 J/kg
r Example 10.39 A particle of mass 1 kg is kept on the surface
GM of a uniform thin spherical shell of mass 20 kg and radius
Acceleration due to gravity at this point, g = 2 = 6.4 m/s2 1 m. Find the work to be done against the gravitational force
r
between them to take the particle away from the thin
|V | GM /r
Clearly, = =r spherical shell.
g GM /r 2
Sol. Potential at the surface of thin spherical shell,
5.12 × 107 J/kg GM
Thus, r= V=−
6.4 m/s2 R
= 8 × 106 m = 8000 km (6.67 × 10−11)(20)
=− J kg −1
Obviously, height of the point from the earth’s surface 1
= (r − R ) = 8000 km − 6400 km = 1600 km = − 1.334 × 10−9 J kg −1
i.e. 1.334 × 10−9 J work is obtained to bring a mass of 1 kg
4. Potential due to a thin spherical shell from infinity to the surface of spherical shell. Hence, the same
Potential at an external point To calculate the potential amount of work will have to be done to take the particle away
at an external point P, a uniform spherical shell may be from the surface of spherical shell. Thus,W = 1.334 × 10−9 J.
Gravitation 449

Example 10.40 A particle of mass M is placed at the centre Sol. Mass of the ring, M = 1 kg
of a spherical shell of same mass and radius a. What will be Radius of the ring, R = 1 m
the magnitude of the gravitational potential at a point Distance of the point, r = 1 m
situated at a / 2 distance from the centre? ∴ Required gravitational potential,
Sol. The given condition is as shown in figure below. − GM − G ×1
V (r = 1) = =
M R +r
2 2
(1)2 + (1)2
a −G − 6.67 × 10−11
= = Jkg −1
M P 2 2
a/2
= − 4.716 × 10−11 Jkg −1

Therefore, the magnitude of gravitational potential at a point


Relation between gravitational
situated at a / 2 distance from the centre can be given as field and potential
− GM GM − 3GM If change in gravitational potential at a point is dV,
VP = Vsphere + Vparticle = − =
a a /2 a gravitational field intensity is E, then during displacement
dr in the field,
5. Potential due to a uniform ring dV = − E ⋅ dr
at a point on its axis where, E = E $i + E $j + E k $
x y z
The gravitational potential at a distance r from the centre
on the axis of a ring of mass M and radius R is given by dr = dx $i + dy $j + dz k
$
GM ∴ dV = − E x dx − E y dy − E z dz
V (r ) = − , 0 ≤r ≤ ∞
R2 +r2 − ∂V − ∂V − ∂V
Also we can write, E x = , Ey = and E z =
∂x ∂y ∂z

where, represents partial differentiation with respect to
∂x
O P
r ∂ ∂
x considering y and z as constants. Similarly, and
R ∂y ∂z
are partial differentiations with respect to y and z
respectively considering other variables as constants.
Fig. 10.30 Potential due to a ring
Note If field is given, the potential can be obtained as
r2
GM V ( r2) − V ( r1) = − ∫ E ⋅ dr
At r = 0,V = − , i.e. at the centre of the ring, r1
R
GM Example 10.42 If gravitational potential isV = xy 2 , find the
gravitational potential is − . gravitational field at (2, 1).
R
Thus, V versus r graph is as shown in figure below. Sol. Given,V = xy 2
Gravitational field intensity in x-direction,
V − ∂V − ∂
Ex = = (xy 2 ) = − y 2
∂x ∂x
O r
Gravitational field intensity in y-direction,
− ∂V − ∂
Ey = = (xy 2 ) = −2 yx
∂y ∂y
− GM
R
∴ E = E $i + E $j = (− y 2 )$i + (−2xy )$j
x y
Fig. 10.31 V versus r graph for a ring
∴ E (x = 2, y = 1) = − (1) i + (−2 × 2 × 1)$j = − $i − 4$j
2$

Example 10.41 Mass of 1 kg is distributed uniformly over a ring ⇒ | E | = (−1)2 + (−4)2


of radius 1 m. Find the gravitational potential at a point lying on
the axis of the ring at a distance of 1m from the centre. = 17 unit
450 OBJECTIVE Physics Vol. 1

⇒ V (2, 1) − V (0, 0) = − ∫ E x dx + 
Example 10.43 If gravitational field is given by 2 1
E = − x$i − 2 y 2 $j. When gravitational potential is zero at  0 ∫ 0 E y dy
(0,0), find potential at (2, 1).
= − ∫ − xdx + 
2 1
∫ 0 − 2 y dy
2
Sol. Given, gravitational field,
 0
E = − x$i − 2 y 2$j, E x = − x
 x 2  2  2 y 3 1 
 4 2 8
and E y = − 2 y 2
= +   +    =  +  = unit
 2  0  3  0   2 3  3
As we know that,  
r2 8 8 8
V (r2 ) − V (r1) = − ∫r
1
E ⋅ dr ⇒ V (2,1) = + V (0, 0) = + 0 = unit
3 3 3

CHECK POINT 10.3


1. Three particles each of mass m are kept at vertices of an 6. A uniform solid sphere of mass m and radius r is
equilateral triangle of side L. The gravitational field at surrounded symmetrically by a uniform thin spherical shell
centre due to these particles is of radius 2r and mass m.
3GM 9 GM 12GM
(a) zero (b) (c) (d) (a) The gravitational field at a distance of1.5r from the centre is
L2 L2 3 L2
2 Gm
.
2. At what height, the gravitational field reduces by 75% the 9 r2
gravitational field at the surface of earth? (where, R is the (b) The gravitational field at a distance of 2.5r from the
radius of earth) 8 Gm
(a) R (b) 2R (c) 3R (d) 4R centre is .
25 r 2
3. Gravitational field due to a solid sphere (c) The gravitational field at a distance of 1.5r from the centre
(a) remains constant throughout the sphere is zero.
(b) increases inside the sphere and decreases outside the (d) The gravitational field between the sphere and spherical
sphere shell is uniform.
(c) increases throughout with distance from the centre
7. Consider three concentric shells of masses M1 , M2 and M3
(d) decreases throughout with distance from the centre
having radii a, b and c respectively are situated as shown in
4. Which one of the following graphs represents correctly the figure.
variation of the gravitational field (E) with the distance (r) M3
from the centre of a spherical shell of mass M and radius R? M2
E E M1
x P
(a) (b) y Q

r=R r r=R r
E E
Gravitational field at a point located at Q and P is
G(M1 + M 2) G(M1 + M 2)
(c) (d) (a) ,
y2 y2
G(M1 + M 2 + M 3) G(M1 + M 2)
(b) ,
r=R r r=R r y2 x2
G(M1 + M 2 + M 3) GM1
5. A mass m is placed inside a hollow sphere of mass M as (c) , 2
shown in figure. The gravitational force on mass m is a2 a
G(M1 + M 2 + M 3) GM 2
(d) ,
c2 b2
r m
8. For a uniform ring of mass M and radius R at its centre,
(a) field and potential both are zero
R GM
(b) field is zero but potential is
R
GMm GMm GMm (c) field is zero but potential is − GM / R
(a) (b) (c) (d) zero GM GM
R2 r2 (R − r)2 (d) magnitude of field is 2 and potential −
R R
Gravitation 451

9. The diagram showing the variation of gravitational potential G Gm


(a) V = − (m + M) (b) V = −
of earth with distance from the centre of earth is d d
GM G
V V (c) V = − (d) V = − ( m + M)2
d d
(a) O R (b) O R
13. A particle is kept on the surface of a uniform sphere of mass
r r 100 kg and radius 10 cm. Find the work to be done per unit
mass against the gravitational force between them, to take
the particle far away from the sphere (you may take
V V h = 6.67 × 10 −11 N-m2kg −2 ) .
(a)13.34 × 10−10 J (b) 3.33 × 10−10 J
R R −9
(c) O
r
(d) O
r
(c) 6.67 × 10 J (d) 6.67 × 10− 8 J
14. By which curve will be variation of gravitational potential of
a hollow sphere of radius R with distance be depicted?
V V
10. If V is the gravitational potential on the surface of the earth,
then what is its value at the centre of the earth?
3 2 R
(a) 2V (b) 3 V (c) V (d) V (a) O (b) O
2 3 R r
r
11. A thin rod of length L is bent to form a semi-circle. The
mass of rod is M. What will be the gravitational potential at
the centre of the circle? V V
GM GM
(a) − (b) −
L 2πL R R
(c) O (d) O
πGM πGM r r
(c) − (d) −
2L L
12. Two particles of masses m and M are placed a distance d
apart. The gravitational potential at the position, where the
gravitational field due to them is zero is V. Then,

GRAVITATIONAL POTENTIAL ENERGY


Gravitational potential energy of a body at a point is Gravitational potential energy
defined as the amount of work done in bringing the given of a two particles system
body from infinity to that point against the gravitational
force. The change in potential energy (dU ) of a system The gravitational potential energy of two particles of
corresponding to a conservative internal force (F) is given masses m1 and m 2 separated by a distance r is given by
r
by dU = − F ⋅ dr m1 m2
f rf
or ∫i dU = − ∫r i
F ⋅ dr Fig. 10.32 System of two point masses
separated by a distance r
rf
or U f − Ui = − ∫r i
F ⋅ dr U=−
Gm1m 2
r
We generally choose the reference point at infinity and
assume potential energy to be zero there, i.e. if we take This is actually the negative of work done in bringing
ri = ∞ (infinite) and Ui = 0, then expression can be written these masses from infinity to a distance r apart by the
as gravitational forces between them.
 Gm1 
r Gravitational potential energy =  −  × m2
Uf = U = − ∫ ∞ F ⋅ dr = − W  r 

Hence, potential energy of a body or system is negative of ∴ Gravitational potential energy =


the work done by the conservative forces in bringing it Gravitational potential × Mass of the body
from infinity to the particular position. Gravitational potential energy is a scalar quantity. Its SI
unit is joule and dimensional formula is [ML2 T −2 ].
452 OBJECTIVE Physics Vol. 1

Example 10.44 Two point masses 1 kg and 4 kg are C


4m
separated by a distance of 10 cm. Find gravitational
potential energy of the two point masses. a a
Sol. Given, m1 = 1 kg, m 2 = 4 kg, r = 10 cm = 10 × 10−2 m
∴ Gravitational potential energy, A B
−11 m a 2m
− Gm1m 2 − 6.67 × 10 × 1 × 4
U = =
r 10 × 10−2 i.e. U = U AB + U BC + U CA
= − 26.68 × 10 −10
J − G (m )(2m ) − G (2m )(4m ) − G (4m )(m )
= + +
a a a
Gravitational potential energy for a −14Gm 2
=
system of more than two particles a
(ii) When the side a is changed to 2a, the potential energy
The gravitational potential energy for a system of particles
−14Gm 2
(say m1, m 2, m 3 and m 4 ) is given by U′ =
2a
m m m m m m m m m m m m  The work done on the system = The work done against
U = − G  4 3 + 4 2 + 4 1 + 3 2 + 3 1 + 2 1
 r43 r42 r41 r32 r31 r21  the gravitational force
n (n − 1) = The work done by the external force
Thus, for a n particle system there are pairs and
2 14Gm 2
=U′ −U =
the potential energy is calculated for each pair and added 2a
to get the total potential energy of the system.
Example 10.45 Three masses of 1 kg, 2 kg and 3 kg are
Gravitational potential energy of
placed at the vertices of an equilateral triangle of side 1 m. an object in different conditions
Find the gravitational potential energy of this system. Object placed at earth’s surface Let an object of mass m
(Take, G = 6.67 × 10 −11 N -m 2 kg −2 ) is placed at a point in the gravitational field of the earth of
Sol. Gravitational potential energy of the three particles system mass M. Then, gravitational potential energy of an object
m m m m m m placed at earth’s surface,
U = − G  3 2 + 3 1 + 2 1
 r32 r31 r21  GMm
U=−
Here, r32 = r31 = r21 = 1 m, R
m1 = 1 kg, m 2 = 2 kg and m 3 = 3 kg Note
(i) If the body of mass m is moved from a point at distance ra to a point at
1 kg
distance rb (ra > rb ), then change in potential energy,
1 1
∆U = GMm  − 
a
r rb

(ii) Gravitational potential energy at the centre of earth relative to


2 kg 3 kg infinity is
− 3  GMm 
Substituting in above, we get U centre =  
2  R 
 3 × 2 3 × 1 2 × 1
U = − (6.67 × 10−11)  + +  Object placed at a point, above the earth’s surface Let
 1 1 1 
an object of mass m is placed at height h above the surface
or U = −7337
. × 10−10 J of earth, then change in gravitational potential energy
Example 10.46 Three particles of masses m, 2m and 4m are when the object is taken from the centre of the earth to
placed at the corners of an equilateral triangle of side a. the height h is
Calculate mgh
(i) the potential energy of the system. ∆U =
(ii) the work done on the system, if the side of the triangle is
1 + h /R
changed from a to 2a. Assume the potential energy to be ∆U is positive, it means the potential energy of a body
zero when the separation is infinity. increases as its height above earth surface increases.
Sol. (i) The potential energy of the system = Sum of the For h < < R, ∆U ≈ mgh
potential energies of all the three possible distinct pairs.
Gravitation 453

Thus, what we observe the mgh is actually the difference Example 10.49 Find the change in the gravitational potential
in potential energy (not the absolute potential energy), that energy when a body of mass m is raised to a height nR above
too for h < < R . the surface of the earth. (Here, R is the radius of the earth)
Note Gravitational potential energy of a body at height x from the earth’s Sol. Gravitational potential energy of mass m at earth’s surface,
surface is GMm
Ue = −
− mgR R
Ux =
1 + x /R Gravitational potential energy of same mass at a height nR
Example 10.47 The mass of the earth is 6 × 10 24 kg. The from the earth’s surface,
constant of gravitation G = 6.67 × 10 −11 N-m 2 kg − 2 . The Uh = −
GMm
=−
GMm
potential energy of the earth and moon system is (R + nR ) R (n + 1)
−779
. × 10 28 J . Determine the mean distance between earth Thus, magnitude of the change in gravitational potential
and moon. (Mass of moon is 7.4 × 10 22 kg) energy,
Sol. The mass of moon, m = 7.4 × 1022 kg ∆U = U h − U e
Potential energy of the earth and moon system, GMm  1 
= 1 − 
− GMm R  (n + 1)
U =
r  n  GMm
− GMm − 6.67 × 10−11 × 6 × 1024 × 7.4 × 1022 = 
⇒ r= =  n + 1 R
U − 7.79 × 1028
 n 
⇒ r = 3.8 × 108 m =  mgR (Q GM = gR 2 )
 n + 1
Example 10.48 A body of mass m is raised to a height 10R
from the surface of the earth, where R is the radius of the Example 10.50 An object is dropped from height h = 2R on
earth. Find the increase in potential energy. (G = universal the surface of earth. Find the speed with which it will
constant of gravitation, M = mass of the earth and collide with ground by neglecting effect of air. (where, R is
g = acceleration due to gravity) radius of earth and M is mass of earth.)
− GMm GMm
Sol. Potential energy at the earth’s surface = Sol. The initial potential energy of object, U i = −
R 3R
Potential energy at a height h above the earth’s surface GMm
Final potential energy, U f = −
− GMm R
=
(R + h ) By law of conservation of energy, ∆KE = − ∆PE
− GMm  GMm  ⇒
1
mv 2 = − (U f − U i ) = U i − U f
∴ Change in potential energy = − − 
(R + h )  R  2
1 GMm GMm
=
GMmh ⇒ mv 2 = − +
R (R + h ) 2 3R R
h = 10R 1 2 2GM
Substituting, ⇒ v =
GMm × 10R 2 3R
∆U = 4GM GM
R (R + 10 R ) ⇒ v= =2
3R 3R
10 GMm
=
11R
OBJECTIVE Physics Vol. 1

CHECK POINT 10.4


1. Consider the two identical particles d
U U
(a) U (b) Ur (c) (d)
shown in the given figure. They are r 2r
released from rest and may move m m
5. Find the potential energy of four-particles, each of mass
towards each other under the influence of mutual
1 kg placed at the four vertices of a square of side length 1
gravitational force. The gravitational potential energy of the
m.
two particle system
(a) + 4.0 G (b) − 7.5 G (c) − 5.4 G (d) + 6.3 G
(a) zero
(b) constant (≠ 0) 6. Energy required in moving a body of mass m from a
(c) decreases as the separation decreases distance 2R to 3R from centre of earth of mass M is
(d) increases as the separation decreases GMm GMm GMm GMm
(a) (b) (c) (d)
12R 2 3R 2 8R 6R
2. The magnitude of gravitational potential energy of a body at
a distance r from the centre of earth is u. Its weight at a 7. When a body is lifted from surface of earth at a height equal
distance 2r from the centre of earth is to radius of earth, then the change in its potential energy is
u u u 4r (a) mgR (b) 2mgR
(a) (b) (c) (d)
r 4r 2r u 1
(c) mgR (d) 4mgR
2
3. A body of mass m is kept at a small height h above
the ground. If the radius of the earth is R and its mass is M, 8. If an object of mass m is taken from the surface of earth
the potential energy of the body and earth system (with (radius R) to a height 5R, then the work done is
h = ∞ being the reference position) is (a) 2mgR (b) mgR (c)
5
mgR (d)
3
mgR
GMm − GMm 6 2
(a) + mgh (b) + mgh
R R 9. If a body of mass m has to be taken from the surface of the
GMm GMm
(c) − mgh (d) − − mgh earth to a height h = 4 R , then the amount of energy
R R required is (R = radius of the earth)
4. The gravitational potential energy of a body of mass m at a mgR 4 mgR mgR
(a) mgR (b) (c) (d)
distance r from the centre of the earth is U. What is the 5 5 12
weight of the body at this distance?

BINDING ENERGY
The minimum energy required to take a particle to an GMm
E = K +U = 0 −
infinite distance from the earth is called the binding R
energy of the earth particle system. GMm
or E =−
R
Binding energy of a particle GMm
∴ Binding energy, BE = |E | =
on the surface of the earth R
Suppose the mass m is placed on the surface of earth. Due to binding energy, the particle is attached to the earth. If
The radius of the earth is R and its mass is M. Then, this much energy is supplied to the particle in any form
the kinetic energy of the particle, K = 0 and potential (normally kinetic) the particle no longer remains bound to the
GMm earth. It goes out of the gravitational field of earth.
energy of the particle is U = − .
R Example 10.51 Find the binding energy of a particle of mass m
m on the surface of earth in term of g. (Here, R = radius of earth
and g = acceleration due to gravity)
GMm
R Sol. As, binding energy = − E =
M R
GM
But g= ⇒ GM = gR 2
Fig. 10.33 A particle on the surface of the earth R2
gmR 2
Therefore, the total mechanical energy of the particle, ∴ Binding energy = |E | = = mgR
R
Gravitation 455

escape velocity, if the body is projected in the


Escape velocity direction in which the earth is moving.
The minimum velocity with which a body must be projected (ii) As the escape velocity depends on the mass and
vertically upwards in order that it may just escape the radius of the planet from the surface of which the
gravitational field of the earth (i.e. v = 0 ) is called escape body is projected, so value of escape velocity is
velocity. different for different planets.
v=0
(iii) A planet will have atmosphere, if the root mean
m ve square velocity of its atmospheric molecules is less
than the escape velocity for the given planet. That is
M
why moon has no atmosphere (v e = 2.3 kms − 1)
R O while jupiter has a thick atmosphere (v e = 60 kms − 1).
Earth Even the lightest hydrogen cannot escape from its
surface.
Fig. 10.34 Escape velocity of a body
(iv) If a body falls freely from infinite height, then it will
A body projected from the surface of the earth with speed reach the surface of the earth with a speed
v e , such that v f = v = 0. approximately equal to the average escape speed
As we have discussed that the binding energy of a particle v e = 11.2 kms −1.
on the surface of earth kept at rest is GMm /R . If this (v) In calculating the escape speed, the air resistance
much energy in the form of kinetic energy is supplied to and the gravitational effect due to other celestial
the particle, it leaves the gravitational field of the earth. bodies are neglected.
So, if v e is the escape velocity of the particle, then from
(vi) For earth to become black hole, the escape speed
conservation of mechanical energy, we have
must become equal to or more than speed of light,
1 GMm
mv e2 = 2GM
2 R i.e. ≥ c. In that case, radius of the earth will
R
2GM become that of the size of a grape.
or ve = ...(i)
R Note If the escape velocity of a body is equal to the velocity of light,
GM then from such bodies nothing can escape, not even light. Such
As, g= bodies are called black hole.
R2
Example 10.52 Calculate the escape velocity from the
⇒ v e = 2gR surface of a planet of mass 14.8 × 10 22 kg. It is given that
radius of the planet is 3.48 × 10 6 m.
Substituting the value of g (= 9.8 ms −2 ) and R (= 6.4 × 10 6 m), Sol. Given, mass of planet,
we get M p = 14.8 × 1022 kg
v e ≈ 11.2 kms −1
and radius of the planet,
Thus, the minimum velocity needed to take a particle
R p = 3.48 × 106 m
infinitely away from the earth is called the escape
velocity. On the surface of earth, its value is 11.2 kms −1. Escape velocity from the surface of planet,
2GM p
2G 4 ve =
From Eq. (i), ve = × πR 3 × ρ Rp
R 3
where, ρ is the mean density of the earth. Substituting the values, we get

8 2 × 6.67 × 10−11 × 14.8 × 1022


Escape velocity, v e = R πGρ ve =
3 3.48 × 106

Important points related to escape velocity = 2.4 × 103 ms−1 or 2.4 kms−1

(i) The escape velocity does not depend on angle of Example 10.53 The ratio of the masses and radii of two
projection from the earth’s surface. But as the earth planets are 4 : 6 and 8 : 18. What is the ratio of the escape
rotates about its axis, so it becomes easier to attain velocity at their surfaces?
456 OBJECTIVE Physics Vol. 1

M1 4 2 R 8 4
Sol. Given, = = and 1 =
M2 6 3
=
R 2 18 9
Escape energy
Minimum energy given to a particle in form of kinetic
2GM
As we know, escape velocity, v = energy, so that it can escape from earth’s gravitational
R field, is called escape energy.
v1 M1 R 2
∴ = × vf = 0
v2 M 2 R1
ve
2 9 3
= × =
3 4 2 Earth
Hence, v1 : v 2 = 3 : 2 O R

Example 10.54 Jupiter has a mass 318 times that of earth,


and its radius is 11.2 times the earth’s radius. Estimate the Fig. 10.35 Particle projected from earth surface
escape velocity of a body from jupiter’s surface, given that
the escape velocity from the earth’s surface is 11.2 kms −1.
GMm
Sol. Escape velocity from the earth’s surface is Escape energy =
R
2GM
ve = = 11.2 kms − 1
R If escape velocity is v e , then
Escape velocity from jupiter’s surface will be 1
2GM ′ Escape energy = mv e2
ve = 2
R′
Given, M ′ = 318 M and R ′ = 11. 2R Example 10.56 A very small groove is made in the earth,
and a particle of mass m 0 is placed at R/2 distance from its
2G (318 M ) 2GM 318 centre as shown in figure. Find the escape energy of the
∴ v e′ = = ×
11. 2R R 11. 2 particle from that place.
Me, R
318 318
= v e′ × = 112
. ×
11. 2 11. 2
R/2
⇒ v e′ = 59.7 kms − 1 m0

Example 10.55 At what temperature, hydrogen molecules


will escape from the earth’s surface? (Take, radius of earth
R e = 6.4 × 10 6 m, mass of hydrogen molecule,
m = 0.34 × 10 − 26 kg, Boltzmann constant Sol. Suppose we projected the particle with speed v e , so that it
k = 1.38 × 10 − 23 JK − 1, acceleration due to gravity just reaches at r → ∞.
3kT Applying energy conservation, Ki + U i = K f + U f
= 9.8ms − 2 and rms speed of gas as v rms = )  GM   R   
2
m 1
⇒ m 0v e2 + m 0 − e
3 
3R 2
−    = 0
3kT 2  2R   2  
Sol. The root-mean-square-velocity of gas is v rms = …(i) 
m
1 11Gm 0M e
Escape velocity of gas molecules is v e = 2gR e …(ii) ⇒ Escape energy = m 0v e2 =
2 8R
As the root-mean-square velocity of gas molecules must be
equal to the escape velocity. Example 10.57 A body is projected upwards with a velocity
of 4 × 11.2 km s − 1 from the surface of earth. What will be
3kT
∴ From Eqs. (i) and (ii), = 2gR e the velocity of the body when it escapes from the
m gravitational pull of earth?
2gR em
⇒ T = Sol. Initial kinetic energy of the body =
1
mv 2
3k 2
2 × 9.8 × 6.4 × 106 × 0.34 × 10−26 Here, v = 4 × 11.2 kms−1
⇒ T = −23
≈104 K
3 (1.38 × 10 ) 1 1
4 So, kinetic energy, KE = × m (4 × 11.2)2 = 16 × mv e2
Therefore, 10 K is the temperature at which hydrogen 2 2
molecules will escape from earth’s surface.
Gravitation 457

1
As, mv e2 energy is used up in coming out from the gravitational (ii) If v < v e , body will attain a certain maximum height
2 and then may either move in an orbit around the
1 planet or may fall down back to the planet.
pull of the earth, so final kinetic energy should be 15 × mv e2.
2
(iii) If v > v e , body will move in interplanetary or
1 1 2
Hence, mv ′ = 15 × mv e
2
interstellar space with velocity v 2 − v e2 .
2 2
∴ v′ 2
= 15v e2 ⇒ v ′ = 15v e = 15 × 11.2 kms−1
Example 10.59 A particle is projected from the surface of
Example 10.58 The earth is assumed to be a sphere of earth with an initial speed of 4 km/s. Find the maximum
radius R. A platform is arranged at a height R from the height attained by the particle. (Take, radius of earth
surface of the earth. The escape velocity of a body from this = 6400 km and g = 9.8 m/s 2 )
platform is f v e , where v e is its escape velocity from the v2
Sol. The maximum height attained by the particle is, h =
surface of the earth. Find the value of f. v2
2g −
Sol. For a platform at a height h, R
escape energy = binding energy of platform Substituting the values, we get
(4 × 103 )2

1 2
mv e′ =
GMm
⇒ v e′ =
2 GM
=
2 GM
( Qh = R ) h= = 9.35 × 105m or h ≈ 935 km
2 R +h R +h 2R (4.0 × 103 )2
2 × 9.8 −
6.4 × 106
2GM
But at surface of earth, v e =
R Example 10.60 A particle is projected vertically upwards
As given, v e′ = f v e from the surface of the earth (radius R e ) with a speed equal
to one-fourth of escape velocity. What is the maximum
2 GM 2 GM 1 f2 1 height attained by it from the surface of the earth?
Hence, =f or = ⇒f =
2R R 2R R 2 Sol. From conservation of mechanical energy,
1 GMm GMm
mv 2 = − ...(i)
Maximum height attained by a particle 2 Re R
Suppose a particle of mass m is projected vertically where, R = maximum height (distance) from centre of the earth.
upwards with a speed v and we want to find the maximum 1 1 2GM
height h attained by the particle. Also, v= ve =
4 4 Re
v
Substituting values in Eq (i), we get
1 1 2GM GMm GMm
⇒ m× × = −
2 16 Re Re R
R 16 R
O ⇒ R= Re ⇒ h = R − Re = e
15 15

Example 10.61 With what velocity must a body be thrown


Fig. 10.36 A particle projected vertically upwards from from earth’s surface, so that it may reach a maximum height
the surface of the earth of 4R e above the earth’s surface? (Take, radius of the earth,
R e = 6400 km, g = 9.8 ms −2 )
Then, we apply conservation of mechanical energy, i.e.
Sol. From conservation of mechanical energy,
decrease in kinetic energy = increase in gravitational
1 GMm 0 GMm 0
potential energy of particle m 0v 2 − =0−
2 Re (R e + 4R e )
1 1 mgh
∴ mv 2 = ∆U or mv 2 = 1 GMm 0 GMm 0
2 2 1 + h /R ⇒ m 0v 2 = − +
2 5R e Re
Solving this, we get 1 4 GMm 0
⇒ m0 v2 =
v2 2 5 Re
h=
2 g − v 2 /R 8 GM 8 gR e2
⇒ v2 = =
5 Re 5 Re
From this, we can obtain that 8
⇒ v = × 9.8 × 6400 × 103 = 108
2

(i) If v = v e or v 2 = v e2 = 2 gR, then h = ∞. 5


∴ v = 10 kms −1
OBJECTIVE Physics Vol. 1

CHECK POINT 10.5


1. In a gravitational field, if a body is bound with earth, then 8. There are two planets and the ratio of radius of the two
total mechanical energy is planets is K but ratio of acceleration due to gravity of both
(a) positive planets is g. What will be the ratio of their escape velocities?
(b) zero (a) (Kg)1 / 2 (b) (Kg)−1 / 2
(c) negative (c) (Kg)2 (d) (Kg)−2
(d) may be positive, negative or zero
9. The escape velocity from earth is v e . A body is projected with
2. The binding energy of an object of mass m placed on the
surface of the earth is (R = radius of earth, g = acceleration velocity 2v e . With what constant velocity will it move in the
due to gravity) inter planetary space?
(a) mgR/ 2 (b) mgR (a) ve (b) 2ve
mgR mgR (c) 3ve (d) 5 ve
(c) (d)
4 8
10. A particle is projected vertically upwards from the surface of
3. When escape velocity is given to a particle on surface of earth (radius R) with a kinetic energy equal to half of the
earth, its total energy is (Take, M = mass of earth, minimum value needed for it to escape. The height to
m = mass of particle and R = radius of earth) which it rises above the surface of earth is
(a) zero (b) greater than zero (a) R (b) 2 R
(c) less than zero (d) − GMm / 2R (c) 3 R (d) 4 R
4. The escape velocity for a body of mass 1 kg from the earth 11. The kinetic energy required to project a body of mass m
surface is 11.2 kms −1 . The escape velocity for a body of mass from the earth surface (radius R) to infinity is
mgR
100 kg would be (a) (b) 2mgR
−1 −1 2
(a) 11.2 × 10 kms
2
(b) 112 kms mgR
(c) mgR (d)
(c) 11.2 kms −1 (d) 11.2 × 10−2 kms −1 2
5. The escape velocity of a body projected vertically upward 12. A body is projected upwards with a velocity of 3 × 11.2 km s −1
−1
from the earth’s surface is 11.2 kms . If the body is from the surface of earth. What will be the velocity of the
projected in a direction making 30° angle to the vertical, its body when it escapes from the gravitational pull of earth?
escape velocity in this case will be (a)11.2 kms−1 (b) 2 × 11.2 km s−1
(a) 11.2 kms −1 (b)
11.2
kms −1 (c) 3 × 11.2 kms−1 (d) 8 × 11.2 kms−1
2
13. With what velocity should a particle be projected, so that its
3 11.2
(c) 11.2 × kms −1 (d) kms −1 maximum height attained becomes equal to radius of
2 3 earth?
1/ 2 1/ 2
6. Escape velocity on earth is 11.2 kms −1 , what would be the (a) 
GM 
 (b) 
8 GM 

escape velocity on a planet whose mass is 1000 times and  R   R 
radius is 10 times that of earth? 1/ 2 1/ 2
(d) 
4 GM 
(c) 
2GM 
(a) 112 kms −1 (b) 11.2 kms −1  
 R   R 
(c) 1.12 kms −1 (d) 3.7 kms −1
14. A body is projected vertically upwards from the surface of a
7. The ratio of the radius of the earth to that of the moon is planet of radius R with a velocity equal to 1/3rd the escape
10. The ratio of the acceleration due to gravity on the earth velocity for the planet. The maximum height attained by
to that on the moon is 6. The ratio of the escape velocity the body is
from the earth’s surface to that from the moon is (a) R/2 (b) R/3
(a) 4 (b) 6 (c) R/5 (d) R/9
(c) 12 (d) None of these
Gravitation 459

MOTION OF SATELLITES Substituting the values of g and R, we get


Just as the planets revolve around the sun, in the same v o = 7.9 kms −1 ≈ 8 kms −1
way few celestial bodies revolve around these planets. Note
(i) Orbital velocity of a satellite is independent of mass of the
These bodies are called satellites. e.g. Moon is a natural satellite.
satellite of earth. The paths of these satellites are elliptical (ii) It decreases with increase in the radius of the orbit and with
with the centre of earth at a focus. increase in the height of the satellite.
(iii) It depends on the mass and radius of the planet about which the satellite
The difference in major and minor axes is not large, so we revolves.
can assume circular orbits. (iv) The angular momentum of a satellite of mass m moving with
velocity v 0 in an orbit of radius r ( = R + h) is given by L = GMm2r .
Orbital velocity of satellites
The velocity with which a satellite moves in its closed Example 10.62 A satellite circled around the earth at a
orbit around the earth is known as orbital velocity. distance of 3400 km. Determine its orbital velocity, if the
radius of the earth is 6400 km and g = 9.8 ms − 2 .
Sol. Given, radius of earth, R = 6400 km = 64 × 105m
vo
M Distance of satellite from earth, h = 3400 km = 34 × 105m
h r m
R Satellite
∴ Orbital velocity of an artificial satellite can be given as
Earth g 9.8
vo = R = 64 × 105
R +h (64 + 34) × 105
Fig. 10.37 Motion of a satellite
⇒ v o = 6400 ms − 1
The necessary centripetal force to the satellite is provided
by the gravitational force exerted by the earth on the Relation between orbital velocity and
mv o2 GMm escape velocity
satellite. Thus, = 2
r r The escape velocity of a body from the earth’s surface,
v e = 2gR
GM
∴ vo = The orbital velocity of a satellite revolving close to earth’s
r
surface,
1 v o = gR
or vo ∝
r
ve 2gR
Here r = R + h, where R is radius of earth and r is the ∴ = = 2
orbital radius. vo gR
GM
∴ vo = ⇒ ve = 2 v o …(i)
R +h
So, Eq. (i) represents that the escape velocity of a body
Further, we know that, GM = gR 2
from the earth’s surface is 2 times its orbital velocity in a
gR 2 circular orbit just above the earth’s surface.
∴ vo =
R +h
Example 10.63 A satellite is launched into a circular orbit
close to the earth’s surface. What additional velocity has
g
⇒ vo = R now to be imparted to the satellite in the orbit to overcome
(R + h ) the gravitational pull?

Hence, the orbital speed (v o ) of the satellite decreases as Sol. Since, orbital velocity near earth’s surface, v o = gR e
the orbital radius (r ) of the satellite increases. Further, the Escape velocity, v e = 2gR e = 1.414 gR e
orbital speed of a satellite close to the earth’s surface
Therefore, additional velocity required
(r = R + h ≈ R ) is,
= v e − v o = (1.414 − 1) gR e
GM = 0.414 9.8 × 6400 × 103
vo = = gR
R
= 3.278 × 103 ms − 1 = 3.278 kms − 1
460 OBJECTIVE Physics Vol. 1

Time period of revolution of satellite Sol. Given, height of satellite from the earth, h = 1000 km = 106 m,
The time taken by a satellite to complete one revolution R = 6.38 × 103 km = 6.38 × 106 m
around the earth, is known as time period of revolution of ⇒ h + R = 738
. × 106m, M = 6 × 1024 kg
satellite.
GM
The period of revolution (T ) is given by ∴ Orbital velocity, v o =
R +h
2πr 2πr
T = or T =
vo GM 6.67 × 10− 11 × 6 × 1024
=
r . × 106
738
= 7364 ms − 1
3
r 2π (R + h )
or T = 2π ∴ Period of revolution, T =
GM vo
2π × 738
. × 106
r3 =
or T = 2π 7364
gR 2 ⇒ T ≈ 6297 s ≈ 1.749 h

(R + h ) 3 Height of satellite
or T = 2π 2
(Q GM = gR 2 and r = R + h)
gR As it is known that the time period of satellite,

2πr 2π (R + h ) r3 ( R + h) 3
Also, T = = T = 2π = 2π …(i)
GM vo GM gR 2
r By squaring both sides of Eq. (i), we get
From this expression of T, we can make the following (R + h ) 3
conclusions T 2 = 4π 2
gR 2
(i) T ∝ r 3/ 2
or T 2
∝r 3
gR 2T 2
which is also the Kepler’s third law. ⇒ = (R + h ) 3
4π 2
(ii) Time period of a satellite very close to earth’s
surface (r ≈ R ) is,  T 2 gR 2 
1/ 3

R ⇒ h=  −R
T = 2π  4π 2 
g
Substituting the values, we get By knowing the value of time period, the height of the
T ≈ 84.6 min ≈ 1.4 h satellite from the earth’s surface can be calculated.
(iii) Suppose the height of a satellite is such that the time Example 10.65 A satellite forms a circle around the earth in
period of the satellite is 24 h and it moves in the same 90 min. Determine the height of the satellite above the
sense as the earth. The satellite will always be earth’s surface.
overhead a particular place on the equator. As seen Sol. Given, T = 90 min = 5400 s,
from the earth, this satellite will appear to be stationary.
R = 6400 km = 6.4 × 106 m
Such a satellite is called a geo-stationary satellite.
Putting T = 24 h in the expression of T, the radius (R + h )3
Q Time period, T = 2π
of geo-stationary satellite comes out to be gR 2
r = 4.2 × 10 4 km. The height above the surface
1/ 3
of earth is about 3.6 × 10 4 km.  gR 2T 2 
⇒ R +h =  2 
 4π 
Example 10.64 A satellite revolves around the earth at a
height of 1000 km. The radius of the earth is 6.38 × 10 3 km. ⇒ R + h = 666813
. km
Mass of the earth is 6 × 10 24 kg and ⇒ h = 666813
. − R = 6668.13 − 6400
G = 6.67 × 10 − 11 N-m 2 kg − 2 . Determine its orbital ⇒ . km −~ 268 km
h = 26813
velocity and period of revolution.
Gravitation 461

Total energy of satellite Binding energy of a satellite


Assume that a satellite of mass m moving around the earth The energy required by a satellite to leave its orbit around
with velocity v o in an orbit of radius r due to gravitational the earth and escape to infinity, is known as binding
pull of the earth. energy of satellite.
The potential energy of the satellite, From above, we know that, the total energy of the satellite
GMm is − GMm /2r . So, if the satellite has to leave the earth’s
U=− orbit and escape to infinity, then an extra energy equal to
r
The kinetic energy of the satellite, + GMm /2r is to be supplied to it, so that its total energy
1 1  GM  becomes zero.
K = m v o2 = m  
2 2  r  GMm
∴ Binding energy of a satellite =
1 GMm 2r
or K =
2 r
Note The total energy of the satellite is negative only when it is
GMm
The total energy, E = K + U = − bound to the earth. It means that its orbit is an ellipse or circle. It
2r is positive when satellite is not bound to any star or its
equivalent. In this case, the path covered by satellite will be
GMm
⇒ E =− hyperbola.
2r
This energy is constant and negative, i.e. the system is Example 10.66 If a spaceship orbits the earth at a height of
closed. The farther the satellite from the earth, the greater 500 km from its surface, then determine its (i) kinetic
is its total energy. energy, (ii) potential energy, (iii) total energy and (iv) binding
energy. (Take, mass of the satellite = 300 kg, mass of the
Variation of energy with distance can be represented by earth = 6 × 10 24 kg, radius of the earth = 6.4 × 10 6 m,
graphs as shown in figure.
G = 6.67 × 10 −11 N-m 2 kg −2 ). Will your answer alter, if the
Energy earth were to shrink suddenly to half its size?
K Sol. Given, height, h = 500 km = 500 × 103 m
O r Mass of spaceship, m = 300 kg
E=K+U Mass of the earth, M = 6 × 1024 kg
E
U
Radius of the earth, R = 6.4 × 106m

Fig.10.38 Variation of energy of a satellite and G = 6.67 × 10−11N-m 2kg −2


1
(i) Kinetic energy = mv 2
Important points to be remembered 2
1 GM  GM 
(i) The total mechanical energy of a satellite is = m⋅ Qv = 
negative, if the reference level of zero potential 2 R +h  R +h
energy is taken when separation is infinity. 1 300 × 6.67 × 10−11 × 6 × 1024
= ×
U 2 6.4 × 106 + 500 × 103
(ii) For a satellite K = − E, E =
2 = 8.7 × 109 J
and U = − 2K GMm
(ii) Potential energy = −
(iii) If we consider elliptical path of a satellite, then (R + h )
maximum and minimum energies are as shown in 6.67 × 10−11 × 6 × 1024 × 300
=−
figure. 6.4 × 106 + 500 × 103
Satellite = − 17.4 × 109 J
Perigee Apogee (iii) Total energy = KE + PE
KE = max KE = min
PE = min Focus PE = max = 8.7 × 109 − 17.4 × 109
= − 8.7 × 109 J
a
r max (iv) Binding energy = − (Total energy) = 8.7 × 109 J
r min
No, there is no change in answer, because r = R + h will
Fig. 10.39 Motion of satellite remain constant.
462 OBJECTIVE Physics Vol. 1

and v e = escape velocity at A.


ARTIFICIAL SATELLITE
The figure given below shows the path of an
Those man-made objects which revolves around the earth artificial satellite launced from A, with different
in a fixed orbit are known as artificial satellites. possible velocities discussed in the above cases.
These satellites are launched from the earth. Artificial
A vo
satellites are used for telecommunication, weather forecast
and other applications. These satellites are of two types :
Geo-stationary satellite and polar satellite. h

Launching of an artificial R
satellite around the earth
Let a satellite be projected from point A with velocity v in the
direction AB. For different values of v, the paths are
different.
Fig. 10.42 Path of an artificial satellite
A B
v
Note
(i) From case (i) to (iv), total energy of the satellite is negative. Hence,
these are the closed orbits. For case (v), total energy is zero and for
case (vi), total energy is positive. In these two cases, orbits are open.
O (ii) If v is not very large the elliptical orbit will intersect the earth and
R Earth the satellite will fall back to earth.

Fig. 10.40 A satellite projected from A Weightlessness in a satellite


Here, are the possible cases A satellite does not produce its own gravity but moves
around the earth in a circular orbit under the action of
(i) If v = 0, path is a straight line from A to O.
gravity. The acceleration of satellite is GM/r 2 towards the
(ii) If 0 < v < v o , path is an ellipse with centre O of the centre of the earth as shown in figure.
earth as a focus.
(iii) If v = v o , path is a circle with O as the centre. Satellite

(iv) If v o < v < v e , path is again an ellipse with O as a h m Rr


r
focus. R
A A Earth GM v
0 < v < vo vo < v < ve
r2
O

Fig. 10.43 Weightlessness in a satellite


O
If a body of mass m placed on a surface inside a satellite
moving around the earth.
Fig. 10.41 Elliptical orbit with focus at lower Then, forces on the body are as follows
side and upper side GMm
The gravitational pull of the earth = 2
(v) If v = v e , satellite escapes from the gravitational pull r
of the earth and path is a parabola. The reaction by the surface = R r
(vi) If v > v e , satellite again escapes from the GMm
By Newton’s law, 2 − R r = ma
gravitational pull of earth but now the path is a r
hyperbola. GMm  GM 
− Rr = m  2 
 GM  r 2 r 
Here, v o = orbital speed   at A
 r  ⇒ Rr = 0
Gravitation 463

Thus, the surface does not exert any force on the body and Sol. In a satellite, the astronaut feels weightlessness and
hence, its apparent weight is zero. therefore, weight becomes zero but mass remains
unchanged.
Note Condition of weightlessness can be experienced only when the mass of
satellite is negligible, so that it does not produce its own gravity. e.g. Therefore, mass of astronaut can be calculated by using
Moon is a satellite of the earth but due to its own weight, it applies weight at the surface of the earth.
gravitational force of attraction on the body placed on its surface and ∴ mg = 850 N
hence, weight of the body will not be equal to zero at the surface of 850
the moon. ⇒ m=
10
Example 10.67 What will be the mass of an astronaut in a space ⇒ m = 85 kg
satellite if at earth’s surface the astronaut weighs 850 N? (Take,
g = 10 ms −2 )

CHECK POINT 10.6


1. The centripetal force on a satellite orbiting round the earth 8. If mean radius of earth is R, its angular velocity is ω and the
and the gravitational force of earth acting on the satellite acceleration due to gravity at the surface of the earth is g,
both equals to F. The net force on the satellite is then the cube of the radius of the orbit of geo-stationary
(a) zero (b) F satellite will be
(c) F 2 (d) 2 F R2 g R2ω2 R2 g Rg
(a) (b) (c) (d)
2. The orbital velocity of a body close to the earth’s surface is ω2 g ω ω2
(a) 8 kms −1 (b) 11.2 kms −1 9. For a satellite orbiting very close to earth’s surface, total
(c) 3 × 108 ms −1 (d) 2.2 × 103 kms −1 energy is
GMm GMm GMm
(a) zero (b) (c) − (d) −
3. Two satellites A and B moves round a planet P in circular R R 2R
orbits having radii 4R and R, respectively. If the velocity of
the satellite A is 3v, the velocity of the satellite B will be 10. An artificial satellite moving in a circular orbit around the
4 3 earth has total mechanical energy E0 . Its kinetic energy is
(a) 12v (b) 6v (c) v (d) v (a) −2E 0 (b) 1.5E 0 (c) 2E 0 (d) − E 0
3 2
4. The orbital velocity of an artificial satellite in a circular orbit 11. In case of an orbiting satellite, if the radius of orbit is
just above the earth’s surface is v. For a satellite orbiting at decreased,
an altitude of half the earth’s radius, the orbital velocity is (a) its kinetic energy decreases
3 3 2 2 (b) its potential energy increases
(a) v (b) v (c) v (d) v (c) Both (a) and (b) are correct
2 2 3 3
(d) Both (a) and (b) are incorrect
5. The period of a satellite in a circular orbit around a planet is
independent of 12. Two satellites A and B, ratio of masses 3 : 1 are in circular
(a) the mass of the planet orbits of radii r and 4r. Then, ratio of total mechanical
(b) the radius of the planet energy of A and B is
(c) the mass of the satellite (a) 1: 3 (b) 31
: (c) 3: 4 (d) 121
:
(d) All the three parameters (a), (b) and (c) 13. Two satellites P and Q, ratio of masses 3 : 1 are in circular
6. Which of the following quantities does not depend upon orbits of radii r and 8r. Then, ratio of total mechanical
the orbital radius of a satellite? (Take, R = radius of orbit) energy of A to B is
T T2 (a) 1: 3 (b) 31
: (c) 3: 4 (d) 24:1
(a) (b)
R R 14. What is the energy required to launch a m kg satellite from
T2 T2 earth’s surface to a circular orbit at an altitude of 7R?
(c) 2 (d) 3
R R (R = radius of the earth)
12 15 1
7. A satellite is revolving in circular orbit of radius r around (a) mgR (b) mgR (c) mgR (d) mgR
13 16 9
the earth of mass M. Time of revolution of satellite is
r5 r3 15. Two identical satellites are orbiting at distances R and 7R
(a) T ∝ (b) T ∝
GM GM from the surface of the earth, R being the radius of the
r r3 earth. The ratio of their
(c) T ∝ (d) T ∝ (a) kinetic energies is 4 (b) potential energies is 4
GM 2 GM
(c) total energies is 4 (d) All of these
3 4
Chapter Exercises
(A) Taking it together
Assorted questions of the chapter for advanced level practice

1 Reason of weightlessness in a satellite is 8 In our solar system, the inter-planetary region has
(a) zero gravity chunks of matter (much smaller in size compared to
(b) no atmosphere planets) called asteroids. They [NCERT Exemplar]
(c) zero reaction force by satellite surface (a) will not move around the sun, since they have very
(d) None of the above small masses compared to the sun
(b) will move in an irregular way because of their small
2 The acceleration due to gravity near the surface of a masses and will drift away into outer space
planet of radius R and density d is proportional to (c) will move around the sun in closed orbits but not obey
d Kepler’s laws
(a) (b) dR 2
R2 (d) will move in orbits like planets and obey Kepler’s laws
d 9 Two satellites of same mass are launched in the
(c) dR (d)
R same orbit of radius r around the earth so as to rotate
3 A satellite of the earth is revolving in a circular orbit opposite to each other. If they collide inelastically
with a uniform speed v. If gravitational force and stick together as wreckage, the total energy of
suddenly disappears, the satellite will the system just after collision is
(a) continue to move with speed v along the original orbit 2GMm GMm
(a) − (b) −
(b) move with the velocity v tangentially to the original r r
orbit GMm
(c) fall downward with increasing velocity (c) (d) zero
2r
(d) ultimately come to rest somewhere on the original orbit
10 Energy required in moving a body of mass m from a
4 A body is projected from earth’s surface to become distance 2R to 4R from centre of earth of mass M is
its satellite, its time period of revolution will not GMm GMm GMm GMm
depend upon (a) (b) (c) (d)
12R 2 3R 2 8R 4R
(a) mass of earth (b) its own mass
(c) gravitational constant (d) radius of orbit 11 Earth orbiting satellite will escape, if
(a) its speed is increased by 41%
5 A planet is revolving round the sun in an elliptical
(b) its kinetic energy is doubled
orbit, If v is the velocity of the planet when its
(c) Both (a) and (b) are correct
position vector from the sun is r, then areal velocity (d) Both (a) and (b) are incorrect
of the planet is
(a) | v × r | (b) 2| r × v | 12 What is the fractional decrease in the value of free
1 fall acceleration g for a particle when it is lifted from
(c) (r × v ) (d) None of these the surface to an elevation h ? (h << R )
2
−h −2h −3h −4h
6 The required kinetic energy of an object of mass m, (a) (b) (c) (d)
R R R R
so that it may escape, will be
1 1 13 The earth is an approximate sphere. If the interior
(a) mgR (b) mgR contained matter which is not of the same density
4 2
(c) mgR (d) 2mgR everywhere, then on the surface of the earth, the
7 A planet of mass m is in an elliptical orbit about the
acceleration due to gravity [NCERT Exemplar]
(a) will be directed towards the centre but not the same
sun with an orbital period T. If A be the area of everywhere
orbit, then its angular momentum would be (b) will have the same value everywhere but not directed
2mA towards the centre
(a) (b) mAT
T (c) will be same everywhere in magnitude directed
mA towards the centre
(c) (d) 2mAT (d) cannot be zero at any point
2T
Gravitation 465

14 Satellites orbiting the earth have a finite life and GM 2GM


(a) (b)
sometimes debris of satellites fall to the earth. This R R
is because [NCERT Exemplar] 1 GM GM
(c) (d)
(a) the solar cells and batteries in satellites run out 4 R 2R
(b) the laws of gravitation predict a trajectory spiralling
inwards 20 Compute the additional velocity required by a satellite
(c) of viscous forces causing the speed of satellite and orbiting around earth with radius 2R to become free
hence, height to gradually decrease from earth’s gravitational field. Mass of earth is M.
(d) of collisions with other satellites 2GM GM
(a) ( 2 − 1) (b) ( 2 − 1)
15 Both the earth and the moon are subject to the R 2R
gravitational force of the sun. As observed from the GM GM
sun, the orbit of the moon [NCERT Exemplar] (c) ( 3 − 1) (d) ( 2 + 1)
R R
(a) will be elliptical
21 The rotation of the earth about its axis speeds up such
(b) will not be strictly elliptical because the total
gravitational force on it is not central
that a man on the equator becomes weightless. In
such a situation, what would be the duration of one
(c) is not elliptical but will necessarily be a closed curve
day?
(d) deviates considerably from being elliptical due to
influence of planets other than the earth R 1 R 1
(a) 2π (b) (c) 2π Rg (d) Rg
16 As observed from the earth, the sun appears to move g 2π g 2π
in an approximate circular orbit. For the motion of 22 The rotation of the earth having radius R about its
another planet like mercury as observed from the axis speeds up to a value such that a man at latitude
earth, this would [NCERT Exemplar] angle 60° feels weightlessness. The duration of the
(a) be similarly true day in such a case is
(b) not be true because the force between the earth and R π R π R g
mercury is not inverse square law (a) π (b) (c) (d) π
g 2 g 3 g R
(c) not be true because the major gravitational force on
mercury is due to the sun 23 An earth’s satellite of mass m revolves in a circular
(d) not be true because mercury is influenced by forces orbit at a height h from the surface of the earth. R is
other than gravitational forces
the radius of the earth and g is acceleration due to
17 Different points in the earth are at slightly different gravity at the surface of the earth. The velocity of
distances from the sun and hence, experience the satellite in the orbit is given by
different forces due to gravitation.
gR 2 gR gR 2
For a rigid body, we know that, if various forces act (a) (b) gR (c) (d)
R +h R +h R +h
at various points in it, the resultant motion is as if a
net force acts on the CM (centre of mass) causing 24 A planet of mass m moves around the sun of mass M
translation and a net torque at the CM causing in an elliptical orbit. The maximum and minimum
rotation around an axis through the CM. For the distance of the planet from the sun are r1 and r 2,
earth-sun system (approximating the earth as a respectively. The time period of the planet is
uniform density sphere) [NCERT Exemplar] proportional to
(a) the torque is zero (a) r13 / 2
3/ 2
(b) (r1 + r2 )3 / 2 (c) (r1 − r2 )3 / 2 (d) r1
(b) the torque causes the earth to spin
(c) the rigid body result is not applicable, since the earth is 25 Particles of masses 2M, m and M are respectively at
not even approximately a rigid body points A, B and C with AB = 1/2 (BC ). m is
(d) the torque causes the earth to move around the sun much-much smaller than M and at time t = 0, they
18 Suppose the gravitational attraction varies inversely
are all at rest as given in figure.
as the distance from the earth. The orbital velocity A B C
of a satellite in such a case varies as nth power of 2M m M
distance, where n is equal to At subsequent times before any collision takes place
(a) − 1 (b) zero (c) + 1 (d) + 2 (a) m will remain at rest [NCERT Exemplar]
19 A body attains a height equal to the radius of the (b) m will move towards M
(c) m will move towards 2M
earth. The velocity of the body with which it was
(d) m will have oscillatory motion
projected is
466 OBJECTIVE Physics Vol. 1

26 Two particles of equal mass m go round a circle of M


32 If the mass of moon is , where M is the mass of
radius R under the action of their mutual 81
gravitational attraction. The speed of each particle is earth, find the distance of the point, where
Gm Gm 1 Gm 4Gm gravitational field due to earth and moon cancel each
(a) v = (b) v = (c) v = (d) v =
R 2R 2 R R other, from the centre of moon. Given that distance
27 Suppose a smooth tunnel is dug along a straight line between centres of earth and moon is 60 R, where R
joining two points on the surface of the earth and a is the radius of earth.
particle is dropped from rest at its one end. Assume (a) 4 R (b) 8 R (c) 12 R (d) 6 R
that mass of earth is uniformly distributed over its 33 The orbital angular momentum of a satellite
volume. Then, revolving at a distance r from the centre is L. If the
(a) the particle will emerge from the other end with distance is increased to 16r, then the new angular
GM e momentum will be
velocity , where M e and R e are earth’s mass and L
2R e (a) 16 L (b) 64 L (c) (d) 4 L
4
radius, respectively
(b) the overlapping will come to rest at centre of the tunnel 34 Two spheres of masses m and 2 m are separated by
(c) potential energy of the particle will be equal to zero at m
distance d. A particle of mass is projected straight
centre of tunnel, if it is along a diameter 5
(d) acceleration of the particle will be proportional to its from 2 m towards m with a velocity v 0 . Which of the
distance from mid-point of the tunnel. following statements is correct?
28 A satellite is moving in a circular orbit round the (a) Velocity of the particle decreases constantly.
earth with a diameter of orbit 2R . At a certain point (b) Velocity of the particle increase constantly.
a rocket fixed to the satellite is fired such that it (c) Acceleration of the particle may become momentarily zero.
increases the velocity of the satellite tangentially. (d) The particle never retraces its path.
The resulting orbit of the satellite would be 35 A ring of mass m1 and radius R is fixed in space at
(a) same as before some location. An external agent brings a point mass
(b) circular orbit with diameter greater than 2R m 2 from infinity to centre of the ring .Work done by
(c) elliptical orbit with minimum distance from the earth the external agent will be
equal to R Gm1m 2 Gm1m 2
(d) elliptical orbit with maximum distance from the earth (a) − (b)
R R
equal to R
G m12 + m 22 Gm1m 2
29 Assuming the radius of the earth to be 6.4 × 10 6 m. (c) (d)
What is the time period T and speed of satellite for R R (m12 + m 22 )
equatorial orbit at 1.4 × 10 3 km above the surface of 36 If an artificial satellite is moving in a circular orbit
the earth? around the earth with a speed equal to half the
(a) 6842 s and 7163 ms−1 (b) 34155 s and 3204 ms−1 magnitude of the escape velocity from the earth, the
(c) 6831 s and 2144 ms−1 (d) 2431 s and 3514 ms−1 height of the satellite above the surface of the earth is
30 A body which is initially at rest at a height R above R R
the surface of the earth of radius R, falls freely (a) 2 R (b) (c) R (d)
2 4
towards the earth, then its velocity on reaching the
37 Two concentric shells of masses M1 and M 2 are
surface of the earth is
concentric as shown in the figure. Which of the
3
(a) (2gR ) (b) (gR ) (c) gR (d) (4gR ) following is correct regarding the gravitational force
2
on m due to M1 and M 2 at points P, Q and R.
31 A rocket is launched vertical from the surface of the
R c
earth of radius R with an initial speed v. If M2
atmospheric resistance is neglected, then maximum
Q a M1
height attained by the rocket is b
R R P
(a) h = (b) h =
 2gR   2gR 
 2 − 1  2 + 1
 v   v  GM1m
2 2 (a) FP = 0 (b) FQ =
R R b2
(c) h = (d) h =
 2gR   2gR  G (M1 + M 2 ) m
 2 − 1  2 + 1 (c) FR = (d) All are correct
 v   v  c2
Gravitation 467

38 Energy of a satellite in circular orbit is E 0 . The 46 Let E be the energy required to raise a satellite to
energy required to move the satellite in a circular height h above earth’s surface and E ′ be the energy
orbit of 3 times the radius of the initial orbit is required to put the same satellite into orbit at that
2 E0 3 height. Then, E /E ′ is equal to
(a) E0 (b) 2E 0 (c) (d) E0 2h 2h R 2R
3 3 2 (a) (b) (c) (d)
(R + 2h ) (2R + 3h ) R +h 2h + R
39 What is the energy required to launch a m kg
satellite from earth’s surface in a circular orbit at an 47 Three particles each of mass m are located at the
altitude of 2R ? (R = radius of the earth ) vertices of an equilateral triangle of side a. At what
2 5 1 speed must they move, if they all revolve under the
(a) mgR (b) mgR (c) mgR (d) mgR
3 6 3 influence of their gravitational force of attraction in
40 A person brings a mass of 1 kg from infinity to a
a circular orbit circumscribing the triangle while still
point A. Initially, the mass was at rest but it moves preserving the equilateral triangle?
at a speed of 2 ms −1 as it reached at A. The work Am
done by the person on the mass is − 3 J . The
potential at A is

30°
(a) − 3 J kg −1 (b) − 12 J kg −1 O
(c) − 5 J kg −1 (d) None of these C B
m m
41 If gravitational attraction between two point masses
m1m 2 Gm 2Gm 3Gm 4Gm
be given by F = G . Then, the period of a (a) (b) (c) (d)
r3 a a a a
satellite in a circular orbit will be proportional to
48 A solid sphere of uniform density and radius R
(a) r (b) r 2 applies a gravitational force of attraction equal to F1
1
(c) r2 (d) independent of r on a particle placed at a distance 2R from the centre
of the sphere. A spherical cavity of radius R /2 is
42 Suppose the gravitational force varies inversely as
now made in the sphere as shown in the figure. The
the nth power of distance. Then, the time period of a
sphere with cavity now applies a gravitational force
planet in circular orbit of radius r around the sun
will be proportional to F
1 1 1
F2 on the same particle. The ratio 2 is
( n + 1) ( n − 1) ( n − 2) F1
(a) r 2 (b) r 2 (c) r n (d) r 2
43 A body is projected vertically upwards from the surface R/2 A
of earth with a velocity equal to half the escape R
velocity. If R be the radius of earth, maximum height
attained by the body from the surface of earth is
2R
R R 2R
(a) (b) (c) (d) R
6 3 3 5 7 3 7
(a) (b) (c) (d)
44 What impulse need to be given to a body of mass m, 9 8 4 9
released from the surface of earth along a straight 49 Suppose a vertical tunnel is along the diameter of
tunnel passsing through centre of earth, at the centre earth (here earth is assumed to be a sphere of
of earth, to bring it to rest? (Mass of earth M, radius uniform mass density ρ). If a body of mass m is
of earth R) thrown in this tunnel, its acceleration at a distance y
GM GMm GM from the centre is given by
(a) m (b) (c) m (d) Zero
R R 2R
y m
45 Four equal masses (each of mass M) are placed at the
corners of a square of side a. The escape velocity of
a body from the centre O of the square is
2GM 8 2GM
(a) 4 (b) 4π 3
a a (a) Gρym (b) πρy
3 4
4GM 4 2GM 4 4
(c) (d) (c) πρy (d) πGρy
a a 3 3
468 OBJECTIVE Physics Vol. 1

50 A solid sphere of mass M and radius R has a 57 Four particles each of mass M, move along a circle of
spherical cavity of radius R /2 such that the centre of radius R under the action of their mutual
cavity is at a distance R /2 from the centre of the gravitational attraction. The speed of each particle is
sphere. A point mass m is placed inside the cavity at GM GM
a distance R /4 from the centre of sphere. The (a) (b) 2 2
R R
gravitational force on mass m is
GM GM  2 2 + 1
11GMm 14 GMm GMm GMm (c) (2 2 + 1) (d)  
(a) (b) (c) (d) R R  4 
R2 R2 2R 2 R2
51 Two particles of masses m and M are initially at rest 58 An object is released from a height twice the radius
at infinite distance. Find their relative velocity of of the earth on the surface of earth. Find the speed
approach due to gravitational attraction when d is with which it will collide with ground by neglecting
their separation at any instant. effect of air. (Take, R as radius of earth and M as
2G (M + m ) G (M + m )
mass of earth)
(a) (b) GM GM
d d (a) 2 (b) 3
3R 2R
G (M + m ) G (M + m )
(c) (d) GM GM
2d 4d (c) 2 (d) 3
R R
52 A satellite is revolving round the earth with orbital
speed v o . If it stops suddenly, the speed with which 59 A planet of mass m revolves in an elliptical orbit
it will strike the surface of earth would be around the sun, so that its maximum and minimum
(v e = escape velocity of a particle on earth’s surface) distance from the sun are equal to r a and r p ,
v e2 respectively. The angular momentum of this planet
(a) (b) 2v o (c) v e2 − v o2 (d) v e2 − 2v o2 relative to the sun is
vo
2GMrpra 4GMrpra
53 Three point masses each of mass m rotate in a circle (a) m (b) m
(rp + ra ) (rp + ra )
of radius r with constant angular velocity ω due to
their mutual gravitational attraction. If at any GMrpra GMrpra
(c) m (d) m
instant, the masses are on the vertex of an (rp + ra ) 2(rp + ra )
equilateral triangle of side a, then the value of ω is
Gm 3Gm 60 The magnitudes of the gravitational force at
(a) (b) distances r1 and r 2 from the centre of a uniform
a3 a3
Gm
sphere of radius R and mass M are F1 and F2,
(c) (d) None of these respectively. Then,
3a 3 F1 r1
(a) = ; if r1 < R and r2 < R
54 Pertaining to two planets, the ratio of escape F 2 r2
velocities from respective surfaces is 1: 2, the ratio of F1 r12
the time period of the same simple pendulum at (b) = ; if r1 > R and r2 > R
F 2 r22
their respective surfaces is 2 :1 (in same order). Then
F r
the ratio of their average densities is (c) 1 = 2 ; if r1 < R and r2 < R
(a) 1:1 (b) 1: 2 (c) 1: 4 (d) 8 :1 F 2 r1
55 The ratio of energy required to raise a satellite to a F1 r22
(d) = ; if r1 > R and r2 < R
height h above the earth’s surface to that required to F 2 r12
put it into the orbit is 61 Two identical thin rings each of radius R are coaxially
(a) h :2R (b) 2h : R placed at a distance R. If the rings have a uniform
(c) R : h (d) h : R
mass distribution and each has masses 2m and 4m
56 A body of supercondense material with mass twice respectively, then the work done in moving a mass m
the mass of earth but size very small compared to from centre of one ring to that of the other is
the size of earth starts from rest from h << R above
2Gm 2
the earth’s surface. It reaches earth in time (a) zero (b) (1 − 2 )
R
h 2h 2h 4h Gm 2 Gm 2
(a) t = (b) t = (c) t = (d) t =
g g 3g 3g (c) ( 2 − 1) (d)
2R 2R
Gravitation 469

62 A mass m is at a distance a from centre of a uniform 63 A particle would take a time t to move down a
rod of length l and M. The gravitational force on the straight tube from the surface of earth (supposed to
mass due to the rod is be a homogeneous sphere) to its centre. If gravity
4GMm 4GmM were to remain constant, then the time would be t.
(a) (b) The ratio of t /t ′ will be
(a + l )2 4a 2 − l 2
π π π
GMm GmM (a) (b) (c) 2π (d)
(c) (d) 2 2 2 2
a2 2(l + a )2

(B) Medical entrance special format questions


Assertion and reason Statement based questions
Directions (Q. Nos. 1-5) These questions consists of two statements
1 Which of the following statement is correct?
each printed as Assertion and Reason. While answering these
(Take, earth as a sphere of uniform density)
questions you are required to choose any one of the following
(a) Acceleration due to gravity increases with altitude.
four responses
(b) Acceleration due to gravity decreases with
(a) If both Assertion and Reason are correct and Reason is the latitude.
correct explanation of Assertion. (c) Acceleration due to gravity depends on the mass of
(b) If both Assertion and Reason are correct but Reason is not the the object.
correct explanation of Assertion. (d) Acceleration due to gravity decreases with
(c) If Assertion is correct but Reason is correct. increasing depth.
(d) If Assertion is incorrect but Reason is correct.
2 When the radius and mass of earth are increased
1 Assertion Gravitational force between two masses in by 0.5%. Which of the following statement is
air is F. If they are immersed in water, force will remain correct at the surface of the earth?
F. (a) g will decrease.
Reason Gravitational force does not depend on the (b) g will increase.
medium between the masses. (c) Escape velocity varies by 15%.
(d) Potential energy varies by 19.99%.
2 Assertion The centre of
Y 3 Which one of the following statement is correct?
semicircular ring of mass m and
radius R is the origin O. The (a) A geo-stationary satellite goes around the earth in
Gm east-west direction.
potential at origin is − . (b) A geo-stationary satellite goes around the earth in
R O X west-east direction.
Reason The gravitational field (c) The first half of the rotation of geo-stationary
strength is towards Y-axis. satellite goes around the earth in north-south
3 Assertion Areal velocity of a planet around sun will direction and then in south-north direction.
become two times, if mass of planet is halved. (d) The first half of the rotation of geo-stationary
L satellite goes around the earth in east-west
Reason Areal velocity = , where L is angular direction, then in west direction.
2m
momentum of planet about centre of sun. 4 Which one of the following statement is in correct?
(a) Inertial mass is a measure of difficulty of
4 Assertion If gravitational potential at some point is
accelerating a body by an external force, whereas
zero, then gravitational field strength at that point will the gravitational mass is relevant in determining
also be zero. the gravitational force on it by an external mass.
Reason Except at infinity gravitational potential due to (b) According to an experimental result, gravitational
a system of point masses at some finite distance cannot mass and inertial mass are equal.
be zero. (c) Acceleration due to gravity on the earth is the
same for all bodies is due to equality of
5 Assertion If radius of earth suddenly shrinks to half its
gravitational mass and inertial mass.
present value without changing its mass, then the period
(d) Gravitational mass of a particle like proton can
of an earth’s satellite will not change. depend on the presence of neighbouring heavy
Reason Time period of a satellite does not depend upon objects but the inertial mass cannot.
the mass of earth.
470 OBJECTIVE Physics Vol. 1

5 According to Newton’s law of gravitation, the Codes


magnitude of the force F on point masses m1 and m 2 A B C D A B C D
Gm1m 2 (a) p r q s (b) p t r s
(say) is given by F = . (c) q t r s (d) q s t p
r2
2 On the surface of earth acceleration due to gravity is
I. This equation is not directly applicable for the
g and gravitational potential isV. Match the Column
gravitational force between extended object (like
I with Column II and mark the correct option from
the earth) and a point mass.
the codes given below.
II. This equation can be applied to find the
gravitational force between an extended object Column I Column II
and a point mass provided r is the distance of the
(A) At height h = R, value of g (p) decreases by a factor 1
point mass from the geometric centre of the 4
extended objects.
R 1
Which of the following statement(s) about this (B) At depth h = , value of g (q) decrease by a factor 2
2
equation is/are correct?
(a) Only I (b) Only II 11
(C) At height h = R, value of V (r) decrease by a factor 8
(c) Both I and II (d) Neither I and II
6 I. The geo-stationary satellite always has circular orbit. R
(D) At depth h = , value of V (s) increases by a factor 2
2
II. The time period of satellite depends on
mass of the satellite, mass of the earth and height (t) None
of satellite from the surface of earth.
Codes
Which of the following statement(s) is/are incorrect? A B C D
(a) Only I (b) Only II (a) p t s r
(c) Both I and II (d) Neither I nor II (b) q t q r
(c) q r p s
Match the columns (d) p q q r
1 Two concentric spherical shells as shown in figure. 3 In elliptical orbit of a planet, as the planet moves
Match the Column I with Column II and mark the from apogee position to perigee position, match the
correct option from the codes given below. Column I with Column II and mark the correct
B
option from the codes given below.
A
D
Column I Column II
C (A) Speed of planet (p) remains same
(B) Distance of planet from centre of sun (q) decreases
(C) Potential energy (r) increases
Column I Column II (D) Angular momentum about centre of sun (s) cannot say
(A) Potential at A (p) greater than B
Codes
(B) Gravitational field at A (q) less than B
A B C D
(C) As one moves from C to D (r) potential remains constant (a) p r q s
(D) As one moves from D to A (s) gravitational field decreases (b) p s q r
(c) r q q p
(t) None
(d) r p q s
(C) Medical entrances’ gallery
Collection of questions asked in NEET & various medical entrance exams

1 A body weighs 72 N on the surface of the earth. B


What is the gravitational force on it, at a height A C
equal to half of radius of the earth? [NEET 2020] S
(a) 32 N (b) 30 N (c) 24 N (d) 48 N
2 What is the depth at which the value of acceleration (a) KB < K A < KC (b) K A > KB > KC
due to gravity becomes 1/n times the value that the (c) K A < KB < KC (d) KB > K A > KC
surface of earth? (Radius of earth =R) [NEET 2020] 9 Two satellites A and B revolve round the same planet
(a) R / n 2 (b) R (n −1) / n in coplanar circular orbits lying in the same plane.
(c) Rn / (n − 1) (d) R / n Their periods of revolutions are 1 h and 8 h,
respectively. The radius of the orbit of A is 10 4 km.
3 A body weighs 200 N on the surface of the earth.
The speed of B relative to A, when they are close in
How much will it weigh half way down to the
km/h is [AIIMS 2018]
centre of the earth ? [NEET 2019]
(a) 3π × 104 (b) zero (c) 2π × 104 (d) π × 104
(a) 200 N (b) 250 N
(c) 100 N (d) 150 N 10 A planet is revolving around the sun in a circular
4 The work done to raise a mass m from the surface of orbit with a radius r. The time period is T. If the
the earth to a height h, which is equal to the radius force between the planet and star is proportional to
of the earth, is [NEET 2019] r −3/ 2 , then the square of time period is
1 proportional to [AIIMS 2018]
(a) 2mgR (b) mgR
2 (a) r 3 / 2 (b) r 2 (c) r (d) r 5 / 2
3
(c) mgR (d) mgR 11 If R is the radius of earth, ω is its angular velocity
2 and g p is the value of g at the poles. The effective
5 The time period of a geo-stationary satellite is 24 h, value of g at a latitude λ = 60 ° is [JIPMER 2018]
at a height 6R E (R E is the radius of earth) from 1 1
(a) g p − Rω 2 (b) g p + Rω 2
surface of earth. The time period of another satellite 4 4
whose height is 2.5 R E from surface will be 1 1
(c) g p − Rω 2 (d) g p + Rω 2
[NEET (Odisha) 2019] 2 2
24 12
(a) 6 2 h (b) 12 2 h (c) h (d) h 12 At what distance (in metre) from the centre of the
2.5 2.5 moon, the intensity of gravitational field will be zero?
6 Find gravitational field at a distance of 2000 km (Take, mass of earth and moon as 5.98 × 10 24 kg and
from the centre of earth. (Take, R earth = 6400 km, . × 10 22 kg respectively and the distance between
735
r = 2000 km, M earth = 6 × 10 24 kg) [AIIMS 2019] moon and earth is 3.855 × 10 8 m.) [JIPMER 2018]
(a) 1.53 ms −2 (b) 7.12 ms −2 (a) zero (b) 3. 85 × 107 (c) 8 × 108 (d) 3. 46 × 108
(c) 3.06 ms −2 (d) 1.8 ms −2
13 Two astronauts are floating in gravitation free space
7 If escape velocity on earth surface is 11.1 kmh −1, after having lost contact with their spaceship. The two
then find the escape velocity on moon surface. If mass will [NEET 2017]
1 (a) keep floating at the same distance between them
of moon is times of mass of earth and radius of
81 (b) move towards each other
1 (c) move away from each other
moon is times radius of earth. [JIPMER 2019]
(d) will become stationary
4
(a) 2.46 kmh −1 (b) 3.46 kmh −1 14 The acceleration due to gravity at a height 1 km
(c) 4.4 kmh −1 (d) None of these above the earth is the same as at a depth d below the
8 The kinetic energies of a planet in an elliptical orbit surface of earth. Then, [NEET 2017]
1
about the sun at positions A, B and C are K A , K B and (a) d = km (b) d = 1 km
K C , respectively. AC is the major axis and SB is 2
3
perpendicular to AC at the position of the sun S as (c) d = km (d) d = 2 km
shown in the figure. Then, [NEET 2018] 2
472 OBJECTIVE Physics Vol. 1

15 A spaceship is launched into a circular orbit close to 22 Kepler’s third law states that square of period of
earth’s surface. What additional velocity has now to revolution (T ) of a planet around the sun is
be imparted to the spaceship in the orbit to overcome proportional to third power of average distance r
the gravitational pull? between the sun and planet, i.e. T 2 = Kr 3 , here K is
(Take, radius of earth = 6400 km and g = 9.8 ms −2 ) constant. If the masses of the sun and planet are M
[AIIMS 2017] and m respectively, then as per Newton’s law of
(a) 3.28 kms −1 (b) 12 kms −1 gravitational force of attraction between them is
(c) 10 kms −1 (d) 40 kms −1 GMm
F = 2 , here G is gravitational constant. The
16 What is the maximum height attained by a body r
projected with a velocity equal to one-third of the relation between G and K is described as
escape velocity from the surface of the earth? [CBSE AIPMT 2015]
(Radius of the earth = R) [AIIMS 2017] (a) GK = 4π 2 (b) GMK = 4π 2 (c) K = G (d) K = I /G
(a) R /2 (b) R /3 (c) R /5 (d) R /8
23 Two spherical bodies of masses M and 5M and radii
17 The planets with radii R 1 and R 2 have densities ρ1 R and 2R are released in free space with initial
and ρ 2 , respectively. Their atmospheric pressures separation between their centres equal to 12 R. If
are p 1 and p 2 , respectively. Therefore, the ratio of they attract each other due to gravitational force
masses of their atmospheres, neglecting variation of only, then the distance covered by the smaller body
g within the limits of atmosphere is [JIPMER 2017] before collision is [CBSE AIPMT 2015]
(a) ρ1R 2 p1 / ρ2R1p 2 (b) p1R 2ρ2 / pP2R1ρ1 (a) 2.5 R (b) 4.5 R (c) 7.5 R (d) 1.5 R
(c) p1R1ρ1 / p 2R 2ρ2 (d) p1R1ρ2 / p 2R 2ρ1 24 The reading of a spring balance corresponds to
18 Starting from the centre of the earth having radius R, 100 N while situated at the north pole and a body is
the variation of g (acceleration due to gravity) is kept on it. The weight record on the same scale, if it
shown by [NEET 2016] is shifted to the equator, is (Take, g = 10 ms −2 and
radius of the earth, R = 6.4 × 10 6 m) [AIIMS 2015]
g g (a) 99.66 N (b) 110 N (c) 97.66 N (d) 106 N
(a) (b)
25 The gravitational field due to a uniform solid sphere
of mass M and radius a at the centre of the sphere is
O R r O R r [UK PMT 2015]
GM GM GM
(a) 2 (b) 3 (c) (d) zero
a a 2a
g g 26 What would be the value of acceleration due to
(c) (d) gravity at a point 5 km below the earth’s surface?
(Take, R E = 6400 km, gE = 9.8 ms −2 ) [UK PMT 2015]
O R r O R r
(a) 9.6 ms −2 (b) 9.79 ms−2 (c) 9.89 ms −2 (d) 10 ms −2
19 A satellite of mass m is orbiting the earth (of radius 27 Two particles of equal masses go round a circle of
R) at a height h from its surface. The total energy of radius R under the action of their mutual
the satellite in terms of g 0 , the value of acceleration gravitational attraction. What would be the speed of
due to gravity at the earth’s surface is [NEET 2016] each particle? [UK PMT 2015]
mg 0R 2 mg 0R 2 2 mg 0R 2 2 mg 0R 2 GM GM GM 2GM
(a) (b) − (c) (d) − (a) (b) (c) (d)
2(R + h ) 2(R + h ) R +h R +h 2R R 4R R
20 At what height from the surface of earth, the 28 What would be the escape velocity from the moon, if
gravitation potential and the value of g are the mass of the moon is 7.4 × 10 22 kg and its radius
− 5.4 × 10 7 J kg −2 and 6 ms −2 , respectively? (Take, is 1740 km? [UK PMT 2015]
the radius of earth is 6400 km) [NEET 2016] (a) 2.4 ms −1 (b) 2.4 kms −1 (c) 240 kms −1 (d) 0.24 kms −1
(a) 1600 km (b) 1400 km (c) 2000 km (d) 2600 km
29 Two spheres of masses 16 kg and 4 kg are separated
21 The ratio of escape velocity at earth (v e ) to the by a distance 30 m on a table. Then, the distance
escape velocity at a planet (v p ) whose radius and from sphere of mass 16 kg at which the net
mean density are twice as that of earth is [NEET 2016] gravitational force becomes zero is [EAMCET 2015]
(a) 1 : 2 2 (b) 1 : 4 (c) 1 : 2 (d) 1 : 2 (a) 10 m (b) 20 m (c) 15 m (d) 5 m
Gravitation 473

30 Orbital velocity of earth satellite does not depend on 37 A body of mass m is raised to a height 10R from the
[Kerala CEE 2015] surface of the earth, where R is the radius of the earth.
(a) mass of the earth The increase in potential energy is (G = universal
(b) mass of the satellite constant of gravitation, M = mass of the earth and
(c) radius of the earth g = acceleration due to gravity) [MHT CET 2014]
(d) acceleration due to gravity GMm GMm mgR 10 GMm
31 Gravitational potential energy of a body of mass m at (a) (b) (c) (d)
11R 10R 11G 11R
a height of h above the surface of earth (M = mass of
earth R = radius of earth) is [Kerala CEE 2015] 38 An artificial satellite moves in a circular orbit around
GMm GMm the earth. Total energy of the satellite is given by E.
(a) (b) The potential energy of the satellite is [WB JEE 2014]
h (R + h )
− GM 2E 2E
(c) (d) −
GMm (a) − 2E (b) 2E (c) (d) −
(R + h ) (R + h ) 3 3
39 What is a period of revolution of the earth satellite?
32 According to Kepler’s law of planetary motion, if T
represents time period and r is orbital radius, then Ignore the height of satellite above the surface of the
for two planets these are related as [Manipal 2015] earth. Given,
T 
3/ 2
r1 T1  r1  (i) the value of gravitational acceleration, g = 10 ms −2 .
(a)  1  = (b) =
 T2  r2 T2 r2  (ii) radius of the earth, R e = 6400 km
2 3 (Take, π = 314. ) [KCET 2014]
T  r  T12 r13
(c)  1  =  1  (d) = (a) 85 min (b) 156 min (c) 83.73 min (d) 90 min
 2
T r2  T2 r1
40 The time period of the earth’s satellite revolving at a
33 If the mass of a body is M on the surface of the height of 35800 km is [Kerala CEE 2014]
earth, then mass of the same body on the surface of (a) 24 h (b) 100 min (c) 12 h (d) 48 h
the moon is [KCET 2015]
41 At a height H from the surface of earth, the total
M energy of a satellite is equal to the potential energy
(a) 6 M (b)
6 of a body of equal mass at a height 3R from the
(c) zero (d) M surface of the earth(R = radius of the earth). The
34 Dependence of intensity of gravitational field (E ) of value of H is [EAMCET 2014]
earth with distance (r ) from centre of earth is 4R R
(a) R (b) (c) 3R (d)
correctly represented by [CBSE AIPMT 2014] 3 3
42 A body of mass m taken from the earth’s surface to
E E the height equal to twice the radius (R ) of the earth.
R The change in potential energy of body will be
(a) O (b) O
r R r [NEET 2013]
2 1
(a) mg2R (b) mgR (c) 3mgR (d) mgR
3 3
E E 43 Infinite number of bodies, each of mass 2 kg are
(c) O
R
(d) O situated on X-axis at distance 1m, 2 m, 4 m, 8 m
r R r respectively from the origin. The resulting
gravitational potential due to this system at the
origin will be [NEET 2013]
35 The force of gravitation is [UK PMT 2014] (a) − G (b) − (8 /3) G (c) − (4 /3) G (d) − 4 G
(a) repulsive (b) electrostatic 44 The value of acceleration due to gravity at the
(c) conservative (d) non-conservative surface of earth [J&K CET 2013]
36 Keeping the mass of the earth as constant, if its (a) is maximum at the poles
radius is reduced to 1/4 th of its initial value, then (b) is maximum at the equator
the period of revolution of the earth about its own (c) remains constant everywhere on the surface of the earth
axis and passing through the centre (in hours) is (d) is maximum at the international time line
(assume the earth to be a solid sphere and its initial 45 The escape velocity of a particle from the surface of
period of rotation as 24 h) [EAMCET 2014] the earth is given by [J&K CET 2013]
(a) 12 (b) 3 (a) (gR )1/ 2 (b) (2gR )1/ 2
(c) 6 (d) 1.5 (c) (3gR )1/ 2 (d) (gR /2)1/ 2
474 OBJECTIVE Physics Vol. 1

46 If earth were to rotate on its own axis such that the 53 A geo-stationary satellite is orbiting the earth at a
weight of a person at the equator becomes half the height of 5R above that surface of the earth, R being
weight at the poles, then its time period of rotation the radius of the earth. The time period of another
is (g = acceleration due to gravity near the poles and satellite (in hours) at a height of 2R from the surface
R is the radius of earth) (Ignore equatorial bulge) of the earth is [CBSE AIPMT 2012]
[EAMET 2013] (a) 5 (b) 10 (c) 6 2 (d) 6 / 2
2R R R R
(a) 2π (b) 2π (c) 2π (d) 2π 54 Two identical thin rings A and B each of radius R
g 2g 3g g are co-axially placed at a distance R. If mass of rings
47 Earth is moving around the sun in elliptical orbit as are m1, m 2 respectively, then the work done in
shown. The ratio of OB and OA is R. Then, the ratio moving a mass m from centre of one ring to that of
of earth’s velocities at A and B is [KCET 2013] the other is [UP CPMT 2012]
Gm G m (m1 − m 2 )
A (a) ( 2 + 1)m (b) ( 2 − 1)
m 2R 2R
Gm 2
O B (c) (m1 + m 2 ) (d) zero
Sun R
55 The distance between the sun and the earth be r,
then the angular momentum of the earth around the
(a) R −1 (b) R (c) R (d) R 2 / 3 sun is proportional to [UP CPMT 2012]
48 If two planets of radii R 1 and R 2 have densities d 1 (a) r (b) r 3 / 2
and d 2 , then the ratio of their respective acceleration (c) r (d) None of these
due to gravity is [Kerala CEE 2013] 56 When a satellite is moving around the earth with
(a) R1d1 : R 2d 2 (b) R12d1 : R 22d 2 velocity v, then to make the satellite escape, the
(c) R13d1 : R 23d 2 (d) R1d12 : R 22d 22 minimum percentage increase in its velocity should
be [BHU 2012]
49 The weight of an object is 90 kg at the surface of the (a) 100% (b) 82.4%
earth. If it is taken to a height equal to half of the (c) 41.4% (d) None of these
radius of the earth, then its weight will beocme
[MPPET 2013] 57 A launching vehicle carrying an artificial satellite of
(a) 135 kg (b) 45 kg mass m is set for launch on the surface of the earth of
(c) 60 kg (d) 40 kg mass M and radius R. If the satellite is intended to move
50 The escape velocity on earth is 11.2 kms −1. If the in a circular orbit of radius 7R, the minimum energy
body is projected out with twice this velocity, then required to be spent by the launching vehicle on the
the speed of the body far away from the earth, satellite is (Gravitational constant = G) [Manipal 2012]
ignoring the presence of any other object in GMm 13 GMm GMm GMm
(a) (b) − (c) (d)
universe, will be [MP PET 2013] R 14R 7R 14R
(a) 11.2 kms −1 (b) 22.4 kms −1 58 Consider a satellite orbiting the earth as shown in
(c) 19.4 kms −1 (d) 15.2 kms −1 the figure below. Let L a and L p represent the
51 A satellite of mass m is circulating around the earth angular momentum of the satellite about the earth
with constant angular velocity. If the radius is R 0 when at aphelion and perihelion, respectively.
and mass of earth is M, then the angular momentum Consider the following relations. [AMU 2012]
about the centre of the earth is [UP CPMT 2013] Satellite a (Aphelion)
(a) m GM /R 0 (b) M GmR 0 ra

(c) m GMR 0 (d) M GM /R 0

52 A spherical planet has a mass M p and diameter D p . rp


A particle of mass m falling freely near the surface of Earth
this planet will experience an acceleration due to
gravity, equal to [CBSE AIPMT 2012] p (Perihelion)

(a) 4 GM p /D p2 (b) GM pm /D p2 (i) L a = L p


(ii) L a = −L p
(c) GM pm /D p2 (d) 4GM pm /D p2
(iii) ra × L a = rp × L p
Gravitation 475

Which of the above relation(s) is/are true? 61 The mass of the moon is 1/81 of the earth but the
(a) (i) only (b) (ii) only gravitational pull is 1/6 of the earth. It is due to the
(c) (iii) only (d) (i) and (iii) fact that [JCECE 2012]
59 A body is projected vertically upward from the 9
(a) the radius of earth is of the moon
surface of the earth with a velocity equal to half the 6
escape velocity. If R is radius of the earth, then 81
(b) the radius of moon is of the earth
maximum height attained by the body is [AMU 2012] 6
(a) R /6 (b) R /3 (c) moon is the satellite of the earth
(c) 2R /3 (d) R (d) None of the above
60 The imaginary angular velocity of the earth for 62 The height vertically above the earth’s surface at
which the effective acceleration due to gravity at the which the acceleration due to gravity becomes 1% of
equator shall be zero is equal to [AMU 2012] its value at the surface is [WB JEE 2011]
(Take, g = 10 ms −2 for the acceleration due to (a) 8R (b) 9R (c) 10R (d) 20R
gravity, if the earth were at rest, radius of earth 63 Two satellites of mass m and 9m are orbiting a planet
equal to 6400 km and λ = 60) in orbits of radius R. Their periods of revolution will
−3 −1 −3 −1
(a) 1.25 × 10 rad s (b) 2.50 × 10 rad s be in the ratio of [KCET 2011]
−3 −1 −3 (a) 9 : 1 (b) 3 : 1
(c) 3.75 × 10 rad s (d) 5 × 10 rad s −1
(c) 1 : 1 (d) 1 : 3

ANSWERS
CHECK POINT 10.1
1. (d) 2. (b) 3. (c) 4. (c) 5. (d) 6. (b) 7. (c) 8. (c) 9. (d) 10. (a)
11. (c) 12. (d) 13. (c) 14. (c) 15. (d) 16. (d)

CHECK POINT 10.2


1. (c) 2. (b) 3. (d) 4. (c) 5. (c) 6. (b) 7. (c) 8. (a) 9. (b) 10. (b)
11. (a) 12. (a) 13. (a) 14. (b) 15. (a) 16. (b)

CHECK POINT 10.3


1. (a) 2. (a) 3. (b) 4. (d) 5. (d) 6. (b) 7. (b) 8. (c) 9. (c) 10. (c)
11. (d) 12. (d) 13. (d) 14. (c)

CHECK POINT 10.4


1. (c) 2. (b) 3. (b) 4. (c) 5. (c) 6. (d) 7. (c) 8. (c) 9. (c)

CHECK POINT 10.5


1. (c) 2. (b) 3. (a) 4. (c) 5. (a) 6. (a) 7. (d) 8. (a) 9. (c) 10. (a)
11. (c) 12. (d) 13. (a) 14. (d)

CHECK POINT 10.6


1. (b) 2. (a) 3. (b) 4. (c) 5. (c) 6. (d) 7. (b) 8. (a) 9. (d) 10. (d)
11. (d) 12. (d) 13. (d) 14. (c) 15. (d)
476 OBJECTIVE Physics Vol. 1

(A) Taking it together


1. (c) 2. (c) 3. (b) 4. (b) 5. (c) 6. (c) 7. (a) 8. (d) 9. (a) 10. (d)
11. (c) 12. (b) 13. (d) 14. (c) 15. (b) 16. (c) 17. (a) 18. (b) 19. (a) 20. (a)
21. (a) 22. (a) 23. (d) 24. (b) 25. (c) 26. (c) 27. (d) 28. (c) 29. (a) 30. (b)
31. (a) 32. (d) 33. (d) 34. (c) 35. (a) 36. (c) 37. (d) 38. (a) 39. (c) 40. (c)
41. (b) 42. (a) 43. (b) 44. (a) 45. (b) 46. (a) 47. (a) 48. (d) 49. (d) 50. (c)
51. (a) 52. (d) 53. (b) 54. (c) 55. (b) 56. (c) 57. (d) 58. (a) 59. (a) 60. (a)
61. (b) 62. (b) 63. (c)

(B) Medical entrance special format questions


l Assertion and reason
1. (a) 2. (b) 3. (d) 4. (d) 5. (c)

l Statement based questions


1. (d) 2. (a) 3. (b) 4. (d) 5. (a) 6. (a)

l Match the columns


1. (c) 2. (d) 3. (c)

(C) Medical entrances’ gallery


1. (a) 2. (b) 3. (c) 4. (b) 5. (a) 6. (c) 7. (a) 8. (b) 9. (d) 10. (d)
11. (a) 12. (b) 13. (b) 14. (d) 15. (a) 16. (d) 17. (d) 18. (b) 19. (b) 20. (d)
21. (a) 22. (b) 23. (c) 24. (a) 25. (d) 26. (b) 27. (c) 28. (b) 29. (b) 30. (b)
31. (d) 32. (c) 33. (d) 34. (b) 35. (c) 36. (a) 37. (d) 38. (a) 39. (c) 40. (a)
41. (a) 42. (b) 43. (d) 44. (a) 45. (b) 46. (a) 47. (c) 48. (a) 49. (d) 50. (c)
51. (c) 52. (a) 53. (c) 54. (b) 55. (a) 56. (c) 57. (b) 58. (a) 59. (b) 60. (b)
61. (a) 62. (b) 63. (c)
Hints & Explanations
l CHECK POINT 10.1 11 (c) ∴ F =
Gm1m 2
or F ∝ m1m 2
1 (d) Kepler’s second law is based on conservation of angular r2
dA L On M due to m force is F and due to mass 2m force is 2F.
momentum, = = constant. Therefore, net force is 3 F.
dt 2m
12 (d) Force will be zero at the point P located at
2 (b) In elliptical orbit, velocity both in magnitude and direction
keep on changing. Therefore, momentum does not remain D
constant.
P
x
3 (c) Since, Kepler’s second law is same as the law of m1 m2
conservation of angular momentum.
d1v1 = d 2v 2 (from conservation of angular momentum at these
m1 81M 9
two points) x= D= D= D
d 1v1 m1 + m 2 81M + M 10
∴ v2 =
d2 14 (c) Let masses of two balls are m1 = m 2 = m (given) and the
density is ρ. Distance between their centres = AB = 2 R.
4 (c) Kepler’s third law states that square of the time period of
any planet about the sun is directly proportional to R 3.
5 (d) T ∝ r 3/ 2 A
R R
B
13  3/ 2
T1  10
∴ =  = 10 10
T2  1012 
Thus, the magnitude of the gravitational force F that two balls
6 (b) According to Kepler’s third law, T 2 ∝ r 3
separated by a distance 2R exert on each other is
T 8
Q TA = 8TB ⇒ A = (Given) 4 3 
2
TB 1  π R ρ
(m ) (m ) m2 3 
2 3 3 F =G =G 2 =G
 TA  r   8
2
r  r (2R ) 2
4R 4R 2
⇒   =  A ⇒   =  A ⇒ A = 4
 TB   rB   1  rB  rB ∴ F ∝ R4
⇒ rA = 4 rB Gm1m 2 Gm1m 2 F
15 (d) As we know, F = and F ′ = =
7 (c) According to Kepler’s third law, T ∝ r 2 3 r2 (3r )2 9
 F − F ′ 8
T12 : T22 : T32 = r13 : r23 : r33 Percent decrease in F =   × 100 = × 100 = 89%
 F  9
3 3
 1  3
=   : (1)3 :   = 1: 8 : 27 16 (d) Consider the equilateral triangle as PQR with centroid at
 2  2
O. Given, OP = OQ = OR = 2 m
8 (c) By Kepler’s third law of planetary motion,
P
T 2 ∝ a3 2 kg

Given, T1 = 1day (geostationary) FOP


a1 = a, a 2 = 2a
O 4 kg
T12 a13 a3 (2a )3 FOQ cos 30° FOR cos 30°
∴ 2
= 3 ⇒ T22 = 23 T12 = 3 × 1 = 8
T2 a 2 a1 a FOQ sin 30°
Q 30° R
⇒ T2 = 2 2 days 2 kg
30°
2 kg
FOR sin 30°
9 (d) Gravitational constant does not depend on the nature of
the medium in which bodies are kept.
The gravitational force on mass 2 kg due to mass 1 kg at P,
Gm1m 2 4×2
10 (a) F = FOP = G = 4 G along OP
r2 ( 2 )2
Gm1m 2 6.67 × 10 −11 × 1 × 1 4×2
∴ r= = Similarly, FOQ = G = 4 G along OQ
F 9.8 × 10 −9 ( 2 )2
= 0.082 m = 8.2 cm − ~ 8 cm
478 OBJECTIVE Physics Vol. 1

4×1
and FOR = G 10 (b) According to the question,
( 2 )2  2h   d 
= 4G along OR g 1 −  = 1 −  (At h << R )
 R   R
FOQ cos 30° and FOR cos 30° are equal and acting in opposite 2h d
directions, hence cancel out each other. Then, the resultant or 1− = 1−
force on the mass 4 kg at O, R R
F = FOP − (FOQ sin 30 ° + FOR sin 30 ° ) = 0 (zero). ∴ d = 2h
 d 1 g
CHECK POINT 10.2 11 (a) Given, g 1 −  =
 R 2 
l
2
h
GM M 1 + 
1 (c) g = or g ∝ 2  R
R2 R
d 1 16 d 34
Given, mass is 2 times and diameter is 3 times. Hence, or 1− = 2
= ⇒ =
2g R  1600  50 R 50
g′ = . 21 + 
9  6400 
GM
2 (b) g = 2 . If radius shrinks to half of its present value, then  34
∴ d = (6.4 × 10 6 )   = 4.3 × 10 6 m
R  50 
g will become four times.
12 (a) At centre of earth, value of g is zero. Hence, weight is zero.
GM G R2
3 (d) g = 2 or = 13 (a) At poles, value of g is maximum. Since, there is no effect
R g m
of rotation of earth.
G m2
∴ will have the unit . 14 (b) When a body is taken from equator to poles, its weight
g kg
increases, because acceleration due to gravity increases.
4 
G  πR 3 ρ 15 (a) At equator, g′ = g − Rω 2 ⇒ 0 = g − Rω 2
GM 3 
4. (c) g = 2 = 2
R R g 9.8
∴ ω= = = 1.23 × 10 −3 rad s −1
∴ ρ=
g
=
3g R 6400 × 10 3
R 4πGR
G ⋅ 4π 16 (b) Acceleration due to gravity at latitude φ is given by
3
4 g ρR 1 4 2 g′ = g − Rω 2 cos 2 φ
5 (c) g = GπRρ ⇒ 1 = 1 1 = × =
3 g2 ρ 2R2 2 1 1 3 2
At φ = 30 °, g30 ° = g − Rω 2 cos 2 30 ° = g − Rω
4
6 (b) g ∝ ρR. If density of earth is increased four times and its
3 2
radius become half, then the value of acceleration due to ∴ g − g30 ° = ωR
gravity becomes double due to which our weight get doubled. 4
g g CHECK POINT 10.3
7 (c) Given, g′ = 2
= (At h = R ) l

 h  4 A
1 +  1 (a) Due to three particles, net
 R intensity at the centre,
72 I = IA + IB + IC = 0. Because these
∴ w′ = = 18N three intensities are equal in
IA
4 IB IC
g magnitude and the angle between
8 (a) Since, it is known that g′ = 2 each other is 120°. 120º
 h B C
1 + 
 R 2 (a) Gravitational field reduces by 75%,
g means 25% is left.
Given, g′ = GM 1  GM 
64 =  2
where, g′ is the value of acceleration due to gravity at height h. r2 4 R 
h h or r = 2R
∴ 1 + = 8 or =7
R R ∴ h = r − R = 2R − R = R
∴ h = 7R = 44.8 × 10 6 m ≈ 45 × 10 6 m 3 (b) Gravitational field due to a solid sphere,
 h  d GM
9 (b) At depth, g′ = g 1 −  or g 1−  E (r ) = 3 r (∴ r ≤ R )
 R  R R
g′ 1  d  ⇒ E (r ) ∝ r, i.e. E (r ) increases inside the sphere,
⇒ = = 1 −  GM
g n  R E (r ) = 2 (∴ r > R )
r
 n − 1 1
or d =R  ⇒ E (r ) ∝ 2 , i.e. E (r ) decreases outside the sphere.
 n  r
Gravitation 479

4 (d) Intensity will be zero inside the spherical shell. x m m


Hence, = ⇒ x= ⋅d
1 d −x M m + M
E = 0 upto r = R and E ∝ 2 when r > R
r M
This is best shown in option (d). and d − x = ⋅d
m + M
5 (d) Inside a shell, field strength is zero. Therefore, force on a Since, gravitational potential between two bodies of masses M
particle is zero. and m respectively, is given by
5 Gm GM
6 (b) At distance 2.5 r or r V =− −
2 x d −x
m, 2r
Gm ( m + M ) GM ( m + M )
m, r =− −
m ⋅d M ⋅d
G
= − ( m + M )2
d
GM
13 (d) Gravitational potential, Vi = −
r
6.67 × 10 −11 × 100
G (m + m ) 8Gm Vi = −
E = 2
= 0.1
5  25r 2
 r 6.67 × 10 −9
2  Vi = − = − 6.67 × 10 −8 J
0.1
7 (b) Gravitational field at an external point due to spherical Q Vf = 0
 GM  ∴ Work done per unit mass,
shell of mass M is  2  while at an internal point is zero.
 r  W = ∆V = V ( f −Vi ) = 6.67 × 10 −8 J
(i) Point Q is external to shell M1, M2 and M3
14 (c) For hollow sphere,
So, field at Q will be
GM GM GM G − GM − GM − GM
E Q = 21 + 22 + 23 = 2 (M1 + M2 + M3 ) Vin = , Vsurface = ,Vout =
y y y y R R r
i.e. Gravitational potential remains constant inside the sphere
(ii) Field at P will be
and it is equal to potential at the surface. Its value increases
GM GM G
E P = 2 1 + 2 2 + 0 = 2 (M1 + M2 ) when the point moves away from the surface of sphere.
x x x
8 (c) For a uniform ring of mass M and radius R at its centre,
l CHECK POINT 10.4
GM 1 (c) When particles are released from rest their separation
the field is zero but potential is − .
R decreases. Therefore, gravitational potential energy of the
− Gm  r 
2 system decreases.
− GM − GM
9 (c) Vin = 3 −   , Vsurface = ,Vout = GMm
2R  R  R r 2 (b) Potential energy, u =
r
3 GM GM GM u
10 (c) Gravitational potential at centre,Vc = − At distance 2r, g = = =
2 R (2r )2 4r 2 4mr
GM u
Gravitational potential on surface,V = − Now, w = mg =
R 4r
3
∴ Vc = V 3 (b) On surface of earth, U = −
GMm
2 R
11 (d) Since, length of rod is equal to the circumference of At height h (<< R ), increase in potential energy is mgh.
semi-circle, ∴ The potential energy of body and the earth is given by
L
πR = L ⇒ R = Uh = −
GMm
+ mgh
π R
Therefore, the gravitational potential at the centre of circle GMm
will be 4 (c) Magnitude of gravitational potential energy, U =
r
GM πGM
V =− =− GMm  GM 
R L or U = 2 × r or U = g × mr Q g = 2 
r  r 
12 (d) E net = 0 U
∴ U = (mg )r or mg =
Gm GM r
∴ = , where x is distance from m.
x2 (d − x )2
480 OBJECTIVE Physics Vol. 1

5 (c) Here, AB = BC = CD = DA = 1m l CHECK POINT 10.5


D C 1 (c) Total mechanical energy of any closed system is always
1kg 1kg negative.
2 (b) The energy required to remove an object from the surface
of the earth to infinity is called binding energy of the object.
It is equal to negative of gravitational potential energy of the
1kg 1kg
A 1m B object.
GMm
Thus, binding energy = − E =
BD = AC = 12 + 12 = 2 m R
Total potential energy, GM gmR 2
But, g = 2 ⇒ GM = gR ⇒ BE = 2
= mgR
R R
 − G × 1 × 1  − G × 1 × 1  − G × 1 × 1
U= + + 3 (a) At v < v e , total energy is negative
 AB   BC   CD 
 − G × 1 × 1  − G × 1 × 1  − G × 1 × 1 v = v e , total energy is zero
+ + + and v > v e total energy is positive
 DA   BD   AC 
So, with reference to the above given cases, it can be said
 − G × 1 × 1  − G × 1 × 1
=4× +2 that when the escape velocity is given to a particle, its total
 1   2  energy is zero.
= − 5.4G 4 (c) Escape velocity is independent of mass of projectile which
6 (d) Change in potential energy in displacing a body from r1 to is projected. Therefore, escape velocity of body of mass
100 kg will also be 11.2 kms −1.
r2 is given by
 1 1  1 1 5 (a) Escape velocity is independent of angle from which
∆U = GMm  −  = GMm  −  (Q r1 = 2R, r2 = 3R) particle is projected. Therefore, when the body is projected in
r1 r2   2R 3R 
a direction making 30° angle to the vertical, then its escape
=
GMm velocity will remain same, i.e. 11.2 kms −1.
6R GM M
mgh 6 (a) v e = 2gR = 2 2
⋅ R or v e ∝
7 (c) Since, potential energy, ∆U = ,h =R (given) R R
 h
1 +  Mass is 1000 times and radius is 10 times that of earth,
 R
(1000M )G GM
mgR v e′ = ⇒v e′ = 10
∴ ∆U = 10R R
2
⇒ v e′ = 10v e ⇒ v e′ = 10 × 11.2 ⇒ v e′ = 112 kms −1
mgh
8 (c) Work done, W = ∆U =
h 7 (d) Escape velocity, v = 2gR
1+  
 R ve ge Re 6 10  Re 10 ge 6
Substituting h = 5R in above equation, we get ∴ = × = × Q = , = 
vm gm Rm 1 1  Rm 1 gm 1
mg × 5R 5mgR
∆U = = = 60
1+ 5 6
GMe m 8 (a) Escape velocity, v e = 2gR
9 (c) Initial gravitational potential energy, Ui = − …(i)
R where, g is acceleration due to gravity and R is radius.
1 GMe m
Final gravitational potential energy, U f = − ⋅ …(ii) R1 g
5 R Given, = K, 1 = g
R2 g2
GMe m GMe m
∴ Energy required, E = U f − Ui = − + v1 gR v1
5R R ⇒ = 1 1
= Kg ⇒ = (Kg )1/ 2
5GMe m − GMe m 4GMe m v2 g2R2 v2
= =
5R 5R 9 (c) Ui + Ki = U f + Kf
But, acceleration due to gravity ( g ) in terms of gravitational GMm 1 1
constant ( G ) is ⇒ − + m (2v e )2 = 0 + mv 2
R 2 2
GM
g = 2e …(iii) GM 1 2GM 8GM
R or − + 2v e 2 = v 2 ⇒ − + =v2
R 2 R R
GMe 4m 4m
∴ 2
× R2 × = g × R2 × [From Eq. (iii)] 6GM  2GM 
R 5R 5R ∴ v= = 3 
R  R 
4gmR 4mgR
= or
5 5 = 3(2gR ) = 3 v e
Gravitation 481

10 (a) Decrease in kinetic energy = Increase in potential energy l CHECK POINT 10.6
2
1  ve  mgh v e2 gh 1 (b) The gravitational force provides the centripetal force.
∴ m  = or =
2  2 1+
h 4 1+
h Therefore, the net force on the satellite will be F.
R R 2 (a) Since, orbital velocity of body close to earth can be given as
2gR gh R h
or = ⇒ = (Q v e = 2gR ) v o = rg = 6400 × 10 3 × 10 = 64 × 10 6 = 8 × 10 3 m s −1
4 h 2 1+ h
1+
R R v o = 8 km s −1
Solving this equation, we get h = R.
3 (b) Velocity v of a satellite varies inversely as the square root
Note Kinetic energy is half the value required to escape. Therefore, of radius R of the orbit.
speed is1 / 2 times the value required to escape. 1
i.e. v∝
11 (c) The kinetic energy required to project a body of mass m R
from the earth surface to infinity is given by vA RB R
1 1 ∴ = =
KE = mv e2 = m 2Rg vB RA 4R
2 2
3v 1
(Q Escape velocity, v e = 2Rg ⇒ v e2 = 2Rg ) ⇒ = ⇒ v B = 6v
vB 2
⇒ KE = mgR
GM 1
1 2 4 (c) Since, orbital velocity, v = or v ∝
12 (d) Initial kinetic energy of the body = mv r r
2
Here, v = 3 × 11.2 kms −1 Q
v2 r r
= 1 ⇒ v 2 = 1 ⋅v 1 =
R
⋅v =
2
v
1 1 v1 r2 r2 R 3
So, kinetic energy, KE = × m (3 × 11.2)2 = 9 × mve2 R+
2 2 2
1 5 (c) The time period of satellite in a circular orbit around a
As, mv e2 energy is used up in coming out from the gravitational
2 planet is independent of mass of the satellite.
1
pull of the earth, so final kinetic energy should be 8 × mv e2. 6 (d) From Kepler’s third law, T 2 ∝ R 3
2
1 1 T2
Hence, mv ′ 2 = 8 × mv e2 ∴ = constant
2 2 R3
∴ v ′ 2 = 8v e2
7 (b) The time period of revolution of satellite is given by
⇒ v ′ = 8 v e = 8 × 11.2 kms −1
r3 r3
T = 2π ⇒ T∝
13 (a) Decrease in kinetic energy = Increase in potential energy GM GM
1 2 mgh 2πr 3/ 2 2π 3/ 2
∴ mv = 8 (a) Time period of satellite, T = or GM = ⋅r
2 h
1+ GM T
R
 2π 
2gR ∴ ωr 3/ 2 = GM Q = ω
∴ v2 = ` (Given, h = R)  T 
2
GM  GM  GM gR 2
= gR = Q g = 2  ⇒ r3 = = 2
R  R  ω2 ω
GMm
GM 9. (d) Energy of a satellite is given by E = −
or v= 2r
R
For a satellite very close to earth, r = R
14 (d) Let h be the maximum height attained by the body, then GMm
from third equation of motion, v 2 = u 2 + 2gh ∴ E =−
2R
When u = 0, then
10 (d) Kinetic energy is negative of the mechanical energy, i.e.
v = 2 gh − E0.
v GMm
Given, v = e , where v e = 2gR 11 (d) As, kinetic energy, K =
3 2r
1 GMm
⇒ 2 gh = 2 gR Potential energy, U = −
3 r
On squaring both sides of equation, we get GMm
Total energy, E = −
R 2r
h=
9 If r is decreased, K will increase but U and E will decrease.
482 OBJECTIVE Physics Vol. 1

GMm m 7 (a) Considering Kepler’s second law, we know that,


12 (d) Energy of satellite is given by E = − or E ∝
2r r A L 2mA
= ⇒ L=
E A m A rB 3 4r  mA 3 rB 4r  T 2m T
⇒ = × = × Q = , = 
E B m B rA 1 r  mB 1 rA r 8 (d) Asteroids are also being acted upon by central
E A 12 gravitational forces, hence they are moving in circular orbits
⇒ = like planets and obey Kepler’s laws.
EB 1
9 (a) If they collide inelastically and stick together as wreckage,
13 (d) Total mechanical energy of satellite,
then
− GMm E m rQ
E = ⇒ P = P × KE = 0, only PE is present.
2r E Q m Q rP
 GMm  2GMm
∴ Total energy, E = U = 2 −  =−
E P 3 8r 24  r  r
⇒ = × =
EQ 1 r 1
10 (d) Change in potential energy in displacing a body from r1 to
14 (c) E = Energy of satellite − Energy of satellite on surface of r2 is given by
earth  1 1  1 1  GMm
∆U = GMm  −  = GMm  −  =
GMm  GMm   1 2
r r  2R 4R 4R
=− − − 
2(8R )  R 
11 (c) Esacpe velocity, v e = 2v o = 1.414v o
15 GMm 15  GM 
= = mgR Q = gR Further speed is to increase 2 times or orbital speed is to be
16 R 16  R 
increased by 41% and kinetic energy is to increase two times.
15 (d) Let r1 and r2 be the respective distance of two satellites GM dg −2GM
12 (b) We know that, g = 2 ⇒ =
from the earth’s surface. R dR R3
r1 = R + R = 2R and r2 = 7R + R = 8R dg −2GM 1
⇒ = ⋅
(Q R is the radius of the earth) dR R2 R
K=
GMm dg h
Now, ⇒ = −2   (Q dR ≈ h )
2r g  R
K1 r2
⇒ = =4 13 (d) If we assume the earth as a sphere of uniform density,
K2 r1
then it can be treated as point mass placed at its centre. In
GMm U1 r2 this case, acceleration due to gravity g = 0, at the centre.
U =− , = =4
r U 2 r1 It is not so, if the earth is considered as a sphere of
GMm non-uniform density, in that case, value of g will be different
and E =− at different points and cannot be zero at any point.
2r
E1 r2 14 (c) As the total energy of the earth satellite bounded system is
or = =4  − GM 
E 2 r1 negative   , where a is radius of the satellite and M is
 2a 
mass of the earth. Due to the viscous force acting on satellite,
(A) Taking it together energy decreases continuously and radius of the orbit or
height decreases gradually.
1 (c) Reason of weightlessness in a satellite is a zero reaction
force by satellite surface. 15 (b) As observed from the sun, two types of forces are acting
4 on the moon : one is due to gravitational attraction between
2 (c) g = πρGR ⇒ g ∝ dR ( Q ρ = d given in the problem) the sun and the moon and the other is due to gravitational
3 attraction between the earth and the moon. Hence, total force
3 (b) The satellite is revolving around earth because the on the moon is not central.
centripetal force is balanced by earth’s gravitational pull. If 16 (c) As observed from the earth, the sun appears to move in an
the gravitational pull disappears, the satellite is free of approximate circular orbit. The gravitational force of
centripetal force. attraction between the earth and the sun always follows
So, it will travel with its instantaneous velocity, i.e. in the inverse square law. Due to relative motion between the earth
direction tangential to the circular path. and mercury, the orbit of mercury as observed from the earth
will not be approximately circular, since the major
2π (r )3/ 2
4 (b) T = , so T is independent of m, the mass of satellite. gravitational force on mercury is due to the sun.
GM
17 (a) As the earth is revolving around the sun in a circular
dA vr sin θ 1
5 (c) According to Kepler’s second law, = = |r × v | motion due to gravitational attraction. The force of attraction
dt 2 2 will be of radial nature, i.e. angle between position vector r
1 2 1 and force F is zero.
6 (c) Here, KE = mv e = m (2gR ) = mgR
2 2 So, torque, | τ | = | r × F | = rF sin 0 ° = 0
Gravitation 483

k
18 (b) F = (where,k = constant) 28 (c) The initial position will become the perigee (nearest)
r position.
mv 2 k
∴ = or v ∝ r 0 ⇒ n = 0 So, the resulting orbit of the satellite will become elliptical
r r orbit with minimum distance from the earth equal to R.
19 (a) Applying conservation of mechanical energy, we get 29 (a) r = Rearth + h = (6.4 × 10 6 + 1.4 × 10 6 ) m
1 2 mgh
mv = ⇒ r = 7.8 × 10 6 m
2 1 + h /R 1/ 2
 4π 2r 3 
Substituting h = R, we get ∴ T =  = 6842 s
GM GMearth 
v = gR =
R GMearth
Speed of satellite, v = = 7162.93 ≈ 7163 ms −1
r
20 (a) Required additional velocity,
30 (b) Increase in kinetic energy = Decrease in potential energy
∆v = v e − v o = 4gR − 2gR
1 2 mgR mgR  mgh 
2GM ∴ mv = = Q ∆U = 
= (2 − 2 ) gR = ( 2 − 1) 2 1 + R /R 2  1 + h /R 
R
1 2 mgR
21 (a) At equator, g′ = g − R ω 2 = 0 ⇒ mv = ⇒ v = gR
2 2
g 2π g R 1 2 mgh
∴ ω= ⇒ = ⇒ T = 2π 31 (a) mv = ...(i)
R T R g 2 h
1+
R
22 (a) 0 = g − Rω 2 cos 2 60 °
On solving Eq. (i) for h, we get
ω2 =
4g
or ω = 2
g v2 R
or ∴ h= =
R R v 2
 2gR 
2g −  2  −1
2π g R R  v 
⇒ =2 ⇒T =π
T R g GM
GM
GM gR 2 32 (d) Equating gravitational field intensities, 81 =
23 (d) Orbital velocity, v o = = r2 (60R − r )2
r R+h
Solving above equation, we get
r1 + r2 60R − r = 9r ⇒ r = 6 R
24 (b) Semi-major axis, a =
2
GM
Now, T 2 ∝ a3 33 (d) L = mvr = m r = m GMr
r
3/ 2
r + r  ∴ L∝ r
⇒ T ∝ a 3/ 2 ∝  1 2  ⇒ T ∝ (r1 + r2 )3/ 2
 2  Q L1 = L, 
L1 r  
G (2Mm ) ⇒ = 1  r1 = r, 
25 (c) Force on B due to A = FBA = towards BA L2 r2  
(AB )2  r2 = 16r 
GMm L r L 1
Force on B due to C = FBC = towards BC ⇒ = ⇒ = ⇒ L2 = 4 L
(BC )2 L2 16r L2 4
As, (BC ) = 2AB
34 (c) Acceleration between them is zero, where force between
GMm GMm
⇒ FBC = = < FBA m m
2m and is equal to the force between m and or the net
(2AB )2 4(AB )2 5 5
Hence, m will move towards 2M. m
force on is zero. Suppose this point is P. Now, if velocity v 0
26 (c) As gravitational force provides the centripetal force to the 5
satellite to revolve around the earth, is less than the value necessary to cross point P, it will retrace
its path otherwise not.
v
35 (a) Work done,W = ∆U = U f − Ui = U f (QUi = 0)
R R
⇒ W = − (Gm1m 2 / R )
1
36 (c) We know that, v o = × v e …(i)
v 2
G ⋅ m ⋅ m mv 2 GMe 2GMe
= ⇒ v=
1 Gm Q vo = and v e =
4R 2 (R ) 2 R Re + h Re
484 OBJECTIVE Physics Vol. 1

2gR gh
Putting these values in Eq. (i), we get or = ...(i) (Qv e = 2gR )
8 1 + h /R
GMe 1 2GMe
= R
Re + h 2 Re On solving Eq. (i), we get h =
3
GMe 1 1 2GMe
⇒ = × ×
Re + h 2 2 Re 44 (a) Conservation of mechanical energy gives,
⇒ Re + h = 2Re 1 2  GM 3GM  GMm
mv = m  − +  =
⇒ h = Re or R 2  R 2R  2R
37 (d) At P, FP = 0 (Q Increase in KE = Decrease in PE)
GMm GM
At Q, FQ = 1 ⇒ v=
b2 R
G (M1 + M2 )m GM
At R, FR = Momentum, mv = m
c2 R
GMm GM
38 (a) Ei = E 0 = − ∴ Impulse required = m
2r R
GMm GMm
or E f = − =− 4GM −4GM 4 2GM
2(3r ) 6r 45 (b) Potential at centre = − = =−
r a/ 2 a
GMm 2
∴ W = E f − Ei = − = E0 M
a
M
3r 3
r =a/ √2
39 (c) E = Energy of satellite − Energy of satellite on surface of
m
earth O
GMm  GMm  Q h = 2R 
=− − −   M
2(3R )  R 
M
∴ total distance = 3R
−4 2GMm
5 GMm 5  GM  Potential energy =
= = mgR Q = gR a
6 R 6  R 
4 2GMm
Binding energy =
40 (c) W = E A − E ∞ = (U A + KA ) − (U ∞ + K∞ ) a
 1  ∴
1 2 4 2GMm
mv e =
∴ −3 = (1)VA + × (1)(2)2 − (0 + 0 ) (Q W = − 3J)
 2  2 a
∴ VA = −5 J kg −1 or
8 2GM
ve =
 1 − 3 a
mv 2 Gm1m 2  
 2 
41 (b) As, = or v ∝ r 46 (a) E = ∆U =
mgh
=
mghR
r r3 1 + h /R (R + h )
2πr r
Now, T= or T ∝ E′ = Energy of satellite − Energy of satellite on surface
v v
1+ 3 of earth
r
∴ T∝ ⇒ T ∝r 2 = r2 GMm  GMm 
 1 − 3 =− − − 

 2 
 2(R + h )  R 
r
1 1  mg (R + 2h )R
42 (a) As, gravitational force provides the necessary centripetal = mgR 2  − = (Q GM = gR 2 )
force  R 2(R + h )  2(R + h )
mv 2 E 2h
∝ r −n Now, =
r E ′ (R + 2h )
⇒ v ∝ r (1 − n )/ 2 47 (a) From the figure,
2πr Gm 2  Gm 2 
Also, T = or T ∝ rv −1 FA = FAB + FAC = 2  2  cos 30 ° =  2 ⋅ 3 
v  a   a 
or T ∝ r ⋅ r (n − 1)/ 2 a
Also, r=
or T ∝ r (n + 1)/ 2 3
43 (b) Applying conservation of mechanical energy, mv 2 mv 2 3 Gm 2
2
Now, =| FA| or = 2 3
1 v e  mgh v 2 gh r a a
m  = or e =
2  2 1+
h 8 1+
h
∴ v=
Gm
R R a
Gravitation 485

GMm GMm
48 (d) F1 = =
(2R )2 4R 2  M 3F = mr ω 2
G   (m )
GMm  8  Gmm  a
F2 = F1 − Fcavity = − 3 2  = m ⋅ω 2
4R 2 3 
2
 a  3
 R
2  3Gm
∴ ω=
⇒ F2 =
7GMm a3
36R 2 54 (c) Time period of simple pendulum,
F2 7
∴ = l 1
F1 9 T = 2π or T ∝
g g
 h  R − h g
49 (d) a = g′ = g 1 −  = g   = ⋅y (Q R − h = y ) g2 2
 R  R  R Q
T1
= =
g
⇒ 2 =4 ...(i)
T2 g1 1 g1
 GM 
 2
R  G 4  4 ∴ v e = 2gR or v e ∝ gR
= ⋅ y = 3  πR 3ρ ⋅ y = πρGy
R R 3  3
v e1 g1 R1
⇒ = ⋅ = 1/ 2
50 (c) Field strength is uniform inside the cavity. Let us find its v e2 g2 R2
value at centre.
1 R1 1  g 
ET = E O + E C (T = Total, O = Remaining, C = Cavity) ∴ × =  from Eq. (i), 2 = 4
GM R GM 4 R2 2  g1 
∴ E O = ET − E C = 3 −0= 2
R 2 2R R1
Hence, =1
GMm R2
∴ F = mFR = 4 
2R 2 G  πR 3ρ
Gm 3 
1 Further, g= 2 = or g ∝ Rρ
51 (a) µv r2 = Ui − U f R R 2
2
g1 R1 ρ1  1 ρ ρ 1
mM ∴ = ⋅ or   = (1) 1 or 1 =
Here, µ = reduced mass = g2 R2 ρ 2  4 ρ2 ρ2 4
M+m
1  mMv r2   mgh 
 GMm  55 (b) E1 = ∆U =  
∴   =0− −   1 + h /R 
2  m + M  d 
2G (M + m ) E 2 = Energy of satellite–Energy of satellite on surface of earth
Hence, relative velocity, v r = GMm GMm
d =− +
2(R + h ) R
1 
52 (d) U 2 = 2  mv o2
2   1 
At ∞ = mgR 1 −  (Q GM = gR 2 )
 2 (1 + h /R ) 
 2h 
mgR  + 1
R 
⇒ E2 =
 h 
− U1 −U2 2 1 + 
PE = 0  R 
1 2 mv e2  h
Now, mv = − U 2 + U1 = U1 − U 2 = − mv o2 2 1 +  Q h < < R 
2 2 E1 mgh  R  2h  
∴ = × =
E 2 1+ h ∴ 1 + 2h ≈ 1
∴ v = v e2 − 2v o2 mgR R
 
R R
AB
53 (b) In right angled ∆AOB, cos 30° = 56 (c) Acceleration of supercondense material = g 2m
OB
F
But acceleration of earth = 2g (towards the mass)
∴ Relative acceleration = 3g F
1 m
O Since, we know that, h = ut + a rt 2
2
r
B a/2
But the body starts from rest.
A
So, u=0
1 1 2h
Now, h = a rt 2 or h = (3g )t 2 or t =
3 a a 2 2 3g
⇒ = ⇒r=
2 2r 3
486 OBJECTIVE Physics Vol. 1

57 (d) Side of square, r = 2R cos 45° = 2R F1 g1 r1


If r1 < R and r2 < R, then = =
F1 F2 g2 r2
F2 2
F1 g1  r2 
If r1 > R and r2 > R, then = = 
F2 g2  r1 
R F1 61 (b) Work done,W = U 2 − U1 = mv 2 − mv1
45º
 Gm 2 Gm1   Gm1 Gm 2  
r = m  − −  − − − 
 R 2R   R 2R  
Mv 2
Now, 2F1 cos 45° + F2 = Gm (m1 − m 2 ) 2Gm 2
R = ( 2 − 1) = (1 − 2 )
2R R
R
⇒ v= ( 2 F1 + F2 ) GmdM dX
M 62 (b) dF =
x2 m
R  GMM GMM  x=a+l
⇒ v= 2 + ∴ F =∫ x
M  (2R )2 
dF
( 2R )2 x=a
dM

 2 2 + 1 M 
GM G ⋅ m  ⋅ dx
=   a+l  l  4GmM
R  4  =∫ =
a − l /2 x2 4a 2 − l 2
GMm
58 (a) The initial potential energy (Ui ) of object, Ui = − 63 (c) The force on particle for varying g is given by
3R
GMm GMmx
Final potential energy, U f = − F = = mω 2x
R R3
By law of conservation of energy, ∆KE = − ∆PE GM
⇒ ω2 = 3
1 2 R
⇒ mv = − (U f − Ui ) = Ui − U f 2
2  2π  GM  2π 
  = 3 Q ω = 
1 2 GMm GMm  t  R  t 
⇒ mv = − +
2 3R R R3
1 2 2GM ⇒ t = 2π
⇒ v = GM
2 3R
When g is constant,
4GM GM
⇒ v= =2 1
s = gt ′ 2 (For initial velocity, u = 0)
3R 3R 2
59 (a) Using conservation of angular momentum, 1  GM   GM 
⇒ R =  2  (t ′ )2 Q g = 2 
mv prp = mv ara 2 R   R 
As velocities are perpendicular to the radius vectors at apogee
2R 3 R3
and perigee. (t ′ )2 = ⇒t′ = 2
⇒ v prp = v ara GM GM
Using conservation of energy, t 2π R 3 / GM 2π
∴ = = = 2π
GMm 1 2 −GMm 1 2 t′ 2 R 3 / GM 2
− + mv p = + mv a
rp 2 ra 2
By solving the above equations,
2GMra
(B) Medical entrance special format
vp = questions
rp (rp + ra )

2GMrpra l Assertion and reason


Hence, L = mv prp = m dA L mvr sinθ vr sinθ
(rp + ra ) 3 (d) = = constant = =
dt 2m 2m 2
4
60 (a) As, g = πρGr i.e.
dA
is independent of m.
3 dt
∴ g ∝ r , if r < R
4 (d) Gravitational potential due to a point mass at some finite
GM
g= 2 distance is always negative and non-zero.
r 2π 3/ 2
1 5 (c) T = r . Time period does not depend upon the
∴ g ∝ 2 , if r > R GM
r radius of earth (R ) but it depends upon its mass (M).
Gravitation 487

l Statement based questions As r increase,V increase and E decreases.


Therefore, potential at A will be less than B. Gravitational
1 (d) Acceleration due to gravity at altitude h, field inside the shell will be zero. As one moves from C to D,
g  2h 
gh = ≈ g 1−  ... (i) the gravitational potential remains constant and when one
(1+ h /R )2  R moves from D to A, gravitational field decreases.
At depth d, Hence, A → q, B → t, C → r, D → s.
 d
gd = g 1−  ....(ii) GM
2 (d) As g = 2 ,V = −
GM
 R R
R
g g GM
In both Eqs. (i) and (ii), if h and d increases, g decreases. At height h = R, g′ = 2
= ⇒ V′ = −
At latitude φ, g φ = g – ω 2R cos 2 φ  h 4 2R
1 + 
 R 
As, φ increases, gφ also increases.
i.e. g decrease by a factor 1/4 andV′ also decreases by a factor of
Thus, option (d) is correct but rest are incorrect. 1/2.
GM 2GM R
2 (a) As we know that, g = 2 , v e = At depth h = ,
R R 2
−GMm  h  1 g
U= g′ = g 1 −  = g 1 −  =
and  R  2 2
R
GM GM  1  11 GM
∴ g∝
M
, ve ∝
M
and U ∝
M V′ = − . R 2 − 0.5r 2 ) = − 3 15
(15 . R 2 − R 2 = −
R2 R R R3 R  8  8 R
1
If both mass and radius are increased by 0.5%, then escape i.e. g decrease by a factor andV′ also decreases by a
velocity and potential energy remains unchanged, whereas g 2
11
decreases by 0.5%. factor .
8
Thus, option (a) is correct but rest are incorrect.
Hence, A → p, B → q, C → q, D → r.
3 (b) A geo-stationary satellite have same rotation as that of
earth, i.e. in west-east direction 3 (c) In the figure given below, it can be seen that at perigee
position planet is nearest to the sun. So, as the planet moves
in the elliptical orbit from apogee position to perigee position,
Earth
then speed of planet increases, distance of planet from centre
of sun decreases, potential energy decreases. Angular
momentum about centre of sun remains the same.
E
Geo-stationary
satellite Perigee
Sun r2 Apogee
4 (d) Gravitational mass of proton is equivalent to its inertial A B
mass and is independent of presence of neighbouring heavy r1 c a
objects.
5 (a) For the gravitational force between an extended object D
(like the earth) and a point mass, the Newton’s universal law
of gravitation is not directly applicable. Hence, A → r, B → q, C → q, D → p.
Thus, Statement I is correct but Statement II is incorrect.

6 (a) Time period of satellite =


2π (RE + h )3/ 2 (C) Medical entrance gallery
GME 1 (a) Given, w = mg = 72 N (on the surface of earth)
From the above equation, it is clear that the time period of  R
At height equal to half of radius of the earth, i.e. h = 
satellite depends on the mass of earth (ME ), radius of orbit  2
(r = RE + h ) and height of the satellite from the surface of
Acceleration due to gravity,
earth (h). 2 2
The geo-stationary satellite can have circular or elliptical orbit.  R   R   4R 2 
g′ = g   = g  = g 2
Thus, only Statement I is incorrect.  R + h  R + R /2  9R 
4
l Match the columns ⇒ g′ = g
GM 9
1 (c) Inside a shell, gravitational potential,V = − = constant
R Gravitational force on body at height h,
and gravitational field intensity, E = 0 4 4
F = mg′ = mg = × 72 = 32 N
GM GM 9 9
Outside the shell,V = − and E = 2
r r or w′ = 32 N
488 OBJECTIVE Physics Vol. 1

2 (b) Radius of earth = R Here, h = R (Given)


GMm
Let at depth d, acceleration due to gravity be gd which ⇒ U2 = − … (ii)
g 2R
becomes . Thus, the change in potential energy, ∆U = U 2 − U1
n
Substituting the values from Eqs. (i) and (ii), we get
g  d g
i.e. gd = ⇒ g 1−  = GMm GMm
n  R n ∆U = − +
2R R
d 1 1 d
⇒ 1− = ⇒ 1− = GMm gR 2m  GM 
R n n R = = Q g = 2 
2R 2R  R 
n −1 d  n − 1
⇒ = ⇒ d = R mgR
n R  n  =
2
3 (c) Given, weight of the body, w = 200 N Thus, the work done in raising the mass to a height R is
As we know w = mg, where m is the mass of the body and equals to
mgR
.
g (≈ 10 ms −2) is acceleration due to gravity of the body at the 2
surface of the earth. 5 (a) From Kepler’s third law, the time period of revolution of
Since, mass m remains constant irrespective of the position of satellite around earth is
the body on the earth. However, g is not constant and its T 2 ∝ r 3 or T ∝ r 3/ 2 …(i)
value at a depth d below the earth’s surface is given as
where, r is the radius of satellite’s orbit.
 d
g′ = g 1 −  … (i) Here, r1 = 6RE + RE , T1 = 24 h
 R
r2 = 2 .5RE + RE , T2 = ?
where, R is the radius of the earth.
So, from Eq. (i), we get
Multiplying m on both sides of Eq. (i), we get 3/ 2
T1  r1 
 d = 
mg′ = mg 1 −  T2  r2 
 R
3/ 2
24  6RE + RE 
3/ 2
 R  7
Now, the weight of the body at half-way down i.e. d =  to =  = 
 2 T2  2 .5RE + RE   3.5
the centre of the earth is 24 24 12
⇒ T2 = = = =6 2h
 R / 2 (2 )3/ 2 2 2 2
mg′ = 200 × 1 − 
 R 
6 (c) Given, radius of earth, Re = 6400 km = 6.4 × 10 6 m
 1 1
= 200 1 −  = 200 × = 100 N Distance from centre of earth, r = 2000 km
 2 2
Distance from the surface of earth,
∴The body will weigh 100 N half-way down to the centre of
the earth. d = Re − r = 6400 − 2000
= 4400 km = 4.4 × 10 6 m
4 (b) As we know, the potential energy of body of mass m on
the surface of earth, If g′ be the gravitational field below the surface of earth at a
GMm depth d, then
U1 = − … (i)
R  d  4.4 × 10 6 
where, G = gravitational constant, g′ = g 1 −  = 9 . 8 1−  (Q g = 9.8 ms −2)
 Re   6.4 × 10 6 
M = mass of earth and R = radius of earth.
= 9.8 × 0.3125 = 3.06 ms −2
When the mass is raised to a height h from the surface of the
earth, then the potential energy of the body becomes, 7 (a) Given, escape velocity on the surface of earth,
v e = 11. 1km/h
h =R
i.e. v e = 2gRe
R where, Re is radius of earth.
M 2 GMe  GM 
As, ve = Q g = 2 e 
Re  Re 
where, Me is mass of earth.
GMm M
U2 = − Mass of moon, Mm = e
(R + h ) 81
Gravitation 489

Re The speed of B relative to A when they are close,


Radius of moon, Rm =
4 v BA = v A − v B = 2π × 10 4 − π × 10 4
∴Escape velocity on the surface of moon, = π × 10 4 km/h
Me GMm
2G 10 (d) As force, F = = mω 2r
vm =
2 G Mm
= 81 r 3/ 2
Rm Re GMm 4π 2mr  2π 
⇒ = Q ω = 
4 r 3/ 2 T2  T
2 2 GMe 2 2GMe 2
= = = ve  4π 2  5/ 2
9 Re 9 Re 9 ⇒ T2 =   ⋅r ⇒ T 2 ∝ r 5/ 2
 GM 
2
= × 11. 1 = 2 . 46 km h −1 11 (a) The acceleration due to gravity at a latitude λ is
9
g′ = g − Rω 2 cos 2 λ
8 (b) According to the question,
vC At the poles, λ = 90 °
vB
B ∴ g′ = g = gp (Q cos 90 ° = 0 )
S
A C For λ = 60 °, g′ = gp − Rω 2 cos 2 λ

vA
or g′ = gp − Rω 2 cos 2 60 °
2
 1 1
The figure above shows an ellipse traced by a planet around Hence, g′ = gp − Rω 2   = gp − Rω 2
 2 4
the sun S. The closest point A is known as perihelion and the
farthest point C is known as aphelion. 12 (b) Given, Me = 5.98 × 10 24 kg, Mm = 7.35 × 10 22kg and
Since, as per the Kepler’s second law of area, the planet will
r = 3.855 × 10 8 m
move slowly (vmin ) only, when it is farthest from the sun and
more rapidly (vmax ), when it is nearest to the sun. Let x be the distance of the point from the centre of earth,
Thus, v A = vmax , v C = vmin where gravitational intensity is zero. Therefore,
Therefore, we can write , v A > v B > v C GMe GMm
…(i) =
Kinetic energy of the planet at any point is given as x 2
(3.855 × 10 8 − x )2
1 x Me 5.98 × 10 24
K = mv 2 or = = ≅9
2 3.855 × 10 − x
8
Mm 7.35 × 10 22
1
Thus, at point A, KA = mv A2 or
x
+ x = 3.855 × 10 8
2 9
1
At point B, KB = mv B2 or x = 9 × 3.855 × 10 8 / 10 = 3.469 × 10 8 m
2
1 Distance from moon = 3.855 × 10 8 − 3.469 × 10 8
At point C, KC = mv C2
2 = 3.86 × 10 7 m
From Eq. (i), we can write ≅ 3.85 × 10 7m
KA > KB > KC 13 (b) In gravitation free space, there is no external gravity. Due
9 (d) Given, TA = 1h, TB = 8h to masses of the astronauts, there will be small gravitational
attractive force between them. Thus, these astronauts will
and rA = 10 4 km move towards each other.
TA2 rA3 12 (10 4 )3 14 (d) Acceleration due to gravity at height h above earth’s
From Kepler’s law, 2
= 3⇒ 2=
TB rB 8 rB3 surface,
 2h 
⇒ rB3 = 64 × (10 4 )3 gh = g 1 −  (Q h << R )
 R
∴ rB = 4 × 10 4 km Acceleration due to gravity at depth d below earth’s surface,
 d
2π rA 2π × 10 4 gd = g 1 − 
Speed of satellite A, v A = =  R
TA 1
Given, h = 1km and gd = gh
= 2π × 10 4 km/h
 d  2h 
2π rB 2π × 4 × 10 4 or g 1 −  = g 1 − 
Speed of satellite B, v B = =  R  R
TB 8
⇒ d = 2h or d = 2 km
= π × 10 4 km/h
490 OBJECTIVE Physics Vol. 1

GM 18 (b) Acceleration due to gravity at a depth d below the surface


15 (a) The orbital velocity of spaceship in circular orbit, v o =
r of the earth is given by
 d
If spaceship is very close to earth surface, then r = radius of gdepth = gsurface 1− 
 R
earth (R ).
R 2g Also, for a point at height h above surface of the earth,
GM  GM 
∴ vo = = = Rg  Since, g = 2   R2 
R R  R  gheight = gsurface  
 (R + h )2 
Here, R = 6400 km = 6.4 × 10 6 m and g = 9.8 ms −2
∴ v o = 6.4 × 10 6 × 9. 8 Therefore, we can say that value of g increases from centre of
earth and it is maximum at its surface and then decreases
= 7.92 × 10 3 m/s = 7.92 km s −1 with height.
The escape velocity of spaceship, Hence, graph given in option (b) is correct.
v e = (2Rg ) = 2 v o = 2 × 7.92 = 11.20 km s −1 19 (b) Total energy of a satellite at height h is
∴Additional velocity required GMm GMm
= KE + PE = −
= 11.20 − 7.92 = 3.28 km s −1 2 (R + h ) (R + h )

Therefore, velocity of 3.28 kms −1 must be added to orbital − GMm − GMmR 2 − mg0R 2  GM 
= = = Q g0 = 2 
velocity of spaceship to overcome the gravitational pull. 2(R + h ) 2R 2 (R + h ) 2 (R + h )  R 
16 (d) From conservation of energy, 20 (d) Gravitational potential at some height h from the surface
1 2 GMem 1 2 GMem of the earth is given by
mv1 − = mv 2 − GM
2 R 2 R+H V =− …(i)
R+h
Here, v 2 = 0 (At maximum height)
and acceleration due to gravity at some height h from the
1 2 GMem GMem
∴ mv1 − =− surface of the earth can be given as
2 R R+H GM
g= …(ii)
Solving, we get (R + h )2
R R
H= = 2 (Qv e = 2gR ) From Eqs. (i) and (ii), we get
2gR v
−1 e −1 |V | GM (R + h )2 |V |
v12 v12 = × ⇒ =R + h …(iii)
g (R + h ) GM g
where, v e is escape velocity.
1 ve Q V = − 5.4 × 10 7 J kg −2, g = 6 ms −2
Here, v1 = v e ⇒ =3
3 v1 and R = 6400 km = 6.4 × 10 6 m
R R Substituting these values in Eq. (iii), we get
∴ H= =
9−1 8 5.4 × 10 7
= 6.4 × 10 6 + h
17 (d) Acceleration due to gravity, 6
GM 4  M 4  ⇒ h = (9 − 6.4) × 10 6 = 2.6 × 10 6 m ⇒ h = 2600 km
g = 2 = πρGR Q ρ = andV = πR 3
R 3  V 3 
21 (a) Since, the escape velocity of the earth can be given as
g1 ρ1R1
∴ = …(i)  GM  8
g2 ρ 2R2 v e = 2gR = 2 2  R = R πGρ
R  3
w 4 
Atmospheric pressure can be given by p = …(ii) [Q mass of earth, m = ρ  πR 3 ]
S 3 
where, w = weight of atmosphere
S = surface area of the planet. 8
and ⇒ ve = R πGρ …(i)
From w = mg, we get 3
m1 w1g2 As it is given that, the radius and mean density of planet are
= twice as that of the earth. So, escape velocity at planet will be
m 2 w 2g1
8
m1 p1 S1g2 v p = 2R πG 2ρ …(ii)
∴ = [From Eq. (ii)] 3
m 2 p 2 S2g1
On dividing Eq. (i) by Eq. (ii), we get
p1 ⋅ (4πR12 ) ρ 2R2
= ⋅ [From Eq. (i)] 8
p 2 (4πR22 ) ρ1R1 R πGρ
ve 3 v 1
= ⇒ e =
p1R1 ρ 2 vp 8 vp 2 2
= 2R πG 2ρ
p 2R2 ρ1 3
Gravitation 491

22 (b) The gravitational force of attraction between the planet 28 (b) The escape velocity is given by
and sun provides the centripetal force,
2GM
GMm mv 2 GM ve = (By putting value)
i.e. = ⇒ v= R
r2 r r
The time period of planet will be 2 × 6.67 × 10 −11 × 7.4 × 10 22
ve = ~ 2.4 kms −1

T=
2πr 1740 × 10 3
v
29 (b) Consider the diagram as shown below.
4π 2r 2 4π 2r 3
⇒ T2 = = …(i)
GM GM
16 kg 4 kg
r
P
Also, from the Kepler's third law, T 2 = Kr 3 …(ii)
30 m
From Eqs. (i) and (ii), we get x
4π 2r 3 Let P be a point at a distance x from 16 kg sphere, where the
= Kr 3 ⇒ GMK = 4π 2
GM net gravitation field (and hence, force) due to two spheres is
zero, then
23 (c) Let the smaller body cover a distance x before collision,
16 G 4G
then − =0
5M x 2
(30 − x )2
x M 4 1
⇒ =
2R x 2 (30 − x )2
R
⇒ 3600 + 3x 2 − 240 x = 0
9R
⇒ x 2 + 1200 − 80 x = 0
12R
Mx = 5M (9R − x ) 80 ±
(− 80 )2 − 4 × 1200 × 1
⇒ x=
45R 2
or x = 45R − 5x or x = = 7.5 R
6 80 ± 1600 80 ± 40
⇒ x= = = 60 m or 20 m
24 (a) At the pole, the weight is same as the actual weight. 2 2
As, x = 60 m in not possible, so x = 20 m
Thus, 100 N = m (10 ms −2) ⇒ m = 10 kg
At the equator, the apparent weight is given by 30 (b) Orbital velocity of earth, v o = gR , so the orbital velocity
mg′ = mg − m ω 2 R of earth satellite does not depend on mass of the satellite.

Also, the angular speed of an equatorial point on the earth's 31 (d) Gravitational potential energy of a body of mass m at a
GMm
surface is height h above the surface of earth is given by − .
(R + h )
ω = (2π / 24 × 60 × 60 )
⇒ ω = 7.27 × 10 −5 rad/s 32 (c) By Kepler’s third law of planetary motion, T 2 ∝ r 3
mg′ = 100 − 10 (7.27 × 10 −5 )2 × 6.4 × 10 6
2 3
Hence, now,  T1   r1 

~ 99.66 N T  = r 
 2  2
25 (d) The gravitational field at the centre of a solid sphere of 33 (d) Mass of a body is its inherent property. It is independent
mass M is zero. of location of the body. Thus, mass of the body remains same
26 (b) The value of g at a depth d from the earth's surface, (i.e. M), on the surface of the earth or on the surface of the
moon.
 d 
g = gE 1 −  34 (b) The dependence of intensity of graviational field (E ) is as
 RE 
shown in the figure.
where, gE is acceleration due to gravity at the earth’s surface E
and RE is the radius of earth.
 5   6395 −2
⇒ g = 9.8 1 −  = 9.8   = 9.79 ms
 6400   6400  E∝r E ∝ 12
r
27 (c) The gravitational force between the particles will give the
O
required centripetal force. r=R
GM 2 Mv 2 GM 2 Mv 2 GM
So, 2
= ⇒ = ⇒ v= 35 (c) As the work done by the gravitational force in closed path
(2R ) R 4R 2 R 4R is zero, so it is conservative in nature.
492 OBJECTIVE Physics Vol. 1

36 (a) As we know that, 41 (a) We know that, the total energy of the satellite,
Time period of revolution of earth at initial condition, 1 GMem
E =−
Re 2 R
T = 2π …(i)
g At a height H from the surface of the earth, the total energy
1 G Mem
So, if radius is reduced, R′ =
Re of satellite, E =− …(i)
4 2 (R + H )
R and the potential energy of the satellite,
Time period becomes T ′ = 2π e …(ii)
4g G Mem GMem
U =− =− …(ii)
On dividing Eq. (i) by Eq. (ii), we get (R + 3R ) 4R

2π Re / g 1 GMem GMem
T From given condition, E = U ⇒ − ⋅ =−
= =2 2 (R + H ) 4R
T′ 2π Re / 4g
1 1
T 24 ⇒ = ⇒ 2H = 4R − 2R ⇒ H = R
⇒ T′ = = = 12 h 2R + 2H 4R
2 2
− GMm 42 (b) Change in potential energy of body,
37 (d) Potential energy at the earth surface =
R GMm  GMm 
∆U = − − − 
Potential energy at a height h above the earth’s surface (R + 2R )  R 
− GMm
= …(i) GMm GMm 2GMm 2  GM 
(R + h ) =− + = = mgR Q g = 2 
3R R 3R 3  R 
− GMm  GMm 
∴ Change in potential energy, ∆PE = − −  43 (d) The resulting gravitational potential,
(R + h )  R 
GMmh 1 1 1 1 
= …(i) V = − 2G  + + + + K
R (R + h )  1 2 4 8 
Substituting the value of h = 10R in Eq. (i), we get  1 1 1 
⇒ V = − 2G 1 + + 2 + 3 K
GMm × 10R 10 GMm  2 2 2 
∆PE = =
R (R + 10 R ) 11R −1
 1
Using Binomial expansion,V = − 2G 1 + 
38 (a) We know that, the potential energy of the satellite,  2
GMem 2G −2G
U =− ⇒ V =− = = − 4G
Re  1  1
1 −   
1 GMem  2  2
The kinetic energy of the satellite, K =
2 Re GM
44 (a) Value of acceleration due to gravity is g =
GMem 1 GMem 1 GMem R2
The total energy, E = U + K = − + =− At poles, R is minimum and at equator R is maximum, hence
Re 2 Re 2 Re
GMem g is maximum at the poles.
⇒ 2E = − ⇒ − 2E = U
Re 45 (b) Escape velocity, v e = 2gR
So, potential energy = − 2 (total energy) 46 (a) Here, g0 = g′ − Rω 2
⇒ PE = − 2E g
Given, g0 = and g′ = g
39 (c) Given, Re = 6400 km = 6.4 × 10 6 m, π = 3.14, g = 10 ms −2 2
g g
We know that, the period of revolution of the earth satellite, So, = g − Rω 2 ⇒ ω =
2 2R
(Re + h )3 r 2π  v
T = 2π [if h << Re , then (Re + h = Re )] We have, T = 2π   ⇒ T = Q ω = 
gRe2 v  ω  r
Re3 6.4 × 10 6 2R
So, T = 2π
R
= 2π e = 2 × 3.14 Hence, T = 2π
gRe2
g 10 g
A
= 5.024 × 10 3 = 5024 s 47 (c) In central force motion, the angular
r1
= 83.73 min momentum of body remains conserved.
O B
40 (a) Given, H = 35800 km which is height of geo-stationary i.e. Angular momentum, L = r × p = mvr r2

orbit. (At minor and major axis, v and r are


perpendicular)
Therefore, T = time period of earth’s satellite at this height
So, if v1 and v 2 be the speed of earth at positions A and B,
= 24 h
respectively.
Gravitation 493

Then, r = mv 2r2
mv11 − Gm1  − Gm 2
⇒ VA = − 
v1 r2 OB v R  2R 
⇒ = = ⇒ 1 =R
v 2 r1 OA v2
Gm (m1 − m 2 )
WA→ B = m V
( B −VA ) = ( 2 − 1)
48 (a) We know that gravity, acceleration due to gravity, 2R
4
g = πGRd 55 (a) According to Kepler’s third law,
3
Here, given densities be d1 and d 2 and radii be R1 and R2. T 2 ∝ r 3 ⇒ T ∝ r 3/ 2
4
πGRd 2π
g1 1 1 g Rd We have, ω= ⇒ ω ∝ r −3/ 2
So, = 3 ⇒ 1= 11 T
g2 4 πGR d g2 R2d 2
2 2 Now, L = mr 2ω ⇒ L ∝ r 2 and L ∝ ω
3
⇒ L ∝ r 2 ⋅ r −3/ 2
49 (d) Weight of the object at height h above the surface of the
w w ∴ L∝ r
earth, w ′ = ⇒ w ′ (h = R / 2) =
 h
2
(1+ 1/ 2)2 56 (c) The orbital velocity of a satellite moving around the earth,
1+ 
 R v o = gr
4 4
⇒ w′ = w × = 90 × = 40 kg The escape velocity is given by v e = 2gr
9 9
∴ Increase in velocity = 2gr − gr
50 (c) Using energy conservation law,
(PE )i + (KE )i = (PE )f + (KE )f = ( 2 − 1) gr = 0.414 gr
− GMm 1 1 So, percentage increase in velocity
+ m (2v e )2 = 0 + m (v ′ )2
R 2 2 0.414
= gr × 100 = 41.4%
1  1 1  GM v e2  gr
⇒ − m  v e2 + m (2v e )2 = m (v ′ )2 Q = 
2  2 2  R 2
57 (b) The energy of artificial satellite at the surface of the earth,
⇒ v ′ = 3v e = 3 × 11.2 = 19.4 km/s GMm
E1 = −
51 (c) Angular momentum = Linear momentum × Perpendicular R
distance from the axis of rotation When the satellite is intended to move in a circular orbit of radius
= Mass of satellite × Orbital velocity × Radius of earth 7R, then energy of artificial satellite,
1 GMm
GM E2 = −
=m × × R0 = m GMR0 2 7R
R0
The minimum energy required,
GMem
52 (a) Force due to particle, F = GMm 1  GMm 
Rp2 E = E1 − E 2 = − +  
R 2  7R 
GMpm 4 GMpm −14 GMm + GMm
Here, F = 2
⇒ F = =
(Dp /2) Dp2 14R
F = mg 13 GMm
Also, =−
F 4 GMp 14R
∴Acceleration due to gravity, g = =
m Dp2 58 (a) Using the law of conservation of angular momentum,
Angular momentum of planet at p
53 (c) From Kepler’s third law, T 2 ∝ r 3 = Angular momentum of planet at a
Hence, T12 ∝ r13 and T22 ∝ r23 i.e. Lp = La
T22 r23 (3R )3 T2 1 But ra > rp . So, ra × L a > rp × L p
So, 2
= 3= 3
or 22 =
T1 r1 (6 R ) T1 8 59 (b) Maximum height attained by body,
1 2  v2 
⇒ T22 = T1 h = R  2 2
8 v e − v 
24
⇒ T2 = =6 2h  v 2/4   v 
2 2 ⇒ h =R  2 e 2  Q v = e 
v e − v e / 4   2
54 (b) VB = Potential at B due to A + Potential at A due to B
− Gm 2  − Gm1  v 2/4  R
VB = −  = R  e2  ⇒ h =
R  2R   3v e / 4  3
494 OBJECTIVE Physics Vol. 1

GM
60 (b) Acceleration due to gravity, g′ =
(R + h )2
g′ = g − ω 2R cos 2 λ
g
⇒ 0 = g − ω 2R cos 2 60 ° Given, g′ = 1% of g =
100
ω 2R
0=g− ⇒
g
=
GM
4 100 (R + h )2
g 10
⇒ ω=2 =2 (R + h )2 GM (R + h )2  GM 
R 6400 × 1000 = or = R2 Q g = 2 
100 g 100  R 
1
Angular velocity, ω = = 2.50 × 10 −3 rad s −1 ∴ R + h = 10R
400
61 (a) Gravitational pull depends upon the acceleration due to h = 9R
gravity on that planet. where, R is the radius of the earth.
1 1
Mm = Me , gm = ge 63 (c) As we know,
81 6
4π 2R 3
GM From Kepler’s law, time period of satellite is T 2 =
By the relation, g = 2 GM
R
1/ 2 (R + h )2
Re  Me gm 
1/ 2
 1 Tsat = 2π
∴ = ×  =  81 ×  GMe
Rm  Mm ge   6
9 As two satellites of masses m and 9m are orbiting a planet of
∴ Re = Rm radius R, their time periods will remain same, i.e. in the ratio
6
1 : 1, because time period of satellite does not depend on its
62 (b) Acceleration due to gravity at a height h from earth’s mass.
surface,
CHAPTER
11

Simple Harmonic
Motion
In this chapter, we are going to study periodic and a basic type of oscillatory
motion called simple harmonic motion and its various aspects with the help of
some fundamental concepts like period, frequency, amplitude and phase, etc.

PERIODIC AND OSCILLATORY MOTION


On the basis of path traced and various other properties, motion can be classified
into several categories.
Two main categories of motion are given below

Periodic motion
A motion which repeats itself after a regular intervals of time is termed as
periodic motion. e.g. Circular motion, motion of a bouncing ball, motion of planets
around the sun, rotation of earth about its polar axis, motion of a pendulum, etc.
For a particle executing periodic motion, its displacement can be represented by a
mathematical function of time. One of the simplest periodic function is given by
f (t ) = A cos ωt Inside
If the argument of this function ωt is increased by an integral multiple of 2π rad, 1 Periodic and oscillatory motion
the value of the function remains the same. 2 Simple harmonic motion
2π Kinematics of SHM
Then, the function x (t ) is periodic and its period is given as T = .
ω Force law for SHM
3 Energy in SHM
Thus, any function x (t ) is periodic with period T, if x (t ) = x (t + T ).
4 Some systems executing SHM
Further, a linear combination of sine and cosine functions like Simple pendulum
f (t ) = A sin ωt + B cos ωt …(i) Spring block system
is also periodic function with the same time period T. The physical pendulum

Take, A = D cosφ and B = D sinφ 5 SHM in some special cases


Free oscillations
Eq. (i) can be written as f (t ) = D sin(ωt + φ )
Damping and damped
where, D and φ are constants which are given as oscillations
B Maintained oscillations
D = A 2 + B 2 and tan −1   , respectively.
 A Forced oscillations
Resonance
Thus, any periodic function can be expressed as a superposition of sine and cosine
functions of different time periods with suitable coefficients.
496 OBJECTIVE Physics Vol. 1

Oscillatory (or vibratory) motion e.g. Motion of a bob of simple pendulum.


A motion in which a particle moves to and fro (back and
forth or up and down) about a fixed point after regular
interval of time, is termed as oscillatory (or vibratory) θ
motion. The fixed point about which the particle oscillates
(or vibrates) is called mean position or equilibrium
position.
Note Every oscillatory motion is periodic, but every periodic motion
need not to be oscillatory. e.g. Motion of the planet around the Fig. 11.2 Motion of a pendulum
sun is periodic but not oscillatory.
Some basic terms related to SHM
SIMPLE HARMONIC (i) Restoring force (F) The force acting on the particle
MOTION (executing SHM) which tends to bring it towards its
mean position, is called restoring force. Restoring
Of all the oscillatory motions, the most important and force is always directed towards mean position or
simplest form of motion is simple harmonic motion acts in the direction opposite to that of displacement.
(SHM). In this type of oscillatory motion, displacement, Restoring force, F = − kx
velocity, acceleration and force vary (w.r.t. time) in a way
that can be described by either the sine or the cosine where, k is a constant and x is displacement about
function collectively called sinusoids. mean position.
Note Any oscillatory motion that cannot be described, in terms of (ii) Time period (T) The time taken by the particle to
single harmonic function, (i.e. sine or cosine function) is called complete one oscillation is called time period. Its
anharmonic oscillation. unit is second.
The particle executing simple harmonic motion oscillates (iii) Frequency (n ) The number of oscillations made by
such that its acceleration is always directed towards the the particle (executing SHM) in one second is called
mean position and magnitude of acceleration is proportional frequency.
1
to the displacement of the particle from mean position. Frequency, n =
Time period (T )
Mathematically, a simple harmonic motion can be
expressed as Its unit is cycles per second or hertz (Hz).
y = A sin ωt = A sin 2πt /T ...(i) (iv) Angular frequency (ω ) The rate of change of phase
or y = A cos ωt = A cos 2 π t /T ...(ii) angle of particle (executing SHM) with respect to time
Here, y = displacement of body from mean position at any is called angular frequency. Its unit is radian/second.

instant t, Angular frequency, ω =
A = amplitude or maximum displacement of the body, Time period (T )
ω = angular frequency = 2 π × Frequency(n )
and T = time period of SHM. (v) Amplitude (A) The maximum displacement of particle
from its mean position is defined as amplitude.
Types of SHM (vi) Phase (φ ) Time varying quantity, which can fully
SHM are of two types as follows describe displacement and velocity of particle is
(i) Linear SHM When a particle moves to and fro called phase. Its unit is radian.
about a fixed point (called equilibrium position) In the given graph, two SHM in one second having
along a straight line, then its motion is called linear same amplitude A and same angular frequency ω are
simple harmonic motion. e.g. Motion of a mass shown.
x T
connected to spring.
π
φ=–
Displacement

A 4 2
m 0 1
t
Fig. 11.1 Motion of a spring block system φ=0
–A
(ii) Angular SHM When a system oscillates angularly x′ T′
with respect to a fixed axis, then its motion is called Fig. 11.3 The curves 1 and 2 are for φ = 0 and −π /4, respectively
angular simple harmonic motion. Their phase angle φ are different.
Simple Harmonic Motion 497

Example 11.1 A nurse in a hospital, noted for a patient that


heart was beating 75 times a minute. Find its frequency and
1. Displacement
time period. The displacement of a particle executing SHM at any
Sol. Number of oscillations (beats) in 60 s = 75
instant (t ) is given by
75 x = A sinωt or x = A cosωt
Number of oscillations (beats) in 1 s = = 1.25
60 First relation is valid when the time is measured from
∴ Beat frequency of heart, n = 1.25 s−1 = 1.25 Hz mean position and second relation is valid when time is
1 1 measured from extreme position.
∴ Time period, T = = = 0.8 s
n 1.25 As shown in figure, consider a particle P moving in
anti-clockwise direction with uniform angular velocity ω
Necessary conditions to execute SHM along a circle of radius A and centre O. Let at time t = 0, it
(i) Motion of particle should be oscillatory. makes an angle φ with X-axis. In time t, it covers an angle
ωt. Then, at position P ′, it makes an angle (ωt + φ ) with
(ii) Total mechanical energy of particle should be
positive X-axis. The position of P ′ on the axis as the
conserved (Kinetic energy + Potential energy
particle moves on the circle is given by
= constant).
(iii) Extreme position should be well defined. x = A sin (ωt + φ ) or x = A cos (ωt + φ )
(iv) In linear SHM The restoring force (or acceleration) The term (ωt + φ ) is called phase and φ is called initial
acting on the particle should always be proportional phase or ‘epoch’.
to the displacement of the particle and directed
towards the equilibrium position. Y
w
∴ F ∝ − y or a ∝ − y P¢
Negative sign shows that direction of force and A P (t = 0)
wt
acceleration is towards equilibrium position and y is f
displacement of particle from equilibrium position. X
O
(v) In angular SHM The restoring torque (or angular
acceleration) acting on the particle should always be
proportional to the angular displacement of the
particle and directed towards the equilibrium position. Fig. 11.5 Circle of reference
∴ τ ∝ − θ or α ∝ − θ
Example 11.2 An object performs SHM of amplitude 5 cm
and time period 4 s. If timing is started when the object is at
Kinematics of SHM the centre of the oscillation, i.e. x = 0, then calculate
Consider a particle P moving along a circle of radius A (i) frequency of oscillation,
with uniform angular velocity ω in anti-clockwise sense. (ii) displacement at 0.5 s.
Then, motion of N (foot of the perpendicular drawn from 1 1
the point P) about O represents simple harmonic motion. Sol. (i) Frequency, n = = = 0.25 Hz
T 4
In the given figure, the particle P is called reference (ii) The displacement equation of object,
particle and the circle along which the particle P revolves x = A sin ωt
is called circle of reference.
= A sin(2πnt )
Thus, SHM can be defined as, “the straight line motion of So, at t = 0.5 s,
the foot of perpendicular drawn from the particle on the x = 5 sin (2π × 0.25 × 0.5)
diameter of the circle.” π 5
Y = 5 sin = cm
4 2
ω
P
A Example 11.3 A particle executes SHM from extreme
y
ωt position and covers a distance equal to half of its amplitude
X′ X in 1 s. Determine the time period of motion.
x N
Sol. For particle starting SHM from extreme position,
x = A cos ω t
Y′ A
At t = 1 s, x=
Fig. 11.4 A particle moving along a circle 2
498 OBJECTIVE Physics Vol. 1

A
⇒ = A cos(ω × 1) 3. Acceleration
2
⇒ cos ω = 1/2
The acceleration of the particle executing SHM at any
π π instant is defined as the time rate of change of its velocity
⇒ cos ω = cos ⇒ ω= at that instant, i.e.
3 3
2 π 2π × 3 a (t ) = dv (t )/dt
∴ Time period, T = = =6s
ω π
or Acceleration, a = − ω 2 x
Example 11.4 A harmonic oscillation is represented by
x = 0.34 cos (3000t + 0.74), where x and t are in mm and Special Cases
second, respectively. Deduce (i) amplitude (ii) angular (i) When the particle is at the mean position, i.e. x = 0,
frequency (iii) initial phase. then acceleration is zero.
Sol. Given, x = 0.34 cos (3000t + 0.74) ∴ amean position = 0
Comparing with general equation x = A cos (ωt + φ ), we get (ii) When the particle is at the extreme position, i.e.
(i) Amplitude, A = 0.34 mm = 0.034 cm
x = A, then acceleration is maximum.
(ii) Angular frequency, ω = 3000 rads−1
(iii) Initial phase, φ = 0.74 rad
∴ a (extreme position) = − ω 2A

Example 11.5 Find the equation of simple harmonic motion


of a particle whose amplitude is 0.04 and whose frequency Graphical representation of displacement,
is 50 Hz. The initial phase is π/3. Assume that motion of velocity and acceleration in SHM
particle is started from mean position. If a particle is executing SHM and is passing through its
Sol. From equation of SHM, x = A sin (ωt + φ ) positive extreme position (x = + A) at t = 0, then its
π displacement equation can be written as
Here, A = 0.04 m, ν = 50 Hz, φ =
3 x = A cosωt
 π
∴ x = 0.04 sin 2π × 50t +  (Qω = 2πν ) dx
 3 Velocity, v = = −ωA sin ωt
dt
 π
or x = 0.04 sin 100πt +   π
 3 = ωA cos  ωt + 
 2
2. Velocity dv
Acceleration, a = = −ω 2A cos ωt
The velocity of a particle executing SHM at any instant, is dt
defined as the time rate of change of its displacement at = ω 2A cos (ωt + π )
that instant, i.e.
v (t ) = dx (t )/dt The following table gives the different values of
displacement, velocity and acceleration for various values
of t for one complete cycle
or Velocity, v = ω A 2 − x 2
T T 3T T
Special Cases Time, t 0
4 2 4
(i) When the particle is at the mean position, i.e. x = 0, Phase angle, 0 π π 3π 2π
then its velocity is maximum. 2π 2 2
ωt = t
∴ v max = ωA T
(ii) When the particle is at the extreme position, i.e. Displacement, x (t ) + A (max) 0 (min) −A (max) 0 (min) +A
x = ± A, then its velocity is zero. (max)

Velocity, v (t ) 0 (min) −ωA 0 (min) + ωA 0 (min)


∴ v = ω A2 − A2 = 0 (max) (max)
Therefore, the velocity of a particle executing Acceleration, a (t ) −ω2A 0 (min) + ω2A 0 (min) −ω2A
SHM is zero at the either of its extreme positions. (max) (max) (max)
Simple Harmonic Motion 499

Thus, on the basis of the above mentioned values, π


Amplitude, A = 10 m, ω = π s−1 and φ =
following graphs are plotted between x (t ) versus t, v(t ) 6
versus t and a(t) versus t for different values of t, 2π
∴ Time period, T = ⇒ T = 2s
ω
+A
∴ Maximum velocity, v max = ω A = 10 π ms −1
Displacement

3T/4
0 t Example 11.8 Amplitude of a harmonic oscillator is A.
T/4 T/2 T
When particle’s velocity of this oscillator is half of maximum
velocity, then determine position of particle.
–A
v max Aω
T
Sol. As, v = ω A2 − x 2 but it is given that v = =
2 2

⇒ = ω A2 − x 2 ⇒ A2 = 4 (A2 − x 2 )
ωA 2
4A2 − A2 3A
⇒ x2 = ⇒ x=±
Velocity

T/2 4 2
0 t
T/4 3T/4 T
Example 11.9 A particle executes SHM with a time period of
– ωA 2 s and amplitude 10 cm. Find the magnitude of its
(i) displacement (ii) velocity (iii) acceleration after (1/6) s,
starting from mean position.
ω2A Sol. (i) Given, T = 2 s, A = 10 cm

Acceleration

Displacement, x = A sin t
T/4 3T/4 T
0 t
T/2 T 2π 1 π
= 10 sin × = 10 sin = 5 cm
− ω2A
2 6 6
dx 2π 2π
Fig. 11.6 Relation between displacement, velocity (ii) Velocity, v = =A cos t
and acceleration in SHM dt T T
2π 2π 1
= 10 × cos ×
Important points regarding kinematics of SHM 2 2 6
(i) In linear SHM, the length of SHM path is 2 A. 3
= 10 × 3.14 × = 27.19 cm/s
(ii) The total work done and displacement in one 2
complete oscillation is zero but total traversed length dv 4π 2 2π
is 4 A. (iii) Acceleration, a = = 2 A sin t
dt T T
(iii) The velocity and acceleration varies with the same 4 × (3.14)2 2π 1
frequency as displacement. = × 10 × sin ×
4 2 6
(iv) Velocity is ahead of displacement by phase angle of = 49.3 cm/s 2
(π /2) rad, acceleration is ahead of velocity by phase
angle of (π /2) rad and acceleration is ahead of Example 11.10 A body oscillates with SHM according to the
displacement by phase angle of π rad. equation, x = (5 m ) cos [(2π rad s −1 ) t + π /4].
At t = 1.5 s, calculate the (i) displacement, (ii) speed and
Example 11.6 The particle is executing SHM on a line 4 cm (iii) acceleration of the body.
long. If its velocity at mean position is 12 cm/s, then
determine its frequency. Sol. The angular frequency ω of the body = 2π rad s −1 and its
Sol. Given, amplitude, A = 2 cm time period, T = 1 s
At mean position, velocity is maximum, v max = Aω = 12 At t = 1.5 s,
⇒ 2 × 2πn = 12 ⇒ n = (3 / π ) Hz (i) Displacement = (5 m) cos [(2π rad s−1) × 1.5 s + π / 4]
Example 11.7 If a SHM is represented by the equation = (5 m) cos [(3π + π / 4) rad]
 π = − 5 × 0.707 m = − 3.535 m
x = 10 sin  πt +  in SI units. Determine its amplitude,
 6 (ii) Using v = − ωA sin (ωt + φ ),
time period and maximum velocity v max . Speed of the body
Sol. Comparing the above equation with  π
= − (5 m) (2π s−1) sin (2π rad s −1) × 1.5 s + 
x = A sin (ωt + φ ), we get  4
500 OBJECTIVE Physics Vol. 1

 π A
= A sin ωt
= − (5 m) (2πs −1) sin 3π +   Given,
  4  3
= 10π × 0.707 ms−1 = 22.2 ms–1 A
and − = A sin (ωt + φ )
3
(iii) Acceleration of the body
1
= − (2π rads−1)2 × Displacement which gives, sin ωt = K (iii)
3
−1 2 −2
= − (2π rads ) × (−3.535 m) = 139.41 ms 1
sin (ωt + φ ) = − K (iv)
3
Example 11.11 A particle executes simple harmonic motion
about the point x = 0 . At time t = 0, it has displacement From Eq. (iv), we get
x = 2 cm and zero velocity. If the frequency of motion is sin ωt cos φ + cos ωt sin φ = −
1
0.25 s −1, find (i) the time period, (ii) angular frequency, (iii) 3
the amplitude, (iv) maximum speed, (v) the displacement at 1 1 1
t = 3 s, (vi) the velocity at t = 3 s and (vii) maximum ⇒ cos φ + 1 − sin φ = −
3 9 3
acceleration.
1 1 Solving this equation, we get
Sol. (i) Time period, T = = =4s 7
f 0.25 s−1 cos φ = − 1,
9
2π 2π π
(ii) Angular frequency, ω = = = rad s −1 = 1.57 rad s −1
 7
T 4 2 ⇒ φ = π or cos−1  
(iii) Amplitude is the maximum displacement from mean  9
position. Hence, A = 2 − 0 = 2 cm. Differentiating Eqs. (i) and (ii), we obtain
(iv) Maximum speed, v1 = Aω cos ωt
π v 2 = Aω cos (ωt + φ )
v max = A ω = 2 ⋅ = π cms −1 = 3.14 cms −1 and
2 If we put φ = π, we find v1 and v 2 are of opposite signs.
(v) The displacement is given by Hence, φ = π is not acceptable.
x = A sin (ωt + φ )  7
∴ φ = cos−1  
Initially at t = 0, x = 2 cm, then  9
2 = 2 sin φ
or sin φ = 1 = sin 90°
Force law for SHM
The acceleration of a particle undergoing SHM is
or φ = 90°
a (t ) = − ω 2 x (t )
Now at t = 3 s,
From Newton’s second law of motion, force acting on the
π π
x = 2 sin  × 3 +  = 0 particle of mass m in SHM is F (t ) = ma (t ), we get
2 2
(vi) At t = 3s, x = 0
F (t ) = − mω 2 x (t ) = − kx (t )
As, velocity at x = 0 is v max . k
 π
where, k = mω 2 or ω =
∴ v max = ωA =   (2) = 3.14 cms−1 m
 2
Thus, in SHM, the force is directly proportional and
(vii) Maximum acceleration, opposite to the displacement and is always directed
a = − ω 2A = − (1.57)2 × 2 = − 4.93 cms −2 towards the mean position.
This force is called restoring force.
Example 11.12 Two particles move parallel to X-axis about
the origin with the same amplitude and frequency. At a 2π k
certain instant, they are found at distance A/3 from the
Further, since ω = =
T m
origin on opposite sides but their velocities are found to be in
the same direction. What is the phase difference between the m
two ? ∴ Time period, T = 2π
k
Sol. Let equations of two SHM be
x1 = A sin ωt …(i) 1 k
and frequency, n=
x 2 = A sin (ωt + φ ) K (ii) 2π m
Simple Harmonic Motion 501

In different types of SHM, the quantities m and k will go Example 11.13 Frequency of oscillation of a body is 6 Hz
on taking different forms and names. In general, m is when force F1 is applied and 8 Hz when F 2 is applied. If
called inertial factor and k is called spring factor. both forces F1 and F 2 are applied together, then find out the
frequency of oscillation.
In linear SHM, the general formula for time period is
Sol. According to question, F1 = −k1x and F 2 = − k 2x
Inertial factor
T = 2π 1 k1
Spring factor So, n1 = = 6 Hz,
2π m
In linear SHM, the spring factor stands for force per unit 1 k2
displacement and inertial factor for mass of the body n2 = = 8Hz
2π m
executing SHM.
Now, F = F1 + F 2 = − (k1 + k 2 )x
Thus, in linear SHM,
1 k1 + k 2
m m Therefore, n=
T = 2π = 2π 2π m
k  Force  4π 2n12m + 4π 2n 22m
  ⇒ n=
1
 Displacement  2π m
(As, k = 4π 2n 2m )
Displacement
or T = 2π = n12 + n 22 = 82 + 62 = 10 Hz
Acceleration

CHECK POINT 11.1


1. The motion which is not simple harmonic is 6. The equation of a simple harmonic motion is
(a) vertical oscillations of a spring x = 0.34cos(3000 t + 0.74), where x and t are in mm
(b) motion of simple pendulum and second, respectively. The frequency of the motion
(c) motion of a planet around the sun (in Hz) is
(d) oscillation of liquid column in a U-tube (a) 3000 (b) 3000/ 2π (c) 0.74/ 2π (d) 3000/π
2. Select the wrong statement regarding simple harmonic 7. A body is executing simple harmonic motion with an
motion. angular frequency of 2 rad s −1 . The velocity of the body at
(a) The body is uniformly accelerated. 20 mm displacement when the amplitude of the motion is
(b) The velocity of the body changes smoothly at all instants.
60 mm, is
(c) The amplitude of oscillation is symmetric about the
(a) 131 mms −1 (b) 118 mms −1
equilibrium position.
(c) 113 mms −1 (d) 90 mms −1
(d) The frequency of oscillation is independent of amplitude.
3. The amplitude and the time period in a SHM is 0.5 cm and 8. The maximum velocity of a simple harmonic motion
 π
0.4 s, respectively. If the initial phase is π /2 rad, then the represented by y = 3 sin 100t +  is given by
equation of SHM will be  6
(a) y = 0.5 sin 5πt (b) y = 0.5 sin 4 πt 3π
(a) 300 units (b) units
(c) y = 0.5 sin 2.5πt (d) y = 0.5 cos 5πt 6
π
4. A particle is moving with constant angular velocity along (c) 100 units (d) units
the circumference of a circle. Which of the following 6
statement is true? 9. A body is executing SHM with an amplitude of 0.1 m. Its
(a) The particle executes SHM
velocity while passing through the mean position is 3 ms −1.
(b) The projection of the particle on any one of the diameters
executes SHM Its frequency (in Hz) is
15
(c) The projection of the particle on any of the diameters (a) 15π (b) (c) 30π (d) 25π
executes SHM π
(d) None of the above 10. Velocity at mean position of a particle executing SHM is v.
5. The displacement of a particle in SHM is indicated by Velocity of the particle at a distance equal to half of the
equation y = 10sin(20t + π / 3), where y is in metre. The value amplitude will be
of time period of vibration will be (in second) v v 3 3
(a) (b) (c) v (d) v
(a) 10/π (b) π/10 (c) 2π/10 (d) 10/ 2π 2 2 2 4
502 OBJECTIVE Physics Vol. 1

11. A particle executes simple harmonic motion with an v02


(a) (b) a 0 v0
amplitude of 4 cm. At the mean position, the velocity of the a0
particle is 10 cms −1 . The distance of the particle from the a 02 1
mean position when its speed becomes 5 cms −1 is (c)
v0
(d)
a 0 v0
(a) 3 cm (b) 5 cm
(c) 2 3 cm (d) 2 5 cm 15. A particle executing simple harmonic motion has an
amplitude of 6 cm. Its acceleration at a distance of 2 cm
12. When a particle executes SHM, there is always a constant
from the mean position is 8 cms −2. The maximum speed of
ratio between its displacement and
(a) velocity (b) acceleration
the particle is
(a) 8 cms −1 (b) 12 cms −1
(c) mass (d) time period
(c) 16 cms −1 (d) 24 cms −1
13. What is the maximum acceleration of the particle
executing SHM whose equation of motion is given below?
16. The average acceleration of a particle performing SHM over
one complete oscillation is
 πt 
y = 2 sin + φ , where y is in cm and t is in second. ω2 A ω2 A
 2  (a) (b)
2 2
π π2
(a) cms −2 (b) cms −2 (c) zero (d) Aω2
2 2
π π2 17. In SHM, the acceleration is ahead of velocity by a phase
(c) cms −2 (d) cms −2 angle
4 4 π
(a) 0° (b)
14. The maximum acceleration of a body moving in SHM is a0 2
and maximum velocity is v 0 . The amplitude is given by (c) π (d) 2π

3. ENERGY IN SHM
The particle executing SHM has both types of energy: (ii) In terms of time Kinetic energy of a particle
potential energy and kinetic energy. When a body is executing SHM at any time t is given by
displaced from its equilibrium position by doing work upon
it, the body acquires potential energy. However, when the 1
K = mω 2A 2 cos 2 ωt
body is released, it begins to move back with a velocity, thus 2
acquiring kinetic energy.
i.e. kinetic energy of a particle executing SHM is a
Kinetic energy periodic function of time.
The relation for kinetic energy of a particle executing SHM Kinetic energy versus time graph is given below
can be expressed in two forms, i.e. in terms of displacement
1
and in terms of time. KEmax KEmax = mω2A2
2
(i) In terms of displacement If mass of the particle
Kinetic energy

executing SHM is m and its displacement at any


1
instant is x, then KE = mω 2 (A 2 − x 2 )
2
0 T T 3T T Time (t)
where, A = amplitude of SHM. 4 2 4
k 1 Fig. 11.7 Kinetic energy versus time graph for one oscillation
As, ω= ⇒ mω 2 = k ∴ KE = k (A 2 − x 2 )
m 2
From the graph 11.7, it is clear that at t = 0, T/2,
Special cases T, etc., kinetic energy has its maximum value. For
• Kinetic energy is maximum at mean position, i.e. at T 3T
x = ± A or t = , , the kinetic energy has its
x=0 4 4
1 minimum value.
KE max = mω 2A 2
2 Clearly, twice in each cycle, kinetic energy acquire its
• Kinetic energy is minimum (zero) at extreme position, peak values. It is a periodic function of time, where the
i.e. at x = ± A , KE min = zero time period equals to T/2.
Simple Harmonic Motion 503

Potential energy which is a constant quantity, i.e. it remains same at all


instant and at all displacement.
The relation for potential energy of a particle executing
SHM in terms of displacement and in terms of time are as Fig. 11.9 shows the variation of kinetic energy (K),
follows potential energy (U) and total energy (E) as a function of
time [shown in Fig. 11.9(a)] and displacement [shown in
(i) In terms of displacement If the displacement of Fig. 11.9(b)] for a particle executing SHM.
particle executing SHM at any instant is x, then
E = K(t) + U(t)
1 1
Potential energy, U = mω 2 x 2 = kx 2 E
2 2
K(t)

Energy
Special cases
U(t)
• Potential energy is maximum at extreme position,
T
i.e. at x = ± A O T/2
Time (t)
1
Umax = mω 2A 2 (a)
2 E, U, K
1 2
• Potential energy is minimum (zero) at mean E=
1
kA2
U=
2
kx
position, i.e. at x = 0, Umin = 0 2

(ii) In terms of time Potential energy of a particle


executing SHM at any time t is given by
1
K= k (A2–x2)
1 2
U = mω 2A 2 sin 2 ωt
2 x
–A –A A +A
Potential energy versus time graph is given below 2 (b) 2
Umax 1 Fig. 11.9 K, U and E as functions of time and
Umax = kA2
Potential energy

2 displacement x for a particle executing SHM


U

∴ From the above graph, it can be concluded that


(i) The total energy remains constant at all t or x. So, it
0 T T 3T
is shown by straight line parallel to time.
T Time (t)
4 2 4 1 1
(ii) Also, U = kx 2 and K = k (A 2 − x 2 ), so the graphs
Fig. 11.8 Potential energy versus time graph for one oscillation 2 2
of U and K versus x are parabolic.
From the graph, it is clear that at mean position (i.e.
x = 0 or t = 0, T/2, T K), potential energy is (iii) Both kinetic energy and potential energy reach
minimum and at extreme position maximum value twice, during each period of SHM.
(i.e. t x = ± A or t = T /4, 3T /4), its value is (iv) In the course of motion, kinetic energy increases at
maximum. Clearly, twice in each cycle, potential the expense of potential energy or vice-versa.
energy acquires its peak values. Therefore, we may say that during an oscillation,
Hence, the potential energy is the periodic function there is a continuous exchange of kinetic and
of time with a period of T/2. potential energies.
Important points regarding energy in SHM
Total energy (i) The frequency of oscillation of potential energy and
Total energy can be obtained by adding potential and kinetic energy is twice as that of displacement or
kinetic energies. Therefore, velocity or acceleration.
1 1
E = K + U = mω 2 (A 2 − x 2 ) + mω 2 x 2 (ii) The average value of kinetic energy and potential
2 2 energy is K av = U av
1 1
E = mω 2A 2 or E = kA 2 1 1
2 2 = E = kA 2
2 4
504 OBJECTIVE Physics Vol. 1

Example 11.14 The potential energy of a particle executing T 2π T π A


Sol. At t = , x = A sin × = A sin =
SHM is 2.5 J, when its displacement is half of the amplitude, 12 T 12 6 2
then determine total energy of particle. 1 3 1
So, kinetic energy, KE = k (A2 − x 2 ) = × kA2
1 2 2 4 2
Sol. As, potential energy = kx
2 1 2 1 1 2
and potential energy, PE = kx = × kA
1 A2  A 2 4 2
⇒ k = 2.5 J Q x = 
2 4  2 KE 3
∴ =
where, x is the displacement of the particle and A is the PE 1
amplitude. Example 11.19 A particle of mass 0.2 kg is executing SHM
1
⇒ Total energy = kA2 = 2.5 × 4 = 10 J of amplitude 0.2 m. When it passes through the mean
2 position, its kinetic energy is 64 × 10 −3 J. Obtain the
Example 11.15 A harmonic oscillator of force constant equation of motion of this particle, if the initial phase of
4 × 10 6 Nm −1 and amplitude 0.01 m has total energy 240 J. oscillation is π/4.
What is maximum kinetic energy and minimum potential π
energy? Sol. Here, A = 0.2 m, φ = , KE = 64 × 10−3 J
4
Sol. Given, k = 4 × 106 N/m, A = 0.01 m, total energy = 240 J 1
Maximum kinetic energy As we know that, KE = mω 2A2
2
1 1 1
= mω 2A2 = kA2 = × 4 × 106 × (0.01)2 = 200 J where, m = mass of particle = 0.2 kg
2 2 2 1
Minimum potential energy ⇒ 64 × 10−3 = × 0.2 × ω 2 × (0.2)2
2
= Total energy − Maximum kinetic energy = 40 J 128 × 10−3
⇒ ω2 = or ω = 4 s−1
Example 11.16 A linear harmonic oscillator has a total 0.2 × 0.2 × 0.2
mechanical energy of 200 J. Potential energy of it at mean ∴ Equation of motion can be written as
position is 50 J. Find  π
(i) the minimum potential energy, x = A sin(ω t + φ ) = 0.2 sin 4t + 
 4
(ii) the maximum kinetic energy,
(iii) the potential energy at extreme positions. Example 11.20 A particle executes SHM with amplitude A
Sol. (i) At mean position, potential energy is minimum. and time period T. When the displacement from the
Hence, U min = 50 J equilibrium position is half the amplitude, what fractions of
the total energy are kinetic and potential?
(ii) At mean position, kinetic energy is maximum.
1 2
∴ K max = E − U min = 200 − 50 = 150 J Sol. Total energy, E = kA
(iii) At extreme positions, kinetic energy is zero and 2
A
potential energy is maximum. When x = ± , then
∴ U max = E = 200 J 2
2
1 1  A
(From law of conservation of energy) Potential energy, U = kx 2 = k ⋅  ± 
2 2  2
Example 11.17 A particle executes SHM, at what value of
1 1 2 1
displacement are the kinetic and potential energies equal? =
⋅ kA = E = 25% of E
1 4 2 4
Sol. We know that, kinetic energy = mω 2 (A2 − x 2 ) and 1 3
2 ∴ Kinetic energy, E − U = E − E = E = 75% of E
1 4 4
potential energy U =
mω 2x 2
2 Example 11.21 The potential energy of a particle oscillating
Since, K =U on X-axis is given as, U = 20 + (x − 2) 2
1 1
⇒ mω (A − x ) = mω 2x 2 or 2x 2 = A2
2 2 2
Here, U is in joule and x is in metre. Total mechanical
2 2 energy of the particle is 36 J.
A
or x= = 0.707A (i) State whether the motion of the particle is simple harmonic
2 or not.
(ii) Find the mean position.
Example 11.18 A particle starts oscillating simple (iii) Find the maximum kinetic energy of the particle.
harmonically from its equilibrium position with time period T.
dU
Determine ratio of kinetic energy and potential energy of the Sol. (i) F = − = − 2(x − 2)
T dx
particle at time t = .
12 By assuming x − 2 = X, we get
Simple Harmonic Motion 505

F = − 2X (iii) At x = 2 m (mean position),


Since, F ∝−X U min = 20 + (2 − 2)2 = 20 J
The motion of the particle is simple harmonic. ∴ Maximum kinetic energy of the particle,
(ii) The mean position of the particle is X = 0 or x − 2 = 0, K max = E − U min = 36 − 20 = 16 J
which gives x = 2 m
Note U min is 20 J at mean position or at x = 2 m.

CHECK POINT 11.2


1. A body of mass 1 kg is executing simple harmonic motion. are measured as a function of displacement x. Which of the
Its displacement y (cm) at t seconds is given by following statement is true?
 π (a) PE is maximum when x = 0
y = 6 sin 100 t +  . Its maximum kinetic energy is
 4 (b) KE is maximum when x = 0
(a) 6 J (b) 18 J (c) 24 J (d) 36 J (c) TE is zero when x = 0
(d) KE is maximum when x is maximum
2. The total vibrational energy of a particle in SHM is E. Its
kinetic energy at half the amplitude from mean position will 7. In SHM for how many times, potential energy is equal to
be kinetic energy during one complete period?
(a) E/2 (b) E/3 (a) 1 (b) 2 (c) 4 (d) 8
(c) E/4 (d) 3E/4 8. In a simple oscillating pendulum, the work done by the
3. A particle undergoing SHM has the equation pendulum in one oscillation will be
x = A sin (ωt + φ), where x represents the displacement of (a) equal to the total energy of the pendulum
the particle. The kinetic energy oscillates with time period (b) equal to the kinetic energy of the pendulum
2π π (c) equal to the potential energy of the pendulum
(a) (b)
ω ω (d) zero

(c) (d) None of these 9. The total energy of a vibrating particle in SHM is E. If its
ω amplitude and time period are doubled, its total energy will
4. A particle executes SHM on a line 8 cm long. Its kinetic be
energy and potential energy will be equal when its distance (a) 16 E (b) 8E (c) 4E (d) E
from the mean position is
10. Energy of particle executing SHM depends upon
(a) 4 cm (b) 2 cm (c) 2 2 cm (d) 2 cm
(a) amplitude only (b) amplitude and frequency
5. A particle starts SHM from the mean position. Its amplitude (c) velocity only (d) frequency only
is A and total energy E. At one instant, its kinetic energy is
3E 11. A particle of mass 0.10 kg executes SHM with an amplitude
. Its displacement at that instant is 0.05 m and frequency 20 vib/s. Its energy of oscillation is
4
(a) 2 J (b) 4 J (c) 1 J (d) zero
A A A
(a) (b) (c) 3 (d) zero
2 2 2 12. Amplitude of a particle in SHM is 6 cm. If instantaneous
potential energy is half the total energy, then distance of
6. A body executes simple harmonic motion. The potential particle from its mean position is
energy (PE), the kinetic energy (KE) and total energy (TE) (a) 3 cm (b) 4.2 cm (c) 5.8 cm (d) 6 cm

SOME SYSTEMS EXECUTING SHM


A pure simple harmonic motion (SHM) is not possible O
unless some conditions are not applied on it. Let us
consider some of the examples of pure simple harmonic θ l (Effective length)
motion under certain conditions.

1. Simple pendulum m (Bob)


It consists of a point mass suspended by a massless and
inextensible string tied from a rigid support. Consider a
Mean position
simple pendulum as shown in figure.
Fig. 11.10 SHM of simple pendulum in one oscillation
A small bob of mass m is tied to an inextensible massless
string of length l. When the bob is pulled sidewards and Let θ be the angle which the string makes with the
released, it oscillates in a circular arc with the centre at vertical, when it is displaced from its mean position.
the point of suspension O.
506 OBJECTIVE Physics Vol. 1

At mean position θ = 0 ° and v is maximum. At extreme l


position, v is minimum. For the smaller value of θ, the T ′ = 2π
 σ
motion of bob is simple harmonic. 1 −  g
 ρ
l
Time period of simple pendulum, T = 2π
g (iii) Effect of change in length
Note that the time period is independent of the mass of the • As T ∝ l , so if a person stand on a swing while
pendulum. This formula is valid only when radius of point swinging, l decreases and thus T decreases.
mass of pendulum is negligible as compared to length of • If the length of simple pendulum (l ) is comparable
pendulum. to radius of earth (R ), then time period,

Variation in time period of simple 1


T = 2π
pendulum in different cases 1 1 
g + 
l R 
(i) Effect of acceleration
If length becomes infinite, then time period,
• Above or below the surface of earth, g decreases.
Hence, time period of simple pendulum increases. T = 2π R /g which is approximately equal to
84.6 min.
• At the centre of earth or in a satellite or in a freely
falling lift, the value of g is zero, so time period of If the length is comparable to R, (i.e. l −
~ R ), then
simple pendulum becomes infinity. That means, it will
R ~
not oscillate and remain as it is, where it is left. time period, T = 2π −1 h
2g
• If a simple pendulum is suspended at the ceiling of a
lift which is accelerating upwards with acceleration a, (iv) Effect of temperature
l
then time period of simple pendulum, T ′ = 2π , Suppose the thread of simple pendulum is replaced by a
g +a metal wire.
hence time period will decrease. l l (1 + αt )
• If a simple pendulum is suspended at the ceiling of a Now, T = 2π and T ′ = 2π
g g
lift which is accelerating downwards with acceleration
a, then time period of simple pendulum, Here, t represents the rise in temperature and α
l represents the coefficient of linear expansion.
T ′ = 2π . i.e. Time period will increase. T′
g −a Now, = (1 + αt ) 1/ 2
T
• If a simple pendulum in a car which is accelerating
Using Binomial theorem, (1 + x ) n = 1 + nx + …
horizontally with acceleration ‘a’, then time period of
l (Neglecting higher terms)
pendulum will be T ′ = 2π T′ 1 T ′ −T 1
g eff = 1 + αt or = αt
T 2 T 2
where g eff = g 2 + a 2 , i.e. time period will decrease.
Oscillations of a simple pendulum
q
a
q g
in an electric field
geff If the bob of simple pendulum has positive charge q and
is made to oscillate in a uniform electric field E, then
Fig. 11.11 (i) If the field is acting in upward direction, then
In this case, equilibrium position will not be vertical
but will make an angle θ = tan −1 (a / g ) to the vertical
qE
θ l
in a direction opposite to the acceleration. E

(ii) Effect of buoyancy


mg
If the bob of simple pendulum of density ρ is made to
oscillate in a liquid of density σ, then its time period will
increase as compared to that of air. Fig. 11.12 A pendulum in upward electric field
Simple Harmonic Motion 507

l l1 36 × 36 36
Time period of pendulum, T = 2π or = =
qE l 2 30 × 30 25
g−
m l1 22
Also, l1 − l 2 = 22 cm, −1=
Hence, the time period will increase. l2 l2
(ii) If the field is in downward direction, then 22 36 l 2 25
or = −1 or =
l 2 25 22 11
q l or l 2 = 50 cm
E
∴ l1 = 22 + l 2 = (22 + 50) cm = 72 cm

mg + qE Example 11.23 The bob of a simple pendulum executes SHM


in water with a period t. The period of oscillation of the bob
in air is t 0 . What is the relation between t and t 0 ?
Fig. 11.13 A pendulum in downward electric field 4000
Given, density of the bob is kg m −3 . Neglect frictional
l 3
Time period of pendulum, T = 2π force of water.
qE
g+
m 4000
Sol. Here, ρ = kgm−3, σ = 1000 kgm−3
Hence, the time period will decrease. 3
(iii) If the direction of electric field is horizontal, then  σ  1000 × 3 g
time period of pendulum, ∴ g ′ = 1 −  g = 1 −  g=
 ρ  4000  4
E l
θ Q t 0 = 2π …(i)
l g′
qE
l×4 l
and t = 2π = 2 × 2π
mg g g
Fig. 11.14 A pendulum in horizontal electric field or t = 2t 0 [from Eq. (i)]

l Example 11.24 Determine the period of small oscillations of


T = 2π 1/ 2
a simple pendulum, whose bob is suspended by a thread
 2  qE  2  l = 20 cm in length, such that it is located in a liquid whose
g +    density is η = 3 times less than that of the bob. The
 m 
 resistance of the liquid is to be neglected.
Hence, the time period will decrease. Sol. LetV = volume of bob,
ρ = density of material of the bob
Second pendulum and g ′ = effective value of acceleration due to gravity.
If the time period of a simple pendulum is 2 second, it is Now, effective weight = weight of bob − upthrust of liquid
called second pendulum. ρ
l Vρ g ′ = Vρ g − V g
Time period of second pendulum, T = 2 s = 2π η
g  1  η − 1
⇒ g ′ = 1 −  g or g ′ =   g
As, on the earth’s surface, g = 9.8 ms −2 ≈ π 2 ms −2  η  η 
∴ The length of second pendulum on earth’s surface l l
Again, T = 2π or T = 2π
is 1 m. g′  η − 1
 g
Example 11.22 Two pendulums whose lengths differ by  η 
22 cm oscillate at the same place, so that one of them makes ηl
30 oscillations and the other 36 oscillations during same or T = 2π
(η − 1) g
time. Find the lengths of the pendulum.
Substituting the given values, we get
Sol. According to the question,
22 3 × 20 × 10−2 44
l l
30 × 2π 1 = 36 × 2π 2 T =2× s= × 0.175 s = 1.1 s
g g 7 (3 − 1) 9.8 7
508 OBJECTIVE Physics Vol. 1

Example 11.25 The length of a simple pendulum is 16 cm. It Example 11.28 A simple pendulum consists of a small
is suspended by the roof of a lift which is moving upwards sphere of mass m suspended by a thread of length l. The
with an acceleration of 6.2 ms −2 . Find the time period of sphere carries a positive charge q. The pendulum is placed
pendulum. in a uniform electric field of strength E directed vertically
Sol. Given, length of the pendulum, l = 16 cm = 0.16 m upwards. With what period will pendulum oscillate, if the
electrostatic force acting on the sphere is less than the
Acceleration of the lift, a = 6.2 ms−2 gravitational force?
l 0.16 Sol. The two forces acting on the bob are shown Fe = qE
Q Time period, T = 2π = 2 × 3.14
(g + a ) (9.8 + 6.2) in figure.
w − Fe
0.16 1 6.28 g eff in this case will be
= 6.28 × = 6.28 × = m
16 100 10 mg − qE qE
= 0.628 s or g eff = =g−
m m
l l w = mg
Example 11.26 A simple pendulum of length l is suspended ∴ T = 2π = 2π
from the ceiling of a cart which is sliding without friction on an g eff qE
g−
inclined plane of inclination θ. What will be the time period of m
the pendulum?
Sol. Here, point of suspension has an acceleration, a = g sin θ 2. Spring block system
(down the plane). Further, g can be resolved into two Let us consider a massless spring placed on a frictionless
components g sin θ (along the plane) and g cos θ horizontal surface. Its one end is attached to a rigid
(perpendicular to plane). support and other end to a body of mass m. When it is
pulled on one side through a small distance x and released,
it moves to and fro about a mean position. Let x = 0 be
q
in
q
gs
in
os
q the position of the centre of the body when the spring is at
gs gc mean position.
a=
q q

∴ g eff = g cos θ (perpendicular to plane) m

l l
⇒ T = 2π = 2π
| g eff | g cos θ m
x
l Stretched
Note If θ = 0 °, T = 2 π which is quite obvious.
g m
x
Example 11.27 A bob of simple pendulum is suspended by a Compressed
metallic wire. If α is the coefficient of linear expansion and Mean position (x = 0)
dθ is the change in temperature, then prove that percentage Fig. 11.15
change in time period is 50αd θ.
Sol. With change in temperature dθ, the effective length of wire By Hooke’s law, a restoring force F is developed in the
becomes l ′ = l (1 + αd θ ) spring on stretching and compressing it through a small
displacement x,
l′ l F = −kx
T′ = 2π and T = 2 π
g g k
⇒ a=− x
T′ l′ m
Hence, = = (1 + αdθ )1/ 2 The time period of such oscillations is given by
T l
1 m
= 1 + αdθ T = 2π
2
k
Percentage increase in time period
where, k is force constant of spring and m is mass of the
 T ′−T  T ′ 
=  × 100 =  − 1 × 100 object connected to the spring.
 T  T 
If the stretch in a vertically loaded spring is y, then for
 αdθ 
= 1 + − 1 × 100
 2  equilibrium of mass m, T = 2 π y /g
= 50 αd θ
Simple Harmonic Motion 509

Special cases Sol. (i) Frequency = 2π m /k


(i) If the spring has a mass M and a mass m is (Frequency is independent of g in spring mass system)
suspended from it, then time period is given by mg
(ii) Initial extension in spring in equilibrium =
k
Extension in spring in equilibrium when lift starts
M m (g + a )
accelerating upwards =
M k
m m (g + a ) mg ma
∴ Amplitude = − =
k k k

m Example 11.31 A body of mass m attached to a spring which


is oscillating with time period 4 s. If the mass of the body is
Fig. 11.16
increased by 4 kg, its time period increases by 2 s.
M eff M Determine the value of initial mass m.
T = 2π , where M eff = m + m m
k 3 Sol. In 1st case, T = 2π ⇒ 4 = 2π ...(i)
k k
(ii) If two masses m1 and m 2 are connected by a spring
of spring constant k and made to oscillate on m+4
In 2nd case, 6 = 2π ...(ii)
horizontal surface. Then, time period is given by k
µ On dividing Eq. (i) by Eq. (ii), we get
T = 2π 4 m 16 m
k = ⇒ = ⇒ m = 3.2 kg
m1 m2 6 m+4 36 m + 4
k
Example 11.32 A block with a mass of 3 kg is suspended
from an ideal spring having negligible mass and stretches the
spring upto 0.2 m.
Fig. 11.17 Two blocks connected by a spring
(i) What is the force constant of the spring?
m1m 2 (ii) What is the period of oscillation of the block, if it is pulled
Here, µ = reduced mass =
m1 + m 2 down and released?
Sol. (i) In equilibrium, kl = mg
Example 11.29 A 2.5 kg collar is attached to a spring of mg
spring constant 250 Nm −1. It slides without friction over a ∴ k=
l
horizontal surface. It is displaced from its equilibrium
Substituting the given values, we get
position by 20 cm and released. Calculate the period of
(3)(9.8)
oscillation and the maximum speed. k= = 147 Nm −1
0.2
Sol. The period of oscillation,
l 0.2
m 2.5 (ii) T = 2π = 2π = 0.897 s
T = 2π = 2π g 9.8
k 250
2 × 314
. Example 11.33 Two rigid bodies A and B
= = 0.63 s A
10 of masses 1 kg and 2 kg respectively are
k 250 rigidly connected to a spring of force
Maximum speed, v m = Aω = 0.2 = 0.2 ×
m 2.5 constant 400 Nm −1. The body B rests on a k = 400 Nm–1

= 0.2 × 10 = 2 ms−1 horizontal table. From the rest position, the


body A is compressed by 2 cm and then B
Example 11.30 A spring mass system is hanging from the released. Deduce (i) the frequency of
ceiling of an elevator in equilibrium. The elevator suddenly oscillation, (ii) total oscillation energy,
starts accelerating upwards with acceleration a, find (iii) the amplitude of the harmonic vibration of the reaction
of the table on body B.
Sol. Since, it is the body A which is oscillating, m = 1 kg
k
1 k
(i) Frequency of oscillation, ν =
m 2π m
1 400 10
(i) the frequency and or ν= = = 3.185 Hz
2π 1 π
(ii) the amplitude of the resulting SHM.
510 OBJECTIVE Physics Vol. 1

1 1 2
(ii) Total oscillation energy, E = mA2ω 2 E 2 = E1 =kA1
2 2
1 k 1 and the amplitude is still A1.
or E = mA2   = kA2
2 m  2 M +m
1 Thus, A2 = A1 and T2 = 2π .
= × 400 × (0.02)2 (Q A = 2 cm = 0.02 m) k
2
= 0.08 J Example 11.35 In the following arrangements, block is
(iii) Total force acting on the table slightly displaced vertically downwards from its
= (1 + 2) kg-wt = 3 kg-wt = 3 × 9.8 N = 29.4 N equilibrium position and released. Find time period of
vertical oscillations. Assume the pulley to be light.
This is the mean upward reaction. Now, due to oscillations,
the maximum tension developed in the spring is given by
F = kA = 400 × 0.02 N = 8 N
Therefore, the net reaction of the table will vary from
(29.4 + 8) N to (29.4) N.
m
Example 11.34 A block with mass M attached to a
m
horizontal spring with force constant k is moving with simple
harmonic motion having amplitude A1. At the instant when the m
block passes through its equilibrium position, a lump of putty (i) (ii) (iii)
with mass m is dropped vertically on the block from a very
Sol. (i) Let at some instant extension of spring is x, tension in
small height and sticks to it. the string T = kx
(i) Find the new amplitude and period.
(ii) Repeat part (i) for the case in which the putty is dropped on
the block when it is at one end of its path.
Sol. (i) Before the lump of putty is dropped the total mechanical T
1
energy of the block and spring is E1 = kA 12 m
2
Since, the block is at the equilibrium position,U = 0 and
the energy is purely kinetic. Let v1 be the speed of the
block at the equilibrium position, we get Restoring force, F = T = − kx (Hooke's law)
1 1 k where, k = spring constant
E1 = Mv12 = kA 12 ⇒ v1 = A1
2 2 M m
During the process, momentum of the system in Time period, T0 = 2π
k
horizontal direction is conserved. Let v 2 be the speed of
the combined mass, then (M + m )v 2 = Mv1 (ii) Let block move down by x, pulley will move by x /2.
M M  k 
∴ v2 = v1 =  A1
M +m M +m  M  kx
2
Now, let A2 be the amplitude afterwards. Then,
1 1
E 2 = kA22 = (M + m )v 22
2 2
T
Substituting the value of v 2, we get
m
M T T
A2 = A1
M +m
Note E2 < E1 , as some energy is lost into heating up the block and kx kx
M+ m 2T = ⇒ T =
putty. Further, T2 = 2 π . 2 4
k
k
(ii) When the putty drops on the block, the block is Restoring force, F = − x
4
instantaneously at rest. All the mechanical energy is
stored in the spring as potential energy. Again, the Comparing it with equation F = −kx, we get
momentum in horizontal direction is conserved during m
the process. But now it is zero just before and after k ′ = k /4 ∴ T0 = 2π
k′
putty is dropped. So, in this case, adding the extra mass
of the putty has no effect on the mechanical energy, i.e. m m
⇒ T0 = 2 π = 4π
k /4 k
Simple Harmonic Motion 511

(iii) If block moves by x, pulley also moves x and hence, Then, spring constant, k p = k1 + k 2
spring is stretched by 2x.
m m
2kx 2kx Time period, T = 2π = 2π
kp (k1 + k 2 )

Important points regarding spring block system


T (i) If the length of the spring is made n times, then the
m effective force constant becomes 1/n times and the
Tension produced in spring, T = 4 kx time period becomes n times.
Restoring force, F = − 4 kx (ii) If a spring of spring constant k is divided into n equal
Comparing it with equation F = − k ′ x, we get parts, the spring constant of each part becomes nk
k ′ = + 4k 1
and time period becomes times.
m m m n
∴ T0 = 2 π ⇒ T0 = 2 π =π
k′ 4k k (iii) The force constant of a stiffer spring is more than
that of a soft spring.
Oscillations of spring combination
Series combination Two springs of spring constants k1 Example 11.36 One body is suspended from a spring of
length l, spring constant k and has time period T. Now, if
and k 2 are joined in series as shown in figure.
spring is divided in two equal parts which are joined in
parallel and the same body is suspended from this
arrangement, then determine new time period.
k1 Sol. When a spring of spring constant k is divided into 2 parts,
k1 k2 the spring constant of each part becomes 2k.
m ∴ Spring constant in parallel combination k ′ = 2k + 2k = 4k
k2 m m
(a)
∴ T = 2π = 2π
k′ 4k
m
m 1 T T
= 2π × = =
(b) k 4 4 2
Fig. 11.18 Series combination of springs
Example 11.37 Two identical springs of spring constant k
are attached to a block of mass m and to fixed supports as
Then, resultant spring constant of the combination is given by shown in figure. Show that when the mass is displaced from
1 1 1 k1 + k 2 kk its equilibrium position on either side, it executes a simple
= + = ⇒ ks = 1 2 harmonic motion. Find the period of oscillations.
k s k1 k 2 k1k 2 k1 + k 2

m (k + k 2 )m k k
and hence, T = 2π = 2π 1 m
ks k1k 2
Sol. Let the mass be displaced by a small distance x to the right
Parallel combination Two springs of spring constants k1 side of the equilibrium position as shown in figure.
and k 2 are joined in parallel as shown in figure. F

x
k1 k2
k1 k2 k1 m Under this situation, the spring on the left side gets elongated
by a length equal to x and that on the right side gets
m k1 compressed by the same length. The forces acting on the mass
m
are then
m
k2 F1 = − kx (force exerted by the spring on the left side, trying to
k2 pull the mass towards the mean position)
F 2 = − kx (force exerted by the spring on the right side, trying to
Fig. 11.19 Parallel combination of springs push the mass towards the mean position)
512 OBJECTIVE Physics Vol. 1

The net force F acting on the mass is then given by, F = − 2kx. that it can oscillate without friction about an axis passing
Hence, the force acting on the mass is proportional to the through O.
displacement and is directed towards the mean position,
In equilibrium, the centre of gravity (G ) is directly below
therefore the motion executed by the mass is simple
O. In the position shown in figure, the body is displaced
m
harmonic. The time period of oscillations is T = 2π . from equilibrium by an angle θ. The distance from O to the
2k
centre of gravity is l. Let, the moment of inertia of the body
Example 11.38 Periodic time of oscillation T1 is obtained about the axis of rotation through O is I and the total mass is
when a mass is suspended from a spring and if another m. In the displaced position, the weight mg causes a
spring is used with same mass, then periodic time of restoring torque, τ = − (mg )(l sin θ )
oscillation is T 2 . Now, if this mass is suspended from series
combination of above springs, then calculate the time period. I
q O
m l
Sol. As, T1 = 2π G
k1
CM I0
m 4π 2m mg cos q
⇒ T12 = 4π2
⇒ k1 = mg sin q mg
k1 T12
Fig. 11.20 Physical pendulum
m m 4π 2m
and T2 = 2π ⇒ T22 = 4π 2 ⇒ k2 =
k2 k2 T22 The negative sign shows that the restoring torque is
clockwise when the displacement is counterclockwise and
vice-versa.
For small oscillations,
k1
k1 k2 sin θ ≈ θ and Στ = Iα ⇒ − (mgl ) θ = Iα
As α is proportional to − θ, the motion is simple harmonic,
m m the time period of which is
k2

θ I
m T = 2π or T = 2π
α mgl
m
Now, T = 2π
k′
1 1 1 Note Time period of some common physical pendulum
where, = +
k ′ k1 k 2 (i) Bar,T = 2 π
2l
(ii) Ring,T = 2 π
2R
 4π 2m   4π 2m  3g g
 2  2 
k1k 2  T1   T2  (iii) Disc, T = 2 π
3R
⇒ k′ = = 2g
k1 + k 2 4π 2m 4π m 2
+
T12 T22 Example 11.39 A uniform circular disc of radius R
 4π m 
2 oscillates in a vertical plane about a horizontal axis. Find
4π 2m  2 2  the distance of the axis of rotation from the centre for which
 T1 T2  4π 2m the period is minimum. What is the value of this period?
⇒ k′ = =
 1 1 T12 + T22 Sol. The time period of a compound pendulum is minimum
4π 2m  2 + 2 
 T1 T2  when its length is equal to the radius of gyration about its
centre of gravity, i.e. l = K
m m
∴ T = 2π = 2π = T12 + T22 Since, the moment of inertia of a disc about an axis
k′ 4π 2m perpendicular to its plane and passing through its centre is
T12 + T22 equal to,
1
I = MK 2 = MR 2 ⇒ K =
R
2 2
Thus, the disc will oscillate with the minimum time period
3. The physical pendulum when the distance of the axis of rotation from the centre is
R
.
Any rigid body suspended from a rigid support capable of 2
oscillating about a horizontal axis passing through it And the value of this minimum time period will be
constitutes a physical pendulum (or compound pendulum). 2R / 2 2R 1.414R
The figure given below shows a rigid body pivoted at O, so Tmin = 2π = 2π or Tmin ≈ 2π
g g g
Simple Harmonic Motion 513

Example 11.40 Find the period of small oscillations of a Example 11.41 A ring of radius r is suspended from a point on
uniform rod with length l, pivoted at one end. its circumference. Determine its angular frequency of small
Sol. Time period of a uniform rod which is same as physical oscillations.
pendulum,
O

Sol. It is a physical pendulum, the time period of which is,


OG = l/2
I
T = 2π
I0 mgl
T = 2π
mg (OG ) Here, I = moment of inertia of the ring about point of suspension
1 l
Here, I 0 = ml 2 and OG = = mr 2 + mr 2 = 2mr 2
3 2
and l = distance of point of suspension from centre of gravity
 1 2 =r
 ml 
3 
∴ T = 2π 2 mr 2 2r
l ∴ T = 2π = 2π
(m )(g )   mgr g
 2
2π g
2l Hence, angular frequency, ω = or ω=
or T = 2π T 2r
3g

CHECK POINT 11.3


1. If the metal bob of a simple pendulum is replaced by a 6. A simple pendulum is suspended from the roof of a trolley
wooden bob, then its time period will which moves in a horizontal direction with an acceleration a,
(a) increase then the time period T is given by T = 2π l / g′ , where g′ is
(b) decrease
equal to
(c) remains the same
(d) may increase or decrease (a) g (b) g − a (c) g + a (d) g2 + a 2

2. The length of a simple pendulum is (39.2/π 2)m. If 7. A man measures the period of a simple pendulum inside a
g = 9.8 m /s 2, the value of time period is stationary lift and finds it to be T second. If the lift
(a) 4 s (b) 8 s accelerates upwards with an acceleration g/ 4 , then the
(c) 2 s (d) 3 s period of the pendulum will be
T 2T
3. The length of a simple pendulum is increased four times of (a) T (b) (c) (d) 2T 5
4 5
its initial value, its time period with respect to its previous
value will 8. A simple pendulum is made of a body which is a hollow
(a) become twice (b) not be different sphere containing mercury suspended by means of a wire.
(c) be halved (d) be 2 times If a little mercury is drained off, the period of pendulum
will
4. The length of a simple pendulum executing simple (a) remains unchanged (b) increase
harmonic motion is increased by 21%. The percentage (c) decrease (d) become erratic
increase in the time period of the pendulum of increased
length is 9. The time period of a simple pendulum of infinite length is
(a) 11% (b) 21% (R = radius of earth)
(c) 42% (d) 10.5% R
(a) infinite (b) 2π
g
5. A cabin is falling freely under gravity, what is the time
period of a pendulum attached to its ceiling? g 1 R
(c) 2π (d)
(a) zero (b) ∞ R 2g g
(c) 1 s (d) 2 s
514 OBJECTIVE Physics Vol. 1

10. An object of mass m is suspended from a spring and it 15. The spring constants of two springs of same length are k1
executes SHM with frequency n. If the mass is increased and k2 as shown in figure. If an object of mass m is
4 times, the new frequency will be suspended and set in vibration, the time period will be
(a) 2n (b) n/2 (c) n (d) n/4
11. A mass m is suspended from a spring. Its frequency of
oscillation is f. The spring is cut into two halves and the k1 k2
same mass is suspended from one of the two pieces of the
spring. The frequency of oscillation of the mass will be
f m
(a) 2 f (b) (c) f (d) 2 f
2 mk1 m
(a) 2π (b) 2π
12. Three masses 0.1 kg, 0.3 kg and 0.4 kg are suspended at end k2 k1 k2
of a spring. When the 0.4 kg mass is removed, the system
m
oscillates with a period 2 s. When the 0.3 kg mass is also (c) 2π (d) 2π m/(k1 + k2)
removed, the system will oscillate with a period k1 − k2
(a) 1 s (b) 2 s (c) 3 s (d) 4 s
16. Five identical springs are used in the three configurations as
13. Time period of a spring mass system is T. If this spring is shown in figure.
cut into two parts whose lengths are in the ratio 1 : 3 and The time periods of vertical oscillations in configurations
the same mass is attached to the longer part, the new time (a), (b) and (c) are in the ratio
period will be
3 T 3T
(a) T (b) (c) (d) 3 T
2 3 2
14. If both spring constants k1 and k2 are increased to 4 k1 and
4 k2 respectively, what will be the new frequency, if f was
the original frequency? m a m c

k1 k2
m
m b
1
(a) f (b) 2f (c) f (d) 4 f 1 1 1 1
2 (a) 1 : 2 : (b) 2 : 2 : (c) : 2 :1 (d) 2 : :1
2 2 2 2

SHM IN SOME SPECIAL CASES


Here, we will discuss motion of some objects being (ii) Motion of a liquid in U-shape tube Let an
reduced to simple harmonic motion under specific incompressible and non-viscous liquid is filled in a
condition U-tube of uniform cross-section. If the liquid is
(i) Vertical motion of a straight wire If a mass m is pressed in one limb, then it starts to perform SHM
suspended from a massless wire of length l, along limb of the tube, where time period is given by
cross-section area A and Young’s modulusY and is
h
pulled along the length of the wire, then restoring T = 2π
force will be developed by the elasticity of the wire. g

Here, h is height of liquid in one limb.


l Wire (iii) Motion of ball in bowl If a small ball of mass m is
placed at a small distance from O inside the smooth
m concave surface of radius R of a bowl and released,
∆l then it will oscillate about O.
m S
Fig. 11.21 Mass suspended from a wire
R
Here, restoring force is linear, so motion is simple
harmonic in nature, where time period is given by
R sin θ P
ml
T = 2π O
mg
YA
Fig. 11.22 A ball is moving in a bowl
Simple Harmonic Motion 515

R ∴ Restoring torque, τ = − mg R sin θ


Time period of such oscillations is T = 2π
g As I = mR 2 ⇒ τ = − mg Rθ (Since, sin θ −~ θ )

(iv) Motion of a ball in a tunnel through the earth If mgR


∴ Iα = − mgRθ ⇒ α = − θ = − ω 2θ
a ball moves through a tunnel along a diameter of I
earth, then due to gravitational force between ball Time period of such oscillation is
and earth a restoring force is set up, due to which
the ball performs SHM, whose time period is given by I mR 2 R
T = 2π = 2π ⇒ T = 2π
mgR mgR g

R Example 11.44 If the earth were a homogeneous sphere and


a straight hole was bored in it through its centre, so when a
body is dropped in the hole, it will execute SHM. Determine
the time period of its oscillation. Radius of the earth is
Fig. 11.23 A ball in a tunnel through the earth 6.4 ×10 5 m and g = 9.8 ms −2
Sol. Given, R = 6.4 × 105 m, g = 9.8 ms −2
T = 2π R /g
∴ Time period of oscillation of body will be
where, R = radius of earth. R 6.4 × 105
T = 2π = 2π = 1604.86 s
Example 11.42 A liquid of mass m is set into oscillations in a g 9.8
U-tube of cross-section A. Its time period recorded is T,
where T = 2π l /2g , here l is the length of liquid column. If
the liquid of same mass is set into oscillations in U-tube of
Free oscillations
cross-section A/16, then determine time period of oscillation. When a body oscillates with its own natural frequency, it is
Sol. As mass is constant. said to be executing free oscillations. The frequency and
 Mass  the time period of free oscillations depends on the
∴ Volume × density = constant Q Density = 
 Volume dimensions of the body and the force constant, i.e. the

inertia factor and the spring factor.
Vd1 = V2d
where,V1 andV2 are the volume of liquid in U-tube in two +A
cases-respectively.
A
(Al ) d = l ′ d ⇒ l ′ = 16 l O
y (t)

16 t

l
Q T = 2π –A
g
Fig. 11.24 Free oscillations
T′ l′ 16 l
∴ = = = 4 ⇒ T ′ = 4T The frequency of free oscillations (or natural oscillations) is
T l l
given by
Example 11.43 A small block oscillates back and forth on a
1 k
smooth concave surface of radius R. Find the time period of ν0 =
small oscillation. 2π m
e.g. The vibrations of string of a sitar when pulled aside
and released, etc.
Sol. For the oscillation of small block on a smooth concave
surface, following figure can be drawn, where R is the radius
of the concave surface. Damping and damped oscillations
O Damping is an influence within or upon an oscillatory system,
which has the effect of reducing, restricting and preventing
θ its oscillations. In physical systems, damping is produced by
R processes that dissipates the energy stored in the oscillation.
The oscillations in which the amplitude decreases gradually
with the passage of time are called damped oscillations. Most
N of the oscillations occurring in viscous media like water, air,
etc., are damped oscillations.
516 OBJECTIVE Physics Vol. 1

So, in this type of oscillations part of the energy is dissipated oscillations. The external agent which exerts the
in overcoming resistive forces. periodic force is called the driver and the oscillating
Consequently, the amplitude of oscillations goes on system under consideration is called the driven body.
decreasing exponentially with time and eventually, e.g.
these oscillations die out. e.g. Oscillations of simple pendulum (i) When the stem of a vibrating tunning fork is
in air, a swinging ball left to itself will eventually stop, etc. pressed against a table, a loud sound is heard.
Gradually falling amplitude This is because the particles of table are forced to
+A
vibrate with the frequency of the tunning fork.
(ii) When the free end of the string of a simple
O
y(t)

t pendulum is held in hand and the pendulum is


made to oscillate by giving jerks by the hand, the
–A pendulum executes forced oscillations.
Fig. 11.25 Damped oscillations
Resonance
Amplitude of damped oscillations A = A0 e − γ t When a body oscillates with its own natural frequency,
with the help of an external periodic force whose
Energy of damped oscillations E = E 0 e −2γt frequency is equal to the natural frequency of the body,
then the amplitude of the body increases and this
b
where, γ = damping coefficient = phenomenon is known as resonance.
2m
Condition for resonance ν 0 = ν d
Here, m is the mass of body undergoing damped oscillations.
where, ν 0 = natural frequency
Damping force Fd = − bv
and ν d = frequency of external periodic force.
where, b = damped constant and v = velocity of oscillation.
Equation of damped oscillations Example 11.45 The amplitude of a damped oscillator
becomes half in one minute. The amplitude after 3 min
md 2 y dy will be 1/x times of the original. Determine the value of
+b + ky = 0 x.
dt 2 dt
Sol. Amplitude of damped oscillations is A = A0 e − γt
Displacement of damped oscillations
As, A = A0 /2 at t = 1 min
y = y m e −bt/ 2m sin (ω ′ t + φ ) So,
A0
= A0e − γ
2
where, ω′ = angular frequency of damped oscillations or eγ = 2
After 3 min, the amplitude will be A0 / x.
= ω 20 − (b /2m ) 2
A0
So, = A0e − γ 3
x
Maintained oscillations or x = e 3γ = (e γ )3 = 23 = 8
The oscillations of a system are said to be maintained or
sustained oscillations, if the system vibrates with its own Vector method of combining two or more
natural frequency and with constant amplitude even in the simple harmonic motions in same direction
presence of damping. If energy is supplied to the oscillator at
A simple harmonic motion is produced when a force
the same rate at which it is dissipated, the amplitude of
(called restoring force) proportional to the displacement
oscillations remains unchanged.
acts on a particle. If a particle is acted upon by two
e.g. Balance wheel of watch, electronic oscillator, electrically such forces, the resultant motion of the particle is a
maintained tunning fork. combination of two simple harmonic motions. Suppose
the two individual motions are represented by
Forced oscillations x 1 = A1 sin ωt
When a body is maintained in a state of oscillation by a and x 2 = A2 sin (ωt + φ )
strong periodic force of frequency other than the natural
Both the simple harmonic motions have same angular
frequency of the body, the oscillations are called forced
frequency ω.
Simple Harmonic Motion 517

 π
The resultant displacement of the particle is given by and x 3 = 6 sin ωt + 
 3
A2 A Sol. Let, the resultant equation is,
x = A sin (ωt + φ )
ΣAx = 2 + 4 cos 30° + 6 cos 60° = 8.5
f
a A3 = 6
A1 Y

Fig. 11.26 A2 = 4
x = x1 + x2 30°
30° X
= A1 sin ωt + A2 sin (ωt + φ ) A1 = 2
= A sin (ωt + α )
Here, A = A12 + A22 + 2A1A2 cos φ
and ΣAy = 4 sin 30° + 6 sin 60° = 7.2
A2 sin φ
and tan α = ∴ A = (ΣAx )2 + (ΣAy )2
A1 + A2 cos φ
Thus, we can see that this is similar to the vector addition. = (8.5)2 + (7.2)2
Hence, the same method of vector addition can be applied = 11.11
to the combination of more than two simple harmonic ΣAy
motions. and tan φ =
ΣAx
Note For SHM’s moving in perpendicular directions, if
7.2
(i) φ = π, the resultant motion is along a straight line. = = 0.85
(ii) φ = π / 4, the resultant motion is in oblique ellipse. 8.5
(iii) φ = π /2 , the resultant motion is along a circle. or φ = tan−1 (0.85) = 40.4°

Example 11.46 Find the displacement equation of the simple Thus, the displacement equation of the combined motion is,
harmonic motion obtained by combining the motions. x = A sin(ω t + φ )
 π x = 11.11 sin (ωt + φ )
x1 = 2 sin ωt, x 2 = 4 sin ωt + 
 6 where φ = 40.4°

CHECK POINT 11.4


1. A point mass m is suspended at the end of a massless wire R π R
(a) π (b)
of length L and cross-sectional area A. If Y is the Young’s g 2 g
modulus of the wire. Then, the frequency of the simple R 2 R
harmonic oscillation along the vertical direction is (c) 2π (d)
g π g
1 LA 1 LAm
(a) (b)
2π mY 2π Y 4. In damped oscillations, damping force is directly
proportional to speed of oscillator. If amplitude becomes
1 YA 1 mY
(c) (d) half of its maximum value in 1s, then after 2 s, amplitude
2π mL 2π AL will be (A0 = initial amplitude)
2. A rectangular block of mass m and area of cross-section A 1 1
(a) A0 (b) A0
floats in a liquid of density ρ. If it is given small vertical 4 2
displacement from equilibrium, it undergoes oscillation 3 A0
with a time period T, then (c) A 0 (d)
2
(a) T ∝ m (b) T ∝ ρ
5. In damped oscillations, the amplitude of oscillations is
1 1
(c) T ∝ (d) T ∝ reduced to one-third of its initial value a0 at the end of 100
A ρ oscillations. When the oscillator completes 200 oscillations,
3. Imagine a narrow tunnel between the two diametrically its amplitude must be
opposite points of the earth. A particle of mass m is released (a) a 0 / 2 (b) a 0 / 4
in this tunnel. The time period of oscillation is (c) a 0 / 6 (d) a 0 / 9
518 OBJECTIVE Physics Vol. 1

6. A weakly damped harmonic oscillator of frequency n1 is (d) mean of frequency of driving force and natural frequency
driven by an external periodic force of frequency n2. When 9. In case of a forced vibration, the resonance wave becomes
the steady state is reached, the frequency of the oscillator very sharp when the
will be (a) applied periodic force is small
(a) n1 (b) n2 (b) quality factor is small
n + n2
(c) . 1 (d) (n1 + n2) (c) damping force is small
2 (d) restoring force is small
7. In the case of sustained forced oscillations, the amplitude of 10. Two sources of sound are in resonance when
oscillations (a) they look alike
(a) decreases linearly (b) they are situated at a particular distance from each other
(b) decreases sinusoidally (c) they produce the sound of same frequency
(c) decreases exponentially (d) they are excited by the same exciting device
(d) always remains constant
11. During the phenomenon of resonance,
8. In forced oscillations, a particle oscillates simple (a) the amplitude of oscillation becomes large
harmonically with a frequency equal to (b) the frequency of oscillation becomes large
(a) frequency of driving force (c) the time period of oscillation becomes large
(b) natural frequency of body (d) All of the above
(c) difference of frequency of driving force and natural frequency
Chapter Exercises
(A) Taking it together
Assorted questions of the chapter for advanced level practice

1 Which of the following quantities is always negative 7 A body oscillates in SHM according to the equation
in SHM?  π
(a) F ⋅ a (b) v ⋅ s (c) a ⋅ s (d) F ⋅ v
(in SI unit), x = 5 cos  2πt +  . Its instantaneous
 4
Here, s is displacement from mean position. displacement at t = 1s is
2 In a simple pendulum, the period of oscillation T is 2 1 5 1
(a) m (b) m (c) m (d) m
related to length of the pendulum l as 5 3 2 2
l l2
(a) = constant (b) = constant 8 A body is vibrating in simple harmonic motion. If its
T T
acceleration is 12 cms −2 at a displacement 3 cm from
2
l l the mean position, then time period is
(c) = constant (b) = constant
T2 T2 (a) 6.28 s (b) 3.14 s (c) 1.57 s (d) 2.57 s
3 A particle executing simple harmonic motion along 9 A horizontally placed spring mass system has time
Y-axis has its motion described by the equation period T. The same system is now placed on a car
y = A sin (ωt ) + B. The amplitude of the simple moving with acceleration a in horizontal direction.
harmonic motion is Then,
(a) A (b) B (c) A + B (d) A+B (a) time period will increase
(b) time period will decrease
4 Two particles are executing SHMs. The equations of (c) time period will remain constant
their motions are (d) no conclusion can be drawn
 π  3 π 10 The acceleration a of a particle undergoing SHM is
y 1 = 10 sin  ωt +  and y 2 = 5 sin  ωt + 
 4  4  shown in the figure. Which of the labelled points
corresponds to the particle being at −x max ?
What is the ratio of their amplitudes? a
(a) 1 : 1 (b) 2 : 1 1
(c) 1 : 2 (d) None of these
2 4
5 The velocity-time graph of a harmonic oscillator is t
shown in the figure below. The frequency of
oscillation is 3
v ms–1
+4 (a) 4 (b) 3
(c) 2 (d) 1
0.02 0.04
11 Two particles are executing simple harmonic motion.
0.01 0.03
T (in second) At an instant of time t, their displacements are
y 1 = A cos (ωt ) and y 2 = A sin (ωt )
–4 Then, the phase difference between y 1 and y 2 is
(a) 25 Hz (b) 50 Hz (a) 120° (b) 90° (c) 180° (d) zero
(c) 12.25 Hz (d) 33.3 Hz 12 A particle executing SHM of amplitude 4 cm and
6 The ratio of frequencies of two pendulums are 2 : 3, T = 4 s. The time taken by it to move from positive
then their lengths are in ratio extreme position to half the amplitude is
2 3 1
(a) (b) (a) 1 s (b) s
3 2 3
4 9 2 3
(c) (d) (c) s (d) s
9 4 3 2
520 OBJECTIVE Physics Vol. 1

13 The maximum acceleration of a particle in SHM is 3T


(a) the force is zero at time
made two times keeping the maximum speed to be 4
constant. It is possible when T
(b) the velocity is maximum at time
(a) amplitude of oscillation is doubled while frequency 2
remains constant (c) the acceleration is maximum at time T
(b) amplitude is doubled while frequency is halved T
(d) the potential energy is equal to total energy at time
(c) frequency is doubled while amplitude is halved 2
(d) frequency of oscillation is doubled while amplitude 20 The time period of a simple pendulum inside a
remains constant
stationary lift is 5 s. What will be the time period
14 Under the action of a force F = − kx 3, the motion of g
when the lift moves upward with an acceleration ?
a particle is (k = a positive constant) 4
(a) simple harmonic motion (a) 5s (b) 2 5 s (c) (2 + 5 ) s (d) 2 s
(b) uniformly accelerated motion
(c) not periodic 21 A linear harmonic oscillator of force constant
(d) periodic but not simple harmonic 2 × 10 6 Nm−1 and amplitude 0.01 m has a total
−1
15 A light spring of force constant 8 Nm is cut into two mechanical energy of 160 J. Its
equal halves and the two are connected in parallel; (a) maximum potential energy is 160 J
the equivalent force constant of the system is (b) maximum potential energy is 100 J
(a) 16 Nm−1 (b) 32 Nm−1 (c) 8 Nm−1 (d) 24 Nm−1 (c) minimum potential energy is zero
(d) minimum potential energy is 100 J
16 In order that the resultant path on superimposing
22 In case of a simple pendulum, time period versus
two mutually perpendicular SHM be a circle, the
length is depicted by
conditions are that
(a) the amplitudes on both SHM should be equal and they
π
should have a phase difference of
2 (a) T (b) T
(b) the amplitudes should be in the ratio 1 : 2 and the
phase difference should be zero l l
(c) the amplitudes should be in the ratio 1 : 2 and the
π
phase difference should be
2
(d) the amplitudes should be equal and the phase (c) T (d) T
difference should be zero
l l
17 The period of a particle executing SHM is 8 s. At
t = 0, it is at the mean position. The ratio of the 23 A second pendulum is moved to moon, where
distances covered by the particle in the 1st second to acceleration due to gravity is 1/6 times that of the
the 2nd second is earth, the length of the second pendulum on moon
1 1 would be
(a) (b) 2 (c) (d) 2 +1
2 +1 2 (a) 6 times (b) 12 times
−1
1 1
18 Force constant of a weightless spring is 16 Nm . A (c) times (d) times
6 12
body of mass 1 kg suspended from it is pulled down
through 5 cm from its mean position and then 24 Two identical springs of spring constant k each are
released. The maximum kinetic energy of the system connected in series and parallel as shown in figure.
(spring + body) will be A mass M is suspended from them. The ratio of their
(a) 2 × 10−2 J (b) 4 × 10−2 J (c) 8 × 10−2 J (d) 16 × 10−2 J
frequencies of vertical oscillation will be

19 The graph shows the variation of displacement of a


particle executing SHM with time. We infer from
this graph that
y
M
T/2 3T/2
T/4 T t M

3T/4 (a) 1 : 2 (b) 2 : 1 (c) 4 : 1 (d) 1 : 4


Simple Harmonic Motion 521

25 The relation between acceleration and displacement 32 The motion of a particle is given
of four particles are given below [NCERT Exemplar] x = a sin ωt + b cos ωt. The motion of the particle is
(a) a x = + 2x (b) a x = + 2x 2
[NCERT Exemplar]
(c) a x = − 2x 2 (d) a x = − 2x (a) not simple harmonic
(b) simple harmonic with amplitude a + b
Which one of the particle is exempting simple (a + b )
harmonic motion? (c) simple harmonic with amplitude
2
26 A particle executing SHM has a maximum speed of (d) simple harmonic with amplitude a 2 + b 2
30 cm s −1 and a maximum acceleration of 60 cm s −2 .
33 The displacement equation of a particle is
The period of oscillation is [NCERT Exemplar]
π x = 3 sin 2t + 4 cos 2t. The amplitude and maximum
(a) π s (b) s velocity will be respectively
2
π (a) 5, 10 (b) 3, 2 (c) 4, 2 (d) 3, 4
(c) 2π s (d) s
t 34 The displacement-time graph of a particle executing
SHM as shown in the figure.
27 A particle is attached to a vertical spring and is
pulled down a distance 0.04 m below its equilibrium
position and is released from rest. The initial upward y

acceleration of the particle is 0.30 ms −2 . The period


of the oscillation is
(a) 4.08 s (b) 1.92 s O t
(c) 3.90 s (d) 2.29 s
28 A disc of radius R and mass M is pivoted at the rim The corresponding force-time graph of the particle is
and is set for small oscillations about an axis F F
perpendicular to plane of disc. If a simple pendulum
has to have the same time period as that of the disc, (a) (b)
the length of the pendulum should be
t t
5 2 0 0
(a) R (b) R
4 3 F F
3 3
(c) R (d) R
4 2 (c) (d)
29 If the length of second’s pendulum is decreased by 0 t 0 t
2%, how many seconds, it will lose per day?
(a) 3927s (b) 3727s 35 A tunnel has been dug through the centre of the
(c) 3427s (d) 864s
earth and a ball is released in it. It will reach the
30 A clock which keeps correct time at 20°C, is other end of the tunnel after
subjected to 40°C. If coefficient of linear expansion (a) 84.6 min
of the pendulum is 12 × 10 −6 /° C. How much will it (b) 42.3 min
gain or loose in time? (c) 1 day
(d) will not reach the other end
(a) 10.3 s/day (b) 20.6 s/day
(c) 5 s/day (d) 20 min/day 36 Two bodies M and N of equal masses are suspended
31 The equation of a damped simple harmonic motion is from two separate massless springs of spring
2 constants k1 and k 2 , respectively. If the two bodies
d x dx
m 2
+b + kx = 0. Then, the angular frequency oscillate vertically such that their maximum
dt dt velocities are equal, the ratio of the amplitude of
of oscillation is vibration of M to that on N is
1/ 2
k b2 
1/ 2
k b  k1 k1
(a) ω =  −  (b) ω =  −  (a) (b)
 m 4m 2   m 4m  k2 k2
1/ 2 −1/ 2 k2 k2
k b2  k b2  (c) (d)
(c) ω =  −  (d) ω =  −  k1 k1
 m 4m   m 4m 2 
522 OBJECTIVE Physics Vol. 1

37 A particle is executing simple harmonic motion with k k k k


(a) and (b) and
a period of T seconds and amplitude in metre. The 1− α α α 1− α
a (c) αk, (1 − α ) k (d) k and k
shortest time it takes to reach a point m from its
2 43 A block of mass 0.2 kg which slides without friction
mean position in seconds is on a 30° incline, is connected to the top of the incline
(a) T (b)
T
(c)
T
(d)
T by a massless spring of force constant 80 Nm −1 as
4 8 16 shown in figure.
38 A mass M is suspended from a massless spring. An If the block is pulled slightly from its mean position,
additional mass m stretches the spring further by a what is the period of oscillations?
distance x. The combined mass will oscillate with a
period
(M + m ) x mg
(a) 2π (b) 2π
mg (M + m ) x M
(M + m ) π mg
(c) 2π (d)
mgx 2 (M + m ) x 30°

39 A particle executes linear simple harmonic motion


π 2π π
with an amplitude of 2 cm. When the particle is at (a) π s (b) s (c) s (d) s
1 cm from the mean position the magnitude of its 10 5 2
velocity is equal to that of its acceleration. Then its 44 Two simple harmonic motions are represented by
time period in seconds is the following equations
1 y 1 = 40 sin ωt
(a) (b) 2π 3
2π 3 and y 2 = 10 (sin ωt + C cos ωt )
2π 3 If their displacement amplitudes are equal, then the
(c) (d)
3 2π value of C (in appropriate units) is
40 A spring has a natural length of 50 cm and a force (a) 13 (b) 15
constant of 2 × 10 3 Nm−1. A body of mass 10 kg is (c) 17 (d) 4
suspended from it and the spring is stretched. If the 45 The potential energy of a particle of mass 2 kg in
body is pulled down to a length of 58 cm and SHM is (9x 2 ) joule. Here, x is the displacement from
released, it executes simple harmonic motion. What mean position. If total mechanical energy of the
is the net force on the body when it is at its particle is 36 J, then maximum speed of the particle
lowermost position of its oscillation? is
(Take, g = 10 ms −2 ) (a) 4 ms −1 (b) 2 ms −1
(a) 20 N (b) 40 N (c) 60 N (d) 80 N (c) 6 ms −1 (d) 10 ms −1
41 Two point masses of 3 kg and 6 kg are attached to 46 A particle of mass 0.1 kg is executing SHM of
opposite ends of horizontal spring whose spring amplitude 01 . m. When the particle passes through
constant is 300 Nm−1 as shown in the figure. The the mean position, its kinetic energy is 8 × 10 −3 J.
natural vibration frequency of the system is Find the equation of motion of the particle, if the
approximately initial phase of oscillation is 45°.
k = 300 Nm–1  π  π
(a) y = 0.1 cos 3t +  (b) y = 0.1 sin  6t + 
6 kg 3 kg  4  4
 π  π
(c) y = 01
. sin  4t +  (d) y = 0.1 cos  4t + 
 4  4
(a) 4 Hz (b) 3 Hz (c) 2 Hz (d) 1 Hz
42 In a spring-mass system, the length of the spring is 47 Molten-wax of mass m drops on a block of mass M,
L, and it has a mass M attached to it and oscillates which is oscillating on a frictionless table. Select the
with an angular frequency ω. The spring is then cut correct option.
(a) amplitude does not change
into two parts, one (i) with relaxed length αL and the
(b) amplitude increases
other (ii) with relaxed length (1 − α ) L. The force (c) time period decreases
constants of the two springs A and B are (d) time period increases
Simple Harmonic Motion 523

48 The displacement of a particle is represented by the 55 A particle of mass 2 kg moves in simple harmonic
π  motion and its potential energy U varies with
equation y = 3 cos  − 2ωt  . The motion of the
4  position x as shown in the figure. The period of
particle is [NCERT Exemplar]
oscillation of the particle is
U (J)
(a) simple harmonic with period 2π / ω 1
(b) simple harmonic with period π / ω
(c) periodic but not simple harmonic
(d) non-periodic
49 The displacement of a particle is represented by the
equation y = sin 3 ωt. The motion is x (m)
[NCERT Exemplar] 0 0.4
(a) non-periodic 2π 2 2π 2π 4π
(b) periodic but not simple harmonic (a) s (b) s (c) s (d) s
5 5 5 5
(c) simple harmonic with period 2π /ω
(d) simple harmonic with period π / ω 56 Maximum kinetic energy of a particle of mass 1 kg
50 Motion of an oscillating liquid column in a U-tube is in SHM is 8 J. Time period of SHM is 4 s. Maximum
[NCERT Exemplar] potential energy during the motion is 10 J. Then,
(a) periodic but not simple harmonic (a) amplitude of oscillations is approximately 4.53 m
(b) non-periodic (b) minimum potential energy of the particle is 4 J
(c) simple harmonic and time period is independent of the (c) maximum acceleration of the particle is approximately
density of the liquid 6.3 ms−2
(d) simple harmonic and time period is directly (d) minimum kinetic energy of the particle is 2 J
proportional to the density of the liquid 57 A simple pendulum has a time period T in vacuum.
51 A particle is acted simultaneously by mutually Its time period when it is completely immersed in a
perpendicular simple harmonic motion x = a cosωt liquid of density one-eight of the density of material
and y = a sin ωt. The trajectory of motion of the of the bob is
particle will be [NCERT Exemplar] 7 5 3 8
(a) T (b) T (c) T (d) T
(a) an ellipse (b) a parabola 9 8 8 7
(c) a circle (d) a straight line
58 A simple pendulum has time period T. The bob is
52 The equation of motion of a particle is given negative charge and surface below it is given
x = a cos (αt ) 2 . The motion is [NCERT Exemplar] positive charge. The new time period will be
(a) periodic but not oscillatory (a) less than T (b) greater than T
(b) periodic and oscillatory (c) equal to T (d) infinite
(c) oscillatory but not periodic
(d) Neither periodic nor oscillatory
59 Two simple pendulums whose lengths are 100 cm
and 121 cm are suspended side by side. Their bobs
53 A block of mass m, attached to a spring of spring are pulled together and then released. After how
constant k, oscillates on a smooth horizontal table. many minimum oscillations of the longer pendulum,
The other end of the spring is fixed to a wall. The will the two be in phase again?
block has a speed v when the spring is at its natural (a) 11 (b) 10 (c) 21 (d) 20
length. Before coming to an instantaneous rest, if the
block moves a distance x from the mean position, then 60 Some springs are combined in series and parallel
arrangement as shown in the figure and a mass m is
m 1 m m mv
(a) x = (b) x = (c) x = v (d) x = suspended from them. The ratio of their frequencies
k v k k k will be
54 In SHM, potential energy of a particle at mean
position is E 1 and kinetic energy is E 2 , then k k k
(a) E1 = E 2
3A 3E
(b) total potential energy at x = is E1 + 2
2 4 k
k
3A 3E 2
(c) total kinetic energy at x = is
2 4 m
A E2 m
(d) total kinetic energy at x = is
2 4 (a) 1 : 1 (b) 2 : 1 (c) 3 : 2 (d) 4 : 1
524 OBJECTIVE Physics Vol. 1

61 A body of mass 0.01 kg executes simple harmonic 66 A mass is suspended separately by two springs of
motion (SHM) about x = 0 under the influence of a spring constants k1 and k 2 in successive order. The
force as shown in figure. The period of the SHM is time periods of oscillations in the two cases are T1
F (N)
and T 2 , respectively. If the same mass be suspended
by connecting the two springs in parallel (as shown
8 in figure), then the time period of oscillations is T.
The correct relation is
0 +2 x (m)
–2
–8 k1 k2

(a) 1.05 s (b) 0.52 s M


(c) 0.25 s (d) 0.31 s
62 The vertical motion of a ship at sea is described by (a) T 2 = T12 + T22 (b) T −2 = T1−2 + T2−2
d 2x (c) T −1 = T1−1 + T2−1 (d) T = T1 + T2
the equation 2
= − 4x, where x is the vertical
dt 67 A horizontal platform with an object placed on it is
height of the ship (in metre) above its mean position. executing SHM in the vertical direction. The
If it oscillates through a height of 1 m, its maximum amplitude of oscillation is 4 × 10 −3 m. What must
vertical be least period of these oscillations, so that the
(a) speed will be 1 ms −1 object is not detached from the platform?
(b) speed will be 2 ms −1 (Take, g = 10 ms −2 )
(c) acceleration is 2 ms −2
π π
(d) acceleration is 1 ms −2 (a) s (b) s
25 5
63 A simple pendulum is suspended from the ceiling of π π
(c) s (d) s
a car and its period of oscillation is T when the car is 10 50
at rest. The car starts moving on a horizontal road
68 Two masses 8 kg and 4 kg are suspended together by
with a constant acceleration g (equal to the
a massless spring of spring constant 1000 Nm −1.
acceleration due to gravity, in magnitude) in the
When the masses are in equilibrium 8 kg is removed
forward direction. To keep the time period same, the
without disturbing the system. The amplitude of
length of the pendulum will have to be
oscillation is
(a) increased by 2 l
(a) 0.5 m (b) 0.08 m
(b) increased by ( 2 − 1) l (c) 0.4 m (d) 0.04 m
(c) decreased by 2 l 69 A particle executes SHM with amplitude of 20 cm
(d) decreased by ( 2 − 1) l and time period of 12 s. What is the minimum time
required for it to move between two points 10 cm on
64 A small ball is dropped from a certain height on the
either side of the mean position?
surface of a non-viscous liquid of density less than
(a) 1 s (b) 2 s
the density of ball. The motion of the ball is
(c) 3 s (d) 4 s
(a) SHM
(b) periodic but not SHM 70 A uniform spring whose unstretched length is l has a
(c) not periodic force constant k. The spring is cut into two pieces of
(d) SHM for half a period and non-periodic for rest half of unstretched lengths l1 and l 2 , where l1 = nl 2 and n is
the period being an integer. Now, a mass m is made to oscillate
65 Two particles undergoing SHM of the same with first spring. The time period of its oscillation
amplitude and frequency along the same straight would be
line. They pass one another when going in opposite mn m
directions each time their displacement is half of (a) T = 2π (b) T = 2π
k (n + 1) k (n + 1)
their amplitude. What is the phase difference
between them? m m (n + 1)
(c) T = 2π (d) T = 2π
(a) 60° (b) 30° (c) 90° (d) 120° nk nk
Simple Harmonic Motion 525

U U
71 A mass M = 5 kg is attached to a spring as shown in
the figure and held in position, so that the spring (c) (d)
remains unstretched. The spring constant is 200 Nm −1. x1 O x2 x1 O x2
The mass M is then released and begins to undergo
small oscillations. The amplitude of oscillation is 75 A solid cube floats in water half immersed and has
π
small vertical oscillations of time period s. Its mass
5
(in kg) is (Take, g = 10 ms −2 )
(a) 4 (b) 2 (c) 1 (d) 0.5
76 In the figure, the block of mass m attached to the
k M spring of stiffness k is in contact with the completely
elastic wall, and the compression in the spring is e.
The spring is compressed further by e by displacing
(a) 0.5 m (b) 0.25 m
the block towards left and is then released. If the
(c) 0.2 m (d) 0.1 m collision between the block and the wall is
completely elastic, then the time period of
72 Period of small oscillations in the two cases shown in oscillations of the block will be
figure is T1 and T 2 , respectively. Assume that, fluid
does not have any viscosity, then Wall

m
k
k
2π m m π m π m
m
(a) (b) 2π (c) (d)
m
3 k k 3 k 6 k
77 A cubical block of mass M vibrates horizontally with
amplitude of 4 cm and a frequency of 2 Hz. A small
(a) T1 = T2 (b) T1 < T2 block of mass m is placed on the bigger block. In
(c) T1 > T2 (d) Cannot say anything order that the smaller block does not slide on the
73 The variation of potential energy of harmonic bigger block, the minimum value of the coefficient
oscillator as shown in figure. The spring constant is of static friction between the two blocks is
U (joules) (a) 0.36 (b) 0.40 (c) 0.64 (d) 0.72
78 Two pendulums of time periods 3 s and 7 s
0.04
respectively, start oscillating simultaneously from
two opposite extreme positions. After how much
0.01
time they will be in same phase?
21 21 21 21
x (mm)
(a) s (b) s (c) s (d) s
8 4 2 10
0 20
79 A particle under the action of a SHM has a period of
−1 −1
(a) 1 × 10 Nm
2
(b) 1.5 × 10 Nm 2 3 s and under the effect of another it has a period
4 s. What will be its period under the combined
(c) 2 × 102 Nm −1 (d) 3 × 102 Nm −1
action of both the SHM’s in the same direction?
74 A particle of mass m oscillates with simple harmonic (a) 7 s (b) 5 s (c) 2.4 s (d) 0.4 s
motion between points x 1 and x 2 , the equilibrium 80 A particle performs SHM in a straight line. In the
position being at O. Its potential energy is plotted. It first second starting from rest, it travels a distance a
will be as given below in the graph. and in the next second, it travels a distance b on the
U U same side of mean position. The amplitude of the
SHM is
2a − b
(a) x1
x2
(b) x1
x2 (a) a − b (b)
O O 3
2a 2
(c) (d) None of these
3a − b
526 OBJECTIVE Physics Vol. 1

81 A particle is in linear SHM of amplitude A and time 86 One end of a long metallic wire of length L is tied to
period T. If v refers to its average speed during any the ceiling. The other end is tied to massless spring of
interval of T /3, then the maximum possible value of v spring constant k. A mass m hangs freely from the
is free end of the spring. The area of cross-section and
3 3 3A 2 3 3A Young’s modulus of the wire are A and Y,
(a) A (b) (c) A (d)
T T T T respectively. If the mass is slightly pulled down and
released, it will oscillate with a time period T equal to
82 A block whose mass is 2 kg is fastened to a spring
m m (YA + kL )
whose spring constant is 100 Nm−1. It is pulled to a (a) 2π (b) 2π
distance of 0.1 m from over a frictionless surface and k YAk
is released at t = 0. Calculate the kinetic energy of m (YA + kL ) ( + kL )
mY
the block when it is 0.05 m away from its mean (c) 2π (d) 2π
Ak YAk
position.
(a) 3 J (b) 0.37 J (c) 4.16 J (d) Zero 87 A particle of mass m is attached to three identical
springs A,B and C each of force constant k as shown
83 A uniform rod of length l is suspended from any in figure. If the particle of mass m is pushed slightly
point P. It is then made to undergo small oscillations. against the spring A and released, then the time
If P is the period of oscillation is
(a) centre of mass, T is zero B C
l
(b) end point, T is 2π 90°
3g
45°
l m
(c) centre of mass, T = 2π O
g
(d) end point, T is infinite
84 The system shown in figure is in A
equilibrium. The mass of the container k 2m m m m
with liquid is M, density of liquid in (a) 2π (b) 2π (c) 2π (d) 2π
k 2k k 3k
the container is ρ and the volume of A
the block is V. If the container is now 88 A mass M is attached to a horizontal spring of force
displaced downwards through a constant k fixed one side to a rigid support as shown
distance x 0 and released such that the in figure. The mass oscillates on a frictionless
block remains well inside the liquid, surface with time period T and amplitude A. When
then during subsequent motion, the mass M is in equilibrium position, another mass
M m is gently placed on it. What will be the new
(a) time period of SHM of the container will be 2π amplitude of oscillations?
k
(b) time period of SHM of the container m
M + ρV k
will be 2π M
k
(c) amplitude of SHM of the container is x 0
(d) amplitude of SHM of the container is 2x 0 (M + m ) (M − m )
(a) A (b) A
85 A block of mass 100 g attached to a spring of spring M M
constant 100 Nm −1 is lying on a frictionless floor as (c)
M
A (d)
M
A
shown in the figure. The block is moved to compress (M + m ) (M − m )
the spring by 10 cm and then released. If the
collisions with the wall in front are elastic, then the 89 Four pendulums A, B, C and D are suspended from
time period of the motion is the same elastic support as shown in figure.
[NCERT Exemplar]
G G
k = 100 Nm–1
100 g

5 cm B A
C
D
(a) 0.2 s (b) 0.1 s (c) 0.15 s (d) 0.132 s
Simple Harmonic Motion 527

A and C are of the same length, while B is smaller y


than A and D is larger than A. If A is given a P (t = 0)
transverse displacement, T = 30 s
B
(a) D will vibrate with maximum amplitude
(b) C will vibrate with maximum amplitude O x
(c) B will vibrate with maximum amplitude
(d) All the four will oscillate with equal amplitude
90 Figure shows the circular motion of a particle. The
 2 πt   πt 
radius of the circle, the period, sense of revolution (a) x (t ) = B sin   (b) x (t ) = B cos  
 30  15 
and the initial position are indicated on the figure.
The simple harmonic motion of the x-projection of  πt π   πt π 
(c) x (t ) = B sin  +  (d) x (t ) = B cos  + 
the radius vector of the rotating particle P is 15 2  15 2 
[NCERT Exemplar]

(B) Medical entrance special format questions


Assertion and reason 5 Assertion Time period of a spring block system is
T. If length of spring is decreased, time period will
Directions (Q. Nos. 1-5) These questions consists of two decrease.
statements each printed as Assertion and Reason. While Reason If length is decreased, then the block will
answering these questions you are required to choose any have to travel less distance and it will take less time.
one of the following four responses
(a) If both Assertion and Reason are correct and Reason is the Statement based questions
correct explanation of Assertion.
(b) If both Assertion and Reason are correct but Reason is not 1 The displacement-time graph of a particle executing
the correct explanation of Assertion. SHM as shown in figure. Which of the following
(c) If Assertion is correct but Reason is incorrect. statement(s) is/are false?
(d) If Assertion is incorrect but Reason is correct.
1 Assertion In the x-t graph of a particle in SHM, y
acceleration of particle at time t 0 is positive but
velocity is negative. t
x T 3T T
4 T 4
2
t0 t

(a) The acceleration is maximum at t = T .


3T
(b) The force is zero at t = .
4
Reason a ∝ −x and velocity is slope of x-t graph.
(c) The potential energy equals the total oscillation
2 Assertion If a pendulum is suspended in a lift and T
energy at t = .
lift accelerates upwards, then its time period will 2
decrease. (d) None of the above
Reason Effective value of g will be 2 A mass of 0.2 kg is attached to the lower end of a
g eff = g + a
massless spring of force constant 200 Nm −1, the
3 Assertion If amplitude of SHM is increased, time upper end of which is fixed to a rigid support.
period of SHM will increase. Which of the following statement(s) is/are true?
Reason If amplitude is increased, body will have to (a) In equilibrium, the spring will be stretched by 1 cm.
travel more distance in one complete oscillation. (b) If the mass is raised till the spring becomes unstretched
and then released, it will go down by 2 cm before
4 Assertion In SHM, v-x graph is an ellipse, where v
moving upwards.
is velocity and x is displacement from mean position. (c) The frequency of oscillation will be nearly 5 Hz.
v2 x2 (d) All of the above
Reason Equation between v and x is 2 + = A2.
ω 1
528 OBJECTIVE Physics Vol. 1

3 The speed (v ) of a particle moving along a straight II. In SHM, speed is constant.
line, when it is at a distance (x ) from a fixed point on Which of the following statement(s) is/are correct?
the line, is given by v 2 = 144 − 9x 2 . Which of the (a) Both I and II (b) Only I
(c) Only II (d) Neither I nor II
following statement(s) is/are correct?
(a) The magnitude of acceleration at a distance 3 units 8 I. Average kinetic energy in one oscillation during
from the fixed point is 27 units. 1
2π SHM of a body is mω 2A 2 .
(b) The motion is simple harmonic with T = units. 4
3 1
(c) The maximum displacement from the fixed point is II. Maximum kinetic energy is mω 2A 2 .
4 units. 2
(d) All of the above Which of the following statement(s) is/are correct?
(a) Only I (b) Only II
4 The potential energy of a particle of mass 0.1 kg, (c) Both I and II (d) Neither I nor II
moving along the X-axis, is given by U = 5x (x − 4) J,
9 I. In a spring block system, if length of spring and
where x is in metre. Which of the following
mass both are halved, time period of oscillation
statement(s) is/are incorrect?
will remain unchanged.
(a) The speed of the particle is maximum at x = 2 m.
(b) The particle executes simple harmonic motion. k
π
II. Angular frequency of SHM is ω = .
(c) The period of oscillation of the particle is s. m
7 where, k is spring constant and m is mass of block.
(d) All are correct
Which of the following statement(s) is/are correct?
5 In the two block spring system, force constant of (a) Only I
spring is k = 6 Nm −1. Spring is stretched by 12 cm (b) Only II
towards left. (c) Both I and II
(d) Neither I nor II
1 kg 2 kg
10. F-x and x-t graph of a particle in SHM as shown in
figure.
Now, choose the correct statement(s). F (N)
(a) Angular frequency of oscillation is 3 rad s −1. 10
(b) Maximum kinetic energy of 1 kg is 4.8 mJ.
x (m)
(c) Maximum kinetic energy of 2 kg is 4.8 mJ.
–1
(d) Both (a) and (c)
6 A particle of mass m is dropped from a great height h
above the hole in the earth dug along its diameter.
Then, which of the following statement is correct? x (cm)
4
(a) The motion of the particle is simple harmonic.
(b) The motion of the particle is periodic. 8 t (s)
(c) The speed of the particle at the centre of earth equals 4
2GM
, where R and M are the radius and mass of
(R + h )
the earth, respectively.
(d) The speed of the particle at the centre of earth equals Now, study the following statements.
GM (R + 3h )  160 
, where R and M are the radius and mass I. Mass of the particle is  2  SI unit.
R (R + h ) π 
of the earth, respectively. II. Maximum kinetic energy of the particle is
7 I. In SHM to find time taken in moving from one 6 × 10 −3 SI unit.
point to another point, we cannot apply the Which of the following statement(s) is/are correct?
Distance (a) Only I (b) Only II
relation, Time =
Speed (c) Both I and II (d) Neither I nor II
Simple Harmonic Motion 529

Codes
Match the columns A B C D A B C D
1 In SHM, match the Column I with Column II and (a) p q r s (b) s q p r
mark the correct option from the codes given below. (c) q s s s (d) r p s p
Column I Column II 3 In case of second’s pendulum, match the Column I
(A) Displacement and velocity (p) Phase difference of zero with Column II and mark the correct option from the
(B) Displacement and (q) Phase difference of π codes given below. (Consider shape of earth also).
acceleration 2 Column I Column II
(C) Velocity and acceleration (r) Phase difference of π
(A) At pole (p) T > 2s
Codes
(B) On a satellite (q) T < 2s
A B C A B C
(a) r p q (b) q r q (C) At mountain (r) T = 2s
(c) p q r (d) p q p (D) At centre of earth (s) T =0
2 In SHM, match the Column I with Column II and (t) T =∞
mark the correct option from the codes given below.
Codes
Column I Column II
A B C D
(A) Acceleration-displacement graph (p) Parabola (a) q t p t
(B) Velocity-acceleration graph (q) Straight line (b) r p s t
(c) p q r s
(C) Velocity-time graph (r) Circle
(d) t s q p
(D) Acceleration-time graph (s) None

(C) Medical entrances’ gallery


Collection of questions asked in NEET & Various Medical Entrance Exams

1 Identify the function which represents a periodic 6 A pendulum is hung from the roof of a sufficiently
motion. [NEET 2020] high building and is moving freely to and fro like a
(a) e ωt (b) log e (ωt ) simple harmonic oscillator. The acceleration of the
(c) sin ωt + cos ωt (d) e − ωt bob of the pendulum is 20 m/s 2 at a distance of 5 m
from the mean position. The time period of
2 The phase difference between displacement and
oscillation is [NEET 2018]
acceleration of a particle in a simple harmonic
(a) 2 s (b) π s (c) 2 π s (d) 1 s
motion is [NEET 2020]
3π π 7 A block of rectangular size of mass m and area of
(a) rad (b) rad (c) zero (d) π rad cross-section A, floats in a liquid of density ρ. If we
2 2
give a small vertical displacement from equilibrium,
3 The distance covered by a particle undergoing SHM
it undergoes SHM with time period T, then
in one time period is (amplitude = A) [AIIMS 2018]
[NEET (Odisha) 2019]
1
(a) zero (b) A (c) 2A (d) 4A (a) T ∝
2
(b) T ∝ ρ
2
ρ
4 The displacement of a particle executing simple 1
harmonic motion is given by (c) T 2 ∝ m −1 (d) T 2 ∝
A −2
y = A0 + A sin ωt + B cos ωt [NEET 2019]
8 A particle executing SHM having amplitude 5cm,
Then, the amplitude of its oscillation is given by
mass 0.5 kg and angular frequency 5 rad/s is at 1cm
(a) A2 + B 2 (b) A02 + (A + B )2 from mean position. Find potential energy and
(c) A + B (d) A0 + A2 + B 2 kinetic energy. [JIPMER 2018]
(a) KE = 6.25 × 10−4 J, PE = 150 × 10−3 J
5 Average velocity of a particle executing SHM in one (b) KE = 150 × 10−4 J, PE = 6.25 × 10−4 J
complete vibration is [NEET 2019] (c) KE = 6.25 × 10−4 J, PE = 6.25 × 10−4 J
2
(a)Aω (b)

(c) zero (d)
Aω (d) KE = 150 × 10−3 J, PE = 150 × 10−4 J
2 2
530 OBJECTIVE Physics Vol. 1

π
9 Given equation of SHM isY = 5 cos (100 t − 2x ), (a) v = v 02 + ω 2 x 2 (b) v = v 02 − ω 2 x 2
what is time period? 2 [JIPMER 2018] 3
(c) v = 3 v 03 + ω 3 x 3 (d) v = v 0 − (ω 3 x 3e x )1/ 3
(a) 0.04 s (b) 1 s
(c) 0.06 s (d) 0.02 s 16 A particle of mass 200 g executes simple harmonic
10 A particle executes linear simple harmonic motion motion. The restoring force is provided by a spring
with an amplitude of 3 cm. When the particle is at of spring constant 80 Nm −1. What would be the time
2 cm from the mean position, the magnitude of its period? [UK PMT 2015]
velocity is equal to that of its acceleration. Then, its (a) 0.93 s (b) 0.62 s
time period in seconds is [NEET 2017] (c) 0.31 s (d) None of these
5 5 4π 2π 17 A body of mass 40 g executing simple harmonic
(a) (b) (c) (d)
π 2π 5 3 motion of amplitude 2 cm. If the time period is
0.20 s, what would be the total mechanical energy of
11 The displacement of a particle along the X-axis is
the system? [UK PMT 2015]
given by x = a sin 2 ωt. The motion of the particle
(a) 14.8 × 10−3 J (b) 7.9 × 10−3 J
corresponds to [JIPMER 2017]
(a) simple harmonic motion of frequency ω / π (c) 23.7 × 10−3 J (d) 3.9 × 10−3 J
(b) simple harmonic motion of frequency 3 ω /2π 18 A copper sphere attached to the bottom of a vertical
(c) non-simple harmonic motion spring is oscillating with time period 10 s. If the
(d) simple harmonic motion of frequency ω / 2π copper sphere is immersed in a fluid (assume the
12 A body of mass m is attached to the lower end of a viscosity of the fluid is negligible) of specific gravity
spring whose upper end is fixed. The spring has 1
of that of the copper, then time period of the
negligible mass. When the mass m is slightly pulled 4
down and released, it oscillates with a time period of oscillation is [EAMCET 2015]
3 s. When the mass m is increased by 1 kg, the time (a) 5 s (b) 10 s (c) 2.5 s (d) 20 s
period of oscillations becomes 5 s. The value of m
19 Out of the following functions representing motion
(in kg) is [NEET 2016]
3 4 16 9 of a particle, which functions/function represent
(a) (b) (c) (d) SHM?
4 3 9 16
A. y = sin ωt − cos ωt B. y = sin 3 ωt
13 Two similar springs P and Q have spring constants
kP and kQ , such that kP > kQ . They are stretched,  3π 
C. y = 5 cos − 3ωt  D. y = 1 + ωt + ω 2t 2 + ω 3t 3
first by the same amount (case a), then by the same  4  [CG PMT 2015]
force (case b). The work done by the springsWP and (a) A and B (b) A and C
WQ are related as, in case (a ) and case (b ), (c) A only (d) A, B and C not D
respectively [CBSE AIPMT 2015]
20 The oscillation of a body on a smooth horizontal
(a)WP = WQ ; WP > WQ (b)WP = WQ ; WP = WQ
surface is represented by the equation x = A cos ωt.
(c)WP > WQ ; WQ > WP (d)WP < WQ ; WQ < WP
where, x = displacement at time t
14 A particle is executing SHM along a straight line. Its and ω = frequency of oscillation.
velocities at distances x 1 and x 2 from the mean
Which one of the following graphs shows correctly
position are v 1 and v 2 , respectively. Its time period is
the variation of a with t? [CBSE AIPMT 2014]
[CBSE AIPMT 2015; EAMCET 2013]
Here, a = acceleration at time t
x 12 + x 22 x 22 − x12 and T = time period.
(a) 2π (b) 2π
v12 + v 22 v12 − v 22
a a
v12 + v 22 v12 − v 22
(c) 2π (d) 2π (a) 0 (b) 0
x12 + x 22 x12 − x 22 T t T t

15 The velocity vector v and displacement vector x of a


particle executing SHM are related as
v dv a a
= − ω 2 x with the initial condition v = v 0 at (c) 0 (d) 0 T
dx t t
x = 0. The velocity v when displacement is x, is
[AIIMS 2015]
Simple Harmonic Motion 531

21 The ratio of kinetic energy to the potential energy of 1 µg 1 µg


(a) (b)
a particle executing SHM at a distance equal to half 2π A 4π A
its amplitude, the distance being measured from its A A
equilibrium position is [KCET 2015; Kerala CEE 2014] (c) 2π (d) 4π
µg µg
(a) 2 : 1 (b) 3 : 1 (c) 8 : 1 (d) 4 : 1
(e) 1 : 1 29 The displacement of a particle in a periodic motion
22 A body oscillates with SHM according to the is given by y = 4 cos 2 (t /2) sin (1000t ). This
 π displacement may be considered as the result of
equation (in SI unit), x = 5 cos  2πt +  . Its
 4 superposition of n independent harmonic oscillations.
instantaneous displacement at t = 1s is Here, n is [WB JEE 2014]
[Kerala CEE 2014] (a) 1 (b) 2 (c) 3 (d) 4
2 1 1 5
(a) m (b) m (c) m (d) m 30 The time period of a simple pendulum of length
5 3 2 2 9.8 m is [JK CET 2013]
1 (a) 0.159 s (b) 3.14 s (c) 6.5 s (d) 6.28 s
(e) m
2
31 A particle executing simple harmonic motion covers
23 The amplitude of a particle executing SHM is 4 cm. At a distance equal to half its amplitude in one second.
the mean position, the speed of the particle is 16 cms −1. Then, the time period of the simple harmonic motion
The distance of the particle from the mean position at is [Kerala CEE 2013]
which the speed of the particle becomes (a) 4 s (b) 6 s (c) 8 s (d) 12 s
8 3 cms −1 will be [AFMC 2014]
(e) 20 s
(a) 2 3 cm (b) 3 cm (c) 1 cm (d) 2 cm 32 The displacement, velocity and acceleration in a simple
harmonic motion are related as the [JK CET 2013]
24 What is the phase difference between two simple
(a) displacement, velocity and acceleration all act in the
harmonic motions represented by same direction
 π
x 1 = A sin  ωt +  and x 2 = A cosωt ? (b) displacement and velocity act in the same direction but
 6 [WB JEE 2014] acceleration in the opposite direction
π π π 2π (c) velocity and acceleration are parallel and both are
(a) (b) (c) (d) perpendicular to the displacement
6 3 2 3
(d) displacement and acceleration are anti-parallel and both
25 When a particle executing SHM oscillates with a perpendicular to the velocity
frequency ν, then the kinetic energy of the particle
[WB JEE 2014] 33 If T1 and T 2 are the time periods of oscillation of a
(a) changes periodically with a frequency of ν simple pendulum on the surface of earth (of radius R)
(b) changes periodically with a frequency of 2 ν and at a depth d, the d is equal to [Kerala CEE 2013]
(c) changes periodically with a frequency of ν/2  T 2  T 2
(d) remains constant (a) 1 − 12  R (b) 1 − 22  R
 T2   T1 
26 If a body is executing simple harmonic motion and  T   T 
its current displacement is 3 /2 times the amplitude (c) 1 − 1  R (d) 1 − 2  R
 T2   T1 
from its mean position, then the ratio between T12
potential energy and kinetic energy is [EAMCET 2014] (e)
T22
(a) 3 : 2 (b) 2 : 3 (c) 3 : 1 (d) 3 : 1
34 In case of a forced vibration, the resonance wave
27 A 10 kg metal block is attached to a spring of spring becomes very sharp when the [UP CPMT 2013]
constant 1000 Nm −1. A block is displaced from (a) restoring force is small
equilibrium position by 10 cm and released. (b) damping force is small
The maximum acceleration of the block is [KCET 2014] (c) quality factor is small
(d) applied periodic force is small
(a) 10 ms−2 (b) 100 ms−2 (c) 200 ms−2 (d) 0.1 ms−2
35 When a boy is playing on a swing in the sitting
28 A block resting on the horizontal surface executes
SHM in horizontal plane with amplitude A. The position, the time period of oscillations of the swing
frequency of oscillation for which the block just is T . If the boy stands up, the time period of
starts to slip is (µ = coefficient of friction, oscillation of the spring will be [UP CPMT 2013]
(a) more than T (b) less than T
g = gravitational acceleration) [MHT CET 2014]
(c) equal to T (d) Cannot be predicted
532 OBJECTIVE Physics Vol. 1

36 To make the frequency of an oscillator double, one 41 If the equation y 1 = A sinωt and
has to [UP CPMT 2013] A A
(a) half the mass y2 = sin ωt + cos ωt represents SHM, then the
2 2
(b) quadruple the mass
ratio of the amplitudes of the two motions is
(c) double the mass [BCECE 2012]
(d) reduce the mass to one-fourth
(a) 0.5 (b) 2 (c) 1 (d) 2
37 A simple harmonic oscillator consists of a particle of
42 Assertion If a bob of a simple pendulum is kept in
mass m and an ideal spring with spring constant k.
The particle oscillates with a time period T. The a horizontal electric field, its period of oscillation
spring is cut into two equal parts. If one part will remain same.
oscillates with the same particle, the time period will Reason If bob is charged and kept in horizontal
be [AIIMS 2012] electric field, then the time period will be
(a) 2 T (b) 2 T
decreased. [BCECE 2012]
(a) If both Assertion and Reason are correct but Reason is
T T
(c) (d) the correct explanation of Assertion.
2 2 (b) If both Assertion and Reason are correct but Reason is
38 A SHM is given by y = 5 [sin (3 πt ) + 3 cos (3 πt )]. not the correct explanation of Assertion.
(c) If Assertion is correct and Reason is incorrect.
What is the amplitude of the motion of y in metre? (d) If Assertion is incorrect and Reason is correct.
[UP CPMT 2012]
(a) 10 (b) 20 (c) 1 (d) 5 43 The displacements of two particles of same mass
39 If a simple pendulum is taken to a place, where g executing SHM are represented by the equations
decreases by 2%, then time period  π
[UP CPMT 2012] x 1 = 4 sin10 t +  and x 2 = 5 cos (ωt ).
(a) increase by 5%  6
(b) increase by 1% The value of ω for which the energies of both the
(c) increase by 2% particles remain same is [Manipal 2012]
(d) decrease by 5%
(a) 16 unit (b) 6 unit (c) 4 unit (d) 8 unit
40 If a simple pendulum executing SHM falls freely
44 The amplitude and the periodic time of a SHM are
along with the support, then [BCECE 2012]
5 cm and 6 s, respectively. At a distance of 2.5 cm
(a) it does not oscillate at all
away from the mean position, the phase will be
(b) its periodic time increases [JCECE 2012]
(c) its periodic time decreases π π π 5π
(d) None of the above (a) (b) (c) (d)
3 4 6 12

ANSWERS
l CHECK POINT 11.1
1. (c) 2. (a) 3. (d) 4. (c) 5. (b) 6. (b) 7. (c) 8. (a) 9. (b) 10. (c)
11. (c) 12. (b) 13. (b) 14. (a) 15. (b) 16. (c) 17. (b)

l CHECK POINT 11.2


1. (b) 2. (d) 3. (b) 4. (c) 5. (b) 6. (b) 7. (c) 8. (d) 9. (d) 10. (b)
11. (a) 12. (b)

l CHECK POINT 11.3


1. (c) 2. (a) 3. (a) 4. (d) 5. (b) 6. (d) 7. (c) 8. (b) 9. (b) 10. (b)
11. (a) 12. (a) 13. (b) 14. (b) 15. (d) 16. (a)
Simple Harmonic Motion 533

l CHECK POINT 11.4


1. (c) 2. (d) 3. (c) 4. (a) 5. (d) 6. (b) 7. (d) 8. (a) 9. (c) 10. (c)
11. (a)

(A) Taking it together


1. (c) 2. (c) 3. (a) 4. (b) 5. (a) 6. (d) 7. (c) 8. (b) 9. (c) 10. (d)
11. (b) 12. (c) 13. (c) 14. (d) 15. (b) 16. (a) 17. (c) 18. (a) 19. (d) 20. (d)
21. (a) 22. (b) 23. (c) 24. (a) 25. (d) 26. (a) 27. (d) 28. (d) 29. (d) 30. (a)
31. (a) 32. (d) 33. (a) 34. (d) 35. (b) 36. (d) 37. (c) 38. (a) 39. (c) 40. (c)
41. (c) 42. (b) 43. (b) 44. (b) 45. (c) 46. (c) 47. (d) 48. (b) 49. (b) 50. (c)
51. (c) 52. (c) 53. (c) 54. (b) 55. (d) 56. (c) 57. (d) 58. (a) 59. (b) 60. (c)
61. (d) 62. (b) 63. (a) 64. (b) 65. (d) 66. (b) 67. (a) 68. (b) 69. (b) 70. (a)
71. (b) 72. (b) 73. (b) 74. (c) 75. (a) 76. (a) 77. (c) 78. (a) 79. (c) 80. (c)
81. (a) 82. (b) 83. (b) 84. (b) 85. (d) 86. (b) 87. (b) 88. (c) 89. (b) 90. (a)

(B) Medical entrance special format questions


l Assertion and reason
1. (a) 2. (a) 3. (d) 4. (a) 5. (c)

l Statement based questions


1. (d) 2. (d) 3. (d) 4. (c) 5. (d) 6. (d) 7. (b) 8. (c) 9. (b) 10. (a)

l Match the columns


1. (b) 2. (c) 3. (a)

(C) Medical entrances’ gallery


1. (c) 2. (d) 3. (d) 4. (a) 5. (c) 6. (b) 7. (a) 8. (b) 9. (a) 10. (c)
11. (c) 12. (d) 13. (c) 14. (b) 15. (b) 16. (c) 17. (b) 18. (b) 19. (b) 20. (c)
21. (b) 22. (d) 23. (d) 24. (b) 25. (b) 26. (d) 27. (a) 28. (a) 29. (c) 30. (d)
31. (d) 32. (b) 33. (a) 34. (b) 35. (b) 36. (d) 37. (c) 38. (a) 39. (b) 40. (a)
41. (b) 42. (b) 43. (d) 44. (c)
OBJECTIVE Physics Vol. 1

Hints & Explanations


l CHECK POINT 11.1 12 (b) Q a = − ω 2x
1 (c) The motion of planets around the sun is periodic but not x 1
∴ = − 2 = constant
simple harmonic motion. a ω
2 (a) The body is non-uniformly accelerated in case of simple π
13 (b) Given, ω = rad s −1 and A = 2 cm
harmonic motion. 2
 2π  π2
3 (d) y = A sin (ωt + φ ) = A sin  t + φ ∴ Maximum acceleration = ω 2A = cms −2
T  2
 2π π
⇒ y = 0.5 sin  t+  14 (a) Q ω 2A = a 0 ...(i)
 0.4 2
and ωA = v 0 ...(ii)
 π
y = 0.5 sin  5πt +  = 0.5 cos 5πt On solving Eqs. (i) and (ii), we get
 2
v2
A= 0
4 (c) When the particle moves along the circumference of a a0
circle with constant angular velocity, the projection of the
a 8
particle on any of the diameters executes SHM. 15 (b) a = ω 2 y ⇒ ω = = = 2 rads −1
y 2
5 (b) Given equation, y = 10 sin (20t + π / 3) ...(i)
Comparing Eq. (i) with general equation of motion of simple Now, vmax = Aω = 6 × 2 = 12 cms −1
harmonic x = A sin(ωt + φ ) , we get 16 (c) The average acceleration of a particle performing SHM
A = 10 m ω = 20 rad s −1 over one complete oscillation is zero. Because acceleration is a
function of sine and average value of sine for a complete
2π 2π π
As, ω= ⇒ T= = second oscillation is zero.
T ω 10
17 (b) As, x = A cos ωt
6 (b) Given equation, x = 0.34cos (3000 t + 0.74) …(i)
 π
Velocity, v = Aω cos ωt +  and the acceleration,
Comparing Eq. (i) with general equation of motion of simple  2
harmonic x = A cos (ωt + φ ), we get a = Aω cos (ωt + π )
2

A = 0.34 mm, ω = 3000 rad s −1


Here, A = amplitude.
ω 3000
∴ Frequency, ν = = Hz
2π 2π x
(i)
t
−1
7 (c) ∴ Velocity, v = ω A − x = 2 (60 ) − (20 ) ≈ 113 mms
2 2 2 2

v0 (ii)
8 (a) ∴ Maximum velocity, vmax = Aω = 3 × 100 = 300 units t
3 3
9 (b) Here, ωA = 3 ms −1 or ω =
= = 30 rad s −1 a0
t
(iii)
A 0.1
ω 30  15
∴ Frequency, f = = =   Hz ∴ In SHM, the acceleration is ahead of velocity by a phase
2π 2π  π
π
angle .
10 (c) Velocity at mean position v = Aω, velocity at a distance of 2
half amplitude,
A2 3 3
l CHECK POINT 11.2
v ′ = ω A2 − y 2 = ω A2 − ⇒ v′ = Aω = v
4 2 2 1 (b) Given, A = 6 cm, ω = 100 rad s −1
v 10 1
11 (c) ∴ vmax = Aω ⇒ ω = max = Maximum kinetic energy, Kmax = mω 2A2
A 4 2
v = ω A2 − y 2 1
Now, = × 1 × (100 )2 × (6 × 10 −2 )2 = 18 J
2
⇒ v 2 = ω 2 (A2 − y 2 )
v2 1 1  A2   A
⇒ y 2 = A2 − 2 2 (d) KE = m ω 2 (A2 − x 2 ) ⇒ KE = m ω 2 A2 −  Q x = 
2 2  4  2
ω
3 1 3
v2 52 = × m ω 2A2 = E
⇒ y = A2 − = 42
− = 2 3 cm 4 2 4
ω2 (10 / 4)2
Simple Harmonic Motion 535

3 (b) Frequency of oscillation of kinetic energy is doubled, i.e. 1 1 1 


⇒ mω 2 y 2 = ×  mω 2A2
2ω. 2 2 2 
2π π A 6
∴ T= = ⇒ y= =
2ω ω 2 2
4 (c) Amplitude, A = 4 cm = 4.2 cm
For, KE = PE l CHECK POINT 11.3
1 1
⇒ m ω 2 (A2 − x 2 ) = m ω 2x 2 1 (c) Time period will remains the same because time period T
2 2
of simple pendulum is independent of mass of the bob.
A2 A
⇒ x2 = ⇒ x= l 39.2
2 2 2 (a) T = 2π ⇒ T = 2π =4s
4 g π 2 × 9.8
∴ x= = 2 2 cm
2 3 (a) According to question, l′ = 4 l
1
mω 2 (A2 − y 2 ) 4l
K 2 A2 − y 2 y2 Q T ′ = 2π = 2T
5 (b) = = 2
= 1− 2 g
E 1 A A
mω 2A2
2 l ∆T 1 ∆l
4 (d) As, T = 2π ⇒ =
 3E  g T 2 l
 
 4 y2 ∆l
So, = 1− 2 Given, = 21%
E A l
y2 3 1 A ∆T 1
⇒ = 1− = ⇒ y= ⇒ = × 21% = 10.5%
A2 4 4 2 T 2
6 (b) Potential energy is minimum and kinetic energy is l
5 (b) T = 2π . Here, geff = g − g, which is zero in a freely
maximum, at mean position, i.e. at x = 0. geff
E falling lift. Hence, T = ∞
7 (c) U = K or U =
2 6 (d) The pendulum is suspended from the roof of a trolley
1 2 1  1 2 which is moving in horizontal direction with acceleration a.
∴ kx =  kA 
2 2 2  Hence, the force acting will be
A F = ma$i − mg$j
i.e. at x = ± , U =K
2
Let θ angle between the horizontal and vertical, then
and this situation will occur four times in one complete ma a
period. tanθ = =
mg g
8 (d) The work done by the pendulum in one oscillation will be
zero because displacement is zero. Then, the value of g′ = g 2 + a 2 .
1
9 (d) As, E = m ω 2A2 l
2 7 (c) In stationary lift, T = 2π
2
1  2π  2 2π  g

or E = m  A Q ω = 
2 T  T In upward motion of lift,
As, T ′ = 2T and A′ = 2A T ′ = 2π
l
(a = acceleration of lift)
1 1  2π 
2 (g + a )
∴ E ′ = m ×   × 4 (A)2 = E
2 4 T  T′ g g  g
Q = =  given, a = 
1 T g+a  g  4
10 (b) Energy in SHM, E = mω 2A2 g + 
2  4
i.e. E depends on ω and A both. 4 2T
= ⇒ T′ =
1 1 5 5
11 (a) E = mω 2A2 = m (2πf )2 A2 = 2π 2f 2mA2
2 2 8 (b) When a little mercury is drained off, the position of centre
= 2π 2 (20)2 (0.10)(0.05)2 ~
− 2J of mass of ball falls (with respect to fixed end), so that
12 (b) If at any instant, displacement is y, then it is given that effective length of pendulum increases, hence T increases.
1 R
U = ×E 9 (b) T = 2π = 84.6 min (If l is infinite)
2 g
536 OBJECTIVE Physics Vol. 1

1 k 1 4 (a) Amplitude of damped oscillation is A = A0e − γt


10 (b) n= ⇒ n∝
2π m m A0
∴ n′ = n/2 (Q m′ = 4m ) Given, A = , at t = 1s
2
1 A0
11 (a) k ∝ and f ∝ k So, = A0e −γ1 or e γ = 2
l 2
Spring constant of each part will be 2k. A0
∴ f′ = 2 f After 2 s, A = A0e − γ 2 ⇒ A =
4
m 5 (d) In damped oscillation, amplitude goes on decaying
12 (a) T = 2π or T ∝ m
k exponentially as a = a 0e − bt , where b = damping coefficient.
T m1 a0
∴ = Initially, = a 0e − b × 100 T , T = time of one oscillation
T′ m2 3
(0.8 − 0.4) 1

T
= =
2
(Q Given, T = 2 s) or = e −100 bT ...(i)
T′ (0.8 − 0.4 − 0.3) T ′ 3
Finally, a = a 0e − b × 200T
0.4 2 2
⇒ = ⇒ = 2 ⇒ T ′ = 1s  1
2
0.1 T ′ T′ or a = a 0 [e −100 bT ]2 or a = a 0 × [From Eq. (i)]
 3 
13 (b) The spring mass system oscillates in SHM, its time period
or a = a0 / 9
is given by
m 6 (b) With weak damping frequency (n1) of the oscillator, the
T = 2π
k system reaches frequency of driving force (n 2) at steady state.
When spring is cut into ratio 1 : 3, the new time constant is
7 (d) Amplitude always remains constant in case of sustained
k′ = 3k
T 3k T forced oscillations.
∴ = ⇒ T′ =
T′ k 3 8 (a) In forced oscillations, particle oscillates with a frequency
equal to the frequency of driving force.
1 k
14 (b) f = or f ∝ k 9 (c) In resonant vibrations of a body, the frequency of external
2π m
force applied on the body is equal to its natural frequency. If
k = k1 + k2 and k′ = 4(k1 + k2 ) = 4k
on increasing or decreasing the frequency of external force
So, f ∝ k′ ⇒ f ′ ∝ 4k from the natural frequency by a large factor, the amplitude of
f ′ ∝ 2 k ⇒ f ′ ∝ 2f vibrations reduces very much. In this case, sharp resonance
will take place.
15 (d) The springs are connected in parallel. So, the effective But if it reduces by a small factor, then flat resonance will
force constant will be keff = k1 + k2 take place. The sharp and flat resonance will depend on
m m damping present in the body executing resonant vibrations.
∴ T = 2π = 2π
keff k1 + k2 Lesser the damping, greater will be sharpness.

m 10 (c) Two sources of sound are in resonance when they produce


16 (a) Ta = 2π the sound of same frequency.
k
m 11 (a) During the phenomenon of resonance, the amplitude of
Tb = 2π (Q Springs are connected in series) oscillation becomes large. Because applied frequency is equal
(k / 2)
to natural frequency.
m
Tc = 2π (Q Springs are connected in parallel)
2k (A) Taking it together
l CHECK POINT 11.4 1 (c) a and s are always in opposite directions.
YA l l g
1 (c) Here, k = 2 (c) T = 2π ⇒ 2= = constant
L g T 4π 2
1 k 1 YA
∴ f = = 3 (a) The amplitude is the maximum displacement of the
2π m 2π mL particle from the mean position, i.e. A .
m 1
2 (d) T = 2π ⇒T ∝ 4 (b) There is no effect of phase angle on amplitude. Hence,
ρAg ρ A1 10 2
= =
R A2 5 1
3 (c) The time period in this case is given by T = 2π
g 1 1
5 (a) Frequency of oscillation, f = = = 25 Hz
where, R is the radius of earth. T 0.04
Simple Harmonic Motion 537

1 2π π π
6 (d) Frequency, n ∝ 17 (c) ω = = s. Therefore, y = A sin t. Now, put t = 1s
l T 4 4
y 1
n1 l l n2 l (2)2 and then t = 2 s, we get 1 = .
⇒ = 2 ⇒ 2 = 12 ⇒ 2 = 2 y2 2
n2 l1 l1 n 2 l1 (3)
l2 4 l 9 k 16
⇒ = ⇒ 1= 18 (a) ω = = = 4 rad s −1
l1 9 l2 4 m 1
1
 π Now, Kmax = mω 2A2
7 (c) Given, x = 5 cos  2πt +  (where, x is displacement) 2
 4
 π 1
⇒ x = 5 cos  2π +  (At t = 1s) = × 1 × (4)2 (5 × 10 −2 )2 = 2 × 10 −2 J
 4 2
π T
19 (d) At time , v = 0
⇒ x = 5 cos [Q cos (360° + θ ) = cos θ]
4 2

⇒ x =5×
1
=
5
m ∴ Total energy = Potential energy
2 2 l l 2
20 (d) T = 2π and T ′ = 2π or T ′ = T = 2s
8 (b) | a | = ω x
2
g  g 5
g + 
12 = ω 2 (3)  4
2π 1 2 1
∴ ω = 2 rad s −1 = 21 (a) kA = × 2 × 10 6 × (0.01)2 = 100 J
T 2 2
or T = (π ) s or 3.14 s At mean positions,
U = 60 J and K = 100 J
9 (c) In case of spring block system, time period does not change
because restoring force is still kx. At extreme positions,
U = 160 J and K = 0
10 (d) Using acceleration, a = − ω 2x
l
At −xmax , a will be maximum and positive. 22 (b) T = 2π or T 2 ∝ l
g
 π
11 (b) y1 = A cos ωt = A sin ωt +  i.e. T-l graph is a parabola.
 2
y 2 = A sin ωt 23 (c) For second’s pendulum at the surface of earth,
π π l
∴ ∆φ = ωt + − ωt = = 90 ° 2 = 2π e ...(i)
2 2 ge
12 (c) Equation of motion, y = A cos ωt For second’s pendulum at the surface of moon,
A π l
⇒ = A cos ωt ⇒ ωt = = 2π m ...(ii)
2 3 gm
π
×T
2πt π 4 2 From Eqs. (i) and (ii), we get
⇒ = ⇒t= 3 = = s
T 3 2π 3×2 3 le l g 
= m ⇒ lm =  m  le
13 (c) ωA = constant and ω A is made two times.
2 ge gm  ge 
This is possible when frequency (ω) is doubled and amplitude 1 l 1
As, gm = ge ⇒ lm = e = times (Q le = 1m)
(A) is halved. 6 6 6
14 (d) At x = 0, F = 0. For x > 0, force is in negative direction k ×k k
24 (a) ks = =
and for x < 0, force is in positive x-direction. Therefore, k+k 2
motion of the particle is periodic about mean position x = 0. kp = k + k = 2k
1
15 (b) k ∝ , hence k of individual halves will be 16 Nm −1. When 1 ks
= π
l ns 2 M = ks = k /2 = 1
they are connected in parallel, effective value of k will ∴
np 1 kp kp 2k 2
become 32 Nm−1.
2π M
16 (a) Suppose x = A sin ωt and y = A cos ωt = A sin (ωt + π / 2)
25 (d) For motion to be SHM, acceleration of the particle must
Then, by squaring and adding these two equations, we get be proportional to negative of displacement.
x 2 + y 2 = A2 (circle) i.e. a ∝ − y (or x )
Thus, option (a) is correct. ⇒ a = − [y (or x)]
i.e. x (ory) has to be linear.
538 OBJECTIVE Physics Vol. 1

26 (a) Let equation of a SHM is represented by y = A sin ωt d 2y


⇒ = − A ω 2 sin (ωt + φ ) = − Ay ω 2 = (−Aω 2 ) y
dy dt 2
v= = A ω cos ωt
dt d 2y
⇒ ∝ (− y )
⇒ (v )max = Aω = 30 ...(i) dt 2
dy 2 Hence, it is an equation of SHM with amplitude, A = a 2 + b 2 .
Acceleration (a) = 2 = − Aω 2 sin ωt
dt
33 (a) x = 3 sin 2t + 4 cos 2t. From given equation,
a max = ω 2A = 60 ...(ii)
4 A
From Eqs. (i) and (ii), we get
ω (ωA) = 60 ⇒ ω (30 ) = 60
⇒ ω = 2 rad s −1

⇒ = 2 rad s −1 ⇒ T = π s
T
3
27 (d) Here, A = 0.04 m, ω 2A = 0.30 ms −2 π
A1 = 3, A2 = 4 and φ =
∴ ω = 2.74 rads −1 2

Now, T= = 2.29 s ∴ A = A12 + A22 = 32 + 42 = 5
ω
⇒ vmax = Aω = 5 × 2 = 10
I l
28 (d) T = 2π = 2π
MgL g 34 (d) Q y is sinusoidal function.
I (3 / 2 MR 2 ) 3 Acceleration = − ω 2 y, so F = − m ω 2 y .
or l = = = R
ML MR 2 So, F will also be sinusoidal function with phase difference π.
∆T 1 ∆l 0.02
29 (d) T ∝ l ⇒ = = = 0.01 35 (b) Ball will execute SHM inside the tunnel with time period,
T 2 l 2
T = 2π R / g = 84.63 min
⇒ ∆T = 0.01T
Hence, time taken by the ball to reach from one end to the
Loss of time per day = 0.01 × 24 × 60 × 60 = 864 s 84.63
other end of the tunnel, t = = 42.3 min
30 (a) Time period, T ∝ l 2
Also, according to thermal expansion, l′ = l (1 + α∆θ ) 36 (d) Given, ω1A1 = ω 2A2
∆l
= α∆θ A1 ω2 k  k
l ⇒ = = 2 Q ω = 
∆T 1 ∆l 1 A2 ω1 k1  m 
Hence, = = α ∆θ
T 2 l 2 2π
1 37 (c) y = a sin t
= × 12 × 10 −6 × (40 − 20 ) = 12 × 10 −5 T
2 a 2π
⇒ = a sin ⋅t
⇒ ∆T = 12 × 10 −5 × 86400 s/day 2 T
∴ ∆T ≈ 10.3 s/day 2π 1 π
⇒ sin t= = sin
T 2 4
31 (a) Displacement of damped oscillations will be
2π π
x = x me − bt / 2m sin (ω dt + φ ) ⇒ t=
T 4
2
k  b  T
where, ω= −  ∴ t=
m  2m  8
38 (a) mg = kx
32 (d) According to the question, the displacement,
mg
y = a sin ωt + b cos ωt ⇒ k=
x
Let a = A sin φ and b = A cos φ
M+m (M + m ) x
Now, a 2 + b 2 = A2 sin2 φ + A2 cos 2 φ ∴ T = 2π = 2π
k mg
= A2 ⇒ A = a 2 + b 2
39 (c) Velocity, v = ω A2 − x 2
∴ y = A sin φ ⋅ sin ωt + A cos φ ⋅ cos ωt
dy and magnitude of acceleration = ω 2x
= A sin (ωt + φ ) ⇒ = Aω cos (ωt + φ )
dt
Now given, ω 2x = ω A2 − x 2
Simple Harmonic Motion 539

⇒ ω 2 ⋅ 1 = ω 22 − 12 47 (d) T = 2π
m
k
⇒ ω= 3
2π 2π or T ∝ m
∴ T= = Mass is increasing, therefore time period will increase.
ω 3
π 
40 (c) At its lowermost point, spring is stretched by 8 cm or 48 (b) Given, y = 3 cos  − 2ωt
4 
8 × 10 −2 m.
∴ Fnet = kx − mg Velocity of the particle,
= (2 × 10 3 × 8 × 10 −2 ) − (10 × 10 ) = 60 N dy d  π 
v= = 3 cos  − 2ωt 
dt dt  4 
41 (c) Reduced mass of two blocks,
 π 
m1m 2 = 3 (−2 ω ) − sin  − 2ωt 
µ= = 2 kg  4 
m1 + m 2
π 
1 k 1 300 = 6 ω sin  − 2ωt
Now, f = = ≈ 2 Hz 4 
2π µ 2π 2
dv d  π 
1 Acceleration, a = = 6 ω sin  − 2ωt 
42 (b) Force constant, k ∝ dt dt  4 
Length of spring
π 

kA
=
L = 6 ω × (−2ω ) cos  − 2ωt
4 
k αL
k π 
∴ kA = = −12ω 2 cos  − 2ωt
α 4 

kB =
k  π 
Similarly, = − 4 ω 2 3 cos  − 2ωt 
1− α   4 
m 0.2 π ⇒ a = − 4 ω2y
43 (b) T = 2π = 2π = s
k 80 10 As acceleration, a ∝ − y
44 (b) For y 2, Hence, due to negative sign motion is SHM.
10 A Clearly, from the equation
ω′ = 2ω [Q Standard equation y = a cos (ω t + φ)]
2π 2π π
⇒ = 2ω ⇒ T ′ = =
T′ 2ω ω
π
So, motion is SHM with period .
ω
10C
49 (b) Given equation of displacement of the particle,
A = (10 ) + (10C )
2 2
or 10 1 + C = 40
2
y = sin3 ωt = (3 sin ωt − 4 sin 3ωt )/ 4
∴ C = 15 (Q sin 3 θ = 3 sin θ − 4 sin3 θ )
dy  d d 
45 (c) At x = 0, U = 0. Therefore, total mechanical energy is ⇒ = (3 sin ω t) − (4 sin 3 ω t ) / 4
dt dt dt 
equal to the maximum kinetic energy.
dy

1 2
mvmax = 36 ⇒ 4 = 3 ω cos ωt − 4 × (3 ω cos 3 ωt )
2 dt
d 2y
or vmax =
72
=
72
= 6 ms −1 ⇒ 4 × 2 = − 3 ω 2 sin ωt + 12 ω 2 sin 3 ωt
m 2 dt
1 d 2y 3 ω 2 sin ωt + 12 ω 2 sin 3 ωt
46 (c) mω 2A2 = 8 × 10 −3 J ⇒ 2
=−
2 dt 4
1 d 2y
∴ × 0.1 × ω 2 × (0.1)2 = 8 × 10 −3 or ω = 4 rad s −1 ⇒ is not proportional to y.
2 dt 2
Therefore, the equation of motion of the particle, if the initial Hence, motion is not SHM.
phase of the oscillation is 45°, will be As, the expression is involving sine functions, hence it will be
 π periodic.
y = 0.1sin  4t + 
 4
540 OBJECTIVE Physics Vol. 1

50 (c) Consider the diagram in which a liquid column oscillates. 56 (c) Maximum kinetic energy = Energy of oscillation in SHM
In this case, restoring force acts on the liquid due to gravity. 1
Acceleration of the liquid column can be calculated in terms ∴ 8 = kA2
2
of restoring force.
⇒ kA2 = 16 ...(i)
m
Further, 2π =4 (Q T = 4 s)
k
1 4 π2
∴ = 2 or k = ...(ii)
k π 4
Restoring force, f = Weight of liquid column of height 2y From Eqs. (i) and (ii), we get
⇒ f = − (A × 2 y × ρ ) × g = − 2A ρ g y (Q m = ρv ) k ≈ 2.5 Nm −1
⇒ f ∝ − y ⇒ Motion is SHM with force constant,
and A ≈ 2.53 m
k = 2Aρg.
Maximum acceleration of the particle will be
m A × 2h × ρ h
⇒ Time period, T = 2π = 2π = 2π k
k 2Aρg g amax = ω 2A = A
m
l
T = 2π , where l =h  2.5
g =   (2.53) ≈ 6.3 ms −2
 1
which is independent of the density of the liquid.
l
57 (d) In vacuum, T = 2π
51 (c) Given, x = a cos ωt ...(i) g
y = a sin ωt ... (ii)
LetV be the volume and ρ the density of the mass of the bob.
Squaring and adding Eqs. (i) and (ii), we get Net downward force acting on the bob inside the liquid
x2 + y 2 = a2 (Q cos 2 ωt + sin2 ωt = 1) ρ 7
= Weight − Upthrust = Vρg − V ⋅ g = Vρg
This is the equation of a circle. 8 8
ρ
Clearly, the locus is a circle of constant radius a. V g
8
(upthrust)
52 (c) As the given equation is
x = a cos (αt )2
is a cosine function. Hence, it is an oscillatory motion.
Now, putting (t + T ) in place of t, Vρg (weight)
x ( t + T ) = a cos [α (t + T )]2 [Q x (t ) = a cos(αt )2]
7
= a cos [α t 2 + α T 2 + 2α t T ] ≠ x (t ) i.e. Effective value of g is g.
8
where, T is supposed as period of the function ω (t ).
Hence, it is not periodic. So, time period of the bob inside the liquid,
1 2 1 2 l l 8 8
53 (c) mv = kx T1 = 2π = 2π × = T
2 2 (7/ 8)g g 7 7
m 58 (a) In this case, time period of pendulum becomes
∴ x =v
k
1 2
54 (b) U = Umean + kx E
2
qE
1
Given, Umean = E1 and kA2 = E 2 = maximum kinetic energy mg
2
at mean position ++++++++++++++
2
3A 1  3A 3  l
∴ At x = , U = E1 + k   = E1 + E 2 T ′ = 2π
l
⇒ T′ < T Q T = 2π 
2 2  2  4  qE   g
g + 
1 2  m
55 (d) Q kA = 1 J
2
59 (b) Let T1 and T2 be the time period of the two pendulums,
2 2 25
∴ k= 2= = Nm−1 100 121
A (0.4) 2
2 T1 = 2π and T2 = 2π (T1 < T2 because l1 < l2 )
g g
m 2 4π
∴ T = 2π = 2π = s Let at t = 0, they start swinging together. Since, their time
k (25 / 2) 5
periods are different, the swinging will not be same always.
Simple Harmonic Motion 541

Only when number of completed oscillation differs by an m 1 α


integer, the two pendulum will again begin to swing together. 66 (b) T = 2π or T ∝ or k = 2
k k T
Let longer length pendulum complete n oscillation and shorter Now, ke = k1 + k2
length pendulum complete (n + 1) oscillation, for the two be in α α α
same phase, then (n + 1)T1 = nT2. ∴ = +
T 2 T12 T22
100 121
(n + 1) × 2π = n × 2π or T −2 = T1−2 + T2−2
g g
67 (a) As, F = N or m ω 2A = mg ⇒ ω 2A = g
⇒ n = 10
g A
60 (c) For series arrangement, fs =
1 ks
=
1 k ∴ ω= or T = 2π
2π m 2π 2m A g
 kk k 4 × 10 −3
Q ks = 1 2 =  = 2π = 2π × 2 × 10 −2
 k1 + k2 2 10
For parallel and series arrangement, 4π π
⇒ T= = s
1 2k  2k k 2k  100 25
fp = Q kp = = 
2π 3m  2k + k 3 12 × 10  mg 
68 (b) x12 = = 0.12 m Q x = 
fs k 3m 3 1000  k 
= × = ⇒ 3:2
4 × 10
Q
fp 2m 2k 2 x4 = = 0.04 m
1000
F 8
61 (d) From graph, slope k = = =4 ∴ A = x12 − x 4 = 0.08 m
x 2
m 0.01 69 (b) Let x = A sin ωt
∴ T = 2π ⇒ T = 2π = 0.31s
k 4
d 2x
62 (b) Comparing with = − ω 2x, we have t=0 t=t
dt 2 10 cm
ω = 2 rads −1,
∴ vmax = ωA = 2 ms −1 or 10 = 20 sin ωt
 π π
and amax = ω 2A = 4 ms −2 ∴ ωt =   ⇒ t =
 6 6ω
63 (a) g
The desired time will be 2t.
π π T 12
or = = = = 2s
3 ω 3(2π / T ) 6 6
1
70 (a) k∝
l
g 2g l1
Q =n
Hence, to keep time period same, l′ = 2 l l2
64 (b) After attaining terminal velocity, ball will move with  n   n + 1
⇒ l1 =   l or k′ =  k
constant velocity. Therefore, the motion of the ball is periodic  n + 1  n 
but not SHM.
mn
∴ T = 2π
65 (d) According to question, the situation can be represented as k (n + 1)
y
Mg 5 × 10
71 (b) Amplitude of oscillation, A = = = 0.25 m
k 200
1
m m
A 72 (b) T1 = 2π but T2 = 2π
k + ρAg k
60° v1 −v2 x Hence, T1 < T2.
60° A/2
1
A 73 (b) kA2 = (0.04 − 0.01) J = 0.03 J
2
0.06 0.06
2
∴ k= 2 = = 150 Nm −1 = 1.5 × 10 2 Nm−1
A (0.02)2
74 (c) Potential energy graph is parabolic with its minimum value
Hence, here φ = 60 ° + 60 ° = 120 ° at mean position.
542 OBJECTIVE Physics Vol. 1

75 (a) Since, the cube is half immersed. The density of cube On solving, we get
should be half the density of water, i.e. 500 kgm−3 2a 2
A=
3a − b
m
T = 2π (Q k = ρ wAg ) T
ρ wAg 81 (a) means 120° in reference circle. Maximum possible value
3
π a3 × ρ of v is near mean position between P and Q.
= 2π (Q Mass = Density × Volume)
5 ρw × a 2 × g
aρ 1
∴ = B A A C
gρ w 100
60° 60°
a 1
or = or a = (0.2) m 30° 30°
20 100 Q P
Now, m = a 3ρ = 4 kg 3A
A T
76 (a) From e to 2e or to A, t =
2 6
T 2π m
∴ Time period of oscillation = 2t = = Distance 3A 3 3A
3 3 k ∴ v= = =
Time (T / 3) T
77 (c) µg > maximum acceleration in SHM, i.e. ω 2A
82 (b) The block executes SHM, so its angular frequency,
ω 2A
∴ µ>
g k 100 Nm−1
ω= = = 7.07 rads −1
(2πf )2 × A m 2 kg
⇒ µ>
g Its displacement at any time t is
. × 2)2 × 4 × 10 −2
(2 × 314 x (t ) = a cos ωt = 0.1cos(7.07t )
⇒ µ>
10 When the particle is 0.05m away from the mean position.
⇒ µ > 0.631 ≈ 0.64
0.05 = 0.1cos (7.07t)
 π  π
78 (a) y1 = A sin ω1t +  ⇒ y 2 = A sin ω 2t −  or cos (7.07 t) = 0.5
 2  2
π π 3
Now, ω1t + = ω 2t − or sin (7.07 t) = = 0.866
2 2 2
π π Velocity of the block at x = 0.05 m is
or t= =
ω 2 − ω1 (2π / T2 ) − (2π / T1) v = Aω sin ωt
TT 3×7 21 = 0.1 × 7.07 × 0.866 = 0.61 ms −1
= 1 2
= = s
2(T2 − T1) 2 (7 − 3) 8 1 2 1
Hence, KE = mv = × 2 (0.61)2 = 0.37 J
m 1 2 2
79 (c) T = 2π or T ∝
k k I
83 (b) T = 2π . Here, l is distance of point P from centre of
4 9 mgl
T has increased times. Hence, k′ = k
3 16 mass.
When both are combined, When P is the centre, l = 0. Therefore, T = ∞, i.e. rod will not
25 25 oscillate.
knet = k + k′ = k or times
16 16 ml 2
4 When P is end point, I =
Hence, new time period will become times of 3 s or 2.4 s. 3
5 ml 2 l
∴ T = 2π = 2π
80 (c) Particle starts from rest. Hence, x = A cos ωt 3mgl 3g
a = A − x = A − A cos (ω × 1)
84 (b) Here, effective mass of container,
A − a  a
or cos ω = = 1 −  Upthrust
A  A Me = Mass of container +
g
⇒ a + b = A − A cos (ω × 2) ρVg
=M + = M + ρV
= A − A(2 cos 2 ω − 1) g
2
 a Me M + ρV
= 2A − 2A 1 −  Hence, T = 2π = 2π
 A k k
Simple Harmonic Motion 543

m 0.1 A and C are having same length, hence they will be in


85 (d) T = 2π = 2π = 0.198 s resonance, because their time period of oscillation,
k 100
From x = A sin ωt or 5 = 10 sin ωt l
T = 2π and hence, frequency will be same.
π 2π π T g
⇒ ωt = or t= or 2t =
6 T 6 6 So, amplitude of C will be maximum.
T Note In this problem, we have assumed that the support is
The desired time in the question will be + 2t
2 perfectly elastic and there is no damping. Hence, oscillation is
considered as undamped.
T T 2T  2
or + = =   ( 0.198) = 0.132 s
2 6 3  3 90 (a) Let angular velocity of the particle executing circular
motion is ω and when it is at Q, it makes an angle θ as shown
YAk
in the diagram.
k1k2
86 (b) Here, keq = = L k1
k1 + k2 YA + k Y
L P (t=0)
Q
YAk ( + Lk )
mYA k2 =k r T = 30 s
= = 2π θ θ
YA + Lk YAk
m X
O R

87 (b) kx kx kx
2 2
O Clearly, θ = ωt
N M Now, we can write OR
45° 45°
= OQ cos (90 ° − θ )
O′
= OQ sin θ = OQ sin ωt
OO ′ = x (say) = r sin ωt (Q OQ = r )
x
Then O′ M = O′ N ≈ ⇒ x = r sin ωt = B sin ωt (Q r = B)
2 2π  2π 
= B sin t = B sin  t
i.e. Elongation in spring B and C is x / 2, while compression T  30 
in spring A is x. Clearly, this equation represents SHM.
Net restarting force,
 2kx 
F = − kx +
 2
cos 45° = − 2kx

(B) Medical entrance special format

F
a= =−
2k
x
questions
m m l Assertion and reason
T = 2π   = 2π
x m
1 (a) At given time, x is negative. Therefore, acceleration is
a  2k
positive (a ∝ −x ). Further slope is negative. Therefore,
88 (c) According to law of conservation of momentum, velocity is negative.
pi = p f l
Mvmax = (m + M ) v ′max 2 (a) T = 2π
Mvmax ge
⇒ v ′max =
(m + M ) Here, geff = g + a, if the lift accelerates upwards.
Q v ′max = A′ ω′
3 (d) Time period does not depend on amplitude of oscillation.
M ⋅A k k M
⇒ = A′ ⇒ A′ = A 4 (a) v = ω A2 − x 2
(m + M ) M (m + M ) (m + M )
v2 x2
89 (b) According to the question, A is given a transverse ∴ + = A2, i.e. ellipse.
ω 2
1
displacement.
Elastic support 1 1
G G 5 (c) Force constant of a spring, k ∝ and T ∝
l k
∴ T∝ l
e
B l Statement based questions
A
C 1 (d) Acceleration is maximum when displacement is maximum.
D
Force is zero when displacement is zero and potential energy
Through the elastic support, the disturbance is transferred to
is maximum when displacement is maximum.
all the pendulums.
544 OBJECTIVE Physics Vol. 1

mg 0.2 × 10 v = ωA sin ωt
2 (d) At equilibrium, x = = = 0.01 m = 1 cm 8 (c) If
k 200 Then, vmax = ωA
1 k 1 200 1 2 1
∴ f = = ≈ 5 Hz ∴ Kmax = mvmax = mω 2A2
2π m 2π 0.2 2 2
1
3 (d) v = ω A2 − x 2 (In SHM) Further, < K > = < mω 2A2 sin2 ωt >
2
or v =ω A −ω x
2 2 2 2 2
1
= mω 2A2 < sin2 ωt >
Comparing the given equation with the above equation, we 2
get 1
But < sin ωt > = in one oscillation
2

ω2 = 9 2
1
2π ∴ < K > = mω 2A2
∴ ω =3= 4
T
1
2π 9 (b) k ∝
∴ T= units Length of spring
3
Also, ω 2A2 = 144 If length is halved, k will become two times.

∴ A = 4 units, | a | = ω 2x = (9) (3) = 27 units m


From T = 2π
k
Displacement ≤ Distance
dU Time period T will remain half for the given conditions.
4 (c) U = 5x 2 − 20 x ⇒ = 10 x − 20
dx 10 (a) T = 8 s
dU
∴ F =− = (−10 x + 20 ) 2π π
dx ∴ω = = rads −1
T 4
Let us suppose x = (X + 2). Then F = − 10X, F = 0 at X = 0 or
x = 2, i.e. x = 2m is the mean position about which particle is k = − slope of F-x graph = 10 Nm −1
in SHM. Here, k = 10
m
m 0.1 π From T = 2π , we have
∴ T = 2π = 2π = s. At mean position, kinetic k
k 10 5
energy is maximum and force acting on particle in SHM is m
8 = 2π
variable. 10
k 160
5 (d) ω = ∴ m= kg
µ π2
m1m 2 2 1
Here, µ = reduced mass = = kg Kmax = mω 2A2
m1 + m 2 3 2

∴ ω=
6
= 3 rads −1 1 160 π 2
= × × × (4 × 10 −2 )2
2/ 3 2 π2 16
Amplitude 12 cm distributes in inverse ratio of mass. = 8.0 × 10 −3 J
∴ A1 = 8 cm and A2 = 4 cm
Now, maximum kinetic energy,
l Match the columns
1 1 1 (b) Suppose x = A sin ωt
K1 = m1ωA12 = × 1 × 3 × (8 × 10 −2 )2 = 9.6 mJ
2 2 dx
1 Then, v= = ωA cos ωt = ωA sin (ωt + π / 2)
K2 = × 2 × 3 × (4 × 10 −2 )2 = 4.8 mJ dt
2 dv
and a= = − ω 2A sin ωt = ω 2A sin (ωt + π )
6 (d) The motion is simple harmonic only inside earth. Further dt
Hence, A → q, B → r, C → q.
1 2 GMm 3 GMm GM (R + 3h )
mv = − + ⇒ v= 2 (c) a = − ω 2x, i.e. a-x graph is straight line passing through
2 (R + h ) 2 R R (R + h )
origin.
7 (b) In simple harmonic motion, speed is not constant. Thus, in If v = v 0 sin ωt, then
order to calculate the time taken by the particle executing dv
SHM, we cannot use the following relation, a= = v 0ω cos ωt
dt
Distance
i.e. Time = = v 0ω 1 − sin2 ωt
Speed
Simple Harmonic Motion 545

v2 So, from Eqs. (i) and (ii), we can say that A0 be the value of
= v 0ω 1 − ⇒ a = ω v 02 − v 2 mean position, at which y = 0.
v 02
∴Amplitude, R = A2 + B 2 + 2 AB cos θ
So, a-v graph is neither a straight line nor a parabola. Further,
acceleration and velocity-time graphs are sine or cosine As two functions sine and cosine have phase difference
functions. of 90°.
Hence, A → q, B → s, C → s, D → s. ∴ R = A2 + B 2 (Q cos 90 ° = 0 )
3 (a) On a satellite and at centre of earth, g′ = 0 5 (c) The average velocity of a particle executing simple
∴ T =∞ harmonic motion (SHM) is
At pole, value of g is more than the normal value. Total displacement x f − xi
Thus, T < 2s v av = =
Time interval T
At mountain or at some height, value of g is less.
Thus, T > 2 s where, x f and xi are the initial and final position of the
particle executing SHM.
Hence, A → q, B → t, C → p, D → t.
As in SHM, the particle executes motion about its mean
position. So, after one complete oscillation by the particle, it
(C) Medical entrances’ gallery will reaches its initial position, i.e.
1 (c) sin ωt and cos ωt both are periodic function of period

. Displacement, x f − xi = 0
ω 0
∴ v av =
We know that, sum of two periodic functions is also a T
periodic function, hence sin ωt + cos ωt represents periodic Hence, the average velocity is zero.
motion.
6 (b) The acceleration of particle/body executing SHM at any
2 (d) In SHM, equation of displacement of a particle is instant (at position x) is given as
y = a sin ωt a = − ω 2x
and equation of acceleration of a particle is where, ω is the angular frequency of the body.
A = − aω 2 sin ωt = aω 2 sin (ωt + π ) ⇒ | a | = ω 2x …(i)
∴Phase difference between displacement and acceleration of Here, x = 5 m, | a | = 20 ms −2
a particle is
Substituting the given values in Eq. (i), we get
= (ωt + π ) − ωt = π rad
20 = ω 2 × 5
3 (d) In a simple harmonic motion (SHM), the particle oscillates
20
about its mean position on a straight line. ⇒ ω2 = = 4 or ω = 2 rad s −1
5
As shown in the figure below, the particle moves from its
mean position (O) to an extreme position (P ) and then return As we know that, time period, T = 2π / ω …(ii)
to its mean position covering same distance of A. ∴Substituting the value of ω in Eq. (ii), we get
Then, by the conservative force, it is moved in opposite 2π
T= =πs
direction to a point Q by distance A and then back to mean 2
position covering a distance of A. This comprises of one time
7 (a) For a rectangular block, floating in a liquid as shown in the
period as shown below.
figure below, then
At equilibrium, mg = Vρg = Alρg
Q O ⇒ m = Aρl
P

Area (A)
m
l
Extreme Mean Extreme
position position position
In one time period

Hence, in one time period, it covers a distance of


x = OP + PO + OQ + QO where, l = length of part immersed in liquid.
= A + A + A + A = 4A When it is given in downward displacement y, restoring force
(upward direction) on block is
4 (a) The displacement of given particle is F = − [A(l + y ) ρg − mg]
y = A 0 + A sin ωt + B cos ωt … (i) = − [A(l + y ) ρg − Alρg] = − Aρgy
The general equation of SHM can be given as i.e. F ∝ − y or a ∝ − y, so it executes SHM.
x = a sin ωt + b cos ωt … (ii)
546 OBJECTIVE Physics Vol. 1

Inertia factor Differentiating the above expression w.r.t. t, we get


∴ Time period = 2π
Spring factor dx
= aω sin 2ωt
dt
m m
T = 2π ⇒T2 ∝ Again, differentiating x w.r.t. t, we get
Aρg Aρ
d 2x
1 = 2aω 2 cos 2ωt
or T2 ∝ dt 2
ρ
d 2x
or = 2aω 2(1− 2 sin2 ωt)
8 (b) Given, m = 0.5 kg, ω = 5 rad/s, dt 2
x = 1 cm = 10 −2m, d 2x
⇒ = 2aω 2 − 4aω 2 sin2 ωt = 2aω 2 − 4ω 2x ...(ii)
A = 5 cm = 5 × 10 −2m dt 2
1 Therefore, after comparing Eqs. (i) and (ii), we can say that the
∴Potential energy, PE = mω 2x 2 motion of the particle is non-simple harmonic motion.
2
1 12 (d) Time period,
or PE = × 0.5 × (5 )2 × (10 −2 )2
2 m
T = 2π
25 k
= × 10 −4 J = 6.25 × 10 −4 J
4 m
1 Case I T1 = 2π ...(i)
∴Kinetic energy, KE = m ω 2 (A2 − x 2 ) k
2 Case II When the mass m is increased by 1 kg, then net mass
1 =m +1
or KE = × 0.5 × (5 )2 (25 − 1) × 10 −4
2 m +1
= 150 × 10 −4 J ⇒ T2 = 2π ...(ii)
k
9 (a) Given, On dividing Eq. (ii) by Eq. (i), we get
π T2 m +1 5 m +1
Y = 5 sin (100t − 2π ) = 5 sin (50 π t − πx ) …(i) = ⇒ =
2 T1 m 3 m
General equation of SHM is 25 m + 1 25 1
⇒ = ⇒ = 1+
Y = A sin (ω t − kx ) …(ii) 9 m 9 m
After comparing with Eqs. (i) and (ii), 1 16
⇒ =
ω = 50 π m 9
2π 2π 9
∴Time period, T = = = 0.04 s ⇒ m = kg
ω 50 π 16
10 (c) For a particle executing SHM, magnitude of velocity of 13 (c) Given, kP > kQ
particle when it is at displacement x from mean position
In case (a), the elongation is same,
= ω A2 − x 2 i.e. x1 = x 2 = x
Also, magnitude of acceleration of particle in SHM = ω 2x 1 1
So, WP = kP x 2 and WQ = kQx 2
Given, when x = 2 cm 2 2
Magnitude of velocity = Magnitude of acceleration WP kP
∴ = >1 (Q kP > kQ)
WQ kQ
⇒ ω A − x =ω x
2 2 2

⇒ WP > WQ
A2 − x 2 9−4
⇒ ω= = In case (b), the spring force is same,
x 2 i.e. F1 = F2 = F
5 F F
⇒ Angular velocity, ω = So, x1 = , x 2 =
2 kP kQ
∴ Time period of motion,
1 1 F2 1F2
2π 4π ∴ WP = kP x12 = kP 2 =
T= = s 2 2 kP 2 kP
ω 5
1 1 F2 1F2
d 2x and WQ = kQ x 22 = kQ ⋅ 2 =
11 (c) For a SHM, acceleration, a = 2 = − ω 2x ...(i) 2 2 kQ 2 kQ
dt
According to the question, WP kQ
∴ = < 1 ⇒ WP < WQ
 1 − cos 2ωt  WQ kP
x = a sin2 ωt = a  
 2 
Simple Harmonic Motion 547

14 (b) Let A be the amplitude of oscillation, then  2π   T 


a = − ω 2A cos     = − ω 2A cos π = ω 2A
v12 = ω 2 (A2 − x12 ) … (i)  T   2

v 22 = ω 2 (A2 − x 22 ) … (ii) At t = T,
 2π 
On subtracting Eq. (ii) from Eq. (i), we get a = − ω 2A cos   (T )
T
v12 − v 22 = ω 2 (x 22 − x12 )
= − ω 2A cos 2π = − ω 2A
v12 − v 22 2π v12 − v 22 x 22 − x12 ∴Correct graph is depicted in option (c).
⇒ ω= ⇒ = ⇒ T = 2π
x 22 − x12 T x 22 − x12 v12 − v 22
21 (b) Let the amplitude of SHM be A.
dv 1
15 (b) Given, v = − ω 2x Now, potential energy of SHM, U = kx 2
dx 2
v
On integrating within the limit, ∫ vdv = ∫ − ω 2x dx
x
A 1 A2
0
Here, x = ⇒U = k …(i)
v0 2 2 4
v 2 
v x 1 2 1 A2 3
2 x 
2
Kinetic energy, K = kA − k ⇒ K = kA2 ...(ii)
⇒ 2 = − ω 2 2 2 4 8
 v0  0
On dividing Eq. (ii) by Eq. (i), we get
⇒ v 2 − v 02 = − ω 2x 2 K 3 8 K 3
= × ⇒ =
⇒ v = v 02 − ω 2x 2 U 8 1 U 1
 π
16 (c) Time period, T = 2π
m 22 (d) Given, displacement, x = 5 cos  2πt + 
 4
k
 π
200 × 10 −3 ⇒ x = 5 cos  2π +  (At t = 1s)
⇒ T = 2 × 3.14  4
80 π
⇒ x = 5 cos [Q cos (2π + θ ) = cos θ]
1 2 × 3.14 4
or T = 2 × 3.14 or T =
400 20 1
⇒ x =5×
or T = 0.314 s 2
5
17 (b) Total mechanical energy of the system, ∴ x= m
2
2
1 1  2π 
E = mω 2A2 = × 40 × 10 −3   × (2)2 23 (d) At mean position, velocity is maximum,
2 2 T
v 16
2 vmax = ωA ⇒ ω = max = = 4 rad/s
1  2 × 3.14
= × 40 × 10 −3  −3
 × (2) = 7.9 × 10 J
2 A 4
2  20 
∴ v = ω A2 − y 2 ⇒ 8 3 = 4 42 − y 2
m
18 (b) Given, time period, T = 2π = 10 s ⇒ 192 = 16(16 − y 2 ) ⇒ 12 = 16 − y 2 ⇒ y = 2 cm
k
 π
When the spring system is immersed in liquid with specific 24 (b) Given, x1 = A sinωt +  and x 2 = A cos ωt
 6
1
gravity th of the copper, but as the liquid is non-viscous in  π
4 ⇒ x 2 = A sinωt + 
 2
nature, so the time period would not be changed because
π π
there will be no change in m and k. Phase difference, ∆φ = φ 2 − φ1 ⇒ ∆φ = −
2 6
d2y
19 (b) For a simple harmonic motion, a = ∝ (− y ) 3π − π π
dt 2 ⇒ ∆φ = =
6 3
So, only equations, y = sin ωt − cos ωt
1
 3π  25 (b) We know that, kinetic energy, K = mv 2
and y = 5 cos  − 3ωt 2
 4 
dy
are satisfying this condition. Thus, they represent SHM. where, v = = ωA cos ωt
dt
20 (c) a = − ω 2x = − ω 2A cos ωt 1
So, K = mω 2A2 cos 2 ωt
So, at t = 0, 2
a = − ω 2A Hence, kinetic energy varies periodically with double the
frequency of SHM. So, when a particle executing SHM
T
At t= , oscillates with a frequency ν, then the kinetic energy of
2 particle changes periodically with a frequency of 2 ν.
548 OBJECTIVE Physics Vol. 1

3 l
26 (d) The current displacement is times the amplitude, 30 (d) Time period, T = 2π
2 g

i.e. y=
3
A Given, g = 9.8 ms −2, l = 9.8 m
2
9.8
We know that, potential energy of a body in SHM is given by ⇒ T = 2π = 2π = 2 × 3.14 = 6.28s
9.8
1
U = mω 2 y 2
2 31 (d) We know that, equation of SHM, y = a sinωt
where, m and ω are constants. Here in given condition,
a
= a sinω × 1
 3 
2 2 2
1 1 3A
U= mω 2  A = mω 2 × …(i) 1 π
2  2  2 4 = sinω ⇒ ω =
2 6
Similarly, kinetic energy of a body in SHM is given by 2π π
⇒ ω= =
1 T 6
K = mω 2 (A2 − y 2 )
2 Hence, time period of SHM, T = 6 × 2 ⇒ T = 12 s
1  3A2  1 A2 32 (b) For SHM a = − ω 2 y, where a is acceleration, ω is angular
= mω 2 A2 −  = mω 2 …(ii) velocity and y is displacement. Hence, it is clear that
2  4  2 4
acceleration is directly proportional to displacement and is
So, the ratio of PE and KE, always opposite to displacement.
1 3A2 1 A2 While, velocity and displacement act in same direction.
PE : KE = mω 2 × : mω 2 = 3:1
2 4 2 4 33 (a) We know that, the time period of a simple pendulum,
27 (a) We know that in spring SHM, the restoring force is l
T = 2π
proportional to displacement, g
i.e. F = − mω 2 y …(i) 1
⇒ T∝
F = − ky …(ii) g
where, k = force constant of the spring. Here, in given condition,
Given, m = 10 kg, A = 10 cm = 0.1 m, k = 1000 Nm−1 K
T1 = …(i)
Comparing both equations, we get g
K
ω2 =
k
⇒ ω=
k
=
1000
⇒ ω = 10 rads −1 and T2 = …(ii)
m m 10  d
g 1 − 
 R
and acceleration in SHM,
amax = − ω 2⋅ y = − 10 2 × (0.1) = − 10 ms −2 On dividing Eq. (i) by Eq. (ii), we get
⇒ |amax | = 10 ms –2 T1 K/ g
=
T2  d
28 (a) When restoring force will become equal to the frictional K g 1 − 
 R
force block will start to slip.
∴ Restoring force = Friction force T1  d T12 d
or = 1 −  or = 1−
⇒ kA = µmg (QFrestoring = −kx ) …(i) T2  R T22 R
1 k  T 2
Now, frequency, f = ⇒ d = 1 − 12  R
2π m  T2 
From Eq. (i), we get
1 µg 35 (b) As the boy stands up, the centre of gravity of the
f = pendulum is raised up, thereby decreasing the effective
2π A length of the pendulum of the swing. Hence, the time period T
 t decreases.
29 (c) y = 4 cos 2   sin 1000t
 2 1 k
36 (d) Frequency of oscillator, n =
 1 + cos t  2π m
=4  sin 1000t
 2  n2 m1
We get, =
= 2 sin 1000t + 2 cos t sin 1000t n1 m2
1
= 2 sin 1000t + (sin 2t + sin 2000t ) m1 1
2 ∴ 2= or m 2 = m1
m2 4
∴It is a superposition of three waves, i.e. n = 3.
Simple Harmonic Motion 549

37 (c) Mass of the particle = m A


= sin(ωt + 45° )
2
Spring constant = k
∴ The ratio of amplitudes of two motions,
m
The time period of oscillator, T = 2 π A1 A
k = = 2
1 A2 A/ 2
As k ∝ (where, l is the length of spring)
l 42 (b) When the bob is placed in an electric field, the time period
∴ k′ = 2 k of simple pendulum will remain same as the bob is not
m 1 charged. On contrary, if simple pendulum having charged bob
∴ T ′ = 2π = T is placed in a horizontal electric field, then the period will be
2k 2
decreased because there will be an increase in restoring force.
38 (a) Given equation, Hence, Assertion and Reason both are correct but Reason is
y = 5 (sin 3πt + 3 cos 3πt ) not the correct explanation of Assertion.

1 3  43 (d) The equation of displacement,


or y = 5 × 2  sin 3πt + cos 3πt   π
2 2  x1 = 4 sin 10 t + 
 6
 π π
or y = 10 sin 3πt cos + cos 3πt sin The energy of this equation,
 3 3 
1 1
 π E1 = mω12A12 = m × 10 × 10 × 4 × 4
or y = 10 sin  3πt +  2 2
 3 The second equation of displacement,
l 1 1 x 2 = 5 cos (ωt )
39 (b) T = 2π , log T = log 2π + log l − log g
g 2 2 The energy of this equation,
dT dl 1 dg 1 1
Differentiating, =0+ − E 2 = mω 22A22 = mω 22 × 5 × 5
T 2l 2 g 2 2
According to question,
Q l = constant
E 2 = E1
∴ dl = 0
dT −1 dg 1 1
100 × = × 100 mω 22 × 5 × 5 = m × 10 × 10 × 4 × 4
T 2 g 2 2
−1 (−2) × 100 10 × 10 × 4 × 4
= × = +1% increase ⇒ ω2 =
2

2 100 5×5

40 (a) Time period of a simple pendulum, ⇒ ω 22 = 2 × 2 × 4 × 4


l ⇒ ω 2 = 2 × 2 × 4 × 4 = 2 × 4 = 8 unit
T = 2π
g
44 (c) The equation of motion is given as
For freely falling system, effective g = 0 2πt  2π 
T =∞ y = 5 sin Q ω = 
So, 6  T
∴ Pendulum does not oscillate at all.
Here, y = 2.5 cm
A A
41 (b) Given, y1 = A sinωt and y 2 = sin ωt + cos ωt 2πt
2 2 ∴ 2 .5 = 5 sin
A 6
or y 2 = (sin ωt + cos ωt ) π 2πt 1
2 ⇒ = ⇒ t= s
A 6 6 2
= 2 [sin(ωt + 45° )]
2 2πt 2π 1 π
∴ Phase, φ = = × =
6 6 2 6
CHAPTER
12

Elasticity
A solid has a definite shape and size. In order to change the shape or size of a
body, a force is required, such a force is called deforming force. Whenever a
load is attached to a thin hanging wire, it elongates and the load moves
downwards. When load is removed, the wire attains its original size and shape.
The property of a body by virtue of which it tends to regain its original size and
shape, when applied force is removed, is known as elasticity and the deformation
caused is known as elastic deformation. In this chapter, we will study about the
elastic behaviour and mechanical properties of solids.
Some terms related to elasticity are discussed below
(i) Perfectly elastic body A body is said to be perfectly elastic, if it returns
back completely to its original size on removing the external force(s).
(ii) Plastic body If a body remains in the deformed shape and does not even
partially regain its original shape, after removal of external force, is called
plastic body or perfectly inelastic body.
(iii) Elastic limit It is the upper limit of deforming force up to which if
deforming force is removed, the body regains its original form completely
and beyond which if deforming force is increased, the body looses its
property of elasticity and gets permanently deformed.
Note
(i) Elastic limit is the property of body, whereas elasticity is the property of material of a body.
(ii) All rigid bodies are elastic to some extent which means we can change their dimensions slightly
by pulling or pushing them.
(iii) No body is perfectly elastic or perfectly plastic. All the bodies found in nature lie between these Inside
two limits. When the elastic behaviour of body decreases, its plastic behaviour increases or
vice-versa. 1 Stress and strain
2 Hooke’s law
STRESS AND STRAIN 3 Poisson’s ratio
Stress-strain curve
When an external force is applied to a body, then at each cross-section of the Work done or potential energy
body, an internal restoring force is developed which tends to restore the body stored in a stretched wire
back to its original position. The internal restoring force acting per unit area Thermal stresses and strains
of cross-section of the deformed body is called stress.

Restoring force
Thus, Stress =
Area
If there is no permanent change in configuration of the body, the restoring force is
equal and opposite to the external deforming force applied.
Elasticity 551

So, stress can be given as 2. Volumetric stress


External deforming force If equal normal forces are applied on an object all over its
Stress =
Area surface, then its volume changes. The restoring force
In SI system, the unit of stress is Nm −2 or pascal (Pa) and acting per unit area inside the object opposing change in
volume is called volumetric stress.
in CGS system, it is dyne cm −2 .
The dimensions of stress are [ML−1T −2 ]. F

Note The minimum value of stress required to break a wire is called ∆V F (Deforming force)
breaking stress. It depends on nature of material, temperature
F F
and impurities.
Due to external force, shape of the body changes and it is (Volumetric stress)
said to have strain. V
F
Strain is defined as the ratio of change in shape of an F
object to the original shape. Fig. 12.3 An object under effect of normal deforming forces

Change in configuration of the object


Strain = Volumetric strain
Original configuration of the object
It is defined as the change in volume per unit volume of
Strain is a pure number, it has no unit. There are three the object on application of deforming force.
types of stress resulting in three types of strain
Change in volume (∆V )
Volumetric strain =
1. Longitudinal stress Original volume (V )
When the stress is normal to the surface of an object, then
it is known as longitudinal stress. 3. Shearing stress
It is of two types When a force is applied on an object along the tangential
direction of the surface of the object, then stress produced in
(i) Tensile stress If the stress produced in an object is the object is called shearing stress. Such force tends to change
due to increase in its length, then it is called tensile the shape of object. For this, the opposite surface is kept fixed.
stress.
x
l (Initial length) (Change in
length) F
∆l φ
Tensile stress F
(Deforming force) φ φ
L
Fig. 12.1 An object under tensile stress Fixed face

(ii) Compressive stress If the stress produced in an Fig. 12.4 An object under tangential deforming forces
object is due to decrease in its length, then it is
called compressive stress.
Tangential or shearing strain
∆l (Change It is defined as the ratio of displacement (x ) of the upper
in length) F (Deforming
force) surface to the distance (L ) between two faces on the
(Initial l
length) (Compressive stress) application of deforming force.
Displacement (x )
Shearing strain =
Distance (L )
Fig. 12.2 An object under compressive stress
Example 12.1 A rod has a radius of 100 mm and a length of
Longitudinal strain 10 cm. A 100 N force compresses along its length. Calculate
It is defined as the change in length per unit length of the the longitudinal stress developed in the rod.
object on application of deforming force. Sol. Given, radius of the rod, r = 100 mm = 100 × 10−3 m

Change in length (∆l ) Length of the rod, l = 10 cm = 10 × 10−2m


Longitudinal strain = Force, F n = 100 N
Original length (l )
552 OBJECTIVE Physics Vol. 1

Longitudinal stress developed in the rod Example 12.5 Two blocks of masses 2 kg and 3 kg are
F 100 connected by a metal wire going over a smoother pulley as
= n = ≈ 3185 Nm−2 shown in figure. The breaking stress of the metal is
A π (100 × 10−3 )2
2 × 10 9 Nm −2 . What would be the minimum radius of the
Example 12.2 Find the greatest length of steel wire that can wire used, if it is not to break? (Take, g = 10 ms −2 )
remain hanged vertically without breaking. Breaking stress
of steel = 8 × 10 8 Nm −2 . Density of steel = 8 × 10 3 kgm −3 .
(Take, g = 10 ms −2 )
Sol. Let l be the length of the wire that can hang vertically
without breaking. The stretching force on it is equal to its
own weight.
If A is the area of cross-section and ρ is the density, then
weight
maximum stress or breaking stress (S m ) = 2 kg
A
 Force
Q Stress =  3 kg
 Area 
Weight = mg = Vρg (Q Mass = Volume × Density ) Sol. Equation of motion for 3 kg mass,
= Alρg 30 − T = 3a ...(i)
(Q Volume = Area of cross-section × Length) Equation of motion for 2 kg mass,
(Alρ)g T − 20 = 2a ...(ii)
or Sm =
A
Sm 8 × 108
∴ Length of the steel wire, l = = = 104 m
ρg (8 × 103 )(10)
Example 12.3 Consider a rod of steel having radius of 8 mm
T
and the length of 2 m. If a force of 150 kN stretches it along
a T
its length, then calculate the stress, percentage strain in the
rod, if the elongation in length is 7.46 mm. 2 kg
−3 3 kg
Sol. Given, radius of rod, r = 8 mm = 8 × 10 m, length, L = 2 m 20 a
Applied force, F = 150 kN = 15 × 104 N
30
Cross-section area of wire,
On dividing Eq. (i) by Eq. (ii), we get
A = πr 2 = π × (8 × 10−3 )2 = 201 × 10−6 m2
30 − T
∆ L = 7.46 mm = 7.46 × 10−3 m, percentage strain = ? = 3 /2
T − 20
F 15 × 104 ⇒ 2(30 − T ) = 3(T − 20)
Stress in rod = =
A 201 × 10−6 ⇒ 60 − 2T = 3T − 60
= 7.46 × 108 Nm −2 5T = 120 ⇒ T = 24 N
∆ L 7.46 × 10−3 T  Force
Longitudinal strain = = = 3.73 × 10−3 Breaking stress, S = Q Stress = 
L 2 A  Area 
Percentage strain = 3.73 × 10−3 × 100 = 0.37 % 24
⇒ 2 × 109 =
A
Example 12.4 If the angle of shear is 30° for a cubical body
and the change in length is 250 cm, then what must be the Area, A = 12 × 10−9m2 ⇒ πr 2 = 12 × 10−9 (Q A = πr 2)
volume of this cubical body? ∴ Minimum radius of the wire,
Sol. Given, angle of shear, φ = 30° 12 × 10−9 
1/ 2

r=  . × 10−5m
= 618
and change in length, ∆L = 250 cm = 2.5 m  314. 
∆L 2.5
∴ Shear strain, tan φ = ⇒ tan 30° =
L L Example 12.6 The two wires shown in figure are made of
2.5 2.5 the same material which has a breaking stress of
⇒ L= = = 4.332
tan 30° 0.577 8 × 10 8 Nm −2 . The area of cross-section of the upper wire
is 0.006 cm 2 and that of the lower wire is 0.003cm 2 . The
Volume of cubical body, V = L3 = 81.295 m3
mass m1 = 10 kg, m 2 = 20 kg and the hanger is light.
Elasticity 553

Find the maximum load that can be put on the hanger without Q T2 ≤ 480
breaking a wire.Which wire will break first, if the load is ∴ 300 + mg ≤ 480 or mg ≤ 180 or m ≤ 18 kg
increased?
Again, T1 ≤ 240 or 100 + mg ≤ 240
or m ≤ 14 kg
m2 Maximum mass, m max = 14 kg. When load is increased, T1
increases, hence lower wire will break first.

Example 12.7 Consider a solid cube which is subjected to a


m1 pressure of 6 × 10 5 Nm −2 . Due to this pressure, the each side
of the cube is shortened by 2%. Find out the volumetric
strain of the cube.
Sol. Maximum tension that the upper wire tolerate Sol. Let L be the initial length of the each side of the cube.
= Breaking stress × Area Volume,V = L × L × L = L3 = Initial volume (Vi say)
T2 = 8 × 108 × 0.006 × 10−4 = 480 N If each side of the cube is shortened by 2%, then final length
of the cube = L − 2% of L
 2L   2 
T2 = L −  = L 1 − 
 100  100
20 kg
3 3
 2   2 
∴ Final volume, Vf = L3 1 −  = V 1 − 
T1  100  100
200 N
T1 Change in volume, ∆V = Vf − Vi
 2 
3  2 
3 
10 kg = V 1 −  −V =V 1 −  − 1
 100   100  
3
100 N ∆V  2   2 × 3
mg ⇒ = 1 −  − 1 −~ 1 − 100  − 1
V  100
Maximum tension that the lower wire has
[Q (1 − x )n −~ 1 − nx for x << 1]
T1 = 8 × 108 × 0.003 × 10−4 = 240 N
∆V
∴ Volumetric strain, = 1 − 0.06 − 1 = 0.06
We have, T2 = T1 + 200 and T1 = 100 + mg V
∴ T2 = 300 + mg (Take positive sign)

CHECK POINT 12.1


1. Breaking stress depends on 4. A metallic cube whose each side is 10 cm is subjected to a
(a) length of wire shearing force of 100 kgf. Calculate the shearing stress produced.
(b) area of cross-section of wire (a) 9.8 × 104 Nm−2 (c) 10 Nm−2
−2
(c) Both (a) and (b) (c) 9.8 Nm (d) 9.8 × 102 Nm−2
(d) independent of length and area of cross-section
5. A wire of length 2.5 m has a percentage strain of 0.012% under a
2. The lower surface of a cube is fixed. On its upper tensile force. The extension produced in the wire will be
surface, force is applied at an angle of 30° from its (a) 0.03 mm (b) 0.3 mm (c) 0.3 m (d) 0.03 m
surface. The change will be in its
(a) shape (b) size 6. A cube is subjected to a uniform volume compression. If the side
(c) volume (d) Both shape and size of the cube decreases by 2%, the bulk strain is
(a) 0.02 (b) 0.03
3. When a spiral spring is stretched by force, strain (c) 0.04 (d) 0.06
produced is
(a) longitudinal strain 7. A metallic cube of side 10 cm is subjected to a shearing force of
(b) volumetric strain 300 kgf. The top face is displaced through 0.25 cm with respect to
the bottom? Calculate the shearing strain produced.
(c) shear strain
(a) 0.25 (b) 2.5
(d) Both (a) and (c)
(c) 0.025 (d) 0.08
554 OBJECTIVE Physics Vol. 1

HOOKE’S LAW Substituting in above equation, we have


According to Hooke’s law, Mgl
Y =
“For small deformation, the stress in a body is proportional (πr 2 )∆l
to the corresponding strain.”
Young’s modulus is only defined for solids, and not for
i.e. Stress ∝ Strain or Stress = (E ) (Strain)
liquids and gases. Larger the value ofY for a material,
stress
Here, E = is a constant called the modulus of more elastic it would be. For this reason, steel is more
strain elastic than rubber.
elasticity. The material which has smaller value of Y is more ductile,
The SI unit of modulus of elasticity is Nm −2 and its i.e. it offers less resistance in framing it into wire. Thus, for
dimensions are [ML−1 T −2 ]. making wire, we choose a material having less value of Y.
The modulus of elasticity depends on the type of material Note For a perfectly rigid body, value of Y is infinite.
and temperature. It does not depend upon the values of Force constant of wire
stress and strain.
Force required to produce unit elongation in a wire is
Depending upon the nature of force applied on the body, called force constant of material of wire. It is denoted by k.
the modulus of elasticity is classified in the following three
F YA  F YA 
types k= = Q = 
∆l l  ∆l l 
1. Young’s modulus of elasticity (Y ) If a spring is stretched or compressed by an amount ∆l, the
When a wire is acted upon by two equal and opposite restoring force produced in it, is
forces in the direction of its length, the length of the body Fs = k ∆l …(i)
 ∆l  Here, k = force constant of spring.
is changed. The change in length per unit length   is
 l  Similarly, if a wire is stretched by an amount ∆l, the
called the longitudinal strain and the restoring force restoring force produced in it, is
(which is equal to the applied force in equilibrium) per YA 
unit area of cross-section of the wire is called the F =   ∆l …(ii)
 l 
longitudinal stress.
Comparing Eqs. (i) and (ii), we can see that force constant
For a small change in the length of the wire, the ratio of
of a wire,
the longitudinal stress to the corresponding strain is called
YA
the Young’s modulus of elasticity (Y ) of the wire. k= …(iii)
l
F
Fl YA
Thus, Y = A or Y = i.e. A wire is just like a spring of force constant .
∆l A∆l l
So, all formulae which we use in case of a spring can
l be applied to a wire also.
Example 12.8 If a wire of length 4 m and cross-sectional
l area of 2 m 2 is stretched by a force of 3 kN, then determine
the change in length due to this force. (Take, Young’s
modulus of material of wire is 110 × 10 9 Nm −2 )
M
Sol. Given, area of cross-section, A = 2m2
∆l Applied force, F = 3 kN = 3 × 103 N, length, L = 4 m
M Young’s modulus,Y = 110 × 109 Nm −2
Fig. 12.5 FL
⇒ Y =
A∆L
Let there be a wire of length l and radius r. Its one end is ∴ Change in length,
clamped to a rigid support and a mass M is attached at the FL 3 × 103 × 4
other end. Then, ∆L = = = 0.0545 × 10− 6 m
AY 2 × 110 × 109
F = Mg and A = πr 2
⇒ ∆L = 54.5 × 10− 6 mm
Elasticity 555

Example 12.9 A steel wire of length 4 m and diameter elasticity of the wire is Y and area of cross-section of the
5 mm is stretched by 5 kg-wt. Find the increase in its length, wire is A.
if the Young’s modulus of steel is 2.4 × 1012 dyne cm −2 . P
Sol. Hence, length of the wire, l = 4 m = 400 cm
Diameter of wire, 2r = 5 mm
or radius of wire, r = 2.5 mm = 0.25 cm
Force, F = 5 kg-wt = 5000 g-wt Q
= 5000 × 980 dyne
Young’s modulus,Y = 2.4 × 1012 dyne cm −2 Sol. Consider a small section dx of the bar at a distance x from Q.
F l The weight of the bar for a length x is,
As we have, Y = ×
πr 2 ∆l P

Fl
Increase in length, ∆l =
πr 2Y
dx
(5000 × 980) × 400
= = 0.0041 cm x
(22 / 7) × (0.25)2 × 2.4 × 1012
Q
Example 12.10 Determine the elongation of the steel bar 1m
2
long and 1.5 cm cross-sectional area when subjected to a  mg 
pull of 1.5 × 10 4 N. (Take,Y = 2 × 1011Nm −2 ) w= x
 l 
F /A
Sol. Young’s modulus,Y =  w   mg 
∆l / l Elongation in section dx will be, dl =   dx =   x dx
 AY   lAY 
Fl
∴ Elongation of the steel wire, ∆l = Total elongation in the bar can be obtained by integrating this
AY
expression for x = 0 to x = l .
(1.5 × 10 )(1) 4
∆l = x =l  mg  l
∆l = ∫
 lAY  ∫ 0
(1.5 × 10− 4 ) (2 × 1011) ∴ dl =   x dx
x= 0
= 0.5 × 10−3 m mgl
or ∆l = 0.5 mm or Increase in length, ∆l =
2AY
Example 12.11 A cable is replaced by another one of same Example 12.13 A rod AD consisting of three segments AB,
length and material but twice the diameter. How will this BC and CD joined together is hanging vertically from a fixed
effect the elongation under a given load? How does this support at A. The lengths of the segments are respectively
effect the maximum load, it can support without exceeding
0.2m, 0.3 m and 0.15 m. The cross-section of the rod is
the elastic limit?
uniformly 10 −4 m 2 . A weight of 10 kg is hung from D.
Mgl Mgl 4Mgl
Sol. Young’s modulus,Y = = = Calculate the displacements of points B, C and D, if
πr ⋅∆l πD 2 ⋅ ∆l Y AB = 3.5 × 1010 Nm −2 ,YBC = 5 × 1010 Nm −2 ,
2 2
D 
π   ∆l
 2 YCD = 2 × 1010 Nm −2 . (Neglect the weight of the rod)
where, D is the diameter of the wire. Sol. Given, area, A = 10−4 m2,
∴ Elongation, ∆l =
4Mgl
, i.e. ∆l ∝ 2
1 Y AB = 3.5 × 1010 Nm−2
πD ⋅Y
2
D
YBC = 5 × 1010 Nm−2
Clearly, if the diameter is doubled, the elongation will become
one-fourth. YCD = 2 × 1010Nm−2
M mg
Also, maximum stress, S m =
(πD 2 / 4) A
0.2 m
or Mm ∝ D 2
B
Clearly, if the diameter is doubled, the wire can support four 0.3 m
times the original load. C
Example 12.12 A bar of mass m and length l is hanging 0.15 m
from point A as shown in figure. Find the increase in its D
length due to its own weight. The Young’s modulus of 10 kg
556 OBJECTIVE Physics Vol. 1

Increase in length, Example 12.15 A 2 kg mass is suspended by a rubber cord


FL MgL 10 × 10L L 2 m long and of cross-section 0.5 cm 2 . It is made to describe
∆L = = = = 106 a horizontal circle of radius 50 cm, 4 times in a second.
AY AY 10−4Y Y
Find the extension of the cord. (Take, Young’s modulus,
Now, increase in length of AB segment, Y = 7 × 10 8 Nm −2 )
L 0.2
(∆LAB ) = 106 × AB = 106 × = 5.7 × 10−6m Sol. Given, m = 2 kg, ω = 2πf = 2π × 4 = 8π rad s−1, r = 0.5 m
Y AB 3.5 × 1010
O
LBC
Increase in length of BC segment, (∆L )BC = 106 ×
YBC
θ θ
0.3
= 106 × = 6 × 10−6m
5 × 1010 T L

Increase in length of CD segment,


L 015
. T cos θ
(∆L )CD = 106 CD = 106 × = 7.5 × 10−6 m
YCD 2 × 1010 T sin θ
−6
So, displacement of B = ∆LAB = 5.7 × 10 m
mg
Displacement of C = ∆LAB + ∆LBC = 11.7 × 10−6m r

Displacement of D = ∆LAB + ∆LBC + ∆LCD = 19.2 × 10−6 m From the figure, T cos θ = mg
mv 2
and T sin θ = = mω 2r
Example 12.14 A block of weight 10 N is fastened to one r
end of wire of cross-sectional area 4 mm 2 and is rotated in
a vertical circle of radius 30 cm. The speed of the block at Tension in cord, T = (T sin θ )2 + (T cos θ )2
the bottom of the circle is 3 ms −1. Find the elongation of the
= (mω 2r )2 + (mg )2
wire when the block is at the bottom of the circle. (Take,
Young’s modulus of the material of the wire = 2 × 1011 Nm −2 T = [2 × (8π )2 × 0.5]2 + [2 × 10]2 ≈ 631 N
and g = 10 ms −2 )
The extension of the cord,
Sol. Given, weight, w = 10 N
TL 631 × 2
∆L = =
Area, A = 4 mm2 = 4 × 10−6 m 2 AY 0.5 × 10−4 × 7 × 108
Radius, R = 30 cm = 0.3 m = 360 × 10−4 m = 3.60 cm
−1
Speed of block, v = 3 ms
Example 12.16 A light rod of length 2m is suspended from
the ceiling horizontally by means of two vertical wires of equal
R length tied to its ends. One of the wires is made of steel and
is of cross-section 10 −3 m 2 and the other is of brass of
T cross-section 2 × 10 −3 m 2 . Find out the position along the rod
at which a weight may be hang to produce
(i) equal stresses in both wires,
v (ii) equal strains on both wires.
(Young’s modulus for steel is 2 × 1011Nm −2 and for brass
is 1011Nm −2 )
mg

mv 2 Sol. (i) Given, stress in steel = stress in brass


Tension of the wire, T − mg =
R
mv 2
10 × 32
or T = mg + = 100 + = 400 N Steel Brass
R 0.3
The elongation of the wire when the block is at the bottom of TS TB
the circle, D
FR A C
∆L =
AY
Here, force (F ) is equal to the tension in the wire. x 2−x
TR 400 × 0.3
So, ∆L = = = 1.5 × 10−4 m ∴
TS TB
=
AY 4 × 10−6 × 2 × 1011 AS AB
Elasticity 557

TS AS 10−3 1 Now, using the relation,


⇒ = = = …(i)
TB AB 2 × 10−3 2 F = S S AS + SBAB
As the system is in equilibrium, taking moments about D, we or 5000 = S S × 16 + SB × 20 …(ii)
have Solving Eqs. (i) and (ii), we get
TS ⋅ x = TB (2 − x ) SB = 120.9 kg cm−2
TS 2 − x ∴ From Eq. (i), we get
∴ = …(ii)
TB x 4
SS = × 120.9 = 161.2 kg cm −2
From Eqs. (i) and (ii), we get 3
x = 1.33 m
Stress 2. Bulk modulus of elasticity (B )
(ii) QStrain =
Y When a uniform pressure (normal force) is applied all over
Given, strain in steel = strain in brass the surface of a body, the volume of the body changes.
TS /AS TB /AB The change in per unit volume of the body is called the
∴ = volume strain and the normal force acting per unit area
YS YB
of the surface (pressure) is called the normal stress or
TS ASY S (1 × 10−3 )(2 × 1011) volume stress. For small strains, the ratio of the volume
∴ = = =1 …(iii)
TB ABYB (2 × 10−3 )(1011) stress to the volume strain is called the bulk modulus of
From Eqs. (ii) and (iii), we have the material of the body. It is denoted by B.
Pressure = p0
x=1m
Example 12.17 A steel rod of cross-sectional area 16 cm 2
and two brass rods each of cross-sectional area 10 cm 2 Initial state
together support a load of 5000kg cms −2 as shown in figure. of the object Volume
Find the stress in the rods. (Take, Young’s modulus for steel V0
= 2 × 10 6 kg cm −2 and for brass = 1 × 10 6 kg cm −2 )
Pressure, p = p0 + ∆p
5000 kg cms-2 F⊥
F⊥
Brass
Steel
Brass

30 cm 20 cm

F⊥ Volume F⊥
Object under
V
bulk stress
F⊥
F⊥ V = V0 + ∆V
Sol. Given, area of steel rod, AS = 16 cm2
Fig. 12.6
Area of two brass rods, AB = 2 × 10 = 20 cm2
Load, F = 5000 kg cms −2
Consider a solid cube body of volumeV and surface area
A. In order to compress the body, let a force F be applied
Young’s modulus for the steel, Y S = 2 × 106 kg cm−2
normally on the entire surface of the body and its volume
Young’s modulus for the brass,YB = 1 × 106 kg cm−2 ∆V
decreases by ∆V, then volumetric strain = − .
Length of the steel rod, l S = 30 cm V
Length of the brass rod, lB = 20 cm Here, negative sign shows the volume is decreasing when
Let S S = stress in steel and SB = stress in brass normal force is applied.
Decrease in length of the steel rod Stress = F /A
= Decrease in length of the brass rod
F /A
SS S
× l S = B × lB ∴ B=
or
Y Y − ∆ V /V
S B

Y S lB 2 × 106 20 −p
or SS = × × SB = × × SB Then, B=
YB l S 1 × 106 30 ∆V /V
4 Here, negative sign implies that when the pressure
∴ S S = SB …(i)
3 increases, volume decreases and vice-versa.
558 OBJECTIVE Physics Vol. 1

The SI unit of bulk modulus is Nm −2 and its dimensions Example 12.20 The bulk modulus of water is 2.3 × 10 9 Nm
−2
are [ML−1 T −2 ]. The relation between density ρ, pressure .
(i) Find its compressibility.
∆p and bulk modulus (B ) is given by (ii) How much pressure in atmospheres is needed to compress a
sample of water by 0.1%?
 ∆p 
ρ′ = ρ 1 +  Sol. Here,|B| = 2.3 × 109 Nm −2
 B 
2.3 × 109
In case of gas The isothermal bulk modulus of the gas = = 2.27 × 104 atm
1.01 × 10 5
(BT ) is equal to the pressure of the gas at that instant. 1 1
(i) Compressibility = =
i.e. BT = p |B | 2.27 × 104
The adiabatic bulk modulus of gas (B s ) is equal to γ times = 4.4 × 10−5 atm−1
the pressure of the gas at that instant of time, i.e. B s = γp. ∆V
(ii) Here, = − 0.1% = − 0.001
Note For a perfectly rigid body, B = ∞ V
∴ Required increase in pressure,
Example 12.18 How much should the pressure on a litre of
 ∆V 
water be changed to compress it by 0.10%? (Take, bulk ∆ p = B × − 
modulus of water = 2.2 × 10 9 Pa)  V 
p = 2.27 × 104 × 0.001
Sol. Bulk modulus, |B | =
∆V = 22.7 atm
V
Example 12.21 What will be the decrease in volume of
∆V 0.10
Here, = 0.10% = = 1 × 10−3 100 cm 3 of water under pressure of 100 atm, if the
V 100
compressibility of water is 4 × 10 −5 per unit atmospheric
 ∆V  −3
Change in pressure, p = |B |   = (2.2 × 10 Pa) × 1 × 10
9 pressure?
V 
1 1
Sol. Bulk modulus, | B | = =
= 2.2 × 106 Pa Compressibility k

Compressibility 1
= = 0.25 × 105 atm
4 × 10−5
The reciprocal of the bulk modulus of the material of a body
is called the compressibility of that material. Thus, = 0.25 × 105 × 1.013 × 105 Nm−2
= 2.533 × 109 Nm−2
1 ∆V
Compressibility = = =k Volume, V = 100 cm3 = 10−4 m3
|B| pV
Pressure, p = 100 atm = 100 × 1.013 × 105 Nm−2
−1
The SI unit of compressibility is N m and its 2
= 1.013 × 107 Nm−2
dimensions are [M −1 LT 2 ]. 1 ∆V
Now, apply =k =
The bulk modulus is defined for all solids, liquids and |B | pV
gases. The value of B for solids are much larger than for pV 1.013 × 107 × 10−4
liquids and that for liquids are much larger than for gases. ∴ ∆V = =
|B | 2.533 × 109
Thus, solids are least compressible while gases are most
compressible. Decrease in volume,
∆V ≈ 0.4 × 10−6 m3
Example 12.19 What is the density of lead under a
pressure of 2 × 10 8 Nm −2 , if the bulk modulus of lead is = 0.4 cm3
8 × 10 9 Nm −2 and initially the density of lead is Example 12.22 Find the decrease in the volume of a sample
11.4 g cm −3 ? of water from the following data. Initial volume = 2000 cm 3 ,

Sol. The changed density, ρ′ = ρ1 +
∆p  initial pressure = 10 5 Nm −2 , final pressure = 10 6 Nm −2

 B and compressibility of water = 25 × 10 −11 N −1 m 2 .
Substituting the values, we have Sol. Bulk modulus,
 2 × 108  1 1
ρ′ = 11.41 +  |B | = = = 4 × 10 9
 8 × 109  Compressibility 25 × 10 −11

ρ′ = 11.69 g cm−3 −∆p


or and |B | =
∆V /V
Elasticity 559

∴ Decrease in the volume, A solid will have all the three moduli of elasticity Y, B and
∆pV (106 − 105 )(2000 × 10−6 ) η. But in case of liquid or gas only B can be defined as
∆V = =
|B | 4 × 109 liquid or gas cannot be framed into a wire or no shear
force can be applied on them.
= 4.5 × 10 −7 m 3 = 4.5 × 10 −7 × 10 6 cm 3
Example 12.23 A 4 cm cube has its upper face displaced by
= 0.45 cm 3 0.1 mm by a tangential force of 8 kN. Calculate the shear
modulus of the cube.
3. Modulus of rigidity ( η) Sol. Here, each side of the cube, L = 4 cm
When a body is acted upon by an external force tangential Area of the face over which the force is applied,
to a surface of the body, where the opposite surface is kept A = L2 = 16 cm 2
fixed, it suffers a change in its shape but the volume
Displacement, ∆L = 01
. mm = 0.01 cm
remains unchanged. Then, the body is said to be sheared.
∆L Force applied, F = 8 kN = 8000 × 105 dyne
K K′
F
= 8 × 108 dyne
FL
L As, η=
A∆L
θ θ Shear modulus of the cube,
8 × 108 × 4
F η= = 2 × 1010 dyne/cm 2
N M 16 × 0.01
Fig. 12.7
The ratio of the displacement of a layer in the direction of Example 12.24 A square lead slab of side 50 cm and
the tangential force to the distance of the layer from the thickness 10.0 cm is subjected to a shearing force (on its
fixed surface is called the shearing strain and the narrow face) of magnitude 9.0 × 10 4 N. The lower edge is
tangential force acting per unit area of the surface is called riveted to the floor as shown in figure. How much is the
upper edge is displaced, if the shear modulus of lead is
the shearing stress.
5.6 × 10 9 Pa?
For small strain, the ratio of the shearing stress to the
shearing strain is called the modulus of rigidity of the F
material of the body. It is denoted by η.
50 cm

F /A
Thus, η=
KK ′ /KN
KK ′
Here, = tan θ ≈ θ
KN Sol. Here, L = 50 cm = 50 × 10−2m,
F /A η = 5.6 × 109 Pa, F = 9.0 × 104 N
∴ η=
θ Area of the face on which force is applied,
F
or η= ...(i) A = 50 × 10 cm2 = 500 cm2 = 0.05 m2

If ∆L is the displacement of the upper edge of the slab due to
KK ′ ∆L tangential force F applied, then
Since, θ ≈ tan θ = =
KN L FL
η=
Therefore, Eq. (i) will become A∆ L
FL 9 × 104 × 50 × 10−2
F L ⇒ ∆L = =
η= ⋅ ηA 5.6 × 109 × 0.05
A ∆L
⇒ ∆L = 1.6 × 10−4 m
The SI unit of modulus of rigidity is Nm −2 and its
dimensions are [ML−1T −2 ] . Example 12.25 Consider an Indian rubber cube having
modulus of rigidity of 2 × 10 7 dyne cm −2 and of side
Shear modulus (or modulus of rigidity) is generally
8 cm. If one side of the rubber is fixed, while a tangential
less than Young’s modulus (from table). For most force equal to the weight of 300 kg is applied to the opposite
material, η ≈Y /3. Shear modulus is defined for solids face, then find out the shearing strain produced and distance
only. through which the strained side moves.
560 OBJECTIVE Physics Vol. 1

Sol. Given, modulus of rigidity, η = 2 × 107 dyne cm−2 (ii) Effect of impurities The addition of impurities
affects the elastic properties depending on whether
H H′ G G′ impurities are themselves more or less elastic.
F
When carbon is added to iron and potassium to gold,
E
their elasticities are strengthened.
E′ F F′
(iii) Effect of annealing By annealing (i.e. heating and
8 cm θ then cooling gradually) large crystal grains are
D θ C
formed and hence, the elasticity of the material
decreases.
A B
(iv) Effect of hammering and rolling By hammering
Side of the cube, l = 8 cm and rolling, crystal grains break up into smaller units
Area, A = l 2 = 64 cm 2 and hence, the elasticity of the material increases.
Force or load, F = 300 kgf = 300 × 1000 × 981 dyne Example 12.26 The stress-strain graph for a metallic wire is
F F shown at two different temperatures T1 and T 2 , then which
Modulus of rigidity, η = ⇒ θ=
Aθ Aη temperature is high T1 or T 2 ?
300 × 1000 × 981 Sol. If the slope of stress-strain curve with strain at X-axis gives
Shearing strain, θ = = 0.23 rad
64 × 2 × 107 the value of Young’s modulus.
F l ∆l
As, η= ⇒ =θ Strain
A ∆l l T2
Distance through which the strained side moves, T1
∆ l = l θ = 8 × 0.23
⇒ ∆ l = 1.84 cm
Stress
Factors affecting elasticity In the above graph, strain is taken along Y-axis. Therefore,
(i) Effect of temperature Almost for all materials, the the slope of graph at temperature T1 is less than the slope of
modulus of elasticity decreases with the rise in graph at temperature T2.
temperature, but the elasticity of invar remains Now, as we know with increase in temperature, the value of
unchanged with the change in temperature. modulus of elasticity decreases. Therefore, temperature T1 is
less than temperature T2.

CHECK POINT 12.2


1. Amongst the following, which option correctly represents cross-section of the first wire is 4 mm2, then the area of
the increasing order of coefficient of elasticity for Copper, cross-section of the second wire is
Steel, Glass and Rubber. (a) 6 mm2 (b) 8 mm2 (c)10 mm2 (d)12 mm2
(a) Steel, Rubber, Copper, Glass
(b) Rubber, Copper, Glass, Steel 5. A 5 m long aluminium wire (Y = 7 × 1010 Nm−2) of diameter
3 mm supports a 40 kg mass. In order to have the same
(c) Rubber, Glass, Steel, Copper
elongation in a copper wire (Y = 1.2 × 1010 N m−2) of the same
(d) Rubber, Glass, Copper, Steel
length under the same weight, the diameter should now be
2. The Young’s modulus of the material of a wire is equal to (in mm)
the (a) 1.75 (b) 1.5 (c) 2.29 (d) 5.0
(a) stress required to increase its length four times
6. Which one of the following statement(s) is/are incorrect?
(b) stress required to produce unit strain
(a) Young’s modulus for a perfectly rigid body is zero.
(c) strain produced in it
(b) Bulk modulus is relevant for solids, liquids and gases.
(d) half the strain produced in it
(c) Rubber is less elastic than steel.
3. A rubber cord 10 m is suspended vertically. How much does (d) The Young’s modulus and shear modulus are relevant for
it stretch under its own weight? (Density of rubber is solids.
1500 kg m−3, Y = 5 × 108 Nm−2and g = 10 ms −2 )
7. Choose the incorrect statement.
(a)15 × 10−4 m (b) 7.5 × 104 m (a) The bulk modulus for solids is much larger than for liquids.
(c)12 × 10−4 m (d) 25 × 10−4 m (b) Gases are least compressible.
(c) The incompressibility of the solids is due to the tight
4. Two similar wires under the same load yield elongation of coupling.
0.1 mm and 0.05 mm, respectively. If the area of (d) The reciprocal of the bulk modulus is called compressibility.
Elasticity 561

Q By definition, Poisson’s ratio,


POISSON’S RATIO  1 dl 
− 
When a wire is loaded, its length increases but − dD / D  2 l 1
σ= =− = = 0.5
its diameter decreases. The strain produced in dl / l dl 2
D
the direction of applied force is called l
longitudinal strain and strain produced in the
perpendicular direction is called lateral L Example 12.28 A material is having Poisson’s ratio 0.2.
strain. Within the elastic limit, the ratio of A load is applied on it due to which it suffers the
lateral strain to the longitudinal strain is called longitudinal strain 3 × 10 −3 , then find out the percentage
Poisson’s ratio. It is represented by (σ ). change in its volume.
Sol. Given, Poisson’s ratio, σ = 0.2
Let the length of the loaded wire increases
from L to L + ∆ L and its diameter decreases ∆L ∆L
Longitudinal strain, = 3 × 10−3
from D to D − ∆D . L
∆L D – ∆D − ∆D / D
Longitudinal strain = Q Poisson’s ratio, σ =
L Fig. 12.8 ∆L / L
− ∆R / R
−∆D ⇒ σ=
Lateral strain = ∆L / L
D ∆R ∆L
Lateral strain −∆ D / D ⇒ = −σ × = − 0.6 × 10−3
Poisson’s ratio is σ = = R L
Longitudinal strain ∆L /L Q Volume,V = πR 2L
Therefore, percentage change in volume,
−L ∆ D
Poisson’s ratio, σ = ⋅  ∆V   2 ∆R ∆L 
D ∆L  × 100 =  +  × 100
V   R L
The negative sign indicates that longitudinal and lateral
= [2 × (− 0.6 × 10−3 ) + 3 × 10−3] × 100 = 018
. %
strain are in opposite sense. It has no units and
dimensions.
−1 < σ < 0.5, for theoretical purpose Stress-strain curve
 When a wire is stretched by a load as shown in
 0 < σ < 0.5, for practical purpose
Fig. 12.9 (a), it is seen that for small value of load, the
Relation between Y, B, η and σ extension produced in the wire is proportional to the load.
∴ Stress ∝ Strain
There are four relations betweenY, B, η and σ, which can
be given as follows When a graph between stress and strain is plotted, then
Y Y (i) The stress is found to be proportional to strain
(i) B = (ii) η = (% elongation) up to point A. Thus, Hooke’s law is
3 (1 − 2σ ) (1 + σ ) 2
fully obeyed in this region, the point A is known
3B − 2η 9 1 3 as point of proportional limit.
(iii) σ = (iv) = +
2η + 6B Y B η (ii) When stress is increased beyond A, then for small
Example 12.27 Determine the Poisson’s ratio of the material
stress, there is a large strain in the wire upto
of a wire whose volume remains constant under an external point B.
normal stress. (iii) When the load is gradually removed between points
πD 2 O to B, the wire return to its original length.
Sol. Volume of a wire,V = l
4 The wire regains its original dimension only when
As volume remains constant, the differentiation of above load applied is less than or equal to certain limit.
equation gives This limit is called elastic limit. The point B on
stress-strain curve is known as elastic limit or yield
πl πD 2
0= ⋅ 2D ⋅ dD + ⋅ dl point. The material of the wire in the region OB
4 4 shows the elastic behaviour, hence known as elastic
− 2ldD = Ddl region.
dD 1 dl (iv) If the stress or load increases beyond point B, the
⇒ =−
D 2 l strain further increases. This increase in strain
represented by BC part of the curve. Now, if the load
562 OBJECTIVE Physics Vol. 1

is removed, the wire does not regain its original Ductile and brittle materials
length. But the increase in the length of the wire is
permanent. On the basis of elastic and plastic properties, materials can
be classified in two ways; ductile materials and brittle
In other words, there is permanent strain equal to
OO ′ in the wire even when the stress is zero. This materials.
permanent strain in the wire is known as The materials which have large plastic range of extension
permanent set. are called ductile materials. As shown in the stress-strain
curve, the fracture point is widely separated from the elastic
Elastic limit or Fracture limit. Such materials undergo an irreversible increase in
Yield point point length before snapping. So, they can be drawn into thin wires,
e.g. copper, silver, iron, aluminium, etc.
C D
B Plastic deformation
A Breaking
Stress

Proportional limit point


Plastic behaviour
BC D
Elastic behaviour A

Stress
Load = mg Permanent set Elastic limit
O
(a) 0 <1% O′ 30%
Strain
(b) O Strain
Fig. 12.9 A typical stress-strain curve Fig. 12.10 Stress-strain curve for a ductile material

(v) Now, as the stress beyond C is increased, there is The materials which have very small range of plastic
huge strain in the wire. This huge increase in the extension are called brittle materials. Such materials break
strain for small stress is represented by CD part of as soon as the stress is increased beyond the elastic limit.
the curve. The wire breaks at point D which is also
known as fracture point. Elastic limit
A D
The material of the wire from point C to point D
shows the plastic behaviour or plastic Breaking
Stress

point
deformation. The stress needed to cause the actual
fracture of the material is known as breaking stress
or ultimate tensile strength. O
Note Strain
(i) Elastic limit and limit of proportionality are very close to each other, Fig. 12.11 Stress-strain curve for a brittle material
so that Hooke’s law is nearly applicable upto elastic limit.
(ii) In the yield region, strains is 15 to 20 times than those strains that Their breaking point lies just close to their elastic limit as
takes place upto the proportional limit that occur during yielding. shown in figure, e.g. cast iron, glass, ceramics, etc.
Breaking Stress of Some Materials
Material Breaking stress (in Nm−2)
Malleability
When a solid is compressed, a stage is reached beyond
Aluminium 2.2 × 108
which it cannot regains its original shape after the deforming
Iron 3.0 × 108 force is removed.
Brass 4.7 × 108 This is the elastic limit point (A′ ) for compression. The
Phosphor bronze 5.6 × 108 solid then behaves like a plastic body.
Steel 5 to 20 × 108
Compression
Glass 10 × 108

Different nature of the stress-strain curve Crushing


point Applied
The stress-strain curve differ considerably for different A′
force
materials. The steeper curve indicates a stiffer material, B′
line parallel to the strain axis indicates a ductile material Elastic
limit
and in the absence of yield point or plastic behaviour refer
to a brittle material. Fig. 12.12 Load compression curve for a metal
Elasticity 563

The yield point (B′) obtained under compression is called The magnitude of the energy absorbed is proportional to
crushing point. the area of the loop. The material having low elastic
After this stage, metals are said to be malleable, i.e. they hysteresis have low elastic relaxation time.
can be hammered or rolled into thin sheets, e.g. gold, Example 12.29 The stress-strain graphs for two materials A
silver, lead, etc. and B are shown in the figure. The graphs are drawn to the
same scale.
Elastomers
The materials which can be elastically stretched to large Q
P P
values of strain are called elastomers. e. g. Rubber can be

Stress

Stress
Q
stretched to several times its original length but still it A
B
can regain its original length when the applied force is
removed. There is no well defined plastic region, rubber
O O
just breaks when pulled beyond a certain limit. Strain Strain

Elastic region in such cases is very large, but the material (i) Which material has a greater Young’s modulus?
does not obey Hooke’s law. (ii) Which material is more ductile?
(iii) Which material is more brittle?
In our body, the elastic tissue of aorta (the large blood
(iv) Which of the two is the stronger material?
vessel carrying blood from the heart) is an elastomer.
For which the stress-strain curve is shown in figure.
Sol. (i) Material A has a greater Young’s modulus because the
slope of the linear portion of the stress-strain curve is
1.0 greater for the material A.
Stress (106 Nm–2)

(ii) Material A is more ductile because it has a greater


plastic region (from elastic limit P to breaking point Q)
0.5 than material B.
(iii) Material B is more brittle because it has a smaller plastic
0 region.
0 0.5 1.0 (iv) Material A is stronger because it can withstand a greater
Strain stress before it breaks.
Fig. 12.13 Stress-strain curve

Elastic hysteresis
Work done or potential energy
As a natural consequence of the elasticity after effect, the stored in a stretched wire
strain in the body tends to lag behind the stress applied to When a wire is stretched, work is done against the
the body, so that during a rapidly changing stress, the interatomic forces. This work is stored in the wire in the
strain is greater for the same value of stress. This lag of form of elastic potential energy.
strain behind the stress is called elastic hysteresis. Thus, stored elastic energy is
Due to elastic hysteresis, the original curve (OAB) is not 1
U = Stress × Strain × Volume
retraced when the deforming force is removed, although 2
the body finally acquires natural length. The figure clearly 1
indicates that the work done by the material in returning or U = Y (Strain) 2 × Volume
2
to its original shape is less than the work done by the
1
deforming force. Hence, some amount of energy is U= Load × Elongation
absorbed by the material in the cycle which appears as 2
heat.
B Example 12.30 Calculate the work done in stretching a steel
wire of Young’s modulus of 2 × 1011 Nm −2 , length of 200 cm
and area of cross-section 0.06 cm 2 slowly by applying a
Stress

A
load of 40 kg without the elastic limit being reached.
C Sol. Here, force, F = mg = 40 × 9.8 N
Length of wire, l = 200 cm = 2 m
O Strain
Area of cross-section, A = 0.06 cm2 = 0.06 × 10−4 m2
Fig. 12.14 Vulcanised rubber
564 OBJECTIVE Physics Vol. 1

Young’s modulus,Y = 2 × 1011 Nm−2 Example 12.33 A rubber cord has a cross-sectional area
1 1 1 mm 2 and total unstretched length 10.0 cm. It is
Work done = × Stretching force × Extension = F∆l stretched to 12.5 cm and then released to project a missile
2 2
of mass 5.0 g. (Take, Young’s modulusY for rubber as
1
= F ×
Fl 5.0 × 10 8 Nm −2 ). Calculate the velocity of projection.
2 AY Sol. Equivalent force constant of rubber cord,
F 2l (40 × 9.8)2 × 2 YA (5.0 × 108 )(1.0 × 10−6 )
⇒ W= = = 0.128 J k= = = 5.0 × 103 Nm−1
2AY 2 × 0.06 × 10−4 × 2 × 1011 l (0.1)
Example 12.31 A steel wire 4 m in length is stretched Now, from conservation of mechanical energy, elastic
through 2 mm. The cross-sectional area of the wire is potential energy of cord = kinetic energy of missile
2.0 mm 2 . If Young’s modulus of the steel is 2 × 1011 Nm −2 . ∴
1 1
k (∆l )2 = mv 2
Find 2 2
(i) the energy density of wire,  k  5.0 × 103 
∴ v=  ∆l =   (12.5 − 10.0) × 10−2
(ii) the elastic potential energy stored in the wire.
 m  5.0 × 10−3 
 
Sol. Here, length of wire, l = 4.0 m, ∆l = 2 × 10−3 m
= 25 ms−1
Area, A = 2.0 × 10−6 m2
Note Following assumptions have been made in this example
Young’s modulus, Y = 2.0 × 1011 Nm−2 (i) k has been assumed constant, even though it depends on the length
(i) The energy density of stretched wire, ( l ).
1 1 (ii) The whole of the elastic potential energy is converting into kinetic
u = × Stress × Strain = × Y × (Strain) 2 energy of missile.
2 2
 −3 2 
1 (2 × 10 ) Example 12.34
= × 2.0 × 1011 ×  2 
2  (4)  (i) A wire 4 m long and 0.3 mm in diameter is stretched by a
force of 100 N. If extension in the wire is 0.3 mm, calculate
= 0.25 × 105 Jm −3
the potential energy stored in the wire.
= 2.5 × 104 Jm−3
(ii) Find the work done in stretching a wire of cross-section
(ii) Elastic potential energy = Energy density × Volume 1mm 2 and length 2 m through 0.1 mm. Young’s modulus for
= 2.5 × 104 × ( 2.0 × 10−6 ) × 4.0 J
the material of wire is 2.0 × 1011 Nm −2.
= 20 × 10−2 = 0.20 J
Sol. 1
(i) Energy stored, U = (Stress)(Strain) (Volume)
Example 12.32 A 45 kg boy whose leg bones are 5 cm 2 in area 2
and 50 cm long falls through a height of 2 m without breaking 1 F   ∆l  1
his leg bones. If the bones can stand a stress of 0.9 × 10 8 Nm −2 , or U =     (Al ) = F ⋅ ∆l
2  A  l  2
then what will be the Young’s modulus for the material of the
1
bone? = (100)(0.3 × 10−3 ) = 0.015 J
2
Sol. Here, mass, m = 45 kg, height of leg bones, h = 2 m
(ii) Work done = Potential energy stored
Length, L = 0.50 m, area of bones, A = 5 × 10−4 m2
1 1 YA  YA
= k ( ∆l )2 =   ( ∆l )2 Q k = 
Loss in gravitational energy 2 2 l   l 
= Gain in elastic energy in both leg bones
Substituting the values, we have
1 
So, mgh = 2 ×  × stress × strain × volume 1 (2.0 × 1011)(10−6 )
2  W= ( 0.1 × 10−3 )2
2 (2)
Here, volume = AL = 5 × 10−4 × 0.50 = 2.5 × 10−4 m3 = 5.0 × 10−4 J
1 
∴ 45 × 10 × 2 = 2 ×  × 0.9 × 108 × strain × 2.5 × 10−4
 2  Thermal stresses and strains
45 × 10 × 2
or Strain = = 0.04 Whenever there is some increase or decrease in the
0.9 × 2.5 × 104 temperature of the body, it causes the body to expand or
∴ Young’s modulus of bones material, contract. If the body is allowed to expand or contract
Stress 0.9 × 108 freely, with the rise or fall in the temperature, no stresses
Y = =
Strain 0.04 are induced in the body. But, if the deformation of the
= 2.25 × 109 Nm−2
body is prevented, some stresses are induced in the body.
Elasticity 565

Such stresses are called thermal stresses or temperature But since, the rod is fixed at the supports, so a
stresses. compressive strain will be produced in the rod due to
The corresponding strains are called thermal strains or which a force is applied on the supports, given
temperature strains. F = YAα∆t
Consider a rod AB fixed at two supports as shown in figure.
Let l = length of the rod, Example 12.35 A steel wire of length 20 cm and uniform
cross-section 1 mm 2 is tied rigidly at both the ends. If
A = area of cross-section of the rod,
temperature of the wire is altered from 40°C to 20°C,
Y = Young’s modulus of elasticity of the rod calculate the change in tension. (Take, coefficient of linear
and α = thermal coefficient of linear expansion of the rod. expansion of steel is 1.1 × 10 −5 ° C −1 and Young’s modulus
for steel is 2.0 × 1011 Nm −2 )
Sol. The change in length l of the wire when wire is cooled by
l temperature ∆t is given by
Fig. 12.15 Rod fixed between two supports ∆l = l α ∆ t or ∆ l /l = α ∆ t …(i)
where, α is the coefficient of linear expansion. Change in the
Let the temperature of the rod be increased by an amount tension of the wire when it is cooled is given by
t. The length of the rod would had increased by an amount F = YA ∆ l /l = YAα ∆ t [From Eq. (i)]
∆l, if it were not fixed at two supports. Here, = ( 2.0 × 1011) × (10−6 ) × (1.1 × 10−5 ) × (40°−20° )
∆l = l α t = 44 N

CHECK POINT 12.3


1. There is no change in the volume of a wire due to change in 7. If a spring extends by x on loading, then the energy stored
its length on stretching. The Poisson’s ratio of the material by the spring is (if T is tension in the spring and k is spring
of the wire is constant)
(a) + 0.50 (b) − 0.50 (c) 0.25 (d) − 0.25 T2 T2 2x 2T 2
(a) (b) (c) (d)
2. For a given material, the Young’s modulus is 2.4 times that 2x 2k T2 k
of rigidity modulus. Its Poisson’s ratio is 8. A wire suspended vertically from one of its ends is stretched
(a) 2.4 (b) 1.2 (c) 0.4 (d) 0.2 by attaching a weight of 200 N to the lower end. The weight
3. Stress-strain curve for four metals are shown in figure. The stretches the wire by 1 mm. Then, the energy stored in the
maximum Young's modulus of elasticity for metal, is wire is
(a) 0.1 J (b) 0.2 J (c) 10 J (d) 20 J
Strain
D 9. Wires A and B are made from the same material. A has
C twice the diameter and three times the length of B. If the
elastic limits are not reached, when each is stretched by the
B
same tension, the ratio of energy stored in A to that in B is
A (a) 2 : 3 (b) 3 : 4 (c) 3 : 2 (d) 6 : 1
Stress 10. A wire of length 50 cm and cross-sectional area of
1 sq. mm is extended by 1 mm. The required work will be
(a) A (b) B (c) C (Take, Y = 2 × 1010 Nm−2 )
(d) D
(a) 6 × 10−2 J (b) 4 × 10−2 J
4. A metallic rod of length l and cross-sectional area A is made
(c) 2 × 10−2 J (d)1 × 10−2 J
of a material of Young’s modulus Y. If the rod is elongated
by an amount y, then the work done is proportional to 11. When a force is applied on a wire of uniform cross-sectional
(a) y (b)1/y (c) y 2 (d)1 / y 2 area 3 × 10−6 m2 and length 4 m, the increase in length is
1 mm. Energy stored in it will be (Take, Y = 2 × 1011 N m−2 )
5. The work done per unit volume to stretch the length by 1%
of a wire with cross-sectional area of 1 mm2 will be (a) 6250 J (b) 0.177 J
(Take, Y = 9 × 1011 Nm−2) (c) 0.075 J (d) 0.150 J

(a) 9 × 1011 J (b) 4.5 × 107 J 12. When a 4 kg mass is hung vertically on a light spring that
obey’s Hooke’s law, the spring stretches by 2 cms. The work
(c) 9 × 107 J (d) 4.5 × 1011 J required to be done by an external agent in stretching this
6. When a load of 10 kg is hung from the wire, then extension spring by 5 cm will be (Take, g = 9.8 m s −2 )
of 2 m is produced. Then, work done by restoring force is (a) 4. 9 J (b) 2.4 J
(a) 200 J (b) 100 J (c) 50 J (d) 25 J (c) 0.495 J (d) 0.24 J
Chapter Exercises
(A) Taking it together
Assorted questions of the chapter for advanced level practice

1 The maximum load, a wire can withstand without 8 The graph shows the behaviour of a length of wire
breaking, when its length is reduced to half of its in the region for which the substance obeys Hooke’s
original length, will [NCERT Exemplar] law. P and Q represent
(a) be double (b) be half
(c) be four times (d) remain same Q
−2
2 For steelY = 2 × 10 Nm . The force required to
11

double the length of a steel wire of area 1 cm 2 is


(a) 2 × 107 N (b) 2 × 106 N
P
(c) 2 × 10 N 8
(d) 2 × 105 N
(a) P = applied force, Q = extension
3 Longitudinal stress of 1 kg mm −2 is applied on a (b) P = extension, Q = applied force
wire. The percentage increase in length is (c) P = extension, Q = stored elastic energy
(Take, Y = 10 11 Nm −2 ) (d) P = stored elastic energy, Q = extension
(a) 0.002 (b) 0.001 9 A metal block is experiencing an atmospheric
(c) 0.003 (d) 0.01 pressure of 10 5 Nm−2 . When the same block is
4. The Young’s modulus of a wire isY, if the energy placed in a vacuum chamber, the fractional change
per unit volume is E, then the strain will be in its volume, is (the bulk modulus of metal is
2E 1.25 × 10 11 Nm−2 )
(a) (b) E 2Y
Y (a) 4 × 10−7 (b) 2 × 10−7
−7
(c) EY (d)
E (c) 8 × 10 (d) 1 × 10−7
Y
10 The increase in length on stretching a wire is 0.05%.
5 When a force is applied at one end of an elastic If its Poisson’s ratio is 0.4, then its diameter
wire, it produces a strain E in the wire. IfY is the (a) reduce by 0.02% (b) reduce by 0.1%
Young’s modulus of the material of the wire, the (c) increase by 0.02% (d) decrease by 0.4%
amount of energy stored per unit volume of the wire
is given by 11 An elastic material with Young’s modulusY is
1 subjected to a tensile stress S, elastic energy stored
(a) Y × E (b) (Y × E ) per unit volume of the material is
2
1 YS S2 S2 S
(c) Y × E 2 (d) Y ( × E2) (a) (b) (c) (d)
2 2 Y 2Y 2Y

6 Young’s modulus of the material of a wire of length 12 When a metal wire elongates by hanging a load Mg
−2 on it, the gravitational potential energy of mass M
L and radius r isY Nm . If the length is reduced to
L r decreases by Mgl. This energy appears
and radius to , the Young’s modulus will be (a) as elastic potential energy completely
2 2 (b) as thermal energy completely
Y Y
(a)Y (b) 2Y (c) (d) (c) half as elastic potential energy and half as thermal energy
4 2 (d) as kinetic energy of the load completely
7. The Young’s modulus of a wire is numerically equal 13 A 5 m long wire is fixed to the ceiling. A weight of
to the stress which will 10 kg is hang at the lower end and is 1m above the
(a) not change the length of the wire floor. The wire was elongated by 1mm. The energy
(b) double the length of the wire stored in the wire due to stretching is
(c) increase the length by 50% (a) 0.01 J (b) 0.05 J
(d) change the radius of the wire to half (c) 0.02 J (d) 0.04 J
Elasticity 567

14 If the Young’s modulus of the material is 3 times its 24 A rectangular block of size 10 cm × 8 cm × 5 cm is
modulus of rigidity, then its volume elasticity will be kept in three different positions P, Q and R in turn
(a) zero (b) infinity as shown in the figure. In each case, the shaded area
(c) 2 × 1010 N /m2 (d) 3 × 1010 N /m2 is rigidly fixed and a definite force F is applied
15 A cable that can support a load w is cut into two tangentially to the opposite face to deform the block.
equal parts. The maximum load that can be The displacement of the upper face will be
supported by either part is 8 8
w w 10 5
(a) (b) (c) w (d) 2w
4 2
16 The longitudinal extension of any elastic material is 5 10
very small. In order to have an appreciable change,
the material must be in the form of (P)
(Q )
(a) long thick wire (b) short thick wire
10
(c) long thin wire (d) short thin wire
5
17 Two wires of same diameter of the same material
having the length l and 2 l. If the force F is applied
8
on each, the ratio of the work done in the two wires
will be
(R)
(a) 1 : 2 (b) 1 : 4 (c) 2 : 1 (d) 1 : 1
18 The temperature of a wire of length 1 m and area of (a) same in all the three cases
cross-section 1 cm 2 is increased from 0°C to 100°C. If (b) maximum in P position
the rod is not allowed to increase in length, the force (c) maximum in Q position
required will be (α = 10 −5 /° C andY = 10 11 Nm −2 ) (d) maximum in R position

(a) 103 N (b) 104 N (c) 105 N (d) 109 N 25 A force of 20 N is applied at one end of a wire of
length 2 m and having area of cross-section 10 −2 cm2 .
19 The temperature of a wire is doubled. The Young’s
modulus of elasticity [NCERT Exemplar] The other end of the wire is rigidly fixed. If the
(a) will also double (b) will become four times coefficient of linear expansion of the wire
(c) will remain same (d) will decrease α = 8 × 10 −6 /° C and Young’s modulus
20 When a pressure of 100 atm is applied on a spherical Y = 2.2 × 10 11 Nm−2 and its temperature is increased
ball, then its volume reduces to 0.01%. The bulk by 5°C, then the increase in the tension of the wire
modulus of the material of the rubber (in dyne cm −2 ) will be
is (a) 4.2 N (b) 4.4 N
(c) 2.4 N (d) 8.8 N
(a) 10 × 10 12
(b) 100 × 10 12
(c) 1 × 10 12
(d) 20 × 10 12

26 A load suspended by a massless spring produces an


21 Two wires of copper having the length in the ratio extension of x cm in equilibrium. When it is cut into
4 : 1 and their radii ratio as 1 : 4 are stretched by two unequal parts, the same load produces an
the same force. The ratio of longitudinal strain in the extension of 7.5 cm when suspended by the larger
two will be part of length 60 cm. When it is suspended by the
(a) 1 : 16 (b) 16 : 1 (c) 1 : 64 (d) 64 : 1 smaller part, the extension is 5.0 cm. Then,
22 The interatomic distance for a metal is 3 × 10 −10 m. (a) x = 12.5 cm
−9 (b) x = 3.0 cm
If the interatomic force constant is 3.6 × 10 N/Å,
(c) the length of the original spring is 90 cm
then the Young’s modulus (in Nm −2 ) will be (d) the length of the original spring is 80 cm
(a) 1.2 × 1011 (b) 4.2 × 1011 (c) 10.8 × 108 (d) 2.4 × 1010 27 A steel wire of cross-sectional area 3 × 10 −6 m2 can
23 Young’s modulus of rubber is 10 4 Nm−2 and area of withstand a maximum strain of 10 −3 . Young’s
cross-section is 2 cm . If force of 2 × 10 dyne is modulus of steel is 2 × 10 11Nm−2 . The maximum
2 5

applied along its length, then its initial length l mass this wire can hold is
becomes (a) 40 kg (b) 60 kg
(a) 3 l (b) 4 l (c) 80 kg (d) 100 kg
(c) 2 l (d) None of these
568 OBJECTIVE Physics Vol. 1

28 The load versus elongation graph for four wires of 35 The stress versus strain graphs for wires of two
the same material is shown in the figure. The materials A and B are as shown in the figure. IfYA
thickest wire is represented by the line andYB are the Young’s moduli of the materials, then
Load
A
D
C

Stress
B B
60°
A
30°
O Strain
Elongation
(a) YB = 2Y A (b) Y A = YB (c) YB = 3Y A (d) Y A = 3YB
(a) OD (b) OC (c) OB (d) OA
36 An elevator cable is to have a maximum stress of
29 Two wires of same material and length but diameter
7 × 10 7 Nm −2 to allow for appropriate safety
in the ratio 1 : 2 are stretched by the same force.
The potential energy per unit volume for the two factors. Its maximum upward acceleration is
wires when stretched will be in the ratio 1.5 ms −2 . If the cable has to support the total weight
(a) 16 : 1 (b) 4 : 1 (c) 2 : 1 (d) 1 : 1 of 2000 kg of a loaded elevator, the area of
30 A stress of 10 6 Nm −2 is required for breaking a cross-section of the cable should be
material. If the density of the material is (a) 3.23 cm2 (b) 2.38 cm2 (c) 0.32 cm2 (d) 8.23 cm2
3 × 10 3 kgm−3 , then what should be the length of the
wire made of this material, so that it breaks under its 37 The strain-stress curves of three wires of different
own weight? materials are shown in the figure. P, Q and R are
(a) 10 m (b) 33.3 m elastic limits of the wires.
(c) 5 m (d) 66.6 m P

31 A cable is replaced by another one of the same Q


Strain

length and material but of twice the diameter. The


R
maximum load that the new wire can support
without exceeding the elastic limit, as compared to
O
the load that the original wire could support, is Stress
(a) half (b) double The figure shows that
(c) four times (d) one-fourth (a) elasticity of wire P is maximum
32 A uniform steel bar of cross-sectional area A and (b) elasticity of wire Q is maximum
length L is suspended, so that it hangs vertically. (c) tensile strength of R is maximum
The stress at the middle point of the bar is (ρ is the (d) None of the above
density of steel) 38 Figure shows the strain-stress curve for a given
L Lρ g LA material. What are (i) Young’s modulus and
(a) ρg (b) (c) (d) L ρg
2A 2 ρg (ii) approximate yield strength for this material?
33 A thick rope of rubber of length 8 m and density
1.5 × 10 3 kg m−3 has Young’s modulus 5 × 10 6 Nm−2 . 300
Stress (106 Nm–2)

When hung from ceiling of a room, the increase in 250


length due to its own weight is 200
(a) 96 × 10−3 m (b) 19.2 × 10−5 m 150
(c) 9.4 cm (d) 9.6 mm 100
34 In the given figure, if the 50
A
dimensions of the wires are the 0
same and materials are 0 0.001 0.002 0.003 0.004
Load

B
Strain
different, Young’s modulus is
more for (a) 5 × 105 Nm−2 , 3 × 106 Nm−2
Extension (b) 2 × 10− 6 Nm−2, 4 × 104 Nm−2
(a) A (b) B
(c) Both (d) None of these (c) 7.5 × 1010 Nm−2, 3 × 108 Nm−2
(d) 7.5 × 1010 Nm−2, 3 × 106 Nm−2
Elasticity 569

39 Consider two cylindrical rods of identical 43 A rigid bar of mass M is supported symmetrically by
dimensions, one of rubber and the other of steel. three wires each of length L. Those at each end are
Both the rods are fixed rigidly at one end to the roof. of copper and the middle one is of iron. The ratio of
A mass M is attached to each of the free ends at the their diameters, if each is to have the same tension, is
centre of the rods. Then, [NCERT Exemplar] equal to
(a) both the rods will elongate but there shall be no Y copper Yiron Y 2 iron Y iron
perceptible change in shape (a) (b) (c) (d)
(b) the steel rod will elongate and change shape but the Yiron Y copper Y 2 copper Y copper
rubber rod will only elongate
44 One end of a long metallic wire of length L is tied to
(c) the steel rod will elongate without any perceptible
change in shape, but the rubber rod will elongate and the ceiling, the other end is tied to a massless spring
the shape of the bottom edge will change to an ellipse of spring constant k. A mass m hangs freely from the
(d) the steel rod will elongate without any perceptible free end of the spring. The area of cross-section and
change in shape, but the rubber rod will elongate with Young’s modulus of the wire are A andY,
the shape of the bottom edge tapered to a tip at the centre respectively. If the mass is slightly pulled down and
40 The graph shown below gives the extension (∆L ) of a released, it will oscillate with a time period T equal to
wire of length 1 m suspended from the top of a roof m m (YA + kL )
(a) 2 π (b) 2 π
at one end with a load w connected to the other end. k YAk
If the cross-sectional area of the wire is 10 −6 m2 , (c) 2 π
mYA
(d) 2 π
mL
calculate the Young’s modulus of the material of the kL YA
wire. 45 The length of an elastic string is a metre when the
tension is 4 N and b metre when the tension is 5 N.
4 The length, (in metre), when the tension is 9 N, is
∆L (a) (a + b ) (b) (4b − 5a ) (c) (5b − 4a ) (d) (9b − 9a )
(1×10–4 m) 3
46 A uniform elastic plank moves due to a constant
2 force F placed over a smooth surface. The area of
1 end face is S and Young’s modulus of the material is
E. What is average strain produced in the direction
20 40 60 80 w (N)
of the force?
F F F
(a) (b) (c) (d) Zero
(a) 2 × 1011 Nm−2 (b) 2 × 10−11 Nm−2 SE 2SE 4SE
(c) 3 × 10−12 Nm−2 (d) 2 × 10−13 Nm−2 47 Two wires of the same material (Young’s
41 A brass rod of length 2 m and cross-sectional area modulus =Y ) and same length L but radii R L, 2R
2
2 cm is attached end-to-end to a steel rod of length and 2R respectively, are joined end-to-end
and a weight w is suspended from the
L and cross-sectional area 1 cm2 . The compound rod
combination as shown in the figure. L, R
is subjected to equal and opposite pulls of magnitude
The elastic potential energy in the system w
5 × 10 4 N at its ends. If the elongations of the two
is
rods are equal, the length of the steel rod (L ) is 3w 2L 3w 2L 5w 2L w 2L
(Ybrass = 1 × 10 11 Nm−2 andYsteel = 2 × 10 11 Nm−2 ) (a) (b) (c) (d)
4πR 2Y 8πR 2Y 8πR 2Y πR 2Y
(a) 1.5 m (b) 1.8 m
(c) 1 m (d) 2 m 48 A mild steel wire of length 2L and cross-sectional
area A is stretched, well within elastic limit,
42 One end of a uniform wire of length L and weight w
horizontally between two pillars (figure ). A mass m
is attached rigidly to a point in roof and a weight w 1
is suspended from the mid-point of the wire. Strain
is suspended from its lower end. If S is the area of
in the wire is [NCERT Exemplar]
cross-section of the wire, the stress in the wire at a
3L 2L
height of from its lower end is x
4
w  3w 
w1 +  w1 + 
4  (d) w1 + w
w m
(a) 1 (b) 4 (c) 
S S  S  S x2 x
  (a) (b) (c) x 2 / L (d) x 2 / 2L
2L2 L
OBJECTIVE Physics Vol. 1

(B) Medical entrance special format questions


Assertion and reason
Directions (Q. Nos. 1-5) These questions consist of two Which of the following statement(s) is/are incorrect?
statements each printed as Assertion and Reason. While (a) Work done by the external force is Fl.
(b) Some heat is produced in the wire in the process.
answering these questions you are required to choose any
Fl
one of the following four responses. (c) The elastic potential energy of the wire is .
2
(a) If both Assertion and Reason are true and Reason is the
correct explanation of Assertion. (d) None of the above
(b) If both Assertion and Reason are true but Reason is not the 3 A copper wire (Y = 10 11 Nm −2 ) of length 8 m and a
correct explanation of Assertion.
steel wire (Y = 2 × 10 11 Nm −2 ) of length 4 m, each
(c) If Assertion is true but Reason is false.
(d) If Assertion is false but Reason is true. of 0.5 cm2 cross-section are fastened end-to-end
1 Assertion Upto the elastic limit, strain ∝ stress. and stretched with a tension of 500 N. Choose the
correct statement.
Reason Upto elastic limit, material returns to its (a) Elongation in copper wire is 0.8 mm.
original shape and size, when external force is 1
removed. (b) Elongation in steel is th the elongation in copper
4
2 Assertion If length of a wire is halved, its Young’s wire.
modulus of elasticity will become two times. (c) Total elongation is 1.0 mm.
Reason The ratio of longitudinal stress and (d) All of the above
longitudinal strain is called Young’s modulus of 4 The diagram shows a
elasticity. force-extension graph for a rubber

Extension
3 Assertion If a wire is stretched, only half of the band. Consider the following
work done in stretching the wire remains stored as statements.
elastic potential energy. I. It will be easier to compress this
Reason Potential energy stored in the wire is rubber than expand it. Force
(1/2) × stress × strain. II. Rubber does not return to its
original length after it is stretched.
4 Assertion Bulk modulus of an incompressible fluid
III. The rubber band will get heated, if it is stretched
is infinite.
and released.
Reason Density of incompressible fluid remains
constant. Which of the following statement(s) is/are correct
regarding the graph?
5 Assertion If radius of cylindrical wire is doubled, (a) Only III (b) II and III
then this wire can bear four times stress. (c) I and III (d) Only I
Reason By doubling the radius, area of
5 Which of the following statement(s) is/are correct?
cross-section will become four times.
I. Incompressible liquids have finite value of bulk
Statement based questions modulus of elasticity.
II. Compressibility is inverse of bulk modulus of
1 The length of an iron wire is L and area of elasticity.
cross-section is A. The increase in length is l on (a) Only I (b) Only II
applying the force F on its two ends. Which of the (c) Both I and II (d) None of these
following statement(s) is/are correct?
(a) Increase in length is inversely proportional to its 6 Which of the following statement(s) is/are correct?
length L. I. The materials having low value of Young’s
(b) Increase in length is directly proportional to area of modulus of elasticity are more ductile.
cross-section A. II. If Young’s modulus is less, then they can be easily
(c) Increase in length is inversely proportional to A. stretched as wires.
(d) Increase in length is directly proportional to Young’s (a) Only I
modulus.
(b) Only II
2 When a steel wire fixed at one end is pulled by a (c) Both I and II
constant force F at its other end, its length increases (d) None of the above
by l.
Elasticity 571

Codes
Match the columns A B C D
1 Match the following columns and choose the correct (a) r q p s
option from the codes given below. (b) q r p s
(c) p s r q
Column I Column II
(d) q r s p
(A) Longitudinal stress (p) volume changes
3 With regard to dependence of quantities given in
(B) Shear stress (q) shape changes
Columns I and II, match the following columns and
(C) Volumetric stress (r) volume does not changes choose the correct option from the codes given
(D) Tensile stress (s) shapes does not change below.
Codes Column I Column II
A B C D
(A) Young’s modulus of a (p) depends on
(a) p,s q,r p,s p,s substance temperature
(b) q,r p,s p,s p,s
(c) p,s p,s q,r p,s (B) Bulk modulus of a substance (q) depends on length
(d) p,s p,s p,s q,r (C) Modulus of rigidity of a (r) depends on area of
substance cross-section
2 Match the following columns and choose the correct
option from the codes given below. (D) Volume of a substance (s) depends on the
nature of material
Column I Column II
(A) Stress × Strain (p) J Codes
YA A B C D
(B) (q) Nm−1 (a) p,s q,s p,s p,q
l
(C) Yl 3 (r) Jm−3 (b) p,s p,s p,s p,q
Fl
(c) p,s p,s p,s p,q,r
(D) (s) m (d) p,q,r p,s p,s p,r
AY

(C) Medical entrances’ gallery


Collection of questions asked in NEET & various medical entrance exams

1 A wire of length L, area of cross-section A is hanging 4 Two wires are made of the same material and have
from a fixed support. The length of the wire changes the same volume. The first wire has cross-sectional
to L1 when mass M is suspended from its free end. area A and the second wire has cross-sectional area
The expression for Young’s modulus is [NEET 2020] 3A. If the length of the first wire is increased by ∆l
Mg (L1 − L ) MgL Mg L MgL1 on applying a force F, how much force is needed to
(a) (b) (c) (d)
AL AL1 A(L1 − L ) AL stretch the second wire by the same amount?
[NEET 2018]
2 The stress-strain curves are drawn for two different (a) 4F (b) 6F
materials X and Y. It is observed that, the ultimate (c) 9F (d) F
strength point and the fracture point are close to 5 If compressibility of a material is 4 × 10 −5 per atm,
each other for material X but are far apart for
pressure is 100 atm and volume is 100 cm3 , then
material Y. We can say that, materials X and Y are
find the value of ∆V.
likely to be (respectively) [NEET (Odisha) 2019]
(a) 0.4 cm 3 (b) 0.8 cm 3 [JIPMER 2018]
(a) ductile and brittle (b) brittle and ductile
(c) brittle and plastic (d) plastic and ductile (c) 0.6 cm 3 (d) 0.2 cm 3

3 Two wires of same material having radius in ratio 6 The bulk modulus of a spherical object is B. If it is
2 : 1 and lengths in ratio 1 : 2. If same force is subjected to uniform pressure p, the fractional
applied on them, then ratio of their change in length decrease in radius is [NEET 2017]
p B 3p p
will be [JIPMER 2019] (a) (b) (c) (d)
(a) 1 : 1 (b) 1 : 2 (c) 1 : 4 (d) 1 : 8 B 3p B 3B
572 OBJECTIVE Physics Vol. 1

7 The length of a metal wire is l1 when the tension in it 14 The Young’s modulus of a rope of 10 m length and
isT1 and is l 2 when the tension isT 2 . The natural length having diameter of 2 cm is 20 × 10 11 dyne cm −2 . If
of the wire is [AIIMS 2015, WBJEE 2015, Manipal 2015] the elongation produced in the rope is 1 cm, the
l1 + l 2 force applied on the rope is [J&K CET 2013 ]
(a) (b) l1 l 2
2 (a) 6.28 × 105 N (b) 6.28 × 104 N
l1T2 − l 2T1 l1T2 + l 2T1 (c) 6.28 × 104 dyne (d) 6.28 × 105 dyne
(c) (d)
T2 − T1 T1 + T2 15 A steel wire of length l and cross-section area A is
8 A load of 4 kg is suspended from a ceiling through a stretched by 1 cm under a given load. When the
steel wire of length 2 m and radius 2 mm. It is found same load is applied to another steel wire of double
that, the length of the wire increases by 0.031 mm its length and half of its cross-section area, the
as equilibrium is achieved. What would be the amount of stretching is [Kerala CEE 2013]
Young’s modulus of steel? (Take, g = 31. π ms −2 ) (a) 0.5 cm (b) 2 cm (c) 4 cm (d) 1.5 cm
[UK PMT 2015] (e) 2.5 cm
(a) 2.0 × 1011 Nm−2 (b) 2.82 × 1011 Nm−2 16 Theoretically, the value of Poisson’s ratio σ lies
−2 −2 between
(c) 0.20 × 10 Nm 11
(d) 0.028 × 10 Nm 11 [MP PMT 2013]
(a) 0 < σ < 1 (b) − 1 < σ < 0.5
9 Two wires of same length and same material but of (c) 0.2 < σ < 0.4 (d) −1 < σ < 1
radii r and 2r are stretched by forces F and f
respectively to produce equal elongation. The ratio F 17 A steel ring of radius r and cross-sectional area A is
to f is [Kerala CEE 2015]
fitted on to a wooden disc of radius R (R > r ). If
(a) 1 : 1 (b) 1 : 2 (c) 2 : 1 (d) 1 : 4
Young’s modulus of the steel isY, then the force
(e) 4 : 1 with which the steel ring is expanded is
[MP PMT 2013]
10 The elastic potential energy of a stretched wire is AYr Y (R − r ) AY (R − r ) AY (R − r )
(a) (b) (c) (d)
given by [CG PMT 2015] (R − r ) r R r
AL 2 AY 2
(a) U = l (b) U = l 18 A rubber cord L metre long and having A m 2 area of
2Y 2L cross-section is suspended vertically. If the wire
1  ALl  1 YL
(c) U =  l (d) U = ⋅ ⋅l extends 1 m under its own weight, then change in
2 Y  2 2A length (l ) is (Take, density of rubber = D kgm −3 and
11 A uniform cylindrical rod of length L and Young’s modulus of rubber = E Nm −2 ).
cross-sectional area A having Young’s modulusY is [UP CPMT 2013]
acted upon by forces as shown in figure. [CG PMT 2015] L2Dg L2Dg L2Dg
2 1
(a) (b) (c) (d) L
L L 2E E 4E
3 3
19 The density of a metal at normal pressure is ρ. Its
3F 2F density when it is subjected to an excess pressure p
is ρ′. If B is the bulk modulus of the metal, then the
L
ratio ρ′/ρ is [UP CPMT 2013]
The elongation produced in the rod is 1 1
3F L 3F L 8F L 5F L (a) 1 + p /B (b) 1 + B / p (c) (d)
(a) (b) (c) (d) (1 − B / p ) (1 − p /B )
8 AY 5 AY 3 AY 3 AY
20 In designing, a beam for its use to support a load.
12 For most of the material, Young’s modulus (Y ) and The depression at centre is proportional to (where,Y
rigidity modulus (G ) are related as [UK PMT 2014] is Young’s modulus). [AIIMS 2012]
Y 3 Y 1 1
(a) G = 3Y (b) G = (c) G = Y (d) G = (a) Y 2 (b) Y (c) (d)
3 2 8 Y Y2
13 The following four wires are made of the same 21 If to break a wire of 1 m length, minimum 40 kg-wt
material. Which of these will have the largest extension is required, then to break a wire of same material
when the same tension is applied? [NEET 2013] 6 m in length and double in radius, the breaking
(a) Length = 50 cm, diameter = 0.5 mm weight required will be
(b) Length = 100 cm, diameter = 1 mm [BCECE (Mains) 2012, Haryana PMT 2010]
(c) Length = 200 cm, diameter = 2 mm (a) 40 kg-wt (b) 80 kg-wt
(d) Length = 300 cm, diameter = 3 mm (c) 160 kg-wt (d) 320 kg-wt
Elasticity 573

1
22 If Poisson’s ratio σ for a material is − , then the cross-section 10 −3 m−2 . The change of temperature
2 required to produce the same elongation, if the
material is [BCECE (Mains) 2012] steel rod is heated is (Take, modulus of elasticity is
(a) elastic fatigue (b) incompressible 3 × 10 11 Nm −2 and coefficient of linear expansion of
(c) compressible (d) None of these . × 10 −5 /°C.)
steel is 11 [AFMC 2012]
23 Two block of masses of 1 kg and 2 kg are connected (a) 20°C (b) 15°C (c) 10°C (d) 0°C
by a metal wire going over a smooth pulley. The −2
25 A stress of 6 × 10 Nm 6
required for breaking a
40
breaking stress of metal is × 10 6 Nm −2 . What material. The density ρ of the material is
3π 3 × 10 3 kg m−3 . If the wire is to break under its own
should be the minimum radius of wire used, if it
should not break? (Take, g = 10 ms −2 ). [Manipal 2012] weight, the length of the wire made of that material
should be (Take, g = 10 ms −2 ) [Manipal 2011]
(a) 0.5 mm (b) 1 mm (c) 1.5 mm (d) 2 mm
(a) 20 m (b) 200 m
24 There is some change in length when a 33000 N (c) 100 m (d) 2000 m
tensile force is applied on a steel rod of area of

ANSWERS
l CHECK POINT 12.1
1. (d) 2. (d) 3. (d) 4. (a) 5. (b) 6. (d) 7. (c)

l CHECK POINT 12.2


1. (d) 2. (b) 3. (a) 4. (b) 5. (c) 6. (a) 7. (b)

l CHECK POINT 12.3


1. (a) 2. (d) 3. (a) 4. (c) 5. (b) 6. (b) 7. (b) 8. (a) 9. (b) 10. (c)
11. (c) 12. (b)

(A) Taking it together


1. (d) 2. (a) 3. (b) 4. (a) 5. (d) 6. (a) 7. (b) 8. (c) 9. (c) 10. (a)
11. (c) 12. (c) 13. (b) 14. (b) 15. (c) 16. (c) 17. (a) 18. (b) 19. (d) 20. (c)
21. (b) 22. (a) 23. (c) 24. (c) 25. (d) 26. (a) 27. (b) 28. (a) 29. (a) 30. (b)
31. (c) 32. (b) 33. (c) 34. (a) 35. (d) 36. (a) 37. (d) 38. (c) 39. (d) 40. (a)
41. (d) 42. (c) 43. (b) 44. (b) 45. (c) 46. (b) 47. (c) 48. (a)

(B) Medical entrance special format questions


l Assertion and reason
1. (d) 2. (d) 3. (c) 4. (b) 5. (d)
l Statement based questions
1. (c) 2. (d) 3. (d) 4. (a) 5. (b) 6. (c)
l Match the columns
1. (a) 2. (a) 3. (c)

(C) Medical entrances’ gallery


1. (c) 2. (b) 3. (d) 4. (c) 5. (a) 6. (d) 7. (c) 8. (a) 9. (d) 10. (b)
11. (c) 12. (b) 13. (a) 14. (b) 15. (c) 16. (b) 17. (d) 18. (a) 19. (d) 20. (c)
21. (c) 22. (b) 23. (c) 24. (c) 25. (b)
Hints & Explanations
l CHECK POINT 12.1 5 (c) ∆l =
FL
2 (d) This is the case of shear strain in which both shape and πr 2Y
size changes. 1
⇒ r2 ∝ (If F, L and ∆l are constants)
3 (d) On stretching a spiral spring, both types of strains are Y
1/ 2 1/ 2
produced, i.e. longitudinal and shear strain. r2 Y   7 × 1010 
=  1 = 
4 (a) Here, L = 10 cm = 10 × 10 −2 m r1 Y2   1.2 × 1010 
and F = 100 kgf = 100 × 9.8 N  7
1/ 2
Radius, r2 = 1.5 ×   = 1.145 mm
F 100 × 9.8  12
∴ Shearing stress = = = 9.8 × 10 4 Nm−2
L2
(10 × 10 −2 )2 ∴ d = 2.29 mm
5 (b) Here, original length, L = 2.5 m 6 (a) For a perfectly rigid body, Young’s modulus is infinite.
∆L 0.012 7 (b) Solids are least compressible, whereas gases are most
Strain = = 0.012% =
L 100 compressible.
∴ ∆L = Strain × L l CHECK POINT 12.3
0.012 dV dL
or ∆L = Extension = ×L 1 (a) We know that, = (1− 2σ )
100 V L
0.012 × 2.5
= = 3 × 10 −4 m = 0.3 mm If
dV
= 0 ⇒ 1 − 2σ = 0
100 V
6 (d) We have,V = l 3 1
or σ=
∆V 3∆l  2  2
= = 3  = 0.06 ∴ σ = + 0.50
V l  100 

7 (c) Here, L = 10 cm = 10 × 10 −2 m 2 (d) We have,


Y = 2η (1+ σ )
∆x = 0.25 cm = 0.25 × 10 −2 m
2.4η = 2η (1 + σ)
Displacement (∆x ) 0.25 × 10 −2
∴ Shearing strain = = = 0.025 1.2 = (1 + σ )
Distance (L ) 10 × 10 −2
Poisson’s ratio, σ = 0.2
CHECK POINT 12.2 Stress
l
3 (a) As,Y =
Strain
2 (b) Young’s modulus of material, 1
Linear stress ∴ Slope of strain and stress graph =
Y = Y
Longitudinal strain
Thus, A has maximum Young’s modulus of elasticity as its
If longitudinal strain is equal to unity, thenY = linear stress slope is minimum.
produced.
4 (c) Volume V( ) = Cross-sectional area (A) × Length (l ) orV = Al
3 (a) Elongation in rubber cord, Elongation y
Strain = =
L2dg (10 )2 × 1500 × 10 Original length l
l= = = 15 × 10 −4 m
2Y 2 × 5 × 10 8 Stress
Young’s modulus,Y =
FL Strain
4 (b) Elongation, ∆l =
AY 1
Work done,W = × Stress × Strain × Volume
1 2
∴ ∆l ∝ (If F, L andY are constants)
A 1
W = ×Y × (Strain )2 × Al
A 2 ∆l1 2
or =
A1 ∆l2 1  y
2
1 YA
= ×Y ×   × Al =   y 2
2  l  2 l 
 0.1 
⇒ A2 = A1  = 2 A1 = 2 × 4 = 8 mm
2
 0.05 ⇒ W ∝ y2
Elasticity 575

F
5 (b) The work done per unit volume, 2 (a) Y = or F = Y ⋅ A = 2 × 1011 × 10 −4 = 2 × 10 7 N
2
A
1 1  1 
u= ×Y × (Strain )2 = × 9 × 1011 ×   Stress 10 6
2 2  100  3 (b) Longitudinal strain = = 11 = 10 −5
Y 10
= 4.5 × 10 7 J Percentage increase in length = 10 −5 × 100 = 0.001%
1
6 (b) W = (Maximum stretching force) × (Elongation produced) 1
2 4 (a) Energy per unit volume, E = × Y × (Strain)2
2
1
= F ∆l 2E
2 ∴ Strain =
Y
Here, F = mg = 10 × 10, ∆l = 2 m
1

1
W = (10 × 10 ) × 2 = 100 J 5 (d) Energy per unit volume = × stress × strain
2 2
1 1
7 (b) Energy stored by the spring, = × (Y × strain) × strain = × Y × E 2
2 2
F2 T2
U= = 6 (a) Young’s modulus is the property of material. So, it is
2k 2k
independent of the length.
8 (a) The energy stored in the wire, Stress
7 (b) Y = ,Y = Stress for unit strain or ∆l = l
1 1
U = × F × l = × 200 × 10 −3 = 0.1 J Strain
2 2 Hence, stress will double the length of the wire.
1 F 2l 8 (c) Graph between applied force and extension will be
9 (b) U = F∆l =
2 2AY straight line because in elastic range.
U∝ 2
l
(Q F andY are constants) Applied force ∝ Extension
r But the graph given is parabolic in nature, so it will be
2 between extension and stored elastic energy.
U A  lA   rB 
2
 1 3
∴ =   ×   = (3) ×   = As, U = (1/ 2) kx 2 or U ∝ x 2
U B  lB   rA   2 4
∆p
YA(∆L )2 9 (c) As, |B | =
10 (c) Required work,W = ∆V /V
2L
∆V ∆p 10 5
2 × 1010 × 10 −6 × (10 −3 )2 ∴ = = = 8 × 10 −7
= V |B | 1.25 × 1011
2 × 50 × 10 −2
Lateral strain
= 2 × 10 −2 J 10 (a) Poisson' s ratio =
Longitudinal strain
1 YA(∆L )2
11 (c) Energy stored, U = × Lateral strain = 0.4 ×
0.05
2 L 100
1 2 × 1011 × 3 × 10 −6 × (1 × 10 −3 )2 So, it will be reduced by 0.02 %.
= ×
2 4 Stress S
11 (c) Strain = =
= 0.075 J Y Y
F mg 1
12 (b) We have, k = = Now, energy stored per unit volume = × stress × strain
x x 2
4 × 9.8 1 S S2
= = 19.6 × 10 2 = ×S× =
2 × 10 −2 2 Y 2Y
1 2 1 13 (b) The energy stored in the wire,
Work done = kx = × 19.6 × 10 2 × (0.05)2 = 2.4 J
2 2 1 1 1
U = × F × l = mg l = × 10 × 10 × 1 × 10 −3 =0.05 J
2 2 2
(A) Taking it together 14 (b) Y = 2η(1 + σ )
1 (d ) We know that, 3 1
⇒ 3η = 2η(1 + σ ) ⇒ σ = − 1=
Breaking force 2 2
Breaking stress = ...(i)
Area of cross - section Now, substituting the value of σ in the following expression
When length of the wire changes, area of cross-section Y = 3K (1 − 2σ )
remains same.
Y
Hence, breaking force will be same when length changes. ⇒ K= =∞
3(1 − 2σ )
576 OBJECTIVE Physics Vol. 1

15 (c) (Stress)1 = (Stress)2 For maximum displacement, area at which force applied
Maximum load does not depend on the length. should be minimum and vertical side of the block should be
∴The maximum load that can be supported by either part isw. maximum. This condition is satisfied when the block is kept
in position (Q).
Fl l
16 (c) We have, ∆l = or ∆l ∝ ...(i) 25 (d) Increase in tension of wire
AY A
As per the information given in the question, the longitudinal =YA α ∆θ
extension of any material of wire is small. = 2.2 × 1011 × 10 −2 × 10 −4 × 8 × 10 −6 × 5 = 8.8 N
It means that if ∆l is small, then A will be large, which
Fl
indicates that in order to have an appreciable change, the 26 (a) Elongation of the wire, ∆l =
material must be in the form of a long thin wire. AY
1 (Can also be applied for a spring)
17 (a) Work done,W = F l
2 ∴ ∆l ∝ l
∴Work done,W ∝ l (If F is constant) 7.5 60
= ⇒ l2 = 40 cm
W1 l1 l 1 5.0 l2
⇒ = = =
W2 l2 2l 2 ∴Length of original spring is (60 + 40) cm = 100 cm
18 (b) Force developed, F = YAα (∆θ ) x 100
Now, =
7.5 60
= 1011 × 10 −4 × 10 −5 × 100 = 10 4 N
∴ x = 12.5 cm
19 (d) We know that, with increase in temperature, length of a 27 (b) Maximum stress = Y × (Maximum strain)
wire changes as
L t = Lo ( 1+ α ∆T ) Mg
= 2 × 1011 × 10 −3 = 2 × 10 8 Nm −2
where, ∆T is change in the temperature, Lo is original length, A
α is coefficient of linear expansion and L t is length at ⇒ Maximum mass of this wire,
temperature T. 2 × 10 8 × 3 × 10 −6
M= = 60 kg
Now, we can write, ∆L = L t − L o = L o α ∆T 10
Stress F Lo FL o 1 FL
Young’s modulus (Y) = = = ∝ 28 (a) Elongation of the wire, ∆l =
Strain A × ∆L AL o α ∆T ∆T AY
1 1
As, Y ∝ ∴ ∆l ∝ 2 (IfY, L and F are constants)
∆T r
When temperature (∆T ) increases, henceY decreases. i.e. For the same load, thickest wire will show minimum
elongation. So, line OD represents the thickest wire.
20 (c) Q Bulk modulus of the material of the rubber,
∆p Fl ∆l  F 
|B | = =
100
= 10 6 atm = 1011 Nm−2 29 (a) ∆l = ⇒ = 
AY l  AY 
∆V /V 0.01/ 100
= 1012 dyne cm−2 1
Potential energy per unit volume =
× Stress × Strain
F 2
21 (b) Strain ∝ Stress ∝ 1 F F 1 1
A u= × × or u ∝ 2 or u ∝ 4
2 2 A AY A d
r 
2
A2  4 16
Ratio of strain = =  2 =   = u1  d 2 
4
A1  r1   1 1 ∴ =  =
16
u 2  d1  1
3.6 × 10 −9 N /Å
22 (a) Young’s modulus, Y = F mg (Alρ ) g
3 × 10 −10 m 30 (b) Maximum breaking stress = = = = Smax
A A A
3.6 × 10 −9 10 6
= = 1.2 × 1011 Nm −2 ∴ lmax =
Smax
= = 33.33 m
3 × 10 −10 × 10 −10 ρg (3 × 10 3 ) (10 )
23 (c) Elongation of the wire, w w′
31 (c) We have, S = =
Fl (2) l −5 πr 2 π ( 2r )2
∆l = = =l (Q 1 dyne = 10 N)
AY (2 × 10 −4 ) (10 4 ) ∴Load, w ′ = 4w
∴ New length, l′ = l + ∆l = l + l = 2l 32 (b) At middle, T = weight of half the length of steel bar
F /A L F
24 (c) η = ⇒ x= ×  L
x /L η A =   Aρg
 2
L
If η and F are constants, then x ∝ . T Lρ g
A ∴ Stress = =
A 2
Elasticity 577

mgl (lAρ ) gl wL
33. (c) Due to own weight, ∆l = = Elongation, ∆L =
2AY 2AY AY
l 2ρg (8)2 (1.5 × 10 3 ) (9.8) ⇒ Y =
L w
= =
2Y 2 × 5 × 10 6 A ∆L
1 20
= 9.4 × 10 −2 m = 9.4 cm Young’s modulus,Y = × = 2 × 1011 Nm −2
10 −6 10 −4
YA
34 (a) F =   ⋅ ∆l, i.e. F -∆l graph is a straight line with slope 41 (d) ∴ Since, the elongations of the two rods are equal
 l 
YA (∆ L ) b = (∆ L ) s
, i.e. slope is proportional toY.
l
F F
(Slope)A > (Slope)B
∴ YA > YB  FL   FL 
⇒   =  (Q Fb = Fs )
YA tan θ A tan 60 °  AY  b  AY  s
35 (d) = =
YB tan θ B tan 30 °
 L   L 
or   = 
3  AY  b  AY  s
= = 3 ⇒ YA = 3YB
1/ 3 ∴ The length of the steel rod,
36 (a) Tension, Tmax = m (g + a ) = (2000 ) (9.8 + 1.5) = 22600 N  AsYs   1.0 × 2.0 × 1011
Ls =   Lb =   (2) = 2 m
Tmax  AbYb   2.0 × 1.0 × 1011
Q Maximum stress =
Area 3L
Tmax 22600 42 (c) At length from lower end, tension in the wire,
∴ Area = = 4
Maximum stress 7 × 10 7 3 3w
T = suspended load + × weight of wire = w1 +
= 3.23 × 10 −4 m2 = 3.23 cm2 4 4
3w
37 (d) As stress is shown on X-axis and strain onY-axis. w1 +
T 4
∴ Stress = =
1 1 S S
So, we can say thatY = cot θ = =
tan θ slope
43 (b ) We know that, Young’s modulus,
So, elasticity of wireP is minimum and of wire R is maximum. Stress F /A F L
Y = = = ×
38 (c) The slope of straight line portion of strain-stress curve for Strain ∆L / L A ∆L
a given material represents its Young’s modulus. F L 4 FL
= × =
(i) Young’s modulus of the given material Y
( ) π (D / 2)2 ∆L π D 2∆L
= Slope of strain-stress curve 4 FL
⇒ D2 =
150 × 10 6 π ∆LY
Y = = 75 × 10 9 = 7.5 × 1010 Nm−2
0.002 4 FL
(ii) Yield strength of the given material ⇒ D=
π∆LY
= Maximum stress that material can sustain L
As F and are constants.
= 300 × 10 6 = 3 × 10 8 Nm−2 ∆L
39 (d ) Consider the diagram. 1
Hence, D ∝
A mass M is attached at the centre of the two cylindrical rods Y
as shown. As the mass is attached to both the rods, both rods Now, we can find ratio as,
will be elongated, but due to different elastic properties of Dcopper Yiron
material rubber changes shape also and bottom edge becomes =
Diron Ycopper
tapered to a tip at the centre.
44 (b) Wire is equivalent to a spring of spring
AY L
constant, k′ =
M M L
k′ and k are in series,
k
k′ k (AY / L ) k AYk
keq = = =
k′ + k  AY  AY + kL
Steel
  +k m
 L 
Rubber
m m (AY + kL )
∆L (4 − 1) × 10 −4 10 −4 T = 2π = 2π
40 (a) We have, = = keq AYk
w 80 − 20 20
578 OBJECTIVE Physics Vol. 1

 YA
45 (c) Let l be the natural length and k  =  be the force
 l  (B) Medical entrance special format
constant of wire. Then, questions
4  F
a =l + ...(i) Q F = k ∆l or ∆l =  l Assertion and reason
k  k
5 1 (d) Upto proportional limit, strain ∝ stress.
and b =l + ...(ii) A wire regains its original dimension only when load applied
k
is less than its elastic limit.
On solving Eqs. (i) and (ii), we get
1 2 (d) Young’s modulus is the property of material. It does not
or = (b − a ) and l = (5a − 4b ) depend on length of the wire.
k
1
Now, when T = 9 N 3 (c) Potential energy per unit volume is × stress × strain.
9 2
l′ = l + = (5a − 4b ) + 9 (b − a ) = (5b − 4a )
k 4 (b) Incompressible fluid is that which cannot be compressed
F (∆V = 0 ) by applying pressure on it. Thus, bulk modulus for
46 (b) Tension at distance x from end A, Tx = F − x these types of fluid is infinite.
L
x Restoring force
5 (d) Stress =
Area
F
Force F
B A S= = 2 …(i)
L Tx dx FL πr 2 πr
Total change in length, ∆L = ∫ = When the radius of cylindrical wire is doubled,
0 ES 2ES
∆L F
∴ Average strain = =
F S′ =
L 2ES π (2r )2
wL wL F S
47 (c) We have, ∆l1 = and ∆l2 = ⇒ S′ = = [From Eq. (i)]
(4πR 2 )Y πR 2Y 4πr 2 4
The elastic potential energy in the system, Now, A = πr 2 …(ii)
1 1 By doubling the radius, area of cross-section,
U = k1 (∆l1)2 + k2 (∆l2 )2
2 2 A′ = π (2r )2 = 4πr 2
2 2
1 Y (4πR )  wL 
2
1 Y (πR )  wL  2
⇒ A′ = 4A [From Eq. (ii)]
= × × + × ×
2 L  2 
 4πR Y  2 L  πR 2Y 
Therefore, area of cross-section will become four times.
 YA
Q k =  Statement based questions
 L
l

5w 2 L FL
= 1 (c) Elongation, ∆l =
8πR 2Y YA
1
48 (a ) Consider the diagram below ⇒ ∆l ∝
A
L L
B D C 3 (d) Elongation in the copper wire,
90°– θ 90°– θ
 Fl  500 × 8
(∆ l ) C =   =
x  AY  C 0.5 × 10 −4 × 1011

θ θ = 0.8 × 10 −3 m = 0.8 mm
O Elongation in the steel wire,
 Fl  500 × 4
m (∆ l ) S =   =
 AY  S 0.5 × 10 −4 × 2 × 1011
Hence, change in length, ∆L = BO + OC − (BD + DC )
= 2BO − 2BD (Q BO = OC, BD = DC ) = 0.2 × 10 −3 m = 0.2 mm
1
= 2 [BO − BD] = 2 [(x 2 + L2 )1/ 2 − L ] (∆ l ) S = (∆ l ) C
4
 x 2
1/ 2   1 x2  x2 and total elongation, ∆l = 0.8 + 0.2 = 1.0 mm
= 2L 1 + 2  − 1 ≈ 2L 1 + − 1 = (Q x << L)
 L    2 L2  L 4 (a) Energy loss ∝ Area of hysteresis loop
∆L x 2 / L x2 Also, energy loss will appear as heat.
∴ Strain = = = 2 For rubber, it is difficult to compress than expand.
2L 2L 2L
Elasticity 579

−∆p r1 2
5 (b) Bulk modulus, B = 3 (d) Given, ratio of radius of two wires, =
∆V /V r2 1
For incompressible fluids, ∆V = 0 l1 1
and ratio in their lengths, =
∴ B=∞ l2 2
Apart from this, it is true to say that compressibility is inverse We know that, Young’s modulus,
of bulk modulus of elasticity. Fl
Y =
A∆l
l Match the columns
Fl
1 (a) Volume changes in case of volumetric stress, longitudinal ⇒ ∆l =
AY
stress and tensile stress, whereas shape gets changed in shear
l
stress. ∆l ∝
A
Hence, A → p,s; B → q,r; C → p,s; D → p,s.
When same force is applied on them, then ratio of change in
m3
2 (a) (A) Stress × Strain = Nm−2 ⋅ = Jm−3 their lengths,
m3 ∆l1 l1 l2 lA
= / = 12
YA Nm−2m2 ∆l2 A1 A2 l2A1
(B) = = Nm−1
l m 2
∆l1 l1 r22 1  1 1
(C)Yl 3 = Nm−2m3 = Nm = J ⇒ = ⋅ = ⋅  =
∆l2 l2 r12 2  2 8
Fl Nm
(D) = =m 4 (c) According to the question,
AY m2 Nm−2
For wire 1 Area of cross-section = A1
Hence, A → r, B → q, C → p, D → s.
Force applied = F1
( , B, η) depends on the temperature
3 (c) Modulus of elasticity Y
and increase in length = ∆l
and nature of material. Volume of a substance depends on
From the relation of Young’s modulus of elasticity,
temperature, length and area of cross-section. Fl
Hence, A → p,s; B → p,s; C → p,s; D → p,q,r. Y =
A∆l
Substituting the values for wire 1 in the above relation, we
(C) Medical entrances’ gallery get
Fl
1 (c) Here, change in length, ∆L = (L1 − L ) ⇒ Y1 = 11 ...(i)
A1∆l
Area = A For wire 2 Area of cross-section = A2
Force, F = Mg Force applied = F2
Young’s modulus, and increase in length = ∆l
Normal stress Fl
Y = Similarly,Y2 = 2 2 ...(ii)
Longitudinal strain A2∆l
V
Mg Q Volume,V = Al or l =
(F / A) MgL A
⇒ Y = = A = Substituting the value of l in Eqs. (i) and (ii), we get
 ∆L   L1 − L  A(L1 − L )
    FV FV
 L   L  Y1 = 21 and Y2 = 22
A1 ∆l A2 ∆l
2 (b) The stress-strain curve for a material is shown in figure. As it is given that the wires are made up of same material,
i.e. Y1 = Y2
Elastic Ultimate
Stress D FV FV
B limit strength point
⇒ 1
= 2
A C
E Fracture point A12∆l A22∆l
Plastic behaviour
Elastic behaviour F1 A12 A2 1
⇒ = = = (Q A1 = A and A2 = 3A)
O F2 A22 9A2 9
Strain
or F2 = 9F1 = 9F (Given, F1 = F )
This curve specifies the behaviour of material.
−5
For the material X, as the distance between strength point 5 (a) Given, k = 4 × 10 per atm,
and fracture point is small, so it is brittle and will break easily p = 100 atm
on the application of some extra stress after point D.
and V = 100 cm 3
For materialY, as the distance between strength point and
fracture point is large, so it is a ductile material and can = 100 × 10 −6 m 3
withstand for some extra stress beyond point D.
580 OBJECTIVE Physics Vol. 1

1 F f
Q Compressibility = So, = 2 (Q l, ∆l andY are same)
| Bulk modulus of elasticity| πr12
πr2
1 ∆V F r12 F r2 F 1
k= = ⇒ = 2 ⇒ = 2 ⇒ =
∆p
V ∆ pV f r2 f 4r f 4
∆V
∴ ∆V = k (∆pV ) = 4 × 10 −5 (100 × 100 × 10 −6 ) 10 (b) Elastic potential energy of a stretched wire is given by
1
= 0.4 × 10 −6 m3 = 0.4 cm 3 U = × Stress × Strain × Volume
2
6 (d) The object is spherical and the bulk modulus is 1
represented by B. It is the ratio of normal stress to the = × Y × Strain × Strain × Volume
2
volumetric strain. 2
1 l
F /A  F = × Y ×   × AL
Hence, B= Q p =  2  L
∆V /V  A
AYl 2
∆V p ∆V p ∴ U=
⇒ = ⇒ = 2L
V B V B 2
∆V 11 (c) Elongation produced, if the part of length L of rod is
Here, p is applied pressure on the object and is volume 3
V stretched by a force of 3F, is given by
strain. 2
∆V ∆R 4 3F × L
=3 (Q V = πR 3 ) FL 3 = 2 FL
Fractional decrease in volume, ∆L1 = 1 1
=
V R 3 AY AY AY
Volume of the sphere decreases due to the decrease in its Elongation produced, if the remaining (1/3) part of length L of
radius, hence rod is stretched by a force of 2F, is given by
∆V 3∆R p ∆R p 1
= = ⇒ = 2F × L
V R B R 3B F2L2 3 2 FL
∆L2 = = =
Stress AY AY 3 AY
7 (c) As, Young’s modulus of elasticity,Y =
Strain Total elongation produced in the rod,
T1 × L FL 2 FL
⇒ Y = … (i) ∆L = ∆L1 + ∆L2 = 2 +
A × (l1 − L ) AY 3 AY
8 FL
where, L is the original length of the wire and also ∴ ∆L =
T2 × L 3 AY
Y = … (ii)
A × (l2 − L ) 12 (b) We know that,
Relation between Y and G
From Eqs. (i) and (ii), we get
Y Y
T1 L T2 L T l − Tl Y = 2 G (1+ σ ) ⇒ G = ⇒ G≅ (Q σ ∼
− 0.5)
= ⇒L = 21 12 2(1+ σ ) 3
A (l1 − L ) A (l2 − L ) T2 − T1
FL L  πd 2 
8 (a) Given, mass of load = 4 kg 13 (a) ∆L = or ∆L ∝ 2 Q A = 
AY d  4 
Length of wire, l = 2 m
L
Radius of wire (cross-section) = 2 mm = 2 × 10 −3 m Therefore, ∆L will be maximum for that wire for which 2 is
d
and change in length, ∆l = 0.031mm = 0.031 × 10 −3 m maximum. Here by checking options, we get option (a) has
Young’s modulus, maximum extension.
mg 4 × 31. π FL
−3 2
14 (b) Young’s modulus of a rope,Y =
Stress π ( 2 × 10 ) A∆l
Y = = A =
Strain  ∆l  (∆l / l ) Given, L = 10 m, A = πr 2 = π (1)2 = π
 
 l 
Y = 20 × 1011 dyne cm−2, ∆l = 1 cm

4 × 31 2
⇒ Y = × Y ⋅ A ⋅ ∆l 20 × 1011 × π × 1
π × 4 × 10 −6 0.031 × 10 −3 ⇒ F = ⇒F =
L 10 × 10 2
. ×2
31 F = 6.28 × 10 dyne
9
= × 10 9 = 2 × 1011 Nm −2
0.031
or F = 6.28 × 10 4 N
9 (d) Given, r1 = r , r2 = 2r FL
F l F l 15 (c) We know that, ∆l =
Now, we know that,Y = ⋅ = ⋅ AY
A ∆l πr 2 ∆l L
Here, ∆l ∝
A
Elasticity 581

∆l1 L1 A2 21 (c) Breaking force = Breaking stress × Area of cross-section of


According to question, = ×
∆l2 L2 A1 wire
∆l1 1 1 ∆l1 1 ∴ Breaking force ∝ r 2
⇒ = × ⇒ =
∆l2 2 2 ∆l2 4 If radius becomes doubled, then breaking force becomes
4 times, i.e. 40 × 4 = 160 kg-wt
⇒ ∆l2 = 4 ∆l1 ⇒ ∆l2 = 4 × 1= 4 cm
1
17 (d) Change in length of ring = 2πR − 2πr 22 (b) We know that, compressibility, C =
|B |
2πR − 2πr R − r dV dL
Longitudinal strain = = Also, = (1+ 2σ )
2πr r V L
F /A 1
Young’s modulus,Y = Given, σ = − , then
R −r 2
r dV
Fr YA (R − r ) = 0, i.e. volume is constant.
⇒ Y = ⇒ F = V
A (R − r ) r
or B = ∞, i.e. the material is incompressible.
18 (a) Volume of the rubber cord = AL 23 (c) Breaking stress = =
F 40
× 10 6 Nm −2
Weight of the rubber cord = ALDg A 3π
ALDg According to question,
∴ Stress = = LDg 40 3 × 10
A × 10 6 =
The weight of the rubber cord acts at the centre of gravity 3π πr 2
L 9
which is from the top. ⇒ r2 =
2 4 × 10 6
l 2l
∴ Strain = = ⇒ r = 1.5 × 10 −3 m
L /2 L
= 1.5 mm
Stress LDg L2Dg L2Dg Force
So, Young’s modulus, E = = = or l = 24 (c) Modulus of elasticity = ×
l
Strain 2l /L 2l 2E Area ∆l
19 (d) Magnitude of bulk modulus, 33000 l
3 × 1011 = ×
p pV 10 −3 ∆l
B= or dV = ∆l 33000 1
dV /V B = ×
M M
l 10 −3 3 × 1011
∴ ρ′ = =
V − dV V − (pV /B ) = 11 × 10 −5
ρ  M ∆l
=
M
= Q ρ =  Change in length, = α ∆T
V (1 − p /B ) (1 − p /B )  V l
11 × 10 −5 = 1.1 × 10 −5 × ∆T
ρ′ 1
∴ = ⇒ ∆T = 10°C
ρ (1 − p /B )
wl 25 (b) The minimum stress after which the wire breaks, i.e.
20 (c) We have, δ = Force mg
4Y bd 3 Stress = =
Area A
where, b and d are cross-sectional dimensions.
Vρg LAρg
w = = = Lρ g
A A
Given, stress = 6 × 10 6 Nm−2,

δ
ρ = 3 × 10 3 kgm−3 and g = 10 ms −2
Stress 6 × 10 6
l ⇒ L= =
ρg 3 × 10 3 × 10
1
or δ∝ = 2 × 10 2 = 200 m
Y
CHAPTER
13

Fluid Mechanics
The term ‘fluid’ refers to a substance which cannot withstand shear, so it has no
definite shape of its own and it has the ability to flow. Thus, the term fluid
applies to both liquids and gases. In this chapter, we will study some important
properties of fluids like pressure, viscosity, surface tension, etc.

PRESSURE
Normal force acting per unit area of a surface is called pressure. It is generally
represented by p.
F

Fig. 13.1
If surface area is A and normal force acting on the surface is F (shown in figure),
then pressure on the surface is
Normal force (F )
p=
Area (A)

∆F dF
If area is very small, then p = lim =
∆A → 0 ∆A dA
It is clear from the above formula that pressure acting on a surface depends upon
the normal force acting on the surface and its area. Thus, force of same magnitude
exerts different pressure on different area that is for the same force, if area is
lesser, then pressure will be greater.
SI unit of pressure is Nm −2 or pascal (Pa). Pressure is a scalar quantity and its
dimensional formula is [ML−1T −2 ] .

Pressure in a fluid
When a fluid (either liquid or gas) is at rest, it exerts a force perpendicular to any
surface in contact with it, such as a container wall or a body immersed in the
fluid.
While the fluid as a whole is at rest, the molecules or atoms that make up the
fluid are in motion. These molecules or atoms collide continuously with the
Fluid Mechanics 583

walls of container. Pressure of a fluid is due to molecules The weight of the air isw air = m air g = (72) (9.8) = 705.6 N
colliding with the container walls. (ii) The weight of water, w water = m water g = ρwater ⋅Vwater g
The pressure applied by a fluid can be defined as the = (103 )(60)(9.8) = 5.9 × 105 N
magnitude of the normal force (applied by fluid) acting per Pressure at the base due to this weight,
unit surface area. F (= w ) 5.9 × 105
p= = = 2.95 × 104 Nm−2
If the pressure is same at all points of a finite plane A 4×5
surface with area A, then
(iii) The downward force on the base is
F = (Air pressure) (Surface area)
p= ⊥
A = (1.013 × 105 ) (4.0 × 5.0) ~ 2.0 × 106 N

where, F⊥ is the normal force on the surface.


Unit of fluid pressure French scientist Pascal
Density of a liquid
contributed many researches in the field of fluid pressure. Density (ρ) of a liquid or any substance is defined as the
Thus, in his honour, SI unit of pressure is known as pascal mass per unit volume.
which is denoted by Pa. 1 Pa = 1 N m −2 Mass ∆m dm
∴ Density, ρ = = lim =
Volume ∆V → 0 ∆V dV
One unit used practically in meterology is the bar which is
equal to 10 5 Pa. 1 bar = 10 5 Pa SI unit of density is kgm −3 and its dimensions are [ML−3 T 0 ]
and CGS unit is gcm −3 with 1gcm −3 = 10 3 kg m −3 .
Atmospheric pressure (p atm )
It is the pressure of the earth’s atmosphere. This changes Relative density (RD)
with weather and elevation. Normal atmospheric pressure
In case of a liquid (or a substance), sometimes an another
at sea level (an average value) is 1.013 × 10 5 Pa. Thus,
term relative density (RD) or specific gravity is defined.
1 atm = 1.013 × 10 5 Pa It is the ratio of density of the liquid (or a substance) to the
Other practical units of atmospheric pressure are bar and density of water at 4°C.
torr (mm of Hg). 1 atm = 1.01 bar = 760 torr
Density of liquid or substance
Absolute pressure and gauge pressure Hence, RD =
Density of water at 4° C
The excess pressure above atmospheric pressure is usually
called gauge pressure and the total pressure is called Relative density is a pure ratio, so it has no units.
absolute pressure. Thus, Density of water at 4°C in CGS is 1 gcm −3 . Therefore,
Gauge pressure = Absolute pressure − Atmospheric pressure numerically the relative density and density of a substance
in CGS are equal. In SI unit, the density of water at 4°C
Absolute pressure is always greater than or equal to zero. is 1000 kgm −3 .
While gauge pressure can be negative also.
Example 13.1 An open container has dimensions of Example 13.2 Relative density of an oil is 0.8. Find the
4.0 m × 5.0 m and height of 3.0 m. absolute density of oil in CGS and SI units.
(i) Find the weight of the air in the container at 20 °C. Sol. Density of oil (in CGS) = (RD) gcm−3 = 0.8 gcm−3
(ii) What is the weight of an equal volume of water? Also, find and in SI unit = 800 kgm−3
pressure at the base of container due to this weight of water.
(iii) What is the total downward force on the base of the Density of a mixture of two or more liquids
container due to air pressure of 1.0 atm?
(Take, the densities of air and water as 1.2 kg/m 3 and
Case I Suppose two liquids of densities ρ1 and ρ 2 having
masses m1 and m 2 are mixed together. Then, the density of
103 kg/m 3, respectively.)
the mixture will be
Sol. It is given that, density of air, ρair = 1.2 kg /m3
Total mass
and density of water, ρwater = 103 kg /m3 ρ=
Total volume
(i) The volume of air in the container (m + m 2 ) (m1 + m 2 )
= (4.0)(5.0)(3.0) = 60 m3 = 1 =
(V1 + V2 )  m1 m 2 
∴ The mass of the air is m air = ρair ⋅Vair  + 
= (1.2) (60) = 72 kg
 ρ1 ρ 2 
584 OBJECTIVE Physics Vol. 1

2ρ1ρ 2 ρ′
1
If m1 = m 2, then ρ = . or =
ρ1 + ρ 2 ρ dp
1−
Case II If two liquids of densities ρ1 and ρ 2 having B
volumes V1 andV2 are mixed, then the density of the Here, dp = change in pressure
mixture, and B = bulk modulus of elasticity of the liquid.
Total mass m + m 2 ρ1V1 + ρ 2V2
ρ= = 1 = ρ
Total volume V1 + V2 V1 + V2 Therefore, ρ′ =
dp
ρ1 + ρ 2 1−
If V1 = V2, then ρ = B
2
Example 13.4 The thermal coefficient of volume expansion of
Example 13.3 Two liquids of densities ρ and 3ρ having a liquid is 5 × 10 −4 K −1. If its temperature is increased by
volumes 3V andV are mixed together. Find density of the 30° C, find the ratio of new density to the previous one.
mixture. Sol. Given, thermal coefficient of volume expansion,
Sol. Given, density of first liquid, ρ1 = ρ γ = 5 × 10−4 K −1
Density of second liquid, ρ2 = 3ρ Rise in temperature, ∆θ = 30° C
Volume of first liquid,V1 = 3V ρ
Volume of second liquid,V2 = V ∴ ρ′ =
1 + γ∆θ
∴ Density of the mixture,
ρ′ 1
ρV + ρ2V2 ⇒ =
ρm = 1 1 ρ 1 + γ∆θ
V1 + V2
1
ρ × 3V + 3 ρ × V 6ρ  3 = −~ 0.98
= = =  ρ 1 + (5 × 10−4 )(30)
3V + V 4  2
Example 13.5 The bulk modulus of a liquid is 8 × 10 9 Nm −2
Effect of temperature on density and its density is 11 g cm −3 . What will be the density of
When the temperature of a liquid is increased, the mass liquid under a pressure of 20, 000 Ncm −2 ?
remains the same while the volume is increased and Sol. Given, bulk modulus of liquid, B = 8 × 109 Nm−2
 1 Density of liquid, ρ = 11 g cm−3
hence, the density of the liquid decreases  as ρ ∝  .
 V
and pressure, p = 20000 Ncm −2
Density after increase of temperature by ∆θ ρ′
Thus, = ρ ρ
Initial density ρ ∴ New density of liquid, ρ′ = or
 dp   p
V V V ρ′ 1 1 −  1 − 
= = = or =  B  B
V ′ V + dV V + V γ ∆θ ρ 1 + γ ∆θ 11
=
20000
1−
Here, γ = thermal coefficient of volume expansion 8 × 109 × 10−4
and ∆θ = rise in temperature. 11 × 40 440
= = gcm−3
ρ 39 39
Therefore, ρ′ =
1 + γ ∆θ Variation of fluid pressure with depth
Effect of pressure on density Consider a fluid of density ρ is kept at rest, in a cylindrical
vessel of height h as shown in figure.
As pressure is increased, volume decreases and hence,
1
density will increase. Thus, ρ ∝
V Density
ρ Cylindrical
Density after increase of pressure ρ′
∴ = h
vessel
Initial density ρ
V V V
= = =
V ′ V + dV  dp 
V −  V Fig. 13.2
B
Fluid Mechanics 585

Pressure applied by liquid at the bottom of the container is (iii) The pressure exerted by a liquid depends only on
the height of fluid column and is independent of the
p = hρ g ...(i) shape of the containing vessel.
From the above formula, we can say that pressure applied
by a liquid column is proportional to height of liquid column.
hA hB hC
If the container is open, then atmospheric pressure will
also work on the top surface, so total pressure on the A B C
bottom will be equal to
Fig. 13.5 The three vessels A, B and C containing
p total = p 0 (atmospheric pressure ) + ρgh different amounts of liquids

From Eq. (i), we can also find the difference of pressure If h A = hB = hC , then p A = p B = p C
between any two points in the liquid column. (iv) Consider following shapes of vessels
p0 p0 p0
h1
A
h2 h ρ ρ ρ
h
B
( x) ( y) (z)
Fig. 13.6
Fig. 13.3 Pressure at the base of each vessel,
Consider two points A and B in a liquid column situated at p x = p y = p z = p 0 + ρgh but weight, w x ≠ w y ≠ w z
h1 and h 2 respectively, below free surface of liquid. where, ρ = density of liquid in each vessel,
Pressure due to liquid column at point A, p A = h1ρg h = height of liquid in each vessel
Pressure due to liquid column at point B, p B = h 2 ρg and p 0 = atmospheric pressure.
∴ Pressure difference (v) In the figure, a block of mass m floats over a fluid surface
p B − p A = h 2ρg − h1ρg = (h 2 − h1 ) ρg p0
Block of
mass m
p B − p A = h ρg
ρ
Hence, the pressure difference depends on the vertical h
height ( i.e. distance between points A and B), density of
the fluid and the acceleration due to gravity. Fig. 13.7
According to Eq. (i), pressure increases linearly with If ρ = density of the liquid,
depth, if ρ and g are uniform. A = area of the block, then
A graph between p and h is shown below Pressure at the base of the vessel is
p mg
p = p 0 + ρgh +
A
p = p0 + ρgh
Example 13.6 Find the pressure exerted below a column of
p0 water, open to the atmosphere, at depth
h (i) 10 m (ii) 30 m
(Take, density of water = 1 × 10 3 kgm −3, g = 10 ms −2 )
Fig. 13.4 Graph between pressure and height
Sol. (i) Pressure at depth of 10 m,
Important points related to fluid pressure p = p a + ρgh
Important points related to fluid pressure are given below = 1.013 × 105 Pa + (1 × 103 kgm−3 )(10 ms−2 )(10 m)
(i) At a point in the liquid column, the pressure applied = 1.013 × 105 Pa + 1 × 105 Pa
on it is same in all directions. = 2.013 × 105 Pa
(ii) In a liquid, pressure will be same at all points at the
≈ 2 atm
same level.
586 OBJECTIVE Physics Vol. 1

(ii) Pressure at depth of 30 m, Substituting these values in Eq. (i), we get


p = p a + ρgh F1 = (1.01 × 105 + 103 × 10 × 0.1) × 10−3
= 1.013 × 105 Pa + (1 × 103 kgm−3 )(10 ms−2 )(30 m)
or F1 = 102 N (downwards)
= 1.013 × 105 Pa + 3 × 105 Pa (ii) Force exerted by atmosphere on water, F 2 = (p 0 )A2
= 4.013 × 105 Pa ≈ 4 atm Here, A2 = area of top = 30 cm2 = 3 × 10–3 m2
Example 13.7 For the arrangement shown in the figure, what ∴ F 2 = (1.01 × 105 )(3 × 10−3 ) = 303 N (downwards)
is the density of oil?
Force exerted by bottom on the water,
C F 3 = − F1 or F 3 = 102 N (upwards)
d = 12.3 mm Weight of water, w = (Volume)(Density)(g)
Oil = (10−3 )(103 )(10) = 10 N (downwards)
l = 135 mm
Let F be the force exerted by side walls on the water
B (upwards). Then, for equilibrium of water (FBD),
Net upward force = Net downward force
Water or F + F3 = F2 + w
Sol. According to the figure, for the equilibrium of liquid, ∴ F = F 2 + w − F 3 = 303 + 10 − 102
or F = 211 N (upwards)
Net downward force = Net upward force
F2
p 0 + ρw gl = p 0 + ρoil (l + d ) g
ρwl 1000 × (135)
⇒ ρoil = = = 916 kgm −3
l + d (135 + 12.3) w

Example 13.8 A curved glass vessel full of water upto a


height of 10 cm has a bottom of area 10 cm 2 , top of area
F
30 cm 2 and volume 1 L. F3
(i) Find the force exerted by the water on the bottom. (FBD of water)
(ii) Find the resultant force exerted by the sides of the glass vessel (iii) If the air inside the jar is completely pumped out,
on the water. F1 = ( ρgh ) A1 (Q p 0 = 0)
(iii) If the glass vessel is covered by a jar and the air inside the jar −3
= (10 )(10)(0.1)(10 )
3
is completely pumped out, what will be the answers to parts (i)
= 1 N (downwards)
and (ii)?
In this case, F 2 = 0
(iv) If a glass vessel of different shape is used provided the height,
the bottom area and the volume are unchanged, will the and F 3 =1 N (upwards)
answers of parts (i) and (ii) change? ∴ F = F 2 + w − F 3 = 0 + 10 − 1
(Take, g = 10 ms −2, density of water = 103 kgm −3 and = 9 N (upwards)
atmospheric pressure = 1.01 × 105 Nm −2) (iv) No, the answers will remain the same. Because the
answers depend upon p 0, ρ, g, h, A1 and A2.
Sol. (i) Force exerted by the water on the bottom,
F1 = (p 0 + ρgh ) A1 …(i) Pascal’s law
Here, p 0 = atmospheric pressure = 1.01 × 105 Nm−2 It states that, “pressure applied to an enclosed fluid is
ρ = density of water = 103 kgm−3 transmitted undiminished to every portion of the fluid and
g = 10 ms−2 , h = 10 cm = 0.1 m the walls of the containing vessel.”
and A1 = area of base = 10 cm2 = 10−3 m2 F
30 cm2 = A2
(Top)
A
h = 10 cm

Volume = 1 L
10 cm2 = A1
(Bottom) Fig. 13.8 Pressure applied to an enclosed fluid
Fluid Mechanics 587

Consider an example in which a flask fitted with a piston and ρ = 750 kgm−3
is filled with a liquid. Let an external force F is applied on 20 × 9.8 F
the piston. If the cross-sectional area of the piston is A, the Thus, 1.5 × 750 × 9.8 = −
π × (5 × 10−2 )2 π × (17.5 × 10−2 )2
pressure just below the piston is increased by F/A. By
Pascal’s law, the pressure applied by piston will be which gives, F = 13
. × 103 N
transmitted equally at all points in the flask. If we make Note Atmospheric pressure is common to both pistons and has been
some holes in the flask, then the liquid from all the holes ignored.
will emerge out with same intensity. Example 13.10 Two pistons of a hydraulic machine have
Hydraulic lift diameters 20 cm and 2 cm. Find the force exerted on the
larger piston when 50 kg-wt is placed on the smaller piston.
A well known application of Pascal’s law is the hydraulic When the smaller piston moves in through 5 cm, by what
lift used to support or lift heavy objects. It is schematically distance, the other piston moves out?
illustrated in figure.
Sol. For smaller piston, area, A1 = π × (1)2
F1 F2
Car For larger piston, area, A 2 = π × (10)2
A1
∴ Force exerted on the larger piston,
A2
A2
F2 = × F1
A1
Fig. 13.9 Hydraulic lift π(10) 2
= × 50 × 9.8
A piston with small cross-section area A1 exerts a force F1 π(1) 2
on the surface of a liquid such as oil. The applied pressure = 100 × 50 × 9.8
p = F1 /A1 is transmitted through the connecting pipe to a −~ 5 × 104N
larger piston of area A2 . It pushes larger piston with force
This is the force exerted on the larger piston. The liquids are
F2 . The applied pressure is the same in both cylinders, so
considered incompressible. Therefore, volume covered by
F1 F2 A
= or F2 = 2 ⋅ F1 movement of smaller piston inwards equal to the outward
A1 A2 A1 movement of larger piston.
Now, since A2 >> A1, therefore F2 >> F1. Thus, hydraulic ∴ L1A1 = L 2A 2
lift is a force multiplying device with a multiplication  A1 
factor equal to the ratio of the areas of the two pistons. ⇒ L2 =   L1
A 2
Dentist’s chairs, car lifts and jacks, hydraulic elevators and
hydraulic brakes all use this principle. π (1 cm)2
= × 5 cm
π (10 cm)2
Example 13.9 Figure shows a hydraulic press with the larger
piston of diameter 35 cm at a height of 1.5 m relative to the 1
= × 5 cm
smaller piston of diameter 10 cm. The mass on the smaller 100
piston is 20 kg. What is the force exerted on the load by the = 0.05 cm
larger piston? The density of oil in the press is 750 kgm −3 .
So, the distance moved out by the larger piston is 0.05 cm.
(Take, g = 9 . 8 ms −2 )
Measurement of pressure
Pressure can be measured by using following two devices
1.5 m
20 kg (i) Barometer It is a device used to measure
atmospheric pressure. In principle, any liquid can be
used to fill the barometer, but mercury is the
substance of choice because its high density makes
possible an instrument of reasonable size.
20 × 9.8
Sol. Pressure on the smaller piston = −2 2
Nm−2 A barometer is an inverted evacuated tube, put over
π × (5 × 10 )
a mercury volume. Outside pressure pushes mercury
F
Pressure on the larger piston = −2 2
N m −2 into tube till the weight of liquid column equalises
π × (17.5 × 10 ) the force due to external pressure. In given diagram,
The difference between the two pressures = hρg in equilibrium,
where, h =1.5 m p1 = p 2
588 OBJECTIVE Physics Vol. 1

Here, p 1 = atmospheric pressure (p 0 ) Example 13.11 What will be the length of mercury column in
a barometer tube, when the atmospheric pressure is 76 cm of
and p 2 = 0 + ρgh = ρgh
mercury and the tube is inclined at an angle of 30° with the
horizontal direction?
Vacuum
(p = 0) Sol. Here, h = 76 cm, θ = 30°
If l is the length of mercury column in a barometer tube, then
h
1
h = sin 30°
l
2 76 cm 1
⇒ =
l 2
⇒ l = 2 × 76 = 152 cm
Example 13.12 A manometer tube contains a liquid of density
Fig. 13.10 Barometer 3 × 10 3 kg m −3 . When connected to a vessel containing a
gas, the liquid level in the other arm of the tube is higher by
where, ρ = density of mercury 20 cm. When connected to another sample of enclosed gas,
the liquid level in the other arm of the manometer tube falls
∴ p 0 = ρgh 8 cm below the liquid level in the first arm. Which of the
Thus, the mercury barometer reads the atmospheric two samples exerts more pressure and by what amount?
pressure (p 0 ) directly from the height of the mercury Sol. For sample 1
column. Difference in level of liquids, h1 = 20 cm = 0.2 m
e.g. If the height of mercury in a barometer is Pressure of the gas in the left arm, p1 = p a + ρgh1 …(i)
760 mm, then atmospheric pressure will be
pa
p 0 = ρgh
= (13.6 × 10 3 )(9.8)(0.760)
= 1.01 × 10 5 Nm −2
(ii) Manometer It is a device used to measure the
20 cm
pressure of a gas inside a container.
Gas
1
p0 p1
Gas
h
1 2

Hg
For sample 2
In this case, level of the liquid in the left arm is higher than
Fig. 13.11 Manometer
that in the right arm by 8 cm.
∴ Atmospheric pressure p a is greater than the pressure
The U-shaped tube often contains mercury. Let 1 exerted by the sample, i.e.
and 2 are points on same horizontal level, then
p a = p 2 + ρgh 2 ⇒ p 2 = p a − ρgh 2 …(ii)
p1 = p 2 Comparing Eqs. (i) and (ii), it is clear that p1 > p 2.
Here, p 1 = pressure of the gas in the container (p )
and p 2 = atmospheric pressure (p 0 ) + ρgh
∴ p = p 0 + ρgh
This can also be written as Gas
p − p 0 = gauge pressure = ρgh 2
p2
Here, ρ is the density of the liquid used in U-tube. 8 cm
Thus, by measuring h, we can find absolute (or pa
gauge) pressure in the vessel.
Fluid Mechanics 589

Therefore, the gas in sample 1 exerts greater pressure than Special cases Three possibilities may now arise
that in sample 2.
(i) If ρ S < ρL , then in this condition,
The difference in the two pressures is the upthrust applied by the liquid
p1 − p 2 = (p a + ρgh1) − (p a − ρgh 2 ) will be greater than the weight of
= ρg (h1 + h 2 ) = ρg (28 cm) the body. That means, if the body
= (3 × 103 kg m−3 ) × (9.8 ms−2 )(0.28 m) is completely immersed in liquid,
= 8.23 × 103 Pa ≈ 8 kPa it will experience a net upward
force. When released, the body
comes up to the fluid surface till Fig. 13.13
Archimedes’ principle the upthrust becomes equal to the
If a heavy object is immersed in water, it seems to weigh weight of the body, at this point, the body floats
less than when it is in air. This is because the water exerts partially immersed in the fluid.
a net upward normal force called buoyant force. (ii) If ρ S = ρL , then w app = 0 in this condition, upthrust
The Archimedes principle gives the magnitude of buoyant of the liquid balances the weight of the body. The
force on a body. It states that, “a body wholly or partially body floats completely submerged just below the
submerged in a fluid experiences an upward force which surface of the fluid.
is equal to the weight of the displaced fluid.”
Thus, the magnitude of buoyant force (F ) which is also
called upthrust is given by
F = Vi ρL g

Fig. 13.14

(iii) If ρ S > ρL , then in this condition, the upthrust of the


liquid is less than the weight of the liquid, i.e. it is
not sufficient to balance the weight of the body, so
the body sinks.
Fig. 13.12 Net upward force on immersed object

where, Vi = immersed volume of solid,


ρL = density of liquid
and g = acceleration due to gravity.
The upthrust act vertically upwards through the centre of
gravity of displaced fluid. Fig. 13.15

Example 13.13 An ornament weighing 50 g in air weights


Apparent weight of a body inside a liquid only 46 g in water. Assuming that some copper is mixed
If a body is completely immersed in a liquid, its effective with gold to prepare the ornament. Find the amount of
weight gets decreased. The decrease in its weight is equal copper in it. Specific gravity of gold is 20 and that of copper
to the upthrust on the body. is 10.
Sol. Let m be the mass of the copper in ornament.
Hence, apparent weight of body = w app = w actual − upthrust
Then, mass of gold in it is (50 − m ).
or w app = Vρ S g − VρL g m  Mass 
Volume of copper, V1 = Q Volume = 
Here, V = total volume of the body, 10  Density 
ρ S = density of body and ρL = density of liquid. 50 − m
and volume of gold, V2 =
20
Thus, w app = Vg (ρ S − ρL ) When immersed in water (ρw = 1 gcm−3 ),
 ρ  decrease in weight = upthrust
or w app = V ρ S g 1 − L  ∴ (50 − 46)g = V
( 1 + V2 ) ρw g
 ρS 
m 50 − m
or 4= + or 80 = 2m + 50 − m
 ρ  10 20
∴ w app = w actual 1 − L 
 ρS  ∴ m = 30 g
590 OBJECTIVE Physics Vol. 1

Example 13.14 Density of ice is 900 kg m −3 . A piece of ice Buoyant force in accelerating fluids
is floating in water (of density 1000 kg m −3 ). Find the
If a body is dipped inside a liquid of density ρL placed in
fraction of volume of the piece of ice outside the water.
an elevator moving upward with an acceleration a as
Sol. Let V be the total volume andVi the volume of ice piece shown in figure.
immersed in water. For equilibrium of ice piece,
weight = upthrust
∴ Vρi g = Vi ρw g …(i) a
Here, ρi = density of ice = 900 kgm−3
and ρw = density of water = 1000 kgm−3 Elevator

Substituting in Eq. (i), we get


Vi ρi 900
= = = 0.9
V ρw 1000 Fig. 13.16 A body dipped in a liquid is placed inside
i.e. The fraction of volume outside the water, f = 1 − 0.9 = 0.1 an elevator

Example 13.15 A piece of ice is floating in a glass vessel Now, consider the force acting on the liquid replaced by
filled with water. How will the level of water in the vessel the body. For upward motion of the replaced liquid, we
changes when the ice melts? can write, F − w = ma
Sol. Let m be the mass of ice piece floating in water.
In equilibrium, weight of ice piece = upthrust
m
or mg = Vi ρw g or Vi = …(i) a
ρw w
Here,Vi is the volume of ice piece immersed in water.
When the ice melts, let V be the volume of water formed by
F
m mass of ice.
m Fig. 13.17 Force acting on the displaced liquid
Then, V= …(ii)
ρw
where, w = weight of the displaced liquid
From Eqs. (i) and (ii), we see that, Vi = V
Hence, the level will not change. F = buoyant force acting on the body
∴ F = w + ma = m (g + a ) = ρLV (g + a )
Example 13.16 A piece of ice having a stone frozen in it,
floats in a glass vessel filled with water. How will the level F = VρL g eff
of water in the vessel changes when the ice melts?
Here, g eff = | g + a |
Sol. Let m 1 = mass of ice, m 2 = mass of stone,
e.g. If the lift is moving upwards with an acceleration a,
ρS = density of stone and ρw = density of water.
the value of g eff is g + a and if it is moving downwards
In equilibrium, when the piece of ice floats in water, with acceleration a, the g eff is g − a. In a freely falling
weight of (ice + stone) = upthrust lift, g eff is zero (as a = g) and hence, net buoyant force is
(m1 + m 2 )g = Vi ρw g zero. This is why, in a freely falling vessel filled with
m m some liquid, the air bubbles do not rise up (which
∴ Vi = 1 + 2 …(i)
ρw ρw otherwise move up due to buoyant force).
Here, Vi = volume of ice immersed. Example 13.17 The tension in a string holding a solid block
When the ice melts, m1 mass of ice converts into water and below the surface of a liquid (of density greater than that of
stone of mass m 2 is completely submerged. solid) as shown in figure is T 0 when the system is at rest.
m
Volume of water formed by m1 mass of ice,V1 = 1 What will be the tension T in the string, if the system has an
ρw upward acceleration a?
Volume of stone (which is also equal to the volume of water
displaced), a
m
V2 = 2
ρS
Since, ρS > ρw , soV1 + V2 < Vi or the level of water will
decrease.
Fluid Mechanics 591

F F′
Sol. Let m be the mass of block.
Initially for the equilibrium of block, F = T0 + mg …(i)
a
Here, F is the upthrust on the block.
When the lift is accelerated upwards, g eff becomes g + a
instead of g.
g + a T0 + mg T + mg
Hence, F′ =F   …(ii)
 g  Solving Eqs. (i), (ii) and (iii), we get
From Newton’s second law,  a
T = T0 1 + 
F ′ − T − mg = ma …(iii)  g

CHECK POINT 13.1


1. Pins and nails are made to have pointed end because 7. The U-tube in figure contains two different liquids in static
(a) it transmits very small pressure equilibrium, water in the right arm and oil of unknown
(b) it transmits a large pressure density ρ x in the left. If l = 135 mm and
(c) it provide a large area d = 15 mm. Density of the oil is
(d) None of the above
2. The two thigh bones each of cross-sectional area 15 cm 2 d
support the upper part of a person of mass 70 kg. The
pressure sustained by these thigh bones is l
(a) 2.5 × 105 N m −2 (b) 1.33 × 105 N m −2
(c) 4.66 × 105 × 105 Nm −2 (d) 2.33 × 105 N m −2
Water
3. Which of the following is correct? (a) 1000 kgm −3
(b) 920 kg m−3
(a) Gauge pressure = Absolute pressure −3
+ Atmospheric pressure (c) 895 kg m (d) 900 kgm−3
(b) Absolute pressure = Gauge pressure
8. Increase in pressure at one point of the enclosed liquid in
− Atmospheric pressure
equilibrium at rest is transmitted equally to all other points.
(c) Gauge pressure = Absolute pressure
This is as per
− Atmospheric pressure
(d) Absolute pressure = Atmospheric pressure (a) impulse
− Gauge pressure (b) Pascal’s law
(c) conservation of momentum
4. At a depth of 500 m in an ocean, what is the absolute (d) None of the above
pressure? Given that the density of sea water is
9. In a vehicle lifter, the enclosed gas exerts a force F on a
1.03 × 10 3 kg m −3 and g = 10 ms −2. small piston having a diameter of 8 cm. This pressure is
(a) 40 atm (b) 52 atm transmitted to a second piston of diameter 24 cm. If the
(c) 32 atm (d) 62 atm mass of the vehicle to be lifted is 1400 kg, then value of F is
5. Four vessels A , B , C and D have different shapes and hold (a) 1200 N (b) 1800 N
(c) 1600 N (d) 700 N
different amount of water. Which of the following is
correct? 10. What will be the length of mercury column in a barometer
A B C D tube when the atmospheric pressure is x cm of mercury and
the tube is inclined at an angle φ with the vertical direction?
x x
(a) (b)
sin φ cos φ
E G x
F (c) (d) x
H tan φ
(a) pE > pF > pG > pH (b) pE < pF < pG < pH 11. An iron casting containing a number of cavities weights
(c) pE = pF = pG = pH (d) pE = pF > pG = pH 6000 N in air and 4000 N in water. What is the volume of
6. Two liquids of densities 2ρ and ρ having their volumes in the cavities in the casting? Density of iron is 7.87 gcm −3.
(Take, g = 9.8 ms −2 and density of water
the ratio 3 : 2 are mixed together. Density of the mixture
will be = 103 kgm −3)
2ρ ρ 8ρ 4ρ (a) 0.16 m 3 (b) 0.2 m 3
(a) (b) (c) (d)
3 2 5 5 (c) 0.12 m 3 (d) 0.14 m 3
592 OBJECTIVE Physics Vol. 1

FLOW OF FLUIDS Turbulent flow


Till now we have studied about the fluids at rest. In this In rivers and canals, where speed of water is quite high or
section, we will study fluids in motion. When a fluid is in the boundary surfaces cause abrupt changes in velocity of
flow, its motion can either be smooth or irregular the flow, then the flow becomes irregular. Such flow of
depending on its velocity of flow. liquid is known as turbulent flow.
Thus, the flow of fluid in which velocity of all particles
Steady flow crossing a given point is not same and the motion of the
Consider a liquid flowing in a tube as shown in the figure fluid becomes irregular or disordered. This is called
given below. turbulent flow.
A few examples of turbulent flow are
v3
v1
(i) a jet of air striking a flat plate.
2 3
1
(ii) the smoke rising from a burning stock of wood.
v2 (iii) when a fast flowing stream encounters rocks, foamy
whirlpool like regions are formed.
Fig. 13.18 Steady flow of liquid through a tube

If the velocity of fluid particles at any point does not vary


Reynolds number
with time, the flow is said to be steady. Steady flow is also Reynolds number (R e ) is a dimensionless number, whose
called streamlined or laminar flow. The velocity at value gives us an idea whether the flow would be laminar
different points may be different. Hence, in the figure, (streamlined) or turbulent.
Inertial force
we can say that velocity of fluid particles at different Re =
points 1, 2 and 3 remains same with time. Viscous force
i.e. v 1 = constant, v 2 = constant, v 3 = constant, Thus, R e represents the ratio of inertial force of moving
but v1 ≠ v 2 ≠ v 3 fluid to viscous force offered by the fluid.
Streamlines The path followed by a fluid particle in The expression for R e is
steady flow is called streamline. Velocity of fluid particle ρvD
at any point of the streamline is along tangent to the curve Re =
at that point. η
Properties of streamline are given below Here, ρ = density of fluid,
(i) In streamline flow, no two streamlines can cross v = speed of fluid,
each other. If they do so the particles of the liquid at D = diameter of tube
the point of intersection will have two different and η = viscosity of fluid (later on it is discussed in detail)
directions for their flow, which will decrease the
e < 1000, flow is streamline or laminar.
● For R

steady nature of the liquid flow.


e → 1000 to 2000, flow is unsteady.
● For R
(ii) The greater is the crowding of streamlines at a place
e > 2000, flow is turbulent.
● For R
greater is the velocity of the liquid particles at that
place and vice-versa. Example 13.18 Water is flowing in a pipe of diameter 6 cm
(iii) Streamline flow is possible only if the liquid velocity with an average velocity 7.5 cms −1 and its density is
does not exceed a limiting value called critical 10 3 kg m −3 . What is the nature of flow? Given, coefficient
velocity. of viscosity of water is 10 −3 kgm −1 s −1.
(iv) The bundle of streamlines forming a tubular region Sol. Reynold’s number for the given situation is given as
is called a tube of flow. ρvD
Re =
η
Here, density, ρ = 103kgm−3
Streamlines

Coefficient of viscosity, η = 10−3 kgm−1s−1


Average velocity of water, v = 7.5 cms −1
Tube of flow
= 0.075 ms−1
Fig. 13.19 Tube of flow formed by streamlines
Diameter of pipe, D = 6 cm = 0.06 m
Fluid Mechanics 593

103 × 0.075 × 0.06 Given, r1 = 1 cm = 1 × 10−2 m, r2 = 2 cm = 2 × 10−2 m,


Hence, Re =
10−3 v1 = 2 ms−1
= 10 × 0.0045 = 4500
6
Substituting the above values, we get
R e > 2000 2
 1.0 × 10−2 
Q
−1
Therefore, the flow is turbulent. v2 =   (2) or v 2 = 0.5 ms
 2.0 × 10−2 

Equation of continuity BERNOULLI’S THEOREM


When an incompressible and non-viscous liquid flows in a
streamlined motion through a tube of non-uniform Bernoulli’s theorem is based on the law of conservation of
cross-section, then mass flow rate is same at every section energy and applied to ideal fluids.
of the tube. It states that the sum of pressure energy per unit volume,
Q
kinetic energy per unit volume and potential energy per
P unit volume of an incompressible, non-viscous fluid in a
v2 streamlined irrotational flow remains constant at every
v1
A2 cross-section throughout the liquid flow.
A1 Mathematically, it can be expressed as
Fig. 13.20 A liquid is flowing through a tube of
1 2
non-uniform cross-section p+ ρv + ρgh = constant
2
Thus, A1v 1 = A2v 2
where, p represents the pressure energy per unit volume
1 1
or Av = constant or v ∝ (or pressure), ρv 2 the kinetic energy per unit volume
A 2
This is called equation of continuity which is basically and ρgh the potential energy per unit volume.
derived from the law of conservation of mass. Proof Consider an ideal fluid having streamline flow through
We can generalise the continuity equation for the case in a pipe of varying area of cross-section as shown in figure.
which the fluid is not incompressible. If ρ1 and ρ 2 are the
B
densities at sections 1 and 2, then
a2
ρ1A1v 1 = ρ 2A2v 2 p2

A
So, this is the continuity equation for an incompressible v2
fluid. a1
dV p1
Note The product Av is the volume flow rate , the rate at which h2
v1
dt
volume crosses a section of the tube. Hence, h1
dV
= volume flow rate = Av
dt
Fig. 13.21 Flow of an ideal fluid
The mass flow rate is the mass flow per unit time through a
dm dV
cross-section. This is equal to = ρ⋅ If p 1 and p 2 are the pressures at two ends of the tube
dt dt
respectively, the work done by pressure difference in
Example 13.19 Water is flowing through a horizontal tube of pushing the volume ∆V of fluid from the points A to B
non-uniform cross-section. At a place, the radius of the tube through the tube,
is 1.0 cm and the velocity of water is 2 ms −1. What will be W = p∆V = (p 1 − p 2 ) ∆V …(i)
the velocity of water, where the radius of the pipe is 2.0 cm?
The change in potential energy of mass ∆m (of volume ∆V),
Sol. Using equation of continuity,
∆U = ∆mg (h 2 − h1 ) …(ii)
 A1 
1 1 = A2v 2 or v 2 = 
Av  v1 The change in kinetic energy,
 A2 
1
 πr 2  r 
2 ∆K = ∆m (v 22 − v 12 ) …(iii)
or v 2 =  12  v1 =  1  v1 2
 πr2   r2 
594 OBJECTIVE Physics Vol. 1

By conservation of energy,W = ∆K + ∆U
Putting the values from Eqs. (i), (ii) and (iii), we get
Applications based on
1 Bernoulli’s theorem
(p 1 − p 2 ) ∆V = ∆m (v 22 − v 12 ) + ∆mg (h 2 − h1 ) Applications based on Bernoulli’s theorem are given below
2
1 2 1  ∆m 
p 1 + ρv 1 + ρgh1 = p 2 + ρv 22 + ρgh 2 Q ρ =  1. Venturimeter
2 2  ∆V 
Figure shows a venturimeter used to measure flow speed
1
p + ρv 2 + ρgh = constant …(iv) in a pipe. We apply Bernoulli’s equation to the wide
2 (point 1) and narrow (point 2) parts of a horizontal pipe
This is called Bernoulli’s equation. 1 1
with h1 = h 2 , p 1 + ρv 12 = p 2 + ρv 22
On dividing both sides of Eq. (iv) by ρg, we get 2 2
p v 2 constant
+h + = = new constant …(v)
ρg 2g ρg
Here, p /ρg is called pressure head, h is called
h
gravitational head and v 2 /2g is called velocity head. p1
p2
Note Bernoulli’s equation for the fluid at rest When a fluid is at rest,
i.e. the velocity is zero everywhere, then the Bernoulli’s equation 1 v1 2 v2
becomes A2
p1 + ρgh1 = p 2 + ρgh2 ⇒ p1 − p 2 = ρg (h2 − h1) A1

Example 13.20 Calculate the rate of flow of glycerine of Fig. 13.22 Venturimeter
−3
density 1.25 × 10 kgm through the conical section of a
3

horizontal pipe, if the radii of its ends are 0.1 m and 0.04 m A1v 1
From the continuity equation, v 2 =
and the pressure drop across its length is 10 Nm −2 . A2
Sol. According to the question, we draw the following diagram. Substituting and rearranging, we get
A1, p1
1  A2 
A2, p2 p 1 − p 2 = ρv 12  12 − 1 …(i)
2  A2 
v1 v2 Because A1 is greater than A2, v 2 is greater than v 1 and
hence, the pressure p 2 is less than p 1. A net force to the
right accelerates the fluid as it enters the narrow part of
From continuity equation,
the tube (called throat) and a net force to the left slows as
v1 A2 πr22 it leaves. The pressure difference is also equal to ρgh,
A1v1 = A2v 2 or = = where h is the difference in liquid level in the two limbs
v 2 A1 πr12
2
of a manometer tube. Substituting in Eq. (i), we get
2
v1  r2   0.04 4
=  =  = …(i)
v 2  r1   01.  25 2gh
v1 =
1 1 2
From Bernoulli’s equation, p1 + ρv12 = p 2 + ρv 22 (As h1 = h 2 )  A1 
2 2   −1
2(p1 − p 2 )  A2 
or v 2 − v1 =
2 2
ρ Note
2 × 10 (i) The discharge or volume flow rate can be obtained as
or v 22 − v12 = = 1.6 × 10−2 m2s−2 …(ii)
1.25 × 103 dV
= A1v1 = A1
2 gh
2
dt  A1 
Solving Eqs. (i) and (ii), we get   −1
 A2 
1.6 × 10−2 × 625
v2 = ≈ 0128
. ms −1 (ii) The venturi effect can be used to give a qualitative understanding of
(625 − 16) the lift of an airplane wing and the path of a pitcher’s curve ball. An
∴ Rate of volume flow through the tube, airplane wing is designed, so that air moves faster over the top of the
wing that it does under the wing, thus making the air pressure less
Q = A2v 2 = (πr22 )v 2 = π (0.04)2(0128
. ) on top than underneath. This difference in pressure results in a net
upward lifting force on the wing.
= 6.43 × 10−4 m3s−1
Fluid Mechanics 595

Example 13.21 The flow of blood in a large artery of an Range The range of liquid (R ), i.e. the horizontal distance
anesthetised dog is diverted through a venturimeter. The covered by liquid coming out of the hole is given by
wider part of the meter has a cross-sectional area equal to
that of the artery, A = 16 mm 2 . The narrower part has an R = 2 h (H − h )
area a = 8mm 2 . The pressure drop in the artery is 24 Pa.
What is the speed of the blood in the artery? R 2 = 4(Hh − h 2 )
Sol. Bernoulli’s equation for the horizontal flow of blood is
1 1 dR 2
p1 + ρv12 = p 2 + ρv 22 For R to be maximum, =0
2 2 dh
By equation of continuity, H
or H − 2h = 0 or h =
Av1 = av 2 or v 2 = Av1 /a 2
1 ρ A2v12 1 2 H
∴ p1 − p 2 = − ρv1 i.e. R is maximum at h =
2 a2 2 2
1 2 A2  H  H
=
ρv1  2 − 1 and R max = 2 H − 
2 a  2  2
Here, p1 − p 2 = 24 Pa ⇒ R max = H
ρ (blood) = 1.06 × 103 kg m−3, A /a = 16 / 8 = 2
i.e. The maximum horizontal distance covered by liquid
2 (p1 − p 2 ) coming out of a hole is equal to the height of the liquid
∴ v1 =
 A2  column.
ρ 2 − 1
a  Time taken to empty a tank is given by the formula

2 × 24 A 2H
= t=
1.06 × 103 × (22 − 1) a g
= 0.123 ms−1
Example 13.22 If the water emerge from an orifice in a
2. Speed of efflux tank in which the gauge pressure is 4 × 10 5 Nm −2 before the
The outflow of a fluid is called efflux and the speed of the flow starts, then what will be the velocity of the water
liquid coming out is called speed of efflux. emerging out? (Take, density of water is 1000 kg m −3 )
Consider a closed vessel filled with a liquid upto height H Sol. Here, p = 4 × 105 N m −2 and ρ = 1000 kg m −3, g = 10 m s−2
and a small hole is made in the wall of the vessel at a depth h p 4 × 105
below the surface of liquid. Then, the speed of efflux is, Apply, p = hρg ⇒ h = =
ρg 1000 × 10
v = 2gh 2 × 10 × 4 × 105
Velocity of efflux, v = 2gh =
1000 × 10
= 800 = 28.28 ms −1
h
v Example 13.23 A cylindrical vessel of 90 cm height is kept
H v H O
O filled with water upto the rim. It has four holes 1, 2, 3, 4
H–h which are respectively at heights of 10 cm, 20 cm, 45 cm
and 50 cm from the horizontal floor PQ. Through which of
the holes water is falling at the maximum horizontal
R distance?
Fig. 13.23 Liquid coming out from the orifice of a tank

From the above formula, it is clear that 4


“The velocity of efflux of a liquid issuing out of an orifice 3
is the same as it would attain, if allowed to fall freely through 2
the vertical height between the liquid surface and orifice.” 1

The above statement is also known as Torricelli’s theorem.


P Q
596 OBJECTIVE Physics Vol. 1

Sol. Different holes of the cylindrical vessel are shown in the Sol. As we know that, the time in which level of liquid in a
figure. Water comes out from different holes with different tank falls from H1 to H 2 is
speeds. Therefore, range will be different corresponding to A 2
different holes. t= ( H1 − H 2 )
a g
where, A = area of cross-section of the tank
4
and a = area of hole.
3
H
2 H = 90 cm It is given that, H1 = H and H 2 =
2
1
A 2  H
∴ t1 =  H −  …(i)
P Q a g  2
H 90 Similarly, time taken to empty the rest half of the tank,
Horizontal range will be maximum when h = = = 45 cm
2 2
A 2 (H /2) A H
This height corresponds to hole 3. t2 = = …(ii)
a g a g
Example 13.24 A tank is filled with a liquid upto a height H.
From Eqs. (i) and (ii), we get
A small hole is made at the bottom of this tank. Let t1 be the
time taken to empty first half of the tank and t 2 the time t1 t
= 2 − 1 or 1 = 0.414
taken to empty rest half of the tank. t2 t2
t1 Note From here we see that t 1 < t 2. This is because initially the
Then, find the value of .
t2 pressure is high and the liquid comes out with greater speed.

CHECK POINT 13.2


1. Consider streamline flow of a liquid flowing through a tube (d) conservation of angular momentum
as shown in the figure, which of the following is correct 5. Water is in streamline flows along a horizontal pipe with
regarding velocities of liquid at different points? non-uniform cross-section. At a point in the pipe, where the
v3 area of cross-section is 10 cm2, the velocity of water is
v1 1 ms −1 and the pressure is 2000 Pa.
v2
The pressure at another point, where the cross-sectional
area is 5 cm 2, is
(a) v1 = constant, v2 = constant, v3 = constant (a) 4000 Pa (b) 2000 Pa (c) 1000 Pa (d) 500 Pa
(b) v1 ≠ v2 ≠ v3
6. If the velocity head of a stream of water is equal to 10 cm,
(c) v1 = v2 = v3
(d) Both (a) and (b) are correct
then its speed of flow is approximately
(a) 1.0 ms−1 (b) 1.4 ms−1 (c) 140 ms−1 (d) 10 ms−1
2. If Re is the Reynold’s number, then which of the following
7. Water falls from a tap with A0 = 4 m , 2
is incorrect?
(a) For Re < 1000, flow is laminar A = 1 m2 and h = 2 m, then volume rate at
A0, v0
(b) For1000 < Re < 2000, flow is steady velocity v is
−1 −1
(c) For Re > 2000, flow is turbulent (a) 2.5 ms (b) 6.5 ms h
(d) All are incorrect (c) 4.5 ms −1 (d) 1.5 ms −1
A, v
3. An incompressible liquid is flowing through a horizontal 8. The lift of an aeroplane is based on
pipe as shown in figure. The value of speed v is (a) Torricelli’s theorem (b) Bernoulli’s principle
6 m/s (c) law of gravitation (d) continuity equation
A 9. The velocity of efflux of a liquid through an orifice in the
8
0. bottom of a tank does not depend upon
8 m/s A (a) density of liquid
3A (b) height of the liquid column above orifice
(c) acceleration due to gravity
v (d) None of the above
(a) 1.1 ms −1 (b) 2.1 ms −1 (c) 3.1 ms −1 (d) 5.1 ms −1
10. A tank is filled to a height H. The range of water coming out
4. Bernoulli’s theorem is a consequence of of a hole which is a depth H/4 from the surface of water
(a) conservation of mass level is
(b) conservation of energy 2H 3H 3H
(c) conservation of linear momentum (a) (b) (c) 3H (d)
3 2 4
Fluid Mechanics 597

VISCOSITY Coefficient of viscosity of water at 10°C is


The characteristic of fluid by virtue of which relative η = 1.3 × 10 −3 Nsm−2 . Experiments show that coefficient
motion between different layers is opposed is known as of viscosity of a liquid decreases as its temperature rises.
viscosity. Example 13.25 A plate of area 2m 2 is made to move
Viscosity is internal friction in a fluid which occurs due to horizontally with a speed of 2 ms −1 by applying a horizontal
diffusion of molecules of one layer into another. Viscous tangential force over the free surface of a liquid. The depth
forces opposes the motion of one portion of a fluid relative of the liquid is 1 m and the liquid in contact with the bed is
to the other. The simplest example of viscous flow is stationary. Coefficient of viscosity of liquid is 0.01 poise.
motion of a fluid between two parallel plates. Find the tangential force needed to move the plate.
2−0
F
v Sol. Velocity gradient = = 2 s −1
1− 0
v = 2 ms–1

F
y 1m

x From Newton’s law of viscous force,


Fig. 13.24 ∆v
|F | = ηA = (0.01 × 10−1) (2) (2)
The bottom plate is stationary and the top plate moves ∆y
with constant velocity v. The fluid in contact with each = 4 × 10−3 N
surface has same velocity as that of the surface. The flow
So, to keep the plate moving a force of 4 × 10−3 N must be applied.
speeds of intermediate layers of fluid increase uniformly
from bottom to top, as shown by arrows. So, the fluid Flow of liquid through a cylindrical pipe
layers slide over one another.
Figure shows the flow speed profile for laminar flow of a
According to Newton, the frictional force F (or viscous force)
viscous fluid in a long cylindrical pipe.
between two layers depends upon the following factors
The speed is greatest along the axis and zero at the pipe
(i) Force F is directly proportional to the area (A) of the
walls.
layers in contact, i.e. F ∝ A.
L
(ii) Force F is directly proportional to the velocity
 dv  R v
gradient   between the layers. r
 dy  Axis

Combining these two, we have


p1 p2
dv Fig. 13.25 A viscous fluid flowing in a long cylindrical pipe
F ∝A
dy
The flow speed v at a distance r from the axis of a pipe of
dv radius R is
or F = − ηA
dy p − p2 2
v = 1 (R − r 2 )
4ηL
Here, η is constant of proportionality and is called
coefficient of viscosity. Its value depends on the nature where, p 1 and p 2 are the pressure at the two ends of a
of the fluid. pipe with length L. The flow is always in the direction of
decreasing pressure.
The negative sign in the above equation shows that the
direction of viscous force F is opposite to the direction of At r = R (along the walls), v = 0
relative velocity of the layer. The velocity of viscous fluid along the wall of the pipe is
The SI unit of η is Nsm−2 . It is also called decapoise or (p − p 2 )R 2
zero and at r = 0 (along the axis), v = 1 = v max
pascal second. 4ηL
Thus, 1 decapoise = 1 Nsm −2 = 1 Pa-s = 10 poise The velocity of viscous fluid flowing along a pipe is
maximum along its axis.
Dimensions of η are [ML−1 T −1 ].
598 OBJECTIVE Physics Vol. 1

 dV  By analogy, total flow resistance is


Volume flow rate Q or  : Poiseuille equation 8η
 dt  X eq = X1 + X 2 = (L1 + L 2 )
To find the total volume flow rate through the pipe, we πR 4
consider a ring with inner radius r, outer radius r + dr and Also, p 1 − p 2 = QX1, p 2 − p 3 = QX 2
cross-sectional area dA = 2πr dr . The volume flow rate and p 1 − p 3 = QX eq
through this element is v dA. The total volume flow rate is
found by integrating from r = 0 to r = R .
(ii) Parallel combination
(a) The pressure difference across each tube is same.
dV π  R 4   p1 − p 2  (b) The rate of flow across each tube is different.
The result is Q= =    
dt 8  η   L  Flow resistance of tubes are p1 p2
8ηL Q1 1 R1
The relation was first derived by Poiseuille and is called X1 =
πR 14 Q2 2
Poiseuille’s equation. R2
8ηL
Some important points related to Poiseuille equation are and X 2 = L
given below πR 24 Fig. 13.27
(i) Poiseuille’s equation can also be written as By electrical analogy, total flow
p − p 2 ∆p 8ηL 1
=
1
+
1
Q= 1 = , here X = resistance of parallel tubes is given by, ,
 8ηL  X πR 4 X eq X1 X 2
 
 πR 4  p 1 − p 2 = Q1X1 = Q 2X 2
This equation can be compared with the current Also, p 1 − p 2 = (Q1 + Q 2 ) X eq
∆V Following examples illustrates above discussion.
equation through a resistance, i.e. i = .
R Example 13.26 Water is flowing through a horizontal tube
Here, ∆V = potential difference 8 cm in diameter and 4 km in length at the rate of 20 litre/s.
and R = electrical resistance. Assuming only viscous resistance. Find the pressure required
For current flow through a resistance, potential to maintain the flow in terms of mercury column.
difference is a requirement, similarly for flow of (Take, coefficient of viscosity of water is 0.001 Pa-s)
liquid through a pipe, pressure difference is must. Sol. Here, 2r = 8 cm = 0.08 m or r = 0.04m, l = 4 km = 4000 m,
(ii) Problems of series and parallel combination of pipes V = 20 litre/s = 20 × 10−3 m3s−1, η = 0.001 Pa-s, p = ?
can be solved in the similar manner as is done in
πpr 4 8Vηl
case of an electrical circuit. The only difference is As, V= or p =
8ηl πr 4
(a) Potential difference (∆V ) is replaced by the pressure
difference (∆p ). 8 × (20 × 10−3 ) × 0.001 × 4000
= = 7.954 × 104 Pa
 L  
22
  × (0.04)
4
(b) The electrical resistance R  = ρ  is replaced by  7
 A
 8ηL  ∴ Height of mercury column for pressure difference p will be
flow resistance X  = .
 πR 4  p 7.954 × 104
h= = = 0.5968 m = 59.68 cm
(c) The electrical current i is replaced by volume flow ρg (13.6 × 103 ) × 9.8
rate Q or dV /dt.
Example 13.27 A liquid is flowing through horizontal pipes
as shown in figure.
Combination of capillary (thin) tubes
E F
(i) Series combination
(a) The rate of flow (Q ) will be same. Q

(b) Pressure difference across each tube is different. A B C D

p2
p1 p3 G H
1 Q Q 2 R
Length of different pipes has the following ratio
L1 L2
L L
Fig. 13.26 L AB = LCD = EF = GH
2 2
Fluid Mechanics 599

Similarly, radii of different pipes has the ratio,


R STOKES’ LAW
R AB = R EF = R CD = GH
2 When an object moves through a fluid, it experiences a
Pressure at A is 2p 0 and pressure at D is p 0 . The volume flow viscous force which acts in opposite direction of its
rate through the pipe AB is Q. Find velocity.
F
(i) volume flow rates through EF and GH
(ii) pressure at E and F.
Sol. The equivalent electrical circuit can be drawn as under,
L  8ηL  r
X ∝ 4 QX =  v
R  πR 4 
Fig. 13.28 An object is moving through a fluid
16X
E F
Q The formula for the viscous force on a sphere was first
8X 8X derived by the English physicist G. Stokes in 1843.
A B 17 C D
2p0 Q Q p0 According to him, a spherical object of radius r moving at
G velocity v experiences a viscous force given by
16 H
Q X
17 F = 6πηrv (where, η = coefficient of viscosity)

 1
This law is called Stokes’ law.
1
 
 2 (1) (1)
∴ X AB : X CD : X EF : X GH = 24 : 4
: 4
: Terminal velocity (vT )
 1  1  1 (1)4
      Consider a small sphere falling from rest through a large
 2  2  2
column of viscous fluid.
= 8 : 8 : 16 : 1 Ft + Fv
(i) As the current is distributed in the inverse ratio of the
resistance (in parallel), the Q will be distributed in the
inverse ratio of X.
Q v
Thus, volume flow rate through EF will be and that
17
16
from GH will be Q.
17 w
(weight, acting
 (16X )(X )  288 vertically downward)
(ii) X net = 8X +   + 8X = X
 (16X ) + (X ) 17 Fig. 13.29 Force acting on a small sphere falling
through a liquid
∆p  ∆V 
∴ Q= Q i =  The forces acting on the sphere are
X net  R 
(i) weight w of the sphere acting vertically downwards.
(2p 0 − p 0 ) 17p 0
= = (ii) upthrust buoyant force Ft acting vertically upwards.
288 288X
X (iii) viscous force Fv acting vertically upwards, i.e. in a
17
Now, let p1 be the pressure at E, then direction opposite to velocity of the sphere.
8 × 17p 0 Initially, Fv = 0 (Q v = 0)
2p 0 − p1 = 8 QX =
288 and w > Ft
 17 × 8 and the sphere accelerates downwards. As the velocity of
∴ p1 = 2 −  p0
 288  the sphere increases, Fv increases. Eventually a stage is
= 1.53 p0 reached when w = Ft + Fv …(i)
Similarly, if p 2 be the pressure at F, then After this net force on the sphere is zero and it moves
downwards with a constant velocity called terminal
p 2 − p 0 = 8 QX
velocity ( vT ).
8 × 17
∴ p2 = p0 + p0 It can be defined as the maximum constant velocity
288
or p2 = 1.47p0 acquired by a body while falling through a viscous
medium.
600 OBJECTIVE Physics Vol. 1

Substituting proper values in Eq. (i), we get Sol. The terminal velocity of the bubble is given by
4 3 4 2 r 2 ( ρ − σ )g
πr ρg = πr 3σg + 6πηrv T …(ii) vT =
3 3 9 η
Here, ρ = density of sphere, σ = density of fluid Here, r = 0.4 × 10−3 m, σ = 0.9 × 103 kgm−3
and η = coefficient of viscosity of fluid. ρ = 1.293 kgm−3, η = 0.15 Nsm−2
From Eq. (ii), we get and g = 9.8 ms−2
Substituting the values, we get
2 r 2 (ρ − σ ) g
vT = 2 (0.4 × 10−3 )2 (1.293 − 0.9 × 103 ) × 9.8
9 η vT = ×
9 0.15
Figure shows the variation of the velocity v of the sphere = − 0.0021 m s−1 or vT = − 0.21 cms−1
with time
Note Here, negative sign implies that the bubble will rise up.
v
Example 13.29 Two spherical raindrops of equal size are
falling vertically through air with a terminal velocity of
vT 1 ms −1. What would be the terminal speed, if these two
drops were to coalesce to form a large spherical drop?
Sol. As, vT ∝ r 2 …(i)
O Time
Let r be the radius of small raindrops and R the radius of large
Fig. 13.30 Variation of velocity of a sphere
falling through a liquid, with time drop.
Equating the volumes, we get
Note From the above expression, we can see that terminal velocity of a 4 4 
spherical body is directly proportional to the difference in the πR 3 = 2  πr 3 
densities of the body and the fluid (ρ − σ ). 3 3 
If the density of fluid is greater than that of body (i.e. σ > ρ), the R
∴ R = (2)1/ 3 ⋅ r or = (2)1/ 3
terminal velocity is negative. This means that the body instead of r
falling, moves upward. This is why air bubbles rise up in water. 2
vT′  R 
Q =   = (2)2/3
Example 13.28 With what terminal velocity will an air vT  r 
bubble 0.8 mm in diameter rise in a liquid of viscosity
0.15 Nsm −2 and specific gravity 0.9 kgm −3 ? (Take, density ∴ vT′ = (2)2/3 (1.0) ms−1 = 1.587 ms−1
of air is 1.293 kg m −3 )

CHECK POINT 13.3


1. The rate of flow of liquid in a tube of radius r, length l, 3. The terminal velocity v of a small steel ball of radius r falling
whose ends are maintained at a pressure difference p is under gravity through a column of viscous liquid of
πNpr 4 coefficient of viscosity η depends on mass of the ball m,
V = , where η is coefficient of the viscosity and acceleration due to gravity g, coefficient of viscosity η and
ηl
radius r. Which of the following relations is dimensionally
N is
1
correct?
(a) 8 (b) mgr mg ηmg
8 (a) v ∝ (b) v ∝ mg η r (c) v ∝ (d) v ∝
η ηr r
1
(c) 16 (d)
16 4. The ratio of the terminal velocities of two drops of radii R
2. Two capillary tubes of the same length but different radii r1 and R / 2 is
(a) 2 (b) 1
and r2 are fitted in parallel to the bottom of a vessel. The (c) 1/2 (d) 4
pressure head is p. What should be the radius of a single
tube that can replace the two tubes, so that the rate of flow 5. An air bubble rises from the bottom of a lake of large depth.
is same as before? The rising speed of air bubble will
r r (a) go on increasing till it reaches surface
(a) r1 + r2 (b) 1 2
r1 + r2 (b) go on decreasing till it reaches surface
r1 + r2 (c) increases in the beginning, then will become constant
(c) (d) None of these
2 (d) be constant all throughout
Fluid Mechanics 601

6. Two equal drops of water are falling through air with a p p


steady velocity v. If the drops coalesce, the new velocity will
be (c) (d)
v
(a) 2v (b) 2v (c) 22/ 3 v (d)
2 x x
7. From amongst the following curves, which one shows the
variation of the velocity v with time t for a small sized 8. A spherical ball of radius 3.0 × 10 −4 m and density104 kgm
−3
spherical body falling vertically in a long column of a falls freely under gravity through a distance h before
viscous liquid? entering a tank of water. If after entering the water, the
p p velocity of the ball does not change, what is the value of h
(Take, viscosity of water is
(a) (b) 9.8 × 10 − 6 Nsm −2)?
(a) 1.65 × 103 m (b) 2.65 × 102 m
x x (c) 3.65 × 104 m (d) 1.45 × 102 m

SURFACE TENSION
It is the property of liquid at rest by virtue of which Force
or Surface tension, S =
a liquid surface tends to occupy a minimum surface Length
area and behaves like stretched membrane.
A small drop of liquid is spherical in shape, because From this expression, surface tension can be defined as the
for given volume, sphere has minimum surface area. force acting per unit length of an imaginary line drawn on
This stretched behaviour is due to net downward the liquid surface, the direction of force being
force acting on a molecule on the surface of liquid as perpendicular to this line and tangential to the liquid
shown in figure. surface.
It is denoted by S and it is a scalar quantity.
Units and dimension of surface tension
SI unit of surface tension = Nm −1, CGS unit of surface tension
= dyne cm−1 ,
Dimensions of surface tension
Force [MLT − 2 ]
= = = [ML0 T − 2 ]
Length [L]
Fig. 13.31 Forces between molecules of a liquid
Note The surface tension of a particular liquid usually decreases as
temperature increases. To wash cloth throughly, water must be forced
A molecule well inside the liquid experiences no net through the tiny spaces between the fibres. This requires increasing the
force. surface area of the water, which is difficult to do because of surface
tension. Hence, hot water and soapy water is better for washing.
Now, consider a line AB on the free surface of the
liquid. The small elements of the surface on this line Example 13.30 A liquid is kept in a beaker of radius 4 cm.
are in equilibrium because they are acted upon by Consider a diameter of the beaker on the surface of the water.
equal and opposite forces, acting perpendicular to Find the force by which the surface on one side of the diameter
the line from either side as shown in figure. pulls the surface on the other side. (Take, surface tension of liquid
= 0.075Nm −1)
The force acting on this line is proportional to the
length of this line. If l is the length of imaginary line Sol. The length of the diameter, l = 2r = 8 cm = 0.08 m
and F the total force on either side of the line, then The surface tension is S = F / l .
F ∝ l ⇒ F = Sl Thus, F = Sl = (0.075 Nm−1) × (0.08 m) = 6 × 10−3 N

F B
Surface energy
l F The free surface of a liquid always has a tendency to contract
A
and possess minimum surface area. To increase the surface area
Fig. 13.32 Force on a line at the surface of a liquid of the liquid work has to be done. This work done is stored in
the surface film of the liquid as its potential energy.
602 OBJECTIVE Physics Vol. 1

This potential energy per unit area of the surface film is Here, ∆A = 2 [4π{(2.5 × 10−2)2 − (1.0 × 10−2 )2}]
called the surface energy. = 1.32 × 10−2 m2
Hence, the surface energy may be defined as the ∴ Work done, W = (3.0 × 10−2)(132
. × 10−2) J
amount of work done in increasing the area of the
surface film through unity. Thus, = 3.96 × 10–4 J

Example 13.32 Calculate the energy released when 1000


Work done in increasing the surface area
Surface energy = small water drops each of radius 10 −7 m coalesce to form
Increase in surface area one large drop. (Take, surface tension of water is
. × 10 −2 Nm −1)
70
The SI unit of surface energy is Jm −2 .
Sol. Let r be the radius of smaller drops and R of bigger one.
Equating the initial and final volumes, we get
Relation between surface energy 4 4 
πR 3 = (1000)  πr 3
and surface tension 3 3 
Consider a rectangular frame PQRS. Here, wire QR is or R = 10 r = (10 )(10−7 ) m or R = 10−6 m
movable. A soap film is formed on the frame. The film
Further, the water drops have only one free surface.
pulls the movable wire QR inward due to surface tension.
Therefore, ∆A = 4πR 2 − (1000)(4πr 2 )
force F ′
As, surface tension = = ⇒ F ′ = S × 2l = 4π [(10−6 )2 − (103 )(10−7 )2]
length 2l
z = − 36π(10−12 ) m2
P Q Here, negative sign implies that surface area is decreasing.
Hence, energy released in the process,
l F′ l
F U = T | ∆A | = ( 7 × 10−2 )(36π × 10−12 ) J
S
(a)
R
(b) = 7.9 × 10−12 J
Equilibrium position of the film Stretched position of the film
Fig. 13.33 Excess pressure
If QR is moved through a distance z by an external force F Due to surface tension, a drop or bubble tends to contract
very slowly, then some work has to be done against this force. and so compresses the matter enclosed. This in turn
increases the internal pressure which prevents further
∴ External work done = Force × Distance contraction and equilibrium is achieved. So, in
= S × 2l × z (Q F ′ = F ) equilibrium, the pressure inside a bubble or drop is greater
Increase in surface area of film = 2l × z than outside and the difference of pressure between two
(As soap film has two sides) sides of the liquid surface is called excess pressure.
Following ways are used to calculate the excess pressure
Work done S 2l z inside a bubble or drop
Surface energy = = =S
Surface area 2lz Excess pressure inside a soap bubble A soap bubble
consists of two spherical surface films with a thin layer of
So, value of surface energy of liquid is numerically liquid between them. Because of surface tension, the film
equal to the value of surface tension. tend to contract in an attempt to minimize their surface
area. But as the bubble contracts, it compresses the inside
Note
(i) Work done in forming a drop, W = S × 4 πr 2
air, eventually increasing the interior pressure to a level
that prevents further contraction.
(As, ∆A = 4 πr 2 − 0 = 4 πr 2)
(ii) Work done in forming a bubble, W = S × 4 πr 2 × 2 (As, bubble in air
has two surfaces)
O
Example 13.31 How much work will be done in increasing R
the diameter of a soap bubble from 2 cm to 5 cm.
(Take, surface tension of soap solution is 3.0 × 10 −2 Nm −2 )
Sol. Soap bubble has two surfaces.
Fig. 13.34 A soap bubble of radius R
Hence, W = T ∆A
Fluid Mechanics 603

Excess pressure inside the soap bubble in terms of its This excess pressure acts from concave to convex side, the
radius R and the surface tension T of the liquid is given by interface will be concave towards smaller bubble and convex
towards larger bubble. Let R be the radius of interface, then
4T
p0 p= …(ii)
R
p From Eqs. (i) and (ii), we get
rr (0.004)(0.002)
R R = 12 = = 0.004 m
r1 − r2 (0.004 − 0.002)

Fig. 13.35 Pressures inside and outside of a soap bubble


Excess pressure inside a liquid drop A liquid drop has
only one surface film, so excess pressure inside the liquid
4T drop is given by
or ∆p = p − p 0 = 2T
R ∆p =
R
Example 13.33 0.04 cm liquid column balances the excess
pressure inside a soap bubble of radius 6 mm. Evaluate Some Important points regarding
density of the liquid. (Take, surface tension of soap solution excess pressure
= 0.03 Nm −1)
(i) If we have an air bubble inside a liquid, a single
Sol. The excess pressure inside a soap bubble,
surface is formed. There is air on the concave side
∆p = 4T / R and liquid on the convex side. The pressure in the
4 × 0.03 Nm−1 concave side (i.e. in the air) is greater than the
= −3
= 20 Nm−2
6 × 10 m pressure in the convex side (i.e. in the liquid) by an
The pressure due to 0.04 cm of the liquid column, amount 2T /R .
∆p = hρg = (0.04 × 10−2m) ρ (10 ms−2 )
Thus, 20 Nm−2 = (0.04 × 10−2m) ρ(10 ms−2 )
p2
∴ Density of the liquid, ρ = 5 × 103kgm−3 p1

Example 13.34 Two separate soap bubbles (radii 0.004 m


and 0.002 m) formed of the same liquid (surface tension Fig. 13.36 Excess pressure in an air bubble inside a liquid
0.07 Nm −1) come together to form a double bubble. Find the 2T
∴ p 2 − p1 =
radius and the sense of curvature of the internal film surface R
common to both the bubbles.
The above expression has been written by assuming
4T p 1 to be constant from all sides of the bubble. For
Sol. Excess pressure inside first soap bubble, p1 = p 0 +
r1 small size bubbles this can be assumed.
4T (ii) From the above discussion, we can make a general
Excess pressure inside second soap bubble, p 2 = p 0 +
r2 statement. The pressure on the concave side of a
spherical liquid surface is greater than the convex
r2
side by 2T /R as shown below.
p p2 p0 p0
A A
p1
r1 p0 B B
p0 − 2T p0 − 2T
R R
Q r2 < r1, therefore p 2 > p1
i.e. Pressure inside the smaller bubble will be more. The R R
2T 2T
excess pressure, pA − pB = pB − pA =
R R
r − r 
p = p 2 − p1 = 4T  1 2  …(i) Fig. 13.37
 rr12 
604 OBJECTIVE Physics Vol. 1

(iii) Excess pressure inside a soap bubble can also be


understood in terms of excess pressure inside a Angle of contact
curved surface as shown below. The surface of liquid near the plane of contact with
p2 another medium is in curved shape.
p′ The angle of contact is defined as the angle that the
tangent to the liquid surface at the point of contact makes
p1 with the solid surface. It is denoted by θ.
The value of contact angle determines whether a liquid will
spread on the surface of a solid or it will form droplets on it.
When a liquid is put on a solid surface, then there are three
R
interfacial tensions at all the three interfaces; liquid-air,
Fig. 13.38 solid-air and solid-liquid denoted by S la , S sa and S sl.
2T At the line of contact, the surfaces between the three
p1 − p ′ =
R media must be in equilibrium.
2T The following cases arise
⇒ p′ − p2 =
R (i) If the surface tension at the solid-liquid Ssl , interface
4T is greater than the surface tension at the liquid-air
Excess pressure, ∆p = p 1 − p 2 =
R S la interface, i.e. Ssl > S la , then θ > 90° (the angle of
(iv) If two bubbles of different sizes are connected with contact is obtuse angle).
each other through a thin tube (as shown in figure),
then the air will rush from smaller to larger bubble
(pressure inside smaller bubble will be greater than Sla
pressure inside larger bubble), so that the smaller
will shrink while the larger will expand till the
θ
smaller bubble reduces to a droplet.
Ssa Ssl
Tube
Fig. 13.40

The molecules of a liquid are attracted strongly to


Air
Smaller Larger themselves and weakly to those of solid. It costs a lot
bubble bubble of energy to create a liquid-solid surface. The liquid
Fig. 13.39 then does not wet the solid.
Example 13.35 What should be the pressure inside a small e.g. Water-leaf or glass-mercury interface.
air bubble of 01
. mm radius situated just below the water (ii) If the surface tension at the solid-liquid Ssl interface
. × 10 −2 Nm −1
surface? (Take, surface tension of water = 72 is less than the surface tension at the liquid-air S la
and atmospheric pressure = 1.013 × 10 Nm −2 )
5
interface, i.e. Ssl < S la , then 0 < 90 ° (the angle of
Sol. Surface tension of water, contact is acute angle).
T = 7.2 × 10−2 Nm −1
Sla
Radius of air bubble, R = 0.1 mm
= 10–4 m
The excess pressure inside the air bubble is given by θ
2T Ssa Ssl
p 2 − p1 = Fig. 13.41
R
2T The molecules of the liquid are strongly attracted to those
∴ Pressure inside the air bubble, p 2 = p1 +
R of solid and weakly attracted to themselves. It costs less
Substituting the values, we get energy to create a liquid-solid surface and liquid wets the
(2 × 7.2 × 10−2 ) solid.
p 2 = ( 1.013 × 105 ) +
10−4 e.g. When soap or detergent is added to water, the angle of
= 1.027 × 105 Nm−2 contact becomes small.
Fluid Mechanics 605

Shape of liquid surface Formula of capillary rise


The curved surface of the liquid is called meniscus. The When a capillary tube is dipped in a liquid, then the level
shape of the meniscus (convex or concave) is determined by of liquid in capillary tube rises or falls w.r.t. free surface
the relative strengths of cohesive and adhesive forces. The of liquid outside the capillary.
force between the molecules of the same material is known
as cohesive force and the force between the molecules of r
different kinds of material is called adhesive force. θ
R
When the adhesive force between solid and liquid molecules
is more than the cohesive force between liquid-liquid
molecules (as with water and glass), shape of the meniscus is θ
concave and the angle of contact θ is less than 90°. In this
case, the liquid wets or adheres to the solid surface.
Fig. 13.44 Shape of meniscus
Angle of
contact
Angle of This phenomena of rises or fall of liquid is called capillary
contact action.
θ
θ The formula in capillary motion is
2T cos θ
h=
rρg
Fig. 13.42
where, h = height of liquid column rises or falls,
When the adhesive force between solid and liquid r = radius of capillary tube,
molecules is less than the cohesive force between ρ = density of liquid,
liquid-liquid molecules than shape of meniscus is convex. g = acceleration due to gravity,
Note The angle of contact between water and clean glass is zero and θ = angle of contact and T = surface tension.
that between mercury and clean glass is 137°.
The result has following notable features
(i) If the contact angle θ is greater than 90°, the term
Capillarity cos θ becomes negative and hence, h is negative. The
The term capilla means hair which is Latin word. A tube result, then gives the depression of the liquid in the
of very fine (hair-like) bore is called a capillary tube. tube.
(ii) Suppose a capillary tube is held vertically in a liquid
which has a concave meniscus, then capillary rise is
given by
2T cos θ 2T  r 
h= = Q R = 
rρg Rρ g  cos θ 
Water Mercury
2T
or hR =
Fig. 13.43 Elevation and depression of liquid in a capillary tube ρg

If a capillary tube of glass is dipped in liquid like water, Note


(i) If the tube is of a length l less than h (l < h), the liquid does not
the liquid rises in the tube, but when the capillary tube is overflow. The angle made by liquid surface with the tube adjusts
dipped in a liquid like mercury, the level of liquid falls in in such a way that rise will stop when it is equal to length of tube.
the tube. (ii) If the tube makes an angle φ units with vertical, rise will be
This phenomenon of rise or fall of a liquid in the capillary
h
is called capillarity. h′ =
cos φ
Some examples are
(i) Small capillaries in fibres of towels soaks water from Example 13.36 A capillary tube whose inside radius is 0.5 mm
our skin. is dipped in water having surface tension 70. × 10 −2 Nm −1. To
(ii) In trees sap rises in stem due to capillary action. what height is the water raised above the normal water level?
Angle of contact of water with glass is 0°. Density of water is
10 3 kg m −3 and g = 9.8 ms −2 .
606 OBJECTIVE Physics Vol. 1

2T cos θ Sol. For glass-water, angle of contact, θ = 0°.


Sol. Height raised, h =
rρg Now, height of water in capillary,
Substituting the proper values, we get 2T cos θ (2)(7.0 × 10−2 ) cos 0°
−2 h= =
(2)(7.0 × 10 ) cos 0° rρg (0.4 × 10−3 )(103 )(9.8)
h= = 2.86 × 10−2 m
(0.5 × 10−3 )(103 )(9.8) = 3.57 × 10−2 m
= 2.86 cm = 3.57 cm
Example 13.37 A glass tube of radius 0.4 mm is dipped Length of capillary occupied after tilting it is,
vertically in water. Find upto what height, the water will rise in h 3.57
the capillary. If the tube is inclined at an angle of 60° with the l= =
cos 60° 1/2
vertical, how much length of the capillary is occupied by water?
(Take, surface tension of water = 7.0 × 10−2 Nm −1, density of = 7.14 cm
water = 103 kgm −3)

CHECK POINT 13.4


1. Which of the following is not the unit of surface tension? 5. If work W is done is blowing a bubble of radius R from soap
(a) N/m (b) J/m 2 solution, then the work done in blowing a bubble of radius
(c) kg/s 2 (d) W/m 2R from the same solution is
(a) W / 2 (b) 2W
2. A square frame of side L is dipped in a liquid. On taking 1
out, a membrane is formed. If the surface tension of the (c) 4W (d) 2 W
3
liquid is T , the force acting on one side of the frame will be
(a) 2TL (b) 4 TL 6. If two soap bubbles of equal radii r coalesce, then the radius
TL of curvature of interface between two bubbles will be
(c) TL (d)
2 (a) r
(b) 0 (c) infinity
3. A 10 cm long wire is placed horizontally on the surface of r
(d)
water and is gently pulled up with a force of 2 × 10−2 N to 2
keep the wire in equilibrium. The surface tension (in Nm−1)
of water is 7. The surface tension of soap solution is 25 × 10−3 Nm −1 . The
(a) 0.1 (b) 0.2 excess pressure inside a soap bubble of diameter 1 cm is
(c) 0.001 (d) 0.002 (a) 10 Pa (b) 20 Pa
(c) 5 Pa (d) None of these
4. If temperature increases, the surface tension of a liquid
(a) increases 8. Water rises in a capillary tube to a height of 2.0 cm. In
(b) decreases another capillary tube whose radius is one-third of it, how
(c) remains the same much the water will rise?
(d) first increases then decreases (a) 6.0 cm (b) 2.0 cm
(c) 4.0 cm (d) 8.0 cm
Chapter Exercises
(A) Taking it together
Assorted Questions of the Chapter for Advanced Level Practice

1 Hair of shaving brush cling together when it is


removed from water due to (c) (d) None of these
(a) force of attraction between hair
(b) surface tension
8 Which of the following diagrams does not represent
(c) viscosity of water
a streamline flow?
(d) characteristic property of hair
2 Coatings used on raincoat are waterproof because they
(a) increases angle of contact (a) (b)
(b) decreases angle of contact
(c) does not alters angle of contact
(d) forms a smooth surface
3 Small droplets of liquid are usually more spherical in
shape than larger drops of the same liquid because
(a) force of surface tension is equal and opposite to the (c) (d)
force of gravity
(b) force of surface tension predominates the force of
gravity 9 A body measures 5 N in air and 2 N when put in
(c) force of gravity predominates the force of surface
tension
water. The buoyant force is
(d) force of gravity and force of surface tension act in the (a) 7 N (b) 9 N (c) 3 N (d) None of these
same direction and are equal 10 The reading of spring balance when a block is
4 Water does not wet an oily glass because suspended from it in air, is 60 N. This reading is
(a) cohesive force of oil > adhesive force between oil and changed to 40 N when the block is immersed in
glass water. The specific gravity of the block is
(b) cohesive force of oil > cohesive force of water (a) 3 (b) 2 (c) 6 (d) 3 /2
(c) oil repels water
11 An iron block is on a boat which floats in a pond.
(d) cohesive force of water > adhesive force between
water and oil molecules The block is thrown into the water. The level of
water in the pond will be
5 If two identical mercury drops are combined to form (a) equal to the earlier level
a single drop, then its temperature will (b) less than the earlier level
(a) decrease (b) increase (c) more than the earlier level
(c) remains the same (d) None of these (d) depends on how large the block is
6 A water proofing agent changes the angle of contact 12 The correct curve between the height or depression
(a) from acute to 90° h of liquid in a capillary tube and its radius r is
(b) from obtuse to 90° h h
(c) from an acute to obtuse value
(d) from an obtuse to acute value
(a) (b)
7 A viscous fluid is flowing through a cylindrical tube.
The velocity distribution of the fluid is best r r
represented by the diagram h h

(a) (b) (c) (d)

r r
608 OBJECTIVE Physics Vol. 1

13 When two capillary tubes of different diameters are 22 A barometer kept in an elevator reads 76 cm when it
dipped vertically, the rise of the liquid is is at rest. If the elevator goes up with increasing
(a) same in both the tubes speed, the reading will be
(b) more in the tube of larger diameter (a) zero (b) 76 cm (c) > 76 cm (d) < 76 cm
(c) less in the tube of smaller diameter
23 A hole is made at the bottom of the tank filled with
(d) more in the tube of smaller diameter
water (density = 1000 kgm −3 ). If the total pressure at
14 The surface tension of liquid at its boiling point the bottom of the tank is three atmospheres
(a) becomes zero
(b) becomes infinity (1 atmosphere = 10 5 Nm−2 ), then the velocity of
(c) is equal to the value at room temperature efflux is nearest to
(d) is half to the value at the room temperature (a) 400 ms−1 (b) 200 ms−1
15 When the temperature is increased, the angle of (c) 600 m s−1 (d) 500 m s−1
contact of a liquid
(a) increases 24 Assuming that the atmosphere has the same density
(b) decreases anywhere as at sea level (ρ = 1.3 kgm−3 ) and g to be
(c) remains the same
(d) first increases and then decreases
constant (g = 10 ms −2 ). What should be the
approximate height of atmosphere?
16 The surface tension of a soap solution is
(p 0 = 1.01 × 10 5 Nm−2 )
2 × 10 −2 Nm −1 . To blow a bubble of radius 1 cm, the
(a) 6 km (b) 8 km (c) 12 km (d) 18 km
work done is
(a) 4π × 10−6 J (b) 8π × 10−6 J 25 Two capillary tubes P and Q are dipped in water.
(c) 12π × 10−6 J (d) 16π × 10−6 J The height of water level in capillary P is 2/3 to the
height in Q capillary. The ratio of their diameters is
17 A liquid rises in a vertical tube. The relation
between the weight of the liquid in the tube, surface (a) 2 : 3 (b) 3 : 2
tension of the liquid T and radius of the tube r is (c) 3 : 4 (d) 4 : 3
given by (if the angle of contact is zero) 26 In a surface tension experiment with capillary tube,
(a) w = πr 2T (b) w = 2πrT water rises upto 0.1 m. If the same experiment is
3 repeated on an artificial satellite, which is revolving
(c) w = 2r 2 πT (d) w = πr 3T
4 around the earth, water will rise in the capillary
18 If the angle of contact is less than 90°, the pressure
tube upto a height of
(a) 0.1 m
just inside the surface of a meniscus
(b) 0.2 m
(a) is less than atmospheric pressure
(c) 0.98 m
(b) is greater than atmospheric pressure
(d) full length of the capillary tube
(c) is same as the atmospheric pressure
(d) None of the above 27 Two parallel glass plates are dipped partly in the
19 If two glass plates are quite nearer to each other in
liquid of density d keeping them vertical. If the
water, then there will be force of distance between the plates is x, surface tension for
(a) attraction the liquid is T and angle of contact θ, then rise of
(b) repulsion liquid between the plates due to capillary will be
(c) attraction or repulsion T cos θ 2T cos θ 2T T cos θ
(d) Neither attraction nor repulsion (a) (b) (c) (d)
xd xdg xdg cos θ xdg
20 The density of ice is x g cm−3 and that of water is
y g cm−3 . When m gram of ice melts, then the 28 A balloon has volume of 2000 m3 . It is filled with
change in volume is hydrogen (ρ = 0.009 gL−1 ). If the density of air is
y−x . gL−1 , it can lift a total weight of
129
(a) m ( y − x ) (b)
m (a) 600 kg (b) 2400 kg
m m
(c) my ( y − x ) (d) − (c) 300 kg (d) 1800 kg
y x
29 A raft of mass M = 600 kg floats in calm water with
21 If a capillary tube is dipped into liquid and the levels 7 cm submerged. When a man stands on the raft, it is
of the liquid inside and outside are same, then the 1.4 cm more submerged, then the man's mass is
angle of contact is (a) 30 kg (b) 60 kg
(a) 120° (b) 90° (c) 45° (d) 30° (c) 90 kg (d) 120 kg
Fluid Mechanics 609

30 A piston of cross-sectional area 100 cm2 is used in a If the water pressure measured at the three points
7 A, B and C below the object are p A , p B and p C ,
hydraulic pressure to exert a force of 10 dyne on
respectively. Then,
the water. The cross-sectional area of the other
piston which supports an object having a mass of
2000 kg is
(a) 9.8 × 102 cm2 (b) 9.8 × 103 cm2
(c) 1.96 × 103 cm2 (d) 1.96 × 104 cm2
31 An ideal fluid flows through two pipes of circular
A B C
cross-section with diameters 2.5 cm and 3.75 cm
connected one after another. The ratio of the (a) p A > pB > pC (b) p A > pB < pC
velocities in the two pipes is (c) p A = pB = pC (d) p A = pB < pC
(a) 9 : 4 (b) 3 : 2 36 Which graph represents the variation of surface
(c) 3 : 2 (d) 2 : 3 tension with temperature over small temperature
32 The angle of contact at the interface of water-glass is ranges for water?
0°, ethyl alcohol-glass is 0°, mercury-glass is 140°
ST ST
and methyliodide-glass is 30°. A glass capillary is
put in a trough containing one of these four liquids. (a) (b)
It is observed that the meniscus is convex. The
x
liquid in the trough is Temp Temp
(a) water (b) ethylalcohol
(c) mercury (d) methyliodide ST ST
33 A uniformly tapered vessel is filled with a liquid of (c) (d)
density 900 kgm−3 . The force that acts on the base
Temp
of the vessel due to the liquid (excluding Temp
atmospheric force) is (g = 10 ms −2 )
37 A solid shell loses half of its weight in water. If
Area = 10−3 m2 relative density of shell is 5, then what fraction of
its volume is hollow?
3 2 1 4
0.4 m (a) (b) (c) (d)
5 5 5 5
38 A tank full of water has a small hole at its bottom.
2 × 10−3 m2
Let t1 be the time taken to empty the first half of the
tank and t 2 be the time needed to empty the rest
(a) 3.6 N (b) 7.2 N half of the tank, then
(c) 9.0 N (d) 12.6 N (a) t1 = t 2 (b) t1 > t 2
34 For the arrangement shown in the figure, the force (c) t1 < t 2 (d) t1 = 0.523t 2
at the bottom of the vessel is 39 An open U-tube contains mercury. When 11.2 cm of
water is poured into one of the arms of the tube,
Vacuum
1 cm2
how high does the mercury rise in the other arm
99 cm from its initial level?
(a) 0.82 cm (b) 1.35 cm
(c) 0.41 cm (d) 2.32 cm
ρw = 103 kgm−3 1cm
40 A small ball (mass m) falling under gravity in a
100 cm2 viscous medium experiences a drag force
proportional to the instantaneous speed u such that
(a) 200 N (b) 100 N (c) 20 N (d) 2 N
Fdrag = ku. Then, the terminal speed of ball within
35 An object of uniform density is allowed to float in viscous medium is
water kept in a beaker. The object has triangular k mg mg
cross-section as shown in the figure. (a) (b) (c) (d) None of these
mg k k
610 OBJECTIVE Physics Vol. 1

41 If a capillary tube of radius r is immersed in water, 49 In the figure shown,


the mass of water rise in capillary is M. If the radius p p0
of capillary be doubled, the mass of water rise in the
capillary will be ρ 3h h

(a) M/2 (b) M (c) 2M (d) 4M
42 The pressure of water in a pipe when tap is closed is
5.5 × 10 5 Nm −2 . When tap gets open, pressure
(a) p 0 > p (b) p > p 0 (c) p = p 0 (d) p = 0
reduces to 5 × 10 5 Nm −2 . The velocity with which 50 A boat having a length 3 m and breadth 2 m is
water comes out on opening the tap is floating on a lake. The boat sinks by 1 cm when a
(a) 10 ms−1 (b) 5 ms−1 man gets on it. The mass of the man is
(c) 20 ms−1 (d) 15 ms−1 (a) 60 kg (b) 72 kg (c) 52 kg (d) 65 kg
43 The level of water in a tank is 5 m high. A hole of 51 A small block of wood of relative density 0.5 is
2 submerged in water. When the block is released, it
area 1 cm is made at the bottom of the tank. The rate
of leakage of water from the hole is starts moving upwards, the acceleration of the block
(Take, g = 10 ms −2 ) is (Take, g = 10 ms −2 )
(a) 10−3 m3 s−1 (b) 10−4 m3 s−1 (a) 5 ms−2 (b) 10 ms−2 (c) 7.5 ms−2 (d) 15 ms−2
(c) 10 m3 s−1 (d) 10−2 m3 s−1
52 A raft of wood of mass 120 kg floats in water. The
44 Water is flowing through two horizontal pipes of weight that can be put on the raft to make it just
different diameters which are connected together. sink, should be (Take, d raft = 600 kgm −3 )
The diameters of the two pipes are 3 cm and 6 cm,
(a) 80 kg (b) 50 kg (c) 60 kg (d) 30 kg
respectively. If the speed of water in the narrower
tube is 4 ms −1 , then the speed of water in the wider 53 A stone of relative density K is released from rest on
tube is the surface of a lake. If viscous effects are ignored,
the stone sinks in water with an acceleration of
(a) 16 ms−1 (b) 1 m s−1 (c) 4 m s−1 (d) 2 m s−1
(a) g (1 − K ) (b) g (1 + K )
45 A block of wood floats in water with (4/5) th of its  1  1
(c) g 1 −  (d) g 1 + 
volume submerged. If the same block just floats in a  K  K
liquid, the density of the liquid is (in kgm −3 )
54 A body floats in a liquid contained
(a) 1250 (b) 600 (c) 400 (d) 800
in a beaker. The whole system
46 Water flows along a horizontal pipe of non-uniform shown in figure is falling under
cross-section. The pressure is 1 cm of Hg, where the gravity, the upthrust on the body
velocity is 35 cms −1. At a point, where the velocity is due to the liquid is
65 cms −1 , the pressure will be (a) zero
(b) equal to weight of the body in air
(a) 0.89 cm of Hg (b) 0.62 cm of Hg
(c) equal to weight of liquid displaced
(c) 0.5 cm of Hg (d) 1 cm of Hg
(d) equal to the weight of the immersed part of the body
47 Three liquids of equal masses are taken in three
55 The relative density of ice is 0.9 and that of sea
identical cubical vessels A, B and C . Their densities
water is 1.125. What fraction of the whole volume
are ρ A , ρB and ρC respectively but ρ A < ρB < ρC .
of an iceberg appears above the surface of the sea?
The force exerted by the liquid on the base of the
(a) 1/ 5 (b) 2 / 5
cubical vessel is (c) 3 / 5 (d) 4 / 5
(a) maximum in vessel C (b) minimum in vessel C
(c) the same in all the vessels (d) maximum in vessel A 56 A metallic sphere floats in immiscible mixture of
water (density 10 3 kgm−3 ) and a liquid
48 An object weights m1 in a liquid of density d 1 and
that in liquid of density d 2 is m 2 . The density of the (density 8 × 10 3 kgm−3 ) such that its (2/3)part is in
object is water and (1/3) part in the liquid. The density of the
m 2d 2 − m1d1 m1d1 − m 2d 2 metal is
(a) (b) 5000 10000
m 2 − m1 m 2 − m1 (a) kgm−3 (b) kgm−3
m 2d1 − m1d 2 m1d 2 − m 2d1 3 3
(c) (d) (c) 5000 kgm−3 (d) 2000 kgm−3
m1 − m 2 m1 − m 2
Fluid Mechanics 611

57 In the given figure, the velocity v 3 will be 63 A vessel whose bottom has round holes with
v2 = 2 ms−1 diameter of 1.0 mm is filled with water. The
A1 = 0.2 m2 maximum height to which the water can be filled
A2 = 0.2 m2
without leakage is (Take, surface tension of water
v1 = 4 ms−1 = 75 dyne cm −1, g = 1000 cms −2 )
v3 (a) 10 cm (b) 7.5 cm (c) 5 cm (d) 3 cm
A3 = 0.4 m2
64 If a water drop is kept between two glass plates,
−1 −1 then its shape is
(a) 2 ms (b) 4 ms
(c) 1 ms−1 (d) 3 ms−1
(a) (b)
58 A water tank standing on the floor has two small
holes punched in the vertical wall one above the
other. The holes are 2.4 cm and 7.6 cm above the
floor. If the jet of water from the holes hit the floor
(c) (d) None of these
at the same point, then the height of water in the
tank is
(a) 10 cm (b) 5 cm (c) 20 cm (d) 4.8 cm 65 Two small drops of mercury each of radius r form a
59 A body of volumeV and density ρ is initially single large drop. The ratio of surface energy before
submerged in a non-viscous liquid of density σ (> ρ). and after this change is
If it rises by itself through a height h in the liquid, its (a) 2 : 22/3 (b) 22/3 : 1 (c) 2 : 1 (d) 1 : 2
kinetic energy will 66 A non-viscous liquid is flowing through a frictionless
(a) increase by hV (σ − ρ)g (b) increase by hV (ρ + σ )g duct of varying cross-section as shown in figure.
hVρg hVρg
(c) increase by (d) decrease by
σ σ X
O
60 The pressure of the gas in a cylindrical chamber is
p 0 . The vertical force exerted by the gas on its
hemispherical end is Which of the following graph represents the
variation of pressure p along the axis of tube?
p p
r

(a) (b)
X X
Gas
p p

(a) p 0r 2 (b) 4p 0 πr 2 (c) (d)


(c) 2p 0 πr 2
(d) p 0 πr 2 X X

61 Water rises in a capillary tube to a certain height 67 There is hole of area a at the bottom of a cylinder of
such that the upward force due to surface tension is area A. Water is filled upto a height h and water
balanced by 75 × 10 −4 N force due to the weight of flows out in t second. If water is filled to a height 4h,
the liquid. If the surface tension of water is it will flow out in time
6 × 10 −2 N m−1, the inner circumference of the (a)
t
(b) 2t (c) 4t (d)
t
capillary must be 4 2
(a) 1.25 × 10−2 m (b) 0.50 × 10−2 m 68 A container has a small hole at its bottom. Area of
(c) 6.5 × 10 −2
m (d) 12.5 × 10 −2
m cross-section of the hole is A1 and that of the
container is A2 . Liquid is poured in the container at a
62 Water rises to a height of 30 mm in a capillary tube. constant rate Q m3s −1. The maximum level of liquid
If the radius of the capillary tube is made 3/4 of its in the container will be
previous value. The height to which the water will Q2 Q2 Q Q2
rise in the tube is (a) (b) (c) (d)
2g AA
1 2 2g A12 2g AA
1 2 2g A22
(a) 30 mm (b) 20 mm (c) 40 mm (d) 10 mm
612 OBJECTIVE Physics Vol. 1

69 Air stream flows horizontally past an aeroplane wing 75 A metal sphere connected by a string is dipped in a
of surface area 4 m2 . The speed of air over the top liquid of density ρ as shown in figure. The pressure
surface is 60 ms −1 and under the bottom surface is at the bottom of the vessel will be (p 0 = atmospheric
40 ms −1. The force of lift on the wing is pressure)
(Take, density of air = 1 kgm−3 )
(a) 800 N (b) 1000 N (c) 4000 N (d) 3200 N
70 Water from a tap emerges vertically down with an h
initial speed of 1.0 ms −1 . The cross-sectional area of
tap is 10 −4 m2 . Assume that, the pressure is constant
throughout the stream of water and that the flow is (a) p = p 0 + ρgh (b) p > p 0 + ρgh
steady. The cross-sectional area of the stream 0.15 m (c) p < p 0 + ρgh (d) p 0
below the tap is
76 An air bubble of radius 1 mm is formed inside water
(a) 5.0 × 10−4 m2 (b) 1.0 × 10−5 m2
at a depth 10 m below free surface (where, air
(c) 5.0 × 10−5 m2 (d) 2.0 × 10−5 m2
71 There are two holes O1 and O 2 in a tank of height H.
pressure is 10 5 Nm −2 ). The pressure inside the
The water emerging from O1 and O 2 strikes the bubble is (surface tension of water = 7 × 10 −2 Nm−1)
ground at the same points as shown in figure. Then, (a) 2.28 × 105 Nm−2 (b) 2.0028 × 105 Nm−2
(c) 2.14 × 105 Nm−2 (d) 2.0014 × 105 Nm−2

h2
h1
O1
77 A block of mass 4 kg and volume 5 × 10 −4 m 3 is
H suspended by a spring balance in a lift which is
O2
accelerating. The apparent weight shown by the
spring balance is 3 kg. Now, the block is immersed
(a) H = h1 + h 2 (b) H = h 2 − h1
in water in a container inside the lift. The apparent
weight (in kg) shown by the spring balance is
(c) H = h1h 2 (d) None of these (a) 2.375 (b) 0.25 (c) 2.5 (d) 3125
.
72 Two rain drops of same radii r falling with terminal 78 A cubical block of wood of specific gravity 0.5 and
velocity v merges and form a bigger drop of radius R. chunk of concrete of specific gravity 2.5 are
The terminal velocity of the bigger drop will be fastened together. The ratio of the mass of wood to
R R2 the mass of concrete, which makes the combination
(a) v (b) v (c) v (d) 2v
r r2 to float with its entire volume submerged under
73 A cubical block of steel of each side equal to l is water is
floating on mercury in a vessel. The densities of (a) 1/5 (b) 1/3 (c) 3/5 (d) 2/3
steel and mercury are ρ s and ρm . The height of the 79 Two identical cylindrical vessels, each of base area
block above the mercury level is given by A, have their bases at the same horizontal level.
 ρ   ρ   ρ   ρ  They contain a liquid of density ρ. In one vessel, the
(a) l 1 + s  (b) l 1 − s  (c) l 1 + m  (d) l 1 − m 
 ρm   ρm   ρs   ρs  height of the liquid is h1 and in the other h 2 (> h1 ).
When the two vessels are connected, the work done
74 A cylindrical tank contains water up to a height H. If by gravity in equalising the levels is
the tank is accelerated upwards with acceleration a,
(a) 2ρAg (h 2 − h1 )2 (b) ρAg (h 2 − h1 )2
the pressure at the point A is p 1. If the tank is
1 1
accelerated downwards with acceleration a, the (c) ρAg (h 2 − h1 )2 (d) ρAg (h 2 − h1 )2
pressure at A is p 2 .Then, 2 4
80 A small ball of density ρ is immersed in a liquid of
density σ (σ > ρ) to a depth h and then released. The
height above the surface of water up to which the
H ball will jump is
σh σ 
A (a) (b)  − 1  h
ρ ρ 
 σh  ρh
(a) p1 < p 2 (b) p1 = p 2 (c) 1 − h (d)
(c) p1 > p 2 (d) Data insufficient  ρ σ
Fluid Mechanics 613

81 A U-tube of base length l filled with same volume of 85 The liquid inside the container has density ρ. Choose
two liquids of densities ρ and 2ρ is moving with an the correct option.
acceleration a on the horizontal plane. If the height
A B
difference between the two surfaces (open to
atmosphere) becomes zero, then height h is given by
a=g
L

ρ

h C L D
a
(a) p A − pC = 2ρgL (b) pC − pB = 2ρgL
(c) pC − pD = ρgL (d) p A − pD = 0
l 86 A tank contains water on top of mercury as shown in
al 3al 2al al figure. A cubical block of side 10 cm is in equilibrium
(a) (b) (c) (d)
g 2g 3g 2g inside the tank. The depth of the block inside
mercury is (Take, relative density of the material of
82 In a U-tube experiment, a column AB of water is
block = 8.56, relative density of mercury = 13.6)
balanced by a column CD of paraffin. The relative
density of paraffin is
Water

C A Cubical block
h1 h2
Mercury
D B
(a) 6 cm (b) 5 cm (c) 7 cm (d) 8 cm
87 If T is the surface tension of a liquid, the energy
h h h − h1 h2 needed to break a liquid drop of radius R into 64
(a) 2 (b) 1 (c) 2 (d)
h1 h2 h1 h1 + h 2 drops is
(a) 6πR 2T (b) πR 2T (c) 12πR 2T (d) 8πR 2T
83 A U-tube of uniform cross-section shown in figure is
partially filled with liquid I. Another liquid II which 88 Two solid spheres of same metal but of mass M and
does not mix with I is poured into one side. The 8M fall simultaneously on a viscous liquid and their
liquid levels of the two sides are found to be same, terminal velocities are v and nv, then value of n is
while the level of liquid I has risen by 2 cm. If the (a) 16 (b) 8 (c) 4 (d) 2
specific gravity of liquid I is 1.1, then specific 89 A cubical block is floating in a
g
gravity of liquid II must be liquid with half of its volume 3
immersed in the liquid. When
the whole system accelerates
upwards with acceleration of
I g/3, the fraction of volume
II
immersed in the liquid will be
(a) 1/2 (b) 3 / 8 (c) 2 /3 (d) 3 / 4
90 A cylindrical vessel is filled
y
with a liquid upto a height
(a) 1. 2 (b) 1.1 (c) 1. 3 (d) 1. 0 H. A small hole is made in H
the vessel at a distance y
84 A closed rectangular tank is A D
below the liquid surface as
completely filled with water and a x
shown in figure. The liquid
is accelerated horizontally with
emerging from the hole strike the ground at distance
an acceleration towards right. B C
x, then
Pressure is (a) x is equal, if hole is at depth y or H − y
(i) maximum at and (ii) minimum at (b) x is maximum for y = H /2
(a) (i) B (ii) D (b) (i) C (ii) D (c) Both (a) and (b ) are correct
(c) (i) B (ii) C (d) (i) B (ii) A (d) Both (d ) and (d ) are incorrect
614 OBJECTIVE Physics Vol. 1

91 Two capillary of lengths L and 2L and of radius 96 A body of density ρ is dropped from rest from a
R and 2R are connected in series. The net rate of height h into a lake of density σ (σ > ρ). The
flow of fluid through them will be (given rate of the maximum depth to which the body sinks inside the
flow through single capillary, X = πpR 4 / 8ηL) liquid is (neglect viscous effect of liquid)
8 9 5 7 hρ hσ hρ hσ
(a) X (b) X (c) X (d) X (a) (b) (c) (d)
9 8 7 5 σ −ρ σ −ρ σ ρ

92 A soap bubble is blown with the help of a 97 A candle of diameter d is floating on a liquid in a
mechanical pump at the mouth of a tube. The pump cylindrical container of diameter D (D > > d ) as
produces a constant increase per minute in the shown in figure. If it is burning at the rate of
radius of the bubble, irrespective of its internal 2 cmh −1, then the top of the candle will
pressure. The graph between the excess pressure
inside the soap bubble and time t will be
L
p p

L
(a) (b) d

t t D

p p (a) remain at the same height


(b) fall at the rate 1 cmh −1
(c) (d) (c) fall at the rate of 2 cmh −1
(d) go up at the rate of 1 cmh −1
t t 98 A spherical object of mass 1kg and radius 1m is
falling vertically downward inside a viscous liquid in
93 Two soap bubbles of radii r1 and r 2 equal to 4 cm a gravity free space. At a certain instant, the
and 5 cm are touching each other over a common velocity of the sphere is 2 m s −1 . If the coefficient of
surface S1S 2 (shown in figure). Its radius will be 1
viscosity of the liquid is SI units, then velocity of

S1 ball will become 0.5 ms −1 after a time is
(a) ln (4) s (b) 2 ln (4) s
4 cm 5 cm
(c) 3 ln (4) s (d) 3 ln (2) s
99 A wooden block of mass 8 kg is tied to a string
S2
attached to the bottom of the tank. In the
equilibrium, the block is completely immersed in
(a) 4 cm (b) 20 cm (c) 5 cm (d) 4.5 cm water. If relative density of wood is 0.8 and
g = 10 ms −2, the tension T in the string is
94 Two substances of relative densities ρ1 and ρ 2 are
mixed in equal volume and the relative density of (a) 120 N (b) 100 N (c) 80 N (d) 20 N
mixture is 4. When they are mixed in equal masses, 100 A metal ball immersed in alcohol weighs w 1 at
the relative density of the mixture is 3. The values 0° C and w 2 at 59° C. The coefficient of cubical
of ρ1 and ρ 2 are
expansion of the metal is less than that of alcohol.
(a) 6 and 2 (b) 3 and 5
(c) 12 and 4 (d) None of these
Assuming that the density of the metal is large
compared to that of alcohol, it can be shown that
95 A large block of ice 10 cm thick with a vertical hole (a) w1 > w 2 (b) w1 = w 2
drilled through it is floating in a lake. The minimum (c) w1 < w 2 (d) w1 = (w1 /2)
length of the rope required to scoop out a bucket full 101 The surface energy of a liquid drop is S. It is sprayed
of water through the hole is (Take, density of ice into 1000 equal droplets, then its surface energy
= 0.9 g cm−3 ) becomes
(a) 0.5 cm (b) 1.0cm (c) 12
. cm (d) 1.8 cm (a) S (b) 10 S (c) 100 S (d) 1000 S
Fluid Mechanics 615

102 An open tank containing non-viscous liquid to a 108 A sphere of solid material of specific gravity 8 has a
height of 5 m is placed over the ground. A heavy concentric spherical cavity and just sinks in water.
spherical ball falls from height 40 m over the ground Then, the ratio of the radius of the cavity to the
in the tank. Ignoring air resistance, find the height to outer radius of the sphere must be
which ball will go back. Collision between ball and 3 3 3
3 5 7 2
bottom of tank is perfectly elastic (a) (b) (c) (d) 3
2 2 2 7
(a) 45 m (b) 35 m
(c) 40 m (d) 20 m 109 A cubical block of side 10 cm floats at the interface
103 A large open tank has two holes in the wall. One is a of an oil and water. The pressure above that of
square hole of side L at a depth y from the top and atmosphere at the lower face of the block is
the other is a circular hole of radius R at a depth 4y
from the top. When the tank is completely filled
with water, the quantities of water flowing out per Oil 0.6 gcc–1
10 cm
second from holes are the same. Then, R is equal to
2 cm
(a) L/ 2π (b) 2πL 10 cm 1.0 gcc–1
(c) L (d) L/2π
104 A piece of steel has a weight w in air, w 1 when
completely immersed in water and w 2 when (a) 200 Nm−2 (b) 680 N m−2
completely immersed in an unknown liquid. The (c) 400 Nm−2 (d) 800 Nm−2
relative density (specific gravity) of liquid is 110 A liquid stands at the plane level in U-tube when at
w − w1 w − w2 w1 − w 2 w1 − w 2 rest. If areas of cross-section of both the limbs are
(a) (b) (c) (d)
w − w2 w − w1 w − w1 w − w2 equal, what will be the difference in heights h of the
105 Two cylinders of same cross-section and length L but liquid in the two limbs of U-tube, when the system
made of two materials of densities d 1 and d 2 are is given an acceleration a in horizontal direction
connected together to form a cylinder of length 2L. towards right as shown in figure?
The combination floats in a liquid of density d with
a length L/2 above the surface of the liquid. If
d 1 < d 2 , then a
3 d d d H
(a) d1 < d (b) > d1 (c) > d1 (d) d1 > d
4 2 4 4
106 A block of wood is floating on the surface of water
L
in a beaker. The beaker is covered with a bell jar
2
and the air is evacuated. What will happen to the g L La L2 a Lg
(a) (b) (c) (d)
block? a H g H g a
(a) Sink a little (b) Rise a little
(c) Remain unchanged (d) Sink completely 111 A capillary tube is dipped in a liquid. Let pressure at
points A, B and C be p A , p B , p C respectively, then
107 A beaker containing water is kept
on a spring scale. The mass of S2
water and beaker is 5 kg. A block B
of mass 2 kg and specific gravity C
A
10 is suspended by means of
thread from a spring balance as
shown in figure.
The readings of scales S1 and S 2
are respectively
(Take, g = 10 ms −2 ) S1 (a) p A = pB = pC (b) p A = pB < pC
(a) 52 N and 20 N (c) p A = pC < pB (d) p A = pC > pB
(b) 50 N and 18 N 112 The volume of an air bubble becomes three times as
(c) 52 N and 18 N it rises from the bottom of a lake to its surface.
(d) 52 N and 22 N Assuming temperature to be constant and
616 OBJECTIVE Physics Vol. 1

atmospheric pressure to be 75 cm of Hg and the 117 A thread is tied slightly loose to a wire frame as
density of water to be 1/10 of the density of the shown in figure and the frame is dipped into a soap
mercury, the depth of the lake is solution and taken out. The frame is completely
(a) 5 m (b) 10 m covered with the film.
(c) 15 m (d) 20 m
Frame
113 Figure shows how the stream of A
water emerges from a faucet
necks down as it falls. The area A0
B
changes from A0 to A through a h
fall of h. At what rate does the A Thread
water flow from the tap?
2ghA2 ghA2
When A is pricked
(a) A0 (b) 2A0 (a) thread will become concave on seeing from side A
A02 − A2 A02 − A2 (b) thread will become concave on seeing from side B
(c) thread will become straight
gh ghA02
(c) A0 (d) 2A (d) thread will remain as it is
2 A02 − A2
118 A tank filled with water has two taps to exhaust
114 A pump is designed as a horizontal cylinder with a and pour. A hollow spherical ball is half submerged
piston area A and an outlet orifice arranged near the in water. Through one tap, water is taken out and
axis of the cylinder. Find the velocity of outflow of through another tap, a liquid of density double the
liquid from pump, if the piston moves with a density of water is poured in tank such that volume
constant velocity under the action of a constant force of liquid in tank remains constant. Sphere will
F and the density of liquid is ρ.

F 2F Aρ Aρ
(a) (b) (c) (d)
Aρ Aρ F 2F (a) go down (b) go up
(c) maintain same height (d) sink to bottom
115 If cross-sectional area of limb I is A1 and that of limb
119 A uniform long tube is bent into a circle of radius R
II is A2, then velocity of the liquid in the tube will
and it lies in vertical plane. Two liquids of same
be (cross-sectional area of tube is very small)
volume but densities ρ and δ fill half the tube. The
I angle θ is
II

R
x
y δ
v θ R

Tube ρ
A1
(a) 2g (x − y ) (b) 2g (x − y )  ρ − δ  ρ
A2 (a) tan−1   (b) tan−1  
A2  ρ + δ  δ
(c) 2g (x − y ) (d) None of these
A1  δ  ρ + δ
(c) tan−1   (d) tan−1  
 ρ  ρ − δ
116 Two capillaries of same length and radii in the ratio1:2
are connected in series. A liquid flows through them in 120 A container has two immiscible liquids of densities
streamlined condition. If the pressure across the two ρ1 and ρ 2 (> ρ1 ). A capillary tube of radius r is
extreme ends of the combination is 1 m of water, then inserted in the liquid, so that its bottom reaches upto
pressure difference across first capillary is the denser liquid. The denser liquid rises in the
(a) 9.4 m (b) 4.9 m (c) 0.49 m (d) 0.94 m capillary and attains a height h from the interface
Fluid Mechanics 617

of the liquids, which is equal to the column v2 v2


(a) (b) + h0
length of the lighter liquid. Assuming angle of contact 2g 2g
to be zero, the surface tension of denser liquid is v2 v2
(c) − h0 (d) − h0
2g 4g

ρ1 124 A solid ball of density half that of water falls freely


h
under gravity from a height of 19.6 m and then
enters water. How much time will it take to come
ρ2 again to the water surface? Neglect air resistance
and viscosity effects in water. (Take, g = 9.8 ms −2 )
ρ2 rgh (a) 4 s (b) 8 s
(a) 2 πr ρ2 gh (b)
2 (c) 6 s (d) 2 s
r
(c) (ρ2 − ρ1 )gh (d) 2πr (ρ2 − ρ1 )gh 125 A large tank is filled with water (density
2 = 10 3 kgm−3 ). A small hole is made at a depth 10 m
121 A ball of mass 1 kg falls from a height of 5m above below water surface. The range of water flowing out
the free surface of water. The relative density of the of the hole is R on ground. What extra pressure must
solid ball is s = 2/3. The ball travels a distance of 2m be applied on the water surface, so that the range
under water and becomes stationary. The work done become 2R ? (Take, 1 atm = 10 5 Pa and g =10 ms −2 )
by the resistive forces of water is
(a) −50 J (b) −20 J (c) −40 J (d) −30 J
122 Under isothermal condition, two soap bubbles of 10m
radii r1 and r 2 coalesce to form a single bubble of
radius r. The external pressure is p 0 . The surface
tension of the soap in terms of the given parameters
is
2p 0 (r 3 − r13 − r23 ) p 0 (r 3 − r13 − r23 ) R
(a) (b)
4(r12 + r22 − r 2 ) 4 (r12 + r22 − r 2 )
(a) 1 atm (b) 2 atm
p (r 3 + r12 + r23 ) (c) 4 atm (d) 3 atm
(c) 0 2 (d) None of these
4(r1 + r22 + r 2 ) 126 Air is blown through a pipe AB at a rate of
123 A bent tube is lowered into the stream as shown in 15 Lmin. −1 The cross-sectional area of the broad
the figure below. The velocity of the stream relative portion of the pipe AB is 2 cm2 and that of the
to the tube equal to v. The closed upper end of the narrow portion is 0.5 cm2 . The difference in water
tube located at the height h 0 . To what height, h will level h is (Take, density of air = 1.32 kg m−3 )
the water jet spurt?
A
2
B
h

h0 h
Water

1 (a) 16 mm (b) 1.5 mm


(c) 10 mm (d) 3.2 mm
OBJECTIVE Physics Vol. 1

(B) Medical entrance special format questions


Assertion and reason
Direction (Q. Nos. 1-4) These questions consist of two 2 Which of the following statement is true in case when
statements each printed as Assertion and Reason. While two water drops coalesce and make a bigger drop?
answering these questions, you are required to choose any (a) Energy is released.
one of the following four responses. (b) Energy is absorbed.
(c) The surface area of the bigger drop is greater than the
(a) If both Assertion and Reason are correct and Reason is sum of the surface areas of both the drops.
the correct explanation of Assertion.
(d) The surface area of the bigger drop is same that of the
(b) If both Assertion and Reason are correct but Reason is sum of the surface areas of both the drops.
not the correct explanation of Assertion.
(c) If Assertion is correct but Reason is incorrect. 3 Which of the following statement(s) is/are correct?
(d) If Assertion is incorrect but Reason is correct. I. An ice ball is floating in water. Some stone pieces
1 Assertion If angle of contact is 90°, then liquid are embedded inside the ice. When ice will melt,
will neither rise nor fall in a capillary. level of water will fall.
Reason When angle of contact is 90°, surface is II. In floating condition, stone pieces will displace
neither convex nor concave inside the capillary. It is more liquid compared to the condition when they
flat. sink.
(a) Only I
2 Assertion When an ideal fluid flows through a pipe
(b) Only II
of non-uniform cross-section, then pressure is more (c) Both I and II
at that section, where area is more, if the pipe is (d) None of the above
horizontal.
Reason According to Bernoulli’s theorem, speed at Match the columns
broader cross-section will be less.
1 A tube is inverted in a mercury vessel as shown in
3 Assertion A solid is floating in a liquid of density
figure. If pressure p is increased, then
ρ1. When the solid melts its density becomes ρ 2 in p
liquid state.
If ρ1 > ρ 2 level of liquids will increase after h
melting. O
Reason In liquid state, volume always increases
after a solid melts.
4 Assertion A solid sphere and a hollow sphere both
of same material are immersed in a liquid, then Column I Column II
change in weight in both the spheres will be same. (A) Height h (p) will increase
Reason Upthrust depends upon the volume of the (B) Pressure at O (q) will decrease
solid immersed not the mass. (C) Pressure at 1cm above O (r) will remain same

Statement based questions Match the columns and marks the correct option
from the codes given below.
1 Which of the following statement is incorrect Codes
regarding surface tension? A B C
(a) Dancing of a camphor piece over the surface of water. (a) p q r
(b) Small mercury drop itself becomes spherical. (b) p r q
(c) A liquid surface comes at rest after stirring. (c) q r r
(d) Mercury does not wet the glass vessel. (d) r p q
Fluid Mechanics 619

2 In the figure shown velocity of liquid which comes 3 A cube is floating in liquid as shown in figure.
out is v, time of liquid to fall to ground is t and Match the following columns and mark the correct
range on ground is R. If the vessel is taken to a option from the codes given below.
mountain, match the following (consider all cases
which might possible) columns and mark the correct x
option from the codes given below.

Column I Column II
R (A) If density of liquid decreases, x (p) increase
Column I Column II will
(A) v (p) will increase (B) If size of cube is increased, x will (q) decrease
(B) t (q) will decrease (C) If the whole system is (r) remain same
accelerated upwards, x will
(C) R (r) will remain same
Codes
Codes
A B C A B C
A B C A B C
(a) p p r (b) q r p
(a) p q r (b) q p r
(c) r p r (d) p r p
(c) p r q (d) q r p

(C) Medical entrances’ gallery


Collection of questions asked in NEET & various medical entrance exams
1 A capillary tube of radius r is immersed in water and
water rises in it to a height h. The mass of the water
in the capillary tube is 5 g. Another capillary tube of Water
radius 2r is immersed in water. The mass of water 15 cm 20 cm
Oil
that will rise in this tube is [NEET 2020]
(a) 5.0 g (b) 10.0 g
(c) 20.0 g (d) 2.5 g (a) 1200 kgm −3 (b) 750 kgm −3
2 A liquid does not wet the solid surface, if the angle (c) 1000 kgm −3 (d) 1333 kgm −3
of contact is [NEET 2020]
(a) equal to 45° 5 Two small spherical metal balls having equal masses
(b) equal to 60° are made from materials of densities ρ1 and
(c) greater than 90° ρ 2 ( ρ1 = 8 ρ 2 ) and have radii of 1 mm and 2 mm,
(d) zero respectively. They are made to fall vertically (from
rest) in viscous medium whose coefficient of
3 A barometer is constructed using a liquid (density viscosity equals η and whose density is 0.1 ρ 2 . The
= 760 kg/m 3 ). What would be the height of the ratio of their terminal velocities would be
liquid column, when a mercury barometer reads [NEET (Odisha) 2019]
76 cm? (Take, density of mercury = 13600 kg/m 3 ) 79 19 39 79
(a) (b) (c) (d)
[NEET 2020] 72 36 72 36
(a) 1.36 m (b) 13.6 m 6 If a small orifice is made at a height of 0.25 m from
(c) 136 m (d) 0.76 m the ground, the horizontal range of water stream will
4 In a U-tube as shown in the figure, water and oil are be [AIIMS 2019]
in the left side and right side of the tube,
respectively. The heights from the bottom for water
1m
and oil columns are 15 cm and 20 cm, respectively.
The density of the oil is (Take, ρ water = 1000 kg/m 3 ) 0.25 m
[NEET (Odisha) 2019]
(a) 46.5 cm (b) 56.6 cm (c) 76.6 cm (d) 86.6 cm
620 OBJECTIVE Physics Vol. 1

7 Determine the pressure difference in tube of 13 A solid floats with (1/4)th of its volume above the
non-uniform cross-sectional area as shown in figure. surface of water, the density of the solid is
∆p = ?, d1 = 5 cm, v1 = 4m/s, d 2 = 2 cm, v 2 = ? [AIIMS 2019] [JIPMER 2018]
(a) 750 kg m−3 (b) 650 kg m−3
(c) 560 kg m−3 (d) 450 kg m−3

d1 v1 d2 v2
14 A rain drop of radius 0.3 mm has a terminal velocity
in air 1 ms −1. The viscosity of air is 18 × 10 −5 poise.
Find the viscous force on the rain drops. [JIPMER 2018]
(a) 2.05 × 10−7 N (b) 1.018 × 10−7 N
∆p
(c) 1.05 × 10−7 N (d) 2.058 × 10−7 N
(a) 304200 Pa (b) 304500 Pa 15 At what speed will the velocity of a stream of water
(c) 302500 Pa (d) 303500 Pa be equal to 20 cm of mercury column?
8 Assertion Sometimes insects can walk on water. (Take, g = 10 ms −2 ) [JIPMER 2018]
Reason The gravitational force on insect is balanced (a) 6.4 ms−1 (b) 7.3756 ms−1
by force due to surface tension. [AIIMS 2019] (c) 6.4756 ms−1 (d) None of these
(a) Both Assertion and Reason are correct and Reason is
16 Water flows through a horizontal pipe of variable
the correct explanation of Assertion.
(b) Both Assertion and Reason are correct but Reason is not cross-section at the rate of 20 L min −1. What will be
the correct explanation of Assertion. the velocity of water at a point, where diameter is
(c) Both Assertion and Reason are incorrect. 4 cm? [JIPMER 2018]
(d) Assertion is incorrect but Reason is correct. (a) 0.2639 ms−1 (b) 0.5639 ms−1
9 Assertion Water drops take spherical shape when (c) 0.4639 ms−1 (d) 0.3639 ms−1
falling freely. 17 The work done in blowing a soap bubble of surface
Reason Water has minimum surface tension among tension 0.06 Nm−1 from 2 cm radius to 5 cm radius
all liquids. [AIIMS 2019] is [JIPMER 2018]
(a) Both Assertion and Reason are correct and Reason is (a) 0.004168 J (b) 0.003168 J
the correct explanation of Assertion. (c) 0.003158 J (d) 0.004568 J
(b) Both Assertion and Reason are correct but Reason is not
the correct explanation of Assertion. 18 A U-tube with both ends open to the atmosphere, is
(c) Both Assertion and Reason are incorrect. partially filled with water. Oil, which is immiscible
(d) Assertion is incorrect but Reason is correct. with water, is poured into one side until it stands at
10 In an isothermal process, 2 water drops of radius a distance of 10 mm above the water level on the
1 mm are combined to form a bigger drop. Find the other side. Meanwhile the water rises by 65 mm
energy change in this process, if T = 01
. N/m. from its original level (see diagram). The density of
[AIIMS 2019] the oil is [NEET 2017]
(a) 1 µJ (b) 0.5 µJ (c) 0.25 µJ (d) 0.75 µJ Pa Pa
11 A small sphere of radius r falls from rest in a F
A
10 mm
viscous liquid. As a result, heat is produced due to E Final water level
viscous force. The rate of production of heat when 65 mm
Oil D Initial water level
the sphere attains its terminal velocity, is
proportional to [NEET 2018] 65 mm
B C
(a) r 5 (b) r 2 (c) r 3 (d) r 4
Water
12 Find density of ethyl alcohol. [JIPMER 2018]

(a) 650 kg m−3 (b) 425 kg m−3


10 cm 12 cm (c) 800 k g m−3 (d) 928 kg m−3
Water Ethyl alcohol
19 An ice-berg of density 900 kgm− 3 is floating in
water of density 1000 kgm− 3 . The percentage of
Hg volume of ice-berg outside the water is [JIPMER 2017]
−3 (a) 20% (b) 35% (c) 10% (d) 11%
(a) 0.83 gcm (b) 0.5 gcm (c) 1.83 gcm −3 (d) 0.12 gcm −3
−3
Fluid Mechanics 621

20 A liquid is flowing into a tube of truncated cone on the roof and the direction of the force will be
shape. Identify the correct statement from the (Take, ρ air = 1.2 kgm−3 ) [CBSE AIPMT 2015]
following. [JIPMER 2017] (a) 4.8 × 105 N, downwards
(a) The speed is high at the wider end and low at the (b) 4.8 × 105 N, upwards
narrow end.
(b) The speed is low at the wider end and high at the (c) 2.4 × 105 N, upwards
narrow end. (d) 2.4 × 105 N, downwards
(c) The speed is same at both ends in a streamline flow. 26 The approximate depth of an ocean is 2700 m. The
(d) The liquid flows with uniform velocity in the tube.
compressibility of water is 45.4 × 10 −11 Pa −1 and
21 Two soap bubbles coalesce. It is noticed that, while density of water is 10 3 kgm−3 . What fractional
joined together, the radii of the two bubbles are a
compression of water will be obtained at the bottom
and b, where a > b . Then, the radius of curvature of
interface between the two bubbles will be of the ocean? [CBSE AIPMT 2015]
[JIPMER 2017] (a) 0.8 × 10−2 (b) 1 × 10−2
−2
(a) a − b (b) a + b (c)
ab
(d)
ab (c) 1.2 × 10 (d) 1.4 × 10−2
(a − b ) (a + b )
27 Determine the height above the dashed line XX′
22 A rectangular film of liquid is extended from attained by the water stream coming out through the
(4 cm × 2cm ) to (5cm × 4 cm ). If the work done is hole situated at point B in the diagram given below.
3 × 10 −4 J, the value of the surface tension of the Given that h = 10 m, L = 2 m and α = 30 °.
liquid is [NEET 2016] [AIIMS 2015]

(a) 0.250 Nm−1 (b) 0.125 Nm−1


(c) 0.2 Nm−1 (d) 8.0 Nm−1 h
23 Three liquids of densities ρ1, ρ 2 and ρ 3 ( with L
α=30°
ρ1 > ρ 2 > ρ 3 ), having the same value of surface X′ B X

tension T, rise to the same height in three identical


(a) 10 m (b) 7.1 m
capillaries. The angles of contact θ 1, θ 2 and θ 3 obey
(c) 5 m (d) 3.2 m
[NEET 2016]
π 28 A water drop of radius 1 cm is broken into 1000
(a) > θ1 > θ 2 > θ 3 ≥ 0 equal droplets. What would be the gain in surface
2
π energy, if the surface tension of water is
(b) 0 ≤ θ1 < θ 2 < θ 3 <
2 0.075 Nm −1? [UK PMT 2015]
π (a) 16.96 × 10−4 J (b) 8.48 × 10−4 J
(c) < θ1 < θ 2 < θ 3 < π
2 (c) 4.24 × 10−4 J (d) 2.12 × 10−4 J
π
(d) π > θ1 > θ 2 > θ 3 >
2 29 The lower end of a capillary tube is dipped into water
and it is seen that the water rises through 7.5 cm in
24 Two non-mixing liquids of densities ρ and nρ(n > 1)
capillary tube. What would be the radius of capillary
are put in a container. The height of each liquid is h. tube, if surface tension of water is 7.5 × 10 −2 Nm −1?
A solid cylinder of length L and density d is put in The contact angle between water and glass is 0° and
this container.
g = 10 ms −2 . [UK PMT 2015]
The cylinder floats with its axis vertical and length
(a) 0.2 cm (b) 0.1 cm
pL (p < 1) in the denser liquid. The density d is equal
(c) 0.4 mm (d) 0.2 mm
to [NEET 2016]
30 A soap bubble of diameter a is produced using the
(a) [2 + (n + 1)p] ρ (b) [2 + (n − 1)p] ρ
soap solution of surface tension T. Find the energy
(c) [1 + (n − 1)p] ρ (d) [1 + (n + 1)p] ρ required to double the radius of the bubble without
25 A wind with speed 40 ms −1 blows parallel to the change of temperature. [EAMCET 2015]
roof of a house. The area of the roof is 250 m2 . (a) 2 π a 2T (b) 6 π a 2T
Assuming that the pressure inside the house is (c) 3 π a 2T (d) 12 π a 2T
atmospheric pressure, the force exerted by the wind
622 OBJECTIVE Physics Vol. 1

31 A boat carrying a few number of big stones floats in 38 A flow of liquid is streamline, if the Reynolds’
a water tank. If the stones are unloaded into water, number is [CBSE AIPMT 2014]
the water level [Kerala CEE 2015] (a) less than 1000
(a) rises till half the number of stones are unloaded and (b) greater than 1000
then begins to fall (c) between 2000 to 3000
(b) rises (d) between 4000 to 5000
(c) fall till half the number of stones are unloaded and then
begins to rise 39 A certain number of spherical drops of a liquid of
(d) falls radius r coalesce to form a single drop of radius R
(e) remains unchanged and volume V. If T is the surface tension of the
32 Choose the correct statement. [Kerala CEE 2015] liquid, then [CBSE AIPMT 2014]
(a) Terminal velocities of rain drops are proportional to 1 1 
(a) energy = 4VT  −  is released
square of their radii. r R 
(b) Water proof agents decrease the angle of contact 1 1 
between water and fibres. (b) energy = 3VT  +  is absorbed
r R 
(c) Detergents increase the surface tension of water.
(d) Hydraulic machines work on the principle of  1 1
(c) energy = 3VT  −  is released
Torricelli’s law. R r 
(e) Venturimeter measures the flow speed of compressible (d) energy is neither released nor absorbed
fluids.
40 A bubble is at the bottom of the lake of depth h. As
33 A 20 cm long capillary tube is dipped vertically in
the bubble comes to sea level, its radius increases
water and the liquid rises up to 10 cm. If the entire
system is kept is a freely falling platform, the length three times. If atmospheric pressure is equal to l
of the water column in the tube will be [WB JEE 2015] metre of water column, then h is equal to
[UK PMT 2014]
(a) 5 cm (b) 10 cm (a) 26 l (b) l
(c) 15 cm (d) 20 cm (c) 25 l (d) 30 l
34 By sucking through a straw, a student can reduce the 41 A wooden block is floating on water kept in a
pressure in his lungs to 750 mm of Hg beaker. 40% of the block is above the water surface.
(density 13.6 gcm −3 ). Using the straw, he can drink Now, the beaker is kept inside a lift that starts going
water from a glass upto a maximum depth of upward with acceleration equal to g/2 . The block
[UP CPMT 2015] will then [WB JEE 2014]
(a) 10.2 cm (b) 75.3 cm (a) sink
(c) 13.6 cm (d) 1.96 cm
(b) float with 10% above the water surface
35 If 1000 drops are combined to form a larger drop, (c) float with 40% above the water surface
then the ratio of surface energy of smaller drop to (d) float with 70% above the water surface
the larger drop will be [UP CPMT 2015]
42 A drop of some liquid of volume 0.04 cm3 is placed
(a) 1 : 100 (b) 1 : 1000
on the surface of a glass slide. Then, another glass
(c) 1 : 10 (d) 0 ⋅ 1 : 1
slide is placed on it in such a way that the liquid
36 A solid floats such that its 1/3 part is above the water forms a thin layer of area 20 cm2 between the
surface. Then, the density of solid is [UP CPMT 2015] surfaces of the two slides. To separate the slides a
1000
(a) 744 kgm−3 (b) kgm−3 force of 16 × 10 5 dyne has to be applied normal to
3 the surfaces. The surface tension of the liquid is
2000
(c) kgm−3 (d) 910 kgm−3 (in dyne cm −1 ) [WB JEE 2014]
3
(a) 60 (b) 70
37 The amount of work done in blowing a soap bubble (c) 80 (d) 90
such that its diameter increases from d to D is
43 The wettability of a surface by a liquid depends
(S = surface tension of the solution) [Manipal 2015]
primarily on [NEET 2013]
(a) 2π (D 2 − d 2 ) S
(a) viscosity
(b) π (D 2 − d 2 ) S (b) surface tension of liquid and air
(c) 4 π (D 2 − d 2 ) S (c) density
(d) 8 π (D 2 − d 2 ) S (d) angle of contact between the surface and the liquid
Fluid Mechanics 623

44 There are two identical small holes on the opposite 50 The unit of viscosity in the CGS system is poise (P)
sides of a tank containing a liquid. The tank is open and that in SI is poiseuille (Pl). Which of the
at the top. The difference in height between the two following statement is correct? [UP CPMT 2013]
holes is h. As the liquid comes out of the two holes, (a) 1 P = 1 Pl (b) 1 P = 10 Pl
the tank will experience a net horizontal force (c) 10 P = 1 Pl (d) None of these
proportional to [J&KCET 2013] 51 Liquid rises to a height of 2 cm in a capillary tube
and the angle of contact between the solid and the
liquid is zero. If the tube is depressed more now, so
that top of capillary is only 1 cm above the liquid,
h then the apparent angle of contact between the solid
and the liquid is [UP CPMT 2013]
(a) 0° (b) 30°
(c) 60° (d) 90°
(a) h (b) h (c) h 3 / 2 (d) h 2
52 Assertion The water rises higher in a capillary
45 Water rises in a capillary tube up to a height of 10 cm, tube of small diameter than in the capillary tube of
whereas mercury depresses in it by 3.42 cm. If the large diameter.
angle of contact and density of mercury are 135° and Reason Height through which liquid rise in capillary
13.6 gcc −1 respectively, then the ratio of the surface tube inversely proportional to the diameter of
tension of water and mercury will be nearly capillary tube. [AIIMS 2012]
[EAMCET 2013] (a) If both Assertion and Reason are correct and Reason is
(a) 13 : 2 (b) 5 : 16 the correct explanation of Assertion.
(c) 16 : 5 (d) 2 : 13 (b) If both Assertion and Reason are correct but Reason is
46 Two capillary tubes of lengths in the ratio 2 : 1 and not the correct explanation of Assertion.
radii in the ratio 1 : 2 are connected in series. (c) If Assertion is correct but Reason is incorrect.
(d) If both Assertion and Reason are incorrect.
Assume the flow of the liquid through the tube is
steady. Then, the ratio of pressure difference across 53 The ratio of radii of two bubbles is 2 : 1. What is the
the tubes is [EAMCET 2013] ratio of excess pressures inside them? [UP CPMT 2012]
(a) 1 : 8 (b) 1 :16 (a) 1 : 2 (b) 1 : 4
(c) 32 : 1 (d) 1 : 1 (c) 2 : 1 (d) 4 : 1
47 Two spherical soap bubbles of diameters 10 cm and 54 10 min are taken to empty a rectangular vessel of
6 cm are formed, one at each end of a narrow height h through an orifice in its bottom. How much
horizontal glass tube. If the surface tension of the time will it take to be emptied the vessel when half
soap solution is 0.03 Nm−1, then the pressure filled? [BCECE (Mains) 2012]
difference (in pascal) between the two ends of the (a) 3 min (b) 5 min
tube is [Kerala CEE 2013] (c) 7 min (d) 9 min
(a) 16 (b) 1.6 55 Water is moving with a speed of 5.18 ms −1 through
(c) 0.016 (d) 0.08 a pipe with a cross-sectional area of 4.20 cm 2 . The
(e) 0.16 water gradually descends 9.66 m as the
48 The excess pressure inside one soap bubble is three cross-sectional area of pipe is increased to 7.60 cm 2 .
times that inside a second bubble. The ratio of the The speed of the flow of water at lower level is
volume of first bubble to that of the second [BCECE (Mains) 2012]
[MPPMT 2013] (a) 2.86 ms −1 (b) 3.0 ms −1
(a) 1 : 27 (b) 27 : 1 (c) 1 : 9 (d) 9 : 1 (c) 3.82 ms −1 (d) 5.7 ms −1
49 Water rises to a height of 20 mm in a capillary. If 56 A block of ice in which a piece of stone is embedded is
the radius of the capillary is made one-third of its floating on water contained in a beaker. When all the
previous value, then the new value of capillary rise ice melts, the level of water in the beaker [BHU 2012]
will be [MPPMT 2013] (a) rises
20 (b) falls
(a) mm (b) 60 mm
3 (c) remains unchanged
20 (d) None of the above
(c) mm (d) 180 mm
9
624 OBJECTIVE Physics Vol. 1

57 The excess pressure inside a spherical soap bubble of reduced to a /2 and the pressure increased to 2p. The
radius 1 cm is balanced by a column of oil (specific rate of flow becomes [AMU 2012]
gravity = 0.8), 2 mm high, the surface tension of the (a) 4 Q (b) Q
bubble is [Manipal 2012] Q Q
(c) (d)
(a) 3.92 Nm −1 (b) 0.0392 Nm −1 4 8
(c) 0.392 Nm −1 (d) 0.00392 Nm −1 61 If two soap bubbles of different radii are connected
58 Water from a tap emerges vertically downwards by a tube, then [Manipal 2012]
with initial velocity 4 ms −1 . The cross-sectional area (a) air flow from bigger bubble to the smaller bubble till
of the tap is A. The flow is steady and pressure is sizes becomes equal
constant throughout the stream of water. The (b) air flow from bigger bubble to the smaller bubble till
distance h vertically below the tap, where the sizes are interchanged
cross-sectional area of the stream becomes (2/3) A, is (c) air flow from smaller bubble to bigger bubble
(Take, g = 10 m s −2 ) [Manipal 2012]
(d) there is no flow of air

(a) 2 m (b) 1 m 62 A body floats in water with 40% of its volume


(c) 0.5 m (d) 4 m outside water. When the same body floats in an oil,
60% of its volume remains outside oil. The relative
59 The terminal speed attained by an aluminium density of oil is [WB JEE 2011]
sphere of radius 1 mm falling through water at (a) 0.9 (b) 1
20°C will be close to [AMU 2012]
(c) 1.2 (d) 1.5
(a) 9.2 ms −1 (b) 6.9 ms −1
(c) 4.6 ms −1 (d) 2.3 ms −1 63 A vertical tank with depth H is full with water. A
hole is made on one side of the walls at a depth h
(Assume laminar flow, specific gravity of Al = 2.7
below the water surface. At what distance from the
and ηwater = 8 × 10 −4 PI) foot of the wall does the emerging stream of water
60 Water flows in a streamlined manner through a strike the foot? [DUMET 2011]
capillary tube of radius a, the pressure difference (a) h (H − h ) (b) 2 h (H − h )
being p and the rate of flow Q. If the radius is (c) 2(H − h ) h / (H − h ) (d) 2h / (H − h )
ANSWERS
CHECK POINT 13.1
1. (b) 2. (d) 3. (c) 4. (b) 5. (c) 6. (c) 7. (d) 8. (b) 9. (c) 10. (b)
11. (c)

CHECK POINT 13.2


1. (d) 2. (b) 3. (a) 4. (b) 5. (d) 6. (b) 7. (b) 8. (b) 9. (a) 10. (b)

CHECK POINT 13.3


1. (b) 2. (d) 3. (c) 4. (d) 5. (c) 6. (c) 7. (d) 8. (a)

CHECK POINT 13.4


1. (d) 2. (a) 3. (a) 4. (b) 5. (c) 6. (c) 7. (b) 8. (a)

(A) Taking it together


1. (b) 2. (a) 3. (b) 4. (d) 5. (b) 6. (c) 7. (c) 8. (d) 9. (c) 10. (a)
11. (b) 12. (b) 13. (d) 14. (a) 15. (b) 16. (d) 17. (b) 18. (a) 19. (a) 20. (d)
21. (b) 22. (d) 23. (a) 24. (b) 25. (b) 26. (d) 27. (b) 28. (b) 29. (d) 30. (d)
31. (a) 32. (c) 33. (b) 34. (b) 35. (c) 36. (b) 37. (a) 38. (c) 39. (c) 40. (d)
41. (c) 42. (a) 43. (a) 44. (b) 45. (d) 46. (a) 47. (c) 48. (d) 49. (a) 50. (a)
51. (b) 52. (a) 53. (c) 54. (a) 55. (a) 56. (b) 57. (c) 58. (a) 59. (a) 60. (d)
61. (d) 62. (c) 63. (d) 64. (c) 65. (a) 66. (b) 67. (b) 68. (b) 69. (c) 70. (c)
71. (a) 72. (b) 73. (b) 74. (c) 75. (a) 76. (d) 77. (b) 78. (c) 79. (d) 80. (b)
81. (b) 82. (a) 83. (b) 84. (a) 85. (c) 86. (a) 87. (c) 88. (c) 89. (a) 90. (c)
91. (a) 92. (a) 93. (b) 94. (a) 95. (b) 96. (a) 97. (b) 98. (a) 99. (d) 100. (c)
101. (b) 102. (c) 103. (a) 104. (b) 105. (a) 106. (c) 107. (c) 108. (c) 109. (d) 110. (b)
111. (d) 112. (c) 113. (a) 114. (b) 115. (a) 116. (d) 117. (a) 118. (b) 119. (a) 120. (c)
121. (c) 122. (b) 123. (c) 124. (a) 125. (d) 126. (b)

(B) Medical entrance special format questions


l Assertion and reason
1. (b) 2. (b) 3. (c) 4. (a)
l Statement based questions
1. (c) 2. (a) 3. (c)
l Match the columns
1. (c) 2. (b) 3. (a)

(C) Medical entrances’ gallery


1. (d) 2. (c) 3. (b) 4. (b) 5. (d) 6. (d) 7. (b) 8. (a) 9. (c) 10. (b)
11. (a) 12. (a) 13. (a) 14. (b) 15. (b) 16. (a) 17. (b) 18. (d) 19. (c) 20. (b)
21. (c) 22. (b) 23. (b) 24. (c) 25. (c) 26. (c) 27. (d) 28. (b) 29. (d) 30. (a)
31. (d) 32. (a) 33. (d) 34. (c) 35. (a) 36. (c) 37. (a) 38. (a) 39. (c) 40. (a)
41. (b) 42. (c) 43. (d) 44. (b) 45. (d) 46. (c) 47. (b) 48. (a) 49. (b) 50. (c)
51. (c) 52. (a) 53. (a) 54. (c) 55. (a) 56. (b) 57. (b) 58. (b) 59. (c) 60. (d)
61. (c) 62. (d) 63. (b)
Hints & Explanations
l CHECK POINT 13.1 11 (c) Let v be the volume of cavities and V be the volume of
solid iron.
1 (b) Pointed ends have very small area, therefore transmits a
large pressure. mass  6000 / 9.8
Then, V = =  = 0.078 m
3

F 70 × 10 7 × 10 4 density  7.87 × 10 3 
2 (d) Pressure, p = = Mg /A = = × 10 2
A −4
15 × 10 × 2 30 Further, decrease in weight = upthrust
−2 ∴ (6000 − 4000 ) = V
( + v )ρ wg
= 2.33 × 10 N m 5
or 2000 = (0.078 + v ) × 10 3 × 9.8
3 (c) Gauge pressure = Absolute pressure − Atmospheric
pressure or 0.078 + v −
~ 0.2
∴ v = 0.12 m 3
4 (b) Absolute pressure, p = p a + ρgh
Here, p a = 1.01× 10 5 Pa, l CHECK POINT 13.2
−3 −2
ρ = 1.03 × 10 kgm , h = 500 m , g = 10 ms
3
1 (d) In the streamline flow,
∴ p = 1.01× 10 Pa + 1.03 × 10 × 10 × 500
5 3 v1 = constant, v 2 = constant, v 3 = constant
but, v1 ≠ v 2 ≠ v 3
= 1.01× 10 5 Pa + 51.5 × 10 5 Pa
2 (b) For 1000 < Re < 2000, flow is unsteady.
= 52.5 × 10 5 Pa ≈ 52 atm
3 (a) By equation of continuity,
5 (c) The shape of the container does not affect the pressure.
The four vessels A, B, C and D have different shapes and hold Av = constant
different amount of water. The free surface of water in all ∴ 1 1 = A 2v 2 + A 3v 3
Av
vessels are in same horizontal plane. The pressure at the four ⇒ A × 8 = 0.8A × 6 + 3A × v
points E, F, G and H lying in the same horizontal plane will be ⇒ 4A = 2.4A + 1.5Av ⇒ 4 = 2.4 + 1.5v
same. 1.6
ρV + ρ 2V2 (2ρ ) (1) + (ρ ) (2 / 3) 8 ρ ⇒ v= = 1.1ms −1
6 (c) Qρ = 1 1 = = 1.5
V1 + V2 1+ 2 / 3 5
4 (b) Bernoulli’s theorem is based upon conservation of
7 (d) If the pressure at the oil-water interface in the left arm is mechanical energy.
p, then the pressure in the right arm at the level of the 5 (d) According to equation of continuity,
interface will also be p.
A1v 1 = A 2 v 2
In the left arm, p = p 0 + ρ x g (l + d ) …(i)
10 × 1 = 5 ×v 2
In the right arm, p = p 0 + ρ wgl …(ii)
⇒ v 2 = 2 ms −1
Equating Eqs. (i) and (ii), we get
l 135 mm Now, applying Bernoulli’s equation, we get
ρx = ρw = × 1000 kgm−3 = 900 kgm−3
l+d 135 mm + 15 mm p1 v12 p 2 v 22
+ = +
ρ 2 ρ 2
8 (b) It is due to Pascal’s law.
2000 12 p 22
9 (c) Here, r1 = 4 cm and r2 = 12 cm ⇒ + = 2 + ⇒ p 2 = 500 Pa
1000 2 1000 2
F1 = F and F2 = mg = 1400 × 10 N
v2
According to Pascal’s law, 6 (b) Given, = h = 10 cm = 0.1m
2
2g
r12  4
F1 = F2 × = 14 × 10 3 ×   ∴ Speed, v = 2gh = 2 × 10 × 0.1 = 2 ms −1 = 1.4 ms −1
r22  12
1 7 (b) As water falls from a tap, the velocity of water goes on
= 14 × 10 3 × −
~ 1600 N increasing, hence from the equation of continuity, the
9
cross-section of water-stream goes on decreasing. If A0 is area
10 (b) Given, h = x cm, θ = φ of cross-section of the mouth of the tap and v 0 is the velocity
Let l = length of mercury column. of water at this point and A, v are corresponding quantities at
h depth h, then
∴ = sin(90 ° − φ ) dV
l A0v 0 = Av = (volume rate of flow)
x dt
⇒ = cos φ ⇒ l = x / cos φ and v 2 − v 02 = 2gh
l
Fluid Mechanics 627

dV 2gh 2 × 10 × 2 CHECK POINT 13.4


∴ = Av = A0A 2 = 4×1 l

dt A0 − A 2
16 − 1 ∆W
2 1 (d) Surface tension T = F / l or , i.e. unit of surface
=8 ms −1 = 6.5 ms −1 ∆A
3 tension is N/m or J/m 2.
8 (b) The lifting of an aeroplane is based on Bernoulli’s N kg - ms 2 kg
= = 2
principle. When airplane moves through air, pressure m m s
difference is created due to higher speed of air in the region
above the wings. 2 (a) Force on each side = 2TL (due to two surfaces)

9 (a) Velocity of efflux = 2gh, which is independent of F 2 × 10 −2


3 (a) Surface tension, T = = = 0.1 Nm−1
density of liquid. 2l 2 × 10 × 10 −2

H 3H 3H 5 (c) W = 8πR 2T
10 (b) Range of water = 2 h (H − h ) = 2 × =
4 4 2 ∴ W ∝ R2
If radius is doubled, then work done will become four times.
l CHECK POINT 13.3
r1r2
πpr 4 6 (c) Radius of curvature, r = = ∞. (Q r1 = r2)
1 (b) Rate of flow through a tube is given byV = r2 − r1
8ηl
4T 4 × 2 × 25 × 10 −3
∴ N = 1/ 8 7 (b) Excess pressure, ∆p = = (Q r = d / 2)
r 1× 10 −2
2 (d) When the tubes are fitted in parallel,V = V1 + V2
= 20 Nm−2 = 20 Pa
πpr 4 πpr14 πpr24
⇒ = + 2T cos θ
8ηl 8ηl 8ηl 8 (a) h=
rρg
⇒ r 4 = r14 + r24 2T cos θ h1 r1
∴ hr = = constant ⇒ hr
11 = h2r2 or h2 =
∴ r = (r14 + r24 )1/ 4 ρg r2
F mg Substituting the values, we get
3 (c) F = 6πηrv , i.e. v has the dimensions of or .  r2 1
ηr ηr h2 = (2.0)(3) Q = 
 r1 3
4 (d) Terminal velocity, vT ∝ r 2 = 6.0 cm
2
(vT )1  R  (A) Taking it together
∴ =  =4
(vT )2  R / 2
1 (b) As brush is removed, water surface formed over bristle
5 (c) Velocity first increases and then becomes constant when tends to take smallest area due to surface tension.
terminal velocity is attained.
2 (a) Waterproof surface coatings are made up of materials
6 (c) Let r be the radius of smaller drop and R the radius of which increases angle of contact and water does not wet the
bigger drop. By equating the volume, we get surface.
4  4 3 (b) For small drops, force of surface tension predominates
2 πr 3 = πR 3 or R = (2)1/ 3 ⋅ r
3  3 gravitational force and so, they are more spherical.
Now, terminal velocity ∝ (radius) 2
4 (d) Cohesive force of water is greater than adhesive force and
∴ v ′ = (2)2/ 3v hence, water does not wets the surface.
7 (d) When the small sized spherical body is falling through the 5 (b) Surface energy of combined drop will be lowered, so
viscous liquid first, its velocity increases and then becomes excess surface energy will raise the temperature of the drop.
constant which is called terminal velocity as shown in (d). 7 (c) Velocity of different layers of a flowing fluid decreases
8 (a) Before entering the water, the velocity of ball is 2gh. If with distance from the axis of the tube. It is maximum at
centre and minimum at walls. So, correct graph is (c).
after entering the water, this velocity does not change, then
this value should be equal to the terminal velocity. Therefore, 8 (d) In a streamline flow at any given point, the velocity of
2 r 2 (ρ − σ )g each passing fluid particles remains constant. If we consider a
2gh = Fv cross-sectional area, then a point on the area cannot have
9 η
different velocities at the same time, hence two streamlines of
2
 2 r 2 (ρ − σ )g  a flow cannot cross each other.
9 η  9 (c) F = Wair − Wwater = 5 N − 2 N = 3 N
 = 2 × r (ρ − σ ) g
4 2
∴ h=
2g 81 η2 w+T 10 (a) Specific gravity or relative density
2 (3 × 10 −4 )4 (10 4 − 10 3 )2 × 9.8 =
Weight in air
=
60
=3
= × = 1.65 × 10 3m
81 (9.8 × 10 −6 )2 Change of weight in water 20
628 OBJECTIVE Physics Vol. 1

11 (b) When iron block was floating, it displacesV1 volume of = 2Tb cos θ …(i) x
Weight of the liquid rises in b
water such that weight of thisV1 volume of water is equal to
weight of block. But since density of iron is more than density between the plates
h
of water, this volumeV1 will be greater than volumeV2 of iron = Vdg = (bxh )dg …(ii)
block. When iron block sinks in the pond, it displaces water Equating Eqs. (i) and (ii), we get
of volume equal to its own volumeV2. 2Tb cos θ = bxhdg
SinceV1 > V2, displaced volume in first case was more than 2T cos θ
displaced volume in second case. Hence, level will fall. ∴ h=
xdg
2T cos θ 1
12 (b) h = ⇒ h ∝ . So, graph will be hyperbolic. 28 (b) 1 gL−1 = 1 kgm−3
rdg r
1 Upthrust = Total downward force
13 (d) h ∝ , so for less r, h will be more. Vρ 0 g = Vρg + mg
r
∴ 2000 × 1.29 × g = (2000 × 0.09 × g + mg )
14 (a) Surface tension decreases with temperature and becomes
zero at boiling point. ∴ m = 2400 kg

15 (b) Cohesive force decreases with increase in temperature, so 29 (d) Upthrust on 7 cm = 600 kg
angle of contact of a liquid decreases. 600
∴Upthrust on further 1.4 cm = × 1.4 = 120 kg
16 (d) Work done,W = 8πR T 2 7
= 8 × π × (10 −2 )2 × 2 × 10 −2 = 16π × 10 −6 J F1 F2
30 (d) Q = (Pascal’s law)
A1 A2
17 (b) Weight of liquid in tube is balanced by force of surface
tension. ⇒ Cross-sectional area of the other piston,
∴ w = 2πrT F  2000 × 9.8 × 10 5 
A2 = 2 ⋅ A1 =   (100) = 1.96 × 10 4 cm2
18 (a) If the angle of contact is less than 90°, the surface tension F1  10 7 
of liquid-surface interface is less than that of air-liquid
interface. And the pressure difference is proportional to the 31 (a) As given
surface tension, so pressure inside liquid is less than d1 = Diameter of Ist pipe is 2.5 cm.
atmospheric pressure. d 2 = Diameter of IInd pipe is 3.75 cm.
19 (a) Due to force of attraction between water molecules, it is Applying equation of continuity for cross-sections A1 and A2,
not easier to separate the two glass plates. A1v 1 = A2v 2
2
20 (d) Volume of ice > Volume of water v1 A2 π (r22 )  r2 
⇒ = = = 
m m v 2 A1 π (r12 )  r1 
∴ ∆V = Vw − Vi = −
y x 2
 3.75 
   3.75
2
9
22 (d) Initially, upthrust p 0A supports the weight of mercury = 2  =   =
 2.5   2.5  4
column in barometer. While in second case, it provides
 2 
acceleration too. So, the reading will decrease.
23 (a) Neglecting gravitational head, difference in pressure 32 (c) According to the question,the observed meniscus is of
energy = difference in kinetic energy convex shape. Which is only possible when angle of contact is
1 obtuse. Hence, the combination will be of mercury-glass (140°).
∴ (3p 0 − p 0 ) = ρv 2
2
p0 10 5
or v =2 =2 = 20 ms −1 or 400 ms −1 Convex
ρ 10 3
24 (b) 1 atmosphere = 1.01 × 10 5 N m−2 = ρgh 140°

1.01 × 105
∴ h= = 7769 m ≈ 8 km
1.3 × 10 Mercury
1 r hQ r h 3
25 (b) r ∝ ⇒ P = ⇒ P = =
h rQ hP rQ 2 h 2
33 (b) Force on the base,
3
F = ρ ghA = 900 × 10 × 0.4 × 2 × 10 −3 = 7.2 N
26 (d) In artificial satellite, gravity is absent, so capillary will be
fully filled. 34 (b) Force at the bottom,
27 (b) Let the width of each plate is b and due to surface tension F = p × A = (ρghtotal )A
liquid will rise upto height h, then upward force due to = (10 3 × 10 × 1.0) (100 × 10 −4 ) = 100 N
surface tension
Fluid Mechanics 629

35 (c) In horizontal direction, pressure remains constant unless 45 (d) Submerged part = Replaced water
the fluid volume is accelerated. 4
hρ w = h × ρl
36 (b) Tc = T0 (1 − αt ), i.e. surface tension decreases linearly with 5
increase in temperature. 4 4
∴ ρl = × ρ w = × 1000 ⇒ ρl = 800 kgm −3
37 (a) Loss of weight = Upthrust 5 5
w 46 (a) Applying Bernoulli’s equation, we get
= Total volume × ρ w × g
2 1 1
p1 + ρv12 = p 2 + ρv 22
( − VC ) 5 × ρ w × g
V 2 2
or =V × ρw × g V
( C = volume of cavity)
1
2 or (10 −2 × 13.6 × 10 3 × 10) + × 10 3 × (0.35)2
2 3 2
or V − VC = V ⇒ VC = V
5 5 1
= p 2 + × 10 3 × (0.65)2
38 (c) Initially, due to more pressure at the hole, the velocity of 2
efflux will be more. So, the liquid drains out more quickly for −2 1210 × 10 2
or p 2 = 1210 Nm = cm of Hg
first half. 13.6 × 10 3 × 10
39 (c) Equating pressure, we get = 0.89 cm of Hg
(11.2) (1) = (13.6) (2x ) 47 (c) Force exerted by the liquid on the base of the vessel is
F = mg
x
Here, mA = mB = mC
x Original ∴ FA = FB = FC
level
48 (d) Given that an object weights m1 in a liquid of density d1
and m 2 in a liquid of density d 2, so when the density of the
object is d, then we get
∴ x = 0.41 cm
V (d − d1) = m1 andV (d − d 2 ) = m 2
40 (d) Weight − Upthrust = Drag force. d − d1 m1
But no information is given regarding the upthrust. Thus, =
d − d 2 m2
2T cos θ 1
41 (c) h = or h ∝ m1d 2 − m 2d1
rρg r So, we get d =
m1 − m 2
m = (πr 2h ) ρ or m ∝ r 2h
or m ∝ r 49 (a) Pressure at same level in two limbs will be same.
∴ M2 = 2M ∴ p + ρg (3h ) = p 0 + 2ρg (h )
∴ p = p 0 − ρgh or p < p 0
42 (a) Given, p1 = 5.5 × 10 5 Nm −2 and p 2 = 5 × 10 5 Nm −2
50 (a) Weight of man = Extra upthrust
∴ ∆p = p1 − p 2 = (5.5 − 5) × 10 5
∴ mg = Vρwg = (3 × 2 × 10 −2 ) (10)3 (10)
= 0.5 × 10 5 Nm −2
or m = 60 kg
Decrease in pressure energy = Increase in kinetic energy
Upthrust − Weight V ρ wg − V ρg
1 51 (b) a = =
or ∆p = ρv 2 Mass Vρ
2
 ρ − ρ  1 − 0.5 −2
2(∆p ) 2 × 0.5 × 10 5 = w  g =   (10) = 10 ms
∴ v= = = 10 ms −1  ρ  0.5 
ρ 10 3
52 (a) For just sink, (mraft + m )g = Vraft ρ wg
43 (a) Rate of leakage of water from the hole
∴ m = Vraft ρ w − mraft
= Av = A 2gh
120
= 10 −4 2 × 10 × 5 = 10 −3 m 3 s −1 = × 1000 − 120 = 80 kg
600
1 1
44 (b) From continuity equation, v ∝ or v ∝ 2 53 (c) The net force acting on the stone in a downward direction
A d would be equal to F = ma, where m is the mass of the stone.
(d = diameter of pipe) The force acting downwards would be equal toVσg due to its
2 weight and the force acting upwards would be equal toVρg
vnarrow  d w 
2
 6
∴ =  =  =4 due to the buoyant force.
v wider  d N   3
 ρ
Hence, we get F = Vσg −Vρg = Vσg 1− 
vnarrow 4  σ
⇒ v wider = = = 1ms −1
4 4
630 OBJECTIVE Physics Vol. 1

 ρ  1
ma = mg 1−  = mg 1−  66 (b) As we know according to equation of continuity, when
 σ  K cross-section of duct decreases, the velocity of flow of liquid
 1 increases and in accordance with Bernoulli’s theorem in a
Thus, a = g 1− 
 K horizontal pipe, graph (b) correctly represents the variation of
pressure.
( ρgeff ) is zero, because
54 (a) In a freely falling vessel upthrust V
A 2h
effective value of g is zero. 67 (b) Time to empty a tank, t =
a g
ρ 0.9
55 (a) Fraction immersed = solid = = 0.8 h2 4h
ρ liquid 1.125 or t ∝ h ⇒ t2 /t1 = = = 2 ⇒ t2 = 2t
h1 h
Therefore, fraction outside the water = 0.2 = 1/5
68 (b) Level in the container will become maximum, when rate
56 (b) Weight of sphere = Upthrust from water + Upthrust from of inflow = rate of outflow
liquid
2 3  V  Q = A1v = A1 2ghmax
∴ Vρg =  V × 10 × g +  × 8 × 10 × g 3
3  3  Q2
∴ hmax =
10000 2gA12
or ρ= kgm−3
3
1
69 (c) Pressure difference, ∆p = ∆ KE = × ρ(v12 − v 22 )
57 (c) A1v 1 = A2v 2 + A3v 3 2
A1v 1 − A 2v 2 (0.2 × 4) − (0.2 × 2) ∆p =
1
× 1.0 × (3600 − 1600 ) = 1000 Nm−2
∴ v3 = = = 1 ms −1 or
2
A3 0.4
⇒ F = (∆p ) × A = 4000 N
58 (a) Rh = RH − h , since range from both the holes is same.
70 (c) Decrease in potential energy = Increase in kinetic energy
h = 2.4 cm and H − h = 7.6 cm 1
Adding these two, we get H = 10 cm ∴ ρgh = ρ (v f2 − vi2 )
2
59 (a) Work done by all the forces, W = Change in kinetic energy or 2 (10 ) (0.15) = v f2 − (1.0 )2
Here, two forces acting are weight and upthrust.
or v f = 2 ms −1
∴ W = F × h = (σ − ρ )g (hV )
1
As, σ > ρ , so kinetic energy will increase. Now, from continuity equation, A1v1 = A2v 2 or A ∝
v
60 (d) Force exerted by gas on hemispherical end = Pressure Velocity has become two times. Hence, area of cross-section
of gas × Projected area = p 0 (πr 2 ) will become half, i.e. 0.5 × 10 −4 m2 or 5 × 10 −5 m2.
61 (d) 6 × 10 −2 × Circumference = Force 71 (a) Q Time of flight, t = 2g (H − h )
75 × 10 −4
∴ Circumference = = 12.5 × 10 −2 m ∴Range, R = 2g h (H − h )
6 × 10 −2
If R1 = R2 ⇒ h1 (H − h1) = h2 (H − h2 )
11 = h2r2
62 (c) Qhr
⇒ H (h1 − h2 ) = h12 − h22
r  ⇒ h1 + h2 = H
⇒ h2 = h1 1  = 30 × 4/ 3 = 40 mm
 r2 
2 r 2 (ρ − σ )g
2T 2 × 75 72 (b) Terminal velocity, v =
63 (d) h = ⇒ h= = 3 cm 9 η
rdg 0.05 × 1 × 10 3
∴ v ∝ r2
 1 mm 
Q r = = 0.05 cm v r2
 2  ⇒ = 2
v′ R
64 (c) Angle of contact is acute. So, the pulling force on glass
vR 2
will attract the water towards them and the water drop takes ⇒ v′ =
concave shape. r2
hi ρ solid ρ
65 (a) Suppose R be the radius of bigger drop. Then, by equating 73 (b) = = s
volumes, we get l ρ liquid ρ m
2(4/ 3πr 3 ) = (4/ 3)πR 3 or R = (2)1/ 3 r ρs
∴ hi = ⋅l
Now, surface energy ∝ surface area ρm
U1 A1 2⋅ r 2 2  ρ 
∴ = = 2 = 2/ 3 ∴ Height above the mercury level = 1 − s  l
U 2 A2 R 2  ρm 
Fluid Mechanics 631

dp
74 (c) When moving upwards = ρ(g + a ) and when moving 81 (b) Writing pressure equation from one end to other end of
dh tube,
dp l l
downwards = ρ(g − a ). p 0 + 2ρgh − 2ρ ⋅ ⋅ a − ρ . a − ρgh = p 0
dh 2 2
In first case, pressure decreases with h more rapidly. 3al
∴ h=
75 (a) Upthrust on sphere from the liquid makes equal and 2g
opposite pair of forces, so p = p 0 + ρgh.
82 (a) Pressure is same in a horizontal level, p D = p B
76 (d) Pressure inside the bubble
∴ p 0 + ρ1gh1 = p 0 + ρ wgh2
2T 2T
= Pressure outside it + = (p 0 + ρgh ) + ρ1 h
r r or = relative density of paraffin = 2
ρw h1
2 × 7 × 10 −2
= 10 + (10 × 10 × 10 ) +
5 3
10 −3 83 (b) As, liquids in the two limbs are at same level, so pressure
−2 is equal and ρ1 = ρ 2 = 1.1.
= 2.0014 × 10 Nm 5
dp
77 (b) ∴ F = mg − ma 84 (a) = ρa (along horizontal towards right), i. e . in the
dx
3g direction of acceleration along horizontal, the pressure will
⇒ 3g = 4 (g − a ) ⇒ g − a = decrease or p AB > p CD
4
dp
 3g  = ρg
Upthrust force =v × ρ (g − a ) = (5 × 10 −4 ) (10 3 )   dy
 4
= 3.75 kg i.e. Along vertically upwards direction pressure will decrease.
Apparent weight = (4 − 3.75) = 0.25 kg Hence, pressure at B is maximum and pressure at D is minimum.
∆p
78 (c) w = Upthrust 85 (c) = ρa (in horizontal direction)
∆x
or (m1 + m 2 ) g = [m1/(0.5 × ρ w ) + m 2 / (2.5 × ρ w )] ρ wg Pressure decreases in the direction of acceleration in
or m1 + m 2 = 2m1 + 0.4 m 2 horizontal.
or m1 = 0.6 m 2 ∴ pD < pC
m1 3 ⇒ p C − p D = ρgL
∴ = 0.6 =
m2 5 ∆p
Also, = ρg (in vertical direction)
∆x
79 (d) Common height after they are connected can be
Pressure increases with depth in vertical direction.
determined by equating the volumes, hence
∴ p C − p A = ρgL
(A1 + A2 ) h = Ah
1 1 + A2h2 (Q A1 = A2 = A)
h1 + h2 86 (a) Weight = Upthrust from mercury + Upthrust from water
∴ h=
2 ∴ 10 × 8.56 = (x × 13.6) + (10 − x ) × 1.0
 h + h2  Here, x = depth of block inside mercury.
The decrease in height h1 , ∆h = h1 −  1 
 2  Solving, we get
h1 − h2 x = 6 cm
=
2 87 (c) Total volume of 64 drops = Volume of big drop
h − h 
Mass of liquid in ∆h column, m =  1 2  ρA 4 3 4 3
 2  64 × πr = πR ⇒ R 3 = 64r 3
3 3
h − h  h − h 
∴Work done by gravity = mg∆h =  1 2  ρAg  1 2  ⇒ R = 4r ⇒ r=
R
 2   2 
4
ρAg Energy of big drop, E1 = T ⋅ 4πR 2
= (h2 − h1)2
4
  R 
2

80 (b) 2a1h1 = 2a 2h2 Energy of 64 drops, E 2 = 64(T × 4πr 2 ) = 64T × 4π   


 4
 
Here, a1 = acceleration inside the liquid
= 16πTR 2
upthrust − weight Vσg − Vρg  σ 
= = =  − 1 g ∴ Energy needed to break the drop into 64 droplets,
mass Vρ ρ 
∆W = E 2 − E1 = 12πR 2T
a 2 = retardation in air = g
a σ  88 (c) The terminal velocity is given by v, where v ∝ r 2
∴ h2 = 1 ⋅ h1 =  − 1 h
a2 ρ  (r = radius of the sphere).
632 OBJECTIVE Physics Vol. 1

The mass of the sphere can be given by m =


4 3
πr ρ. 95 (b) In equilibrium,weight = upthrust, i.e.Vρi g = Viρ wg
3
l
Thus, m ∝r3
3
m1 1  r1  r 1
So, according to the question, = =  ⇒ 1 =
m 2 8  r2  r2 2
2
v1  r1 
2
 1 1 v 1 Vi ρi 0.9
and since, =  =  = ⇒ = ⇒n = 4 or = = = 0.9
v 2  r2   2 4 nv 4 V ρ w 1.0
89 (a) Let x be the fraction of volume immersed in first case and i.e. 90% or 9 cm of ice block is inside the water. Therefore,
y the fraction in second case. minimum length l required to scoop out a bucket of water is
Then, in first case,W = Upthrust 10 − 9 = 1.0 cm.
ρ
or Vρ bg = (xV )ρl g or x = b …(i) 96 (a) v = 2gh
ρl Vσg − Vρg  σ − ρ
In second case, Retardation in liquid, a = =  g
Vρ  ρ 
Upthrust − Weight = Mass × Acceleration
v2 2gh ρh
 g ρ ∴ dmax = = =
or ( yV ) ρl (g + g / 3) − V ( ρ b )   or y = b …(ii)
( ρ b )g = V 2a  σ − ρ σ −ρ
 3 ρl 2× g
 ρ 
From Eqs. (i) and (ii), we see that x = y or fraction of volume
immersed does not change by the acceleration of vessel. 97 (b) Fraction of length will remain unchanged.
So, fraction of volume immersed in liquid = 1/ 2. yi 1
= constant =
H y 2
90 (c) At y = , maximum range is obtained, which is equal to
2 ∴ If total melting rate is 2 cmh−1, then top of candle will fall
H. Further x y = x H − y .
at the rate of 1 cmh−1.
8ηl
91 (a) Fluid resistance is given by R = 4 .  1
πr 98 (a) F = (6πηrv ) = 6π   (1) (v ) = v
When two capillary tubes of same size are joined in series,  6π 
then equivalent fluid resistance is F dv
∴ Retardation, a = − = − v or = −v
8ηL 8η × 2L  8ηL  9 m dt
Re = R1 + R2 = + = 4 × t 0 .5 dv
 πr  8
πR 4 π (2R )4 ⇒ ∫ 0 dt = − ∫ 2 v or t = ln (4) s
Equivalent resistance becomes 9/8 times, so rate of flow will
be (8/ 9)X. 99 (d) Upthrust, F = w + T
4T 1
92 (a) ∆p = ⇒ ∆p ∝ F
r r
As, radius of soap bubble increases constantly with time.
1
∴ ∆p ∝
t
Hence, the graph will be as shown in option (a).
2T 2T 2T w T
93 (b) Q + =
r2 R r1
1 1 1 1 1 1 1 1 1 1 ⇒ T = F − w = V ρ wg − V ρ bg = (ρ w − ρ b )Vg
⇒ + = ⇒ + = ⇒ = − =
 8 
r2 R r1 5 R 4 R 4 5 20 = (1000 − 800 )   (10 ) = 20 N
 800 
⇒ R = 20 cm
94 (a) Density of the mixture, 100 (c) Upthrust = Volume of metal ball × Density of liquid × g.
ρV + ρ 2V With increase in temperature, volume of ball will increase
4= 1 or ρ1 + ρ 2 = 8 …(i) and density of liquid will decrease.
2V
m+m But coefficient of cubical expansion of liquid is more.
3= Hence, second effect is more dominating.
(m /ρ1) + (m /ρ 2 )
Therefore, upthrust at higher temperatures will be less or
ρ1 ρ 2 3
or = …(ii) apparent weight will be more.
ρ1 + ρ 2 2
101 (b) Let r be the radius of smaller drop and R the radius of
Solving these two equations, we get
bigger drop. Then, by equating volume, we get
ρ1 = 6 and ρ 2 = 2
Fluid Mechanics 633

4  4 R 2
× 1 × 10 = 2 N
1000  πr 3 = πR 3 or r = 107 (c) Upthrust =
3  3 10 10
Surface energy ∝ Surface area Reading of S1 will increase by 2N, i.e. it will become
Si = 4πR 2
or Sf = 1000 (4πr ) = 10 (4πR ) = 10 Si
2 2 50 + 2 = 52 N, while that of S2 will decrease by 2N, i.e.
20 − 2 = 18 N.
As surface area has increased 10 times, surface energy will
also become ten times. 108 (c) Weight = Upthrust

102 (c) From conservation of mechanical energy, it will return to


same height.
103 (a) A1v1 = A2v 2 or A1 2gh1 = A2 2gh2 r

L R
or A12h1 = A22h2 ⇒ (L2 )2 ( y ) = (πR 2 )2 (4 y ) or R =

Weight in air 4 4 3  4 3
104 (b) Relative density of steel =  π R − π r  (8 × ρ w × g ) =  π R  ρ w g
3
Change in weight in water 3 3  3 
w
= or 8R 3 − 8r 3 = R 3
w − w1
1/ 3
Now, change in weight in the given liquid = upthrust in this liquid r  7 3
7
∴ 8r 3 = 7R 3 or =  =
 w /g  R  8 2
or w −w2 =  d 2 × ρ w × g
 (
w /w − w ) ρ
1 w 109 (d) Pressure, p = p 0 + ρ1gh1 + ρ 2gh2
w −w2 or p − p 0 = ρ1gh1 + ρ 2 gh2
∴ d2 = = relative density of given liquid
w − w1 = (600 ) (10 ) (0.1) + (1000 ) (10 ) (0.02) = 800 Nm−2
105 (a) Weight of combined cylinder = Upthrust 110 (b) Change in pressure along horizontal direction,
dp
= ρa
dX
d1
L
2
d h
d2 a

 3L 
∴ d1LAg + d 2LAg = d   Ag A L B
 2
3 ∴ ∆p AB = ρaL = ρgh
⇒ d1 + d 2 = d La
2 ∴ h=
But since d1 < d 2 g
3 111 (d) As, A and C are at the same level inside the liquid, thus
⇒ d1 < d
4 the pressure must be equal at these points, i.e. p A = p C .
106 (c) Also, pressure at C, p C = p B + ρgh, where ρ is the density of
p0 the liquid.
∴ pB < pC
h1 h2 ⇒ pB < pA = pC
p p2
112 (c) If temperature remains constant, then
In the first case, 1 1 = p 2V2
pV
Initially (p − p 0 )A = weight of block (75 + h )V = (75) (3V )
w ∴ h = 150 cm of Hg = 1.5 m of Hg or 15 m of water
or [(p 0 + ρgh1) − p 0 ]A = w or h1 = …(i)
Aρg 113 (a) As, A0v 0 = Av and v 2 = v 02 + 2gh
In the second case,
2ghA2
p 2A = w or (ρgh2 )A = w ⇒ v0 =
A02 − A2
w
or h2 = …(ii)
ρAg 2ghA2
or R = A0 v 0 = A0
From Eqs. (i) and (ii), we see that h1 = h 2 . A02 − A2
634 OBJECTIVE Physics Vol. 1

F
114 (b) We know that, ∆p = . 120 (c) Let us write pressure equation in path ABCDE.
A
Difference in pressure energy is equal to difference in kinetic A
F 1 2 2F E
energy, i.e. = ρv or v =
A 2 ρA B
h
115 (a) p 2 = p 3 and p1 > p 2
h'
C D

2T
x p0 − + ρ 2g (h + h ' ) − ρ 2gh ' − ρ1gh = p 0
r
y r
∴ T = (ρ 2 − ρ1 ) gh
v 2
1
2 3 121 (c) From work-energy theorem,
p1 = p 0 + ρgx ⇒ p 2 = p 0 + ρgy work done by all the forces = change in kinetic energy.
∴ ∆p = p1 − p 2 = ρg (x − y ) ∴(Work done by gravity for 7 m) + (Work done by upthrust
Between 1 and 2, for 2 m) + (Work done by resistive forces) = 0 as ∆ KE = 0
Difference in pressure energy = Difference in kinetic energy  1 
∴ (1) (10) (7) cos 0 ° +   (ρ wg ) (2) cos 180°
1  2/ 3 × ρ w 
or ρg (x − y ) = ρv 2 or v = 2g (x − y )
2 + Work done by resistive forces = 0
r 1 ∴ Work done by resistive forces = − 40 J
116 (d) Given, l1 = l2 = 1m and 1 =
r2 2 122 (b) As, mass of the air is conserved,
πp r 4 πp r 4
V = 11 = 2 2 p
8ηl 8ηl p2
4 p1
p1  r2 
⇒ =   = 16 ⇒ p1 = 16p 2 + =
p 2  r1  r1 r2 r
Since, both tubes are connected in series, hence pressure
difference across combination,
17p1 16
p = p1 + p 2 ⇒ p = ⇒ p1 = = 0.94 m ∴ n1 + n 2 = n (Q pV = nRT )
16 17
pV pV pV
117 (a) Due to surface tension, B will keep its area as minimum as ∴ 11
+ 22=
possible. RT1 RT2 RT
As temperature is constant,
T1 = T2 = T
A
∴ pV
11 + p 2 2 = pV
V
 4S   4   4S   4   4S   4 
∴  p 0 +   πr13 +  p 0 +   πr23 =  p 0 +   πr 3
 r1   3   r2   3   r  3 

So, the thread will become concave on seeing from side A. Solving this, we get
w p (r 3 − r13 − r23 )
118 (b) w = Upthrust =Vi ρ l g orVi = S= 0 2
ρl g 4(r1 + r22 − r 2 )
If density of liquid ρ l is increased, immersed volumeVi will Note To avoid confusion with the temperature, surface tension here is
decrease or the ball will go up. represented by S.

119 (a) From the given figure, we get 123 (c) Let tube’s entrance be at depth y below the surface. Take
δgR (cos θ + sin θ ) = ρgR (cos θ − sin θ ) point 1 at entry, 2 at the maximum height of the fountain.
⇒ δ cos θ + δ sin θ = ρ cos θ − ρ sin θ Applying Bernoulli’s theorem,
⇒ sin θ (δ + ρ ) = cos θ (ρ − δ) 1 1
p1 + ρgh1 + ρv12 = p 2 + ρgh2 + ρv 22 …(i)
ρ −δ 2 2
⇒ tanθ =
ρ+δ Taking, h1 = y , h2 = ( y + h0 + h ), v1 = v, v 2 = 0, p 2 = p1 = p 0
ρ −δ Hence, pressure term on both sides are equal to atmospheric
⇒ θ = tan−1  pressure.
 ρ + δ
Fluid Mechanics 635

1
Substituting these values in Eq. (i), we get or hρ w g = ρ air (v 22 − v12 )
2
1
p 0 + ρgy + ρv 2 = p 0 + ρg ( y + h0 + h ) ρ
2 ∴ h = air (v 22 − v12 )
2ρ wg
1 2
⇒ ρv = ρg (h0 + h ) 1.32
2 = [(5)2 − (1.25)2]
v 2  2 × 10 3 × 10
⇒ h =  − h0  = 1.5 × 10 −3 m ≈ 1.5 mm
 2g 

124 (a) v = 2gh = 2 × 9.8 × 19.6 = 19.6 ms −1 (B) Medical entrance special format
Let ρ be the density of ball and 2ρ the density of water. Net questions
retardation inside the water,
l Assertion and reason
g 2T cos θ
1 (b) h = ⇒ h ∝ cos θ ⇒ h = 0, when θ = 90 °
rρg
h = 19.6 m
Also, when angle of contact is 90°, then surface is flat.
4 (a) Upthrust = Weight of fluid displaced
v
= Volume of body × ρ fluid × g
a=g Hence, both Assertion and Reason are correct and Reason is
the correct explanation of Assertion.
l Statement based Questions
upthrust − weight V (2ρ )g − V (ρ )(g ) 1 (c) The liquid comes to rest due to viscosity.
a= =
mass V (ρ ) 2 (a) Energy is released in process of coalescing of water drops.
(where,V = volume of ball) 3 (c) Water level fall as stone pieces displaces more liquid when
= g = 9.8 ms −2 embedded in ice compared to condition when they sinks.
Hence, the ball will go upto the same depth 19.6 m below the l Match the columns
water surface. Further, time taken by the ball to come back to
1 (c) Pressure at O is the atmospheric pressure which will remain
v   19.6
water surface is t = 2  = 2   = 4s same.
 a  9.8 
Also, as pressure is increased the height h decreases but the
1 2 2(∆p ) pressure at 1 cm above O will remain same.
125 (d) ∆p = ρv or v =
2 ρ Hence, A → q, B → r, C → r.
2h
2 (b) v = 2gh , t = and R = 2 h (H − h )
p1 − p2 = Dp g
At mountain, value of g will be less. Hence, v will decrease, t
1 2
will increase and R will remain same.
Hence, A → q, B → p, C → r.
3 (a) In equilibrium, upthrust = weight
R = vt or R ∝ ∆p
If density of liquid is decreasing, more volume should be
To make R two times ∆p should be 4 times. Initially, immersed in the liquid, so x will increase.
∆p = ρ wgh = 10 3 × 10 × 10 Similarly, if size of cube is increased, volume of cube
increases and again it immerses more, so x will increase.
= 10 5 Nm−2 = 1 atm
The x remains same, when whole system is accelerated upwards.
In second case, ρ wgh + extra pressure = 4 ( ∆p ) = 4 atm
Hence, A → p, B → p, C → r.
or 1 atm + extra pressure = 4 atm
∴ Extra pressure = 3 atm
(C) Medical entrances’ gallery
15 × 10 −3
126 (b) Q = A1v1 = A2 v 2 = 15 Lmin−1 = = 2.5 × 10 −4 m3s −1 1 (d) Relation for height of water in capillary tube is
60
2S cos θ 1
2.5 × 10 −4 2.5 × 10 −4 h= ⇒ h∝
v1 = −4
= 1.25 ms −1 and v 2 = = 5 ms −1 ρgr r
2 × 10 0.5 × 10 −4
h1 r2 2r
Now, difference in pressure = difference in kinetic energy ⇒ = = =2
h2 r1 r
636 OBJECTIVE Physics Vol. 1

As, m = A ⋅ h ⋅ρ  8 ρ − 0.1ρ2   r1 
2

m 2 Ah2ρ h2 1 = 2   …(iii)
∴ = = =  ρ2 − 0.1ρ2   r2 
m1 Ah1ρ h1 2
(Given, σ = 0 . 1ρ2)
m 5 Here, r1 = 1mm and r2 = 2 mm
⇒ m 2 = 1 = = 2.5 g
2 2 Substituting these values in Eq. (iii), we get
2 (c) A liquid does not wet the solid surface, if the angle of v t1  7.9 ρ2   1 2 79
contact is obtuse, i.e. θ > 90º. ⇒ =   =
v t 2  0.9 ρ2   2 36
3 (b) Given, density of liquid, ρl = 760 kgm−3
6 (d) Given, height of small orifice from ground, h = 0.25 m
Density of mercury, ρ m = 13600 kgm−3
Height of liquid column in mercury barometer,
hm = 76 cm = 0.76 m
H
If height of liquid in liquid column be hl , then
p liquid = p mercury 0.25 m h
⇒ hl ρl g = hmρ mg
h ρ 0.76 × 13600 Total height of water tank, H = 1m
⇒ hl = m m = = 13.6 m
ρl 760 ∴Range of water stream,
4 (b) In the given situation as shown in the figure below. R = 2 h (H − h )
= 2 (1 − 0.25 )0.25 = 2 0.75 × 0.25
= 0.866 m = 86.6 cm
20 cm 7 (b) Given, diameter of tube at first end, d1 = 5 cm
15 cm
d
∴ Radius, r1 = 1 = 2 .5 cm
2
Water Oil = 2 .5 × 10 −2 m
From Pascal's law, Diameter of tube at second end, d 2 = 2 cm
d
Pressure due to water column of height 15 cm ∴ Radius, r2 = 2 = 1cm = 1 × 10 −2 cm
= Pressure due to oil column of height 20 cm 2
⇒ hw ρwg = ho ρog Velocity of fluid at first end, v1 = 4 m/s
15 By the principle of continuity,
15 ρw = 20 ρo ⇒ ρo = ρw
20 A1v1 = A2v 2
15 πr12v1 = πr22v 2
⇒ ρo = × 1000 = 750 kgm −3
20
(2 .5 × 10 −2 )2 × 4 = (10 −2 )2 ⋅v 2
5 (d) The terminal velocity achieved by ball in a viscous fluid is
⇒ v 2 = 25 m/s
2(ρ − σ )r 2g
vt = From Bernoulli’s theorem,
9η 1 1
where, ρ = density of metal ball, p1 + ρv12 = p 2 + ρv 22
2 2
σ = density of viscous medium, 1
r = radius of ball p1 − p 2 = ρ(v 22 − v12 )
2
and η = coefficient of viscosity of medium.
1
Terminal velocity of first ball, = × 10 3 (252 − 42 )
2
2(ρ − σ )r12g
v t1 = 1 (Q Density of water, ρ = 10 3 kg/ m3 )

= 304500 Pa
2 (8 ρ2 − σ )r12g
= …(i) (Given, ρ 1 = 8 ρ2) 8 (a) Sometimes insects can walk on the surface of water due to

surface tension, when legs of insects are not being wet. In this
Similarly, for second ball, situation, the gravitational force on insect is balanced by force
2 (ρ2 − σ )r22g due to surface tension.
v t2 = …(ii)
9η Hence, both Assertion and Reason are correct and Reason is
On dividing Eq. (i) by Eq. (ii), we get the correct explanation of Assertion.
v t1 2(8ρ2 − σ )r12g 9η 9 (c) Surface tension is responsible for the shape of tiny water
= × drops. When falling freely in the absence of other forces like
v t2 2(ρ2 − σ )r22g 9η
gravity, shape of water drops is spherical because droplets of
Fluid Mechanics 637

water tend to be pulled into a spherical shape by the cohesive The mercury column in both arms of U-tube are at same level,
forces of the surface layer. therefore pressure in both arms will be same.
In the presence of gravity, the shape of heavy water drops is or p1 = p 2 ⇒ p 0 + ρ1 = p 0 + 1.2 ρ 2
not spherical when falling freely. It is oval shaped. Surface ∴ Density of ethyl alcohol,
tension of methyl alcohol is lower than surface tension of
ρ 1000
water. ρ2 = 1 = = 833.3 kgm −3 = 0.83 gcm −3
Hence, water does not have minimum surface tension among 1.2 1.2
all liquids. 13 (a) Let V and ρ be volume and density of solid respectively
Therefore, both Assertion and Reason are incorrect. and ρ′ be the density of water,
10 (b) Given, surface tension of water, T = 0.1N/m i.e. ρ′ = 10 3 kgm−3
Radius of small drops, r = 1mm = 10 −3 m Weight of body =Vρg
Volume of solid body outside water =V/4
If R be the radius of bigger drops, then volume remains
conserved. ∴Volume of solid body inside water = V − V / 4 = 3V / 4
V1 = V2 3V
i.e. Weight of water displaced by solid = × 10 3 × g
4 3 4 4
πR = 2 × πr 3 As solid body is floating, then
3 3
R = 2r = 2 × (10 −3 )3
3 3 Weight of body = Weight of water displaced by it
3V
∴ R = 21/ 3 × 10 −3 m Vρg = × 10 3 g
4
∴Change in energy during isothermal combination, 3
⇒ ρ = × 1000 = 750 kg m−3
∆E = T ⋅ ∆A = T (A2 − A1) 4
= 0. 1 (2 × 4πr 2 − 4πR 2 ) 14 (b) Here, r = 0.3 mm = 0.3 × 10 −3 m,v = 1 ms −1 ,
= 0.1× 4 π (2r − R )
2 2
η = 18 × 10 −5 poise = 18 × 10 −6 decapoise
= 0.4 π (2 × 10 −6 − 22/ 3 × 10 −6 ) Viscous force, F = 6 π η rv
22
= 0.4π × 10 −6 × 0.41 =6× × (18 × 10 −6 ) × (0.3 × 10 −3 ) × 1
7
= 0.52 × 10 −6 J = 1.018 × 10 −7 N

= 0.52 µJ −
~ 0.5 µJ 15 (b) Here, velocity head = 20 cm of Hg = 20 × 13.6 cm of water

11 (a) The rate of heat generation is equal to the rate of work As, velocity head = v 2 / 2g
done by the viscous force which in turn is equal to its power. v2
∴ 20 × 13.6 =
Rate of heat produced, 2 × 1000
dQ
= F × vT ⇒ v = 20 × 13.6 × 2 × 1000
dt
where, F is the viscous force and vT the terminal velocity. = 737.56 cms −1 = 7.3756 ms −1
As, F = 6πηr vT 16 (a) Volume of the water flowing per second,V = 20 L min−1
dQ 20 × 1000 3 −1 1
⇒ = 6πηrvT × vT = 6πηrvT2 ...(i) = m s = × 10 −3 m3s −1
dt 60 × (100 )3 3
From the relation of terminal velocity,
4
2 r 2 (ρ – σ ) Radius of the pipe, r = = 2 cm = 0.02 m
vT = ⋅ g, we get 2
9 η 22
Area of cross-section, A = πr 2 = × (0.02 )2 m2
vT ∝ r 2 ...(ii) 7
From Eq. (ii), we can rewrite Eq. (i) as Let v be the velocity of the flow of water at the given point.
dQ dQ Clearly,V = Av
∝ r ⋅ (r 2 )2 or ∝ r5 1 22
dt dt ⇒ × 10 −3 = × (0.02)2 × v
3 7
12 (a) Pressure at left hand of a U-tube,
7 × 10 −3
p1 = p 0 + ρ1gh1 = p 0 + ρ1 × 10 (10 × 10 −2 ) ⇒ v=
3 × 22 × (0.02)2
or p1 = p 0 + ρ1
~ 0.2639 ms −1

Pressure at right hand of a U-tube,
p 2 = p 0 + ρ 2gh2 = p 0 + ρ 2 × 10 (12 × 10 −2 ) 17 (b) Here, S = 0.06 Nm−1,
or p 2 = p 0 + 1.2 ρ 2 r1 = 2 cm = 0.02 m , r2 = 5 cm = 0.05 m
Since, bubble has two surfaces.
638 OBJECTIVE Physics Vol. 1

Initial surface area of the bubble between A and B on either side of the common surface
= 2 × 4πr12 = 2 × 4π (0.02)2 4S 4S
= −
= 32 π × 10 −4 m2 b a
Final surface area of the bubble This will be equal to (4S / r ).
= 2 × 4πr22 = 2 × 4π (0.05)2 = 200 π × 10−4 m2 4S 4S 4S 1  1 1 a − b
∴ = − or = −  =
Increase in surface area r b a r  b a ba
= 200 π × 10 −4 − 32 π × 10 −4 or r=
ab
= 168 π × 10 −4 m2 a −b
∴ Work done = Surface tension × Increase in surface area 22 (b) Increase in surface area,
= 0.06 × 168π × 10 −4 ∆A = (5 × 4 − 4 × 2) × 2 (Q Film has two surfaces)
= 0.003168 J = (20 − 8) × 2 cm2
18 (d) Pressure of two points lie in the same horizontal level = 24 cm2 = 24 × 10 −4 m2
should be same and p = hdg. So, work done,W = T ∆A
Both ends of the U-tube are open, so the pressure on both the 3 × 10 −4 = T × 24 × 10 −4
free surfaces must be equal, 1
⇒ T = = 0.125 N m−1
i.e. p1 = p 2 8
hoil ⋅ Soil g = hwater ⋅ Swater ⋅ g 23 (b) Ascent formula for capillary tube,
where, Soil = specific density of oil. 2T cos θ
h ⋅S ⋅g h=
⇒ Soil = water water ρgr
hoil ⋅ g
cos θ1 cos θ 2 cos θ 3
(65 + 65) × 1000 ∴ = =
From figure, Soil = ρ1 ρ2 ρ3
(65 + 65 + 10 )
Thus, cos θ ∝ ρ
= 928 kgm−3
∴ cos θ1 > cos θ 2 > cos θ 3 (Q ρ1 > ρ 2 > ρ 3 )
19 (c) Let the volume of ice-berg isV and its density is ρ. If this π
ice-berg floats in water with volumeVin inside it, then 0 ≤ θ1 < θ 2 < θ 3 <
2
Vin σg = Vρg
24 (c) According to question, the situation can be drawn as
 ρ following.
⇒ Vin =   V
 σ
 σ − ρ A
⇒ Vout = V − Vin =  V
 σ  (1–p)L d ρ
 1000 − 900  V nρ
= V = pL
 1000  10
Vout
⇒ = 0.1 = 10% Applying Archimedes’ principle,
V
weight of cylinder = (upthrust)1 + (upthrust) 2
20 (b) For an incompressible liquid, equation of continuity, i.e. ALdg = (1 − p ) LAρg + (pLA) nρg
1
Av = constant or A ∝ ⇒ d = (1 − p ) ρ + pnρ = ρ − pρ + n pρ
v
= ρ + (n − 1) p ρ = ρ [1 + (n − 1) p]
where, A = area of cross-section of tube
1 1
and v = speed of liquid. 25 (c) From Bernoulli’s theorem, p1 + ρv12 = p 2 + ρv 22
2 2
So, the speed of liquid flow is not uniform, but changes with
area of cross-section of tube. where, p1, p 2 are pressure inside and outside the roof and
Therefore, at the wider end speed will be low and at the v1, v 2 are velocities of wind inside and outside the roof.
narrow end speed will be high. Neglect the width of the roof. Pressure difference is
1 1
21 (c) According to given figure, p1 − p 2 = ρ (v 22 − v12 ) = × 1.2 (40 2 − 0 ) = 960 Nm −2
B A 2 2
Let the radius of curvature of the
Force acting on the roof is given by
common internal film surface of the
double bubble formed by two bubbles a F = (p1 − p 2 )A = 960 × 250 = 24 × 10 4 N = 2 .4 × 10 5 N
b
be r. Excess of pressure as compared to As the pressure inside the roof is more than outside to it. So,
atmosphere inside A is (4s / a ). In the force will act in the upward direction.
double bubble, the pressure difference r
i.e. F = 2.4 × 10 5 N (upward).
Fluid Mechanics 639

26 (c) Given, d = 2700 m, ρ = 10 3 kgm −3 32 (a) Statement given in option (a) is correct, while rest are
−11 −1 incorrect whose corrected form is as
Compressibility = 45.4 × 10 Pa
Water proof agents increase the angle of contact between
The pressure at the bottom of ocean is given by water and fibres.
p = ρgd = 10 3 × 10 × 2700 = 27 × 10 6 Pa Detergents decrease the surface tension of water.
So, fractional compression = compressibility × pressure Hydraulic machines work on the Pascal's law.
= 45.4 × 10 −11 × 27 × 10 6 = 1.2 × 10 −2 Venturimeter measures the flow speed of fluid in a pipe,
which can be compressible or incompressible.
27 (d) Let the velocity at point B is v B .
33 (d) In freely falling platform, a body experience
From conservation of total mechanical energy, weightlessness. So, the liquid will rise up to length of the
1
mg (h − L sin α ) = mv B2 capillary, i.e. height raised by the liquid will be 20 cm.
2
34 (c) Pressure difference between atmosphere and lungs
 1
⇒ v B2 = 2g (h − L sin α ) = 2g 10 − 2 ×  = Pressure difference at a depth h of fluid
 2
⇒ 760 − 750 mm × ρ Hg × g = h ρ w g
⇒ v B2 = 18g 13.6
h = 10 mm × = 13.6 cm
Now, let maximum height attained by water stream be H. 1
v 2 sin2 α 35 (a) Let R = radius of bigger drop
∴ H = L sin α + B
2g
and r = radius of smaller drop.
2
 1 4 4
18g   Then, πR 3 = 1000 × πr 3 ⇒ R = 10r
1  2 18 3 3
⇒ H =2× + = 1+ = 3.2 m
2 2g 8 The surface energy of smaller drop, E1 = S × 4πr 2 and the
28 (b) Gain in surface energy, surface energy of bigger drop,
E 1
E = 4π R 2 (n1/ 3 − 1)T E 2 = S × 4π (10r )2 ⇒ 1 =
E 2 100
= 4 × 3.14 × (10 −2 )2 [(1000 )1/ 3 − 1] × 0.075
V
= 8.48 × 10 −4
J 36 (c) Volume of part above the water surface =
3
29 (d) Given, height raised by liquid, V 2V
Volume of solid inside the water surface = V − =
h = 7.5 cm = 7.5 × 10 −2 m 3 3
2V
Surface tension, S = 7.5 × 10 −2 Nm −1 Weight of displaced water, w = × 10 × g
3
3
and contact angle, θ = 0 ° Now, weight of body = weight of displaced water
2S cos θ
As, r= ⇒ Vρ g =
2V
× 10 3 g
hρ g 3
where, ρ = density of water = 1000 kgm −3 2 2000
⇒ ρ = × 1000 = kg m−3
2 × 7.5 × 10 −2 × cos 0 ° 3 3
⇒ r=
7.5 × 10 −2 × 1000 × 10 37 (a) Change in surface area
2 2  D  2  d  2 
= = 2 × 10 −4 m = × 10 −3 m = 0. 2 mm = 2 × 4π   −    = 2π (D 2 − d 2 )
10 4 10    2
 2 
2
 a a2 ∴ Work done = Surface tension × Change in area
30 (a) The area of soap bubble = 4π   = 4π ⇒ A = π a2
 2 4 = 2πS (D 2 − d 2 )
work done TA
As, surface energy = = =T 38 (a) Re ≤ 1000 : Streamlined flow
area A
∴ Surface energy is equal to surface tension. 1000 < Re < 2000 ; Unstable flow
Total energy associated with surface, U = TA Re ≥ 2000 ; Turbulent flow
As soap bubbles are of two layers, so 39 (c) Here, if the surface area changes, it will change the
U = T × 2A = T × 2 π a 2 = 2π a 2 T surface energy as well. If the surface area decreases, it means
that energy is released and vice-versa.
31 (d) When the stones are unloaded into water, the water level
Change in surface energy = ∆A × T …(i)
falls because the volume of the water displaced by stones in
water will be less than the volume of water displaced when Let we have n number of drops initially.
stones are in the boat. So, ∆A = 4πR 2 − n (4πr 2 ) …(ii)
640 OBJECTIVE Physics Vol. 1

Volume is constant. From Eqs. (i) and (ii), we get


4 4
n πr 3 = πR 3 = V  3 3
So, ...(iii)  × V ( × ρl × g ) = Vi × ρl × g
3 3  2 5
From Eqs. (ii) and (iii), we get 9
or Vi = V
3 4π 3  4π 3 3 3 10
∆A = × R 3 − n r  = ×V − V
R 3 r  3  R r So, volume of the block above liquid
 1 1 1
= V = 10% of total volume.
⇒ ∆A = 3V  − 
R r 10
As R > r, so ∆A is negative. It means that surface area is 42 (c) Let thickness of layer be x.
decreased, so energy must be released. So, volume,V = area × x
 1 1 ⇒ V = A× x
Energy released = ∆A × T = 3VT  − 
R r V
x = V / A ⇒ 2r = (Q x = 2r )
40 (a) Assuming temperature is constant. A
As, from the Boyle’s law pV = constant. V
⇒ r= ...(i)
So, from the two cases at depth h and at the surface, 2A
T
pV1 1 = p 2V2 ...(i) and ∆p =
where, p1 = pressure at depth h, V1 = volume at depth h r
T
p 2 = pressure at surface, V2 = volume at surface. We know that, F = ∆p × A = ×A
r
4
⇒ p1 = (h + l ) ρg, V1 = πr 3 T
3 ⇒ F = ×A [From Eq. (i)]
 
V
4  
p 2 = lρg,V2 = π(3r )3  2A
3
F ×V
Putting these values in Eq. (i), we get ⇒ T=
2A2
 4  4
(h + l ) ρg   πr 3 = lρg ×   π (3r )3 where, F = 16 × 10 5 dyne,V = 0.04 cm3, A = 20 cm2
 3  3
So, (h + l ) = 27l 16 × 10 5 × 0.04
Q T=
⇒ h = 26 l 2 × 20 2
8 × 10 5 × 4 8 × 10 5 × 4
41 (b) According to Archimedes’ principle, weight of the wooden = =
block = weight of the liquid displaced 20 2 × 100 400 × 100
 60  3 = 8 × 10 × 10 = 80 dyne cm −1
5 −4

⇒ mg =  V × ρl × g = V × ρl × g …(i)
 100  5 43 (d) The value of angle of contact determines whether a liquid
[Here,V = total volume of block, ρl = density of liquid] will spread on the surface.
44 (b)
When beaker is kept in a lift, A
vA
Net weight of the block = mg + mg / 2
h
3mg  mg  B
= Q R − mg =  vB
2  2

Vρg dp dp
Net horizontal force (reaction), F = FB − FA = B − A
dt dt
= av Bρ × v B − av Aρ × v A
g/2
∴ F = aρ(v B2 − v A2 ) …(i)
According to Bernoulli’s theorem,
1 1
mg p A + ρv A2 + ρgh = p B + ρv B2 + (p A − p B )
2 2
1 2
⇒ ρ (v B − v A ) = ρgh ⇒v B2 − v A2 = 2gh
2
Again, for float condition, 2
Net weight of the block = Buoyant force From Eq. (i), we get
3mg F = 2aρgh ⇒ F ∝ h
⇒ = Vi × ρl × g …(ii)
2 45 (d) Given, h1 = 10 cm, h2 = 3.42 cm,
Here,Vi = volume of block inside liquid.
θ 2 = 135° and d 2 = 13.6 gcc −1
Fluid Mechanics 641

We know that, angle of contact of water, θ1 = 0 °. ∴


2
=
1
T1 h1d1 cos θ 2 cos 0 ° cos θ′
Here, =
T2 h2d 2 cos θ1 2 1
⇒ =
T1 10 × 1 cos 135° 1 cos θ′
⇒ =
T2 3.42 × 13.6 cos 0 ° 1
or cos θ′ =
T1 2 2
⇒ –~ ⇒ θ′ = 60 °
T2 13
52 (a) The height of capillary rise is inversely proportional to
r4
46 (c) We know that, pressure, p ∝ 1
radius (or diameter) of capillary tube, i.e. h ∝ .
l r
Given, l1 : l2 = 2 : 1and r1 : r2 = 1: 2 So, for smaller r, the value of h is higher.
Let l1 = 2l, l2 = l and r1 = r, r2 = 2r Hence, both Assertion and Reason are correct and Reason is
p1 r4 l p r4 l the correct explanation of Assertion.
Now, = 1 × 24 ⇒ 1 = ×
p2 l1 r2 p2 2l (2r )4 53 (a) The excess pressure inside the bubble,
p1 1 p 1 4T 4T
⇒ = ⇒ 1 = p= , then p1 = ...(i)
p2 2 × (2) 4
p2 32 r r1
∴Pressure across the tubes has ratio of 1: 32 or 32 : 1. 4T
and p2 = ...(ii)
4T r2
47 (b) We know that, the pressure at the end of tube, p =
r On dividing Eq. (i) by Eq. (ii), we get
Here, the pressure difference at the both end of tube, p1 4T / r1 r2 1
= = =
4T 4T r − r  p 2 4T / r2 r1 2
∆ p = ∆ p1 − ∆ p 2 = − = 4T  2 1 
r1 r2  r1 r2  54 (c) Time taken to empty a vessel of height h,
Given, r1 = 5 cm = 0.05 m and r2 = 3 cm = 0.03 m t = 2h / g
 0.05 − 0.03  ∴ Time taken in emptying a vessel of height h /2,
∆p = 4 × 0.03
 0.05 × 0.03 
2h / 2 h
4 × 0.03 × 0.02 0.0024 t′ = =
= = = 1.6 Pa g g
0.05 × 0.03 0.0015
4T t′ 1
48 (a) We know that, for soap bubble, p = ∴ =
r t 2
Given, p1 = 3p 2
t 10
4T  4T  or t′ = = ≈ 7 min
Here, = 3   ⇒ r2 = 3r1 …(i) 2 2
r1  r2 
4 3 4 55 (a) From the principle of continuity, A1v1 = A2 v 2
V1 = πr1 and V2 = πr23 A1v1 4.20 × 5.18
3 3 ⇒ v2 = = = 2.86 ms −1
4 4 A2 7.60
= π (3r1) = π (27r13 )
3
3 3 56 (b) Volume of liquid displaced is more than the volume of
V1 (4/ 3) πr13 1 water formed. So, the level of water in the beaker falls.
⇒ = =
V2 (4/ 3) π 27r13 27 57 (b) The excess pressure of soap bubble,
49 (b) We know that, capillary rises, 4T
p=
2S cos θ 1 R
h= ⇒h ∝
ρgr r or hρ g =
4T
(Q p = hρg )
r R
⇒ h' = h = 20 × 3 = 60 mm
r′ Rhρg 1 × 10 −2 × 2 × 10 −3 × 0.8 × 10 3 × 9.8
⇒ T= =
4 4
50 (c) We have, 1 P = 1 g cm −1s −1
= 0.0392 Nm−1
∴ 1 Pl = 10 gcm−1s −1 = 10 P
58 (b) The equation of continuity,
51 (c) If a liquid can rise to a height h, but the tube has A1v1 = A2 v 2
insufficient height h′, then the angle of contact increases from
h′ 2
θ to θ′ given by
h
= ⇒ A × 4 = A ×v2
cos θ cos θ′ 3
Here, h = 2 cm, h′ = 1cm, θ = 0 ° ⇒ v 2 = 6 ms −1
642 OBJECTIVE Physics Vol. 1

From Bernoulli’s theorem, 62 (d) Using the law of buoyancy, we get


1 1 Vσg = 0.6 Vσ1g , for the part of body inside water and
p + ρgh1 + ρv12 = p + ρgh2 + ρv 22
2 2 Vσg = 0.4Vσ 2 g , for the part of body inside oil, hence we get
1 2 0.6Vσ1g = 0.4Vσ 2g
or g (h1 − h2 ) = (v 2 − v1 )
2
2 0.6σ1
1 ⇒ 1=
or g × h = [(6)2 − (4)2] (Q h1 − h2 = h ) 0.4σ 2
2 So, we get
1 σ2 6 3
or 10 × h = [36 − 16] = = = 1.5
2 σ1 4 2
20
or h= = 1m 63 (b) Time taken by water to reach the foot,
20
2(H − h )
59 (c) Specific gravity of Al = 2.7 t=
g
Density of Al = 2.7 × 10 3 kg m−3
2 r 2 (ρ − σ )g
Terminal velocity, v = h

2 × (1 × 10 −3 )2 (2.7 × 10 3 − 1000) × 9.8 H
v=
9 × 8 × 10 −4
⇒ v = 4.6 ms −1
πpr 4
60 (d) As,V = x
8ηl
Velocity of water coming out of hole,
∴ V ∝ pr 4 (η and l are constants)
4
v = 2gh
V2  p 2   r2 
4
 1 1
∴ =    =2×  = ∴ Emerging stream of water strike the foot at distance,
V1  p1   r1   2 8
x =v × t
Q
⇒ V2 = 2(H − h )
8 = 2gh ×
g
61 (c) Air flow from smaller bubble to bigger bubble as the
pressure in smaller bubble is higher. = 2 h (H − h )
CHAPTER
14

Thermometry, Thermal
Expansion and Kinetic
Theory of Gases
The branch of science which deals with the measurement of temperature of a
substance is known as thermometry.
Almost all substances (solids, liquids and gases) expand on heating and contract on
Inside
cooling. Few exceptions are: water from 0°C to 4°C and silver iodide from 80°C 1 Temperature
to 140°C contract on heating and expand on cooling. The expansion of a Scales of temperature
substance on heating is called thermal expansion of that substance. Thermometers
Further, in this chapter, we will study the kinetic theory of gases, which relates the 2 Thermal expansion
macroscopic properties (pressure, temperature, etc.) of gases to the microscopic Types of thermal expansion
properties (velocity, speed, etc.) of gas molecules. Variation of density with
temperature
3 Kinetic theory of ideal gases
TEMPERATURE Pressure of an ideal gas
(Concept of pressure)
The degree of hotness or coldness of a body is known as the temperature of the Ideal gas equation
body. An object at higher temperature is said to be hotter than the one at lower Molecular velocities
temperature. Mean free path (λ)
4 Ideal gas laws
Scales of temperature Boyle’s law
Charles’ law
The measurement of temperature is done by some specified scales. In any scale of Gay Lussac’s law or pressure law
temperature, two fixed points are considered: one is ice point (Lower Fixed Dalton’s law of partial pressure
Point-LFP) and other is steam point (Upper Fixed Point-UFP). Generally, these Graham’s law of diffusion
points are freezing point and boiling point of water at 1 atm. Behaviour of real gases
van der Waals’ equation
Some of the commonly used temperature scales are discussed below
5 Degree of freedom
(i) The Celsius scale (°C) This scale was designed by Anders Celsius. It
Maxwell’s law of equipartition
defines the ice point temperature as 0°C and the steam point temperature as of energy
100°C. The space between 0°C and 100°C is divided into 100 equal Critical temperature, pressure
intervals and each interval is known as 1°C. and volume
100 ° C
0° C ←→ 100°C (100 equal parts)
644 OBJECTIVE Physics Vol. 1

(ii) The Fahrenheit scale (°F) This scale was designed Example 14.3 On a new scale of temperature (which is
by Fahrenheit. It defines the ice point temperature linear) and called the W scale, the freezing and boiling
as 32°F and the steam point as 212°F. The space points of water are 39°W and 239°W, respectively. What will
between 32°F and 212°F is divided into 180 equal be the temperature on the new scale, corresponding to a
intervals and each interval represents 1°F. temperature of 39°C on the Celsius scale?
Sol. In general, whenever we have to go from any known scale
180 ° F
32° F ←→ 212° F (180 equal parts) to any unknown scale, then we follow the equation
(iii) The Kelvin scale (K) This scale was designed by (Temperature on one scale) − (LFP for known scale)
Kelvin. It defines the ice point temperature as (UFP − LFP)known
273.15 K and the steam point temperature as (Temperature on other scale) − (LFP for unknown scale)
373.15 K. The space between 273.15 K and =
(UFP − LFP)unknown
373.15 K is divided into 100 equal intervals and
39 − 0 t − 39
each interval represents 1 K. Kelvin is the SI unit of ⇒ = ⇒ t = 117° W
temperature. 100 − 0 239 − 39
100 K
273.15 K ←→ 373.15 K (100 equal parts) Example 14.4 An arbitrary scale has the ice point at −20°
and the steam point at 180°. When the thermometer reads
Conversion of temperature from one 5°, then find the reading of centigrade thermometer.
scale to another Sol. In order to convert the temperature from one scale to
In order to convert the temperature from one scale to another,
another, the following relation should be kept in mind, i.e. C −0 t − (−20)
=
Temperature on one scale − LFP (ice point) 100 − 0 180 − (−20)
UFP (steam point) − LFP (ice point) Here, t = 5°
C 5 + 20
Temperature on other scale − LFP (ice point) ⇒ =
= 100 200
UFP (steam point) − LFP (ice point)
∴ Reading of centigrade thermometer,
In simple terms, the temperature measured by different C = 12.5°C
scales is given as
Example 14.5 A faulty scale has its fixed points marked as
C − 0 F − 32 K − 273
= = 5° and 95°. The temperature of a body as measured by the
100 180 100 faulty scale is 59°. Find the correct temperature of the body
Temperature difference measured by different scales is on Celsius scale.
given as Sol. Let θ 0 be the lower fixed point of the faulty scale and n be
∆C = ∆K = (5/9) ∆F the number of divisions between its lower and the upper
Example 14.1 Express the temperature of 60°F in degree fixed points. If a temperature C on Celsius scale corresponds
celsius and in kelvin. to temperature θ on the faulty scale, then
C − 0 F − 32 C −0 θ − θ0
Sol. Temperature in degree celsius, = = …(i)
100 180 100 n
5 5 Here, θ 0 = 5° , n = 95 − 5 = 90 and θ = 59°
C = (F − 32) = (60° − 32°) = 15.55° C Therefore, the Eq. (i) becomes
9 9
Temperature in kelvin, K = C + 273.15 C − 0 59 − 5
=
= 15.55° C + 273.15 100 90
= 288.7 K 54
or C = × 100 = 60° C
90
Example 14.2 The temperature of an iron piece is raised from
30°C to 90°C. What is the change in its temperature on the
Fahrenheit scale and on the Kelvin scale? Thermometers
Sol. Given, ∆C = 90° − 30° = 60°C The thermometers work on the thermometric property,
i.e. the property which changes with temperature like any
Temperature difference on Fahrenheit scale, physical quantity such as length, volume, pressure, resistance,
9 9
∆F = ∆C = (60°C) = 108°F etc. which varies linearly with a certain range of
5 5 temperature. Let X denotes the thermometric physical
Temperature difference on Kelvin scale, ∆K = ∆C = 60 K
Thermometry, Thermal Expansion and Kinetic Theory of Gases 645

quantity and X 0 , X100 and Xt be its values at 0°C, 100°C and intensity of radiations received from the body.
t°C respectively. Then, They are based on the fact that the amount of
radiation emitted from a body per unit area per
 X − X0  second is directly proportional to the fourth power
Temperature, t =  t  × 100° C
 X100 − X 0  of temperature, i.e. Stefan’s law. These can be used
to measure temperature ranging from 800°C to
6000°C.
Different types of thermometers
Some different types of thermometers are given below Example 14.6 The pressures of the gas filled in the bulb of a
constant volume gas thermometer are 66 cm and 88 cm of
(i) Mercury thermometer In this thermometer, the
mercury column at 0°C and 100°C, respectively. When its
length of a mercury column from some fixed point is bulb is immersed in a liquid placed in a vessel, its pressure is
taken as thermometric property. If length of mercury 82.5 cm of mercury column. Calculate the temperature of
column at 0° and 100° are l 0 and l100 respectively the liquid.
and at t°, the length of mercury is lt . Then, Sol. Given, pressure at 0°C, p 0 = 66 cm of Hg (mercury) column
 l − l0  Pressure at 100°C, p100 = 88 cm of Hg column
t= t  × 100 ° C
 l100 − l 0  Pressure at unknown temperature t, pt = 82.5 cm of Hg column
(ii) Constant volume gas thermometer It works on According to formula, the unknown temperature is
the principle of change in pressure with temperature  p − p0 
t= t  × 100° C
when the volume is kept constant. If p 0 , p 100 and  p100 − p 0 
p t are the pressures of gas at temperatures 0°C,  82.5 − 66
100°C and unknown temperature (t°C) respectively =  × 100° C
 88 − 66 
keeping the volume constant, then 16.5
 p − p0  = × 100° C = 75° C
t= t × 100  ° C 22
 p 100 − p 0  Example 14.7 The following observations were recorded on a
V − V0 platinum resistance thermometer. Resistance at melting point
Note For constant pressure gas thermometers, t = t × 100° C
V100 − V0 of ice = 3.70 Ω, resistance at boiling point of water at
normal pressure = 4.71 Ω and resistance at t ° C = 529. Ω.
(iii) Platinum resistance thermometer It works on the Calculate
principle of variation of resistance of metals with (i) temperature coefficient of resistance of platinum.
temperature. (ii) value of temperature t.
If R 0 , R 100 and R t are the resistances of a platinum Sol. Given, resistance at melting point of ice, R 0 = 3.70 Ω
wire at temperature 0°C, 100°C and unknown
temperature (t° C), respectively. Resistance at boiling point of water at normal pressure,
R100 = 4.71 Ω
 R − R0  R  Resistance at t ° C, Rt = 5.29 Ω
Then, t =  t × 100  ° C =  t × 273.16 K
 R 100 − R 0   R tr  (i) According to the formula, temperature coefficient of
resistance is given by
where, R tr = resistance of wire at 273.16 K.
R − R 0 4.71 − 3.70 1.01
Here, temperature coefficient of resistance (α ) is α = 100 = = = 2.73 × 10−3/° C
R 0 × 100 3.70 × 100 370
given by
(ii) According to the formula, for temperature t, we have
R 100 − R 0 R − R0
α= t = 100° C × t
R 0 × 100 R100 − R 0
5.29 − 3.70 1.59
(iv) Pyrometers Pyrometers are the devices used to = 100° C × = 100° C × = 157.4° C
measure the temperature by measuring the 4.71 − 3.70 1.01
OBJECTIVE Physics Vol. 1

CHECK POINT 14.1


1. A device used to measure temperature is (a) 40 (b) 313
(a) hygrometer (b) thermometer (c) 574.25 (d) 301.25
(c) barometer (d) calorimeter 6. A faulty thermometer has its fixed points marked 5° and
2. A difference of temperature of 25°C is equivalent to a 95°. lf the temperature of a body as shown on the Celsius
difference of scale is 40°C, then its temperature shown on this faulty
(a) 45°F (b) 72°F (c) 32°F (d) 25°F thermometer is
(a) 39° (b) 40°
3. At what temperature, does the temperature in Celsius and (c) 41° (d) 44.4°
Fahrenheit equalise?
(a) − 40° (b) 40° 7. The readings of a bath on Celsius and Fahrenheit
(c) 36.6° (d) 38° thermometers are in the ratio 2 : 5. The temperature of the
bath is
4. The freezing point on a thermometer is marked as − 20° and (a) − 26.66°C (b) 40°C
the boiling point as130°. A temperature of human body (c) 45.71°C (d) 26.66°C
(34°C) on this thermometer will be read as
(a) 31° (b) 51° 8. The reading of air thermometer at 0°C and100°C are
(c) 20° (d) None of these 50 cm and 75 cm of mercury column, respectively.
The temperature at which its reading is 80 cm of mercury
5. The temperature of a body on Kelvin scale is found to be x column is
Kelvin. When it is measured by Fahrenheit thermometer, it (a) 105°C (b) 110°C
is found to be x°Fahrenheit, then the value of x is (c) 115°C (d) 120°C

∆l
THERMAL EXPANSION Then, increase in length, ∆l = l 0 α∆T ⇒ α =
l 0 ∆T
When matter is heated without any change in its state, it Therefore, final length, l = l 0 + ∆l
usually expands. This phenomenon of expansion of matter
on heating is called thermal expansion. l = l 0 [1 + α ∆T ]
On heating the matter, the energy of atoms increases and
hence, the average distance between them also increases Here, the constant α is called the coefficient of linear
which results in thermal expansion. Thermal expansion is expansion of the material of the rod and its unit is K –1 or
minimum in solids and maximum in gases because (° C ) –1. Its dimensions are [M 0 L 0 T 0 θ −1].
intermolecular forces are maximum in solids and minimum Note Test tubes, beakers and cubicles are made up of pyrex-glass or
in gases. silica because they have very low value of coefficient of linear
expansion.

Types of thermal expansion Example 14.8 A steel ruler exactly 20 cm long is graduated
Solids can expand in one dimension (linear expansion), two to give correct measurements at 20° C.
dimensions (superficial expansion) and three dimensions (i) Will it give readings that are too long or too short at lower
(volume expansion) while liquids and gases usually expand temperatures?
only in three dimensions. (ii) What will be the actual length of the ruler be when it is used
in the desert at a temperature of 40° C ?
Thermal expansion in solids (Take, α steel = 1.2 × 10−5 ° C –1)
There are three types of thermal expansion Sol. (i) If the temperature decreases, the length of the ruler
also decreases due to thermal contraction. Below 20°C,
1. Linear expansion each centimetre division is actually somewhat shorter
Suppose the temperature of a thin uniform rod of length l than 1.0 cm, so the steel ruler gives readings that are
is changed from T to T + ∆T , then its length becomes too long.
l + ∆l . (ii) Given, change in temperature,
∆l
∆T = T2 − T1 = 40° − 20° = 20° C
Linear expansion Length, l = 20 cm
At 40°C, the increase in length of the ruler,
Fig. 14.1 Thermal expansion in rod ∆l = lα∆T = (20) (1.2 × 10−5 ) (20)
= 0.48 × 10−2 cm
Thermometry, Thermal Expansion and Kinetic Theory of Gases 647

∴ The actual length of the ruler, Example 14.10 A metal ball having a diameter of 0.4 m is
l ′ = l + ∆l = 20 + 0.48 × 10−2 heated from 273K to 360 K. If the coefficient of areal
expansion of the material of the ball is 0.000034 K −1, then
= 20.0048 cm determine the increase in surface area of the ball.
0.4
Example 14.9 The length of a steel rod is 5 cm longer than Sol. Given, diameter = 0.4 m and radius, r = = 0.2 m
2
that of a brass rod. If this difference in their lengths is to
remain same at all temperatures, then find the length of brass ∴ Area of ball, A 0 = 4πr 2 = 4 × π × (0.2)2 = 0.5024 m2
rod. (Coefficients of linear expansion for steel and brass are Change in temperature, ∆T = T2 − T1 = 360 K − 273 K = 87 K
12 × 10 −6 /°C and 18 × 10 −6 /°C, respectively.)
Coefficient of areal expansion, β = 0.000034 K −1
Sol. Given, ∆l s − ∆l b = ∆l
Increase in surface area, ∆A = βA 0∆T
Let l s = l cm ⇒ l b = (l − 5) cm ⇒ ∆A = 0.000034 × 0.5024 × 87 = 0.001486
(Q Steel rod is 5 cm longer than that of a brass rod) = 1.486 × 10−3 m2
α s = 12 × 10−6 /°C and α b = 18 × 10−6 /° C
3. Volume expansion
As, ∆l s = l s αt1
Suppose the temperature of a metal block of volume V0 is
∆l s
∴ αs = increased from T to T + ∆T , then its volume becomes
l s × t1 V + ∆V .
∆l Volume expansion ∆V
⇒ 12 × 10−6 = …(i)
l ×t
Similarly , ∆l b = l b αt 2
∆l b
⇒ αb =
lb × t2
∆l
⇒ 18 × 10−6 = …(ii)
(l − 5) × t
Fig. 14.3 Thermal expansion in solid cube
Dividing Eq. (i) by Eq . (ii), we get
12 × 10−6 ∆l /l × t Now, increase in volume, ∆V = V0 γ∆T
=
18 × 10 −6
∆l /(l − 5) × t Therefore, new volume,V = V0 + ∆V = V0 [1 + γ∆T ]
2 l−5 Here, γ is called the coefficient of volume expansion
⇒ = and its unit is K –1 or (°C) −1.
3 l
⇒ 2l = 3l − 15 ⇒ l =15 cm
So, l s = 15 cm and l b = l − 5 = 15 − 5 = 10 cm
Relation between α, β and γ
For solids, α, β and γ are related as
2. Superficial (areal) expansion
α : β : γ = 1: 2 : 3
Suppose the temperature of a metal sheet of an initial
surface area A0 is changed from T to T + ∆T , then its area i.e. γ = 3 α, β = 2 α, 2 γ = 3 β
becomes A + ∆A. or Percentage change in area = 2 × Percentage change in length
Superficial expansion
Percentage change in volume = 3 × Percentage
change in length
Example 14.11 On heating a glass block of 10000 cm 3 from
25° C to 40° C, its volume increases by 4 cm 3 . Determine the
coefficient of linear expansion of glass.
∆A Sol. Given, volume,V = 10000 cm 3
Fig. 14.2 Thermal expansion in thin sheet
Increase in temperature, ∆T = (40° − 25° ) = 15° C, ∆V = 4 cm 3
Now, increase in area, ∆A = A0 β∆T ∆V 4
∴ Coefficient of cubical expansion, γ = =
Therefore, final area, A = A0 + ∆A V ⋅ ∆T 10000 × 15
A = A0 [1 + β∆T ] = 26.67 × 10−6 ° C−1
∴ Coefficient of linear expansion,
where, β is the coefficient of superficial (areal) γ 26.67 × 10−6
α= = = 8.89 × 10−6 ° C−1
expansion and its unit is (°C) −1 or K −1. 3 3
648 OBJECTIVE Physics Vol. 1

Example 14.12 The volume of mercury in the bulb of a Hence, if temperature increases, then time period
thermometer is 10 −6 m 3 . The area of cross-section of the increases and vice-versa. On increasing the
capillary tube is 2 × 10 −7 m 2 . If the temperature is raised by temperature, pendulum clock runs slowly and
100° C, then find the increase in the length of the mercury vice-versa.
column. (Take, γ Hg = 18 × 10 −5 / ° C )
(iii) If a metal rod fixed at both the ends with rigid
Sol. From volume expansion relation, ∆V = V × γ ∆T …(i)
support is heated, then due to thermal expansion, it
−5
where, γ = 18 × 10 /°C, exerts force on rigid supports.
initial volume,V = 10−6m3 l, α
and increase in temperature, ∆T = 100°C
Putting these values in Eq. (i), we get
∆V = 10−6 × 18 × 10−5 × 102 = 18 × 10−9 m3 Fig. 14.5 A rod with a rigid support

Now, ∆V = A × ∆l ⇒ 18 × 10−9 = 2 × 10−7 × ∆l F /A F l F l


−2
Now,Y = = × = ×
or 9 × 10 = ∆l or ∆l = 9 cm ∆l / l A ∆ l A l α∆T
where, ∆T is rise in temperature, α is coefficient of
Some effects of thermal expansion linear expansion,Y is Young’s modulus of material of
in solids rod, l is length of rod and A is area of cross-section
Some effects of thermal expansion in solids are given below of rod.
(i) If a bimetallic strip (strip made up of two strips of Force on rigid supports,
different materials placed one over another and rigidly F = YAα∆T
joined) is heated, then it bends such that the strip of
material having high value of α will be on convex side. Example 14.13 A clock which keeps correct time at 20°C is
subjected to 40°C. If coefficient of linear expansion of the
Cu pendulum is 12 × 10−6 / ° C. Determine the gain or loss in time
Cu
period.
Sol. Given, coefficient of linear expansion, α = 12 × 10−6/ ° C
Fe θ2 > θ1 Fe
Change in temperature, ∆θ = θ 2 − θ1 = 40° − 20° = 20 °C
(θ1) ( θ 2)
Initial time period, T = 24 × 60 × 60 = 24 × 3600 = 86400 s
Fig. 14.4 A bimetallic strip is heated from θ 1 to θ 2 Now, change in time period,
1 1
Since α Cu > α Fe , increase in length of copper strip ∆T = T α ∆θ = × 86400 × 12 × 10−6 × 20
2 2
will be greater, hence strip of copper will be on
= 12 × 10 × 86400 = 10368 × 10−3
−5
convex side.
= 10.4 s−1
(ii) If the temperature of a pendulum increases, then
its length increases, hence its time period changes. Example 14.14 A second’s pendulum clock has a steel wire.
Time period of pendulum, The clock is calibrated at 20°C. How much time does the
∆T ∆l l α∆θ clock loss or gain in one week when the temperature is
T = 2π l / g ⇒ = = increased to 30°C? (Take, α steel = 1.2 × 10 −5 ° C −1 )
T 2l 2l
Sol. The time period of second’s pendulum is 2 s. As the
Here, ∆θ is change in temperature and α is coefficient temperature increases, length and hence, time period
of linear expansion of material of pendulum. increases. Clock becomes slow and it loses the time. The
1 change in time period,
Change in time period, ∆T = α ∆θT
2 1  1
∆T = Tα∆θ =   (2) (1.2 × 10−5 )(30 − 20)
2  2
Time lost by the clock in a day (T = 86400 s),
1 1 = 1.2 × 10−4 s
∆t = × α∆θ × T = × α∆θ ×86400 = 43200 α∆θ s ∴ New time period,
2 2
T ′ = T + ∆T = (2 + 1.2 × 10−4 ) = 2.00012 s
Change in time (shown by pendulum clock),
∴ Time lost in one week,
∆T
∆t = t  ∆T  (1.2 × 10−4 )
T ∆t =  t = (7 × 24 × 3600) = 36.28 s
 T′  (2.00012)
Thermometry, Thermal Expansion and Kinetic Theory of Gases 649

Example 14.15 An aluminium cylinder 10 cm long, with a 4°C, water contracts and beyond 4°C, it expands. Thus,
cross-sectional area of 20 cm 2 , is to be used as a spacer density of water reaches a maximum value of 1000 kgm−3
between two steel walls. At 17.2°C, it just slips in between at 4°C.
the walls. When it warms to 22.3°C, calculate the stress in
the cylinder and the total force it exerts on each wall.
Assuming that, the walls are perfectly rigid and a constant
distance apart. (For aluminium, α = 2.4 × 10 −5 K −1)

Density
Volume
Sol. As we know, for aluminium,Y = 7.0 × 1010 Pa
and α = 2.4 × 10−5 K −1 4°C 4°C
Change in temperature, ∆T = T2 − T1 = (22.3 − 17.2)° C Temperature Temperature
= 5.1° C = 5.1 K (a) (b)
Area, A = 20 cm2 = 20 × 10−4 m2 Fig. 14.6 (a) Thermal expansion of water
(b) Variation of density of water with temperature

A= 20 cm2 Example 14.16 A non-conducting body floats in a liquid at


25° C with one-third of its volume immersed in the liquid.
When liquid temperature is increased to 105°C, half of
10 cm
body’s volume is immersed in the liquid. Then, find the
F
∴ Stress = = Yα∆T = (0.70 × 1011Pa)(2.4 × 10−5 K −1)(5.1 K) coefficient of real expansion of the liquid (neglecting the
A expansion of container of the liquid).
= 1.68 × 5.1 × 106 = 8.6 × 106 Pa V2 − V1
Sol. Coefficient of real expansion, γ r = …(i)
The total force F is the product of cross-sectional area and the V1(T2 − T1)
stress.
1 1
F = Stress × Area = (8.6 × 106 Pa ) (20 × 10−4 m2 ) = 1.7 × 104N Here, V2 = , V1 =
2 3
Thermal expansion of liquids and (T2 − T1) = (105 − 25) = 80° C

Liquids do not have linear and superficial expansion but Putting these values in Eq. (i), we get
they only have volumetric expansion.  1 1
 − 
 2 3 1
Since, liquids are always heated in a vessel, so initially on γr = = = 62.5 × 10−4 ° C−1
heating the system (liquid + vessel), the level of liquid in 1/3 (80) 160
vessel falls (as vessel expands more since it absorbs heat
and liquid expands less) but later on, it starts rising due to Example 14.17 The coefficient of apparent expansion of a
liquid when determined using two different vessels A and B
faster expansion of the liquid.
are γ 1 and γ 2 , respectively. If the coefficient of linear
Thus, liquids have two coefficients of volume expansion expansion of vessel A is α. Find the coefficient of linear
(i) Coefficient of real expansion (γ r ), which is due to expansion of the vessel B.
the actual increase in volume of liquid due to heating. Sol. From thermal expansion in liquid, γ real = γ app + γ vessel
Real increase in volume Given, γ vessel = 3 α
γr =
Original volume × Change in temperature For vessel A, γ real = γ1 + 3α A
(ii) Coefficient of apparent expansion (γ a ), which is For vessel B, γ real = γ 2 + 3αB
due to apparent increase in the volume of liquid, if Hence, γ1 + 3α A = γ 2 + 3αB
expansion of vessel containing the liquid is also taken γ − γ2
⇒ αB = 1 + αA
into account. 3
Observed (or apparent) increase in volume
γa = Example 14.18 A glass vessel of volumeV0 is completely filled
Original volume × Change in temperature with a liquid and its temperature is raised by ∆T. What volume
of the liquid will overflow ? Coefficient of linear expansion of
Now, γr = γ a + γ g glass = α g and coefficient of volume expansion of the liquid = γ l .
Here, γ g = expansion of vessel. Sol. Volume of the liquid that will overflow
= Increase in the volume of the liquid
Anomalous behaviour of water
− Increase in the volume of the vessel
Generally, with increasing temperature, the coefficient of = [V0 (1 + γ l ∆T ) − V0] − [V0 (1 + γ g ∆T ) − V0]
volume expansion of liquids is about ten times greater than
that of solids. Water is an exception to this rule. From 0°C to = V0∆T (γ l − γ g ) = V0∆T (γ l − 3α g ) (Q γ g ≈ 3α g )
650 OBJECTIVE Physics Vol. 1

Example 14.19 Assume that one early morning when the Example 14.20 A vertical column whose density is 50 g cm −3
temperature is 10°C, a driver of an automobile gets his at 50°C balances another column of same liquid whose density
gasoline tank which is made of steel, filled with 75 L of is 60 g cm −3 at 100°C. Find the coefficient of absolute
gasoline, which is also at 10°C. During the day, the expansion of the liquid.
temperature rises to 30°C, how much gasoline will overflow?
Sol. Given, density of vertical column, ρ = 50 g cm−3
(Take, α for steel = 1.2 × 10 −5 ° C −1 and γ for gasoline
Temperature, T1 = 50 °C
= 9.5 × 10 −4 ° C −1)
Density of another column, ρ′ = 60 g cm−3
Sol. Change in volume of gasoline, ∆Vg = γ gV∆T
Temperature, T2 = 100° C
= 9.5 × 10−4 × V × 20 = 190 × 10−4V
Now, variation of density with temperature is given by
Change in volume of steel tanks, ∆Vs = γ sV∆T ρ′ = ρ(1 − γ∆T )
= 3α sV∆T = 3 × 1.2 × 10−5 × V × 20 = 7.2 × 10−4 V ⇒ 60 = 50 [1 − γ (100 − 50)]
∴ Volume of gasoline that will overflow ⇒
60
= [1 − γ (50)] ⇒ 50 γ = 1 −
6
= ∆Vg − ∆ Vs = (190 × 10−4 − 7.2 × 10−4 )V 50 5
1 1
= 182.8 × 10−4 × 75 = 1.37 L ⇒ γ=− =−
5 × 50 250
Thermal expansion of gases ⇒ γ = − 0.004 /° C
On heating, gases expand more than solids or liquids and Example 14.21 A sphere of diameter 7 cm and mass 266.5 g
equal volumes of different gases expand equally, when floats in a bath of liquid. As the temperature is raised, the
heated by the same amount. sphere just sinks at a temperature of 35°C. If the density of
All gases have coefficient of volume expansion γ V with the liquid at 0° C is 1.527 g/cm 3 , find the coefficient of
volume variation given by cubical expansion of the liquid.
Sol. The sphere will sink in the liquid at 35°C when its density
V = V0 (1 + γ V ∆T ) becomes equal to the density of liquid at 35° C
and pressure variation given by p = p 0 (1 + γ p ∆T ) The density of sphere,
Mass 266.5
ρ35 = =
Volume 4  22  7 3
Variation of density with ×  × 
3  7   2
temperature = 1.483 g/cm 3
Most of the substances expand when they are heated, i.e. Now, ρ0 = ρ35 [1 + γ∆T ]
volume of a given mass of a substance increases on 1.527 = 1.483 [1 + γ × 35]
 1 1.029 = 1 + γ × 35
heating, so the density should decrease  as ρ ∝  .
 V 1.029 − 1
⇒ γ=
Let us see how the density (ρ) varies with increase in 35
temperature. = 0.00083 / ° C
m 1
ρ= or ρ ∝ (For a given mass) Example 14.22 The coefficient of volume expansion of
V V glycerine is 49 × 10 −5 ° C −1. What is the fractional change
ρ′ V V V 1 in its density for a 30°C rise in temperature?
∴ = = = =
ρ V ′ V + ∆V V + γV∆T 1 + γ∆T Sol. Given, coefficient of volume expansion,
ρ γ = 49 × 10−5 ° C−1
∴ ρ′ =
1 + γ∆T Temperature, T = 30° C
This expression can also be written as Let ρ0 is density at 0°C and ρt is density at t°C.
∴ Fractional change in density,
ρ′ = ρ (1 + γ∆T ) –1
ρ0 − ρt
= γ ∆T
As γ is small, (1 + γ∆T ) −1 ≈ 1 − γ∆T ρ0
∴ ρ′ ≈ ρ (1 − γ∆T ) = 49 × 10−5 × 30 = 0.0147
CHECK POINT 14.2
1. A bar of iron is 10 cm at 20°C. At 19°C, it will be in temperature. There is no bending of the rods. If
(α Fe = 11 × 10−6 / °C) α 1 : α 2 = 2 : 3, the thermal stresses developed in the two
rods are equal provided Y1 : Y2 is equal to
(a)11 × 10−6 cm longer (b)11 × 10−6 cm shorter
(c)11 × 10−5 cm shorter (d)11 × 10−5 cm longer
2. ABCD is a thin iron sheet. A hole with radius r is made in it
at the middle. If we heat up the sheet, the radius of the hole
(a) will increase
(b) will decrease
(c) will remain constant
(a) 2 : 3 (b) 1 : 1 (c) 3 : 2 (d) 4 : 9
(d) can increase or decrease depending upon size
10. A steel rod of diameter 1 cm is clamped firmly at each end
3. A steel tape gives correct measurement at 20°C. A piece of when its temperature at 25°C, so that it cannot contract on
wood is being measured with the steel tape at 0°C. The cooling.The tension in the rod at 0°C is approximately
reading is 25 cm on the tape, the real length of the given
(α = 10−5 / ° C , Y = 2 × 1011 Nm−2)
piece of wood must be
(a) 25 cm (b) < 25 cm (a) 4000 N (b) 7000 N (c) 7400 N (d) 4700 N
(c) > 25 cm (d) Cannot say 11. A solid ball of metal has a spherical cavity inside it. If the
4. Two rods of lengths l1 and l2 are made of materials whose ball is heated, the volume of the cavity will
(a) increase (b) decrease
coefficients of linear expansions are α 1 and α 2 , respectively.
(c) remain unchanged (d) Data insufficient
If the difference between two lengths is independent of
temperature, then 12. The coefficient of linear expansion of crystal in one direction
l α l α α2 α2 is α 1 and that in other two directions perpendicular to it is
(a) 1 = 1 (b) 1 = 2 (c) l 22 α1 = l12 α (d) 1 = 2 α 2. The coefficient of cubical expansion is
l2 α 2 l 2 α1 l1 l2
(a) α1 + α 2 (b) 2α1 + α 2
5. The radius of a ring is R and its coefficient of linear (c) α1 + 2α 2 (d) None of these
expansion is α. If the temperature of ring increases by θ,
then its circumference will increase by 13. Ratio among linear expansion coefficient (α), area
(a) π Rα θ (b) 2π Rα θ expansion coefficient (β) and volume expansion coefficient
θ θ (γ) is
(c) π Rα (d) πRα
2 4 (a) 1:2:3 (b) 3:2:1 (c) 4:3:2 (d) All of these
6. A metre rod of silver at 0°C is heated to 100°C. Its length is 14. On heating a liquid having coefficient of volume expansion
increased by 0.19 cm. Coefficient of volume expansion of α in a container having coefficient of linear expansion α / 2,
the silver rod is the level of the liquid in the container would
(a) 5.7 × 10−5/°C (b) 0.63 × 10−5/°C (a) rise
(c)1.9 × 10−5/°C (d)16.1× 10−5/°C (b) fall
(c) remains almost stationary
7. A uniform metal rod is used as a bar pendulum. If the room
(d) Cannot be predicted
temperature rises by 10°C and the coefficient of linear
expansion of the metal of the rod is 15. A beaker is completely filled with water at 4°C. It will
2 × 10−6/°C, the period of the pendulum will have percentage overflow, if
increase of (a) heated above 4°C
−3 −3
(a) − 2 × 10 (b) − 1 × 10 (b) cooled below 4°C
(c) 2 × 10−3 (d) 1 × 10−3 (c) Both heated and cooled above and below 4°C, respectively
(d) None of the above
8. If two rods of lengths L and 2L having coefficients of linear
16. A glass flask of volume 200 cm 3 is just filled with mercury
expansion α and 2α respectively are connected end-to-end,
at 20°C. The amount of mercury that will overflow when
the average coefficient of linear expansion of the composite
the temperature of the system is raised to 100°C is
rod is equal to
(γ glass = 1.2 × 10 −5 ° /C andγmercury = 1.8 × 10 −4 / ° C)
3 5
(a) α (b) α
2 2 (a) 2.15 cm3 (b) 2.69 cm3 (c) 2.52 cm3 (d) 2.25 cm3
5
(c) α (d) None of these 17. Coefficient of volume expansion of mercury is
3 0.18 × 10−3/ °C. If the density of mercury at 0°C is 13.6 g/cc,
9. Two rods of different materials having coefficients of then its density at 200°C is
thermal expansions α 1 , α 2 and Young’s moduli Y1 , Y2 (a) 13.11 g/cc (b) 52.11 g/cc
respectively are fixed between two rigid massive walls. The (c) 16.11 g/cc (d) 26.11 g/cc
rods are heated such that they undergo the same increase
652 OBJECTIVE Physics Vol. 1

KINETIC THEORY OF IDEAL GASES


Kinetic theory of gases explains the behaviour of gases Due to the continuous movement, the gas molecules
based on the idea that the gas consists of rapidly moving collide with the walls of the container, thus applying
atoms of molecules. It correlates the macroscopic pressure on the walls.
properties of gases such as pressure, temperature, etc., to Z
the microscopic properties of gas molecules such as speed,
momentum, kinetic energy of molecules, etc.
v
Postulates of kinetic theory X
O
The kinetic theory of an ideal gas is based on the
following postulates l
Y
(i) A gas consists of large number of tiny particles, Fig. 14.7 A cubical box with sides of length l containing an
either atoms or molecules moving around randomly. ideal gas. The molecule shown moves with velocity v.
(ii) The volume of the particles themselves is negligible
as compared to the total volume of the gas. Most of The pressure exerted by gas molecules on the wall of
the volume of a gas is empty space. container is given by
(iii) The gas particles act independently of one another; 1 mN 2 1 2
there are no attractive or repulsive forces between p= v = ρv
3 V 3
particles.
Here, m is mass of one gas molecule, N is total number of
(iv) Collisions of the gas particles, either with other
gas molecules,V is volume of gas, v is root mean square
particles or with the walls of the container,
velocity of gas molecules and ρ is density of gas.
are elastic, i.e. the total kinetic energy of the
gas particles is constant at constant temperature. The above expression can be written as
Particles have no energy in the form of potential 1 N  21 N
p = m   v 2 =  mv 2 
energy because particles do not have mutual 3  V  
3 2 V
attraction or repulsion except gravitational attraction 1
which is very feeble considering their masses. Here, mv 2 is the kinetic energy of one gas molecule.
2
(v) The average kinetic energy of the gas particles is
proportional to the temperature of the sample 2N 2  N mv 2 
∴ p= (KE) or p =  
(in kelvin). 3V 32 V 
(vi) The pressure exerted by a gas is the result of collisions
of the particles with the walls of the container. 2  1 2
or p=  ρv 
(vii) At any particular time, different particles in a gas 3 2 
have different speeds and hence, different kinetic 1 2
energy. Here, ρv is average kinetic energy of gas molecules
2
However, average kinetic energy of the molecules
per unit volume.
remains constant at a particular temperature.
2
∴ p = × average kinetic energy per unit volume
Pressure of an ideal gas 3
(Concept of pressure) ∴ p= E
2
On the basis of kinetic theory of gases, we can do 3
mathematical calculations to find the expression for If E mol be the energy of one mole of gas, then
pressure exerted by a gas.
2 E mol
Consider an ideal gas consisting of N molecules each of p=
mass m in a container of volume V having edges of 3 V
length l. where,V is volume of one mole gas which is equal
to 22.4 L.
Thermometry, Thermal Expansion and Kinetic Theory of Gases 653

2 Pressure, p1 = 1.5 × 105 Pa, p 2 = 105 Pa


Now, pV = E mol = RT
3 Temperature, T1 = 400 K, T2 = 300 K
−1 −1
where, gas constant, R = 8.314 J mol K . 1 = n1RT1
From ideal gas equation, pV
pV
3 ⇒ n1 = 1
⇒ E mol = RT RT1
2
p 2V
⇒ E mol ∝T Similarly, p 2V = n 2RT2 ⇒ n 2 =
RT2
Example 14.23 If the mass of each molecule of a gas is ∴ Number of moles leaked,
halved and speed is doubled. Find the ratio of initial and
p p V
final pressure. ∆n = n1 − n 2 =  1 − 2 
Sol. Given, m1 = m , v1 = v,V1 = V , m 2 = m /2, v 2 = 2 v, V2 = V1 = V  T1 T2  R

∴ Pressure exerted by first gas molecule,  1.5 1  105 × 10−3


= −  ×
1 m1 2  400 300 8.3
p1 = v1
3 V1 = 5.02 × 10−3
and pressure exerted by second gas molecule, Mass of gas leaked = ∆n M 0 = 5.02 × 10−3 × 32 = 0.16 g
1 m2 2
p2 = v2
3 V2 Example 14.25 Find the coefficient of volume expansion for
an ideal gas at constant pressure and at temperature T.
p1 m1 V2 v12
∴ = × × Sol. For an ideal gas, pV = nRT
p 2 m 2 V1 v 22
As p is constant, we have
Putting all the values in above equation, we get
2 p ⋅ dV = nRdT
p1 m V v 1 1
= × ×   =2× = ∴
dV nR
=
p 2 m /2 V  2v  4 2 dT p
According to volume expansion, dV = VγdT
Ideal gas equation or γ= ⋅
1 dV nR
= =
nR
=
1
Pressure of an ideal gas is given by V dT pV nRT T
1 mN 2 2N 1 2 1
p= v or pV =  mv  ...(i) ∴ γ=
3 V 3 2  T
For ideal gas, kinetic energy of gas molecules is
proportional to temperature of the gas. Molecular velocities
1 In gaseous state, molecules tend to move in different
∴ mv 2 ∝ T or v 2 ∝ T ⇒ v 2 = kBT directions with different speeds.
2
where, kB is a constant. Based upon the calculations, there are three types of
molecular velocities
2 1  R
From Eq. (i), pV = N  mkBT  = RT = N T
3 2  N 1. Average velocity (v or v av )
We can write the above equation as It is the arithmetic mean of velocities of the different
R molecules of a gas.
pV = NkBT where, kB = v + v 2 + v 3 + … + vn
N Mathematically, v = 1
n
where, kB is called Boltzmann’s constant. where, v 1, v 2, v 3, …, v n are the velocities of the individual
molecules and n = n 1 + n 2 + n 3K is the total number of
Example 14.24 Oxygen is filled in a closed metal jar of
molecules.
volume 10 −3 m 3 at a pressure of 1.5 × 10 5 Pa and temperature
400 K. The jar has a small leak in it. The atmospheric pressure This is related to temperature T of the gas as
is 10 5 Pa and the atmospheric temperature is 300 K. Find the
8RT 8p 8kBT
mass of the gas that leaks out by the time, the pressure and the v = = =
temperature inside the jar equalise with the surrounding. πM πρ πM
Sol. Given, volume,V = 10−3 m3
where, M is molar mass in kg/mol and ρ is density.
654 OBJECTIVE Physics Vol. 1

Example 14.26 Find the average speed of nitrogen Example 14.29 Determine the rms velocity of oxygen
molecules at 27° C. molecules at STP. The molecular weight of oxygen is 32.
Sol. Mass of 1 mole of nitrogen gas, M = 14 g = 14 × 10−3 kg Sol. At STP, pressure, p = 1 atm = 1.013 × 105 Nm −2
8RT Mass, M = 32 g = 32 × 10−3 kg
Average speed, v =
πM Molar volume,V = 22.4 L
where, T = temperature of the gas (in kelvin) = 22.4 × 10−3 m 3
= 273 + 27° = 300 K Root-mean-square-velocity of oxygen molecules at STP,
R = gas constant = 8 .314 J mol −1 K −1 3p 3pV
v rms = =
8 × 8 .314 × 300 ρ M
∴ v = = 673 . 72 ms−1
3 .14 × 14 × 10−3 3 × 1.013 × 105 × 22.4 × 10−3
=
32 × 10−3
2. Root-mean-square velocity (u or v rms )
. ms −1
v rms = 46123
It is the square root of the mean of the squares of the
velocities of the molecules of gas. Example 14.30 The molecules of a given mass of a gas have
v 12 + v 22 + v 32 +… + v n2 root-mean-square-speed of 100 ms −1 at 27°C and 1.00 atm
Mathematically, v rms = pressure. What will be the root-mean-square-speed of the
n molecules of the gas at 127°C and 2.0 atm pressure?
From kinetic gas equation, Sol. Given, at 27°C, root-mean-square-speed,
(v rms )1 = 100 ms−1
3RT 3 pV 3p 3p 3p1 3pV
v rms = = = = (v rms )1 = = 11
…(i)
M M M /V ρ ρ1 M
where, p = pressure (RT /V ) and ρ = density (M /V ). According to ideal gas equation,
pV
11 pV
= 2 2
T1 T2
Example 14.27 In a container, two gases neon and argon are
V1 p 2T1 2 × 300 3
filled. Find the ratio of the root-mean-square speed of the ⇒ = = =
molecules of the two gases. Given, M Ne = 20.2 µ and V2 p1T2 400 2
M Ar = 39.9 µ. The temperature of the system is 30°C. ∴ At 127°C, root mean square speed,
Sol. From kinetic gas equation, 3p 2 3p 2V2
(v rms )2 = = …(ii)
3RT ρ2 M
v rms =
M From Eqs. (i) and (ii), we get
1 V p
∴ 2
v rms ∝ (v rms )22 = (v rms )12 × 2 × 2
M V1 p1
(v rms )Ne (M )Ar 39.9 2
∴ = = ~1.41 = (100)2 × ×2
(v rms )Ar (M ) Ne 20.2 3
200
or (v rms )2 = ms −1
Example 14.28 What is rms velocity of O 2 gas at 127° C? 3
The molecular weight of oxygen is 32.

Sol. As, v rms =


3RT
…(i)
3. Most probable velocity (v mp or ump )
M It is the velocity possessed by the largest number of
−1 −1 molecules of gas (Maxwell speed distribution law).
Given, R = 8.3143 J mol K ,
T = 127° C = (273 + 127) K = 400 K From kinetic gas equation,
M = 32 g mol−1 = 32 × 10−3 kg mol−1 2RT 2pV 2p
v mp = = =
Putting these values in Eq. (i), we get M M ρ
3 × 8.3143 × 400
v rms =
32 × 10−3 Note The ratio in different molecular velocities is
−1 8
= 5.5838 × 10 ms 2
v rms : v av : v mp = 3 : : 2 = 3 : 2.5 : 2
π
Thermometry, Thermal Expansion and Kinetic Theory of Gases 655

Example 14.31 Calculate the root mean square, average and E 300
⇒ =
most probable speeds of oxygen molecules at 27° C. (E / 2) T2
Sol. Given, molar mass of oxygen, ⇒ T2 = 150 K
M = 32 g mol−1 = 0.032 kg mol−1 = 150 − 273 = − 123° C
Temperature, T = 27° C = (27 + 273) K = 300 K
Example 14.33 Prove that the pressure of an ideal gas is
3RT 3 (8.314)(300)
(i) Root mean square speed, v rms = = numerically equal to two-third of the mean translational kinetic
M (0.032) energy per unit volume of the gas.
= 483.6 ms−1 Sol. Translational kinetic energy per unit volume,
8RT 8(8.314)(300) 1
(ii) Average speed, v = = = 445.6 ms−1 E = (Mass per unit volume)(v 2 )
πM (3.14)(0.032) 2
1  3p  3
(iii) Most probable speed, = (ρ)   = p
2  ρ 2
2RT 2(8.314)(300)
v mp = = = 394.8 ms−1 2
M (0.032) or p= E
3
Kinetic interpretation of temperature Example 14.34 A balloon has 5.0 moles of helium at 7°C.
2 Calculate
According to kinetic theory of gases, pV = E, (i) the number of atoms of helium in the balloon.
3
(ii) the total internal energy of the system.
but according to equation of state for an ideal gas,
(Avogadro’s number, N A = 6.02 × 1023
pV = RT
and Boltzmann’s constant, kB = 1.38 × 10−23 Jmol −1K −1).
2
⇒ E = RT = NkBT Sol Given, n = 5.0, T = 7° C = 7 + 273 = 280 K
3
(i) Number of atoms = nN A = 5.0 × 6.02 × 1023
3
∴ Total internal energy, E = NkBT ≈ 30 × 1023
2
3
where, k B is the Boltzmann’s constant. Its value is (ii) Average kinetic energy per molecule = kBT
1.38 × 10 –23 J mol–1K −1. 2
3
3 ∴ Total internal energy = kBT × N
Mean translational kinetic energy, E = kBT 2
2 3
= . × 10−23 × 280 × 30 × 1023
× 138
i.e. The mean translational kinetic energy of a gas 2
molecule depends only on its temperature and is 3
= × 30 × 280 × 1.38 = 1.74 × 104 J
independent of its nature or mass. 2
On this basis, we can say that absolute zero temperature
Example 14.35 A tank used for filling helium balloons has a
is the temperature at which translational kinetic volume of 3.0 m 3 and contains 2.0 moles of helium gas at
energy of a gas molecule becomes zero, i. e. at which 20.0°C. Assuming that the helium behaves like an ideal gas,
the molecular motion ceases altogether. (i) what is the total translational kinetic energy of the molecules
of the gas?
Example 14.32 At what temperature, the kinetic energy of a
(ii) what is the average kinetic energy per molecule?
gas molecule is half of the value at 27°C?
3
Sol. Given, T1 = 27° C = 27 + 273 = 300 K Sol. (i) Using (KE)trans = nRT
2
Kinetic energy of a gas molecule, Given, n = 2.0 mol and T = 293 K
3 3
E = kBT ∴ (KE)trans = (2.0) (8.31) (293) = 7.3 × 103 J
2 2
where, k is Boltzmann’s constant. (ii) The average kinetic energy per molecule,
∴ E ∝T 3
(KE)av = kBT = (138
3
. × 10–23) (293)
E1 T1 2 2
⇒ =
E 2 T2 = 6.07 × 10–21 J
656 OBJECTIVE Physics Vol. 1

Example 14.36 Calculate the ratio of the mean free path of


Mean free path (λ) the molecules of two gases, if the ratio of the numbers density
Every gas consists of a large number of molecules per cm 3 of the gases is 5 : 3 and the ratio of the diameters of
undergoing frequent collisions, therefore they cannot move the molecules of the gases is 4 : 5.
straight or unhindered, these molecules are in a state of Sol. Given, ratio of the numbers density,
continuous and random motion. Also, they undergo n1 5
perfectly elastic collision against each other. =
n2 3
The average distance travelled by a molecule between Ratio of the diameters,
two successive collisions is known as the mean free d1 4
=
path of the molecule. d2 5
Mean free path of the molecules of first gas,
1
Mean free path, λ = …(i) λ1 =
1
2 nπ d 2 2πn1d12
where, d = diameter of each molecule Similarly, mean free path of the second gas,
and n = number of molecules per unit volume. λ2 =
1
2πn 2d 22
Relation between mean free path (λ), temperature
(T ) and pressure (p) ∴ Ratio of λ1 and λ 2,
λ1 n 2d 22 3 5 5 15
For the derivation of Eq. (i), we have assumed the other = = × × =
molecules to be at rest. Taking into consideration the λ 2 n1d12 5 4 4 16
motion of all the gas molecules, the mean free path comes ⇒ λ1 : λ 2 = 15 : 16
out to be
1 Example 14.37 For a molecule of an ideal gas,
λ= n = 3 × 10 8 cm −3 and mean free path is 10 −2 cm.
2 nπ d 2 Calculate the diameter of the molecule.
If m is the mass of each gas molecule, then the density of Sol. Given, n = 3 × 108 cm−3, λ = 10−2 cm
the gas is
ρ = mn Mean free path is given by
ρ
or n = λ=
1
m 2πnd 2
m
∴ λ= ⇒ d2 =
1
2πd 2ρ 2 π nλ
For one mole of a gas, pV = RT 1
⇒ d2 =
RT N R 2 × 3.14 × 3 × 108 × 10−2
or p= = × × T = nkB T
V V N 10−6
=
1.414 × 3.14 × 3
p
or n= ⇒ d = 7.5 × 10−8 = 7.5 × 10−4 cm
kBT
⇒ d = 2.7 × 10−4 cm
kBT
⇒ λ= Hence, the diameter of the molecule of the gas
2πd 2 p is 2.7 × 10−4 cm.
CHECK POINT 14.3
1. Which one of the following is not an assumption in the 11. The molecules of a given mass of gas have rms speed
kinetic theory of gases? 200 ms −1 at 27°C and105 Nm−2 pressure. When the absolute
(a) The volume occupied by the molecules of the gas is temperature is doubled and the pressure is halved, the rms
negligible speed of the molecules of the same gas is
(b) The force of attraction between the molecules is negligible (a) 200 ms−1 (b) 400 ms−1
(c) The collisions between molecules are elastic −1
(d) All molecules have same speed (c) 200 2 ms (d) 400 2 ms−1

2. A closed vessel is maintained at a constant temperature. It 12. The velocities of ten particles (in ms −1) are 0, 2, 3, 4, 4, 4, 5,
is first evacuated and then vapour is injected into it 5, 6, 9. The most probable speed is
continuously. The pressure of the vapour in the vessel (a) 2 ms−1 (b) 4ms−1
(a) increases continuously (c) 5ms−1 (d) 9ms−1
(b) first increases and then becomes constant 13. If at the same temperature and pressure, the densities of
(c) first increases and then decreases two diatomic gases are d1 and d 2, respectively. The ratio of
(d) None of the above mean kinetic energy per molecule of gases will be
3. A vessel contains 1 mole of O 2 gas (molar mass 32) at a (a) 1 : 1 (b) d1 : d2
temperature T . The pressure of the gas is p. An identical (c) d1 : d2 (d) d2 : d1
vessel containing one mole of He gas (molar mass 4) at
temperature 2T has a pressure of 14. Two vessels A and B having equal volume contain equal
(a) p/ 8 (b) p (c) 2p (d) 8p masses of hydrogen in A and helium in B at 300 K. Then,
mark the correct statement.
4. If Maxwell distribution is valid and v p denotes the most (a) The pressure exerted by hydrogen is half of that exerted by
probable speed, v av is the average speed and vrms is the helium.
root-mean-square speed, then (b) The pressure exerted by hydrogen is equal to that exerted
(a) vav < vp < vrms (b) vav < vrms < vp by helium.
(c) vp < vav < vrms (d) vp < vrms < vav (c) Average kinetic energy of the molecules of hydrogen is half
the average kinetic energy of the molecules of helium.
5. What will be the temperature when the rms velocity is (d) The pressure exerted by hydrogen is twice that exerted by
double of that at 300 K? helium.
(a) 300 K (b) 600 K (c) 900 K (d) 1200 K
15. A sealed container with negligible coefficient of volumetric
6. By what factor the rms velocity will change, if the expansion contains helium (a monoatomic gas). When it is
temperature is raised from 27°C to 327°C? heated from 300 K to 600 K, the average kinetic energy of
(a) 2 (b) 2 (c) 2 2 (d) 1 helium atoms is
7. The velocities of three molecules are 3v , 4 v and 5v, (a) halved (b) unchanged
(c) doubled (d) increased by factor 2
respectively. Their rms speed will be
50 5 7 5 16. The gases carbon monoxide (CO) and nitrogen at the same
(a) v (b) v (c) v (d) v
3 2 2 2 temperature have translational kinetic energies E1 and E2,
respectively. Then,
8. The temperature at which the root-mean-square speed of a (a) E1 = E 2 (b) E1 > E 2
gas will be half of its value at 0°C is (assume the pressure (c) E1 < E 2 (d) None of these
remains constant)
(a) − 86.4° C (b) − 204.75° C 17. Pressure of an ideal gas is increased by keeping temperature
(c) − 104.75° C (d) − 68.25° C constant. What is the effect on kinetic energy of molecules?
(a) Increases (b) Decreases
9. Four molecules of a gas have speeds (in kms −1 ) 1, 2, 3 and (c) No change (d) Cannot be determined
4. The value of the root-mean-square speed of the gas
molecules is 18. Some gas at 300 K is enclosed in a container. Now, the
1 1 container is placed on a fast moving train. While the train
(a) 15 kms−1 (b) 10 kms−1 is in motion, the temperature of the gas
2 2
. kms−1
(c) 25 (d) 15/ 2 kms−1 (a) rises above 300 K (b) falls below 300 K
(c) remains unchanged (d) becomes unsteady
10. The root-mean-square (rms) speed of oxygen molecules (O 2) 19. The mean free path for air molecule with average speed
at a certain absolute temperature is v. If the temperature is 18.5 ms −1 at STP is (d = 2 × 10−10 m and n = 2.7 × 10 25 m−3)
doubled and the oxygen gas dissociates into atomic oxygen,
the rms speed would be (a) 3.5 × 10−7 m (b) 4 × 10−7 m
(a) v (b) 2v (c) 2 v (d) 2 2 v (c) 2.08 × 10−7 m (d) 5 × 10−7 m
658 OBJECTIVE Physics Vol. 1

IDEAL GAS LAWS Then, V2 = 3V1


Ideal gases irrespective of their nature obey the following Change in volume, ∆V = V2 − V1 = 2V1
laws ∆V 2V
Percentage change = × 100 = 1 × 100 = 200%
V V1
Boyle’s law
According to this law, for a given mass of a gas, the Charles’ law
volume of a gas at constant temperature (called isothermal According to this law, for a given mass of a gas, the
process) is inversely proportional to its pressure, i.e. volume of a gas at constant pressure (called isobaric
process) is directly proportional to its absolute
1
V ∝ (T = constant) temperature, i.e.
p V ∝T (p = constant)
V
or pV = constant or p iVi = p fVf or = constant
T
Thus, p-V graph in an isothermal process is a rectangular
Vi Vf
hyperbola or pV versus p or V graph is a straight line or =
parallel to p or V-axis. Ti Tf
p pV
Thus, V-T graph in an isobaric process is a straight line
T = constant
passing through origin orV /T versus V or T graph is a
T = constant straight line parallel to V or T axis.
V V
T
p = constant p = constant
V p or V

Fig. 14.8 p versus V graph and pV versus p or V graph

T (in K) T or V
Example 14.38 A gas at 27°C in a cylinder has a volume of
−2
4 L and pressure 100 Nm . If the gas is first compressed at Fig. 14.9 V versus T graph and V/T versus T or V graph
constant temperature, so that the pressure is 150 Nm −2 .
Estimate the change in volume. If volume of ideal gas at 0°C isV0 and at t° C isVt , then
Sol. Given,V1 = 4 L,V2 = ?, p1 = 100 Nm , −2 Vt = V0 (1 + α t )
p 2 = 150 Nm −2, ∆V = ? where, α is volume coefficient of the gas. For all gases, its
1
Using Boyle’s law for constant temperature, we have value is nearly /° C.
273
1 1 = p 2V2
pV
 t 
pV 100 × 4 ∴ Vt = V0 1 + 
⇒ V2 = 11
= = 2.667 L  273 
p2 150
∴ Change in volume, ∆V = V1 − V2 Example 14.40 Air is filled in a container at 333 K.
= 4 −2.667
Calculate the temperature upto which it should be heated, so
that one-third of air may escape out of the vessel.
= 1.33 L
V
Sol Given, T1 = 333 K, T2 = ?,V1 = V, V2 = V +
Example 14.39 The pressure of a given mass of a gas filled in 3
a vessel of volumeV at constant temperature is reduced to (Q According to question, one-third of air may escape out the vessel)
one-third of its initial value. Calculate the percentage 4
change in its volume. = V
3
1
Sol. According to the question, p 2 = p1 According to Charles’ law, we get
3 V2 T2
=
1 1 = p 2V2
Applying Boyle’s law, pV V1 T1
1
∴ 11=
pV pV
1 2 ⇒
V 4
T2 = T1 × 2 = 333 × = 444 K
3 V1 3
Thermometry, Thermal Expansion and Kinetic Theory of Gases 659

100 T
Gay Lussac’s law or pressure law ⇒ =
101 T + 1
According to this law, for a given mass of a gas, the
⇒ 100T + 100 = 101 T
pressure of a gas at constant volume (called isochoric
process) is directly proportional to its absolute ⇒ T = 100 K
temperature.
i.e. p ∝T (V = constant) Dalton’s law of partial pressure
It states that, the total pressure of a mixture of
p
or = constant non-interacting ideal gases is the sum of partial pressures
T exerted by individual gases in the mixture.
pi pf i.e. p = p1 + p 2 + p 3 + L .
or = Consider a mixture of non-interacting ideal gases. Let gas
Ti Tf
1 has µ 1 moles, gas 2 has µ 2 moles and so on.
Thus, p-T graph in an isochoric process is a straight line Suppose the net volume of the mixture isV, temperature
passing through origin or p /T versus p or T graph is a of mixture is T and pressure of mixture is p.
straight line parallel to p or T-axis. Equation of state for the mixture,
p p pV = ( µ 1 + µ 2 + L ) RT
T
µ 1RT µ 2RT
V = constant p= + +L
V V
V = constant
Dalton’s law, p = p 1 + p 2 + p 3 + L

T (in K) p or T µ 1RT
Here, p 1 = is the pressure of gas 1 exerted at the
Fig. 14.10 p versus T graph and p/T versus p or T graph V
same condition of volume and temperature, if no other gases
are present.
If pressure of an ideal gas at 0°C is p 0 and at t° C is p t ,
then p t = p 0 (1 + βt ) Example 14.42 One mole each of hydrogen, oxygen and
where, β is pressure coefficient of gas. For all gases, its nitrogen are mixed in a close container of volume 10 L and
1 temperature 27°C. Calculate the pressure exerted by the
volume is nearly /° C. mixture. (R = 8.314 J mol −1 K −1)
273
Sol. Given, R = 8.314 J mol−1 K −1
 t  Temperature, T = 27° C = 27 + 273 = 300 K
∴ p t = p 0 1 + 
 273  Volume,V = 10 L = 10 × 10−3 m3
According to Dalton’s law of partial pressure,
Example 14.41 The pressure of a gas filled in a closed
p = p1 + p 2 + p 3
container increases by 1% when temperature is increased by
1°C. Find the initial temperature of the gas. µRT µRT µRT 3µRT
⇒ p= + + =
Sol. Given, p1 = p, T1 = T K V V V V
Here, µ = 1 mol
1 101 3RT 3 × 8.314 × 300
Increased pressure, p 2 = p + p= p ∴ p= =
100 100 V 10 × 10−3
Increased temperature, T2 = (T + 1) K = 7.48 × 105 Nm −2
According to the Gay Lussac’s law, we get
p1 T1 Example 14.43 4 g hydrogen is mixed with 11.2 L of
= helium at STP in a container of volume 20 L. If the final
p 2 T2
temperature is 300 K, find the pressure.
p T
⇒ = Sol. Q 4 g hydrogen = 2 moles hydrogen.
101
p (T + 1) 1
100 ∴ 11.2 L helium at STP = mole of helium
2
660 OBJECTIVE Physics Vol. 1

According to Dalton’s law of partial pressure, At low temperature and high pressure, gas molecules
RT are close to each other, so intermolecular attraction
p = p H + p He = (n H + n He )
V cannot be ignored between them.
 1 8.31 × (300 K ) (ii) For ideal gas, volume of gas molecules is negligible as
= 2 + 
 2 (20 × 10−3 ) m3 compared to volume of gas.
= 3.12 × 105 N/m 2 But at high pressure and low temperature, total
volume of gas decreases (total volume changes but
volume of molecules remains same), so volume of gas
Graham’s law of diffusion molecules is not negligible as compared to volume of
It states that the rate of diffusion of a gas is inversely gas.
1
proportional to the square root of its density, i.e. r ∝
ρ van der Waals’ equation
where, r = rate of diffusion and ρ = density of the gas. Based on the deviation in behaviour of real gas from ideal
Let r1 and r 2 be the rates of diffusion of gases 1 and 2 gas, van der Waals’ modified ideal gas equation by taking
respectively, then according to Graham’s law, into consideration the intermolecular atomic forces and the
volume occupied by the gas molecules and introduced a
r1 ρ2 new equation known as van der Waals’ equation which
=
r2 ρ1 explains behaviour of real gas correctly.
van der Waals’ equation is
where, ρ1 is density of gas 1 and ρ 2 is density of gas 2.
r1 ρ2 M2  an 2 
= =  p + 2  (V − nb ) = nRT
r2 ρ1 M1  V 
IfV is the volume of gas diffused in t seconds, then Here, a and b are called van der Waals’ constants. In this
V r V t equation, p, V, T and n represent the measured values of
r = ⇒ 1 = 1 × 2 pressure, volume, temperature (expressed on the absolute
t r 2 V2 t1 scale) and number of moles respectively, just as in the ideal
gas equation.
an 2
Behaviour of real gases Here, the term
V2
is called pressure correction for
If a gas obeys the gas laws at all conditions, it is called an molecular attraction and term nb is called volume correction
ideal gas and if it does not, it is called the real gas. for finite size of molecules.
(i) All the existing gases are real and the ideal gases
are hypothetical. Significance of van der Waals’ constants
(ii) Real gases obey the gas laws under the moderate The quantities a and b are experimentally derived constants
conditions of temperature and pressure. At low that differ for different gases. When a and b are both zero,
temperature and high pressure, the real gases the van der Waals’ equation reduces to the ideal gas
deviate from the ideal gas behaviour significantly equation (pV = nRT ).
(i.e. start disobeying gas laws). (i) Larger molecules have greater value of b. For
(iii) Real gases behave ideally at high temperature and instance, H 2 (a diatomic molecule) has greater value
low pressure. of b than that of He (a monoatomic smaller molecule).
n 2a
Reasons for deviation of real gas from (ii) 2 affects internal pressure of the gas and a is
ideal gas behaviour V
a measure of force of attraction between gas
Real gases deviate from ideal gas behaviour because some molecules.
of the assumptions of kinetic theory of gases do not hold (iii) Greater the value of a, greater the liquefaction of the
good for real gases. gas. That is why, H 2 and He having smaller value of
(i) For ideal gas, it has been considered that there is a are difficult to liquify.
no force of attraction between gas molecules. But (iv) b is also called co-volume or excluded volume, i.e.
this assumption is not valid for all pressures and b = 4VN 0
temperatures.
Thermometry, Thermal Expansion and Kinetic Theory of Gases 661

where, V is the volume of a spherical molecule of Example 14.46 The p-V diagram of two different masses m1
radius r,V = (4/3 ) πr 3 . and m 2 are drawn (as shown) at constant temperature T.
State whether m 1 > m 2 or m 2 > m 1 ?
(v) a is expressed in atm L2 mol−2 and b is expressed in
p
L mol −1 .
T T

Example 14.44 If pressure of CO 2 (real gas) in a container


RT a
is given by p = − 2 , then compute the mass of the
2V − b 4b m2
gas in container. m1
V
Sol. van der Waals’ gas equation for n mole of real gas is
 n 2a  Sol. The ideal gas equation is
p + 2  V ( − nb ) = nRT
m
 V  pV = nRT = RT
M
n 2a nRT
p+ = M
V 2
V − nb ∴ m = (pV )  
 RT 
nRT n 2a m ∝ pV , if T = constant
∴ p= − 2 or
V − nb V
From the graph, we can see that p 2V2 > pV
1 1 (for same p or V ).
 RT a 
Given equation, p= −  Therefore, m 2 > m1.
 2V − b 4b 2 
On comparing the given equation with the standard equation, Example 14.47 The p-T graph for the given mass of an
ideal gas is shown in figure. What inference can be drawn
we get n = 1/2
regarding the change in volume (whether it is constant,
m
As, n= increasing or decreasing)?
M p
1
⇒ Mass of gas, m = nM = × 44 (Q Molar mass of CO 2 = 44 g) B
2
= 22 g
A
Example 14.45 The p-V diagrams of same mass of a gas
are drawn at two different temperatures T1 and T 2 . Explain
whether T1 > T 2 or T 2 > T1. T (K)

p
Sol. From the given graph, we can write the p-T equation as
p = aT + b (Q y = mx + c )
Here, a and b are positive constants. Further,
p b
T2 =a +
T1 T T
V
Now, TB > TA
b b
Sol. The ideal gas equation is ∴ <
TB TA
pV = nRT
pV  p  p
or T = or   < 
nR T B T  A

T ∝ pV , if number of moles of the gas are kept constant. Here, T  T 


or   >   or VB > VA
mass of the gas is constant, which implies that number of  p B  p  A
moles are constant, i. e .T ∝ pV .
Thus, as we move from A to B, volume of the gas is
In the given diagram, product of p and V for T2 is more than T1 increasing.
at all points (keeping either p or V same for both graphs).
Hence, T2 > T1.
662 OBJECTIVE Physics Vol. 1

DEGREE OF FREEDOM Maxwell’s law of equipartition


The number of independent ways in which a molecule or of energy
an atom can exhibit motion or have energy is called its The total kinetic energy of a gas molecule is equally
degree of freedom. A rigid body of finite size (gas distributed among its all degrees of freedom and the
molecule) can have both translational and rotational energy associated with each degree of freedom at absolute
motions. Like translational motion, rotational motion can 1
be resolved into mutually perpendicular components. temperature T is kBT , where kB is Boltzmann’s constant.
2
Hence, a rigid body has 6 degrees of freedom; three for For a gas in thermal equilibrium at temperature T, the
translational motion and three for rotational motion. Out of average value of translational energy of the molecule is
three possible rotational degrees, only two are applicable
for linear molecules. For non-linear molecules, third 1 2 1 1
Et = mv x + mv y2 + mv z2
rotational degree is allowed. Apart from 3 rotational and 3 2 2 2
translational, there are 3 N – 6 degree of freedom which Translational energy of the molecules, (for monoatomic
are vibrational. gas)
A very important relation regarding degree of freedom is 3
2 Et = k t
γ = 1 + ; γ = C p /C V 2 B
f
It is same as the case with an ideal gas. It distributes its
where, f = degree of freedom
internal energy equally in all degrees of freedom. In each
Degrees of freedom for different gases according to degree of freedom, energy of one mole of an ideal gas is
atomicity of gas at low temperature is given below 1
RT, where T is the absolute temperature of the gas.
Atomicity 2
Translational Rotational Total
of gas Thus, if f be the number of degrees of freedom, the
Monoatomic, 3 0 3 y f
internal energy of 1 mole of the gas will be RT or
e.g. Ar, Ne, 2
ideal gas, etc. n
x internal energy of n moles of the gas will be f RT . Thus,
2
z n
Internal energy, U = fRT
Diatomic, e.g. 3 2 5 2
O2, Cl2, N 2, etc.
Equipartition of energy of gases is given below
Translational Internal
Types of gases f energy of energy of
one molecule one mole
Triatomic 3 2 5 3 3
(linear), e.g. Monoatomic gas 3 k BT RT
2 2
CO2, C2 H2, etc.
O
C
O Diatomic gas
5 5 5
(a) excluding vibration k BT RT
2 2
Triatomic 3 3 6 7 7
(Non-linear) or (b) including vibration 7 k BT RT
2 2
Polyatomic, e.g.
6 6
H2O, NH3, CH4 , Polyatomic gas like CO2, H2O 6 k B T = 3k B T RT = 3RT
etc. 2 2

Different energies of a system of degree of freedom f are


Note Degree of freedom for different gases can also be given by the as follows
formula, f = 3 NK
where, N = number of particles in the system f
(i) Total energy associated with each molecule = kBT
and K = number of independent relations between the 2
particles.
Thermometry, Thermal Expansion and Kinetic Theory of Gases 663

f Example 14.52 Given, Avogadro's number, N = 6.02 × 10 23


(ii) Total energy associated with N-molecule = NkBT and Boltzmann’s constant, kB = 1.38 × 10 −23 J/K. Calculate
2
f (i) the average kinetic energy of translation of the molecules of
(iii) Total energy associated with 1 mole = RT an ideal gas at 0°C and at 100°C
2
(ii) and the corresponding energies per mole of the gas.
f
(iv) Total energy associated with µ moles = µRT Sol. (i) According to the kinetic theory, the average kinetic
2 energy of translation per molecule of an ideal
f  3
(v) Total energy associated with each gram = rT monoatomic gas at temperature T (in kelvin) is   kBT ,
2  2
f where kB is Boltzmann’s constant.
(vi) Total energy associated with m gram = mrT
2 At 0° C (T = 273 K ),
3
Example 14.48 Calculate the ratio of kinetic energy of the kinetic energy of translation = kBT
2
molecule of oxygen and neon gas at 27°C. 3 −23
= × (138 . × 10 ) × 273
Sol. For oxygen gas, number of degrees of freedom is 5. 2
5
∴ (KE)oxygen = kBT = 5.65 × 10−21 J/molecule
2
At 100° C (T = 373 K ), the energy is
Neon is monoatomic, therefore it has 3 degrees of freedom.
3
3 = × (1.38 × 10−23 ) × 373 = 7.72 × 10−21 J/molecule
∴ (KE)neon = kBT 2
2
(ii) 1 mole of gas contains N (= 6.02 × 1023 ) mol
Hence, the ratio of their kinetic energy
(KE)oxygen (5 /2) kBT 5 Therefore, at 0° C, the kinetic energy of translation of
= = = 1 mole of the gas,
(KE)neon (3 /2) kBT 3
= (5.65 × 10−21) (6.02 × 1023 ) ≈ 3401 J/mol
Example 14.49 Calculate the average kinetic energy of At 100°C, the kinetic energy of translation of 1 mol of
oxygen molecule at 0°C. (R = 8.314 J mol −1 K −1, gas
N A = 6.023 × 10 23 ) = ( 7.72 × 10−21) (6.02 × 1023 ) ≈ 4647 J/mol

Sol. Given, T = 0° C = 273 K Example 14.53 One mole of an ideal monoatomic gas is taken
Oxygen is diatomic molecule, therefore it has 5 degrees of at a temperature of 300 K. Its volume is doubled keeping its
freedom, i.e. 3 translational and 2 rotational. pressure constant. Find the change in internal energy.
5 RT 5 8.314 Vi Vf
∴ KE = = × × 273 = 9.4 × 10−21 J Sol. Since, pressure is constant,V ∝ T ⇒ =
2 N A 2 6.023 × 1023 Ti Tf
Vf
Example 14.50 Find the average kinetic energy per molecule or Tf = Ti ⇒ Tf = 2 Ti = 600 K
Vi
at temperature T for an equimolar mixture of two ideal
gases A and B, where A is monoatomic and B is diatomic. f 3
∴ ∆U = n ⋅ R∆T = R (600 − 300) = 450 R
Sol. Number of degrees of freedom per molecule for A = 3 2 2

Number of degrees of freedom per molecule for B = 5 Critical temperature, pressure and volume
Since, the mixture is equimolar, the average kinetic energy
Gases cannot be liquified above a temperature called
per molecule will be the simple average of the two values, i. e .
 3 + 5
critical temperature (TC ), howsoever large the pressure
  k T = 4kBT , where kB is Boltzmann’s constant. may be.
 2  B
The pressure required to liquify the gas at critical
Example 14.51 Calculate the change in internal energy of temperature is called critical pressure (p C ) and the volume
3.0 moles of helium gas when its temperature is increased by of the gas at critical temperature and pressure is called
2.0 K.
critical volume (VC ). Value of critical constants in terms of
Sol. Helium is a monoatomic gas. van der Waals’ constants a and b are as follows
3
Internal energy of n-moles of the gas, U = nRT a 8a
2 VC = 3b, p C = 2
and TC =
3 27b 27Rb
∴ ∆U = nR (∆T )
2 RTC 8
Further, = is called critical coefficient and is
 3 p CVC 3
Substituting the values, ∆U =   (3) (8.31) (2.0) = 74.8 J
 2 same for all gases.
OBJECTIVE Physics Vol. 1

CHECK POINT 14.4


1. If temperature of the gas remains constant, but pressure is 9. The average translational kinetic energy of O 2 (molar mass
decreased by 20%. The percentage change in volume 32) at a particular temperature is 0.048 eV. The average
(a) increases by 20% translational kinetic energy of N 2 (molar mass 28)
(b) decreases by 20% molecules (in eV) at the same temperature is
(c) increases by 25% (a) 0.0015 (b) 0.003
(d) decreases by 25% (c) 0.048 (d) 0.768
2. The adjoining figure shows graph of pressure and volume of 10. A gas has volume V and pressure p. The total translational
a gas at two temperatures T1 and T 2. Which of the following
kinetic energy of all the molecules of the gas is
is correct?
p 3
(a) pV only if the gas is monoatomic
2
3
(b) pV only if the gas is diatomic
2
T2 3
T1 (c) pV in all cases
2
V (d) None of the above
(a) T1 > T2
11. A vessel contains a mixture of one mole of oxygen and two
(b) T1 = T2 moles of nitrogen at 300 K. The ratio of the average
(c) T1 < T2 rotational kinetic energy per O 2 molecule to that per N 2
(d) Nothing can be said about temperatures molecule is
(a) 1 : 1 (b) 1 : 2
3. A perfect gas at 27°C is heated at constant pressure, so as to (c) 2 : 1 (d) 8 : 7
double its volume. The increase in temperature of the gas
will be 12. Two monoatomic gases are at absolute temperatures 300 K
(a) 600°C (b) 327°C and 350 K, respectively. Ratio of average kinetic energy of
(c) 54°C (d) 300°C their molecules is
(a) 7 : 6 (b) 6 : 7
4. One litre of an ideal gas at 27°C is heated at a constant (c) 36 : 49 (d) 49 : 36
pressure till it reaches 297°C. Then, the final volume is the
approximately 13. At 27°C temperature, the kinetic energy of an ideal gas is E1 .
(a) 1.2 L (b) 1.9 L If the temperature is increased to 327°C, then the kinetic
(c) 19 L (d) 2.4 L energy will be
E1 E1
5. In the adjacent V -T diagram, what is the relation between (a) (b) 2 E1 (c) 2E1 (d)
2 2
p1 and p 2?
14. The gas in a vessel is subjected to a pressure of 20 atm at a
V p2 temperature 27°C. The pressure of the gas in a vessel after
one-half of the gas is released from the vessel and the
p1
temperature of the remaining gas is raised by 50°C is
(a) 8.5 atm (b) 10.8 atm
(c) 11.7 atm (d) 17 atm

T
15. The graph which represents the variation of mean kinetic
energy of molecules with temperature t°C is
(a) p2 = p1 (b) p2 > p1
E E
(c) p2 < p1 (d) Cannot be predicted
6. 16 g of oxygen, 14 g of nitrogen and 11 g of carbon dioxide (a) (b)
are mixed in an enclosure of volume 5 L and temperature
27°C. The pressure exerted by the mixture is t ºC t ºC
(a) 4 × 105 Nm−2 (b) 5 × 105 Nm−2 E E
(c) 6 × 105 Nm −2 (d) 9 × 105 Nm−2 (c) (d)
7. What is the degree of freedom in case of a monoatomic gas?
(a) 1 (b) 3 t ºC t ºC
(c) 5 (d) None of these
16. A balloon is filled at 27 °C and 1 atm pressure by
8. The degrees of freedom of a molecule of a non-linear 500 m3 He. At −3°C and 0.5 atm pressures, the volume of
triatomic gas is (ignore vibrational motion) He-gas contained in balloon will be
(a) 2 (b) 4 (a) 700 m3 (b) 900 m3
(c) 6 (d) 8 (c)1000 m3 (d) 500 m3
Thermometry, Thermal Expansion and Kinetic Theory of Gases 665

17. The figure shows graphs of pressure versus density for an (M2 = 2 kgK −1 mol −1) at a pressure of 3 atm contained in the
ideal gas at two temperatures T1 and T 2, then same cylinder at same temperature is
(a) 1.08 kg (b) 0.86 kg
T1 (c) 0.68 kg (d) 1.68 kg
p
20. Two different isotherms representing the relationship
T2 between pressure p and volume V at a given temperature of
the same ideal gas are shown for masses m1 and m2, then
p
ρ

(a) T1 > T2 (b) T1 = T2


m2
(c) T1 < T2 (d) None of these m1
18. From the given p - T graph, what conclusion can be drawn?
V
p (a) m1 > m2
V2 (b) m1 = m2
(c) m1 < m2
V1 (d) Nothing can be predicted
θ2
θ1 21. A gas is found to obey the law p 2V = constant. The initial
T
temperature and volume are T 0 and V0 . If the gas expands
(a) V2 = V1 (b) V2 < V1 to a volume 3 V0 , its final temperature becomes
(c) V2 > V1 (d) None of these T0 T0
(a) (b)
3 3
19. A cylinder contains 20 kg of N 2 gas (M1 = 28 kgK −1 mol −1) at (c) 3T0 (d) None of these
a pressure of 5 atm. The mass of hydrogen
OBJECTIVE Physics Vol. 1

Chapter Exercises
(A) Taking it together
Associated questions of the chapter for advanced level practice

1 The absolute zero temperature in Fahrenheit scale is 9 A gas at the temperature 250 K is contained in a
(a) − 273°F (b) − 32°F closed vessel. If the gas is heated through 1 K, then
(c) − 460°F (d) − 132°F the percentage increase in its pressure will be
2 On which of the following scales of temperature, the (a) 0.4% (b) 0.2% (c) 0.1% (d) 0.8%
temperature is never negative? 10 An electron tube was sealed off during manufacture
(a) Celsius (b) Fahrenheit at a pressure of 1.2 × 10 −7 mm of mercury at 27°C.
(c) Reaumur (d) Kelvin
Its volume is 100 cm 3 . The number of molecules
3. Boyle’s law is applicable for an [NCERT Exemplar] that remain in the tube is
(a) adiabatic process (b) isothermal process (a) 2 × 1016 (b) 3 × 1015
(c) isobaric process (d) isochoric process
(c) 3.94 × 1011 (d) 5 × 1011
4. Heat is associated with [NCERT Exemplar]
(a) kinetic energy of random motion of molecules 11 At what temperature is the root mean square
(b) kinetic energy of orderly motion of molecules velocity of the gaseous hydrogen molecules equal to
(c) total kinetic energy of random and orderly motion of that of oxygen molecules at 47°C?
molecules (a) 20 K (b) 80 K (c) − 73 K (d) 3 K
(d) kinetic energy of random motion in some cases and
12 The root-mean-square velocity of the molecules in a
kinetic energy of orderly motion in other
sample of helium is (5/7)th that of the molecules in a
5 A centigrade and a Fahrenheit thermometers are sample of hydrogen. If the temperature of hydrogen
dipped in boiling water. The water temperature is sample is 0°C, then the temperature of the helium
lowered until the Fahrenheit thermometer registers sample is about
140°. What is the fall in temperature as registered (a) 5.57 °C (b) 0° C
by the centigrade thermometer? (c) 273°C (d) 100°C
(a) 30° (b) 40°
(c) 60° (d) 80° 13 At room temperature, the rms speed of the molecules
of certain diatomic gas is found to be 1930 ms −1.
6 A constant volume gas thermometer shows pressure
The gas is
reading of 50 cm and 90 cm of mercury at 0°C and
(a) H2 (b) F2
100°C, respectively. When the pressure reading is (c) O2 (d) Cl2
60 cm of mercury, the temperature is
(a) 25°C (b) 40°C 14 In a mercury thermometer, the ice point (lower fixed
(c) 15°C (d) 12.5°C point) is marked as 10° and the steam point (upper
3 fixed point) is marked as 130°. At 40°C temperature,
7 The volume of gas at 20°C is 100 cm at normal
what will this thermometer read?
pressure. If it is heated to 100°C, its volume
(a) 78° (b) 66°
becomes 125 cm 3 at the same pressure, then volume (c) 62° (d) 58°
coefficient of the gas at normal pressure is
15 A clock with a metal pendulum beating seconds
(a) 0.0015/°C (b) 0.0045/°C
(c) 0.0025/°C (d) 0.0033/°C keeps correct time at 0°C. If it loses 12.5 s a day at
25°C, the coefficient of linear expansion of metal
8 Coefficient of real expansion of mercury is pendulum is
0.18 × 10 −3 /°C. If the density of mercury at 0°C is 1 1
(a) /°C (b) /°C
13.6 g/cc, its density at 200°C is 86400 43200
(a) 13.11 g/cc (b) 26.22 g/cc 1 1
(c) /°C (d) /°C
(c) 52.11 g/cc (d) None of these 14400 28800
Thermometry, Thermal Expansion and Kinetic Theory of Gases 667

16 The graph AB shown in figure is a plot of (a) remains the same because 500 ms −1 is very much
temperature of a body in degree celsius and degree smaller than v rms of the gas
fahrenheit, then (b) remains the same because motion of the vessel as a
whole does not affect the relative motion of the gas
molecules and the walls
100°C B 2
(c) will increase by a factor equal to (v rms + (500)2 ) / v rms
2
Centigrade

where v rms was the original mean square velocity of the


gas.
(d) will be different on the top wall an bottom wall of the
32°F 212°F Fahrenheit vessel
A
22 The steam point and the ice point of a mercury
(a) slope of line AB is 9/5 thermometer are marked as 80° and 10°. At what
(b) slope of line AB is 5/9 temperature on centigrade scale will the reading of
(c) slope of line AB is 1/9 this thermometer be 59° ?
(d) slope of line AB is 3/9
(a) 70° C (b) 60° C
17 A chamber containing a gas was evacuated till the (c) 40° C (d) 50° C
vacuum attained was 10 −14 m of Hg. If the 23 Two uniform brass rods A and B of lengths l and 2l
temperature of the chamber was 30°C, the number and radii 2r and r respectively are heated to the
of molecules that remain in it per cubic metre is same temperature. The ratio of the increase in
(a) 3.2 × 1011 (b) 3.2 × 1012
volume of A to that of B is
(c) 2.3 × 1012 (d) 2.3 × 1010 (a) 1 : 1 (b) 1 : 2
18 A bimetallic strip is made of aluminium and steel (c) 2 : 1 (d) 1 : 4
(α Al > α steel ). On heating, the strip will 24 The expansion of an ideal gas of mass m at a
[NCERT Exemplar]
(a) remains straight constant pressure p is given by the straight line B.
(b) get twisted Then, the expansion of the same ideal gas of mass
(c) will bend with aluminium on concave side 2m at a pressure 2p is given by the straight line
(d) will bend with steel on concave side
Volume A
19 A uniform metallic rod rotates about its
perpendicular bisector with constant angular speed. B
If it is heated uniformly to raise its temperature C
slightly, [NCERT Exemplar]
(a) its speed of rotation increases
(b) its speed of rotation decreases Temperature
(c) its speed of rotation remains same
(d) its speed increases because its moment of inertia (a) C (b) A
increases (c) B (d) None of these
20 As the temperature is increased, the period of a 25 An ideal gas is initially at temperature T and volume
pendulum [NCERT Exemplar] V. Its volume is increased by ∆V due to an increase
(a) increases as its effective length increases even though in temperature ∆T, pressure remaining constant.
its centre of mass still remains at the centre of the bob The quantity δ = ∆V /(V ∆T ) varies with
(b) decreases as its effective length increases even though temperature as
its centre of mass still remains at the centre of the bob
δ δ
(c) increases as its effective length increases due to
shifting to centre of mass below the centre of the bob (a) (b)
(d) decreases as its effective length remains same but the
centre of mass shifts above the centre of the bob
T T
21 A cubic vessel (with face horizontal + vertical) δ δ
contains an ideal gas at NTP. The vessel is being
carried by a rocket which is moving at a speed of (c) (d)
500 ms −1 in vertical direction. The pressure of the
gas inside the vessel as observed by us on the ground T T
[NCERT Exemplar]
668 OBJECTIVE Physics Vol. 1

26 Variation of internal energy with density of one 30 A cylindrical steel plug is inserted into a circular
mole of monoatomic gas is depicted in the adjoining hole of diameter 2.6 cm in a brass plate. When the
figure. Corresponding variation of pressure with plug and the plates are at a temperature of 20°C, the
volume can be depicted as (Assuming the curve is diameter of the plug is 0.01 cm smaller than that of
rectangular hyperbola) the hole. The temperature at which the plug will just
U fit in it is
 11 × 10 −6 19 × 10 −6 
 Given, α steel = and α brass = 
 °C °C 
(a) − 48° C (b) − 20° C (c) − 10° C (d) − 458° C
ρ
31 The apparent coefficient of expansion of a liquid
p p
when heated in a copper vessel is C and when
heated in a silver vessel, it is S. If A is the linear
(a) (b)
coefficient of expansion of copper, then the linear
coefficient of expansion of silver is
V V C + S − 3A C − S + 3A
(a) (b)
p p 3 3
S + 3A − C C + S + 3A
(c) (d) (c) (d)
3 3
32 Two identical containers joined by a small pipe
V V initially contain the same gas at pressure p 0 and
27 Volume versus temperature graphs for a given mass absolute temperature T 0 . One container is now
of an ideal gas are shown in figure. At two different maintained at the same temperature while the other
values of constant pressure, what can be inferred is heated to 2T 0 . The common pressure of the gases
about relation between p 1 and p 2 ? [NCERT Exemplar] will be
V (L) 3 4 5
(a) p0 (b) p0 (c) p0 (d) 2 p 0
p2 2 3 3
40
33 The graph between two temperature scales A and B
30
p1 as shown in figure. Between upper fixed point and
20 lower fixed point, there are 150 equal divisions on
scale A and 100 on scale B. The relationship for
10
conversion between the two scales is given by
[NCERT Exemplar]
100 200 300 400 500 T (K)
(a) p1 > p 2 (b) p1 = p 2
180
(c) p1 < P2 (d) Data is insufficient
Temperature (°A)

28 The radius of a metal sphere at room temperature T ∆tA=150°


is R and the coefficient of linear expansion of the
metal is α. The sphere is heated a little by a
temperature ∆T, so that its new temperature is ∆tB = 100°
T + ∆T . The increase in the volume of the sphere is O Temperature (°B) 100
approximately, [NCERT Exemplar]
(a) 2πRα∆Τ (b) πR 2α∆T (c) 4πR 3α∆T /3 (d) 4πR 3α∆T t A − 180 t t A − 30 t
(a) = B (b) = B
100 150 150 100
29 An aluminium sphere is dipped into water. Which of tB − 180 t tB − 40 t
the following statement is true? [NCERT Exemplar] (c) = A (d) = A
150 100 100 180
(a) Buoyancy will be less in water at 0° C than that in water
at 4° C. 34 1 mole of H2 gas is contained in a box of volume
(b) Buoyancy will be more in water at 0° C than that in V = 100
. m3 at T = 300 K. The gas is heated to a
water at 4° C.
(c) Buoyancy in water at 0° C will be same as that in temperature of T = 3000 K and the gas gets
water at 4° C. converted to a gas of hydrogen atoms. The final
(d) Buoyancy may be more or less in water at 4° C pressure would be (considering all gases to be ideal)
depending on the radius of the sphere. [NCERT Exemplar]
Thermometry, Thermal Expansion and Kinetic Theory of Gases 669

(a) same as the pressure initially (a) the pressure on EFGH would be zero
(b) 2 times the pressure initially (b) the pressure on all the faces will be equal
(c) 10 times the pressure initially (c) the pressure of EFGH would be double the pressure on
(d) 20 times the pressure initiallly ABCD
(d) the pressure on EFGH would be half that on ABCD
35 Pressure versus temperature graph of an ideal gas of
equal number of moles of different volumes are 39 Figure shows two flasks connected to each other.
plotted as shown in figure. Choose the correct The volume of the flask 1 is twice that of flask 2.
alternative. The system is filled with an ideal gas at
p 4 temperatures 100 K and 200 K, respectively. If the
mass of the gas in flask 1 is m, then what is the mass
3 2
of the gas in flask 2?

1
100 K 200 K
T
(a) V1 = V2, V3 = V4 andV2 > V3
(b) V1 = V2, V3 = V4 andV2 < V3 (a) m (b) m /2 (c) m /4 (d) m /8
(c) V1 = V2 = V3 = V4 40 The given curve represents the variations of
(d) V4 > V3 > V2 > V1 temperature as a function of volume for one mole of
36. An inflated rubber balloon contains one mole of an an ideal gas. Which of the following curves best
ideal gas, has a pressure p, volumeV and represents the variation of pressure as a function of
temperature T. If the temperature rises to 1.1 T, and volume?
the volume is increased to 1.05V, the final pressure T
will be [NCERT Exemplar]
45º
(a) 1.1 p (b) p
(c) less than p (d) between p and 1.1 p
37 A ring shaped tube contains two ideal gases with
V
equal masses and relative molar masses M1 = 32 and
M 2 = 28. The gases are separated by one fixed p p
partition P and another movable stopper S which can (a) (b)
move freely without friction inside the ring.
M2 V V
P p p
α
S (c) (d)

V V
M1

The angle α in equilibrium as shown in the figure 41 The mass of hydrogen molecule is 3.32 × 10 −27 kg.
(in degrees) is If 10 23 hydrogen molecules strike per second at
(a) 291 (b) 219 (c) 129 (d) 192 2 cm2 area of a rigid wall at an angle of 45° from the
38 1 mole of an ideal gas is contained in a cubical normal and rebound back with a speed of 1000 ms −1,
volumeV, ABCDEFGH at 300 K (see figure). One then the pressure exerted on the wall is
face of the cube (EFGH ) is made up of a material
which totally absorbs any gas molecule incident on
it. At any given time, [NCERT Exemplar]
B θ
D Wall
θ
A C
G
F (a) 2.34 × 103 Pa (b) 0.23 × 106 Pa
E
H (c) 0.23 × 103 Pa (d) 23.4 × 103 Pa
670 OBJECTIVE Physics Vol. 1

42 Three rods of equal length l are joined to form an 45 A cylinder containing an ideal gas is in vertical
equilateral triangle PQR. O is the mid-point of PQ. position and has a piston of mass M that is able to
Distance OR remains same for small change in move up or down without friction (see figure). If the
temperature. Coefficient of linear expansion for PR temperature is increased, then [NCERT Exemplar]
and RQ is same, i.e. α 2 but that for PQ is α 1. Then, M
R

P O Q
(a) both p andV of the gas will change
(a) α 2 = 3 α1 (b) α 2 = 4 α1 (b) only p will increase according to Charles’ law
(c) α1 = 3 α 2 (d) α1 = 4 α 2 (c) V will change but not p
43 A piece of metal weighs 46 g in air. When it is (d) p will change but notV
immersed in a liquid of specific gravity 1.24 at 27°C, 46 A cylindrical tube of uniform cross-sectional area A
it weighs 30 g. When the temperature is raised to is fitted with two air tight frictionless pistons.
42°C, the metal piece weighs 30.5 g. If the specific
gravity of the liquid at 42°C is 1.20, the coefficient
of linear expansion of the metal is Wire
−3 −5
(a) 1.6 × 10 /° C (b) 2.3 × 10 /° C
−3
(c) 4.3 × 10 /° C (d) 3.4 × 10−3 /° C
44 A vertical cylinder closed at both ends is fitted with The pistons are connected to each other by a
a smooth piston dividing the volume into two parts metallic wire. Initially, the pressure of the gas is p 0
each containing one mole of air. At the equilibrium and temperature is T 0 , atmospheric pressure is also
temperature of 320 K, the upper and lower parts are p 0 . Now, the temperature of the gas is increased to
in the ratio 4 :1. The ratio will become 3 : 1 at a 2T 0 , the tension in the wire will be
temperature of p 0A
(a) 2 p 0A (b) p 0A (c) (d) 4 p 0A
(a) 450 K (b) 228 K (c) 420 K (d) 570 K 2

(B) Medical entrance special format questions


Assertion and reason Reason Translational kinetic energy of any
Directions (Q. Nos. 1-4) These questions consists of two 3
type of gas at temperature T would be RT for
statements each printed as Assertion and Reason. While 2
answering these questions you are required to choose any one mole.
one of the following four responses.
(a) If both Assertion and Reason are correct and Reason is 3. Assertion Degree of freedom of a monoatomic gas
the correct explanation of Assertion. is always three, whether we consider vibrational
(b) If both Assertion and Reason are correct but Reason is effects or not.
not the correct explanation of Assertion. Reason At all temperatures (low or high),
(c) If Assertion is correct but Reason is incorrect. vibrational kinetic energy of an ideal gas is zero.
(d) If Assertion is incorrect but Reason is correct.
1 Assertion In summers, a metallic scale will read 4. Assertion Total internal energy of oxygen gas at a
3
more than the actual. given temperature is E. Of this energy, E is
Reason In summers, length of metallic scale 2 5
translational kinetic energy and E is rotational
increases. 5
kinetic energy.
2 Assertion Translational kinetic energy of any gas
1 Reason Potential energy of an ideal gas is zero.
2
at temperature T would be mv rms .
2
Thermometry, Thermal Expansion and Kinetic Theory of Gases 671

Statement based questions Match the columns


1 From the following statements, concerning ideal gas 1 In the process T 2 ∝ 1/V , pressure of the gas increases
at any given temperature T, which of the following from p 0 to 4p 0 . Match the Column I with Column II and
statement is incorrect?. mark the correct option from the codes given below.
(a) The coefficient of volume expansion at constant
pressure is same for all ideal gases. Column I Column II
(b) The average translational kinetic energy per molecule (A) Temperature of the gas (p) Positive
of oxygen gas is kB T (kB being Boltzmann constant).
(c) In a gaseous mixture, the average translational kinetic (B) Volume of the gas (q) Negative
energy of the molecules of each component is same. (C) Energy of the gas (r) Two times
(d) The mean free path of molecules increases with the (s) None of these
decrease in pressure.
2 A bimetallic strip is formed out of two identical Codes
strips, one of copper and other of brass. The A B C A B C
(a) s r p (b) r s p
coefficients of linear expansion of the two metals are
(c) r p s (d) s p r
α c and α b . On heating, the temperature of the strip
goes up by ∆T and the strip bends to form an arc of 2 In the V-T graph shown in the figure. Match the
radius of curvature R. Then, which of the following Column I with Column II and mark the correct
statement is correct about the radius of curvature? option from the codes given below.
(a) It is proportional to ∆T. V
(b) It is inversely proportional to ∆T . A
(c) It is proportional to α b – α c .
(d) It is inversely proportional to α b + α c . B
3. I. The pressure of an ideal gas depends on the
volume of the gas. T

II. The pressure of an ideal gas depends on the


temperature of the gas. Column I Column II
Which of the following statement(s) is/are correct? (A) Gas A is (p) Monoatomic
(a) Only I (b) Only II (B) pA /pB is (q) Diatomic
(c) Both I and II (d) Neither I nor II (C) n A / n B is (r) >1
4 I. Atomic theory relates molecular properties with (D) Gas B is (s) <1
bulk properties of a gas. (t) Cannot say anything
II. According to kinetic theory, the interatomic forces
are negligible in gases. Codes
III. According to kinetic theory, the gas consists of A B C D
rapidly moving atoms. (a) t p r t
(b) t s t t
Which of the following statement(s) is/are correct? (c) t t t t
(a) Both I and II (b) Only III (d) s t p t
(c) Both II and III (d) I, II and III

(C) Medical entrances’ gallery


Collection of questions asked in NEET & various medical entrance exams

1 The average thermal energy for a monoatomic gas is (a) 0.2 kg/m 3 (b) 0.1 kg/m 3
(where, kB is Boltzmann constant and T is absolute (c) 0.02 kg/m 3 (d) 0.5 kg/m 3
temperature.) [NEET 2020] 3 The mean free path λ for a gas, with molecular
3 5 7 1 diameter d and number of density n can be
(a) kBT (b) kBT (c) kBT (d) kBT
2 2 2 2 expressed as [NEET 2020]
2 A cylinder contains hydrogen gas at pressure of 1 1 1 1
(a) (b) (c) 2 2 2
(d)
249 kPa and temperature 27°C. Its density is 2 nπ d 2
2 n πd
2 2
2n π d 2 nπ d
. J mol −1 K −1)
(Take, R = 83 [NEET 2020]
672 OBJECTIVE Physics Vol. 1

4 An ideal gas equation can be written as 11 The wooden wheel of radius R is made of two
ρRT semicircular parts (see figure). The two parts are
p=
M0 held together by a ring made of a metal strip of
cross-sectional area S and length L. L is slightly less
where, ρ and M 0 are respectively, [NEET 2020]
than 2πR. To fit the ring on the wheel, it is heated,
(a) mass density, mass of the gas so that its temperature rises by ∆T and it just steps
(b) number density, molar mass
over the wheel. As it cools down to surrounding
(c) mass density, molar mass
temperature, it presses the semicircular parts
(d) number density, mass of the gas
together. If the coefficient of linear expansion of the
5 The mean free path l for a gas molecule depends metal is α and its Young’s modulus isY, the force
upon diameter d of the molecule as [NEET 2020] that one part of the wheel applies on the other part
1 1 is [JIPMER 2019]
(a) l ∝ (b) l ∝ d (c) l ∝ d 2
(d) l ∝
d2 d
6 A copper rod of length 88 cm and an aluminium rod
of unknown length have their increase in length R
independent of increase in temperature. The length
of aluminium rod is [NEET (National) 2019]
(a) 113.9 cm (b) 88 cm
(c) 68 cm (d) 6.8 cm (a) 2πSYα∆T (b) SYα∆T
(c) πSYα∆T (d) 2SYα∆T
7 Increase in temperature of a gas filled in a container
would lead to [NEET 2019] 12 At what temperature will the rms speed of oxygen
(a) increase in its kinetic energy molecules become just sufficient for escaping from
(b) decrease in its pressure the earth’s atmosphere? [NEET 2018]
(c) decrease in intermolecular distance (Take, mass of oxygen molecule,
(d) increase in its mass m = 2.76 × 10 −26 kg and Boltzmann’s constant,
8 An ideal gas initially at pressure 1 bar is being kB = 1.38 × 10 −23 J K −1 )
compressed from 30 m3 to 10 m3 volume and its
(a) 5.016 × 104 K (b) 8.326 × 104 K
temperature decreases from 320 K to 280 K, then
find the value of final pressure of the gas. [AIIMS 2019] (c) 2.508 × 104 K (d) 1.254 × 104 K
(a) 2.625 bar (b) 3.4 bar 13 The coefficient of volume expansion of glycerine is
(c) 1.325 bar (d) 4.5 bar 49 × 10 −5 ° C −1. What is the fractional change in
9 Assertion NH3 is liquified more easily than CO 2 . density for a 30° C rise in temperature?
[JIPMER 2018, UP CPMT 2015]
Reason Critical temperature of NH3 is more than
(a) 0.0155 (b) 0.0145
CO 2 . [AIIMS 2019] (c) 0.0255 (d) 0.0355
(a) Both Assertion and Reason are correct and Reason is the 14 A given sample of an ideal gas occupies a volumeV
correct explanation of Assertion.
at a pressure p and absolute temperature T. The
(b) Both Assertion and Reason are correct but Reason is not
the correct explanation of Assertion. mass of each molecule of the gas is m. Which of the
(c) Assertion is correct but Reason is incorrect. following gives the density of the gas? [NEET 2017]
(d) Both Assertion and Reason are incorrect. p pm p
(a) (b) (c) (d) mkT
kT kT kTV
10 Assertion Vibrational degree of freedom of a
diatomic gas molecule appears at every high 15 Assertion The molecules of a monoatomic gas have
temperature. three degrees of freedom.
Reason Diatomic gas has two vibrational degree of Reason The molecules of diatomic gas have five
freedom in one direction. [AIIMS 2019] degrees of freedom. [NEET 2017]
(a) Both Assertion and Reason are correct and Reason is the (a) Both Assertion and Reason are correct and Reason is the
correct explanation of Assertion. correct explanation of Assertion.
(b) Both Assertion and Reason are correct but Reason is not (b) Both Assertion and Reason are correct but Reason is not
the correct explanation of Assertion. the correct explanation of Assertion.
(c) Assertion is correct but Reason is incorrect. (c) Assertion is correct but Reason is incorrect.
(d) Both Assertion and Reason are incorrect. (d) Both Assertion and Reason are incorrect.
Thermometry, Thermal Expansion and Kinetic Theory of Gases 673

16 A gas mixture consists of 2 mol of O 2 and 4 mol of 22 The rms speed of oxygen molecule in a gas at 27°C
Ar at temperature T. Neglecting all vibrational would be given by [UK PMT 2015]
modes, the total internal energy of the system is (a) 483 ms −1 (b) 966 ms −1
[NEET 2017]
(c) 4.83 ms −1 (d) 9.66 ms −1
(a) 4 RT (b) 15 RT (c) 9 RT (d) 11 RT
17 The coefficient of cubical expansion of mercury is 23 The pressure of an ideal gas is directly proportional
0.00018/°C and that of brass 0.00006 /°C. If a to [UK PMT 2015]
barometer having a brass scale were to read 74.5 cm (a) total kinetic energy (b) translational kinetic energy
(c) rotational kinetic energy (d) vibrational kinetic energy
at 30°C, find the true barometric height at 0°C. The
scale is supposed to be correct at 15°C. [AIIMS 2017] 24 Two metal rods of lengths L 1 and L 2 and coefficients
(a) 74.122 cm (b) 79.152 cm (c) 42.161 cm (d) 142.39 cm of linear expansion α 1 and α 2 respectively are
18 Mercury boils at 367°C. However, mercury welded together to make a composite rod of length
thermometers are made such that they can measure (L 1 + L 2 ) at 0°C. Find the effective coefficient of
temperature upto 500°C. This is done by [JIPMER 2017] linear expansion of the composite rod. [EAMCET 2015]
(a) maintaining vacuum above mercury column in the L1α12 − L 2 α 22 L12 α1 − L22 α 2
stem of the thermometer (a) (b)
+L12 L22 L12 + L22
(b) filling nitrogen gas at high pressure above the mercury
column L α + L2 α 2 L α + L2 α 2
(c) 1 1 (d) 1 1
(c) filling oxygen gas at high pressure above the mercury L1 − L 2 L1 + L 2
column
(d) filling nitrogen gas at low pressure above the mercury 25 The ratio of rms speed of an ideal gas molecules at
column pressure p to that at pressure 2p is [Kerala CEE 2015]
19 A graph between pressure p (alongY-axis) and (a) 1 : 2 (b) 2 : 1 (c) 1 : 2 (d) 2 : 1
absolute temperature T(along X-axis) for equal moles (e) 1 : 1
of two gases has been drawn. Given that volume of 26 The rms speed of oxygen is v at a particular
second gas is more than volume of first gas. Which temperature. If the temperature is doubled and
of the following statement(s) is/are correct? oxygen molecules dissociate into oxygen atoms, the
p [JIPMER 2017]
1
rms speed becomes [WB JEE 2015]
(a) v (b) 2v (c) 2v (d) 4 v
2 27 The deviation of a real gas from the ideal one is
minimum at [UP CPMT 2015]
(a) low pressure and high temperature
T
(b) low temperature and low pressure
(a) Slope of gas 1 is less than gas 2.
(c) high pressure and low temperature
(b) Slope of gas 1 is more than gas 2. (d) high pressure and high temperature
(c) Both have some slopes.
(d) None of the above 28 The density of hydrogen gas at STP is 0.09 kg m −3 .
The mean kinetic energy of one gram molecule of
20. Coefficient of linear expansion of brass and steel hydrogen gas is approx [UP CPMT 2015]
rods are α 1 and α 2 . Lengths of brass and steel rods
(a) 3403 J (b) 3500 J
are l1 and l 2 , respectively. If (l 2 − l1 ) is maintained (c) 3704 J (d) 3207 J
same at all temperatures, which one of the following
29 At constant pressure, the ratio of increase in volume
relations holds good ? [NEET 2016]
of an ideal gas per degree rise in kelvin temperature
l = α 2l 2 (d) α1l 2 = α 2l1
(a) α1l 22 = α 2l12 (b) α12l 2 = α 22l1 (c) α11 to its volume is [Manipal 2015]
21 The molecules of a given mass of a gas have rms (a) 1/T (b) 1/T 2
−1 −2
velocity of 200 ms at 27°C and 10 . × 10 Nm 5
(c) T (d) T 2
pressure. When the temperature and pressure of the 30 Water is heated from 0° C to 10° C, then its volume
gas are respectively, 127°C and 0.05 × 10 5 Nm−2 , the (a) does not change [KCET 2015]
rms velocity of its molecules (in ms −1 ) is [NEET 2016] (b) decreases
400 100 2 100 (c) first decreases and then increases
(a) (b) (c) (d) 100 2 (d) increases
3 3 3
674 OBJECTIVE Physics Vol. 1

31 In anomalous expansion of water, at what 38 The average kinetic energy of a gas molecule at
temperature, the density of water is maximum? absolute temperature T is [Kerala CEE 2013]
[KCET 2014] 2
(a) directly proportional to T
(a) 4°C (b) < 4° C (c) > 4° C (d) 10° C (b) inversely proportional to T 2
32 A metal rod is fixed rigidly at two ends so as to (c) directly proportional to T
prevent its thermal expansion. If L, α and Y (d) inversely proportional to T
respectively denote the length of the rod, coefficient (e) inversely proportional to T 3
of linear thermal expansion and Young’s modulus of 39 At what temperature, the rms velocity of gas
its material, then for an increase in temperature of
molecules would be double of its value at NTP, if
the rod by ∆T, the longitudinal stress developed in
pressure is remaining constant? [MP PMT 2013]
the rod is [AMU 2014]
(a) 819 °C (b) 819 K (c) 546 K (d) 546°C
(a) inversely proportional to α
(b) inversely proportional toY 40 The average kinetic energy per mole of hydrogen at
(c) directly proportional to ∆T /Y given temperature is [MP PMT 2013]
(d) independent of L (a) equal of that of oxygen (b) 16 times that of oxygen
33 Which one of the following is a wrong statement in 1 1
(c) times that of oxygen (d) times that of oxygen
kinetic theory of gases? [Kerala CEE 2014] 16 8
(a) The gas molecules are in random motion. 41 In kinetic theory of gases, it is assumed that
(b) The collision between molecules is inelastic. molecular collisions are [UP CPMT 2013]
(c) The volume occupied by the molecules of a gas is (a) for negligible duration
negligible. (b) inelastic
(d) The force of attraction between the molecules is (c) one-dimensional (head on)
negligible. (d) unable to exert mutual force
(e) Both (b) and (c)
42 The number of molecules in a litre of a gas at
34 The average pressure of an ideal gas is [J&K CET 2013] temperature of 27°C and a pressure of 10 6 dyne cm −2
(a) p = (1 / 3) mnVav2 (b) p = (1 / 2) mnVav is [UP CPMT 2013]
(c) p = (1 / 4) mnVav2 (d) p = (1 / 3) mnVav (a) 2.4 × 1020 (b) 2.4 × 1021 (c) 2.4 × 1022 (d) 2.4 × 1023

35 According to equipartition law of energy, each 43 A non-conducting body floats in a liquid at 20°C
particle in a system of particles have thermal energy 2
with of its volume immersed in the liquid. When
E equal to [J&K CET 2013] 3
(a) kBT (b) (1 / 2) kBT 3
liquid temperature is increased to 100°C, of body’s
(c) 3 kBT (d) (3 / 2) kBT 4
36 When a liquid is heated in a glass vessel, its volume is immersed in the liquid. Then, the
coefficient of apparent expansion is 1.03 × 10 −3 /° C. coefficient of real expansion of the liquid is
When the same liquid is heated in a copper vessel, (neglecting the expansion of container of the liquid)
[AIIMS 2012]
its coefficient of apparent expansion is
(a)15.6 × 10−4/°C (b)156 × 10−4/°C
1.006 × 10 −3 /° C. If the coefficient of linear
(c) 1.56 × 10−4/°C (d) 0156
. × 10−4/°C
expansion of copper is 17 ×10 −6 /° C, then the
44 During an experiment an ideal gas is found to obey
coefficient of linear expansion of glass is
[EAMCET 2013] an additional lawVp 2 = constant. The gas is initially
(a) 8 .5 × 10−4 /° C (b) 9 × 10−6 /° C at temperature T and volumeV. When it expands to
volume 2V, the resulting temperature is
(c) 27 × 10−6 /° C (d) 10 × 10−4 /° C [UP CPMT 2012]
37 A horizontal uniform tube, open at both ends, is T T
(a) (b) 2 T (c) 2T (d)
containing a liquid of certain length at some 2 2
temperature. When the temperature is changed, the 45 Two temperature scales A and B are related by
length of the liquid in the tube is not changed. If α is A − 42 B − 72
the coefficient of linear expansion of the material of = . At which temperature, two scales
the tube and γ is the coefficient of volume expansion 110 220
of the liquid, then [EAMCET 2013]
have the same readings? [WB JEE 2011]
(a) − 42° (b) − 72° (c) + 12° (d) − 40°
(a) γ = 2α (b) γ = 3 α (c) γ = 4 α (d) γ = α
ANSWERS
CHECK POINT 14.1
1. (b) 2. (a) 3. (a) 4. (a) 5. (c) 6. (c) 7. (c) 8. (d)

CHECK POINT 14.2


1. (c) 2. (a) 3. (b) 4. (b) 5. (b) 6. (a) 7. (d) 8. (c) 9. (c) 10. (a)
11. (a) 12. (c) 13. (a) 14. (b) 15. (c) 16. (b) 17. (a)

CHECK POINT 14.3


1. (d) 2. (b) 3. (c) 4. (c) 5. (d) 6. (a) 7. (a) 8. (b) 9. (d) 10. (c)
11. (c) 12. (b) 13. (a) 14. (d) 15. (c) 16. (a) 17. (c) 18. (c) 19. (c)

CHECK POINT 14.4


1. (c) 2. (c) 3. (d) 4. (b) 5. (c) 6. (c) 7. (b) 8. (c) 9. (c) 10. (c)
11. (a) 12. (b) 13. (c) 14. (c) 15. (c) 16. (b) 17. (a) 18 (c) 19. (b) 20. (c)
21. (d)

(A) Taking it together


1. (c) 2. (d) 3. (b) 4. (a) 5. (b) 6. (a) 7. (d) 8. (a) 9. (a) 10. (c)
11. (a) 12. (a) 13. (a) 14. (d) 15. (a) 16. (b) 17. (a) 18. (d) 19. (b) 20. (a)
21. (b) 22. (a) 23. (c) 24. (c) 25. (c) 26. (d) 27. (a) 28. (d) 29. (a) 30. (d)
31. (b) 32. (b) 33. (b) 34. (d) 35. (a) 36. (d) 37. (d) 38. (d) 39. (c) 40. (a)
41. (a) 42. (d) 43. (b) 44. (a) 45. (c) 46. (b)

(B) Medical entrance special format questions


l Assertion and reason
1. (d) 2. (b) 3. (c) 4. (b)

l Statement based questions


1. (b) 2. (b) 3. (c) 4. (c)

l Match the columns


1. (b) 2. (c)

(C) Medical entrances’ gallery


1. (a) 2. (a) 3. (a) 4. (a) 5. (a) 6. (c) 7. (a) 8. (a) 9. (a) 10. (b)
11. (d) 12. (b) 13. (b) 14. (b) 15. (b) 16. (d) 17. (a) 18. (b) 19. (b) 20. (c)
21. (a) 22. (a) 23. (b) 24. (d) 25. (c) 26. (c) 27. (a) 28. (a) 29. (a) 30. (c)
31. (a) 32. (c) 33. (b) 34. (a) 35. (b) 36. (b) 37. (b) 38. (c) 39. (a) 40. (a)
41. (a) 42. (c) 43. (a) 44. (c) 45. (c)
Hints & Explanations
l CHECK POINT 14.1
9 9
2 (a) ∆TF = ∆TC = × 25° C = 45° F 7 (d) Fractional change in period,
5 5 ∆T 1 1
= α∆θ = × 2 × 10 −6 × 10 = 10 −5
3 (a) Let the required temperature is θ. T 2 2
θ − 0 θ − 32 ∆T
∴ = ⇒ θ = − 40 ° % change = × 100 = 10 −5 × 100 = 10 −3 %
100 180 T
4 (a) On this thermometer, 34°C will read as, 8 (c) Lα∆θ + 2L (2α ) (∆θ ) = (3L ) (α av ) ∆θ
t − (−20 ) 34 − 0
= L 2L 3L, αav
130 − (−20 ) 100 − 0 ⇒
150 α 2α
⇒ t = −20 + × 34 = 31°
100 5
F − 32 K − 273 x − 32 x − 273 or α av = α
5 (c) = ⇒ = 3
9 5 9 5 ∆l lα∆θ
9 (c) Strain = = = α∆θ
⇒ x = 574.25 l l
6 (c) Temperature shown on faulty thermometer will be Stress = Y × Strain =Yα ∆θ
 95 − 5 (Stress)1 = (Stress)2
t = 5+   × 40 = 41°
 100  ∴ Y1α 1 = Y2α 2 (Q ∆θ → same)
Y1 α 2 3
T
7 (c) Given, C =
2 5
⇒ TF = TC or = =
TF 5 2 Y2 α 1 2
TC (5/ 2) TC − 32 ∆l
∴ = 10 (a) Strain = = α∆θ
100 180 l
25 Stress = Y × Strain =Yα∆θ
⇒ 9TC = TC − 160
2 ∴ Force or tension, T = Stress × Area =YA α∆θ
⇒ TC = 45.71° C πYαd 2 ∆θ  π d 2
= Q A = 
 80 − 50  4  4 
8 (d) t =   × 100 ° C
 75 − 50 
π × 2 × 1011 × 10 −5 × 10 −4 × 25
or T =
∴ t = 120 ° C 4
= 3925 N ≈ 4000 N
l CHECK POINT 14.2
11 (a) Volumetric expansion will take place. So, if the ball is
1 (c) Change in length, ∆l = l0α ∆T = 10 × 11 × 10 −6 (19 − 20 )
heated, then the volume of the cavity will increase.
= − 11 × 10 −5 cm 12 (c) V = V0 (1 + γ∆θ )
The negative sign shows that, it is shorter in size. L3 = L0 (1 + α 1 ∆θ ) L20 (1 + α 2∆θ )2
2 (a) As, metal expands on heating in all directions, so the = L30 (1 + α 1∆θ ) (1 + α 2∆θ )2
radius of the hole will also increase in size.
Since, L30 = V0 and L3 = V
3 (b) With decreasing temperature, length of the steel tape
decreases and hence the tape gives reading greater than the Hence, 1 + γ∆θ = (1 + α 1∆θ ) (1 + α 2 ∆θ )2
real reading. So, real length of wood must be less than 25 cm. ≅ (1 + α 1∆θ ) (1 + 2 α 2∆θ )
l α (Using Binomial expansion)
4 (b) ∆l1 = ∆l2 ⇒ l1α 1∆θ = l2α 2∆θ ⇒ 1 = 2
l2 α 1 ≅ (1 + α 1∆θ + 2α 2 ∆θ )
5 (b) Original value of circumference, l = 2 πR ⇒ γ = α1 + 2 α 2
∴ ∆l = lαθ = (2πR ) αθ 14 (b) Coefficient of volume expansion of container will become
α
6 (a) As, ∆L = L α ∆T ⇒ 0.19 = 100 × α × 100 3   , which is greater than the coefficient of volume
 2
⇒ α = 0.19 × 10 −4 = 1.9 × 10 −5 ° C−1 expansion of liquid. Hence, container will expand more and
⇒ γ = 3 α = 5.7 × 10 −5 / ° C the level of the liquid in the container would fall.
Thermometry, Thermal Expansion and Kinetic Theory of Gases 677

15 (c) Water has maximum density at 4°C, so if the water is 11 (c) v rms ∝ T . If temperature is doubled, rms speed will
heated above 4°C or cooled below 4°C, density decreases, i.e.
become 2 times.
volume increases. In other words, it expands and overflows in
both the cases. ∴ v rms = 200 2 ms −1
16 (b) Volume of mercury that will overflow, 12 (b) From the given velocities, the most probable velocity
∆V = V∆T (γmercury − γ glass ) is 4 ms −1.
13 (a) At a given temperature (T ), all the ideal gas molecules no
. × 10 −4 − 0.12 × 10 −4 ) = 2.688 ~− 2.69 cm 3
= 200 × 80 (18 matter what their masses, have the same average translational
17 (a) ρ 200 = ρ 0 (1− γ m∆T ) = 13.6(1− 0.18 × 10 −3 × 200 ) = 1311
. g/cc kinetic energy.
3
i.e. E = kT
l CHECK POINT 14.3 2
So, E does not depend upon density. (Q Mass = ρV )
1 (d) All molecules have same speed. This statement is true but
E1
this is not an assumption of kinetic theory of gases. = 1: 1
E2
2 (b) Total pressure inside the container is given as
1 mN 2 14 (d) As, pV = nRT
p= v rms ⇒ p ∝ m …(i)
3 V m RT m RT
⇒ p H2 = ⋅ and p He = ⋅
(When volume and temperature are constants) MH 2 V MHe V
From Eq. (i), p H2 MHe 4 × 10 −3
Pressure first increases and then becomes constant. ∴ = = = 2 ⇒ p H2 = 2p He
p He MH2 2 × 10 −3
3 (c) pV = nRT ⇒ p ∝ nT (QV and R are constants )
15 (c) The kinetic energy of gas is given as
p 2 n 2 T2
⇒ = × 1 2
p1 n1 T1 K = mvrms
2
p He 1 2T
⇒ = × ⇒ p He = 2p 3RT
p 1 T Also, vrms =
M
3RT 8RT 2RT
4 (c) vrms = , v av = and v p = 2
 T 
2
M πM M K2  (vrms )2 
Now, vrms ∝ T ⇒ =  =  2
⇒vrms :v av :vp = 3 : 8 / π : 2 K1  (vrms )1   T1 
∴ v rms > v av > v p T   600 
⇒ K2 = K1 2  ⇒ K2 = K1  
5 (d) As, vrms ∝ T , so vrms will become two times when  T1   300 
temperature is made four times. ∴ K2 = 2K1
6 (a) T1 = 273 + 27 = 300 K ⇒ T2 = 273 + 327 = 600 K 16 (a) The gases carbon monoxide (CO) and nitrogen (N2 ) have
T2 = 2T1 and vrms ∝ T equal translational kinetic energy as both are diatomic gases,
i.e.
v ′ rms T T1
⇒ = 1= E1 = E 2
v ′′rms T2 2T1
17 (c) Kinetic energy of an ideal gas depends only on its
v ′ rms 1
⇒ = ⇒ v ′′rms = 2 v ′ rms temperature. Hence, it remains constant whether its pressure
v ′′rms 2 is increased or decreased.
(3v )2 + (4v )2 + (5v )2 50 18 (c) The kinetic energy of gas molecules is the internal
7 (a) vrms = = v property, which is independent of the motion of frame. The
3 3
temperature depends on the kinetic energy, so it will remains
8 (b) vrms ∝ T unchanged, when the container is kept in a moving train.
root-mean-square velocity will become half, if temperature 19 (c) For air at STP, n = 2.7 × 10 25 m −3
 273 
becomes (1/4)th or  − 273 ° C or − 204.75° C. and d = 2 × 10 −10 m
 4 
1
(1)2 + (2)2 + (3)2 + (4)2 15 Mean free path, λ =
9 (d) vrms = = kms −1 2nπd 2
4 2
On putting values, we get
T
10 (c) vrms ∝ λ=
1
M
2 × 2.7 × 10 25
. × (2 × 10 −10 )2
× 314
T is doubled and M has become half. Therefore, vrms will
= 2.08 × 10 −7 m
become two times.
678 OBJECTIVE Physics Vol. 1

l CHECK POINT 14.4 ∴ f = 3 × 1− 0 = 3


where, N = number of particles in the system
1 1 = p 2V2
1 (c) According to Boyle’s law, pV
and K = number of independent relations between the
As, the pressure is decreased by 20%, therefore particles.
80 80 80 8 (c) f = 3N − K
p2 = p1 ⇒ pV 1 1= 1 2 ⇒ V1 =
pV V2
100 100 100 For non-linear triatomic gas, f = 3 × 3 − 3 = 6
∴ Percentage increase in volume 9 (c) Average translational kinetic energy per molecule is
V −V 100 − 80 (3/ 2) kT which depends only on temperature and not on
= 2 1 × 100 = × 100 = 25%
V1 80 molecular mass.

2 (c) For a given pressure, volume will be more, if temperature 10 (c) Translational degrees of freedom for any type of gas is three.
is more (Charles’ law). 1  3
∴ Total translational kinetic energy = 3  nRT  = pV
p 2  2
11 (a) Both O 2 and N 2 are diatomic gases. Rotational degree of
freedom in both cases is two. Therefore, average rotational
Constant 1 
kinetic energy per molecule for each of them is 2  kBT  or
pressure 2 
T2
the ratio is 1 : 1.
T1
V 12 (b) In monoatomic gases, only translational kinetic energy is
V1 V2
present, which is proportional to temperature and is given by
From the graph, it is clear thatV2 > V1 ⇒ T2 > T1 (3/ 2) kBT .
300 6
∴ Ratio of average kinetic energy = =
3 (d) For perfect gas,V ∝ T 350 7
V1 T1 3
∴ = …(i) 13 (c) E = kT ⇒ E ∝ T
V2 T2 2
E 2 T2 600
According to the question, ∴ = = = 2 ⇒ E 2 = 2E1
E1 T1 300
V1 = V , thenV2 = 2V and T1 = 300 K
m m
1 300 14 (c) As, pV = µRT = RT ⇒ 20 × V = × R × 300 ...(i)
∴ = ⇒ T2 = 600 K ⇒ T2 = 327° C M M
2 T2
When m = m/2 and p = p′, then
So, ∆T = 327 − 27 = 300°C
 m /2
As, p′ × V =   R × 350 ...(ii)
4 (b) From Charles’ law,
V1 T1
=  M 
V2 T2
From Eqs. (i) and (ii), we get
VT
⇒ V2 = 1 2 p′ =
140
≈ 11.7 atm
T1 12
(297 + 273) 570
∴ V2 = 1 × = = 1.9 L 15 (c) Mean kinetic energy of gas molecules,
(27 + 273) 300
f f f  f
E = kBT = kB (t + 273) =  kB  t + × 273 kB
5 (c) V-T graph is a straight line passing through origin. Hence, 2 2 2  2
process is isobaric.
Comparing it with standard equation of straight line
 nR  nR
V =   T ⇒ Slope = y = mx + c, we get
 p p f f
m = k and c = 273 k. So, the graph between E and t will
Slope of 2 > Slope of 1 2 2
∴ p 2 < p1 be straight line with positive intercept on E-axis and positive
slope with t-axis.
 nRT   nRT   nRT 
6 (c) p = p1 + p 2 + p 3 =   +  +  pV p V pVT
 V 1  V  2  V  3 16 (b) As, 1 1 = 2 2 or V2 = 1 1 2
T1 T2 p 2T1
RT
= (n1 + n 2 + n 3 ) 1× 500 × (273 − 3)
V ∴ Volume of He, V2 =
(0.5 + 0.5 + 0.25)(8.31)(300) 0. 5 × (273 + 27)
= 1× 500 × 270
5 × 10 −3 = = 900 m3
0.5 × 300
= 6.23 × 10 5 Nm−2 ≈ 6 × 10 5 Nm −2
m
17 (a) As, pV = nRT = RT
7 (b) f = 3N − K M
For monoatomic gas, N = 1, K = 0 where, M = molecular weight.
Thermometry, Thermal Expansion and Kinetic Theory of Gases 679

p RT p RT p ⇒ pV = constant (At constant temperature)


⇒ = ⇒ = ⇒ ∝T
m /V M ρ M ρ 1
i. e . p∝ (where, p = pressure andV = volume)
(Q for a given gas, M is constant) V
Temperature is directly proportional to the slope of So, this process can be called as isothermal process.
p -ρ graph, so T1 > T2 . 4 (a) We know that as temperature increases, vibration of
18 (c) As θ 2 > θ1 ⇒ tan θ 2 > tan θ1 molecules about their mean position increases. Hence, kinetic
energy associated with random motion of molecules increases.
⇒ V2 > V1
∆TC ∆TF 212 − 140
19 (b)
V nR
= 5 (b) = =
T p 100 180 180
72
V and T for both cases are same. i.e. ∆TC = 100 × = 40 ° C
180
n1 n 2 m1 m2
Hence, = or = ∴ Fall in temperature = 40 °
p1 p 2 p1M1 p 2M2
(p t − p 0 ) (60 − 50 )
p 2M2 (3) (2) 6 (a) t = × 100 ° C = × 100 ° C = 25° C
or m2 = ⋅ m1 = ⋅ 20 = 0.86 kg (p100 − p 0 ) (90 − 50 )
p1M1 (5) (28)
V1 1 + γT1 100 1 + γ × 20
m 7 (d) = ⇒ =
20 (c) pV = µRT = RT V2 1 + γT2 125 1 + γ × 100
M
p ⇒ γ = 0.0033 /°C
8 (a) ρ = ρ 0 (1 − γ ⋅ ∆θ ) = 13.6 [1 − 0.18 × 10 −3 (473 − 273)]
m2
= 13.6 [1 − 0.036] = 13.11g/cc
m1
p T p − p1 T2 − T1
9 (a) p ∝ T ⇒ 1 = 1 ⇒ 2 =
p 2 T2 p1 T1
 ∆p   251 − 250 
V1 V2 V ⇒  % =  × 100 = 0.4%
 p   250 
m1 RT
For 1st plot, p= …(i)
M V1 10 (c) Gas equation for N molecules pV = NkT

p=
m 2 RT pV hρgV 1.2 × 10 −10 × 13.6 × 10 3 × 10 × 10 −4
For 2nd plot, …(ii) ⇒ N= = =
M V2 kT kT 1.38 × 10 −23 × 300
From Eqs. (i) and (ii), we get = 3.94 × 1011
m1 V1
= ⇒ m ∝V 3RT
m 2 V2 11 (a) vrms = ⇒ T ∝M (Q vrms and R are constants )
M
As, V2 > V1 ⇒ m1 < m 2
TH2 MH 2 TH2 2
21 (d) p V = constant
2 ⇒ = ⇒ = ⇒ TH2 = 20 K
TO 2 MO 2 (273 + 47) 32
nRT CT
p= or p = (C = constant) 3RT T
V V 12 (a) vrms = ⇒ vrms ∝
M M
T2
∴ = constant or T ∝ V
V v He 5 THe MH2
= = ×
V is increased to 3 times. Hence, T will increase 3 times. v H2 7 MHe TH2
25 4
⇒ THe = × × 273 = 278.57 K ≈ 5.57° C
49 2
(A) Taking it together
3RT 3RT
13 (a) vrms = ⇒ M= 2
F − 32 K − 273 F − 32 0 − 273 M vrms
1 (c) = ⇒ =
9 5 9 5 At room temperature, T = 27° C = 300 K
F = − 459.4° F ≈ − 460 ° F 3 × 8.3 × 300
∴ M= ≈ 2 × 10 −3 kg = 2 g
(1930 )2
2 (d) Zero kelvin = − 273° C (absolute temperature). As no
matter can attain this temperature, hence temperature can ⇒ Gas is hydrogen.
never be negative on Kelvin scale. T −0 T − 10
14 (d) Using C =
3 (b) Boyle’s law is applicable when temperature is constant. 100 − 0 130 − 10
i. e . pV = nRT = constant 40 − 0 T − 10
⇒ =
100 − 0 130 − 10
680 OBJECTIVE Physics Vol. 1

40 T − 10 2 T − 10
⇒ = or = 20 (a) As the temperature is increased, length of the pendulum
100 120 5 120 increases but its centre of mass remains at the centre of bob.
⇒ 5T − 50 = 240 The time period of pendulum,
290
⇒ 5T = 290 ⇒ T = = 58°
5
1 L
15 (a) Number of seconds lost in a day, ∆t = α∆θ × 86400
2
The coefficient of linear expansion of metal pendulum, Pendulum
2∆t 2 × 12.5
α= = T = 2π
L
⇒ T∝ L
∆θ × 86400 25 × 86400 g
1
⇒ α= /° C So, as L increases, time period (T ) will also increases.
86400
21 (b) As the motion of the vessel as a whole does not affect the
16 (b) Relation between Celsius and Fahrenheit scale of relative motion of the gas molecules with respect to the walls
C F − 32
temperature is = of the vessel, hence pressure of the gas inside the vessel, as
5 9 observed by us, on the ground remains the same.
5 160
⇒ C= F − 22 (a) Let the relation between the thermometer reading and
9 9 centigrade scale be y = ax + b .
Equating above equation with standard equation of the line Given, at x = 100, y = 80 and at x = 0, y = 10
y = mx + c, we get ∴ 80 = 100a + b, 10 = b ⇒ a = 0.7
5 Now, we have to find x when y = 59
Slope of the line AB is m = .
9 59 = 0.7x + 10 ⇒ x = 70
pV ∴At 70° C, the reading of given thermometer be 59°.
17 (a) pV = nRT or n =
RT
23 (c) Let the original temperature be 0°C.
pV
Now, N = n × 6.02 × 10 23 = × 6.02 × 10 23 Volume of A,V1 = l × π (2r )2; V1′ = V1 (1 + γ ∆T )
RT
(V1′ − V1)
(10 −14 × 13.6 × 10 3 × 10) (1) = γ∆T ⇒ V
( 1′ − V1 ) ∝ V1
or N = × 6.02 × 10 23 V1
(8.31) (273 + 30)
(V ′ − V2 )
= 3.2 × 1011 Similarly, 2 = γ∆T ⇒ V
( 2′ − V2 ) ∝ V2
V2
18 (d) As α Al > α steel , aluminium will expand more. So, it should
∆V1 l (2 r )2 2
have larger radius of curvature. Hence, aluminium will be on Q = =
∆V2 2 lr 2 1
convex side and steel on concave side.
Aluminium  nR   mR 
24 (c) V =   T =  T
 p  Mp 
m
∴Slope of V-T graph ∝
p
θ
m
Steel Since, ratio is same in both cases. So, the expansion will be
p
Q
represented by the same straight line B.
19 (b) As the rod is heated, it expands. No external torque is 25 (c) From ideal gas equation, pV = RT …(i)
acting on the system, so angular momentum should be
conserved. or p∆V = R∆T …(ii)
On dividing Eq. (ii) by Eq. (i), we get
∆V ∆T ∆V 1
= ⇒ = =δ (Given)
V T V ∆T T
1
∴ δ=
T
Rod
So, the graph between δ and T will be rectangular hyperbola.
Angular momentum, L = Iω = constant ⇒ I1ω1 = I2ω 2
1 1
Due to expansion of the rod, I2 > I1 26 (d) U ∝ or T ∝ ⇒ T ∝V
ω 2 I1 ρ (1/V )
⇒ = < 1 ⇒ ω 2 < ω1
ω1 I2 Hence, from ideal gas equation (pV = nRT ), p is constant.
So, angular velocity (speed of rotation) decreases. Further, ρ is increasing in U-ρ, thereforeV should decrease.
Thermometry, Thermal Expansion and Kinetic Theory of Gases 681

27 (a) We know for an ideal gas, 32 (b) (n1 + n 2 )i = (n1 + n 2 )f


 nR  p 0V p 0V pV pV
pV = nRT ⇒ V =   T + = +
 p RT0 RT0 RT0 2RT0
where, p = pressure,V = volume, ∴ p = (4/ 3) p 0
n = number of moles of gases
33 (b) It is clear from the graph that lowest point for scale A is
R = gas constant 30° and lowest point for scale B is 0°. Highest point for the
and T = temperature. scale A is 180° and for scale B is 100°. Hence, correct relation is
Slope of the V-T graph, m =
dV nR
= tA − (LFP )A tB − (LFP )B
=
dT p (UFP )A − (LFP )A (UFP )B − (LFP )B
1 where, LFP = lower fixed point
⇒ m∝ (Q nR = constant)
p and UFP = upper fixed point.
1
⇒ p∝ 180
(+A) B
(+B)
m C
p1 m 2

Temperature (°A)
Hence, = >1
p 2 m1 ∆tA = 150°

where, m1 is slope of the graph corresponding to p1 and m 2 is °–θ


90
slope corresponding to p 2. θ
⇒ p 2 < p1 or p1 > p 2 ∆tB = 100° 30°
O Temperature (°B) 100
28 (d) Let the radius of the sphere be R. As
the temperature increases, radius of the tA − 30 t −0 t − 30 t
⇒ = B ⇒ A = B
sphere increases as shown in figure. 180 − 30 100 − 0 150 100
R
4
Original volume,V = πR 3 34 (d) Consider the diagram, when the molecules breaks into
3 dV atoms, the number of moles would become twice.
Coefficient of linear expansion = α Now, from ideal gas equation,
∴ Coefficient of volume expansion = 3α pV = nRT
1 dV Here, p = pressure of gas, n = number of moles, R = gas
∴ = 3 α ⇒ dV = 3VαdT ≈ 4πR 3α∆T
V dT constant, T = temperature and V = volume of the container.
= Increase in the volume
29 (a) Let volume of the sphere beV and ρ be its density, then H2 H2
V H2
we can write buoyant force as molecules

F = V ρ g (g = acceleration due to gravity)


⇒ F ∝ρ (QV and g both are constants)
F4 ° C ρ 4 ° C
⇒ = >1 (Q ρ 4 ° C > ρ 0 ° C)
F0 ° C ρ 0 ° C V H
atoms
Hence, buoyancy will be less in water at 0° C than that in HH
water at 4° C. As, number of moles becomes twice of initial, so n 2 = 2n1.
30 (d) Diameter of brass plate = 2 .6 cm So, p ∝ nT
and diameter of steel plate = (2.6 − 0.01) cm = 2.59 cm ⇒
p 2 n 2T2 (2n1) (3000 )
= = = 20
Now, (d + ∆d )b = (d + ∆d )s p1 n1T1 n1(300 )
or d b (1 + α∆ θ )b = d s (1 + α∆ θ )s ⇒ p 2 = 20 p1
∴ 2.6 (1 + 19 × 10 −6 ∆θ ) = 2.59 (1 + 11 × 10 −6 ∆θ ) Hence, final pressure of the gas would be 20 times the
∆ θ ≈ − 478° C or θ f = − 478°+20 ° = − 458° C pressure initially.

31 (b) γ r = γ a + γ v , where γ r = coefficient of real expansion, 35 (a) From ideal gas equation, pV = µRT
γ a = coefficient of apparent expansion p µR 1
⇒ Slope of p -T curve, = ⇒ Slope ∝
and γ v = coefficient of expansion of vessel. T V V
For copper, γ r = C + 3α Cu = C + 3A It means line of smaller slope represents greater volume of
For silver, γ r = S + 3α Ag gas. In the given problems 1 and 2 are on the same line, so
they will represent same volume, i.e.V1 = V2.
⇒ C + 3A = S + 3α Ag
Similarly, points 3 and 4 are on the same line, so they will
C − S + 3A
⇒ α Ag = represent same volume, i.e.V3 = V4 .
3
682 OBJECTIVE Physics Vol. 1

2
Also, slope of line 1-2 is less than that of line 3-4. l 
⇒ (OR )2 = [l (1 + α 2t )]2 − (1 + α 1t )
Hence, ( 1 = V2 ) > V
V ( 3 = V4 )  2 
36 (d) For an ideal gas, pV = nRT l2 l2
l2 − = l 2 (1 + α 22 t 2 + 2α 2t ) − (1 + α 12 t 2 + 2α 1t )
Here, n = number of moles, p = pressure,V = volume, 4 4
R = gas constant, T = temperature ⇒ n =
pV Neglecting α 2 t and α 1 t , we get
2 2 2 2

RT
l2
As number of moles of the gas remains fixed, hence we can write 0 = l 2 (2α 2 t ) − (2α 1t )
4
pV pV T 
11
= 2 2 ⇒ p 2 = (p1V1)  2  2α 1
RT1 RT2 V2T1 ⇒ 2α 2 =
4
(p ) V . T ) Given, p1 = p, V2 = 1.05 V 
( ) (11 ⇒ α 1 = 4α 2
=
(1.05V )(T ) and T2 = 11
.T 
 43 (b) Change in weight = Upthrust (F )
 1.1  where, F = Vs ρl g
=p×  = p (1.0476) −
~ 1.05 p
 1.05 ⇒ F ′ = Vs′ ρ′l g
Hence, final pressure p 2 lies between p and 1.1 p. F ′ Vs ′ ρ′ l
Q = ⋅
37 (d) Pressure on both sides should be same. F Vs ρl


n1RT n 2RT
= ⇒
m
=
m F′ ρ′
or = (1 + γ s ∆θ ) l
V1 V2 MV
11 MV2 2 F ρl
m m  46 − 30.5  1.20 
⇒ =   = (1 + 3 × α s × 15)  
32 (360 ° − α V
) 28 αV  46 − 30   1.24
32 × 360
⇒ 28 α = 32(360 °−α ) ⇒ α = or α = 192° or α s = 2.3 × 10 −5/ °C
60
38 (d) In an ideal gas, when a molecule collides elastically with a 44 (a) (p 2 − p1) A = mg
wall, the momentum or pressure transferred to each molecule mg RTi RTi 3RTi
will be twice the magnitude of its normal momentum. As face or = − = …(i)
A V1 4V1 4V1
EFGH absorbs the molecule incident on it, so for the face
EFGH, it transfers only half of that on other faces. Similarly, in second case,
m V m T mg RTf RTf 2RTf
39 (c) pV = RT ⇒ V ∝ mT ⇒ 1 = 1 ⋅ 1 = − = …(ii)
M V2 m 2 T2 A V2 3V2 3V2
2 V m 100 m Further, 5V1 = 4V2
⇒ = × ⇒ m2 =
V m 2 200 4 Equating Eqs. (i) and (ii), we get
3Ti 2Tf
40 (a) Slope of line is 1. Therefore, T-V equation can be written =
4V1 3V2
as T = V + T0
9 V2
pV nRT0 ⇒ Tf = × × Ti
∴ = V + T0 ⇒ p = (nR ) + 8 V1
nR V
∴ p versusV graph is a sort of rectangular hyperbola. 9 5
= × × 320
∆p 8 4
41 (a) F = rate of change of momentum = = (nm ) (2v cos θ ) = 450 K
∆t
F (nm ) (2v cos θ ) 45 (c) Consider the diagram, where an ideal gas is contained in a
Pressure, p = =
A A cylinder, having a piston of mass M. Friction is absent.
10 23 × 3.32 × 10 −27 × 2 × 1000 × cos 45°
= M
2 × 10 −4
= 2.34 × 10 3 Nm−2 or Pa pa
2
 l
42 (d) (OR )2 = (PR )2 − (PO )2 = l 2 −   pa Mg/A
 2 p
R
A p
l
The pressure inside the gas will be p = p a + Mg /A
where, p a = atmospheric pressure,
P l/2 O Q
Thermometry, Thermal Expansion and Kinetic Theory of Gases 683

A = area of cross-section of the piston 2 (b) Let L0 be the initial length of each strip before heating.
and Mg = weight of piston.
Hence, p = constant Brass strip
As, pV = nRT
⇒ When temperature increases, volume V ( ) increases at
constant pressure.
Copper strip
46 (b) Volume of the gas V is constant. d

θ
F R

O
pA p0 A
Length after heating will be
Lb = L0 (1 + α b∆T ) = (R + d ) θ … (i)
(Q Length of an arc = Radius × Angle)
∴ p ∝ T , i.e. pressure will be doubled when temperature is
Lc = L0 (1 + α c ∆T ) = Rθ … (ii)
doubled.
On dividing Eq. (i) by Eq. (ii), we get
∴ p = 2p 0
R + d 1 + α b ∆T d
Now, let F be the tension in the wire. Then, equilibrium of ⇒ = ⇒ 1 + = 1 + (α b − α c ) ∆T
R 1 + α c ∆T R
any one piston will give
F = (p − p 0 ) A d
⇒ R=
= (2p 0 − p 0 ) A (α b − α c ) ∆T
= p 0A 1
⇒ R∝
∆T
(B) Medical entrance special format 1
and R∝
questions (α b − α c )
i.e. R is inversely proportional to ∆T and (α b − α c ).
l Assertion and reason
3 (c) An ideal gas is that which satisfy the equation,
1 (d) In summers, a metallic scale will read less than the actual
pV = µRT
because length of metallic scale increases in summers.
where, p = pressure,V = volume,
1 2
2 (b) The translational kinetic energy is mvrms . µ = number of moles, R = gas constant
2
and T = temperature.
Further, translational degree of freedom of any type of gas is
Here, µ and R are constants for a gas, so
three. Therefore, translational kinetic energy of any type of
T
3
gas (of one mole) is RT. p∝
2 V
Hence, pressure of an ideal gas depends on both the
3 (c) In ideal gases, there is no inter-particle interaction. So, no temperature and volume of the gas. Thus, both the statements
vibrational energy is there. are correct.
∴ Only the kinetic energy contribute to the internal energy.
4 (c) Statements II and III are correct but statement I is
4 (b) Translational degree of freedom of a diatomic gas (O 2 ) is incorrect and it can be corrected as,
three and rotational degree of freedom is two. Therefore, The kinetic theory relates the molecular properties like
K 3
according to law of equipartion of energy, T = . viscosity, conduction and diffusion with bulk properties of a
KR 2 gas.
Further, intermolecular force between ideal gas molecules is
zero. Hence, potential energy is zero.
l Match the columns
1 1
1 (b) T 2 ∝ or T 2 ∝ or T ∝ p
l Statement based questions V 1/ p
1 (b) All statements are correct except the statement given in p has become four times. Therefore, T will become two times
option (b) and it can be corrected as, 1
orV will become times.
The average translational kinetic energy per molecule is same 4
for molecules of all gases like here O 2 and for each molecule, Further, T is increasing, hence ∆U will be positive.
3
it is k B T. Hence, A → r, B → s, C → p.
2
684 OBJECTIVE Physics Vol. 1

2 (c) From the givenV -T graph, we cannot tell the nature of 6 (c) Due to change in temperature, the thermal strain produced
gas. in a rod of length L is given by
nRT  nR  ∆L
V= = T = α ∆T ⇒ ∆L = L α ∆T
p  p L
If p is constant,V -T graph is a straight line. Slope of this line where, L = original length of rod and α = coefficient of linear
nR expansion of solid rod.
is .
p As, the change in length (∆l ) of the two given rods of copper
Here, (Slope)A > (Slope)B and aluminium are independent of temperature change, i.e.
 nR   nR  ∆T is same for both copper and aluminium.
⇒   > 
 p A  p B LCuα Cu = LAlα Al … (i)
nA p Here, . × 10 −5 K−1
α Cu = 17
So, either may be greater than 1 or A may be less
nB pB α Al = 2.2 × 10 −5 K−1
than 1. and LCu = 88 cm
n n
But it certain that   >   . Substituting the given values in Eq. (i), we get
 p A  p B α L . × 10 −5 × 88
17
LAl = Cu Cu = −
~ 68 cm
Hence, A → t, B → t, C → t, D → t. α Al 2.2 × 10 −5
7 (a) The kinetic energy of gas molecules is given by
(C) Medical entrances’ gallery KE = RT
3
1 (a) The average thermal energy of a system with degree of 2
freedom f is equal to its average energy, which is given as ⇒ KE ∝ T
f Thus, increase in temperature would lead to increase the
= kB ⋅ T kinetic energy of molecules.
2
For monoatomic gas, f = 3 Option (b) is incorrect as increase in temperature will lead to
3 increase in pressure as p ∝ T . Other options (c) and (d) are
∴Average thermal energy = kB ⋅ T also incorrect as molecular distance increases while mass
2
remains the same for increase in the temperature.
2 (a) Given, pressure, p = 249 kPa = 249 × 10 3 Pa
8 (a) Given, initial pressure of ideal gas,
Temperature, T = 27° C = 273 + 27 = 300 K p1 = 1bar = 1Nm −2
Density, ρ = ?
Initital volume,V1 = 30 m 3
From ideal gas equation, pV = nRT
m  m Final volume,V2 = 10 m 3
⇒ pV = RT Q n = 
M  M Initial temperature, T1 = 320 K
⇒ pVM = mRT Final temperature, T2 = 280 K
m  m  Final pressure of gas, p 2 = ?
⇒ pM = RT = ρRT Q = ρ
V  V  From ideal gas equation,
pV pV
pM 249 × 10 3 × 2 × 10 −3 11
= 22
⇒ ρ= = T1 T2
RT 8.3 × 300
1 × 30 p 2 × 10
(Q For hydrogen gas, M = 2 g = 2 × 10 −3 kg) ⇒ =
320 280
⇒ ρ = 0.2 kg/m 3 ⇒ p 2 = 2 .625 bar
4 (a) Ideal gas equation is given as 9 (a) A gas can be liquified by applying pressure only when it is
ρRT M cooled below critical temperature. Critical temperature of NH3
p= ⇒ p ⋅ 0 = RT is more than CO 2, i.e. TNH3 = 405 K and TCO2 = 304.1K.
M0 ρ
Therefore, NH3 is liquified more easily than CO 2.
 M0 
⇒ pV = RT QV = 
 ρ  10 (b) There are two vibrational degree of freedom of diatomic
gas molecules at high temperature. Hence, vibrational degree
Here, ρ and M0 are mass density and mass of the gas, of freedom of a diatomic gas molecule appears at very high
respectively. temperature because vibration in gas molecules is directly
5 (a) The mean free path l for a gas molecule is given as proportional to the square root of its temperature.
1 1 11 (d) Elongation due to change in temperature,
l= ⇒ l∝ 2
2πnd 2
d ∆l = Lα∆ T ...(i)
where, d = diameter of molecule of gas. where, α = coefficient of linear expansion,
Thermometry, Thermal Expansion and Kinetic Theory of Gases 685

which is compensated by elastic strain. 15 (b) A monoatomic gas molecules like He consists of single
When temperature becomes normal, then atom. It can have translational motion in any direction in
TL space.
∆l = ...(ii)
YS Thus, it has three translational degrees of freedom f = 3. It
From Eqs. (i) and (ii), we get can also rotate but due to its small moment of inertia,
TL rotational kinetic energy is neglected.
= Lα∆T The molecules of a diatomic gas (like O 2 , CO 2 , H2 ) cannot
YS
only move but also rotate about any one of the three
⇒ T = Y Sα ∆ T coordinates.
At equilibrium, force exerted by one-half on other, Hence, it can have two rotational degree of freedom.
F = 2T = 2YSα∆T
Thus, a diatomic molecule has 5 degree of freedom, i.e. 3
12 (b) The minimum velocity with which the body must be translational and 2 rotational.
projected vertically upwards, so that it could escape from the
16 (d) Total internal energy of system
earth’s atmosphere is its escape velocity (v e ).
= Internal energy of oxygen molecules + Internal energy of
As, v e = 2gR argon molecules
Substituting the value of g (9.8 ms −2 ) and radius of earth f f
= 1 n1RT + 2 n 2RT
(R = 6.4 × 10 6 m), we get 2 2
5 3
v e = 2 × 9.8 × 6.4 × 10 6 = × 2RT + × 4RT = 11RT
2 2
≅ 11.2km s −1 = 11.2 × 10 3 m s −1 γ brass 0.00006
17 (a) As, α brass = =
Let the temperature of molecule be T when it attains v e . 3 3
According to the question, v rms = v e = 0.00002 = 2 × 10 − 5 / °C
where, v rms is the rms speed of the oxygen molecule. The brass scale is true at 15°C, therefore at 30°, its
3kBT graduations will increase in length and hence, observed
⇒ = 11.2 × 10 3
mO 2 reading will be less than actual reading at 30°.
∴The change in reading,
. × 10 3 )2 (m O 2 )
(112
or T= ∆l = lα brass (∆T ) = 74. 5 × 2 × 10 − 5 (30 − 15)
(3kB )
= 0. 02235 cm
Substituting the given values, i.e.
kB = 1.38 × 10 −23 JK−1 ∴ Actual reading at 30°C,
and m O 2 = m = 2.76 × 10 −26
kg, l30 = lobserved + ∆l = 74. 5 + 0. 02235 = 74. 522 cm
Assuming area of cross-section to be constant, we have
(11.2 × 10 3 )2 (2.76 × 10 −26 ) ~
we get T= − 8.326 × 10 4 K V0 ρ 0 = V30 ρ 30
(3 × 1.38 × 10 −23 ) or ah0 ρ 0 = ah30 ρ 30
13 (b) Given, γ = 49 × 10 −5 ° C−1, Therefore, true height at 0°C,
ρ h30
∆T = 30 ° C h0 = h30 30 =
ρ0 (1 + γ Hg ∆T )
V ′ = V + ∆V = V (1+ γ ∆T )
∴ V ′ = V (1 + 49 × 10 −5 × 30 ) = 1.0147V =
74.522
=
74.522
= 74.122 cm
m m m 1 + 0.00018 × 30 10054
.
Q ρ= and ρ′ = = = 0.9855 ρ
V V ′ 1.0147 V 18 (b) If we fill nitrogen gas at high pressure above the mercury
Hence, fractional change in density level, the boiling point of mercury will get increased which
ρ − ρ′ ρ − 0.9855 ρ can extend to the range upto 500°C.
= = = 0.0145
ρ ρ 19 (b) According to ideal gas equation, pV = nRT
RT p nR
14 (b) We know that, pV = RT ⇒ V = …(i) ⇒ =
p T V
M mN A p
∴ Density, ρ = = [From Eq. (i)] where, represents slope of the graph.
V RT T
p As, the number of moles are same for the two gases.
p 1
=
mpN A
(Q R = kN A ) ⇒ ∝
kN AT T V
Q V2 > V1 (Given)
pm
⇒ ρ= ∴ (Slope)2 < (Slope)1 or (Slope)1 > (Slope)2
kT
686 OBJECTIVE Physics Vol. 1

20 (c) According to question, In the second condition,


Coefficient of linear expansion of brass = α 1 1

2p = ρ vrms …(ii)
Coefficient of linear expansion of steel = α 2 3
Length of brass and steel rods are l1 and l2, respectively. On dividing Eq. (i) by Eq. (ii), we get
As the increase in length (l′2 − l1′) is same for all temperatures. 2
v rms 1 v 1
So, l′2 − l1′ = l2 − l1 = ⇒ rms =
′2
vrms 2 ′
vrms 2
⇒ l2 (1 + α 2∆t ) − l1 (1 + α 1 ∆t ) = l2 − l1 ⇒ l2α 2 = l1α 1
26 (c) The rms speed is given by
21 (a) It is given that,
(v rms )1 = 200 ms −1, T1 = 300 K, p1 = 10 5 Nm −2 vrms =
3RT
−2 M
T2 = 400 K, p 2 = 0.05 × 10 Nm , (v rms )2 = ?
5

As, rms velocity of gas molecules, T


⇒ vrms ∝
 3RT  M
v rms ∝ T Q v rms = 
 M  When temperature is doubled and molecules dissociate into
atoms, then
For two different cases,
T T
(v rms )1 T 200 300 3
⇒ = 1 ⇒ = = (vrms )1 M = M = T 1
(v rms )2 T2 (v rms )2 400 4 = =
(vrms )2 2T 4T 4T 2
2 400 M /2 M
⇒ (v rms )2 = × 200 = ms −1
3 3
If (vrms )1 is v, then (vrms )2 will be 2v.
22 (a) The rms speed is given by
28 (a) Given, ρ = 0.09 kg m −3
3 RT
vrms = Pressure, p (at STP) = 1.01 × 10 5 Pa
Molecular weight
According to kinetic theory of gas,
3 × 8.3 × (273 + 27) 3 × 8.3 × 300
= = p = (1/ 3) ρv 2
2 × 16 × 10 −3 32 × 10 − 3
3p 3 × 1.01 × 10 5
⇒ vrms = 483 ms −1 ⇒ v= =
ρ 0.09
23 (b) Translational kinetic energy, = 1834.85 ms −1
1
K = Mvrms 2
…(i) Now, volume occupied by one mole of hydrogen at STP
2
= 22.4 L = 22.4 × 10 −3 m3
3pV
and vrms = …(ii) ∴ Mass of hydrogen = Volume × Density
M
= 22.4 × 10 −3 × 0.09
From Eqs. (i) and (ii), we get
1 3pV 3 = 2.016 × 10 −3 kg
K= M = pV
2 M 2 1 2
∴ Average kinetic energy per mol = mv
2K 2
⇒ p= ⇒ p ∝K (If,V = constant) 1
3V = × 2.016 × 10 − 3 × (1834.85)2
2
24 (d) Given, the lengths of rods are L1 and L2 −
~ 3403 J
and coefficients of linear expansions are α 1 and α 2.
29 (a) According to the ideal gas law,
As, ∆L1 = α 1 L1 ∆t and ∆L2 = α 2 L2 ∆t
 R
and for combined rod, ⇒ pV = RT ⇒ V =   T
 p
∆L = α (L1 + L2 ) ∆t
Now, ∆L = ∆L1 + ∆L2 V1 T1
⇒ V ∝T ⇒ =
⇒ α (L1 + L2 ) ∆t = α 1 L1∆t + α 2 L2∆t V2 T2
⇒ α (L1 + L2 ) = α 1 L1 + α 2 L2 ⇒
V2 T
− 1= 2 − 1
α L + α 2 L2 V1 T1
⇒ α= 1 1
L1 + L2 V2 − V1 T2 − T1
⇒ =
V1 T1
25 (c) According to the question,
1 2 V2 − V1 1 1
In the first condition, p = ρ vrms …(i) ⇒ = =
3 V1 T1 T
Thermometry, Thermal Expansion and Kinetic Theory of Gases 687

30 (c) Water is heated from 0°C to 10°C, then its volume first 41 (a) In kinetic theory of gases, it is assumed that time taken in
decreases and then increases. a collision is negligible compared to the average time taken in
free travel between any two collisions, to be precise.
42 (c) The number of molecules (n ) in a volume V
( ) at pressure p
Volume and temperature T is given by
of 1 kg
pV
water n=
kBT
Here, p = 10 6 dyne cm −2,V = 1000 cm3,
4°C Temperature
T = 273 + 27 = 300 K,
31 (a) When cooled from room temperature, liquid water kB = 1.38 × 10 −16 erg per kelvin
becomes dense as compared to other substances, but at
approximately 4° C (39° F), pure water reaches its maximum (10 6 ) (1000 )
∴ n= ≈ 2.4 × 10 22
density. If it is cooled further, it expands to become less dense. 1.38 × 10 −16 × 300

32 (c) Strain = α ⋅ ∆T ⇒ Stress ×Y ∝ ∆T 43 (a) Coefficient of real expansion,


where, L = length of the rod, V −V
γR = 2 1
V1(T2 − T1)
α = coefficient of linear thermal expansion
and Y = Young’s modulus of its material. 3 2
Here, V2 = , V1 =
4 3
So, the longitudinal stress developed in the rod is directly
∆T and ( T2 − T1) = (100 − 20 ) = 80 °C
proportional to .
Y  3 2
 − 
 4 3 1
33 (b) The kinetic theory of gases assume the collision between ∴ γR = = = 15.6 × 10 − 4 ° C−1
2 640
molecules as perfectly elastic. (80 )
3
35 (b) From law of equipartition of energy, for any system in
thermal equilibrium, the total energy is equally distributed 44 (c) pV = RT = constant
among its various degrees of freedom and each degree of RT
freedom is associated with energy (1/ 2) kBT . ⇒ p=
V
36 (b) Here, γr = γ ag + 3α g = γ ac + 3α c Vp 2 = constant
1.03 × 10 −3 + 3 α g = 1.006 × 10 −3 + 3 × (17 × 10 −6 )
2
⇒  RT 
⇒ V   = constant
V 
⇒ α g = 9 × 10 −6 / ° C
R 2T 2
3 ⇒ = constant
38 (c) We know that, kinetic energy, KE = kBT V
2
T2 T 2 T ′2
3
where, kB is constant, so KE ∝ T. ⇒ = constant ⇒ =
2 V V 2V
Now, we can say that the average kinetic energy of a gas ⇒ T ′ 2 = 2T 2 ⇒ T ′ = 2 T
molecule at absolute temperature T is directly proportional
to T. 45 (c) The relation between the temperature scales A and B
is given as
3 RT
39 (a) From gas equation, vrms = A − 42 B − 72
M = , so for the two scales to have the same
110 220
In given condition,
(vrms )T
=
T reading, A = B
(vrms )0 T0 A − 42 A − 72
⇒ =
T 110 220
⇒ 4=
T0 A − 42 A − 72
⇒ =
1 2
⇒ T = 4 T0 = 4 × 273
⇒ 2A − 84 = A − 72
= 1092 K = 819° C
⇒ 2A − A = 84 − 72
40 (a) For an ideal gas, the mean kinetic energy is proportional to
⇒ A = 12°
the absolute temperature. It does not depend on the kind of
gas.
CHAPTER
15

Laws of
Thermodynamics
Thermodynamics is the branch of Physics that deals with the concepts of heat,
work and interconversion of heat and other forms of energy. In this chapter, we
will study the laws of thermodynamics, various thermodynamic processes, basic
theory of heat engines, refrigerators and Carnot engine.

THERMODYNAMIC TERMS Inside


Thermodynamics studies transformation of energy taking place with physical and 1 Thermodynamic terms
chemical changes. In order to understand these transformations, we need to 2 Zeroth law of thermodynamics
understand the terms given below
3 First law of thermodynamics
1. Thermodynamic system Explanation of the terms
used in first law
It is an assembly of an extremely large number of particles (atoms or molecules) Applications of first law of
which is capable of exchange of energy with its surroundings. A thermodynamic thermodynamics
system may contain matter in solid, liquid or gaseous form or a combination of two 4 Heat capacity
or more of these states. Thermodynamic parameters for
Everything outside the system which has a direct effect on the system is called its a mixture of gases
surroundings. All space in universe outside the system is surroundings. e.g. 5 Thermodynamic processes
Environment. Isobaric process
Isochoric process
Isothermal process
Boundary
system Adiabatic process
Cyclic process
Thermodynamic
system 6 Heat engine
Surroundings 7 Refrigerator
8 Second law of thermodynamics
Reversible and irreversible
Fig. 15.1 A thermodynamic system having boundary and surroundings processes
Carnot engine
A system is separated from its surrounding by a boundary. It is a real or Entropy
imaginary two-dimensional closed surface that enclose the volume or region that a
thermodynamic system occupies.
Laws of Thermodynamics 689

A thermodynamic system can be classified by the nature of the 4. Thermodynamic process


transfer of heat that are allowed to occur across its boundary When state of a system changes, then thermodynamic
(i) Open system It exchanges both heat (or energy) variables associated with the system like pressure p,
and matter with its surroundings. volume V and temperature T also changes. This process is
Matter in or out known as thermodynamic process.

Container without 5. Thermal equilibrium


lid and with Two systems are said to be in thermal equilibrium, if there
conducting walls
is no net flow of heat between them when they are brought
into thermal contact.
Heat in or out
Fig. 15.2 Open system 6. Adiabatic and diathermic wall
Adiabatic wall is an insulating wall between two
(ii) Closed system It exchanges only heat (or energy),
thermodynamic systems which does not allow flow of
but no matter with its surroundings.
Container with
energy (or heat) from one system to another system.
lid and Diathermic wall is a conducting wall between two
conducting walls thermodynamic systems which allows energy flow (or
No matter heat) from one system to another system.
in or out

Heat in or out
ZEROTH LAW OF
Fig. 15.3 Closed system THERMODYNAMICS
(iii) Isolated system It exchanges neither heat (or The Zeroth law of thermodynamics states that, if two
energy) nor matter with its surroundings. systems A and B are separately in thermal equilibrium
Fixed insulated lid Container with with a third system C, then A and B are in thermal
insulating walls equilibrium with each other.
Hence, from Zeroth law, if systems A and B are separately
No matter in thermal equilibrium with C, then
in or out
T A = TC and TB = TC ⇒ T A = TB
So, systems A and B will also be in thermal equilibrium
No heat in or out with each other. Now, to understand Zeroth law of
Fig. 15.4 Isolated system thermodynamics, let us consider two systems A and B
2. Thermodynamic or state variables separated by a fixed adiabatic wall.
The quantities like pressure (p), volume (V) and The two systems A and B are in contact with a third
temperature (T) which help us to completely, specify state system C through diathermic wall. The macroscopic
of thermodynamic system are called thermodynamic variables of A and B will vary until both A and B come in
variables. Thermodynamic variables may be extensive or thermal equilibrium with the third system C .
intensive in nature. This shows that two systems A and B are separately in
Extensive variables are those which depends on the size thermal equilibrium with a third system C.
of the system and intensive variables are those which do This condition is shown in Fig. (a).
not depend on the size of the system. e.g. Internal energy
(U), volume (V) and total mass (M) are extensive variables C C
and pressure (p), temperature (T) and density (ρ) are
intensive variables. Adiabatic
wall
A B A B
3. Equation of state Diathermic
wall
The equation which represents the relationship between
Adiabatic wall Diathermic wall
the state variables of a system is called its equation of state.
(a) (b)
e.g. For an ideal gas, the equation of state is pV = µRT .
Fig. 15.5
For a fixed amount of gas given, there are only two
independent variables, say p and V or T and V.
690 OBJECTIVE Physics Vol. 1

Let the adiabatic wall between A and B is replaced by a


diathermic wall and an adiabatic wall insulates C from A Explanation of the terms used in
and B. Now, A and B are found in thermal equilibrium first law
with each other. This condition is shown in Fig. (b). This First law of thermodynamics basically revolves around the
experiment proves the Zeroth law of thermodynamics. three terms ∆Q, ∆U and ∆W. If you substitute these three
terms correctly with proper signs in the equation
FIRST LAW OF ∆Q = ∆U + ∆W, then you are able to solve most of the
THERMODYNAMICS problems of first law.
Let us understand each term one by one, for an ideal gas
First law of thermodynamics is a statement of conservation system
of energy applied to any system in which energy transfer
from or to the surroundings is taken into account. 1. Work done by a thermodynamic system
It states that, if some quantity of heat is supplied to a By work, we mean work done by the system or on the
system capable of doing external work, then the quantity system. Suppose a gas is confined in a cylinder with a
of heat absorbed by the system is equal to the sum of the movable piston, if p is pressure on the piston and A is area
increase in the internal energy of the system and the of piston, then force exerted by the gas on the piston of
external work done by the system. cylinder, F = pA.
i.e. Heat supplied, ∆Q = ∆U + ∆W …(i)
Piston
where, ∆Q = heat supplied to the system by the
surroundings. It is a path dependent function. dx
∆W = Work done by the system on the surroundings. It is
a path dependent function.
F Gas
∆U = Change in internal energy of the system. It depends
only on the initial and final states.
Fig. 15.6 Work done during expansion
Often the first law is used in its differential form, which is
When the piston is pushed outward by the gas through an
dU = dQ − dW
infinitesimal distance dx, then the work done by the gas
This can also be written as dW = Fdx = pA dx = pdV
dQ = dU + dW and for a finite volume change fromVi toVf ,
Vf
Following sign conventions are used in the measurement Work done,W = ∫Vi
dW
of heat, work and internal energy Vf
Thermodynamic Sign Conventions Work done,W = ∫Vi
pdV

Process Convention Here, p is constant.


From the equation of state, pV = nRT
Heat added to the system ∆Q > 0, i.e. positive
∴ p∆V = nR∆T
Heat removed from the system ∆Q < 0, i.e. negative
Work done,W = p ∆V = nR∆T
Work done by the system (volume increases) ∆W > 0, i.e. positive
Tf
Work done on the system (volume decreases) ∆W < 0, i.e. negative
or Work done,W = nR ∫ ∆T = nR (T f − Ti )
Ti

When temperature of the system rises ∆U > 0, i.e. positive When variation of pressure with the volume is known to
When temperature of the system falls ∆U < 0, i.e. negative
us, then the area bounded by p-V graph (indicator diagram)
represents the work done.
● In the Eq. (i), all three quantities ∆Q, ∆U and ∆W must be Vf
expressed in same unit either in joule or in calorie. Work = ∫V
i
pdV = Area under p-V graph
Laws of Thermodynamics 691

● Work done is a path dependent quantity. Example 15.3 Consider the process on a system shown in
So, work done is not a state function. figure below. During the process, predict the nature of work
● If volume is constant in a thermodynamic process, then done by the system.
work done is zero.

Pressure
● If volume increases, then work done is positive otherwise 1 2
negative.
● For a cyclic process, work done is positive, if cycle is
clockwise and negative, if cycle is anti-clockwise.
Example 15.1 Two moles of an ideal gas are in a rubber Volume
balloon at 30°C in equilibrium at 1atm pressure. The balloon
is fully expanded and can be assumed to require no energy Sol. We know that, the area under the p-V diagram is equal to
in its expansion. The temperature of the balloon increases the work done. So, if we move from 1 to 2, the shaded area
slowly to 35°C. What is the work done of expansion of (ABCD ) under the graph is continuously increasing, which
balloon? (Take, R = 8.31 J mol −1K −1). means the work done by the system continuously increases.
Sol. If the balloon is fully expanded (i.e. does not develop
significant tension as it expands), the pressure inside the

Pressure (p)
balloon is always nearly equal to the pressure outside the
balloon. And the pressure outside the balloon is 1 atm 1 2
B
A
(constant). So, the pressure inside the balloon is constant at
about 1 atm. According to the question, the limits of
temperature are 30°C and 35°C.
∴ T1 = 30° C = 30 + 273 = 303 K D C
Shaded area Volume (V)
T2 = 35° C = 35 + 273 = 308 K
We know that, when the limits of temperature are given and
pressure is constant, then Example 15.4 An ideal gas is taken around the cycle ABCA
as shown in the p-V diagram.
Work done, W = nR∆T = nR (T2 − T1) = 2 × 8.31 [308 − 303]
(Q Number of moles of the gas, n = 2) p
= 2 × 8.31 × (5) = 83.1 J 4p B

Example 15.2 (i) 2 mol of an ideal gas is heated at constant


pressure, so that its temperature increases from 27°C to p
C
127°C. Find work done by the gas. A
(ii) For the same gas in a process, if its volume increases from V 3V
V
1000 cc to 2000 cc at constant pressure 100 kPa. Find work
done by the gas.
Evaluate the total work done by the gas during the cycle.
Sol. Given, pressure, p = 100 kPa = 100 × 103 Pa
Sol. Work done = Area enclosed by triangle under the
Temperature T1 = 27° C, T2 = 127° C p-V diagram
Volume,V1 = 1000 cc,V2 = 2000 cc 1
= AB × AC
According to the question, pressure is constant. 2
∴ ∆W = p∆V = nR∆T Here, side AB of the triangle is parallel to pressure (p ) axis,
(i) Work done by the gas, therefore
∆W = nR∆T = nR (T2 − T1) AB = Change in pressure, ∆p = pB − p A
= 2 × 8.3 × (127 − 27) = 4p − p = 3p
(Q Number of moles of the gas, n = 2) Similarly, side AC of the triangle is parallel to volume V
( ) axis,
= 1660 J therefore
(ii) Work done by the gas, AC = Change in volume, ∆V = VC − VA
∆W = p∆V = pV ( 2 − V1) = 3V − V = 2V
−6 1
= 100 × 10 (2000 − 1000) × 10
3
∴ Work done, W = 3p × 2V = 3pV
2
= 100 J
692 OBJECTIVE Physics Vol. 1

Example 15.5 The figure shows a p-V graph of the Sol. As we know that, area of a circle of radius R is πR 2.
thermodynamic behaviour of an ideal gas. Find out from this p
graph (i) work done by the gas in the process A → B, B → C, B
F (p1)
C → D and D → A, (ii) work done by the gas in complete cycle R1
A → B → C → D → A. A C
G R2
E (p2)
14
A B V
12 O K (V1) D H (V2)
p(105 Nm–2)

10
8 From the diagram, it is clear that,
6 Vertical radius of the circle,
4 R1 = GB
D
2 C EF OF − OE p1 − p 2
A′ B′ = AF = = =
1.0 2.0 3.0 4.0 5.0 6.0 7.0 2 2 2
V (Litre) Horizontal radius of the circle,
R 2 = GC
Sol. (i) The work done in a thermodynamic process is equal to KH OH − OK V2 − V1
the area enclosed between the p-V curve and the = DH = = =
2 2 2
volume axis.
Work done by the gas in the process A → B is We know that, the area of an ellipse is πR1R 2, where R1 and R 2
are semi-major and semi-minor axis respectively as shown in
W1 = area of ABB ′ A′ = AB × A′ A figure.
= ( 7 − 1) litre × (12 × 105 ) Nm−2
2R1

= 6 × 10−3 m3 × (12 × 105 ) Nm−2 2R2


= 7200 N-m
= 7200 J
If R1 = R 2, the ellipse becomes a circle as shown in figure.
Work done in the process B → C is zero, since volume p
remains constant.
p1
Work done by the gas in the process C → D is
W2 = area of DCB ′ A′
= DC × A′ D
p2
= (− 6 × 10−3 ) × (2 × 105 )
V
= − 1200 J V1 V2
(Negative sign is taken because volume decreases)
So, here the work done = area of the circle
Work done in the process D → A is also zero, because
= area of shaded portion
volume remains constant.
 p − p 2  V2 − V1
(ii) WorkW1 is positive, whileW2 is negative. Hence, the = πR1R 2 = π  1  
net work done in the whole cycle by the gas is  2  2 
W = W1 + W2 The positive sign is due to the fact that the cycle is clockwise.
= 7200 − 1200 Example 15.7 Calculate the work done by a gas, when it
= 6000 J is taken from 1 to 2, 2 to 3 and 3 to 1 as shown in the
figure. Also, calculate the work done in cyclic process
Example 15.6 Find the work done during the perfectly 1 → 2 → 3 → 1.
circular cyclic process as shown in the figure.
p 3p0 3

p1
p p0 2
1

p2
V0 2V0
V V
V1 V2
Laws of Thermodynamics 693

Sol. The work done in a process is the area below the process Internal energy of an ideal gas
in p-V diagram.
In an ideal gas, there are no intermolecular forces, hence
Process 1 → 2 the gas does not possess intermolecular potential energy,
thus UP = 0.
p0 1 2
p So, internal energy of an ideal gas is just the sum of kinetic
energies associated with various random motions
(i.e. rotational, translational and vibrational) of its molecules.
Therefore, internal energy of an ideal gas depends only on
V0 2V0
3
V its temperature and is given by U = UK = nRT
2
∆W1→ 2 = p 0 (2V0 − V0 ) = p 0V0
3
In the process 2 → 3, volume remains same, ∆W2 → 3 = 0 ∴ Change in internal energy, ∆U = nR∆T
Process 3 → 1 2
f nR
Note For any gas, ∆U = nC V ∆T = n  R∆T = ∆T
2 γ −1
3p0 3
Cp
Here, f = degree of freedom, γ =
CV
p p0 1
Change in internal energy is independent of path followed
by a thermodynamic system.
V0 2V0 ∴ Change in internal energy, ∆U = U f − Ui
V

∆W3 → 1 = area below the process 3 → 1 Example 15.8 1 kg of water at 373 K is converted into steam
1 at same temperature. Volume of 1 cm 3 of water becomes
= − (p 0 + 3p 0 )(2V0 − V0 ) = − 2p 0V0 1671 cm 3 on boiling. What is the change in the internal
2
energy of the system, if the latent heat of vaporisation of
Work done is negative because volume is decreasing. water is 5.4 × 10 5 cal kg −1?
Work done in cyclic process,
Sol. Volume of 1 kg of water = 1000 cm3 = 10−3 m3
∆Wcyclic = ∆W1→ 2 → 3 → 1
Volume of 1 kg of steam = 103 × 1671 cm3 = 1.671 m3
= ∆W1→ 2 + ∆W2 → 3 + ∆W3 → 1
Change in volume, ∆V = (1.671 − 10−3 ) m3 = 1.670 m3
= p 0V0 + 0 − 2p 0V0 = − p 0V0
Pressure, p = 1 atm = 1.01 × 105 Nm−2
2. Internal energy of a In expansion, work done, ∆W = p∆V = 1.01 × 105 × 1.670 J
thermodynamic system 1.686 × 105
= cal = 4.015 × 104 cal
Internal energy of a system is defined as the total energy 4.2
possessed by the system due to molecular motion and But ∆U = ∆Q − ∆W (From first law of thermodynamics)
molecular configuration. or∆U = (5.4 × 105 − 0.4015 × 105 ) cal = 4.9985 × 105 cal
It is represented by U. The energy due to molecular Example 15.9 A certain amount of an ideal gas passes from
motion is called internal kinetic energy UK . state A to state B first by means of process 1, then by means
The molecular motion includes translation, rotation and of process 2. In which of the process is the amount of heat
vibrational motion of the molecules. absorbed by the gas greater?
p
The energy due to molecular configuration is called internal
potential energy UP . 1

Then, internal energy, U = UK + UP A B


● Internal energy is sum of the kinetic energies and potential 2
energies of all the constituent particles of the system. V
● Internal energy is a state variable which does not depend
on the path taken to arrive at that state. Sol. From the first law of thermodynamics,
∆Q1 = ∆W1 + ∆U1 and ∆Q 2 = ∆W2 + ∆U 2
694 OBJECTIVE Physics Vol. 1

U is a state function. Hence, ∆U depends only on the initial From first law of thermodynamics,
and final positions. Therefore, ∆Q = ∆U + ∆W ⇒ 10 = ∆U + 4
p p
1 ∴ Increase in internal energy, ∆U = 6 J

A B A 2 B Example 15.12 The pressure in monoatomic gas increases


linearly from 4 × 10 5 Nm −2 to 8 × 10 5 Nm −2 , when its
W1
W2 volume increases from 0.2 m 3 to 0.5 m 3 . Calculate
p
V V
8 B
∆U1 = ∆U 2
But ∆W1 > ∆W2

(105 Nm–2)
4 C
As the area under process 1 is greater than area under process 2. A
Hence, ∆Q1 > ∆Q 2
E D
Example 15.10 When a system goes from state A to state B, 0 V (m3)
0.2 0.5
it is supplied with 400 J of heat and it does 100 J of work.
(i) For this transition, what is the system’s change in (i) work done by the gas.
internal energy? (ii) increase in the internal energy.
(ii) If the system moves from B to A, what is the change in (iii) amount of heat supplied.
internal energy?
Sol. Given, p1 = 4 × 105 Nm−2, p 2 = 8 × 105 Nm−2, V1 = 0.2 m3,
(iii) If in moving from A to B along a different path in which
W AB′ = 400 J of work is done on the system, how much V2 = 0.5 m3
heat does it absorb? (i) Work done by the gas, ∆W = Area under p-V graph
Sol. (i) From the first law of thermodynamics, (Area ABCDEA)
∆UAB = QAB − WAB = (400 − 100) J = 300 J 1
= (AE + BD ) × AC
(ii) Consider a closed path that passes through the state A 2
and state B. Internal energy is a state function, so ∆U is 1
= (4 × 105 + 8 × 105 ) × (0.5 − 0.2)
zero for a closed path. 2
Thus, ∆U = ∆UAB + ∆UBA = 0 1
= × 12 × 105 × 0.3 = 1.8 × 105 J
or ∆U BA = − ∆UAB = − 300 J 2
(iii) The change in internal energy is the same for any path, so (ii) Increase in the internal energy,
∆UAB = ∆ U AB′ = Q AB ′ − W AB ′ C C
′ − (− 400 J)
300 J = Q AB ∆U = C V (T2 − T1) = V R (T2 − T1) = V (p 2V2 − pV1 1)
R R
and the heat exchanged is, Q ′AB = − 100 J 3
For monoatomic gas, C V = R
The negative sign indicates that the system loses heat in this 2
transition. 3
∴ ∆U = [(8 × 10 × 0.5) − (4 × 105 × 0.2)]
5

Example 15.11 A gas is enclosed in a cylindrical vessel 2


fitted with a frictionless piston. The gas is slowly heated for 3
= [4 × 105 − 0.8 × 105] = 4.8 × 105 J
sometime. During the process, 10 J of heat is supplied and 2
the piston is found to move out 10 cm. Find the increase in (iii) Amount of heat supplied,
the internal energy of the gas. The area of cross-section of ∆Q = ∆U + ∆W = 4.8 × 105 + 1.8 × 105 = 6.6 × 105 J
the cylinder = 4 cm 2 and the atmospheric pressure
= 100 kPa. Example 15.13 A thermodynamical p
Sol. In given process, piston moves against atmospheric process is shown in figure with B C
−2
process, so p A = 3 × 10 Nm ,
4

VA = 2 × 10 −3 m 3 , A
−2
p0 x = 10 cm pB = 8 × 10 Nm ,
4

VC = 5 × 10 −3 m 3 . In the processes V
AB and BC, 600 J and 200 J of
p
heat is added to the system, respectively. Find the change in
internal energy of the system in process AC.
DQ
Sol. Since, internal energy is a state function, so the change in
Work done by the gas = ∆W = p∆V = p 0Ax internal energy during the process AC will be same as that
= 100 × 103 × 4 × 10−4 × 0.1 = 4 J during the total processes AB plus BC. Calculate the total
Laws of Thermodynamics 695

work ∆W during AB plus BC and total heat ∆Q during these (iii) Then, the change in internal energy,
processes. Then, calculate ∆U = ∆Q − ∆W . 3 3
∆U = U 2 − U1 = R (T2 − T1) = RT1( 2 − 1)
During the process AB, the volume does not change, so work 2 2
done is zero. Work done by the gas,
During BC, the pressure is constant, so total work done
∆W = 2A V1 ( 2 − 1) = 2RT1( 2 − 1)
= pB (VC − VB ) = pB (VC − VA ) (Q VB = VA )
= 8 × 104 Nm−2 (5 × 10−3 m3 − 2 × 10−3 m3 ) Example 15.15 When a system is taken from state a to state
b (in figure) along the path a → c → b, 60 J of heat flow into
= 240 J
the system and 30 J of work is done.
∴ ∆W during process AB plus BC = 240 J p
Now, given during the same whole process, c b
∆Q = 600 + 200 = 800 J
∴ ∆U = ∆Q − ∆W = 800 − 240 = 560 J
a d
Example 15.14 Consider a p-V diagram in which the path V
followed by one mole of perfect gas in a cylindrical container
as shown in figure. (i) How much heat flows into the system along the path
a → d → b, if the work done is 10 J ?
p
1( p1 , V1 ,T1 )
(ii) When the system is returned from b to a along the curved path,
the work done on the system is 20 J. Does the system absorb or
pV 1/2= constant liberate heat and how much?
Sol. For the path a → c → b,
2( p2 , V2 ,T2 )
∆U = ∆Q − ∆W = 60 − 30 = 30 J or U b − U a = 30 J
(i) Along the path a → d → b,
V1 V2
V ∆Q = ∆U + ∆W = 30 + 10 = 40 J
(ii) Along the curved path b → a,
(i) Find the work done when the gas is taken from state 1 to state 2. ∆Q = (U a − U b ) + ∆W = (−30) + (−20) = − 50 J, heat
(ii) What is the ratio of temperatures T1 /T2, if V2 = 2V1? flows out of the system.
(iii) Given that internal energy for one mole of gas at temperature T Example 15.16 In the previous example, if U a = 0 and
is (3/2) RT, find the change in internal energy and work done U d = 22 J , then find the heat absorbed in the process a → d
by the gas when it is taken from states 1 to 2, withV2 = 2V1. and d → b.
Sol. (i) It is given that Sol. For the path a → d → b
1
Wadb = Wa→ d + Wd→ b = Wa→ d + 0
pV 2 = constant
⇒ Wa→ d = Wadb = 10 J
Let this constant is A,
Now, applying first law of thermodynamics for the path a → d ,
∴ pV 1/ 2 = A
∆Q a→ d = ∆U a→ d + ∆Wa→ d
A A = U d − U a + 10 = 22 − 0 + 10 = 32 J
⇒ p= 1/ 2
=
V V Similarly, for the path d → b,
Also, we can write
∆Q d→ b = ∆U d→ b + ∆Wd→ b = U b − U d + 0 = U b − U d
1/ 2 1/ 2
pV
11 = p 2V2 =A Given, U b − U a = 30 J ⇒ U b − 0 = 30 ⇒ U b = 30 J
∴ Work done by the gas, ∴ Q d→ b = 30 − 22 = 8 J
V
V2 V2 dV  V 2
∆W = ∫V
1
pdV = A∫
V1 V
=A 
1/2 V1
Example 15.17 50 cal of heat should be supplied to take a
system from the state A to the state B through the path ACB
as shown in figure. Find the quantity of heat to be supplied
= 2A ( V2 − V1 ) to take it from A to B via ADB.
p
= 2pV ( 2 − V11/ 2 )
1/ 2 1/ 2
11 V
155 kPa D B
A
(ii) Since, T = pV /nR = ⋅ V
nR 50 kPa A C
T2 V2
Thus, = = 2 V
T1 V1 100 cc 300 cc
696 OBJECTIVE Physics Vol. 1

Sol. In process ACB, Therefore, work done by external force on the ice,
∆WACB = ∆WAC + ∆WCB W = − p × ∆V = (1.01 × 105 Nm−2 ) × (−182 × 10−6 m3 )
= 50 × 103 × (300 − 100) × 10−6 + 0 = −18.4 J
10 Because work is done on the ice, therefore it is negative.
= 10 J = ≅ 2.4 cal
4.2 According to first law of thermodynamics, change in internal
Given, ∆Q = 50 cal energy of ice,
From the first law of thermodynamics, ∆U = Q − W = 6.8 × 105 J − (−18.4 J)
∆Q = ∆U + ∆W = U B − U A + ∆W = (6.8 × 105 + 18.4) J ≈ 6.8 × 105 J
⇒ 50 = (U B − U A ) + 2.4 ∆U is positive. Therefore, internal energy of ice increases
⇒ U B − U A = 47.6 cal when it melts.
In process ADB, Boiling process
∆WADB = ∆WAD + ∆WDB
When a liquid is heated, it changes into vapour at constant
= 0 + 155 × 103 × (300 − 100) × 10−6 temperature (called boiling point) and pressure. Let us
31 consider the vaporisation of liquid of mass m. LetVl and
= 31 J = ≅ 7.4 cal
4.2 Vv be the volumes of liquid and vapours, respectively.
∴ Quantity of heat supplied, The work done in expanding at constant temperature and
∆Q = ∆U + ∆W = U B − U A + 7.4 pressure p,
= 47.6 + 7.4 = 55 cal ∆W = p∆V = pV ( v − Vl )
Let the latent heat of vaporisation = L
Applications of first law of ∴ Heat absorbed during boiling process, ∆Q = mL
Let Ul and Uv be the internal energies of the liquid and
thermodynamics vapours respectively, then ∆U = Uv − Ul
Applications of first law of thermodynamics are given below According to the first law of thermodynamics,
Melting process ∆Q = ∆U + ∆W
When a substance is heated and it melts, the change in ∴ mL = (Uv − Ul ) + p (Vv − Vl )
volume (dV) is very small and can, therefore be neglected.
The temperature of the substance remains unchanged Example 15.19 When water is boiled at 2 atm pressure, then
latent heat of vaporisation is 2.2 × 10 6 Jkg −1 and boiling
during melting process.
point is 120°C. At this temperature, volume of 1 kg of water
Heat absorbed during melting process, ∆Q = mL is 10 −3 m 3 and that of 1 kg vapours is 0.824 m 3 . When
where, L is latent heat of fusion. 1 kg water is converted to the vapour at 120°C temperature,
then find the work done and increase in internal energy.
By the first law of thermodynamics, (1 atm pressure = 1.013 × 10 5 Nm −2 )
∆Q = ∆U + ∆W Sol. Heat required to convert water into vapour at 120°C is
⇒ mL = ∆U (Q ∆W = p∆V = p × 0 = 0) ∆Q = m × L = 1 kg × 2.2 × 106 Jkg −1 = 2.2 × 106 J
So, the internal energy increases by mL during the melting Change in volume of water = (0.824 − 0.001) m3 = 0.823 m3
process. Therefore, work done against atmospheric pressure,
∆W = p∆V
Example 15.18 At normal pressure and 0°C temperature
when 2 kg of ice melts, its volume decreases by 182 cm 3 . = (2 × 1.013 × 105 Nm−2 ) × (0.823 m3 )
Latent heat of ice for melting is 3.4 × 10 5 J kg −1. Find out by = 166739.8 N-m = 1.67 × 105 J
what value, internal energy of the ice changes when it melts at From first law of thermodynamics, ∆Q = ∆W + ∆U
normal pressure and 0°C temperature. (Normal pressure
or ∆U = ∆Q − ∆W = 2.2 × 106 J − 166739.8 J
= 1.01 × 10 5 Nm −2 )
Sol. Heat required to melt 2 kg of ice, Q = mL = 2033260.2 = 2.03 × 106 J
= 2 kg × (3.4 × 105 ) J kg −1 = 6.8 × 105 J ∴ Required work = 1.67 × 105 J
and change in internal energy = 2.03 × 106 J
Decrease in volume of ice = ∆V = 182 cm3
CHECK POINT 15.1
1. First law of thermodynamics corresponds to 7. Work done by the gas in the process as shown in figure, is
(a) conservation of energy p
(b) conservation of momentum
B
(c) law of conservation of angular momentum
(d) conservation of mass
2. In a process for an ideal gas, ∆W = 0 and ∆Q < 0. Then, for C A
the gas,
(a) the temperature will decrease D
(b) the volume will increase V
(c) the pressure will remain constant (a) positive (b) negative
(d) the temperature will increase (c) zero (d) may be positive or negative
3. A thermodynamic system is taken from state (p1 , V1) to 8. In thermodynamic process, 200 J of heat is given to a gas
(p 2 , V2) by two different process. The quantity which will and 100 J of work is also done on it. The change in internal
remain same will be energy of the gas is
(a) ∆Q (b) ∆W (a) 100 J (b) 300 J (c) 419 J (d) 24 J
(c) ∆Q + ∆W (d) ∆Q − ∆W 9. Corresponding to the process ABCA as shown in figure,
4. An ideal monoatomic gas is taken around the cycle as what is the heat given to the gas?
shown in p-V diagram. The work done during the cycle is B C
4
given by 3

p (Nm–2)
p 2
A
2p0 1

V (m3)
0 1 2
p0
3 1
(a) 1 J (b) J (c) J (d) 0
2 2
10. An ideal gas is taken through the cycle A → B → C → A as
0 V shown in the figure. If the net heat supplied to the gas in
V0 3V0
the cycle is 5 J, the work done on the gas in the process
1 C → A is
(a) p0 V0 (b) p0 V0
2
3
(c) 2p0 V0 (d) None of these V (m )
5. A thermodynamic system is taken through the cyclic 2 C B
process PQRSP. The net work done by the system is
p
S R 1 A
200 kPa
0
p (Nm– )
10 2

100 kPa
(a) −5J (b) −10 J (c) −15J (d) −20 J
Q
P p
11. The p-V diagram of a system
A
undergoing thermodynamic
V
100 cc 300 cc transformation is shown in figure.
The work done on the system in
(a) 20 J (b) −20 J (c) 400 J (d) −400 J going from A → B → C is 50 J and 20
cal heat is given to the system. The B C
6. An ideal gas is taken around ABCA as shown in the p-V
diagram. The work done during the cycle is change in internal energy between A V
p
and C is
B (3p, 3V) (a) 34 J (b) 70 J
(c) 84 J (d) 134 J
12. A closed system undergoes a change of state by process
1→ 2 for which Q12 = 10 J and W12 = − 5 J. The system is now
A C returned to its initial state by a different path 2 → 1 for
(p,V) (p, 3V) which Q21 is −3 J. The work done by the gas in the process
V 2 → 1 is
(a) 8 J (b) zero
pV (c) −2J (d) + 5 J
(a) 2pV (b) pV (c) (d) zero
2
698 OBJECTIVE Physics Vol. 1

HEAT CAPACITY
If an amount of heat ∆Q is needed to change the (ii) Molar heat capacity at constant volume
temperature of a body by ∆T, then heat capacity of the
material of the body is given by  ∆Q 
CV =  
∆Q  n ∆T  constant volume
Heat capacity, S =
∆T
Note In case of solids and liquids, volume expansion is small
Heat capacity is equal to the heat energy required to change (i. e. ∆W = 0), therefore we define only one specific heat for
the temperature of a body by unity. Its unit is JK −1. solids and liquids.

Note Heat capacity of a body is not constant over a large temperature


range. Different amounts of heat may be needed for a unit rise in Relation between C p and CV
temperature at different temperature values. For an ideal gas, the relation between C p and C V is

Specific heat capacity C p − CV = R


The amount of heat required to change the temperature of Here, C p and C V are molar specific heat capacities of an
unit mass of a substance by unity is known as specific heat ideal gas at constant pressure and volume respectively and
capacity of the substance. R is the universal gas constant. This relation is known as
S 1 ∆Q Mayer’s formula.
Specific heat capacity, s = = The ratio of C p and C V is notified by γ.
m m ∆T
1 ∆Q γ = C p /C V
Specific heat capacity, s =
m ∆T It is known as heat capacity ratio.
It depends on the nature of substance and its temperature. From the above two relation between C p and C V , we get
Its unit is J/kg-K. R
CV =
γ −1
Molar specific heat capacity
γR
The amount of heat required to raise the temperature of and Cp =
one gram mole of a substance by unity is called molar γ −1
specific heat capacity of the substance. The heat capacity ratio in terms of degrees of freedom is
2
In other words, heat capacity per mole is known as molar γ = 1+
specific heat capacity or molar heat capacity. f
S 1  ∆Q  where, f = degrees of freedom.
C = =   Values of f, C V , C p and γ for different types of gases are
n n  ∆T 
given below
1 ∆Q Atomicity of gas f CV Cp γ
Molar specific heat capacity, C =
n ∆T Monoatomic 3
3
R
5
R
5
= 1.67
2 2 3
where, n = number of moles. It depends on the nature of Diatomic, triatomic and 5 7 7
R R = 1.4
the substance, its temperature and the conditions under triatomic linear (at normal 5 2 2 5
which heat is supplied. Its unit is J/mol-K. temperature)
On the basis of condition under which the heat is supplied Polyatomic triangular 6 8 4
6 R = 3R R = 4R = 1.33
to a gas, molar specific heat capacity is of two types (Non-linear) 2 2 3

(i) Molar heat capacity at constant pressure For n moles of a gas, internal energy at temperature T is
 ∆Q  nfRT
Cp =   U= or U = nC VT
 n ∆T  constant pressure 2
and change in internal energy, ∆U = nC V ∆T .
Laws of Thermodynamics 699

Example 15.20 5 mol of oxygen is heated at constant volume Change in internal energy,
from 10°C to 20°C. What will be change in the internal ∆U = nC V ∆T
energy of the gas? (The gram molecular specific heat of ⇒ ∆U = 0.2 × 5 × (337 − 37)
oxygen at constant pressure is C p = 8 cal mol −1°C −1 and
(Q C V = C p − R = 7 − 2 = 5 cal /mol ° C)
R = 8.36 J/mol°C) = 0.2 × 5 × 300 = 300 cal
Sol. According to the Mayer’s formula, C V = C p − R Work done by the gas,
= 8 − 2 = 6 cal/mol ° C ∆W = ∆Q − ∆U = 420 − 300 = 120 cal
(Q R = 8.36 J/mol ° C −~ 2 cal/mol ° C ) = 120 × 4.2 = 504 J
∴ Heat absorbed by 5 mol of oxygen at constant volume, Example 15.23 An ideal gas has a specific heat or molar
Q = nC V ∆T = 5 × 6 (20 − 10) = 30 × 10 = 300 cal heat capacity at constant pressure C p = 5R /2. The gas is
At constant volume, ∆V = 0 kept in a closed vessel of volume 0.0083 m 3 at a
∴ ∆W = 0 temperature of 300 K and a pressure of 1.6 × 10 6 Nm −2 . An
From first law of thermodynamics, Q = ∆U + ∆W amount of 2.49 × 10 4 J heat energy is supplied to the gas.
⇒ 300 = ∆U + 0 ⇒ ∆U = 300 cal
Calculate the final temperature and pressure of the gas.
Sol. Given, C p = 5R /2, ∆V = 0, T1 = 300 K,
Example 15.21 At normal temperature and pressure (NTP),
density of air is 1.293 kg m −3 and specific heat at constant V = 0.0083 m3, p1 = 1.6 × 106 Nm−2 and Q = 2.49 × 104 J
volume is 169 cal kg −1K −1. Find the value of specific heat at According to the Mayer’s formula,
constant pressure. 5R 3R
CV = C p − R = −R =
Mass Mass 2 2
Sol. Density = or Volume = From first law of thermodynamics, Q = ∆U + p∆V
Volume Density
1 ⇒ ∆U = Q = 2.49 × 104 J
Volume of 1 kg of air,V = m3
1.293 pV 1.6 × 106 × 0.0083 16
From gas equation, n = 1
= =
Normal pressure, i.e. 76 cm mercury column pressure, RT1 8.3 × 300 3
p = hρg = 76 × 10−2 × 13.6 × 103 × 9.8 Nm−2 As, ∆U = nC V ∆T
Normal temperature, T = 273 K ∆U 2.49 × 104 × 6
⇒ ∆T = = = 375 K
For 1 kg air, pV = RT nC V 3 × 8.3 × 16
 1  Final temperature = 300 + 375 = 675 K
(76 × 10−2 × 13.6 × 103 × 9.8)  
pV 1.293 p 2 T2
or R= = As volume remains constant, p ∝ T ⇒ =
T 273 p1 T1
76 × 136 × 98 × 10−1 1.6 × 106 × 675
= = 286.96 Jkg −1 K −1 ⇒ p2 =
T2
× p1 =
273 × 1.293 T1 300
From Mayer’s formula,
= 3.6 × 106 Nm−2
R
C p = CV + (where, J = 4.2 J cal−1)
J Example 15.24 Find molar specific heat for the process
286.96 a  3 
= 169 + = 169 + 68.32 p= for a monoatomic gas. a = constant, C V = R .
4.2 T  2 
= 237.32 cal kg −1 K −1 Sol. From first law of thermodynamics,
∆Q = ∆W + ∆U
Example 15.22 Find the work done in increasing
∆Q ∆W ∆U
temperature of 0.2 mol nitrogen at constant pressure from ⇒ C = = +
37°C to 337°C. If C p = 7 cal / mol ° C . In this process, what ∆T ∆T ∆T
is the work done by the gas? ∆W
or C= + CV (Q ∆U = C V ∆T )
Sol. Heat given to increase temperature of n mole of a gas by dT, ∆T
∆Q = nC pdT p ∆V
= + CV
= 0.2 × 7(337 − 37) ∆T
= 0.2 × 7 × 300 = 420 cal From gas equation,
Equivalent work done on nitrogen, pV = RT
∆W = J∆Q = 4.2 × 420 = 1764 J
700 OBJECTIVE Physics Vol. 1

RT RT 2  a n 1 (C V1 + R ) + n 2 (C V2 + R ) n 1C p 1 + n 2 C p 2
⇒ V= = Q p =  = =
p a  T n1 + n 2 n1 + n 2
∆V dV 2RT n 1C p + n 2 C p
⇒ = =
∆T dT a Thus, C p = 1 2

p ∆V  2 RT 
n1 + n 2
So, C = + CV = p   + CV
∆T  a  (v) γ of the mixture
a  2RT  From Eq. (i),
=   + CV (n 1 + n 2 ) C V = n 1C V1 + n 2 C V2
T  a 
3 7 (n 1 + n 2 ) R nR n R
⇒ C = 2R + C V = 2R + R= R or = 1 + 2
2 2 γ −1 γ1 −1 γ 2 −1
n1 + n 2 n1 n2
or = +
Thermodynamic parameters for a γ −1 γ1 −1 γ 2 −1
mixture of gases Thus, γ of the mixture is given by above equation.
Thermodynamic parameters for a mixture of gases are Example 15.25 A gaseous mixture enclosed in a vessel
given below  5
consists of 1g mole of a gas A  γ =  and some amount of
(i) Equivalent molar mass When n 1 moles of a gas  3
with molar mass M1 are mixed with n 2 moles of a  7
gas with molar mass M 2 , the equivalent molar mass gas B  γ =  at a temperature T. The gases A and B do not
 5
n M + n 2M 2
of the mixture is given by M = 1 1 react with each other and are assumed to be ideal. Find the
n1 + n 2 number of gram moles of the gas B, if γ for the gaseous
(ii) Internal energy of the mixture The total energy 19
mixture is   .
of the mixture is U = U1 + U 2 ...(i) 13
(iii) CV of the mixture Sol. Using formula for γ of a mixture of two gases, we get
As, U = U1 + U 2 ⇒ ∆U = ∆U1 + ∆U 2 n A + nB nA nB
or nC V ∆T = n 1 C V1 ∆T + n 2 C V2 ∆T …(ii) ⇒ = +
γ −1 γ A − 1 γB − 1
or (n 1 + n 2 ) C V = n 1 C V1 + n 2C V2 (As, n = n 1 + n 2 )
n 1C V1 + n 2 C V2 1 + nB 1 nB
⇒ = +
∴ CV = 19   5   7 
n1 + n 2  − 1  − 1  − 1
13   3   5 
(iv) C p of the mixture
⇒ 13 + 13 n B = 9 + 15 n B
n 1C V1 + n 2 C V2
As, C p = C V + R or C p = +R ⇒ n B = 2 g-mol
n1 + n 2

CHECK POINT 15.2


1. If 10000 cal of heat is added to 4 mol of a monoatomic gas 4. The molar specific heats of an ideal gas at constant pressure
without changing its volume, then change in temperature of and volume are denoted by C p and C V , respectively. If
the gas is γ = C p / C V and R is the universal gas constant, then C V is
(a) 233 K (b) 833 K (c) 123 K (d) 433 K equal to
1+ γ R (γ − 1)
2. The specific heat at constant pressure is greater than that of (a) (b) (c) (d) γR
the same gas at constant volume because 1−γ (γ − 1) R
(a) at constant volume, work is done in expanding the gas 5. The molar heat capacity of a gas at constant volume is C V . If
(b) at constant pressure, work is done in expanding the gas
n moles of the gas undergo ∆T change in temperature, its
(c) the molecular attraction is more at constant pressure internal energy will change by nC V ∆T
(d) the molecular vibration is more at constant pressure (a) only if the change of temperature occurs at constant
3. For hydrogen gas C p − C V = a and for oxygen gas C p − C V = b, volume
then the relation between a and b is (where, C p and C V are (b) only if the change of temperature occurs at constant
gram specific heats) pressure
(a) a = 16 b (b) b = 16 a (c) in any process which is not adiabatic
(c) a = b (d) None of these (d) in any process
Laws of Thermodynamics 701

6. Two moles of a gas at temperature T and volume V are 13 18 24


(a) 6g (b) g (c) g (d) g
heated to twice its volume at constant pressure. If 7 7 7
C p / C V = γ, then increase in internal energy of the gas is 9. The equivalent value of γ in the above question is
RT 2RT 2RT 2T
(a) (b) (c) (d) (a) 1.59 (b) 1.53
γ −1 γ −1 3(γ − 1) γ −1 (c) 1.56 (d) None of these
7. A gas mixture consists of 2 mol of oxygen and 4 mol of 10. A gaseous mixture enclosed in a vessel consists of 3 g mole
argon at temperature T. Neglecting all vibrational modes, of a gas X with γ = (5/ 3) and some amount of gas Y with
the total internal energy of the system is γ = (7/ 5) at a temperature T . The gases X and Y do not react
(a) 4RT (b)15RT (c) 9RT (d) 11RT with each other and are assumed to be ideal. If γ for the
 29
8. Five moles of helium are mixed with two moles of hydrogen gaseous mixture is   , then the number of gram moles of
to form a mixture. Take, molar mass of helium M1 = 4 g and  23
that of hydrogen M2 = 2 g. The equivalent molar mass of the the gas Y is
mixture is (a) 0.5 (b) 0.4 (c) 0.8 (d) 1

THERMODYNAMIC PROCESSES
When state of a thermodynamic system changes or the V
(ii) V-T graph In isobaric process, = constant
state variables change with time, then the process is T
known as thermodynamic process. ∴ V ∝T
Few of thermodynamic processes are discussed below
i.e. V-T graph is a straight line passing through origin
as shown below.
1. Isobaric process
A thermodynamic process occurring at constant V
pressure, is known as isobaric process. e.g. Boiling of
water in an open container.
θ
Equation of state In this process, volume V and T
temperature T changes but pressure p remains constant. Fig. 15.8 V-T graph for isobaric process
V
= constant (Charles’ law) Note Slope of the V-T graph, tanθ =
V nR
= = constant.
T T p
Graphs Graphs related to isobaric process are given (iii) p-T graph In isobaric process, pressure remains
below
constant, so volume changes resulting in change of
(i) p-V graph In an isobaric process, pressure of the temperature as shown below.
gas remains constant. p
Therefore, p-V graph associated with this process 1 2
is a straight line parallel toV-axis (having slope, p
dp dp
= 0) as shown below. Slope= =0
dT
dV
O T
p T1 T2
Fig. 15.9 p-T graph for isobaric process

dp
Work done Suppose the pressure p of a gas remains constant
Slope = —— =0 and its volume changes fromVi toVf , then the work done by the
dV
O Isobaric expansion
V gas is

p
Work done,W = p (Vf − Vi )
or Work done,W = nR (T f − Ti )
dp First law of thermodynamics for isobaric process
Slope =
dV
=0 For isobaric process, ∆W = p∆V or∆W = nR∆T
O
Isobaric compression
V
From ∆Q = ∆U + ∆W
Fig. 15.7 p-V graph for isobaric expansion and compression So, ∆Q = ∆U + nR∆T
702 OBJECTIVE Physics Vol. 1

∆Q ∆Q Graph Graphs related to isochoric process are given below


Example 15.26 Find the ratio of and in an isobaric
∆U ∆W (i) p-V graph In an isochoric process, volume of the
Cp gas remains constant. Therefore, p-V graph is a
process. The ratio of molar heat capacities = γ.
CV straight line parallel to p-axis with slope,
Sol. In an isobaric process, p = constant. Therefore, C = C p .
dp π
= tan = ∞ as shown below.
dV 2
∆Q nC p ∆T C p p p
Now, = = =γ
∆U nC V ∆T C V
dp dp
∆Q ∆Q I Slope = ——
dV =∞ II Slope = ——
dV =∞
and =
∆W ∆Q − ∆U V V
Increase in pressure Decrease in pressure
nC p ∆T
= Fig. 15.10 Graph for isochoric process
nC p ∆T − nC V ∆T
p
Cp C p / CV γ (ii) p-T graph For isochoric process, = constant.
= = = T
C p − CV C p / CV − 1 γ −1
∴ p ∝T
Example 15.27 Suppose 1.0 g of water vaporises isobarically i.e. p-T graph is a straight line passing through origin.
at atmospheric pressure (1.01 × 10 5 Pa ). Its volume in the
liquid state isVi = Vliquid = 1.0 cm 3 and its volume in
vapour state isVf = Vvapour = 1671 cm 3 . Find the work done p
in the expansion and the change in internal energy of the
system. Ignore any heat transfer of the steam and the θ
surrounding air. (Take, latent heat of vaporisation, T
L v = 2.26 × 10 6 Jkg −1) Fig. 15.11 p-T graph for isochoric process
Sol. Because expansion takes place at constant pressure, the p nR
Note Slope of the p-T graph, tanθ = = = constant.
work done is T V
Vf Vf
W= ∫Vi
p 0dV = p 0 ∫Vi
dV = p 0 (Vf − Vi ) (iii) V-T graph In isochoric process, volume remains
constant. So, pressure changes resulting in change of
= (1. 01 × 105 ) (1671 × 10−6 − 1.0 × 10−6 ) ≅169 J temperature as shown below.
Heat of vaporisation, V
−3
Q = mLv = (1.0 × 10 ) (2.26 × 10 ) = 2260 J 6
1 2
Hence, from the first law of thermodynamics, the change in
internal energy, Slope =dV = 0
dT
∆U = Q − W = 2260 − 169 = 2091 J T
T1 T2
Note The positive value of ∆U indicates that the internal energy of the
 2091 J  Fig. 15.12 V-T graph for isochoric process
system increases. We see that most  = 93%  of the
 2260 J 
Work done As volume remains constant,
energy transferred to the liquid goes into increasing the internal
energy of the system only
169 J
= 7% leaves the system, by
i.e. V = constant ⇒ ∆V = 0
2260 J ∴ Work done, ∆W = p∆V = 0
work done through the steam in pushing the surrounding
atmosphere. First law of thermodynamics for isochoric process
For isochoric process, ∆Q = nC V ∆T
2. Isochoric process From first law of thermodynamics, we get
A thermodynamic process occurring at constant volume, is ∆Q = ∆U + ∆W
known as isochoric process. e.g. Heat given to a system As dW = 0 (for isochoric process), ∆Q = ∆U = nC V ∆T
with fixed walls. For one mole of a gas, n = 1⇒ ∆Q = C V ∆T = ∆U
Equation of state In this process, p and T changes butV Example 15.28 What is the heat input needed to raise the
remains constant. temperature of 2 mol of helium gas from 0°C to 100°C
p (i) at constant volume? (ii) at constant pressure?
p ∝T ⇒ = constant (Gay-Lussac’s law) (iii) What is the work done by the gas in part (ii)?
T
Give your answer in terms of R.
Laws of Thermodynamics 703

Sol. Helium is monoatomic gas. (iii) p-T graph Similar toV-T graph, p-T graph is
3R 5R parallel to p-axis.
Therefore, C V = and C p =
2 2
p
 3R 
(i) At constant volume, Q = nC V ∆T = (2)   (100) = 300R p2
 2 2
dp
Slope = =∞
 5R  dT
(ii) At constant pressure, Q = nC p ∆T = (2)   (100) p1 1
 2
= 500R T
T
(iii) At constant pressure,W = Q − ∆U = nC p ∆T − nC V ∆T Fig. 15.15 p-T graph for isothermal process
= nR∆T = (2) (R ) (100) = 200R
Slope of p-V graph For isothermal process,
3. Isothermal process pV = constant
A thermodynamic process occurring at constant On differentiating, we get
temperature is known as isothermal process. e.g. Freezing dp
of water at 0°C to form ice at 0°C. pdV + Vdp = 0 ⇒ = − p /V
dV
Equation of state For an ideal gas, the equation of state dp p
for isothermal process is given by Slope of graph = − tanφ = =−
dV V
pV = constant p

So, gas follows Boyle’s law,


⇒ p iVi = p fVf , for isothermal process
Note Slow processes are treated as isothermal.
φ
Graph Graphs related to isothermal process are given below V

(i) p-V graph For isothermal process p -V = constant. Fig. 15.16 Slope of isothermal curve
Therefore, p-V graph is a rectangular hyperbola as
Specific and molar heat capacity During isothermal
shown below.
process, ∆T = 0
p ∆Q ∆Q
Isotherms ∴ Specific heat capacity, s = = =∞
m∆T m (0 )
Isothermal expansion
i
∆Q
∴ Molar heat capacity, C iso = =∞
f
n∆T
T = 325 K
Vf Vf  nRT 
Work done W = ∫ p dV = ∫ 
T = 310 K
T = 290 K Vi Vi  V 
 dV
V
O Vi Vf  nRT 
Q p = 
Fig. 15.13 An isothermal expansion process  V 
Vf dV
Note An isotherm is a curve that connects points of same temperature.
= nRT ∫ (Q T = constant)
Vi V
(ii) V-T graph Temperature of the gas remains constant  Vf 
during isothermal process. Thus,V-T graph is W = nRT ln  
parallel toV-axis as shown in figure.  Vi 
p 
V
or W = nRT ln  i  (Q p iVi = p fVf )
V2 2  pf 
Slope = dV = ∞ Note
dT
V1 1 (i) If Vf > Vi , then W > 0, hence Q > 0, i.e. gas absorbs heat and work
is done by the gas on the surrounding during expansion.
T
T (ii) If Vf < Vi , then W < 0, hence Q < 0, i.e. during compression, work
Fig. 15.14 V-T graph for isothermal process is done on the gas by the surrounding and heat is released.
704 OBJECTIVE Physics Vol. 1

First law of thermodynamics for isothermal According to the question,


process As temperature remains constant in isothermal n = 2 g-mol
process, T = 273 + 27 = 300 K
∆U = 0 and Vf / Vi = 3
So, by first law of thermodynamics, ∆Q = ∆W + ∆U W = 2.3026 × 2 × 8.3 × 300 × log10 3
= 2.3026 × 2 × 8.3 × 300 × 0.4771
Vf 
⇒ ∆Q = ∆W = nRT ln   = 5.47 × 103 J
 Vi 
Now, W = JQ
Vf  W 5.47 × 103 J
= 2.3026 nRT log 10   ∴ Absorbed heat, Q = =
 Vi  J 4.18 J cal−1
where, n = number of moles, = 1.31 × 103 cal
R = gas constant and T = temperature.
Example 15.29 Five moles of an ideal gas at constant
4. Adiabatic process
temperature 1200K is compressed from 200 L to 20L. A thermodynamic process which takes place in such a way
Evaluate the work done in the process. that there is no transfer of heat into or out of a system is
known as adiabatic process.
Sol. It is given that, number of moles, n = 5
e.g. Rapid compression like filling of a cycle tube by a
Temperature of the gas, T = 1200 K
hand pump, rapid expansion.
∴ Work done in an isothermal process,
Equation of state For an ideal gas, equation of state for
V 
W = 2.3026 nRT log10  2  adiabatic process is
 V1 
pV γ = constant
 20 
= 2.3026 × 5 × 8.3 × 1200 log10  
 200 Proof of the p-V relation In adiabatic process, dQ = 0
≅ −11.5 × 10 J
4
and dW = − dU
∴ pdV = − C V dT (For n = 1)
Example 15.30 If 1 g mole of a gas is expanded isothermally
at 127°C such that its volume increases two times, then find pdV
⇒ dT = − …(i)
the work done and heat absorbed by the gas. CV
Sol. Given, temperature of the gas, Also, for 1 mole of an ideal gas,
T = 127°C = 127 + 273 = 400 K d (pV ) = d (RT )
Initial volume,V1 = V (Assume)
or pdV + Vdp = RdT
Final volume,V2 = 2V pdV + Vdp
V  or dT = …(ii)
Work done,W = 2.303 nRT log10  2  R
 V1 
 2V  From Eqs. (i) and (ii), we get
= 2.303 × 1 × 8.3 × 400 log10  
V  C VVdp + (C V + R ) pdV = 0
= 2.303 × 8.3 × 400 × 0.3010 ≅ 2302 J or C VVdp + C p pdV = 0
For isothermal process, dU = 0 Dividing this equation by pV, we are left with
∆Q = dU + ∆W = 0 + 2302 = 2302 J
dp dV
⇒ ∆Q =
2302
≅ 548 cal CV + Cp =0
4.2 p V
dp dV
Example 15.31 2 g mol ideal gas is expanded isothermally at or +γ =0
27°C. Its volume increases three times of initial volume. p V
Find the work done and heat absorbed by the gas. (Take, dp dV
R = 8.3 J mol −1K −1) or ∫ p
+γ ∫
V
=0
Sol. Work done by n moles of an ideal gas at isothermal
temperature T, when its volume changes fromVi toVf is or ln (p ) + γ ln (V ) = constant
Vf or ln (p ) + ln (V γ ) = constant
W = 2.3026 nRT log10
Vi
Laws of Thermodynamics 705

or ln (pV γ ) = constant (iii) p-T graph For adiabatic process with decrease in
temperature, pressure decreases as shown below.
We can write this in the form
pV γ = constant p
p2
This equation is the condition that must be obeyed by an
ideal gas in an adiabatic process. e.g. If an ideal gas makes
an adiabatic transition from a state with pressure and
p1
volume p i andVi to a state with p f andVf , then
O
piVi γ = p f Vfγ
T
T1 T2
g
p1–g T =constant
The equation pV γ = constant can be written in terms of
Fig. 15.19 p-T graph for adiabatic process
other pairs of thermodynamic variables by combining it
with the ideal gas law (pV = nRT ).
Slope of p-V graph In an adiabatic process
RT
Putting p = , we get (From ideal gas equation) (pV γ = constant ), the slope of p-V diagram at any point is
V
dp d  constant 
TV γ −1
= constant = − tan φ =  
dV dV  V γ 
RT
Again puttingV = , we get (From ideal gas equation) p
p =−γ  
V 
T γ p 1 − γ = constant
p
Graphs Graphs related to adiabatic process are given below
(i) p-V graph For no heat exchange in adiabatic process,
pressure decreases exponentially with increase in
volume as shown below.
p φ
p1 O V
Fig. 15.20

p
Thus, (Slope) adiabatic = − γ  
p2
V 
V
O V V2
1 Also, we know that, in an isothermal process,
pV γ = constant p
(Slope) isothermal = − . Because γ > 1, the isothermal curve
Fig. 15.17 p-V graph for adiabatic process V
is not as steep as that for the adiabatic expansion.
(ii) V-T graph In adiabatic process with increase in
temperature, volume decreases exponentially as p p 1 Monoatomic
shown below. 2 Diatomic
3 Polyatomic
V
V1 Isothermal γ = 1.33
3
2 γ = 1.4
1
γ = 1.67
Adiabatic
V V
V2 O O
T Adiabatic and isothermal Adiabatic expansion of mono, dia
O T T2 expansion of an ideal gas and polyatomic gases
1
γ–1
TV = constant
Fig. 15.21 Adiabatic and isothermal expansion
Fig. 15.18 V-T graph for adiabatic process
706 OBJECTIVE Physics Vol. 1

In similar way for adiabatic and isothermal compression Example 15.33 Two samples of a gas initially at same
of an ideal gas, p-V diagram as shown, below implies temperature and pressure are compressed from a volume V to
that, the curve which is more vertical is adiabatic. V /2. One sample is compressed isothermally and the other
p adiabatically. In which sample is the pressure greater?
Adiabatic Sol. Let initial volume,V1 = V and pressure, p1 = p
V
Isothermal Final volume,V2 = and final pressure, p 2 = ?
2
O V For isothermal compression, p 2V2 = pV
11
Fig. 15.22 pV pV
⇒ p2 = 1 1 = = 2p
Specific and molar heat capacity During adiabatic V2 V /2
process, ∆Q = 0 V 
γ

∆Q For adiabatic compression, p 2′ = p1  1 


∴ Specific heat capacity, s = =0 V2 
m∆T γ
V 
∆Q ⇒ p 2′ = p  γ γ
 = 2 p ⇒ p 2′ = 2 p
∴ Molar heat capacity, C adi = =0 V /2
n∆T
Q γ > 1 ∴ 2γ > 2 and p 2′ > p 2
Work done Let initial state of gas is (p 1, V1, T1 ) and after
adiabatic change final state of gas is (p 2, V2, T 2 ), then Pressure during adiabatic compression is greater than the
p V − p 2V2 pressure during isothermal compression.
work done W = 1 1
γ −1 Example 15.34 An ideal monoatomic gas at 300 K expands
adiabatically to twice its volume. What is the final
On putting pV = nRT , we get temperature?
nR Sol. For an ideal monoatomic gas, γ = 5 /3
⇒ W = (T1 − T2 )
(γ − 1)
In an adiabatic process, TV γ −1
= constant
Note
(i) If work is done by the gas in an adiabatic process (W > 0), then ∴ Tf Vfγ − 1 = Ti Vi γ −1

T2 < T1.
γ −1 5
−1
(ii) If work is done on the gas (W < 0), we get T2 > T1, i.e. the V   1 3
temperature of the gas rises. or Tf = Ti  i  = (300)   ≈ 189 K
Vf   2
First law of thermodynamics for adiabatic
process Now, by first law of thermodynamics,
∆Q = ∆U + ∆W
Bulk modulus of gas
Bulk modulus of gas is given by
As, ∆Q = 0 ⇒ ∆U = − ∆W
dp
So, if work is done by the system, on the expense of B=−
internal energy and so the temperature of system falls, (dV /V )
i.e.W > 0, T f < Ti . Conversely, if work is done on the  dp 
or B = −V  
system, the internal energy and so the temperature of  dV 
system increases, i.e.W < 0, T f > Ti .
dp
Example 15.32 Initial pressure of a gas is 5 × 10 5 Nm −2 . where, is slope of p-V graph.
dV
Under an adiabatic process, its volume becomes 1/9 of its
dp p
initial value. What is the pressure of the gas in this For isothermal process, =−
situation? (Take, γ = 3/2) dV V
γ
V   p
Sol. For adiabatic process of the gas, pV γ γ
1 1 = p 2V2 ⇒ p 2 = p1  
1 ∴ Bulk modulus of gas, B = −V  − 
V2   V
Given, γ = 3/2, V1 = V , V2 =
V
and p1 = 5 × 105 Nm−2 B=p
9 dp γp
 V 
3/ 2 For adiabatic process, =−
∴ p 2 = 5 × 105 ×   dV V
V / 9
 γp 
= 5 × 105 × (9)3/ 2 = 5 × 105 × (32 )3/ 2 ∴ Bulk modulus of gas, B = −V  −  or B = γp
 V
= 5 × 105 × 33 = 135 × 105 Nm−2
Laws of Thermodynamics 707

Therefore, isothermal bulk modulus is equal to pressure of Summary of ∆Q, ∆U and ∆W for different processes
the gas and adiabatic bulk modulus is γ times the pressure Name of the ∆Q ∆U ∆W
of gas. process
Isothermal ∆Q = ∆W 0 V  p 
p-V Diagram of Different Processes nRT ln  f  = nRT ln  i 
p 
 Vi   f
p-V diagram of different processes is shown as follows. We
can compare the work done in each process from area under Adiabatic 0 nC V ∆T i i − pfVf
pV
= − ∆U
p-V curve. γ −1
p
Adiabatic Isobaric n C p ∆T nC V ∆T p (Vf − Vi )
(compression)
Isochoric ∆Q = ∆U nC V ∆T 0
Isobaric = nC V ∆T
O
Isothermal
Adiabatic (expansion) Example 15.35 14 g of nitrogen gas is compressed in an
Isochoric adiabatic process such that its temperature increases by
O V
60°C. Find the work done on the gas.
Fig. 15.23 Combined p-V diagram of different processes
Sol. Given, m = 14 g, T2 − T1 = 60° C, γ = 1.5
∆p p
We know that, in an isothermal process, =− . Work done on the gas,W =
nR
(T1 − T2 )
∆V V γ −1
∆p γp
For an adiabatic process, =− it means that at a m 14 1
n= = =
∆V V Q
M 28 2
particular point, slope (value) of adiabatic curve is more 1 8.3
than that for isotherm or we can say adiabatic curve is ∴ W= × × (− 60) = − 498 J
2 1.5 − 1
steeper than an isotherm for expansion and just reverse for
Negative sign shows that work is done on the gas.
compression. It is clear from the figure that for expansion
that occurs within same limits. Example 15.36 Temperature of a 5 mol of hydrogen gas
Wisobaric > Wisothermal > Wadiabatic > Wisochoric decreases from 273 K to 260 K while expanded through
adiabatic process. Find the work done by the gas. What is
Note In general many thermodynamic processes follow a rule of type, the change in internal energy of the gas?
pV n = constant.
nR
These processes are called polytropic process and ‘n’ is called Sol. Work done by the gas, W = (Ti − Tf )
γ −1
polytropic index.
According to the question,
Process n
n = 5 mol, R = 8.31 J/mol-K,
p = constant 0 γ = 1.4, Ti = 273 K, Tf = 260 K
V = constant ∞ 5 × 8.31
Work done,W = (273 − 260) K ≈ 1350 J
1.4 − 1
T = constant 1
According to first law of thermodynamics,
∆Q = 0 r
∆U = ∆Q − W = 0 − (1350) = − 1350 J
(i) Work done in a polytropic (1 → 2) process, Example 15.37 2 m 3 volume of a gas at a pressure of
p V − p 2V2 4 × 10 5 Nm −2 is compressed adiabatically, so that its
∆W = ∫ pdV = 1 1
n −1 volume becomes 0.5 m 3 . Find the new pressure. Compare
(ii) Heat transfer in a polytropic process, this with the pressure that would result, if the compression
R(T1 − T2) was isothermal. Calculate work done in each process.
Q= − C V(T1 − T2)
n −1 (Take, γ = 14)
.
R
where, C V = Sol. Given,V1 = 2 m3, p1 = 4 × 105 Nm−2,V2 = 0.5 m3
γ −1
γ γ

 γ − n
Q=  W = C n(T2 − T1) 1 1 = p 2V2
In adiabatic process, pV
 γ − 1 1.4
 2 
(iii) Specific heat capacity of gas in a polytropic process, ⇒ p 2 = 4 × 105   = 4 × 105 (4)1.4
C n = CV +
R  γ − n
= CV 
 0.5

1− n  1− n 
≅ 2.8 × 106 Nm−2
708 OBJECTIVE Physics Vol. 1

1 1 = p 2V2
In isothermal process, pV
pV 4 × 105 × 2
5. Cyclic process
⇒ p2 = 11
= = 1.6 × 106 Nm−2 A single process or a series of processes in which after
V2 0.5
certain interchanges of heat and work, the system is
Now, work done in adiabatic process , restored to its initial state are known as a cyclic process.
1 1 − p 2V2
pV (4 × 105 × 2 − 2.8 × 106 × 0.5)
W= = As both initial and final states are same in a cyclic process,
γ −1 1.4 − 1
∆U = Uf − Ui = 0
= −1.5 × 106 J p p
Work done in isothermal process ,
V V2
W = 2.3026 RT log 2 = 2.3026 pV
1 1 log
V1 V1
 0.5
= 2.3026 × 4 × 105 × 2 × log   V V
 2.0  Clockwise cyclic Anti-clockwise
 1 process cyclic process
= 2.3026 × 4 × 105 × 2 log  
 4 Fig. 15.24 Cyclic processes
= − 1.1 × 10 J 6
For a cyclic process, p-V graph is a closed curve and area
Example 15.38 Two moles of helium gas (γ = 5/3), assumed enclosed by the curve is equal to the work done.
ideal, are initially at 27°C and occupy a volume of 20 L. From first law of thermodynamics,
The gas is first expanded at constant pressure till its volume ∆Q = ∆U + ∆W
is doubled. It then undergoes an adiabatic change until the
temperature returns to its initial value. Determine the final For cyclic process, ∆U = 0 or ∆Q = ∆W
pressure and volume of the gas. Also, calculate the work So, heat supplied to the system is converted into work
done under isobaric and adiabatic processes. done and vice-versa. Heat is supplied to the system for
(Take, R = 8.3 J mol −1 K −1) clockwise cycle and heat is taken from the system for
Sol. anti-clockwise cycle.
p
T1 Example 15.39 Figure shows a process ABCA performed on
p1 T2 = 2T1 an ideal gas. Find the net heat given to the system during
A B V1 = 20 H 10–3
m,3
the process.
T3 = T1 = 300 K V
p2 T3 = T1 C
C V2

V1 V2 V3 V A
B
V1
For process AB (p = constant),
T2 = 2T1 = 600 K (Q V2 = 2V1) T2 T
T1
For process BC, TV γ −1
= constant
Sol. Since, the process is cyclic, hence the change in internal
Therefore, (2T1)(V2 )5/ 3 − 1 = (T1)(V3 )5/ 3 − 1 energy is zero.
⇒ V3 = (V2 )(2)3/ 2 = 2 2V2 The heat given to the system is then equal to the work done
by it.
= 80 2 × 10−3 m3 = 80 2 L The work done in part AB isW1 = 0 (the volume remains
nRT3 (2)(8.3)(300) constant). The part BC represents an isothermal process, so
⇒ p3 = = = 0.44 × 105 Nm−2 the work done by the gas during this part is
V3 80 2 × 10−3
V
Work done under isobaric process = p∆V = p1V
( 2 − V1) W2 = nRT2 ln 2
V1
nRT1 2 × 8.3 × 300
= ( 2 − V1) =
V (40 × 10 −3 − 20 × 10 −3 ) = 4980 J
V1 20 × 10 −3 During the part CA , V ∝ T . So, V/T is constant and hence,
nRT
nR (T2 − T3 ) p= is constant.
Work done under adiabatic process = V
(γ − 1)
The work done by the gas during the part CA is
2 × 8.3 × (300) W3 = pV ( 1 − V2 ) = nRT1 − nRT2
= = 7470 J
(5/3 − 1) = − nR (T2 − T1)
Laws of Thermodynamics 709

The net work done by the gas in the process ABCA is According to the p-V diagram, work done
 V  1
∆Wcyclic = − (150 − 50) × 103 × (2.5 − 2.0)
W = W1 + W2 + W3 = nR T2 ln 2 − (T2 − T1) 2
 V1 
= − 25000 J
The same amount of heat is given to the gas.
In the cyclic process,
Example 15.40 Two moles of helium gas undergo a cyclic ∆Q = ∆W = − 25000 J
process as shown in figure. Assuming the gas to be ideal, Negative sign shows that heat is extracted from the gas.
calculate the following quantities in the process:
(i) the net work done, Efficiency of a cycle
(ii) the net change in internal energy, Efficiency (η) of a cycle can be defined as
−1 −1
(iii) the net change in heat energy. (Take, R = 8.32 J mol K )
 Work done by the working substance 
 during a cycle 
2 atm A
B η=   × 100
 Heat supplied to the gas during the cycle 
 
p
1 atm C
D Wtotal |Q | − |Qreleased |
300 K 400 K
η= × 100 = absorbed × 100
|Q absorbed | |Q absorbed |
T
Wtotal  Qreleased 
Sol. Given, n = 2, C V = 3R /2, C p = 5R /2, Thus, η= × 100 =
1 −  × 100
|Q absorbed |  Q absorbed 
TA = TD = 300 K, TB = TC = 400 K
(i) A → B (isobaric process) : There cannot be a cycle whose efficiency is 100%. Hence,
∆WA → B = p∆V = nR∆T = 2R (400 − 300) = 200R η is always less than 100%.
B → C (isothermal expansion): Thus, Wtotal ≠ Q absorbed
∆WB → C = nRTB ln (pB / pC ) = 2R × 400 ln (2 /1) = 800 R ln 2 Example 15.42 N moles of a monoatomic gas is carried round the
C → D (isobaric process) : rectangular cycle ABCDA as shown in the diagram. The
temperature at A is T0. Find the thermodynamic efficiency of the
∆WC → D = nR∆T = 2R (300 − 400) = − 200R
cycle.
D → A (isothermal compression): p
∆WD → A = nRTA ln (pD / p A ) = 2R × 300 ln (1/2) 2p0 B C
= − 600 R ln 2
∆Wcyclic = ∆WAB + ∆WBC + ∆WCD + ∆WDA p0
A
D
= 200R + 800R ln 2 − 200R − 600 R ln 2
= 200R ln 2 = 200 × 8.3 × 0.693 ≅ 1150 J V0 2V0
V
(ii) For cyclic process, ∆T = 0, ∆U = 0
(iii) For cyclic process, ∆Q = ∆W = 1150 J Sol. First of all note that point C is at highest temperature and A is
at lowest temperature. So, during process A to B and B to C,
Example 15.41 A gas is initially at a pressure of 50 kPa. Its heat is being added to the cycle. Work output can be calculated
pressure is kept constant and the volume is changed from by calculating the area under the cycle of p-V diagram. Then,
2 m 3 to 2.5 m 3 . Its volume is now kept constant and the efficiency can be calculated as work output upon heat input.
pressure is increased from 50 kPa to 150 kPa. The gas is Note that the temperature at A is T0, that at B is 2T0 and that
brought back to its initial state via a process in which, the at C is 4T0. Also, note C V for monoatomic gas is 3R /2.
pressure varies linearly with its volume. Whether the heat is For A-B, ∆Q = NC V ∆T (Q constant volume process)
supplied to or extracted from the gas in the complete cycle? 3R N
=N (2T0 − T0 ) = 3RT0
Sol. First draw p-V diagram as shown below. 2 2
150 3 For B-C, ∆Q = NC p ∆T (Q constant pressure process)
5R
p 50 =N (4T0 − 2T0 ) = 5NRT0
2 2
(kPa) 1
3  13
Now, total heat input, Q =  + 5 NRT0 = NRT0
2  2
2 2.5
V(m3) Work output = Area under the cycle
710 OBJECTIVE Physics Vol. 1

= (2p 0 − p 0 ) (2V0 − V0 ) Therefore, (∆U )whole cycle = 0


= p 0V0 = NRT0 = ∆U AB + ∆U BC + ∆U CA
W NRT0 2 and from first law of thermodynamics,
∴ Efficiency = = = or 15.38% Q AB + QBC + QCA = WAB + WBC + WCA
Q 13 NRT 13
0 Substituting the values,
2
700 + 0 − 100 = 700 + 400 + ∆WCA
Example 15.43 An ideal gas expands isothermally along AB ∴ ∆WCA = −500 J
and does 700 J of work.
Negative sign implies that work is done on the gas.
p
Different values in different processes
A

B Process Q (J) W (J) ∆U (J)

AB 700 700 0
C
BC 0 400 − 400
V
CA − 100 − 500 400
(i) How much heat does the gas exchange along AB?
(ii) The gas then expands adiabatically along BC and does 400 J of For complete cycle 600 600 0
work. When the gas returns to A along CA, it exhausts 100 J of
heat to its surroundings. How much work is done on the gas Note Total work done is 600 J, which implies that area of the closed
along this path? curve is also 600 J.
Sol. (i) AB is an isothermal process.
Example 15.44 In example 15.43, find the efficiency of the
Hence, ∆U AB = 0
given cycle.
and Q AB = WAB = 700 J
Sol. From table of the previous example, we can see that
(ii) BC is an adiabatic process.
Q absorbed during the cycle is 700 J, while the total work done
Hence, QBC = 0 ⇒ WBC = 400 J (given) in the cycle is 600 J.
∴ ∆U BC = − WBC = − 400 J
Wtotal  600
ABCA is a cyclic process and internal energy is a state function. ∴ η= × 100 =   × 100 = 85.71%
|Q absorbed|  700

CHECK POINT 15.3


1. Choose the incorrect statement related to an isobaric process. 9 2
(a) (b)
V 59 3
(a) = constant
T 3
(c) (d) None of these
(b) W = p ∆V 4
(c) Heat given to a system is used up in raising the 5. In an isothermal expansion of an ideal gas, select the wrong
temperature only statement.
(d) None of the above (a) There is no change in the temperature of the gas.
2. When 1 g of water at 0°C and1 × 105 Nm −2 pressure is (b) There is no change in the internal energy of the gas.
(c) The work done by the gas is equal to the heat supplied
converted into ice of volume1.091 cm3, the work done will be to the gas.
(a) 0.0091 J (b) 0.0182 J (d) The work done by the gas is equal to the change in its
(c) 0.091 J (d) Data insufficient internal energy.
3. An ideal gas of volume1.5 × 10−3 m3 and at pressure1.0 × 105 6. Which of the p-V diagrams best represents an isothermal
Pa is supplied with 70 J of energy. The volume increases to process?
1.7 × 10−3 m3, the pressure remaining constant. The internal p p
energy of the gas is
(a) increased by 90 J (a) (b)
(b) increased by 70 J
V V
(c) increased by 50 J p p
(d) decreased by 50 J
p1 (c) (d)
4. In an isochoric process, if T1 = 27°C and T 2 = 127°C, then
p2
will be equal to V V
Laws of Thermodynamics 711

7. A cycle tyre bursts suddenly. This represents an 11. Two identical samples of a gas are allowed to expand
(a) isothermal process (b) isobaric process (i) isothermally (ii) adiabatically. Work done is
(c) isochoric process (d) adiabatic process (a) more in the isothermal process
(b) more in the adiabatic process
8. A gas for which γ = 1.5 is suddenly compressed to (1/4)th (c) zero in both of them
of the initial volume. Then, the ratio of the final to the (d) equal in both processes
initial pressure is
12. The volume of a gas is reduced adiabatically to (1/4)th of its
(a) 1 : 16 (b) 1 : 8 (c) 1 : 4 (d) 8 : 1
volume at 27°C. If γ = 1.4, the new temperature will be
5 (a) (300) 20.4 K (b) (300) 41.4 K (c) (300) 4 0.4 K (d) (300) 21/ 4 K
9. For a gas γ = and 640 cc of this gas is suddenly
3
compressed to 80 cc. If the initial pressure is p, then the 13. An ideal gas at a pressure of 1 atm and temperature of 27°C
final pressure will be is compressed adiabatically until its pressure becomes
8 times the initial pressure, then the final temperature is
(a) 8p (b) 32p (c) 16p (d) 64p
(γ = 3 / 2)
10. During an adiabatic process, the pressure of a gas is found (a) 627°C (b) 527°C (c) 427°C (d) 327°C
to be proportional to the cube of its absolute temperature. 14. In an adiabatic expansion, a gas does 25 J of work while in
Cp
The ratio = γ for the gas is an adiabatic compression 100 J of work is done on a gas. The
CV change of internal energy in the two processes respectively are
(a) 2 (b)
3
(c)
5
(d)
4 (a) 25 J and −100 J (b) −25J and 100 J
2 3 3 (c) −25J and −100 J (d) 25 J and 100 J

HEAT ENGINE
A heat engine is a device which converts heat energy into Thermal efficiency
mechanical energy. Thermal efficiency of a heat engine is defined as the ratio
The essential parts of a heat engine are as follows of net work done per cycle by the engine to the total
Source It is a reservoir of heat at high temperature and amount of heat absorbed per cycle by the working
infinite thermal capacity. Any amount of heat can be substance from the source. It is denoted by η.
extracted from it at constant temperature. W Q1 − Q 2 Q
Thus, η = = = 1− 2
Sink It is a reservoir of heat at low temperature and infinite Q1 Q1 Q1
thermal capacity. Any amount of heat can be given to the
Q2
sink at constant temperature. Thermal efficiency, η = 1 −
Q1
Working substance A material which absorbs heat
energy from the source and converts it into mechanical where,
energy (useful work) by rejecting some of the heat to sink Q1 = heat absorbed by the working substance,
is called working substance. e.g. Steam, petrol, etc. Q 2 = heat released by the working substance to
the sink,
Working η = thermal efficiency of heat engine
The working substance absorbs heat Q1 (as shown in and W = work done.
figure) from the source, does an amount of workW,
For an ideal heat engine, Q 2 = 0
returns the remaining amount of heat Q 2 to the sink and
comes back to its original state and there occurs no change So η = 1, i.e. 100% ( practically not possible)
in its internal energy. Example 15.45 The p-V diagram of 0.2 mol of an ideal
Source is always at higher temperature than sink. diatomic gas used as working substance in heat engine is as
shown in figure. Process BC is adiabatic. The value of γ for
By repeating the same cycle over and over again, work this gas is 1.4.
can be continuously obtained from an engine, p
600 K
Source Sink B
Engine T2
T1
Q1 Q2
A C
1.0 atm 450 K
W = Q1 − Q2 300 K
V
O
Fig. 15.25 Working of heat engine
712 OBJECTIVE Physics Vol. 1

(i) Find the pressure and volume at points A, B and C. ∆QCA  Cp
(ii) Calculate ∆Q, ∆W and ∆U for each of the three processes. ∆U CA = nC V ∆T = Q γ = 
γ  C V
(iii) Find the thermal efficiency of the cycle. 872
(Take, 1 atm = 1.01 × 105 Nm −2) =− ≈ − 623 J
1.4
Sol. (i) Given, p A = pC = 1 atm = 1.01 × 105 Nm−2 ∴ ∆WCA = ∆QCA − ∆U CA = − 872 J + 623 J = − 249 J
Process AB is an isochoric process.
Process ∆Q (in J) ∆W (in J) ∆U (in J)
p T
∴ p ∝ T or B = B AB 1246 0 1246
p A TA
BC 0 623 − 623
T   600
∴ pB =  B  p A =   (1 atm) = 2 atm CA − 872 − 249 − 623
 TA   300 
Total 374 374 0
= 2.02 × 105 Nm−2
nRT (iii) Efficiency of the cycle,
From ideal gas equation,V = W 374
p η = total × 100 = × 100 = 30%
|Q + ve | 1246
nRTA (0.2) (8.31) (300)
∴ VA = VB = =
pA (1.01 × 105 )
≈ 5.0 × 10−3 m3 = 5 L REFRIGERATOR
nRTC (0.2) (8.31) (450)
and VC = = It is a device which works in the reverse direction of heat
pC (1.01 × 10 5 ) engine. Refrigerator takes heat Q 2 (as shown in figure)
≈ 7.4 × 10 –3 m3 = 7.4 L from a cold body (sink), work (W ) is done on it and the
work done together with the heat absorbed (i.e.Q1 ) is
State p V
rejected to the source.
A 1 atm 5L
B 2 atm 5L Q1 Q2
Source Working Sink
C 1 atm 7.4 L T1 substance T1

(ii) Process AB is an isochoric process.


W
Hence, ∆WAB = 0
5  Fig. 15.26 Working of refrigerator
∆Q AB = ∆U AB = nC V ∆T = n  R  (TB − TA )
2 
 5 Coefficient of performance (COP)
= (0.2)   (8.31) (600 − 300) ≈ 1246 J
 2
The performance of a refrigerator is expressed by means of
Process BC is an adiabatic process. coefficient of performance β which is defined as the ratio
Hence, ∆QBC = 0 of the heat extracted from the cold body to the work
∴ ∆WBC = − ∆U BC needed to transfer it to the hot body.
∆U BC = nC V ∆T = nC V (TC − TB ) Heat extracted
5  i.e. β=
= (0.2)  R  (450 − 600) Work done
2 
Q2 Q2
 5
= (0.2)   (8.31) (− 150) J ≈ − 623 J
= =
 2 W Q1 − Q 2
∴ ∆WBC = − ∆U BC = 623 J Q2
Process CA is an isobaric process. Coefficient of performance, β =
Q1 − Q 2
7 
Hence, ∆QCA = nC p ∆T = n  R  (TA − TC )
2  ● A perfect refrigerator is one which transfers heat from cold
 7 to hot body without doing work.
= (0.2)   (8.31) (300 − 450) ≈ − 872 J
 2 i.e. W = 0, so that Q1 = Q 2 and hence, β = ∞
Laws of Thermodynamics 713

● For an ideal refrigerator or heat engine, we can take


Q 2 T2
=
SECOND LAW OF
Q1 T1 THERMODYNAMICS
● Air conditioners are basically refrigerators whose The second law of thermodynamics gives a fundamental
refrigerated space is a room or a building. limitation to the efficiency of a heat engine and the
● In domestic refrigerator, inside portion acts as a cold coefficient of performance of a refrigerator . It says that
reservoir and the surrounding act as hot reservoir. efficiency of a heat engine can never be unity (or 100%).
This implies that heat released to the cold reservoir can
Relation between coefficient of performance and never be made zero.
efficiency of heat engine For a refrigerator, the second law says that the coefficient
We know, β=
Q2
=
Q2 / Q1
…(i) of performance can never be infinite. This implies that
Q1 − Q2 1 − Q2 / Q1 external work can never be zero.
But η = 1−
Q2
or
Q2
= 1− η …(ii) Second law of thermodynamics may be stated in various
Q1 Q1 ways, but two statements are worth mentioning which are
1− η as follows
From Eqs. (i) and (ii), we get β =
η
Kelvin-Planck statement
No process is possible which can completely convert heat
Example 15.46 An ideal refrigerator runs between −23° C absorbed from a reservoir into work.
and 27° C. Find the heat rejected to atmosphere for every
joule of work input. Clausius statement
Sol. Let the heat rejected to the atmosphere, Q1 = x No process is possible which can transfer heat from a cold
body to a hot body without doing any external work.
and givenW = 1 J. Now, Q 2 = Q1 − W = x − 1
Given, hot temperature, T1 = 273 + 27 = 300 K
and cold temperature, T2 = 273 − 23 = 250 K Reversible and irreversible processes
For an ideal refrigerator, we know that, Reversible process
Q1 T1
= A process which could be reversed in such a way that the
Q 2 T2 system and its surrounding returns exactly to their initial
x 300 states with no other changes in the surroundings is known
∴ =
x − 1 250 as reversible process.
or x=6J e.g. If heat is absorbed in the direct process, then same
amount of heat should be given out in the reverse process.
Example 15.47 Calculate the amount of work that must
be done to freeze one gram of water at 0°C by means of an If work is done on the working substance in the direct
ideal refrigerator. Temperature of surroundings is 27°C. process, then the same amount of work should be done by
How much heat is passed to the surroundings in this process. the working substance in the reverse process.
(Take, latent heat of fusion, L = 80 cal g −1.) The conditions for reversibility are as follows
Sol. Heat released during freezing, (i) There must be complete absence of dissipative forces
Q 2 = mL = 1 × 80 = 80 cal such as friction, viscosity, electric resistance, etc.
T2 = 0° C = 273 K (ii) The direct and reverse processes must take place
and T1 = 27° C = 300 K infinitely slowly.
Q2 Q2 T2
Since, = = (iii) The temperature of the system must not differ
W Q1 − Q 2 T1 − T2 appreciably from its surroundings.
Q 2 (T1 − T2 )
∴ Work done, W =
T2 Irreversible process
80 (300 − 273) Any process which is not reversible exactly is an irreversible
= = 7.91 cal
273 process. All natural processes such as conduction,
⇒ Heat passed to the surrounding, radiation, radioactive decay, etc. are irreversible processes.
Q1 = Q 2 + W = (80 + 7.91) = 87.91cal
714 OBJECTIVE Physics Vol. 1

p
( p1, V1, T1)
Carnot engine A Q1 ( p2, V2, T1)
B
The reversible engine which operates between two Isothermal T1

compression
expansion
temperatures of source and sink is known as Carnot heat

Adiabatic

Ad ansio
exp
engine. The designed engine is a theoretical engine which W=Q1 –Q2

iab
atic n
is free from all the defects of a practical engine.
This engine cannot be realised in actual practice, however D
this can be taken as a standard against which the ( p4, V4, T2) Isothermal
compression Q2 C(p3, V3, T2) T2
performance of an actual engine can be judged.
O E F G H V
Cylinder with
Fig. 15.28 Various processes in Carnot cycle
conducting base

Work done = Heat absorbed by the gas


V2 V 
W1 = Q1 = ∫V 1
pdV = RT1 log e  2 
 V1 
Ideal gas
Working = Area of ABGE
substance
Second stroke (Adiabatic expansion) (Curve BC) The
Source Insulated Sink cylinder is then placed on the non-conducting stand and
T1 stand T2
the gas is allowed to expand adiabatically till the
Fig. 15.27 A Carnot engine temperature falls from T1 to T 2 .
V3 R
W2 = ∫ pdV = (T1 − T 2 )
The main parts of Carnot engine are as follows
V2 (γ − 1)
Cylinder The cylinder has conducting base and = Area of BCHG
insulating walls. It contain the gas as a working substance Third stroke (Isothermal compression) (Curve CD) The
and an insulating and frictionless piston is attached with it cylinder is placed on the sink and the gas is compressed at
as shown in the figure. constant temperature T 2 .
Source It is a hot reservoir at a temperature T1 with Work done = Heat released by the system
conducting walls. It has infinite thermal capacity. Any V4 V
amount of heat can be taken from it without changing the W3 = Q 2 = − ∫ pdV = − RT 2 log e 4
V3 V3
temperature.
V3
Sink It is a cold reservoir at temperature T 2 . It has an = RT 2 log e = Area of CDFH
V4
infinite thermal capacity. So, any amount of heat can be
rejected to it without changing the temperature of the sink. Fourth stroke (Adiabatic compression) (Curve
DA) Finally, the cylinder is again placed on
Working substance We use an ideal gas as a working
non-conducting stand and the compression is continued, so
substance in the cylinder.
that gas returns to its initial stage.
Insulating stand The base of the cylinder could be V1 R
placed on the insulated stand, to isolate it completely from W4 = − ∫ pdV = − (T 2 − T1 )
V4 γ −1
the surroundings.
R
= (T − T 2 ) = Area of ADFE
Carnot cycle γ −1 1
As the engine works, the working substance of the engine
undergoes a cycle known as Carnot cycle. Efficiency of the carnot cycle
The Carnot cycle consists of the following four strokes The efficiency of engine is defined as the ratio of net work
done to the heat supplied, i.e.
First stroke (Isothermal expansion) (Curve AB) The
Net work done Wnet
cylinder containing one mole of an ideal gas as working η= =
substance is allowed to expand slowly at the constant Heat input Q1
temperature T1 by putting it on the source. W1 +W2 +W3 +W4
=
Q1
Laws of Thermodynamics 715

Q2  T 
= 1− Efficiency of the engine, η = 1 − 2  × 100
Q1  T1 
On further calculation, we get  300 
or η = 1 −  × 100 = 72.72%
Q 2 T2  1100
=
Q1 T1 Example 15.50 The efficiency of a Carnot engine at a
particular source and sink temperature is 1/2. When the
T2
∴ Efficiency of Carnot engine, η = 1 − sink temperature is reduced by 100°C, the engine’s
T1 efficiency becomes 2/3. Find the source temperature.
T2 1
● The efficiency of the Carnot engine depends only on the Sol. According to Carnot engine, 1 − = …(i)
T1 2
temperatures of source and sink.
(T2 = sink temperature, T1 = source temperature)
● The efficiency of Carnot engine does not depend on the
(T − 100) 2
nature of the working substance. and 1− 2 = …(ii)
T1 3
● The efficiency of Carnot engine will be 100%, if the
T2 1
temperature of the sink is 0 K. As practically, we cannot or, = [From Eq. (i)]
attain a sink at 0 K, so it is not possible to have 100% T1 2
efficiency. (T2 − 100) 1
and = [From Eq. (ii)]
T1 3
Carnot theorem On dividing Eq. (i) by Eq. (ii), we get
According to Carnot theorem, T2 3
= or T2 = 300 K
(i) A heat engine working between the two given T2 − 100 2
temperatures T1 of hot reservoir, i.e. source and T 2 of So, T1 = 600 K
cold reservoir, i.e. sink cannot have efficiency more
than that of the Carnot engine. Example 15.51 If a Carnot engine whose heat sink is at
27°C has an efficiency of 40%. By how many degrees should
(ii) The efficiency of the Carnot engine is independent
the temperature of the source be changed to increase the
of the nature of working substance. efficiency by 10% of the original efficiency?
Example 15.48 A Carnot engine takes in 3000 kcal of heat Sol. Given, T2 = 27 + 273 = 300 K, η = 40 %
from a reservoir at 627°C and gives a part of it to a sink at T2
27°C. Find the work done by the engine. As, η=1−
T1
Sol. Here, T1 = 273 + 627 = 900 K and T2 = 273 + 27 = 300 K
T2 40 60 3
W T ∴ =1− η=1− = =
Now, η= =1− 2 T1 100 100 5
Q1 T1
5 5
W 300 ⇒ T1 = × T2 = × 300 = 500 K
⇒ =1− 3 3
3000 kcal 900 ∴ New efficiency, η′ = 40 + 4 = 44%
∴ Work done by the engine, W = 2000 kcal (Q Increase in efficiency = 10% of 40 = 4%)
= 2000 × 4.2 kJ = 8.4 × 106 J Let the new temperature of the source be T1′ K, then
T
Example 15.49 Carnot engine takes in a thousand kilo η′ = 1 − 2
T1′
calories of heat from a reservoir at 827°C and exhausts it to
a sink at 27°C. How much work does it perform? What is 44 300
⇒ =1−
the efficiency of the engine? 100 T1′
Sol. Given, Q1 = 106 cal, T1 = (827 + 273) = 1100 K 300 44 56
⇒ =1− =
and T2 = (27 + 273) = 300 K T1′ 100 100
Q 2 T2 100 × 300
Q = ⇒ T1′ = = 535.7 K
Q1 T1 56
T2  300  ∴ Increase in the temperature of the source,
⇒ Q2 = ⋅ Q1 =   (10 ) = 2.73 × 10 cal
6 5
= 535.7 − 500
T1 1100
= 35.7 K or 35.7° C
716 OBJECTIVE Physics Vol. 1

Example 15.52 A scientist claims to have developed 60% (ii) When temperature of a substance changes from T1
efficient engine while working between 27°C and 327°C. dQ
T2
T 
dT
Does he claim right? to T 2 , then dS = ∫ = ms ∫ = ms ln  2 
Sol. Here, sink temperature, T2 = 273 + 27 = 300 K
T T1
T  T1 
T 
(Low temperature happens to be sink temperature) = 2.303 ms log 10  2 
and source temperature, T1 = 327 + 273 = 600 K  T1 
So, within this temperature difference, the maximum possible where, m = mass of the substance
T
efficiency is ηmax = 1 − 2 = 1 −
300
= 0.5 = 50% and s = specific heat of the substance.
T1 600
Example 15.53 200 g of ice at 0°C is converted into water
But the scientist is claiming more efficiency. So, as per Carnot vapour at 100°C. Evaluate the change in entropy.
theorem his claim is not correct.
Sol. Consider conversion of ice into water
ice → water
Entropy 0° C 0° C

Like pressure, volume, temperature, internal energy, etc., Change in entropy during this conversion,
we have another thermodynamic variable of a system, ∆Q mL ice 200 × 80
∆S1 = = =
named entropy. It is related to the disorder of molecular T1 T1 273
motion of the system. Greater the randomness or disorder, = 58.6 cal/K
greater is the entropy.
Again, consider rise of temperature of water from 0°C to
The change in entropy, 100°C.
Heat absorbed by the system ∆Q water → water
i.e. ∆S = = 0° C 100 ° C
Absolute temperature T
Change in entropy for this increase in temperature of the
This relation is also called mathematical form of second law water,
of thermodynamics. T  373
∆S 2 = msw ln 2 = 200 × 1 × ln  
SI unit of ∆S is J/K and its dimensional formula is T1  273
[ML2 T −2 K −1]. = 200 ln (1.4) cal/K
Note We always define change in entropy rather than its absolute Again, consider conversion of water to vapour
value. water → vapour
100 ° C 100 ° C
Change in entropy for solids and liquids Change in entropy for this conservation is
(i) When heat is supplied to a solid and its state changes ∆Q mL steam 200 × 540
∆S 3 = = =
such that temperature remains constant, then T2 T2 373
∆Q = 289.5 cal/K
Change in entropy, ∆S =
T Net change in entropy, ∆S = ∆S1 + ∆S 2 + ∆S 3
dQ ± |mL| = 58.6 + 200 ln(1.4) + 289.5
⇒ dS = = = 348.1 + 200 × 2.303 log10 (1.4)
T T
= 415.4 cal/K
Positive sign is used for heat absorption and negative
sign is used for heat rejection.

CHECK POINT 15.4


1. A system undergoes a cyclic process in which it absorbs Q1 6. For which combination of temperatures, the efficiency of
heat and gives out Q2 heat. The efficiency of the process is η Carnot’s engine is highest?
and work done is W. Select the correct statement. (a) 80 K, 60 K
Q1 − Q2 (b) 100 K, 80 K
(a) W = (c) 60 K, 40 K
Q2
W (d) 40 K, 20 K
(b) η =
Q1 7. A Carnot engine is made to work between 200°C and 0°C
(c) Both (a) and (b) are correct first and then between 0° and − 200° C. The ratio of
(d) Both (a) and (b) are wrong  η 2
efficiencies   of the engine in the two cases is
2. An ideal heat engine is operating between 227°C and 127°C.  η1 
It absorbs104 J amount of heat at the higher temperature. (a) 1 : 1.5 (b) 1 : 1
The amount of heat converted into work is (c) 1 : 2 (d) 1.73 : 1
(a) 2000 J (b) 4000 J
(c) 8000 J (d) 5600 J
8. A Carnot engine with its cold body at 17°C has 50%
efficiency. If the temperature of its hot body is now
3. The inside and outside temperature of a refrigerator are increased by 145°C, the efficiency becomes
273 K and 303 K, respectively. Assuming that refrigerator (a) 55% (b) 60%
cycle is reversible, for every joule of work done, the heat (c) 40% (d) 45%
delivered to the surroundings will be
9. The heat reservoir of an ideal Carnot engine is at 800 K and
(a) 10 J (b) 20 J
its sink is at 400 K. The amount of heat taken in it in one
(c) 30 J (d) 50 J
second to produce useful mechanical work at the rate of
4. A Carnot engine has an efficiency of 50% at sink 750 J is
temperature 50°C. Calculate the temperature of source. (a) 2250 J (b) 1125 J
(a) 133°C (b) 143°C (c) 1500 J (d) 750 J
(c) 100°C (d) 373°C
10. Entropy of a system is related with
5. If a Carnot engine functions at source temperature = 127° C (a) disorder of molecular motion of the system
and sink temperature = 87° C, what is its efficiency? (b) volume of the system
(a) 10% (b) 25% (c) pressure of the system
(c) 40% (d) 50% (d) None of the above
Chapter Exercises
(A) Taking it together
Assorted questions of the chapter for advanced level practice
1 Two identical containers joined by a small pipe 7 The p-V diagram of a system undergoing
initially contain the same gas at pressure p 0 and thermodynamic transformation as shown in figure.
absolute temperature T 0 . One container is now The work done by the system is going from
maintained at the same temperature while the other A → B → C is 30 J and 40 J heat is given to the
is heated to 2T 0 . The common pressure of the gases system. The change in internal energy between A
will be and C is
3 4 5 p C
(a) p0 (b) p0 (c) p0 (d) 2p 0
2 3 3
2 An ideal monoatomic gas is compressed (no heat
being added or removed in the process), so that its
A B
volume is halved. The ratio of the new pressure to
V
the original pressure is
3/ 5 4/ 3 3/ 4 5/ 3 (a) 10 J (b) 70 J (c) 84 J (d) 134 J
(a) (2) (b) (2) (c) (2) (d) (2)
3 If a Carnot’s engine functions at source temperature 8 A thermally insulated rigid container contain an
= 137° C and sink temperature = 97° C, what is its ideal gas heated by a filament of resistance 100 Ω
efficiency? through a current of 1 A for 5 min, then change in
(a) 9.75% (b) 10.5% (c) 40% (d) 50% internal energy is
4 A gas is compressed at a constant pressure of 50 Nm −2 (a) 0 kJ (b) 10 kJ (c) 20 kJ (d) 30 kJ
3 3 9 In the p-V diagram as p
from a volume of 10 m to a volume of 4 m . Energy
of 100 J then added to the gas by heating. Its shown in figure, the net
internal energy is amount of work done will
be 1 2
(a) increased by 400 J (b) increased by 200 J
(c) increased by 100 J (d) decreased by 200 J (a) positive
(b) negative
5 An ideal gas undergoes four V
(c) zero
different processes from the same 4 (d) infinity
initial state (figure). Four p 3
processes are adiabatic, 10 An ideal gas is allowed to expand freely against
2 vacuum in a rigid insulated container. The gas
isothermal, isobaric and 1
isochoric. Out of 1, 2, 3 and 4 undergoes
which one is adiabatic? V (a) an increase in its internal energy
(a) 4 (b) 3 (c) 2 (d) 1 (b) a decrease in its internal energy
(c) Neither increase nor decrease in temperature or
6 The process ∆U = 0, for an ideal gas can be best internal energy
represented in the form of a graph (d) an increase in temperature
p p
B A 11 An ideal gas mixture filled inside a balloon expands
according to the relation pV 2/ 3 = constant. The
(a) A (b) temperature inside the balloon is
B (a) increasing (b) decreasing
T V (c) constant (d) Cannot be defined
p p
B
A B 12 The internal energy of a gas is given by U = 2pV . It
(c) (d)
expands fromV0 to 2V0 against a constant pressure
p 0 . The heat absorbed by the gas in the process is
A (a) 2p 0V0 (b) 4p 0V0 (c) 3p 0V0 (d) p 0V0
V T
Laws of Thermodynamics 719

13 An ideal monoatomic gas undergoes the process AB 18 One mole of a perfect gas in a cylinder fitted with a
as shown in the figure. If the heat supplied and the piston has a pressure p, volume V and temperature
work done in the process are ∆Q and ∆W, T. If the temperature is increased by 1 K keeping
respectively. The ratio ∆Q : ∆W is pressure constant, the increase in volume is
V
2V V V
B (a) (b) (c) (d) V
273 91 273
19 Ideal monoatomic gas is taken through a process
dQ = 2dU. The molar heat capacity for the process is
A (where, dQ is heat supplied and dU is change in
T internal energy)
(a) 2.50 (b) 1.67 (a) 2.4 R (b) 3R (c) R (d) 2R
(c) 1.25 (d) 0.40 20 Heat energy absorbed by a system is going through a
14 The figure shows two paths for the change of state cyclic process as shown in figure, is
of a gas from A to B. The ratio of molar heat

V (in litre)
capacities in path 1 and path 2 is 30

p
2
10
A B
1
10 30
V p (in kPa)

(a) < 1 (b) > 1 (a) 107 π J (b) 104 π J (c) 102 π J (d) 103 π J
(c) 1 (d) data insufficient
21 Unit mass of a liquid with volumeV1 is completely
15. An ideal gas expands in such a manner that its changed into a gas of volumeV2 at a constant
pressure and volume can be related by equation external pressure p at temperature T. If the latent
pV 2 = constant. During this process, the gas is heat of evaporation is L, then the increase in the
(a) heated internal energy of the system is
(b) cooled (a) zero (b) p (V2 − V1 )
(c) neither heated nor cooled (c) L − p V
( 2 − V1 ) (d) L
(d) first heated and then cooled 22 A monoatomic ideal gas, initially at temperature T1,
16 p-V diagram of a diatomic gas is a straight line is enclosed in a cylinder fitted with a frictionless
passing through origin. The molar heat capacity of piston. The gas is allowed to expand adiabatically to
the gas in the process will be a temperature T 2 by releasing the piston suddenly. If
(a) 4R (b) 2. 5R L1 and L 2 are lengths of the gas column before and
4R after expansion respectively, then T1 /T 2 is given by
(c) 3R (d)
3 2/ 3 5/ 3 5/ 3 2/ 3
L  L  L  L 
17 Identify the graph(s) which correctly represents an (a)  1  (b)  1  (c)  2  (d)  2 
 L2   L2   L1   L1 
isotherm at two temperatures T1 and T 2 (> T1 ).
p
23 A refrigerator, whose coefficient of performance β is
p
4, extracts heat from the cooling compartment at the
rate of 400 J per cycle. How much work per cycle is
(a) (b) required to operate the refrigerator?
T2 T1
T1 T2 (a) 100 J (b) 200 J
O V O V (c) 50 J (d) 25 J
p T2 T1 p T1 T2 24 Two pistons can move freely
inside a horizontal cylinder
having two sections of
(c) (d)
unequal cross-sections. The
pistons are joined by an inextensible, light string and
O V O V
some gas is enclosed between the pistons.
720 OBJECTIVE Physics Vol. 1

On heating the system, the piston will 32 One mole of an ideal monoatomic gas is at 360 K
(a) move to the left and a pressure of 10 5 Pa. It is compressed at
(b) move to the right constant pressure until its volume is halved. Taking
(c) remain stationary
R as 8.3 J mol−1 K −1 and the initial volume of the gas
(d) (a) or (c) depending upon the initial pressure of the gas
as 3.0 × 10 −2 m3 , the work done on the gas is
25 Starting with the same initial conditions, an ideal gas (a) −1500 J (b) + 1500 J
expands from volumeV1 toV2 in three different (c) −3000 J (d) + 3000 J
ways. The work done by the gas isW1, if the process
is isothermalW2 , if isobaric andW3 , if adiabatic, then 33 If an average person jogs, he produces
(a) W2 > W1 > W3 (b) W2 > W3 > W1 145 × 10 3 cal/min. This is removed by the
(c) W1 > W2 > W3 (d) W1 > W3 > W2 evaporation of sweat. The amount of sweat
evaporated per minute (assuming 1 kg requires
26 The pressure (1 × 10 5 Nm−2 ) of the air filled in a
580 × 10 3 cal for evaporation) is
vessel is decreased adiabatically so much as to
(a) 0.25 kg (b) 2.25 kg
increase its volume three times. (c) 0.05 kg (d) 0.20 kg
The air pressure is (γ for air = 14
.,
log 10 3 = 0.4771, log 10 2.148 = 0.33206) 34 An ideal monoatomic gas undergoes a process in
−2 −2 which its internal energy U and density ρ vary as
(a) 2.14 × 10 Nm4
(b) 2.19 × 10 Nm
5
Uρ = constant. The ratio of change in internal energy
(c) 1.14 × 105 Nm−2 (d) 3.14 × 104 Nm−2 and the work done by the gas is
27 The equation of a state of a gas is given by 3 −2 1 3
(a) (b) (c) (d)
( − b ) = nRT . 1 mol of a gas is isothermally
pV 2 3 3 5
expanded from volume V to 2V, the work done 35 The molar heat capacity in a process of a diatomic
during the process is gas, if it does a work of Q /4 when a heat of Q is
 2V − b  V − b  supplied to it is
(a) RT ln   (b) RT ln  
V − b   V  2 5 10 6
(a) R (b) R (c) R (d) R
5 2 3 7
V − b   V 
(c) RT ln   (d) RT ln   36 The amount of heat required to raise the
 2V − b  V − b 
temperature of 1 mole of a monoatomic gas from
28 A monoatomic gas is supplied the heat Q very slowly 20°C to 30°C at constant volume is H. Then, the
keeping the pressure constant. The work done by amount of heat required to raise the temperature of
the gas will be 2 mol of a diatomic gas from 20°C to 25°C at constant
2 3 pressure is
(a) Q (b) Q
3 5 4 5 7
(a) 2H (b) H (c) H (d) H
2 1 3 3 3
(c) Q (d) Q
5 5 37 p-V diagram of a cyclic p
A
29 70 cal of heat are required to raise the temperature process ABCA is as shown
of 2 mol of an ideal gas at constant pressure from in figure. Choose the
30°C to 35°C. The amount of heat required in calories correct option.
to raise the temperature of the same gas through the (a) ∆Q A → B = negative
same range (30°C-35°C) at constant volume is B
(b) ∆U B → C = positive C
(a) 30 cal (b) 50 cal (c) 40 cal (d) 90 cal V
(c) ∆WCAB = negative
30 The temperature of a hypothetical gas increases to (d) All of the above
2 times when compressed adiabatically to half the 38 Two different ideal diatomic gases A and B are
volume. Its equation can be written as initially in the same state. A and B are then
(a) pV 3 / 2 = constant (b) pV 5 / 2 = constant expanded to same final volume through adiabatic
and isothermal process, respectively. If p A , p B and
(c) pV 7 / 3 = constant (d) pV 4 / 3 = constant
T A , TB represents the final pressure and
31 A gas mixture consists of 4 mol of oxygen and 6 mol temperatures of A and B respectively, then
of argon at temperature T. Neglecting all vibrational (a) p A < pB and TA < TB (b) p A > pB and TA > TB
modes, the total internal energy of the system is (c) p A > pB and TA < TB (d) p A < pB and TA > TB
(a) 4RT (b) 14RT (c) 8RT (d) 19RT
Laws of Thermodynamics 721

39 One mole of a monoatomic gas is carried along (a) 300 K (b) greater than 300 K
process ABCDEA as shown in the diagram. Find the (c) less than 300 K (d) Data insufficient
net work done by gas 44 A Carnot engine whose low temperature reservoir is
at 7° C has efficiency of 50%. It is desired to
B C
4 increase the efficiency to 70%. By how many
degrees should the temperature of the high
)
—2

2 temperature reservoir be increased?


p (Nm

A
(a) 933 K (b) 432 K
1 E (c) 373 K (d) 267 K
D
3 45 A certain mass of an ideal gas is at pressure p 1 and
V (m )
1 2 3 volumeV1. It is compressed isothermally and then
3 1 allowed to expand adiabatically until its pressure
(a) J (b) 1 J (c) J (d) 0 J returns to p 1. The gas is then allowed to expand its
2 2
original volume.
40 The p-V diagram of 2 g of helium gas for a certain Upon which of the following p-V graphs are these
process A → B is shown in the figure. What is the processes correctly shown?
heat given to the gas during the process A → B ?
p p
p
2p0 B
(a) p1 (b) p1
p0
A
V V V
V0 2V0 V1 V1
p p
(a) 4p 0V0 (b) 6p 0V0 (c) 3p 0V0 (d) 2p 0V0
41 A gas undergoes A to B through three different
(c) p1 (d) p1
processes 1, 2, and 3 as shown in the figure. The
heat supplied to the gas is Q1, Q 2 and Q 3
respectively, then V V
V1 V1
p
B
1 3 46 Two different masses of a gas m and 2m are heated
2 separately in vessels of equal volume. The T-p curve
for mass 2m makes angle α with T-axis and that for
mass m makes angle β with T-axis, then
A
V (a) tan α = tan β (b) tan α = 2 tan β
(c) tan β = 2 tan α (d) None of these
(a) Q1 = Q 2 = Q 3 (b) Q1 < Q 2 < Q 3
(c) Q1 > Q 2 > Q 3 (d) Q1 = Q 3 > Q 2 47 In a cyclic process shown in the figure, an ideal gas
42 When an air bubble rises from the bottom to the is adiabatically taken from B to A, the work done on
surface of a lake, its radius becomes double. Find the the gas during the process B → A is 30 J, when the
depth of the lake. (Given that the atmospheric gas is taken from A → B, the heat absorbed by the
pressure is equal to the pressure due to a column of gas is 20 J. The change in internal energy of the gas
water 10 m high. Assume constant temperature and in the process A → B is
disregard surface tension.) p
(a) 30 m (b) 40 m (c) 70 m (d) 80 m A
20 J
43 A container of volume 1 m3 is divided into two equal
compartments by a partition. One of these 30 J B
compartments contains an ideal gas at 300 K. The
other compartment is vacuum. The whole system is V
thermally isolated from its surroundings. The
(a) 20 J (b) −30 J
partition is removed in the gas container. Its (c) 50 J (d) −10 J
temperature now would be
722 OBJECTIVE Physics Vol. 1

48 n moles of an ideal gas undergo a process in which 53 When a system is taken from state i to a state f along
the temperature changes with volume as T = KV 2 . path iaf, Q = 50 J andW = 20 J. IfW = −13 J for the
The work done by the gas as the temperature curved return path fi, Q in this path is
changes from T 0 to 4T 0 is
p
 5
(a) 3nRT0 (b)   nRT0
 2 a f
 3
(c)   nRT0 (d) zero
 2
49 One mole of a gas expands with temperature T such
i
that its volumeV = kT 2, where k is a constant. If the V
temperature of the gas changes by 60°C, then the
work done by the gas is (a) 33 J (b) 23 J (c) −7 J (d) −43 J
(a) 120R (b) R ln 60 (c) kR ln 60 (d) 60kR 54 Two Carnot’s engines A and B are operated in
50 An ideal monoatomic gas undergoes a process in which succession. The first one, A receives heat from a
the gas volume relates to temperature asVT = constant. source at T1 = 800 K and rejects to sinks at T 2 K.
Then, molar specific heat of gas in this process is The second engine B receives heat rejected by
R the first engine and rejects to another sink at
(a) (b) R
2 T 3 = 300 K. If the work outputs of two engines are
3R equal, then the value of T 2 is
(c) (d) None of these (a) 100 K (b) 300 K
2
(c) 550 K (d) 700 K
51 One mole of an ideal gas with heat capacity at
constant pressure C p undergoes the process 55 Consider p-V diagram for an ideal gas shown in
T = T 0 + αV , where T 0 and α are constants. If its figure.
volume increases fromV1 toV2 , the amount of heat p 1
transferred to the gas is constant
p=
V 
(a) C p RT0 ln 2  V
 V1 
V  2
( 2 − V1 ) − RT0 ln 2 
(b) αC p V V
 V1 
V  Out of the following diagrams, which figure
( 2 − V1 ) + RT0 ln 2 
(c) αC p V represents the T-p diagram?
 V1 
T T
V 
(d) RT0 ln 2  − αC p V
( 2 − V1 ) 2 2
 V1  (i) (ii)
52 A cyclic process for 1 mol of an ideal gas is 1 1
shown in the V-T diagram. The work done in
p p
AB, BC and CA respectively are
T T
V1 C

(iii) 2 1 (iv) 1 2
V
A
V2 B p p

(a) (iv) (b) (ii) (c) (iii) (d) (i)


T1 T2
T 56 A sample of 100 g water is slowly heated from 27°C
to 87°C. What is the change in the entropy of the
V  V 
(a) 0, RT2 ln 1 , R (T2 − T1 ) (b) R (T1 − T2 ), 0, RT1 ln 1  water? (Take, specific heat capacity of water
V2  V2  = 4200 J/kg-K)
V  V  (a) 420 ln (1.2) JK −1 (b) 220 ln (1.4) JK −1
(c) 0, RT2 ln 1 , R (T1 − T2 ) (d) 0, RT2 ln 2 , R (T2 − T1 )
 2
V  V1  (c) 120 ln (1.5) JK −1 (d) 320 ln (1.2) JK −1
Laws of Thermodynamics 723

p
57 An ideal gas is taken through a cyclic 6p0 B
thermodynamic process through four steps. The
amounts of heat involved in these steps are 3p0
A
Q1 = 5960 J, Q 2 = − 5585 J, Q 3 = 2980 J and
Q 4 = 3645 J, respectively. The corresponding works V
involved areW1 = 2200 J,W2 = − 825 J,W3 = − 1100 J V0 5V0

andW4 = 482.4 J, respectively. 3R 13R 5R


(a) (b) (c) (d) 2R
What is the efficiency of the cycle? 2 6 2
(a) 10.82 % (b) 11.42 % 61 Carbon monoxide is carried around a closed cycle
(c) 9.32 % (d) 5.82 % abca in which bc is an isothermal process as shown
58 Consider two containers A and B containing identical in the figure. The gas absorbs 7000 J of heat as its
gases at the same pressure, volume and temperature. temperature increases from 300 K to 1000 K in
The gas in container A is compressed to half of its going from a to b.
original volume isothermally while the gas in p
container B is compressed to half of its original value p2
b
adiabatically. The ratio of final pressure of gas in B
to that of gas in A is
γ −1 2 2
 1  1   1 
(a) 2γ −1
(b)   (c)   (d)  
 2 1 − γ   γ − 1 p1
a
c

59 A sample of an ideal gas is taken through a cycle as V


shown in figure. It absorbs 50 J of energy during the V1 V2
process AB, no heat during BC, rejects 70 J during The quantity of heat rejected by the gas during the
CA. 40 J of work is done on the gas during BC. process ca is approximately
Internal energy of gas at A is 1500 J, the internal (a) 4200 J (b) 5000 J
energy at C would be (c) 9000 J (d) − 9800 J
p
B 62 Six moles of an ideal gas performs a cycle as shown
in figure. If the temperatures are T A = 600 K,
TB = 800 K, TC = 2200 K and TD = 1200 K, the
work done per cycle is approximately
C A p B C
V

(a) 1590 J (b) 1620 J (c) 1540 J (d) 1570 J A D

60 One mole of a monoatomic ideal gas undergoes the


process A → B in the given p-V diagram. The molar T
heat capacity for this process is (a) 20 kJ (b) 30 kJ (c) 40 kJ (d) 60 kJ

(B) Medical entrance special format questions


Assertion and reason
1 Assertion During melting of ice, work done by
Directions (Q. Nos. 1-4) These questions consists of two statements
surrounding on (ice + water) system is positive.
each printed as Assertion and Reason. While answering these
questions you are required to choose any one of the following four Reason Volume of the given system decreases
responses on melting of ice.
(a) If both Assertion and Reason are correct and Reason is the 2 Assertion If initial and final volumes are
correct explanation of Assertion. equal, then work done by gas is zero.
(b) If both Assertion and Reason are correct but Reason is not the Reason In isochoric process, initial and final
correct explanation of Assertion. volumes are equal and work done by gas is
(c) If Assertion is correct but Reason is incorrect. zero.
(d) If Assertion is incorrect but Reason is correct.
724 OBJECTIVE Physics Vol. 1

3 Assertion If volume of a gas is increasing but III. Change in the internal energy in cycle = 0
temperature of the gas is decreasing, then heat given Which of these are correct?
to the gas may be positive, negative or zero. (a) Only I (b) Only II
(c) II and III (d) I, II and III
Reason Heat given to a gas is a state function. It is
not path function. 4 Which of the following statement(s) is/are correct?
4 Assertion There are two processes : Process-1 is I. Efficiency of any heat engine cannot be greater
than the efficiency of Carnot engine.
pV = constant and process-2 is pV 2 = constant. In
II. Heat flow never takes place from a body at lower
both the processes, volume of gas is increased from temperature to a body at higher temperature.
V1 toV2 . Initial coordinates (p 1, V1 ) of the gas were (a) Only I (b) Only II
same. Then, more work is done by the gas in (c) Both I and II (d) None of these
process-1.
Reason In second process, pressure drops more Match the columns
rapidly with increase in volume.
1 Match the following columns and mark the correct
option from the codes given below.
Statement based questions
1 Pressure versus density graph of an ideal gas is Column I Column II
shown in figure. Which of the following statement is (A) Adiabatic bulk modulus (p) − (p /V )
correct? (B) Slope of p-V graph in isothermal (q) 2
p
C process γ −1

B (C) Degree of freedom (r) γp

(D) Molar heat capacity at constant (s) γ


D pressure divided by R γ −1
A
ρ
Codes
(a) During the process, AB work done by the gas is positive. A B C D
(b) During the process, AB work done by the gas is (a) s r p q
negative. (b) r p q s
(c) During the process, BC internal energy of the gas is (c) r s q p
increasing. (d) s p q r
(d) None of the above
2. For one mole of a monoatomic gas, match the
2 At 27°C, two moles of an ideal monoatomic gas following columns and mark the correct option from
occupy a volume V. The gas expands adiabatically to the codes given below.
a volume 2V. Which of the following statement(s)
is/are correct? Column I Column II
(a) Final temperature of the gas is 189 K. (A) Isothermal bulk modulus (p) − RT
(b) Change in its internal energy, − 2767.235. V2
(c) The work done by the gas during the process is 2767.235.
(d) All are correct (B) Adiabatic bulk modulus (q) − 5p
3V
3 In the cyclic process ABCDA as shown in the figure,
(C) Slope of p-V graph in (r) T
consider the following statements.
isothermal process V
p A
(D) Slope of p-V graph in (s) 4T
B
adiabatic process 3V

(t) None
D
C Codes
V
A B C D A B C D
(a) p q r s (b) p s r q
I. Area ABCD = Work done on the gas
(c) t t p q (d) t s r p
II. Area ABCD = Net heat absorbed
(C) Medical entrances’ gallery
Collection of questions asked in NEET & Various Medical Entrance Exams

1 Two cylinders A and B of equal capacity are (a) Both Assertion and Reason are correct and Reason is the
connected to each other via a stop cock. A contains correct explanation of Assertion.
an ideal gas at standard temperature and pressure. B (b) Both Assertion and Reason are correct but Reason is not
the correct explanation of Assertion.
is completely evacuated. The entire system is (c) If Assertion is correct but Reason is incorrect.
thermally insulated. The stop cock is suddenly (d) Both Assertion and Reason are incorrect.
opened. The process is [NEET 2020]
(a) adiabatic (b) isochoric 8 The temperature of food material in refrigerator is
(c) isobaric (d) isothermal 4°C and temperature of environment is 15°C. If
Carnot cycle is used in its working gas, then find its
2 The efficiency of a Carnot engine depends upon Carnot efficiency. [AIIMS 2019]
[NEET 2020]
(a) the temperature of the sink only (a) 0.038 (b) 0.028 (c) 0.053 (d) 0.072
(b) the temperatures of the source and sink Cp
9 If 7 g N 2 is mixed with 20 g Ar, the of mixture
(c) the volume of the cylinder of the engine CV
(d) the temperature of the source only
will be [AIIMS 2019]
3 The p-V diagram for an ideal gas in a piston cylinder 17 11 17 17
assembly undergoing a thermodynamic process is (a) (b) (c) (d)
6 7 11 13
shown in the figure. The process is [NEET 2020]
10 In an isobaric process, the work done by a di-atomic
p Initial gas is 10 J, the heat given to the gas will be
state Final [AIIMS 2019]
state (a) 35 J (b) 30 J (c) 45 J (d) 60 J
11 A sample of 0.1 g of water at 100° C and normal
V pressure (1.013 × 10 5 Nm−2 ) requires 54 cal of heat
energy to convert to steam at 100° C. If the volume
(a) adiabatic (b) isochoric (c) isobaric (d) isothermal of the steam produced is 167.1 cc, the change in
3
4 1g of water of volume 1 cm at 100°C is converted internal energy of the sample is [NEET 2018]
into steam at same temperature under normal (a) 42.2 J (b) 208.7 J (c) 104.3 J (d) 84.5 J
atmospheric pressure (~− 1 × 10 5 Pa). The volume of 12 The efficiency of an ideal heat engine working
steam formed equals 1671 cm 3 . If the specific latent between the freezing point and boiling point of
heat of vaporisation of water is 2256 J/g, the change water is [NEET 2018]
in internal energy is [NEET (Odisha) 2019] (a) 6.25% (b) 20% (c) 26.8% (d) 12.5%
(a) 2423 J (b) 2089 J 13 The volume (V ) of a monoatomic gas varies with its
(c) 167 J (d) 2256 J temperature (T ) as shown in the graph. The ratio of
5 In which of the following processes, heat is neither work done by the gas, to the heat absorbed by it,
absorbed nor released by a system? [NEET 2019] when it undergoes a change from state A to state B,
(a) Adiabatic (b) Isobaric is [NEET 2018]
V
(c) Isochoric (d) Isothermal
 Cp 
6 The value of γ  =  , for hydrogen, helium and
 CV  A
B

another ideal diatomic gas X (whose molecules are


O T
not rigid but have an additional vibrational mode)
are respectively equal to [NEET (Odisha) 2019] (a) 1/3 (b) 2/3 (c) 2/5 (d) 2/7
7 5 9 5 7 9 5 7 7 7 5 7 14 Assertion In isothermal process, whole of the heat
(a) , , (b) , , (c) , (d) , ,
5 3 7 3 5 7 3 5 5 5 3 5 energy supplied to the body is converted into
7 Assertion In adiabatic process, work is independent internal energy.
of path. Reason According to the first law of
thermodynamics,
Reason In adiabatic process, work done is equal to
negative of change in internal energy. [AIIMS 2019] ∆Q = ∆U + ∆W [AIIMS 2018]
726 OBJECTIVE Physics Vol. 1

(a) Both Assertion and Reason are correct and Reason is 21 Thermodynamic processes are indicated in the
the correct explanation of Assertion. following diagram
(b) Both Assertion and Reason are correct but Reason is not p
the correct explanation of Assertion.
(c) Assertion is correct but Reason is incorrect.
(d) Assertion is incorrect but Reason is correct. IV
f
15 An ideal gas of mass m in a state A goes to another I III
state B via three different processes as shown in II f
f 700 K
figure. If Q1, Q 2 and Q 3 denote the heat absorbed by f 500 K
the gas along the three paths, then [AIIMS 2018] 300 K
p V
Match the Column I with Column II and mark the correct
A option from the codes given below.
3 Column I Column II
2
1 A. Process I p. Adiabatic
B B. Process II q. Isobaric
V C. Process III r. Isochoric
(a) Q1 < Q 2 < Q 3 (b) Q1 < Q 2 = Q 3 D. Process IV s. Isothermal
(c) Q1 = Q 2 > Q 3 (d) Q1 > Q 2 > Q 3 [NEET 2017]
16 A gas consisting of a rigid diatomic molecules was Codes
initially under standard condition. Then, gas was A B C D A B C D
compressed adiabatically to one-fifth of its initial (a) p r s q (b) r p s q
volume. What will be the mean kinetic energy of a (c) r s q p (d) s q p r
rotating molecule in the final state? [AIIMS 2018] 1
(a) 1.44 J (b) 4.55 J 22 A Carnot engine having an efficiency of as heat
10
(c) 787.98 × 10−23 J (d) 757.3 × 10−23 J engine, is used as a refrigerator. If the work done on
17 The efficiency of an ideal gas with adiabatic the system is 10 J, then the amount of energy
exponent γ for the shown cyclic process would be absorbed from the reservoir at lower temperature is
[JIPMER 2018] [NEET 2017]
V (a) 1 J (b) 90 J (c) 99 J (d) 100 J
2V0 C
23 One mole of an ideal diatomic gas undergoes
transition from A to B along a path AB as shown
below. [AIIMS 2017]

V0 A
B 5 A
p (kPa)

T
T0 2T0
2 B
(2 ln 2 − 1 ) (1 − 1 ln 2) (2 ln 2 + 1 ) (2 ln 2 − 1 )
(a) (b) (c) (d)
γ / (γ − 1 ) γ / (γ − 1 ) γ / (γ − 1 ) γ / (γ + 1 ) 0
4 6
18 A Carnot engine absorbs 6 × 10 cal at 227°C. The 5
V (in m3)
work done per cycle by the engine, if its sink is The change in internal energy of the gas during the
maintained at 127°C is [JIPMER 2018]
transition is
(a) 15 × 108 J (b) 15 × 104 J (a) 20 kJ (b) − 12 kJ (c) − 20 kJ (d) 20 J
(c) 5 × 105 J (d) 2 × 104 J 24 Assertion Molar heat capacity cannot be defined for
19 If the efficiency of an engine is 50% and its work isothermal process.
output is 500 J, then find the value of input work. Reason In isothermal process, p-V versus T graph is
[JIPMER 2018] a dot. [AIIMS 2017]
(a) 1000 J (b) 500 J (c) 100 J (d) 250 J (a) Both Assertion and Reason are correct and Reason is
20 The efficiency of a heat engine is 1/6. Its efficiency the correct explanation of Assertion.
doubled when the temperature of sink decreased by (b) Both Assertion and Reason are correct but Reason is not
62°C, then what is the temperature of source? the correct explanation of Assertion.
[JIPMER 2018] (c) Assertion is correct but Reason is incorrect.
(a) 470 K (b) 372 K (c) 542 K (d) 1042 K (d) Reason is correct but Assertion is incorrect.
Laws of Thermodynamics 727

25 Assertion In adiabatic expansion process, the (c) which of the case (whether compression through
product of p andV always decreases. isothermal or through adiabatic process) requires more
Reason In adiabatic expansion process, work is done work will depend upon the atomicity of the gas
by the gas at the cost of internal energy of gas. (d) compressing the gas isothermally will require more
[AIIMS 2017] work to be done
(a) Both Assertion and Reason are correct and Reason is
the correct explanation of Assertion. 31 A refrigerator works between 4°C and 30°C. It is
(b) Both Assertion and Reason are correct but Reason is not required to remove 600 cal of heat every second in
the correct explanation of Assertion. order to keep the temperature of the refrigerated
(c) Assertion is correct but Reason is incorrect. space constant. The power required is
(d) Reason is correct but Assertion is incorrect. (Take, 1 cal = 4. 2 J) [NEET 2016]
26 p-V plots for two gases during adiabatic processes as (a) 23.65 W (b) 236.5 W
shown in figure. Plots 1 and 2 should correspond (c) 2365 W (d) 2.365 W
respectively to [JIPMER 2017] 32 Figure below shows two paths that may be taken by
a gas to go from a state A to a state C .
In process AB, 400 J of heat is added to the system
1
and in process BC , 100 J of heat is added to the
p
system. The heat absorbed by the system in the
2 process AC will be [CBSE AIPMT 2015]
V p
4 B C
(a) He and O2 (b) O2 and He 6×10 Pa
(c) He and Ar (d) O2 and N 2
27 The temperature of source and sink of a heat engine
A
are 127°C and 27°C, respectively. An inventor 2×104 Pa
claims its efficiency to be 26%, then [JIPMER 2017] V
2×10–3m3 4×10–3m3
(a) it is impossible
(b) it is possible with high probability (a) 380 J (b) 500 J (c) 460 J (d) 300 J
(c) it is possible with low probability Cp
(d) Data are insufficient 33 The ratio of the specific heats = γ in terms of
CV
28 One mole of an ideal monoatomic gas undergoes a degrees of freedom (n ) is given by [CBSE AIPMT 2015]
process described by the equation pV 3 = constant.  1  n
(a) 1 +  (b) 1 + 
The heat capacity of the gas during this process is  n  3
[NEET 2016]
 2  n
3 5 (c) 1 +  (d) 1 + 
(a) R (b) R  n  2
2 2
(c) 2R (d) R 34 The molar specific heat of a gas as given from the
29 The temperature inside a refrigerator is t 2 ° C and the kinetic theory is
5
R. If it is not specified whether it
room temperature is t1 ° C. The amount of heat 2
delivered to the room for each joule of electrical is C p or C V , one could conclude that the molecules
energy consumed ideally will be [NEET 2016] of the gas [AIIMS 2015]
t1 t1 + 273 t 2 + 273 t1 + t 2 (a) are definitely monoatomic
(a) (b) (c) (d) (b) are definitely rigid diatomic
t1 − t 2 t1 − t 2 t1 − t 2 t1 + 273
(c) are definitely non-rigid diatomic
30 A gas is compressed isothermally to half its initial (d) can be monoatomic or rigid diatomic
volume. The same gas is compressed separately
35 If ∆U represents the increases in internal energy and
through an adiabatic process until its volume is again
W the work done by the thermodynamic system,
reduced to half. Then [NEET 2016]
then [Kerala CEE 2015]
(a) compressing the gas through adiabatic process will (a) ∆U = − W is an adiabatic process
require more work to be done (b) ∆U = W is an isothermal process
(b) compressing the gas isothermally or adiabatically will (c) ∆U = W is an adiabatic process
require the same amount of work (d) ∆U = W is an isochoric process
728 OBJECTIVE Physics Vol. 1

36 If the energy input to a Carnot engine is thrice the 44 A monoatomic gas at a pressure p, having a volume
work it performs, then the fraction of energy V expands isothermally to a volume 2V and then
rejected to the sink is [Kerala CEE 2015] adiabatically to a volume 16V. The final pressure of
(a) 1/3 (b) 1/4 (c) 2/5 (d) 2/3 5
(e) 1/6 the gas is (Take, γ = )
3 [CBSE AIPMT 2014]
37 A gas with γ = 1.4 undergoes the adiabatic process of (a) 64 p (b) 32 p
compression, if the pressure is increased by 0.5%, p
then the volume decreases by (about) [UP CPMT 2015] (c) (d) 16p
64
(a) 0.25% (b) 01
.% (c) 0.5% (d) 0.36%
45 If for hydrogen C p − C V = m and for nitrogen
38 An ideal monoatomic gas is taken round the cycle
C p − C V = n, where C p and C V refer to specific
PQRS as shown in p-V diagram.
heat per unit mass respectively at constant pressure
2p, V 2p, 2V and constant volume, the relation between m and n
S R is (Molecular weight of hydrogen = 2 and molecular
p weight of nitrogen = 28) [AIIMS, UP CPMT 2014]
P Q (a) n = 14 m (b) n = 7m
p, V p, 2V (c) m = 7n (d) m = 14n

V
46 Assertion For monoatomic gas atom, the number of
degrees of freedom is 3.
The work done during the cycle is [CG PMT 2015]
Cp 5
(a) pV (b) 2pV (c) p (d) zero Reason =γ =
CV 3 [AIIMS 2014]
39 The efficiency of a Carnot engine which operates
between the two temperatures T1 = 500 K and (a) Both Assertion and Reason are correct and Reason is
the correct explanation of Assertion.
T2 = 300 K is [KCET 2015]
(b) Both Assertion and Reason are correct but Reason is
(a) 75 % (b) 50% (c) 40% (d) 25% not the correct explanation of Assertion.
40 An ideal diatomic gas is heated at constant pressure. (c) Assertion is correct but Reason is incorrect.
What fraction of heat energy is utilised to increase (d) Assertion is incorrect but Reason is correct.
its internal energy? [EAMCET 2015] 47 The change in internal energy of a thermodynamic
5 2 3 3 system which has absorbed 2 kcal of heat and done
(a) (b) (c) (d)
7 5 5 7 400 J of work is (Take, 1 cal = 4.2 J) [Kerala CEE 2014]
41 How much heat (in joules) is required to raise the (a) 2 kJ (b) 8 kJ
temperature of 1g of water from 14.5°C to 15.5°C at (c) 3.5 kJ (d) 5.5 kJ
1 atmospheric pressure? [UK PMT 2015] (e) 2.5 kJ
(a) 41.86 J (b) 4.186 J 48 The p-T relation for an adiabatic expansion is
(c) 418.6 J (d) None of these
[UK PMT 2014]
42 The increase in internal energy of 1kg of water at (a) p γ T γ −1 = constant (b) p γ −1T γ = constant
100°C when it is converted into steam at the same
(c) p γ T1− γ = constant (d) p1− γ T γ = constant
temperature and at 1 atmospheric pressure will be
(Take, density of steam is 0.6 kg m−3 , latent heat of 49 A Carnot refrigerator extracts heat from water at
vaporisation of water = 2.25 × 10 6 Jkg −1) 0° C and rejects it to room at 24.4° C. The work
[UK PMT 2015] required by the refrigerator for every 1 kg of water
(a) 1.04 × 106 J (b) 2.08 × 106 J converted into ice (latent heat of ice = 336 kJ kg −1)
(c) 4.16 × 10 J
6
(d) 8.32 × 106 J [EAMCET 2014]
(a) 30 kJ (b) 336 kJ
43 Specific heats of one mole of hydrogen at constant
(c) 11.2 kJ (d) 24.4 kJ
pressure and at constant volume are 450 JK −1 and
300 JK −1, respectively. Then, what is the density of 50 A Carnot engine working between 300 K and 600 K
has work output of 800 J cycle −1. The amount of
the gas at STP? (Take, p atm = 1.013 × 10 5 Nm−2 )
heat energy supplied from the source of engine in
[EAMCET 2015]
each cycle is [WB JEE 2014]
(a) 2.47 kg m−3 (b) 3 kg m−3
(a) 800 J (b) 1600 J
(c) 3.58 kg m−3 (d) 4 kg m−3 (c) 3200 J (d) 6400 J
Laws of Thermodynamics 729

51 A gas is taken through the cycle A → B → C → A, Codes


as shown. What is the net work done by the gas? A B C A B C
[NEET 2013] (a) q p r (b) p s q
p (105 Pa) (c) q p s (d) s p q
7 57 Two cylinders A and B fitted with pistons contain
6 B equal amounts of an ideal diatomic gas at 300 K.
5 Piston A is free to move and piston of B is fixed.
4 Same amount of heat is given to the gases in the two
3
2 A cylinders. Temperature of the gas in cylinder A
C
1
increases by 30 K, then increase in temperature of
0 V (10–3 m3)
the gas in the cylinder B is (γ = 14
. for diatomic gas)
2 4 6 8 [EAMCET 2013]
(a) 24 K (b) 36 K (c) 54 K (d) 42 K
(a) 2000 J (b) 1000 J (c) Zero (d) − 2000 J
58 For which combination of working temperatures of
52 During an adiabatic process, the pressure of a gas is
source and sink, the efficiency of Carnot’s heat
found to be proportional to the cube of its
engine is maximum? [KCET 2013]
Cp
temperature. The ratio of for the gas is (a) 600 K, 400 K (b) 400 K, 200 K
CV [NEET 2013] (c) 500 K, 300 K (d) 300 K, 100 K
4 5 3 59 During an adiabatic process of an ideal gas, if p is
(a) (b) 2 (c) (d)
3 3 2 1
proportional to , then the ratio of specific heat
53 In the given V-T diagram, what is the relation V 1.5
between pressures p 1 and p 2 ? [NEET 2013] capacities at constant pressure to that at constant
V
volume for the gas is [Kerala CEE 2013]
p2 (a) 1.5 (b) 0.25
(c) 0.75 (d) 0.4
p1
(d) 1.3
θ2  5
θ1 60 One litre of a gas  with γ =  at NTP is
T  3
(a) p 2 = p1 (b) p 2 > p1 compressed adiabatically to one cubic centimetre,
(c) p 2 < p1 (d) Cannot be predicted then the resulting pressure is [MP PMT 2013]
54 The amount of heat energy required to raise the (a) 10 atm (b) 103 atm
temperature of 1 g of helium at NTP, from T1 K to (c) 105 atm (d) 100 atm
T 2 K is [NEET 2013] 61 The specific heat of an ideal gas for an isothermal
3 3 process is [MP PMT 2013]
(a) N AkB (T2 − T1 ) (b) N AkB (T2 − T1 )
8 2 3
(a) zero (b) R
3 3  T2  2
(c) N AkB (T2 − T1 ) (d) N AkB  
4 4  T1  (c)
5
R (d) infinite
2
55 The efficiency of a Carnot engine kept at the
temperatures of 27°C and 127°C is [ J & K CET 2013] 62 A thermodynamic system is taken through the cycle
(a) 20% (b) 25% (c) 30% (d) 40% ABCD as shown in figure. Heat rejected by the gas
during the cycle is [CBSE AIPMT 2012]
56. Match the Column I with Column II and mark the
correct option from the codes given below. 2p D C
[EAMCET 2013]

Column I Column II p
A B
(A) Work done in isobaric process (p) nRT log e (V2 /V1)
(B) Work done in isothermal process (q) p (V2 − V1) V 3V
(C) Work done in adiabatic process (r) nR (T1 − T2 ) (a) 2 pV (b) 4 pV
γ −1 1
(c) pV (d) pV
(s) zero 2
730 OBJECTIVE Physics Vol. 1

63 One mole of an ideal gas goes from an initial state A B, C and D respectively, then which of the following
to final state B via two processes. It first undergoes is not true? [Manipal 2012]
isothermal expansion from volumeV to 3V and then

Pressure
its volume is reduced from 3V toV at constant 2p A B
pressure. The correct p-V diagram representing the
two processes is [CBSE AIPMT 2012] p C
D
p B p A
T 2T
Temperature
(a) A (b) B
(a) U A = U D (b) U B < U A (c) U B = U C (d) U C > U D
V V
V 3V V 3V 69 During an adiabatic process, the cube of the pressure
is found to be inversely proportional to the fourth
p A p A
power of the volume. Then, the ratio of specific
heats is [Manipal 2012]
(c) B (d) B
(a) 1 (b) 1.33 (c) 1.67 (d) 1.4
V V 70 A Carnot’s engine has an efficiency of 50% at sink
V 3V V 3V
temperature 50°C. Calculate the temperature of
64 If an ideal gas changes from state 1 to state 2 as source. [Manipal 2012]
shown in figure, then the work done by the gas in (a) 133°C (b) 143°C (c) 100°C (d) 373°C
the process will be [BHU (Screening) 2012] 3
71 The water of volume 4 m at the height 20 m is
p
2 pressed by 2 × 10 5 N pressure. The work done by
motor is [Manipal 2012]
(a) 8 × 105 J (b) 16 × 105 J
1
(c) 12 × 105 J (d) 32 × 105 J
T
72 In adiabatic process, the work done by system is
(a) positive (b) negative
50 J, then

You might also like